Engineering mathematics : a tutorial approach 9780070146150, 0070146152

3,807 423 59MB

English Pages [1689] Year 2010

Report DMCA / Copyright

DOWNLOAD FILE

Polecaj historie

Engineering mathematics : a tutorial approach
 9780070146150, 0070146152

Table of contents :
Title
Contents
1 Complex Numbers
2 Differential Calculus I
3 Differential Calculus II
4 Partial Differentiation
5 Infinite Series
6 Integral Calculus
7 Gamma and Beta Functions
8 Multiple Integral
9 Vector Calculus
10 Differential Equations
11 Matrices
12 Laplace Transform
13 Fourier Series
14 Fourier Transform
15 Z-transform
Appendix 1 Differential Formulae
Appendix 2 Integral Formulae
Appendix 3 Standard Curve
Index

Citation preview

Engineering Mathematics A Tutorial Approach

About the Authors Ravish R Singh is presently Vice-Principal and Head, Department of Electronics and Telecommunication Engineering at Thakur College of Engineering and Technology, Mumbai. He obtained his BE degree from University of Mumbai, in 1991 and MTech from IIT Bombay, in 2001. He is pursuing PhD from Faculty of Technology, University of Mumbai. He has published two books, namely, Electrical Networks, and Basic Electrical and Electronics Engineering with Tata McGraw Hill Education Private Limited. He is a member of IEEE, ISTE, IETE and CSI, and has published research papers in national journals. His fields of interest include Circuits, Signals & Systems and Engineering Mathematics. Mukul Bhatt is presently Senior Lecturer, Department of Humanities and Sciences at Thakur College of Engineering and Technology, Mumbai. She obtained her MSc (Mathematics) degree from H N B Garhwal University, in 1992. She has fifteen years of teaching experience at various levels in engineering colleges of Mumbai. Her fields of interest include Integral Calculus, Complex Analysis and Operation Research. She is a member of ISTE.

Engineering Mathematics A Tutorial Approach

Ravish R Singh Vice Principal and Head Department of Electronics and Telecommunication, Engineering Thakur College of Engineering and Technology, Mumbai Mukul Bhatt Senior Lecturer Department of Humanities and Sciences Thakur College of Engineering and Technology, Mumbai

Tata McGraw Hill Education Private Limited NEW DELHI New Delhi New York St Louis San Francisco Auckland Bogotá Caracas Kuala Lumpur Lisbon London Madrid Mexico City Milan Montreal San Juan Santiago Singapore Sydney Tokyo Toronto

Tata McGraw-Hill Published by the Tata McGraw Hill Education Private Limited, 7 West Patel Nagar, New Delhi 110 008. Copyright© 2010, by Tata McGraw Hill Education Private Limited. No part of this publication may be reproduced or distributed in any form or by any means, electronic, mechanical, photocopying, recording, or otherwise or stored in a database or retrieval system without the prior written permission of the publishers. The program listings (if any) may be entered, stored and executed in a computer system, but they may not be reproduced for publication. This edition can be exported from India only by the publishers, Tata McGraw Hill Education Private Limited ISBN (13): 978-0-07-014615-0 ISBN (10): 0-07-014615-2 Managing Director: Ajay Shukla Head—Higher Education Publishing: Vibha Mahajan Manager: Sponsoring—SEM & Tech Ed: Shalini Jha Editorial Executive: Tina Jajoriya Jr Executive: Editorial Services: Dipika Dey Sr Production Executive: Suneeta S Bohra General Manager: Marketing—Higher Education: Michael J Cruz Sr Product Manager—SEM & Tech Ed: Biju Ganesan Asst Product Manager—SEM & Tech Ed: Amit Paranjpe General Manager—Production: Rajender P Ghansela Asst General Manager—Production: B L Dogra Information contained in this work has been obtained by Tata McGraw Hill, from sources believed to be reliable. However, neither Tata McGraw Hill nor its authors guarantee the accuracy or completeness of any information published herein, and neither Tata McGraw Hill nor its authors shall be responsible for any errors, omissions, or damages arising out of use of this information. This work is published with the understanding that Tata McGraw Hill and its authors are supplying information but are not attempting to render engineering or other professional services. If such services are required, the assistance of an appropriate professional should be sought. Typeset at Tej Composers, WZ-391, Madipur, New Delhi - 110 063 and printed at Gopsons, A-2&3, Sector-64, Noida, U.P. 201 301 Cover Printer: Gopsons RYZYYRYZDARCL

Dedicated To Our Parents Late Shri Ramsagar Singh and Shrimati Premsheela Singh Ravish R Singh

Shri Ved Prakash Sharma and Late Shrimati Vidyavati Hemdan Mukul Bhatt

Preface

O Engineering Mathematics is a key area in the study of an engineering course. It is the study of numbers, structures, and associated relationships using rigorously defined literal, numerical and operational symbols. A sound knowledge of the subject develops analytical skills, thus enabling engineering graduates to solve numerical problems encountered in daily life, as well as apply mathematical principles to physical problems, particularly in the area of engineering.

Rationale We have observed that many students who opt for engineering find it difficult to conceptualise the subject since very few available texts have syllabus compatibility and right pedagogy. Feedback received from students and teachers have highlighted the need for a comprehensive textbook on Mathematics that covers all topics of first year engineering along with suitable solved problems. This book—an outcome of our vast experience of teaching the undergraduate students of engineering—provides a solid foundation in mathematical principles, enabling students to solve mathematical, scientific and associated engineering principles.

Users This book on Engineering Mathematics, meant for first year engineering students, covers both Mathematics-I and Mathematics-II papers (first year engineering mathematics course) in a single volume. The structuring of the book takes into account the commonly featuring topics in the syllabi of major Indian universities.

Intent An easy-to-understand and student-friendly text, it presents concepts in adequate depth using step-by-step problem solving approach. The text is well supported with plethora of solved examples at varied difficulty levels, practice problems and engineering applications. It is intended that students will gain logical understanding from solved problems and then through solving similar problems themselves.

Features Each topic has been thoroughly covered from the examination point of view. The theory part of the text is explained in a lucid manner. For each topic, problems of all

viii

Preface

possible combinations have been worked out. This is followed by an exercise with answers. Objective type questions provided in each chapter help students in mastering concepts. Salient features of the book are summarised below:

Multiple Choice Questions (350). maxima and minima under Partial Differential Equation) have been provided.

the text.

Organisation The contents of this book are divided into 15 chapters, keeping in mind the syllabus structure in major Indian universities. first chapter on Complex Numbers covers De Moivre’s theorem, hyperbolic functions and logarithm of complex number. Chapter 2 on Differential Calculus I offers a detailed exposition of successive differentiation, mean value theorems, expansion of functions and indeterminate forms. Chapter 3 on Differential Calculus II are tangents and normals, radius of curvature, evolutes, envelopes and curve tracing. Chapter 4 on Partial Differentiation elucidates composite function, homogemaxima and minima and Lagrange’s multipliers. Chapter 5 on Infinite Series deals with various tests to check the convergence of the series. Chapter 6 on Integral Calculus explains reduction formulae, rectification of curves, area under the curves, volume and surface area of solid of revolution. Chapter 7 gives a clear understanding of Gamma and Beta functions and their properties. Chapter 8 on Multiple Integrals includes double and triple integrals and their applications. Chapter 9 on Vector Calculus provides comprehensive coverage of vector differentiation and integration. Chapter 10 on Differential Equations explains first order differential equations, linear differential equations of higher order, homogeneous differential equations and applications of differential equations. Chapter 11 on Matrices covers inverse, rank, normal form, solution of homogeneous and non homogeneous equations, eigen values, eigen vectors and quadratic forms. Chapter 12 on Laplace Transform explains properties of Laplace transform, inverse Laplace transform and its applications.

Preface

ix

Chapter 13 on Fourier Series gives a detailed account of orthogonal functions, trigonometric and exponential Fourier series and half range Fourier series. Chapter 14 on Fourier Transform covers Fourier integral theorem, Fourier sine and cosine transforms, and finite Fourier transforms. Chapter 15 on Z-Transform deals with properties of Z-Transform, inverse Z-Transform and applications of Z-Transform.

Exhaustive OLC Supplements The website accompanying the book http://www.mhhe.com/ravish/mukul/em provides valuable resources such as additional solved examples. Instructors can access a solution manual, chapter wise PowerPoint slides with diagrams and notes for effective lecture presentations, and a test bank. Students can avail a sample chapter and link to reference material.

Acknowledgements We would like to express our gratitude to our colleagues in Thakur college of Engineering and Technology for their support and suggestions. We extend our appreciation

with us during the editorial, copyediting and production stages of the book. We would also like to thank our family members for encouraging, inspiring and supporting us while the making of the book was in progress. A note of acknowledgement is due to the following reviewers for their valuable suggestions. S B Singh, G. B. Pant University of Agriculture & Technology, Pantnagar Vinai K Singh, R. D. Engineering College (UPTU), Ghaziabad K H Patil, University of Pune, Pune S Jha, National Institute of Technology, Jamshedpur

Debdas Mishra, C V Raman College of Engineering, Bhubaneswar G Prema, Amrita Vishwa Vidyapeetham (Deemed University), Coimbatore BV Appa Rao, Koneru Lakshmaiah College of Engineering, Guntur Y P Anand, Kakinada Institute of Engineering and Technology, Kakinada

Ravish R Singh Mukul Bhatt

Publisher’s Note: Tata McGraw Hill Education looks forward to receiving from teachers and students their valuable views, comments and suggestions for improvements, all of which may be sent to [email protected] (mentioning the title and author’s name). Also, please inform any observations on piracy related issues.

Contents

O Preface

vii

1. COMPLEX NUMBERS

1.1

1.1 Introduction 1.1 1.2 Complex Numbers 1.1 1.3 Geometrical Representation of Complex Numbers (Argand’s Diagram) 1.2 1.4 Algebra of Complex Numbers 1.2 1.5 Different Forms of Complex Numbers 1.2 1.6 Modulus and Argument (or Amplitude) of Complex Numbers 1.3 1.7 Properties of Complex Numbers 1.3 1.21 1.9 Applications of De Moivre’s Theorem 1.37 1.10 Circular and Hyperbolic Functions 1.71 1.11 Inverse Hyperbolic Functions 1.74 1.12 Separation Into Real and Imaginary Parts 1.86 1.13 Logarithm of a Complex Number 1.108 Formulae 1.122 Multiple Choice Questions

1.123

2. DIFFERENTIAL CALCULUS I 2.1 2.2 2.3 2.4 2.5 2.6 2.7

Introduction 2.1 Successive Differentiation 2.1 Leibnitz’s Theorem 2.22 Mean Value Theorem 2.42 Rolle’s Theorem 2.42 Lagrange’s Mean Value Theorem (L.M.V.T.) 2.53 Cauchy’s Mean Value Theorem (C.M.V.T.) 2.71 78 2.9 Maclaurin’s Series 2.90

2.1

xii

Contents 2.10 Indeterminate Forms 2.121 Formulae 2.161 Multiple Choice Questions

2.162

3. DIFFERENTIAL CALCULUS II 3.1 3.2 3.3 3.4 3.5 3.6 3.7

3.1

Introduction 3.1 Tangent and Normal 3.1 Length of an Arc and its Derivative 3.28 Curvature 3.31 Centre and Circle of Curvature 3.50 Evolute 3.51 Envelopes 3.64 3.76 Formulae 3.109 Multiple Choice Questions

3.110

4. PARTIAL DIFFERENTIATION 4.1 4.2 4.3 4.4 4.5 4.6 4.7

4.1

Introduction 4.1 Partial Derivative 4.1 Higher Order Partial Derivatives 4.2 Variables to be Treated as Constants 4.33 Composite Function 4.40 Implicit Functions 4.63 Homogeneous Functions and Euler’s Theorem 4.68 4.98 Formulae 4.154 Multiple Choice Questions

4.155

5. INFINITE SERIES 5.1 5.2 5.3 5.4 5.5 5.6 5.7

5.1

Introduction 5.1 Sequence 5.1 Infinite Series 5.2 Geometric Series 5.4 Standard Limits 5.5 Comparison Test 5.5 D’Alembert’s Ratio Test 5.11 5.18

5.9 5.10 5.11 5.12

Logarithmic Test 5.23 Cauchy’s Root Test 5.27 Cauchy’s Integral Test 5.31 Alternating Series 5.34

xiii

Contents 5.13 Absolute Convergence of a Series 5.35 5.14 Uniform Convergence of a Series 5.38 Formulae 5.41 Multiple Choice Questions

5.42

6. INTEGRAL CALCULUS 6.1 6.2 6.3 6.4 6.5 6.6

6.1

Introduction 6.1 Reduction Formulae 6.1 Rectification of Curves 6.16 Areas of Plane Curves (Quadrature) 6.47 Volume of Solid of Revolution 6.68 Surface of Solid of Revolution 6.90 Formulae 6.106 Multiple Choice Questions

6.108

7. GAMMA AND BETA FUNCTIONS 7.1 7.2 7.3 7.4 7.5 7.6

7.1

Introduction 7.1 Gamma Function 7.1 Properties of Gamma Function 7.2 Beta Function 7.10 Properties of Beta Function 7.11 Beta Function as Improper Integral 7.25 Formulae 7.32 Multiple Choice Questions

7.33

8. MULTIPLE INTEGRAL

8.1

8.1 8.1 Order of Integration 8.21 8.39 Variables of Integration 8.57 8.67 Multiple Integrals 8.85 Multiple Choice Questions

133

9. VECTOR CALCULUS 9.1 9.2 9.3 9.4

Introduction 9.1 Unit Vector 9.1 Components of a Vector 9.1 Triple Product 9.2

9.1

xiv

Contents 9.5 Product of Four Vectors 9.8 9.6 Vector Function of a Single Scalar Variable 9.12 9.7 Velocity and Acceleration 9.13 9.13 9.9 Tangent Vector to a Curve at a Point 9.14 9.10 Scalar and Vector Point Function 9.24 9.11 Gradient 9.25 9.12 Divergence 9.46 9.13 Curl 9.48 9.14 Properties of Gradient, Divergence and Curl 9.60 9.15 Second Order Differential Operator 9.64 9.16 Line Integrals 9.81 9.17 Green’s Theorem in the Plane 9.98 9.115 9.19 Volume Integral 9.121 9.20 Stoke’s Theorem 9.124 9.21 Gauss Divergence Theorem 9.147 Formulae 9.165 Multiple Choice Questions

9.165

10. DIFFERENTIAL EQUATIONS

10.1

10.1 Introduction 10.1 10.2 Differential Equation 10.1 10.3 Ordinary Differential Equations of First Order and First Degree 10.2 10.4 Homogeneous Linear Differential Equations of Higher Order with Constant Coefficients 10.77 10.5 Non-Homogeneous Linear Differential Equations of Higher Order with Constant Coefficients 10.85 10.6 Higher Order Linear Differential Equations with Variable Coefficients 10.111 10.7 Method of Variation of Parameters 10.125 10.131 10.9 Simultaneous Linear Differential Equations with Constant Coefficients 10.142 10.10 Applications of Ordinary Differential Equations of First Order and First Degree 10.150 10.11 Applications of Higher Order Linear Differential Equations 10.175 Formulae 10.197 Multiple Choice Questions

10.200

Contents

11. MATRICES 11.1 11.2 11.3 11.4 11.5 11.6 11.7 11.9 11.10 11.11 11.12 11.13 11.14 11.15 11.16

xv

11.1

Introduction 11.1 Matrix 11.1 Some Definitions Associated with Matrices 11.1 Adjoint of a Square Matrix 11.19 Inverse or Reciprocal of a Matrix 11.23 Elementary Transformations 11.38 Rank of a Matrix 11.45 11.63 Homogeneous Linear Equations 11.73 Linear Dependence and Independence of Vectors 11.81 Eigen Values and Eigen Vectors 11.90 Cayley–Hamilton Theorem 11.112 Minimal Polynomial and Minimal Equation of a Matrix 11.120 Function of Square Matrix 11.124 Similarity of Matrices 11.131 Quadratic Form 11.152 Multiple Choice Questions

11.173

12. LAPLACE TRANSFORM

12.1

12.1 12.2 12.3 12.4 12.5 12.6 12.7

Introduction 12.1 Laplace Transform 12.1 Laplace Transform of Some Standard Functions 12.2 Properties of Laplace Transform 12.6 Evaluation of an Integral using Laplace Transform 12.37 Heaviside’s Unit-step Function 12.44 Dirac Delta or Unit Impulse Function 12.50 Transform of Periodic Functions 12.53 12.9 Inverse Laplace Transform 12.58 12.10 Application of Laplace Transform to Differential Equations with Constant Coefficients 12.87 12.11 Application of Laplace Transform to a System of Simultaneous Differential Equations 12.100 Formulae 12.108 Multiple Choice Questions

12.109

13. FOURIER SERIES 13.1 Introduction 13.1 13.2 Orthogonality of Functions 13.1 13.3 Fourier Series 13.10

13.1

xvi

Contents 13.4 13.5 13.6 13.7

Parseval’s Identity 13.14 Fourier Series of Even and Odd Functions 13.37 Half-range Fourier Series 13.52 Complex Form of Fourier Series 13.62 Formulae 13.70 Multiple Choice Questions

13.71

14. FOURIER TRANSFORM 14.1 14.2 14.3 14.4 14.5

14.1

Introduction 14.1 Fourier Integral Theorem 14.1 Fourier Transform 14.9 Properties of the Fourier Transform 14.11 Finite Fourier Transforms 14.29 Formulae 14.35 Multiple Choice Questions

14.35

15. Z-TRANSFORM 15.1 15.2 15.3 15.4 15.5 15.6

15.1

Introduction 15.1 Sequence 15.1 Z-transform 15.6 Properties of Z-transform 15.6 Inverse Z-transform 15.18 Application of Z-transform to Difference Equations 15.32 Formulae 15.138 Multiple Choice Questions

Appendix A Differential Formulae Appendix B Integral Formulae Appendix C Standard Curves Index

15.140 A.1.1 A.2.1 A.3.1 I.1

Visual Guide

O 4.2.1 Geometrical Interpretation The function u = f (x, y) represents a surface. The point P [x1, y1, f (x1, y1)] on the surface corresponds to the values x1, y1 of the independent variables x, y. The intersection of the plane y = y1 (parallel to the zox–plane) and the surface u = f (x, y) is the curve shown by the dotted line in the Figure. On this curve, x and u vary according to the relation u = f (x, y1). The ordinary derivative of f (x, y1) w.r.t. x at x1

Lucid Text

Fig. 4.1

⎛ ∂u ⎞ ⎛ ∂u ⎞ is ⎜ ⎟ . Hence, ⎜ ⎟ is the slope of the tangent to ⎝ ∂x ⎠ ( x, y1 ) ⎝ ∂x ⎠ ( x1 , y1 ) the curve of the intersection of the surface u = f (x, y) with the plane y = y1 at the point P[x1, y1, f (x1, y1)]. ⎛ ∂u ⎞ Similarly, ⎜ ⎟ is the slope of the tangent to the curve of the intersection of the ⎝ ∂y ⎠ ( x , y ) 1 1

surface u = f (x, y) with the plane x = x1 at the point P[x1, y1, f (x1, y1)].

4.3 HIGHER ORDER PARTIAL DERIVATIVES Partial derivatives of higher order, of a function u = f (x, y), are obtained by partial differentiation of first order partial derivative. Thus, if u = f (x, y), then ∂ 2 u ∂ ⎛ ∂u ⎞ = ⎜ ⎟ ∂x 2 ∂x ⎝ ∂x ⎠ ∂2 u ∂ ⎛ ∂u ⎞ = ⎜ ⎟ ∂y ∂x ∂y ⎝ ∂x ⎠

O

Chapter

5

Organised Sections

In this chapter, we will learn about the convergence and divergence of an infinite series. There are various methods to test the convergence and divergence of an infinite series. In this chapter, we will study Comparision Test, D’Alembert’s ratio test, Raabe’s test, Logarithmic test, Cauchy’s root test and Cauchy’s integral test. We will also study alternating series, absolute and uniform convergence of the series.

An ordered set of real numbers as u1, u2, u3, ……..un, …… is called a sequence and is denoted by {un}. If the number of terms in a sequence is infinite, it is said to be infinite sequence, otherwise it is a finite sequence and un is called the nth term of the sequence. A sequence is said to be monotonically increasing if un 1 un for each value of n and is monotonically decreasing if un 1 un for each value of n, whereas the sequence is called alternating sequence if the terms are alternate positive and negative. e.g. (i) 1, 2, 3, 4, … is a monotonically increasing sequence. 1 1 1 (ii) 1, , , , … is a monotonically decreasing sequence. 2 3 4 (iii) 1, –2, 3, – 4, … is an alternating sequence.

Example 18: If ` = i + 1, a = 1 - i and tanφ =

Solved Examples

( x + ` )n - ( x + a )n = sin ne cosec ne . ` -a

Solution: a

1, b

i

i, tan =

1 cot f

n

x

1, x

1 x +1 cot f

1 , then prove that x +1

1

n

(x + α ) − (x + β ) (cot φ − 1 + i + 1) n − (cot φ − 1 + 1 − i ) n = α −β i +1−1+ i n

n

⎛ cos φ ⎞ ⎛ cos φ ⎞ +i⎟ −⎜ −i⎟ ⎜ ⎝ sin φ ⎠ ⎝ sin φ ⎠ 2i n (cos f + i sin f ) − (cos f − i sin f ) n = 2i sin n f

=

=

(e if ) n − (e − if ) n e inf − e − inf 2i sin nf = = 2i sin n f 2i sin n f 2i sin n f

= sin nf cosec n f Example 19: If (1 + cos p + i sin p ) (1 + cos 2p + i sin 2p ) = u + iv, prove that θ v 3 (ii) . (i) u2 + v 2 = 16 cos 2 cos 2θ = tan 2 u 2 Solution: u iv (1 cos q i sin q ) (1 cos 2q i sin 2q ) q q q ⎛ ⎞ = ⎜ 2 cos 2 + i 2 sin cos ⎟ (2 cos 2 q + i 2 sin q cos q ) ⎝ 2 2 2⎠ = 2 cos

q⎛ q q⎞ ⎜ cos + i sin ⎟⎠ 2 cos q (cosq + i sin q ) 2⎝ 2 2

xviii

Visual Guide

4.8 APPLICATIONS OF PARTIAL DIFFERENTIATION 4.8.1 Jacobians If u and v are continuous and differentiable functions of two independent variables x

and y, i.e., u

f1(x, y) and v

f2(x, y), then the determinant

u x

u y

v x

v y

Application Focus is called the

Jacobian of u, v with respect to x, y and is denoted as J = (u, v) . ( x, y ) Similarly, if u, v and w are continuous and differentiable functions of three independent variables x, y, z, then the Jacobian of u, v, w with respect to x, y, z is u x (u, v, w) = ( x, y , z )

u y

u z

v v v x y z w w w x y z

Jacobian is useful in transformation of variables from cartesian to polar, cylindrical and spherical coordinates in multiple integrals.

Exercise 2.2 1. Find the nth order derivative w.r.t. x (i) xex (ii) x2e2x (iii) x log (x 1) (iv) x3 sin 2x (v) y x2 sin x ⎡ Ans. : (i) e x ( x + n) ⎢ 2x n 2 n n −1 ⎢ (ii) e [2 x + 2 nx + n ( n − 1) 2 ] ⎢ ( −1) n − 2 ( n − 2)!( x + n) ⎢(iii) ( x + 1) n ⎢ ⎢ ⎢(iv) 2n x 3 sin ⎛⎜ 2 x + np ⎞⎟ + ⎝ ⎢ 2 ⎠ ⎢ p⎤ ⎡ ⎢ 3n x 2 2n −1 sin ⎢ 2 x + ( n − 1) ⎥ ⎢ 2⎦ ⎣ ⎢ p⎤ ⎡ ⎢ + 3n ( n − 1) x 2n − 2 sin ⎢ 2 x + ( n − 2) ⎥ ⎢ 2⎦ ⎣ ⎢ + n ( n − 1) ( n − 2) 2n − 3 ⎢ ⎢ n p ⎡ ⎤ ⎢ sin ⎢ 2 x + ( n − 3) ⎥ 2 ⎦ ⎢ ⎣ ⎢ ⎢( v) x 2 sin ⎛ x + np ⎞ ⎜ ⎟ ⎝ ⎢ 2 ⎠ ⎢ p⎤ ⎡ ⎢ + 2nx sin ⎢ x + ( n − 1) ⎥ ⎢ 2⎦ ⎣ ⎢ p ⎡ ⎤ ⎢ + ( n2 − n) sin ⎢ x + ( n − 2) ⎥ 2⎦ ⎣ ⎣⎢

Exercises

3. If y e ax [a2 x2 - 2nax n (n prove that yn an 2 x2 eax. 4. If y ⎤ ⎥ ⎥ ⎥ ⎥ ⎥ ⎥ ⎥ ⎥ ⎥ ⎥ ⎥ ⎥ ⎥ ⎥ ⎥ ⎥ ⎥ ⎥ ⎥ ⎥ ⎥ ⎥ ⎥ ⎥ ⎥ ⎥ ⎥ ⎥⎦

1)],

x2 sin x, prove that

np ⎞ ⎛ yn = ( x 2 − n2 + n) sin ⎜ x + ⎟ ⎝ 2 ⎠ np ⎞ ⎛ − 2nx cos ⎜ x + ⎟ ⎝ 2 ⎠ 5. If x

tan log y, prove that

(1 + x 2 ) yn +1 + (2nx − 1) yn + n (n − 1) yn −1 = 0 ⎡ Hint : log y = tan −1 x, y = e tan ⎣

−1

x

⎤ ⎦

6. If y cos (m sin−1 x), prove that (1 - x2) yn 2 - (2n 1) xyn 1 (m2 - n2) yn 0 Hence, obtain yn (0). ⎡ Ans. : yn (0) = ( n2 − m 2 )........... ⎤ ⎥ ⎢ ( 4 2 − m 2 )( 22 − m 2 )( − m 2 ) ⎥⎦ ⎢⎣ 7. If x sin q, y sin 2q, prove that 1) xyn 1 (1 - x2) yn 2 - (2n (n2 - 4) yn 0 ⎡ Hint : y = 2 sin q cos q = 2 x 1 − x 2 ⎤ ⎣ ⎦

1.8 DE MOIVRE’S THEOREM Statement: For any real number n, one of the values of (cos q cos nq i sin nq. Hence, (cos q

i sin q )n

cos nq

i sin q )n is

i sin nq

Proof: Case I: If n is a positive integer Let z1 r1 (cos q1

i sin q1), z2 r2 (cos q2

z1 z2 r1 (cos q1

i sin q1) r2 (cos q2

r1 r2 [(cos q1 cos q2 r1 r2 [cos (q1 Similarly, z1 z2……. zn

r1 (cos q1

q2)

i sin q2) , …… , zn rn (cos qn

sin q1 sin q2) i sin (q1

i sin qn).

i sin q2) i(sin q1 cos q2

cos q1 sin q2)]

q2)]

i sin q1) r2 (cos q2

i sin q2)……. rn (cos qn

i sin qn)

(r1 r2…….. rn) (cos q1 i sin q1) (cos q2 i sin q2)….. (cos qn i sin qn) (r1 r2…….. rn)[cos (q1 q2 …… qn) If z1

z2

…….

zn rn (cos q (cos q

zn z i sin q )n

i sin q )n

r (cos q

(cos nq

i sin (q1

q2…… qn)] … (1)

i sin q ) , then Eq. (1) reduces to

rn (cos nq

i sin nq )

i sin nq ), where n is a positive integer.

Theorems and Derivations

xix

Visual Guide

FORMULAE

Important Formulae

Tangent and Normal Equation of the tangent at any point (x, y): Y – y = fÄ (x) (X – x) Equation of the normal at any point 1 (x, y): Y – y = − (X – x) f ′( x )

Length of polar normal

Angle of Intersection of Curves

Derivative of Length of an arc

= tan −1

m2 − m1 1 + m2 m1

=

dr d

2

(i)

⎛ dx ⎞ Length of tangent = y 1 + ⎜ ⎟ ⎝ dy ⎠

dy dx

ds ⎛ dy ⎞ = 1 + ⎜ ⎟ Cartesian form ⎝ dx ⎠ dx ⎛ dx ⎞ ds = 1+ ⎜ ⎟ dy ⎝ dy ⎠

2

dx dy

⎛ dy ⎞ Length of normal = y 1 + ⎝⎜ ⎠⎟ dx Length of sub-normal = y

2

Length of polar sub-normal =

Length of Tangent, Sub-tangent, Normal and Sub-normal

Length of sub-tangent = y

⎛ dr ⎞ r2 + ⎜ ⎟ ⎝d ⎠

2

2

2

(ii)

ds ⎛ dx ⎞ ⎛ dy ⎞ = ⎜ ⎟ + ⎜ ⎟ Parametric ⎝ dt ⎠ ⎝ dt ⎠ form dt

(iii)

ds ⎛ dr ⎞ = r 2 + ⎜ ⎟ Polar form ⎝d ⎠ d

2

2

ds ⎛d ⎞ = 1 + r2 ⎜ ⎟ ⎝ dr ⎠ dr

2

MULTIPLE CHOICE QUESTIONS Choose the correct alternative in each of the following: 1. The equation of the tangent to the curve y = 2 sin x + sin 2x at x = p is 3 equal to (a) 2y = 3 3 (b) y = 3 3 (c) 2y + 3 3 = 0 (d) y + 3 3 = 0 2. The sum of the squares of the intercept made on the co-ordinate axis by the tangents to the curve 2

2

2

x 3 + y 3 = a 3 is (a) a2 (b) 2a2 (c) 3a2 (d) 4a2 3. The equation of the normal to the curve y = x (2 – x) at the point (2, 0) is (a) x – 2y = 2 (b) 2x + y = 4 (c) x – 2y + 2 = 0 (d) none of these 4. The length of the normal at t on the curve x = a (t + sin t), y = a(1 – cos t) is

(b) 2a sin3 t sec t 2 2 (c) 2a sin t tan t 2 2 t (d) 2a sin 2 5. The length of the sub-tangent to the curve x2 + xy + y2 = 7 at (1, –3) is (a) 3 (b) 5 (c) 15 (d) 3 5 6. The angle of intersection of the curves y = 4 – x2 and y = x2 is 4 p (b) tan–1 (a) 3 2 (d) none of these (c) tan–1 4 2 7 7. The length of the sub-normal to the parabola y2 = 4ax at any point is equal to

()

2a (b) 2 2a a (d) 2a 2 8. If x = a (q + sin q) and y = dy will be equal to a (1 – cos q), then dx (a) (c)

(a) a sin t

Exhaustive Online Learning Center

Multiple Choice Questions

Complex Numbers

Complex Numbers Chapter

1.1

1

1.1 INTRODUCTION The complex numbers are an extension of the real numbers obtained by introducing an imaginary unit i, where i = 1 . The operations of addition, subtraction, multiplication and division are applicable on complex numbers. A negative real number can be obtained by squaring a complex number. With a complex number, it is always possible to find solutions to polynomial equations of degree more than one. Complex numbers are used in many applications, such as control theory, signal analysis, quantum mechanics, relativity, etc.

1.2 COMPLEX NUMBERS A complex number z is an ordered pair (x, y) of real numbers x and y. It is written as z = (x, y) orz = x + iy, where i =

1 is known as the imaginary unit. Here, x is called

the real part of z and is written as “Re (z)” and y is called the imaginary part of z and is written as “Im (z)”. If x = 0 and y 0, then z = 0 + iy = iy which is purely imaginary. If x 0 and y = 0, then z = x + i 0 = x which is real. Hence, z is purely imaginary, if its real part is zero and is real, if its imaginary part is zero. This shows that every real number can be written in the form of a complex number by taking its imaginary part as zero. Hence, the set of real numbers is contained in the set of complex numbers. The even power of i is either 1 or 1 and odd power of i is either i or i. i2 = i.i = 1, i3 = i2.i = i, i4 = (i2)2 = ( 1)2 = 1, i5 = i. i4 = i, etc. Two complex numbers are equal if and only if their corresponding real and imaginary parts are equal. If z = x + iy as z = x iy.

1.2

Engineering Mathematics

1.3 GEOMETRICAL REPRESENTATION OF COMPLEX NUMBERS (ARGAND’S DIAGRAM) Any complex number z = x + iy can be represented as a point P(x, y) in the xy-plane with reference to the rectangular x and y axes. The plot of a given complex number z = x + iy, as the point P(x, y) in the xy-plane is known as Argand’s diagram. The x-axis is called the real axis, y-axis is called the imaginary axis and the xy-plane is called the complex plane.

y P(x, y)

x'

x

O

y'

Fig. 1.1

1.4 ALGEBRA OF COMPLEX NUMBERS Let z1 = x1 + iy1 and z2 = x2 + iy2 be two complex numbers. (a) Addition: z1 + z2 = (x1 + iy1) + (x2 + iy2) = (x1 + x2) + i (y1 + y2) (b) Subtraction: z1 z2 = (x1 + iy1) (x2 + iy2) = (x1 x2) + i (y1 y2) (c) Multiplication: z1 z2 = (x1 + iy1) (x2 + iy2) = (x1 x2 y1 y2) + i (x2 y1 + y2 x1) (d) Division:

[∵ i2 = –1]

z1 x + iy1 = 1 z2 x2 + iy2 = =

( x1 + iy1 ) ( x2 − iy2 ) ⋅ ( x2 + iy2 ) ( x2 − iy2 ) x1 x2 + y1 y2 x22 + y22

+i

( y1 x2 − x1 y2 ) ( x22 + y22 )

1.5 DIFFERENT FORMS OF COMPLEX NUMBERS 1.5.1 Cartesian or Rectangular Form If x and y are real numbers, then z = x + iy is called the Cartesian form of the complex number.

1.5.2 Polar Form The complex number z = x + iy can be represented by the point P whose cartesian coordinates are (x, y). We know that if polar coordinates of the same point P are (r, q ), then x = r cos q and y = r sin q.

Complex Numbers

1.3 y

Hence, polar form of z is z = r cos q + ir sin q = r (cos q + i sin q ) Polar form can also be written as r q.

P(r, ) r x'

We know that eiq = cos q + i sin q Using polar form, z = r (cos q + i sin q) = reiq This is called the exponential form or Euler’s form of a complex number z. Note: eiq = cos q + i sin q, e iq = cos q i sin q. 1 1 Hence, cos = (ei + e −i ) and sin = (ei 2i 2

x

O

1.5.3 Exponential Form

y'

Fig. 1.2

e

i

)

1.6 MODULUS AND ARGUMENT (OR AMPLITUDE) OF COMPLEX NUMBER Let z be a complex number such that z = x + iy = r (cos q + i sin q ) where, x = r cos q, y = r sin q y r = x 2 + y 2 and tan = or then x

⎛ y⎞ = tan −1 ⎜ ⎟ ⎝x⎠ Here ‘r’ is called the modulus or absolute value of z and is denoted by |z| or mod (z) and q is called argument or amplitude of z and is denoted by arg (z) or amp (z). Hence,

z = r = x2 + y 2

y x Note: The value of q which satisfies both the equations x = r cos q and y = r sin q, gives the argument of z. Argument q has infinite number of values. The value of q lying between p and p is called the principal value of argument. arg (z) =

= tan −1

1.7 PROPERTIES OF COMPLEX NUMBER Let z = x + iy and z = x iy. 1 (a) Re (z) = x = ( z + z ) 2 1 (b) Im (z) = y = (z z ) 2i (c) ( z1 + z2 ) = z1 + z2

1.4

Engineering Mathematics

(d) ( z1 z2 ) = z1 z2 ⎛ z1 ⎞ z1 ⎟ = z z 2 ⎝ 2⎠

(e) ⎜

(f ) z z = |z|2 = | z |2

[∵ z = | z | = x 2 + y 2 ]

(g) |z1z2|= |z1| |z2| and arg (z1z2) = arg (z1) + arg (z2) Proof: Let z1 = r1 eiq1 , z2 = r2 eiq 2 z1z2 = r1 eiq1 r2 eiq 2 = (r1 r2) ei(q1 +q 2 ) Comparing with exponential form, |z1 z2| = r1 r2 = |z1| |z2| and arg (z1 z2) = q1 + q2 = arg (z1) + arg (z2) (h)

and

z1 z1 = z2 z2 ⎛z ⎞ arg ⎜ 1 ⎟ = arg (z1) ⎝ z2 ⎠

arg (z2)

z1 r1ei 1 ⎛ r1 ⎞ i ( 1 − 2 ) = = ⎜ ⎟e z2 r2 ei 2 ⎝ r2 ⎠ Comparing with exponential form, z1 z1 r = 1 = z2 r2 z2

Proof:

⎛z ⎞ arg ⎜ 1 ⎟ = q1 q2 = arg (z1) arg (z2) ⎝ z2 ⎠ Example 1: Find the modulus and principal value of argument. and

(ii) (4 + 2i ) ( -3 + 2 i )

(i) -1 + i 3 ⎛ 4 - 5i ⎞ ⎛ 3 + 2i ⎞ (iii) ⎜ . ⎝ 2 + 3i ⎟⎠ ⎜⎝ 7 + i ⎟⎠ Solution: (i)

z= 1+ i 3 Re (z) = x = 1, Im (z) = y = r = |z| =

3 ( −1) 2 + ( 3 ) = 2 2

Complex Numbers

q = arg (z) = tan

1

y = tan x

1

⎛ 3⎞ ⎜⎜ ⎟⎟ = tan ⎝ −1 ⎠

1.5

1

( − 3 ) = 2p 3

⎡⎣∵ Point ( −1, 3 ) lies in the second quadrant ⎤⎦

(

)

(ii) z = (4 + 2i ) −3 + 2i = z1 z2

r = z = z1 z2 = z1 z2 = 4 + 2i −3 + 2i = ( 16 + 4 ) ( 9 + 2 ) = 220 = 2 55 = arg ( z ) = arg ( z1 z2 ) = arg ( z1 ) + arg ( z2 ) = arg (4 + 2i ) + arg ( −3 + 2i ) ⎛ 2⎞ ⎛2⎞ = tan −1 ⎜ ⎟ + tan −1 ⎜ ⎜ −3 ⎟⎟ ⎝4⎠ ⎝ ⎠ ⎛ 2⎞ ⎛1⎞ = tan −1 ⎜ ⎟ − tan −1 ⎜ ⎜ 3 ⎟⎟ 2 ⎝ ⎠ ⎝ ⎠

⎛ 1 2 − ⎜ 2 3 = tan ⎜ 1 2 ⎜ 1+ ⋅ 2 3 ⎝ −1 ⎜

⎞ ⎟ ⎟ ⎟ ⎟ ⎠

⎛ 3− 2 2 ⎞ = tan −1 ⎜ ⎜ 6 + 2 ⎟⎟ ⎝ ⎠ (iii) z =

(4 − 5i )(3 + 2i ) z1 z2 = z3 z4 (2 + 3i )(7 + i )

r= z =

z1 z2 4 − 5i 3 + 2i z1 z2 = = = 2 + 3i 7 + i z3 z4 z3 z4

q = arg ( z ) = arg

( 16 + 25 ) ( 9 + 4 ) = ( 4 + 9 ) ( 49 + 1 )

z1 z2 z3 z4

= arg ( z1 z2 ) − arg ( z3 z4 ) = arg ( z1 ) + arg ( z2 ) − [ arg ( z3 ) + arg ( z4 ) ] = arg ( 4 − 5i ) + arg (3 + 2i ) − arg ( 2 + 3i ) − arg (7 + i ) ⎛ −5 ⎞ ⎛2⎞ ⎛3⎞ ⎛1⎞ = tan −1 ⎜ ⎟ + tan −1 ⎜ ⎟ − tan −1 ⎜ ⎟ − tan −1 ⎜ ⎟ 4 3 2 ⎝ ⎠ ⎝ ⎠ ⎝ ⎠ ⎝7⎠ 5 1⎞ ⎛ 2 3⎞ ⎛ = − ⎜ tan −1 + tan −1 ⎟ + ⎜ tan −1 − tan −1 ⎟ 4 7⎠ ⎝ 3 2⎠ ⎝

41 50

1.6

Engineering Mathematics

⎛ 5 1 ⎞ ⎛ 2 3 ⎞ − + ⎜ ⎟ ⎜ ⎟ ⎛ 39 ⎞ ⎛ 5⎞ = − tan −1 ⎜ 4 7 ⎟ + tan −1 ⎜ 3 2 ⎟ = − tan −1 ⎜ ⎟ + tan −1 ⎜ − ⎟ 5.1 2.3 ⎝ 23 ⎠ ⎝ 12 ⎠ ⎜1− ⎟ ⎜ 1+ ⎟ ⎝ ⎠ ⎝ ⎠ 4 7 3 2 ⎛ 39 5 ⎞ + ⎜ ⎟ = − tan −1 ⎜ 23 12 ⎟ = − tan −1 (7.19) 39 . 5 ⎜ 1− ⎟ ⎝ 23 12 ⎠ Example 2: Express in polar form ⎛ 2+ i ⎞ (i) ⎜ ⎟ ⎝ 3−i ⎠

2

(ii) 1 + sin ` + i cos ` 2

Solution: (i)

4 + i 2 + 4i 3 + 4i ⎛ 2 + i⎞ = z=⎜ = ⎟ ⎝ 3−i⎠ 9 + i 2 − 6i 8 − 6i =

3 + 4i 8 + 6i 1 ⋅ = i 8 − 6i 8 + 6i 2

Comparing with polar form, 2

1 ⎛1⎞ r = z = 02 + ⎜ ⎟ = 2 ⎝2⎠

and

⎛1⎞ ⎜⎝ ⎟⎠ p 2 = tan −1 ∞ = q = tan −1 0 2 2

Hence, (ii)

1⎛ ⎛ 2+i ⎞ ⎞ ⎜ 3 − i ⎟ = 2 ⎜ cos 2 + i sin 2 ⎟ ⎝ ⎠ ⎝ ⎠ z = 1 + sin a + i cos a ⎛p ⎞ ⎛p ⎞ = 1 + cos ⎜ − a ⎟ + i sin ⎜ − a ⎟ ⎝2 ⎠ ⎝2 ⎠ ⎛p a ⎞ ⎛p a ⎞ ⎛p a ⎞ = 2 cos 2 ⎜ − ⎟ + 2i sin ⎜ − ⎟ cos ⎜ − ⎟ ⎝ 4 2⎠ ⎝ 4 2⎠ ⎝ 4 2⎠ q q⎤ ⎡ 2q ⎢∵1 + cos q = 2 cos 2 , sin q = 2 sin 2 cos 2 ⎥ ⎦ ⎣ ⎛π α ⎞⎡ ⎛π α ⎞ ⎛ π α ⎞⎤ z = 2 cos ⎜ − ⎟ ⎢cos ⎜ − ⎟ + i sin ⎜ − ⎟ ⎥ ⎝ 4 2 ⎠⎣ ⎝ 4 2 ⎠ ⎝ 4 2 ⎠⎦

Complex Numbers

1.7

Comparing with polar form, ⎛π α ⎞ r = 2 cos ⎜ − ⎟ ⎝4 2⎠ π α θ= − 4 2 ⎛π α ⎞⎡ ⎛π α ⎞ ⎛ π α ⎞⎤ Hence, 1 + sin a + i cos a = 2 cos ⎜ − ⎟ ⎢cos ⎜ − ⎟ + i sin ⎜ − ⎟ ⎥ ⎝ 4 2 ⎠⎣ ⎝ 4 2 ⎠ ⎝ 4 2 ⎠⎦ Example 3: Find the value of Solution: Let x + iy =

- 5 + 12i .

− 5 + 12i

(x + iy)2 = 5 + 12i (x2 y2) + i (2xy) = 5 + 12i Comparing real and imaginary parts on both the sides, x2 y2 = 5, 2xy = 12, xy = 6 6 Putting y = in Eq. (1), x 36 x 2 − 2 = −5 x 4 2 x + 5x 36 = 0 (x2 + 9) (x2

4) = 0 x2 = 9, x2 = 4

Since x is real,

x=±2

When

x = 2, y =

When

x = 2, y =

Hence,

5 12i = 2 + 3i or 2

6 =3 2 6 = −3 −2 3i

Example 4: If x and y are real, solve the equation Solution:

... (1)

iy 3 y + 4i − = 0. ix + 1 3 x + y

iy 3 y + 4i − =0 ix + 1 3x + y iy (3 x + y ) − (3 y + 4i )(ix + 1) =0 (ix + 1)(3x + y ) (−3 y + 4 x) + i (3 xy + y 2 − 3 xy − 4) = 0 + i0 (ix + 1)(3 x + y )

1.8

Engineering Mathematics

Comparing real and imaginary parts on both the sides, 3y + 4x = 0 and y2 4 = 0, y = ± 2 3 x=± 2 3 Hence, x = ± , y = ± 2. 2 Example 5: Prove that Re (z) > 0 and |z - 1| < |z + 1| are equivalent, where z = x + iy. z = x + iy

Solution:

Now,

Re (z) > 0 x>0 |z 1| < |z + 1| |x + iy 1| < |x + iy + 1|

... (1)

( x − 1) 2 + y 2 < ( x + 1) 2 + y 2 x2 + 1

2x + y2 < x2 + 1 + 2x + y2 2x < 2x 0 < 4x 0 < x or x > 0

From Eqs. (1) and (2), Re (z) > 0 and |z

... (2)

1| < |z + 1| are equivalent.

a + ib 1 + iz , then prove that a2 + b2 + c2 = 1, = 1 + c 1 − iz where a, b and c are real numbers and z is a complex number. Example 6: If b + ic = (1 + a) z and

Solution: We have b + ic = (1 + a) z b + ic z= 1+ a a + ib 1 + iz and = 1 + c 1 − iz Substituting z in the above equation, ⎛ b + ic ⎞ 1+ i ⎜ ⎟ a + ib ⎝ 1+ a ⎠ = 1+ c ⎛ b + ic ⎞ 1− i ⎜ ⎟ ⎝ 1+ a ⎠ =

1 + a + ib + i 2 c

1 + a − ib − i 2 c (1 + a − c) + ib = (1 + a + c) − ib

[∵i 2 = −1]

Complex Numbers

1.9

(a + ib) [(1 + a + c) ib] = (1 + c) [(1 + a c) + ib] a (1 + a + c) i ab + ib (1 + a + c) i2 b2 = 1 + a c + c + ac c2 + ib (a + a2 + ac + b2) + i (b + bc) = (1 + a + ac c2) + ib Comparing real parts on both the sides, a + a2 + ac + b2 = 1 + a + ac c2 a2 + b2 + c2 = 1 π 2π Example 7: Find z if arg ( z + 1) = and arg ( z − 1) = . 6 3 Solution: Let z = x + iy arg (z + 1) = arg (x +iy + 1) =

6 6

arg[( x + 1) + iy ] = tan −1

6 y = x +1 6 y 1 = tan = x +1 6 3

x − y 3 = −1

Also,

2 3 2 arg ( x + iy − 1) = 3 2 arg [( x − 1) + iy ] = 3 y 2 tan −1 = x −1 3 y 2 = tan =− 3 x −1 3 arg ( z − 1) =

x 3+y= 3 Solving Eqs. (1) and (2), x=

1 , 2

y=

3 2

⎛z+i⎞ π Example 8: Find z if |z + i| = |z| and arg ⎜ ⎟= . ⎝ z ⎠ 4 Solution: We have |z + i| = |z| z+i =1 z

1.10

Engineering Mathematics

⎡ z1 z1 ⎤ = ⎢∵ ⎥ z2 ⎥⎦ ⎢⎣ z2 ⎛ z +i ⎞ arg ⎜ ⎟= ⎝ z ⎠ 4

z +i =1 z Also, Let

z +i = rei z z +i =1 z

where,

r=

and

⎛ z +i ⎞ π θ = arg ⎜ ⎟= ⎝ z ⎠ 4 iπ

⎛ z +i ⎞ iθ 4 ⎜ z ⎟ = re = 1.e ⎝ ⎠

Hence,



z + i = ze 4 iπ ⎛ z ⎜1 − e 4 ⎜ ⎝

−i

z=

1− e

ip 4



1− e 1− e

⎞ ⎟ = −i ⎟ ⎠

ip − 4 −

ip 4

ip − ⎞ ⎛ −i ⎜1 − e 4 ⎟ ⎝ ⎠

=

ip

1− e 4 − e



ip 4

i ⎛ − −i ⎜1 − e 4

=

⎜ ⎝

2 − 2 cos

=

⎞ ⎟ ⎟ ⎠

+1

[∵ ei + e −i = 2 cos ]

4

⎛ ⎞ −i ⎜ 2 sin 2 + i 2 sin cos ⎟ 8 8 8 ⎝ ⎠

⎛ ⎞ −i ⎜1 − cos + i sin ⎟ 4 4 ⎝ ⎠

= ⎛ ⎞ ⎛ ⎞ 2 ⎜ 2 sin 2 ⎟ 2 ⎜1 − cos ⎟ 8⎠ 4⎠ ⎝ ⎝ 1⎛ 1 ⎞ ⎛ ⎞ = ⎜ −i − i 2 cot ⎟ = ⎜ −i + cot ⎟ 2⎝ 8 ⎠ 2⎝ 8⎠

Example 9: Determine the locus of z if |z - 3| - |z + 3| = 4. Solution: Let z = x + iy |z

3|

|z + 3| = 4 |z

|x + iy

3| = 4 + |z + 3| 3| = 4 + |x + iy + 3|

Complex Numbers

|(x

3) + iy| = 4 + |(x + 3) + iy|

( x − 3) 2 + y 2 = 4 + ( x + 3) 2 + y 2 Squaring both the sides, (x x2 + 9

3)2 + y2 = 16 + (x + 3)2 + y2 + 8 ( x + 3) 2 + y 2 6x + y2 = 16 + x2 + 9 + 6x + y2 + 8 ( x + 3) 2 + y 2 12x = 8 ( x + 3) 2 + y 2

16

(4 + 3x) = 2 ( x + 3) 2 + y 2 Squaring again both the sides, 16 + 9x2 + 24x = 4 (x2 + 9 + 6x + y2) 5x2 4y2 = 20 x2 y 2 − =1 4 5 x2 y 2 Hence, locus of z is − = 1, which represents a hyperbola. 4 5 z+i Example 10: If u = and z = x + iy, then show that z+2 (i) locus of (x, y) is a straight line, if u is real. (ii) locus of (x, y) is a circle, if u is purely imaginary. Find the centre and radius of the circle. Solution:

u= u= = Re (u ) = Im (u ) =

z +i and z = x + iy z+2 x + iy + i x + i ( y + 1) ( x + 2 − iy ) = ⋅ x + iy + 2 ( x + 2) + iy ( x + 2) − iy [ x ( x + 2) + y ( y + 1)] + i [( y + 1)( x + 2) − xy ] ( x + 2) 2 + y 2 x ( x + 2) + y ( y + 1) ( x + 2) 2 + y 2 ( y + 1)( x + 2) − xy 2

( x + 2) + y

2

=

x + 2y + 2 ( x + 2) 2 + y 2

(i) If u is real, then Im (u) = 0 x + 2y + 2 =0 ( x + 2) 2 + y 2 x + 2y + 2 = 0 Hence, locus of (x, y) is x + 2y + 2 = 0, which represents a straight line.

1.11

1.12

Engineering Mathematics

(ii) If u is purely imaginary, then Re (u) = 0 x ( x + 2) + y ( y + 1) =0 ( x + 2) 2 + y 2 x2 + y2 + 2x + y = 0 Hence, locus of (x, y) is x2 + y2 + 2x + y = 0, which represents a circle with centre 5 1⎞ ⎛ at ⎜ −1, − ⎟ and radius unit. 2 2⎠ ⎝ Example 11: If sum and product of two numbers are real, show that the two numbers must be either real or conjugate. Solution: Let z1 = x1 + iy1 and z2 = x2 + iy2 are two complex numbers. Let z1 + z2 = a, where a is real (x1 + iy1) + (x2 + iy2) = a + i · 0 (x1 + x2) + i (y1 + y2) = a + i · 0 Comparing real and imaginary parts on both the sides, x1 + x2 = a y1 + y2 = 0

… (1) … (2)

Let z1 z2 = b, where b is real (x1x2

(x1 + iy1) (x2 + iy2) = b + i · 0 y1y2) + i (x2y1 + x1y2) = b + i · 0

Comparing real and imaginary parts on both the sides, x1x2 y1y2 = b x2y1 + x1y2 = 0

… (3) … (4)

Substituting y2 =

y1 from Eq. (2) in Eq. (4), x2y1 x1y1 = 0 y1(x2 x1) = 0 y1 = 0 or x2 If y1 = 0, then y2 = 0 Hence, z1 = x1 and z2 = x2 If x1 = x2, then z1 = x1 + iy1 and z2 = x1 iy1

x1 = 0, x1 = x2

Hence, z1 and z2 both are either real or conjugate. Example 12: If z1 and z2 are two complex numbers such that |z1 + z2| = |z1 - z2|, prove that the difference of their amplitude is . 2 Solution: Let z1 = x1 + iy1 and z2 = x2 + iy2 are two complex numbers. |z1 + z2| = |z1

z2|

|x1 + iy1 + x2 + iy2| = |x1 + iy1

x2

iy2|

Complex Numbers |(x1 + x2) + i (y1 + y2)| = |(x1

1.13

x2) + i (y1

y2)|

( x1 + x2 ) 2 + ( y1 + y2 ) 2 = ( x1 − x2 ) 2 + ( y1 − y2 ) 2 Squaring both the sides,

x12 + x22 + 2 x1 x2 + y12 + y22 + 2 y1 y2 = x12 + x22 − 2 x1 x2 + y12 + y22 − 2 y1 y2 4x1x2 + 4y1y2 = 0 x1x2 + y1y2 = 0 Now, amp (z1)

amp (z2) = amp (x1 + iy1)

… (1) amp (x2 + iy2)

⎛y ⎞ ⎛y ⎞ = tan −1 ⎜ 1 ⎟ − tan −1 ⎜ 2 ⎟ ⎝ x1 ⎠ ⎝ x2 ⎠ ⎛ y1 y2 − ⎜ x x −1 1 2 = tan ⎜ y y 1 ⎜1 + ⋅ 2 ⎜⎝ x1 x2

⎞ ⎟ ⎛x y −x y ⎞ ⎟ = tan −1 ⎜ 2 1 1 2 ⎟ ⎝ x1 x2 + y1 y2 ⎠ ⎟ ⎟⎠

⎛x y −x y ⎞ = tan −1 ⎜ 2 1 1 2 ⎟ ⎝ ⎠ 0

[Using Eq. (1)]

p 2 Hence, the difference of amplitude of z1 and z2 is = tan −1 ( ∞) =

Example 13: Show that

2

.

z − 1 ≤ arg ( z ) . z

Solution: Let z = reiq, where |z| = r and arg (z) = q z rei −1 = − 1 = |eiq z r = |cos q + i sin q = −2 sin 2 = 2 sin ≤2

q 2

q 2

arg( z )

1| 1| = |cos q

1+ i sin q |

q q q q q q + i 2 sin cos = 2 sin − sin + i cos 2 2 2 2 2 2 sin 2

q q q + cos 2 = 2 sin 2 2 2 ⎡ sin ⎢⎣∵

⎤ ≤ 1⎥ ⎦

1.14

Engineering Mathematics

π α Example 14: If sin ` = i tan p, prove that cos p + i sin p = tan ⎛⎜ + ⎞⎟ . ⎝4 2⎠ Solution: i tan q = sin i sin θ sin α = 1 cos θ Applying componendo—dividendo, cos q + i sin q 1 + sin a = cos q − i sin q 1 − sin a iq

e = e − iq

e 2iq =

⎛p ⎞ 1 + cos ⎜ − a ⎟ ⎝2 ⎠ ⎛p ⎞ 1 − cos ⎜ − a ⎟ ⎝2 ⎠ a⎞ ⎛ 2 cos 2 ⎜ p − ⎟ ⎝4 2⎠ a⎞ ⎛ 2 sin 2 ⎜ p − ⎟ ⎝4 2⎠

⎡ ⎛ p a ⎞⎤ (e iq ) 2 = ⎢cot ⎜ − ⎟ ⎥ ⎣ ⎝ 4 2 ⎠⎦

2

⎡p ⎛ p a ⎞⎤ ⎛p a ⎞ e iq = cot ⎜ − ⎟ = tan ⎢ − ⎜ − ⎟ ⎥ ⎝4 2⎠ ⎣ 2 ⎝ 4 2 ⎠⎦ ⎛p a ⎞ = tan ⎜ − ⎟ ⎝4 2⎠ ⎛p a ⎞ cos q + i sin q = tan ⎜ − ⎟ ⎝4 2⎠ -

ip

Example 15: Prove that (1 - e ) i

Solution: (1 − e )



1 2

+ (1 − e

−i

)



1 2

1 2

+ (1 - e

- ip

-

)

q q q ⎛ = ⎜ 2 sin 2 − i 2 sin cos ⎟ ⎝ 2 2 2⎠

q⎞ ⎛ = ⎜ 2 sin ⎟ ⎝ 2⎠



1 2



1 2

1

p ⎞2 ⎛ = ⎜ 1 + cosec ⎟ . 2⎠ ⎝

= (1 − cos − i sin ) 1 − ⎞ 2

q⎞ ⎛ = ⎜ 2 sin ⎟ ⎝ 2⎠

1 2



1 2

+ (1 − cos + i sin )



q q q⎞ ⎛ + ⎜ 2 sin 2 + i 2 sin cos ⎟ ⎝ 2 2 2⎠

1 2



1 2

1 1 ⎡ − − ⎤ 2 2 q q q q ⎛ ⎞ ⎛ ⎞ ⎢ sin − i cos + ⎜ sin + i cos ⎟ ⎥ ⎟ ⎢⎣ ⎜⎝ ⎥⎦ ⎠ ⎝ ⎠ 2 2 2 2

1 1 − ⎤ − ⎡ 2 2 ⎢⎧cos ⎛ p − q ⎞ − i sin ⎛ p − q ⎞ ⎫ + ⎧cos ⎛ p − q ⎞ + i sin ⎛ p − q ⎞ ⎫ ⎥ ⎨ ⎬ ⎨ ⎬ ⎜ ⎟ ⎜ ⎟ ⎜ ⎟ ⎜ ⎟ ⎢⎩ ⎝ 2 2 ⎠ ⎝ 2 2 ⎠⎭ ⎝ 2 2 ⎠⎭ ⎥ ⎩ ⎝ 2 2⎠ ⎢⎣ ⎥⎦

Complex Numbers

q⎞ ⎛ = ⎜ 2 sin ⎟ ⎝ 2⎠



1 2

q⎞ ⎛ = ⎜ 2 sin ⎟ ⎝ 2⎠



1 2

q⎞ ⎛ = ⎜ 2 sin ⎟ ⎝ 2⎠



1 2

1.15

1 1 1 − ⎤ − ⎡ − ⎛p q ⎞ ⎛p q ⎞ ⎛p q ⎞ ⎛p q ⎞ 2 2 ⎧ ⎫ ⎫ ⎧ − − i i −i ⎜ − ⎟ ⎤ − ⎜ ⎟ ⎜ ⎟ q ⎞ 2 ⎡ i ⎜⎝ 4 − 4 ⎟⎠ ⎪ ⎝ 2 2⎠ ⎪ ⎥ ⎛ ⎢⎪ ⎝ 2 2 ⎠ ⎪ ⎝ 4 4⎠ e + e 2 sin e e = + ⎢ ⎥ ⎬ ⎨ ⎬ ⎥ ⎜ ⎟⎠ ⎢⎨ ⎝ 2 ⎢ ⎥⎦ ⎪ ⎪ ⎪ ⎪ ⎩ ⎭ ⎩ ⎭ ⎣ ⎢⎣ ⎥⎦

⎡ q⎞ ⎛ p q ⎞⎤ ⎛ ⎢ 2 cos ⎜⎝ 4 − 4 ⎟⎠ ⎥ = ⎜⎝ 2 sin 2 ⎟⎠ ⎦ ⎣



1 2

1

⎡ q ⎞⎤ 2 2 ⎛p ⎢ 4 cos ⎜⎝ 4 − 4 ⎟⎠ ⎥ ⎦ ⎣

1

⎡ 2 ⎤ ⎢∵1 + cos = 2 cos 2 ⎥ ⎣ ⎦

⎡ ⎧ ⎛ p q ⎞ ⎫⎤ 2 ⎢ 2 ⎨1 + cos ⎜⎝ − ⎟⎠ ⎬⎥ 2 2 ⎭⎦ ⎣ ⎩

1

q ⎤2 ⎡ 1 ⎢1 + sin 2 ⎥ q ⎤2 ⎡ =⎢ ⎥ = ⎢cosec + 1⎥ 2 ⎦ ⎣ ⎢ sin q ⎥ ⎣ 2 ⎦ Example 16: If a = cos ` + i sin ` and b = cos a + i sin a, then show that (a + b )(ab − 1) sinα + sinβ . = (a − b )(ab + 1) sinα − sinβ

a = cos a + i sin a = eia, b = cos b + i sin b = eib

Solution:

( a + b)( ab − 1) (e ia + e ib )(e ia e ib − 1) = ( a − b)( ab + 1) (e ia − e ib )(e ia e ib + 1) =

(e 2ia e ib + e 2ib e ia − e ia − e ib ) e − i ( b +a ) ⋅ (e 2ia e ib − e 2ib e ia + e ia − e ib ) e − i ( b +a )

e ia + e ib − e − ib − e − ia (e ia − e − ia ) + (e ib − e − ib ) = e ia − e ib + e − ib − e − ia (e ia − e − ia ) − (e ib − e − ib ) 2i sin a + 2i sin b sin a + sin b = = 2i sin a − 2ii sin b sin a − sin b

=

Example 17: If a = cos ` + i sin `, b = cos a + i sin a, c = cos f + i sin f , then prove that

(b + c )(c + a )(a + b ) ⎛ β −γ = 8cos ⎜ abc ⎝ 2

⎞ ⎛ γ −α ⎞ ⎛α − β ⎞ ⎟ cos ⎜ 2 ⎟ cos ⎜ 2 ⎟ . ⎠ ⎝ ⎠ ⎝ ⎠

Solution: a = cos a + i sin a = eia, b = cos b + i sin b = e ib, c = cos g + i sin g = e ig, (b + c)(c + a)( a + b) (e ib + e ig )(e ig + e ia )(e ia + e ib ) = abc e ia e ib e ig =

e ib + e ig eig + e ia eia + eib ⋅ ig ia ⋅ ia ib i b ig e2e2 e2e2 e2e2

ia ia ⎡ ia ⎤ 2 2 ⎢∵ e = e e etc.⎥ ⎣ ⎦

1.16

Engineering Mathematics

− i ( b −g ) − i (g −a ) − i (a − b ) ⎡ i ( b −g ) ⎤ ⎡ i (g −a ) ⎤ ⎡ i (a − b ) ⎤ = ⎣e 2 + e 2 ⎦ ⎣e 2 + e 2 ⎦ ⎣e 2 + e 2 ⎦

⎛ b −g ⎞ ⎛g − a ⎞ ⎛a − b ⎞ 2 cos ⎜ 2 cos ⎜ = 2 cos ⎜ ⎝ 2 ⎟⎠ ⎝ 2 ⎟⎠ ⎝ 2 ⎟⎠ ⎛ b −g ⎞ ⎛g − a cos ⎜ = 8 cos ⎜ ⎝ 2 ⎟⎠ ⎝ 2

⎞ ⎛a − b ⎞ ⎟⎠ cos ⎜⎝ ⎟ 2 ⎠

Example 18: If ` = i + 1, a = 1 - i and tanφ = ( x + ` )n - ( x + a )n = sin ne cosec ne . ` -a

1 x +1 cot f = x + 1, x = cot f

Solution: a = i + 1, b = 1

1 , then prove that x +1

i, tan =

1

(x + α ) − (x + β ) (cot φ − 1 + i + 1) − (cot φ − 1 + 1 − i ) n = α −β i +1−1+ i n

n

n

n

n

⎛ cos φ ⎞ ⎛ cos φ ⎞ +i⎟ −⎜ −i⎟ ⎜ ⎝ sin φ ⎠ ⎝ sin φ ⎠ = 2i (cos f + i sin f ) n − (cos f − i sin f ) n = 2i sin n f =

(e if ) n − (e − if ) n e inf − e − inf 2i sin nf = = 2i sin n f 2i sin n f 2i sin n f

= sin nf cosec n f Example 19: If (1 + cos p + i sin p ) (1 + cos 2p + i sin 2p ) = u + iv, prove that θ v 3 (i) u2 + v 2 = 16 cos 2 cos 2θ (ii) . = tan 2 u 2 Solution: u + iv = (1 + cos q + i sin q ) (1 + cos 2q + i sin 2q ) q q q⎞ ⎛ = ⎜ 2 cos 2 + i 2 sin cos ⎟ (2 cos 2 q + i 2 sin q cos q ) 2 2 2⎠ ⎝

= 2 cos

q⎛ q q⎞ cos + i sin ⎟ 2 cos q (cosq + i sin q ) 2 ⎜⎝ 2 2⎠

= 4 cos

i q cos q ⋅ e 2 ⋅ eiq 2

= 4 cos

q cosq ⋅ e 2

q

= reif

i 3q 2

Complex Numbers

r = u + iv = 4 cos

where,

u 2 + v 2 = 4 cos

2

u 2 + v 2 = 16 cos 2

tan −1

cos

cos

2

cos 2

φ = arg(u + iv) =

and

2

1.17

3θ 2

v 3 = u 2 v 3 = tan u 2

Example 20: If (a1 + ib1)(a2 + ib2) . . . . . (an + ibn) = A + iB, prove that (i) (a12 + b12 )(a22 + b22 )…… (an2 + bn2 ) = A2 + B 2

⎛b (ii) tan −1 ⎜ 1 ⎝ a1

⎞ −1 ⎛ b2 ⎟ + tan ⎜ ⎠ ⎝ a2

⎞ −1 ⎛ bn ⎞ −1 ⎛ B ⎞ ⎟ + ...... + tan ⎜ ⎟ = tan ⎜ ⎟ . ⎝ A⎠ ⎠ ⎝ an ⎠

Solution: (a1 + ib1) (a2 + ib2) . . . . . (an + ibn) = A + iB (i) Taking modulus of Eq. (1) on both the sides, |(a1 + ib1)(a2 + ib2) . . . . . (an + ibn)| = |A + iB | |a1 + ib1| |a2 + ib2| . . . . . |an + ibn| = | A + iB | a12 + b12 a22 + b22

an2 + bn2 =

… (1)

A2 + B 2

Squaring both the sides, (a12 + b12 )(a22 + b22 )

(an2 + bn2 ) = A2 + B 2

(ii) Taking argument of Eq. (1) on both the sides, Arg [(a1 + ib1) (a2 + ib2) . . . . . . (an + ibn)] = Arg (A + iB) Arg (a1 + ib1) + Arg (a2 + ib2) + . . . . . . + Arg (an + ibn) = Arg (A + iB) ⎛b ⎞ ⎛b tan −1 ⎜ 1 ⎟ + tan −1 ⎜ 2 ⎝ a1 ⎠ ⎝ a2 Example 21: If

⎞ −1 ⎛ bn ⎟ + .... + tan ⎜ ⎠ ⎝ an

⎞ −1 ⎛ B ⎞ ⎟ = tan ⎜ ⎟ ⎝ A⎠ ⎠

1 1 + = 1, where ` , a , a and b are real, express b in ` + i a a + ib

terms of ` and a. Solution:

1 1 + =1 a + i b a + ib 1 1 a + ib − 1 = 1− = a + ib a + ib a + ib

1.18

Engineering Mathematics a + ib a + ib (a − 1) − i b = ⋅ (a − 1) + i b (a − 1) + i b (a − 1) − i b

a + ib = =

a (a − 1) − i 2 b 2 + i b (a − 1) − iab (a − 1) 2 + b 2

=

a (a − 1) + b 2 b −i 2 2 (a − 1) + b (a − 1) 2 + b 2

Comparing the imaginary part on both the sides, b=

Example 22: If x Solution:

iy = 3 a

−β (α − 1) 2 + β 2

ib , prove that

a b + = 4( x 2 − y 2 ). x y

x + iy = 3 a + ib , 1

(a + ib) 3 = x + iy a + ib = ( x + iy )3 = x3 + i 3 y 3 + 3 x 2 iy + 3 xi 2 y 2 [ i 3 = −i ]

= ( x3 − 3 xy 2 ) + i (3 x 2 y − y 3 ) Comparing real and imaginary parts on both the sides,

Hence,

a = x3 3xy2 and b = 3x2y y3 a b = x 2 − 3 y 2 and = 3 x 2 − y 2 x y a b + = 4( x 2 − y 2 ) x y

⎛π Example 23: If xr = cos ⎜ r ⎝2

⎛π ⎞ ⎟ + i sin ⎜ r ⎝2 ⎠

⎞ x1 x2 x3 ......xn = -1. ⎟ , show that nlim ãÇ ⎠ i

xr = cos

Solution:

2r i

lim x1 ⋅ x2 ⋅ x3

n →∞

+ i sin

2r

i

i 2

lim x1 ⋅ x2 ⋅ x3

e2

n →∞

⎛1 1 1 i ⎜ + 2+ 3+ ⎝2 2 2

n →∞

⋅⋅⋅ xn = lim

n →∞

⎡ ⎛ 1 ⎞n ⎤ i ⎢1−⎜ ⎟ ⎥ e ⎢⎣ ⎝ 2 ⎠ ⎥⎦

r

i 3

⋅ xn = lim e 2 ⋅ e 2 ⋅ e 2 = lim e

n →∞

= e2

+

1 ⎞ ⎟ 2n ⎠

n

Complex Numbers

⎡ ⎢ 1 ⎢ 1 1 ⎢∵ 2 + 2 + 3 + 2 2 ⎢ ⎣

1⎡ ⎛1⎞ ⎢1 − 2 ⎢⎣ ⎜⎝ 2 ⎟⎠ 1 ⋅⋅+ n = 1 2 1− 2

1.19

⎤ ⎥ n⎥ ⎥⎦ ⎛1⎞ ⎥ = 1− ⎜ ⎟ ⎥ ⎝2⎠ ⎥ ⎦

n⎤

= lim e ip e

⎛1⎞ − ip ⎜ n ⎟ ⎝2 ⎠

n→∞

= e ip e

= (cos + i sin )e





ip 2∞

i ∞

= (−1 + i.0)e0 Hence,

lim x1 ⋅ x2 ⋅ x3 ..... xn = −1

n →∞

Example 24: Prove that e 2 ai cot Solution:

e 2 ai cot

−1

b

b

⎛ bi − 1 ⎞ ⎜⎝ ⎟ bi + 1 ⎠

−a

= 1.

−a

=1

bi − 1 bi + i 2 b + i = = bi + 1 bi − i 2 b − i i = re iq

Let b + i = reiq, then b r = |b + i| =

⎛ bi − 1 ⎞ ⎜ ⎟ ⎝ bi + 1 ⎠

−1

b 2 + 1 and q = arg (b + i) = tan

1

1 = cot 1b b

−1 bi − 1 rei = −i = e 2i = e 2i cot b bi + 1 re Substituting in the given equation,

e 2 ai cot Hence,

e

−1

b

2 ai cot −1 b

⎛ bi − 1 ⎞ ⎜ ⎟ ⎝ bi + 1 ⎠ ⎛ bi − 1 ⎞ ⎜ ⎟ ⎝ bi + 1 ⎠

−a

= e 2 ai cot

−1

b

(e 2i cot

−1

b −a

)

= e0 = 1

−a

= 1.

Exercise 1.1 1. Find the modulus and principal value of the argument 1 + 2i 1 + 2i (i) 3 i (ii) (iii) 1 − 3i 1 − (1 − i ) 2 1+ i (iv) (v) tan a i. 1 i

−5p 1 3p ⎡ ⎢ Ans. : (i) 2, 6 (ii) 2 , 4 ⎢ p ⎢ (iii) 1, 0 (iv) 1, ⎢ 4 ⎢ p ⎢ ( v) sec a , a − ⎢⎣ 2

⎤ ⎥ ⎥ ⎥ ⎥ ⎥ ⎥ ⎥⎦

1.20

Engineering Mathematics

2. Express in polar form (i) (iii)

3 i

1+ i 1− i

(ii)

2 + 6 3i 5 + 3i

.

⎡ ⎛ ⎞⎤ ⎢ Ans. : (i) 2 ⎜ cos − i sin ⎟ ⎥ 6 6⎠⎥ ⎝ ⎢ ⎢ ⎥ (ii) cos − i sin ⎢ ⎥ 2 2 ⎢ ⎥ ⎢ ⎛ ⎞⎥ (iii) 2 ⎜ cos − i sin ⎟ ⎥ ⎢ 3 3 ⎠ ⎥⎦ ⎝ ⎢⎣ 3. Find the value of

3 4i. i or 2 + i]

[Ans. : 2 4. Find z if arg ( z + 2i ) = arg ( z − 2i ) =

4

,

3 . 4 [Ans. : 2]

5. Find the locus of z, if

z 1 is purely z +i

imaginary. [Ans. : circle x2 + y2

x

y = 0]

6. Find the locus of z if (i)

z 1 =1 z +1

(ii) arg

z 1 = . z +1 4

⎤ ⎡ Ans. : (i) x = 0 ⎥ ⎢ 2 2 ⎣(ii) circle x + y − 2 y − 1 = 0 ⎦ 7. Find two numbers whose sum is 4 and product is 8. [Ans. : 2 ± 2i] 8. If x + iy = a + ib , prove that (x2 + y2)2 = a2 + b2. ⎡ Hint : square both the sides and ⎤ ⎢ ⎥ then take modulus ⎣ ⎦ z 1 9. If |z| = 1, z ó 1, prove that is z +1 purely imaginary.

⎡ Hint : z = x + iy, | z | = 1 ∴ x 2 + y 2 = 1, ⎤ ⎢ ⎥ z − 1 ( x − 1) + iy ( x + 1) − iy ⎥ ⎢ = ⋅ ⎢⎣ z + 1 ( x + 1) + iy ( x + 1) − iy ⎥⎦ z 10. If |z1 + z2| = |z1 z2|, prove that 2 is z1 purely imaginary. 11. If a = ei2a, b = ei2b, c = ei2g, then prove ab c 2 cos( ). that c ab 12. If a = cos a + i sin a, b = cos b + i sin b, then prove that 1 a b sin(a ). 2i b a 13. If a = cos a + i sin a, then prove that 2 1 i tan (i) 1+ a 2 1+ a = i cot . (ii) 1 a 2 14. If a = a + ib, b = c + id, 1 , then show that if = +1 a 2 + b2 =

(c 1) 2

d2

(c + 1) 2 + d 2

.

c + id − 1 (c − 1) + id ⎡ ⎢ Hint : a + ib = c + id + 1 = (c + 1) + id ⎢ ⎢ | (c − 1) + id | a + ib = ⎢ | (c + 1) + id | ⎣

⎤ ,⎥ ⎥ ⎥ ⎥ ⎦

15. If x2 + y2 = 1, then prove that 1 + x + iy = x + iy . 1 + x − iy ⎡ Hint : x 2 + y 2 = 1, ( x + iy )( x − iy ) = 1,⎤ ⎢ ⎥ ⎢ ⎥ x + iy 1 = ⎢ ⎥ 1 x − iy ⎢ ⎥ ⎢ ⎥ x + iy 1 = ⎢ Apply dividendo ⎥ 1 + + 1 + − x iy x iy ⎢ ⎥ ⎢⎣ ⎥⎦

Complex Numbers 16. If (1 + ai)(1 + bi)(1 + ci) = p + iq, prove that

19. Find the value of (i) x2 6x + 13, when x = 3 + 2i

(i) p tan [tan 1a + tan 1b + tan 1c] = q (ii) (1 + a2)(1 + b2)(1 + c2) = p2 + q2. 1 17. If (α + i β ) = , prove that a + ib ( 2 + 2) (a2 + b2) = 1. ⎡ 1 ⎤ ⎢ Hint : α + i β = ⎥ a + ib ⎥⎦ ⎢⎣ 18. If a = cos a + i sin a, b = cos b + i sin b, where 0 < , find polar form of


0. 4 ⎝ x− y⎠ π 20. If cot ⎛⎜ + iα ⎞⎟ = x + iy, prove that ⎝6 ⎠ x2 + y2

2x 3

= 1.

π 21. If cot ⎛⎜ + iα ⎞⎟ = x + iy, prove that ⎝8 ⎠ 2 2 x + y – 2x = 1. iπ ⎞ ⎛ 22. If tanh ⎜ α + ⎟ = x + iy, 6⎠ ⎝ prove that x 2 + y 2 +

2y 3

= 1.

23. If tanh (a + ib ) = x + iy, prove that (i) x2 + y2 – 2x coth 2a = 1 (ii) x2 + y2 + 2y cot 2b = 1. ⎡ Hint : tanh (a − i b ) = x − iy, ⎤ ⎢ tanh 2a = tanh [(a + i b ) ⎥ ⎢ ⎥ ⎢ ⎥ + (a − i b )] ⎢ ⎥ ⎢ tanh 2i b = tanh [(a + i b ) ⎥ ⎢ ⎥ − (a − i b )], ⎢ ⎥ ⎢ tanh 2i b = − i tan i (2i b ) ⎥ ⎢ ⎥ = − i tan ( − 2b ) ⎢ ⎥ = i tan 2b ⎢⎣ ⎥⎦ 24. If cot (a + ib ) = x + iy, prove that (i) x2 + y2 – 2x cot 2a = 1 (ii) x2 + y2 + 2y coth 2b + 1 = 0. 25. Separate real and imaginary parts of cos 1 (eiq ). ⎡ Ans. : sin −1 sin q + ⎢ ⎢ i log 1 + sin q − sin q ⎣

(

26. Prove that

(

)

⎤ ⎥ ⎥ ⎦

)

2 sin 1 (ix) = i log x + x + 1 + 2n .

27. Separate into real and imaginary parts ⎛ 5i ⎞ (ii) cos −1 ⎜ ⎟ (i) cos 1 (i) ⎝ 12 ⎠ 3i (iii) sin −1 ⎛⎜ ⎞⎟ (iv) sinh 1 (ix) ⎝4⎠

1.108

Engineering Mathematics

(v) tanh 1 (i). p ⎡ ( )⎤ ⎢ Ans. : (i) 2 + i log 2 − 1 ⎥ ⎥ ⎢ 2 p ⎥ ⎢ (ii) + i log ⎥ ⎢ 3 2 ⎥ ⎢ (iii) i log 2 ⎥ ⎢ ⎥ ⎢ p ip −1 (iv) cosh x + ⎥ ⎢ 2 ⎥ ⎢ ip ⎥ ⎢ (v) ⎥⎥⎦ ⎢⎢⎣ 4 28. Prove that sin (cosec q) =

29. If log cos (x – iy) = a + ib, then prove that 1 ⎛ cosh 2 y + cos 2 x ⎞ = log ⎜ ⎟, 2 2 ⎝ ⎠ tan b = – tan x tanh y. 30. If log sin (x + iy) = a + ib, then prove that =

1 ⎛ cosh 2 y − cos 2 x ⎞ log ⎜ ⎟, 2 2 ⎝ ⎠

tan b = cot x tanh y.

1

2

+ i log cot

. 2 [Hint : Let sin (cosec q ) = a + ib ]

1.13 LOGARITHM OF A COMPLEX NUMBER If z and w are two complex numbers and z = ew, then w = log z is called logarithm of the complex number z. Let z = x + iy = reiq y where r = z = x 2 + y 2 and q = arg (z) = tan 1 x log z = log (r eiq) = log r + log eiq = log r + iq log e = log r + iq = log x 2 + y 2 + i tan −1 y x 1 y log ( x 2 + y 2 ) + i tan −1 x 2 This is called principal value of log (x + iy). The general value of log (x + iy) is given as log (x + iy) = log r + i (2np + q ) 1 y⎞ ⎛ = log ( x 2 + y 2 ) + i ⎜ 2n + tan −1 ⎟ . 2 x⎠ ⎝

Hence,

log ( x + iy ) =

Example 1: Find the value of (i) log i

(ii) log-3 (-2 )

(iii) log (-5)

(iv) log (1 + i).

Complex Numbers

1.109

Solution: (i) log i = 1 log 1 + i tan −1 1 = 0 + i = i 2 0 2 2 1 ⎛ 0 ⎞ log 4 + i tan −1 ⎜ ⎟ ⎝ −2 ⎠ = log 2 + i (ii) log −3 (−2) = log e (−2) = 2 0 ⎞ log 3 + i log e (−3) 1 ⎛ log 9 + i tan −1 ⎜ ⎟ 2 ⎝ −3 ⎠ (iii) log ( −5 ) =

1 ⎛ 0 ⎞ log 25 + i tan −1 ⎜ ⎟ = log 5 + i 2 ⎝ −5 ⎠

(iv) log (1 + i ) =

1 i ⎛1⎞ 1 log 2 + i tan −1 ⎜ ⎟ = log 2 + 2 4 ⎝1⎠ 2

Example 2: Prove that log i i =

4n + 1 , where n, m are integers. 4m + 1

p⎞ 1 ⎛ log 1 + i ⎜ 2np + ⎟ ⎝ log i 2 2⎠ Solution: log i i = = p⎞ log i 1 ⎛ log 1 + i ⎜ 2mp + ⎟ ⎝ 2 2⎠ =

i ( 4n + 1) p

i ( 4m + 1) p

=

4n + 1 4m + 1

Example 3: Prove that log (1 + cos 2p + i sin 2p) = log (2 cos p) + ip. Solution: log (1 + cos 2q + i sin 2q ) =

1 ⎛ sin 2q ⎞ 2 log ⎡⎣(1 + cos 2q ) + sin 2 2q ⎤⎦ + i tan −1 ⎜ ⎝ 1 + cos 2q ⎟⎠ 2

1 ⎛ 2 sin q cos q ⎞ log(4 cos 4 q + 4 sin 2 q cos 2 q ) + i tan −1 ⎜ ⎝ 2 cos 2 q ⎟⎠ 2 1 = log ⎡⎣ 4 cos 2 q (cos 2 q + sin 2 q ) ⎤⎦ + i tan −1 (tan q ) 2 1 = log (4 cos 2 q ) + iq 2 = log (2 cos q ) + iq ⋅

=

Example 4: Simplify log (ei` + eia). Solution: log (eia + eib ) = log [cos a i sin a ) + (cos b + i sin b )] log [(cos a + cos b ) + i (sin a + sin b )]

1.110

Engineering Mathematics

⎡ ⎛ a + b ⎟⎞ ⎛ a − b ⎞⎟⎤ ⎛ a + b ⎞⎟ ⎛ a − b ⎞⎟ ⎜⎜ ⎜⎜ ⎥ cos ⎜ i cos + 2 sin = log ⎢ 2 cos ⎜⎜ ⎟ ⎟ ⎢ ⎜⎝ 2 ⎟⎟⎠ ⎜⎜⎝ 2 ⎟⎟⎠⎥ ⎜⎝ 2 ⎟⎠ ⎜⎝ 2 ⎟⎠ ⎣ ⎦ ⎡ ⎤ ⎛ a − b ⎞⎟ ⎛ a + b ⎞⎟ ⎛ a + b ⎞⎟ ⎥ cos ⎜ = log ⎢ 2 cos ⎜⎜ + i sin ⎜⎜ ⎜⎝ 2 ⎟⎟⎠ ⎜⎜⎝ 2 ⎟⎟⎠ ⎢ ⎜⎝ 2 ⎟⎟⎠⎥ ⎣ ⎦ ⎡ ⎛a + b ⎞ ⎤ ⎛ a − b ⎞⎟ ⎟⎟ + i sin ⎛⎜⎜ a + b ⎞⎟⎟⎥ ⎢ cos ⎜⎜ log = log 2 cos ⎜⎜ + ⎟ ⎢ ⎜⎝ 2 ⎟⎠ ⎜⎝ 2 ⎟⎠⎥ ⎝⎜ 2 ⎟⎠ ⎣ ⎦ ⎛ a + b ⎞⎟ ⎟ 2 ⎟⎠

i ⎜⎜ ⎛ a − b ⎞⎟ ⎜⎝ + log e = log 2 cos ⎜⎜ ⎟ ⎝⎜ 2 ⎟⎠

⎛ a − b ⎞⎟ ⎛ a + b ⎞⎟ + i⎜ = log 2 cos ⎜⎜ ⎜⎝ 2 ⎟⎟⎠ ⎜⎜⎝ 2 ⎟⎟⎠

[ log e = 1]

⎛ x−i ⎞ -1 Example 5: Prove that i log ⎜ ⎟ = o - 2 tan x. x + i ⎝ ⎠ Solution: ⎛ x −i⎞ i log ⎜ = i [ log( x − i ) − log( x + i ) ] ⎝ x + i ⎟⎠ ⎡1 1⎤ ⎛ −1⎞ 1 = i ⎢ log( x 2 + 1) + i tan −1 ⎜ ⎟ − log ( x 2 + 1) − tan −1 ⎥ ⎝ x⎠ 2 x⎦ ⎣2 − 1 − 1 − 1 = i ⎡⎣i ( − cot x − cot x) ⎤⎦ = 2 cot x ⎛p ⎞ = 2 ⎜ − tan −1 x⎟ = p − 2 tan −1 x ⎝2 ⎠ Example 6: Simplify tanh-1 (x + iy). Solution: tanh −1 ( x + iy ) =

1 ⎛ 1 + x + iy ⎞ 1 log ⎜ = [log (1 + x + iy ) − log (1 − x − iy ) ] 2 ⎝ 1 − x − iy ⎟⎠ 2

{

}

{

1 ⎡1 y 1 log (1 + x) 2 + y 2 + i tan −1 − log (1 − x) 2 + y 2 2 ⎢⎣ 2 1+ x 2 −y ⎤ −i tan −1 1 − x ⎥⎦ ⎤ (1 + x) 2 + y 2 1 ⎡1 ⎛ −1 y −1 y ⎞ = ⎢ log + + i tan tan ⎥ ⎜ ⎟ ⎝ 2 ⎣⎢ 2 1+ x 1 − x ⎠ ⎦⎥ (1 − x) 2 + y 2 ⎡∵ tan −1 ( − x) = − tan −1 x ⎤ ⎣ ⎦ =

}

Complex Numbers

1.111

⎡ y y ⎞⎤ ⎛ ⎢ ⎥ + 2 2 1 1 (1 + x) + y ⎜ −1 1 + x 1 − x ⎟ ⎥ = ⎢ log + i tan ⎜ ⎟ 2 ⎢2 y2 ⎟ ⎥ (1 − x) 2 + y 2 ⎜⎜ 1− ⎢ ⎟⎥ ⎝ ⎢⎣ 1 − x 2 ⎠ ⎥⎦ ⎤ 1 ⎡1 2y (1 + x) 2 + y 2 = ⎢ log + i tan −1 2 2 2 2⎥ 2 ⎣2 1− x − y ⎦ (1 − x) + y p ⎞ ⎛o p ⎞ ⎛ 1 ⎞ ⎛1 Example 7: Prove that log ⎜ = log ⎜ cosec ⎟ + i ⎜ − ⎟ . ip ⎟ ⎝1− e ⎠ ⎝2 2⎠ ⎝ 2 2⎠ Solution: ⎛ 1 ⎞ log ⎜ = log (1 − eiq ) −1 = − log(1 − eiq ) ⎝ 1 − eiq ⎟⎠ q q q⎞ ⎛ = − log (1 − cos q − i sin q ) = − log ⎜ 2 sin 2 − 2i sin cos ⎟ ⎝ 2 2 2⎠ ⎡ ⎛ q⎞⎛ q q ⎞⎤ = − ⎢log ⎜ 2 sin ⎟ ⎜ sin − i cos ⎟ ⎥ ⎝ ⎠ ⎝ 2 2 2⎠⎦ ⎣ ⎡ ⎛p q ⎞ q⎞ ⎛p q ⎞⎤ ⎛ = − log ⎜ 2 sin ⎟ − log ⎢cos ⎜ − ⎟ − i sin ⎜ − ⎟ ⎥ ⎝ ⎠ ⎝ 2 2⎠⎦ ⎝ ⎠ 2 2 2 ⎣ q⎞ ⎛ = log ⎜ 2 sin ⎟ ⎝ 2⎠

−1

− log e

⎛p q ⎞ −i⎜ − ⎟ ⎝ 2 2⎠

q ⎞ ⎛p q ⎞ ⎛1 = log ⎜ cosec ⎟ + i ⎜ − ⎟ ⎝2 2⎠ ⎝ 2 2⎠ ⎛ π ix ⎞ Example 8: Prove that log tan ⎜ + ⎟ = i tan-1 (sinh x). ⎝4 2⎠ ix ⎛ ⎜ 1 + tan 2 ix ⎞ ⎛ Solution: log tan ⎜ + ⎟ = log ⎜ ⎝4 2⎠ ⎜⎜ 1 − tan ix 2 ⎝ x ⎛ ⎜ 1 + i tanh 2 = log ⎜ ⎜⎜ 1 − i tanh x 2 ⎝ 1 ⎛ = log ⎜ 1 + tanh 2 2 ⎝

⎞ ⎟ ⎟ ⎟⎟ ⎠

⎞ ⎟ x⎞ x⎞ ⎛ ⎛ ⎟ = log ⎜1 + i tanh ⎟ − log ⎜1 − i tanh ⎟ 2⎠ 2⎠ ⎝ ⎝ ⎟⎟ ⎠ x⎞ x⎞ x⎞ x⎞ 1 ⎛ ⎛ ⎛ + i tan −1 ⎜ tanh ⎟ − log ⎜1 + tanh 2 ⎟ − i tan −1 ⎜ − tanh ⎟ 2 ⎟⎠ 2 2 2 2 ⎝ ⎠ ⎝ ⎠ ⎝ ⎠

1.112

Engineering Mathematics

x x ⎛ ⎜ tanh 2 + tanh 2 ⎡ −1 ⎛ x⎞ x ⎞⎤ −1 ⎛ −1 = i ⎢ tan ⎜ tanh ⎟ + tan ⎜ tanh ⎟ ⎥ = i tan ⎜ 2⎠ 2 ⎠⎦ ⎝ ⎝ ⎣ ⎜⎜ 1 − tanh 2 x 2 ⎝ x ⎞ ⎛ ⎜ 2 tanh 2 ⎟ −1 −1 = i tan ⎜ ⎟ = i tan (sinh x) x ⎜⎜ 1 − tanh 2 ⎟⎟ 2⎠ ⎝

⎞ ⎟ ⎟ ⎟⎟ ⎠

⎡ sin( x + iy ) ⎤ -1 Example 9: Prove that log ⎢ ⎥ = 2i tan (cot x tanh y). ⎣ sin( x − iy ) ⎦ Solution:

⎡ sin ( x + iy ) ⎤ log ⎢ ⎥ = log [sin (x + iy)] – log [sin (x – iy)] ⎣ sin ( x − iy ) ⎦ log (sin x cosh y i cos x sinh y) – log (sin x cosh y – i cos x sinh y) =

⎛ cos x sinh y ⎞ 1 log ( sin 2 x cosh 2 y + cos 2 x sinh 2 y ) + i tan −1 ⎜ ⎟ 2 ⎝ sin x cosh y ⎠ ⎛ − cos x sinh y ⎞ 1 − log ( sin 2 x cosh 2 y + cos 2 x sinh 2 y ) − i tan −1 ⎜ ⎟ 2 ⎝ sin x cosh y ⎠ i tan 1 (cot x tanh y) + i tan 1 (cot x tanh y) 2i tan 1 (cot x tanh y).

⎡ a + ib ⎞ ⎤ a 2 − b 2 Example 10: Prove that cos ⎢ i log ⎛⎜ = . ⎝ a − ib ⎟⎠ ⎥⎦ a 2 + b 2 ⎣ Solution: ⎡ ⎛ a + ib ⎞ ⎤ cos ⎢i log ⎜ = cos[i log (a + ib) − i log (a − ib)] ⎝ a − ib ⎟⎠ ⎥⎦ ⎣ ⎡1 b 1 ⎛ −b ⎞ ⎤ = cos i ⎢ log (a 2 + b 2 ) + i tan −1 − log (a 2 + b 2 ) − i tan −1 ⎜ ⎟ ⎥ ⎝ a ⎠⎦ 2 a 2 ⎣ b⎞ b b⎞ b⎞ ⎛ ⎛ ⎛ = cos i ⎜ i tan −1 + i tan −1 ⎟ = cos ⎜ −2 tan −1 ⎟ = cos ⎜ 2 tan −1 ⎟ ⎝ ⎠ ⎝ ⎝ a⎠ a a a⎠ = cos 2q , where tan q =

b a

b2 2 2 1 − tan q a2 = a − b = = b2 a 2 + b2 1 + tan 2 q 1+ 2 a 2

1−

Complex Numbers

1.113

2ab a − ib ⎞ ⎛ Example 11: Prove that tan ⎜ i log . ⎟= ⎝ a + ib ⎠ a 2 − b2 Solution: ⎛ a − ib ⎞ ⎤ ⎡ log ⎛⎜ a − ib ⎞⎟ − log ⎜ ⎟ ⎝ a + ib ⎠ ⎢ − e ⎝ a + ib ⎠ ⎥ a − ib ⎞ a − ib ⎞ e ⎛ ⎛ = = tan ⎜ i log i tanh log i ⎥ ⎢ ⎜⎝ ⎟ ⎟ a − ib ⎛ a − ib ⎞ ⎝ a + ib ⎠ a + ib ⎠ − log ⎜ ⎥ ⎢ log ⎛⎜⎝ a + ib ⎞⎟⎠ ⎝ a + ib ⎟⎠ +e ⎥⎦ ⎢⎣ e ⎡ ⎛ a − ib ⎞ ⎛ a + ib ⎞ ⎤ ⎛ a + ib ⎞ ⎤ ⎡ log ⎛⎜ a − ib ⎞⎟ log ⎜ ⎟ ⎢ ⎜⎝ a + ib ⎟⎠ − ⎜⎝ a − ib ⎟⎠ ⎥ ⎢ e ⎝ a + ib ⎠ − e ⎝ a −ib ⎠ ⎥ ⎥ = i⎢ ⎥ = i⎢ ⎛ a − ib ⎞ ⎛ a + ib ⎞ ⎢ ⎛ a − ib ⎞ ⎛ a + ib ⎞ ⎥ log ⎜ ⎢ log ⎜⎝ a + ib ⎟⎠ ⎥ ⎟ ⎢ ⎜⎝ a + ib ⎟⎠ + ⎜⎝ a − ib ⎟⎠ ⎥ + e ⎝ a − ib ⎠ ⎥⎦ ⎢⎣ e ⎦ ⎣

⎡ (a − ib) 2 − (a + ib) 2 ⎤ 2ab ⎡ −2aib ⎤ = i⎢ = 2 = i⎢ 2 2 2⎥ 2⎥ ⎣ a − b ⎦ a − b2 ⎢⎣ (a − ib) + (a + ib) ⎥⎦ Example 12: If tan [log (x + iy)] = a + ib, where a2 + b2 ñ 1, prove that 2a tan ⎡⎣ log( x 2 + y 2 ) ⎤⎦ = . 1 − ( a 2 + b2 ) tan [log (x + iy)] = a + ib log (x + iy) = tan–1 (a + ib) log (x – iy) = tan–1 (a – ib) Adding Eqs. (1) and (2),

Solution:

... (1) ... (2)

log (x + iy) + log (x – iy) = tan–1 (a + ib) + tan–1 (a – ib) a + ib + a − ib 1 − (a + ib)(a − ib) 2a −1

log [ ( x + iy )( x − iy ) ] = tan −1 log ( x 2 + y 2 ) = tan tan ⎡⎣log ( x 2 + y 2 ) ⎤⎦ =

1 − (a 2 + b 2 ) 2a

1 − (a 2 + b 2 )

Example 13: If log [log (x + iy)] = a + ib, prove that y = x tan ⎡⎣ tan b log x 2 + y 2 ⎤⎦ . Solution:

log (x + iy) = ea+ib

1 y log ( x 2 + y 2 ) + i tan −1 = e a (cos b + i sin b) 2 x Comparing real and imaginary parts on both the sides, 1 log ( x 2 + y 2 ) = e a cos b 2

... (1)

1.114

Engineering Mathematics

tan −1

and

y = e a sin b x

... (2)

Dividing Eq. (2) by Eq. (1), tan −1 tan b =

tan −1

y x

1 log ( x 2 + y 2 ) 2

y = tan b log x 2 + y 2 x y = tan tan b log x 2 + y 2 x

(

)

(

)

y = x tan tan b log x 2 + y 2 . Example 14: Separate real and imaginary parts of (i)log1–i (1 + i) (ii) (1 + i)i. Solution: (i) Let x + iy = log1–i (1 + i) =

log (1 + i ) log (1 − i )

ip 1 1 log 2 + i tan −1 1 log 2 + 2 4 2 = = ip 1 1 −1 log 2 + i tan ( −1) log 2 − 2 2 4 2 ip ⎞ ⎛ ip ⎞ ⎛ 2 + i log 2 ⎜⎝ log 2 + ⎟⎠ ⎜⎝ log 2 + ⎟⎠ (log 2) − 2 2 4 = = 2 ip ⎞ ⎛ ip ⎞ ⎛ (log 2) 2 + ⎜⎝ log 2 − ⎟⎠ ⎜⎝ log 2 + ⎟⎠ 2 2 4 Comparing real and imaginary part on both the sides, x=

(log 2) 2 − (log 2) 2 +

2

4 , 2

log 2

y=

(log 2) 2 +

4

2

4

(ii) Let x + iy = (1 + i)i Taking logarithm on both the sides, ⎛1 ⎞ log (x + iy) = i log (1 + i) = i ⎜ log 2 + i tan −1 1⎟ ⎝2 ⎠ i π = log 2 − 2 4 i

x + iy = e 2

log 2 −

e

π 4

= e i log 2 e



π 4

=e



π 4

⎡⎣cos (log 2 ) + i sin (log 2 ) ⎤⎦

1.115

Complex Numbers

Comparing real and imaginary part on both the sides, x=e



y=e



4

cos (log 2 )

4

sin (log 2 )

Example 15: If i`+ia = ` + ia, prove that ` 2 + a 2 = e-(4k+1)oa. Solution: ia+ib = a + ib Taking logarithm on both the sides, (a + ib ) log i = log (a + ib ) p ⎞ pa pb ⎛ log (a + i b ) = (a + i b ) i ⎜ 2kp + ⎟ = i (4k + 1) − (4k + 1) 2⎠ 2 2 ⎝ a + ib = e

i ( 4 k +1)

pa pb − ( 4 k +1) 2 e 2

r = a + ib = e

Then,

a2 + b2 = e

− ( 4 k +1)

− ( 4 k +1)

=e

− ( 4 k +1)

pb pa i ( 4 k +1) 2 e 2

= reiq , say

pb 2

pb 2

a 2 + b 2 = e − ( 4 k +1)pb . Example 16: If i

log (1+i)

= A + iB, prove that one value of A is

−π 2 e 8

Solution: A + iB = ilog (1+i)

⎛π ⎞ cos ⎜ log 2 ⎟ . ⎝4 ⎠

Taking logarithm on both the sides, log (A + iB) = log (1 + i) log i p⎤ ⎛1 ⎞ ip ip ⎡ 1 = ⎜ log 2 + i tan −1 1⎟ ⋅ log (2) + i ⎥ = ⎝2 ⎠ 2 2 ⎢⎣ 2 4⎦ =

p p ip p 2 −p 2 ip ip 1 log 2 − = + log 2 ⋅ log 2 + i 2 ⋅ = 2 4 4 8 8 4 2 2 p2 8

ip

p2 8

⎡ ⎛p ⎛p ⎞ ⎞⎤ ⎢cos ⎜⎝ log 2⎟⎠ + i sin ⎜⎝ log 2⎟⎠ ⎥ 4 4 ⎦ ⎣ Comparing real and imaginary part on both the sides, A + iB = e



⋅e 4

log 2

=e



A=e



2

8

⎛ ⎞ cos ⎜ log 2 ⎟ ⎝4 ⎠

Example 17: By considering only principle value, express (1 + i 3 )1+ i form of (a + ib). Solution: Let a + ib = (1 + i 3 )1+ i

3

3

in the

1.116

Engineering Mathematics

Taking logarithm on both the sides, log (a + ib) = (1 + i 3 ) log (1 + i 3 ) ⎡1 ⎤ log (a + ib) = (1 + i 3 ) ⎢ log (1 + 3) + i tan −1 3 ⎥ ⎣2 ⎦ ip ⎞ ip ⎞ ⎛ ⎛1 = (1 + i 3 ) ⎜ log 4 + ⎟ = (1 + i 3 ) ⎜ log 2 + ⎟ ⎝2 ⎠ ⎝ 3 3⎠ = log 2 − a + ib = e

log 2 −

p 3 ⎛ p⎞ + i ⎜ 3 log 2 + ⎟ ⎝ 3 3⎠ 3

3 i ⎛ 3 log 2 + ⎞ ⎜ 3 ⎟⎠ e⎝ 3

⎡ ⎛ ⎞ ⎛ ⎞⎤ ⎢cos ⎜ 3 log 2 + ⎟ + i sin ⎜ 3 log 2 + ⎟ ⎥ 3⎠ 3 ⎠⎦ ⎝ ⎣ ⎝ Comparing real and imaginary parts on both the sides, =e

3

log 2 −

p 3 3

p⎞ ⎛ cos ⎜ 3 log 2 + ⎟ ⎝ 3⎠

log 2 −

p 3 3

p⎞ ⎛ sin ⎜ 3 log 2 + ⎟ ⎝ 3⎠

a=e b=e

log 2 −

Example 18: Prove that (1 + i tan `)-i = e2mo +` [cos (log cos `) + i sin (log cos `)]. Solution: Let x + iy = (1 + i tan a )

i

Taking logarithm on both the sides, log (x + iy) = i log (1 + i tan a ) ⎡1 ⎤ = −i ⎢ log (1 + tan 2 a ) + i ( 2mp + tan −1 tan a ) ⎥ ⎣2 ⎦ ⎡1 ⎤ = −i ⎢ log sec 2 a + i ( 2mp + a ) ⎥ ⎣2 ⎦ = i log (sec2 a )



1 2

+ (2mp + a ) = i log (cos a ) + (2mp + a )

x + iy = ei log cosα e( 2 mπ +α ) = e 2 mπ +α [cos (log cos α ) + i sin (log cos α )] Hence, (1 + i tan a ) i = e2mp +a [cos (log cos a ) + i sin (log cos a )] Example 19: Prove that if (1 + i tan `)1+i tan a can have only real values, one of them is (sec α )sec

2

β

considering only principle value.

1.117

Complex Numbers

Solution: Let x = (1 + i tan a)1+i tan b, where x is real. Taking logarithm on both the sides, log x = (1 + i tan b ) log (1 + i tan a) ⎡1 ⎤ = (1 + i tan b ) ⎢ log (1 + tan 2 a ) + i tan −1 (tan a ) ⎥ 2 ⎣ ⎦ = (1 + i tan b ) ( log sec a + ia ) = (log sec a − a tan b ) + i (a + tan b log sec a ) Comparing real and imaginary parts on both the sides, log x = log sec a x=e

a tan b

and a + tan b log sec a = 0

and a = tan b log sec a

(log sec a a tan b)

Substituting a in x, x = elog sec a + tan = e(logseca )sec

2

2

b logseca b

2

= elogseca (1+ tan

2

b)

b logseca

2

= (sec a )sec b b 2 tan −1 y a . Example 20: If (a + ib) p = mx+iy, prove that = x log(a 2 + b 2 ) Solution: (a + ib) p = mx+iy = esec

Taking logarithm on both the sides, p log (a + ib) = (x + iy) log m p ⎡1 b⎤ log (a 2 + b 2 ) + i tan −1 ⎥ = x + iy log m ⎢⎣ 2 a⎦ Comparing real and imaginary parts on both the sides, x=

p log (a 2 + b 2 ) 2 log m

p b tan −1 log m a b b 2 tan −1 tan −1 y a a . = = 2 2 x 1 2 2 log(a + b ) log(a + b ) 2 y=

Example 21: If

(1 + i ) x + iy = a + i a , then by considering only principle values, (1 − i ) x − iy

⎛β ⎞ πx prove that tan −1 ⎜ ⎟ = + y log 2. 2 ⎝α ⎠

1.118

Engineering Mathematics

(1 + i ) x + iy (1 − i ) x − iy Taking logarithm on both the sides, Solution: a + i b =

⎡ (1 + i ) x + iy ⎤ log (a + ib ) = log ⎢ = ( x + iy ) log (1 + i ) − ( x − iy ) log (1 − i ) x − iy ⎥ ⎣ (1 − i ) ⎦ ⎛1 ⎞ ⎡1 ⎤ = ( x + iy ) ⎜ log 2 + i tan −1 1⎟ − ( x − iy ) ⎢ log 2 + i tan −1 ( −1) ⎥ ⎝2 ⎠ ⎣2 ⎦

1 β iπ ⎛ log (α 2 + β 2 ) + i tan −1 = ( x + iy ) ⎜ log 2 + ⎝ 2 4 α

iπ ⎞ ⎛ ⎞ ⎟⎠ − ( x − iy ) ⎜⎝ log 2 − ⎟⎠ 4

p y⎞ ⎛ p x⎞ ⎛ = ⎜ x log 2 − ⎟ + i ⎜ y log 2 + ⎟ ⎝ 4⎠ ⎝ 4⎠ p y⎞ ⎛ p x⎞ ⎛ − ⎜ x log 2 − ⎟ + i ⎜ y log 2 + ⎟ ⎝ 4⎠ ⎝ 4⎠ p x⎞ ⎛ = 2i ⎜ y log 2 + ⎟ ⎝ 4⎠ Comparing imaginary part on both the sides, px px b tan −1 = 2 y log 2 + = y log 2 + . a 2 2 Example 22: Separate real and imaginary parts of ( i ) i . Solution: Let a + ib = i Taking logarithm on both the sides, log (a + ib) = log i =

1 ip 1 log i = ⋅ 2 2 2

i

a + ib = e 4 i=

Hence, Let ( i )

i

i e4

= x + iy

Taking logarithm on both the sides, i log i = log ( x + iy ) ip e4

ip

log e 4 = log ( x + iy )

p p ⎞ ip ⎛ 1 i ⎞ ip ip p ⎛ log ( x + iy ) = ⎜ cos + i sin ⎟ ⋅ = + = − ⎟ ⎝ 4 4 ⎠ 4 ⎜⎝ 2 2⎠ 4 4 2 4 2

1.119

Complex Numbers

−p

ip

x + iy = e 4 2 e 4

2

=e



p 4 2

p p ⎞ ⎛ + i sin ⎜⎝ cos ⎟ 4 2 4 2⎠

Comparing real and imaginary parts on both the sides,

x=e y=e





π 4 2

cos

π 4 2

sin

π 4 2 π 4 2 ⎛

i

Example 23: Prove that i i = cos p + i sin p, where p = ( 4n + 1) Solution: Let ii = x + iy Taking logarithm on both the sides, i log i = log (x + iy) p⎞ ⎛ i ⋅ i ⎜ 2mp + ⎟ = log ( x + iy ) ⎝ 2⎠ ( x + iy ) = e ii = e

1⎞ ⎛ − ⎜ 2m+ ⎟ p ⎝ 2⎠ 1⎞ ⎛ − ⎜ 2m+ ⎟ p ⎝ 2⎠

i

cos q + i sin q = i i = ie Taking logarithm on both the sides, log (cos q + i sin q ) = e =

1⎞ ⎛ −⎜ 2 m + ⎟p 2⎠ ⎝



1⎞ ⎛ ⎜⎝ 2 m + ⎟⎠ p 2

log i = e

1⎞ ⎛ −⎜ 2 m + ⎟p 2⎠ ⎛ ⎝ i 2np

⎜ ⎝

1⎞ ⎛ −⎜ 2 m + ⎟p p 2⎠ ⎝ e i (4n + 1)

2

+

p ⎞ 2 ⎟⎠

= if , say

cos q + i sin q = eif eiq = eif Comparing both the sides,

θ =φ = e Example 24: If i i (i) z

2

1⎞ ⎛ −⎜ 2 m + ⎟π 2⎠ ⎝

(4n + 1)

π . 2

= z, where z = x + iy, prove that

= e − ( 4 n + 1)π y , n ∈ I

(ii) tan

1⎞

o − ⎜⎝ 2 m + 2 ⎟⎠ o e . 2

πx y = and x 2 + y 2 = e −π y . 2 x

1.120

Engineering Mathematics

= x + iy ii ix+iy = x + iy Taking logarithm on both the sides, (x + iy) log i = log (x + iy) Solution:

p⎞ ⎛ log ( x + iy ) = ( x + iy ) i ⎜ 2np + ⎟ , n ∈ I ⎝ 2⎠ p⎞ ⎛ ⎛ 4 n + 1⎞ py = i ⎜ 2np + ⎟ x − ⎜ ⎝ ⎝ 2 ⎟⎠ 2⎠ x + iy = e

i ( 4 n +1)

p x − ⎛ 4 n +1⎞ p y ⎜ ⎟ 2e ⎝ 2 ⎠

= reiq , say

⎡ − ⎛⎜ 4 n +1⎞⎟ p y ⎤ r = | x + iy | = | x – iy | = ⎢⎣e ⎝ 2 ⎠ ⎥⎦

Then

⎡ − ⎛⎜ 4 n +1 ⎞⎟p y ⎤ | x − iy | = ⎣⎢e ⎝ 2 ⎠ ⎦⎥

2

2

| z |2 = e − ( 4 n +1) p y.

θ = tan −1

and For n = 0,

tan −1

y x = x 2 y x = tan . x 2

| z |2 = e −

and

y ⎛ 4n + 1 ⎞ = πx x ⎜⎝ 2 ⎟⎠

y

, x 2 + y 2 = e−

y



Exercise 1.8 1. Find the general value of 2. Considering principal values only, (i) log ( i) prove that (ii) log 3 − i log 3 + i i (iii) log2 5 (iv) sin (log i ) log 2 (−3) = . log 2 (v) cos (log i i). ⎡ ⎤ 3. Find the general value of log (1 + i) + p⎞ ⎛ ⎢ Ans. : (i) i ⎜⎝ 2p n − 2 ⎟⎠ ⎥ log (1 – i). ⎢ ⎥ [Ans. : log 2] p⎞ ⎛ ⎢ ⎥ ⎢(ii) log 2 + i ⎜⎝ 2p n − 6 ⎟⎠ ⎥ 4. Find the general value of ⎢ ⎥ 2 log 1 + i 3 + log 1 − i 3 ⋅ ⎢(iii) [(log5 log 2 + 4p mn) ⎥ ⎢ + i ( n log 2 − m log 5)2p ] / [(log 2) 2 + 4p 2 m 2 ]⎥ [Ans. : 2log 2] ⎢ ⎥ ( v) 0 ⎣ (iv) − 1 ⎦

(

)

(

)

(

)

1.121

Complex Numbers

5. Prove that sin loge (i i) = 1. 6. Prove that α −β ⎞ log (eiα − eiβ ) = log ⎛⎜ 2 sin 2 ⎟⎠ ⎝ ⎛ π +α + β ⎞ +i ⎜ ⎟. 2 ⎝ ⎠ 7. Show that log (–log i) = log

2

15. Find the principal value of (x + iy)i and show that it is entirely real if 1 log ( x 2 + y 2 ) is a multiple of p. 2

i . 2

8. Prove that log (1 + i tan a) = log sec a + ia. 9. Prove that log (1 + eiq) = ⎡ ⎛ ⎞⎤ i log ⎢ 2 cos ⎜ ⎟ ⎥ + . ⎝ 2 ⎠⎦ 2 ⎣ 10. Prove that ⎛ 1 ⎞ ⎛1 ⎞ i = log ⎜ sec ⎟ − . log ⎜ i ⎟ 2 2 ⎝ 1+ e ⎠ ⎝ ⎠ 2 11. If sin–1 (x + iy) = log (A + iB), prove that x2

y2

= 1 where A2 + B 2 = e 2u . sin 2 u cos 2 u 12. If (a + ib) p = mx+iy, prove that one of ⎛b⎞ 2 tan −1 ⎜ ⎟ y ⎝a⎠ . the values of is x log(a 2 + b 2 ) 13. Separate i(1–i) into real and imaginary parts. −

p⎞⎤ ⎛ ⎡ ⎜ 2 np + ⎟⎠ 2 ⎥ ⎢ Ans.: ie⎝ ⎥ ⎢ ⎦ ⎣ 14. Considering only the principal values, separate real and imaginary parts of ( x + iy )α + iβ . ( x − iy )α −iβ

⎡ Ans.: cos 2q + i sin 2q , where ⎤ ⎢ ⎥ ⎢q = a tan −1 y + b log x 2 + y 2 ⎥ ⎢⎣ ⎥⎦ x

1 ⎡ ⎤ 2 2 ⎢ Hint : put 2 log ( x + y ) = n ⎥ ⎣ ⎦ ⎤ ⎡ ⎛ y⎞ −1 ⎢ Ans.: e tan ⎜⎝ x ⎟⎠ [cos log ( x 2 + y 2 ) ⎥ ⎥ ⎢ ⎢⎣ + i sin log ( x 2 + y 2 ) ⎥⎦ 16. If ia+ib = a + ib, prove that a 2 + b 2 = e–(4n+1)pb .

i

17. If

i

= a + ib, prove that

a +b = e 2

2

πβ 2

.

⎡ Hint : ⎣⎢

( i)

α +iβ

⎤ = α + iβ ⎥ ⎦

x

18. If x = a (cos a + i sin a), prove that the general value of x is given by r (cos q + i sin q ) where (2nπ + α ) sin α + (cos α ) log a log r = a (2nπ + α ) cos α − (sin α ) log a and θ = . a [Hint : xa (cos a + i sin a) = a (cos a + i sin a) = aeia ] ⎡ ( a − b) + i ( a + b) ⎤ 19. Prove that log ⎢ ⎥ ⎣ ( a + b) + i ( a − b) ⎦ 2ab ⎛ = i ⎜ 2n + tan −1 2 a − b2 ⎝

⎞ ⎟. ⎠

[Hint : put a – b = x, a + b = y]

1.122

Engineering Mathematics

FORMULAE Algebra of Complex Numbers (i) Addition: z1 + z2 = (x1 + x2) + i (y1 + y2) (ii) Subtraction: z1 z2 = (x1 x2) + i (y1 y2) (iii) Multiplication: z1 z2 = (x1 x2 y1y2) + i (x2y1 + y2x1) (iv) Division: (y x − x y ) z1 x1 x2 + y1 y2 = 2 + i 1 2 2 12 2 2 z2 x2 + y2 ( x2 + y2 ) Different Forms of Complex Numbers (i) Cartesian or Rectangular Form: z = x + iy (ii) Polar Form: z = r (cos q + i sinq ) =r q (iii) Exponential Form: z = reiq Modulus and Argument (or Amplitude) of Complex Numbers Modulus: | z | = r = x 2 + y 2 Argument (or Amplitude): y arg( z ) = q = tan −1 x Properties of Complex Numbers 1 (i) Re (z) = x = (z + z ), Im (z) = 2 1 y = (z z ) 2i (ii) ( z1 + z2 ) = z1 + z2 (iii)

( z1 z2 ) = z . z 1

2

⎛z ⎞ z (iv) ⎜ 1 ⎟ = 1 ⎝ z2 ⎠ z2 (v) z z = |z|2 = | z |2 (vi)

z1 z2 = | z1 | | z2 |

(vii) arg (z1 z2) = arg (z1) + arg (z2)

(viii)

z1 z1 = z2 z2

⎛z ⎞ (ix) arg ⎜ 1 ⎟ = arg (z1) ⎝ z2 ⎠

arg (z2)

De Moivre’s Theorem (cosq + i sin q )n = cos nq + i sin nq where n is any real number Circular and Hyperbolic Functions iz (i) sin z = e

cos z =

e 2i

iz

,

e iz + e − iz 2

e z − e− z , 2 e z + e− z cosh z = 2

(ii) sinh z =

Relation between Circular and Hyperbolic Functions (i) sin iz = i sinh z, sinh z = i sin iz (ii) cos iz = cosh z (iii) tan iz = i tanh z, tanh z = i tan iz Formulae on Hyperbolic Functions (i) cosh2 z – sinh2 z = 1 (ii) coth2 z – cosech2 z = 1 (iii) sech2 z + tanh2 z = 1 (iv) sinh 2z = 2 sinh z cosh z (v) cosh 2z = cosh2 z + sinh2 z = 2 cosh2 z – 1 = 1 + 2 sinh2 z 2tanh z (vi) tanh 2z = 1 + tanh 2 z (vii) sinh 3z = 3 sinh z + 4 sinh3 z (viii) cosh 3z = 4 cosh3 z – 3 cosh z 3tanh z + tanh3 z (ix) tanh 3z = 1 + 3tanh 2 z

1.123

Complex Numbers (x) sinh (z1 ± z2) = sinh z1 cosh z2 ± cosh z1 sinh z2 (xi) cosh (z1 ± z2) = cosh z1 cosh z2 ± sinh z1 sinh z2 tanh z1 ± tanh z2 (xii) tanh (z1 ± z2) = 1 ± tanh z1tanh z2 (xiii) sinh z1 + sinh z2 = z +z z −z 2 sinh 1 2 cosh 1 2 2 2 (xiv) sinh z1 – sinh z2 = z +z z −z 2 cosh 1 2 sinh 1 2 2 2 (xv) cosh z1 + cosh z2 = z +z z −z 2 cosh 1 2 cosh 1 2 2 2 (xvi) cosh z1 – cosh z2 = z1 + z2 z −z sinh 1 2 2 2 (xvii) 2 sinh z1 cosh z2 = sinh (z1 + z2) + sinh (z1 – z2) (xviii) 2 cosh z1 sinh z2 = sinh (z1 + z2) – sinh (z1 – z2) (xix) 2 cosh z1 cosh z2 = cosh (z1 + z2) + cosh (z1 – z2) (xviii) 2 sinh z1 sinh z2 = cosh (z1 + z2) – cosh (z1 – z2) Inverse Hyperbolic Functions

(

(iii) tanh

1

x=

)

1 ⎛1 + x ⎞ log ⎜ ⎝ 1 − x ⎟⎠ 2

Separation into Real and Imaginary Parts (i) sin (x ± iy) = sin x cosh y ± icos x sinh y (ii) cos (x ± iy) = cos x cosh y ± i sin x sinh y sin 2 x ± isinh 2 y (iii) tan (x ± iy) = cos 2 x + cosh 2 y (iv) sinh (x ± iy) = sinh x cos y ± i cosh x sin y

2 sinh

2 (i) sinh 1 x = log x + x + 1

(

2 (ii) cosh 1 x = log x + x − 1

)

(v) cosh (x ± iy) = cosh x cos y ± i sinh x sin y sinh 2 x ± i sin 2 y (vi) tanh (x ± iy) = cosh 2 x + cos 2 y Logarithm of a Complex Numbers Principle Value: log (x + iy ) 1 y = log (x 2 + y 2 ) + i tan 1 2 x 1 log r + iq 2 General Value: log (x + iy) =

=

y⎞ 1 ⎛ log (x 2 + y 2 ) + i ⎜ 2np + tan −1 ⎟ ⎝ x⎠ 2

=

1 log r + i (2np + q ) 2

MULTIPLE CHOICE QUESTIONS Choose the correct alternative in each of the following questions: 1. The value of 10

sin n =1

(a) (c)

1 i

2n 11

i cos

2n 11

(b) 0 (d) i

is

2. If the area of the triangle on the complex plane formed by complex numbers z, iz and z + iz is 50 square units, then |z| is (a) 5 (b) 10 (c) 15 (d) none of these

1.124

Engineering Mathematics

2 lies on z (a) a circle (b) an ellipse (c) a parabola (d) a straight line 4. The region in the Argand’s diagram defined by z 2i z 2i 5 is the interior of the ellipse with major axis along (a) the real axis (b) the imaginary axis (c) y = x (d) y = x 5. If w is an imaginary cube root of unity, then (1 + w – w 2)7 is equal to (a) 128w (b) 128w (c) 128w 2 (d) 128w 2 3. If z lies on |z| = 1, then

6. If

z 8i = z+6

then z lies on the

curve (a) x2 + y2 + 6x – 8y = 0 (b) 4x – 3y + 24 = 0 (c) x2 + y2 – 8 = 0 (d) none of these 7. If 2 + i 3 is a root of the quadratic equation x2 + ax + b = 0, where a b R then the values of a and b are respectively, (a) 4, 7 (b) 4, 7 (c) 4, 7 (d) 4, 7 8. If z1, z2, z3 are vertices of an equilateral triangle inscribed in the circle |z| then = 2 and if z1 = + i (a) z2 (b) z2

z3 z3

(c) z2 (d) z2

i i

z3 i

i z3

i

9. The triangle formed by the points, 1+ i 1, and i as vertices in the Argand 2 diagram is

(a) scalene (c) isosceles

(b) equilateral (d) right-angled 1 + c + is 10. If c2 + s2 = 1, then is equal 1 c is to (a) c + is (c) s + ic

(b) c – is (d) s – ic

11. The value of i i is (a) w (b) –w2 (c)

(d) none of these 2 12. If x + iy = c sin(u + iv) , then u = constant represents family of (a) confocal ellipses (b) confocal circles (c) confocal hyperbolas (d) none of these 1 13. If then cosh 2x is x= 2 2 4 (a) (b) 3 3 1 (c) 1 (d) 3 14. The value of sin(log i i ) is (a) –1 (b) 1 (c) 0 (d) none of these 15. If log[log(x + iy)] = p + iq, then the y value of tan 1 is x (a) e pcos q (b) e qsin p q (c) e cos p (d) e psin q 16. If (a + ib) p = m x+iy then the value of y is p b tan 1 (a) log m a p 1 a (b) tan log m b p (c) log(a 2 + b 2 ) 2 log m (d) none of these 17. The general value of which satisfies the equation (cos + i sin 3 )

1.125

Complex Numbers (cos 3 + isin3 )…[cos(2n – 1) + isin(2n – 1) ] = 1 is r (r 1) (a) 2 (b) n n2 (2r + 1) 2r (c) (d) 3 n n2 18. The smallest positive integer n for which (1 + i)2n = (1 – i)2n is (a) 4 (b) 8 (c) 2 (d) 12

(d) an ellipse

23. If the complex number z and its conjugate z satisfy z z + 2(z – z ) = 12 + 8i, then the values of z are

3 3w 3

6w 4

(a) 1 (c) 0

21. The complex numbers sinx + icos2x and cosx – sin2x are conjugate to each other for (a) x = n (b) x = n + 2 (c) x = (d) no value of x 3

b c a b

3. 10. 17. 24.

3 +i

)

= 299 (a + ib), then (a2

2

+ i sin

2

(b) 4

5 3

(c) 4 cos

(c) a parabola y = 2x 2. 9. 16. 23.

(

(a) 4 cos

5 , 0 3

2

Answers 1. d 8. a 15. d 22. a

(d) 2 ± 3i

100

27. If P is a point in the Argand diagram representing the complex number, 4 4 4 cos + i sin and OP is ro3 3 2 tated through an angle in the 3 anticlockwise direction, then P in the new position represents

z +1 22. The locus determined by =2 z 1 where z 1, z = x + iy is

(b) a circle with centre 0,

(c) 2 ± 2i

26. A root of x3 – 8x2 + px + q = 0 where p and q are real numbers, is 3 i 3. The real root is (a) 2 (b) 6 (c) 9 (d) 12

(b) –1 (d) none of these

(a) a circle with centre

(b) 2 2 ± 2i

+ b2) is equal to (a) 1 (b) 2 (c) 3 (d) 4 is a complex number such that 25. If 2 + + 1 = 0, then 31 is equal to (a) (b) 2 (c) 1 (d) 0

20. If w is the cube root of unity, then the value of the 2w 2 4w 3

(a) 2 ± 2 2i

24. If

19. The cube roots of unity lie on a circle (a) | z | = 1 (b) | z – 1| = 1 (c) | z + 1| = 1 (d) | z – 1| = 2

1 w 2 2w 2

x2 y 2 + =1 2 3

3

+ i sin

3

(d) 3 + 2i a a d d

4. 11. 18. 25.

b d c a

5. 12. 19. 26.

d c a a

6. 13. 20. 27.

a b c b

7. c 14. a 21. d

Differential Calculus I Chapter

2

2.1 INTRODUCTION Differential calculus is the study of derivative, i.e., the study of change of functions w.r.t. the change in inputs. It is the mathematical study of change, motion, growth or decay, etc. In this chapter, we will study successive differentiation, mean value theorems, such as Rolle’s theorem, Lagrange’s mean value theorem, Cauchy’ mean value theorem, expansion of functions and indeterminate forms.

2.2 SUCCESSIVE DIFFERENTIATION If y = f (x) be a differentiable function of x, then its derivative order derivative of y and is in general a function of x. If

dy is called the first dx

dy is differentiable, then its dx

derivative is called the second order derivative of y and is denoted by the derivative of

d2 y dx 2

d2 y dx 2

. Similarly,

is called the third order derivative of y and is denoted by

and so on. The successive differential coefficients of the function y = f (x) are denoted by dy d 2 y dn y , 2 ,… , n , … dx dx dx Alternative methods of writing the differential coefficients are d Dy, D2y, D3y, ……, Dny, … where D = dx f (x), f (x), f (x), …, f n(x), … y (x), y (x), y (x), … , y n(x), … y1 (x), y2 (x), y3 (x), … , yn (x), …

d3 y dx 3

2.2

Engineering Mathematics

The value of nth differential coefficient at x = a is denoted by ⎛ dn y ⎞ or ⎜ n⎟ ⎝ dx ⎠ x = a

(yn)a

or f n(a)

or

y n(a).

2.2.1 nth Order Derivative of Some Standard Functions 1. y = (ax + b)m, where m is any real number. Proof: y = (ax + b)m Differentiating w.r.t. x successively, y1 = ma (ax + b)m 1 y2 = m (m 1) a2 (ax + b)m

2

y3 = m (m 1) (m 2) a (ax + b)m 3, ........................................................... ............................................................ yn = m (m 1) (m 2) … (m n + 1) an (ax + b)m 3

dn

Hence,

dx n

(ax + b) m = m (m

1) (m

2) … (m

n + 1) an (ax + b)m

n

n

=

m (m − 1)… (m − n + 1)[(m − n)(m − n − 1)… 3 ⋅ 2 ⋅1] a n (ax + b) m − n (m − n) (m − n − 1)… 3 ⋅ 2 ⋅1

=

a n m !(ax + b) m − n , (m − n)!

= n! an , = 0,

if n < m if n = m if n > m

2. y = (ax + b)−m, where m is any positive integer. Proof: y = (ax + b) m Differentiating w.r.t. x successively, y1 = ( 1) ma (ax + b)

m 1

y2 = ( 1)2 m (m + 1) a2 (ax + b) m 2 y3 = ( 1)3 m (m + 1) (m + 2) a3 (ax + b) m 3 ........................................................... ............................................................ yn = ( 1)n m (m + 1) (m + 2) … (m + n 1) an (ax + b) = ( −1) n

Hence,

dn dx n

(ax + b) − m = ( −1) n

m n

an (m + n − 1)… m(m − 1)(m − 2)… 2 ⋅1 (m − 1)(m − 2)… 2 ⋅1 (ax + b) m + n (m + n − 1)! an . (m − 1)! (ax + b) m + n

Differential Calculus I

dn

Corollary 1: Putting m = 1, we get

(ax + b) −1 = ( −1) n n !

dx n

3. y = log (ax + b) Proof: y = log (ax + b) Differentiating w.r.t. x, a ax + b

y1 = Differentiating (n

1) times w.r.t. x, d n −1 dx

n −1

y1 =

d n −1 ⎛ a ⎞ ⎜ ⎟ dx n −1 ⎝ ax + b ⎠

n −1

n −1 n −1 ⎛ dy ⎞ a ( −1) (n − 1)!a = ⎜ ⎟ dx n −1 ⎝ dx ⎠ (ax + b) n

d

Hence,

dn dx n

log (ax + b) =

( −1) n −1 (n − 1)!a n (ax + b) n

4. y = eax Proof: y = eax Differentiating w.r.t. x successively, y1 = aeax y2 = a2eax y3 = a3eax ................. ................. yn = aneax Hence,

dn dx n

(e ax ) = a n e ax

5. y = amx Proof: y = amx Differentiating w.r.t. x successively, y1 = mamx log a y2 = m2amx (log a)2 y3 = m3amx (log a)3 ............................. ............................. yn = mnamx (log a)n Hence,

dn dx n

(a mx ) = m n a mx (log a ) n

2.3

an (ax + b)1+ n

.

2.4

Engineering Mathematics

6. y = sin (ax + b) Proof: y = sin (ax + b) Differentiating w.r.t. x successively,

⎛p ⎞ y1 = a cos ( ax + b) = a sin ⎜ + ax + b ⎟ ⎝2 ⎠ ⎛p ⎞ ⎛ 2p ⎞ + ax + b ⎟ y2 = a 2 cos ⎜ + ax + b ⎟ = a 2 sin ⎜ ⎝2 ⎠ ⎝ 2 ⎠ ⎛ 2p ⎞ ⎛ 3p ⎞ + ax + b ⎟ = a3 sin ⎜ + ax + b ⎟ y3 = a3 cos ⎜ ⎝ 2 ⎠ ⎝ 2 ⎠ .................................................................................. ..................................................................................

⎛ np ⎞ yn = a n sin ⎜ + ax + b ⎟ ⎝ 2 ⎠ Hence,

dn ⎛ np ⎞ [sin ( ax + b)] = a n sin ⎜ + ax + b ⎟ ⎝ 2 ⎠ dx n

dn ⎛ np ⎞ n + ax + b ⎟ Corollary 2: n [cos ( ax + b)] = a cos ⎜⎝ ⎠ 2 dx 7. y = eax cos (bx + c) Proof: y = eax cos (bx + c) Differentiating w.r.t. x, y1 = aeax cos (bx + c) + (-1) b eax sin (bx + c) y1 = eax [a cos (bx + c) b sin (bx + c)] Let a = r cos q, b = r sin q Then

b a y1 = eax [r cos q cos (bx + c) = reax cos (bx + c + q ) r = a 2 + b 2 , q = tan

1

r sin q sin (bx + c)]

Differentiating w.r.t. x, y2 = aeax r cos (bx + c + q ) - b eax r sin (bx + c + q ) = reax [r cos q cos (bx + c + q ) r sin q sin (bx + c + q )] = r2eax cos (bx + c + 2q ) Differentiating n times w.r.t. x, yn = r neax cos (bx + c + nq ), b where r = a 2 + b 2 , q = tan −1 . a

Differential Calculus I

Hence,

dn

2.5

[eax cos (bx + c)] = rneax cos (bx + c + nq ),

dx n

b where r = a 2 + b 2 ,q = tan −1 . a Corollary 3:

d n ax [e sin (bx + c)] = rneax sin (bx + c + nq ), dx n

2 2 −1 b . where r = a + b , q = tan a

Example 1: Find yn if y = Solution: y =

xn − 1 . x− 1

x n − 1 ( x − 1)( x n −1 + x n − 2 + x n − 3 + ……… + 1) = x−1 ( x − 1)

= x n −1 + x n − 2 + x n − 3 + ……… +1

Differentiating n times w.r.t. x, yn =

dn dx n

( x n −1 + x n − 2 + x n − 3 + ……… + 1) ⎡ dn ⎤ m ⎢∵ n (ax + b) = 0, if n > m ⎥ ⎢⎣ dx ⎥⎦

=0

Example 2: Prove that

d2n ( x2 dx 2 n

1) n = ( 2n)! .

Solution: (x2 - 1)n = (x2)n - nC1 (x2)n 1 + nC2 (x2)n 2 - …........ (-1)n = x2n - nC1 x2n 2 + nC2 x2n

4

1

- …........

Differentiating 2n times w.r.t. x,

d 2n dx 2 n

( x 2 − 1) n =

d 2n dx 2 n

(x2n - nC1 x2n 2 + nC2 x2n

Solution:

y= =

x . ( x + 1)4

x ( x + 1) 1

………)

n ⎡ dn m = m ! a , if n = m ⎢∵ n (ax + b) if n > m = 0, ⎢⎣ dx

= (2n) ! Example 3: Find yn, if y =

4

4

( x + 1)3

= −

( x + 1) − 1 ( x + 1) 4 1 ( x + 1) 4

⎤ ⎥ ⎥⎦

2.6

Engineering Mathematics

Differentiating n times w.r.t. x, ( −1) n (n + 2)!

yn =

2 ! ( x + 1) n + 3



( −1) n (n + 3)! 3! ( x + 1) n + 4

n+3 ⎤ ( −1) n (n + 2)! ⎡ 1− ⎥ n+3 ⎢ 2 ! ( x + 1) ⎣ 3 ( x + 1) ⎦ ( −1) n (n + 2)! ⎡ 3 x − n ⎤ = ⎢ ⎥. 2 ! ( x + 1) n + 3 ⎣ 3 ( x + 1) ⎦ =

Example 4: Find yn, if y =

x2 + 4x + 1 . x3 + 2x2 - x - 2

Solution: y= =

x2 + 4 x + 1 x3 + 2 x 2 − x − 2 x2 + 4x + 1

=

=

( x 2 + 4 x + 1) x 2 ( x + 2) − ( x + 2)

x2 + 4 x + 1 ( x + 2) ( x + 1) ( x − 1)

( x + 2) ( x 2 − 1) A B C + + = x + 2 x +1 x −1

[By partial fraction expansion]

(x2 + 4x + 1) = A(x + 1) (x – 1) + B(x + 2) (x – 1) + C(x + 2) (x + 1) Putting x = 2, A= 1 Putting x = 1, Putting x = 1, y=

B=1 C=1

1 1 −1 + + x + 2 x +1 x −1

Differentiating n times w.r.t. x, yn = −

( −1) n n ! ( x + 2) n +1

Example 5: Find yn, where y =

Solution: y =

x2 + 4

(2 x + 3) ( x − 1) 2 1 1 = + 2 x + 3 ( x − 1) 2

+

( −1) n n ! ( x + 1) n +1

+

x2 + 4 . ( 2 x + 3)( x - 1)2

=

( x − 1) 2 + (2 x + 3) (2 x + 3) ( x − 1) 2

( −1) n n ! ( x − 1) n +1

[Using Cor.1]

Differential Calculus I

2.7

Differentiating n times w.r.t. x, ⎡ (n !)2n (n + 1)! ⎤ yn = ( −1) n ⎢ + ⎥ n +1 ( x − 1) n + 2 ⎥⎦ ⎢⎣ (2 x + 3) Example 6: Find yn, where y =

x4 . ( x − 1) ( x − 2)

x4 x4 = 2 ( x − 1) ( x − 2) x − 3 x + 2 15 x − 14 = x 2 + 3x + 7 + 2 x − 3x + 2

Solution: y =

[By dividing]

15 x − 14 ( x − 1) ( x − 2) A B = x2 + 3x + 7 + + x−1 x− 2

= x 2 + 3x + 7 +

[By partial fraction expansion]

16 ⎛ −1 ⎞ = x 2 + 3x + 7 + ⎜ + ⎝ x − 1⎟⎠ x − 2 Differentiating n times w.r.t. x, yn = −

( −1) n n ! ( x − 1) n +1

+

16 ( −1) n n ! ( x − 2) n +1

⎡ ⎤ 16 1 = ( −1) n n ! ⎢ − n +1 ⎥ n +1 ( x − 1) ⎦ ⎣ ( x − 2) Example 7: If y = Solution: y =

x3 ⎧−( n !) if n is odd ⎫ , then prove that (yn)0 = ⎨ ⎬. x2 - 1 ⎩ 0 if n is even ⎭

x3 2

x −1

= x+

= x+

x 2

x −1

1⎛ 1 1 ⎞ 1 ⎡ x −1+ x +1 ⎤ x + = x+ ⎢ ⎥ = x+ ⎜ 2 ⎝ x + 1 x − 1 ⎟⎠ ( x − 1) ( x + 1) 2 ⎣ ( x − 1)( x + 1) ⎦

Differentiating n times w.r.t. x, yn =

⎡ ⎤ 1 1 1 (−1) n n ! ⎢ + ⎥ [Using Cor.1] + 1 + 1 n n 2 ( x − 1) ⎥⎦ ⎢⎣ ( x + 1)

⎡ 1 1 ⎤ ! ⎢ n +1 + ⎥ (−1) n +1 ⎥⎦ ⎢⎣ (1) = −(n !), if n is odd. = 0, if n is even.

( yn ) 0 =

1 (−1) n n 2

2.8

Engineering Mathematics

Example 8: Find yn, where y = y=

Solution:

=

=

=

1 . 1 + x + x2 1 1 + x + x2 1 2

1⎞ 3 ⎛ ⎜⎝ x + ⎟⎠ + 2 4

⎛ 1 3⎞ ⎛ 1 3⎞ ⎜x+ 2 +i 2 ⎟ −⎜x+ 2 −i 2 ⎟ ⎠ ⎝ ⎝ ⎠

1 = ⎞ ⎛ ⎛ 1 3 ⎛ 1 3⎞ 1 3⎞ ⎛ 1 3⎞ ⎜x+ 2 +i 2 ⎟ ⎜x+ 2 −i 2 ⎟ ⎜x+ 2 +i 2 ⎟ ⎜x+ 2 −i 2 ⎟ ⎠ ⎠ ⎠⎝ ⎠⎝ ⎝ ⎝ 1 ⎛ 1 1 ⎞ − 1 3 1 3⎟ i 3 ⎜ ⎟ ⎜ x+ −i x+ +i ⎝ 2 2 2 2 ⎠

Differentiating n times w.r.t. x, yn =

Let

1 1 ⎤ ⎡ − ( −1) n n ! ⎢ n +1 n +1 ⎥ i 3 ⎛ 1 3⎞ 1 3⎞ ⎥ ⎢⎛ ⎜x+ 2 +i 2 ⎟ ⎥ ⎢ ⎜⎝ x + 2 − i 2 ⎟⎠ ⎠ ⎦ ⎝ ⎣ 1

1⎞ 3 1⎞ 3 ⎛ ⎛ = re iq , ⎜ x + ⎟ − i = re − iq ⎜⎝ x + ⎟⎠ + i ⎝ ⎠ 2 2 2 2

3 2 1 3 ⎛ ⎞ − 1 2 where r = ⎜ x + ⎟ + , q = tan ⎝ 1⎞ 2⎠ 4 ⎛ ⎜⎝ x + ⎟⎠ 2 x+

1 3 = cotq 2 2

Substituting in r,

Hence,

r=

3 2 3 3 cot q + = cosec q 4 4 2

yn =

( −1) n n ! ⎡ 1 1 ⎤ ⎢ ( re − iq ) n +1 − ( re iq ) n +1 ⎥ i 3 ⎣ ⎦

=

( −1) n n! ⎡ e i ( n +1)q − e − i ( n +1)q ⎤ ⎥ ⎢ ⎦ r n +1 i 3 ⎣

[Using Cor.1]

Differential Calculus I

=

2.9

( −1) n n ! 2i sin ( n + 1)q ⎞ ⎛ 3 i 3⎜ cosecq ⎟ ⎠ ⎝ 2

=

n +1

( −1) n n ! 2n + 2 sin n +1 q sin ( n + 1)q 3

n+ 2 2

where q = tan −1

,

3 ( 2 x + 1)

1 , find yn. Example 9: If y = 4 x - a4 Solution:

y=

1 x4 − a4

=

1 ( x 2 + a 2 )( x 2 − a 2 )

=

1 2a 2



( x2 + a2 − x2 + a2 ) ( x 2 + a 2 )( x 2 − a 2 )

⎤ 1 ⎡ 1 1 − 2 2 ⎢ 2 2 2 ⎥ 2a ⎢⎣ ( x − a ) ( x + a ) ⎥⎦ ⎤ 1 ⎡ 1 1 = 2⎢ − ⎥ 2a ⎣ ( x + a ) ( x − a ) ( x + ia ) ( x − ia ) ⎦

=

1 ⎡ 1 ⎧ ( x + a ) − ( x − a ) ⎫ 1 ⎧ ( x + ia ) − ( x − ia ) ⎫⎤ ⎢ ⎨ ⎬− ⎨ ⎬⎥ 2a 2 ⎢⎣ 2a ⎩ ( x + a )( x − a ) ⎭ 2ia ⎩ ( x + ia )( x − ia ) ⎭⎥⎦ 1 ⎞ 1 ⎛ 1 1 ⎞ 1 ⎛ 1 − = − ⎟ ⎟− 3 ⎜ 3 ⎜ 4ia ⎝ x + ia x − ia ⎠ 4a ⎝ x + a x − a ⎠ Differentiating n times w.r.t. x, (−1) n n ! ⎡ 1 1 ⎤ yn = − Hence, ⎥ 3 ⎢ n +1 4ia ⎣ ( x + ia ) ( x − ia ) n +1 ⎦ =

− Let where

x + ia = reiq, x

ia = re

iq

1 (−1) n n ! ⎡ 1 ⎤ − ⎢ 3 n +1 n +1 ⎥ ( x − a) ⎦ 4a ⎣ ( x + a )

⎛a⎞ r = x 2 + a 2 , q = tan −1 ⎜ ⎟ . ⎝x⎠ yn =

n ( −1) n n ! ⎡ 1 1 1 1 ⎤ ( −1) n ! ⎡ ⎤ − − − 3 − iq n +1 ⎥ iq n +1 n +1 n +1 ⎥ 3 ⎢ ⎢ ( x − a) ⎦ 4ia ⎣ ( re ) ( re ) ⎦ 4 a ⎣ ( x + a)

=

( −1) n n! 1 ( −1) n n ! ⎡ 1 1 ⎤ ⋅ n +1 [ e − i ( n +1)q − e i ( n +1)q ] − − n +1 n +1 ⎥ 3 3 ⎢ 4ia 4 a ⎣ ( x + a) ( x − a) ⎦ r

=

( −1) n n ! ⋅ 4ia3

1 (x + a ) 2

2

n +1 2

[ −2i sin ( n + 1)q ] −

=

( −1) n +1 n ! n +1 2 2

2a 3 ( x 2 + a )

1 ( −1) n n ! ⎡ 1 ⎤ − n +1 n +1 ⎥ 3 ⎢ 4 a ⎣ ( x + a) ( x − a) ⎦

sin( n + 1)q −

1 1 ( −1) n n ! ⎡ ⎤ − n +1 n +1 ⎥ 3 ⎢ 4 a ⎣ ( x + a) ( x − a) ⎦

2.10

Engineering Mathematics

Example 10: Find nth order derivatives of (i) y = sin 2x sin 3x cos 4x (ii) y = cos4 x 5 3 (iv) y = ex (sin x + cos x) (iii) y = sin x cos x x cosa cos (x sin ` ). (v) y = e Solution: y = sin 2x sin 3x cos 4x 1 = (cos x cos 5x) cos 4x 2

(i)

=

1 (cos x cos 4x 2

cos 5x cos 4x)

1 (cos 5x + cos 3x cos 9x cos x) 4 Differentiating n times w.r.t. x, 1⎡ np ⎞ np ⎞ ⎛ ⎛ n yn = ⎢5n cos ⎜ 5 x + ⎟ + 3 cos ⎜⎝ 3 x + ⎟ ⎝ 4⎣ 2 ⎠ 2 ⎠ =

np np ⎞ ⎛ ⎛ − 9n cos ⎜ 9 x + ⎟ − cos ⎜⎝ x + ⎝ 2 ⎠ 2

⎞⎤ ⎟⎠ ⎥ [Using Cor. 2] ⎦

y = cos4 x

(ii)

⎛ 2 cos 2 x ⎞ =⎜ 2 ⎟⎠ ⎝

2

1 (1 + cos 2 x) 2 4 1 = (1 + cos 2 2 x + 2 cos 2 x) 4 1 ⎛ 1 + cos 4 x ⎞ + 2 cos 2 x⎟ = ⎜1 + ⎝ ⎠ 2 4 1 = (3 + cos 4 x + 4 cos 2 x) 8 =

Differentiating n times w.r.t. x,

yn = (iii)

1⎡ n np ⎞ np ⎛ ⎛ n 4 cos ⎜ 4 x + ⎟⎠ + 4.2 cos ⎜⎝ 2 x + ⎢ ⎝ 8⎣ 2 2 y = sin5 x cos3 x = sin2 x (sin x cos x)3 =

sin 2 x 23

sin 3 2 x

⎞⎤ ⎟⎠ ⎥ ⎦

[Using Cor. 2]

Differential Calculus I

2.11

1 ⎛ 1 − cos 2 x ⎞ ⎛ 3 sin 2 x − sin 6 x ⎞ ⎜ ⎟⎠ ⎜⎝ ⎟⎠ 8⎝ 2 4 1 = 6 (3 sin 2 x − sin 6 x − 3 cos 2 x sin 2 x + cos 2 x sin 6 x) 2 1 1 ⎡ 3 ⎤ = 6 ⎢3 sin 2 x − sin 6 x − sin 4 x + (sin 8 x + sin 4 x) ⎥ 2 2 2 ⎣ ⎦ 1 ⎡ 1 ⎤ = 6 ⎢3 sin 2 x − sin 4 x − sin 6 x + sin 8 x ⎥ 2 2 ⎣ ⎦

=

Differentiating n times w.r.t. x, yn =

1 26

⎡ n np ⎛ ⎢ 2 ⋅ 3 sin ⎜⎝ 2 x + 2 ⎣ np ⎛ − 6 n sin ⎜ 6 x + ⎝ 2

np ⎞ ⎞ n ⎛ ⎟⎠ − 4 sin ⎜⎝ 4 x + ⎟ 2 ⎠ np ⎞ 1 n ⎛ ⎟⎠ + 8 sin ⎜⎝ 8 x + 2 2

⎞⎤ ⎟⎠ ⎥ ⎦

[Using result (6)]

(iv) y = ex (sin x + cos x) Differentiating n times w.r.t. x, n

yn = (1 + 1) 2 ⋅ e x [sin ( x + n tan −1 1) + cos ( x + n tan −1 1)] [Using result (7) and Cor. 3] n ⎡ ⎛ np = 2 2 e x ⎢sin ⎜ x + 4 ⎣ ⎝

np ⎞ ⎤ ⎞ ⎛ ⎟⎠ + cos ⎜⎝ x + ⎟ 4 ⎠ ⎥⎦

n ⎡ ⎛ np ⎞ np ⎛p = 2 2 e x ⎢sin ⎜ x + + sin ⎜ + x + ⎟ ⎝2 4 ⎠ 4 ⎣ ⎝

p np ⎛ 2x + + ⎜ 2 2 = 2 e ⋅ 2 sin ⎜ 2 ⎜ ⎝ n 2

x

n

= 22 ex ⋅ =2

n +1 2

⎞⎤ ⎟⎠ ⎥ ⎦

⎞ p ⎟ ⎟ cos 4 ⎟ ⎠

⎤ ⎡ p sin ⎢ x + ( n + 1) ⎥ ⎣ ⎦ 4 2

2

p⎤ ⎡ e x sin ⎢ x + ( n + 1) ⎥ ⎣ 4⎦

(v) y = e xcosa cos (x sin a) Differentiating n times w.r.t. x, n sin a ⎞ ⎛ yn = (cos 2 a + sin 2 a ) 2 e x cos a ⋅ cos ⎜ x sin a + n tan −1 ⎟ [Using result (7)] ⎝ cos a ⎠

= e x cos a cos ( x sin a + na )

2.12

Engineering Mathematics

1 n n Example 11: If y(x) = sin px + cos px, prove that yn ( x ) = p [1 + ( -1) sin 2 px ]2 . 1 Hence, find y8(p ), when p = . 4

Solution: y(x) = sin px + cos px Differentiating n times w.r.t. x, ⎡ ⎛ np ⎞ np ⎛ yn ( x ) = p n ⎢sin ⎜ px + ⎟⎠ + cos ⎜⎝ px + ⎝ 2 2 ⎣

⎞⎤ ⎟⎠ ⎥ ⎦

⎡⎧ ⎛ np ⎞ np ⎛ = p n ⎢⎨sin ⎜ px + ⎟⎠ + cos ⎜⎝ px + ⎝ 2 2 ⎢⎣⎩

[ Using result (66) and Cor.2] 1

2 2 ⎞⎫ ⎤ ⎟⎠ ⎬ ⎥ ⎭ ⎥⎦

⎡ np np ⎞ np ⎞ np ⎞ ⎛ ⎛ ⎛ 2 ⎛ = p ⎢sin 2 ⎜ px + ⎟⎠ + cos ⎜⎝ px + ⎟⎠ + 2 sin ⎜⎝ px + ⎟ cos ⎜⎝ px + ⎝ ⎠ 2 2 2 2 ⎣ n

1

⎞⎤ 2 ⎟⎠ ⎥ ⎦

1

= p n [1 + sin ( 2 px + np )] 2 1

= p n [1 + sin 2 px cos np + cos 2 px sin np ] 2 1

= p n [1 + ( −1) n sin 2 px ] 2 Putting n = 8, p =

1 and x = p, 4 ⎛1⎞ y8 (p ) = ⎜ ⎟ ⎝4⎠

8

1

⎡ ⎛ p ⎞⎤ 2 8 ⎢1 + ( −1) sin 2 ⎜⎝ 4 ⎟⎠ ⎥ ⎦ ⎣ 1

8

31

⎛1⎞ ⎡ ⎛ p ⎞⎤ 2 ⎛ 1 ⎞ 2 = ⎜ ⎟ ⎢1 + sin ⎜ ⎟ ⎥ = ⎜ ⎟ ⎝4⎠ ⎣ ⎝2⎠ ⎝ 2 ⎠⎦

( -1) n -1 ( n - 1)! -1 ⎛ x ⎞ Example 12: If y = tan ⎜ ⎟ , prove that yn = sin np sin n p , ⎝a⎠ an ⎛a⎞ where p = tan -1 ⎜ ⎟ . ⎝ x⎠ ⎛ x⎞ Solution: y = tan −1 ⎜ ⎟ ⎝ a⎠ Differentiating w.r.t. x, y1 =

1 2

1+

x a2



a a 1 = = a a 2 + x 2 ( x + ia )( x − ia )

Differential Calculus I

=

1 ⎡ ( x + ia ) − ( x − ia ) ⎤ 2i ⎢⎣ ( x + ia ) ( x − ia ) ⎥⎦

=

1⎛ 1 1 ⎞ − ⎜ 2i ⎝ x − ia x + ia ⎟⎠

2.13

Differentiating (n - 1) times w.r.t. x, yn = Let

x + ia = reiq, x

( −1) n −1 (n − 1)! ⎡ 1 1 ⎤ − ⎢ n n⎥ 2i ( x + ia ) ⎦ ⎣ ( x − ia )

ia = re

iq

a ⎛a⎞ r = x 2 + a 2 , q = tan −1 ⎜ ⎟ , tan q = , x = a cot q ⎝x⎠ x

where,

r = a 2 cot 2 q + a 2 = a cosecq Hence,

yn = =

( −1) n −1 ( n − 1)! ⎡ 1 1 ⎤ − iq n ⎥ − iq n ⎢ 2i ⎣ ( re ) ( re ) ⎦ ( −1) n −1 ( n − 1)! 1 ( inq e − e − inq ) 2i rn

( −1) n −1 ( n − 1)! 2i sin nq ⋅ 2i rn sin nq = ( −1) n −1 ( n − 1)! rn sin nq = ( −1) n −1 ( n − 1)! n a cosec n q ( −1) n −1 ( n − 1)! = sin nq sin n q . n a =

where,

⎛a⎞ q = tan −1 ⎜ ⎟ . ⎝x⎠

⎛ 1 + x2 - 1 ⎞ -1 Example 13: If y = tan ⎜⎝ ⎟⎠ , prove that x yn =

[Using Cor. 1]

1 (–1)n−1 (n – 1) ! sinn q sin nq, where q = cot−1 x. 2

⎛ 1 + x 2 − 1⎞ Solution: y = tan −1 ⎜ ⎟ ⎝ ⎠ x Putting x = tan f,

2.14

Engineering Mathematics

⎛ sec f − 1 ⎞ ⎛ 1 − cos f ⎞ y = tan −1 ⎜ = tan −1 ⎜ ⎟ ⎝ tan f ⎠ ⎝ sin f ⎟⎠ f ⎞ ⎛ 2 sin 2 ⎜ 2 ⎟ = tan −1 tan f = f = tan ⎜ f f⎟ 2 2 ⎜ 2 sin cos ⎟ ⎝ 2 2⎠ −1

y=

1 −1 tan x 2

y1 =

1 1 ⋅ 2 (1 + x 2 )

Differentiating w.r.t. x,

=

1 1 1 ( x + i) − ( x − i) ⋅ = ⋅ 2 ( x + i )( x − i ) 4i ( x + i )( x − i )

=

1⎛ 1 1 ⎞ − ⎜⎝ ⎟ 4i x − i x + i ⎠

Differentiating (n - 1) times w.r.t. x,

yn = Let where,

x + i = r eiq, x

i = re

( −1) n −1 (n − 1)! ⎡ 1 1 ⎤ − ⎥ ⎢ n 4i ( x + i)n ⎦ ⎣ ( x − i)

iq

1 ⎛1⎞ r = x 2 + 1, q = tan −1 ⎜ ⎟ , tan q = , x = cot q ⎝x⎠ x r = cot 2 q + 1 = cosecq

Hence,

yn =

( −1) n −1 ( n − 1)! ⎡ 1 1 ⎤ ⎢ − iq n − iq n ⎥ 4i ( ) ( ) ⎦ re re ⎣

( −1) n −1 ( n − 1)! 1 inq (e − e −inq ) 4i rn ( −1) n −1 ( n − 1)! 2i sin nq = ⋅ 4i rn =

( −1) n −1 ( n − 1)!sin nq 2r n n −1 ( −1) ( n − 1)!sin nq = 2 cosec n q 1 ⎛1⎞ = ( −1) n −1 ( n − 1)! sin n q sin nq , where q = tan −1 ⎜ ⎟ . ⎝x⎠ 2 =

Differential Calculus I

2.15

-1 ⎛ 1 + x ⎞ , prove that yn = (-1)n−1 (n - 1) ! sinnp sin np Example 14: If y = tan ⎜ ⎝ 1 - x ⎟⎠ ⎛1⎞ where θ = tan −1 ⎜ ⎟ . ⎝ x⎠

−1 ⎛ 1 + x ⎞ Solution: y = tan ⎜ ⎝ 1 − x ⎟⎠

Putting x = tan f, ⎛ 1 + tan f ⎞ y = tan −1 ⎜ ⎝ 1 − tan f ⎟⎠ p ⎛ ⎞ tan + tan f ⎜ ⎟ 4 = tan ⎜ ⎟ p ⎜ 1 − tan ⋅ tan f ⎟ ⎝ ⎠ 4 −1

⎡ ⎛p p ⎞⎤ p = tan −1 ⎢ tan ⎜ + f ⎟ ⎥ = + f = + tan −1 x ⎠⎦ 4 4 ⎣ ⎝4 Proceeding as in Example 13, we get yn = ( 1)n 1 (n

1)! sinn q sin nq.

⎛ x - x -1 ⎞ . ⎝ x + x -1 ⎟⎠

-1 Example 15: Find the nth order derivative of y = cos ⎜

⎛ x − x −1 ⎞ ⎛ 2 ⎞ −1 x − 1 Solution: y = cos −1 ⎜ cos = ⎟ ⎜ 2 ⎟ ⎝ x + x −1 ⎠ ⎝ x + 1⎠ Putting x = tan f, ⎛ tan 2 f − 1 ⎞ = cos −1 ( − cos 2f ) y = cos −1 ⎜ 2 ⎝ tan f + 1 ⎟⎠ = cos −1 [cos(p − 2f )] = p − 2f = p − 2 tan −1 x

Proceeding as in Example 13, we get ⎛ x − x −1 ⎞ . y = cos −1 ⎜ ⎝ x + x −1 ⎟⎠

Example 16: If y = x log (1 + x), prove that yn =

( -1) n 2 ( n - 2)!( x + n) . ( x + 1) n

Solution: y = x log (1 + x) Differentiating w.r.t. x, y1 = log (1 + x) +

x 1 = log (1 + x) + 1 − 1+ x 1+ x

2.16

Engineering Mathematics

Differentiating (n - 1) times w.r.t. x,

yn = = = =

( −1) n − 2 (n − 2)! ( x + 1) n −1

+0−

( −1) n −1 (n − 1)! ( x + 1) n

[Using result (3) and Cor. 1]

( −1) n − 2 (n − 2)! ⎡ 1 ( −1)1 (n − 1) ⎤ − ⎢ ⎥ −1 1 ( x + 1) n ⎣ ( x + 1) ⎦ ( −1) n − 2 (n − 2) ! ( x + 1) n

( x + 1 + n − 1)

( −1) n − 2 (n − 2) ! ( x + n) ( x + 1) n

.

x+n ⎤ ⎛ x - 1⎞ ⎡ x-n , prove that yn = (-1)n (n - 2) ! ⎢ , Example 17: If y = x log ⎜ ⎟ n ⎝ x + 1⎠ ( x + 1) n ⎥⎦ ⎣ ( x - 1) n ê 2.

⎛ x − 1⎞ Solution: y = x log ⎜ ⎝ x + 1⎟⎠ = x log( x − 1) − x log( x + 1) Differentiating y w.r.t. x, x x − − log( x + 1) x −1 x +1 1 1 = log( x − 1) + 1 + −1+ − log(xx +1) x −1 x +1 Differentiating (n - 1) times w.r.t. x, y1 = log( x − 1) +

yn =

( −1) n − 2 (n − 2)! ( x − 1) n −1

+

( −1) n −1 (n − 1)! ( x − 1) n

+

( −1) n −1 (n − 1)! ( x + 1) n



( −1) n − 2 (n − 2)! ( x + 1) n −1

[if n – 2 0]

[Using result (3) and Cor. 1] =

=

(−1) n (n − 2)! ⎡ (−1) −2 (−1) −1 (n − 1) ⎤ + ⎢ ⎥ n −1 1 ( x − 1) ⎢⎣ ( x − 1) ⎥⎦ − 1 n (−1) (n − 2)! ⎡ (−1) (n − 1) (−1) −2 ⎤ − + ⎢ ⎥ 1 ( x + 1) n ⎢⎣ ( x + 1) −1 ⎥⎦ (−1) n (n − 2) ! ( x − 1) n

( x − 1 − n + 1) +

(−1) n (n − 2)! ( x + 1) n

x+n ⎤ ⎡ x−n = (−1) n (n − 2)! ⎢ − ⎥ , n ≥ 2. n ( x + 1) n ⎦ ⎣ ( x − 1)

(−n + 1 − x − 1)

Differential Calculus I

2.17

Example 18: If y = x coth−1 x, prove that yn =

( -1) n ( n - 2)! ⎡ x + n x-n ⎤ , n ≥ 2. n ⎢ 2 ( x - 1) n ⎥⎦ ⎣ ( x + 1)

y = x coth−1 x

Solution:

⎛ 1⎞ = x tanh −1 ⎜ ⎟ ⎝ x⎠ 1 x 1 1− x x ⎛ x + 1⎞ = log ⎜ ⎝ x − 1⎟⎠ 2 1 = x ⋅ log 2

1+

1 [ x log ( x + 1) − x log ( x − 1)] 2 Proceeding as in Example 17, we get y=

yn =

( −1) n (n − 2)! ⎡ x + n x−n ⎤ . ⎢ ( 1) n − ( 1) n ⎥ , n ≥ 2 2 x− ⎦ ⎣ x+

Example 19: If y = (x – 1)n, prove that y + Solution: y = (x

y1 y2 y3 y + + + ... + n = x n . n! 1! 2 ! 3 !

1)n

Differentiating w.r.t. x successively, y1 = n(x 1)n 1 y2 = n (n 1) (x 1)n 2 y3 = n (n 1) (n 2) (x 1)n 3 ……………………………… ...……………………………. yn = n ! Hence,

y+

= ( x − 1) n +

y1 y2 y3 + + + 1! 2 ! 3!

+

yn n!

n n(n − 1) n(n − 1)(n − 2) ( x − 1) n −1 + ( x − 1) n − 2 + ( x − 1) n −3 + 1! 2! 3!

= ( x − 1) n + nC1 ( x − 1) n −1 + nC2 ( x − 1) n − 2 + nC3 ( x − 1) n −3 +

+

n! n!

+ nCn

= [1 + ( x − 1) ]

n

= xn .

[ Using Binomial Expansion ]

2.18

Example 20: If I n =

Engineering Mathematics

dn ( x n log x ), prove that In = n In−1 + (n - 1) ! dx n

1 1 1⎞ ⎛ Hence, prove that I n = n ! ⎜ log x + 1 + + + ... + ⎟ . ⎝ 2 3 n⎠ In =

Solution: For

n=1 I1 =

In = =

dn dx n

( x n log x)

d ( x log x) = log x + 1 dx

d n −1 ⎡ d n ⎤ ( x log x) ⎥ n −1 ⎢ d x ⎦ ⎣ dx d n −1 ⎛ n −1 n ⎜ nx log x + x dx n −1 ⎝

=n

1⎞ ⎟ x⎠

d n −1

d n −1

dx

d x n −1

( x n −1 log x) + n −1

( x n −1 )

I n = nI n −1 + (n − 1)!

In I 1 = n −1 + n ! (n − 1)! n Putting n = 2, 3, 4, ……. in Eq. (1), I 2 I1 1 = + 2 ! 1! 2 I3 I 2 1 = + 3! 2 ! 3 I 4 I3 1 = + 4 ! 3! 4 ……………… .……………...

I n −1 I 1 = n−2 + (n − 1)! (n − 2)! n − 1 From Eq. (1),

In − 1 In 1 = + n ! (n − 1)! n

... (1)

Differential Calculus I

2.19

Adding all the above equations,

In 1 1 1 1 = I1 + + + + … + n! n 2 3 4 1 1 1 1⎞ ⎛ I n = n !⎜ log x + 1 + + + + … + ⎟ ⎝ n⎠ 2 3 4

(

1

)

1 dn ( -1) n e x n-1 x x e Example 21: Prove that = . x n+1 dx n

Solution:

dn dx n

(x

1 n −1 x e

)

⎡ n −1 ⎛ 1 ⎤ 1 1 1 + +… + ⎢ x ⎜⎜1 + + ⎥ 2 3 − 1 n 3! x (n − 1)! x ⎥ d ⎢ ⎝ x 2! x = n⎢ ⎥ dx ⎢ ⎞⎥ 1 1 1 + + … + + ⎟⎟ ⎥ ⎢ n ! x n (n + 1)! x n +1 (n + 2)! x n + 2 ⎠⎦ ⎣ n

=

d n ⎡ n −1 1 1 x n −3 +… + + x + xn−2 + n ⎢ − 2 ! ( 1 )! ( n x n !) dx ⎣ 1 1 ⎤ + 2 + + …⎥ x (n + 1)! x3 (n + 2)! ⎦

= 0+

= =

1(−1) n n ! n ! x n +1

+

(−1) n (n + 1)! (n + 1)! (1!) x n + 2

+

(−1) n (n + 2)! (n + 2)! (2 !) x n + 3

+ ....

[Using result (1) and (2)]

1 (−1) n ⎛ 1 ⎞ 1+ + + …⎟ n +1 ⎜ 2 x 2! x x ⎠ ⎝ (−1) n x n +1

1 ex.

Exercise 2.1 1. Find the nth order derivative of x x +1 (i) y = 2 (ii) y = . 1 − 4 x2 x −4

⎡ 3 ( −1) n n ! 1 ( −1) n n ! ⎤ + ⎢ Ans. : (i) ⎥ 4 ( x − 2) n +1 4 ( x + 2) n +1 ⎥ ⎢ ⎢ ⎥ 1 ⎡ ( −1) n n ! ( −2) n ⎢ ⎥ (ii) ⎢ n +1 4 ⎣ (1 − 2 x ) ⎢ ⎥ ⎢ ⎥ n n ( −1) n ! 2 ⎤ ⎥ ⎢ − ⎥ ⎢ (1 + 2 x ) n +1 ⎦ ⎥⎦ ⎣

2.20

Engineering Mathematics

2. Find the nth order derivative of x y= . ( x − 1)( x − 2)( x − 3) ⎡ ⎤ 1 ⎡ n ⎢ Ans. : ( −1) n ! ⎢ 2( x − 1) n +1 ⎥ ⎣ ⎢ ⎥ ⎢ 2 3 ⎤⎥ − + ⎢ ⎥ ( x − 2) n +1 2( x − 3) n +1 ⎥⎦ ⎥⎦ ⎢⎣ 3. Find the nth order derivative of x y= . 1 + 3x + 2 x 2 ⎡ ⎤ 1 ⎡ n ⎢ Ans. : (−1) n ! ⎢ n +1 ⎥ ⎣ ( x + 1) ⎥ ⎢ ⎢ ⎤ ⎥⎥ 2n ⎢ − ⎥ ⎢ (2 x + 1) n +1 ⎥⎦ ⎥⎦ ⎣ 4. Find the nth order derivative of x2 . y= ( x + 2)(2 x + 3) ⎡ −4( −1) n n ! 9( −1) n n ! (2) n −1 ⎤ + ⎢ Ans. : ⎥ ( x + 2) n +1 (2 x + 3) n +1 ⎥⎦ ⎢⎣ 5. Find the nth order derivative of 2x + 3 y= . ( x − 1) 2 ⎡ 2 ( −1) n n ! 5 ( −1) n (n + 1)!⎤ + ⎢ Ans. : ⎥ ( x − 1) n +1 ( x − 1) n + 2 ⎥⎦ ⎢⎣ 6. Find the nth order derivative of x y= . ( x − 1) 2 ⎡ ( −1) n n ! ( −1) n (n + 1)!⎤ + ⎢ Ans. : ⎥ ( x − 1) n +1 ( x − 1) n + 2 ⎥⎦ ⎢⎣

7. Find the nth order derivative of x +1 y= . ( x − 1) n ( x − 1) + 2 ⎤ ⎡ ⎥ ⎢ Hint : y = ( 1) n x − ⎥ ⎢ ⎢ 1 2 ⎥ = + ⎥ ⎢ ( x − 1) n −1 ( x − 1) n ⎥⎦ ⎢⎣

⎡ ⎡ (2n − 2)! (2n − 1)! ⎤ ⎤ −1) n ⎢ + ⎢ Ans. : (− ⎥⎥ 2 n −1 ( x − 1) 2 n ⎦ ⎥⎦ ⎢⎣ ⎣ ( x − 1) 8. Find the nth order derivative of 4x . y= ( x − 1) 2 ( x + 1) ⎡ ⎤ 1 ⎡ n ⎢ Ans. : ( −1) n ! ⎢ ⎥ n +1 ⎣ ( x − 1) ⎢ ⎥ ⎢ 2 (n + 1) n! ⎤ ⎥ ⎢ + − ⎥⎥ ⎢⎣ ( x − 1) n + 2 ( x + 1) n +1 ⎦ ⎥⎦ 9. Find the nth order derivative of 1 . y= (3 x − 2)( x − 3) 2 ⎡ ⎤ ⎡ 3n + 2 n Ans. : ( − 1 ) n ! ⎢ ⎢ n +1 ⎥ ⎣ 49 (3 x − 2) ⎥ ⎢ ⎢ 3 ( n + 1) ⎤ ⎥ ⎢− ⎥ + n +1 7 ( x − 3) n + 2 ⎥⎦ ⎥⎦ ⎢⎣ 49 ( x − 3) 10. If y =

x2 2 x2 + 7 x + 6

, find y . n

⎡ Hint : Divide x 2 by 2 x 2 + 7 x + 6, ⎢ 1 7x + 6 ⎢ y= − ⎢⎣ 2 2 ( x + 2) ( 2 x + 3) ⎡ ⎤ 8 ⎡ n ⎢ Ans.. : ( −1) n ! ⎢ − ( x + 2) n +1 ⎥ ⎣ ⎢ ⎥ n ⎢ ⎤⎥ 9( 2) ⎢ ⎥ + n +1 ⎥ ( 2 x + 3) ⎦ ⎥⎦ ⎢⎣ 4 ⎛ x3 ⎞ 11. Prove that d = 0. ⎜ ⎟ dx 4 ⎝ x 2 − 1⎠ x = 0 x , prove that 12. If y = 2 x + a2 yn = ( 1)n n ! a n 1 (sin q )n+1 cos (n + 1) q. x , prove that 13. If y = 2 x +1

yn = (-1) n n! sinn+1 q cos (n + 1) q ⎛1⎞ where q = tan −1 ⎜ ⎟ . ⎝x⎠

⎤ ⎥ ⎥ ⎥⎦

Differential Calculus I 14. Find the nth order derivative of 1 . y= 1 + x + x 2 + x3 1 ⎡ ⎤ ⎢ Hint : y = (1 + x ) (1 + x 2 ) ⎥ ⎢ ⎥ 1 ⎢ ⎥ = ⎢ (1 + x ) ( x + i ) ( x − i ) ⎥⎦ ⎣ ⎡ ⎤ 1 ( −1) n n ! ⎡ Ans. : ⎢ ⎥ n +1 ⎢ 2 ⎣ (1 + x ) ⎢ ⎥ ⎢ 1 ⎤⎥ ⎢ + n +1 {sin ( n + 1)q − cos ( n + 1)q }⎥ ⎥ ⎦⎦ ⎣ 2r 15. Find the nth order derivative of x . y= 1 + x + x2

⎡ ⎤ ⎤ ⎡cos( n + 1)q n ⎢ Ans. : ( −1) n ! ⎢ ⎥ ,⎥ 1 ⎢ sin( n + 1)q ⎥ ⎥ r n +1 ⎢ − ⎢ ⎥⎦ ⎥ ⎢⎣ 3 ⎢ ⎥ ⎢ where r = x 2 + x + 1, ⎥ ⎢ ⎥ ⎛ 3 ⎞ ⎢ ⎥ −1 q = tan ⎜ ⎟ ⎢ ⎥ ⎝ 2x + 1⎠ ⎣ ⎦ 16. Prove that

dn dx n

tan

1

x

1⎞ ⎛ sin ⎜ n tan −1 ⎟ x⎠ = (−1) n −1 (n − 1)! ⎝ . n ( x 2 + 1) 2 17. Find the nth order derivatives of ⎛ 2x ⎞ y = tan −1 ⎜ . ⎝ 1 − x 2 ⎟⎠ ⎡ Ans. : 2( −1) n −1 ( n − 1) ! (sin q ) n sin nq ,⎤ ⎢ ⎥ ⎢ where q = tan −1 ⎛ 1 ⎞ . ⎥ ⎜⎝ ⎟⎠ ⎢⎣ ⎥⎦ x −1 ⎛ 2 x ⎞ 18. If y = sin ⎜⎝ ⎟ , prove that 1 + x2 ⎠

2.21

yn = 2 (-1)n−1 (n - 1) ! sinn q sin nq, 1 where q = tan −1 ⎛⎜ ⎞⎟ . ⎝x⎠ 2⎞ ⎛ −1 1 + x y = sec 19. If ⎜ ⎟ , prove that ⎝ 1 − x2 ⎠ yn = 2 (-1)n−1 (n - 1) ! sinn q sin nq. 20. Find the nth order derivative w.r.t. x of (ii) sin7 x (i) sin4 x 7 ⎡ ⎡ 1 ix 7 − ix ⎤ ⎢ Hint : sin x = ⎢ (e − e ) ⎥ , ⎦ ⎣ 2i ⎢ ⎢⎣expand using binomial expaansion

(iii) sin3 x cos2 x

⎤ ⎥ ⎥ ⎥⎦

(iv) sin3 3x.

⎡ np ⎞ ⎤ ⎛ n −1 ⎢ Ans. : (i) − 2 cos ⎜⎝ 2 x + 2 ⎟⎠ ⎥ ⎢ ⎥ ⎢ ⎥ n p ⎛ ⎞ + 22 n − 3 cos ⎜ 4 x + ⎢ ⎟⎠ ⎥ ⎝ 2 ⎥ ⎢ ⎢ ⎥ np ⎞ 1 ⎡ ⎛ ⎢(ii) − ⎢7n sin ⎜ 7 x + ⎥ ⎟ ⎝ 2 ⎠ 64 ⎣ ⎢ ⎥ ⎢ ⎥ ⎢ −7.5n sin ⎛⎜ 5 x + np ⎞⎟ + 21.3n sin ⎤ ⎥ ⎥⎥ ⎝ ⎢ 2 ⎠ ⎥⎥ ⎢ ⎥⎥ p n p n ⎛ ⎞ ⎛ ⎞ ⎢ 3x + ⎟⎠ − 35 sin ⎜⎝ x + ⎟⎠ ⎥ ⎥ ⎢ ⎜⎝ 2 2 ⎦ ⎢ ⎥ 1 ⎡ np ⎞ n ⎛ ⎢ ⎥ ⎢(iii) 16 ⎢ 2 sin ⎜⎝ x + 2 ⎟⎠ + 3 sin ⎥ ⎣ ⎢ ⎥ ⎢⎛ np ⎞ n np ⎞ ⎤ ⎥ ⎛ ⎢ ⎜ 3x + ⎟ − 5 sin ⎜⎝ 5 x + ⎟ ⎥ 2 ⎠ 2 ⎠ ⎥⎦ ⎥ ⎢⎝ ⎢ ⎥ 3n +1 np ⎞ ⎛ ⎢(iv) ⎥ sin ⎜ 3 x + ⎟ ⎝ 4 2 ⎠ ⎢ ⎥ ⎢ ⎥ np ⎞ 1 ⎛ ⎢ ⎥ − ⋅ 32n sin ⎜ 9 x + ⎟ ⎝ 4 2 ⎠ ⎣⎢ ⎦⎥ 21. Find the nth order derivative w.r.t. x of (i) sin 2x cos 6x (ii) sin x cos 3x (iii) cos x cos 2x cos 3x.

2.22

Engineering Mathematics

⎡ 1⎡ n np ⎞ ⎛ ⎢ Ans. : (i) ⎢8 sin ⎜⎝ 8 x + ⎟ 2 2 ⎠ ⎣ ⎢ ⎢ np ⎞ ⎤ ⎛ ⎢ − 4 n sin ⎜ 4 x + ⎟ ⎝ 2 ⎠ ⎥⎦ ⎢ ⎢ 1⎡ np ⎞ ⎛ ⎢ (ii) ⎢ 4 n sin ⎜ 4 x + ⎟ ⎢ ⎝ 2⎣ 2 ⎠ ⎢ np ⎞ ⎤ ⎛ ⎢ − 2n sin ⎜ 2 x + ⎟ ⎢ ⎝ 2 ⎠ ⎥⎦ ⎢ ⎢ 1⎡ np ⎞ ⎛ (iii) ⎢6 n cos ⎜ 6 x + ⎢ ⎟ ⎝ 2 ⎠ 4⎣ ⎢ ⎢ np ⎞ n np ⎛ ⎛ ⎢ +4 n cos ⎜ 4 x + ⎟⎠ + 2 cos ⎜⎝ 2 x + ⎝ 2 2 ⎢⎣

⎤ ⎥ ⎥ ⎥ ⎥ ⎥ ⎥ ⎥ ⎥ ⎥ ⎥ ⎥ ⎥ ⎥ ⎥ ⎥ ⎞⎤⎥ ⎟⎠ ⎥ ⎥ ⎦ ⎥⎦

22. Find the nth order derivative w.r.t. x of (i) e5x cos x cos 3x (ii) ex cos x cos 2x (iii) eax cos2 x sin x (iv) 2x sin2 x cos x (v) 2x sin (3x + 1). 1 5x ⎡ ⎤ ⎢ Ans. : (i) 2 e ⎥ ⎢ ⎥ n ⎡ 4 ⎢ ⎥ ⎛ ⎞ 1 − 2 ⎢( 41) cos ⎜⎝ 4 x + n tan ⎟⎠ ⎥ ⎢ 5 ⎣ ⎢ ⎥ n ⎢ ⎥ ⎤ 2 ⎛ ⎞ − 1 ⎢ ⎥ + ( 29) 2 cos ⎜ 2 x + n tan ⎥ ⎟ ⎝ 5 ⎠ ⎦ ⎥⎦ ⎢⎣

⎢ ⎥ n ⎢ ⎥ 1 x⎡ (ii) e ⎢(10) 2 cos (3 x + ⎢ ⎥ 2 ⎣ ⎢ ⎥ n ⎢ ⎥ ⎤ n p ⎛ ⎞ ⎢ ⎥ n tan −1 3) + ( 2) 2 cos ⎜ x + ⎥ ⎟ ⎝ 4 ⎠⎦⎥ ⎢ ⎢ ⎥ n e ax ⎡ 2 ⎢ ⎥ 2 (iii) ⎢ ( a + 9) ⎢ ⎥ 4 ⎣ ⎢ ⎥ ⎢ ⎥ ⎛ −1 3 ⎞ sin ⎜ 3 x + n tan ⎟⎠ ⎢ ⎥ ⎝ a ⎢ ⎥ n ⎢ ⎥ ⎤ 1 ⎛ ⎞ 1 − 2 ⎢ +( a + 1) 2 sin ⎜ x + n tan ⎥ ⎥ ⎟ ⎝ a ⎠⎦ ⎢ ⎥ ⎢ ⎥ 1 ⎢ ⎥ (iv) − r1n 2 x cos (3 x + nf1) 4 ⎢ ⎥ ⎢ ⎥ 1 n x + r2 2 cos ( x + nf 2) ⎢ ⎥ 4 ⎢ ⎥ ⎢ where r1 = (log 2) 2 + 32 , ⎥ ⎢ ⎥ ⎢ ⎥ ⎛ ⎞ 3 f1 = tan −1 ⎜ , ⎢ ⎥ ⎟ ⎝ log 2 ⎠ ⎢ ⎥ ⎢ ⎥ 2 r2 = (log 2) + 1, ⎢ ⎥ ⎢ ⎥ ⎛ ⎞ 1 ⎢ ⎥ f 2 = tan −1 ⎜ ⎝ log 2 ⎟⎠ ⎥ ⎢ ⎢ ⎥ n 2 x ⎢ ⎥ 2 ( v) 2 [(log 2) + 9] ⎢ ⎥ ⎢ ⎛ 3 ⎞ ⎥ −1 sin ⎜ 3 x + 1 + n tan ⎢ ⎥ ⎝ log 2 ⎟⎠ ⎦ ⎣

2.3 LEIBNITZ’S THEOREM Statement: If y = uv, where u and v are two functions of x whose nth derivatives are known, then yn = (uv) n = un v + n C1 un −1v1 + nC2 un − 2 v2 + nC3un − 3 v3 + … + nCn uvn . Proof:

y = uv

Differential Calculus I

2.23

Differentiating w.r.t. x successively, y1 = u1v + uv1 = 1C 0 u1v + 1C1u v1 y2 = u2v + 2 u1v1 + uv2 = 2C 0 u2 v + 2C1u1v1 + 2C 2 u v2 This shows that theorem is true for n = 1 and n = 2. Let the theorem is true for n = m. ym = (uv) m = mC 0 um v + mC1um −1v1 + mC2 um − 2 v2 + ........... + mCm uvm . Differentiating ym w.r.t. x, d d ym = (uv) m = mC 0 (um +1v + um v1 ) + mC1 (um v1 + um −1v2 ) + m C2 (um −1 v2 + um − 2 v3 ) dx dx + ........... + m C m (u1vm + uvm +1 ) = mC 0 u m + 1 v + ym +1 = ( uv )m +1 = +… +

(

)

C 0 + mC1 um v1 +

m

m +1

C 0 u m + 1v +

m +1

(

)

C1 + mC 2 um −1v2 + ........... + mC m uvm + 1

m

C1 um v1 +

m +1

C 2 um −1v2 ⎡∵ mC + mC = r −1 r ⎣

m +1

C m +1 uvm +1 .

Cr ⎤ ⎦

m +1

This shows that theorem is true for n = m + 1 also. By mathematical induction, theorem is true for all integer values of n. Hence, yn = (uv) n = un v + n C1 un −1v1 + n C2 un − 2 v2 + n C3un − 3 v3 + … + n Cn uvn . Example 1: If y = x2 e2x, prove that (yn)0 = 2n-2 n (n - 1). Solution:

y = x2 e2x

Differentiating n times using Leibnitz’s Theorem, n (n − 1) yn = x 2 2n e 2 x + n ⋅ 2 x 2n −1 e 2 x + ⋅ 2 ⋅ 2n − 2 e 2 x 2! Putting x = 0, (yn)0 = 2n 2 n (n

1)

Example 2: If u is a function of x, and y = eax u, prove that Dny = eax (D + a)n u, d where D = . dx Solution:

y = eax u Dn (eax u) = (Dn eax) u + nC1 (Dn−1 eax) (Du) + nC2 (Dn−2 eax) (D2u) + nC3 (Dn−3 eax) (D3u) + ………. + eax (Dnu)

2.24

Engineering Mathematics = (an eax) u + nC1 (an−1 eax) (Du) + nC2 (an−2 eax) (D2u) + nC3 (an−3 eax) (D3u) + ………. + eax (Dnu) = eax [an + nC1 an−1 (D) + nC2 an−2 (D2) + nC3 an−3 (D3) + … + Dn] u = eax (D + a)n u, where D = d dx

[Using Binomial Expansion]

Example 3: Find the nth order derivative of (ii) x3 cos x (iii) x2 ex cos x (i) y = x2 eax

(iv) x2 tan−1 x.

Solution: (i) y = x2 eax Let u = eax, v = x2 Differentiating n times using Leibnitz’s Theorem, yn = un v + nC1 un −1v1 + nC2 un − 2 v2 + … + nCn u vn = a n e ax ⋅ x 2 + n a n −1e ax ⋅ 2 x +

n(n − 1) n − 2 ax a e ⋅2 2!

= e ax [ x 2 a n + 2nxa n −1 + n(n − 1)a n − 2 ] (ii) y = x3 cos x Let u = cos x, v = x3 Differentiating n times using Leibnitz’s Theorem, yn = un v + nC1 un −1v1 + nC2 un − 2 v2 + … + nCn u vn

np ⎛ = cos ⎜ x + ⎝ 2

( n − 1)p ⎞ 3 ⎡ ⎟⎠ x + n cos ⎢ x + 2 ⎣

⎤ 2 ⎥ 3x ⎦ n( n − 1)( n − 2) ( n − 3)p n( n − 1) ( n − 2)p ⎤ ⎡ ⎡ cos ⎢ x + 6x + + cos ⎢ x + ⎥ 2 ⎦ 3! 2 2! ⎣ ⎣

np ⎛ = x 3 cos ⎜ x + ⎝ 2

⎤ ⎥6 ⎦

( n − 1)p ⎤ ⎞ ⎡ 2 ⎟⎠ + 3nx cos ⎢ x + 2 ⎥⎦ ⎣

( n − 3)p ⎤ ( n − 2)p ⎤ ⎡ ⎡ + 3 x n( n − 1) cos ⎢ x + + n( n − 1)( n − 2) cos ⎢ x + ⎥ 2 ⎥⎦ 2 ⎦ ⎣ ⎣ (iii) y = x2 ex cos x Let u = ex cos x, v = x2. n

np ⎞ ⎛ un = e x ( 2) 2 cos ⎜ x + ⎟ ⎝ 4 ⎠ Differentiating n times using Leibnitz’s Theorem,

[Using result (7)]

yn = un v + nC1un −1v1 + nC2 un − 2 v2 + … + nCn u vn n

np ⎞ 2 ⎛ x = e x ( 2) 2 cos ⎜ x + ⎟⎠ x + ne ( 2) ⎝ 4

n −1 2

( n − 1)p ⎤ ⎡ cos ⎢ x + 2x ⎣ 4 ⎦⎥

Differential Calculus I

+

n( n − 1) n −2 2 ( n − 2)p ⎡ ( 2) cos ⎢ x + 2! 4 ⎣

n

np ⎞ 2 ⎛ x = e x ( 2) 2 cos ⎜ x + ⎟ x + nxe ( 2) ⎝ 4 ⎠ +

2.25

n +1 2

⎤ ⎥2 ⎦

( n − 1)p ⎤ ⎡ cos ⎢ x + 4 ⎥⎦ ⎣

n( n − 1) 2n ( n − 2)p ⎡ ( 2) cos ⎢ x + 2! 4 ⎣

⎤ ⎥. ⎦

(iv) y = x2 tan−1 x Let u = tan−1 x, v = x2. sin nq 1 , whereq = tan −1 , r = 1 + x 2 n x r Differentiating n times using Leibnitz’s Theorem,

un = ( −1) n −1 ( n − 1)!

[ Refer Ex.12, page 12]

yn = un v + nC1un −1v1 + nC2 un − 2 v2 + … + nCn u vn sin nq 2 sin( n − 1)q ⋅ x + n( −1) n − 2 ( n − 2)! ⋅ 2x n r r n −1 n( n − 1) sin( n − 2)q ⋅2 + ( −1) n − 3 ( n − 3)! 2! r n− 2 sin nq sin( n − 1)q = ( −1) n −1 ( n − 1)! n ⋅ x 2 + 2nx( −1) n − 2 ( n − 2)! r n −1 r sin( n − 2)q + n( n − 1) ( −1) n − 3 ( n − 3)! r n− 2 = ( −1) n −1 ( n − 1)!

Example 4: Find nth order derivative of y = x2 ex and hence, prove that

yn =

1 1 n ( n - 1) y2 - n ( n - 2) y1 + ( n - 1) ( n - 2) y . 2 2

Solution: y = x2 ex Let u = ex, v = x2 Differentiating n times using Leibnitz’s Theorem,

yn = e x ⋅ x 2 + n e x ⋅ 2 x +

n(n − 1) x e ⋅2 2!

= e x [ x 2 + 2nx + n(n − 1)] Putting

n = 1 and 2 successively in Eq. (1), y1 = e x ( x 2 + 2 x),

Consider ,

y 2 = e x ( x 2 + 4 x + 2)

1 1 n(n − 1) y2 − n(n − 2) y1 + (n − 1) (n − 2) y 2 2

... (1)

2.26

Engineering Mathematics

1 1 n(n − 1) e x ( x 2 + 4 x + 2) − n (n − 2) e x ( x 2 + 2 x) + (n − 1) (n − 2) x 2 e x 2 2 2 x = e [ x + 2nx + n (n − 1)] = yn =

Example 5: If y = xn log x, prove that yn+ 1 =

n! . x

Solution: y = xn log x Differentiating w.r.t. x, 1 + nx n −1 ⋅ log x x xy1 = x n + nx n log x y1 = x n

= x n + ny Differentiating the above equation n times using Leibnitz’s Theorem, xyn+1 + nyn = n ! + nyn yn+1 =

n! . x

Example 6: If y = (x2 – 1)n, prove that (x2 – 1) yn+2 + 2x yn+1 - n (n + 1) yn = 0. Solution: y = (x2

1)n

Differentiating w.r.t. x, y1 = n( x 2 − 1) n −1 ⋅ 2 x ( x 2 − 1) y1 = n ( x 2 − 1) n 2 x = 2nyx Differentiating again w.r.t. x, (x2 1) y2 + 2xy1 = 2 (ny1x + ny) (x2 1) y2 + (2x 2nx) y1 = 2ny Differentiating the above equation n times using Leibnitz’s Theorem, n(n − 1) ( x 2 − 1) yn + 2 + n ⋅ 2 xyn +1 + ⋅ 2 yn + (2 x − 2nx) yn +1 + n.2(1 − n) yn = 2nyn 2! ( x 2 − 1) yn + 2 + 2 xyn +1 − n(n + 1) yn = 0. Example 7: If y = sin [log (x2 + 2x + 1)], prove that (x + 1)2 yn+2 + (2n + 1) (x + 1) yn+1 + (n2 + 4) yn = 0. Solution: y = sin [log (x2 + 2x + 1)] = sin [log (x + 1)2] = sin [2 log (x + 1)] Differentiating w.r.t. x, 2 x +1 ( x + 1) y1 = 2 cos[2 log( x + 1)] y1 = cos[2 log( x + 1)] ⋅

Differential Calculus I

2.27

Differentiating again w.r.t. x, ( x + 1) y2 + y1 = −2 sin[ 2 log( x + 1)] ⋅

2 ( x + 1)

(x + 1)2 y2 + (x + 1) y1 = 4y Differentiating the above equation n times using Leibnitz’s Theorem, n(n − 1) ( x + 1) 2 yn + 2 + n ⋅ 2 ( x + 1) yn +1 + ⋅ 2 yn + ( x + 1) yn +1 + n ⋅ yn = −4 yn 2! (x + 1)2 yn+2 + (2n + 1) (x + 1) yn+1 + (n2 + 4) yn = 0. Example 8: If y = a cos (log x) + b sin (log x), prove that x2 yn+2 + (2n + 1) xyn+1 + (n2 + 1) yn = 0. y = a cos (log x) + b sin (log x)

Solution: Differentiating w.r.t. x,

1 1 y1 = − a sin (log x) ⋅ + b cos (log x) ⋅ x x xy1 = -a sin (log x) + b cos (log x) Differentiating again w.r.t. x, 1 1 xy2 + y1 = − a cos (log x) ⋅ − b sin (log x) ⋅ x x x2y2 + xy1 = -y Differentiating the above equation n times using Leibnitz’s Theorem, n (n − 1) x 2 y n + 2 + n ⋅ 2 x y n +1 + ⋅ 2 yn + xyn +1 + nyn = − yn 2! x2yn+2 + (2n + 1) x yn+1 + (n2 + 1) yn = 0. n

⎛ y⎞ ⎛ x⎞ Example 9: If cos ⎜ ⎟ = log ⎜ ⎟ , prove that x2 yn+2 + (2n + 1)xyn+1 + 2n2yn = 0. ⎝b⎠ ⎝ n⎠ -1

n

Solution:

⎛ y⎞ ⎛ x⎞ ⎛ x⎞ cos −1 ⎜ ⎟ = log ⎜ ⎟ = n log ⎜ ⎟ ⎝b⎠ ⎝n⎠ ⎝n⎠ ⎡ y ⎛ x ⎞⎤ = cos ⎢ n log ⎜ ⎟ ⎥ b ⎝ n ⎠⎦ ⎣

Differentiating w.r.t. x,

⎡ ⎛ x ⎞⎤ y = b cos ⎢ n log ⎜ ⎟ ⎥ ⎝ n ⎠⎦ ⎣

⎡ 1 1 −bn ⎡ ⎛ x ⎞⎤ ⎛ x ⎞⎤ y1 = −b sin ⎢ n log ⎜ ⎟ ⎥ ⋅ n ⋅ ⋅ = sin ⎢ n log ⎜ ⎟ ⎥ x n x ⎝ n ⎠⎦ ⎝ n ⎠⎦ ⎣ ⎣ n ⎡ ⎛ x ⎞⎤ xy1 = −bn sin ⎢ n log ⎜ ⎟ ⎥ ⎝ n ⎠⎦ ⎣

2.28

Engineering Mathematics

Differentiating again w.r.t. x,

⎡ ⎛ x ⎞⎤ 1 1 xy2 + y1 = −bn cos ⎢ n log ⎜ ⎟ ⎥ n ⋅ ⋅ ⎝ n ⎠⎦ x n ⎣ n ⎡ ⎤ −bn 2 x ⎛ ⎞ = cos ⎢ n log ⎜ ⎟ ⎥ x ⎝ n ⎠⎦ ⎣ ⎡ ⎛ x ⎞⎤ x 2 y2 + xy1 = −bn 2 cos ⎢ n log ⎜ ⎟ ⎥ = −n 2 y ⎝ n ⎠⎦ ⎣ Differentiating the above equation n times using Leibnitz’s Theorem, n(n − 1) x 2 yn + 2 + n ⋅ 2 xyn +1 + ⋅ 2 yn + xyn +1 + nyn = − n 2 yn 2! x2 yn+2 + (2n + 1) xyn+1 + 2n2 yn = 0 1 m



1 m

Example 10: If y + y = 2x, prove that (x2 - 1) yn+2 + (2n + 1) x yn+1 + (n2 - m2) yn = 0. 1

1 m

ym + y

Solution:

1

= 2x

1

ym +

y

= 2x

1 m

2 m

1

y + 1 = 2x y m 2

1

1

Hence,

ym =

1

2x y m + 1 = 0, equation is quadratic in y m .

ym

2x ± 4x2 − 4 = x ± x2 − 1 2

(

y = x ± x2 − 1

)

m

Differentiating w.r.t. x,

(

y1 = m x ± x 2 − 1

(

= m x ± x −1 =

2

(x ± x −1 m 2

y1 x 2 − 1 = my ( x 2 − 1) y12 = m 2 y 2

)

m −1

)

m −1

⎛ 2x ⎞ ⎜1 ± ⎟ 2 ⎝ 2 x −1 ⎠

(

x2 − 1

x2 − 1 ± x x −1 2

)

m

)

Differential Calculus I

2.29

Differentiating again w.r.t. x, (x2 - 1) 2y1 y2 + 2x y12 = m2 2y y1 (x2 - 1) y2 + xy1 = m2 y Differentiating the above equation n times using Leibnitz’s Theorem, n(n − 1) ⋅ 2 yn + x yn +1 + nyn = m 2 yn 2! (x2 - 1) yn+2 + (2n + 1) x yn+1 + (n2 - m2) yn = 0

( x 2 − 1) yn + 2 + n ⋅ 2 x yn +1 +

⎛1 ⎞ Example 11: If x = cosh ⎜ log y ⎟ , prove that ⎝m ⎠ (x2 - 1) yn+2 + (2n + 1) xyn+1 + (n2 - m2) yn = 0.

Solution:

⎛1 ⎞ x = cosh ⎜ log y ⎟ ⎝m ⎠ 1 cosh −1 x = log y m

)

(

(

log y = m log x + x 2 − 1 = log x + x 2 − 1

(

y = x + x2 − 1

)

)

m

m

Differentiating w.r.t. x,

(

y1 = m x + x 2 − 1

( m( x + =

)

m −1

) x −1)

= m x + x −1 2

2

m −1

⎛ 2x ⎞ ⎜1 + ⎟ 2 ⎝ 2 x −1 ⎠

(

x2 − 1 + x

)

x −1 2

m

x −1 2

=

my x2 − 1

y1 x 2 − 1 = my ( x 2 − 1) y12 = m 2 y 2 Differentiating again w.r.t. x, (x2 - 1) 2y1 y2 + 2x y12 = m2 2y y1 (x2 - 1) y2 + xy1 = m2 y

2.30

Engineering Mathematics

Differentiating the above equation n times using Leibnitz’s Theorem, n(n − 1) ⋅ 2 yn + x yn +1 + nyn = m 2 yn 2! (x2 - 1) yn+2 + 2nx yn+1 + n2 yn - nyn + x yn+1 + nyn = m2 yn

( x 2 − 1) yn + 2 + n ⋅ 2 x yn +1 +

(x2 - 1) yn+2 + (2n + 1) x yn+1 + (n2 - m2) yn = 0. Example 12: If y =

sin 1 x 1 x2 y=

Solution:

, prove that (1 - x2) yn+1 – (2n + 1) xyn – n2 yn–1 = 0.

sin −1 x 1 − x2

y 1 − x 2 = sin −1 x y 2 (1 − x 2 ) = (sin −1 x) 2 Differentiating the above equation w.r.t. x, 2 yy1 (1 − x 2 ) + y 2 ( −2 x) = 2 sin −1 x ⋅

1 1 − x2

= 2y

(1 - x2) y1 - xy = 1 Differentiating the above equation n times using Leibnitz’s Theorem, n(n − 1) (1 − x 2 ) yn +1 + n (− 2 x) yn + (−2) yn −1 − xyn − nyn −1 = 0 2! (1 - x2) yn+1 - (2n + 1) xyn n2 yn-1 = 0. Example 13: If y = sec−1 x, prove that x (x2 - 1) yn+2 + [(2 + 3n) x2 - (n + 1)] yn+1 + n (3n + 1) xyn + n2 (n – 1) yn–1 = 0. Solution: y = sec−1 x Differentiating w.r.t. x, y1 = x2 (x2

−1 x x2 − 1

1) y12 = 1

Differentiating again w.r.t. x, 2x (x2 - 1) y12 + x2 2x y12 + x2 (x2 - 1) 2y1 y2 = 0 (x2 - 1) y1 + x2 y1 + x (x2 - 1) y2 = 0

Differential Calculus I

2.31

Differentiating the above equation n times using Leibnitz’s Theorem, n(n − 1) n(n − 1) ⋅ 2 yn −1 + x 2 yn +1 + n ⋅ 2 x yn + ⋅ 2 yn −1 2! 2! n(n − 1) n(n − 1) (n − 2) + x ( x 2 − 1) yn + 2 + n (3 x 2 − 1) yn +1 + ⋅ 6 x yn + ⋅ 6 yn − 1 = 0 2! 3!

( x 2 − 1) yn +1 + n ⋅ 2 x yn +

x (x2 - 1) yn+2 + [(2 + 3n) x2

(n + 1)] yn+1 + n (3n + 1) xyn + n2 (n

1) yn−1 = 0

Example 14: If y = sinh−1 x, prove that (1 + x2) yn+2 + (2n + 1) xyn+1 + n2yn = 0.

(

Solution: y = sinh −1 x = log x + x 2 + 1

)

Differentiating w.r.t. x,

⎛ 2x ⎞ 1 y1 = ⋅ ⎜⎜1 + ⎟⎟ = 2 2 x + x + 1 ⎝ 2 x + 1 ⎠ x + x2 + 1 (x2 + 1) y12 = 1 1

(

x2 + 1 + x x +1 2

Differentiating again w.r.t. x, (x2 + 1) 2y1 y2 + 2xy12 = 0 (x2 + 1) y2 + xy1 = 0 Differentiating the above equation n times using Leibnitz’s Theorem, n (n − 1) ⋅ 2 yn + xyn +1 + nyn = 0 2! (x2 + 1) yn+2 + (2n + 1) xyn+1 + n2yn = 0

( x 2 + 1) yn + 2 + n ⋅ 2 yn +1 +

a sin Example 15: If y = e

1

x

, prove that

(1 – x2) yn+2 – (2n + 1) xyn+1 – (n2 + a2) yn = 0. Solution: y = e a sin

−1

x

Differentiating w.r.t. x, y1 = e a sin

−1

y1 1 − x 2 = ay (1 - x2) y12 = a2 y2

x



a 1 − x2

)

2.32

Engineering Mathematics

Differentiating again w.r.t. x, (1 - x2) 2y1 y2 + (-2x) y12 = a2 2y y1 (1 - x2) y2 - xy1 = a2 y Differentiating the above equation n times using Leibnitz’s Theorem, (1 − x 2 ) yn + 2 + n(− 2 x) yn +1 +

n (n − 1) (− 2) yn − xyn +1 − nyn = a 2 yn 2!

(1 - x2) yn+2 - (2n + 1) xyn+1 - (n2 + a2) yn = 0.

⎛a+ x⎞ , prove that Example 16: If y = tan -1 ⎜ ⎝ a - x ⎟⎠ (a2 + x2) yn+2 + 2 (n + 1) x yn+1 + n (n + 1) yn = 0. Solution:

⎛ a + x⎞ y = tan −1 ⎜ ⎝ a − x ⎟⎠

Putting x = a tan q, ⎛ 1 + tan q ⎞ ⎛p ⎞ y = tan −1 ⎜ = tan −1 tan ⎜ + q ⎟ ⎝ 1 − tan q ⎟⎠ ⎝4 ⎠ =

p p ⎛x⎞ + q = + tan −1 ⎜ ⎟ ⎝a⎠ 4 4

Differentiating w.r.t. x, y1 =

1 1 a ⋅ = 2 2 x a x + a2 1+ 2 a (x2 + a2) y1 = a

(x2 + a2) y2 + 2x y1 = 0 Differentiating the above equation n times using Leibnitz’s Theorem, n(n − 1) ( x 2 + a 2 ) y n + 2 + n ⋅ 2 x y n +1 + ⋅ 2 y n + 2 x y n +1 + n ⋅ 2 y n = 0 2! (x2 + a2) yn+2 + 2(n + 1) x yn+1 + n (n + 1) yn = 0.

1+ x , prove that y = (1 – x2) y1 and hence, deduce that 1- x (1 – x2) yn – [2 (n – 1) x + 1] yn–1 – (n – 1) (n – 2) yn – 2 = 0.

Example 17: If y =

Solution:

y=

1+ x 1− x

log y = log =

1+ x 1− x

1 [log (1 + x) − log (1 − x)] 2

Differential Calculus I

2.33

Differentiating w.r.t. x,

1 1⎛ 1 1 ⎞ 1 ⋅ y1 = ⎜ + ⎟= y 2 ⎝ 1 + x 1 − x ⎠ 1 − x2 (1

x2) y1 = y

Differentiating the above equation n times using Leibnitz’s Theorem, (1 − x 2 ) yn +1 + n ( −2 x) yn + Replacing n by n

n(n − 1) ( −2) yn −1 = yn 2!

1,

(1 − x 2 ) yn − [2 (n − 1) x + 1] yn −1 − (n − 1) (n − 2) yn − 2 = 0. Example 18: If f (x) = tan x, prove that

f n (0) − nC2 f n − 2 (0) + nC4 f n − 4 (0) + …… = sin

no . 2

f ( x) = tan x sin x = cos x cos x ⋅ f ( x) = sin x

Solution:

Differentiating the above equation n times using Leibnitz’s Theorem, cos x f n (x) + nC1 ( sin x) f n 1 (x) + nC2 ( cos x) f n 2 (x) + nC3 (sin x) f n 3 (x) np ⎞ np ⎞ ⎛ + nC4 (cos x) f n 4 (x) + ……… + f ( x ) ⋅ cos ⎛⎜ x + ⎟ = sin ⎜⎝ x + ⎟ ⎝ 2 ⎠ 2 ⎠ Putting x = 0, np ⎛ np ⎞ f n (0) − nC2 f n − 2 (0) + nC4 f n − 4 (0) + ………… + f (0) cos ⎜ ⎟ = sin . ⎝ 2 ⎠ 2

)

(

2

2 Example 19: If y = ⎡⎢log x + 1 + x ⎤⎥ , prove that yn+2 (0) = – n2yn (0). ⎣ ⎦

)

(

Solution: y = ⎡log x + 1 + x 2 ⎤ ⎢⎣ ⎥⎦

2

Differentiating w.r.t. x,

)

(

1 y1 = 2 ⎡log x + 1 + x 2 ⎤ ⎣⎢ ⎦⎥ x + 1 + x 2 1 = 2 log x + 1 + x 2 ⋅ 1 + x2

(

)

⎛ ⎞ 1 ⋅ 2 x ⎟⎟ ⎜⎜1 + 2 ⎝ 2 1+ x ⎠

2.34

Engineering Mathematics

(

y1 1 + x 2 = 2 log x + 1 + x 2

) )

(

2

(1 + x 2 ) y12 = 4 ⎡log x + 1 + x 2 ⎤ = 4 y ⎥⎦ ⎣⎢ Differentiating again w.r.t. x, (1 + x2) 2y1 y2 + 2xy12 = 4y1 (1 + x2) y2 + xy1 = 2 Differentiating the above equation n times using Leibnitz’s Theorem, n (n − 1) ⋅ 2 yn + xyn +1 + nyn = 0 2! (1 + x2) yn+2 + (2n + 1) xyn+1 + n2 yn = 0

(1 + x 2 ) yn + 2 + n ⋅ 2 xyn +1 +

Putting x = 0, (yn+2)0 = -n2 (yn)0.

(

2 Example 20: If y = x + 1 + x

)

m

, prove that

(i) (y2n)0 = [m2 - (2n - 2)2] [m2 - (2n - 4)2] … [m2 - 22] m2. (ii) (y2n+1)0 = [m2 - (2n - 1)2] [m2 - (2n - 3)2] … [m2 - 12] m.

(

Solution: y = x + 1 + x 2

)

m

Differentiating w.r.t. x,

(

)

(

)

y1 = m x + 1 + x 2 1 + x 2 ⋅ y1 = m x + 1 + x 2

m −1

m

⎛ 2x ⎜⎜1 + 2 ⎝ 2 1+ x

= my

⎞ ⎟⎟ ⎠ ... (1)

(1 + x2) y12 = m2 y2 Differentiating again w.r.t. x, (1 + x2) 2y1 y2 + 2xy12 = m2 2yy1 (1 + x2) y2 + xy1 = m2 y

… (2)

Differentiating the above equation n times using Leibnitz’s Theorem, n(n − 1) ⋅ 2 yn + xyn +1 + nyn = m 2 yn 2! (1 + x2) yn+2 + (2n + 1) xyn+1 + (n2 m2) yn = 0

(1 + x 2 ) yn + 2 + n ⋅ 2 xyn +1 +

… (3)

Putting x = 0 in Eqs (1), (2) and (3), and

(y1)0 = m, (y2)0 = m2 (yn+2)0 = (m2 n2) yn(0)

[∵ y(0) = 1] … (4)

Differential Calculus I

2.35

Putting n = 1, 2, 3, 4, … in Eq. (4), y3 (0) = (m2 12) y1 (0) = (m2 12) m y4 (0) = (m2 22) y2 (0) = (m2 22) m2 y5 (0) = (m2 32) y3 (0) = (m2 32) (m2 12) m y6 (0) = (m2 42) y4 (0) = (m2 42) (m2 22) m2 and so on. In general, Even derivative, y2n (0) = [m2 - (2n - 2)2] [m2 - (2n - 4)2] … (m2 22) m2 Odd derivative, y2n+1 (0) = [m2 - (2n - 1)2] [m2 - (2n - 3)2] … (m2 12) m.

(

)

2 Example 21: If y = log x + x + 1 , prove that

2 y2n (0) = 0 and y2n+1 (0) = (-1)n [1 ⋅ 3 ⋅ 5 … ( 2n − 1) ] . 2

(

2 Solution: y = log x + x + 1

2

2

)

Differentiating w.r.t. x,

y1 =

⎛ 2x ⎞ ⎜⎜1 + ⎟⎟ x + 1 + x ⎝ 2 1 + x2 ⎠ 1

2

1 + x 2 ⋅ y1 = 1 (x2 + 1) y12 = 1 Differentiating again w.r.t. x, (x2 + 1) 2y1 y2 + 2xy12 = 0 (x2 + 1) y2 + xy1 = 0

... (1)

… (2)

Differentiating the above equation n times using Leibnitz’s Theorem, n(n − 1) ( x 2 + 1) yn + 2 + n ⋅ 2 xyn +1 + ⋅ 2 yn + xyn +1 + nyn = 0 2! (x2 + 1) yn+2 + (2n + 1) xyn+1 + n2yn = 0 … (3) Putting x = 0 in Eqs (1), (2) and (3), y1(0) = 1 and y2(0) = 0 and yn+2(0) = -n2 yn(0) Putting n = 1, 2, 3, 4, … in Eq. (4), … (4) y3(0) = -12 y1(0) = y4(0) = -22 y2(0) = 0 2 2 ( 1) 2 12 32 y5(0) = -32 y3(0) y6(0) = 0 y7(0) = -52 y5(0) = 52 ( 1) 2 12 32 ( 1)3 12 32 52 etc. In general, Even derivative, y2n(0) = 0 Odd derivative, y2n+1(0) = ( 1) n [12 32 52 … (2n 1) 2 ] Example 22: If y = (sin–1 x)2, prove that (i) (1 - x2) yn+2 - (2n + 1) xyn+1 - n2yn = 0. (ii) y2n+1(0) = 0 and y2n(0) = 22n–1 [(n – 1)!]2.

2.36

Engineering Mathematics

Solution: (i) y = (sin−1 x)2 Differentiating w.r.t. x, y1 = 2 sin −1 x ⋅ (1

1

... (1)

1 − x2

x 2)y12 = 4 (sin−1 x)2 = 4y

Differentiating again w.r.t. x, (1

x 2)2y1 y2 2xy12 = 4y1 (1 x 2) y2 xy1 = 2

… (2)

Differentiating the above equation n times using Leibnitz’s Theorem, (1 − x 2 ) yn + 2 + n ⋅ (−2 x) yn +1 + (1

n(n − 1) (−2) yn − xyn +1 − nyn = 0 2!

x 2)yn+2

(2n + 1) xyn+1

n2yn = 0

… (3)

(ii) Putting x = 0 in Eq. (1), (2) and (3), y1(0) = 0, y2(0) = 2 = 22–1 [(1

1)!]2

yn+2(0) = n2 yn(0) Putting n = 1, 2, 3, 4, …, in Eq. (4) y3(0) = 12 y1(0) = 0 y4(0) = 22 y2(0) = 22 2 = 23 = 24–1 [(2 y5(0) = 0 y6(0) = 42 y4(0) = 42 23 = 26–1 [(3 In general, Even derivative, Odd derivative,

y2n(0) = 22n 1 [(n y2n+1(0) = 0.

… (4) 1)!]2

1)!]2 etc.

1)!]2

Example 23: If y = sin (m sin–1 x), prove that (i) (1 - x2) yn+2 - (2n + 1) xyn+1 - (n2 - m2) yn = 0. (ii) (yn )0 = [(n - 2)2 - m2] [(n - 4)2 - m2] … (1 - m2) m, if n is odd = 0, if n is even. Solution: (i) y = sin (m sin–1 x) Differentiating w.r.t. x, y1 = cos (m sin −1 x) ⋅ 1 − x 2 ⋅ y1 = m cos (m sin −1 x)

m 1 − x2

... (1)

Differential Calculus I

2.37

(1 - x2) y12 = m2 cos2 (m sin–1 x) = m2 [1 – sin2 (m sin–1 x)] (1 - x2) y12 = m2 (1 – y2) Differentiating again w.r.t. x, (1 - x2) 2y1 y2 + y12 (–2x) = m2 ( 2yy1) (1 - x2) y2 - xy1 = -m2y

… (2)

Differentiating the above equation n times using Leibnitz’s Theorem, (1 − x 2 ) yn + 2 + n(−2 x) yn +1 +

n(n − 1) (−2) yn − xyn +1 − nyn = −m 2 yn 2!

(1 - x2) yn+2 - (2n + 1) xyn+1 - (n2 (ii) Putting

m2) yn = 0

… (3)

x = 0 in Eqs (1), (2) and (3), y1 (0) = cos ( m sin −1 0 ) ⋅

m 1− 0

=m

y1(0) = m y2(0) = – m2 y(0) = 0 also, yn+2(0) = (n2 m2) yn(0) Putting n = 1, 2, 3, … in Eq. (4), y3(0) = (12 m2) y1(0) = (12 y4(0) = (22 m2) y2(0) = 0 y5(0) = (32 y6(0) = (42

… (4) m2) m

m2) y3(0) = (32 m2) (12 m2) y4(0) = 0 etc.

m2) m

In general, yn(0) = [(n 2)2 m2] [(n = 0, if n is even.

4)2

m2] … (12

m2) m, if n is odd

Example 24: If y = tan−1 x, prove that (i) (x2 + 1) yn+1 + 2nxyn + n (n – 1) yn–1 = 0 (ii) yn (0) = 0, if n is even

= ( -1)

n -1 2 ( n - 1)!,

if n is odd.

Solution: (i) y = tan−1 x Differentiating w.r.t. x, 1 1 + x2 (x2 + 1) y1 = 1 y1 =

... (1)

Differentiating again w.r.t. x, (x2 + 1) y2 + 2 x y1 = 0

… (2)

2.38

Engineering Mathematics

Differentiating Eq. (1) n times using Leibnitz’s Theorem, ( x 2 + 1) yn +1 + n ⋅ 2 xyn +

n(n − 1) ⋅ 2 yn −1 = 0 2!

(x2 + 1) yn+1 + 2nxyn + n (n

1) yn 1 = 0

... (3)

(ii) Putting x = 0 in Eqs (1), (2) and (3), y1(0) = 1, y2(0) = 0 yn+1(0) = n(n Putting n = 2, 3, 4, … in Eq. (4),

1) yn 1(0)

y3 (0) = −2 (2 − 1) y1 (0) = −2 = −(2 !) = (−1) y4(0) = 3(3

... (4) 3 −1 2

(2 !)

1) y2(0) = 0

y5 (0) = −4 (4 − 1) y3 (0) = −4 (3) (−2) = (−1) 2 (4 !) = (−1)

5 −1 2

y6(0) = 5(4) y4(0) = 0 y7 (0) = −6 (5) y5 (0) = −6 (5) (−1) 2 (4 !) = (−1)3 (6 !) = (−1)

(4!)

7 −1 2

(6 !)

In general, yn = 0, if n is even = ( −1)

n −1 2

(n − 1)!, if n is odd.

-1

Example 25: If y = e m cos x, find ( yn )(0). −1

Solution: y = e m cos x Differentiating w.r.t. x, y1 = e m cos

−1

x

⎛ −m ⎞ ⎜ ⎟ ⎝ 1 − x2 ⎠

... (1)

(1 - x2) y12 = m2 y2 Differentiating again w.r.t. x (1 x2) 2y1 y2 2xy12 = 2m2 yy1 (1 x2) y2 xy1 = m2 y Differentiating the above equation n times using Leibnitz’s Theorem, n (n − 1) (1 − x 2 ) yn + 2 + n(−2 x) yn +1 + (−2) yn − xyn +1 − nyn = m 2 yn 2! (1 x2) yn+2 (2n + 1) xyn+1 (n2 + m2) yn = 0 Putting x = 0 in Eqs (1), (2) and (3), y1 (0) = − me m cos

−1

0

= − me

yn+2(0) = (n2 + m2) yn(0)

mp 2

, y2 ( 0 ) = m 2 y ( 0 ) = m 2 e

… (2)

... (3)

mp 2

... (4)

Differential Calculus I

2.39

Putting n = 1, 2, 3, 4, … in Eq. (4), y3 (0) = (12 + m 2 ) y1 (0) = − me

mp 2

y4 ( 0 ) = ( 2 2 + m 2 ) y2 ( 0 ) = m 2 e

mp 2

y5 (0) = (32 + m 2 ) y3 (0) = − me y6 (0) = ( 4 2 + m 2 ) y4 (0) = m 2 e

(12 + m 2 )

mp 2 mp 2

( 22 + m 2 ) (12 + m 2 ) (32 + m 2 ) ( 22 + m 2 ) ( 4 2 + m 2 )

In general, mp 2

Even derivative,

y2 n ( 0 ) = m 2 e

Odd derivative,

y2 n +1 (0) = − me

( 22 + m 2 ) ( 4 2 + m 2 )…[( 2n − 2) 2 + m 2 ] mp 2

(12 + m 2 ) (32 + m 2 )…[( 2n − 1) 2 + m 2 ]

Example 26: If x + y = 1, prove that 2 2 dn ( x n y n ) = n ! ⎡⎢ y n - ( nC1 ) y n -1 x + ( nC2 ) y n - 2 x 2 n ⎣ dx

- ( nC 3 ) y n - 3 x 3 +… ……… + (-1) n x n ⎤⎥ . ⎦ 2

Solution: x + y = 1, y = 1

x, y1 = 1

Differentiating n times using Leibnitz’s Theorem,

dn n n dn n x y = n [ x (1 − x) n ] dx n dx n! n (n − 1) n ! 2 = n !(1 − x) n + n ⋅ x ⋅ n (1 − x) n −1 (−1) + x n(n − 1) (1 − x) n − 2 (−1) 2 1! 2! 2! n (n − 1) (n − 2) n ! 3 + x n (n −1) (n − 2) (1 − x) n −3 (−1)3 + ……… + (−1) n x n 3! 3! ⎡ d n (ax + b) m a n m !(ax + b) m − n ⎤ = ⎢∵ ⎥ (m − n)! dx n ⎣ ⎦ 2 2 2 n = n ! ⎡⎢ y n − ( n C1 ) y n −1 x + ( nC2 ) y n − 2 x 2 − ( nC3 ) y n −3 x 3 + ……… + ( −1) x n ⎤⎥ ⎣ ⎦ [∵ (1 − x) = y ]

Example 27: By finding two different ways the nth derivative of x2n, prove that

1+

n2 n2 ( n - 1)2 n2 ( n - 1)2 ( n - 2)2 ( 2n)! . + + + …………… = 2 2 2 2 2 2 ( n !)2 1 1 2 1 2 3

2.40

Engineering Mathematics

Let y = x2n

Solution:

yn = =

(2n)! 2 n − n x (2n − n)!

⎡ d n (ax + b) m a n m !(ax + b) m − n ⎤ = ⎢∵ ⎥ (m − n)! dx n ⎦ ⎣

(2n)! n x n!

... (1)

y = xn ⋅ xn

Now,

Differentiating the above equation n times using Leibnitz’s Theorem, n! n (n − 1) n! x n − ( n − 2) x n − ( n −1) + n (n − 1) x n − 2 2! (n − n + 1)! (n − n + 2)! n (n − 1) (n − 2) n! + n (n − 1) (n − 2) x n − 3 x n − (nn − 3) + ………… 3! (n − n + 3)!

yn = x n ⋅ n !+ n ⋅ nx n −1 ⋅

= xn ⋅ n! + n2 ⋅

n ! n n 2 (n − 1) 2 n 2 (n − 1) 2 (n − 2) 2 n x + n ! x + n ! x n +………… 1! (2 !) 2 (3!) 2

⎡ n 2 n 2 (n − 1) 2 n 2 (n − 1) 2 (n − 2) 2 ⎤ = x n ⋅ n ! ⎢1 + 2 + 2 2 + + …………⎥ 2 2 2 1 ⋅2 1 ⋅ 2 ⋅3 ⎣ 1 ⎦

... (2)

n

Equating coefficients of x in Eqs (1) and (2), ⎡ n 2 n 2 (n − 1) 2 n 2 (n − 1) 2 (n − 2) 2 ⎤ n ! ⎢1 + 2 + 2 2 + + …………⎥ = 2 2 2 1 ⋅2 1 ⋅ 2 ⋅3 ⎣ 1 ⎦ 2 2 2 2 2 n n (n − 1) n (n − 1) (n − 2) 2 Hence, 1 + 2 + 2 2 + +……………… …y = 1 1 ⋅2 12 ⋅ 22 ⋅ 32 Example 28: If y = Solution: y =

(2n)! (n !) (2n)! (n !) 2

log x ( -1) n n ! ⎡ 1 1 ⎞⎤ ⎛ log x - ⎜ 1 + + ... + ⎟ ⎥ . , prove that yn = ⎝ x n ⎠⎦ 2 x n + 1 ⎢⎣

log x x

Differentiating n times using Leibnitz’s Theorem, yn =

(−1) n n ! (−1) n −1 (n − 1)! 1 n (n − 1) (−1) n − 2 (n − 2)! ⎛ 1 ⎞ ⋅ log x + n ⋅ + ⎜− 2 ⎟ n +1 2! x x n −1 x xn ⎝ x ⎠ +

n (n − 1) (n − 2) (−1) n −3 (n − 3)! ⎛ 2 ⎞ 1 (−1) n −1 (n − 1)! ⋅ ………… + + ⋅ ⎜ 3⎟ 3! x xn−2 xn ⎝x ⎠ [Using result (2) and (3)]

Differential Calculus I

2.41

(−1) n n ! (−1) n n ! (−1) n n ! (−1) n n ! (−1) n n ! ……… x log ⋅ − − − − … … − x n +1 x n +1 2 x n +1 3x n +1 nx n +1 n (−1) n ! ⎡ 1 ⎞⎤ ⎛ 1 1 = log x − ⎜1 + + + … + ⎟ ⎥ . n ⎠⎦ 2 3 x n +1 ⎢⎣ ⎝ =

Exercise 2.2 3. If y = e ax [a2 x2 - 2nax + n (n + 1)], prove that yn = an+2 x2 eax.

1. Find the nth order derivative w.r.t. x (i) xex (ii) x2e2x (iii) x log (x + 1) (iv) x3 sin 2x 2 (v) y = x sin x. ⎡ Ans. : (i) e x ( x + n) ⎢ n 2 n n −1 2x ⎢ (ii) e [2 x + 2 nx + n ( n − 1) 2 ] ⎢ ( −1) n − 2 ( n − 2)!( x + n) ⎢(iii) ( x + 1) n ⎢ ⎢ ⎢(iv) 2n x 3 sin ⎛⎜ 2 x + np ⎞⎟ + ⎝ ⎢ 2 ⎠ ⎢ p⎤ ⎡ ⎢ 3n x 2 2n −1 sin ⎢ 2 x + ( n − 1) ⎥ ⎢ 2⎦ ⎣ ⎢ p⎤ ⎡ ⎢ + 3n ( n − 1) x 2n − 2 sin ⎢ 2 x + ( n − 2) ⎥ ⎢ 2⎦ ⎣ ⎢ n−3 + n ( n − 1) ( n − 2) 2 ⎢ ⎢ np ⎤ ⎡ ⎢ sin ⎢ 2 x + ( n − 3) ⎥ 2 ⎦ ⎢ ⎣ ⎢ ⎢( v) x 2 sin ⎛ x + np ⎞ ⎜⎝ ⎟ ⎢ 2 ⎠ ⎢ p⎤ ⎡ ⎢ + 2nx sin ⎢ x + ( n − 1) ⎥ ⎢ 2⎦ ⎣ ⎢ p⎤ ⎡ ⎢ + ( n2 − n) sin ⎢ x + ( n − 2) ⎥ ⎢⎣ 2⎦ ⎣ 2. If y = 7x x7, find y5. ⎡ Ans. : (log 7)5 7 x x 7 ⎤ ⎢ ⎥ 4 x 6 + 35 (log 7) 7 x ⎢ ⎥ 3 x 5 ⎥ ⎢ + 420 (log 7) 7 x ⎢ ⎥ ⎢ + 2100 (log 7) 2 7 x x 4 ⎥ ⎢ ⎥ + 4200 (log 7) 7 x x 3 ⎥ ⎢ ⎢ ⎥ + 2520 7 x x 2 ⎣ ⎦

4. If y = x2 sin x, prove that ⎤ ⎥ ⎥ ⎥ ⎥ ⎥ ⎥ ⎥ ⎥ ⎥ ⎥ ⎥ ⎥ ⎥ ⎥ ⎥ ⎥ ⎥ ⎥ ⎥ ⎥ ⎥ ⎥ ⎥ ⎥ ⎥ ⎥ ⎥ ⎥⎦

np ⎞ ⎛ yn = ( x 2 − n2 + n) sin ⎜ x + ⎟ ⎝ 2 ⎠ np ⎞ . ⎛ − 2nx cos ⎜ x + ⎟ ⎝ 2 ⎠ 5. If x = tan log y, prove that (1 + x 2 ) yn +1 + (2nx − 1) yn + n (n − 1) yn −1 = 0. ⎡ Hint : log y = tan −1 x, y = e tan ⎣

−1

x

⎤ ⎦

6. If y = cos (m sin−1 x), prove that (1 - x2) yn+2 - (2n + 1) xyn+1 + (m2 - n2) yn = 0 Hence, obtain yn (0). ⎡ Ans. : yn (0) = ( n2 − m 2 )........... ⎤ ⎥ ⎢ ( 4 2 − m 2 )( 22 − m 2 )( − m 2 ) ⎥⎦ ⎢⎣ 7. If x = sin q, y = sin 2q, prove that (1 - x2) yn+2 - (2n + 1) xyn+1 (n2 - 4) yn = 0. ⎡ Hint : y = 2 sin q cos q = 2 x 1 − x 2 ⎤ ⎣ ⎦ 8. If y = x2ex, prove that yn =

1 n (n − 1) y2 − n (n − 2) y1 2 1 + (n − 1) (n − 2) y. 2

2.42

Engineering Mathematics

2.4 MEAN VALUE THEOREM The roots of the given function as well as equality or inequality of any two or more than two functions can be determined using Mean Value Theorems. These theorems are Rolle’s Theorem, Lagrange’s Mean Value Theorem, and Cauchy’s Mean Value Theorem. For better understanding of these theorems, we shall first learn two type of functions.

2.4.1 Continuous and Differentiable Functions A function f (x) is said to be continuous at x = a if (i) f (a) is finite, i.e., f (a) exists. (ii) lim f ( x) = lim f ( x) = lim f ( x) = f (a) x→a

x→a +

x→ a −

A function f (x) is said to be differentiable at x = a, if Right Hand Derivative (RHD) and Left Hand Derivative (LHD) exists and RHD = LHD. i.e. or

lim

x→a +

lim h →0

f ( x) − f (a) f ( x) − f (a) = lim x → a − x−a x−a f ( a + h) − f ( a ) f ( a − h) − f ( a ) = lim h→ 0 h h

Note: (i) If function f (x) is finitely differentiable (derivative is finite) in the interval (a, b), then it is continuous in the interval [a, b], i.e., every differentiable function is continuous in the given interval. But converse is not necessarily true, i.e., a function may be continuous for a value of x without being differentiable for that value. The interval (a, b) is also called as domain of the function. (ii) Algebraic, trigonometric, inverse trigonometric, logarithmic and exponential function are ordinarily continuous and differentiable (with some exceptions). (iii) Addition, subtraction, product and quotient of two or more continuous and differentiable functions are also continuous and differentiable. (iv) f (x) and f (x) are differentiable and continuous if f (x) exists. (v) A function is said to be differentiable if its derivative is neither indeterminate nor infinite.

2.5 ROLLE’S THEOREM Statement: If a function f (x) is (i) continuous in the closed interval [a, b] (ii) differentiable in the open interval (a, b) (iii) f (a) = f (b) then there exists at least one point c in the open interval (a, b) such that f ( c) = 0.

Differential Calculus I

2.43

Proof: Since function f (x) is continuous in the closed interval [a, b], it attains its maxima and minima at some points in the interval. Let M and m be the maximum and minimum values of f (x) respectively at some points c and d respectively in the interval [a, b]. f (c) = M and f (d) = m Now two cases arise: Case I: If M = m f (x) = M = m for all x in [a, b] f (x) = constant for all x in [a, b] f (x) = 0 for all x in [a, b] Hence, the theorem is true. Case II: If M ó m Since f (a) = f (b), either M or m must be different from f (a) and f (b). Let M is different from f (a) and f (b). f (c) is different from f (a) and f (b). Also, Hence,

f (c ) ≠ f ( a ) , f (c) ≠ f (b)

∴c ≠ a ∴c ≠ b

a 0, b > 0 ⎣ ( a + b) x ⎦

(vii) f (x) = x2 + 1 =3-x (viii) f (x) = x2 - 2 =x-

0

x x

-1

1 x

0

x

Solution: (i) f (x) = (x

a)m (x

b)n in [a, b], where m, n are positive integers.

(a) Since m and n are positive integers, f (x) = (x a)m (x b)n, being a polynomial, is continuous in [a, b]. (b) f (x) = m (x a)m 1 (x b)n + n (x a)m (x b)n 1 = (x a)m 1 (x b)n 1 [m (x b) + n (x a)] = (x a)m 1 (x b)n 1 [(m + n) x (mb + na)] exists for every value of x in (a, b). Therefore, f (x) is differentiable in (a, b). (c) f (a) = f (b) = 0 Thus, f (x) satisfies all the conditions of Rolle’s theorem. Therefore, there exists at least one point c in (a, b) such that f ( c) = 0. (c a)m 1 (c b)n 1 [(m + n) c (mb + na)] = 0 mb + na m+n which represents a point that divides the interval [a, b] internally in the ratio of m : n. Thus, c lies in (a, b). Hence, theorem is verified. c=

(ii) f ( x) = x ( x + 3) e



x 2

in − 3 ≤ x ≤ 0 −

x

(a) f ( x) = x ( x + 3) e 2 , being composite function of continuous function, is continuous in [ 3, 0]. (b) f ′ ( x) = ( x + 3) e



x 2

+ xe



x 2



x ( x + 3) − 2x e 2

2.46

Engineering Mathematics

exists for every value of x in (-3, 0). Therefore, f (x) is differentiable in (-3, 0). (c) f (-3) = f (0) = 0 Thus, f (x) satisfies all the conditions of Rolle’s theorem. Therefore, there exists at least one point c in (-3, 0) such that f ( c) = 0 (c + 3) e



c 2

+ ce



c 2



c (c + 3) − 2c e =0 2

2 (c + 3) + 2c − c (c + 3) = 0

⎡ − 2c ⎤ ⎢∵ e ≠ 0 for anny finite ⎥ ⎢ value of c ⎥⎦ ⎣

- c2 + c + 6 = 0, c = –2, 3 c = –2 lies in (–3, 0) Hence, theorem is verified. (iii) f (x) = | x | in [-1, 1]. | x | = -x, -1 x 0 = x, 0 x 1 (a) f (x) is continuous in [-1, 1]. (b) Left hand derivative at x = 0 f ′(0 − ) = lim− x→0

f ( x) − f (0) −x − 0 = lim− = −1 x→0 x−0 x

Right hand derivative at x = 0 f ′(0+ ) = lim+ x →0

f ( 0−)

f ( x) − f (0) x−0 = lim+ =1 x → 0 x−0 x

f ( 0+)

Thus, function is not differentiable at x = 0 and hence, Rolle’s theorem is not applicable. sin x = e − x sin x ex (a) f (x) = e−x sin x, being product of continuous functions, is continuous in [0, p ] (b) f (x) = -e−x sin x + e−x cos x = e−x (cos x - sin x) exists for every value of x in (0, p ). Therefore, f (x) is differentiable in (0, p ). (c) f (0) = f (p ) = 0 Thus, f (x) satisfies all the conditions of Rolle’s theorem. Therefore, there exists at least one c in (0, p ) such that f ( c) = 0.

(iv) f ( x) =

Differential Calculus I

2.47

f ′(c) = e − c (cos c − sin c) = 0 cos c − sin c = 0 cos c = sin c tan c = 1, c = np +

[∵ e − c ≠ 0 for any finite value off c] p , where n is an integer. 4

n = 0, 1, 2, … p 5p c= , , … 4 4 p c = lies in the interval (0, p ). 4 Hence, theorem is verified. ⎡ p 5p ⎤ x (v) f ( x ) = e (sin x − cos x ) in ⎢ , ⎥ ⎣4 4 ⎦ Putting

(a) f ( x) = e x (sin x − cos x) , being composite function of continuous func⎡ p 5p ⎤ tions, is continuous in ⎢ , ⎥. ⎣4 4 ⎦ (b) f (x) = ex (sin x cos x) + ex (cos x + sin x) = 2ex sin x ⎛ p 5p ⎞ exists for every value of x in ⎜ , . Therefore, f ( x ) is differentiable ⎝ 4 4 ⎟⎠

⎛ p 5p ⎞ in ⎜ , ⎝ 4 4 ⎟⎠ (c)

⎛p ⎞ ⎛ 5p ⎞ f ⎜ ⎟ = f ⎜ ⎟=0 ⎝4⎠ ⎝ 4 ⎠ Thus, f (x) satisfies all the conditions of Rolle’s theorem. Therefore, there ⎛ p 5p ⎞ exists at least one point c in ⎜ , such that f ( c) = 0 ⎝ 4 4 ⎟⎠ 2ec sin c = 0 sin c = 0 [∵ ec 0 for any finite value of x] c = 0, p, 2p, …………… ⎛ p 5p ⎞ . c = p lies in ⎜ , ⎝ 4 4 ⎟⎠

Hence, theorem is verified. ⎡ x 2 + ab ⎤ (vi) f ( x) = log ⎢ ⎥ in [a, b], a > 0, b > 0 ⎣ ( a + b) x ⎦

2.48

Engineering Mathematics (a) f (x) = log (x2 + ab) log x log (a + b), being composite function of continuous functions, is continuous in [a, b]. 2x 1 − x + ab x exists for every value of x in (a, b) [∵ a > 0, b > 0]. Therefore, f (x) is differentiable in (a, b). (c) f (a) = log (a2 + ab) log a log (a + b) = log a + log (a + b) log a log (a + b) =0 f (b) = log (b2 + ab) log b log (a + b) = log b + log (b + a) log b log (a + b) =0 f (a) = f (b) Thus, f (x) satisfies all the conditions of Rolle’s theorem. Therefore, there exists at least one point c in (a, b) such that f ( c) = 0

(b) f ′( x) =

2

2c 1 − =0 c + ab c 2c 2 − c 2 − ab = 0 2

c 2 − ab = 0, c = ± ab Since c = ab lies between a and b [being geometric mean of a and b]. Hence, theorem is verified. (vii) f (x) = x2 + 1 0 x 1 =3-x 1 x 2 0 x 1 (a) f (x) = x2 + 1 =3-x 1 x 2 is defined every where in [0, 2] and hence, continuous in [0, 2]. (b) Left hand derivative at x = 1 f ( x) − f (1) x2 + 1 − 2 = lim− x →1 x →1 x −1 x −1 2 x −1 = lim− = lim( x + 1) = 1 + 1 = 2 x →1 x − 1 x →1−

f ′(1− ) = lim−

Right hand derivative at x = 1

f ( x) − f (1) x −1 f ( x) − f (1) 3− x − 2 = −1 = lim+ = lim+ x →1 x →1 x −1 x −1 f ( 1-) f ( 1+)

f ′(1+ ) = lim+ x →1

Differential Calculus I

2.49

Thus, function is not differentiable at x = 1 and hence, Rolle’s theorem is not applicable. (viii) f (x) = x2 - 2 -1 x 0 =x-2 0 x 1 -1 x 0 (a) f (x) = x2 - 2 =x-2 0 x 1 is defined everywhere in [-1, 1], and hence, is continuous in [-1, 1]. (b) Left hand derivative at x = 0 f ( x ) − f ( 0) x 2 − 2 − ( −2) = lim− x→0 x→0 x−0 x 2 2 x −2+2 x = lim− = lim− =0 x→0 x→0 x x

f ′(0 − ) = lim−

Right hand derivative at x = 0 f ( x) − f (0) ( x − 2) − ( −2) = lim+ x→0 x−0 x x = lim+ = 1 x→0 x f ′ (0 − ) ≠ f ′ (0 + )

f ′(0+ ) = lim+ x→0

Thus, function is not differentiable at x = 0 and hence, Rolle’s theorem is not applicable. Example 2: Prove that between any two roots of ex sin x = 1 there exists at least one root of ex cos x + 1 = 0. Solution: Let f (x) = 1

ex sin x

(a) f (x), being composite function of continuous functions, is continuous in a finite interval. (b) f (x) = (ex sin x + ex cos x ) = ( 1 + ex cos x ) [∵ex sin x =1] exists for every finite value of x. Therefore, f (x) is differentiable in a finite interval. (c) Let a and b are two roots of the equation, f (x) = 1 ex sin x = 0 Then f (a ) = f (b ) = 0 Thus, f (x) satisfies all the conditions of Rolle’s theorem in [a, b ]. Therefore, there exists at least one point c in (a, b ) such that f (c) = 0 1 + ec cos c = 0 This shows that c is the root of the equation ex cos x + 1= 0 which lies between the root a and b of the equation 1 ex sin x = 0. Example 3: Prove that the equation 2x3 – 3x2 – x + 1 = 0 has at least one root between 1 and 2.

2.50

Engineering Mathematics

x4 x2 − x 3 − + x [obtained by integratSolution: Let us consider a function f ( x) = 2 2 ing the given equation] (a) f (x), being an algebraic function, is continuous in [1, 2] (b) f (x) = 2x3 3x2 x + 1 exists for every value of x in (1, 2). Therefore, f (x) is differentiable in (1, 2). (c) f (1) = f (2) = 0 Thus, f (x) satisfies all the conditions of Rolle’s theorem. Therefore, there exists at least one point c in (1, 2) such that f (c) = 0 2c3 3c2 c + 1 = 0 This shows that c is the root of the equation 2x3 3x2 x + 1 = 0 which lies between 1 and 2. Example 4: Prove that if a0 , a1 , a2 , ....... an are real numbers such that a0 a a a + 1+ 2 + + n -1 + an = 0, then there exists at least one real numn+1 n n 1 2 n n -1 + a2 x n - 2 + ....... + an = 0. ber x between 0 and 1 such that a0 x + a1 x

Solution: Let us consider a function f ( x) =

a0 x n +1 a1 x n a2 x n −1 + + + ....... + an x n +1 n n −1

defined in [0, 1]. (a) f (x), being an algebraic function, is continuous in [0, 1]. (b) f (x) = a0 xn + a1 xn−1 + a2 xn−2 + ……. + an exists for every value of x in (0, 1). Therefore, f (x) is differentiable in (0, 1). (c) f (0) = 0 f (1) =

a0 a a a + 1 + 2 + ........ + n −1 + an = 0 n +1 n n −1 2

[given]

f (0) = f (1) Thus, f (x) satisfies all the conditions of Rolle’s theorem. Therefore, there exists at least one point c in (0, 1) such that f (c) = 0 a0 cn + a1 cn−1 + a2 cn−2 + ……….. + an = 0 Replacing c by x, a0 xn + a1 xn−1 + a2 xn−2 + ………. + an = 0 Example 5: If f (x), e (x), Y (x) are differentiable in (a, b), prove that there exists f (a )

e (a )

Y (a )

at least one point c in (a, b) such that f (b )

e (b)

Y (b) = 0 .

f (c )

e (c )

Y (c )

Differential Calculus I

2.51

f ( a)

f ( a)

y ( a)

Solution: Let us consider a function F ( x ) = f (b) f ( x)

f ( b)

y ( b)

f ( x)

y ( x)

(a) Since f (x), f (x), Y (x) are differentiable in (a, b), therefore, will be continuous in [a, b]. F (x), being composite function of continuous functions, is continuous in [a, b]. f ( a)

f ( a)

y ( a)

(b) F ′( x ) = f (b)

f ( b)

y (b) exists for every value of x in (a, b). There-

f ′( x )

y ′( x )

f ′( x )

fore, f (x) is differentiable in (a, b). (c) f (a) = f (b) = 0 Thus, f (x) satisfies all the conditions of Rolle’s theorem. Therefore, there exists at least one point c in (a, b) such that f (c) = 0 f ( a)

f ( a)

y ( a)

f ( b)

f ( b)

y ( b ) = 0.

f ′( c )

f ′( c )

y ′(c )

Example 6: If f (x) = x (x + 1) (x + 2) (x + 3), prove that f (x) = 0 has three real roots. Solution: f (x) = x (x + 1) (x + 2) (x + 3) (a) f (x), being polynomial is continuous, in the intervals [-3, -2], [-2, -1], [-1, 0]. (b) f (x) = (x + 1) (x + 2) (x + 3) + x (x + 2) (x + 3) + x (x + 1) (x + 3) + x (x + 1) (x + 2) exists for every value of x in [-3, -2], [-2, -1] and [-1, 0]. Therefore, f (x) is differentiable in [-3, -2], [-2, -1] and [-1, 0]. (c) f (-3) = f (-2) = f (-1) = f (0) = 0 Thus, f (x) satisfies all the conditions of Rolle’s theorem. Therefore, there exists at least one c1 in (-3, -2), c2 in (-2, -1) and c3 in (-1, 0) such that f (c1) = f (c2) = f (c3) = 0 Thus c1, c2 and c3 are the roots of f (x) = 0 Hence f (x) = 0 has at least 3 real roots. Example 7: If k is a real constant, prove that the equation x3 – 6x2 + c = 0 cannot have distinct roots in [0, 4]. Solution: Let f (x) = x3 Then

6x2 + c = 0 has distinct roots a and b between 0 and 4 i.e. 0 a f (a), i.e., f (x) is strictly (monotonically) increasing function in the closed interval [a, b]. Proof: By Lagrange’s Mean Value theorem f (b) − f (a ) = f ′ (c ) b−a Let f (x) > 0 for all x in (a, b). Then,

f ′(c) > 0, a 0 [∵ b a > 0 being length of the interval] f (b) > f (a), b>a f (x) is strictly (monotonically) increasing function in the closed interval [a, b]. In general, f (x2) > f (x1) for x2 > x1, for every value of x1, x2 in [a, b].

Decreasing Function Statement: If a function f (x) is (i) continuous in the closed interval [a, b], (ii) differentiable in the open interval (a, b), (iii) f (x) < 0 throughout the interval (a, b), then f (b) < f (a), i.e., f (x) is strictly (monotonically) decreasing function in the closed interval [a, b].

2.56

Engineering Mathematics

Proof: By Lagrange’s Mean Value theorem f (b) − f (a ) = f ′ (c ) b−a Let f (x) < 0 for all x in (a, b). f ′ (c ) < 0 a x1, for every value of x1, x2 in [a, b]. Then,

Example 1: Verify Lagrange’s Mean Value Theorem for the following functions: (i) f (x) = x3 in [-2, 2] (ii) f (x) = lx2 + mx + n in [a, b] 2

(iii) f (x) = x 3 in [-8, 8] (iv) f (x) = ex in [0, 1] (v) f (x) = log x in [1, e]. Solution: (i) f (x) = x3 in [-2, 2] (a) f (x) = x3, being an algebraic function, is continuous in [-2, 2]. (b) f (x) = 3x2 exists for every value of x in (-2, 2). Therefore, f (x) is differentiable in (-2, 2). Thus, f (x) satisfies all the conditions of Lagrange’s Mean Value theorem. Therefore, there exists at least one point c in (-2, 2) such that f (2) − f ( −2) = f ′ (c ) 2 − ( −2) (2)3 − ( −2)3 = 3c 2 2 − ( −2) 4 = 3c 2 2 c=± 3 2 c=± liies in ( −2, 2). 3 Hence, theorem is verified.

Differential Calculus I

2.57

(ii) f (x) = lx2 + mx + n in [a, b]. (a) f (x) = lx2 + mx + n, being an algebraic function, is continuous in [a, b]. (b) f (x) = 2lx + m, exists for every value of x in (a, b). Therefore, f (x) is differentiable in (a, b). Thus, f (x) satisfies all the conditions of Lagrange’s Mean Value theorem. Therefore, there exists at least one point c in (a, b) such that f (b) − f (a) = f ′ (c ) b−a (lb 2 + mb + n) − (la 2 + ma + n) = 2lc + m b−a l (b + a ) + m = 2llc + m b+a lies in (a, b) being arithmetic mean of a and b. 2 Hence, theorem is verified. c=

2

(iii) f (x) = x 3 in [-8, 8]. 2

(a) f (x) = x 3 , being an algebraic function, is continuous in [-8, 8]. 2 −1 2 (b) f ′( x) = x 3 = 1 which does not exists at x = 0. 3 3 x3 Hence, theorem is not applicable. (iv) f (x) = ex in [0, 1]. (a) f (x) = ex, being an exponential function, is continuous in [0, 1]. (b) f (x) = ex, exists for every value of x in (0, 1). Therefore, f (x) is differentiable in (0, 1). Thus, f (x) satisfies all the conditions of Lagrange’s Mean Value theorem. Therefore, there exists at least one point c in (0, 1) such that f (1) − f (0) = f ′ (c ) 1− 0 e1 − e0 = ec 1− 0 ec = e - 1 c = log (e - 1) = 0.5413 < 1 c = 0.5413 lies in (0, 1). Hence, theorem is verified. (v) f (x) = log x in [1, e]. (a) f (x) = log x, being a logarithmic function, is continuous in [1, e]. 1 (b) f (x) = , exists for every value of x in [1, e]. Therefore, f (x) is differenx tiable in [1, e].

2.58

Engineering Mathematics

Thus, f (x) satisfies all the conditions of Lagrange’s Mean Value theorem. Therefore, there exists at least one point c in [1, e] such that



f (e) − f (1) = f ′ (c ) e −1 log e − log 1 1 = e −1 c c=e 1 2 0 and (x + 1) > 0 i.e., x > 1 Case II: (x 1) < 0 and (x + 1) < 0 i.e., x < 1 Hence, f (x) is increasing in (- , 1) and (1, ). –¥

x –1

0

1

(ii) f (x) is a decreasing function if f ′( x) < 0,

¥

( x − 1) ( x + 1) < 0, i.e. ( x − 1) ( x + 1) < 0 x2

Now, (x 1) (x + 1) < 0 if Case I: (x 1) < 0 and (x + 1) > 0 i.e., 1 < x < 1 Case II: (x 1) > 0 and (x + 1) < 0 i.e., x > 1 and x < possible. Hence, f (x) is decreasing in ( 1, 1 ).

1 but this is not

a⎞ b ⎛b ⎞ ⎛ Example 10: If 0 < a < b, prove that ⎜ 1 − ⎟ < log < ⎜ − 1⎟ . Hence, prove ⎝ b⎠ a ⎝a ⎠ that

1 1 1 < log (1.2) < and < log 2 < 1. 6 5 2

Differential Calculus I

2.63

Solution: Let f (x) = log x is defined in [a, b] where 0 < a < b. (a) f (x) = log x, being logarithmic function, is continuous in [a, b]. 1 exists for every value of x in (a, b). Therefore, f (x) is differentiable x in (a, b).

(b) f ′( x) =

Thus, f (x) satisfies all the conditions of Lagrange’s Mean Value theorem. Therefore, there exists at least one point c in (a, b), such that

We have,

f (b) − f (a ) = f ′ (c ) b−a log b − log a 1 = b−a c b b−a log = a c a a c b b−a b−a b−a > > a c b b b a − 1 > log > 1 − a a b a b b 1 − < log < − 1 b a a

... (1)

[∵ b > a] [ Using Eq. (1)] … ( 2)

(i) Putting b = 6, a = 5 in Eq. (2), 1−

5 6 6 < log < − 1 6 5 5 1 1 < log 1.2 < 6 5

(ii) Putting b = 2, a = 1 in Eq. (2), 1−

1 2 < log 2 < − 1 2 1

1 < log 2 < 1. 2

Example 11: Prove that if 0 < a < 1, 0 < b < 1 and a < b, then b−a 1− a

(i)

2

< sin −1 b - sin −1 a
1 − c2 > 1 − b2 1 1− a b−a

2

1 − a2 b−a 1− a

(i) Putting b =

... (1)

2

<
0, b > 0]

[∵ b

a > 0]

... (2) [Using Eq. (1)]

2.66

Putting b =

Engineering Mathematics

4 , a = 1 in Eq. (2), 3 4 4 −1 −1 4 3 < tan −1 − tan −1 1 < 3 16 3 1+1 1+ 9 3 4 p 1 < tan −1 − < 25 3 4 6 p 3 4 p 1 + < tan −1 < + . 4 25 3 4 6

−1 Example 13: Prove that tan x >

Solution: Let f ( x) = tan −1 x −

o x 0 < tan −1 x < . 2 if 2 x 1+ 3

x x2 1+ 3

0 < tan −1 x
0, for every value of x in (0,

Differential Calculus I

2.67

Hence, f (x) is strictly increasing function in (0, ). f (x) > f (0)

for

x>0 [∵ f (0) = 0]

f (x) > 0

for x > 0 x tan −1 x − >0 x2 1+ 3 x tan −1 x > . x2 1+ 3

Example 14: Prove that x2 - 1 > 2x log x > 4 (x - 1) - 2 log x, for all x > 1. Solution: (i) Let f (x) = x2 1 2x log x f (x) = 2x 2 log x 2 It is difficult to decide about the sign of f (x) therefore differentiating again w.r.t. x,

2 2( x − 1) = >0 x x Hence, f (x) is strictly increasing function for x > 1. f (x) > f (1) for x>1 f (x) > 0 for x>1 [∵ f (1) = 2 f ′′( x) = 2 −

[∵ x > 1]

2 log 1

2 = 0]

Hence, f (x) is strictly increasing function for x > 1. f (x) > f (1) for x>1 f (x) > 0 for x>1 [ ∵ f (1) = 1 1 2 log 1 = 0] x2 1 2x log x > 0 for x > 1 x2 1 > 2x log x for x > 1 … (1) (ii) Let f (x) = 2x log x

4 (x

1) + 2 log x f ′( x) = 2 log x + 2 − 4 +

2 x

f (x). Therefore, differentiating again w.r.t. x, 2 2 − x x2 2( x − 1) = >0 x2 Hence, f (x) is strictly increasing function for x > 1. f ′′( x) =

f (x) > f (1) f (x) > 0

for for

x>1 x>1

[∵ x > 1]

[∵ f ′ (1) = 2 log 1 + 2 − 4 +

2 = 0] 1

Hence, f (x) is an increasing function for x > 1. f (x) > f (1) f (x) > 0

for for

x>1 x>1

[∵ f (1) = 2 log 1

4 (1

1) + 2 log 1 = 0 ]

Engineering Mathematics

2.68

2x log x 4 (x 2x log x > 4 (x

1) + 2 log x > 0

for

x>1

1)

for

x>1

2 log x

… (2)

From Eqs (1) and (2), we get x2

1 > 2x log x > 4 (x

Example 15: Prove that 2 x < log Solution: (i) Let f (x) = 2 x − log = 2x

1)

2 log x

for x > 1.

⎡ 1 ⎛ x2 ⎞ ⎤ 1+ x < 2 x ⎢1 + ⎜ in (0, 1). 2 ⎟⎥ 1- x ⎣ 3 ⎝ 1 - x ⎠⎦

1+ x 1− x

log (1 + x) + log (1

x)

1 1 2 − 2x − 1 + x − 1 − x − = 1+ x 1− x (1 − x 2 ) 2

f ′ ( x) = 2 − =

−2 x 2 0]

Hence, f (x) is a decreasing function in (0, 1). f (x) < f (0) for x>0 f (x) < 0

x>0

2 x − log

1+ x 0

[∵ f (0) = 0]

log

⎡ 1 ⎛ x2 ⎞⎤ 1+ x − 2 x ⎢1 + ⎜ 0 for ⎤ ⎢ ⎥ all values of x except x = 1⎦ ⎣ 4. Prove that x sin x is strictly increasing in every interval. 5. Separate the intervals in which the polynomial x3 6x2 36x + 7 is increasing or decreasing: ⎡ Ans.: Increasing in (6, ∞), ⎤ ⎢ ⎥ (− ∞, − 2) and ⎢ ⎥ decreasing in (−2, 6) ⎦⎥ ⎣⎢ 6. Separate the intervals in which the following polynomials are increasing or decreasing: (i) x3 3x2 + 24x 31 (ii) 2x3 15x2 36x + 40 (iii) 2x3 9x2 + 12x + 5 ⎡ Ans.: (i) Increasing in ( − ∞, 4), ⎤ ⎢ (2, ∞) and decreasing ⎥⎥ ⎢ ⎢ ⎥ in ( − 2, 4). ⎢ ⎥ (ii) Increasing in ( − ∞, − 1), ⎥ ⎢ ⎢ (6, ∞) and decreasing ⎥ ⎢ ⎥ in ( − 1, 6). ⎢ ⎥ ⎢ (iii) Increasing in ( − ∞, 1), ⎥ ⎢ ⎥ (2, ∞) and decreasing ⎥ ⎢ ⎢ ⎥ in (1, 2). ⎣ ⎦ 7. Find the value of q in Lagrange’s Mean Value theorem for the following: (i) ax2 + bx + c at x = 0 (ii) f (x) = x3, 1 < x < 2 ⎡ Ans. (i) interval is (0, h), ⎤ ⎢ ⎥ 1 7⎥ ⎢ q = (ii) − 1 + ⎢⎣ 2 3 ⎥⎦

8. Prove that the following functions are increasing in the given interval: (i) x3 3x2 + 3x + 1, ( , ) (ii) log x, (a, ), where a > 0

(iii) e x, ( − ∞, ∞) ⎛ p p⎞ (iv) sin x, ⎜ − , ⎟ ⎝ 2 2⎠ (v) cos x, ( , 2 )

⎛ p p⎞ (vi) tan x, ⎜ − , ⎟ ⎝ 2 2⎠

[Hint : Prove that f given interval]

(x) > 0 in the

9. Prove that the following functions are decreasing in the given interval: 2

(i) e − x , (0, ∞) (ii) cos x, (0, p ) ⎛ p⎞ (iii) cosec x, ⎜ 0, ⎟ ⎝ 2⎠

⎛ (iv) sin x, ⎜ , ⎝2

⎞ ⎟ ⎠

⎛ p ⎞ (v) sec x, ⎜ − , 0 ⎟ ⎝ 2 ⎠ (vi) cot x, (0, p ) [Hint : Prove that f (x) < 0 in the given interval] 10. Prove the following: (i) log (1 + x) < x for all x > 1 1 < log x < x − 1 x for all x > 1

(ii) 1 −

(iii) log x < x < tan x

for all x > 1 x (iv) e > 1 + x for all x > 0

(v) 0 < − log (1 − x)
0, ⎥ z ⎢ ⎥ ⎢ f ( x + y ) − f ( x) ⎥ ⎢ ⎥ = f ′ (c), ⎢ ( x + y) − x ⎥ ⎢ log ( x + y ) − log x 1 1 ⎥ ⎢ ⎥ = < y c x ⎢ ⎥ ⎢ ⎥ (∵ c > x) ⎢ ⎥ ⎢ ⎥ y ⎢log ( x + y ) − log x < , ⎥ x ⎢ ⎥ y ⎢ ⎥ log ( ) l og x x + y < + ⎢⎣ ⎥⎦ x 16. If a, b are real numbers, prove that there exists at least one real number

⎛p ⎞ (0, 1) and ⎜ , 1⎟ such that the ⎝2 ⎠

c such that b2 + ab + a2 = 3c2, a < c 0, b > 0 ⎡ o⎤ (ii) sin x and cos x in ⎢ 0, ⎥ . ⎣ 2⎦ Solution: (i) Let f (x) = x2, g (x) = x4

(a) f (x) and g (x), both being algebraic functions, are continuous in the closed interval [a, b]. (b) f (x) = 2x and g (x) = 4x3 exists for all values of x in the open interval (a, b). Therefore, f (x) and g (x) are differentiable in (a, b), and g (x) = 4x3 0 for any x in (a, b) since a > 0, b > 0. Thus, f (x) and g (x) satisfies all the conditions of Cauchy’s Mean Value theorem. Therefore, there exists at least one point c in (a, b) such that f (b) − f (a ) f ′ (c) = g (b) − g (a ) g ′ (c) b2 − a 2 2c 1 = 3 = 2 4 4 b −a 4c 2c (b 2 − a 2 ) 1 = (b 2 + a 2 ) (b 2 − a 2 ) 2c 2 2c2 = b2 + a2

Differential Calculus I

b2 + a 2 2

c=± c=

2.73

b2 + a 2 2

which lies between a and b. (ii) Let f (x) = sin x, g (x) = cos x (a) f (x) and g (x), both being trigonometric functions, are continuous in ⎡ p⎤. ⎢0, 2 ⎥ ⎦ ⎣ ⎛ p⎞ (b) f (x) = cos x, g (x) = sin x exists for all values of x in ⎜ 0, ⎟ and ⎝ 2⎠ g (x) =

sin x

⎛ p⎞ 0 for any x in ⎜ 0, ⎟ . ⎝ 2⎠

Thus, f (x) and g (x) satisfies all the conditions of Cauchy’s Mean Value theorem. ⎛ p⎞ Therefore, there exists at least one point c in ⎜ 0, ⎟ such that ⎝ 2⎠ ⎛p ⎞ f ⎜ ⎟ − f ( 0) ⎝ 2⎠

=

f ′ (c )

g ′ (c ) ⎛p ⎞ g ⎜ ⎟ − g ( 0) ⎝ 2⎠ p sin − sin 0 cos c 2 = p cos − cos 0 − sin c 2 1− 0 = − cot c 0 −1 −1 = − cot c cot c = 1, c =

p 4

p . 2 Hence, theorem is verified. which lies between 0 and

1 , prove that c of Cauchy’s Mean Value x x theorem is the harmonic mean between a and b, a > 0, b > 0.

Example 2: If f ( x ) =

1

2

, and g ( x ) =

Solution: (a) f (x) and g(x) are continuous in the closed interval [a, b] for a > 0, b > 0. 2 1 (b) f ′ ( x) = − 3 and g ′ ( x) = − 2 exists for all x in (a, b) and g (x) 0 for any x x in (a, b). x

Engineering Mathematics

2.74

Thus, f (x) and g (x) satisfies all the conditions of Cauchy’s Mean Value theorem. Therefore, there exists at least one point c in (a, b) such that f (b) − f (a ) f ′(c) = g (b) − g (a ) g ′(c) 1 1 2 − − b 2 a 2 = c3 1 1 1 − − 2 b a c (a 2 − b 2 )(ab) 2 = (a 2 b 2 )(a − b) c a+b 2 = ab c 2 1 1 = + c b a 1 1⎛1 1⎞ = ⎜ + ⎟ c 2⎝a b⎠ Hence, c is the harmonic mean between a and b. Example 3: If f ( x ) =

1

, prove that c of Cauchy’s Mean x Value theorem is geometric mean between a and b, a > 0, b > 0. Solution: (a) f ( x) = (b) f ′ ( x) =

1 2 x

x and g ( x ) =

1

x and g ( x) = 1

, g ′ ( x) = −

2( x)

3 2

are continuous in [a, b] for a > 0, b > 0.

x

exists for all x in (a, b) and g (x)

0 for any x

in (a, b). Thus, f (x) and g (x) satisfies all the conditions of Cauchy’s Mean Value theorem. Therefore, there exists at least one point c in (a, b) such that f (b) − f (a ) f '(c) = g (b) − g (a ) g '(c) 1 b− a = 2 c 1 1 1 − − 3 b a 2(c) 2

(

a− b

(

)

a− b

ab

)

=c

c = ab Hence, c is the geometric mean between a and b.

Differential Calculus I

2.75

Example 4: If f (x) = ex and g (x) = e−x, prove that c of Cauchy’s Mean Value theorem is arithmetic mean between a and b, a > 0, b > 0. Solution: (a) f (x) and g (x), being exponential functions, are continuous in [a, b]. (b) f (x) = ex, g (x) = − e−x exists for all x in (a, b) and g (x)

0 for any x in (a, b).

Thus, f (x) and g (x) satisfies all the conditions of Cauchy’s Mean Value theorem. Therefore, there exists at least one point c in (a, b) such that f (b) − f (a ) f ′ (c) = g (b) − g (a ) g ′ (c) eb − e a ec = −c −b −a e −e −e b a e −e = −e2c 1 1 − eb e a − (e a − e b ) e b e a = −e2c (e a − e b ) ea + b = e2c a + b = 2c a+b c= 2 Hence, c is the arithmetic mean between a and b.

[By comparing ]

Example 5: If 1 < a < b, prove that there exists c satisfying a < c < b such that b b2 - a 2 log = . a 2c 2 Solution: Let f (x) = log x, g(x) = x2 are defined in (a, b). (a) f (x), being logarithmic function and g(x), being algebraic function, are continuous in [a, b] for a > 1, b > 1. 1 (b) f ′ ( x) = , g ′ ( x) = 2 x exists for all x in (a, b) and g (x) 0 for any x in (a, b) x since a > 1, b > 1. Thus, f (x) and g (x) satisfies all the conditions of Cauchy’s Mean Value theorem. Therefore, there exists at least one point c in (a, b) such that

f (b) − f (a ) f ′ (c) = g (b) − g (a ) g ′ (c) 1 log b − log a c = 2c b2 − a 2 Hence,

log

b b2 − a 2 = . a 2c 2

Engineering Mathematics

2.76

Example 6: Using appropriate mean value theorem, prove that sin b − sin a e −e b

a

=

cos c ec

for a < c < b. Hence, deduce that ec sin x = (ex - 1) cos c.

Solution: Let f (x) = sin x, g (x) = ex are defined in (a, b). (a) f (x), being trigonometric function and g (x), being exponential function, are continuous in [a, b]. (b) f (x) = cos x, g (x) = ex exists for all x in (a, b) and g (x) 0 for any x in (a, b). Thus, f (x) and g(x) satisfies all the conditions of Cauchy’s Mean Value theorem. Therefore, there exists at least one point c in (a, b) such that f (b) − f (a ) f ′ (c) = g (b) − g (a ) g ′ (c) sin b − sin a cos c = c eb − e a e

... (1)

Putting b = x, a = 0 in Eq. (1), sin x − sin 0 cos c = c e x − e0 e ec sin x = (ex 1) cos c. , Example 7: Using Cauchy s Mean Value theorem, prove that there exists a num1

⎛b⎞ ber c such that 0 < a < c < b and f (b) - f (a) = c f (c) log ⎜ ⎟ . By putting f ( x ) = x n , ⎝a⎠ 1

(

)

deduce that lim n b n − 1 = log b. n→ ∞

Solution: Let g (x) = log x is defined in [a, b]. (a) Let f (x) is continuous in [a, b] and differentiable in (a, b). Also g (x), being a 1 logarithmic function, is continuous in [a, b] for 0 < a < b. g ′( x) = exists for x all x in (a, b) since 0 < a < b and g (x) 0 for any x in (a, b). Thus, f (x) and g (x) satisfies all the conditions of Cauchy’s Mean Value theorem. Therefore, there exists at least one point c in (a, b) such that f (b) − f (a ) f ′ (c) = g (b) − g (a ) g ′ (c) f (b) − f (a ) f ′ (c) = 1 log b − log a c Hence,

⎛b⎞ f (b) − f (a ) = cf ′ (c) log ⎜ ⎟ ⎝a⎠

Differential Calculus I

1

Putting f ( x) = x n , f ′ ( x) =

2.77

1 1n −1 x in Eq. (1), n 1 1 1 1 −1 ⎛b⎞ (b) n − (a ) n = c ⋅ (c) n log ⎜ ⎟ n ⎝a⎠ 1 1 ⎛ 1 ⎞ n ⎛b⎞ n ⎜ b n − a n ⎟ = c log ⎜ ⎟ ⎝a⎠ ⎝ ⎠

1 1 ⎛ 1 ⎞ n ⎛b⎞ lim n ⎜ b n − a n ⎟ = lim c log ⎜ ⎟ n →∞ n →∞ ⎝a⎠ ⎝ ⎠

= c 0 log

b b = log a a

Putting a = 1, ⎛ 1 ⎞ lim n ⎜ b n − 1⎟ = log b. n →∞ ⎝ ⎠

Exercise 2.5 1. Verify Cauchy’s Mean Value theorem for the following functions : (i) f (x) = 3x + 2, g (x) = x2 + 1 in [1, 4] (ii) f (x) = x2 + 2, g (x) = x3 − 1 in [1, 2] (iii) f (x) = 2x3, g (x) = x6 in [a, b] (iv) f (x) = log x, g (x)

1 in [1, e] x

5 ⎡ ⎢ Ans. : (i) c = 2 ⎢ 1 ⎢ 3 3 3 ⎢(iii) c = ⎛⎜ a + b ⎞⎟ ⎝ 2 ⎠ ⎣⎢

14 ⎤ 9 ⎥ ⎥ ⎥ e ⎥ (iv) c = e − 1 ⎥⎦ (ii) c =

2. Using Cauchy’s Mean Value theo-

rem, find lim x →1

⎛ p x⎞ cos ⎜ ⎟ ⎝ 2⎠ log x

px ⎤ ⎡ ⎢ Hint : Consider f ( x ) = cos 2 , ⎥ ⎢ ⎥ ⎣ g ( x ) = log x in the interval ( x, 1) ⎦ p c⎤ ⎡ ⎢⎣ Ans. : − 2 ⎥⎦ 3. If f (x) is continuous in [a, b], f (x) exists in (a, b), prove that there exists a point c in (a, b) such that f (b) − f (a ) f ′(c) (i) = 2c b2 − a 2 f (b) − f (a ) f ′(c) (ii) = b3 − a 3 3c 2 [Hint : (i) g (x) = x2 (ii) g (x) = x3] 4. Using appropriate theorem, prove that

mean

value

sin b − sin a = cot c, a < c < b. cos a − cos b 5. If f (x) = sin x and g (x) = cos x in [a, b], prove that c of Cauchy’s Mean

Engineering Mathematics

2.78

Value theorem is the arithmetic mean of a and b. 6. If f (x) and g (x) are continuous in [a, b] and differentiable in (a, b), g (a) g (b) and g (x) 0 in (a, b), then there exists at least one c between a and b such that f ′ (c ) f (c ) − f ( a ) = , a > ⎢⎣ ⎥⎦ 1+ c 1+ x 1+1

8. If f (x), g (x), h (x) are three functions differentiable in the interval (a, b), prove that there exists a point c in (a, b) such that f ′(c) g ′(c) h′(c) f ( a ) g ( a ) h( a ) = 0 f (b) g (b) h(b) Hence, deduce Lagrange’s and Cauchy’s Mean Value theorem. ⎡ Hint : Consider F ( x ) ⎤ ⎢ ⎥ f ( x) g ( x) h ( x) ⎥ ⎢ ⎢ = f ( a) g ( a) h ( a) ⎥ ⎢ ⎥ f ( b) g ( b) h ( b) ⎥ ⎢ ⎢ Apply Rolle’s theorem. For ⎥ ⎢ ⎥ ⎢ deduction of Lagrange′s ⎥ ⎢ MVT g( x ) = x, h ( x ) = 1, Keep ⎥ ⎢ ⎥ ⎢ f ( x) as it is in result. For ⎥ ⎢ ⎥ ⎢ deduction of Cauchy′s MVT ⎥ ⎢ take h ( x ) = 1, keep f ( x) and ⎥ ⎢ ⎥ ⎣ g ( x) as it is in the result. ⎦

2.8 TAYLOR’S SERIES Statement: If f (x + h) be a given function of h which can be expanded into a convergent series of positive ascending integral powers of h, then f ( x + h) = f ( x) + h f ′( x) +

h2 h3 hn n f ′′( x) + f ′′′( x) + ........ + f ( x) + ....... 2! 3! n!

Proof: Let f (x + h) be a function of h which can be expanded into positive ascending integral powers of h, then ... (1) f (x + h) = a0 + a1 h + a2 h2 + a3 h3 + a4 h4 +…….…… Differentiating w.r.t. h successively, ... (2) f (x + h) = a1 + a2 · 2h + a3 · 3h2 + a4 · 4h3 + ……. ……… 2 f (x + h) = a2 · 2 + a3 · 6h + a4 · 12h +……. ……… ... (3) f (x + h) = a3 · 6 + a4 · 24h +……. ……… ... (4) and so on

Differential Calculus I

2.79

Putting h = 0 in Eq. (1), (2), (3) and (4), a0 = f (x) a1 = f (x) 1 f ′′ ( x) 2! 1 a3 = f ′′′ ( x) and so on 3!

a2 =

Substituting a0, a1, a2 and a3 in Eq. (1),

f ( x + h) = f ( x) + h f ′( x) +

h2 h3 hn n f ′′( x) + f ′′′( x) + ....... + f x + ....... 2! 3! n!

This is known as Taylor’s Series. Putting x = a and h = x a in above series, we get Taylor’s Series in powers of (x – a) as f ( x) = f (a ) + ( x − a ) f ′(a ) +

( x − a)2 ( x − a )3 f ′′(a ) + f ′′′(a ) + ... ...... 2! 3! ( x − a)n n + f (a ) + ...... n!

Example 1: Prove that f (mx) = f ( x ) + ( m − 1) x f ′( x ) + Solution: f (mx) = f (mx

x + x) = f [x + (m

( m − 1)2 2 x f ′′( x ) + ... . 2!

1) x]

By Taylor’s series, f ( x + h) = f ( x) + h f ′( x) +

Putting

h2 h3 f ′′( x) + f ′′′ ( x) + ................ 2! 3!

h = (m 1) x,

f [ x + (m − 1) x] = f (mx) = f ( x) + (m − 1) x f ′( x) +

(m − 1) 2 2 x f ′′ ( x) + ......... 2!

Example 2: Prove that

⎛ x2 ⎞ x x2 x3 = f ( x) – f⎜ f ′ ( x) + f ′ ′( x ) – f ′′′( x ) + ... . ⎟ 2 ⎜ 1+ x ⎟ 1+ x 2 !(1 + x ) 3 !(1 + x )3 ⎝ ⎠ x2 x = x− , 1+ x 1+ x

Solution: By Taylor’s series,

f ( x + h) = f ( x) + h f ′( x) +

h2 h3 f ′′ ( x) − f ′′′ ( x) + ... 2! 3!

Engineering Mathematics

2.80

x , 1+ x ⎛ x2 ⎞ ⎛ x ⎞ f ⎜x − f = ⎜ ⎟ ⎟ 1+ x⎠ ⎝ ⎝1+ x ⎠ x3 x x2 f ′′ ( x ) − f ′′′( x) + ... = f ( x) − f ′ ( x) + 1+ x 3! (1 + x)3 2 ! (1 + x) 2 h=−

Putting

Example 3: Expand f (x) = x5 - x4 + x3 - x2 + x - 1 in powers of (x - 1) and find f (0.99). Solution: f (x) = x5

x4 + x3

x2 + x

1

By Taylor’s series, f ( x) = f (a) + ( x − a) f ′ (a) +

( x − a)2 ( x − a )3 f ′′ (a ) + f ′′′ (a ) + ...... 2! 3!

Putting a = 1, f (x) = x5

x4 + x3

x2 + x

1

( x − 1) 2 ( x − 1)3 f ′′(1) + f ′′′ (1) 2! 3! ( x − 1)5 v ( x − 1) 4 iv f (1) + f (1) + ....... + 5! 4! f (1) = 1 − 1 + 1 − 1 + 1 − 1 = 0 = f (1) + ( x − 1) f ′ (1) +

... (1)

Differentiating f (x) w.r.t. x successively, f (x) = 5x4 4x3 + 3x2 2x + 1, f (x) = 20x3 12x2 + 6x 2, f (x) = 60x2 24x + 6, f i v (x ) = 120x 24, f v(x) = 120,

f (1) = 5 4 + 3 2 + 1 = 3 f (1) = 20 12 + 6 2 = 12 f (1) = 60 24 + 6 = 42 f iv(1) = 120 24 = 96 f v(1) = 120

Substituting in Eq. (1), ( x − 1) 2 ( x − 1)3 ( x − 1) 4 ( x − 1)5 (12) + (120) (42) + (96) + 5! 2! 3! 4! = 3( x − 1) + 6( x − 1) 2 + 7( x − 1)3 + 4( x − 1) 4 + ( x − 1)5

f ( x) = 0 + ( x − 1) 3 +

Putting x = 0.99, f (0.99) = 3 (0.99 1) + 6 (0.99 1)2 + 7 (0.99 1)3 + 4 (0.99 1)4 + (0.99 = 3 ( 0.01) + 6 ( 0.01)2 + 7 ( 0.01)3 + 4 ( 0.01)4 + ( 0.01)5 = 0.02939

1)5

Differential Calculus I

Example 4: Prove that

2.81

1 1 ( x + 2) ( x + 2) 2 ( x + 2) 3 = + + + + ... . 1– x 3 32 33 34

Solution: Let f ( x) = 1

1− x

By Taylor’s series, f ( x) = f (a ) + ( x − a ) f ′ (a ) + Putting a =

( x − a)2 ( x − a )3 f ′′ (a ) + f ′′′ (a ) + ... .......... 2! 3!

2,

1 ( x + 2) 2 ( x + 2) 3 = f ( −2) + ( x + 2) f ′ ( −2) + f ′′ ( −2) + f ′′′ ( −2) + .......... 1− x 2! 3! ... (1)

f ( x) = f ( −2) =

1 3

Differentiating f (x) w.r.t. x successively, 1 1 , f ′ ( −2) = 2 (1 − x) 2 3 2 2! f ′′ ( x) = , f ′′ ( −2) = 3 3 (1 − x) 3 2.3 3! , f ′′′ ( −2) = 4 and so on f ′′′ ( x) = 4 (1 − x) 3 f ′ ( x) =

Substituting in Eq. (1), f ( x) =

1 1 ( x + 2) ( x + 2 ) 2 ( x + 2 ) 3 = + + + + ............ 1− x 3 32 33 34

Example 5: Expand log (cos x) about

o . 3

Solution: Let f (x) = log (cos x) By Taylor’s series, f ( x) = f (a) + ( x − a) f ′ (a) + Putting

( x − a)2 ( x − a )3 f ′′ (a ) + f ′′′ (a ) + … … 2! 3!

p , 3 f ( x) = log (cos x) a=

2

3

p ⎞ ⎛p ⎞ 1 ⎛ p⎞ p⎞ ⎛p ⎞ ⎛ ⎛p ⎞ 1 ⎛ ⎛p ⎞ = f ⎜ ⎟ + ⎜ x − ⎟ f ′ ⎜ ⎟ + ⎜ x − ⎟ f ′′ ⎜ ⎟ + ⎜ x − ⎟ f ′′′ ⎜ ⎟ + … ... (1) 3⎠ 3 ⎠ ⎝ 3 ⎠ 2! ⎝ 3⎠ ⎝3⎠ ⎝ ⎝ 3 ⎠ 3! ⎝ ⎝3⎠

p⎞ ⎛p ⎞ ⎛ ⎛1⎞ f ⎜ ⎟ = log ⎜ cos ⎟ = log ⎜ ⎟ = − log 2 3 3 ⎝ ⎠ ⎝ ⎠ ⎝2⎠

Engineering Mathematics

2.82

Differentiating f (x) w.r.t. x successively, f ′( x) =

p ⎛p ⎞ f ′ ⎜ ⎟ = − tan = − 3 3 3 ⎝ ⎠ p ⎛p ⎞ f ′′ ⎜ ⎟ = − sec 2 = −4 3 ⎝3⎠

1 (− sin x) = − tan x, cos x

f ′′( x) = − sec 2 x,

f ′′′ ( x) = −2 sec 2 x tan x,

p p ⎛p ⎞ f ′′′ ⎜ ⎟ = −2 sec 2 tan = −2(4) 3 = − 8 3 and so on 3 3 ⎝3⎠

Substituting in Eq. (1), 2

p⎞ p⎞ 1⎛ ⎛ f ( x) = log (cos x) = − log 2 + ⎜ x − ⎟ − 3 + ⎜ x − ⎟ (−4) 3⎠ 2! ⎝ 3⎠ ⎝

(

)

3

+

p⎞ 1⎛ ⎜ x − ⎟ −8 3 + … 3⎠ 3! ⎝

(

)

3

2

4 3⎛ ⎛ ⎞ ⎛ ⎞ ⎞ = − log 2 − 3 ⎜ x − ⎟ − 2 ⎜ x − ⎟ − ⎜ x − ⎟ −… 3⎠ 3⎠ 3 ⎝ 3⎠ ⎝ ⎝ Example 6: Obtain tan-1 x in powers of (x - 1). Solution: Let f (x) = tan 1 x By Taylor’s series,

f ( x) = f (a ) + ( x − a ) f ′(a ) +

( x − a)2 ( x − a )3 f ′′(a ) + f ′′′(a ) + …… 2! 3!

Putting a = 1, f ( x) = tan −1 x = f (1) + ( x − 1) f ′(1) +

( x − 1) 2 ( x − 1)3 f ′′(1) + f ′′′(1) + … 2! 3!

p 4 Differentiating f (x) w.r.t. x successively, f (1) = tan −1 1 =

f ′( x) =

1 , 1 + x2

f ′(1) =

1 2

f ′′ ( x) = −

2x , (1 + x 2 ) 2

f ′′′( x) = −

2 8x2 1 + , f ′′′ (1) = 2 2 2 (1 + x ) (1 + x 2 )3

f ′′ (1) = −

2 1 = − and so on 4 2

... (1)

Differential Calculus I

2.83

Substituting in Eq. (1), 2 3 p ⎛ 1 ⎞ ( x − 1) ⎛ 1 ⎞ ( x − 1) ⎛ 1 ⎞ + ( x − 1) ⎜ ⎟ + ⎜ ⎟+ ⎜− ⎟+ 4 2! ⎝ 2 ⎠ 3! ⎝ 2 ⎠ ⎝2⎠ p 1 1 1 = + ( x − 1) − ( x − 1) 2 + ( x − 1)3 4 2 4 12

f ( x) = tan −1 x =

Example 7: Prove that

log [sin ( x + h)] = log sin x + h cot x –

h2 h3 cos x cosec2 x + + 2 3 sin 3 x

Solution: Let f (x) = log (sin x), f (x + h) = log [sin (x + h)] By Taylor’s series,

f ( x + h) = f ( x) + h f ′( x) +

h2 h3 f ′′( x) + f ′′′( x) + ………… 2! 3!

... (1)

Differentiating f (x) w.r.t. x successively,

1 cos x = cot x sin x f ′′( x) = − cosec 2 x f ′( x) =

f ′′′( x) = 2 cosec2 x cot x =

2 cos x and so on sin 3 x

Substituting in Eq. (1), f (x + h) = log [sin (x + h)] h2 h3 2 cos x cosec 2 x + + ……… 2! 3! sin 3 x h2 h3 cos x = log sin x + h cot x − cosec 2 x + + ……… 2 3 sin 3 x = log sin x + h cot x −

Example 8: Expand tan-1 (x + h) in powers of h and hence, find the value of tan-1 (1.003) up to 5 places of decimal. Solution: Let f (x) = tan 1 x , f (x + h) = tan 1 (x + h) By Taylor’s series, f ( x + h) = f ( x) + h f ′( x) +

h2 h3 f ′′( x) + f ′′′( x) + ………… 2! 3!

Differentiating f (x) w.r.t. x successively, 1 2x f ′( x) = , f ′′( x) = − 1 + x2 (1 + x 2 ) 2

... (1)

Engineering Mathematics

2.84

f ′′′( x) = −

2 2x ⋅ 4x 2(3 x 2 − 1) + = and so on (1 + x 2 ) 2 (1 + x 2 )3 (1 + x 2 )3

Substituting in Eq. (1), f ( x + h) = tan −1 ( x + h) = tan −1 ( x + h) ⋅

1 2x ⎤ h2 ⎡ + ⎢− ⎥ 2 2 ! ⎣ (1 + x 2 ) 2 ⎦ 1+ x h3 ⎡ 2(3 x 2 − 1) ⎤ + ⎢ ⎥ +……… 3! ⎣ (1 + x 2 )3 ⎦

Putting x = 1, h = 0.0003, tan −1 (1 + 0.003) = tan −1 (1.0003) = tan −1 1 +

0.0003 (0.0003) 2 + 2 2!

3 ⎛ 2 ⎞ (0.0003) ⎛ 1 ⎞ + − ⎜ ⎟ ⎜ ⎟ + ……… 3! ⎝ 4⎠ ⎝2⎠

p + 0.00015 − 2.25 × 10−8 + 2.25 × 10−12 [Considering first 4 terms] 4 = 0.78540 =

Example 9: Prove that

1 + x + 2 x2 = 1 +

x 7 x2 + 2 8

7 x3 + 16

.

Solution: Let f ( x) = x, f ( x + h) = x + h By Taylor’s series,

f ( x + h) = f ( x) + h f ′( x) +

h2 h3 f ′′( x) + f ′′′( x) + ……… 2! 3!

Putting x = 1, h = x + 2x 2 , f ( x + h) = x + h = 1 + x + 2 x 2

= f (1) + ( x + 2 x 2 ) f ′(1) + f ( x) =

x,

( x + 2 x 2 )2 ( x + 2 x 2 )3 f ′′(1) + f ′′′(1) + ……… ... (1) 2! 3!

f (1) = 1

Differentiating f (x) w.r.t. x successively, 1 f ′( x) = , 2 x 1⎛ 1⎞ 1 f ′′( x) = ⎜ − ⎟ 3 , 2⎝ 2⎠ 2 x f ′′′( x) =

1 ⎛ 1 ⎞⎛ 3 ⎞ 1 ⎜ − ⎟⎜ − ⎟ , 2 ⎝ 2 ⎠ ⎝ 2 ⎠ 52 x

f ′(1) =

1 2

f ′′(1) = −

1 4

f ′′′(1) =

3 and so on 8

Differential Calculus I

2.85

Substituting in Eq. (1),

1 1 ( x 2 + 4 x 3 + 4 x 4 ) 3 ( x3 + …) 1 + x + 2x2 = 1 + ( x + 2x2 ) − + + ......... 2 4 2 8 6 x 7 x 2 7 x3 − +… = 1+ + 2 8 16 1 + x + 2 x 2 in powers of (x - 1).

Example 10: Expand

1 + x + 2 x 2 = 4 + 2 ( x − 1) 2 + 5 ( x − 1) [Expressing in terms of (x – 1)]

Solution:

Let f ( x) = x, f ( x + h) = x + h By Taylor’s series,

f ( x + h) = f ( x) + h f ′( x) + Putting x = 4, h = 2 (x f ( x + h) =

1)2 + 5 (x

h2 h3 f ′′( x) + f ′′′( x) + ……… 2! 3!

1),

x + h = 4 + 2 ( x − 1) 2 + 5 ( x − 1)

= f (4) + [2 ( x − 1) 2 + 5 ( x − 1)] f ′(4) + f ( x) = x ,

[2 ( x − 1) 2 + 5 ( x − 1)]2 f ′′(4) + … ... (1) 2!

f (4) = 2

Differentiating f (x) w.r.t. x successively,

f ′( x) =

1 2 x

f ′′( x) = Substituting in Eq. (1),

,

f ′(4) =

1⎛ 1⎞ 1 ⎜− ⎟ , 2 ⎝ 2 ⎠ 32 x

1 4

f ′′(4) = −

1 and so on 32

1 4 [2 ( x − 1) 2 + 5( x − 1)]2 ⎛ 1 ⎞ + ⎜ − ⎟ + ……… 2! ⎝ 32 ⎠ 7 5 1 + x + 2 x 2 = 2 + ( x − 1) + ( x − 1) 2 + ………… 64 4 4 + 2 ( x − 1) 2 + 5( x − 1) = 2 + [2 ( x − 1) 2 + 5 ( x − 1)]

Example 11: Using Taylor’s theorem, evaluate up to 4 places of decimals: (i)

1.02

(ii)

25.15

(iii)

9.12

(iv)

10

Engineering Mathematics

2.86

Solution: Let f ( x) = x , f ( x + h) = x + h By Taylor’s series,

f ( x + h) = f ( x) + h f ′( x) + (i) Putting

h2 h3 f ′′( x) + f ′′′( x) + ………… 2! 3!

... (1)

x = 1, h = 0.02, f ( x + h) = x + h = 1 + 0.02 = f (1) + (0.02) f ′(1) +

(0.02) 2 f ′′(1) + ……… 2!

f ( x) = x ,

... (2)

f (1) = 1

Differentiating f (x) w.r.t. x successively, 1 1 , f ′(1) = 2 2 x 1 1 f ′′( x) = − 3 , f ′′(1) = − and so on 4 4x 2 Substituting in Eq. (2) and considering only first 3 terms, f ′( x) =

1 (0.02) 2 1.02 = 1 + (0.02) + 2 2! = 1.0099 approx.

⎛ 1⎞ ⎜⎝ − ⎟⎠ 4

(ii) Putting x = 25, h = 0.15 in Eq. (1), f ( x + h) =

x + h = 25 + 0.15

= f (25) + (0.15) f ′(25) +

f ( x) = x ,

(0.15) 2 f ′′(25) + … 2!

f (25) = 5

Differentiating f (x) w.r.t. x successively,

f ′( x) =

1

,

f ′(25) =

1 = 0.1 10

2 x 1 1 f ′′( x) = − 3 , f ′′(25) = − = − 0.002 and so on 500 2 4x Substituting in Eq. (2) and considering only first 3 terms, 25.15 = 5 + (0.15) (0.1) + = 5.0150 approx.

(0.15) 2 ( −0.002) 2

... (3)

Differential Calculus I

2.87

(iii) Putting x = 9, h = 0.12 in Eq. (1),

f ( x + h) = x + h = 9 + 0.12

= f (9) + (0.12) f ′(9) + f ( x) = x ,

(0.12) 2 f ′′(9) + … 2!

... (3)

f (9) = 3

Differentiating f (x) w.r.t. x successively, f ′( x) =

1

f ′(9) =

,

1 6

2 x 1 1 and so on f ′′ ( x) = − 3 , f ′′(9) = − 108 2 4x Substituting in Eq. (2) and considering only first 3 terms, 2 ⎛ 1 ⎞ (0.12) ⎛ 1 ⎞ 9.12 = 3 + (0.12) ⎜ ⎟ + ⎜− ⎟ ⎝ 6⎠ 2 ⎝ 108 ⎠ = 3 + 0.02 − (0.12) (0.06) (0.0093) = 3.0199 approx.

(iv) Putting

x = 9, h = 1 in Eq. (1), f ( x + h) = x + h = 9 + 1 = f (9) + f ′(9) +

10 = 3 +

1 f ′′(9) + … 2!

1 1 − 6 216

... (4) [refer (iii)]

= 3.1620 approx. Example 12: Find the value of tan (43ç). Solution: Let f (x) = tan x,

f (x + h) = tan (x + h)

By Taylor’s series, f ( x + h) = f ( x) + h f ′( x) +

h2 h3 f ′′ ( x) + f ′′′ ( x) + ………… 2! 3!

2p p =− = −0.0349 , 180 90 tan ( x + h) = tan (45° − 2° ) = tan 43°

Putting x = 45° , h = −2° = −

= f (45° ) + (−0.0349) f ′(45° ) + f ( x) = tan x ,

(−0.0349) 2 f ′′(45° ) + ……… ... (1) 2!

f (45° ) = tan (45° ) = 1

Engineering Mathematics

2.88

Differentiating f (x) w.r.t. x successively, f ′ ( x) = sec 2 x, f ′ (45° ) = sec 2 45° = 2 f ′′(45° ) = 2 sec 2 45° tan 45° = 4

f ′′ ( x) = 2 sec 2 x tan x,

and so on

Substituting in Eq. (1) and considering only first 3 terms,

tan 43° = 1 + (−0.0349)(2) +

(−0.0349) 2 (4) 2!

= 0.9326 approx. Example 13: Find cosh (1.505) given sinh (1.5) = 2.1293 and cosh (1.5) = 2.3524. Solution: Let f (x) = cosh x By Taylor’s series, f ( x + h) = f ( x ) + h f ′ ( x ) +

h2 h3 f ′′ ( x) + f ′′′ ( x) + ………… 2! 3!

Putting x = 1.5, h = 0.005, f (x + h) = cosh (x + h) = cosh (1.5 + 0.005) (0.005)3 (0.005) 2 f ′′(1.5) + f ′′′ (1.5) + 3! 2! f ( x) = cosh x, f (1.5) = cosh (1.5) = 2.3524 = f (1.5) + (0.005) f ′(1.5) +

Differentiating f (x) w.r.t. x successively, f (x) = sinh x, f (1.5) = sinh (1.5) = 2.1293 f (x) = cosh x, f (1.5) = cosh (1.5) = 2.3524

... (1)

and so on

Substituting in Eq. (1) and considering only first 3 terms,

(0.005) 2 coosh (1.5) + … 2! = 2.3524 + (0.005)(2.1293) + (12.5) (10−6 )(2.3524) = 2.3631 approx.

cosh (1.505) = cosh (1.5) + (0.005) sinh (1.5) +

Exercise 2.6 1. Expand ex in powers of (x − 1). ⎡ ⎛ ( x − 1) 2 ⎤ ⎢ Ans.: e ⎜1 + ( x − 1) + ⎥ 2! ⎥ ⎝ ⎢ ⎢ ⎞ ⎥ ( x − 1)3 ⎢ + + …⎟ ⎥ ⎢⎣ 3! ⎠ ⎥⎦ 2. Expand 2x3 + 7x2 + x − 1 in powers of x − 2. [Ans.: 45 + 53 (x 2) + 19 (x 2)2 + 2 (x 2)3]

3. Expand x5 − 5x4 + 6x3 − 7x2 + 8x in powers of (x 1).

9

⎡ Ans. : − 6 − 3 ( x − 1) − 9 ( x − 1) 2 ⎤ ⎢ ⎥ − 4 ( x − 1)3 + ( x − 1)5 ⎥⎦ ⎢⎣ 4. Expand x4 − 3x3 + 2x2 − x + 1 in powers of (x 3). ⎡ Ans. :16 + 38 ( x − 3) + 29 ( x − 3) 2 ⎤ ⎢ ⎥ + 9 ( x − 3)3 + ( x − 3) 4 ⎥⎦ ⎢⎣

Differential Calculus I 5. Expand x3 − 2x2 + 3x − 5 in power of (x 2). [Ans.: 11 + 7 (x

2) + 4 (x 2)2 + (x − 2)3]

6. Expand 2x3 + 3x2 − 8x + 7 in terms of (x 2). [Ans.: 19 + 28 (x 7. Expand ⎡ ⎢ Ans.: ⎣

8. Expand (x 1).

2) + 15 (x 2)2 + 2 (x − 2)3]

⎤ a+ − − …⎥ 2 a 8a a ⎦ ( x − a )3

1 + x + 2x 2 in powers of

5 7 ⎡ ⎤ 2 ⎢ Ans.: 2 + 4 ( x − 1) + 32 ( x − 1) + …⎥ ⎣ ⎦

9. Expand sin x in powers of (x − a). ⎡ Ans.: sin a + ( x − a ) ⎤ ⎢ ⎥ 2 ( x − a) ⎥ ⎢ cos a − sin a ⎥ ⎢ 2! ⎥ ⎢ 3 ( x − a) ⎢ cos a + …⎥ − ⎢⎣ 3! ⎦⎥ p 10. Expand cos x in powers of ⎛⎜ x − ⎞⎟ . 2⎠ ⎝

⎡ p ⎞ 1⎛ p⎞ ⎛ ⎢ Ans.: − ⎜ x − ⎟ + ⎜ x − ⎟ 2 ⎠ 3! ⎝ 2⎠ ⎝ ⎢ 5 ⎢ 1⎛ p⎞ ⎢ − ⎜ x − ⎟ +… ⎢⎣ 5! ⎝ 2⎠

⎤ 3 1 x2 x− ⋅ ⎥ 2 2 2! ⎥ ⎥ 3 x3 1 x 4 ⋅ + ⋅ +… ⎥ 2 3! 2 4 ! ⎦

⎛p ⎞ 13. Expand tan ⎜ + x ⎟ in powers of x 4 ⎝ ⎠ upto x4 and hence find the value of tan (46 36').

x in powers of (x − a). ( x − a)

⎡ 1 ⎢ Ans.: + 2 ⎢ ⎢ − ⎢ ⎣

2.89

3

⎤ ⎥ ⎥ ⎥ ⎥ ⎥⎦

p 11. Expand tan x in powers of ⎛⎜ x − ⎞⎟ . 4⎠ ⎝ 2 ⎡ ⎤ p⎞ p⎞ ⎛ ⎛ Ans.: 1 + 2 − + 2 − x x ⎢ ⎜ ⎟ ⎜ ⎟ +…⎥ 4⎠ 4⎠ ⎝ ⎝ ⎢⎣ ⎥⎦

⎛p ⎞ 12. Expand sin ⎜ + x ⎟ in powers of x 6 4 ⎝ ⎠ upto x .

⎡ 8 3 ⎤ ⎛ 2 ⎢ Ans.: ⎜1 + 2 x + 2 x + 3 x ⎥ ⎝ ⎢ ⎥ ⎢ ⎥ 10 4 ⎞ + x + … ⎟ , 1.0574⎥ ⎢ 3 ⎠ ⎣ ⎦ 14. Using Taylor’s theorem find approximate value of cos 64 . [Ans.: 0.4384] 15. Using Taylor’s theorem find approximate value of sin (30 30 ). [Ans.: 0.5073] 16. Expand log x in powers of (x

2).

⎡ 1 1 ( x − 2) 2 2 Ans.: log + ( x − 2 ) − ⋅ ⎢ 2 2! 4 ⎢ ⎢ 1 ( x − 2) 3 + ⋅ +… ⎢ 3! 4 ⎣

⎤ ⎥ ⎥ ⎥ ⎥ ⎦

17. Expand log sin x in powers of (x 2). ⎡ Ans.: log sin 2 + ( x − 2) cot 2 ⎤ ⎢ ⎥ 1 ⎢ − ( x − 2) 2 cosec 2 x + …⎥ ⎢⎣ ⎥⎦ 2 ⎛p ⎞ 18. Expand log tan ⎜ + x ⎟ in powers ⎝4 ⎠ of x. 4 3 4 5 ⎤ ⎡ ⎢ Ans.: 2 x + 3 x + 3 x + … ⎥ ⎦ ⎣ 19. Arrange in powers of x, by Taylor’s theorem, 7 + (x + 2) + 3 (x + 2)3 + (x + 2)4. [Ans.: 49 + 69x + 42x2 + 11x3 + x4]

2.90

Engineering Mathematics

20. Arrange in powers of x, by Taylor’s theorem, 17 + 6 (x + 2) + 3 (x + 2)3 + (x + 2)4 − (x + 2)5. [Ans.: 37 − 6x − 38x2 − 29x3 − 9x4 − x5] 21. Arrange in powers of (x + 1), by Taylor’s theorem, (x + 2)4 + 5 (x + 2)3 + 6 (x + 2)2 + 7 (x + 2) + 8. ⎡ Hint : f ( x) = x 4 + 5 x 3 + 6 x 2 + 7 x + 8, ⎤ ⎥ ⎢ f [( x + 1) + 1] = f (1) ⎥ ⎢ ⎥ ⎢ ( x + 1) 2 ⎢ f ′′ (1) + …⎥ + ( x + 1) f ′(1) + 2! ⎦ ⎣

⎡ Ans. : 27 + 38 (x + 1) + 27 (x + 1) 2 ⎤ ⎥ ⎢ + 9 (x + 1)3 + (x + 1) 4 ⎥⎦ ⎢⎣ 22. Prove that sinh (x + a) = sinh a + x2 sinh a + … x cosh a + 2! Given sinh (1.5) = 2.1293, cosh (1.5) = 2.3524, find the value of sinh (1.505). [Ans. 2.1411]

2.9 MACLAURIN’S SERIES Statement: If f (x) be a given function of x which can be expanded in positive ascending integral powers of x, then f ( x) = f (0) + x f ′(0) +

x2 x3 xn n f ′′ (0) + f ′′′ (0) + ……… + f (0) + ……… 2! 3! n!

Proof: Let f (x) be a function of x which can be expanded into positive ascending integral powers of x, then f (x) = a0 + a1x + a2 x2 + a3 x3 + a4 x4 +……. …

... (1)

Differentiating w.r.t. x successively, f (x) = a1 + a2· 2x + a3 · 3x2 + a4 · 4x3 + ……. ……… f (x) = a2 · 2 + a3 · 6x + a4 · 12x2 + ……. ……… f (x) = a3 · 6 + a4 · 24x + ……. ………

... (2) ... (3) ... (4)

and so on Putting x = 0 in Eq. (1), (2), (3) and (4), a0 = f (0) a1 = f (0) 1 a2 = f ′′(0) 2! 1 a3 = f ′′′(0) 3!

and so on.

Substituting a0, a1, a2 and a3 in Eq. (1), f ( x) = f (0) + x f ′(0) +

x2 x3 xn n f ′′ (0) + f ′′′ (0) + ……… + f (0) + ……… 2! 3! n!

Differential Calculus I

2.91

This is known as Maclaurin’s Series. This series can also be written as, y = y (0) + xy1 (0) +

x2 x3 xn y2 (0) + y3 (0) + …… + yn (0) + ……… 2! 3! n!

2.9.1 Standard Expansions Using Maclaurin’s series, expansion of some standard functions can be obtained. These expansions can be directly used while solving the examples. (1) Expansion of ex (Exponential series) Proof: Let y = ex, y (0) = e0 = 1 dn x (e ) = e x , yn (0) = e0 = 1 for all values of n . dx n Substituting in Maclaurin’s series, Now yn =

x 2 x3 + + ……… 2 ! 3!

ex = 1 + x +

This series is known as the exponential series. Note: In the above series (i) Replacing x by −x, e− x = 1 − x +

x 2 x3 − + ……… 2 ! 3!

(ii) Replacing x by ax, a 2 x 2 a3 x3 + +……… 2! 3!

eax = 1 + ax + (2) Expansion of sin x (Sine series) Proof: Let y = sin x, y (0) = sin 0 = 0

yn =

Now

dn np ⎞ ⎛ (sin x) = sin ⎜ x + ⎟ n 2 ⎠ dx ⎝

⎛ np ⎞ yn (0) = sin ⎜ ⎟ ⎝ 2 ⎠ Putting

n = 1, 2, 3, 4, 5, ..…..

y1 (0) = 1, y2 (0) = 0, y3 (0) =

1, y4 (0) = 0, y5 (0) = 1, and so on.

Substituting in Maclaurin’s series, sin x = x −

This series is known as the sine series.

x3 x5 + − ……… 3! 5 !

Engineering Mathematics

2.92

(3) Expansion of cos x (Cosine series) Proof: Let y = cos x, y (0) = cos 0 = 1 dn np ⎞ ⎛ Now yn = n (cos x) = cos ⎜ x + ⎟ 2 ⎠ dx ⎝ ⎛ np ⎞ yn (0) = cos ⎜ ⎟ ⎝ 2 ⎠ Putting n = 1, 2, 3, 4, ….. y1 (0) = 0, y2 (0) = 1, y3 (0) = 0, y4 (0) = 1, Substituting in Maclaurin’s series, x 2 x 4 ……… cos x = 1 − + − 2! 4! This series is known as the cosine series.

and so on.

(4) Expansion of tan x (Tangent series) Proof: Let y = tan x, y1 = sec 2 x = 1 + tan 2 x = 1 + y 2 ,

y(0) = 0 y1 (0) = 1

y2 = 2 yy1 ,

y2 (0) = 2 y (0) y1 (0) = 2(0)((1) = 0

y3 = 2 y + 2 yy2 ,

y3 (0) = 2(1) 2 + 2(0)(0) = 2

y4 = 4 y1 y2 + 2 y1 y2 + 2 yy3

y4 (0) = 6(1)(0) + 2(0)(2)

2 1

= 6 y1 y2 + 2 yy3 ,

=0

y5 = 6 y2 + 6 y1 y3 + 2 y1 y3 + 2 yy4 2

= 6 y2 + 8 y1 y3 + 2 yy4 , 2

y5 (0) = 0 + 8(1)(2) + 0 = 16

Substituting in Maclaurin’s series, x3 x5 tan x = x + (2) + (16) + ……… 3! 5! 3 5 x 2x = x+ + + ……… 3 15 This series is known as the tangent series. Note: This series can also be obtained by dividing the sine and cosine series sin x since tan x = . cos x (5) Expansion of sinh x Proof: We have sinh x =

e x − e− x 2

Substituting ex and e x from above exponential series, ⎛ ⎞ ⎛ ⎞ x 2 x3 x 2 x3 ⎜1 + x + + + … ⎟ − ⎜1 − x + − + … ⎟ 2! 3! 2! 3! ⎠ ⎝ ⎠ sinh x = ⎝ 2 x3 x5 = x + + +……… 3! 5!

Differential Calculus I

2.93

(6) Expansion of cosh x e x + e− x 2 Substituting exponential series ex and e x,

Proof: We have sinh x =

⎛ ⎞ ⎛ ⎞ x 2 x3 x 2 x3 ⎜ 1 + x + + + …⎟ + ⎜ 1 − x + − + …⎟ 2 ! 3! 2 ! 3! ⎠ ⎝ ⎠ cosh x = ⎝ 2 x2 x4 = 1 + + +……… 2 ! 4! (7) Expansion of tanh x Proof: Expansion of tanh x can be obtained by dividing the series of sinh x and cosh x. x3 x5 x 7 + + +… sinh x 3! 5 ! 7 ! tanh x = = cosh x x2 x4 x6 1+ + + +… 2! 4! 6! x2 2 5 = x − + x − ……… 3 15 x+

Note: This series can also be obtained by using Maclaurin's series (refer tangent series) (8) Expansion of log (1 + x) (Logarithmic series) Proof: Let y = log (1 + x), y (0) = log 1 = 0

yn =

Now

dn (n − 1)! [ log (1 + x)] = (−1) n −1 ⋅ dx n ( x + 1) n

yn (0) = ( −1) n −1 ⋅ (n − 1)!

Putting

n = 1, 2, 3, 4, ….. y1(0) = 1, y2 (0) =

1, y3 (0) = 2! and so on

Substituting in Maclaurin’s series, log (1 + x) = x −

x 2 x3 + − ……… 2 3

This series is known as the Logarithmic series and is valid for 1 < x < 1. Note: In above series replacing x by −x, we get expansion of log (1 − x) log (1 − x) = − x −

(9) Expansion of (1 + x)m (Binomial series) Proof: Let y = (1 + x) m , y (0) = (1 + 0) m = 1

x 2 x3 x 4 x5 − − − −… 2 3 4 5

Engineering Mathematics

2.94

yn = m (m − 1) (m − 2)……… (m − n + 1)(1 + x) m − n

Now

yn (0) = m(m − 1)(m − 2)……… (m − n + 1)

Putting

n = 1, 2, 3, 4, …..

y1(0) = m, y2(0) = m (m − 1), y3(0) = m (m − 1) (m − 2) and so on Substituting in Maclaurin’s series, m(m − 1) 2 m(m − 1)(m − 2) 3 (1 + x) m = 1 + mx + x + x + ……… 2! 3! This series is known as the Binomial series and is valid for 1 < x < 1.

x

Example 1: Expand 5 up to the first three non-zero terms of the series. x

f (x) = 5 , f (0) = 50 = 1 x f (x) = 5 log 5, f (0) = 50 log 5 = log 5 x f (x) = 5 (log 5)2, f (0) = 50 (log 5)2 = (log 5)2 Substituting in Maclaurin’s series,

Solution: Let

f ( x) = f (0) + x f ′ (0) + 5 x = 1 + x log 5 + Aliter:

x2 f ′′(0) + ……… 2!

x2 (log 5) 2 + ……… 2!

x

f (x) = 5x = e log 5 = e x log 5 ( x log 5) 2 = 1 + x log 5 + + ……… 2!

[Using Exponential series]

⎛ 1+ x ⎞ Example 2: Obtain the series log (1 + x) and find the series log ⎜ ⎟ and ⎝ 1- x ⎠ ⎛ 11 ⎞ hence, find the value of loge ⎜ ⎟ . ⎝ 9 ⎠ Solution: Let y = log (1 + x) y1 =

1 1 (2 !) (3!) etc. , y2 = − , y3 = , y4 = − 1+ x (1 + x) 2 (1 + x)3 (1 + x) 4

At x = 0, y = 0, y1 = 1, y2 = −1, y3 = 2 !, y4 = −(3!) etc. Substituting in Maclaurin’s series,

x2 x3 x4 y2 (0) + y3 (0) + y4 (0) + ……… 2! 3! 4! x4 x 2 x3 = 0 + x − + (2 !) − (3!) + ……… 2 ! 3! 4! x 2 x3 x 4 log (1 + x) = x − + − + ……… 2 3 4 y = y (0) + xy1 (0) +

Differential Calculus I

2.95

Replacing x by −x, log (1 − x) = − x −

x 2 x3 x 4 − − − ……… 2 3 4

⎛1+ x⎞ log ⎜ = log (1 + x) − log (1 − x) ⎝ 1 − x ⎟⎠ ⎛ ⎞ x3 x5 = 2 ⎜ x + + + …⎟ 3 5 ⎝ ⎠

Now,

Putting x =

1 , and considering first three terms, 10 ⎡1 1 1 1 1 ⎤ ⎛ 11 ⎞ + ⋅ log e ⎜ ⎟ = 2 ⎢ + ⋅ ⎥ = 0.20067 3 9 10 3 5 (10)5 ⎦ ( ) 10 ⎝ ⎠ ⎣

Example 3: If x 3

y3

xy 1

0, prove that y

1

x 3

26 3 x … 81

Solution: x3 + y3 + xy 1 = 0, Putting x = 0, y (0) = 1 Differentiating w.r.t. x, 3 x 2 + 3 y 2 y1 + xy1 + y = 0 −1 3 Differentiating Eq. (1) w.r.t. x, 6x + 6yy12 + 3y2y2 + 2y1 + xy2 = 0

...(1)

Putting x = 0, y1 (0) =

Putting x

0, 6

1 3

2

3 y2 (0) 2

1 3

0

y2 (0) = 0 Differentiating Eq. (2) w.r.t. x, 6 + 6y13 + 12yy1y2 + 3y2y3 + 6yy1y2 + 3y2 + xy3 = 0 Putting x = 0,

⎛ −1 ⎞ 6 + 6 ⎜ ⎟ + 0 + 3 y3 (0) = 0 ⎝ 27 ⎠ − 52 y3 (0) = and so on. 27 Substituting in Maclaurin’s series, x2 x3 y2 (0) + y3 (0) + 2! 3! x x2 x 3 ⎛ − 52 ⎞ y = 1 − + ( 0) + ⎜ ⎟+ 3 2! 3! ⎝ 27 ⎠ x 26 3 =1− − x − 3 81 y = y (0) + xy1 (0) +

… (2)

Engineering Mathematics

2.96

Example 4: If x3 + 2xy2 - y3 + x - 1 = 0, expand y in ascending powers of x. x3 + 2xy2

Solution: Putting x = 0,

y3 + x

1=0

y(0) = – 1

Differentiating w.r.t. x, 3x2 + 2y2 + 4xyy1 Putting x = 0, 2

3y2y1 + 1 = 0

… (1)

3y1 (0) + 1 = 0 y1 (0) = 1

Differentiating Eq. (1) w.r.t. x, 6x + 4yy1 + 4yy1 + 4xy12 + 4xyy2 Putting x = 0, 8+6

6yy12

3y2y2 = 0

3y2 (0) = 0 y2 (0)

2 and so on. 3

Substituting in Maclaurin’s series, x2 y 2 ( 0) + 2! x2 ⎛ 2 ⎞ y = −1+ x + ⎜− ⎟ + 2! ⎝ 3 ⎠ x2 + = −1 + x − 3 y = y (0) + xy1 (0) +

Example 5: If x = y (1 + y2), prove that y = x - x3 + 3x5 + … . Solution:

x = y (1 + y2)

Putting x = 0, y (0) = 0 Differentiating w.r.t. x, Putting

x = 0,

1 = y1 + 3y2y1

1 = y1 (0) y1 (0) = 1 Differentiating Eq. (1) w.r.t. x, 0 = y2 + 6yy12 + 3y2y2 Putting x = 0, y2 (0) = 0, Differentiating Eq. (2) w.r.t. x, 0 = y3 + 12yy1y2 + 6y13 + 6yy1y2 + 3y2y3 0 = y3 (1 + 3y2) + 18yy1y2 + 6y13 Putting x = 0, 0 = y3 (0) + 6 y3 (0) = −6

… (1)

… (2)

… (3)

Differential Calculus I

2.97

Differentiating Eq. (3) w.r.t. x, 0 = (1 + 3y2) y4 + 6yy1y3 + 18y12 y2 + 18yy22 + 18yy1y3 + 18y12 y2 = (1 + 3y2) y4 + 24yy1y3 + 36y12 y2 + 18yy22 Putting x = 0, y4 (0) = 0, Differentiating Eq. (4) w.r.t. x, 0 = (1 + 3y2) y5 + 6yy1y4 + 24y12 y3 + 24yy2y3 + 24yy1y4 + 72y1y22

… (4)

+ 36y12y3 + 36yy2y3 + 18y1y22 Putting x = 0,

0 = y5 (0) + 24 (−6) + 36 (−6) y5 (0) = 360 and so on. Substituting in Maclaurin’s series, x2 x3 x4 x5 y 2 ( 0) + y3 (0) + y 4 ( 0) + y5 (0) + 2! 3! 4! 5! x2 x3 x4 x5 = 0 + x.1 + ⋅ 0 + (− 6) + ⋅0+ ⋅ 360 + 2! 3! 4! 5! = x − x3 + 3x5 +

y = y (0) + xy1 (0) +

2 Example 1: Obtain the expansion of 1 + x 1 + x4 1 + x2 Solution: = (1 + x 2 )(1 + x 4 ) 1 1 + x4 = (1 + x2) (1 − x4 + x8 − x12 + x16 − …) = 1 + x2 − x4 − x6 + x8 + x10 − …

Example 2: If x y = x+

y

y2 2

y3 3

y4 4

..... , prove that

x2 x3 x4 + + + ......... and conversely. 2! 3! 4!

Solution:

x = log (1 + y ) 1 + y = ex y

ex 1 = x+

x 2 x3 + + ....... 2! 3!

Conversely, y

ex 1

ex = 1 + y x = log (1 + y ) y

y2 2

y3 3

y4 4

.....

2.98

Engineering Mathematics

Example 3: Expand 1 + sin x . Solution:

1 + sin x = sin

x x + cos 2 2

2 4 ⎡ x 1 ⎛ x ⎞3 ⎤ ⎡ 1 ⎛x⎞ 1 ⎛x⎞ = ⎢ − ⎜ ⎟ + ⎥ + ⎢1 − ⎜ ⎟ + ⎜ ⎟ − 4 ⎝2⎠ ⎢⎣ 2 3! ⎝ 2 ⎠ ⎥⎦ ⎣⎢ 2 ! ⎝ 2 ⎠ 2 3 4 x x x x = 1+ − − + − 2 8 48 384 1 4 2 6 x x ..... . Example 4: Prove that cos 2 x 1 x 2 3 45 1 Solution: cos 2 x = (1 + cos 2 x) 2

=

(2 x) 2 (2 x) 4 (2 x)6 1⎡ + − + − + 1 1 ⎢ 2⎣ 2! 4! 6!

⎤ ⎥ ⎥⎦

⎤ ⎥ ⎦

1 4 2 6 x − x + 3 45 ∞ 1 32 n + 3 2 n Example 5: Prove that cosh3 x = x . 4 n = 0 ( 2n)! = 1 − x2 +



Solution:

1 (cosh 3 x + 3 cos h x) 4 1 ⎡⎛ (3 x) 2 (3x) 4 = ⎢⎜ 1 + + + 4 ⎣⎝ 2! 4!

cosh3 x =

⎞ ⎛ x2 x4 + + ⎟ + 3 ⎜1 + 2! 4! ⎠ ⎝

=

32 + 3 2 34 + 3 4 1⎡ x + x + ⎢(1 + 3) + 4⎣ 2! 4!

=

1 4

⎞⎤ ⎟⎥ ⎠⎦

⎤ ⎥ ⎦

32 n + 3 2 n x n = 0 ( 2n)! ∞



Example 6: Prove that sin x sinh x = x 2 −

8 6 32 10 x + x − ... . 6! 10 !

⎛ ⎞ ⎛ ⎞ x3 x5 x7 x9 x3 x5 x 7 x9 Solution: sin x sin h x = ⎜ x − + − + − ... ⎟ ⋅ ⎜ x + + + + + ... ⎟ ! ! ! ! ! ! ! 3 5 7 9 3 5 7 9! ⎝ ⎠ ⎝ ⎠

⎡2 1 ⎤ 10 ⎡ 2 2 1 ⎤ = x 2 + x6 ⎢ − +x ⎢ − + + ... 2⎥ 2⎥ ⎣ 5! (3!) ⎦ ⎣ 9 ! 7 !3! (5!) ⎦ 8 32 10 x − ... = x 2 − x6 + 6! 10 !

Differential Calculus I

2.99

Example 7: Expand log (1 + x + x2 + x3) up to a term in x8. Solution: log (1 + x + x2 + x3) = log [(1 + x) (1 + x2)] = log (1 + x) + log (1 + x2) ⎤ ⎡ ⎤ ⎡ x 2 x 3 x 4 x 5 x 6 x 7 x8 ( x 2 ) 2 ( x 2 )3 ( x 2 ) 4 + ...⎥ = ⎢ x − + − + − + − + ...⎥ + ⎢ x 2 − + − 2 3 4 5 6 7 8 2 3 4 ⎦ ⎣ ⎦ ⎣ 2 3 5 6 7 x x 3 4 x x x 3 8 = x + + − x + + + − x + ... 2 3 4 5 6 7 8 x2 x3 x4 Example 8: Prove that log (1 + x + x2 + x3 + x4) = x + + + – . 2 3 4 Solution:

⎛ 1 − x5 ⎞ log (1 + x + x2 + x3 + x4) = log ⎜ ⎟ ⎝ 1− x ⎠ = log (1 − x 5 ) − log (1 − x)

[Using sum of G.P.]

⎛ ⎞ ⎛ x10 x15 x 20 x 2 x3 x 4 ⎞ = ⎜ − x5 − − − − ⎟ − ⎜− x − − − ⎟ 2 3 4 2 3 4 ⎠ ⎝ ⎠ ⎝ x 2 x3 x 4 =x+ + + − 2 3 4 2 3 x 1 x 1 x Example 9: Prove that log x log 2 1 1 1 2 2 2 3 2 x Solution: log x log 2 2 x = log 2 + log 2 ⎡ ⎛x ⎞⎤ = log 2 + log ⎢1 + ⎜ − 1⎟ ⎥ 2 ⎝ ⎠⎦ ⎣ 2

3

1⎛ x ⎞ ⎞ ⎛x ⎞ 1⎛ x = log 2 + ⎜ − 1⎟ − ⎜ − 1⎟ + ⎜ − 1⎟ − … 2 2 2 3 2 ⎝ ⎠ ⎝ ⎝ ⎠ ⎠ ⎛ sinh x ⎞ x 2 x 4 Example 10: Prove that log ⎜ − + ... . ⎟= ⎝ x ⎠ 6 180 Solution:

⎡1 ⎛ ⎛ x2 x4 ⎞ ⎞⎤ x3 x5 ⎛ sinh x ⎞ log ⎜ ⎟ = log ⎢ ⎜ x + + + ... ⎟ ⎥ = log ⎜1 + + + ... ⎟ 3! 5! 3! 5! ⎝ x ⎠ ⎝ ⎠ ⎠⎦ ⎣x⎝

x2 3!

x4 ... 5!

x2 6

x4

x2 6

x4 ..... 180

1 120

1 x2 2 3! 1 72

x4 ... 5!

......

2

.....

... .

Engineering Mathematics

2.100

x2 3

Example 11: Prove that log ( x cot x ) Solution: log ( x cot x)

log

7 4 x 90

.... .

1 x cot x

⎛ tan x ⎞ = − log ⎜ ⎟ ⎝ x ⎠ ⎛ x2 2 ⎞ = − log ⎜1 + + x 4 + ... ⎟ 3 15 ⎝ ⎠ 2 2 ⎡⎛ x ⎤ ⎞ 1 ⎛ x2 2 ⎞ 2 = − ⎢⎜ + x 4 + .... ⎟ − ⎜ + x 4 + ... ⎟ + ...⎥ ⎢⎣⎝ 3 15 ⎥⎦ ⎠ 2 ⎝ 3 15 ⎠ ⎡ x2 ⎤ ⎛ 2 1⎞ = − ⎢ + x 4 ⎜ − ⎟ + ...⎥ ⎝ 15 18 ⎠ ⎣3 ⎦ 2 7 x = − − x 4 + ... 3 90 ⎛ 1 + e2x Example 12: Prove that log ⎜ x ⎝ e

⎞ x2 x4 x6 – – + ⎟ = log 2 + 2 12 45 ⎠

.

⎛ 1 + e2 x ⎞ Solution: log ⎜ x ⎟ = log (e − x + e x ) = log (2 cosh x) ⎝ e ⎠ = log 2 + log cosh x ⎛ x2 x4 x6 ⎞ = log 2 + log ⎜1 + + + + ... ⎟ 2! 4! 6! ⎝ ⎠ 2

3

⎞ ⎛ x2 x4 x6 ⎞ 1 ⎛ x2 x4 ⎞ 1 ⎛ x2 = log 2 + ⎜ + + + ... ⎟ − ⎜ + + ... ⎟ + ⎜ + ... ⎟ + ... ⎠ ⎝ 2! 4! 6! ⎠ 2 ⎝ 2! 4! ⎠ 3 ⎝ 2! ⎛ x2 x4 x6 ⎞ 1 ⎛ x4 ⎞ 1 ⎛ x6 ⎞ x6 = log 2 + ⎜ + + + ... ⎟ − ⎜ + 2 ⋅ + ... ⎟ + ⎜ + ... ⎟ + ... 48 ⎝ 2! 4! 6! ⎠ 2⎝ 4 ⎠ 3⎝ 8 ⎠ 2 x 1 1 ⎞ ⎛ 1 1⎞ ⎛ 1 = log 2 + + x 4 ⎜ − ⎟ + x 6 ⎜ − + ⎟ + ... 2 24 8 720 48 24 ⎝ ⎠ ⎝ ⎠ = log 2 +

x2 x4 x6 − + − ... 2 12 45

Example 13: Prove that log (1 e x )

Solution:

log 2

x 2

x2 8

⎛ x 2 x3 x 4 log (1 + e x ) = log ⎜1 + 1 + x + + + + 2 ! 3! 4 ! ⎝

x4 192 ⎞ ⎟ ⎠

...... .

Differential Calculus I

2.101

⎡ ⎛ x x 2 x3 x 4 ⎞⎤ = log ⎢ 2 ⎜1 + + + + + ... ⎟ ⎥ ⎠⎦ ⎣ ⎝ 2 4 12 48 ⎞ ⎛ x x 2 x3 x 4 = log 2 + log ⎜1 + + + + + ... ⎟ 2 4 12 48 ⎝ ⎠ ⎛ x x 2 x3 x 4 ⎞ 1 ⎛ x x 2 x3 ⎞ = log 2 + ⎜ + + + + ... ⎟ − ⎜ + + + ... ⎟ ⎝ 2 4 12 48 ⎠ 2 ⎝ 2 4 12 ⎠ 3

2

4

⎞ 1⎛ x 1 ⎛ x x2 ⎞ + ⎜ + + ... ⎟ − ⎜ + ... ⎟ + ... 3⎝ 2 4 4 2 ⎝ ⎠ ⎠ ⎛x⎞ ⎛1 1⎞ ⎛1 1 1 ⎞ = log 2 + ⎜ ⎟ + x 2 ⎜ − ⎟ + x 3 ⎜ − + ⎟ 2 4 8 ⎝ ⎠ ⎝ ⎠ ⎝ 12 8 24 ⎠ 1 1 1 1 ⎞ ⎛ 1 + x 4 ⎜ − − + − ⎟ + ... ⎝ 48 32 24 16 64 ⎠ = log 2 +

x x2 ⎛ 1 ⎞ 4 + +0+⎜− ⎟ x + ... 2 8 ⎝ 192 ⎠

= log 2 +

x x2 x4 + − + ... 2 8 192

1 ⎤ ⎡ x 5 x2 x3 251 4 Example 14: Prove that log ⎢ log (1 + x ) x ⎥ = − + − + x + ... . 2 24 8 2880 ⎢⎣ ⎥⎦ 1 1 Solution: log (1 + x) x = log (1 + x) x ⎞ 1⎛ x 2 x3 x 4 x5 = ⎜x − + − + − ⎟ x⎝ 2 3 4 5 ⎠

x x 2 x3 x 4 + − + − 2 3 4 5 ⎛ x x 2 x3 x 4 − + =1− ⎜ − + 4 5 ⎝ 2 3 = 1− y =1−

Now,

⎞ ⎟ ⎠

1 ⎡ ⎤ y 2 y3 y 4 log ⎢log (1 + x) x ⎥ = log (1 − y ) = − y − − − − ... 2 3 4 ⎣ ⎦ 2 ⎛ x x 2 x3 x 4 ⎞ 1 ⎛ x x 2 x3 ⎞ =−⎜ − + − + ... ⎟ − ⋅ ⎜ − + − ... ⎟ 4 5 4 ⎝2 3 ⎠ 2 ⎝2 3 ⎠ 3

4

⎞ 1 ⎛ x x2 ⎞ 1 ⎛ x x2 − ... ⎟ − ⎜ − + ... ⎟ − ... − ⎜ − 3⎝2 3 ⎠ 4⎝ 2 3 ⎠

Engineering Mathematics

2.102

x ⎛1 1⎞ ⎛1 1 1 ⎞ ⎛1 1 1 1 1 ⎞ = − + x 2 ⎜ − ⎟ − x 3 ⎜ − + ⎟ + x 4 ⎜ − − + − ⎟ + ... 2 ⎝3 8⎠ ⎝ 4 6 24 ⎠ ⎝ 5 18 8 12 64 ⎠ x 5 x 2 x 3 251 4 =− + − + x + ... 2 24 8 2880 1+ ex

Example 15: Expand

1 2

2e x

up to the term containing x2.

1

Solution:

1

⎛ 1 + ex ⎞ 2 ⎛ 1 − x 1 ⎞ 2 =⎜ e + ⎟ ⎜ x ⎟ 2⎠ ⎝2 ⎝ 2e ⎠ ⎞ ⎟+ ⎠

⎛ x2 1 = ⎜1 − x + − 2 4 ⎝

⎞2 ⎟ ⎠

⎡ ⎛ x x2 + = ⎢1 − ⎜ − ⎣ ⎝2 4

⎞⎤ 2 ⎟⎥ ⎠⎦

1 x 1 2 2 x 4 x 1 4 1

x2 4

Solution: e

=e

x 1

1⎤ 2 ⎥ 2⎦

1

1

...

x2 1 x2 8 8 4 3 2 x ... 32

Example 16: Prove that e x cos x = 1 + x + x cos x

1

⎡1 ⎛ x2 = ⎢ ⎜1 − x + − 2! ⎣2 ⎝

1 1 1 2 2 2!

x 2

x2 4

2

...

...

...

x 2 x 3 11 4 x 5 . − − x − 2 3 24 5

x2 x4 ... 2! 4!

2

⎞ ⎞ 1⎛ ⎞ ⎛ 1⎛ x3 x5 x3 x3 = 1 + ⎜ x − + − ... ⎟ + ⎜ x − + ... ⎟ + ⎜ x − + .... ⎟ 2! 4! 2! 2! ⎠ 3! ⎝ ⎠ 2! ⎝ ⎠ ⎝ 4

3

5

⎞ ⎞ 1⎛ 1⎛ x3 x3 + ⎜ x − − ... ⎟ + ⎜ x − − ... ⎟ 4! ⎝ 2! 2! ⎠ 5! ⎝ ⎠ 2 1 ⎞ x ⎛ 1 1⎞ ⎛ 1 1 ⎞ ⎛ 1 1 = 1 + x + + x3 ⎜ − + ⎟ + x 4 ⎜ − + ⎟ + x5 ⎜ − + ⎟ + ... 2 ⎝ 2 6⎠ ⎝ 2 24 ⎠ ⎝ 24 4 120 ⎠ = 1+ x +

x 2 x 3 11 4 x5 − − x − + ... 2 3 24 5

Differential Calculus I

2.103

⎛ 5x3 ex 2 + Example 17: Prove that e = e ⎜ 1 + x + x + 6 ⎝ ex

Solution: e = e

⎛ x 2 x3 + + ⎜⎜1 + x + 2! 3! ⎝

= ee

x+

⎞ ⎟⎟ ⎠

x 2 x3 + + 2 ! 3!

⎡ ⎛ x 2 x3 = e ⎢1 + ⎜ x + + + 2 ! 3! ⎢⎣ ⎝ ⎡ ⎛1 1⎞ = e ⎢1 + x + x 2 ⎜ + ⎟ + ⎝2 2⎠ ⎣ 5 ⎛ = e ⎜1 + x + x 2 + x3 + 6 ⎝ Example 18: Prove that (1 1

Solution:

1

(1 + x) x = e x =e =e

1 x) x

e

2

⎞ 1 ⎛ x2 + ⎟ + ⎜x + 2! ⎠ 2! ⎝

⎞ 1 3 ⎟ + (x + … ) + 3! ⎠

⎛1 1 1⎞ x3 ⎜ + + ⎟ + ⎝6 2 6⎠

⎤ ⎥ ⎦

⎤ ⎥ ⎥⎦

⎞ ⎟ ⎠ e x 2

11e 2 x 24

...... .

log (1+ x )

⎞ 1 ⎛ x 2 x3 ⎜ x − + −... ⎟⎟ x ⎜⎝ 2 3 ⎠ ⎛ x x2 ⎞ ⎜⎜1− + −... ⎟⎟ ⎝ 2 3 ⎠

= ee

⎛ x x 2 x3 x 4 ⎞ ⎜⎜ − + − + −... ⎟⎟ ⎝ 2 3 4 5 ⎠

⎡ ⎛ x x 2 x3 x 4 = e ⎢1 + ⎜ − + − + − 4 5 ⎢⎣ ⎝ 2 3 ⎡ x ⎛1 1⎞ = e ⎢1 − + x 2 ⎜ + ⎟ + ⎝3 8⎠ ⎣ 2 11e 2 e =e− x+ x + 2 24

Example 19: Prove that sin (e x

Solution:

⎞ ⎟. ⎠

1)

⎞ 1 ⎛ x x 2 x3 − + ⎟ + ⎜− + 4 ⎠ 2! ⎝ 2 3

⎤ ⎥ ⎦

x

x2 2

5 4 x 24

⎛ x 2 x3 x 4 sin (e x − 1) = sin ⎜ x + + + + 2 ! 3! 4 ! ⎝

⎞ ⎟ ⎠

....... .

2

⎞ ⎟ + ⎠

⎤ ⎥ ⎥⎦

Engineering Mathematics

2.104

⎛ x 2 x3 x 4 =⎜x + + + + 2 ! 3! 4 ! ⎝ x2 ⎛1 1⎞ =x+ + x3 ⎜ − ⎟ + 2 ⎝6 6⎠ =x+ Example 20: Expand x ex + 1 x2 x4 =1+ + – x 12 720 2 e –1

Solution:

x ex

⎞ 1 ⎛ x 2 x3 + + ⎟ − ⎜x + 2 ! 3! ⎠ 3! ⎝ ⎛ 1 1⎞ − ⎟+ x4 ⎜ ⎝ 24 4 ⎠

x2 5 4 − x + 2 24 up to x4 and hence, prove that

1 .

x x = 2 3 e − 1 ⎡⎛ x x x 4 x5 + ⎢ ⎜1 + x + + + 2 ! 3! 4 ! 5! ⎣⎝ x = 2 3 ⎛ x x x 4 x5 + + ⎜x+ + + 2 ! 3! 4 ! 5 ! ⎝ x

⎡ ⎛ x x 2 x3 x 4 = ⎢1 + ⎜ + + + + ⎣ ⎝ 2 ! 3! 4 ! 5 ! ⎛ x x 2 x3 x4 =1− ⎜ + + + + ⎝ 2 6 24 120

⎞ ⎤ + ... ⎟ − 1⎥ ⎠ ⎦ ⎞ ⎟ ⎠ −1

⎞⎤ ⎟⎥ ⎠⎦ ⎞ ⎛ x x 2 x3 + + ⎟+⎜ + ⎠ ⎝ 2 6 24

⎛ x x2 −⎜ + + ⎝2 6 x ⎛ 1 1⎞ ⎛ 1 1 1⎞ = 1 − + x 2 ⎜ − + ⎟ + x3 ⎜ − + − ⎟ 2 ⎝ 6 4⎠ ⎝ 24 6 8 ⎠ 1 1 ⎛ 1 + x4 ⎜ − + + ⎝ 120 36 24 2 4 x x x =1− + + x 3 ( 0) − + 2 12 720 x ex + 1 x ⎛ 2 ⎞ = ⎜1 + x ⎟ 2 ex − 1 2 ⎝ e − 1⎠ x x = + x 2 e −1

x x x2 x4 +1− + − + 2 2 12 720 x2 x4 =1+ − + 12 720 =

3

⎞ ⎟ + ⎠

⎞ ⎟ ⎠

2

3

⎞ ⎛x ⎟ +⎜ + ⎠ ⎝2



⎞ ⎟ ⎠

4

1 1⎞ + ⎟+ 8 16 ⎠

...(1)

[Using Eq. (1)]

Differential Calculus I

2.105

Example 21: Prove that tan

1

x sin 1 x cos

= x sin +

x2 x3 sin 2 + sin 3 + .... . 2 3

y = tan

Solution: Let

1

x sin 1 x cos

x sin 1 − x cos eiy − e − iy x sin = i (eiy + e − iy ) 1 − x cos tan y =

eiy − e − iy ix sin = iy − iy 1 − x cos e +e

Applying componendo dividendo, eiy 1 x (cos = e iy 1 x (cos e 2iy = 2iy

1 xe i 1 xei log (1 xe

i sin ) i sin )

i

) log (1 xei )

⎞ ⎛ ⎞ ⎛ x 2 e −2i x 3 e −3i x 2 e 2i x 3 e 3i = ⎜ − xe − i − − − ... ⎟ − ⎜ − xei − − − ... ⎟ 2 3 2 3 ⎠ ⎝ ⎠ ⎝ = x (e i − e − i ) + = x ⋅ 2i sin + y = x sin +

x 2 2i x3 (e − e −2i ) + (e3i − e −3i ) + ... 2 3

x2 x3 ⋅ 2i sin 2 + ⋅ 2i sin 3 + ... 2 3

x2 x3 sin 2 + sin 3 + ... 2 3

Example 22: Prove that e ax cos bx = 1 + ax + x cos cos( x sin ) = and hence, deduce e n= 0

(a 2

b 2 ) 2 a ( a 2 3b 2 ) 3 x + x + ... 2! 3!

xn cos n . n!

Solution: e cos bx = e . Real Part of (eibx) ax

ax

= R.P. of e(a+ib)x ⎡ (a 2 + ib) 2 2 (a + ib)3 3 = R.P. of ⎢1 + (a + ib) x + x + x + 2! 3! ⎣

⎤ ⎥ ⎦

Engineering Mathematics

2.106

⎡ ⎤ (a 2 − b 2 + 2aib) 2 (a 3 − ib3 + 3ia 2 b − 3ab 2 ) 3 = R.P ⎢1 + (a + ib) x + x + x + ...⎥ 2 ! 3 ! ⎣ ⎦ 2 2 2 2 (a − b ) 2 a (a − 3b ) 3 x + x + ... = 1 + ax + 2! 3! Putting a = cos a and b = sin a,

e x cos cos ( x sin ) = 1 + x cos +

− sin 2 ) 2 cos3 x + 2!

(cos 2

− 3 cos ⋅ sin 2 3!

x 3 + ...

− 3 cos (1 − cos 2 ) 3 x + ... 3!

= 1 + x cos +

cos 2 2 cos3 x + 2!

= 1 + x cos +

x2 x3 cos 2 + cos 3 + .... 2! 3!



xn cos n n=0 n!

=∑

Example 23: Prove that e x = 1 + tan x +

1 1 7 tan 2 x - tan 3 x - tan 4 x + ....... . 2! 3! 4!

Solution: Let e x = a0 + a1 tan x + a2 tan 2 x + a3 tan 3 x + a4 tan 4 x + ......... 2

3

...(1) 4

⎛ ⎞ ⎛ ⎞ ⎛ ⎞ ⎛ ⎞ x3 x3 x3 x3 = a0 + a1 ⎜ x + + ... ⎟ + a2 ⎜ x + + ... ⎟ + a3 ⎜ x + + ... ⎟ + a4 ⎜ x + + ... ⎟ + ........ 3 3 3 3 ⎝ ⎠ ⎝ ⎠ ⎝ ⎠ ⎝ ⎠ ⎛ ⎛ ⎞ ⎞ 2x4 x3 + ... ⎟ + a3 ( x 3 + ...) + a4 ( x 4 + ...) + .......... = a0 + a1 ⎜ x + + ... ⎟ + a2 ⎜ x 2 + 3 3 ⎝ ⎝ ⎠ ⎠ ⎛a ⎞ ⎛2 ⎞ = a0 + a1 x + a2 x 2 + ⎜ 1 + a3 ⎟ x 3 + ⎜ a2 + a4 ⎟ x 4 + .......... 3 3 ⎝ ⎠ ⎝ ⎠ ex = 1 + x +

But

x 2 x3 x 4 + + + ...... 2 ! 3! 4 !

...(2) ...(3)

Thus from Eqs (2) and (3) 1+ x +

x 2 x3 x 4 ⎛a ⎞ ⎛2 ⎞ + + + ..... = a0 + a1 x + a2 x 2 + ⎜ 1 + a3 ⎟ x 3 + ⎜ a2 + a4 ⎟ x 4 + ....... 2 ! 3! 4 ! 3 3 ⎝ ⎠ ⎝ ⎠ x, x2, x3 and x4 on both the sides, a0 = 1, a1 = 1, a2 =

1 1 a1 1 1 = , + a3 = = 2! 2 3 3! 6

Differential Calculus I

2.107

1 a1 1 1 1 1 − = − =− =− 6 3 6 3 6 3! 2 1 1 1 2 1 7 7 , a4 = − ⋅ =− =− a2 + a4 = = 3 4 ! 24 24 3 2 24 4! a3 =

Substituting in Eq. (1), e x = 1 + tan x +

1 7 1 tan 2 x − tan 3 x − tan 4 x + ... 2! 3! 4!

Example 24: Find the values of a and b such that the expansion of x (1 + ax ) in ascending powers of x begins with the term x4 and 1 + bx x4 prove that this term is 36 log (1 x )

Solution: Let f ( x)

log (1 x)

x(1 + ax) 1 + bx

x

x2 2

x3 3

x4 4

...

( x ax 2 )(1 bx)

x

x2 2

x3 3

x4 4

...

( x ax 2 )(1 bx b 2 x 2 b3 x3 ...)

x

x2 2

x3 3

x4 4

...

( x bx 2

1 b a x2 2

1 2 b 3

1

b 2 x 3 b3 x 4 1 3 b 4

ab x 3

ax 2

abx 3

ab 2 x 4

ab3 x 5 .......)

ab 2 x 4 ........

If the expansion begins with the term x4, the coefficients of x2 and x3 must be zero.

1 − + b − a = 0, 2

b =a+

1 2

and

1 − b 2 + ab = 0 3

Substituting b in Eq. (1), 2

1 ⎛ 1⎞ 1⎞ ⎛ −⎜a + ⎟ + a⎜a + ⎟ = 0 3 ⎝ 2⎠ 2⎠ ⎝ 1 1 1 − a2 − − a + a2 + a = 0 3 4 2 1 1 1 a= , a= 6 2 12 1 1 4 2 b= + = = 6 2 6 3

... (1)

Engineering Mathematics

2.108

3

2

1 1 ⎛2⎞ 1⎛2⎞ 1 x 4 = − + b3 − ab 2 = − + ⎜ ⎟ − ⎜ ⎟ = − 4 4 ⎝3⎠ 6⎝3⎠ 36 Hence, the expansion begins with the term

x4 . 36

Exercise 2.7 1. Expand ex sec x in powers of x using Maclaurin’s series. [Ans.: 1 + x + x + ...] 2

2. Using Maclaurin’s series, prove that x2 esin x = 1 + x + + … 2 3. Using Maclaurin’s series, prove that x2 a x = 1 + x log a + (log a ) 2 + … 2! 4. Prove that

10. Prove that sin (e x 1)

x

x2 2

5 4 x … 24

11. Prove that cos n x 1 n

x2 x4 n (3n 2) … 2! 4!

Hence, deduce that cos3 x 1

3x 2 2

15 x 4 … 48

12. Prove that sin 2 x

x4 3

x2

2 6 x … 45

5. Prove that sec x = 1 +

x2 5x4 + +… 2 24

1 ⎤ ⎡ −1 ⎢ Hint : sec x = cos x = (cos x) = ⎥ ⎥ ⎢ −1 ⎢ ⎡ ⎛ x2 x4 ⎞⎤ ⎥ ⎢ ⎢1 − ⎜ − + … ⎟ ⎥ ⎥ ⎠ ⎦ ⎥⎦ ⎣ ⎝ 2! 4! ⎣⎢ 6. Prove that x cosec x = 1 +

x2 7 x4 + +… 6 360

7. Prove that x3 … 3 x3 8. Prove that e x cos x 1 x … 3 9. Prove that e x sin 2 x

2x 2x2

cos x cos h x

22 x 4 1 4!

24 x8 … 8!

sinh 3 x = ∑

(3n − 3) − [1 − (− 1) n ] x n 8 ⋅ n!

13. Prove that e x sin x = 1 + x 2 +

x4 x6 + + ……… 3 120

14. Prove that (1 x) x

1 x2

x3 2

5x4 … 6

n (n − 1) 2 ⎤ ⎡ n ⎢ Hint : (1 + x) = 1 + nx + 2 ! x ⎥ ⎢ ⎥ n (n − 1) (n − 2) 3 ⎢ ⎥ + x + …⎥ ⎢ 3! ⎣ ⎦ 15. Prove that (1 + x)1+ x = 1 + x + x 2 +

x3 +… 3

(1.01)1.01 [Ans.: 1.0101]

Differential Calculus I 16. Prove that log x

2.109

23. Prove that 1 ( x 1) 2 2

( x 1)

1 ( x 1)3 … 3

17. Prove that

e x log (1 + x) = x +

x 2 x3 + +… 2 3

24. Prove that 2

log (1 x x 2 )

x 2

x

2x 3

3



log

xe x ex 1

x 2

x2 24

x4 … 2880

18. Prove that [log (1 x)]2

x2

x3

⎞ ⎛p 25. Expand log tan ⎜ + x⎟ upto x 5 . ⎠ ⎝4

11 4 x … 12

19. Prove that log cosh x =

1 2 1 4 1 6 x − x + x −… 2 12 45

20. Prove that log (1 tan x)

x

x2 2

⎡ ⎛p ⎞ ⎛ 1 + tan x ⎞ ⎢ Hint : log tan ⎜⎝ 4 + x⎟⎠ = log ⎜⎝ 1 − tan x ⎟⎠ ⎣

⎤ = log (1 + tan x) − log (1 − tan x) ⎥ ⎦

2 x3 ……… 3

4 3 4 5 ⎤ ⎡ ⎢⎣ Ans. : 2 x + 3 x + 3 x + …⎥⎦

21. Prove that sin x log x

x2 6

x4 180

x6 … 2835

22. Prove that x3 7 tan x = + x 4 + ……… log x 3 90

Example 1: Prove that log (sec x ) =

26. Prove that x = y + if y

x

x2 2

x3 3

y 2 y3 y 4 + + +… 2! 3! 4! x4 4



x2 x4 x6 + + + ........ . 2 12 45

Solution: Let y = log (sec x) dy 1 = ⋅ sec x tan x = tan x dx sec x x3 2 = x + + x 5 + ........ 3 15 Integrating Eq. (1),

x2 x4 2 x6 + + ⋅ + ........ 2 12 15 6 x2 x4 x6 log (sec x) = c + + + + ......... 2 12 45 y = c+

... (1)

Engineering Mathematics

2.110 Putting x = 0,

log (sec 0) = c + 0 c = log 1, c = 0 log (sec x) =

Hence,

Example 2: Prove that sin

x2 x4 x6 + + + ........ 2 12 45 1

x = x+

1 x 3 1.3 x 5 1.3.5 x 7 + + +… . 2 3 2.4 5 2.4.6 7

Solution: Let y = sin–1 x 1 − dy 1 = = (1 − x 2 ) 2 dx 1 − x2 ⎛ 1 ⎞⎛ 3 ⎞⎛ 5 ⎞ ⎛ 1 ⎞⎛ 3 ⎞ − − ⎜ − ⎟⎜ − ⎟⎜ − ⎟ 1 2 ⎜⎝ 2 ⎟⎠ ⎜⎝ 2 ⎟⎠ 2 ⎠⎝ 2 ⎠⎝ 2 ⎠ 2 2 = 1+ x + ( − x 2 )3 + … (− x ) + ⎝ 3! 2! 2

= 1+

x 2 1.3 4 1.3.5 6 + x +… x + 2 2.4 2.4.6 ... (1)

Integrating Eq. (1), 1 2 1 sin −1 x = c + x + 2 y = c+ x+

Putting x = 0,

x 3 1.3 x 5 1.3.5 + + 3 2.4 5 2.4.6 x 3 1.3 x 5 1.3.5 + + 3 2.4 5 2.4.6

x7 + ........ 7 x7 + ........ 7

sin 1 0 = c c=0 sin 1 x = x +

Hence,

–1 Example 3: Prove that cos x =

1 x 3 1.3 x 5 1.3.5 x 7 + + + ........ 2 3 2.4 5 2.4.6 7

⎛ 1 x 3 1.3 x 5 –⎜x + + + 2 ⎝ 2 3 2.4 5

Solution: Let y = cos–1 x dy 1 =− dx 1 − x2 Proceeding as in Ex. 2, we get ⎛ ⎞ 1 x 3 1.3 x 5 cos −1 x = c − ⎜ x + + +…⎟ 2 3 2 4 5 . ⎠ ⎝

⎞ ⎟. ⎠

Differential Calculus I

2.111

Putting x = 0, cos −1 0 = c p c= 2

⎛ 1 x 3 1.3 x 5 −⎜x + + + 2 ⎝ 2 3 2.4 5

cos −1 x =

Hence,

Example 4: Prove that tan

1

x

x3 3

x

x5 5

x7 7

⎞ ⎟ ⎠

........ .

Solution: Let y = tan–1 x dy 1 = = (1 + x 2 ) −1 = 1 − x 2 + x 4 − x 6 + … dx 1 + x 2 Integrating Eq. (1), x3 x5 x 7 y = c + x − + − +… 3 5 7 3 5 x x x7 tan −1 x = c + x − + − + ........ 3 5 7

... (1)

Putting x = 0, tan −1 0 = c c=0 Hence,

tan −1 x = x −

Example 5: Prove that sinh Solution: Let

1

x

x3 x5 x 7 + − +… 3 5 7 x3 6

x

(

3 x5 40

y = sin h −1 x = log x + x 2 + 1

........ .

)

⎛ dy 1 2x ⎞ = ⎜⎜1 + ⎟= 2 dx x + x + 1 ⎝ 2 x 2 + 1 ⎟⎠ = (1 + x 2 )



1 x2 + 1

1 2

⎛ 1⎞⎛ 3⎞ ⎜− ⎟ ⎜− ⎟ 1 2⎠⎝ 2⎠ 2 2 = 1 − x2 + ⎝ (x ) − … 2! 2 1 3 = 1 − x2 + x4 − … 2 8 Integrating Eq. (1),

x3 3 x5 + ⋅ −… 6 8 5 x3 3x5 sinh −1 x = c + x − + −… 6 40 y = c+ x−

... (1)

Engineering Mathematics

2.112

sinh −1 0 = c, c = 0

Putting x = 0,

sinh −1 x = x − Example 6: If x = 1 – Solution:

y2 y4 + 2! 4! x =1−

x3 3x5 + −… 6 40

y6 + ........, find y in a series of x. 6! y2 y4 y6 + − + …… 2! 4! 6!

= cos y y = cos –1 x Proceeding as in Ex. 3, we get y=

⎛ ⎞ x3 3x5 −⎜x + + + …⎟ 2 ⎝ 6 40 ⎠

⎛ x3 3 5 Example 1: Prove that sinh–1 (3x + 4x3) = 3 ⎜ x − x + + 6 40 ⎝ Solution: Let y = sinh 1 (3 x + 4 x 3 ) Putting x = sinh q ,

⎞ ⎟. ⎠

y = sinh −1 (3 sinh q + 4 sinh 3 q ) = sinh −1 (sinh 3q ) = 3q = 3 sinh −1 x ⎞ ⎛ x3 3x5 = 3⎜ x − + −…⎟ 6 40 ⎝ ⎠ ⎛ ⎛ 2x ⎞ x3 x5 x7 − − Example 2: Prove that sin − 1 ⎜ = 2 x + + ⎜ ⎟ 2 3 5 7 ⎝1 + x ⎠ ⎝ Solution: Let y = sin Putting

1

2x 1 + x2 x = tan ,

⎛ 2 tan q ⎞ y = sin −1 ⎜ ⎟ 2 ⎝ 1 + tan q ⎠ = sin −1 (sin 2q ) = 2q = 2 tan −1 x ⎞ ⎛ x3 x5 x 7 = 2 ⎜ x − + − +…⎟ 3 5 7 ⎝ ⎠

⎞ ⎟. ⎠

Differential Calculus II Chapter

3

3.1 INTRODUCTION In Chapter 2, we have studied a few topics of differential calculus such as successive differentiation, mean value theorems, expansion of functions and indeterminate forms. In this chapter, we will study tangents, normals, curvature and envelope of curves. Curve tracing is covered as the last topic of this chapter. Knowledge of curve tracing helps in application of integration in finding length, area, volume and surface area.

3.2 TANGENT AND NORMAL Let P(x, y) and Q(x + h, y + k) be the points on the curve y = f (x). As Q tends to P, the chord PQ tends to the straight line PQ which touches the curve at point P. This straight line is called the tangent to the curve at P. The perpendicular drawn to the tangent at P is called normal to the curve at that point. y ( y + k ) − y f ( x + h) − f ( x ) Slope of line PQ = = ( x + h) − x h when Q P, line PQ tends to the tangent at P. Q(x + h, y + k) f ( x + h) − f ( x ) Slope of tangent at P = lim Q→P h f ( x + h) − f ( x ) = lim P(x, y) h→ 0 h = f ′( x ) dy x = dx Fig. 3.1 Equation of the tangent to the curve at any point (x, y) is given by, Y − y = f ′( x )( X − x ) 1 Slope of normal at P = − f ′( x ) and equation of the normal to the curve at any point (x, y) is given by, 1 Y−y=− ( X − x) f ′( x ) where (X, Y ) is any arbitrary point on the tangent (or normal) to the curve.

3.2

Engineering Mathematics

3.2.1 Angle of Intersection of Curves The angle of intersection of two curves at a point of intersection is defined to be the angle between the tangents to the curves at that point. Let m1 and m2 be the slopes of the tangent to the curves y = f1(x) and y = f2(x) respectively at the point of intersection. Angle of intersection of two curves at the point of intersection is given by, m − m1 q = tan −1 2 . 1 + m2 m1

3.2.2 Length of Tangent, Sub-tangent, Normal and Sub-normal Let P(x, y) be any point on the curve y = f (x). The tangent and normal at the point P meet the x-axis at T and N respectively. Let PM be the ordinate. PT and PN are the lengths of the tangent and normal to the curve. TM and MN are the lengths of sub-tangent and sub-normal to the curve at the point P. Let be the angle which tangent makes with the x-axis. dy tany = dx y Length of tangent = PT = PM cosec y = y 1 + cot 2 y ⎛ dx ⎞ = y 1+ ⎜ ⎟ ⎝ dy ⎠

P(x, y)

2

Length of sub-tangent = TM = PM coty dx dy Length of normal = PN = PM secy

T

=y

M

N

Fig. 3.2

= y 1 + tan 2 y ⎛ dy ⎞ = y 1+ ⎜ ⎟ ⎝ dx ⎠

2

Length of sub-normal = MN = PM tany dy =y dx

3.2.3 Length of Perpendicular from the Origin to the Tangent Let p be the length of the perpendicular drawn from origin to the tangent.

p=

y − x f ′( x ) 1 + [ f ′ ( x)]

2

x

Differential Calculus II

The relation between distance of any point P(x, y) on the curve from the origin and the length of perpendicular from the origin to the tangent at that point is called pedal equation of the curve. Pedal equations can be obtained by eliminating x and y from equations y = f (x), y − x f ′( x ) p= and r2 = x2 + y2 1 + ⎡⎣ f ′( x ) 2 ⎤⎦

3.3

y

P(x, y)

r

x

O p m

Fig. 3.3

Example 1: Find the equations of the tangent and normal to the curve xy = c 2 at ⎛ c⎞ the point ⎜ ct , ⎟ . ⎝ t⎠ xy = c 2

Solution: Differentiating w.r.t. x, x

⎛ c⎞ At the point ⎜ ct , ⎟ , ⎝ t⎠

dy +y=0 dx dy y =− dx x c dy 1 =− t =− 2 dx ct t

⎛ c⎞ Slope of the tangent to the curve at ⎜ ct , ⎟ is ⎝ t⎠

1 and slope of the normal is t2. t2

⎛ c⎞ Equation of the tangent to the curve at ⎜ ct , ⎟ is given by, ⎝ t⎠ c 1 Y − = − 2 ( X − ct ) t t X + t 2Y = 2ct ⎛ c⎞ Equation of the normal to the curve at ⎜ ct , ⎟ is given by, ⎝ t⎠

c = t 2 ( X − ct ) t t 3 X − tY = c(t 4 − 1) . Y−

Example 2: Find the equations of the tangent and normal to the curve x2 y2 − = 1 at the point ( a sec q , b tan q ). a 2 b2 Solution:

x2 y2 − =1 a2 b2

3.4

Differentiating w.r.t. x,

Engineering Mathematics

2 x 2 y dy − =0 a 2 b 2 dx

dy b 2 x = dx a 2 y dy b 2 ( a secq ) b At the point ( a secq , b tan q ), = 2 = dx a (b tan q ) a sin q b Slope of the tangent to the curve at ( a secq , b tan q ) is and slope of the normal a sinq a is sin q . b Equation of the tangent to the curve at ( a secq , b tan q ) is given by, b Y − b tan q = ( X − a secq ) a sin q Y sin 2 q X sin q − = − secq b cos q a ⎛X⎞ ⎛Y ⎞ 2 2 ⎜⎝ ⎟⎠ secq − ⎜⎝ ⎟⎠ tan q = sec q − tan q = 1 a b Equation of the normal to the curve at ( a secq , b tan q ) is given by, a Y − b tan q = − sin q ( X − a secq ) b Y b X a − tan q = − sin q + tan q a a b b a b a2 + b2 ⎛X⎞ ⎛Y ⎞ ⎜⎝ ⎟⎠ cos q + ⎜⎝ ⎟⎠ cot q = + = b a b a ab . Example 3: Find the equations of the tangent and normal to the curve y = 2 x 2 - 4 x + 5 at the point (3, 11). Solution:

y = 2x2 − 4x + 5

Differentiating w.r.t. x,

dy = 4x − 4 dx dy At the point (3, 11), = 8. dx Slope of the tangent to the curve at (3, 11) is 8 and slope of the normal is Equation of the tangent to the curve at (3, 11) is given by, Y − 11 = 8 ( X − 3) 8 X − Y = 13 Equation of the normal to the curve at (3, 11) is given by, 1 Y − 11 = − ( X − 3) 8 X + 8Y = 91.

1 . 8

Differential Calculus II

3.5

Example 4: Find the equations of the tangent and normal to the curve x = sin t , p y = cos 2t at t = . 6 Solution: x = sin t dx = cos t dt y = cos 2t dy = −2 sin 2t dt dy dy / dt 2 sin 2t = =− = −4 sin t dx dx / dt cos t dy ⎛p ⎞ = −4 sin ⎜ ⎟ = −2 ⎝6⎠ dx 1 p Slope of the tangent to the curve at t = is 2 and slope of the normal is . 2 6 p 1 1 At t = , x = and y = . 6 2 2 p Equation of the tangent to the curve at t = is given by, 6 1 1⎞ ⎛ Y − = −2 ⎜ X − ⎟ ⎝ 2 2⎠ 4 X + 2Y = 3 p Equation of the normal to the curve at t = is given by, 6 1 1⎛ 1⎞ Y − = ⎜X − ⎟ 2 2⎝ 2⎠ 2 X − 4Y = −1 . At the point t =

p , 6

n

n

⎛x⎞ ⎛ y⎞ Example 5: Prove that the curve ⎜ ⎟ + ⎜ ⎟ = 2 touches the straight line ⎝a⎠ ⎝b⎠ x y + = 2 at the point (a, b), whatever be the value of n. a b n

Solution:

n

⎛x⎞ ⎛ y⎞ ⎜⎝ ⎟⎠ + ⎜⎝ ⎟⎠ = 2 a b

Differentiating w.r.t. x, n n −1 n n −1 dy =0 x + n y dx an b

dy b n x n −1 = − n n −1 dx a y At the point (a, b),

dy b n a n −1 b = − n n −1 = − dx a a b

3.6

Engineering Mathematics

Equation of the tangent to the curve at (a, b) is given by, b Y − b = − ( X − a) a bX + aY = 2ab X Y + =2 a b n

n

⎛x⎞ ⎛ y⎞ x y Hence, the curve ⎜ ⎟ + ⎜ ⎟ = 2 touches the straight line + = 2 at the point (a, b), ⎝a⎠ ⎝b⎠ a b whatever be the value of n. Example 6: Tangents are drawn from the origin to the curve y = sin x . Prove that their point of contact lies on x 2 y 2 = x 2 - y 2 . Solution: Differentiating w.r.t. x,

y = sin x

dy = cos x dx Equation of the tangent to the curve at the origin is given by, Y − 0 = cos x ( X − 0) Y = cos x X Let (x1, y1) be the point of contact of the curve and the tangent. y1 = sin x1 y1 and = cos x1 x1 Squaring and adding the equations, y2 y12 + 12 = 1 x1 x12 y12 = x12 − y12 Hence, the point of contact lies on x 2 y 2 = x 2 − y 2 . Example 7: Show that the line x cos q + y sin q = p will touch the curve m

m

m

⎛x⎞ ⎛ y⎞ m ⎜⎝ ⎟⎠ + ⎜⎝ ⎟⎠ = 1, provided ( a cos q ) a b

m 1

+ ( b sin q ) m

m

Solution:

m 1

= pm 1 .

m

⎛x⎞ ⎛ y⎞ ⎜⎝ ⎟⎠ + ⎜⎝ ⎟⎠ = 1 a b

Differentiating w.r.t. x, mx m −1 my m −1 dy + m =0 dx am b dy b m x m −1 = − m m −1 dx a y

Differential Calculus II

3.7

Equation of the tangent to the curve is given by, b m x m −1 ( X − x) a m y m −1 x m −1 y m −1 x m y m X m +Y m = m + m = 1 a b a b Let (x1, y1) be the point of contact of the curve and the tangent. sinq cosq x1 + y1 =1 p p Y−y=−

x1 Comparing two equations,

x m −1 y m −1 + y1 m = 1 m a b x m −1 cosq = p am 1

and

⎛ a m cosq ⎞ m −1 x=⎜ p ⎟⎠ ⎝ y m −1 sinq = p bm 1

⎛ b m sinq ⎞ m −1 y=⎜ p ⎟⎠ ⎝ m

m

⎛x⎞ ⎛ y⎞ The point (x, y) lies on the curve ⎜ ⎟ + ⎜ ⎟ = 1 ⎝a⎠ ⎝b⎠ m

m

1 ⎛ a m cos q ⎞ m −1 1 ⎛ b m sin q ⎞ m −1 + m⎜ =1 p ⎟⎠ p ⎟⎠ a m ⎜⎝ b ⎝ m

m

m

( a cos q ) m −1 + (b sin q ) m −1 = p m −1 Example 8: Prove that the sum of intercepts of the tangent to the curve x + y = a on the coordinate axes is constant. x+ y= a Solution: Differentiating w.r.t. x,

1 1 1 1 dy + =0 2 x 2 y dx dy y =− dx x

Equation of the tangent to the curve at (x, y) is given by, Y−y=−

y ( X − x) x

… (1)

3.8

Engineering Mathematics

X intercept is obtained by putting Y = 0 in Eq. (1), −y = −

y ( X − x) x

X − x = xy X = x + xy = x

(

)

x+ y = x a

Y intercept is obtained by putting X = 0 in Eq. (1), Y − y = xy Y = y + xy =

(

y

)

x+ y =

y a

Sum of intercepts = X + Y = x a + y a = a

(

)

x+ y = a

( a)

=a Hence, sum of intercepts of the tangent to the curve on the coordinate axes is constant. Example 9: Show that the length of the portion of the normal to the curve x = a (4cos 3q - 3cosq ), y = a (4sin 3q - 9sinq ) intercepted between the coordinate axes is constant. Solution:

x = a (4cos3q − 3cosq ) dx = a ( −12 cos 2 q sin q + 3 sin q ) dq y = a ( 4 sin 3 q − 9 sin q ) dy = a (12 sin 2 q cos q − 9 cos q ) dq dy dy / d q a (12 sin 2 q cos q − 9 cos q ) = = dx dx / d q a ( −12 cos 2 q sin q + 3 sin q )

3 cos q ( 4 sin 2 q − 3) −3 sin q ( 4 cos 2 q − 1) cos q [2 (1 − cos 2q ) − 3] = − sin q [2 (1 + cos 2q ) − 1] cos q ( 2 cos 2q + 1) = sin q ( 2 cos 2q + 1) = cot q sin q Slope of the normal to the curve = − tan q = − cos q Equation of the normal to the curve is given by, sin q Y − a ( 4 sin 3 q − 9 sin q ) = − [ X − a ( 4 cos3 q − 3 cos q )] cos q =

… (1)

Differential Calculus II

3.9

Y intercept is obtained by putting X = 0 in Eq. (1), sin q Y − a ( 4 sin 3 q − 9 sin q ) = a ( 4 cos3 q − 3 cos q ) cos q Y = a sin q ( 4 cos 2 q − 3 + 4 sin 2 q − 9) = −8a sinq X intercept is obtained by putting Y = 0 in Eq. (1), sin q − a ( 4 sin 3 q − 9 sin q ) = − [ X − a ( 4 cos3 q − 3 cos q )] cos q X = a cos q ( 4 sin 2 q − 9 + 4 cos 2 q − 3) = −8a cos q Length of the portion of the normal to the curve intercepted between co-ordinate axes = X 2 + Y 2 = ( −8a sin q ) 2 + ( −8a cos q ) 2 = 8a

= constant Example 10: Find the angle of intersection of the curves y 2 = 4ax and x 2 = 4by at their point of intersection other than origin. Solution: The points of intersection of the curves are obtained as, y 2 = 4 ax

⎡⎣∵ x 2 = 4by ⎤⎦

= 4 a ⋅ 2 by ⎛ 3 ⎞ y ⎜ y 2 − 8a b ⎟ = 0 ⎝ ⎠ 3

y = 0 and y 2 − 8a b = 0 2

2

1

1 3 ⎛ ⎞ 3 3 y = 0 and y = ⎜ 8ab 2 ⎟ = 4 a b ⎝ ⎠

When y = 0, x = 0. 1

1 2 ⎛ 2 1 ⎞2 When y = 4 a b , x = 2 by = 2b ⎜ 4 a 3 b 3 ⎟ = 4 a 3 b 3 ⎠ ⎝ 2 3

1 3

1 2

2 1 ⎞ ⎛ 1 2 Hence, (0, 0) and ⎜ 4 a 3 b 3 , 4 a 3 b 3 ⎟ are the two points of intersection. ⎠ ⎝ 2 For the curve y = 4 ax,

Differentiating w.r.t. x, 2y

dy = 4a dx dy 2a = y dx

3.10

Engineering Mathematics

1

dy a 3 = 1 dx 2b 3

⎞ ⎛ At the point ⎜ 4 a b , 4 a b ⎟ , ⎠ ⎝ 1 3

2 3

2 3

1 3

For the curve x 2 = 4by, Differentiating w.r.t. x, dy dx

2 x = 4b x dy = dx 2b

1

2 1 ⎞ ⎛ 1 2 At the point ⎜ 4 a 3 b 3 , 4 a 3 b 3 ⎟ , ⎠ ⎝

dy 2a 3 = 1 dx b3 If m1 and m2 be the slopes of the tangents to the curves, then m1 =

1

1

a3

2a 3

2b Angle of intersection = tan

1

1 3

and m2 =

1

b3

m2 m1 1 + m2 m1

= tan

−1

1

1

2a 3

a3

b

1 3



1

2b 3 1

1

2a 3 a 3 1+ 1 . 1 b 3 2b 3 1

= tan

−1

1

3a 3 b 3 2 ⎞ ⎛ 2 2 ⎜a3 + b3 ⎟ ⎠ ⎝

.

Example 11: Show that the condition that the curves ax 2 + by 2 = 1 and 1 1 1 1 a x 2 + b y 2 = 1 should intersect orthogonally is - = - . a b a b Solution: Let P (x1, y1) be the point of intersection of the curves. Hence, ax12 + by12 = 1 and a x12 + b y12 = 1 Solving these two equations, b′ − b … (1) x12 = ab ′ − a ′b a − a′ y12 = … (2) ab ′ − a ′ b

Differential Calculus II

3.11

For the curve ax 2 + by 2 = 1, Differentiating w.r.t. x, 2ax + 2by

At the point of intersection (x1, y1),

dy =0 dx dy ax dx by dy dx

ax1 by1

dy dx

ax by

dy dx

a x1 b y1

For the curve a x 2 + b y 2 = 1,

At the point of intersection (x1, y1),

Since the two curves intersect orthogonally, ⎛ ax1 ⎞ ⎛ a ′x1 ⎞ ⎜⎝ − by ⎟⎠ ⎜⎝ − b ′y ⎟⎠ = −1 1

1

aa x12 + bb y22 = 0 aa ′(b ′ − b) bb ′( a − a ′ ) + =0 ab ′ − a ′b ab ′ − a ′b b b a a + =0 bb aa 1 a

1 b

1 a

[Using Eqs (1) and (2)]

1 . b

Example 12: Show that the curves x 2 = ay and y 2 = 2ax intersect upon the curve x 3 + y 3 = 3axy and find the angle between each pair at the point of intersection. Solution: The points of intersection of the curves x 2 = ay and y 2 = 2ax are obtained as, 1

3

1

x 2 = ay = a 2ax = 2 2 a 2 x 2 1 1 3 ⎛ 3 ⎞ x 2 ⎜ x 2 − 22 a2 ⎟ = 0 ⎝ ⎠ 1

3

1

3

x 2 = 0 and x 2 − 2 2 a 2 = 0

3.12

Engineering Mathematics

2

1 ⎛ 1 3 ⎞3 x = 0 and x = ⎜ 2 2 a 2 ⎟ = 2 3 a ⎝ ⎠

When x = 0, y = 0 1

When x = 2 3 a,

2

2ax = 2 3 a

2 ⎞ ⎛ 1 Hence, (0, 0) and ⎜ 2 3 a, 2 3 a ⎟ are the two points of intersection. On substituting, these ⎠ ⎝ points satisfy the equation of the curve x 3 + y 3 = 3axy , and hence they lie on this curve.

For the curve x 2 = ay , dy 2 x = dx a dy = 0 which indicates that tangent at the origin is x-axis. dx 2 4 ⎞ ⎛ 1 dy At the point ⎜ 2 3 a, 2 3 a ⎟ , = 23 dx ⎠ ⎝

At the point (0, 0),

For the curve y 2 = 2ax, 2y

At the point (0, 0),

dy dx

dy = 2a dx dy a = dx y

which indicates that tangent at the origin is y-axis.

2 ⎞ ⎛ 1 dy At the point ⎜ 2 3 a, 2 3 a ⎟ , =2 dx ⎠ ⎝ Further, for the curve x 3 + y 3 = 3axy,

3x 2 + 3 y 2

2 3

dy dy = 3ay + 3ax dx dx 2 dy ay x = dx y 2 ax

2 2 ⎞ dy ⎞ ⎛ 1 ⎛ 1 = 0 which indicates that tangent at the point ⎜ 2 3 a, 2 3 a ⎟ At the point ⎜ 2 3 a, 2 3 a ⎟ , d x ⎠ ⎠ ⎝ ⎝ 2 ⎞ ⎛ 13 to the curve is parallel to x-axis. Hence, at the point ⎜ 2 a, 2 3 a ⎟ , angle between the ⎠ ⎝ 4 2 3 3 −1 ⎛ 3 ⎞ curves x = ay and x + y = 3axy is tan ⎜ 2 ⎟ and angle between the curves y 2 = 2ax ⎝ ⎠

⎛ −2 ⎞ and x 3 + y 3 = 3axy is tan −1 ⎜ 2 3 ⎟ . ⎠ ⎝

Differential Calculus II

3.13

Example 13: Find the lengths of the tangent, sub-tangent, normal and suba3 ⎛ a⎞ normal to the curve y = 2 at a , . a + x 2 ⎜⎝ 2 ⎟⎠ Solution:

y=

a3 a2 + x2

Differentiating w.r.t. x, dy 2a 3 x =− 2 dx (a + x 2 )2 a At the point ⎛⎜ a, ⎞⎟ , ⎝ 2⎠

dy 2a3 ⋅ a 1 =− 2 =− dx 2 (a + a2 )2 2

⎛ dx ⎞ a 5 ⎛ a⎞ Length of the tangent at ⎜ a, ⎟ = y 1 + ⎜ ⎟ = 1 + ( −2) 2 = a ⎝ 2⎠ ⎝ dy ⎠ 2 2 dx a ⎛ a⎞ Length of the sub-tangent at ⎜ a, ⎟ = y = ( −2) = − a ⎝ 2⎠ dy 2 2

2

a 5 ⎛ dy ⎞ ⎛ 1⎞ ⎛ a⎞ Length of the normal at ⎜ a, ⎟ = y 1 + ⎜ ⎟ = 1+ ⎜− ⎟ = a ⎝ dx ⎠ ⎝ ⎠ ⎝ 2⎠ 2 2 4 ⎛ a⎞ dy a ⎛ 1 ⎞ a Length of the sub-normal at ⎜ a, ⎟ = y = ⎜− ⎟ = − . ⎝ 2⎠ dx 2 ⎝ 2 ⎠ 4 Note: Length cannot be negative, therefore depending upon the value of a, consider always a positive value of length. Example 14: Find the lengths of the tangent, sub-tangent, normal and subp normal to the curve x = a (q - sinq ), y = a (1 - cosq ) atq = . 2 Solution:

x = a (q − sin q ) dx = a (1 − cos q ) dq y = a (1 − cos q ) dy = a sin q dq a sin q dy dy / dq = = = dx dx / dq a (1 − cos q )

Atq =

dy = 1 and y = a. 2 dx ,

q q cos 2 2 = cot q q 2 2 sin 2 2

2 sin

3.14

Engineering Mathematics

2

⎛ dx ⎞ Hence, length of the tangent = y 1 + ⎜ ⎟ = a 1 + 1 = 2 a ⎝ dy ⎠ dx =a dy

Length of the sub-tangent = y

dy dx

Length of the normal = y 1 + Length of the sub-normal = y

2

= a 1+1 = 2 a

dy =a. dx

Example 15: Prove that the sum of the length of the tangent and sub-tangent at any point of the curve y = a log ( x 2 - a 2 ) varies as the product of the coordinates of the point. Solution:

y = a log ( x 2 − a 2 )

Differentiating w.r.t. x, dy dx

a

1 x2

a2

2x

2ax x2 a2 2

2

⎛ x2 − a2 ⎞ ⎛ dx ⎞ ( x2 + a2 ) Length of tangent = y 1 + ⎜ ⎟ = y 1 + ⎜ ⎟ =y 2ax ⎝ dy ⎠ ⎝ 2ax ⎠ Length of sub-tangent = y

dx ( x2 − a2 ) =y 2ax dy

2x2 1 ( x 2 + a2 ) ( x 2 − a2 ) +y =y = xy 2ax 2ax 2ax 2 Hence, sum of the length of the tangent and sub-tangent varies as the product of the coordinates of the point. Sum of the length of tangent and sub-tangent = y

Example 16: Prove that the length of the sub-normal at any point of the curve x 2 y 2 = a 2 ( x 2 - a 2 ) varies inversely as the cube of its abscissa. Solution:

x 2 y 2 = a2 ( x 2 − a2 )

Differentiating w.r.t. x, 2 xy 2 + 2 x 2 y

dy = 2a 2 x dx dy 2a 2 x 2 xy 2 a 2 y 2 = = xy dx 2x2 y

… (1)

Differential Calculus II

Length of sub-normal = y

3.15

dy a 2 − y 2 a 4 1 = = 2⋅ dx x x x

[Using Eq. (1)]

a4 x3 Hence, the length of the sub-normal varies inversely as the cube of its abscissa. =

Exercise 3.1 1. Find the equations of the tangent and normal to the following curves: (i) y 2 = 4ax at (a, −2a ) x2 y 2 + = 1 at (a cos q , b sin q ) a 2 b2 a (iii) ( x 2 + y 2 ) x − ay 2 = 0 at x = 2 2 2 (iv) x ( x − y ) + a ( x + y ) = 0 at (0, 0) (ii)

x (v) y = a cosh a (vi) x = 2a cos q − a cos 2q , y = 2a sin q − a sin 2q at q =

p 2

2at 3 1 2at 2 ,y= at t = 2 2 2 1+ t 1+ t (viii) x = a (q + sin q ), y = a (1 + cos q ) (vii) x =

π . 2 ⎡ Ans.: (i) x + y + a = 0, x − y = 3a ⎤ ⎢ ⎥ x y ⎢ ⎥ (ii) cos q + sin q = 1, ⎢ ⎥ a b ⎢ ⎥ ax by ⎢ ⎥ − = a 2 − b2 ⎢ ⎥ cos q sin q ⎢ ⎥ (iii) 4 x ± 2 y − a = 0, ⎢ ⎥ ⎢ ⎥ 2 x ± 4 y = 3a ⎢ ⎥ (iv) x + y = 0, x − y = 0 ⎢ ⎥ ⎢ ⎥ x (v) y − y1 = sinh ( x − x1 ) , ⎥ ⎢ a ⎢ ⎥ ⎢ ⎥ x x − x1 + ( y − y1 ) siinh = 0 ⎥ ⎢ a ⎢ ⎥ at θ =

⎢ ⎢ ⎢ ⎢ ⎢ ⎢ ⎢ ⎢ ⎢ ⎢ ⎢ ⎣⎢⎢

(vi) x + y = 3a, x − y + a = 0 (vii) 13 x − 16 y = 2a, 16 x + 13 y = 9a a (viii) x + y − p − 2a = 0 2 a x− y− p =0 2

2. Prove that

⎥ ⎥ ⎥ ⎥ ⎥ ⎥ ⎥ ⎥ ⎥ ⎥ ⎥ ⎥⎥⎦

x y + = 1 touches the curve a b

x

y = be a at the point where the curve crosses the y-axis. 3. Prove that all the points of the curve x y 2 = 4a x + a sin at which the a tangent is parallel to the x-axis lie on a parabola. 4. Show that the tangents to the curve x 3 + y 3 = 3axy at the points where it meets the parabola y 2 = ax are parallel to the y-axis. 5. In the curve x m y n = a m + n , prove that the portion of the tangent intercepted between the axes, is divided at its point of contact into segments which are in a constant ratio. x 6. In the curve y = a cosh , prove a that the length of the portion of the normal intercepted between the curve y2 . and the x-axis is a 7. Show that the tangent and normal at any point of the curve x = aeq (sin cos ),

3.16

Engineering Mathematics

y = aeq (sin + cos ) are equidistant from the origin. 8. Show that the distance from the origin of the normal at any point of the curve

q q⎞ ⎛ y = aeq ⎜ cos − 2 sin ⎟ is twice the ⎝ 2 2⎠ distance of the tangent at the point from the origin. 9. Find the angle of intersection of the following curves:

(ii) x 2 = 4ay and 2 y 2 = ax (iii) xy = a 2 and x 2 + y 2 = 2a 2 (iv) x − y = a and x + y = a 2

2

2

2

2

(v) y = sin x and y = cos x (vi) y 2 = ax and x 3 + y 3 = 3ax. ⎡ p −1 ⎛ 1 ⎞ ⎤ ⎢ Ans. : (i) 2 , tan ⎜ 2 ⎟ ⎥ ⎝ ⎠⎥ ⎢ ⎢ p ⎛ 3 ⎞⎥ ⎢ (ii) , tan −1 ⎜ ⎟ ⎥ 2 ⎢ ⎝ 5 ⎠⎥ ⎢ ⎥ (iii) 0 ⎢ ⎥ ⎢ ⎥ p ⎢ ⎥ (iv) ⎢ ⎥ 4 ⎢ ⎥ −1 ( v) tan 2 2 ⎢ ⎥ ⎢ ⎥ −1 3 ( vi) tan 16 ⎥⎦ ⎢⎣ 10. Show that the curves

x2 y 2 + =1 a 2 b2

x2 y2 + = 1 intersect a2 + b2 + orthogonally for all values of . 11. Prove that the curves x2 + 2xy y2 + 2ax = 0 and 3y3 2a2x 4a2 y + a3 = 0 and

9 at the 8

point ( a, a).

[ Ans. : n = −2] 13. Prove that for the curve x m + n = cy 2 n , the m th power of the sub-tangent varies as the n th power of the subnormal. 14. Show that for the curve b y2 = (x + a)3, the square of the sub-tangent varies as the sub-normal.

(i) 2 y 2 = x 3 and y 2 = 32 x

2

1

12. Find the value of n so that the subnormal at any point of the curve xy n = c n +1 is of constant length.

q⎞ ⎛ q x = aeq ⎜ sin + 2 cos ⎟ , ⎝ 2 2⎠

2

intersect at an angle tan

15. Find the lengths of the normal and sub-normal of the curve x x − 1 ⎛ a a ⎞ y = a ⎜e + e ⎟ . 2 ⎝ ⎠ ⎡ ⎛ 2x ⎞⎤ 2 ⎛x⎞ 1 ⎢ Ans. : a cosh ⎜⎝ a ⎟⎠ , 2 a sinh ⎜⎝ a ⎟⎠ ⎥ ⎦ ⎣ 16. Find the sub-tangent, sub-normal, normal and tangent at the point t on the curve x = a (t + sin t), y = a (1 – cos t). 1 ⎡ ⎤ 3 1 ⎢ Ans. : a sin t , 2a sin 2 t sec 2 t , ⎥ ⎢ ⎥ 1 1 1 ⎢ 2a sin t tan t , 2a sin t .⎥ ⎢⎣ 2 2 2 ⎥⎦ 17. Show that the sub-tangent at any point of the curve x m y n = a m + n varies as the abscissa. 18. Prove that the length of the subtangent at any point on the hyperbola xy = c 2 is numerically half the intercept made by the tangent at that point on the x-axis.

Differential Calculus II

3.17

3.2.4 Angle between Radius Vector and Tangent Let P ( r ,q ) be any point on the curve r = f (q ). Polar co-ordinates can be transformed to rectangular co-ordinates by the relation

y

x = r cos q = f (q ) cos q y = r sin q = f (q ) sin q

P(r, q ) r f

Let PT be the tangent to the curve at the point P ( r ,q ). Lety be the angle between tangent PT and positive x-axis. Let be the angle between the radius vector OP and tangent PT. dy dy / dq Slope of the tangent = tany = = dx dx / dq f ′(q ) sin q + f (q ) cos q = f ′(q ) cos q − f (q ) sin q

y

q O

x

T

Fig. 3.4

Dividing the numerator and denominator by f ′ (q ) cos q ,

tany = From Fig. 3.4,

tan q + f (q )/ f ′(q ) 1 − [ f (q )/ f ′(q ) ] tan q

… (1)

y = q +f tany = tan (q + f ) =

tan q + tan f 1 − tan f tan q

… (2)

From Eqs (1) and (2), f (q ) dq =r f ′(q ) dr Corollary: Angle of intersection of curves: If 1 and 2 are the angles between the common radius vector and the tangents to the two curves at the point of intersection, then the angle of intersection of two curves is given by 1 2 . tan f =

3.2.5 Length of Polar Tangent, Polar Sub-tangent, Polar Normal and Polar Sub-normal Let P ( r ,q ) be any point on the curve r = f (q ). Let PT and PN be the tangent and normal to the curve at the point P. Let NT be a straight line through the pole O and perpendicular to the radius vector OP. Then OT and ON are known as the polar sub-tangent and polar sub-normal respectively. Length of the polar tangent = PT = OP sec f = r 1 + tan 2 f ⎛ dq ⎞ = r 1+ r2 ⎜ ⎝ dr ⎟⎠

2

3.18

Engineering Mathematics

Length of the polar sub-tangent y

= OT = OP tanf dq dr d q = r2 dr

N

= r⋅r

f

P(r, q )

Length of the polar normal

f

r

= PN = OP cosecf = r 1 + cot 2 f = r 1+

O

1 ⎛ dr ⎞ ⎜ ⎟ r 2 ⎝ dq ⎠

⎛ dr ⎞ = r2 + ⎜ ⎝ dq ⎟⎠

2

p f

M

x

T

2

Length of the polar sub-normal

Fig. 3.5

= ON = OP cotf 1 dr r dq dr = dq = r⋅

3.2.6 Length of Perpendicular from Pole to the Tangent Let p be the length of the perpendicular OM drawn from pole to the tangent PT. From triangle OPM, p = r sin f 1 1 1 1 cosec 2f = 2 (1 + cot 2 f ) = = p 2 r 2 sin 2 f r 2 r ⎡ 1 ⎢1 + 2 ⎢⎣ r

2 ⎛ dr ⎞ ⎤ ⎜⎝ ⎟⎠ ⎥ dq ⎥⎦

=

1 r2

=

1 1 ⎛ dr ⎞ + ⎜ ⎟ r 2 r 4 ⎝ dq ⎠

2

Pedal Equations An equation connecting p and r is called pedal equation or p-r equation. The pedal equation can be obtained by eliminating between polar equation r = f (q ) and equation

1 1 1 dr = 2+ 4 2 p r r d

2

.

Differential Calculus II

3.19

Pedal equation can also be obtained by eliminating and from equations dq and p = r sin . dr 2 2a q Example 1: For the curve r = , prove that (i) f = p - (ii) p = ar . 1 cosq 2 r = f (q ), tan f = r

Solution: (i)

r=

2a 1 cos

=

2a 2sin 2

= a cosec 2

2

2

Differentiating w.r.t. , dr q⎛ q q⎞ 1 = a ⋅ 2 cosec ⎜ −cosec cot ⎟ ⋅ dq 2⎝ 2 2⎠ 2 q q q = −a cosec 2 cot = − r cot 2 2 2 q q⎞ r dq ⎛ tan f = r = = − tan = tan ⎜ p − ⎟ q 2 2⎠ dr ⎝ −r cot 2 Hence, (ii)

2 q⎞ q ⎛ p = r sin f = r sin ⎜ p − ⎟ = r sin 2⎠ 2 ⎝ a q p 2 = r 2 sin 2 = r 2 ⋅ = ar r 2

Example 2: For the curve r = a sin np , prove that

⎛1 ⎞ (i) f = tan -1 ⎜ tan nq ⎟ ⎝n ⎠ Solution: (i)

(ii) p 2 =

r4

(

r = a sin n

Differentiating w.r.t. , dr = na cos n d dθ a sin nθ 1 = = tan nθ dr na cos nθ n ⎛1 ⎞ φ = tan −1 ⎜ tan nθ ⎟ ⎝n ⎠

tan φ = r

)

n2 a 2 - n2 - 1 r 2

.

3.20

Engineering Mathematics

(ii) We know that, 2

1 1 1 ⎛ dr ⎞ = 2+ 4⎜ ⎟ 2 p r r ⎝ dq ⎠ 1 1 1 = + ( na cos nq ) 2 p2 r 2 r 4 =

1 n2 a 2 + 4 (1 − siin 2 nq ) r2 r

1 n2 a 2 ⎛ r 2 ⎞ 1 n2 2 2 + 4 ⎜1 − 2 ⎟ = 2 + 4 a − r r2 r ⎝ a ⎠ r r 2 2 2 2 2 r +n a −n r = r4 r4 . p2 = 2 2 n a − ( n2 − 1)r 2

(

=

)

Example 3: For the curve r 2 = b 2 sec2q , prove that (i) y =

p -q 2

Solution: (i)

(ii) pr = b 2 . r 2 = b 2 sec 2

Differentiating w.r.t. , dr 2r = 2b 2 sec 2q tan 2q = 2r 2 tan 2q dq dr = r tan 2q dq dq r ⎛p ⎞ = cot 2q = tan ⎜ − 2q ⎟ tan f = r = dr r tan 2q 2 ⎝ ⎠ p f = − 2q 2 ⎛p ⎞ p y =q + f = q + ⎜ − 2q ⎟ = − q Now ⎝2 ⎠ 2 b2 b2 ⎛p ⎞ p = r sin f = r sin ⎜ − 2q ⎟ = r cos 2q = r 2 = r r ⎝2 ⎠ 2 pr = b .

(ii) Hence,

Example 4: For the parabola (i) f =

p q 2 2

(ii) p =

1 = 1 + cosq , show that r

1 q sec . 2 2

Differential Calculus II

3.21

1 = 1 + cos r

Solution: (i) Differentiating w.r.t. , −

1 dr = − sin r2 d dr = r 2 sin d d r tan = r = 2 dr r sin cot

2

2

tan

1 = r sin

2

1 + cos = sin

2 cos 2 = 2sin

2

2

cos

2

2

2

q cos 1 p q ⎛ ⎞ 2 = 1 sec q . sin ⎜ − ⎟ = (ii) p = r sin f = 1 + cos q 2 ⎝ 2 2 ⎠ 2 cos 2 q 2 2 Example 5: Show that the curves r m = a m cos mq , r m = a m sin mq cut each other orthogonally. Solution: For the curve r m = a m cos m , Taking logarithm on both the sides, m log r = m log a + log cos m Differentiating w.r.t. , 1 dr 1 ( − m sin mq ) = r dq cos mq dr = − r tan mq dq dq r ⎛p ⎞ tan f 1 = r = = − cot mq = tan ⎜ + mq ⎟ ⎝ ⎠ 2 dr − r tan mq p f 1 = + mq 2

m⋅

For the curve r m = a m sin m , Taking logarithm on both the sides, m log r = m log a + log sin m

3.22

Engineering Mathematics

Differentiating w.r.t. , 1 dr 1 ( m cos mq ) = r dq sin mq dr = r cot mq dq dq r tan f 2 = r = = tan mq dr r cot mq f 2 = mq

m⋅

Angle between the curves

m m 2 Hence, the curves cut each other orthogonally. 1

2

2

Example 6: Find the angle of intersection of the curves r = sin q + cos q , and r = 2sinq . Solution: The point of intersection is obtained as, sin + cos = 2sin cos = sin tan = 1 =

4

For the curve r = sin q + cos q ,

dr = cos − sin d tan p⎞ ⎛ At the point of intersection ⎜q = ⎟ , 4⎠ ⎝

1

d sin + cos = dr cos sin

=r

tanf 1= ∞ 1

=

2

For the curve r = 2 sinq , dr = 2 cos q dq dq 2 sin q tan f 2 = r = = tan q dr 2 cos q f 2 =q p⎞ ⎛ At the point of intersection ⎜q = ⎟ , 4⎠ ⎝

f 2=

p 4

Angle of intersection of curves = f 1− f 2 =

p p p − = 2 4 4

Differential Calculus II

3.23

Example 7: Show that the two curves r 2 = a 2 cos 2q and r = a (1 + cosq ) intersect 1

⎛ 3⎞ 4 at an angle 3sin ⎜ ⎟ . ⎝ 4⎠ Solution: The point of intersection is obtained as, -1

a 2 cos 2q = a 2 (1 + cos q ) 2 2 cos 2 q − 1 = 1 + 2 cos q + cos 2 q cos 2 q − 2 cos q − 2 = 0 cos q = 1 ± 3 But, –1 < cos q < 1 Hence,

cos 1 2sin 2 sin 2

sin

2 2

2

2

=

=

1

3

1

3 3 2 3 2

= sin

1

1 2

3 4

3 = 4

1 4

1 4

For the curve r 2 = a 2 cos 2q , dr = −2a 2 sin 2q dq dr a 2 sin 2q =− dq r r2 a 2 cos 2q dq ⎛p ⎞ = − cot 2q = tan ⎜ + 2q ⎟ tan f 1 = r = 2 = 2 dr −a sin 2q −a sin 2q ⎝2 ⎠ p f 1 = + 2q 2 For the curve r = a (1 + cos q ), 2r

dr = −a sin q dq dq a (1 + cos q ) q ⎛p q ⎞ = tan f 2 = r = − cot = tan ⎜ + ⎟ dr −a sin q 2 ⎝ 2 2⎠ p q f2= + 2 2

3.24

Engineering Mathematics

1

4 Hence, angle of intersection of curves = f 1− f 2 = p + 2q − p − q = 3q = 3 sin −1 ⎛⎜ 3 ⎞⎟ . ⎝4⎠ 2 2 2 2

Example 8: Show that the curves r 2 cos (2θ - α ) = a 2 sin 2α and r2 = 2a2 sin (2p + `) cut at right angles at their points of intersection. Solution: The points of intersection are obtained as, a 2 sin 2a = 2a 2 sin ( 2q + a ) cos ( 2q − a )

sin 2a = 2 sin( 2q + a ) cos( 2q − a ) = sin 4q + sin 2a sin 4q = 0 4q = 0, p , 2p , …… … For the curve r 2 cos ( 2q − a ) = a 2 sin 2a , Differentiating w.r.t. , 2r

dr cos ( 2q − a ) − 2r 2 sin ( 2q − a ) = 0 dq

dr = r tan ( 2q − a ) dq dq r tan f 1 = r = = cot ( 2q − a ) dr r tan( 2q − a ) 1

2

(2

tan

2

(2

)

)

For the curve r 2 = 2a 2 sin( 2q + a ), Differentiating w.r.t. , 2r

tan f 2 = r

dr = 4 a 2 cos ( 2q + a ) dq dr 2a 2 = cos ( 2q + a ) dq r

sin ( 2q + a ) dq r2 = 2 = = tan ( 2q + a ) dr 2a cos ( 2q + a ) cos ( 2q + a ) f 2 = 2q + a

Angle of intersection 2 At the points of intersection,

1

(2

)

2

(2

)

4

2

p p 3p , , , …… 2 2 2 Hence, the curves cut at right angles at their points of intersection. f 2 − f 1= −

Differential Calculus II

3.25

Example 9: Find the length of polar sub-tangent and polar sub-normal for the 2a curve = 1 - cosq . r 2a r

Solution:

1 cos

Differentiating w.r.t. , −

Length of the polar sub-tangent = r 2

dq = dr

2a dr = sin q r 2 dq dr r 2 sin q =− dq 2a

2a r2 = −2a cosec q =− sin q r sin q − 2a 2

Length of the polar sub-normal dr r 2 sin q 4a 2 sin q = =− =− =− 2 dq 2a (1 − cos q ) 2a

q q cos 2 2 = −a cot q cosec 2 q . q 2 2 4 sin 4 2

2a ⋅ 2 sin

Example 10: Find the length of polar tangent, polar sub-tangent, polar normal and polar sub-normal to the curve r 2 = a 2 sin 2q . r 2 = a 2 sin 2

Solution: Differentiating w.r.t , 2r

dr = 2a 2 cos 2 d dr a 2 = cos 2 d r

Length of the polar tangent 2

a 4 sin 2 2q ⎛ dq ⎞ = r 1+ r2 ⎜ = r 1 + = r 1 + tan 2 2q = r sec 2q = a sin 2 sec 2 ⎟ a 4 cos 2 2q ⎝ dr ⎠ Length of the polar sub-tangent 3 3 r r3 a3 (sin 2q ) 2 dq =r = r2 ⋅ 2 = 2 = 2 = a (sin 2q ) 2 sec 2q dr a cos 2q a cos 2q a cos 2q 2

3.26

Engineering Mathematics

dr d

Length of the polar normal = r 2 + = =

2

= r2 +

a 4 cos 2 2 r2

r4

a 4 (1 sin 2 2 ) = r2

r4

a4 r2

Length of the polar sub-normal =

r4

r4

a4

a 4 sin 2 2 r2

a2 a = r sin 2

=

dr a 2 a 2 cos 2q a cos 2q = . = cos 2q = dq r sin 2q a sin 2q

Example 11: Find the length of the polar tangent, polar sub-tangent, polar nor-

r 2 - a2 a - cos -1 . a r

mal and polar sub-normal for the curve q = r2

Solution:

a2 a

cos

1

a r

Differentiating w.r.t. r,

⎡ ⎢ 1 1 dq 1 1 ⎢ = ⋅ ⋅ 2r − ⎢ − 2 2 2 dr a 2 r − a ⎛a⎞ ⎢ ⎢ 1 − ⎜⎝ r ⎟⎠ ⎣ r a r2

r2

a a2

r r2

Length of the polar tangent = r 1 + r 2

Length of the polar sub-tangent = r 2

a2

ar r 2

2

d dr

= r 1+

dq = r2 dr

⎤ ⎥ ⎥⎛ a ⎞ ⎥ ⎜ − r2 ⎟ ⎠ ⎥⎝ ⎥ ⎦

a2 a2

r 2 (r 2 a 2 ) r 2 = a a2r 2

r 2 − a2 r 2 = r − a2 ar a

2

a2 r 2 ⎛ dr ⎞ Length of the polar normal = r 2 + ⎜ = r2 + 2 = ⎟ r − a2 ⎝ dq ⎠ Length of the polar sub-normal =

dr = dq

ar r − a2 2

r 2 a2 ar

.

r2 r 2 − a2

Differential Calculus II

3.27

Example 12: Find the length of the polar sub-tangent and polar sub-normal for the curve r = aeq cot a . Solution:

r = ae

cot

Differentiating w.r.t. , dr = a cot a eq cot a dq Length of the polar sub-tangent = r2

dq r2 a 2 e 2q cot a = = = a tan a eq cot a dr a cot a eq cot a a cot a eq cot a

Length of the polar sub-normal =

dr = a cot a eq cot a dq

.

Example 13: Show that in the spiral of Archimides r = aq , the length of the polar sub-normal is constant and the polar sub-tangent is aq 2 . r = aq dr =a dq

Solution:

Length of the polar sub-normal =

dr = a = constant d r 2 a 2q 2 d = = = aq 2 . a a dr

Length of the polar sub-tangent = r 2 Exercise 3.2 1. For the curve r = a , prove that a (i) cos = 2 a + r2

3. For the curve r 4 = a 4 cos 4 , prove that (i)

4

r . a + r2 2. For the cardioid r a (1 cos ), prove that (ii) p 2 =

(i)

=

2

2

(iii) 2ap 2 = r 3 .

(ii) p = 2a sin 3

2

=

2

+4

(ii) a 4 p = r 5 . 4. For the curve r 3 = a 3 sin 3 , prove that (i)

=4

3 4 (ii) pa = r .

3.28

Engineering Mathematics

5. Find the angle between the following curves: (i) r = a (1 + cos q ), r = b (1 − cos q )

8. Find the polar sub-tangent to the curve r 3 = a 3 cos 3 .

[ Ans. : − r cot 3q ]

(ii) r 2 = a 2 cosec 2q , r 2 = b 2 sec 2q (iii) r (1 + cos q ) = 2a, r (1 − cos q ) = 2b a (iv) r = a cos q , r = 2 a aq ,r= ( v) r = 1+q (1 + q 2 ) a ( vi) r = a log q , r = log q ( vii) r = aeq , req = b.

⎡ p p p ⎤ (ii) (iii) ⎥ ⎢ Ans. : (i) 2 2 2 ⎥ ⎢ p ⎥ ⎢ (iv) (v) tan −1 3 ⎥ ⎢ 3 ⎥ ⎢ p⎥ ⎢ −1 ⎛ 2e ⎞ (vii) tan (vi) ⎜⎝ 2 −1 ⎠⎟ ⎢ 2 ⎥⎦ e ⎣

6. Prove that the length of the polar subtangent for the curve r (1 − cos q ) = 2b is 2b cosec . 7. Show that for the curve r = a, the polar sub-tangent is constant and the r2 polar sub-normal is . a

9. Find the length of perpendicular from the pole on the tangent to the curve r ( 1) a 2 . ⎡ 1 2 2 4 4⎤ a2 = 2 − 3 + 4 − 5 + 6⎥ Ans. : ⎢ 2 p q q q q q ⎦ ⎣ 10. Find the polar sub-tangent for the l ellipse = 1 + e cos . r length of perpendicular from the pole to the tangent. ⎡ Ans. : ⎢ ⎢(i) l ⎢⎣ e sin

⎤ ⎥ 1 1 ⎛ 2l ⎞ (ii) 2 = 2 ⎜ − 1 + e 2 ⎟ ⎥ p l ⎝ r ⎠ ⎥⎦ 2

11. Prove that for the curve r = ae m , the ratio of polar sub-normal to polar sub-tangent is proportional to 2 . 12. For the curve r 3 = a 3 cos 3 , show that the normal at any point to the curve makes an angle 4 with the initial line.

3.3 LENGTH OF AN ARC AND ITS DERIVATIVE 3.3.1 Derivative of Arc Length in Cartesian Form Let P(x, y) and Q( x + Δx, y + Δy ) be the two neighbouring points on the curve y = f ( x). Let arc AP = s and arc AQ s s, where A is a fixed point on the curve. Then arc PQ s. From the right angled triangle PQR,

Q

A

Δy

Δs

P

R

Δx

PQ 2 = PR 2 + RQ 2 = ( Δx ) 2 + ( Δy ) 2

… (1)

M

Fig. 3.6

N

Differential Calculus II

3.29

Dividing by ( x) 2 , 2

2

2

⎛ PQ ⎞ ⎛ PQ ⎞ ⎛ Δs ⎞ ⎛ Δy ⎞ ⎜ Δx ⎟ = ⎜ Δs ⎟ ⎜ Δx ⎟ = 1 + ⎜ Δx ⎟ ⎝ ⎠ ⎝ ⎠ ⎝ ⎠ ⎝ ⎠ when Q

P,

Also, lim x

0

Thus as Q

2

… (2)

PQ ⎛ chord PQ ⎞ =⎜ ⎟ → 1 and x Δs ⎝ arc PQ ⎠

s ds = and x dx

y dy = x dx

lim x

0

0

P, Eq. (2) reduces to 2

2

⎛ ds ⎞ ⎛ dy ⎞ ⎜⎝ ⎟⎠ = 1 + ⎜⎝ ⎟⎠ dx dx

ds ⎛ dy ⎞ = 1+ ⎜ ⎟ ⎝ dx ⎠ dx

2

Similarly, dividing Eq. (1) by ( y ) 2 and taking limits Q ds dx = 1+ dy dy

P, i.e.,

y

0

2

Corollary: If equation of the curve is given in parametric form x = x(t ), y = y (t ) ds dt

ds dx dx dt dy dx

1

=

dx dt

2

+

2

dx dt dy dt

dx dt

2

dy dx

2

dx dt

2

2

.

3.3.2 Derivative of Arc Length in Polar Form Let P(r , ) and Q( r + Δr , q + Δq ) be the two neighbouring points on the curve r = f ( ). Let arc AP = s and arc AQ = s + Δ s, where A is a fixed point on the curve. Then arc PQ = Δ s. Draw a perpendicular PN on OQ. From the right-angled triangle PQN, PQ 2 = PN 2 + NQ 2 = ( r sin Δq ) 2 + (OQ − ON ) 2 = ( r sin Δq ) 2 + ( r + Δr − r cos Δq ) 2 ⎡ Δq ⎞ ⎤ ⎛ = ( r sin Δq ) 2 + ⎢ Δr + r ⎜ 2 sin 2 ⎟ ⎝ 2 ⎠ ⎥⎦ ⎣

2

… (1)

3.30

Engineering Mathematics

y Q(r +Δr, q +Δq )

N P(r, q ) r Δq

A

q O

x

Fig. 3.7 Rewriting and dividing by (

)2 ,

2

⎛ PQ ⎞ ⎛ PQ ⎞ ⎜⎝ ⎟⎠ = ⎜⎝ ⎟ Δ Δs ⎠

2

⎛ Δs ⎞ ⋅⎜ ⎝ Δ ⎟⎠

2

Δ ⎛ sin Δ Δ r sin ⎜ ⎛ ⎞ 2 ⋅ sin Δ = r2 ⎜ +⎜ +r⋅ ⎟ Δ ⎝ Δ ⎠ 2 Δ ⎜⎝ 2 2

When Q

Also, Thus as Q

P,

PQ ⎛ chord PQ ⎞ =⎜ ⎟ → 1 and Δs ⎝ arc PQ ⎠ lim

0

sin

s

= 1, lim

0

⎞ ⎟ ⎟ ⎟⎠

2

0

=

ds r dr = and lim 0 d d

P, Eq. (2) reduces to ds d

2

dr d

= r2 +

ds dr = r2 + d d Similarly, dividing Eq. (1) by ( r ) 2 and taking limits Q ds d = 1+ r2 dr dr

2

.

2

2

P, i.e., Δ r → 0

… (2)

Differential Calculus II

3.31

3.4 CURVATURE Let P and Q be two neighbouring points on the curve. Let arc AP = s, where A is a fixed point on the curve. Lety be the angle made by the tangent at dy is called the curvature P with the x-axis. Then ds of the curve at point P . Thus, the curvature is defined as the rate of turning of the tangent w.r.t. the arc length.

y Q P A

s

y x

3.4.1 Radius of Curvature

Fig. 3.8

Cartesian Form Radius of curvature of the curve at any point is defined as the reciprocal of the curvature at that point and is denoted by . ds r= dy We know that

dy = tany dx

Differentiating w.r.t. x, d2 y dy = sec 2 y 2 dx dx dy ds dy ds 2 = sec y ⋅ = (1 + tan 2 y ) ⋅ ds dx ds dx

⎡ ⎛ dy ⎞ 2 ⎤ 1 = ⎢1 + ⎜ ⎟ ⎥ ⋅ ⎢⎣ ⎝ dx ⎠ ⎥⎦ r

1

⎡ ⎛ dy ⎞ 2 ⎤ 2 ⎢1 + ⎜ ⎟ ⎥ ⎢⎣ ⎝ dx ⎠ ⎥⎦

3

⎡ ⎛ dy ⎞ 2 ⎤ 2 ⎢1 + ⎜ ⎟ ⎥ ⎢ ⎝ dx ⎠ ⎥⎦ Hence, r=⎣ d2 y dx 2 d2 y Note: The radius of curvature r is positive or negative according as is positive or dx 2 negative. This indicates that the curve is either concave or convex. Since the value of is independent of the choice of axes, interchanging x and y, we get 3

⎡ ⎛ dx ⎞ 2 ⎤ 2 ⎢1 + ⎜ ⎟ ⎥ ⎢ ⎝ dy ⎠ ⎥⎦ r=⎣ d2 x dy 2 This formula is useful when

dx = 0, i.e., the tangent is perpendicular to x-axis. dy

3.32

Engineering Mathematics

Polar Form Let r = f ( ) be the curve. We know that x = r cos dx dr cos r sin d d y = r sin Also, dy dr = sin q + r cos q dq dq dr + r cos q sin q dy dy / dq d q = = dr dx dx / dq − r sin q cos q dq dr ⎛ ⎞ + r cos q sin q d2 y d ⎜ ⎟ dq d q = ⎟ dx dr dx 2 dq ⎜ ⎜ cos q − r sin q ⎟ ⎝ ⎠ dq 2

=

d2r ⎛ dr ⎞ r2 + 2 ⎜ ⎟ − r 2 ⎝ dq ⎠ dq dr ⎛ ⎞ − r sin q ⎟ ⎜⎝ cos q ⎠ dq

3

3

Now,

⎡ ⎛ dy ⎞ 2 ⎤ 2 ⎢1 + ⎜ ⎟ ⎥ ⎢⎣ ⎝ dx ⎠ ⎥⎦ r= d2 y dx 2 3 2 2 ⎡ ⎛ dr ⎞ ⎤ + r cos q ⎥ ⎢ ⎜ sin q ⎟ dq ⎢1 + ⎜ ⎟ ⎥ r d ⎢ ⎜ cos q − r sin q ⎟ ⎥ ⎠ ⎥⎦ ⎢⎣ ⎝ dq = 2 d2r ⎛ dr ⎞ r2 + 2 ⎜ −r 2 ⎟ ⎝ dq ⎠ dq 3

=

dr ⎛ ⎞ − r sin q ⎟ ⎜⎝ cos q ⎠ dq 3 2 ⎤2 ⎡ 2 ⎛ dr ⎞ 2 2 2 2 (cos q + sin q )⎥ ⎢ r (cos q + sin q ) + ⎜ ⎝ dq ⎟⎠ ⎥⎦ ⎢⎣ 2

d2r ⎛ dr ⎞ r2 + 2 ⎜ −r 2 ⎟ ⎝ dq q⎠ dq 3

⎡ 2 ⎛ dr ⎞ 2 ⎤ 2 ⎢r + ⎜ ⎟ ⎥ ⎝ d ⎠ ⎥⎦ ⎢⎣ = 2 d2r . ⎛ dr ⎞ r2 + 2⎜ ⎟ −r 2 d ⎝d ⎠

Differential Calculus II

3.33

3.4.2 Newtonian Method to Find Radius of Curvature at the Origin If a curve passes through the origin and x-axis is the tangent to the curve at origin, then the radius of curvature at the origin is given by x2 x→0 2 y

r = lim

Proof: The curve passes through the origin and x-axis is the tangent to the curve at the origin, dy x = 0, y = 0, =0 At dx 0 ⎡ ⎤ x2 2x x lim = lim = lim ⎢Indeterminate form 0 ⎥ x→0 2 y x→0 d y x → 0 dy ⎢ ⎥ , , 2 ⎣ Applying L Hospital s rule ⎦ dx dx 1 d2 y dx 2 1

= lim

[Applying L’Hospital’s rule]

x→0

=

d2 y dx 2

… (1)

x=0 3

⎡ ⎛ dy ⎞ 2 ⎤ 2 ⎢1 + ⎜ ⎟ ⎥ ⎢⎣ ⎝ dx ⎠ ⎥⎦ r= d2 y dx 2

Now

3

At the origin,

r=

(1 + 0) 2 2

d y dx 2

=

x =0

1

… (2)

2

d y dx 2

x =0

From Eqs (1) and (2), r = lim x→0

x2 2y

Similarly, if the y-axis is the tangent to the curve at the origin, y2 x→0 2x Radius of curvature at the origin can also be found by expanding y in powers of x by algebraic or trigonometric method. Since the curve passes through the origin, f (0) = 0. r = lim

Engineering Mathematics

3.34

Let p and q denote the values of

dy d2 y and 2 at the origin respectively. dx dx 3 2 2

(1 + p ) r (at the origin ) = q

By Maclaurin’s theorem, y = f ( x ) = f (0) + xf ′(0) + = 0+ x⋅ p+ = px +

x2 f ′′(0) + … 2!

x2 ⋅ q +… 2!

qx 2 +… 2!

Equating coefficients of suitable powers of x (generally the lowest two), we obtain equations to determine p and q and hence, is determined at the origin. Example 1: Find the radius of curvature for the following curves s = f (x ) where x is the angle which the tangent to the curve makes with the x axis. Solution: (i) Catenary Radius of curvature (ii) Cycloid Radius of curvature (iii) Tractrix Radius of curvature (iv) Parabola Radius of curvature =a

s = c tany ds r= = c sec 2 y dy s = 4 a siny r=

ds = 4 a cosy dy

s = c log secy r=

ds 1 =c secy tany = c tany dy secy

s = a log (tany + secy ) + a tany secy

r=

ds dy

1 (sec 2 y + secy tany ) + a tany (secy tany ) + a sec3 y tany + secy

= a secy + a secy (tan 2 y + sec 2 y ) = a secy + a secy (sec 2 y − 1 + sec 2 y ) = 2a sec3 y

Differential Calculus II

s = 8a sin 2

(v) Cardioid

3.35

y 6

Radius of curvature

r=

ds d ⎛ d ⎡ ⎛ y y ⎞⎤ 4 2y ⎞ 4 a ⎜1 − cos ⎟ ⎥ = a sin = ⎜ 8a sin ⎟⎠ = ⎢ ⎠ ⎝ dy d y ⎝ d 3 3 y 6 3 ⎦ ⎣

Example 2: Find the radius of curvature of the parabola y 2 = 4ax at any point (x, y). y 2 = 4 ax

Solution: Differentiating w.r.t. x, 2y

dy = 4a dx dy 2 a = dx y

… (1)

Differentiating Eq. (1) w.r.t. x, d2 y 2a dy 4a 2 =− 2 =− 3 2 dx y dx y 3

3

⎡ ⎛ dy ⎞ 2 ⎤ 2 ⎛ 4a 2 ⎞ 2 3 ⎢1 + ⎜ ⎟ ⎥ ⎜1 + 2 ⎟ y ⎠ ⎢⎣ ⎝ dx ⎠ ⎥⎦ ( y 2 + 4a 2 ) 2 ⎝ = = =− d2 y 4a 2 4a 2 − 3 2 dx y 3

3

3

3

(4ax + 4a 2 ) 2 ( 4a ) 2 ( x + a ) 2 2( x + a ) 2 =− = − = − . 4a 2 4a 2 a

Example 3: Find the radius of curvature at any point of catenary y = c cosh y = c cosh

Solution:

x c

Differentiating w.r.t. x, dy x = sinh dx c Differentiating again w.r.t. x, d2 y 1 x = cosh 2 c c dx 3

3 ⎡ ⎛ dy ⎞ 2 ⎤ 2 2 ⎛ 2 x⎞ ⎢1 + ⎜ ⎟ ⎥ ⎜1 + sinh ⎟ ⎢⎣ ⎝ dx ⎠ ⎥⎦ c⎠ = =⎝ x 1 d2 y cosh c c dx 2

x . c

Engineering Mathematics

3.36

x x c = = c cosh 2 1 x c cosh c c y2 = . c cosh 3

Example 4: Find the radius of curvature of the Folium x 3 + y 3 = 3axy at the 3a 3a ⎞ point ⎛⎜ , . 2 2 ⎟ ⎝

Solution:



x 3 + y 3 = 3axy

… (1)

Differentiating Eq. (1) w.r.t. x, dy dy = 3ay + 3ax dx dx d y d y x2 + y 2 = ay + ax dx dx dy ay − x 2 = dx y 2 − ax

3x 2 + 3 y 2

⎛ 3a 3a ⎞ dy At the point ⎜ , = −1 ⎟, 2 ⎠ dx ⎝ 2 Differentiating Eq. (2) w.r.t. x, 2

d2 y dy dy d2 y ⎛ dy ⎞ 2x + 2 y ⎜ ⎟ + y2 2 = a +a + ax 2 dx dx dx dx ⎝ dx ⎠ 3a 3a ⎞ At the point ⎛⎜ , ⎟, 2 ⎠ ⎝ 2 2

2 2 ⎛ 3a ⎞ d y ⎛ 3a ⎞ ⎛ 3a ⎞ ⎛ 3a ⎞ d y = ( − ) + ( − ) + a a a 2 ⎜ ⎟ + 2 ⎜ ⎟ (−1) 2 + ⎜ ⎟ 1 1 ⎜ ⎟ 2 2 ⎝ 2 ⎠ dx ⎝ 2 ⎠ ⎝ 2 ⎠ ⎝ 2 ⎠ dx

d2 y 32 =− 2 3a dx 3a 3a ⎞ At the point ⎛⎜ , ⎟, 2 ⎠ ⎝ 2 3

⎡ ⎛ dy ⎞ 2 ⎤ 2 3 ⎢1 + ⎜ ⎟ ⎥ ⎢⎣ ⎝ dx ⎠ ⎥⎦ [1 + (−1) 2 ] 2 3a = = =− 32 d2 y 8 2 − 2 a 3 dx

… (2)

Differential Calculus II

3.37

Example 5: Find the radius of curvature of the curve x 2 y = a ( x 2 + y 2 ) at ( -2a , 2a ). Solution:

x 2 y = a( x 2 + y 2 )

… (1)

Differentiating Eq. (1) w.r.t. x, 2 xy + x 2

dy dy = 2ax + 2ay dx dx dy 2ax − 2 xy = 2 dx x − 2ay

dy =∞ dx Hence, differentiating Eq. (1) w.r.t. y, At the point ( 2a, 2a ),

2 xy

dx dx + x 2 = 2ax + 2ay dy dy

… (2)

dx 2ay − x 2 = dy 2 xy − 2ax At the point ( 2a, 2a ),

dx =0 dy

Differentiating Eq. (2) w.r.t. y, 2

2

⎛ dx ⎞ ⎛ dx ⎞ d2 x dx dx d2 x 2 xy 2 + 2 x + 2 y ⎜ ⎟ + 2 x = 2a ⎜ ⎟ + 2ax 2 + 2a dy dy dy dy ⎝ dy ⎠ ⎝ dy ⎠ At the point ( 2a, 2a ), 2(−2a )(2a )

d2 x d2 x = 2a (−2a ) 2 + 2a 2 dy dy d2 x 2a 1 = =− 2a dy 2 4 a 2 − 8 a 2

At the point ( 2a, 2a ) , 3

⎡ ⎛ dx ⎞ 2 ⎤ 2 ⎢1 + ⎜ ⎟ ⎥ ⎢ ⎝ dy ⎠ ⎥⎦ =⎣ d2 x dy 2 = −2a. Example 6: Find the radius of curvature of the curve (x2 + y2)2 – 2ax (x2 + y2) – a3 y = 0 at the point (2a, 0).

3.38

Solution:

Engineering Mathematics

( x 2 + y 2 ) 2 − 2ax ( x 2 + y 2 ) − a 3 y = 0

… (1)

Differentiating Eq. (1) w.r.t. x, dy dy ⎞ dy ⎞ ⎛ ⎛ 2( x 2 + y 2 ) ⎜ 2 x + 2 y ⎟ − 2ax ⎜ 2 x + 2 y ⎟ − 2a ( x 2 + y 2 ) − a 3 =0 dxx dx ⎠ dx ⎠ ⎝ ⎝ At the point (2a, 0), dy 2(4a 2 )(4a ) − 4a 2 (4a ) − 2a (4a 2 ) − a 3 =0 dx dy =8 dx

… (2)

Differentiating Eq. (2) w.r.t. x, 2 2 2 ⎡ ⎡ d2 y d2 y dy ⎞ ⎛ dy ⎞ ⎤ ⎛ dy ⎞ ⎤ ⎛ 2( x 2 + y 2 ) ⎢ 2 + 2 y 2 + 2 ⎜ ⎟ ⎥ + 2 ⎜ 2 x + 2 y ⎟ − 2ax ⎢ 2 + 2 y 2 + 2 ⎜ ⎟ ⎥ ⎝ ⎝ dx ⎠ ⎦ ⎝ dx ⎠ ⎦ dx dx dx ⎠ ⎣ ⎣ 2 dy ⎞ d y dy ⎞ ⎛ ⎛ −2a ⎜ 2 x + 2 y ⎟ − 2a ⎜ 2 x + 2 y ⎟ − a3 2 = 0 ⎝ ⎠ ⎝ ⎠ dx dx dx

At the point (2a, 0), 2 ( 4 a 2 ) ( 2 + 128) + 2 (16 a 2 ) − 4 a 2 ( 2 + 128) − 8a 2 − 8a 2 − a3

d2 y =0 dx 2 d 2 y 536 = a dx 2

At the point (2a, 0), 3

⎡ ⎛ dy ⎞ 2 ⎤ 2 3 3 ⎢1 + ⎜ ⎟ ⎥ ⎢⎣ ⎝ dx ⎠ ⎥⎦ (1 + 64) 2 (65) 2 a = . = = 536 536 d2 y a dx 2

Example 7: Find the radius of curvature of the curves x = a log (sec q + tan q ), y = a sec q . Solution:

x = a log(secq + tan q ) dx 1 =a (sec q tan q + sec 2 q ) = a secq dq (sec q + tan q ) y = a secq dy = a secq taanq dq dy dy / dq a secq tan q = = = tan q dx dx / dq a secq d2 y dq sec 2 q 1 2 = secq = sec q = dx a secq a dx 2

Differential Calculus II

3.39

3

⎡ ⎛ dy ⎞ 2 ⎤ 2 3 3 ⎢1 + ⎜ ⎟ ⎥ (1 + tan 2 θ ) 2 (sec 2 θ ) 2 ⎝ dx ⎠ ⎦ ⎣ = = ρ= 1 1 d2 y sec sec θ θ 2 a a dx = a sec 2 θ . ⎛

Example 8: Find the radius of curvature of the curve x = a ⎜ t ⎝

Solution:

⎛ t3 ⎞ x = a ⎜t − ⎟ 3⎠ ⎝

t3 ⎞ 2 ⎟ , y = at . 3⎠

y = at 2

dx = a (1 − t 2 ) dt

dy = 2at dt 2at 2t dy dy / dt = = = dx dx / dt a (1 − t 2 ) 1 − t 2

d 2 y 2 (1 − t 2 ) − 2t ( −2t ) dt = dx (1 − t 2 ) 2 dx 2 =

2 (1 + t 2 ) 1 2 (1 + t 2 ) ⋅ = 2 2 2 (1 − t ) a (1 − t ) a (1 − t 2 )3 3

⎡ ⎛ dy ⎞ 2 ⎤ 2 ⎢1 + ⎜ ⎟ ⎥ ⎢⎣ ⎝ dx ⎠ ⎥⎦ r= = d2 y dx 2

3

⎡ ⎛ 2t ⎞ 2 ⎤ 2 ⎢1 + ⎜ 2 ⎟ ⎥ ⎢⎣ ⎝ 1 − t ⎠ ⎥⎦ 2 (1 + t 2 ) a (1 − t 2 )3 3

3

[(1 − t 2 ) 2 + 4t 2 ] 2 ⋅ a (1 − t 2 )3 a [(1 + t 2 ) 2 ] 2 = = 2 (1 + t 2 ) (1 − t 2 )3 ⋅ 2 (1 + t 2 ) a = (1 + t 2 ) 2 . 2 Example 9: Find the radius of curvature of the curve x = e t + e t, y = e t - e t at t = 0. Solution:

x = et + e − t

y = e t – e–t

dx = et − e − t dt

dy = et + e − t dt

dy dy / dt e t + e − t x = = = dx dx / dt e t − e − t y d 2 y 1 x dy 1 x x y 2 − x 2 = − = − ⋅ = dx 2 y y 2 d x y y 2 y y3

Engineering Mathematics

3.40

3

3

⎡ ⎛ dy ⎞ 2 ⎤ 2 ⎛ x 2 ⎞ 2 3 ⎢1 + ⎜ ⎟ ⎥ ⎜⎝1 + y 2 ⎟⎠ ⎢⎣ ⎝ dx ⎠ ⎥⎦ ( y2 + x2 )2 r= = 2 = y − x2 d2 y ( y2 − x2 ) 3 2 y dx At t = 0, y = 0, x = 2 3

42 r= = −2 . −4 Example 10: Find the radius of curvature at any point of the curve x = a cos 3p , y = a sin 3p . Solution:

x = a cos3 q dx = −3a cos 2 q sin q dq

y = a sin 3 q dy = 3a sin 2 q cos q dq

dy dy / d q 3a sin 2 q cos q = = = − tan q dx dx / dq −3a cos 2 q sin q d2 y dq = − sec 2 q 2 dx dx − sec 2 q = −3a cos 2 q sin q 1 = 4 3a cos q sin q 3

⎡ ⎛ dy ⎞ 2 ⎤ 2 3 ⎢1 + ⎜ ⎟ ⎥ ⎢⎣ ⎝ dx ⎠ ⎥⎦ (1 + tan 2 q ) 2 r= = = 3a cos q sin q . 1 d2 y 3a cos 4 q sin q dx 2

Example 11: For the cycloid x = a (q + sin q ), y = a (1 - cos q ), prove that q r = 4 a cos . 2 y = a(1 − cos q ) Solution: x = a(q + sin q ) dy dx = a sin q = a(1 + cos q ) dq dq q q 2 sin cos dy dy / dq a sin q 2 2 = tan q = = = 2 dx dx / dq a(1 + cos q ) 2q 2 cos 2

Differential Calculus II

d 2 y 1 2 q dq 1 2 q = sec = sec ⋅ 2 dx 2 2 dx 2 2

1 2a cos 2

q 2

3.41

1

=

4 a cos 4

q 2

3

3 ⎡ ⎛ dy ⎞ 2 ⎤ 2 2 ⎛ 2q ⎞ ⎢1 + ⎜ ⎟ ⎥ ⎜⎝1 + tan ⎟⎠ ⎢⎣ ⎝ dx ⎠ ⎥⎦ 2 = 4 a cos q . r= = 2 1 2 d y 4 q dx 2 4 a cos 2

Example 12: Find the radius of curvature of the curve 3a x= (sinh u cosh u + u), y = a cosh3 u. 2 3a Solution: y = a cosh3 u x= (sinh u cosh u + u ) 2 dx 3a dy = (cosh 2 u + sinh 2 u + 1) = 3a cosh 2 u sinh u du 2 du 3a = ⋅ 2 cosh 2 u 2 = 3a cosh 2 u dy dy /du 3a cosh 2 u sinh u = = = sinh u dx dx /du 3a cosh 2 u cosh u 1 d2 y du = = = cosh u 2 2 dx 3a cosh u 3a cosh u dx 3

⎡ ⎛ dy ⎞ 2 ⎤ 2 3 ⎢1 + ⎜ ⎟ ⎥ ⎢⎣ ⎝ dx ⎠ ⎥⎦ (1 + sinh 2 u ) 2 r= = = 3a coshh 4 u . 1 d2 y 3a cosh u dx 2 Example 13: Find the radius of curvature of the cardioid r = a (1 + cos q ). Solution:

r = a (1 + cos q )

Differentiating w.r.t. , dr = − a sin q dq Differentiating again w.r.t. , d2r = − a cos q dq 2

3.42

Engineering Mathematics

3

⎡ 2 ⎛ dr ⎞ 2 ⎤ 2 ⎢r + ⎜ ⎥ ⎝ dq ⎟⎠ ⎥⎦ ⎢⎣ r= 2 d2r ⎛ dr ⎞ − r2 + 2 ⎜ r ⎝ dq ⎟⎠ dq 2 3

[a 2 (1 + coss q ) 2 + a 2 sin 2 q ] 2 = 2 a (1 + cos q ) 2 + 2a 2 sin 2 q + a 2 cos q (1 + cos q ) 3 2

=

[2a 2 (1 + cos q )] = a 2 + 2a 2 + 3a 2 cos q

=

4a q cos . 3 2

⎡ 2 ⎢⎣ 2a

3

2 ⎛ 2 q ⎞⎤ ⎜⎝ 2 cos ⎟⎠ ⎥ 2 ⎦

q⎞ ⎛ 3a 2 ⎜ 2 cos 2 ⎟ ⎝ 2⎠

Example 14: Find the radius of curvature of the curve r = aep cot` . r = ae

Solution:

cot a

Differentiating w.r.t. q, dr = a cot α eθ cot α dθ Differentiating again w.r.t. q, d2r = a cot 2 α eθ cot α dθ 2 3

⎡ 2 ⎛ dr ⎞ 2 ⎤ 2 ⎢r + ⎜ ⎟ ⎥ ⎝ dθ ⎠ ⎥⎦ ⎢⎣ ρ= 2 d2r ⎛ dr ⎞ r2 + 2⎜ ⎟ −r 2 dθ ⎝ dθ ⎠ 3

[a 2 e 2θ cott α + a 2 cot 2 α e 2θ cot α ] 2 = 2 2θ cot α ae + 2a 2 cot 2 α e 2θ cot α − a 2 cot 2 α e 2θ cot α 3

[a 2 e 2θ cot α (1 + cot 2 α )] 2 = 2 2θ cot α ae (1 + cot 2 α ) a 3 e3θ cot α cosec 3α a 2 e 2θ cot α cosec 2α = aeθ cot α cosecα = r cosec α .

=

Differential Calculus II

Example 15: Find the radius of curvature of the curve r = r=

Solution:

3.43

a . p

a q

Differentiating w.r.t. q, dr a =− 2 dq q Differentiating again w.r.t. q, d 2 r 2a = dq 2 q 3 3

3

⎡ 2 ⎛ dr ⎞ 2 ⎤ 2 ⎛ a2 a2 ⎞ 2 ⎢r + ⎜ ⎥ ⎟ ⎜⎝ q 2 + q 4 ⎟⎠ ⎝ dq ⎠ ⎥⎦ ⎢⎣ = 2 r= 2 a a 2 2a 2 d2r ⎛ dr ⎞ 2 + 2 − − r + 2⎜ r 2 q4 q4 ⎝ dq ⎟⎠ dq 2 q 3

⎛ a2 r 2 ⎞ 2 3 3 ⎜⎝ q 2 + q 2 ⎟⎠ (a2 + r 2 ) 2 r(a2 + r 2 ) 2 = = = . a 2q a3 a2 q2 Example 16: Find the radius of curvature of the curve r m = a m cos mp . Solution:

r m = a m cos mq

Taking logarithm on both the sides, m log r = m log a + log cos mq

Differentiating w.r.t. q, 1 dr 1 m⋅ ⋅ = ( − m sin mq ) r dq cos mq dr = − r tan mq dq Differentiating again w.r.t. q, d2r dr = − r ⋅ m ⋅ sec 2 mq − tan mq dq dq 2 = − mr sec 2 mq + r tan 2 mq

Engineering Mathematics

3.44

3

⎡ 2 ⎛ dr ⎞ 2 ⎤ 2 ⎢r + ⎜ ⎥ ⎝ dq ⎟⎠ ⎥⎦ ⎢⎣ r= 2 d2r ⎛ dr ⎞ −r 2 r2 + 2 ⎜ ⎟ ⎝ dq ⎠ dq 3

( r 2 + r 2 tann 2 mq ) 2 = 2 r + 2r 2 tan 2 mq + mr 2 sec 2 mq − r 2 tan 2 mq r 3 sec3 mq r r am = = = = . ( m + 1) r 2 sec 2 mq ( m + 1) cos mq r m ( m + 1)r m −1 ( m + 1) m a

Example 17: Prove that the radius of curvature at any point of the curve

r 2 cos2p = a 2 is –

r3 . a2

Solution:

r 2 cos 2q = a 2

Taking logarithm on both the sides, 2 log r + log cos 2q = log a 2 Differentiating w.r.t. , 1 dr 1 + ( −2 sin 2q ) = 0 r dq cos 2q dr = r tan 2q dq

2⋅

Differentiating again w.r.t. , d2 r dr = 2r sec 2 2q + tan 2q dq dq 2 = 2r sec 2 2q + r tan 2 2q 3

⎡ 2 ⎛ dr ⎞ 2 ⎤ 2 ⎢ r + ⎜⎝ ⎟ ⎥ dq ⎠ ⎦ ⎣ r= 2 d2r ⎛ dr ⎞ r2 + 2 ⎜ −r 2 ⎟ ⎝ dq ⎠ dq 3

( r 2 + r 2 tan 2 2q ) 2 = 2 r + 2r 2 tan 2 2q − r ( r tan 2 2q + 2r sec 2 2q ) 3

( r 2 sec 2 2q ) 2 r2 = = − r sec 2q = − r ⋅ 2 2 2 − r sec 2q a r3 =− 2. a

Differential Calculus II

3.45

Example 18: Prove that at the points in which the Archimedean spiral r = ap intersects the hyperbolical spiral rp = a, their curvatures are in the ratio 3:1. Solution: For the curve r = aq, dr =a dq d2r =0 dq 2 2

Curvature

k1 =

d2r ⎛ dr ⎞ −r 2 r2 + 2 ⎜ ⎟ ⎝ dq ⎠ dq 3

=

⎡ 2 ⎛ dr ⎞ 2 ⎤ 2 ⎢r + ⎜ ⎥ ⎝ dq ⎟⎠ ⎥⎦ ⎢⎣

a 2q 2 + 2a 2 − 0 3

( a 2q 2 + a 2 ) 2

=

q2 +2 3

a (q 2 + 1) 2

For the curve rq = a , r=

a q

dr a =− 2 dq q d 2 r 2a = dq 2 q 3 2

d2r ⎛ dr ⎞ −r 2 r + 2⎜ ⎟ ⎝ dq ⎠ dq 2

Curvature

k2 =

3

a2 a 2 2a 2 + 2 − 2 q4 q4 q4 = =q 3 3 ⎛ a2 a2 ⎞ 2 a (q 2 + 1) 2 ⎜⎝ q 2 + q 4 ⎟⎠

⎡ 2 ⎛ dr ⎞ 2 ⎤ 2 ⎢r + ⎜ ⎥ ⎝ dq ⎟⎠ ⎥⎦ ⎢⎣ The points of intersection of two curves are obtained as, a q 2 q =1 q = ±1 aq =

At q = ±1,

3

k1 =

3

a ⋅ 22

1

k2 =

3

a 22

k1 3 = k2 1 Hence, curvatures are in the ratio 3:1.

Engineering Mathematics

3.46

Example 19: Find the radius of curvature at the origin for the curve x 3 - 2 x 2 y - 4 y 3 + 5 x 2 - 6 xy + 7 y 2 - 8 y = 0. Solution: Equating the lowest degree term in the equation to zero, we get y = 0, i.e., x-axis which is tangent to the curve at the origin. x2 = lim x →0 2 y

2 = lim

Hence,

x

0

x2 y

Dividing the equation by y, x⋅

When x

0, y

x2 x2 − 2x2 − 4 y2 + 5 − 6x + 7 y − 8 = 0 y y

0.

0 ( 2 ) − 2 ( 0) − 4 ( 0) + 5 ( 2 ) − 6 ( 0 ) + 7 ( 0 ) − 8 = 0 4 = . 5

Example 20: Find the radius of curvature at the origin for the curve x 3 y − xy 3 + 2 x 2 y − 2 xy 2 + 2 y 2 − 3 x 2 + 3 xy − 4 x = 0.

Solution: Equating the lowest degree term in the equation to zero, we get x = 0, i.e., y-axis which is tangent to the curve at the origin. y2 = lim x →0 2 x 2 = lim x →0

y2 x

Dividing the equation by x,

x 2 y − y 3 + 2 xy − 2 y 2 + When y

0, x

2 y2 − 3x + 3 y − 4 = 0 x

0, 0 − 0 + 0 − 0 + 2 (2 ) − 0 + 0 − 4 = 0

=1

Example 21: Find the radius of curvature at the origin for the curve y = 2 x + 3 x 2 - 2 xy + y 2 . Solution: Equating the lowest degree terms in the equation to zero, we get y 2 x 0 which is tangent to the curve at the origin.

Differential Calculus II

Substituting y = px + q

3.47

x2 + … in the equation of the curve, 2

⎛ ⎞ ⎛ x2 x2 x2 px + q + … = 2 x + 3 x 2 − 2 x ⎜ px + q + … ⎟ + ⎜ px + q + 2 2 2 ⎝ ⎠ ⎝

⎞ ⎟ ⎠

2

x and x2, p=2 q = 3 − 2 p + p2 2 = 3− 4+ 4 = 3 q=6

and

3

3

(1 + p 2 ) 2 (1 + 4) 2 5 5 r= = = . q 6 6

Example 22: Find the radius of curvature at the origin for the curve y 2 - 3 xy - 4 x 2 + x 3 + x 4 y + y 5 = 0. Solution: Equating the lowest degree terms in the equation to zero, we get y 2 − 3 xy − 4 x 2 = 0 which, indicates that there are two tangents at the origin. Hence, there will be two values of at the origin. Substituting y = px + q

x2 + … in the equation of the curve, 2

2

⎛ ⎞ ⎛ ⎞ ⎞ x2 x2 x2 2 3 4 ⎛ … 3 … 4 px + q + − x px + q + − x + x + x px + q + …⎟ ⎜⎝ ⎟ ⎜ ⎟ ⎜ 2 2 2 ⎠ ⎝ ⎠ ⎝ ⎠ 5

⎛ ⎞ x2 + ⎜ px + q + …⎟ = 0 2 ⎝ ⎠ x 2 and x 3 ,

and From Eq. (1), p = 4, From Eq. (2), When p = 4, q = − When p = −1, q =

2 5

2 5

1

p2 − 3 p − 4 = 0

… (1)

3 pq − q + 1 = 0 2

… (2)

Engineering Mathematics

3.48

2 For p = 4 and q = − , 5 3

3

(1 + p 2 ) 2 (1 + 16) 2 85 r= = =− 17 2 q 2 − 5 For p =

1 and q =

2 , 5 3

3

(1 + p 2 ) 2 (1 + 1) 2 r= = = 5 2. 2 q 5

Example 23: Find the radius of curvature at the origin for the curve x = a (q + sin q ), y = a (1 - cos q ).

Solution: For the cycloid, x = a(q + sin q ), y = a(1 − cos q ), x-axis is the tangent at the origin. x2 x→0 2 y

r = lim

a 2 (q + sin q ) 2 q → 0 2a (1 − cos q )

= lim

[∵q = 0 at the originn ]

a (q + sin q ) 2 ⎡0⎤ lim ⎢⎣ 0 ⎥⎦ → 0 q 2 1 − cos q a 2(q + sin q )(1 + cos q ) = lim → q 0 2 sin q =

[Applying L’Hospital’s rule]

a ⎛ q ⎞ lim 2 ⎜ + 1⎟ (1 + cos q ) ⎠ 2 q → 0 ⎝ sin q a = ( 2) (1 + 1)(1 + 1) 2 = 4 a. =

Exercise 3.3 1. Find the radius of curvature of the following curves: (i) xy = c 2 a a (ii) x + y = a at ⎛⎜ , ⎞⎟ ⎝4 4⎠ x2 y 2 (iii) 2 + 2 = 1 at (a, 0) and (0, b) a b

(iv) x3 + y3 = 2a3 at (a, a) (v) xy 2 = a 3 − x 3 at (a, 0) (vi) y =

log x at x = 1 x

(vii) y = e x at (0, 1)

Differential Calculus II

(viii) x 3

xy 2

6 y2

0 at (3, 3).

3 ⎤ ⎡ 2 2 2 ⎢ Ans.: (i) ( x + y ) (ii) a ⎥ ⎢ 2c 2 2 ⎥ ⎥ ⎢ 2 2 b a a ⎥ ⎢ (iii) , (iv) ⎢ a b 2 ⎥⎥ ⎢ ⎢ 3a 2 2⎥ ( v) − ( vi) ⎥ ⎢ 3 ⎥ 2 ⎢ 3 ⎥ ⎢ ( vii) 2 2 ( viii) 5 2 ⎥⎦ ⎢⎣

2. Find the radius of curvature of the following curves: (i) x = 1 − t 2 , y = t − t 3 at t = ±1 (ii) x = log t , y =

1 ⎛ 1⎞ ⎜t + ⎟ 2⎝ t⎠

(iii) x = a (q − sin q ), y = a (1 − cos q ) (iv) x = a sin 2t (1 + cos 2t ), y = a cos 2t (1 − cos 2t ) (v) x = 3a cos q − a cos 3q , y = 3a sin q − a sin 3q (vi) x = 3 + 4 cos t , y = 4 + 3 sin t at (3, 7) (vii) x = a( 2 cos q + cos 2q ), y = a( 2 sin q − sin 2q ) (viii) x = a cos q , y = a sin q . q q 1 ⎡ ⎤ (ii) (1 + t 2 ) 2 ⎥ ⎢ Ans.: (i) 2 2 4 ⎢ ⎥ ⎢ (iii) − 4a sin (iv) 4a cos 3t ⎥ ⎢ ⎥ 2 ⎢ ⎥ 16 ⎢ ⎥ ( v) 3a sin ( vi) − ⎢ ⎥ 3 ⎢ 3 ⎥ ⎢ 3 a (1 + 2 ) 2 ⎥ ( vii) 8a sin ( viii) ⎢ ⎥ 4 2 ⎣ ⎦

3.49

3. Show that the radius of curvature at any point of curve x = aeq (sin q − cos q ), y = aeq (sin q + cos q ) is twice the distance of the tangent at the point from the origin. 4. Show that the radius of curvature at each point of the curve t⎞ ⎛ x = a ⎜ cos t + log tan ⎟ , y = a sin t ⎝ 2⎠ is inversely proportional to the length of the normal intercepted between the point on the curve and the x-axis. 5. Find the radius of curvature of the following curves: (i) r = e 2q at q = log 2 n n (ii) r = a sin nq

(iii) r = tanq at q =

3p 4

(iv) r (1 + cos q ) = 2a r 2 − a2 a − cos −1 a r (vi) r = a(1 − cos q ) (v) q =

q =b 2 q r cos = a . 2

(vii) r cos 2 (viii)

⎡ ⎤ an (iii) 5 ⎥ ⎢ Ans.: (i) 16 (ii) n −1 (1 + n)r ⎢ ⎥ ⎢ ⎥ r 2 ⎢(iv) 2r ( v) r 2 − a 2 ( vi) 2ar ⎥ a 3 ⎢ ⎥ ⎢ ⎥ ⎢( vii) 2r ( viii) 2r r ⎥ ⎢⎣ ⎥⎦ a b 6. Find the radius of curvature at the origin for the following curves: (i) x 4 − y 4 + x 3 − y 3 + x 2 − y 2 + y = 0

Engineering Mathematics

3.50

(viii) 9a 2 x 2 = 4 y 2 ( y − 3a ) 2

(ii) ( x 2 − y 2 )( x 2 + y 2 ) +( x − y )( x 2 + xy + y 2 ) + ( x − y )( x + y ) + y = 0

(ix) x 3 + y 3 − 3axy = 0 (x) x 2 − 4 xy − 2 y 2 + 10 x + 4 y = 0.

(iii) y = x 4 − 4 x 3 − 18 x 2 (iv) y = x 3 + 5 x 2 + 6 x

4 1 1 17 ⎡ ⎤ ⎢ Ans.: (i) 5 (ii) − 2 (iii) 36 (iv) 10 37 ⎥ ⎢ ⎥ ⎢( v) 1 ( vi) 3 ( vii) ± 2a 2 ( viii) 15 a 5 ⎥ ⎢ ⎥ 4 ⎢ ⎥ 3 29 ⎢(ix) a ( x) ⎥ 29 ⎢⎣ ⎥⎦ 2 6

(v) x y − xy + 2 x y 3

3

2

+ xy − y 2 + 2 x = 0 (vi) x 3 + 3 x 2 y − 4 y 3 + y 2 − 6 x = 0 2 2 3 (vii) a ( y − x ) = x

3.5 CENTRE AND CIRCLE OF CURVATURE Let P(x, y) be any point on the curve y = f ( x). Let the tangent at point P make an with the x-axis. Let C be the point angle on the positive direction of the normal to the curve at point P such that CP = . Then C is called the centre of curvature to the curve at P. The circle with centre C and radius r is called the circle of curvature at P. Let C(X, Y) be the centre of curvature.

y

From Fig. 3.9,

O

C(X, Y) Q

T

y M

y

r P(x, y)

N

Fig. 3.9

X = OM = ON − MN = ON − QP = x − r siny 3

⎡ ⎛ dy ⎞ 2 ⎤ 2 dy ⎢1 + ⎜ ⎟ ⎥ ⎢⎣ ⎝ dx ⎠ ⎥⎦ dx = x− ⋅ 2 d2 y ⎛ dy ⎞ 2 1 + ⎟ ⎜ dx ⎝ dx ⎠

= x−

dy ⎡ ⎛ dy ⎞ ⎢1 + ⎜ ⎟ dx ⎢⎣ ⎝ dx ⎠ d2 y dx 2

2

⎤ ⎥ ⎥⎦

dy ⎤ ⎡ ⎢⎣∵ tany = dx ⎥⎦

x

Differential Calculus II

3.51

Y = MC = MQ + CQ = y + r cosy 3

⎡ ⎛ dy ⎞ 2 ⎤ 2 ⎢1 + ⎜ ⎟ ⎥ ⎢⎣ ⎝ dx ⎠ ⎥⎦ = y+ d2 y dx 2

1 ⎛ dy ⎞ 1+ ⎜ ⎟ ⎝ dx ⎠

2

dy ⎤ ⎡ ⎢⎣∵ tany = dx ⎥⎦

2

⎛ dy ⎞ 1+ ⎜ ⎟ ⎝ dx ⎠ = y+ . d2 y dx 2 The equation of the circle of curvature of the curve at the point P (x, y) with radius and centre C (X, Y ) is given by, ( x − X ) 2 + ( y − Y ) 2 = r 2.

3.6 EVOLUTE The locus of the centres of curvature of the curve is called the evolute of the curve. The evolute of any curve y = f ( x) is obtained by eliminating x and y from the equation 2 2 dy ⎡ ⎛ dy ⎞ ⎤ ⎛ dy ⎞ 1 + ⎢ ⎜ ⎟ ⎥ 1 + ⎜ ⎟ dx ⎢⎣ ⎝ dx ⎠ ⎥⎦ ⎝ dx ⎠ , Y = y+ y = f ( x), X = x − 2 2 y d d y 2 dx 2 dx

Example 1: Show that the circle of curvature at the origin of the curve x2 y = mx + is x 2 + y 2 = a (1 + m2 )( y - mx ). a y = mx +

Solution:

x2 a

Differentiating w.r.t. x, dy 2x = m+ dx a Differentiating again w.r.t. x, d2 y 2 = dx 2 a At the origin,

dy d2 y 2 = m, = dx dx 2 a

Engineering Mathematics

3.52

Let (X, Y) be the centre of curvature at the origin.

X = x−

dy ⎡ ⎛ dy ⎞ ⎢1 + ⎜ ⎟ dx ⎢⎣ ⎝ dx ⎠

2

⎤ ⎥ ⎥⎦

d2 y dx 2

= 0−

m (1 + m 2 ) ma (1 + m 2 ) =− 2 2 a

2

⎛ dy ⎞ 1+ ⎜ ⎟ ⎝ dx ⎠ 1 + m 2 a (1 + m 2 ) = 0 + = Y = y+ 2 2 d2 y 2 a dx At the origin, 3

⎡ ⎛ dy ⎞ 2 ⎤ 2 3 ⎢1 + ⎜ ⎟ ⎥ ⎢⎣ ⎝ dx ⎠ ⎥⎦ (1 + m 2 ) 2 r= = 2 d2 y 2 a dx Hence, the equation of the circle of curvature at the origin is given by, ( x − X )2 + ( y − Y )2 = r 2 2

2

⎡ ma (1 + m 2 ) ⎤ ⎡ a (1 + m 2 ) ⎤ a 2 (1 + m 2 )3 ⎢x + ⎥ + ⎢y − ⎥ = 2 2 4 ⎣ ⎦ ⎣ ⎦ x 2 + y 2 = − ma (1 + m 2 ) x + a (1 + m 2 ) y = a (1 + m 2 ) ( y − mx ). Example 2: Find the centre and circle of curvature of the curve

⎛a a⎞ , ⎟. ⎝4 4⎠

at the point ⎜

x+ y = a

Solution: Differentiating w.r.t. x,

1 − 12 1 − 12 dy x + y =0 2 2 dx 1

dy y2 =− 1 dx x2 Differentiating again w.r.t. x, 1 2

d y =− dx 2

x2

1 1 − 12 dy 1 −1 y − y2 x 2 2 2 dx x

x+

y= a

Differential Calculus II

⎛a a⎞ At the point ⎜ , ⎟ , ⎝4 4⎠

3.53

dy = −1 dx 1

d2 y =− dx 2

⎛ a ⎞2 1 ⎛ a ⎞ ⎜⎝ ⎟⎠ ⎜⎝ ⎟⎠ 4 2 4



1 2

1

⎛ a ⎞2 1 ⎛ a ⎞ ( −1) − ⎜ ⎟ ⎜ ⎟ ⎝4⎠ 2 ⎝4⎠ a 4



1 2

=

4 a

a a Let (X, Y ) be the centre of curvature at the point ⎛⎜ , ⎞⎟ . ⎝4 4⎠ 2 dy ⎡ ⎛ dy ⎞ ⎤ ⎢1 + ⎜ ⎟ ⎥ dx ⎢⎣ ⎝ dx ⎠ ⎥⎦ a ( −1)(1 + 1) 3a = − = X = x− 4 4 4 d2 y a dx 2 2

⎛ dy ⎞ 1+ ⎜ ⎟ ⎝ dx ⎠ a 1 + 1 3a = + = Y = y+ 2 4 4 4 d y a dx 2 ⎛a a⎞ At the point ⎜ , ⎟ , ⎝4 4⎠ 3

⎡ ⎛ dy ⎞ 2 ⎤ 2 3 ⎢1 + ⎜ ⎟ ⎥ ⎢⎣ ⎝ dx ⎠ ⎥⎦ (1 + 1) 2 a r= = = 2 4 d y 2 a dx 2

a a Hence, the equation of the circle of curvature at the point ⎛⎜ , ⎞⎟ is given by, ⎝4 4⎠ ( x − X )2 + ( y − Y )2 = r 2 2

2

3a ⎞ ⎛ 3a ⎞ a2 ⎛ ⎜⎝ x − ⎟⎠ + ⎜⎝ y − ⎟⎠ = . 4 4 2

Example 3: Find the centre and circle of curvature of the curve y = x3 – 6x2 + 3x + 1 at the point (1, –1). Solution:

y = x 3 − 6 x 2 + 3x + 1

Differentiating w.r.t. x, dy = 3 x 2 − 12 x + 3 dx

3.54

Engineering Mathematics

Differentiating again w.r.t. x, d2 y = 6 x − 12 dx 2 At the point (1, 1),

dy d2y = −6 , = −6 dx dx 2

Let (X, Y ) be the centre of curvature at the point (1, 1).

X = x−

dy ⎡ ⎛ dy ⎞ ⎢1 + ⎜ ⎟ dx ⎢⎣ ⎝ dx ⎠

2

2

d y dx 2

⎤ ⎥ ⎥⎦

= 1−

( −6)(1 + 36) = −36 −6

2

⎛ dy ⎞ 1+ ⎜ ⎟ ⎝ dx ⎠ 1 + 36 43 = −1 + =− Y = y+ 2 −6 6 d y dx 2 At the point (1, –1), 3

⎡ ⎛ dy ⎞ 2 ⎤ 2 3 3 ⎢1 + ⎜ ⎟ ⎥ ⎢⎣ ⎝ dx ⎠ ⎥⎦ (1 + 36) 2 (37) 2 r= = =− −6 6 d 2y 2 dx

Hence, the equation of the circle of curvature at the point (1, 1) is given by ( x − X )2 + ( y − Y )2 =

2

2

43 ⎞ (37)3 ⎛ ( x + 36) 2 + ⎜ y + ⎟ = . ⎝ 6 ⎠ 36

Example 4: Show that the parabolas y = - x 2 + x + 1 and x = - y 2 + y + 1 have the same circle of curvature at the point (1, 1). Solution: For the parabola y = − x 2 + x + 1, Differentiating w.r.t. x, dy = −2 x + 1 dx Differentiating again w.r.t. x, d2 y = −2 dx 2 At the point (1, 1),

dy d2 y = −1, = −2 dx dx 2

Differential Calculus II

3.55

Let (X, Y ) be the centre of curvature at the point (1, 1).

X = x−

2 dy ⎡ ⎛ dy ⎞ ⎤ ⎢1 + ⎜ ⎟ ⎥ dx ⎢⎣ ⎝ dx ⎠ ⎥⎦ 2

d y dx 2

= 1−

( −1)(1 + 1) =0 −2

2

⎛ dy ⎞ 1+ ⎜ ⎟ ⎝ dx ⎠ (1 + 1) = 1+ =0 Y = y+ −2 d2 y dx 2

At the point (1, 1), 3

⎡ ⎛ dy ⎞ 2 ⎤ 2 3 ⎢1 + ⎜ ⎟ ⎥ ⎡⎣1 + ( −1) 2 ⎤⎦ 2 ⎢⎣ ⎝ dx ⎠ ⎥⎦ r= = =− 2 −2 d2 y dx 2 Hence, the equation of the circle of curvature at the point (1, 1) is given by, ( x − X )2 + ( y − Y )2 =

2

( x − 0) 2 + ( y − 0) 2 = ( 2 ) 2 x2 + y 2 = 2 For the parabola x = − y 2 + y + 1, Differentiating w.r.t. y, dx = −2 y + 1 dy Differentiating again w.r.t. y, d2 x = −2 dy 2 At the point (1, 1),

d2 x dx = −2 = −1, dy 2 dy

Let (X, Y ) be the centre of curvature at the point (1, 1).

X = x−

dx ⎡ ⎛ d x ⎞ ⎢1 + dy ⎢⎣ ⎜⎝ dy ⎟⎠ d2 x dy 2 2

2

⎤ ⎥ ⎥⎦

= 1−

( −1)(1 + 1) =0 −2

⎛ dx ⎞ 1+ ⎜ ⎟ ⎝ dy ⎠ (1 + 1) Y = y+ = 1+ =0 2 −2 d x dy 2

Engineering Mathematics

3.56

At the point (1, 1),

3

⎡ ⎛ dx ⎞ 2 ⎤ 2 3 ⎢1 + ⎜ ⎟ ⎥ ⎢⎣ ⎝ dy ⎠ ⎥⎦ [1 + ( −1) 2 ]2 r= = =− 2 −2 d 2x dy 2 Hence, the equation of the circle of curvature at the point (1, –1) is given by, ( x − X 2 ) + ( y − Y )2 = r 2 ( x − 0) 2 + ( y − 0) 2 = ( − 2 ) 2 x2 + y2 = 2 2 2 Thus, the parabolas y = − x + x + 1 and x = − y + y + 1 have the same circle of curvature at the point (1, 1).

Example 5: Find the evolute of the parabola y 2 = 4ax . y 2 = 4 ax

Solution: Differentiating w.r.t. x, 2y

dy = 4a dx dy 2a = dx y

Differentiating again w.r.t. x, d2 y 2 a dy 4a 2 =− 2 =− 3 2 dx y dx y Let (X, Y ) be the centre of curvature.

X = x−

2 dy ⎡ ⎛ dy ⎞ ⎤ + 1 ⎢ ⎥ dx ⎢⎣ ⎜⎝ dx ⎟⎠ ⎥⎦

d2 y dx 2

2a ⎛ 4 a 2 ⎞ ⎜1 + 2 ⎟ y ⎝ y ⎠ = x− 2 4a − 3 y

y 2 + 4 a 2 2ax + 4 ax + 4 a 2 = 2a 2a = 3 x + 2a = x+

2

2

4a ⎛ dy ⎞ 1+ 2 1+ ⎜ ⎟ y d x Y = y + ⎝2 ⎠ = y + dy 4a2 − 3 2 dx y = y−

y( y 2 + 4 a 2 ) y3 = − 4a2 4a2

… (1)

Differential Calculus II

3

3.57

3

( 4 ax ) 2 2x 2 =− = − 1 4a 2 a2

… (2)

From Eq. (1), X − 2a 3

x= From Eq. (2), Y2 =

4 x 3 4 ⎛ X − 2a ⎞ = ⎜ a a ⎝ 3 ⎟⎠

3

27aY 2 = 4( X − 2a)3

This is the required evolute of the parabola y 2 = 4 ax. x2 y2 + = 1. a 2 b2

Example 6: Find the evolute of the ellipse x2 y2 + =1 a2 b2

Solution: Differentiating w.r.t. x,

2 x 2 y dy + =0 a 2 b 2 dx dy b2 x =− 2 dx a y Differentiating again w.r.t. x, dy ⎞ ⎛ y−x ⎟ d2 y b2 ⎜ dx =− 2⎜ ⎟ dx 2 a ⎜ y2 ⎟ ⎜ ⎟ ⎝ ⎠

=− =−

⎛ b2 x ⎞⎤ b2 ⎡ b2 ⎛ b2 x 2 + a2 y 2 ⎞ y − x − = − ⎢ ⎥ ⎜ ⎟ ⎜ ⎟ 2 a2 y 2 ⎣ a2 y 2 ⎝ a2 y ⎝ a y ⎠⎦ ⎠

b2 ⎛ a2b2 ⎜ a2 y 2 ⎝ a2 y

⎞ b4 ⎟= − 2 3 a y ⎠

Let (X, Y ) be the centre of curvature. 2 dy ⎡ ⎛ dy ⎞ ⎤ b2 x ⎛ b4 x 2 ⎞ ⎢1 + ⎜ ⎟ ⎥ − 2 ⎜1 + 4 2 ⎟ dx ⎢⎣ ⎝ dx ⎠ ⎥⎦ a y⎝ a y ⎠ X = x− = x− 2 d y b4 − 2 3 2 dx a y x x ( a 4 y 2 + b 4 x 2 ) = x − 4 2 [a 2 b 2 ( a 2 − x 2 ) + b 4 x 2 ] 2 ab ab a2 − b2 3 x = a4 = x−

4

… (1)

Engineering Mathematics

3.58

2

b4 x 2 ⎛ dy ⎞ 1+ 4 2 1+ ⎜ ⎟ a y dx Y = y + ⎝2 ⎠ = y + b4 d y − 2 3 2 a y dx y y ( a 4 y 2 + b 4 x 2 ) = y − 2 4 [a 4 y 2 + b 2 a 2 (b 2 − y 2 )] 4 ab ab b2 − a2 3 … (2) = y b4 = y−

2

From Eq. (1), 1

⎛ a4 X ⎞ 3 x=⎜ 2 2 ⎟ ⎝ a −b ⎠ From Eq. (2), 1

⎛ b 4Y ⎞ 3 y=⎜ 2 2 ⎟ ⎝b −a ⎠ Substituting in equation of the ellipse, 2

2

1 ⎛ a 4 X ⎞ 3 1 ⎛ b 4Y ⎞ 3 ⎜ ⎟ + 2⎜ 2 ⎟ =1 a2 ⎝ a2 − b2 ⎠ b ⎝ b − a2 ⎠ 2

2

2

( aX ) 3 + (bY ) 3 = ( a 2 − b 2 ) 3 This is the required evolute of the ellipse

x2 y2 + = 1. a2 b2

Example 7: Find the evolute of the astroid x = a cos 3 q , y = a sin 3 q . Solution:

x = a cos3 q

dx = −3a cos 2 q sin q dq

y = a sin 3 q dy = 3a sin 2 q cos q dq

dy dy /d q 3a sin 2 q cos q = = = − tan q dx d x /d q −3a cos 2 q sin q

sec 2 q sec 4 q cosec q d2 y dq 2 = − sec q = − = 3a dx −3a cos 2 q sin q dx 2

Differential Calculus II

3.59

Let (X, Y ) be the centre of curvature.

X = x−

dy ⎡ ⎛ dy ⎞ ⎢1 + dx ⎢⎣ ⎜⎝ dx ⎟⎠

2

⎤ ⎥ ⎥⎦

d 2y dx 2 3a tan q (1 + tan 2 q ) = a cos3 q + sec 4 q cosec q = a cos3 q + 3a sin 2 q cos q

… (1)

2

⎛ dy ⎞ 1+ ⎜ ⎟ dx Y = y + ⎝2 ⎠ d y dx 2 3a(1 + tan 2 q ) = a sin 3 q + sec 4 q cosec q = a sin 3 q + 3a cos 2 q sin q

… (2)

Adding Eqs (1) and (2), X + Y = a (cosq + sin q )3 1

1

( X + Y ) 3 = a 3 (cosq + sin q )

… (3)

Subtracting Eqs (2) from (1), X − Y = a(cosq − sin q )3 1

1

( X − Y ) 3 = a 3 (cosq − sin q )

… (4)

Squaring and adding Eqs (3) and (4), 2

2

2

( X + Y ) 3 − ( X − Y ) 3 = 2a 3 This is the required evolute of the astroid x = a cos 2 q , y = a sin 3 q . Example 8: Find the evolute of the curve x = a (cos p + p sinp ), y = a (sin p – p cosp ). Solution:

x = a(cosq + q sin q ) dx = a [− sin q + (q cos q + sin q )] = aq cos q dq y = a (sin q − q cos q ) dy = a [cosq q − ( −q sin q + cos q )] = aq sin q dq

dy d y / dq aq sin q = = = tan q dx d x /dq aq cos q d 2y dq 1 sec 2 q 2 = = = sec q 2 dx aq cos q aq cos3 q dx

Engineering Mathematics

3.60

Let (X, Y ) be the centre of the curvature.

X = x−

dy ⎡ ⎛ dy ⎞ ⎢1 + dx ⎢⎣ ⎜⎝ dx ⎟⎠ d2 y dx 2

2

⎤ ⎥ ⎥⎦ = a(cosq + q sin q ) − tan q (1 + tan 2 q )( aq cos3 q )

= a cos q

… (1) 2

⎛ dy ⎞ 1+ ⎜ ⎟ dx Y = y + ⎝2 ⎠ d y dx 2 = a(sin q − q cos q ) + (1 + tan 2 q ) ( aq cos3 q ) = a sinq

… (2)

From Eqs (1) and (2), X 2 + Y 2 = a2

This is the equation of circle which is required evolute of the curve. t⎞ ⎛ Example 9: Show that the evolute of the tractrix x = a ⎜ cos t + log tan ⎟ , ⎝ 2⎠ x y = a sin t is the catenary y = a cosh . a t⎞ ⎛ Solution: x = a ⎜ cos t + log tan ⎟ y = a sin t 2⎠ ⎝ dy ⎛ ⎞ = a cos t ⎜ dx 1 1 2t⎟ dt = a ⎜ − sin t + ⋅ sec ⎟ t 2 dt 2⎟ ⎜⎜ tan ⎟ ⎝ 2 ⎠

⎛ ⎞ ⎜ ⎟ 1 = a ⎜ − sin t + ⎟ t t ⎜⎜ 2 sin cos ⎟⎟ ⎝ 2 2⎠ 1 ⎞ ⎛ = a ⎜ − sin t + sin t ⎟⎠ ⎝ =a

cos 2 t sin t

dy dy /dt a cos t = = = tan t dx dx /dt cos 2 t a sin t d 2y sec 2 t sin t sin t 2 dt = sec t = = 2 2 dx dx a cos t a cos 4 t

Differential Calculus II

3.61

Let (X, Y ) be the centre of curvature.

X = x−

2 dy ⎡ ⎛ dy ⎞ ⎤ + 1 ⎢ ⎥ dx ⎢⎣ ⎜⎝ dx ⎟⎠ ⎥⎦ 2

dy dx 2

= x−

tan t (1 + tan 2 t ) sin t a coss 4 t

t⎞ ⎛ = a ⎜ cos t + log tan ⎟ − a cos t 2⎠ ⎝ t = a log tan 2

… (1)

2

⎛ dy ⎞ 1+ ⎜ ⎟ 1 + tan 2 t a cos 2 t dx Y = y + ⎝2 ⎠ = y + = a sin t + sin t sinn t d y 4 2 a cos t dx a = sin t From Eqs (1) and (2), tan

X t =ea 2

and

2 tan But

sin t =

sin t =

a Y

t 2

1 + tan 2

t 2

X

a 2e a = 2X Y 1+ e a 2X ⎛ a ⎜1+ e a Y = ⎜ X 2⎜ ⎝ ea X = a cosh h a

⎞ X X ⎟ = a ⎛ e− a + e a ⎞ ⎜ ⎟ ⎟⎟ 2 ⎝ ⎠ ⎠

x Hence, the required evolute of the tactrix is y = a cosh . a Example 10: Show that the evolute of the cycloid x = a(p – sin p ), y = a (1 – cosp ) is another cycloid x = a(q + sin q ), y = - a(1 - cos q ) . Solution:

x = a(q − sin q ) dx = a(1 − cos q ) dq

y = a (1 – cos q ) dy = a sin q dq

… (2)

Engineering Mathematics

3.62

dy dy/ dq a sin q = = = dx dx / dq a(1 − cos q )

q q cos 2 2 = cot q q 2 2 sin 2 2

2 sin

d 2y 1 q dq = − cosec 2 2 2 2 dx dx q ⎡ 1 1 ⎤ = − cosec2 ⎢ =− 2 2 ⎣ a(1 − cos q ) ⎥⎦

cosec 4

q 2

4a

Let (X, Y ) be the centre of curvature.

X = x−

2 dy ⎡ ⎛ dy ⎞ ⎤ + 1 ⎢ ⎥ dx ⎢⎣ ⎜⎝ dx ⎟⎠ ⎥⎦

d2 y dx 2

cot = a(q − sin q ) +

q⎛ 2q ⎞ ⎜1 + cot ⎟ 4 a 2⎠ 2⎝ q cosec 4 2

q q sin 2 2 2 q q = a(q − sin q ) + 4 a sin cos 2 2 = a(q − sin q ) + 2a sin q = a(q + sin q ) = a(q − sin q ) + 4 a cot

2

⎛ dy ⎞ ⎛ 2q ⎞ 1+ ⎜ ⎟ ⎜1 + cot ⎟ 4 a dx ⎠ ⎝ ⎝ 2⎠ Y = y+ = a(1 − cos q ) − d2 y 4 q cos ec 2 dx 2 q = a(1 − cos q ) − 4a sin 2 2 = a(1 − cos q ) − 2a(1 − cos q ) = −a + a cos q = −a(1 − cos q )

Hence, the required evolute of the cycloid is x = a (q + sin q ), y = – a (1 – cos q ). Exercise 3.4 1. Find the centre of curvature of the following curves: ⎛1 1⎞ (i) y = x 3 at ⎜ 2 , 8 ⎟ ⎝ ⎠

2 (ii) y = x + 9 at (3, 6) x 3 3 (iii) x + y = 2a3 at (a, a)

Differential Calculus II

x2 y2 + = 2 at (3, 2) 9 4 3 3 ⎛ 3a 3a ⎞ (v) x + y = 3axy at ⎜ , ⎟ ⎝ 2 2 ⎠ x (vi) y = c cosh at (x, y) c 2 (vii) xy = c at (c, c) (viii) y 3 = a 2 x at (x, y) (ix) x = 3t , y t 2 6 at (a, b) (x) x (1 at ) cos t a sin t , y (1 at ) sin t a cos t. (iv)

⎤ ⎡ Ans. : ⎥ ⎢ ⎛ 7 31 ⎞ ⎛ 15 ⎞ (ii) ⎜ 3, ⎟ ⎥ ⎢ ( i) ⎜ , ⎟ ⎝ 64 48 ⎠ ⎝ 2⎠ ⎥ ⎢ ⎥ ⎢ a a ⎛ 5 −5 ⎞ ⎥ ⎢ (iii) ⎛⎜ , ⎞⎟ (iv) ⎜ , ⎟ ⎥ ⎢ ⎝6 4 ⎠ ⎝2 2⎠ ⎥ ⎢ ⎥ ⎢ ( v) ⎛ 21a , 21a ⎞ ⎜ 16 16 ⎟ ⎥ ⎢ ⎝ ⎠ ⎥ ⎢ 2 2 ⎛ ⎞ ⎥ ⎢ ( vi) ⎜ x − y y − c , 2 y ⎟ ⎥ ⎢ ⎜ ⎟ c ⎝ ⎠ ⎥ ⎢ ⎥ ⎢ ( vii) ( 2c, 2c) ⎥ ⎢ 4 4 4 5 ⎛ y a y + 9y ⎞ ⎥ ⎢ ( viii) ⎜ a + 15 , ⎟ 2 4 2a ⎥ ⎢ ⎝ 6a y ⎠ ⎥ ⎢ 2 ⎡ ⎤ ( 81 + 4 a ) ⎢ (ix) ⎢ −4 a( 20 + a 2 ), b + ⎥ ⎥ ⎢ 18 ⎣ ⎦⎥ ⎢ (x) ( a sin t , − a cos t ) ⎥ ⎥⎦ ⎣⎢ 2. Find n so that the centre of curvature of the curve y = x n at the point (1, 1) lies on the line y = 2. [Ans. : n = −1 ] 3. Show that the centre of curvature at the point P ( a, a) of the curve x 4 + y 4 = 2a 2 xy divides the line OP in the ratio 6:1, O being the origin of co-ordinates. 4. Find the co-ordinates of the centre of curvature at the origin for the curve 5 x 4 − ax 2 y − axy 2 + a 2 y 2 = 0. 2 ⎡ ⎛ a ⎞⎤ ⎢ Ans. : (0, a) and ⎜ 0, 4 ⎟ ⎥ ⎝ ⎠⎦ ⎣

3.63

5. Find the circle of curvature of the following curves: (i) 2 xy + x + y = 4 at (1, 1) (ii) y 2 = 4 ax at (at2, 2at) (iii) x 3 + y 3 = 2 xy at (1, 1) (iv) y = x 3 + 2 x 2 + x + 1 at (0, 1) (v) xy(x + y) = 2 at (1, 1). ⎡ Ans. : 2 2 ⎢ ⎢ (i) ⎛⎜ x − 5 ⎞⎟ + ⎛⎜ y − 5 ⎞⎟ = 9 2⎠ ⎝ 2⎠ 2 ⎢ ⎝ ⎢ (ii) x 2 + y 2 − 6 at 2 x − 4 ax + 4 at 3 y ⎢ = 3a 2 t 4 ⎢ 2 2 2 ⎢ ⎛ ⎞ ⎢ (iii) ⎛ x − 7 ⎞ + ⎛ y − 7 ⎞ = ⎜ 2 ⎟ ⎜ ⎢ 8 ⎟⎠ ⎜⎝ 8 ⎟⎠ ⎜⎝ 8 ⎟⎠ ⎝ ⎢ 2 2 ⎢ (iv) x + y + x − 3 y + 2 = 0 2 2 ⎢⎣ (v) x + y + 5 x − 5 y + 8 = 0

⎤ ⎥ ⎥ ⎥ ⎥ ⎥ ⎥ ⎥ ⎥ ⎥ ⎥ ⎥ ⎥⎦

6. Find the evolute of the following curves: 2 2 (i) x − y = 1 a2 b2 (ii) 2 xy = a 2 3 (iii) x = 4 ay

(iv) x 2 − y 2 = a 2 (v) x = (1 − aq ) cos q + a sin q , y = (1 − aq ) sin q − a cos q (vi) x = a cosh u, y = b sinh u (vii) x = a cot 2 q , y = 2a cotq . ⎡ Ans. : ⎤ ⎢ 2 2 2 ⎥ ⎢ (i) ( aX ) 3 − (bY ) 3 = ( a 2 + b 2 ) 3 ⎥ ⎢ ⎥ 2 2 2 ⎢ ⎥ 3 3 3 ⎢ (ii) ( X + Y ) + ( X − Y ) = 2a ⎥ ⎢ (iii) 4(Y − 2a)3 = 27aX 2 ⎥ ⎢ ⎥ 2 2 2 ⎢ ⎥ ⎢ (iv ) X 3 − Y 3 = ( 2a) 3 ⎥ ⎢ ⎥ 2 2 2 ⎢ ( v) X + Y = a ⎥ 2 2 2⎥ ⎢ 2 2 ⎢ ( vi) ( aX ) 3 − (bY ) 3 = ( a + b ) 3 ⎥ ⎢ ⎥ 2 3 ⎣ ( vii) 27aY = 4( X − 2a) ⎦

Engineering Mathematics

3.64

7. Show that the evolute of the deltoid x = 2 cost + cos 2t, y = 2 sint – sin 2t is another deltoid three times the size of the given deltoid and has the equations. X = 3( 2 cos t − cos 2t ), Y = 3( 2 sin t + sin 2t ) . 8. Show that the evolute of an equian-

gular spiral is an equal equiangular spiral. 9. Show that the radii of curvatures of the curve x = axq (sin q − cos q ), y = aeq (sin q + cos q ) and its evolutes at corresponding points are equal. 10. Prove that normals to a curve are the tangents to its evolute.

3.7 ENVELOPES Consider an equation f ( x, y, a ) = 0. For different values of a we get different curves. This equation represents a one parameter family of curves with a as the parameter. Example: 1 (i) The equation y = mx + represents a family of straight lines where m is the m parameter. (ii) The equation x 2 + y 2 − 2ax = 0 represents a family of circles with their centres on x-axis and which pass through the origin. Here, a is the parameter. In a similar way, a two parameter family of curves is represented by the equation f ( x, y, a , b ) = 0 where and b are the parameters. Example: x2 y2 The equation 2 + 2 = 1 represents a family of ellipse, where a and b are the a b parameters. Envelope of a given family of curves is a curve which touches each member of a family of curves and at each point is touched by some member of the family of curves. A family of curve may have no envelope or unique envelope or several envelopes. Envelope may also be defined as the locus of the limiting positions of the points of intersection of one member of the family with a neighbouring member when one of them tends to coincide with the other which is kept fixed. Determination of envelope (i) The equation of the envelope of the family of curves f ( x, y, a ) = 0, where a is the parameter, is obtained by eliminating between the equations f ( x, y, a ) = 0

where

and

∂ f ( x, y, a ) = 0 ∂a

f is the partial derivative of f w.r.t. a . a

Differential Calculus II

3.65

(ii) For two parameter family of curves f ( x, y, a , b ) = 0 with relation g(a , b ) = 0 between the parameters a and b , the equation of the envelope of the family of curves is obtained by (a) Writing one of the parameter, say b , in terms of a . (b) Using this to reduce the equation of two parameter family of curves into one parameter family of curves. (c) Proceeding as in step (i). (iii) Envelope of the family of normals to a given curve is the evolute of the curve. Example 1: Find the envelope of the family of lines y = mx + am3 , where m is the parameter. y = mx + am3

Solution: Differentiating partially w.r.t. m,

… (1)

0 = x + 3 am 2 1

⎛ x ⎞2 m = ⎜− ⎟ ⎝ 3a ⎠

… (2)

Substituting in the Eq. (1), 1

3

⎛ x ⎞2 ⎛ x ⎞2 y = ⎜− ⎟ x + a⎜− ⎟ ⎝ 3a ⎠ ⎝ 3a ⎠ 3

⎛ x ⎞ ⎛ x ⎞ ⎛ x ⎞ y 2 = ⎜ − ⎟ x 2 + a 2 ⎜ − ⎟ + 2ax ⎜ − ⎟ ⎝ 3a ⎠ ⎝ 3a ⎠ ⎝ 3a ⎠

2

27ay 2 = −9 x 3 + ( − x )3 + 6 x 3 27ay 2 = −4 x 3

This is the equation of the required envelope. Example 2: Find the envelope of the family of lines y = mx + a 2 m2 + b 2 , where m is the parameter. Solution:

y = mx + a 2 m 2 + b 2

… (1)

( y − mx ) 2 = a 2 m 2 + b 2 y 2 + m 2 x 2 − 2mxy = a 2 m 2 + b 2 ( x 2 − a 2 )m 2 − 2mxy + ( y 2 − b 2 ) = 0 Differentiating partially w.r.t. m, 2m( x 2 − a 2 ) − 2 xy = 0 m=

xy x − a2 2

… (2)

… (3)

Engineering Mathematics

3.66

Substituting in Eq. (2), ( x 2 − a2 )

x2 y2 x2 y2 − 2 + ( y 2 − b2 ) = 0 ( x 2 − a2 )2 x 2 − a2 x2 y2 2x2 y2 − 2 + ( y 2 − b2 ) = 0 2 2 x −a x − a2

x2 y2 = y 2 − b2 x 2 − a2 x 2 y 2 = ( x 2 − a 2 )( y 2 − b 2 ) = x 2 y 2 − b 2 x 2 − a 2 y 2 + a 2 b 2 x2 y2 + =1 a2 b2 This is the equation of the required envelope. x y Example 3: Find the envelope of the family of curves cos t + sin t = 1, where a b t is the parameter.

Solution:

x y cos t + sin t = 1 a b

… (1)

Differentiating partially w.r.t. t, x y − sin t + cos t = 0 a b Squaring Eqs (1) and (2) and adding,

… (2)

2

2

y y ⎞ ⎛ x ⎞ ⎛x ⎜ a cos t + b sin t ⎟ + ⎜ − a sin t + b cos t ⎟ = 1 ⎝ ⎠ ⎝ ⎠ x2 y2 (cos 2 t + sin 2 t ) + 2 (sin 2 t + cos 2 t ) = 1 2 a b x2 y2 + =1 a2 b2 This is the equation of the required envelope. Example 4: Find the envelope of the family of circles x2 + y2 – 2ax cos ` – 2ay sin ` = c2, where a is the parameter. Solution:

x 2 + y 2 − 2ax cos a − 2ay sin a = c 2 2ax cos a + 2ay sin a = x 2 + y 2 − c 2

Differentiating partially w.r.t. a , −2ax sin a + 2ay cos a = 0 Squaring Eqs (1) and (2) and adding, 4a 2 ( x 2 + y 2 ) = ( x 2 + y 2 − c 2 )2 This is the equation of the required envelope.

… (1) … (2)

Differential Calculus II

3.67

Example 5: Find the envelope of the system of straight lines 2 y - 3tx + at 3 = 0 , where t is the parameter. 2 y − 3tx + at 3 = 0

Solution:

… (1)

Differentiating partially w.r.t. t, −3 x + 3at 2 = 0 x t2 = a Substituting in Eq. (1), 2y – 3t x + at Substiuting in Eq. (1),

x =0 a y t= x

y3 ⎛ y⎞ 2y − 3⎜ ⎟ x + a 3 = 0 ⎝x⎠ x ay 2 = x 3 This is the equation of the required envelope.

Example 6: Find the envelope of family of parabolas

x + a

y = 1, where the b

parameters a and b are connected by the relation a + b = c. Solution: x y + =1 a b a+b = c

But

x y + =1 a c−a

Differentiating w.r.t. a,

… (1)

⎛ 1⎞ 1 ⎛ 1⎞ 1 x ⎜− ⎟ 3 + y ⎜− ⎟ (−1) = 0 3 ⎝ 2⎠ 2 ⎝ 2⎠ 2 (a) (c − a ) 3

1

⎛ c − a ⎞2 ⎛ y ⎞2 ⎜ ⎟ =⎜ ⎟ ⎝ a ⎠ ⎝x⎠ 1

c − a ⎛ y ⎞3 =⎜ ⎟ a ⎝x⎠ 1

1

c x3 + y3 = 1 a x3 1

a=

cx 3 1

1

x3 + y3

… (2)

Engineering Mathematics

3.68

Substituting Eq. (2) in Eq. (1), 1 ⎡ ⎛ 13 ⎢x ⎜ x + y3 1 ⎢ ⎜ ⎢⎣ ⎝ cx 3

(

1

⎞⎤ 2 1 ⎟⎥ + ⎡ y ⎧ ⎢ ⎪ 1 ⎥ ⎟ 3 ⎠ ⎥ ⎢ ⎨⎪ ⎛ ⎦ ⎢ ⎜ c − cx 1 1 ⎪ ⎢ ⎪ ⎜⎝ 3 3 x y + ⎣ ⎩

)

1

(

1

⎫⎤ 2 ⎥ =1 ⎞ ⎪⎪⎥ ⎟ ⎬⎥ ⎟ ⎪⎥ ⎠ ⎪⎭⎦

)

1

1 1 1 ⎡ 32 13 ⎤2 ⎡ 2 1 ⎤2 ⎢x x + y3 ⎥ + ⎢ y3 x3 + y3 ⎥ = c2 ⎣ ⎦ ⎣ ⎦ 1

1 2 ⎛ 13 ⎞ ⎜ x + y3 ⎟ ⎝ ⎠

1 ⎡ 2 12 2 2⎤ 1 ⎛ ⎞ ⎛ ⎞ ⎢ x3 + y3 ⎥ = c2 ⎜ ⎟ ⎜ ⎟ ⎢ ⎥ ⎢⎣⎝ ⎠ ⎝ ⎠ ⎥⎦ 1

1

1

x3 + y3 = c3 This is the equation of the required envelope. Example 7: Find the envelope of the family of ellipses

x2 y2 + = 1 , where the a 2 b2

parameters a and b are connected by the relation ab = c2. Solution: x2 y2 + =1 a2 b2 ab = c 2

But

x 2 a2 y 2 + 4 =1 a2 c

… (1)

Differentiating w.r.t. a, −

y2 2x2 a + 2 =0 a3 c4 x2 y2 = a4 c4 x2 a4 = 2 c4 y a2 =

x 2 c y

… (2)

Differential Calculus II

3.69

Substituting Eq. (2) in Eq. (1), x 2 2 c y x2 y + =1 x 2 c4 c y xy xy + =1 c2 c2 2 xy =1 c2 2 xy = c 2 This is the equation of the required envelope. x y + = 1 , where a b the parameters a and b are connected by the relation a 2 + b 2 = c 2 .

Example 8: Find the envelope of the family of straight lines

Solution: x y + =1 a b

… (1)

Differentiating w.r.t. a, −

x y db − 2 =0 2 a b da

… (2)

a 2 + b2 = c2

… (3)

Also, Differentiating w.r.t. a,

2 a + 2b

Substituting Eq. (4) in Eq. (2), x y − 2− 2 a b

db =0 da db a =− da b

… (4)

⎛ a⎞ ⎜⎝ − ⎟⎠ = 0 b x y = a3 b3 x y a = b = a2 b2

x=

a3 c2

and

y=

1

a = (c 2 x ) 3

⎛x⎞ ⎛ y⎞ ⎜⎝ ⎟⎠ + ⎜⎝ ⎟⎠ 1 a b = 2 a2 + b2 c

b3 c2 1

and

b = (c 2 y ) 3

… (5)

Engineering Mathematics

3.70

Substituting Eq. (5) in Eq. (3), 2

2

(c 2 x ) 3 + (c 2 y ) 3 = c 2 2

2

2

x3 + y3 = c3 This is the equation of the required envelope. x2 y2 + = 1, where the a 2 b2 parameters a and b are connected by the relation a 2 + b 2 = c. Example 9: Find the envelope of the family of ellipses

Solution:

x2 y2 + =1 a2 b2

… (1)

Differentiating w.r.t. a,

− Also

2 x 2 2 y 2 db − 3 =0 a3 b da a 2 + b2 = c

… (2) … (3)

Differentiating w.r.t. a, 2 a + 2b

db =0 da db a =− da b

… (4)

Substituting Eq. (4) in Eq. (2), −

2x2 2 y2 − 3 a3 b

⎛ a⎞ ⎜⎝ − ⎟⎠ = 0 b

x2 y2 = a4 b4 2 2 x2 y2 ⎛ x ⎞ ⎛ y ⎞ + ⎜ ⎟ ⎜ ⎟ 2 2 a2 = b2 = ⎝ a ⎠ ⎝ b ⎠ = 1 c a2 b2 a2 + b2 2 2 x 1 y 1 = and = a4 c b4 c a 2 = cx Substituting Eq. (5) in Eq. (3),

and

b2 = c y

… (5)

cx + c y = c x+ y = c This is the equation of the required envelope. Example 10: Considering the evolute of a curve as the envelope of its normals, find the evolute of the parabola y 2 = 4ax. Solution:

y 2 = 4ax

Differential Calculus II

3.71

Equation of normal to the parabola is y = mx − 2am − am3 , where m is the parameter. Differentiating partially w.r.t. m, 0 = x − 2a − 3am 2 1

⎛ x − 2a ⎞ 2 m=⎜ ⎝ 3a ⎟⎠ Substituting in equation of the normal, y = m( x − 2a − am 2 ) 1

⎛ x − 2a ⎞ 2 y=⎜ ⎝ 3a ⎟⎠

x − 2a ⎞ ⎛ ⎜⎝ x − 2a − a ⎟ 3a ⎠

1

⎛ x − 2a ⎞ 2 ⎛2⎞ y=⎜ ( x − 2a) ⎜ ⎟ ⎝ 3a ⎟⎠ ⎝3⎠ 27 y 2 = 4( x − 2a)3 This is the equation of the required evolute. Example 11: Considering the evolute of a curve as the envelope of its normals, x2 y2 find the evolute of the ellipse 2 + 2 = 1. a b x2 y2 Solution: … (1) + =1 a2 b2 Equation of normal at any point (a cos , b sin ) on the ellipse is ax by a 2 b 2 , where is the parameter. … (2) cos sin Differentiating partially w.r.t. , ax sin q by cos q + =0 cos 2 q sin 2 q tan 3 q = −

by ax 1

⎛ by ⎞ 3 tanq = − ⎜ ⎟ ⎝ ax ⎠ 1

sin q =

−(by ) 3 1

2 2 2 ⎡ ⎤ 3 3 ( ) ( ) + ax by ⎢ ⎥ ⎣ ⎦ 1

and

cos q =

( ax ) 3 1

2 2 2 ⎡ ⎤ 3 3 ⎢( ax ) + (by ) ⎥ ⎣ ⎦

Engineering Mathematics

3.72

Substituting in Eq. (2), 1

2 2 2 ⎡ ⎤ ax ⎢( ax ) 3 + (by ) 3 ⎥ ⎣ ⎦

ax

1 3

1

+

2 2 2 ⎡ ⎤ by ⎢( ax ) 3 + (by ) 3 ⎥ ⎣ ⎦

(by )

1 3

= a2 − b2

1 2

2 2 2 2 ⎡ ⎤ ⎡ ⎤ 2 2 3 3 3 3 ⎢( ax ) + (by ) ⎥ ⎢( ax ) + (by ) ⎥ = a − b ⎣ ⎦ ⎣ ⎦ 3

2 2 2 ⎤ ⎡ 2 2 3 3 ⎣⎢( ax ) + (by ) ⎦⎥ = ( a − b ) 2

2

2

( ax ) 3 + (by ) 3 = ( a 2 − b 2 ) 3 This is the equation of the required evolute. Example 12: Find the envelope of the straight lines drawn at right angles to the radii vectors of the spiral r = aeq cot a through their extremities. Solution: Let P ( R, f ) be any point on the spiral

y

r = aeq cot a

Q(r, q )

... (1) R = aef cot a Let Q (r, ) be any point on the line PQ, which is drawn through the extremity P of the radius vector OP and is at right angles to the radius vector. From OPQ, R = r cos(q − f )

... (2)

From Eq. (1) and (2), aef cot a = r cos(q − f ),

... (3)

Taking logarithm on both the sides, log a + φ cot α = log r + log cos(θ − φ ) Differentiating partially w.r.t. , 1 sin(q − f ) cos(q − f )

⎛p ⎞ tan ⎜ − a ⎟ = tan(q − f ) ⎝2 ⎠ π −α = θ −φ 2 π φ = θ +α − 2

P (R, f )

q−f R f q

x

O

where is the parameter.

cot a =

r

Fig. 3.10

Differential Calculus II

3.73

Substituting in Eq. (3), π⎞ ⎛ ⎜ θ +α − ⎟ cot α 2⎠

ae⎝

⎛π ⎞ = r cos ⎜ − α ⎟ ⎝2 ⎠

⎛ π⎞ ⎜α − ⎟

ae⎝ 2 ⎠ eθ cot α = r sin α This is the equation of the required envelope. Example 13: Find the envelope of the straight lines drawn at right angles to the radii vectors of the cardioid r = a(1 + cos p ) through their extremities. Solution: Let P( R, ) be any point on the cardioid r = a (1 + cos ) R = a(1 + cos f )

… (1)

Let Q(r , ) be any point on the line PQ, which is drawn through the extremity P of the radius vector OP and is at right angles to the radius vector. From OPQ R = r cos(q − f ) ... (2)

y Q(r, q )

r

a(1+ cos f ) = r cos(q − f ) ,

tan f =

R

... (3)

where is the parameter. Differentiating partially w.r.t. , − a sin f = r sin(q − f ) r sin q cos f − ( r cos q − a) sin f = 0

f q x

O

Fig. 3.11

r sin q r cos q − a

r sin q

sin f = cos f =

and

P(R, f )

q−f

From Eq. (1) and (2),

r + a 2 − 2ar cos q 2

r cos q − a r + a 2 − 2ar cos q 2

Substituting in Eq. (3), a (1+ cos φ ) = r (cos θ cos φ + sin θ sin φ ) ( r cos q − a) cos f + r sin q sin f = a ( r cos q − a) 2 + r 2 sin 2 q r 2 + a 2 − 2ar cos q

=a

r 2 + a 2 − 2ar cosq = a r 2 + a 2 − 2ar cosq = a 2 r = 2a cosq This is the equation of the required envelope.

[Substituting cosf and sinf ]

Engineering Mathematics

3.74

Example 14: Find the envelope of the circles drawn upon the radii vectors of the ellipse

x2 y2 + = 1 as diameter. a 2 b2

Solution: Any point on the ellipse in the parameteric form is P ( a cos q , b sin q ) with as the parameter. Hence, equation of the circle on the radius vector to this point as diameter is given by x 2 + y 2 − ax cos q − by sin q = 0

... (1)

Differentiating partially w.r.t. , ax sin q − by cos q = 0 tanq = sinq = cosq =

and

by ax by a x + b2 y 2 2

2

ax a x + b2 y 2 2

2

Substituting in Eq. (1), x2 + y2 −

a2 x 2 a2 x 2 + b2 y 2



b2 y 2 a2 x 2 + b2 y 2

=0

x 2 + y 2 − a2 x 2 + b2 y 2 = 0 ( x 2 + y 2 )2 = a2 x 2 + b2 y 2 This is the equation of the required envelope. Exercise 3.5 1. Find the envelope of the following family of curves: 1 (i) y = mx + , the parameter being m. m (ii) y = mx + 1 + m 2 , the parameter being m. (iii) y = mx − 2am − am3 , the parameter being m. (iv) x cos q + y sin q = c sin q cos q , the parameter beingq . (v) x tan q + y secq = c, the parameter being . (vi) x sin q − y cos q = aq , the parameter being .

(vii) x cosecq − y cot q = c, the parameter being . ⎡ Ans. : ⎤ ⎢ ⎥ 2 ( ) i y = 4x ⎢ ⎥ ⎢ ⎥ (ii ) x 2 + y 2 = 1 ⎢ ⎥ ⎢ (iii ) 27ay 2 = 4( x − 2a)3 ⎥ ⎢ ⎥ 2 2 2 ⎢ ⎥ (iv ) x 3 + y 3 = a 3 ⎢ ⎥ 2 2 2 (v) y = a + x ⎢ ⎥ ⎢ ⎥ ( ) x a a = cos q + q sin q , vi ⎢ ⎥ ⎢ y = a sin q − aq cos q ⎥ ⎢ ⎥ ⎢⎣ ( viii ) x 2 − y 2 = c 2 ⎥⎦

Differential Calculus II

2. Find the envelope of the family of x y straight lines + = 1, where the a b parameters a and b are connected by the relation. (i) a + b = c

(ii) ab = c 2 .

⎡ Ans. : (i ) x + y = c ⎤ ⎢ ⎥ (ii ) 4 xy = c 2 ⎢⎣ ⎥⎦ 3. Find the envelope of the family x2 y2 of curves + = 1, where the y 2 b2 parameters a and b are connected by the relation (i) a + b = c

(ii) a + b = c . 2

2

2

2 2 2 ⎡ ⎤ 3 3 3 ( ) Ans. : i + = x y c ⎢ ⎥ ⎢ (ii ) x ± y ± c = 0 ⎥⎦ ⎣

⎡ ⎢ Ans. : ⎢ ⎢ ⎢ ⎢ ⎢ ⎢ ⎢ ⎢ ⎢ ⎢ ⎢ ⎢ ⎢ ⎢⎣

3.75

⎤ ⎥ 2 ⎥ = (a2 + b2 ) 3 ⎥ ⎥ 2 2 3 3 (ii ) ( x + y ) + ( x − y ) ⎥⎥ 3 ⎥ = ( 2a) 3 ⎥ ⎥ ⎥ (iii ) y = a cosh x ⎥ a ⎥ 2 2 2 (iv ) x 3 + y 3 = ( 4 a) 3 ⎥ ⎥ ( v) x 2 + y 2 = a2 ⎥⎦ 2

2

(i ) ( ax ) 3 + (by ) 3

6. Find the envelope of the straight lines drawn at right angles to the radii vectors of the following curves through their extremities. (i) r = a + b cosq (ii) r n = a n cos nq .

4. Find the envelope of the family of x y + = 1, where the curves a b parameters a and b are connected by

⎡ Ans. : (i ) r 2 − 2br cos q + (b 2 − a 2 ) = 0⎤ ⎢ ⎥ ⎛ n ⎞ ⎛ n ⎞ ⎜⎝ ⎟⎠ ⎜⎝ ⎟⎠ n ⎞ ⎥ ⎛ ⎢ − − 1 n 1 n (ii ) r cos ⎜ =a q ⎢ ⎝ 1− n ⎟⎠ ⎥⎦ ⎣

the relation ab = c 2 . ⎡⎣ Ans. : 16 xy = c 2 ⎤⎦

7. Find the envelopes of the circles described on the radii vectors of the following curves as diameters 2 (i) y = 4ax l (ii) = l + e cos r (iii) r 3 = a 3 cos 3 .

5. Considering the evolute of a curve as the envelope of its normals, find the evolute of the following curves: x2 y2 (ii) xy = c 2 − =1 a2 b2 t⎞ ⎛ (iii) x = ⎜ a cos t + log tan ⎟ , ⎝ 2⎠ (i)

y = a sin t (iv) x = a(3cos t – 2 cos3 t), y = a(3sin t – 2 sin3 t) (v) x = a(cos t + t cos t ), y = a(sin t − t cos t ) .

⎤ ⎡ Ans. : ⎥ ⎢ 2 2 2 ( i ) ay + x( x + y ) = 0 ⎥ ⎢ ⎢ ( ii ) r 2 (e 2 − 1) − 2ler cos + 2l 2 = 0⎥ ⎥ ⎢ 3 3 ⎥ ⎢ 3 ⎛ ⎞ 4 4 ( iii ) r = a cos ⎜ ⎟ ⎥ ⎢ ⎥⎦ ⎝ 4 ⎠ ⎣⎢ 8. Show that family of circles ( x − a ) 2 + y 2 = a 2 has no envelope.

3.76

Engineering Mathematics

3.8 CURVE TRACING Curve tracing is a procedure to obtain an approximate shape of the curve without plotting a large number of points on it. In this chapter, we will study the tracing of cartesian, parametric and polar curves.

3.8.1 Tracing of Cartesian Curves The points to be taken into consideration while tracing a cartesian curve f (x, y) = 0 are as follows: (i) Symmetry: (a) The curve is symmetric about x-axis if the powers of y occurring in the equation are all even i.e., f (x, –y) = f (x, y). (b) The curve is symmetric about y-axis if the powers of x occuring in the equation are all even i.e., f (–x, y) = f (x, y). (c) The curve is symmetric about the line y = x, if on interchanging x and y, the equation remains unchanged i.e., f (y, x) = f (x, y). (d) The curve is symmetric about the line y = –x if on replacing x by –y and y by –x, the equation remains unchanged.i.e., f (–y, –x) = f (x, y) (e) The curve is symmetric in opposite quadrants or about origin if on replacing x by –x by y and –y, the equation remains unchanged i.e., f (–x, –y) = f (x, y). (ii) Origin: The curve passes through the origin if there is no constant term in the equation. (a) If the curve passes through the origin, the tangents at the origin are obtained by equating the lowest degree term in x and y to zero. (b) If there are two or more tangents at the origin, it is called a multiple point. The multiple point is called a node, a cusp or an isolated point if the tangents at this point are real and distinct, real and coincident or imaginary respectively. (iii) Points of Intersection: (a) The points of intersection of the curve with x and y axis are obtained by putting y = 0 and x = 0 respectively in the equation of the curve. (b) Tangent at the point of intersection is obtained by shifting the origin to this point and then equating the lowest degree term to zero. (iv) Region of Existence: This region is obtained by expressing one variable in terms of other, i.e., y = f (x) [or x = f (y)] and then finding the values of x (or y) at which y(or x) becomes imaginary. The curve does not exist in the region which lies between these values of x (or y). (v) Asymptotes: (a) Asymptotes parallel to x-axis are obtained by equating the coefficient of highest degree term of x in the equation to zero. (b) Asymptotes parallel to y-axis are obtained by equating the coefficient of highest degree term of y in the equation to zero. (c) Oblique asymptotes are obtained by the following method: Let y = mx + c is the asymptote to the curve and f2(x, y), f3(x, y) are the second and third degree terms in the equation.

Differential Calculus II

3.77

x = 1 and y = m in f2(x, y) and f3(x, y) f2(x, y) = f2(1, m) or f2(m) f3 (x, y) = f3(1, m) or f3(m) f ( m) Find c =– 2 f3 ( m) Solve f3(m) = 0 m = m1, m2, … Calculate c at m1, m2, … Substituting the values of m and c in y = mx + c, we get oblique asymptotes to the curve. (vi) Interval of Increasing-decreasing Function: dy (a) The curve increases strictly in the interval in which > 0. dx dy (b) The curve decreases strictly in the interval in which < 0. dx Putting

(c) The curve attains its maximum and minimum values at the points where dy = 0. dx Example 1: Trace the cissoid y2 (2a – x) = x3. Solution: (i) Symmetry: The curve is symmetric about x-axis. (ii) Origin: The curve passes through the origin. Equating the lowest degree term i.e. 2ay2 to zero, we get y = 0. Thus, x-axis is the tangent at the origin. (iii) Points of Intersection: Putting y = 0, we get x = 0. Thus, the curve meets the coordinate axes only at the origin. (iv) Region of Existence: From the equation of the curve, y =

x

x 2a

x

which

becomes imaginary when x < 0 or x > 2a. Therefore, the curve does not exist in the region – < x < 0 and 2a < x < . Thus, the curve lies in the region 0 < x < 2a. (v) Asymptotes: (a) Since coefficient of highest degree term of x is constant, there is no asymptote parallel to x-axis. (b) Equating the coefficient of highest degree term of y to zero, we get 2a – x = 0. Thus, x = 2a is the asymptote parallel to y-axis. (vi) Interval of Increasing-decreasing Function: dy x 2 (3a x ) = dx y ( 2a x ) 2

Fig. 3.12

3.78

Engineering Mathematics

Since the curve is symmetric about x-axis, considering the part of the curve above x-axis (y > 0), dy > 0, when x < 3a i.e., 0 < x < 2a [In the region of existence] dx Thus, curve is strictly increasing in this interval. Example 2: Trace the witch of agnesi xy2 = 4a2 (a – x). Solution: (i) Symmetry: The curve is symmetric about x-axis. (ii) Origin: The curve does not pass through the origin. (iii) Points of Intersection: Putting y = 0, we get x = a. Thus, the curve meets x-axis at A(a, 0). Shifting the origin to A(a, 0) by putting x = X + a and y = Y + 0 in the equation of the curve, (X + a)Y 2 = 4a2 (–X), (X + a)Y 2 + 4a2 X = 0. Equating the lowest degree term i.e. 4a2 X to zero, we get X = 0, x – a = 0. Thus, x = a is the tangent at A(a, 0). a x (iv) Region of Existence: From the equation of the curve, y = ± 2a which x becomes imaginary when x < 0 or x > a. Therefore, the curve does not exist in the region – < x < 0 and a < x < . Thus, the curve lies in the region 0 < x < a. (v) Asymptotes: (a) Equating the coefficient of highest y degree term of x to zero, we get y2 + 4a2 = 0 which gives imaginary values. Thus, there is no asymptote parallel to x-axis. (b) Equating the coefficient of highest dex=a gree term of y to zero, we get x = 0. Thus, y-axis is the asymptote. O x A(a, 0) (vi) Interval of Increasing-decreasing Functions: dy 2a 3 =– 2 dx x y Since the curve is symmetric about xaxis, considering the part of the curve dy above x-axis (y > 0), < 0 for all values dx of x. Thus, the curve is strictly decreasing in the Fig. 3.13 0 < x < a. Example 3: Trace the strophoid y2 (a + x) = x2 (b – x). Solution: (i) Symmetry: The curve is symmetric about x-axis. (ii) Origin: The curve passes through the origin.

Differential Calculus II

3.79

Equating the lowest degree term i.e. ay2 – bx2 to zero, we get y = ±

b x. Thus, a

b are the two tangents at origin. a (iii) Points of Intersection: Putting y = 0, we get x = 0, b. Thus, the curve meets x-axis at 0(0, 0) and A(b, 0). Shifting the origin to A(b, 0) by putting x = X + b and y = Y + 0 in the equation of the curve, Y 2(a + X + b) = (X + b)2 (–X) 2 Y (a + b + X) + X (X 2 + 2bX + b2) = 0 Equating the lowest degree term i.e. b2X to zero, we get X = 0, x – b = 0. Thus, x = b is the tangent at A(b, 0). y= ±

(iv) Region of Existence: From the equation of the curve, y = ± x

b−x which a+x

becomes imaginary when x < – a or x > b. Therefore, the curve does not exist in the region – < x < – a and b < x < . Thus, the curve lies in the region – a < x < b. (v) Asymptotes: (a) Since the coefficient of highest degree term of x is constant, there is no asymptote parallel to x-axis. (b) Equating the coefficient of highest degree term of y to zero, we get x + a = 0. Thus, x = – a is the asymptote parallel to y-axis. Since the curve meets x-axis at two points O(0, 0) and A(b, 0), a loop exists in b the region 0 < x < b. Also, y = ± x are the tangents at the origin, therefore after a passing through the origin, the curve extends towards the asymptote x = – a in the region – a < x < 0. y

y

bx a

−a

y

b

A(b, 0) x

O

x

x

− ba x

Fig. 3.14

Engineering Mathematics

3.80

Example 4: Trace Folium of Descartes x3 + y3 = 3axy. Solution: (i) Symmetry: The curve is not symmetric about the coordinate axes but is symmetric about the line y = x, since after interchanging y and x, equation of the curve remains unchanged. (ii) Origin: The curve passes through the origin. Equating the lowest degree term i.e. xy to zero, we get x = 0 and y = 0. Thus, x = 0 and y = 0 are the tangents at the origin. (iii) Points of Intersection: (a) Putting y = 0, we get x = 0. Thus, the curve meets the coordinate axes only at the origin. (b) Putting y = x, we get 2x3 = 3ax2, x = 0, 3a and y = 0, 3a 2 2 3a 3a , Thus, the curve meets the line y = x at O(0, 0) and A . 2 2

( 32a , 32a ) is given by, ( y − 32a ) = ⎛⎜⎝ ddyx ⎞⎟⎠( ) ( x − 32a )

Tangent at A

(

)

y

y=

3a , 3a 2 2

(

⎡ ay − x ⎤ 3a = ⎢ 2 x− ⎥ 2 ⎣ y − ax ⎦ ( 3a , 3a ) 2

(

)

2

)

(

)

F 3a , 3a I H 2 2K

A

x + y = 3a O

2

3a 2 Thus, x + y = 3a is the tangent at 3a 3a , . A 2 2 = –1 x −

x

x+y+a=0

Fig. 3.15

(iv) Region of Existence: In the equation of the curve, x and y cannot be negative simultaneously, otherwise equation of the curve will not be satisfied. Thus, the curve does not exist in the region where x < 0 and y < 0, i.e., third quadrant. (v) Asymptotes: (a) Since coefficients of highest degree term of x and y are constant, the curve does not have any asymptotes parallel to coordinate axes. (b) Oblique Asymptotes: Let y = mx + c is the asymptote of the curve. Putting x = 1 and y = m in the third and second degree terms of the equation separately f3(m) = 1 + m3, f2(m) = –3am Solving f3(m) = 0, 1 + m3 = 0, m = –1 f ( m) ( 3am) c=– 2 =– = a = –a m f3 ( m) 3m 2 Thus, y = –x – a i.e. x + y + a = 0 is the asymptote of the curve.

Differential Calculus II

3.81

Since no part of the curve lies in the third quadrant and coordinate axes are the tangents at the origin, after passing through the origin, the curve extends towards the asymptote x + y + a = 0 in the second and fourth quadrants. Example 5: Trace the catenary y = c cosh x . c

Solution: Rewriting the equation −x c cx (e + e c ) 2 (i) Symmetry: The curve is symmetric about y-axis, since on replacing x by –x, equation remains unchanged. (ii) Origin: The curve does not pass through the origin. (iii) Points of Intersection: Putting x = 0, we get y = c. From the equation of the curve,

y=

dy −x ⎞ ⎛ x = c ⎜ 1 ec − 1 e c ⎟ ⎠ dx 2 ⎝c c dy dx

=0 ( 0, c )

Thus, the tangent is parallel to x-axis at A(0, c). The curve does not meet x-axis. (iv) Region of Existence: Since x 1 cosh < for – < x < , the curve c lies in the region c y < , – 0, when x > 0 dx Thus, the curve is strictly increasing in 0 < x < dy (b) < 0, when x < 0 dx Thus, the curve is strictly decreasing in – < x < 0.

y

(a)

Example 6: Trace the curve y (x2 + a2) = a3. Solution: (i) Symmetry: The curve is symmetric about y-axis. (ii) Origin: The curve does not pass through the origin.

A (0, c) O

Fig. 3.16

x

3.82

Engineering Mathematics

(iii) Points of Intersection: (a) Putting x = 0, we get y = a. Thus, the curve meets the y-axis at A(0, a). Shifting the origin to A(0, a) by putting x = X + 0 and y = Y + a in the equation of the curve, (Y + a)(X 2 + a2) = a3 (Y + a) X 2 + a2Y = 0 Equating the lowest degree term i.e. a2Y to zero, we get Y = 0, y – a = 0. Thus, y = a is the tangent at A(0, a). (b) The curve does not meet x-axis. y which y becomes imaginary when a – y < 0, i.e., y > 0 and y < 0. Thus, the curve does not exist in the region a < y < and – < y < 0. Therefore, the curve lies in the region 0 < y < a. (v) Asymptotes: (a) Equating the coefficient of highest degree term of x to zero, we get y = 0. Thus, x-axis is the asymptote parallel to x-axis. (b) Since coefficient of highest degree term of x is constant, there is no asymptote parallel to y-axis. (vi) Interval of Increasing-decreasing Function: (iv) Region of Existence: From the equation of the curve, x =

a

dy 2 xa3 =– 2 dx ( x + a2 )2 dy < 0 when x > 0 dx Thus, y is strictly decreasing in the interval 0 < x < . dy > 0 when x < 0 dx Thus, y is strictly increasing in the interval – < x < 0. y

A (0, a)

O

Fig. 3.17

y=a

x

a

Differential Calculus II

3.83

Example 7: Trace the curve (x2 – a2)(y2 – b2) = a2 b2. Solution: (i) Symmetry: The curve is symmetric about both x and y axes. (ii) Origin: The curve passes through the origin. Equating the lowest degree terms i.e. – b2 x2 – a2 y2 to zero, we get imaginary values. Thus, tangents at the origin are imaginary. Therefore, the origin is an isolated point. (iii) Points of Intersection: The curve does not meet x and y axes. ay bx (iv) Region of Existence: From equation of the curve, y = ± ,x=± 2 2 2 y b2 x a y is imaginary when x2 – a2 < 0, i.e., – a < x < a and x is imaginary when y2 – b2 < 0, i.e., – b < y < b. Therefore, the curve does not exist in the region where – a < x < a and – < y < – b, b < y < b. Thus, the curve lies in the region – < x < – a, a < x < and – < y < – b, b < y < . (v) Asymptotes: (a) Equating the coefficient of highest degree term of x to zero, we get y2 – b2 = 0. Thus, y = b are the asympy totes parallel to x-axis. (b) Equating the coefficient of highest degree term of y to zero, we get x2 – a2 = 0. Thus, x = a are the asymptotes parallel to y-axis. x −a x a y b (vi) Interval of Increasing-decreasing O y −b x Function: dy = dx

ba 2 3

( x 2 a2) 2

dy < 0 for all values of x dx in the region of existence. Thus, y is strictly decreasing.

Fig. 3.18

Example 8: Trace the curve y2 = (x – 1) (x – 2) (x – 3). Solution: (i) Symmetry: The curve is symmetric about x-axis. (ii) Origin: The curve does not pass through the origin. (iii) Points of Intersection: Putting y = 0, we get x = 1, 2, 3. Thus, the curve meets the x-axis at A(1, 0), B(2, 0) and C(3, 0). Shifting the origin to A(1, 0), B(2, 0) and C (3, 0) by putting (a) x = X + 1, y = Y + 0 in the equation of the curve, Y 2 = X (X – 1)(X – 2) Equating the lowest degree term i.e. 2X to zero, we get X = 0, x – 1 = 0. Thus, x = 1 is the tangent at A(1, 0).

Engineering Mathematics

3.84

(b) x = X + 2, y = Y + 0 in the equation of the curve, Y 2 = (X + 1) X (X – 1) Equating the lowest degree term i.e. –X to zero, we get x – 2 = 0. Thus, x = 2 is the tangent at B(2, 0). (c) x = X + 3, y = Y + 0 in the equation of the curve, Y 2 = (X + 2) (X + 1) X Equating the lowest degree term i.e. 2X to zero, we get X = 0, x – 3 = 0. Thus, x = 3 is the tangent at C(3, 0). (iv) Region of Existence: From the equation of the curve, y = ± ( x − 1) ( x − 2) ( x − 3) which becomes imaginary when x < 1, 2 < x < 3. Therefore, the curve does not exists in the region – < x < 1 and 2 < x < 3. Thus, the curve lies in the region 1 < x < 2 and x > 3. (v) Asymptotes: Since the coefficients of highest degree of x and y are constants, there are no asymptotes to the curve. (vi) Interval of Increasing-decreasing Function: dy 3 x 2 − 12 x + 11 = ± dx 2 ( x − 1) ( x − 2) ( x − 3) = ±

3 ( x − 1.42) ( x − 2.5)

2 ( x − 1) ( x − 2) ( x − 3) dy (a) > 0 when x > 2.5, i.e., 3 < x < [in region of existence] and when dx x < 1.42, i.e., 1 < x < 1.42 Thus, y is strictly increasing in both the intervals. dy < 0 when 1.42 < x < 2.5, i.e., 1.42 < x < 2 [in region of existence] (b) dx Thus, y is strictly decreasing in this interval. y

x=1

O

A (1, 0)

x=2

x=3

B C (2,0) (3,0)

Fig. 3.19

x

Differential Calculus II

3.85

Example 9: Trace the curve y2 (x + a) = x2 (3a – x). Solution: (i) Symmetry: The curve is symmetric about x-axis. (ii) Origin: The curve passes through the origin. Equating the lowest degree term i.e. ay2 – 3ax2 to zero, we get y = ± x 3 . Thus, y = ± x 3 are two tangents at the origin. (iii) Points of Intersection: Putting y = 0, we get x = 0, 3a. Thus, the curve meets the x-axis at A(3a, 0) and O(0, 0). Shifting the origin to A(3a, 0) by putting x = X + 3a and y = Y + 0 in the equation of the curve, Y 2 (X + 3a + a) = (X + 3a)2 (–X). Equating the lowest degree term i.e. –9a2X to zero, we get X = 0, x – 3a = 0. Thus, x = 3a is the tangent at A(3a, 0). (iv) Region of Existence: From the equation of the curve, y = ± x

3a − x which x+a

becomes imaginary when x > 3a or x < – a. Therefore, the curve does not exist in the region where 3a < x < and – < x < – a. Thus, the curve lies in the region – a < x < 3a. (v) Asymptotes: (a) Since coefficient of highest degree term of x is constant, there is no asymptote parallel to x-axis. (b) Equating the coefficient of highest degree term of y to zero, we get x + a = 0. Thus, x = – a is the asymptote parallel to x-axis. Since the curve meets the x-axis at two points, due to symmetry a loop exists between O(0, 0) and A(3a, 0). Also, y = ± x 3 are the tangents at the origin, after passing through the origin the curve extends towards the asymptote. y

A(3a, 0) x

O

x = −a

Fig. 3.20

3.86

Engineering Mathematics

x2 + 2x . x2 + 4

Example 10: Trace the curve y =

Solution: (i) Symmetry: The curve is not symmetric. (ii) Origin: The curve passes through the origin. Equating the lowest degree term i.e. 4y – 2x to zero, we get x = 2y. Thus, x = 2y is the tangent at the origin. (iii) Points of Intersection: Putting y = 0, we get x = 0, – 2. Thus, the curve meets the x-axis at A(–2, 0) and O(0, 0). Shifting the origin to P(–2, 0) by putting x = X – 2, y = Y + 0 in the equation of the curve, Y (X – 2)2 + 4 = (X – 2)2 + 2(X – 2) Y (X2 – 4X + 8) = X2 + 6X Equating the lowest degree term i.e. 8Y – 6X to zero, we get 4y – 3(x + 2) = 0. Thus, 4y – 3x – 6 = 0 is the tangent at (–2, 0). (iv) Region of existence: y is defined for all values of x. Thus, the curve lies in the region – < x < . (v) Asymptotes: (a) Equating the coefficient of highest degree of x to zero, we get y – 1 = 0. Thus, y = 1 is the asymptote parallel to x-axis. (b) Equating the coefficient of highest degree of y to zero, we get x2 + 4 = 0 which gives imaginary values. Thus, there is no asymptote parallel to y-axis. When y = 1, x = 2. This shows that the curve meets y = 1 at B(2, 1). Thus, the curve approaches the asymptote y = 1 from above when x + and from below when x – . (vi) Interval of Increasing-decreasing Function: dy − 2( x 2 − 4 x − 4 ) − 2( x + 0.83) ( x − 4.83) = = dx ( x 2 + 4) 2 ( x 2 + 4) 2

dy > 0 when – 0.83 < x < 4.83 dx Thus, the curve is strictly increasing in this interval. dy (b) < 0, when – < x < – 0.83 and 4.83 < x < dx Thus, the curve is strictly decreasing in both the intervals. (a)

y

x

4.83

B(2, 1)

O A (0, −2) x −0.83

Fig. 3.21

x

Differential Calculus II

3.87

Example 11: Trace the curve x3 + y3 = 3ax2 (a > 0). Solution: (i) Symmetry: The curve is neither symmetric about the coordinate axes nor about the line y = x. (ii) Origin: The curve passes through the origin. Equating the lowest degree term i.e. 3ax2 to zero, we get x = 0. Thus x = 0 i.e., y-axis is the tangent at origin. (iii) Points of Intersection: Putting y = 0, we get x = 0, 3a. Thus, the curve meets x-axis at O(0, 0) and A(3a, 0). Shifting the origin to A(3a, 0) by putting x = X + 3a, y = Y + 0 in the equation of the curve, (X + 3a)3 + Y 3 = 3a (X + 3a)2 3 2 X + 9a X + 6aX 2 + Y 3 = 0. Equating the lowest degree term i.e. 9a2 X to zero, we get X = 0, x – 3a = 0. Thus, x = 3a is the tangent at A(3a, 0). (iv) Region of Existence: x and y cannot be negative simultaneously, but can take opposite signs. Thus, the curve does not exist in the region where x < 0 and y < 0 i.e., third quadrant. (v) Asymptotes: (a) Since coefficients of highest degree terms in x and y are constant, the curve does not have any asymptotes parallel to x and y-axis. (b) Oblique Asymptote: Let y = mx + c is the asymptote of the curve. Putting x = 1, y = m in the third and second degree terms of the equation separately. f3 (m) = 1 + m3, f2 (m) = –3a Solving f3 (m) = 0, 1 + m3 = 0, m = –1, f ( m) c =– 2 f3 ( m) 3a =a 3m 2 Thus, y = –x + a or y + x = a is the asymptote of the curve and curve meets a 2a , the asymptote at 3 3 =–

(

)

(vi) Interval of Increasing-decreasing Function: dy x ( 2a x ) = dx y2 (a)

dy < 0, when x < 0 and x > 2a dx Thus, the curve is strictly decreasing in –

< x < 0 and 2a < x < .

3.88

(b)

Engineering Mathematics

dy > 0, when 0 < x < 2a dx Thus, the curve is strictly increasing in 0 < x < 2a. y

O

A(3a, 0)

x

x+y=a

Fig. 3.22 Example 12: Trace the curve y3 = a2x – x3. Solution: (i) Symmetry: The curve is symmetric in opposite quadrants, since on replacing x by –x and y by –y, equation remains unchanged. (ii) Origin: The curve passes through the origin. Equating the lowest degree term i.e. a2 x to zero, we get x = 0. Thus, x = 0 i.e., y-axis is the tangent at origin. (iii) Points of Intersection: Putting y = 0, we get x = 0, a. Thus, the curve meets the x-axis at O(0, 0), A(a, 0) and B(– a, 0). Shifting the origin to A(a, 0) and B(– a, 0) by putting (a) x = X + a, y = Y + 0 in the equation of the curve, Y 3 = a2 (X + a) – (X + a)3 Y 3 + X 3 + 3aX 2 + 2a2 X = 0 Equating the lowest degree term i.e. 2a2 X to zero, we get X = 0, x – a = 0. Thus, x = a is the tangent at A(a, 0). (b) x = X – a, y = Y + 0 in the equation of the curve, y3 = a2 (X – a) – (X – a)3 Y 3 + X 3 – 3aX 2 + 2a2 X = 0 Equating the lowest degree term i.e. 2a2 X to zero, we get X = 0, x + a = 0. Thus, x = – a is the tangent at B(– a, 0). (iv) Region of Existence: The curve exists everywhere in the region – < x < .

Differential Calculus II

3.89

(v) Asymptotes: (a) Since coefficients of highest degree term of x and y are constant, the curve does not have any asymptotes parallel to coordinate axes. (b) Oblique Asymptotes: Let y = mx + c is the asymptote of the curve. Putting x = 1 and y = m in the third and second degree terms of the equation separately f3 (m) = m3 + 1, f2 (m) = 0 Solving f3 (m) = 0, m3 + 1 = 0, m = –1, c=–

f 2( m ) =0 f3 ( m)

Thus, y = –x is the asymptote of the curve. (vi) Interval of Increasing-decreasing Function: dy a2 3x 2 = dx 3y2 (a)

dy < 0, when x < dx

a 3

a 3

and x >

Thus, the curve is strictly decreasing in the region – a 0, when dx

a 3

0, b > 0. Solution: (i) Symmetry: The curve is symmetric about the line q = 0, since when q is replaced by –q, equation of the curve remains unchanged. Three different cases arise:

Differential Calculus II

3.101

Case I: a > b (ii) Pole: It does not lie on the curve. If r = 0, cos q = – a < –1 which is not possible. Thus r 0 for any value of q. b (iii) Points of Intersection: Putting q = 0, p , p , we get r = (a + b), a, (a – b) respec2 p tively. Thus, curve meets the line q = 0, q = p and q = p at A (a + b, 0), B a, 2 2 and C (a – b, p) respectively.

( )

(iv) Direction of Tangent: r = a + b cos q dr = –b sin q dq d q tan f = r dr (a + b cos q ) = − b sin q At point A(a + b, 0): tan f ,f= p 2 Thus, the tangent is perpendicular to the initial line q = 0 −a ⎞ p At point B a, : tan f = – a , f = tan–1 ⎛⎜ = p – tan–1 a ⎝ b ⎟⎠ 2 b b –1 a Thus, the tangent makes an angle p – tan with the line q = p . b 2 At point C (a – b, p ): tan f ,f= p 2 Thus, the tangent is perpendicular to the line q = p. (v) Region: Minimum value of r = a – b, since minimum value of cos q = –1. Thus r is always positive. (vi) Asymptote: There is no asymptote of the curve since r is finite for all values of q. (vii) Variation of r: p p 2p q 0 p 3 2 3 b r a+b a+ a a– b a–b 2 2

( )

q=p 2

p B a, 2

C (a – b, p)

A (a + b, 0) q=0

O

Fig. 3.29

Engineering Mathematics

3.102

Case II: a < b (ii) Pole: It lies on the curve. a If r = 0, cos q = > –1 b ⎛ −a ⎞ ⎛ −a ⎞ Thus at q = cos–1 ⎜⎝ ⎟ , r = 0. Therefore, q = cos–1 ⎜⎝ ⎟ is the tangent at origin. b ⎠ b ⎠ (iii) Points of Intersection: The curve meets the line q = 0, q = p and q = p at 2 p A (a + b, 0), B a, and C (a – b, p) respectively. 2 (iv) Direction of Tangent: Same as case I (v) Region: Minimum value of r = a – b < 0, thus r is negative for some values of q. (vi) Asymptote: Same as case I (vii) Variation of r: Same as case I. But, here a – b < 0. Therefore, for some values of q, r is negative. Thus, a smaller loop exists between the points 0 and C.

( )

q=p 2 a, p B 2

O

C {–(b – a), 0}

A (a + b, 0) q=0

Fig. 3.30 Case III: a = b the r = a (1 + cos q) which is a cardioid. (ii) Pole: It lies on the curve, since at q = p, r = 0. Tangent at the pole is the line q = p. p (iii) Points of Intersection: Curve meets the line q = 0, q = p at A(2a, 0) and B a, 2 2 respectively. (iv) Direction of Tangent: From Case I

( )

tan f =

a + b cos q − b sin q

1 + cos q = = − sin q

2 cos 2 q p q 2 + = – cot q = tan q q 2 2 2 2 sin cos 2 2

(

)

f= p + q 2 2 At point A(2a, 0), f = p . Thus, the tangent is perpendicular to the initial line 2 q = 0.

Differential Calculus II

( p2 ) , f = p2

At point B a,

3.103

+ p = 3p . Thus, the tangent makes an angle 3p 4 4 4

with the line q = p . 2 (v) Region: The maximum value of r is 2a. Thus, the whole curve lies within a circle with centre at the pole and radius 2a. (vi) Asymptotes: Same as case I (vii) Variation of r: q

0

r

2a

p 3 3a 2

p 2 a

2p 3 a 2

p 0

q=p 2 a, p B 2

O (0, p)

A (2a, 0) q=0

Fig. 3.31 Example 3: Trace the lemniscate of Bernoulli r2 = a2 cos 2q. Solution: (i) Symmetry: The curve is symmetric about the initial line q = 0 and the line q = p , since when q is replaced by –q and by p – q respectively, equation of the 2 curve remains unchanged. The curve is also symmetric about the pole since power of r is even. (ii) Pole: It lies on the curve since r = 0, at q = p . Tangents at the pole are the lines 4 q=± p . 4 (iii) Points of Intersection: The curve meets the initial line q = 0 at A(a, 0) and B(– a, 0). (iv) Direction of Tangent: r2 = a2 cos 2q 2r

dr = – 2a2 sin 2q dq dr a 2 sin 2q =– dq r

Engineering Mathematics

3.104

tan f = r dq dr

(

)

r2 a 2 cos 2q p + 2q = – = – cot 2q = tan 2 2 sin 2 q a sin 2q f = p + 2q 2 p At point A(a, 0), f = . Thus, the tangent is perpendicular to the initial line q = 0. 2 Due to symmetry, the curve is discussed only between q = 0 to q = p . 2 (v) Region: (a) Since maximum value of cos 2q is 1, the maximum value of r is a. Thus, the whole curve lies within a circle with centre at the pole and radius a. =

⎡ Due to symmetry ⎤ ⎥ p p p ⎢ (b) cos 2q < 0, if < 2q < p, i.e., 0 (or t > 0) at (a, b), then f (x, y) is minimum at (a, b) and the minimum value of the function is f (a, b). (iii) If rt – s2 < 0 at (a, b), then f (x, y) is neither maximum nor minimum at (a, b). Such point is known as saddle point. (iv) If rt – s2 = 0 at (a, b), then no conclusion can be made about the extreme values of f (x, y) and further investigation is required. Example 1: Show that the minimum value of f ( x , y ) = xy + Solution:

f ( x, y ) = xy +

a3 a3 + x y

Step I: For extreme values, ∂f a3 = y− 2 =0 ∂x x ∂f a3 = x− 2 = 0 ∂y y Solving Eqs (1) and (2), x2 y = a3 and

xy2 = a3

Solving Eqs (3) and (4), x=y Substituting in Eq. (3), x3 = a3 x=a y=a Stationary point is (a, a).

a3 a3 + is 3a 2 . x y

... (1) ... (2)

... (3) ... (4)

Partial Differentiation

4.129

Step II: ∂ 2 f 2a 3 = 3 ∂x 2 x 2 ∂ f s= =1 ∂x ∂y

r=

t=

∂ 2 f 2a 3 = 3 ∂y 2 y

At (a, a), r = 2, s = 1, t = 2 Step III: At (a, a), rt – s2 = (2) (2) – (1)2 = 3 > 0 Hence, f (x, y) is minimum at (a, a). ⎛1 1⎞ f min = a 2 + a 3 ⎜ + ⎟ = 3a 2 . ⎝a a⎠ Example 2: Find the stationary value of x3 + y3 – 3axy, a > 0. Solution:

f (x, y) = x3 + y3 – 3axy

Step I: For extreme values, ∂f = 3 x 2 − 3ay = 0 ∂x

... (1)

∂f = 3 y 2 − 3ax = 0 ∂y

... (2)

From Eq. (1), y=

x2 a

Substituting in Eq. (2), x4 – a3x = 0 x (x – a) (x + ax + a2) = 0 x = 0, x = a Then y = 0, y = a. 2

Hence, stationary points are (0, 0) and (a, a). Step II:

∂2 f = 6x ∂x 2 ∂2 f = −3a s= ∂x ∂y

r=

t =

∂2 f = 6y ∂y 2

Step III: At (0, 0) rt – s2 = (0) (0) – (–3a)2 = –9a2 < 0

Engineering Mathematics

4.130

Hence, function f (x, y) is neither maximum nor minimum at (0, 0). At (a, a) rt – s2 = (6a) (6a) – (–3a)2 = 27a2 > 0 and r = 6a > 0 Hence, function f (x, y) is minimum at (a, a). fmin = a3 + a3 – 3a3 = –a3. Example 3: Find the extreme values of u = x3 + 3xy2 – 3x2 – 3y2 + 7, if any. Solution:

u = x3 + 3xy2 – 3x2 – 3y2 + 7

Step I: For extreme values, ∂u = 3x 2 + 3 y 2 − 6 x = 0 ∂x x2 + y2 – 2x = 0

... (1)

∂u = 6 xy − 6 y = 0 ∂y

and

6y (x – 1) = 0 y = 0, x = 1 Substituting y = 0 in Eq. (1), x2 – 2x = 0, x = 0, 2 Stationary points are (0, 0), (2, 0) Substituting x = 1 in Eq. (1), 1 + y2 – 2 = 0, y2 = 1, y = ± 1 Stationary points are (1, 1), (1, –1) ∂2u = 6 x − 6 = 6( x − 1) ∂x 2 ∂2u = 6y s= ∂x ∂y

r=

Step II:

t=

∂2u = 6 x − 6 = 6( x − 1) ∂y 2

Step III: (x, y)

r

s

t

rt – s2

(0, 0)

−6

0

−6

36 > 0 and r < 0

maximum

(2, 0)

6

0

6

36 > 0 and r > 0

minimum

(1, 1)

0

6

0

−36 < 0

neither maximum nor minimum

(1, −1)

0

−6

0

−36 < 0

neither maximum nor minimum

Conclusion

Partial Differentiation

4.131

Hence, u is maximum at (0, 0) and minimum at (2, 0). umax = 0 + 7 = 7 and umin = 23 + 3(2) (0)2 – 3(2)2 – 3(0)2 + 7 = 3. Example 4: Find the extreme values of u = x3 + y3 – 63 (x + y) + 12xy. Solution:

u (x, y) = x3 + y3 – 63x – 63y + 12xy ∂u = 3 x 2 − 63 + 12 y ∂x ∂u = 3 y 2 − 63 + 12 x ∂y

Step I:

For extreme values ∂u =0 ∂x 3x2 – 63 + 12y = 0, 3x2 + 12y = 63 x2 + 4y = 21

... (1)

∂u =0 ∂y

and

3y2 – 63 + 12x = 0, 12x + 3y2 = 63 4x + y2 = 21 Solving Eqs (1) and (2),

x 2 + 4 y = 4 x + y 2 , x 2 − y 2 = 4( x − y ) ( x + y )( x − y ) − 4( x − y ) = 0 ( x − y )( x + y − 4) = 0 x + y − 4 = 0, x − y = 0 y = 4 − x, y = x Substituting y = 4 – x in Eq. (1), x 2 + 4(4 − x) = 21 x 2 − 4 x − 5 = 0, ( x + 1)( x − 5) = 0 x = −1, 5 y = 5, − 1 Stationary points are (–1, 5), (5, –1). Putting y = x in Eq. (1), x 2 + 4 x − 21 = 0, ( x + 7)( x − 3) = 0 x = −7, 3 y = −7, 3 Stationary points are (–7, –7), (3, 3). Hence, all stationary points are: (–1, 5), (5, –1) (–7, –7), (3, 3).

... (2)

Engineering Mathematics

4.132

Step II:

r=

∂2u = 6x ∂x 2

s=

∂2u = 12 ∂x ∂y

t=

∂2u = 6y ∂y 2

Step III: (x, y)

r

s

t

rt – s2

(−1, 5)

−6

12

30

−324 < 0

neither maximum nor minimum

(5, −1)

30

12

−6

−324 < 0

neither maximum nor minimum

(−7, −7)

−42

12

−42

1620 > 0 and r < 0

maximum

(3, 3)

18

12

18

180 > 0 and r > 0

minimum

Conclusion

Hence, at (−7, −7), u is maximum. umax = (−7)3 + (−7)3 – 63 (−7 −7) + 12(−7) (−7) = 2156. and at (3, 3), u is minimum. umin = 33 + 33 – 63 (3 + 3) + 12(3) (3) = −216. Example 5: Find the stationary value of xy (a – x – y). Solution:

f (x, y) = xy (a – x – y) = axy – x2y – xy2

Step I: For extreme values, ∂f = ay − 2 xy − y 2 = 0 ∂x ∂f = ax − x 2 − 2 xy = 0 ∂y From Eqs (1) and (2), we get y (a – 2x – y) = 0 y = 0, a – 2x – y = 0 and x (a – x – 2y) = 0 x = 0, a – x – 2y = 0 Considering four pairs of equations y=0 x=0 y=0 a – x – 2y = 0 a – 2x – y = 0 x=0 a – 2x – y = 0 a – x – 2y = 0

... (1) ... (2)

Partial Differentiation

4.133

Solving these equations, following pairs of values of x and y are obtained. a a (0, 0), (0, a), (a, 0), ⎛⎜ , ⎞⎟ ⎝3 3⎠ ∂2 f Step II: r = 2 = −2 y ∂x ∂2 f = a − 2x − 2 y s= ∂ x ∂y t=

∂2 f = −2 x ∂y 2

Step III: (x, y)

r

s

t

rt – s2 2

Conclusion

(0, 0)

0

a

0

–a 0 3

⎛a ⎜⎝ , 3

a⎞ ⎟ 3⎠

maximum or minimum

⎛a a⎞ Hence, f (x, y) is maximum or minimum at ⎜ , ⎟ depending on whether a > 0 or ⎝3 3⎠ a < 0. a a⎛ a a ⎞ a3 f extreme = ⋅ ⎜ a − − ⎟ = . 3 3⎝ 3 3 ⎠ 27 Example 6: Examine the function u = x3 y2 (12 – 3x – 4y) for extreme values. Solution:

u (x, y) = 12x3 y2

x4 y2 – 4x3 y3

∂u = 36 x 2 y 2 − 12 x 3 y 2 − 12 x 2 y 3 ∂x = 12 x 2 y 2 (3 − x − y ) ∂u = 24 x 3 y − 6 x 4 y − 12 x 3 y 2 = 6 x 3 y (4 − x − 2 y ) ∂y

Step I:

For extreme values, ∂u =0 ∂x 2 2 12x y (3 – x – y) = 0 x = 0, y = 0, x + y = 3

... (1)

Engineering Mathematics

4.134

∂u =0 ∂y 6x3 y (4 – x – 2y) = 0 x = 0, y = 0, x + 2y = 4 Considering six pairs of equations, x=0 y=0 x=0 x + 2y = 4 y=0 x + 2y = 4 x=0 x+y=3 y=0 x+y=3 x+y=3 x + 2y = 4

and

... (2)

Solving these equations, following pairs of stationary points are obtained (0, 0), (0, 2), (4, 0), (0, 3), (3, 0), (2, 1) ∂2u = 72 xy 2 − 36 x 2 y 2 − 24 xy 3 = 12 xy2(6 – 3x – 2y) ∂x 2 ∂2u s= = 72 x 2 y − 24 x 3 y − 36 x 2 y 2 = 12 x 2 y (6 − 2 x − 3 y ) ∂x ∂y

r=

Step II:

t=

∂2u = 24 x 3 − 6 x 4 − 24 x 3 y = 6 x 3 (4 − x − 4 y ) ∂y 2

Step III: (x, y)

r

s

t

rt – s2

(0, 0)

0

0

0

0

no conclusion no conclusion

Conclusion

(0, 2)

0

0

0

0

(4, 0)

0

0

0

0

no conclusion

(0, 3)

0

0

0

0

no conclusion

(3, 0)

0

0

0

0

no conclusion

(2, 1)

−48

−48

−96

2304 > 0 and r < 0

maximum

Hence, function is maximum at (2, 1) umax = (23) (12) (12 – 6 – 4) = 16. Example 7: Find the extreme values of sin x + sin y + sin(x + y). Solution: Step I:

f (x, y) = sin x + sin y + sin(x + y) ∂f = cos x + cos( x + y ) ∂x ∂f = cos y + cos( x + y ) ∂y

Partial Differentiation

4.135

For extreme values,

∂f = 0, cos x + cos( x + y ) = 0 ∂x ∂f = 0, cos y + cos ( x + y ) = 0 ∂y From Eqs (1) and (2), we get cos x + cos( x + y ) = cos y + cos( x + y ) cos x = cos y, x = y

... (1) ... (2)

Substituting x = y in Eq. (1), cos x + cos 2 x = 0, cos x = − cos 2 x = cos(p − 2 x) or cos(p + 2 x) x = p − 2 x or p + 2 x p x = , −p 3 p y = , −p 3 ⎛p p ⎞ Thus, ⎜ , ⎟ , (−p , − p ) are stationary points. ⎝3 3⎠ ∂2 f r = 2 = − sin x − sin( x + y ) Step II: ∂x ∂2 f = − sin( x + y ) s= ∂x ∂y t=

∂2 f = − sin y − sin( x + y ) ∂y 2

Step III: (x, y)

(

, 3 3 ,

r

s 3

)

0

3 2 0

rt – s2

t 3 0

9 > 0 and r < 0 4 0

Conclusion maximum no conclusion

Hence, function is maximum at ⎛⎜ p , p ⎞⎟ . ⎝3 3⎠ f max = sin

3 3 3 3 3 2p p p + sin + sin = + + = . 3 3 3 2 2 2 2

Example 8: Find the extreme values of sin x sin y sin (x + y). Solution:

f (x, y) = sin x sin y sin (x + y)

Engineering Mathematics

4.136

Step I:

∂f = sin y [cos x sin( x + y ) + sin x cos( x + y )] ∂x 1 = sin y sin (2 x + y ) = [cos 2 x − cos( 2 x + 2 y )] 2

∂f = sin x [ cos y sin ( x + y ) + sin y cos( x + y )] ∂y 1 = sin x sin ( x + 2 y ) = [ cos 2 y − cos(2 x + 2 y ) ] 2 For extreme values, ∂f = 0, cos 2 x − cos 2( x + y ) = 0 ∂x

and

... (1)

∂f = 0, cos 2 y − cos 2( x + y ) = 0 ∂y From Eqs (1) and (2), we get

... (2)

and

cos 2x = cos 2y, x = y Putting x = y in Eq. (1), cos 2x − cos 2 (x + x) = 0, cos 2x = cos 4x, cos 2x = 2 cos2 2x – 1 2 cos2 2x – cos 2x – 1 = 0 1± 1+ 8 4 1 = 1, – 2 cos 2x = cos 0,

cos 2x =

cos 2x = cos

x = 0,

x=

y = 0,

y=

2 3

3 3

⎛p p ⎞ Thus, (0, 0), ⎜ , ⎟ are stationary points. ⎝3 3⎠ Step II:

∂2 f = − sin 2 x + sin 2( x + y ) = 2 sin y cos (2 x + y ) ∂x 2 ∂2 f s= = sin 2( x + y ) ∂x ∂y

r=

t=

∂2 f = – sin 2 y + sin 2( x + y ) = 2 sin x cos ( x + 2 y ) ∂y 2

Partial Differentiation

4.137

Step III: (x, y)

r

s

t

rt – s2

(0, 0)

0

0

0

0

⎛ ⎞ ⎜⎝ , ⎟⎠ 3 3

3

3 2

3

9 > 0 and r < 0 4

Conclusion no conclusion maximum

⎛p p ⎞ Hence, function is maximum at ⎜ , ⎟ . ⎝3 3⎠ p p 2p sin sin 3 3 3 3 3 3 3 3 = ⋅ ⋅ = . 2 2 2 8

f max = sin

Example 9: Find the points on the surface z 2 = xy + 1 nearest to the origin. Also find that distance. Solution: Let P (x, y, z) be any point on the surface z2 = xy + 1. Its distance from the origin is given by D = ( x2 + y 2 + z 2 ) D2 = x2 + y 2 + z 2 Since P lies on the surface z 2 = xy + 1 D 2 = x 2 + y 2 + xy + 1 Let f ( x, y ) = x 2 + y 2 + xy + 1 Step I: For extreme values, ∂f = 2x + y = 0 ∂x ∂f = 2y + x = 0 ∂y Solving Eqs (1) and (2), x = 0 and y = 0 Stationary point is (0, 0). Step II:

∂2 f =2 ∂x 2 ∂2 f s= =1 ∂x ∂y

r=

t=

∂2 f =2 ∂y 2

... (1) ... (2)

Engineering Mathematics

4.138

Step III: At (0, 0) rt – s2 = (2) (2) – (1)2 = 3 > 0 r=2>0 and Thus, f (x, y) is minimum at (0, 0) and hence D is minimum at (0, 0). At x = 0, y = 0 z 2 = xy + 1 = 1 z = ±1 Hence, D is minimum at (0, 0, 1) and (0, 0,

1) .

Thus, the points (0, 0, 1) and (0, 0, –1) on the surface z 2 = xy + 1 are nearest to the origin. Minimum distance = 0 + 0 + 1 = 1. Example 10: A rectangular box open at the top is to have a volume 108 cubic meters. Find the dimensions of the box if its total surface area is minimum. Solution: Let x, y and z be the dimensions of the box. Let V and S be its volume and surface area respectively. V = xyz S = xy + 2 xz + 2 yz V Substituting z = , xy V V 2V 2V S = xy + 2 x ⋅ + 2 y ⋅ = xy + + xy xy y x ∂S 2V = y− 2 ∂x x Step I: ∂S 2V = x− 2 ∂y y For extreme values, S =0 x 2V ... (1) y 0 x2 ∂S =0 ∂y

and x

2V y2

Solving Eqs (1) and (2), y=

2V x2

0

... (2)

Partial Differentiation

4.139

⎛ x4 ⎞ x = 2V ⎜ 2 ⎟ = 0 ⎝ 4V ⎠ ⎛ x3 x ⎜1 − ⎝ 2V

⎞ ⎟⎠ = 0 1

x = (2V ) 3 y=

1 2V 2V 3 [since x 0 being the side of the box] ( 2 V ) = = 2 x2 (2V ) 3

1 1 ⎡ ⎤ Hence, stationary point is ⎣⎢(2V ) 3 , (2V ) 3 ⎦⎥

Step II:



1

1



r=

∂ 2 S 4V = 3 ∂x 2 x

s=

∂2 S =1 ∂x ∂y

t=

∂ 2 S 4V = 3 y ∂y 2

At ⎣⎢(2V ) 3 , (2V ) 3 ⎦⎥ , r =

4V 4V = 2 > 0, s = 1, t = =2 2V 2V

1 1 ⎡ ⎤ Step III: At ⎢⎣(2V ) 3 , (2V ) 3 ⎥⎦ ,

rt s 2

(2)(2) (1) 2

0 and r

3

2

0

1

Hence, S is minimum at But

x = y = (2V ) 3 V = 108 m3 1

x and

z=

y

(2 108) 3

6

V 108 = =3 xy 6 × 6

Hence, dimensions of the box which make its total surface area S minimum are x = 6, y = 6, z = 3. Example 11: Show that the rectangular solid of maximum volume that can be inscribed in a given sphere is a cube.

Engineering Mathematics

4.140

Solution: Let x, y, z be the length, breadth and height of the rectangular solid and V be its volume. V = xyz

... (1)

Let given sphere is x2 + y 2 + z 2 = a2 z2

a2

x2

y2

Substituting in Eq. (1), V = xy a 2 − x 2 − y 2 V 2 = x 2 y 2 (a 2 − x 2 − y 2 ) Let Step I:

and

f ( x, y ) V 2 f x

x 2 y 2 (a 2

x2

y 2 [2 x(a 2

x2

2 xy 2 (a 2

2x2

y2 )

... (2)

y 2 ) x 2 ( 2 x)] y2 )

∂f = x 2 [2 y (a 2 − x 2 − y 2 ) + y 2 (−2 y )] ∂y = 2 x 2 y (a 2 − x 2 − 2 y 2 )

For extreme values, ∂f = 0, 2 xy 2 (a 2 − 2 x 2 − y 2 ) = 0 ∂y x = 0, y = 0, 2 x 2 + y 2 = a 2 and

f y

0, 2 x 2 y (a 2

x2

2 y2 )

x = 0, y = 0, x 2 + 2 y 2 = a 2 But x and y are the sides of the rectangular solid, therefore cannot be zero. Solving 2 x 2 + y 2 = a 2 and x 2 + 2 y 2 = a 2 x=

a 3

, y=

z = a2 −

a 3

a2 a2 a − = 3 3 3

Thus, stationary points are ⎛ a , a , a ⎞ . ⎜⎝ ⎟ 3 3 3⎠

... (3)

... (4)

Partial Differentiation

4.141

∂2 f = 2a 2 y 2 − 12 x 2 y 2 − 2 y 4 ∂x 2 ∂2 f = 4a 2 xy − 8 x 3 y − 8 xy 3 s= ∂x ∂y

r=

Step II:

t= a ⎞ ⎛ a , Step III: At ⎜ ⎟, ⎝ 3 3⎠

∂2 f = 2a 2 x 2 − 2 x 4 − 12 x 2 y 2 2 ∂y

2a 4 4a 4 2a 4 8a 4 − − =− 3 3 9 9 4 4 4 4 a 8a 8a 4a 4 s= − − =− 3 9 9 9 4 4 4 2a 2a 12a 8a 4 t= − − =− 3 9 9 9

r=

rt − s 2 =

64a 4 16a 4 48a 4 − = >0 81 81 81

rt – s2 > 0 and r < 0 Therefore, f (x, y) i.e. v2 is maximum at x = y = z and hence, v is maximum when x = y = z, i.e. rectangular solid is a cube. Exercise 4.8 1. Examine maxima and minima of the following functions and find their extreme values: (i) 2 + 2x + 2y − x2 − y2 2 (ii) x2 y2 xy y2 2 2 (iii) x + y + xy + x y (iv) x2 + y2 + 6x = 12 x y) (v) x3 y2 (vi) xy (3a x y) (vii) x3 + 3xy2 x2 y2 + 4 x y)2 (viii) x4 + y4 4 2 2 (ix) x + x y + y (x) x4 + y4 − 4a2 xy (xi) y4 − x4 + 2(x2 − y2) (xii) x3 + 3x2 + y2 + 4xy (xiii) x2y − 3x2 − 2y2 − 4y + 3 (xiv) x4 − y4 − x2 − y2 + 1. ⎡ Ans.: (i) max. at (1, 1); 4 ⎢ (ii) max. at (0, 0); 0 ⎢ ⎢ (iii) min. at ( − 2, 3); − 2 ⎢

⎤ ⎥ ⎥ ⎥ ⎥

⎢ ⎢ ⎢ ⎢ ⎢ ⎢ ⎢ ⎢ ⎢ ⎢ ⎢ ⎢ ⎢ ⎢ ⎢ ⎢ ⎢ ⎢ ⎢ ⎢ ⎢ ⎢ ⎢ ⎢ ⎢⎣

⎥ ⎥ ⎥ ⎛ 1 1⎞ 1 (v) max. at ⎜ , ⎟ ; ⎥ ⎝ 2 3 ⎠ 432 ⎥ 3 ⎥ (vi) max. at ( a, a)); a ⎥ (vii) max. at (0, 0); 4 ⎥ ⎥ (viii) min. at 2, − 2 ⎥ ⎥ and − 2 , 2 ; − 8 ⎥ ⎥ (ix) min. at (0, 0); 0 ⎥ 4⎥ (x) min. at ( a, a) and ( − a, a); a ⎥ ⎥ (xi) No extreme values ⎥ (xii) No extreme values ⎥ ⎥ (xiii) max. at (0, − 1); 5 ⎥ (xiv) max. at (0, 0); 1, min at ⎥ ⎥ ⎛ 1 1 ⎞ 1 ⎥ , ± ± ; ⎟ ⎜ 2 2⎠ 2 ⎝ ⎦⎥ (iv) min at ( − 3, 0); 3

(

(

)

)

4.142

Engineering Mathematics

2. A rectangular box, open at the top, is to have a volume of 32 cc. Find the dimensions of the box requiring least materials for its construction. [Ans. : 4, 4, 2] 3. Divide 120 into three parts so that the sum of their products taken two at a time shall be maximum. [Hint : f = xy + yz + zx where x + y + z = 120] [Ans. : 40, 40, 40] 4. The sum of three positive numbers is ‘a’. Determine the maximum value of their product. a3 a a a Ans. : at , , 27 3 3 3 5. Find the volume of the largest rectangular parallelopiped that can be inscribed in an ellipsoid x2 y 2 z 2 + + = 1. a 2 b2 c2 ⎡ Hint : Let 2x, 2y , 2z be the sides of ⎤ ⎢ the parallelopiped, then its volume ⎥ ⎢ ⎥ ⎢ ⎥ x2 y 2 ⎢ ⎥ v = 8 xyz = 8 xy 1 − 2 − 2 a b ⎣ ⎦

Ans. :

8abc

6. Prove that area of a triangle with constant perimeter is maximum when the triangle is equilateral. [Hint : Area s ( s a ) ( s b) ( s c ) where 2s = a + b + c, c = 2s a b, s is constant] 7. Find the shortest distance from origin to the surface xyz2 = 2. [Ans. : 2] 8. Find the shortest distance from the origin to the plane x − 2y − 2z = 3. [Ans. : 1] 9. Find the shortest distance between the lines x−3 = 1 x +1 = 7

y−5 z −7 = and 1 −2 y +1 z +1 = . −6 1 ⎡ Ans. : 2 29 ⎤ ⎣ ⎦

10. Find the maximum value of cos A cos B cos C, where A, B, C are angles of a triangle. Ans. :

p 3

p 3

p 3

1 8

3 3

4.8.4 Lagrange’s Method of Undetermined Multipliers Let f (x, y, z) be a function of three variables x, y, z, and the variables be connected by the relation f ( x, y , z ) = 0 ... (1) Suppose we wish to find the values of x, y, z, for which f (x, y, z) is stationary (maximum and minimum) For this purpose, we construct an auxiliary equation (x, y, z) ... (2) F (x, y, z) = f (x, y, z) + Differentiating partially w.r.t. x, y, z and equating to zero, ∂F ∂f ∂f ... (3) = +l =0 ∂x ∂x ∂x

Partial Differentiation

4.143

∂F ∂f ∂f = +l =0 ∂y ∂y ∂y

... (4)

∂F ∂f ∂f ... (5) = +l =0 ∂z ∂z ∂z Solving Eqs (1), (3), (4) and (5), we can find the values of x, y, z and l for which f (x, y, z) has stationary value. This method of obtaining stationary values of f (x, y, z) is called the Lagrange’s method of undetermined multipliers and Eqs (3), (4) and (5) are called Lagrange’s equations. The term l is called undetermined multiplier. Example 1: Find the point on the plane ax + by + cz = p at which the function f = x2 + y2 + z2 has a minimum value and find this minimum f. Solution:

f = x2 + y2 + z2

... (1)

ax + by + cz = p f (x, y, z) = ax + by + cz − p = 0 Lagrange’s equations f f +l =0 x x

... (2)

2x + la = 0 x=

la 2

f f +l =0 y y 2y + lb = 0 y=

lb 2

f f +l =0 z z 2z + lc = 0 z=

lc 2

Substituting x, y, z in Eq. (2), ⎛ −l a ⎞ ⎛ −l b ⎞ ⎛ −l c ⎞ a⎜ +b⎜ +c⎜ =p ⎝ 2 ⎟⎠ ⎝ 2 ⎟⎠ ⎝ 2 ⎟⎠ la2 + lb2 + lc2 = −2p l=

−2 p a + b2 + c2 2

4.144

Thus,

The minimum value of

Engineering Mathematics

x=

f = =

ap bp cp , y= 2 , z= 2 2 2 2 2 a +b +c a +b +c a + b2 + c2 2

a2 p2 b2 p 2 c2 p2 + 2 + 2 2 2 2 2 2 2 (a + b + c ) (a + b + c ) (a + b 2 + c 2 ) 2 2

p 2 (a 2 + b 2 + c 2 ) 2 p2 = . (a 2 + b 2 + c 2 ) 2 a 2 + b2 + c2

Example 2: Find the maximum value of f = x2 y3 z4 subject to the condition x + y + z = 5. Solution:

f = x2 y3 z4 x+y+z=5 f (x, y, z) = x + y + z − 5 = 0

... (1) ... (2)

Lagrange’s equations f f +l =0 x x 2xy3z4 + = 0 2xy3 z4 = −

... (3)

f f +l =0 y y 3x2 y2 z4 + = 0 3x2 y2 z4 = −

... (4)

f f +l =0 z z 4x2 y3 z3 + = 0 4x2 y3z3 = − From Eqs (3) and (4), 2xy3 z4 = 3x2 y2 z4 2y = 3x 3 y= x 2 From Eqs 2xy3z4 = 4x2 y3 z3 z = 2x

... (5)

Partial Differentiation

4.145

Substituting y and z in Eq. (2), x+

3 x + 2x = 5 2 9 x = 10 10 x= 9 3 3 ⎛ 10 ⎞ 5 y= x= ⎜ ⎟= 2 2⎝9⎠ 3 ⎛ 10 ⎞ 20 z = 2x = 2 ⎜ ⎟ = ⎝9⎠ 9 2

Maximum value of

3

4

(210 ) (59 ) ⎛ 10 ⎞ ⎛ 5 ⎞ ⎛ 20 ⎞ . f =⎜ ⎟ ⎜ ⎟ ⎜ ⎟ = ⎝ 9 ⎠ ⎝3⎠ ⎝ 9 ⎠ 315

Example 3: Show that the rectangular solid of maximum value that can be inscribed in a sphere is a cube. Solution: Let 2x, 2y, 2z be the length, breadth and height of the rectangular solid. Let r be the radius of the sphere. Volume of solid, Equation of the sphere,

V = 8xyz 2

2

2

… (1) 2

x +y +z =r

… (2)

f (x, y, z) = x + y + z − r = 0 2

2

2

2

Lagrange’s equation V f +l =0 x x 8yz + .2x = 0 2 x = −8yz 2 x2 = −8xyz

… (3)

V f +l =0 y y 8xz + .2y = 0 2 y = −8xz 2 y2 = −8xyz V f +l =0 z z 8xy + .2z = 0

… (4)

Engineering Mathematics

4.146

2 z = −8xy 2 z2 = −8xyz

… (5)

From Eqs (3), (4) and (5), 2 x2 = 2 y2 = 2 z2 x2 = y2 = z2 x=y=z Hence, rectangular solid is a cube. Example 4: A rectangular box open at the top is to have volume of 32 cubic units. Find the dimensions of the box requiring least material for its construction. Solution: Let x, y, z be the dimensions of the box. Volume

V = xyz = 32

The box is open at the top. Therefore, its surface area S = xy + 2xz + 2yz f (x, y, z) = xyz – 32

… (1) … (2) … (3)

Lagrange’s equation S f +l =0 x x y + 2z + yz = 0

… (4)

S f +l =0 y y x + 2z + xz = 0

… (5)

f S +l =0 z z 2x + 2y + xy = 0

… (6)

Multiplying Eq. (4) by x, xy + 2 xz + lxyz = 0 xy + 2 xz + 32l = 0 32l

… (7)

xy + 2 yz + lxyz = 0 xy + 2 yz + 32l = 0 xy 2 yz 32l

… (8)

xy 2 xz Multiplying Eq. (5) by y,

Partial Differentiation

4.147

Multiplying Eq. (6) by z, 2 xz + 2 y z + lxyz = 0 2 xz + 2 yz + 32l = 0 2 xz + 2 yz = −32l From Eqs (7) and (8),

… (9)

xy + 2 xz = xy + 2 yz 2 xz = 2 yz x= y

From Eqs (8) and (9), xy + 2 yz = 2 xz + 2 yz xy = 2 xz y y = 2 z, z = 2 Substituting x, y, z in Eq. (1), y y

y 32 2 y 3 = 64 y=4 x= y=4

y =2 2 Hence, dimensions of the box requiring least material for its construction are 4, 4, 2. z=

Example 5: Find the maximum and minimum distances from the origin to the curve 3x2 + 4xy + 6y2 = 140. Solution: The distance d from the origin (0, 0) to any point (x, y) is given by d=

x2 + y 2 , d 2 = x2 + y 2

Let f (x, y) = x2 + y2 and Lagrange’s equations

2x +

f (x, y) = 3x2 + 4xy + 6y2 − 140

f f +l =0 x x (6x + 4y) = 0

… (1)

f f +l =0 y y 2y +

(4x + 12y) = 0

… (2)

Engineering Mathematics

4.148

Solving Eqs (1) and (2),

Substituting

l

x 3x + 2 y

y 2x + 6 y

l

x2 3 x + 2 xy

y2 2 xy + 6 y 2

2

x2 + y 2 3x + 6 y 2 + 4 xy 2

f ( x, y ) 140

in Eqs (1) and (2), f f (6 x 4 y ) 0, 2 y (4 x 12 y ) 140 140 (140 3 f ) x 2 fy 0 2x

and Substituting x =

0 … (3) … (4)

−2fx + (140 − 6f ) y = 0 2 fy from Eq. (3) in Eq. (4), 140 3 f − 4 f 2 + (140 − 3 f ) (140 − 6 f ) = 0 14 f 2 − 1260 f + (1402 ) = 0 f 2 − 90 f − 1400 = 0 f = 70, 20

Thus, maximum and minimum distances are

70, 20.

Example 6: A wire of length b is cut into two parts which are bent in the form of a square and circle respectively. Find the least value of the sum of the areas so found. Solution: Let x and y be two parts of the wire. x+y=b

… (1)

Let the piece of length x is bent in the form of a square so that each side is Thus, the area of the square, A 1 =

x x x2 ⋅ = . 4 4 16

x . 4

Suppose piece of length y is bent in the form of a circle of radius r so perimeter of the circle is y. y 2p r = y, r = 2p 2

Thus, the area of the circle,

y2 ⎛ y ⎞ = A2 = p ⎜ . ⎝ 2p ⎟⎠ 4p

Let sum of the areas is given as f ( x, y ) =

x2 y 2 + 16 4p

Partial Differentiation

4.149

f (x, y) = x + y – b

and Lagrange’s equations:

f f +l =0 x x 2x l 0, x 16

8l

f f +l =0 y y

and

2y l 4p Substituting x and y in Eq. (1),

0, y

2pl

(–8 ) + (–2p ) = b l= Thus,

b 8 + 2p

8b 4b = 8 + 2p 4 + p 2p b pb y= = 8 + 2p 4 + p x=

Substituting in f (x, y), 1 4b f ( x, y ) = 16 4 + p = =

b2

(4 + p )

2

2

1+

1 + 4p

pb 4+p

2

b 2p (4 + p ) p2 = 2 4p 4p (4 + p )

b2 4(p + 4)

b2 . 4 (p + 4) Example 7: A closed rectangular box has length twice its breadth and has constant volume V. Determine the dimensions of the box requiring least surface area. Hence, the least value of the sum of the areas is

Solution: Let x be the breadth and y be the height of the rectangular box so length of the box will be 2x. Volume of the box V = x . 2x . y = 2x2y Volume of the box is constant 2x2y = V = constant Surface area of the box is given by S = 2 (2x · x + x · y + y · 2x) = 4x2 + 6xy

… (1) … (2)

Engineering Mathematics

4.150

f (x, y) = 2x2y − V Let Lagrange’s equations:

… (3)

∂S ∂f +l =0 ∂x ∂x

8x + 6y +

(4xy) = 0

2x + 3y +

(2xy) = 0

… (4)

∂S ∂f +l =0 ∂y ∂y 6x + (2x2) = 0

and

3 x lx 2

Substituting x

3 in Eq. (4), l 3 2 3y l 2 y l

3 l

3 l

0, x

0

6 l

2 l

3 y, y

Substituting x and y in Eq. (1), 3 l

2

2

2 l

V 1

−36 ⎛ 36 ⎞ 3 , l = −⎜ ⎟ l = ⎝V ⎠ V 3

x

3 l

V 3 36

y

2 l

V 2 36

1 3

1 3

1 3

27V 36 8V 36

1 3

1 3

3V 4 2V 9

1 3

1

1

⎛ 3V ⎞ 3 ⎛ 3V ⎞ 3 Hence, the dimensions of the box requiring least surface area are 2 ⎜ ⎟ , ⎜ ⎟ , ⎝ 4 ⎠ ⎝ 4 ⎠ 2V 9

1 3

.

Example 8: Using the Lagrange’s method find the minimum and maximum distance from the point (1, 2, 2) to the sphere x2 + y2 + z2 = 36.

Partial Differentiation

4.151

Solution: Given sphere is x2 + y2 + z2 = 36 … (1) Let the coordinates of any point on the sphere be (x, y, z), then its distance D from the point (1, 2, 2) is D ( x 1) 2 ( y 2) 2 ( z 2) 2 Let D2 = f (x, y, z) = (x − 1)2 + (y − 2)2 + (z − 2)2 f (x, y, z) = x2 + y2 + z2 − 36

and Lagrange’s equations:

f f +l =0 x x 2 (x − 1) + (2x) = 0 (x − 1) + x = 0

… (2)

f f +l =0 y y 2 (y − 2) + (2y) = 0

and

(y − 2) + y = 0

… (3)

f f +l =0 z z 2 (z − 2) + (2z) = 0 (z − 2) + z = 0

… (4)

Multiplying Eq. (2) by x, Eq. (3) by y and Eq. (4) by z and adding, (x2 + y2 + z2) − (x + 2y + 2z) + (x2 + y2 + z2) = 0 36 (1 + ) − (x + 2y + 2z) = 0 [Using Eq. (1)] From Eq. (2), 1 x= 1+ l From Eq. (3), 2 y= 1+ l From Eq. (4), 2 z= 1+ l Substituting x, y, z in Eq. (5), 36 (1 l )

1+ 4 + 4 1+ l

0

36 (1 + l ) = 9, (1 + l ) = 2

1 1+ l = ± , 2

2

1 , 4

… (5)

… (6)

… (7)

… (8)

Engineering Mathematics

4.152

Substituting in Eqs (6), (7) and (8), x = ± 2, y = ± 4, z = ± 4 Minimum distance = (2 − 1) 2 + (4 − 2) 2 + (4 − 2) 2 = 1 + 4 + 4 = 3 Maximum distance =

(−2 − 1) 2 + (−4 − 2) 2 + (−4 − 2) 2 =

9 + 36 + 36 = 9.

Example 9: Use the method of the Lagrange’s multipliers to find volume of the largest rectangular parallelopiped that can be inscribed in the ellipsoid x2 y2 z2 + + = 1. a 2 b2 c 2

x2 y 2 z 2 + + =1 a 2 b2 c2

Solution:

… (1)

Let 2x, 2y, 2z be the length, breadth and height of the rectangular parallelopiped inscribed in the ellipsoid. Volume of the parallelopiped, V = (2x) (2y) (2z) = 8xyz. Let

f ( x, y , z )

Lagrange’s equations: ∂V ∂f +l ∂x ∂x 2x 8 yz + l 2 a ∂V ∂f +l ∂y ∂y 2y 8 xz + l 2 b ∂V ∂f +l ∂z ∂z 2z 8 xy + l 2 c

x2 a2

y2 b2

z2 c2

1

=0 = 0, 4 yz + l

x =0 a2

… (2)

y =0 b2

… (3)

z =0 c2

… (4)

=0 = 0, 4 xz + l =0 = 0, 4 xy + l

Multiplying Eq. (2) by x, Eq. (3) by y and Eq. (4) by z and adding, ⎛ x2 y 2 z 2 ⎞ 12 xyz + l ⎜ 2 + 2 + 2 ⎟ = 0 b c ⎠ ⎝a 12 xyz + l = 0 l = −12 xyz

[Using Eq. (1)]

Partial Differentiation

4.153

Substituting in Eq. (2), x a2

4 yz 12 xyz 3x 2 a2

1

0, x

0 a 3

Similarly substituting in Eqs (3) and (4), y=

b 3

, z=

c 3

Volume of the largest rectangular parallelopiped that can be inscribed in the ellipsoid ⎛ a ⎞ ⎛ b ⎞ ⎛ c ⎞ 8abc V = 8 xyz = 8 ⎜ = . ⎝ 3 ⎟⎠ ⎜⎝ 3 ⎟⎠ ⎜⎝ 3 ⎟⎠ 3 3 Exercise 4.9 1. Find stationary values of the function f (x, y, z) = x2 + y2 + z2, given that z2 = xy + 1. [Ans. : (0, 0, −1), (0, 0, 1)] 2. Find the stationary value of a3 x2 + b3 y2 + c3 z2 subject to the fulfillment of 1 1 1 the condition + + = 1, given a, x y z b, c are not zero. 1 ⎡ ⎤ ⎢ Ans. : x = a (a + b + c), ⎥ ⎢ ⎥ 1 ⎢ y = (a + b + c), ⎥ ⎢ ⎥ b ⎢ ⎥ 1 ⎢ z = (a + b + c) ⎥ c ⎣⎢ ⎦⎥ 3. Find the largest product of the numbers x, y and z when x + y + z2 = 16. 4096 Ans. : 4. Find the largest product of the numbers x, y and z when x2 + y2 + z2 = 9. Ans. : 3 3 5. Find a point in the plane x + 2y + 3z = 13 nearest to the point (1, 1, 1).

Ans. :

2

, 2,

2

6. Find the shortest distance from the point (1, 2, 2) to the sphere x2 + y2 + z2 = 36. [Ans. : 3] 7. the origin (0, 0) to the curve 3x2 + 3y2 + 4xy – 2 = 0. Ans. : 2 8. Decompose a positive number a into three parts so that their product is Ans. :

a a a , , 3 3 3 xm yn zp

9. when x + y + z = a. Ans. :

a m+ n+ p mm nn p p

( m + n + p )m + n + p

10. Find the dimensions of a rectangular surface area is given when

Engineering Mathematics

4.154

inscribed in the ellipse 4x2 + y2 = 36. ⎡ ⎢ Ans. : (i) ⎢ ⎢ (ii ) ⎢ ⎣

s , 3 s , 6

s 1 s ⎤ , ⎥ 3 2 3 ⎥ s s ⎥ , ⎥ 6 6 ⎦

11. Determine the perpendicular distance of the point (a, b, c) from the plane lx + my + nz = 0. ⎡ Ans. : minimum distance ⎤ ⎢ ⎥ la + mb + nc ⎢ ⎥ l 2 + m2 + n2 ⎣⎢ ⎦⎥

Ans. :

3 2 , 2, Area = 12 2

13. Find the volume of the largest rectangular parallelopiped that can be inscribed in the ellipsoid of revolution 4x2 + 4y2 + 9z2 = 36. Ans. : 16 3 14. Find the extreme volume of x2 + y2 + z2 + xy + xz + yz subject to the conditions x + y + z = 1 and x + 2y + 3z = 3. 1 1 5⎤ ⎡ ⎢⎣ Ans. : 6 , 3 , 6 ⎥⎦

12. Find the length and breadth of a rectangle of maximum area that can be FORMULAE Chain Rule z z dz ∂u dz ∂u ⋅ = ⋅ , = y x du ∂x du ∂y where z = f (u) and u = f (x, y) Total Differential Coefficient du ∂u dx ∂u dy ⋅ ⋅ = + dt ∂x dt ∂y dt where u = f (x, y) and x = f (t), y = y (t) du u dy (ii) = u dx + + u dz dt y dt x dt z dt where u = f (x, y, z) and x = f (t), y = y (t), z = x (t), (i)

Composite Function of Two Variables z z y = z x + u y u x u z z x z y = + v x v y v where z = f (x, y) and x = f (u, v), y = y (u, v)

Implicit Functions f/ x dy = dx f/ y where f (x, y) = c and y is a function of x. Euler’s Theorem and deductions u u (i) x +y = nu y y u (ii) x u + y + z u = nu y x z 2 2 2 u u u (iii) x2 2 + 2xy + y2 2 x y y x = n(n – 1)u f (u ) u =n (iv) x u + y y f (u ) x 2 2 2 u u u (v) x2 2 + 2xy + y2 2 x y y x = g(u) [g (u) – 1] f (u ) where g(u) = n f (u )

Partial Differentiation

4.155

MULTIPLE CHOICE QUESTIONS Choose the correct alternative in each of the following: 1. If z = f (x + ay) + f (x – ay), then (b) zxx = a2 zyy (a) zxx = zyy 2 (c) zyy = a zxx (d) none of these 2. If x = log (x tan–1 y), then fxy is equal to 1 (a) (b) 0 x2 (c) 12 (d) none of these x x2 y 2 z 2 3. If u = x y z , then

4.

5.

6.

7.

8.

1 1 1 ux + uy + uz is equal to (a) 0 (b) x y z (c) x + y + z (d) none of these x x If z = cos ⎛⎜ ⎞⎟ + sin ⎛⎜ ⎞⎟ , then ⎝ y⎠ ⎝ y⎠ x z + y z is equal to x y (a) z (b) 2z (c) 0 (d) none of these 3 3 If u = log (x + y + z3 – 3xyz), then xux + yuy + zuz is equal to (a) 3u (b) 2u (c) 3 (d) none of these If u = x2 + y2 + z2 be such that xux + yuy + zuz = lu then, l is equal to (a) 1 (b) 2 (c) 3 (d) none of these If f (x, y, z) = 0, then the value of ∂x ∂y ∂z is ⋅ ⋅ ∂y ∂z ∂x (a) 1 (b) –1 (c) 0 (d) none of these y If u(x, y) = x2 tan–1 ⎛ ⎞ ⎝x⎠ x – y2 tan–1 ⎛⎜ ⎞⎟ , x > 0, y > 0, then ⎝ y⎠ 2

x2

2 2 u u u + 2xy + y2 2 is equal to 2 x y y x

(a) 0 (b) 2u (c) u (d) 3u 2 9. If f (x, y) = e xy , the total differential of the function at the point (1, 2) is (a) e(dx + dy) (b) e4 (dx + dy) (c) e4 (4dx + dy) (d) 4e4 (dx + dy) dy is equal to 10. If f (x, y) = 0, then dx f f y x (a) (b) f f x y f f y (c) – (d) – x f f x y 11. The function f (x, y) = 2x2 + 2xy – y3 has (a) only one stationary point at (0, 0) (b) two stationary points at (0, 0) 1 1 , and 6 3 (c) two stationary points at (0, 0) and (1, –1) (d) no stationary points 12. If z = f (x, y), dz is equal to

( )

⎛ ∂f ⎞ ⎛ ∂f ⎞ (a) ⎝ ⎠ dx + ⎜ ⎟ dy ∂x ⎝ ∂y ⎠ ⎛ ∂f ⎞ ⎛ ∂f ⎞ (b) ⎜ ⎟ dx + ⎝ ⎠ dy ∂x ⎝ ∂y ⎠ ∂f ∂f (c) ⎛ ⎞ dx – ⎛⎜ ⎞⎟ dy ⎝ ∂x ⎠ ⎝ ∂y ⎠ ∂f ∂f (d) ⎛⎜ ⎞⎟ dx – ⎛ ⎞ dy ⎝ ∂x ⎠ ⎝ ∂y ⎠ 13. The function z = 5xy – 4x2 + y2 – 2x – y + 5 has at x = 1 , y = 18 41 41 (a) maxima (b) saddle point

Engineering Mathematics

4.156

14.

15.

16.

17.

18.

(c) minima (d) none of these If f (x, y) is such that fx = ex cos y and fy = ex sin y, then which of the following is true (a) f (x, y) = ex+y sin (x + y) (b) f (x, y) = ex sin(x + y) (c) f (x, y) does not exist (d) none of these The percentage error in the area of a rectangle when an error of 1% is made in measuring its length and breadth is equal to (a) 1% (b) 2% (c) 0 (d) 3% The function f (x) = 10 + x6 (a) is a decreasing function of x (b) has a minimum at x = 0 (c) has neither a maximum nor a minimum at x = 0 (d) none of these If u = f (y + ax) + f (y – ax), 2 2 u u 2 then – a is y2 x2 (a) 0 (b) a2 2 (c) a (f – f ) (d) a2 (f + f ) With usual notations, the properties of maxima and minima under various conditions are, I II (P) Maxima (i) rt – s2 = 0

Answers 1. (c) 8. (b) 15. (b)

2. (b) 9. (d) 16. (c)

3. (a) 10. (d) 17. (a)

(Q) Minima (ii) rt – s2 < 0 (R) Saddle (iii) rt – s2 > 0, point r>0 (S) Failure (iv) rt – s2 > 0 case r m, i.e., lim un = l. n

(i) If the sequence {un} has a finite limit, i.e., lim un is finite, the sequence is said n to be convergent.

5.2

e.g.

Engineering Mathematics

⎧ ⎫ ⎪ 1 ⎪ {un } = ⎨ ⎬ ⎪1 + 1 ⎪ ⎩ n⎭ lim un = 1

n

Since limit is finite, the sequence is convergent. (ii) If the sequence {un} has infinite limit, i.e., lim un is infinite, the sequence is said n to be divergent. e.g.

{un} = {2n + 1} lim un

n

Since limit is infinite, the sequence is divergent. (iii) If the limit of the sequence {un} does not exist, i.e., lim un is not unique, the n sequence is said to be oscillatory. e.g.

{un} = ( −1) n +

1 2n

lim un = 1, if n is even

n →∞

= –1, if n is odd Since limit is not unique, the sequence is oscillatory. Note 1: Every convergent sequence is bounded but the converse is not true. Note 2: A monotonic increasing sequence converges if it is bounded above and diverges to + if it is not bounded above. Note 3: A monotonic decreasing sequence converges if it is bounded below and diverges to – if it is not bounded below. Note 4: If sequence {un} and {vn} converges to l1 and l2 respectively, then (i) Sequence {un + vn} converges to l1 + l2 (ii) Sequence {un . vn} converges to l1 l2 l ⎧u ⎫ (iii) Sequence ⎨ n ⎬ converges to 1 provided l2 l2 ⎩ vn ⎭

0.

If u1, u2, u3, . . . un, . . . is an infinite sequence of real numbers, then the sum of the terms of the sequence, u1 + u2 + u3 + … + un + … ∞ is called an infinite series. The infinite series u1 + u2 + u3 + . . . un + . . .

is usually denoted by

un or n =1 th

un .

The sum of its first n terms is denoted by Sn and is also known as n partial sum of un .

5.3

Consider the infinite series un u1 u2 u3 …un … and let the sum of the first , three possibilities arise for Sn: n terms is Sn = u1 + u2 + u3 + . . . + un. As n (i) If Sn tends to a finite limit as n (ii) If Sn tends to

as n

, the series un is said to be convergent.

, the series un is said to be divergent.

(iii) If Sn does not tend to a unique limit as n series un is said to be oscillatory.

, i.e., limit does not exist, the

1. The convergence or divergence of an infinite series remains unaffected: (i) by addition or removal of a finite number of its terms. (ii) by multiplication of each term with a finite number. 2. If two series un and vn are convergent, then (un vn ) is also convergent.

If a positive term series

un is convergent, then lim un = 0. n

Note: The converse of this result is not true, i.e., if lim un = 0, it is not necessary that n series will be convergent. un = 1 +

e.g.

lim un = lim

n

2 1

n

Sn = 1 +

Now,

1

Sn >

n 1 2

+

1 3

+… +

1 n

+…

=0 +

1 3

+… +

1 n

> 1+

1 n

+

1 n

+… +

1 n

n n

lim S n = lim n = ∞

n →∞

n →∞

Thus, the series is divergent. Hence, lim un = 0 is a necessary but not sufficient condition for convergence n of un .

5.4

Engineering Mathematics

Consider the geometric series a + ar + ar 2 +

+ ar n 1 +

2

n 1

S n = a + ar + ar +

+ ar

rn ) , r 1) , 1

a (1 1 a(r n r

if r 1 if r 1

lim r n = 0

(i) When | r | 1 ,

n

lim S n =

n

a is finite. 1 r

Hence, the series is convergent. lim r n

(ii) When r > 1,

n

lim S n

lim

n

n

a (r n 1) r 1

Hence, the series is divergent. (iii) When r = 1 Sn = a + a + a +

= na

lim S n

n

Hence, the series is divergent. (iv) When r = –1, series becomes a a a Sn

a a a

( 1) n 1 a

= 0, is n is even = a, if n is odd Hence, the series is oscillatory. (v) When r < –1, let r = –k lim S n =

n

where k > 0

a[1 − ( −1) n k n ] a[1 ( k ) n ] = lim n →∞ 1+ k 1+ k

= – , if n is even = + , if n is odd Hence, the series is oscillatory. From all the above cases, we conclude that the geometric series (1) is (i) Convergent if | r | 1 (ii) Divergent if r 1 (iii) Oscillatory if r

1

... (1)

5.5

1 1 1 1 = p+ p+ p+ p 1 2 3 n =1 n (i) Convergent if p > 1 (ii) Divergent if p 1

Note: The p series

(i) lim n

log n =0 n

(ii) lim 1 + n

1 n

is

(vii) lim x n

n

(viii) lim nx n = 0 if x < 1

=e

n

1 n

(iii) lim(n) = 1 n

(ix) nlim

xn = 0 for all x n!

(x) lim

an 1 = log a n

1

(iv) lim(n !) n n

n! (v) lim n n

n

1 n

if x > 1

n

0

1

an 1 = log a (xi) lim n 1 n

1 = e

(vi) lim x n = 0 if x < 1 n

If un and vn are series of positive terms such that lim n

un = l (finite and non-zero), vn

then both series converge or diverge together. Proof:

un =l vn

lim

n

By definition of limit, for a positive number , however small, there exists an integer m such that un vn

− ∈
m

un − l m

l

l − ∈
m

5.6

Engineering Mathematics

m terms of

un and

l − ∈
1, i.e., p > 0 and divergent if p 1 1,

0.

Hence, by comparison test,

un is also convergent if p > 0 and divergent if p

Test the convergence of the series

0.

14 24 34 + + +…. 13 23 33

nth term of the numerator = a + (n – 1)d = 14 + (n – 1)10 = 10n + 4 n term of the denominator = n3 th

5.9

nth term of the given series, Let vn =

10n + 4 1 4 = 2 10 + 3 n n n

un =

1 n2 lim

n

un 4 = lim 10 + n vn n = 10 (finite and non-zero)

1 is convergent since p = 2 > 1. n2 Hence, by comparison test, un is also convergent. and S vn =

Test the convergence of the series

nth term of the series, un =

Let vn =

lim

un = lim vn n

=

vn

n = a n2 b

1 n a+

b n2

1 n n

and

1 2 3 + + +…. a 12 + b a 22 + b a 32 + b

1 a+

b n2

1 (finite and non-zero) a

1 is divergent since p = 1. n

Hence, by comparison test,

un is also divergent.

Test the convergence of the series

nth term of the series

un =

1 1 1 + p + p +…. p 3 5 7

1 = (2n + 1) p

1 np 2 +

1 n

p

5.10

Engineering Mathematics

Let vn =

1 np

lim

n

un = lim vn n =

1 2+

1 n

p

1 (finite and non-zero) 2p

1 is convergent if p > 1 and divergent if p 1. np Hence, by comparison test, un is also convergent if p > 1 and divergent if p 1 . and

vn

Test the convergence of the series 1 1 1 1 1 + + + + + … , where x is a positive fraction. x x 1 x 1 x 2 x 2 Since it is an infinite series, by ignoring the first term, the series can be rewritten as un =

1

+

1

+

1

+

1

x 1 x 1 x 2 x 2 2x 2x = 2 2+ 2 + …… x 1 x 22 2x = x2 n2 2x 2x un = 2 = x n2 x2 n2 2 1 n

Let vn =

+ ……

1 n2 lim

n

un 2x = lim 2 n vn x 1 n2 = –2x (finite and non-zero)

and

vn

1 is convergent since p = 2 > 1. n2

Hence, by comparison test,

un is also convergent.

5.11 Exercise 5.1 1. Test the convergence of the following series: (i)

1 n2 + 1

(ii)

(

n 1

(iii)

(

n4 1

n

)

n4 1

)

np

(iv)

n +1 + n

(v)

np (n + 1) q

(vi)

1 n +1 log n n

(vii) (viii)

1 n tan

tan

a+

n

b n

1 n

1 n

1+ 2

.

(ii)

un is divergent if l > 1.

2 3 + + …… . 1+ 2 3 1+ 3 4

(1 + a )(1 + b) 1 1 1 1 + p + p + p + p +…… . 1 2 3 3 5 7 9

[Ans. : Convergent if p > 1, Divergent if p

n

un is convergent if l < 1.

+

4. Test the convergence of the series

un is a positive term series and lim (i)

1 2

[Ans. : Divergent]

⎡ Ans. : ⎤ ⎢ (i) Convergent ⎥ ⎢ ⎥ ⎢ (ii) Divergent ⎥ ⎢ ⎥ ⎣ (iii) Convergent ⎦

If

⎤ ⎥ ⎥ 1 ⎥ Divergent if p ≥ − ⎥ 2 ⎥ Convergent if p − q + 1 < 0, ⎥ Divergent if p − q + 1 ≥ 0 ⎥ ⎥ Convergent ⎥ ⎥ Convergent ⎥ Divergent ⎥ ⎥ Convergent if a > 1, ⎥ ⎥⎦ Divergent if a ≤ 1 Convergent if p < −

2. Test the convergence of the series (1 + a )(1 + b) (2 + a )(2 + b) + + 1⋅ 2 ⋅ 3 2 ⋅3⋅ 4 (3 + a )(3 + b) +… . 3⋅ 4 ⋅5 [Ans. : Divergent] 3. Test the convergence of the series 1

1

1

(ix)

⎡ ⎢ (iv) ⎢ ⎢ ⎢ ⎢ ⎢ (v) ⎢ ⎢ ⎢ (vi) ⎢ (vii) ⎢ ⎢(viii) ⎢ (ix) ⎢ ⎢⎣

un +1 = l , then un

1.]

5.12

Engineering Mathematics

Proof: Case I: If lim n

un +1 un

l 1.

Consider a number l < r < 1 such that m terms, ∞

∑u

n

n = m +1

un +1 un

r for all n > m

… (1)

⎞ ⎛ u u u = um +1 + um + 2 + um + 3 + … ∞ = um +1 ⎜1 + m + 2 + m + 3 + m + 4 + … ⎟ ⎠ ⎝ um +1 um +1 um +1 ⎛ u ⎞ u u u u u = um +1 ⎜1 + m + 2 + m + 3 ⋅ m + 2 + m + 4 ⋅ m + 3 ⋅ m + 2 + … ⎟ ⎝ um +1 um + 2 um +1 um + 3 um + 2 um +1 ⎠ [ Using Eq. (1)] < um +1 (1 + r + r ⋅ r + r ⋅ r ⋅ r + …)

1 = um +1 (1 + r + r 2 + r 3 + …) = um +1 ⋅ − 1 r ∞ um + 1 ∑ un < 1 − r (finite) n = m +1

(r < 1)



The series



un is convergent.

n = m +1

The nature of a series remains unchanged if we neglect a finite number of terms in the ∞

beginning. Hence, the series

∑u

n

is convergent.

n =1

lim

Case II: If

n

un +1 un

l 1, un +1 un

1 for all n

m

... (2)

Neglecting the first m terms, ∞

∑u

n = m +1

n

= u m +1 + u m + 2 + u m + 3 + u m + 4 + … ∞ ⎛ u ⎞ u u = um +1 ⎜1 + m + 2 + m + 3 + m + 4 + …⎟ ⎝ u m +1 u m +1 u m +1 ⎠ ⎛ u ⎞ u u u u u = um +1 ⎜1 + m + 2 + m + 3 ⋅ m + 2 + m + 4 ⋅ m + 3 ⋅ m + 2 + …⎟ > um +1 (1 + 1 + 1 + 1 + …) ⎝ u m + 1 u m + 2 u m +1 u m + 3 u m + 2 u m + 1 ⎠

Consider, (um +1 um + 2 … to n terms) Sn

lim S n

n

um +1 (1 1 1… to n terms)

um +1 (n) lim num +1

n

[∵ um +1 is positive]

5.13





The series

un is divergent. The nature of a series remains unchanged if we

n = m +1



neglect a finite number of terms in the beginning. Hence, the series

∑u

n

is divergent.

n =1

un +1 = 1, the ratio test fails, i.e., no conclusion can be drawn about the un convergence or divergence of the series.

Note 1: If lim n

Note 2: It is convenient to use D’Alembert’s ratio test in the following form: u If un is a positive term series and lim n = l , then n un +1 (i) un is convergent if l > 1. (ii) un is divergent if l < 1. (iii) The ratio test fails if l = 1. Test the convergence of the following series: 2! 3! 4! (i) + + +… 3 32 3 3 (iv)

(ii)

2

2 2 ⋅ 5 2 ⋅ 5 ⋅ 8 2 ⋅ 5 ⋅ 8 ⋅ 11 + + + +… 1 1 ⋅ 5 1 ⋅ 5 ⋅ 9 1 ⋅ 5 ⋅ 9 ⋅ 13 (i)

2

2

⎛ 1 ⎞ ⎛ 1⋅ 2 ⎞ ⎛ 1⋅ 2 ⋅ 3 ⎞ + + …∞ (iii) ⎜ ⎟ + ⎜ ⎝ 3 ⎠ ⎝ 3 ⋅ 5 ⎟⎠ ⎜⎝ 3 ⋅ 5 ⋅ 7 ⎟⎠

n! nn

(v)

n!(2) n . nn

(n + 1)! 3n (n + 2)! = n +1 3 (n + 1)! 3n +1 lim n (n + 2)! 3n

un = un +1 lim

n

un un +1

lim

n

3 n+2

0 1

Hence, by ratio test, the series is divergent. un =

(ii)

un +1 =

lim

n

un un +1

n! nn

(n + 1)! (n + 1) n +1 n! nn lim n (n + 1)! (n + 1) n +1

lim

n

(n + 1)(n + 1) n (n + 1)n n

Hence, by ratio test, the series is convergent.

lim 1

n

1 n

n

e 1

5.14

Engineering Mathematics

2

2

2

1 1⋅ 2 ⎞ ⎛ 1⋅ 2 ⋅ 3 ⎞ (iii) The series is given by ⎛⎜ ⎞⎟ + ⎛⎜ + +…∞ ⎝ 3 ⎠ ⎝ 3 ⋅ 5 ⎟⎠ ⎜⎝ 3 ⋅ 5 ⋅ 7 ⎟⎠ ⎡ 1⋅ 2 ⋅ 3… n ⎤ un = ⎢ ⎥ ⎣ 3 ⋅ 5 ⋅ 7 … (2n + 1) ⎦

2

⎡ ⎤ 1⋅ 2 ⋅ 3 … n(n + 1) u n +1 = ⎢ ⎥ ( 2 n + 3 ) 3 5 7 … ( 2 1 )( ⋅ ⋅ n + ⎣ ⎦

lim

n →∞

un u n +1

⎡ 1 ⋅ 2 ⋅ 3… n ⎤ ⎢ 3 ⋅ 5 ⋅ 7 … (2n + 1) ⎥ ⎣ ⎦

2

2

2

3⎞ ⎤ ⎡⎛ 2+ ⎟ ⎥ ⎜ ⎢ ⎝ ⎡ (2n + 3) ⎤ n⎠ ⎥ = 4 >1 = lim = lim ⎢ = lim ⎢ ⎥ 2 n →∞ n →∞ n →∞ ⎛ ⎢ 1+ 1⎞ ⎥ ⎣ (n + 1) ⎦ ⎡ ⎤ 1 ⋅ 2 ⋅ 3… n(n + 1) ⎢⎣ ⎜⎝ n ⎟⎠ ⎥⎦ ⎢ 3 ⋅ 5 ⋅ 7 …(2n + 1)(2n + 3) ⎥ ⎣ ⎦ 2

Hence, by ratio test, the series is convergent. 2 2.5 2 ⋅ 5 ⋅ 8 2 ⋅ 5 ⋅ 8 ⋅ 11 + + + +… 1 1.5 1 ⋅ 5 ⋅ 9 1 ⋅ 5 ⋅ 9 ⋅ 13 2 ⋅ 5 ⋅ 8 ⋅ 11… (3n − 1) un = 1 ⋅ 5 ⋅ 9 ⋅ 13… (4n − 3)

(iv) The series is given by

un+1 =

lim

n →∞

un u n +1

2 ⋅ 5 ⋅ 8 ⋅ 11… (3n − 1)(3n + 2) 1 ⋅ 5 ⋅ 9 ⋅ 13… (4n − 3)(4n + 1)

2 ⋅ 5 ⋅ 8 ⋅ 11… (3n − 1) 1 4+ 4n + 1 1 ⋅ 5 ⋅ 9 ⋅ 13… (4n − 3) n = 4 >1 = lim = lim = lim n →∞ 2 ⋅ 5 ⋅ 8 ⋅ 11…( (3n − 1)(3n + 2) n→∞ 3n + 2 n→∞ 2 3 3+ 1 ⋅ 5 ⋅ 9 ⋅ 13… (4n − 3)(4n + 1) n

Hence, by ratio test, the series is convergent. n !(2) n (v) un = nn ( n + 1)!( 2) n +1 un +1 = ( n + 1) n +1 u lim n n un +1

n !(2) n nn lim n (n + 1)!(2) n +1 (n + 1) n +1 lim

n

1 n +1 2 n

n

lim

n

(n + 1)(n + 1) n n !2n (n + 1)n !2n 2 n n

1 1 lim 1 n 2 n

Hence, by ratio test, the series is convergent.

n

e 2

1

5.15

Test the convergence of 1 +

2p 3p 4p + + + … ,( p > 0). 2! 3! 4!

np n! (n + 1) p = (n + 1)!

un = un +1

lim

n

un un +1

np = lim n ! p n (n + 1) (n + 1)! lim

n

np (n + 1)! p n! (n + 1)

lim

(n + 1)

n

1+

1 n

p

1 Hence, by ratio test, the series is convergent. Test the convergence of the series

un =

u n +1 = un n →∞ u n +1 lim

xn 1 . n n=1 n 3

xn 1 n 3n

xn (n + 1) 3n +1

x n −1 n = lim n ⋅ 3n n →∞ x (n + 1) 3n +1 (n + 1) 3 3 ⎛ 1⎞ = lim ⎜1 + ⎟ n →∞ x⋅n x n→∞ ⎝ n ⎠ 3 = x

= lim

Hence, by ratio test, the series is convergent, if 3 x 3 if 1, i.e., x > 3. x For x = 1, lim n

un un +1

3 1, the series is convergent.

1, i.e., x < 3 and divergent

5.16

Engineering Mathematics

n

Test the convergence of the series

n +1 2

n n x n2 + 1

un =

(n + 1) x n +1 (n + 1) 2 + 1

un +1 lim

n

un n (n + 1) 2 + 1 1 lim 2 xn un +1 n n +1 n +1 x n +1 lim

n

n (n 2 + 2n + 2) 1 (n + 1) x (n 2 + 1) 1+

lim

n

=

1 1+ n

2 2 + n n2

1 x

Test the convergence of the series

u n +1 =

lim

n →∞

un u n +1

x2n

1 x

1 1+ 2 n

Hence, by ratio test, the series is convergent if 1 x 1 1, i.e., x > 1. Ratio test fails for x = 1. x

un =

xn .

1, i.e., x < 1 and is divergent if

1 2 1

+

x2 3 2

+

x4 4 3

+

x6 5 4

+… .

2

(n + 1) n

x2n ( n + 2) n + 1

1 ⎛ 2⎞ 1+ ⎟ 1+ ⎜ ⎝ n⎠ ( n + 2) n + 1 x n 1 = lim = lim ⋅ ⋅ 2 2n n →∞ n →∞ 1 x x ⎛ ⎞ (n + 1) n ⎜⎝1 + ⎟⎠ n 2n− 2

=

1 x2

Hence, by ratio test, the series is convergent if 12 x 1 2 1, i.e., x > 1. Ratio test fails for x = 1. x2

1, i.e., x2 < 1 and is divergent if

5.17 Exercise 5.2 Test the convergence of the following series: 1. 1

2.

22 2!

32 3!

42 … . 4! [Ans. : Convergent]

1 2 1 + 2 1 + 22

3 … . 1 + 23

xn . (2n )!

10.

[Ans. : Convergent] 11.

n +1 n ⋅ x , x > 0. n3 + 1



[Ans. : Convergent] 2! 3. 1 2 2

4.

3! 33

4! … . 44 [Ans. : Convergent]

12. x + 2 x 2 + 3 x 3 + 4 x 4 + … . Ans. : Convergent for x < 1, divergent for x > 1

n2 .. 3n [Ans. : Convergent]

5.

Ans. : Convergent for x < 1, divergent for x > 1

13. 1 +

2n 1 . 3n + 1

x x 2 x3 xn + + +… + 2 +… . 2 5 10 n +1 Ans. : Convergent for x < 1, divergent for x > 1

[Ans. : Convergent] 6.

7.

8.

1 . n!

14. [Ans. : Convergent]

Ans. : Convergent for x < 1, divergent for x > 1

[Ans. : Convergent]

3 2 8 3 15 4 15. x + x + x + x + … . 5 10 17 n2 − 1 n + 2 x + …∞ n +1

n 2 (n + 1) 2 . n!

xn ,x 3 n2 n

x x2 x3 + + + … ∞. 1⋅ 3 3 ⋅ 5 5 ⋅ 7

0.

Ans. : Convergent for x < 1, divergent for x > 1

Ans. : Convergent for x < 3, divergent for x > 3 3n − 2 n −1 ⋅ x , x > 0. 9. ∑ n 3 +1 Ans. : Convergent for x < 1, divergent for x > 3

16.

x 2 3

+

x2 3 4

+

x3 4 5

+…

.

Ans. : Convergent for x < 1, divergent for x > 1

5.18

If

Engineering Mathematics

un 1 un +1

un is a positive term series and nlim n (i)

un is convergent if l > 1

(ii)

un is divergent if l < 1

l , then

(iii) Test fails if l = 1 un

Proof: (i) Consider a number p such that p > 1. The series if p > 1. By comparison test,

un will be convergent if from and after some term un un +1

vn vn +1

un un +1

1

n

un 1 un +1

lim n

un 1 un +1

n

n

(n + 1) p np 1 n

p n

p

1

1 n

1 p n

p

p ( p 1) … 2!n 2

p ( p 1) … 2n 2

lim p

n

l Hence,

1 is convergent np

p ( p 1) … 2n

p 1

un is convergent if l > 1.

(ii) Consider a number p such that p < 1. The series By comparison test,

vn

1 is divergent if p < 1. np

un will be divergent if from and after some term un un +1

vn vn +1

Proceeding as above in the case (i), we get lim n

n

un 1 un +1

lim p

n

l Hence,

p ( p 1) … 2n

p 1

un is divergent if l < 1.

(iii) Raabe’s test fails if l = 1 and other tests are required to check the nature of the series. Note: When Raabe’s test fails, logarithmic test can be applied.

5.19

Test the convergence of the series un =

un + 1 =

2 2⋅5 2⋅5⋅8 + + +… . 7 7 ⋅ 10 7 ⋅ 10 ⋅ 13

2 ⋅ 5 ⋅ 8…… (3n − 1) 7 ⋅10 ⋅13…… (3n + 4)

2 ⋅ 5 ⋅ 8… (3n − 1) (3n + 2) 7 ⋅ 10 ⋅ 13… (3n + 4) (3n + 7)

un 2 ⋅ 5 ⋅ 8… (3n − 1) 7 ⋅10 ⋅13… (3n + 4)(3n + 7) 3n + 7 = = ⋅ un+1 7 ⋅10 ⋅13… (3n + 4) 2 ⋅ 5 ⋅ 8…(3n − 1)(3n + 2) 3n + 2

lim n

n

un 1 un +1

lim n

n

3n + 7 1 3n + 2

lim

n

5n 3n + 2

lim

n

5 3+

2 n

5 1 3

Hence, by Raabe’s test, the series is convergent. Test the convergence of the series

4 7……(3n 1) x n . n!

4 ⋅ 7… (3n + 1) x n n! 4 ⋅ 7… (3n + 1)(3n + 4) x n +1 = (n + 1)!

un = un +1

un 4 ⋅ 7… (3n + 1) x n (n + 1)! n +1 ⋅ = = n! un +1 4 ⋅ 7… (3n + 1)(3n + 4) x n +1 (3n + 4) x

1 1+ un n = 1 lim = lim n →∞ u n →∞ ⎛ 4 3x ⎞ n +1 ⎜⎝ 3 + ⎟⎠ x n By ratio test, the series is

1 1 1 or x 3 3x 1 1 (ii) Divergent if 1 or x 3x 3 1 (iii) Test fails if x = 3 un n +1 3n + 3 Then = = 1 3n + 4 un+1 (3n + 4) 3 (i) Convergent if

5.20

Engineering Mathematics

Applying Raabe’s test, ⎛ ⎞ ⎜ −1 ⎟ ⎛ un ⎞ 1 ⎛ −n ⎞ ⎛ 3n + 3 ⎞ lim n ⎜ − 1⎟ = lim n ⎜ − 1⎟ = lim ⎜ ⎜ ⎟ = − 1 x (iii) Test fails if x = 1 (ii) Divergent if

Then

un (n + 1)(c + n) = un +1 (a + n)(b + n)

5.22

Engineering Mathematics

Applying Raabe’s test,

lim n

n

un 1 un +1

lim n

n

(n + 1)(c + n) 1 (a + n)(b + n)

(c ab) n(1 c a b) = lim n = lim n n (a + n)(b + n)

(c ab) (1 c a b) n a b +1 +1 n n

1 c a b By Raabe’s test, the series is (i) Convergent if 1 c a b 1 or c (ii) Divergent if 1 c a b 1 or c a b.

a b

Hence, the series is convergent if x < 1 and divergent if x > 1. For x = 1, the series is convergent if c > a + b and divergent if c < a + b.

Exercise 5.3 Test the convergence of the following series: 1. 1 +

2.

Ans. : Convergent for x 1 and divergent for x > 1

1 1 1 + + + ... . 2 2 4 2 4 6 [Ans. : Divergent] 12 52 92 …… (4n 3) 2 . 42 82 122 …… (4n) 2

4. 1 +

22 22 4 2 22 4 2 6 2 + + +… . 32 32 52 32 52 72

[Ans. : Divergent]

[Ans. : Convergent] 3. (i) 1 +

22 22 4 2 22 4 2 6 2 + + +… . 3 4 3 4 5 6 3 4 5 6 7 8

[Ans. : Convergent] 2

(1!) (2!) 2 2 (3!) 2 3 x+ x + x +… . 2! 4! 6! Ans. : Convergent for x < 4 and divergent for x 4

(ii) 1 +

(iii) 1 + 3 x + 3 6 x 2 + 3 6 9 x 3 + … . 7 7 10 7 10 13

5. a (a + 1) + (a + 1)(a + 2) 2! 3! (a + 2)(a + 3) + + …. 4! [Ans. : Convergent for a 0 ] 6.

(n !) 2 2 n x . (2n)! Ans. : Convergent for x < 4 and 2 divergent for x 4

5.23

5.9 LOGARITHMIC TEST un is a positive term series and if nlim n log

If

(i) (ii)

un = l , then un +1

un is convergent if l > 1. un is divergent if l < 1.

(iii) Test fails if l = 1. Proof: Comparing the series Let

vn

(i) Let

1 , np

un with the p-series

1 which converges if p > 1 and diverges if p 1. np vn is convergent, then un will also be convergent if un un +1

vn vn +1

(n + 1) p np

un ⎛ 1⎞ > ⎜1 + ⎟ un +1 ⎝ n ⎠

p

⎛ u ⎞ ⎛ 1⎞ log ⎜ n ⎟ > log ⎜ 1 + ⎟ u ⎝ n⎠ ⎝ n +1 ⎠ ⎛ u ⎞ ⎛ 1⎞ log ⎜ n ⎟ > p log ⎜1 + ⎟ ⎝ n⎠ ⎝ un +1 ⎠ p

⎛ u ⎞ 1 1 ⎛1 log ⎜ n ⎟ > p ⎜ − 2 + 3 − ⎝ n 2n 3n ⎝ u n +1 ⎠ ⎛ u ⎞ 1 1 ⎛ n log ⎜ n ⎟ > p ⎜1 − + 2− ⎝ 2n 3n ⎝ u n +1 ⎠

⎞ ⎟⎠ ⎞ ⎟⎠

⎛ u ⎞ lim n log ⎜ n ⎟ > p ⎝ u n +1 ⎠

n →∞

l>p>1 Hence,



un is convergent if l > 1.

(ii) Let

vn is divergent, then

un will also be divergent if

Proceeding as above, we get lim n log

n

un is divergent if l < 1.

un v < n un +1 vn +1

un

1 or x < e x

e < 1 or x > e x e (iii) Test fails if 1 or x = e x (ii) Divergent if

1+

un un +1

Then

1+

2 n 1 n

n +1

n +1

1 e

Applying logarithmic test, log

un u n +1

2 n

(n 1)

1 n

(n 1) 1 lim n log

n

un u n +1

2 n

(n 1) log 1

lim

n

3 2n 1 2

(n 1) log 1

1 22 2 n2

1 23 3 n3

3 7 2n 2 3n3 7 1 3 3n 2 n 2n 2 5 6n

1 n

7 3n 2

log e 1 n

1 2n 2

7 3n3

1 1 2

Example 3: Test the convergence of the series

1 2n

5 6n 2

7 3n3

1

e.

a + x ( a + 2 x )2 ( a + 3 x )3 + + + …. 1! 2! 3!

Solution:

u n +1

1

1

By logarithmic test, the series is divergent if x = e Hence, the series is convergent if x < e and is divergent if x

un

1 3n3

(a + nx) n n! [a + (n + 1) x]n +1 (n + 1)!

5.26

un (a + nx) (n + !) = ⋅ un + 1 n! [a + (n + 1) x] n + 1 nx ⎡ ⎤ a a ⎞ ⎛ ⎢⎜1 + ⎟ ⎥ ⎛⎜1 + 1 ⎞⎟ ⎢⎝ nx ⎠ ⎥ ⎝ n⎠ ⎣ ⎦

n

a ⎞ ⎛ (nx) n ⎜1 + ⎟ (n + 1) 1 nx ⎠ ⎝ = ⋅ = n +1 a+x x a + x⎞ nx ⎛ ⎡ ⎤ x (nnx) n + 1 ⎜1 + ⎟ a + x ⎞ ⎢⎛ a ⎞a + x ⎥ ⎛ nx ⎠ ⎝ ⎟ ⎜1 + ⎟ ⎜1 + ⎥ nx ⎠ ⎢⎝ nx ⎠ ⎝ ⎣ ⎦ a

u lim n n u n +1

ex a

ex 1 ex

+1

n ⎤ ⎡ b ⎢∵ lim ⎛⎜1 + b ⎞⎟ = e ⎥ ⎥ ⎢ n→∞ ⎝ n ⎠ ⎦ ⎣

1 x

By ratio test, the series is (i) Convergent if (iii) Test fails if

1 ex

1 1 > 1 or x < ex e 1 or x

1 1

ex e

(ii) Divergent if

1 e un u n +1

Then

1+

ae n

1+

n

1 e n

1+

ae + 1 n

n +1

Applying logarithmic test,

log

un ⎛ ae + 1 ⎞ ⎛ ae ⎞ ⎛ 1⎞ = n log ⎜1 + ⎟ + log ⎜1 + ⎟ − (n + 1) log ⎜1 + ⎟ + log e un+1 n ⎠ n ⎠ ⎝ ⎝ ⎝ n⎠ ⎛ ae 1 a 2 e 2 1 a 3 e3 = n⎜ − ⋅ 2 + ⋅ 3 − 3 n ⎝ n 2 n

⎞ ⎛1 1 ⎟+⎜ − 2 + ⎠ ⎝ n 2n

⎞ ⎟ ⎠

⎡ ae + 1 1 ⎛ ae + 1 ⎞ 2 1 ⎛ ae + 1 ⎞3 − (n + 1) ⎢ − ⎜ ⎟ − ⎟ + ⎜ 2⎝ n ⎠ 3⎝ n ⎠ ⎢⎣ n ⎛ 1 a 2 e 2 1 a 3 e3 + ⋅ 2 − = ⎜ ae − ⋅ 2 n 3 n ⎝

⎞ ⎛1 1 ⎟+⎜ − 2 + ⎠ ⎝ n 2n

⎤ ⎥ +1 ⎥⎦

⎞ ⎟ ⎠

2 ⎡ 1 (ae + 1) 2 1 (ae + 1)3 (ae + 1) 1 ⎛ ae + 1 ⎞ + + − − ⎢(ae + 1) − ⎜ ⎟ + n n 2 3 n2 2⎝ n ⎠ ⎢⎣

⎤ ⎥ +1 ⎥⎦

5.27

lim n log

n→∞

⎡⎛ a 2 e 2 1 a 3 e3 un = lim ⎢⎜ − + − un + 1 n → ∞ ⎣⎝ 2 3 n

⎞ ⎛ 1 + ⎟ + ⎜1 − n 2 ⎠ ⎝

⎞ ⎟ ⎠

⎧ 1 1 (ae + 1)3 1 (ae + 1) 2 − ⎨− (ae + 1) 2 + (ae + 1) − + n 3 2 n ⎩ 2

⎫⎤ ⎬⎥ ⎭⎦

1 1 a 2 e2 + 1 + (a 2 e 2 + 1 + 2ae) − (ae + 1) = < 1 2 2 2 By logarithmic test, the series is divergent if x 1 . =−

e 1 Hence, the series is convergent if x < and is divergent if x e

1 . e

Exercise 5.4 Test the convergence of the following series: 22 x 2 33 x 3 44 x 4 55 x 5 + + + + 2! 3! 4! 5! 1 Ans.: Convergent if x < and e 1 divergent if x e

1. x +

2. 1 +

.

1 22 33 4 4 55 + + + + + . 22 33 4 4 55 66 [Ans. : Convergent]

3.

4. (a + 1)

2 32 43 54 x + x 2 + x3 + x 4 + 2! 3! 4! 5!

x x2 + ( a + 2) 2 1! 2!

x3 + . 3! Ans. : Convergent if xe < 1 and divergent if xe 1.

.

+ (a + 3) 2

Ans. : Convergent if xe 1 and divergent if xe > 1

5.10 CAUCHY’S ROOT TEST 1

If

un is a positive term series and if lim(un ) n

l , then

n

(i)

un is convergent if l < 1.

(ii)

un is divergent if l > 1.

Proof: 1

Case I: If lim(un ) n n

l 1. 1

Consider a number l < r < 1 such that (un ) n < r for all n > m un < r n for all n > m The geometric series

r

n

r r

2

r

3



... (1)

5.28

r r2

Sn

r3 … rn

r (1 r n ) 1 r r (1 r n ) lim n 1 r r , which is finite 1 r

lim S n

n

⎡∵ r < 1 ⎤ ⎢ ⎥ n r = 0⎥ ⎢⎣∴ nlim →∞ ⎦

r n is convergent.

Hence, the series

un < r n for all n > m

From Eq. (1),

un < rn Since

r n is convergent,

un is also convergent.

1

Case II: If lim(un ) n

l 1.

n

1

(un ) n > 1 for all n > m

… (1)

Neglecting the first m terms, 1

un

(um +1 ) m +1 1

Sn

1

1

(um + 2 ) m + 2

(um + 3 ) m + 3 …

1

(um +1 ) m +1

(um + 2 ) m + 2

1

1

(um + 3 ) m + 3 … (um + n ) m + n

lim S n

n

[Using Eq. (1)]

1 1 1…

1 1 1… n terms

n

lim n

n



The series

∑u

n

is divergent. The nature of a series remains unchanged if we

n = m +1



neglect a finite number of terms in the beginning. Hence, the series

∑u

n

is divergent.

n= 1 1

Note: If lim(un ) n n

1, the root test fails, i.e., no conclusion can be drawn about the

convergence or divergence of the series. Example 1: Test the convergence of the series 1 2 + 3 5 Solution:

2

+

un

3 7

3

n

+…+ n 2n + 1

n

n 2n + 1

+… .

5.29

1

lim(un ) n

n 2n + 1

lim

n

n

1

lim

n

2+

1 2

1 n

1

Hence, by root test, the series is convergent. Example 2: Test the convergence of the series 22 12 Solution:

lim(un )

33 23

1 n

n +1 n

1 n

lim

n

n

3 2

44 34

3

4 3

….

n +1 n 1

n +1

1 1 n

2

n

n +1

n +1 n

un

(un )

1

2 1

n +1 n

1 1 n

1

n

1 1 n

1 1 n

1

n

1 1 n

1 1 n

(e 1)

1

Hence, by root test, the series is convergent. 1

Example 3: Test the convergence of the series

1 1+ n un =

Solution:

⎛ 1⎞ ⎜1 + ⎟ ⎝ n⎠

1

n2

1 1+

1 n

.

1

(un ) n

lim(un ) n

n2

lim

1 n

n

1

n

1+

1 n

n

1 1 e

Hence, by root test, the series is convergent. Example 4: Test the convergence of the series Solution:

un

( n log n) n . 2 n nn

(n log n) n 2n n n

1 e 1

1

5.30

1

(n log n) 1 lim n n n 2n 2 1 1 1 lim n 1 2 n 1 2 Hence, by root test, the series is convergent. lim(un ) n

log n n

lim

[Using L’Hospital’s rule]

( n + 1) n x n . nn + 1

Example 5: Test the convergence of the series Solution:

(n + 1) n x n n n +1 1 ( n + 1) x ( n + 1) x ( un ) n = = n +1 1 nn n.n n 1 1 x lim(un ) n = lim 1 + n n n 1n n =x un

1 ⎡ ⎤ n n = 1 using indeterminate form (∞°) method ⎥ ⎢∵ lim x →∞ ⎣ ⎦

Hence, by root test, the series is convergent, if x < 1 and divergent if x > 1. Root test fails for x = 1. 1 2

Example 6: Test the convergence of the series

⎛ n ⎞ un = ⎜ ⎝ n + 1 ⎟⎠

Solution:

(un )

1 n

n n +1

2 x 3

1 n

lim

n

2

4 5

x2

3

x3 … .

n −1

x n −1 n 1 n

x

n 1 n

1

lim(un ) n

3 4

1 1+

n n +1

1 n

( x)

1 n

1 n 1

1 n

1

( x)

1 n

x

Hence, by root test, the series is convergent, if x < 1 and is divergent if x > 1. Root test fails for x = 1.

5.31

Exercise 5.5 Test the convergence of the following series: 1 1. 1 2 2

1 33

1 44

n

5.

… . [Ans. : Convergent]

2.

1 . (log n) n n= 2

3.

n +1 . 3n

nx . n +1 Ans. : Convergent for x < 1, divergent for x > 1

2 3 6. 1 + x + x + x + … ( x > 0) . 2 32 43 [Ans. : Convergent] [Ans. : Convergent]

n

7.

1 1+ n

n2

.

[Ans. : Convergent] [Ans. : Divergent]

3

1

1+

4.

n2

(1 + nx ) . nn Ans. : Convergent if x < 1 and divergent if x > 1 n

8.

.

n [Ans. : Convergent]

5.11 CAUCHY’S INTEGRAL TEST If 1

f ( n) is a positive term series where f (n) decreases as n increases and let

un

f ( x )dx = I , then (i)

un is convergent if I is finite

(ii)

un is divergent if I is infinite

Proof: Consider the area under the curve y = f (x) from x = 1 to x = n + 1 represented as



n +1

1

f ( x )dx. Plot the terms f (1), f (2), f (3)…….f (n), f (n + 1).

The area



n +1 1

f ( x)dx lies between the sum of the areas of smaller rectangles and sum

of the areas of larger rectangles f ( 2) + f (3) +

f ( n + 1) ≤ ∫

n +1

1

f ( x )dx ≤ f (1) + f ( 2) + f (3) +

Sn +1 − f (1) ≤ ∫

n +1

1

As n

f ( x )dx ≤ Sn

first inequality reduces to ∞

lim Sn +1 ≤ ∫ f ( x ) dx + f (1)

n →∞

1

+ f ( n)

5.32 y



This shows that if ∫ f ( x) dx 1 Σ f ( n) = Σun is convergent. As n

second inequality reduces to





1

f ( x ) dx ≤ lim Sn n →∞

y¢ = f (x)



lim Sn ≥ ∫ f ( x )dx

n →∞

or

1

f (1) f (2) f (3)



This shows that if ∫ f ( x)dx is infinite, 1 Σ f ( n) = Σun is divergent.

f (n)

1 2

f (n+1)

n n+1

3

x

Fig. 5.1

Example 1: Test the convergence of the series

1 . n = 2 n log n

Solution: un =

1 = f ( n) n log n

1 x log x ∞ m 1 1 dx = lim ∫ f ( x ) dx = ∫ dx 2 x log x m → ∞ 2 x log x f ( x) =



∞ 2

= lim log log x m →∞

m

⎡ ⎤ f ′( x ) ⎢∵ ∫ f ( x ) dx = log f ( x ) ⎥ ⎣ ⎦

2

= lim (log log m − log log 2) → ∞ m →∞

Hence, by Cauchy’s integral test, the series is divergent. n2 e

Example 2: Test the convergence of the series

n3

.

n=1

Solution:





1

un = n 2 e − n

3

f ( x) = x 2 e − x

3



f ( x)dx = ∫ x 2 e − x dx 3

1

⎡ 1 m 3 ⎤ = lim ⎢ − ∫ e − x (−3 x 2 )dx ⎥ 1 m →∞ ⎣ 3 ⎦ m 3 ⎡ 1 ⎤ = lim ⎢ − e − x ⎥ m →∞ 1 ⎦ ⎣ 3

(

)

⎡⎣∵ e f ( x ) f ′( x)dx = e f ( x ) ⎤⎦

3 1 ⎡ 1 ⎤ = lim ⎢ − e − m − e −1 ⎥ = − (e −∞ − e −1 ) m →∞ 3 ⎣ 3 ⎦ 1⎛ 1⎞ 1 = − ⎜0 − ⎟ = (finite) e ⎠ 3e 3⎝ Hence, by Cauchy’s integral test, the series is convergent.

5.33

Example 3: Show that the harmonic series of order p, 1 1 = p p 1 n =1 n

1 2p

1 3p



Solution:

un f ( x)

is convergent if p > 1 and is divergent if p

1 np 1 xp

f ( x)dx

1

1.

1

1 dx xp m

⎛ m1− p x − p +1 1 ⎞ = lim = lim ⎜ − m →∞ − p + 1 m →∞ ⎝ 1 − p 1 − p ⎟⎠ 1 1 , 1− p → ∞, =−

If

p >1 p 1 and is divergent if p 1. Exercise 5.6 Test the convergence of the following series. 1

1. n =1

n

2

ne n .

3.

.

n =1

[Ans. : Convergent]

[Ans. : Divergent] 2.

1 . n +1 n =1

4.

2

[Ans. : Convergent]

1 . 2 n =1 n(log n) [Ans. : Convergent]

5.34

5.12 ALTERNATING SERIES An infinite series with alternate positive and negative terms is called an alternating series. n 1 Leibnitz’s test for alternating series: An alternating series ( 1) un is convern =1 gent if (i) each term is numerically less than its preceding term, i.e, un +1 < un or un > un +1 (ii) lim un

0

n

Example 1: Test the convergence of the series 1

1

2

1

3

4

….

1

un =

Solution:

1

n

(i) The given series is an alternating series. (ii)

un

1

un +1

1

n 1

n +1

n

n

n n +1

0

un > un +1 (iii)

lim un = lim

n

n

1 n

=0

Hence, by Leibnitz’s test, the series is convergent. Example 2: Test the convergence of the series x Solution:

un =

x2 2

x3 3

x4 4

……(if x

1) .

xn n

(i) The given series is an alternating series. (ii) un

un +1

xn n

x n +1 x n [( n + 1) − nx ] = n +1 n( n + 1) =

x n [1 (1 x )n] > 0, n( n + 1)

[∵ 0 < x < 1]

un > un +1 xn =0 n n n Hence, by Leibnitz’s test, the series is convergent. (iii)

lim un = lim

∵ lim x n = 0 if x < 1 n

5.35

Example 3: Test the convergence of the series un =

Solution:

1 1p

1 2p

1 3p

1 4p

….

1 np

Case I: If p > 0, (i) The given series is an alternating series. 1 1 (ii) un un +1 p ( n + 1) p n =

( n 1) p n p > 0, n p ( n + 1) p

if p > 0

un > un +1 (iii) lim un = lim 1 = 0, if p > 0 n n np Hence, by Leibnitz’s test, the series is convergent if p > 0. Case II: If

p1 n →∞ | u n →∞ 3 n→ ∞ 1 | n + 1 n +1 1+ n | ln | is convergent and hence, the given series is absolutely

By ratio test, n =1

convergent. Example 2: Test the series for absolute or conditional convergence 2 3

3 1 4 2

4 1 5 3

5 1 …. 6 4

Solution:

un

( 1) n

| un | n =1

| un | =

Let vn =

1 n

n +1 1 ⋅ n+2 n

2 3

1

n +1 1 n+2 n

3 1 4 2

4 1 5 1 …. 5 3 6 4

5.37

1 1+ | un | n +1 n = 1 (finite and non-zzero) lim = lim = lim n →∞ v n →∞ n + 2 n →∞ 2 n 1+ n 1 and vn = is divergent since p = 1. n By comparison test, | un | is divergent. Hence, the given series is not absolutely convergent. To check for conditional convergence we need to check the convergence of the given series. (i) The given series

un is an alternating series.

(ii)

| un | | un +1|

n +1 n( n + 2) =

n+2 ( n + 1)( n + 3)

n2 + 3n + 3 >0 n( n + 1)( n + 2)( n + 3)

| un | > | un +1 | lim | un | = lim

(iii)

n

n

n +1 n( n + 2)

1 n =0 = lim n 2 n 1+ n 1+

By Leibnitz’s test, given series un is convergent. Thus, un is convergent and is divergent. Hence, the given series is conditionally convergent.

| un |

Exercise 5.8 Test the following series for absolute or conditional convergence: 1 1 1 1 1 4. 1 2 3 4 … . …. 2 5 10 17 2 3 4 5 6 [Ans. : Conditionally convergent] [Ans. : Conditionally convergent] 1 1 1 1 1 sin x sin 2 x sin 3 x 2. 1 2 …. 5. …. 2 32 4 2 52 6 2 13 23 33 [Ans. : Absolutely convergent] [Ans. : Absolutely convergent]

1. 1

3. 1

1

1

1

…. 2 3 4 [Ans. : Conditionally convergent]

5.38

5.14 UNIFORM CONVERGENCE OF A SERIES un ( x ) of real valued functions defined in the interval (a, b) is said to

The series n =1

0, there exists a number m

converge uniformly to a function S (x) if for a given independent of x such that for every x ( a, b), for all n > m

| Sn ( x ) S ( x ) |

Sn ( x ) = u1 ( x ) + u2 ( x ) + … + un ( x )

where,

Weierstrass’s M-Test: The series

un ( x ) is said to converge uniformly in an n =1

M n of positive constants such that

interval (a, b), if there exists a convergent series n =1

| un ( x ) | Proof: Let

M n for all x

( a, b) 0, there exists a number m such

M n is convergent, then for a given n =1

that | S − S n | < ∈ for all n > m, where S

M1

then | M n +1 M n +1

M2

M n+ 2 … |

M n+ 2 …

Now | un ( x ) |

M3 …

and Sn = M1 + M 2 + … + M n

for all n > m for all n > m

M n for all x

[∵ Mn is positive constant]

( a, b)

| un +1 ( x ) un + 2 ( x ) … | | un +1 ( x ) | | un + 2 ( x ) | … M n +1

M n+ 2 …

for all n > m | S ( x ) Sn ( x ) | where,

for all n > m

Sn (x) = u1(x) + u2 (x) + … + un (x)

Since m does not depend on x, the series terval (a, b). Example 1: Test the series Solution:

un ( x ) converges uniformly in the inn =1

1 for uniform convergence. 3 2 n=1 n + n x 4

un ( x ) =

1 n 4 + n3 x 2

5.39

| un ( x ) | =
0]

R

1 n4

4 > 1.

Hence, by M-test, the series is uniformly convergent for all real values of x. Example 2: Test the series Solution:

un ( x )

cos( x 2 + n 2 x) n(n 2 + 2) cos( x 2 + n2 x ) | sin( x 2 + n2 x ) | = n( n2 + 2) n( n2 + 2)

| un ( x ) | =

Mn = n =1

cos( x 2 + n2 x ) for uniform convergence. n( n2 + 2) n=1



1 n + 2n


1. 3 n n =1

Hence, by M-test, the series is uniformly convergent for all real values of x. Example 3: Test the series sin x

sin 2 x

sin 3 x

sin 4 x

2 2

3 3

4 4

convergence. Solution:

un ( x )

( 1) n

| un ( x ) | =

1

sin nx n n

sin nx n n

… . for uniform

5.40

1

for all x ∈ R

3 2

Mn =

n 1

⎡∵ −1 ≤ sin q ≤ 1⎤ ⎢ | sin q | ≤ 1⎥⎦ ⎣

3

n2 1

Mn = n =1

n =1

3 2

is convergent since p =

3 >1. 2

n Hence, by M-test, the series is uniformly convergent for all real values of x. r n cos n2 x is uniformly

Example 4: Show that if 0 < r < 1, the series n=1

convergent. Solution:

un ( x ) = r n cos n2 x un ( x ) = r n cos n2 x ≤| r n | for all x ∈ r = rn , 0 < r < 1

⎡∵− 1 ≤ cos ≤ 1⎤ ⎢ cos ≤ 1 ⎥ ⎣ ⎦

Mn = rn Mn = n =1

r n = r + r 2 + r3 + … n =1

which is convergent being a geometric series with 0 < r < 1. Hence, by M-test, the series is uniformly convergent for all real values of x. Exercise 5.9 Test the following series for uniform convergence: 4. Show that if 0 < r < 1, then the series sin( x 2 + nx ) 1. ; for all real x. n( n + 2) n =1 r n sin a n x is uniformly convergent n =1 [Ans. : Uniformly convergent] for all real values of x. 1 ; for all real x and p > 1. 2. 5. Show that p n + nq x 2 n =1 1 1 1 1 … [Ans. : Uniformly convergent] 1+ x2 2 + x2 3 + x2 4 + x2 sin x sin 2 x sin 3 x sin 4 x converges uniformly in the interval 3. + 2 + 2 + +…. 12 2 3 42 x 0 but not absolutely. [Ans. : Uniformly convergent]

5.41

FORMULAE Sequence A sequence {un} is said to be convergent, divergent or oscillatory according un is finite, infinite or not unique as nlim →∞ respectively. Series The infinite series un is said to be convergent, divergent or oscillatory accordor not unique ing as lim S n is finite, n →∞ respectively. If a positive term series un is convergent, then lim un = 0 but converse is not n →∞

true i.e., if lim un = 0, the series may con-

When Raabe’s test fails, Logarithmic test can be applied. Logarithmic test: If

un is a positive u ⎞ ⎛ n log n ⎟ = l, term series and if nlim →∞ ⎜ u n +1 ⎠ ⎝ then (i) un is convergent (ii) un is divergent (iii) Test fails

Cauchy’s root test: If un is a positive term 1

series and if lim (un ) n = l, then n→∞

n →∞

verge or diverge. If lim un n →∞

Comparison test: If un and vn are series of positive terms such that u lim n = l (finite and non-zero), n→∞ v n then both series converge or diverge together. D’Alembert’s ratio test: If un is a posiu tive term series and lim n = l, then n →∞ u n +1 (i) un is convergent if l > 1. (ii) un is divergent if l < 1. (iii) The ratio test fails if l = 1. When Ratio test fails, Raabe’s or Logarithmic Test can be applied. Raabe’s test (higher ratio test): If un is a positive term series and if ⎛ un ⎞ lim n − 1⎟ = l, then n→∞ ⎜ u ⎝ n +1 ⎠ (i) un is convergent (ii) un is divergent (iii) Test fails

(i) (ii)

0, the series

is divergent.

if l > 1 if l < 1 if l = 1

if l > 1. if l < 1. if l = 1.

un is convergent un is divergent

if l < 1. if l > 1.

This test is preferred when un contains nth powers of itself. Cauchy’s integral test: If un = f (n) is a positive term series where f (n) decreases as n increases and let (i) (ii)





1

f (x) dx = I, then

un is convergent if I is finite. un is divergent if I is infinite.

This test is preferred when evaluation of the integral of f (x) is easy. Alternating series: An infinite series with alternate positive and negative terms is called an alternating series. Leibnitz’s test: An alternating series ∞

∑ (−1)

n −1

un is convergent if

n =1

(i) each term is numerically less than its preceding term, i.e, un +1 < un or un > un +1 (ii) lim un = 0 n→∞

5.42

Absolute Convergence





The series

∑u

n

with both positive and

n =1

negative terms (not necessarily alternative) is called absolutely convergent if ∞

the corresponding series

∑ |u | n

with all



∑ | u | is divergent, then the series ∑ u n

n =1

n

n =1

is called conditionally convergent. ∞

Weierstrass’s M-test: The series

∑ u ( x) n

n =1

is said to converge uniformally in an

n =1

positive terms is convergent.

interval (a, b), if there exists a convergent

Conditional Convergence

series





If the series

∑u

∑M

n

of positive constants such

n =1

n

is convergent and

that |un(x)|

Mn for all x

(a, b).

n =1

MULTIPLE CHOICE QUESTIONS Choose the correct alternative in each of the following: ∞

1. The series

∑ cos ⎛⎝ 1n ⎞⎠

is

n =1

(a) convergent (c) oscillatory

(b) divergent (d) none of these



2. The series

n

∑ n x+ 1 3

at x = 1 is

n =1

(a) convergent (c) oscillatory

(b) divergent (d) none of these

3. The series 1 – 12 + 12 – 12 + … 2 3 4 is (a) convergent (b) divergent (c) oscillatory (d) none of these 4. The series 2 – 3 + 4 – 5 + … is 2 3 4 (a) convergent but not absolutely convergent (b) divergent (c) absolutely convergent (d) oscillates finitely 5. The series 1 + 1 + 1 + 1 + 2 3 4 … is (a) convergent but not absolutely convergent (b) oscillatory

(c) divergent (d) absolutely convergent 6. In a series of positive terms un if lim un 0, then series un is n→∞

(a) (b) (c) (d)

convergent divergent not convergent oscillatory 7. The series 1 – 1 + 1 – 3 + 1 – 7 4 2 8 … is (a) (b) (c) (d)

convergent conditionally convergent absolutely convergent oscillatory

8. The series

1 – 1 + 1 a +1 a+2 a+3

1 + … convergent if a+4 (a) a > 0 (b) a < 0 (c) a < –1 (d) none of these



9. The series 1 – 2x + 3x2 – 4x3 + … where 0 < x < 1 is (a) convergent (b) divergent (c) oscillatory (d) none of these 10. The series

1 2 + 1 + 2−1 1 + 2− 2

5.43

3 … is 1 + 2− 3 (a) convergent (c) oscillatory

+

(b) divergent (d) none of these

11. The series whose nth term is

n3 + 1

– n3 is (a) convergent (c) oscillatory

(b) divergent (d) none of these 1 12. The series whose nth term is sin is n (a) convergent (b) divergent (c) oscillatory (d) none of these 13. The series 2 + 3 + 4 + 5 + n +1 + … is 1 4 9 16 n2 (a) convergent (b) divergent (c) oscillatory (d) none of these 14. Which of the following is true? 1 1 1 (a) 1 + 1 + 1 + 1 + … is 2 3 33 4 3 convergent 1 1 + + 2 3 convergent 1 1 (c) 1 + 2 + 2 2 3 convergent 1 1 (d) 1 − k + k 2 3 divergent (b) 1 +

1 + … is 4

does not give absolute convergence of a series? (a) Root Test (b) Comparison Test (c) Ratio Test (d) Leibnitz Test 17. Which one of the following infinite series is convergent! ∞ ∞ 1 1 (b) ∑ 1 (a) ∑ 2 n =1 n − n n =1 n n +n ∞ 1 (c) ∑ n n =1 n − ∞

(d)

1 + … is 4k

15. If un is a series of positive terms, then (a) convergence of (–1)nun implies the convergence of un. (b) convergence of un implies the convergence of (–1)nun. (c) convergence of (–1)nun implies the divergence of un. (d) divergence of un implies the divergence of (–1)nun.

3

n =1

n2 − n 2 + 1)

18. The series 2 2 33 x 3 44 x 4 x+ 2 x + + +… 3! 4! 2! is convergent if

(a) a < x


2 3 1 and divergent if x2 1

20. Which one of the following statement hold? ∞

(a) The series

∑x n=0

16. Which one of the following test

1 e

each x

[–1, 1]

n

converges for

5.44



(b) The series

∑x

n

converges

n=0

uniformly in (–1, 1) ∞

(c) The series

n

∑ xn

converges for

(a) p < q – 1 (c) p q – 1

n =1

each x

[–1, 1 [ ∞

(d) The series

21. If p and q are positive real nump 3p bers, then the series 2q + q + 2 1 4 p + … is convergent for 3q

∑ nx

(b) p < q + 1 (d) p q + 1

n 2

converges

n =1

uniformly in (–1, 1) Answers 1. (b) 8. (a) 15. (b)

2. (a) 9. (a) 16. (d)

3. (a) 10. (b) 17. (a)

4. (d) 11. (a) 18. (a)

5. (c) 12. (b) 19. (c)

6. (c) 13. (b) 20. (c)

7. (d) 14. (c) 21. (a)

Integral Calculus Chapter

6

6.1 INTRODUCTION Integral calculus helps in developing techniques for determination of the integral of a given function. In this chapter, we will study applications of integral calculus, such as finding lengths of arcs of curves, areas of planes, and volumes and surface areas of solids of revolution. The concept of reduction formula also helps in solving the integral of a given function. Integral calculus deals with the derivation of formulas for finding anti-derivatives. It is also useful in solving differential equations.

6.2 REDUCTION FORMULAE Reduction formulae reduce an integral to a simple integral by repeatedly using integration by parts.

6.2.1 Reduction Formula for ∫ sin n x dx ; n > 0

∫ sin

n

x dx = ∫ sin n −1 x sin x dx

Integrating by parts,

∫ sin

n

x dx = sin n −1x( − cos x ) − ∫ ( n − 1) sin n − 2 x cos x ( − cos x ) dx = − cos x sin n −1 x + ( n − 1) ∫ sin n − 2 x (1 − sin 2 x ) d x = − cos x sin n −1 x + ( n − 1) ∫ sin n − 2 x dxx − ( n − 1) ∫ sin n x d x

n ∫ sin n x dx = − cos x sin n −1 x + ( n − 1) ∫ sin n − 2 x dx 1 n −1 x dx = − cos x sin n −1 x + sin n − 2 x dx n n ∫ Note: If n is odd positive integer, the function can be easily integrated using substitution cos x = t.

∫ sin

n

6.2

Engineering Mathematics

6.2.2 Reduction Formula for

∫ cos

n

∫ cos x dx ; n > 0 n

x dx = ∫ cos n −1 x cos x dx

Integrating by parts,

∫ cos x dx = cos n

n −1

x sin x − ∫ ( n − 1) cos n − 2 x ( − sin x ) sin x dx

= sin x cos n −1 x + ( n − 1) ∫ cos n − 2 x (1 − cos 2 x ) dx = sin x cos n −1 x + ( n − 1) ∫ cos n − 2 x dx − ( n − 1) ∫ cos n x dx

n ∫ cos n x dx = sin x cos n −1 x + (n − 1)∫ cos n − 2 x dx n −1 cos n − 2 x dx n ∫ Note: If n is an odd positive integer, the function can be easily integrated using substitution sin x = t. 1

∫ cos x dx = n sin x cos n

n −1

x+

6.2.3 Reduction Formula for

∫ tan

n

tan n x dx

x dx = ∫ tan n − 2 x tan 2 x dx = ∫ tan n − 2 x (sec 2 x − 1) dx = ∫ tan n − 2 x sec 2 x dx − ∫ tan n − 2 x dx =

tan n −1 x − tan n − 2 x dx n −1 ∫

6.2.4 Reduction Formula for

∫ cot

n

⎡ ⎢∵ ⎣

f ′( x ) d x =

[ f ( x )]n +1 ⎤ ⎥ n +1 ⎦

n ∫ [ f ( x)] f ′( x) dx =

[ f ( x )]n +1 ⎤ ⎥ n +1 ⎦

∫ [ f ( x)]

n

cot n x dx

x dx = ∫ cot n − 2 x cot 2 x dx = ∫ cot n − 2 x (cosec 2 x − 1) dx = ∫ cot n − 2 x cos ec 2 x dx − ∫ cot n − 2 x dx =−

cot n −1 x − cot n − 2 x dx n −1 ∫ ⎡ ⎢∵ ⎣

6.2.5 Reduction Formula for

∫ sec x dx = ∫ sec n

n− 2

x sec 2 x dx

sec n x dx

Integral Calculus

6.3

Integrating by parts,

∫ sec x dx = sec n

n− 2

x tan x − ∫ ( n − 2) sec n − 2 x tan x tan x dx

= tan x sec n − 2 x − ( n − 2) ∫ sec n − 2 x (sec 2 x − 1) dx

(

)

= tan x sec n − 2 x − ( n − 2) sec n x − ∫ sec n − 2 x dxx [1 + ( n − 2)]∫ sec x dx = tan x sec n

n− 2

x + ( n − 2) ∫ sec

n− 2

x dx

n− 2

tan x sec x n − 2 + sec n − 2 x dx n −1 n −1 ∫ Note: If n is an even positive integer, the function can be easily integrated using substitution of tan x = t.

∫ sec x dx = n

6.2.6 Reduction Formula for

∫ cosec x dx = ∫ cosec n

n− 2

cosec n x dx

x cosec 2 x dx

Integrating by parts,

∫ cos ec x dx = cosec n

n− 2

x ( − cot x ) − ∫ ( n − 2) cosec n − 3 x ( − cosec x cot x )( − cot x ) dx

= − cot x cosec n − 2 x − ( n − 2) ∫ cosec n − 2 x (cosec 2 x − 1) dx = − cot x cosec n − 2 x − ( n − 2)

(∫ cosec x − ∫ cosec n

n− 2

x dx

)

[1 + ( n − 2)]∫ cosec n x dx = − cot x cosec n − 2 x + ( n − 2) ∫ cosec n − 2 x dx − cot x cosec n − 2 x n − 2 + cosec n − 2 x dx n −1 n −1 ∫ Note: If n is an even positive integer, the function can be easily integrated using substitution of cot x = t. n ∫ cosec x dx =

6.2.7 Reduction Formula for ∫ sin m x cos n x dx ; m, n > 0

∫ sin

m

x cos n x dx = ∫ sin m −1 x (sin x cos n x ) dx

Integrating by parts, m n ∫ sin x cos x dx = −

cos n +1 x m −1 cos n +1 x sin x + ∫ ( m − 1) sin m − 2 x cos x dx n +1 n +1 ⎡ [ f ( x )]n +1 ⎤ n ⎢∵ ∫ [ f ( x )] f ′( x ) d x = ⎥ n +1 ⎦ ⎣

=−

cos n +1 x sin m −1 x m − 1 + sin m − 2 x cos n x (1 − sin 2 x ) dx n +1 n +1 ∫

=−

cos n +1 x sin m −1 x m − 1 m −1 sin m − 2 x cos n x dx − sin m x cos n x dx + ∫ n +1 n +1 n +1 ∫

6.4

Engineering Mathematics

cos n +1 x sin m −1 x m − 1 ⎛ m −1⎞ m n x x x 1 + sin cos d = − + sin m − 2 x cos n x dx ⎟ ⎜⎝ n +1 ⎠ ∫ n +1 n +1 ∫ m n ∫ sin x cos x dx = −

cos n +1 x sin m −1 x m − 1 + sin m − 2 x cos n x dx m+n m+n∫

Similarly, it can be proved that m n ∫ sin x cos x dx =

(i)

sin m +1 x cos n −1 x n − 1 + sin m x cos n − 2 x dx m+n m+n∫

cos n +1 x sin m −1 x m − 1 + sin m − 2 x cos n + 2 x dx n +1 n +1 ∫ This formula is useful when m is positive and n is a negative integer.

∫ sin

(ii)

m

x cos n x dx = −

sin m +1 x cos n −1 x n − 1 + sin m + 2 x cos n − 2 x dx m +1 m +1 ∫ This formula is useful when m is negative and n is a positive integer. m n ∫ sin x cos x dx =

(iii)

cos n +1 x sin m +1 x m + n + 2 + sin m + 2 x cos n x dx m +1 m +1 ∫ This formula is useful when n is a negative integer. m n ∫ sin x cos x dx = −

(iv)

sin m +1 x cos n +1 x m + n + 2 + sin m x cos n + 2 x dx ∫ m +1 m +1 This formula is useful when n is a negative integer. (v)

m n ∫ sin x cos x dx =

Example 1: Evaluate

sin 5 x dx.

Solution: Using reduction formula, 1 4 5 4 3 ∫ sin x dx = − 5 cos x sin x + 5 ∫ sin x dx 1 4⎛ 1 2 ⎞ = − cos x sin 4 x + ⎜ − cos x sin 2 x + ∫ sin x dx⎟ ⎝ ⎠ 5 5 3 3 1 4 8 = − cos x sin 4 x − cos x sin 2 x − cos x 5 15 15 Example 2: Evaluate

sin6 x dx.

Solution: Using reduction formula,

∫ sin

6

1 5 x dx = − cos x sin 5 x + ∫ sin 4 x dx 6 6 3 1 5⎛ 1 ⎞ 5 = − cos x sin x + ⎜ − cos x sin 3 x + ∫ sin 2 x dx ⎟ ⎝ ⎠ 4 6 6 4

Integral Calculus

6.5

1 5 5⎛ 1 1 ⎞ = − cos x sin 5 x − cos x sin 3 x + ⎜ − cos x sin x + ∫ sin 0 x dx ⎟ ⎝ ⎠ 6 24 8 2 2 1 5 5 5 = − cos x sin 5 x − cos x sin 3 x − cos x sin x + x 6 24 16 16

cos 3 x dx.

Example 3: Evaluate

Solution: Using reduction formula,

1

2

∫ cos x dx = 3 sin x cos x + 3 ∫ cos x dx 3

2

1 2 = sin x cos 2 x + sin x 3 3

tan6 x dx.

Example 4: Evaluate

Solution: Using reduction formula,

tan 5 x − ∫ tan 4 x dx 5 ⎞ tan 5 x ⎛ tan 3 x = −⎜ − ∫ tan 2 x dx ⎟ ⎝ 3 ⎠ 5 5 3 tan x tan x ⎛ tan x ⎞ − +⎜ − ∫ tan 0 x dx ⎟ = ⎝ ⎠ 5 3 1 tan 5 x tan 3 x = − + tan x − x 5 3

6 ∫ tan x dx =

Example 5: Evaluate

p 2 p 4

cot 4 x dx.

Solution: Using reduction formula,

cot 3 x − ∫ cot 2 x dx 3 cot 3 x cot 3 x ⎛ cot x ⎞ + cot x + x =− − ⎜− − ∫ cot 0 x dx ⎟ = − ⎠ ⎝ 3 3 1

4 ∫ cot x dx = −



2 4

cot 4 x dx = −

cot 3 x + cot x + x 3

2

4

1 3 −8 = + −1− = 2 3 4 12

6.6

Engineering Mathematics

Example 6: Evaluate

sec 4 x dx .

Solution: Using reduction formula,

tan x sec 2 x 2 + ∫ sec 2 x dx 3 3 tan x sec 2 x 2 = + ( tan x + 0 ) 3 3 2 1 = tan x sec 2 x + tan x 3 3

4 ∫ sec x dx =

Example 7: Evaluate

o 2 o 6

cosec 5 x dx .

Solution: Using reduction formula, cot x cosec3 x 3 + ∫ cosec3 x dx 4 4 cot x cosec3 x 3 ⎛ cot x cosec x 1 ⎞ + ∫ cosec x dx ⎟ =− + ⎜− ⎠ 2 2 4 4⎝ 1 3 3 = − cot x cosec3 x − cot x cosec x + log (cosec x − cot x ) 4 8 8

5 ∫ cosec x dx = −



2 6

2 1 3 3 cosec5x dx = − cot x cosec3 x − cot x cosec x + log(cosec x − cot x ) 4 8 8 6

3 1 3 3 = log 1 + ( 3 ) (8) + ( 3 ) ( 2) − log ( 2 − 3 ) 8 4 8 8 =

11 3 3 − log (2 − 3 ) 4 8

Example 8: Evaluate

sin 2 x cos 4 x dx .

Solution: Using reduction formula,

sin 3 x cos3 x 3 + ∫ sin 2 x cos 2 x dx 6 6 ⎞ 1 1 ⎛ sin 3 x cos x 1 + ∫ sin 2 x cos 0 x dx ⎟ = sin 3 x cos3 x + ⎜ ⎝ ⎠ 6 2 4 4 1 1 1 ⎛ cos x sin x 1 ⎞ = sin 3 x cos3 x + sin 3 x cos x + ⎜ − + ∫ sin 0 x dx ⎟ ⎝ ⎠ 6 8 8 2 2 1 1 1 = sin 3 x cos3 x + sin 3 x cos x + ( x − sin x cos x ) 6 8 16

2 4 ∫ sin x cos x dx =

Integral Calculus

6.7

sin 4 x cos 4 x dx .

Example 9:

Solution: Using reduction formula,

∫ sin

4

x cos 4 x dx

cos5 x sin 3 x 3 + ∫ sin 2 x cos 4 x dx 8 8 ⎞ 1 3 ⎛ − cos5 x sin x 1 + ∫ sin 0 x cos 4 x dx ⎟ = − cos5 x sin 3 x + ⎜ ⎠ 8 8⎝ 6 6 =−

⎞ 1 ⎛ cos3 x sin x 3 1 1 + ∫ cos 2 x dx ⎟ = − cos5 x sin 3 x − cos5 x sin x + ⎜ ⎝ ⎠ 16 4 4 8 16 1 1 1 3 ⎛ sin x cos x 1 ⎞ = − cos5 x sin 3 x − cos5 x sin x + cos3 x sin x + + ∫ cos 0 x dx ⎟ ⎜ ⎠ 8 16 64 64 ⎝ 2 2 1 1 1 3 = − cos5 x sin 3 x − cos5 x sin x + cos3 x sin x + ( x + sin x cos x ) 8 16 64 128 Exercise 6.1 1. Evaluate

sin 4 x cos 2 x d x

(i)

sin 4 x dx

(ii)

tan 3 x dx

(ii)

(iii)

cot 5 x dx

(iv)

sec5 x dx

1 ⎡ ⎤ 3 ⎢ Ans. : (i) − 4 sin x cos x ⎥ ⎢ ⎥ 1 ⎢ + ( x + sin x cos x ) ⎥ ⎢ ⎥ 8 ⎢ ⎥ 1 1 ⎢ (ii) sin 5 x − sin 7 x ⎥ ⎢⎣ ⎥⎦ 5 7

(v)

4

cosec x dx

⎡ 3 ⎤ cos x sin 3 x 3 − cos x sin x + x ⎥ ⎢ Ans. : (i) − 4 8 8 ⎢ ⎥ 1 2 ⎢ ⎥ (ii) tan x − log sec x ⎢ ⎥ 2 ⎥ ⎢ 1 1 ⎢ (iii) − cot 4 x + cot 2 x + log sin x ⎥ ⎥ ⎢ 4 2 ⎥ ⎢ 1 3 ⎥ ⎢ (iv ) tan x sec3 x + tan x sec x 4 8 ⎥ ⎢ ⎥ ⎢ 3 + log (sec x + tan x ) ⎥ ⎢ 8 ⎥ ⎢ ⎥ ⎢ 1 2 2 ( ) − cot x cos x − cot x v ec ⎥ ⎢ ⎣ ⎦ 2 3

2. Evaluate (i)

sin 2 x cos 2 x dx

3. Show that (i)

(ii)

sin 5 x 1 ∫ cos4 x dx = 3 cos3 x 2 − − cos x cos x cos5 x sin 4 x = d x ∫ sin x 4 − sin 2 x + logsin x

6.8

Engineering Mathematics

4. Evaluate (i)

sec x tan 5 x dx

(ii)

sin 3 x sec7 x dx

(iii)

cos3 x cosec 4 x dx

2 0

1 5 2 3 ⎡ ⎤ ⎢ Ans.: (i) 5 sec x − 3 sec x + sec x ⎥ ⎢ ⎥ 1 1 6 4 ⎢ ⎥ (ii) tan x + tan x ⎢ ⎥ 4 6 ⎢ ⎥ 1 ⎢ (iii) − cosec3 x + cosec x ⎥ 3 ⎣⎢ ⎦⎥

sin n x dx and

2 0

cos n x dx , n > 0

Using reduction formula,

∫ sin

n

1 n −1 x dx = − cos x sin n −1 x + sin n − 2 x dx n n ∫ 2 n − 1 2 n− 2 1 I n = ∫ 2 sin n x dx = − cos x sin n−1 x + sin x dx 0 n n ∫0 0

n − 1 2 n− 2 sin x dx n ∫0 n −1 I = n n− 2 Using this recurrence relation, = 0+

In

2

In

4

n n n = n

=

3 In 2 5 In 4

4

6



………



………

2 I1 , if m is odd 3 1 I 2 = I 0 , if m is even 2 I3 =

Substituting these values,



2

0

n −1 n − 3 n − 5 2 ⋅ ⋅ … I1 , n n−2 n−4 3 n −1 n − 3 n − 5 1 = ⋅ ⋅ … I0 , n n−2 n−4 2

sin n x dx =

if n is odd if n is even

Integral Calculus

I1 = ∫ 2 sin x dx = − cos x 02 = 1

Now,

0

p 2

I0 Hence,

6.9



2

0

0

sin n x dx = =

0

p 2

1 dx

2 n −1 n − 3 n − 5 ⋅ ⋅ … , 3 n n−2 n−4

if n is odd

1 n −1 n − 3 n − 5 ⋅ … ⋅ , ⋅ 2 2 n n−2 n−4

if n is even

p 2

x

Putting

p 2

sin 0 x dx

dx

y, dy

⎛ ⎞ I n = ∫ 2 sin n x dx = ∫ 2 sin n ⎜ − y ⎟ ( −dy ) = ∫ 2 cos n y dy 0 0 0 ⎝2 ⎠ = ∫ 2 cos n x dx 0

2 n −1 n − 3 n − 5 n ∫ 02 cos x dx = n ⋅ n − 2 ⋅ n − 4 … 3 , =

1 n −1 n − 3 n − 5 ⋅ … ⋅ , ⋅ 2 2 n n−2 n−4

if n is odd if n is even

Corollary: Certain definite integrals can be evaluated using 2 0

cos n x dx, where n is a positive integer. x = a sin q ,

(i) Putting a 0

xn 1 x2

dx =

p 2 0

= an

a n sin n q a cos q dq cos q p 2 0

sin n q dq

x = a tan q ,

(ii) Putting

0

1 dx = 2 (a + x 2 )n =

p 2 0

a sec 2 q dq a 2 n (sec 2 q ) n p 2

1 a2n

1

0

cos 2 n 2 q dq

2 0

sin n x dx and

6.10

Engineering Mathematics x = a tan q ,

(iii) Putting



0

π

1



(a2 + x 2 )

n+

1 2

dx = ∫ 2 0

sec 2 θ dθ sec 2 n +1 θ

π

= ∫ 2 cos 2 n −1 θ dθ 0



π 2

0

sin m x cos n x dx ; m, n > 0

Using reduction formula,

∫ sin

m

x cos n x dx = −

cos n +1 x sin m +1 x m − 1 + sin m − 2 x cos n x dx ∫ m+n m+n

I m , n = ∫ 2 sin m x cos n x dx = − 0

cos n +1 x sin m +1 x m+n

2

0

m − 1 2 m−2 + sin x cos n x dx m + n ∫0 m − 1 2 m−2 sin x cos n x dx m + n ∫0 m −1 I m − 2, n = m+n Using this recurrence relation, = 0+

m 3 Im m n 2 m 5 I m 4, n = Im m n 4 … ……… Im

2, n

=



4, n

6, n

………

I 3, n =

2 I , 3 + n 1, n

if m is odd

I 2, n =

1 I , 2 + n 0, n

if m is even

Substituting these values,



2

0

2 m −1 m − 3 ⋅ … ⋅ I1, n , 3+ n m+n m+n−2 1 m −1 m − 3 ⋅ … ⋅ I0, n , = m+n m+n−2 2+n

sin m x cos n x dx =

if m is odd if m is even

Integral Calculus

I1, n = ∫ 2 sin x ⋅ cos n x dx = −

Now,

0

p 2

cos n +1 x n +1

p 2

6.11

2

=

0

1 n +1

I 0 , n = ∫ sin x cos x dx = ∫ cos x dx 0

n

0

n

0

2 1 m −1 m − 3 m n ∫02 sin x cos x dx = m + n ⋅ m + n − 2 … 3 + n ⋅ n + 1 , if m is odd and n may be odd or even m −1 m − 3 m−5 1 n −1 n − 3 2 … … , = ⋅ ⋅ × ⋅ m+n n+n−2 m+n−4 2+n n n−2 3 if m is even and n is odd m −1 m − 3 m−5 1 n −1 n − 3 1 p = ⋅ ⋅ × ⋅ … … ⋅ , m+n m+n−2 m+n+4 2+n n n−2 2 2 if m is even and n is even

Hence,

2

Corollary: Certain definite integrals can be evaluated using

0

sin m x cos n x dx,

where m, n are positive integers. x = a tan q , (i) Putting n x a n p2 tan n q d x a sec 2 q dq 0 (a2 x 2 )m a 2 m 0 sec 2 m q = a2n

p 2

2m 1

0

sin n q cos 2 m

n 2

q dq

x = a tan q ,

(ii) Putting



x



0

π

n 1 m+ 2 2

(a + x ) 2

dx = ∫ 2 0

a n sin n θ cos n θ

1 a

2 m +1

2m + 1 (sec θ ) 2

⋅ a sec 2 θ dθ

2

π

= a n − 2 m ∫ 2 sin n θ cos 2 m − n −1 θ dθ 0

x = 2a sin 2 q ,

(iii) Putting



2a

0

2a

x m 2ax − x 2 dx = ∫ x

m+

1 2

0

π 2 0

= ∫ ( 2a)

2a − x dx

1 m+ 2

sin 2 m +1θ ⋅ 2a cos θ 4 a sin θ ⋅ cos θ dθ π

= ( 2a) m + 2 ⋅ 2 ∫ 2 sin 2 m + 2 θ ⋅ cos 2 θ dθ 0

Example 1: Evaluate: (i) (iv)

2 0

2 0

sin 5 x dx

(ii)

cos 7 x dx

(v)

2 0

2 0

sin6 x dx cos 4 x dx

(iii)

2 0

sin 8 x dx

6.12

Engineering Mathematics

Solution: (i)



0

(iii)



0

(v)



0

2

2

2

sin 5 x dx =

4 2 8 ⋅ = 5 3 15

(ii)



0

sin8 x dx =

7 5 3 1 35 ⋅ ⋅ ⋅ ⋅ = 8 6 4 2 2 256

(iv)



0

cos 4 x dx =

3 1 3 ⋅ ⋅ = 4 2 2 16

2

2

sin 6 x dx =

5 3 1 5 ⋅ ⋅ ⋅ = 6 4 2 2 32

cos 7 x dx =

6 4 2 16 ⋅ ⋅ = 7 5 3 35

Example 2: Evaluate: o 4

(i) (iv)

0 o



0

sin 4 2x dx sin 2 p

(ii)

1 − cos p dp 1 + cosp



(v)

⎛ x⎞ sin 5 ⎜ ⎟ dx ⎝2⎠

0 o 4

(iii)



o 2

0

sin 4 p dp (1 + cosp )2

cos6 2t dt

0

Solution: (i)

p 4 0

sin 4 2 x dx 2x = t , 2dx = dt x = 0, t = 0 p p x= , t= 4 2

Putting When

p 4 0

(ii)



0

1 2

sin 4 2 x dx

p 2 0

sin 4 t dt

1 3 1 p 2 4 2 2

⎛x⎞ sin 5 ⎜ ⎟ dx ⎝2⎠ 1 x = t, 2 1 dx = dt 2 x = 0,

Putting

When

x =p,



0

p

(iii)

0

3p 32

sin 4 q dq = (1 + cos q ) 2

p

sin 5

2sin

x 4 2 16 dx = 2 ∫ 2 sin 5 t dt = 2 ⋅ ⋅ = 0 2 5 3 15

q q cos 2 2

0

2 cos 2

t=0 p t= 2

q 2

2

4

dq =

p 0

4 sin 4

q dq 2

Integral Calculus

q = t, 2

Putting

1 dq = dt 2 q = 0,

When

t=0 p t= 2

q =p, p 0

p

(iv)

0

6.13

sin 4 q dq (1 + cos q ) 2

1 cos q sin 2 q dq 1 + cos q

p 0

8

q q 4 sin 2 cos 2 2 2

p 2

sin 4 t dt

0

q 2 dq 2q 2 cos 2 2sin 2

1 dq = dt 2 q = 0,

When

p 0

0

p

2 2

0

3p 2

sin 3

q dq 2

t=0 p t= 2

q =p,

p 4

3 1 p 4 2 2

q = t, 2

Putting

(v)

8

sin 2 q

1 cos q dq 1 + cos q

4 2

p 2 0

sin 3 t dt

4 2

2 3

8 2 3

cos6 2t dt 2t = x, 2dt = dx

Putting

t = 0, x=0 p p t= , x= 4 2 1 1 5 3 1 5 6 6 ∫04 cos 2t dt = 2 ∫02 cos x dx = 2 ⋅ 6 ⋅ 4 ⋅ 2 ⋅ 2 = 64

When

Example 3: Evaluate: 2

(i)

0

sin 5 x cos6 x dx

(ii)

2 0

sin 4 x cos 8 x dx

Solution: (i)



2

0

sin 5 x cos6 x dx =

4 ⋅ 2 ⋅ 5 ⋅ 3 ⋅1 8 = 11 ⋅ 9 ⋅ 7 ⋅ 5 ⋅ 3 ⋅1 693

(iii)

o 4 0

cos 3 2 x sin 4 4 x dx

6.14

(ii)

Engineering Mathematics



2

0 p 4

(iii)

0

sin 4 x cos8 x dx =

3 ⋅1 ⋅ 7 ⋅ 5 ⋅ 3 ⋅1 7 = 12 ⋅10 ⋅ 8 ⋅ 6 ⋅ 4 ⋅ 2 2 2048

cos3 2 x sin 4 4 x dx

p 4 0

cos3 2 x (2sin 2 x cos 2 x ) 4 dx p 4

= 16

0

cos 7 2 x sin 4 2 x dx 2x = t 2dx = dt x = 0, t = 0 p p x= , t= 4 2

Putting When

p 4 0

p 2

cos3 2 x sin 4 4 x dx = 8

0

cos 7 t sin 4 t dt

6 4 2 3 11 9 7 5 3

8

128 1155

Example 4: Evaluate: (i)

1 0

3

x 2 (1 x ) 2 dx 4

(iv)

0

x3 4x

(ii)

2 0

x4 4 x2

dx

(iii) 0

1 dx (1 + x 2 )5

x 2 dx

Solution: (i)

1 0

3

x 2 (1 x ) 2 dx x = sin 2 q ,

Putting

dx = 2sin q cos q dq x = 0,

When

q =0 p q = 2

x = 1, 1 0

p 2

3

x 2 (1 x ) 2 dx

0

=2 2 (ii)

2 0

Putting

x4 4 x2

sin 4 q cos3 q 2sin q cos q dq p 2

0

sin 5 q cos 4 q dq

4 2 3 9 7 5 3

dx x = 2sin q , dx = 2 cos q dq

16 315

Integral Calculus

When



x4

2

0

4 − x2

x = 0,

q =0

x = 2,

q =

π

p 2

( 2 sin θ ) 4

dx = ∫ 2

4 − 4 sin 2 θ

0

6.15

⋅ 2 cos θ dθ

π

= 16 ∫ 2 sin 4 θ dθ 0

3 1 π = 16 ⋅ ⋅ ⋅ = 3π 4 2 2

(iii)

0

1 dx (1 + x 2 )5 x = tan q ,

Putting

dx = sec 2 q dq When

0

x = 0,

q =0

x

q

1 dx = (1 + x 2 )5

, p 2 0

p 2

sec 2 q dq = (1 + tan 2 q )5

7 5 3 1 p 8 6 4 2 2 (iv)

4 0

p 2 0

sec 2 q dq = sec10 q

p 2 0

cos8 q dq

35p 256

x 3 4 x x 2 dx x = 4 sin 2 q ,

Putting

dx = 8sin q cos q dq x = 0,

When

q =0 π θ= 2

x = 4, 4 0

x 3 4 x x 2 dx

p 2 0

7

1

(4) 2 sin 7 q (4 4 sin 2 q ) 2 8sin q cos q dq

27 16 128

p 2 0

sin8 q cos 2 q dq

16 7 5 3 1 p 10 8 6 4 2 2

128 16 28p

1 10

p 2 0

sin8 q dq

6.16

Engineering Mathematics

Exercise 6.2 1. Evaluate (i)

(iv)

sin 4 x dx

2 0

2

(ii)

0

3p Ans. : (i) 16

(ii) (iii) (iv)

2a

(v)

8 (ii) 15

1 0

cos5 x sin 4 x dx

2 0

cos 4 x sin 3 x dx

2 0 p 6

cos6 3 x sin 2 6 x dx

0

4. Evaluate

8 8 (ii) 693 315 13 7p (iii) (iv) 35 384

(i)

0

1

(ii)

0

3. Evaluate

(ii) (iii)

1

x 2 (2 x 2 )

0

1 x2 a

0

x4 a2

1 0

x2

3

Ans.: (i)

Ans. : (i)

(i)

3

x 2 (1 x ) 2 dx 5p 3p 4 (ii) a 16 16 11p p (iii) (iv) 192 32 5p 4 3p (v) a (vi) 8 128

sin 6 x cos5 x dx

2 0

x 2 2ax x 2 dx

0

(vi)

2. Evaluate (i)

0

cos5 x dx

x2 dx (1 + x 2 ) 4

(iii)

dx

0

1 dx (1 + x 2 )5 x5

1+ x2 dx 1 x2

x3 dx (4 + x 2 ) 2 35p 256 1 (iii) 3

Ans.: (i)

dx

x 5 sin 1 x dx

(ii)

3p + 8 24

6.3 RECTIFICATION OF CURVES The process of determining the length of an arc of a plane curve is known as rectification of curves.

Length of Arc in Cartesian Form We

y = f (x)

y

know from differential calculus that for the curve y = f (x), ds ⎛ dy ⎞ = 1+ ⎜ ⎟ ⎝ dx ⎠ dx

2

The length of the arc of the curve y = f (x) between x = a and x = b is given by,

O x=a

x=b

Fig. 6.1

x

Integral Calculus

s=∫

6.17

2

b ds ⎛ dy ⎞ dx = ∫ 1 + ⎜ ⎟ dx a ⎝ dx ⎠ dx

b a

Similarly, the length of the arc of the curve x = f (y) between y = c and y = d is given by,

s=∫

d

c

2

d ds ⎛ dx ⎞ dy = ∫ 1 + ⎜ ⎟ d y c dy ⎝ dy ⎠

y y=d

x = f (y)

Length of Arc in Parametric Form When the equation of the curve is given in parametric form x = f1(t), y = f2(t), we have, from differential calculus, 2

ds ⎛ dx ⎞ ⎛ dy ⎞ = ⎜ ⎟ +⎜ ⎟ ⎝ dt ⎠ ⎝ dt ⎠ dt

y=c x

O

2

Fig. 6.2

The length of the arc of the curve between the points t = t1 and t = t2 is given by,

s=∫

2

2

⎛ dx ⎞ ⎛ dy ⎞ ⎜⎝ ⎟⎠ + ⎜⎝ ⎟⎠ dt dt dt

t2

t1

Length of Arc in Polar Form

For the curve r = f (q ), we have, from dif-

ferential calculus, ds ⎛ dr ⎞ = r2 + ⎜ ⎟ ⎝d ⎠ d

q=p 2

2

P (r, q ) r

The length of the arc of the curve r = f (q ) between the points q = q 1 and q = q 2 is given by, 2

s=∫

2 1

⎛ dr ⎞ r2 + ⎜ ⎟ d ⎝d ⎠

q q1 q2 q =0

Fig. 6.3

Similarly, the length of the arc of the curve q = f (r) between the points r = r1 and r = r2 is given by,

s=∫

r2 r1

2

⎛d ⎞ 1 + r ⎜ ⎟ dr ⎝ dr ⎠ 2

Cartesian Form Example 1: Show that the length of the arc of the curve 4ax = y 2 y2 a from (0, a) to any point (x, y) is given by x. 2a 2

2a 2 log

y a

a2

Engineering Mathematics

6.18

Solution:

y − a2 a a 1 dx 4a = 2 y − 2a 2 ⋅ ⋅ y a dy 4ax = y 2 − 2a 2 log

… (1)

y a y 2 − a2 dx = − = 2ay dy 2 a 2 y For the required arc, y varies from a to y.

s=∫

Length of the arc,

2

⎛ dx ⎞ 1 + ⎜ ⎟ dy ⎝ dy ⎠

y a

2

=∫ =

2

y ⎛ y 2 − a2 ⎞ ( y 2 + a2 ) 1+ ⎜ dy = ∫ dy ⎟ a ⎝ 2ay ⎠ ( 2ay ) 2

y a

1 ⎛ y 1 y y 2 + a2 y dy + a 2 dy = ∫ y 2a ⎜⎝ ∫a 2a a



y a

1 ⎞ dy y ⎟⎠

y

y a2 ⎞ 1 y2 1 ⎛ y2 2 = + a 2 log y = ⎜⎝ + a log − ⎟⎠ a 2 2a 2 2a 2 a =

a2 ⎞ a2 ⎤ 1 ⎡ y2 ⎛ y2 ⎢ + ⎜⎝ − 2ax − ⎟⎠ − ⎥ 2a ⎣ 2 2 2 2⎦

… [From Eq. (1)]

y2 a 1 2 ( y − 2ax − a 2 ) = −x− 2 2a 2a y2 a = − −x 2a 2

=

Example 2: Find the length of the arc of the curve y = ex from the point (0, 1) to (1, e). y = ex dy = ex dx For the required arc, x varies from 0 to 1.

Solution:

Length of the arc AB,

s=∫

1

=∫

1

0

0

Putting

2

⎛ dy ⎞ 1 + ⎜ ⎟ dx ⎝ dx ⎠

y y = ex

B(1, e) A

(0, 1) x

O

1 + e dx

1 + e2x = t2, 2e2xdx = 2t dt t dx = 2 dt t 1

2x

Fig. 6.4

Integral Calculus

When

Length of the arc,

x = 0,

t= 2

x = 1,

t = 1 + e2

s=∫

1+ e 2 2

t⋅

1+ e 2

=∫

2

t dt = ∫ t −1 2

1+ e 2 2

6.19

t 2 −1+1 dt t 2 −1

1 ⎞ 1 t −1 ⎛ ⎜⎝1 + 2 ⎟⎠ dt = t + log 2 t +1 t −1

1+ e 2

2

1⎛ 1 + e2 − 1 2 −1⎞ = 1 + e 2 − 2 + ⎜ log − log ⎟ 2⎝ 2 +1⎠ 1 + e2 + 1 2 ⎡ ⎧ ⎫ ⎧⎪ ( 2 − 1)2 ⎫⎪⎤ 1 ⎢ ⎪ ( 1 + e 2 − 1) ⎪ − log = 1 + e − 2 + log ⎨ ⎬ ⎬⎥ ⎨ 2 − 1 ⎪⎥ 2 ⎢ ⎪ 1 + e2 − 1 ⎪ ⎪ ⎭⎦ ⎩ ⎭ ⎣ ⎩ 1 = 1 + e 2 − 2 + log ( 1 + e 2 − 1) − log e 2 − log ( 2 − 1) 2 2

= 1 + e 2 − 2 + log ( 1 + e 2 − 1) − 1 − log ( 2 − 1)

[∵ log e2 = 2 log e = 2] Example 3: Find the length of the arc of the curve y = log sec x from x = 0 to o x= . 3 Solution: y = log sec x dy 1 = ⋅ sec x tan x = tan x dx sec x For the required arc, x varies from 0 to

p . 3 2

Length of the arc,

s=∫

3

0

⎛ dy ⎞ 1 + ⎜ ⎟ dx ⎝ dx ⎠

p

p

= ∫ 3 1 + tan 2 x dx = ∫ 3 sec x dx 0

= log(sec x + tan x ) = log ( 2 + 3 )

0

p 3 0

⎛ ex − 1⎞ from x = 1 to ⎝ e x + 1 ⎟⎠

Example 4: Find the length of the arc of the curve y = log ⎜ x = 2.

Engineering Mathematics

6.20

Solution:

y = log

ex 1 ex + 1

log (e x

y dy dx

e e

1) log (e x

x

x

e 1 e

x

1)

2e

x

1

e

x

2x

1

For the required arc, x varies from 1 to 2. Length of the arc,

s=∫

2

=∫

2

2

⎛ dy ⎞ 1 + ⎜ ⎟ dx ⎝ dx ⎠

1

1+

1

2x 2 (e 4e 2 x − 1) 2 + 4e 2 x dx = ∫ dx 2 1 (e − 1) (e 2 x − 1) 2 2x

x −x 2x 2⎛e 2⎛e +e ⎞ +1⎞ = ∫ ⎜ 2x dx = ∫ ⎜ x dx ⎟ 1 ⎝e 1 ⎝ e − e− x ⎟ −1⎠ ⎠ ⎡ 2 = log (e x − e − x ) ⎢∵ 1 ⎣ = log (e 2 − e −2 ) − log(e − e −1 )



⎤ f ′( x ) dx = log f ( x ) ⎥ f ( x) ⎦

e 2 − e −2 = log (e + e −1 ) e − e −1 ⎛ 1⎞ = log ⎜ e + ⎟ ⎝ e⎠ = log

Example 5: Show that the length of the arc of the curve ay2 = x3 from the origin 8a 7

to the point whose abscissa is b is Solution:

9b 4a

1

3 2

1 .

ay 2 = x 3 2ay

y

dy = 3x 2 dx

P

dy 3 x 2 = = dx 2ay

=

3x 2 2a

x3 a

O

x=b

3 x 2 a

For the arc OP, x varies from 0 to b. Length of the arc OP,

s=

b 0

1+

dy dx

2

dx

Fig. 6.5

x

Integral Calculus

b

=

9x dx 4a

1+

0

6.21

2 9x = 1+ 3 4a

3 2

b

4a 9 0

8a 27

9b 4a

1

3 2

1

x , prove that the length of the arc s, c measured from its vertex to any point (x, y), is Example 6: For the catenary y = c cosh

(i) s = c sinh

x c

(ii) s 2

y2

c2

y

x c

y = c cosh

Solution: (i)

(iii) s = c tanx

dy x = sinh dx c For the arc AP, x varies from 0 to x. Length of the arc AP,

x

s=∫

0

=∫

x

0

A (0, c)

2

⎛ dy ⎞ 1 + ⎜ ⎟ dx ⎝ dx ⎠ 1 + sinh 2

x

= ∫ cosh 0

= c sinh (ii)

P(x, y)

y x

x dx c

x x dx = c sinh c c

x

0

Fig. 6.6

x c

s 2 = c 2 sinh 2

x x ⎞ x ⎛ = c 2 ⎜ cosh 2 − 1⎟ = c 2 cosh 2 − c 2 ⎝ c c ⎠ c

= y 2 − c2 (iii) The tangent at point P(x, y) makes an angle y with the x-axis. dy x tan = = sinh dx c s = c s = c tan

[From (i)]

Engineering Mathematics

6.22

2

1 Example 7: Prove that the length of the arc of the curve y x 1 x from 3 4 the origin to the point P(x, y) is given by s 2 = y 2 + x 2 . Hence, rectify the loop. 3 2

Solution:

y2

1 x 3

x 1

y

2

P(x, y)

1 1 3 ⎛ x⎞ y = x ⎜1 − ⎟ = x 2 − x 2 ⎝ 3⎠ 3

dy 1 = x dx 2



1 2

A (3, 0) x

O

1 3 (1 − x ) − ⋅ x = 3 2 2 x 1 2

For the arc OP, x varies from 0 to x. Length of the arc OP,

2

⎛ dy ⎞ 1 + ⎜ ⎟ dx ⎝ dx ⎠

s=∫

x

=∫

x

=∫

x

1+ x

0

2 x

0

0

1+

Fig. 6.7

x (1 − x ) 2 (1 + x ) 2 dx = ∫ dx 0 4x 4x

dx =

1 1 x − 12 2 ( x x ) dxx + 2 ∫0

x 1 2

3 2

1 x x + 3 2 1 2 2 0 ⎛ x⎞ = x ⎜1 + ⎟ ⎝ 3⎠ =

2

2

⎛ x⎞ ⎛ x⎞ 4 s 2 = x ⎜1 + ⎟ = x ⎜1 − ⎟ + x 2 ⎝ 3⎠ ⎝ 3⎠ 3 4 = y2 + x2 3 2 1 The points of intersection of the curve y 2 x 1 x and x-axis are obtained as, 3 0

x 1

1 x 3

2

x = 0, 3, 3 and y = 0, 0, 0 Hence, A: (3, 0) Length of the upper half of the loop = 3 1 + Length of the complete loop = 4 3

3 =2 3 3

Integral Calculus

6.23

Example 8: Show that the length of the loop of the curve 9ay 2 is 4 3a. Solution: The points of intersection of the curve 9ay 2 x-axis are obtained as,

( x 2a )( x 5a )2

( x 2a)( x 5a) 2 and

y

0 = ( x − 2a)( x − 5a) 2 x = 2a, 5a and y = 0, 0 Hence, A: (2a, 0) and B: (5a, 0) 9ay 2

( x 2a)( x 5a) 2

B (5a, 0)

A (2a, 0) O

dy 18ay = ( x − 2a) 2( x − 5a) + ( x − 5a) 2 dx = ( x − 5a)(3x − 9a) dy ( x − 5a)( x − 3a) = 6 ay dx

x

Fig. 6.8

For the upper half of the loop, x varies from 2a to 5a. Length of the loop of the curve, s = 2 (Length of upper half of the loop) = 2∫ = 2∫ = 2∫ =

2

5a ( x − 5a) 2 ( x − 3a) 2 ⎛ dy ⎞ dx 1 + ⎜ ⎟ dx = 2 ∫ 1 + 2a ⎝ dx ⎠ 36 a 2 y 2

5a 2a

5a

1+

2a

5a ( x − a) 2 x−a dx = 2 ∫ dx a 2 4 a( x − 2 a ) 2 a ⋅ x − 2a

5a 2a

1 a



5a (xx − 5a) 2 ( x − 3a) 2 ( x − 3a) 2 d x = 2 1 + ∫2a 4a( x − 2a) dx 4 a( x − 2a)( x − 5a) 2

5a 2a

( x − 2a) + a x − 2a

dx =

1 − ⎤ ⎡ 2 2 ( 2 ) x a a x a − + − ⎥ dx ∫ ⎢ a 2a ⎣ ⎦

1

3 1 1 2 ( x − 2a) 2 + 2a( x − 2a) 2 = a 3

5a

5a

= 2a

3 1 ⎤ 1 ⎡2 2 2 ⎢ (3a) + 2a(3a) ⎥ a ⎣3 ⎦

= 2 3 a + 2a 3 = 4 3a

Example 9: In the evolute 27 ay 2

4( x 2a ) 3 of the parabola y 2 = 4ax , show

that the length of the arc from one cusp to the point where it meets the parabola

(

)

is 2a 3 3 1 . Solution: (i) The points of intersection of the parabola y 2 = 4 ax with its evolute 27ay 2 4( x 2a)3 are obtained as,

Engineering Mathematics

6.24

27a ⋅ 4 ax = 4( x − 2a)3

y (8a, ≥32a)

x − 6 ax − 15a x − 8a = 0 3

2

2

3

B

( x + a ) 2 ( x − 8a ) = 0 x = − a, 8 a But x

A

a does not lie on the parabola.

O

x = 8a and y = ± 32a

(

Hence, B: 8a, 32a

)

C

(ii) The points of intersection of the evolute 27ay 2 4( x 2a)3 with the x-axis are obtained as, x = 2a and y = 0 Hence A: (2a, 0) Now, 27ay 2 4( x 2a)3 3 2 y ( x 2a) 2 3 3a 1 dy 2 3 = ⋅ ( x − 2a) 2 = dx 3 3a 2 For the arc AB, x varies from 2a to 8a. Length of the arc AB,

x

(2a, 0)

s=∫

8a 2a

=∫

8a 2a

Fig. 6.9

x − 2a 3a

2

⎛ dy ⎞ 1 + ⎜ ⎟ dx ⎝ dx ⎠ 1+

x − 2a 1 dx = 3a 3a

3 2 = ⋅ ( x + a) 2 3a 3

1

=

(8a, −≥32a)

2 3 3a

3 2

8a

= 2a



8a 2a

x + a dx

3 3 ⎤ 2⎡ ⋅ ⎢(9a) 2 − (3a) 2 ⎥ 3a 3 ⎣ ⎦

1

3 2

(3a) (3 − 1) = 2a (3 3 − 1)

Example 10: Find the length of the parabola x 2 = 4 y which lies inside the circle y

x 2 + y 2 = 6 y.

Solution: The equation of the circle is x2 + y2 = 6y x2 + y2 – 6y = 0 The centre of the circle is (0, 3) and radius is 3. The points of intersection of parabola x2 = 4y and circle x2 + y2 = 6y are obtained as,

A (−2≥2, 2)

B (2≥2, 2)

O

Fig. 6.10

x

Integral Calculus

6.25

4 y + y2 = 6 y y2 − 2 y = 0 y( y − 2) = 0 y = 0, 2 When

(

)

y = 0, y = 2,

(

Hence A : −2 2 , 2 and B : 2 2 , 2 Now,

)

x=0 x = ±2 2

x = 4y dy x = dx 2 2

For the arc, OB, x varies from 0 to 2 2. s = 2( Length of the arc OB) Length of the arc OB, = 2∫

2 2

0

=∫

2 2

0

2

2 ⎛ dy ⎞ 1 + ⎜ ⎟ dx = 2 ∫ 0 ⎝ dx ⎠

x 2 + 4 dx =

(

2

1+

x2 dx 4

(

x 2 x + 4 + 2 logg x + x 2 + 4 2

)

2 2

0

)

= 2 ⋅ 12 + 2 log 2 2 + 12 − 2 log 2 = 2 ⎡ 6 + log ⎣

(

)

2+ 3 ⎤ ⎦

Example 11: Show that the length of the parabola y2 = 4ax from the vertex to the end of the latus rectum is a ⎡ 2 + log (1 + 2 ) ⎤ . Find the length of the arc cut ⎣ ⎦ off by the line 3y = 8x. Solution: (i) The points of intersection of the parabola y 2 = 4 ax and its latus rectum x = a are obtained as, y2

4a a

3y = 8x

y

P (a, 2a) A

4a 2

x=a

y = ± 2a and x = a

Hence, P: (a, 2a) and

Q : ( a,

2a)

O

(a, 0)

x

2

Now,

y 4a dx y = dy 2a x=

Q(a, −2a) y 2 = 4ax

For the arc OP, y varies from 0 to 2a. Fig. 6.11

Engineering Mathematics

6.26

2

Length of the arc OP,

s=∫

2a

=∫

2a

0

0

⎛ dx ⎞ 1 + ⎜ ⎟ dy ⎝ dy ⎠ 1+

1 y = 2a 2

y2 1 2a 2 y + 4 a 2 dy dy = 2 ∫ 0 a 2 4a

(

4a 2 y + 4a + log y + y 2 + 4 a 2 2 2

2

)

2a

0

1 ⎡ a ⋅ 2a 2 + 2a 2 log ( 2a + 2a 2 ) − 2a 2 log 2a ⎤ ⎦ 2a ⎣ ⎛ 2a + 2a 2 ⎞ = a ⎜ 2 + log ⎟⎠ ⎝ 2a =

= a ⎡ 2 + log (1 + 2 )⎤ ⎣ ⎦ (ii) The points of intersection of the parabola y 2 = 4 ax and the line 3 y = 8 x are obtained as, ⎛ 3y ⎞ y 2 = 4a ⎜ ⎟ ⎝ 8 ⎠ 3a ⎞ ⎛ y⎜y− ⎟ = 0 ⎝ 2⎠

y = 0,

3a 9a and x = 0, 2 16

9a 3a , 16 2 3a For the arc OA, y varies from 0 to . 2 1 32a 2 y + 4 a 2 dy Length of the arc OA, s = 2a ∫0 Hence, A :

1 y = 2a 2

(

4a 2 y 2 + 4a2 + log y + y 2 + 4 a 2 2

)

3a 2

0

⎧⎪ ⎛ 3a ⎫⎪⎤ ⎞ 9a 2 1 ⎡ 3a 9a 2 ⎢ = + 4 a 2 + 2a 2 ⎨log ⎜ + + 4 a 2 ⎟ − log 2a⎬⎥ 2 4 2a ⎢ 4 4 ⎠ ⎩⎪ ⎝ ⎭⎪⎥⎦ ⎣ =

⎫⎤ 1 ⎡ 3a 5a ⎛ 3a 5a ⎞ 2⎧ g ⎜ + ⎟ − log 2a⎬⎥ ⎢ ⋅ + 2a ⎨log 2⎠ 2a ⎣ 4 2 ⎩ ⎝ 2 ⎭⎦

⎞ 1 ⎛ 15a 2 + 2a 2 log 2 ⎟ ⎜⎝ ⎠ 2a 8 15 ⎞ ⎛ = a ⎜ log 2 + ⎟ ⎝ 16 ⎠ =

Integral Calculus

6.27

Exercise 6.3 1. Find the length of the arc of following curves: (i) y = log tanh

x 2

from x = 1

4. Find the length of the arc of the parabola y 2 = 8 x cut off by its latus rectum. Find the length of the arc cut off by the line 3 y = 8 x. Ans.: 4

to x = 2 (ii) 24 xy = x + 48 from x = 2

(

2 log 2 +

to x = 4 3 y 2 1 from y = 0

to y = 4 (iv) y

x (2 x ) from x = 0

⎡ Ans.: log ⎡ x + (1 + x 2 ) ⎤ ⎤ ⎣ ⎦ ⎦⎥ ⎣⎢

to x = 2

Ans.: (i) log e +

1 17 , (ii) 6 e

8 (82 82 1) 243 1 (iv) log (2 + 5 ) + 5 2 (iii)

2. Find the length of the curve y 2 (2 x 1)3 cut off by the line x = 4.

⎡ Ans.: 1022 ⎤ ⎢ 27 ⎥⎦ ⎣ 3. Find the arc of the parabola y 2 4 a( a x ) cut off by the y-axis. ⎡ Ans.: a ⎡ 2 2 − log 3 − 2 2 ⎤ ⎤ ⎣ ⎦⎦ ⎣

(

6. Show that if s is the arc of the curve 9 y 2 x(3 x ) 2 measured from the origin to the point P(x, y), then 3s 2 = 3 y 2 + 4 x 2 . 7. Find the length of the loop of the curve (i) 3ay 2 x( x a) 2 (ii) 9 y 2 = ( x + 7) ( x + 4) 2 (iii) 9ay 2 x( x 3a) 2 (iv) ay 2 x 2 ( a x ) Ans.: (i)

)

4 3

Example 1: Find the length of the curve x = a(cosp + p sinp ), y = a(sinp – p cos p ), from p = 0 to p = 2o . x = a(cosq + q sin q )

dx = a( − sin + sin + cos ) = a cos d y = a (sin − cos ) dy = a (cos − cos + sin ) = a sin d

a, (ii) 4 3

(iii) 4 3a, (iv)

Parametric Form

Solution:

15 16

5. Find the length of the arc of the 2 parabola x = 4 ay measured from the vertex to one extremity of the latus rectum.

3

(iii) x

)

2 + log 1 + 2 ,

4

4 3

a

Engineering Mathematics

6.28

For the required arc,

varies from 0 to 2 .

Length of the curve,

s=∫



=∫

2p

2

0

( a q cos q ) 2 + ( a q sin q ) 2 dq

0

= a∫

2p

0

2

⎛ dx ⎞ ⎛ dy ⎞ ⎜⎝ ⎟⎠ + ⎜⎝ ⎟⎠ dθ dθ dθ

q2 q dq = a 2

2p

= 2ap 2 0

p p Example 2: Find the length of the curve x = ep sin + 2cos , 2 2 p p⎞ ⎛ y = ep ⎜ cos − 2 sin ⎟ measured from p = 0 to p = . ⎝ 2 2⎠

Solution:

q⎞ ⎛ q x = eq ⎜ sin + 2 cos ⎟ ⎝ 2 2⎠ dx = eq dq

q ⎞ q ⎛1 q q⎞ 5 q q ⎛ q ⎜⎝ sin + 2 cos ⎟⎠ + e ⎜⎝ cos − sin ⎟⎠ = e cos 2 2 2 2 2 2 2

q q⎞ ⎛ y = eq ⎜ cos − 2 sin ⎟ ⎝ 2 2⎠ dy = eq dq

q q⎞ q ⎛ ⎜⎝ cos − 2 sin ⎟⎠ + e 2 2

5 q q⎞ q ⎛ 1 q ⎜⎝ − sin − cos ⎟⎠ = − e sin 2 2 2 2 2

For the required arc, q varies from 0 to p . Length of the curve

s=∫

π

0

2

2

⎛ dx ⎞ ⎛ d y ⎞ ⎜⎝ ⎟⎠ + ⎜⎝ ⎟⎠ dθ dθ dθ

π 25 2θ θ 25 2θ θ 25 e cos 2 + e 2θ sin 2 dθ = ∫ e dθ 0 4 2 4 2 4 5 π 5 π = ∫ eθ dθ = eθ 0 0 2 2 5 = (eπ − 1) 2

=∫

π

0

Example 3: Find the length of the cycloid from one cusp to the next cusp y x = a(q + sin p ), y = a(1 − cos p ). Solution:

x = a (q + sin q ) dx = a (1 + cos q ) dq y = a (1 − cos q ) dy = a sin q dq

B

A

−ap

O

Fig. 6.12

ap

x

Integral Calculus

6.29

For the arc OB, x varies from 0 to ap , hence q varies from 0 to p. Length of the arc AB,

s = 2 (Length of arc OB)

= 2∫

2

2

π ⎛ dx ⎞ ⎛ dy ⎞ 2 2 2 2 ⎜⎝ ⎟⎠ + ⎜⎝ ⎟⎠ dθ = 2 ∫0 a (1 + cos θ ) + a sin θ dθ dθ dθ

π

0

= 2a ∫

π

0

2(1 + cos θ ) dθ = 4 a



π

0

cos

θ dθ 2

π

θ = 4 a 2 sin 20 = 8a

y

Example 4: Find the length of one arc of the cycloid x = a(p – sinp ), y = a(1 + cosp ). Solution:

x = a (q − sin q ) dx = a (1 − cos q ) dq y = a (1 + cos q )

A

B

O

2ap

x

dy Fig. 6.13 = − a sin q dq For the arc AB, x varies from 0 to 2ap, hence q varies from 0 to 2p. Length of the arc AB,

s=∫



=∫

2p

0

0

= a∫

2p

= a∫

2p

0

0

2

2

⎛ dx ⎞ ⎛ dy ⎞ ⎜⎝ ⎟⎠ + ⎜⎝ ⎟⎠ dθ dθ dθ a 2 (1 − cos q ) 2 + a 2 sin 2 q dq 2 − 2 cos q dq 2 ⋅ 2 sin 2

= 2a −2 cos

q 2

2p q q dq = 2a ∫ sin dq 0 2 2

2p

= −4 a (cos p − cos 0) 0

= 8a ⎡ ⎛ t ⎞⎤ Example 5: Find the length of the tractrix x = a ⎢ cos t + log tan ⎜ ⎟ ⎥ , y = a sin t ⎝ 2 ⎠⎦ ⎣ o from t = to any point t. 2 ⎡ ⎛ t ⎞⎤ x = a ⎢cos t + log tan ⎜ ⎟ ⎥ Solution: ⎝ 2 ⎠⎦ ⎣

Engineering Mathematics

6.30

⎡ ⎤ ⎢ ⎥ dx 1 2 ⎛t ⎞ 1 = a ⎢ − sin t + sec ⎜ ⎟ ⋅ ⎥ ⎝ 2 ⎠ 2⎥ dt ⎛t ⎞ ⎢ tan ⎜ ⎟ ⎢⎣ ⎥⎦ ⎝2⎠ ⎛ ⎞ 1 1 ⎞ ⎜ ⎟ ⎛ = a ⎜ − sin t + = a ⎜ − sin t + ⎟ t t⎟ ⎝ sin t ⎠ 2 sin cos ⎟ ⎜ ⎝ 2 2⎠ (1 − sin 2 t ) cos 2 t =a sin t sin t y = a sin t =a

dy = a cos t dt

For the required arc, t varies from

Length of the curve,

s=∫ =∫

p to t. 2 2

2

⎛ d x ⎞ ⎛ dy ⎞ ⎜⎝ ⎟⎠ + ⎜⎝ ⎟⎠ dt dt dt

t 2 t

a2

2

t cos 4 t + a 2 cos 2 t dt = a ∫ cos t cot 2 t + 1 dt 2 sin t 2

t

t

2

2

= a ∫ cot t dt = a log sin t = a log sin t

Example 6: For the curve x = a ( 2 cos t − cos 2t ), y = a ( 2 sin t − sin 2t ), show that the length of the arc of the curve measured from t = 0 to the point where the x tangent makes an angle x with the tangent, at t = 0 is given by s = 16 a sin 2 . 6 Solution: x = a ( 2 cos t − cos 2t ) dx = a ( −2 sin t + 2 sin 2t ) = 2a (sin 2t − sin t ) dt y = a ( 2 sin t − sin 2t ) dy = a ( 2 cos t − 2 cos 2t ) = 2a (cos t − cos 2t ) dt For the required arc, t varies from 0 to t. Length of the curve,

s=∫

t

0

2

2

⎛ dx ⎞ ⎛ dy ⎞ ⎜⎝ ⎟⎠ + ⎜⎝ ⎟⎠ dt dt dt

Integral Calculus

=∫

t

6.31

4 a 2 [(sin 2t − sin t ) 2 + (cos t − cos 2t ) 2 dt

0

= 2a ∫

t

0 t

= 2a ∫

0

= 2a ∫

t

0

2 − 2(sin 2t sin t + cos t cos 2t ) dt 2 [1 − cos( 2t − t )] dt = 2a ∫

t

0

2 ⋅ 2 sin 2

= 8a − cos = 16 a sin 2

t 2

t 0

2 (1 − cos t ) dt

t t t dt = 4 a ∫ sin dt 0 2 2

t⎞ ⎛ = 8a ⎜1 − cos ⎟ ⎝ 2⎠

t 4

… (1)

d y d y / dt 2a (cos t − cos 2t ) = = d x d x / dt 2a (sin 2t − sin t )

t 3t sin 2 2 = tan 3t = t 3t 2 2 cos sin 2 2 2 sin

At

y t = 0, y = 0, d = 0 dx

Hence, the tangent is x-axis at t = 0. At the point where tangent makes an angle y with the tangent at t = 0, i.e., x-axis, we get dy = tany dx tan

3t = tany 2

3t 2 2y t= 3

y =

Putting t in Eq. (1),

s = 16 a sin 2 = 16 a sin 2

2 12 6

Engineering Mathematics

6.32

2

2

⎛ x⎞3 ⎛ y⎞3 Example 7: Find the total length of the curve ⎜ ⎟ + ⎜ ⎟ = 1. Hence, deduce ⎝a⎠ ⎝b⎠ 2 2 2 o the total length of the curve x 3 + y 3 = a 3 . Also show that the line p = divides 3 2

2

2

the length of the curve x 3 + y 3 = a 3 in the first quadrant in the ratio 1:3. Solution: (i) The parametric equations of the curve 2

y (0, b) B

2

⎛ x ⎞3 ⎛ y ⎞3 ⎜⎝ ⎟⎠ + ⎜⎝ ⎟⎠ = 1 are given by, a b 3 x = a cos3 q , y = b sin q

q= p 3 C p 3

dx = −3a cos 2 q sin q dq dy = 3b sin 2 q cos q dq

(−a, 0)

O

A (a, 0)

(0, −b)

For the arc AB, x varies from a to 0, hence q p varies from 0 to . 2 Total length of the curve, s = 4 (Length of the arc AB) 2

p

Fig. 6.14

2

⎛ dx ⎞ ⎛ dy ⎞ dq = 4∫ 2 ⎜ + 0 ⎝ dq ⎟⎠ ⎜⎝ dq ⎟⎠ p

= 4 ∫ 2 9a 2 sin 2 q cos 4 q + 9b 2 sin 4 q cos 2 q dq 0

p

= 12 ∫ 2 sin q cos q a 2 + (b 2 − a 2 ) sin 2 q dq 0

Putting a 2 + (b 2 − a 2 ) sin 2 q = t 2 , 2(b 2 − a 2 ) sin q cos q dq = 2t dt sin cos

When

d =

t dt b2 − a2

q = 0, t = a p q = , t=b 2 b

s = 12 ∫ t ⋅ a

=

t dt b2 − a2

12 t 3 b2 − a2 3

b

= a

4( b 3 − a 3 ) b2 − a2

4( a 2 + ab + b 2 ) = a+b

x

Integral Calculus

6.33

(ii) Putting b = a,

4( a 2 + a 2 + a 2 ) = 6a 2a 2 2 2 6a 3 = a (iii) Length of the curve x 3 + y 3 = a 3 in the first quadrant = 4 2 2

2

2

Total length of the curve ( x 3 + y 3 = a 3 ) =

p

Length of the arc AC = ∫ 3 3a sin q cos q dq 0

3a p3 3a − cos 2q sin 2q dq = = ∫ 0 2 2 2

p 3 0

9a = 8 Length of the arc BC = length of the arc AB − length of the arc AC 3a 9a 3a − = 2 8 8 Length of the arc BC 1 = Length of the arc AC 3 =

Example 8: Show that the length of the arc of the curve x sin p + y cos p = f (p ), x cos p – y sin p = f (p ) is given by s = f (p ) + f (p ) + C. Solution:

x sin q + y cos q = f ′(q )

… (1)

x cos q − y sin q = f ′′(q )

… (2)

Multiplying Eq. (1) by sinq and (2) by cosq and adding, x = sin q f ′(q ) + cos q f ′′(q )

… (3)

Multiplying Eq. (1) by cosq and (2) by sinq and subtracting, y = cos q f ′(q ) − sin q f ′′(q ) dx = cos q f ′(q ) + sin q f ′′(q ) − sin q f ′′(q ) + cos q f ′′′(q ) dq = cos q [ f ′(q ) + f ′′′(q )] dy = cos q f ′′(q ) − sin q f ′(q ) − cos q f ′′(q ) − sin q f ′′′(q ) dq = − sin q [ f ′(q ) + f ′′′(q )] 2

Length of the arc,

s=∫

2

⎛ dx ⎞ ⎛ dy ⎞ ⎜⎝ ⎟⎠ + ⎜⎝ ⎟ dq dq dq ⎠

= ∫ (cos 2 q + sin 2 q ) [ f ′(q ) + f ′′′(q )]2 dq

Engineering Mathematics

6.34

= ∫ [ f ′(q ) + f ′′′(q )] dq = f (q ) + f ′′(q ) + C Example 9: Show that for the curve 8 a 2 y 2 = x 2 ( a 2 − x 2 ), s=

arc length

a

(2p + sin p cos p) where x = a sinp and that the perimeter of one of the 2 2 y oa loop is . 2 Solution: When x = a sinq 8a 2 y 2 = a 2 sin 2 q ( a 2 − a 2 sin 2 q ) = a 4 sin 2 q cos 2 q a a y= sinq cosq = sin 2q 2 2 4 2 dx = a cos q dq dy a = cos 2q dq 2 2

B (−a, 0)

O

A (a, 0) x

Fig. 6.15

For the upper half of the loop OA, x varies from 0 to a, hence q varies from 0 to Length of one loop,

s = 2( Length of upper half of the loop OA) 2

2

p ⎛ dx ⎞ ⎛ dy ⎞ dq = 2∫ 2 ⎜ + 0 ⎝ dq ⎟⎠ ⎜⎝ dq ⎟⎠ p

= 2 ∫ 2 a 2 cos 2 q + 0

=

a

=

a

=

a

=

a

p 2

∫ 2

0 p 2

∫ 2

0

∫ 2

p 2

0

2 a



p 2

0

a2 cos 2 2q dq 8

8 cos 2 q + ( 2 cos 2 q − 1) 2 dq 4 cos 4 q + 4 cos 2 q + 1 dq ( 2 cos 2 q + 1) dq ( 2 + cos 2q ) dq

sin 2q = 2q + 2 2 pa = 2

p 2 0

=

a 2

(p )

p . 2

Integral Calculus

6.35

Example 10: Show that the length of one complete wave of the curve y = b cos a 2 + b 2 and a.

is equal to the perimeter of the ellipse whose semi-axes are y = b cos

Solution: (i)

x a

x a

dy b x = − sin dx a a x For one complete wave varies from 0 to 2p , i.e., x varies from 0 to 2pa. a Length of one complete wave, s1 = ∫

2p a

=∫

2p a

0

0

2

⎛ dy ⎞ 1 + ⎜ ⎟ dx ⎝ dx ⎠ 1+

b2 x x 1 2p a 2 a + b 2 sin 2 dx sin 2 dx = ∫ 2 0 a a a a

x = t, a

Putting

dx = a dt x = 0, t = 0 x = 2p a, t = 2p 1 2p 2 a + b 2 sin 2 t ⋅ a dt s1 = ∫ a 0

When

=∫

2p

a 2 + b 2 sin 2 t dt

0

⎡∵ a + b sin t dt ⎢ ⎢⎣

p

= 2∫

2

0

= 4∫

p 2

2

2



2a

0

a 2 + b 2 sin 2 t dt

0

(ii) Now, parametric equations of the given

f (t ) dt = 2 ∫ f (t ) dt ⎤ 0 ⎥ if f ( 2a − t ) = f (t ) ⎥⎦ (1)

y

ellipse are x = a 2 + b 2 cost and y = a sin t

B (0, a)

dx dy = − a 2 + b 2 sin t , = a cos t dt dt

A

For the arc AB, x varies from a + b to 0, p hence t varies from 0 to . 2 Perimeter of the ellipse, s2 = 4 (Length of the arc AB) 2

2

a

x

(√a 2 + b 2, 0(

2

Fig. 6.16

2

p p ⎛ dx ⎞ ⎛ dy ⎞ = 4 ∫ 2 ⎜ ⎟ + ⎜ ⎟ dt = 4 ∫ 2 ( a 2 + b 2 ) sin 2 t + a 2 cos 2 t dt 0 0 ⎝ dt ⎠ ⎝ dt ⎠ p

= 4 ∫ 2 a 2 + b 2 sin 2 t dt 0

… (2)

Engineering Mathematics

6.36

From Eqs. (1) and (2), Length of one complete wave = perimeter of the ellipse. Example 11: Show that the perimeter of the ellipse

x2 y2 + = 1 is a 2 b2

⎡ e 2 12 ⋅ 3 ⎤ 12 ⋅ 32 ⋅ 5 2oa ⎢1 − 2 − 2 2 e 4 − 2 2 2 e 6 − …⎥ , where e is the eccentricity of the ellipse. 2 ⋅4 ⋅6 ⎣ 2 2 ⋅4 ⎦

Solution: The parametric equations of the given ellipse are x = a cosq and y = b sinq . dx = − a sin q dq dy = b cos q dq For the arc AB, x varies from a to 0, hence q p varies from 0 to . 2 Perimeter of the ellipse = 4(Length of the arc AB) 2

p 2 0

y B (0, b) A (a, 0) x

O

Fig. 6.17

2

p ⎛ dx ⎞ ⎛ dy ⎞ dq = 4 2 a 2 sin 2 q + b 2 cos 2 q dq ⎜⎝ ⎟⎠ + ⎜⎝ ⎟ ∫0 dq dq ⎠

= 4∫

p

= 4 ∫ 2 a 2 sin 2 q + a 2 (1 − e 2 ) cos 2 q dq 0

p

1

= 4 a ∫ 2 (1 − e 2 cos 2 q ) 2 dq 0

p 2 0

= 4a ∫

⎡ 1 ⎛1 ⎞ ⎢ ⎜⎝ 2 − 1⎟⎠ 1 ⎢1 + ( −e 2 cos 2 q ) + 2 ( −e 2 cos 2 q ) 2 ⎣ 2 2! ⎤ 1 ⎛1 ⎞ ⎛1 ⎞ ⎥ ⎜ − 1⎟ ⎜ − 2 ⎟⎠ 2 ⎝2 ⎠⎝2 ( −e 2 cos 2 q )3 + …⎥ dq + 3! ⎦

1 4 1⋅ 3 6 ⎛ 1 ⎞ = 4 a ∫ 2 ⎜1 − e 2 cos 2 q − e cos 4 q − e cos6 q …⎟ dq 0 ⎝ ⎠ 2 2⋅4 2⋅ 4 ⋅6 p

1 p 1 4 3 1 p 1 ⋅ 3 6 5 ⋅ 3 ⋅1 p ⎡p 1 ⎤ e ⋅ ⋅ − e ⋅ ⋅ – …⎥ = 4a ⎢ − e 2 ⋅ ⋅ − 2 2 2⋅ 4 4 2 2 2⋅ 4 ⋅6 6⋅4⋅2 2 ⎣2 2 ⎦

2p a 1

e2 22

12 3 4 e 22 4 2

12 32 5 6 e … 22 4 2 6 2

⎡ b2 ⎤ ⎢∵ e = 1 − 2 ⎥ a ⎥⎦ ⎢⎣

Integral Calculus

6.37

Exercise 6.4 1. Find the length of the following curves: (i) x = a (2 cos q + cos 2q ), y = a (2sin q + sin 2q ), from q = 0 to any point q . (ii) x a (q sin q ), y a (1 cos q ) from q = 0 to q = 2p

Ans. : (i) 8a sin (ii) 8a

p

(iii)

2(e 2

1)a

(iv) log sec q (v) log cosh t

(iii) x = aeq sin q , y = aeq cos q from p q = 0 to q = 2 (iv) x log (secq tan q ) sin q , y = cos q from

4b aq ( a + b) cos 2b a 4 (vii) a sin q 3 (vi)

2. Prove that the loop of the curve 1 x = t2, y = t – t3 is of length 4 3. 3 3. Show that the length of the arc of

q = 0 to any point q (v) x a (t tanh t ), y = a sech t from t = 0 to any point t. a+b (vi) x = (a + b) cosq – bcos q , b y = (a + b) sinq – b sin ⎛⎜ a + b q ⎞⎟

⎝ b

q 2

the curve x = a(3sinq – sin3 q ), y = a cos3q measured from (0, a) to 3 a(q + sin q cos q ). 2 4. If ‘s’ be the length of the arc of the curve x = a(q + sinq cosq ), y = a(1 + sinq )2, measured from the p point q to a point q , show 2 that s4 varies as y 3 . any point (x, y) is



pb from q = to any point q. a (vii) x = a sin 2q (1 + cos 2q ), y a cos 2q (1 cos 2q ), from q = 0 to any point q

Polar Form Example 1: Find the length of the spiral r = e 2q from q = 0 to p = 2o . r = e 2q dr = 2e 2q dq For the required length of the spiral, q varies from 0 to 2p .

Solution:

Length of the spiral, s = ∫

2p

0

2

2p ⎛ dr ⎞ r2 + ⎜ dq = ∫ e 4q + 4e 4q dq ⎟ 0 ⎝ dq ⎠

6.38

Engineering Mathematics

= 5∫

e 2q e dq = 5 2

2p

2p

2q

0

0

5 4p = (e − 1) 2 Example 2: Find the length of the arc of the equiangular spiral r = aeq cota from the point corresponding to p = 0 to the point corresponding to q = tana . Solution:

r = aeq cot a

dr = a cot a eq cot a dq For the required arc, q varies from 0 to tan a . Length of the arc,

s=∫

tan a

=

tan a

0

0

2

⎛ dr ⎞ dq r2 + ⎜ ⎝ dq ⎟⎠ a 2 e 2q cot a + a 2 cot 2 a e 2q cot a dq

= a 1 + cot 2 a a cosec a

tan a 0

eq cot a dq

tan a

eq cot a cot a

0

a cosec a tan a cot a (e cot a

1)

a sec a (e 1) a (e 1) sec a Example 3: Find the length of the cissoid r = 2a tanq sinq from q = 0 to p q = . 4 Solution: r = 2a tan q sin q dr = 2a (sec 2 q sin q + tan q cos q ) dq = 2a sin q (sec 2 q + 1)

For the required arc length of the cissoid, q varies from 0 to

Length of the curve,

s=∫

p 4

0

=

p 4 0 p 4 0

p . 4

2

⎛ dr ⎞ dq r +⎜ ⎝ dq ⎟⎠ 2

4 a 2 tan 2 q sin 2 q + 4 a 2 sin 2 q (sec 2 q + 1) 2 dq 4 a 2 sin 2 q (sec 2 q 1 sec 4 q

2sec 2 q 1) dq

Integral Calculus

6.39

p

= ∫ 4 4 a 2 sin 2 q sec 2 q (sec 2 q + 3) dq 0

p

= ∫ 4 4 a 2 tan 2 q (tan 2 q + 4) dq 0

p

= ∫ 4 2a tan q tan 2 q + 4 dq 0

Putting

tan q + 4 = t , 2

2

2 tan q sec 2 q dq = 2t dt

t dt t dt t dt = = sec 2 q 1 + tan 2 q t 2 − 3 q = 0, t = 2 p q = , t= 5 4

tan q dq =

When

5

s = ∫ 2a ⋅ 2

= 2a t +

5 t2 3 ⎞ ⎛ dt = ∫ 2a ⎜1 + 2 dt 2 ⎝ t − 3 ⎟⎠ t −3 2

3 2 3

log

5

t− 3 t+ 3

⎛ 3 = 2a ⎜ 5 + log 2 ⎝

2

5− 3 5+ 3

⎡ ⎪⎧ 3 ⎢ = 2a ⎢ 5 − 2 + log ⎨ 2 ⎪⎩ ⎢⎣

(

−2−

3 2− 3⎞ log ⎟ 2 2+ 3⎠

)

(

2 ⎧⎪ 2 − 3 5 − 3 ⎫⎪ ⎬ − log ⎨ 5 − 3 ⎪⎭ ⎪⎩ 4 − 3

) ⎫⎪⎬⎤⎥ 2

⎪⎭⎥ ⎥⎦

⎡ ⎤ 3 = 2a ⎢ 5 − 2 + log 4 − 15 − log 7 − 4 3 ⎥ 2 ⎣ ⎦

(

)

(

)

Note: Only positive values of t are considered since q lies in the first quadrant. Example 4: Find the length of the whole arc of the cardioid r = a (1 + cosq ) and show that the upper half is bisected p by the line q = . 3 Solution:

r = a (1 + cos q )

q= p 2 C

q= p 3 A

D q=p

dr a sin q dq (i) For the arc BACDO, q varies from 0 to p. Length of the whole arc of the curve, s = 2( Length of arc BACDO )

O

Fig. 6.18

B

q=0

Engineering Mathematics

6.40

2

dr dq

2 r2

p

2

0

a 2 (1 cos q ) 2 ( a sin q ) 2 dq

p

2

0

= 4a

p

a 2 2 cos q dq p 0

cos

2

0

q q dq = 4 a 2sin 2 2

a 2 2 cos 2

q dq 2

p

= 8a 0

Length of the upper half of the cardioid = 4a p intersects the cardioid at point A. 3 p q q Length of the arc, BA = 3 2a cos dq = 2a 2sin 0 2 2 (ii) Let q =

p 3

= 2a

0

Hence, the upper half of the cardioid is bisected by the line q =

p 3

Example 5: Find the length of the cardioid r = a (1 - cosq ) lying outside the circle r = a cosq . Solution: The points of intersection of cardioid r r = a cos q is obtained as,

a(1 cos q ) a cos q 1 = 2 cos q cos q =

q= p 3

q= p 2 A

1 2

q =± Hence at A,

a (1 cos q ) and the circle

p 3

q=p B

O

p q = 3 r a(1 cos q ) Fig. 6.19

dr = a sin q dq

p For the arc of the cardioid lying outside the circle, q varies from to p . 3 Length of the cardioid lying outside the circle, s = 2 (Length of arc AB) p

= 2 ∫p

3 p

= 2 ∫p

3

2

⎛ dr ⎞ r2 + ⎜ dq ⎝ dq ⎟⎠ a 2 (1 − cos q ) 2 + ( a sin q ) 2 dq

q=0

Integral Calculus

p

6.41

p

= 2 ∫p a 2 − 2 cos q dq = 2 ∫p a 2 ⋅ 2 sin 2 3

3

q q = 4 a ∫p sin dq = 4 a −2 cos 2 2 3

q dq 2

p

p

p 3

⎛ 3⎞ = −8a ⎜ − ⎟ ⎝ 2 ⎠ = 4a 3 Example 6: Show that the length of the arc of that part of cardioid r = a (1 + cos q ) which lies on the side of the line 4r = 3a sec q away from the pole is 4a. Solution: The points of intersection of cardioid r = a (1 + cos q ) and the line 4 r = 3a secq are obtained as, q= p 2

3a a (1 + cos q ) = secq 4 4 (1 + cos q ) cos q = 3 4 cos q (2 cos q

4 cos 2 q

3

0

3) (2 cos q 1)

0

q= p 3

A

B (2a, 0) q = 0

O

4r = 3a sec q

1 3 and cos q = (does not exist) 2 2 p q =± 3 p Hence at A, q = 3 r = a (1 + cos q )

C

cos q =

Fig. 6.20

dr a sin q dq p For the arc BA, q varies from 0 to . 3 Length of the arc CBA, s = 2 (length of arc BA) = 2∫

2

p 3

⎛ dr ⎞ r +⎜ dq ⎝ dq ⎟⎠ 2

0 p

= 2 ∫ 3 a 2 (1 + cos q ) + ( − a sin q ) dq 2

2

0 p 3

p

0

0

= 2 ∫ a 2 + 2 cos q dq = 2 ∫ 3 a 2 ⋅ 2 cos 2 = 4a ∫

p 3

0

= 4a

q q cos dq = 4 a 2 sin 2 2

p 3 0

q dq 2

Engineering Mathematics

6.42

3 Example 7: Find the total length of the curve r = a sin

q= p 2

q . 3

B

q r = a sin 3 dr q q 1 a 3sin 2 cos dq 3 3 3 q q = a sin 2 cos 3 3 3

Solution:

A C

D

For the arc OABCD, q varies from 0 to 3p . 2 Length of the curve = 2 ( Length of the arc OABCD ) = 2∫

3p 2

0

= 2∫

3p 2

0

= 2∫

3p 2

0

q=0

O

Fig. 6.21

2

⎛ dr ⎞ r2 + ⎜ dq ⎝ dq ⎟⎠ a 2 sin 6 a sin 2

q q q + a 2 sin 4 cos 2 dq 3 3 3

3p q dq = a ∫ 2 0 3

3 2q = a q − sin 2 3

2q ⎛ ⎜⎝1 − cos 3

⎞ ⎟⎠ dq

3p 2 0

3p 3 = a⋅ = pa 2 2 Example 8: Find the perimeter of the 2 2 lemniscate r = a cos 2q .

q= p 2

B

r 2 = a 2 cos 2q

Solution: 2r

dr dq dr dq

A

a 2 ( sin 2q ) 2 O 2

a sin 2q r

For the arc OBA, q varies from 0 to

p . 4

Fig. 6.22

Perimeter of the curve = 4 (Length of the arc OBA) p

2

⎛ dr ⎞ = 4 ∫ 4 r2 + ⎜ dq 0 ⎝ dq ⎟⎠ p

= 4 ∫ 4 a 2 cos 2q + 0

q= p 4

a4 sin 2 2q dq r2

q=0

Integral Calculus

p

= 4 ∫ 4 a 2 cos 2q + 0

p

= 4 ∫ 4 a 2 cos 2q + 0

p

= 4∫ 4 0

a4 sin 2 2q dq r2 a 4 sin 2 2q dq a 2 cos 2q

p a4 1 dq = 4a ∫ 4 dq 0 a cos 2q cos 2q 2

2q = t , 2dq = dt

Putting When

6.43

q = 0, t = 0 p p q = , t= 2 4

Perimeter of the curve =

4 a p2 dt 2 ∫0 cos t

p −1 ⎛1 1⎞ = 2a ∫ 2 sin 0 t ⋅ (cos t ) 2 dt = aB ⎜ , ⎟ 0 ⎝2 4⎠ 2

⎛ ⎞ 1 1 a1 1 a 2 ⎜⎝ 4 ⎟⎠ = 2 4 = 1 1 3 1− 4 4 4 2 2 ⎛ ⎞ ⎛ 1⎞ 1 1 a ⎜ ⎟ a p ⎜ ⎟ 2 ⎝ 4⎠ ⎝ 4⎠ = = p p 2 p sin 4 2 a ⎛ 1⎞ = ⎜ ⎟ 2p ⎝ 4 ⎠ Example 9: Show that for the parabola 2a = 1 + cos q , the arc intercepted between the r vertex and the extremity of the latus rectum is a

(

)

⎡ p ⎤ ⎢∵ n 1− n = sin np ⎥ ⎦ ⎣

q= p 2 B

O

A

2 + log 1 + 2 .

p Solution: The latus rectum is the line q = . 2 2a = 1 + cos q r 2a r= = 1 + cos q

q = a sec 2 q 2 2 cos 2 2 2a

Fig. 6.23

q=0

Engineering Mathematics

6.44

dr q q = a sec 2 ⋅ tan dq 2 2 p For the arc AB, q varies from 0 to . 2 2

p

⎛ dr ⎞ s = ∫ 2 r2 + ⎜ dq 0 ⎝ dq ⎟⎠

Length of the arc AB,

p

= ∫ 2 a 2 sec 4 0

p

= ∫ 2 a sec 2 0

Putting

When

tan

q = t, 2

q q q + a 2 sec 4 tan 2 dq 2 2 2

q q 1 + tan 2 dq 2 2

1 2q q sec dq = dt , sec 2 dq = 2dt 2 2 2 q = 0, t = 0 p q = , t =1 2 1

s = ∫ 2 a 1 + t 2 dt = 2 a 0

(

t 1 1 + t 2 + log t + 1 + t 2 2 2

(

)

(

)

1

0

)

1 ⎡1 ⎤ = 2a ⎢ 2 + log 1 + 2 ⎥ = a ⎡ 2 + log 1 + 2 ⎤ ⎣ ⎦ 2 ⎣2 ⎦ Example 10: Show that the whole length of the limacon r = a cosp + b (a < b) is equal to that of an ellipse whose semi-axes q= p 2 are equal in length to the maximum and B minimum radii vectors of the limacon. Solution:

For the arc ABC,

r = a cos + b

q=p

C (b−a, 0)

dr a sin d varies from 0 to

O

q=0 A (b + a, 0)

Whole length of the limacon Fig. 6.24 = 2(length of the arc ABC ) = 2∫

p

0 p

= 2∫

= 2∫

0 p

0

2

⎛ dr ⎞ r2 + ⎜ dq ⎝ dq ⎟⎠ ( a cos q + b) 2 + ( − a sin q ) 2 dq

a 2 + b 2 + 2ab cosq dq

… (1)

Integral Calculus

6.45

Maximum radius vector of the limacon = a (1) + b = b + a

[∵ Maximum value of cosq

Minimum radius vector of the limacon

a ( 1) b

b a

[∵ Minimum value of cosq

The parametric equations of the ellipse with above radii vectors as semi-axes are given as, x = (b + a) cos q and y (b a) sin q

For the arc AB,

dx = −(b + a) sin q dq dy (b a) cos q dq p varies from 0 to . 2

= 1]

= −1]

q= p 2 B (0, b- a)

O

A (b+a, 0) q =0

Fig. 6.25

Whole length of the ellipse = 4(length of the arc AB) 2

2

p ⎛ dx ⎞ ⎛ dy ⎞ dq = 4∫ 2 ⎜ + 0 ⎝ dq ⎟⎠ ⎜⎝ dq ⎟⎠ p

= 4 ∫ 2 [−(b + a) sin q ]2 + [(b − a) cos q ]2 dq 0

p

= 4 ∫ 2 ( a 2 + b 2 + 2ab sin 2 q − 2ab cos 2 q ) dq 0

p

= 4 ∫ 2 ( a 2 + b 2 − 2ab cos 2q ) dq 0

Putting

2q = t ,

dt 2 = 0,

dq = When

t=0 p q = , t =p 2

Whole length of the ellipse = 2 ∫

p

= 2∫

p

= 2∫

p

0

0

0

a 2 + b 2 − 2ab cos t dt a 2 + b 2 − 2ab cos (p − t ) dt a 2 + b 2 + 2ab cos t dt ⎡∵ a f ( x ) dx = a f ( a − x ) dx ⎤ ∫0 ⎥⎦ ⎢⎣ ∫0

From Eqs. (1) and (2), Whole length of the limacon = Whole length of the ellipse

… (2)

Engineering Mathematics

6.46

Example 11: Find the length of the arc of the hyperbolic spiral r = a from the point r = a to r = 2a. rq = a

Solution: r

Length of the arc,

s=∫ =∫ =∫

Putting

2a a

2a a 2a a

dq +q = 0 dr 2

⎛ dq ⎞ dr 1+ r2 ⎜ ⎝ dr ⎟⎠ 1 + q 2 dr = ∫

a

1+

a2 dr r2

r +a dr r2 2

2

r 2 + a2 = t 2 , t dt

2r dr = 2t dt , dr =

When

2a

r = a, s=∫ =∫

t

t=a 5 t dt

a 2

t 2 − a2

a 5

dt + a 2 ∫

a 2

a2

t=a 2

r = 2a, a 5

t2

t 2 − a2 a 5

a 2

=∫

a 5 a 2

dt = t t − a2 2

t 2 − a2 + a2 dt t 2 − a2 a 5

a 5 a 2

+

a2 t−a log 2a t+a a

)

a⎡ a 5−a a 2 − a⎤ − log ⎢log ⎥ 2⎣ a 2 + a⎦ a 5+a

)

a⎡ 5 −1 2 − 1⎤ − log ⎢log ⎥ 2⎣ 5 +1 2 + 1⎦

)

⎡⎛ 5 − 1⎞ ⎛ 2 + 1⎞⎤ a log ⎢ ⎜ ⎟⎜ ⎟⎥ 2 ⎢⎣ ⎝ 5 + 1 ⎠ ⎝ 2 − 1 ⎠ ⎥⎦

=a

(

5− 2 +

=a

(

5− 2 +

=a

(

5− 2 +

=a

(

⎡ a 5 − 2 + log ⎢ ⎢ 2 ⎢⎣

=a

(

5− 2

) )

( 2( + a log

5 −1

) 2 + 1)

5 +1

5 +1

2

(

)

2 2 +1 ⎤ ⎥ 2 −1 ⎥ ⎥⎦

2

Integral Calculus

6.47

Exercise 6.5 1. Find the perimeter of the following curves: (i) r = a cos (ii) r

a(q 2 1)

(iii) r = a cos3

q 3

(iv) r = ae m (v) r = aq q 2 (vii) r = 4 sin 2 q (vi) r = a sec 2

⎡ Ans. : ⎤ ⎢ ⎥ (i) p a ⎢ ⎥ 8a ⎢ ⎥ (ii) 3 ⎢ ⎥ 3p a ⎢ ⎥ (iii) ⎢ ⎥ 2 ⎢ ⎥ 1 + m2 ⎢ ⎥ r r − (iv) ( 2 1) m ⎢ ⎥ ⎢ ⎥ a ⎢ (v) ⎡q 1 + q 2 + sinh −1 q ⎤ ⎥ ⎦⎥ 2⎣ ⎢ ⎢ (vi) 2a ⎡⎣ 2 + log ( 2 + 1) ⎤⎦ ⎥ ⎢ ⎥ 4 ⎢ ⎥ (vii) 8 + log( 3 + 2) ⎥⎦⎥ 3 ⎣⎢⎢

2p bisects the upper half of 3 the cardioid. [Ans. : 8a] line q =

3. Find the length of the cardioid r = a(1 + cos q ) which lies outside the circle r + a cos q = 0. ⎡ Ans. : 4 3a ⎤ ⎣ ⎦ 4. Prove that the length of the spiral r = ae cot as r increases from r1 to r2 is given by (r2 r1 ) sec . 5. Find the length of the cardioid r a (1 cos ) lying inside the circle r = a cos . Ans. : 8a 1

m 6. Find the length of the spiral r = ae lying inside the circle r = a.

Ans. :

a 1 + m2 m

7. Find the length of the arc of parabola l = 1 + cos cut off by its latus recr tum.

2. Find the perimeter of the cardioid r

3 2

a(1 cos q ) and prove that the

(

6.4 AREAS OF PLANE CURVES (QUADRATURE) The process of determining the area of a plane region is known as quadrature.

Area Bounded by the Curve in Cartesian Form

Let y = f (x) be a curve defined in the interval [a, b]. The area bounded by the curve y = f (x), the x-axis and the two lines x = a and x = b is given by,

)

⎡ Ans. : l ⎡ 2 + log 1 + 2 ⎤ ⎤ ⎣ ⎦ ⎦⎥ ⎣⎢

Fig. 6.26

Engineering Mathematics

6.48

Area,

A =

b

f ( x ) dx =

a

b a

y dx

Similarly, the area bounded by the curve x = f ( y ), the y-axis and the two lines, y = c and y = d is given by,

A=

d

f ( y )dy =

c

d c

x dy

When the portion of the curve under consideration is above x-axis, y is positive and hence, area will be positive. When the portion of the curve under consideration is below x-axis, y is negative and hence, area will be negative. In such a case, A=

b a

Fig. 6.27

f ( x ) dx

However, when the curve f (x) crosses the x-axis several times, the total area bounded by the curve is the sum of the areas above and below the x-axis with the absolute value taken for the areas when the curve is below x-axis. Further, the area bounded by the curves y = f1(x) and y = f2(x) and the lines x = a and x = b is given by, A

b a b a b a

b

f 2 ( x ) dx

[f

2

( y2

( x)

a

f1 ( x ) dx

f1 ( x ) ] dx

y1 ) dx

Fig. 6.28

Area Bounded by the Curve in Parametric Form When the equation of the curve is given in parametric form x = f1 (t ), y = f 2 (t ) with t1 x(t1 ) = a, x(t2 ) = b, the area is given by, b b t2 dx A = f ( x ) dx = y d x = y dt a a t1 dt

Area Bounded by the Curve in Polar Form

t

t2 and

Let r = f ( ) be the equation of the curve and OA, OB be the radii vectors at q = q1 , q = q 2 . The whole area is divided into small sectors, such as OPQ subtending an angle at O. Let P(r, ) and Q(r + r, + ) be the two points on the curve. If dA is the area of the elementary triangular strip OPQ, then 1 δ A = OP ⋅ OQ sin δθ 2 1 = r ( r + δ r ) sin δθ 2 δA 1 sin δθ = r (r + δ r ) δθ 2 δθ Fig. 6.29

Integral Calculus

Taking limits as

6.49

0,

A

1 sin lim r ( r + r ) 2 0 dA 1 2 ⎡ ⎤ sin d q = r ∵ lim = 1 and d q → 0, d r → 0 ⎥ ⎢ d q → 0 d 2 dq ⎣ ⎦ 1 2 2 2 1 2 ∫dA = ∫ 1 2 r d = 2 ∫ 1 r d 1 2 A = ∫ r 2d 2 1

lim

0

=

Cartesian Form Example 1: Find the area bounded by the ellipse

x2 y2 + = 1. a2 b2

Solution: The region is symmetric in all the four quadrants. For the region in the first quadrant, x varies from 0 to a. Area, A = 4 (Area in the first quadrant) a

= 4 ∫ y dx = 0

4b a 2 a − x 2 dx a ∫0 a

⎤ x 4b ⎡ a 2 4b x a 2 − x 2 a 2 + sin −1 = = ⎢ ⋅ ⎥ 2 2 a a ⎣ 2 2⎦ a 0 = ab

Fig. 6.30

Example 2: Find the area bounded by the curve a 2 x 2

y 3 (a

y ).

Solution: The region is symmetric about y -axis. For the region in the first quadrant, y varies from 0 to a. Area, A = 2 (Area bounded by the curve in first quadrant) a

= 2 ∫ x dy = 0

Putting When

2 a 32 y a − y dy a ∫0

y = a sin 2 q , dy = 2a sin q cos q dq y = 0, q = 0 y = a, q = p 2 2 p2 32 3 A a sin q a cos 2 q 2a sin q cos q dq a 0 p 311 p 4a 2 2 sin 4 q cos 2 q dq 4 a 2 0 6 4 2 2 p a2 = 8

Fig. 6.31

Engineering Mathematics

6.50

Example 3: Find the area enclosed by the curve a 4 y 2

x 4 (a 2

x2 ) .

Solution: The region is symmetric in all the quadrants. For the region in the first quadrant, x varies from 0 to a. Area, A = 4(Area in the first quadrant) a

= 4 ∫ y dx 0

1 a2

= 4⋅



a

0

x 2 a 2 − x 2 dx

Putting x = a sin q , dx = a cos q dq When x = 0, x = a, 4 a2

A

4a2 =

q =0 p q = 2 p 2 0

a 2 sin 2 q a cos q a cos q dq

p 2 0

Fig. 6.32

sin 2 q cos 2 q dq

4a2

11 p 4 2 2

1 2 pa 4

Example 4: Find the area enclosed by the curve a 2 y 2

x 2 (2a

x )( x a ).

Solution: The region is symmetric about the x-axis. For the region above the x-axis, x varies from a to 2a. Area, A = 2(Area above x-axis) = 2∫

2a a

y dx

2 2a x ( 2a − x )( x − a) dx a ∫a 2 2a = ∫ x − x 2 + 3ax − 2a 2 dx a a Fig. 6.33 1 2a = − ∫ ( −2 x + 3a − 3a) − x 2 + 3ax − 2a 2 dx a a 2a 1 2a ( − x 2 + 3ax − 2a 2 dx =− ∫ − x 2 + 3ax − 2a 2 ) ( −2 x + 3a ) dx + 3 ∫ a a a =

3 1 2 =− ( − x 2 + 3ax − 2a 2 ) 2 a 3

2a

a

+ 3∫

2a a

2

2

3a ⎞ ⎛a⎞ ⎛ ⎜⎝ ⎟⎠ − ⎜⎝ x − ⎟⎠ dx 2 2 n +1 ⎡ ⎡⎣ f ( x )⎤⎦ ⎤ n ⎢∵ ∫ [ f ( x ) ] f ′( x ) d x = ⎥ n +1 ⎥ ⎢ ⎣ ⎦

Integral Calculus

6.51

2a

3a ⎞ ⎛ x− 1 ⎛a⎞ ⎛ 3a ⎞ a2 −1 ⎜ 2 ⎟ = 3 x ⎜ ⎟ − ⎜ x − ⎟ + sin ⎜ a ⎟ 2 ⎝2⎠ ⎝ 2⎠ 8 ⎜ ⎟ ⎝ 2 ⎠a 2

=

2

3 3a 2 ⎡sin −1 1 − sin −1 ( −1) ⎤⎦ = p a 2 8 ⎣ 8

Example 5: Find the area included between the curve y 2 ( a - x ) = x 3 and its asymptote. Solution: The equation of the curve can be rewritten as, 3

x2

y=

a x

The asymptote is the line, x = a. The region is symmetric about the x-axis. For the region, x varies from 0 to a. Area,

A = 2(Area above x-axis) =2

a 0

y dx 3

=2 Putting

x2

a 0

a x

dx Fig. 6.34

x = a sin q , dx = 2a sin q cos q dq 2

x = 0,

When

x = a,

q =0 p q = 2

3

π 2 0

A = 2∫

a 2 sin 3 θ a cos 2 θ

⋅ 2a sin θ cos θ dθ

π

= 4 a 2 ∫ 2 sin 4 θ dθ = 4 a 2 ⋅ 0

=

3 ⋅1 π ⋅ 4⋅2 2

3 π a2 4

Example 6: Find the area enclosed by the curve x ( x 2 + y 2 ) = a ( x 2 its asymptote. Solution: The equation of the curve can be rewritten as, y=x

a x a+x

y 2 ) and

Engineering Mathematics

6.52

x

The asymptote is the line,

a

The region is symmetric about the x-axis. For the region, x varies from a to 0. Area, A = 2(Area above x-axis)

Putting When

=2

0

=2

0

a

a

y dx = 2 x( a x ) a2

x2

0 a

a x dx a+ x

x

dx

x = a sin , dx = a cos q dq x = 0, x

Fig. 6.35

q =0 p 2 a sin θ ( a − a sin θ ) ⋅ a cos θ dθ a cos θ

a, q

A = 2∫

0 −

π 2

0 ⎡ 0 ⎛ 1 − cos 2θ ⎞ ⎤ = 2a 2 ∫ π (sin θ − sin 2 θ ) dθ = 2a 2 ∫ π ⎢sin θ − ⎜ ⎟⎠ ⎥ dθ − − ⎝ 2 ⎦ 2 ⎣ 2

= 2a 2 − cos θ −

θ sin 2θ + 2 4

0

π − 2

π⎤ ⎡ = 2a 2 ⎢ −1 − ⎥ 4⎦ ⎣

π⎞ ⎛ = −a2 ⎜ 2 + ⎟ ⎝ 2⎠ A = a2 2 +

Neglecting the negative sign,

p 2

Example 7: Find the area enclosed between the curve y( x 2 + 4a 2 ) = 8 a 3 and its asymptote. Solution: The equation of the curve can be rewritten as, x2 = The asymptote is the line,

4 a 2 ( 2a − y ) y

y = 0, i.e., x-axis

The region is symmetric about the y-axis. For the region in first quadrant, x varies from 0 to . Area,

A = 2(Area above x -axis in first quadrant)

Fig. 6.36

Integral Calculus

6.53



= 2 ∫ y dx 0

= 2∫



0



x 8a3 1 dx = 16 a3 ⋅ tan −1 2 2 2a 2a 0 x + 4a

= 8a 2 ⋅

2

= 4 a2

Example 8: Find the area included between the curve x 2 y 2 its asymptotes.

a2 ( y2

x 2 ) and

Solution: The equation of the curve can be written as, ax y= 2 a x2 The asymptotes are lines, x = a and x

a.

Fig. 6.37 The region is symmetric about both the axes. For the region above the x-axis in the first quadrant, x varies from 0 to a. Area, A = 4(Area above x -axis in first quadrant) a

= 4 ∫ y dx 0

= 4a ∫

x

a

a − x2

0

2

dx −1

a

= − 2a ∫ ( a 2 − x 2 ) 2 ( −2 x )dx 0

= −4 a a − x 2

2

a 0

= 4a

⎡ ⎢∵ ∫ ⎢⎣

[ f ( x ) ] f ′ ( x ) dx = n

[ f ( x)] n+1 ⎤⎥ n +1

⎥⎦

2

Example 9: Find the area enclosed between the curve x 2 = 4 y and the line x = 4y – 2.

Engineering Mathematics

6.54

Solution: The points of intersection of the curve x 2 = 4 y and line x

4 y 2 are

obtained as, x2 = x + 2 x2

x 2 ( x 1)( x 2)

x

0 0

1, 2 and y =

1 ,1 4

Hence, P :

1,

1 4

Area,

A

Area PTQRSP Area POQRSP

and Q : (2, 1)

Fig. 6.38 For the regions PTQRSP and POQRSP, x varies from 1 to 2. x+2 dx 4

2

A

1

1 x2 4 2

2

2x 1

x2 dx 1 4

2

1 x3 4 3

1 1 2 4 2 4 2 9 = 8

2

1

1 8 1 4 3 3

Example 10: Find the area bounded by the parabola y = x 2 + 2 and the lines x = 0, x = 1 and x + y = 0. Solution: For the regions TPQST and TOST, x varies from 0 to 1. Area,

A

Area TPQST 1 0

( x2

1

2) dx 1

=

Area TOST 0

x3 x + 2x + 3 2 0

17 = 6

( x) dx

2 1

0

Fig. 6.39

Example 11: Find the area bounded by the parabolas y 2 = 5 x + 6 and x 2 = y. Solution: The points of intersection of the parabolas y 2 = 5 x + 6 and x 2 = y are obtained as, x 4 = 5x + 6

Integral Calculus

x

6.55

y

1, 2 and y = 1, 4

Hence, P : ( 1, 1) and Q : (2, 4) Area, A

Area PMQRSP Area POQRSP

For the regions PMQRSP and POQRSP, x varies from 1 to 2. A= ∫

2 −1

(

)

Q

x2 = y (−1, 1)

M

(2, 4)

P

2

5 x + 6 dx − ∫ x 2 dx

S

−1

O

R

x

2

(5 x + 6 ) = 3 5⋅ 2

3 2

x3 − 3

2

−1

y 2 = 5x + 6

−1

3 ⎤ ⎛8 1⎞ 2 ⎡ 2 ⎢(16) − 1⎥ − ⎜⎝ + ⎟⎠ 15 ⎣ ⎦ 3 3 27 = 5

Fig. 6.40

=

Example 12: Find the area common to the parabola y 2 = x and the circle x 2 + y 2 = 2.

Solution: The points of intersection of the parabola y 2 = x and circle x 2 + y 2 = 2 are obtained as, x2 x 2 0 ( x 1)( x 2) 0

x 1,

2

When x = 1, y = ± 1 and when x = –2, y2 is negative, Hence, P : (1, 1) and Q : (1, 1) The region is symmetric about the x-axis. Area, A = 2(Area above x-axis) 2(Area ORPSO Area ORPO) For the regions ORPSO and ORPO, y varies from 0 to 1. A = 2 ⎡∫ ⎢⎣ 0

1

(

)

Fig. 6.41

2 − y 2 dy − ∫ y 2 dy ⎤ ⎥⎦ 0 1

1 1 ⎡y y y3 ⎤ 1⎞ ⎛1 ⎥ = 2⎜ + − ⎟ = 2⎢ − 2 − y 2 + sin −1 ⎝ 2 4 3⎠ 3 0⎥ 20 ⎢⎣ 2 ⎦ 1 = (3 + 2) 6

Engineering Mathematics

6.56

Exercise 6.6 1. Find the area enclosed by the curve a4 y 2 + b2 x 4 = a2 b2 x 2 . 4 ab 3 2. Prove that the area of a loop of the 16 . curve y 2 x 2 (4 x 2 ) is 3 3. Find the area of the loop of the curve y 2 (4 x ) x( x 2) 2 .

10. Find the area between the curve y2(a + x) = (a – x)3 and its asymptote. Ans. : 3p a 2

Ans. :

[ Ans. : 2(4

)]

4. Find the area in the first quadrant bounded by the curve b4y2 = (a2 – x2)3 and the co-ordinate axes. 3p a 4 Ans. : 16b 2 5. Find the area of the loop of the curve a+ x y2 = x2 . Also find the area a x between the curve and its asymptote. Ans. : a 2 2

p p , a2 2 a 2

6. Prove that area of the loop of the 8a 2 . curve 3ay 2 x( x a ) 2 is 15 3 7. Find the area enclosed by the curve y 2 ( x a )(b x), 0 < a, b. p ( a b) 2 4 8. Find the whole area of the curve a2 x2 y 2 = x2 2 . a + x2 Ans. :

Ans. : a 2 (p

2)

9. Show that the area of infinite region enclosed between the curve x3(1 – y) y = 1 and its asymptote is 2p.

11. Find the area of the loop of the curve y 2 x + ( x + a ) 2 ( x + 2a ) = 0. Ans. :

1 2 a (4 p ) 2

12. Find the area included between the y +8 x 2 and the x-axis. curve x

[ Ans. : 36] 13. Find the area between the parabola y 2 = 4 x and line 2 x 3 y 4 0. 1⎤ ⎡ ⎢⎣ Ans. : 3 ⎥⎦ 14. Find the area bounded by the curves x2 = 4ay and x2 = 4ay. 16 Ans. : a 2 3 15. Find the area enclosed by the curves 8a3 . x2 = 4ay and x 2 + 4 a 2 = y 2 Ans. : (3p 2)a 2 3 16. Show that the area enclosed by the curves xy2 = a2(a – x) and (a – x)y2 = a2x is (p – 2)a2. 17. Find the area between the ellipses x2 y2 x2 y2 + 2 = 1 and 2 + 2 = 1. 2 a b b a b Ans. : 4 ab tan 1 a 18. Find the area between the ellipses x 2 + 2 y 2 = a 2 and 2 x 2 + y 2 = a 2 . Ans. : 4 2a 2 cot 1 2

Integral Calculus

6.57

19. Find the area above the x-axis included between the curves y2 = x(2a – x) and y 2 = ax.

20. Find the area between the curve xy = 2 and the circle x 2 + y 2 = 5 in the first quadrant.

⎡ 2 ⎞⎤ 2 ⎛p ⎢ Ans. : a ⎜⎝ 4 − 3 ⎟⎠ ⎥ ⎦ ⎣

⎡ Ans. : ⎤ ⎢ ⎥ ⎢ 5 ⎛ sin −1 2 − sin −1 1 ⎞ − 2 log 2⎥ ⎟ ⎢⎣ 2 ⎜⎝ ⎥⎦ 5 5⎠

Parametric Form Example 1: Find the area enclosed between one arch of the cycloid x = a(p - sin p ), y = a(1 - cos p ) and its base. Solution:

a(q

x dx dq

sin q )

a(1 cos q )

For the region shown, x varies from 0 to 2 a. x = 0, q = 0 When x = 2pa, q = 2p A=

Area,

2p

y

0

=∫

2p

dx dq dq

Fig. 6.42

a(1 − cos q ) ⋅ a(1 − cos q )dq

0

2p

= a 2 ∫ (1 − 2 cos q + cos 2 q )dq 0

=a

2



2p

0

1 + cos 2q ⎞ ⎛ ⎜⎝1 − 2 cos q + ⎟⎠ dq 2

3q sin 2q =a − 2 sin q + 2 4

2p

2

= 3p a

0

2

Example 2: Find the area of the hypocycloid, x = a cos 3 q , y = b sin 3 q . Solution: x = a cos3 q dx dq

3a cos 2 q ( sin q )

For the region in the first quadrant, x varies from 0 to a. p When x = 0, q = 2 x = a, q = 0 Fig. 6.43

Engineering Mathematics

6.58

The region is symmetric in all the quadrants. Area, A = 4(Area in the first quadrant) 0

= 4 ∫p y 2

dx dq dq

0

= 4 ∫p b sin 3 q ⋅ 3a cos 2 q ( − sin q ) dq 2

p

= 12ab ∫ 2 sin 4 q cos 2 q dq = 12ab 0

3 ⋅1 ⋅1 p ⋅ 6⋅4⋅2 2

3p ab 8

=

1 t Example 3: Find the area bounded by the curve x = a cos t + a log tan 2 , 2 2 y = a sin t . 1 t Solution: x = a cos t + a log tan 2 2 2 dx a 2 1 2t = − a sin t + ⋅ sec t 2 dt 2 2 tan 2 a a sin t t t 2sin cos 2 2 a a (1 sin 2 t ) a sin t sin t sin t a cos 2 t = sin t For the region in the second quadrant, x varies from − ∞ to 0. When x → − ∞, t = 0 p x = 0, t = 2 The region is symmetric in all the quadrants. Area, A = 4(Area in the second quadrant) p

p dx ⎛ a ⎞ dt = 4 ∫ 2 a sin t ⎜ cos 2 t ⎟ dt 0 0 ⎝ ⎠ dt sin t p p ⎛ 1 + cos 2 t ⎞ = 4 a 2 ∫ 2 cos 2 t dt = 4 a 2 ∫ 2 ⎜ ⎟⎠ dt 0 0 ⎝ 2

= 4∫ 2 y

sin 2t = 2a t + 2 2

=pa

2

p 2 0

= 2a 2

p 2

Fig. 6.44

Integral Calculus

6.59

Example 4: Find the area bounded by the curve x = 3 + cos q , y = 4sin q . Solution: x = 3 + cos q dx dq

sin q

For the region in the first quadrant, x varies from 3 to 4. p x = 3, q = When 2 x = 4, q = 0 A = 4(Area BCD)

Area,

0

= 4 ∫p y 2

Fig. 6.45

0 dx dq = 4 ∫p 4 sin q ( − sin q ) dq dq 2

p

p

= 16 ∫ 2 sin 2 q dq = 8∫ 2 (1 − cos 2q ) dq 0

=8q −

0

sin 2q 2

p 2 0

⎛p ⎞ = 8⎜ ⎟ ⎝2⎠

= 4p Exercise 6.7 1. Find the area enclosed between one arch of the cycloid x = a(q + sin q ), y a(1 cos q ) and its base. Ans. : 3 a 2 2. Find the area of the astroid 2 3

2 3

2

3 x +y =a .

3 Ans. : p a 2 8 3. Find the area bounded by the ellipse

x = a cos t , y = b sin t.

[ Ans. : p ab] 4. Find the area bounded by the curve x = 2cosq – cos 2q – 1, y = 2sinq – sin 2q. [Ans. : 6p ] 5. Show that the area bounded by the sin 3 t cissoid x = a sin 2 t , y = a and cos t its asymptote is 3p a 2 . 4

Polar Form Example 1: Find the area bounded by the cardioid r = a(1 + cos q ) . Solution: The region is symmetric about the initial line q = 0. For the region above the initial line, q varies from 0 to p .

Engineering Mathematics

6.60

Area, A = 2(Area above the initial line) p 1 p 2 2 r dq a 2 (1 cos q ) 2 dq 0 2 0 p

a2

0 p

a2 a2

(1 2 cos q

0

3 q 2

1 2 cos q

cos 2 q ) dq 1 cos 2q dq 2 sin 2q 4

2sin q

p

3 2 pa 2

0

Fig. 6.46

Example 2: Find the area bounded by the lemniscate r 2 = a 2 cos 2q . Solution: The region is symmetric in all the quadrants. For the region in the first quadrant, q varies from 0 to p . 4 Area, A = 4(Area in the first quadrant ) p 1 p4 2 4 r dq 2 4 a 2 cos 2q dq 0 2 0 sin 2q = 2a 2 2

=a

p 4

Fig. 6.47

0

2

Example 3: Find the area bounded by the curve r = a cos 3p . Solution: For the region in the first quadrant,

varies from 0 to

p . 6

Area, A = 6(Area in the first quadrant) 1 p6 2 6 r dq 2 0 p

p

⎛ 1 + cos 6q ⎞ = 3∫ 6 a 2 cos 2 3q dq = 3a 2 ∫ 6 ⎜ ⎟⎠ dq 0 ⎝ 0 2 3a 2 q 2

sin 6q 6

p 6 0

1 = p a2 4

Fig. 6.48

Example 4: Find the area of the curve r = a sin 2p . Solution: The region is symmetric in all the quadrants. For the region in the first p quadrant, varies from 0 to . 2

Integral Calculus

Area,

6.61

A = 4(Area in the first quadrant) p 2

1 2

4

0 p 2

= 2a 2

0 p 2

= 2a 2

0

r 2 dq

sin 2 2q dq 1 cos 4q dq 2 sin 4q 4

2

a q

p 2

Fig. 6.49

0

1 = p a2 2 Example 5: Find the area of the smaller loop of the curve r = a ( 2 cos q - 1) . Solution: The region is symmetric about the initial line q = 0. For the region above the initial line, Area,

varies from 0 to

p . 4

A = 2(Area above the initial line) p 2 1 p4 2 4 2( 2 r dq a 2 cos q 1) dq 0 0 2

a2 a2

p 4 0 p 4 0

(2 cos 2 q (1 cos 2q

2 2 cos q 1) dq Fig. 6.50

2 2 cos q 1) dq

1 = a 2q + sin 2q − 2 2 sin q 2 2

p 4 0

1 ⎞ ⎛p 1 = a2 ⎜ + − 2 2 ⎟ ⎝2 2 2⎠

a2 = (p − 3) 2 Example 6: Find the area inside the cardioid r = 1 + cos q and outside the circle r = 1. Solution: The points of intersection of the cardioid r = 1 + cos q and circle r = 1 are obtained as, 1 + cos q = 1 cos q = 0 q =±

p 2

Fig. 6.51

Engineering Mathematics

6.62

p 2 The region is symmetric about the initial line p OACDO, q varies from 0 to . 2 Area, A = 2 (Area above the initial line) q =

Hence, at C,

= 0. In the regions OBCDO and

⎤ ⎡1 p 1 p = 2( Area OBCDO − Area OACDO ) = 2 ⎢ ∫ 2 (1 + cos q ) 2 dq − ∫ 2 (1) 2 dq ⎥ 0 0 ⎦ ⎣2 2 p

p

1 + cos 2q ⎛ = ∫ 2 (1 + 2 cos q + cos 2 q − 1) dq = ∫ 2 ⎜ 2 cos q + 0 0 ⎝ 2 1 sin 2q = q + 2 sin q + 2 4

⎞ ⎟⎠ dq

p 2 0

p = +2 4 Example 7: Find the area common to the circle r = 3cos q and the cardioid r = 1 + cos q . Solution: The points of intersection of the circle r = 3cos q and the cardioid r = 1 + cos q are obtained as, 3cos q = 1 + cos q cos q =

1 2

q =±

p 3

p 3 The region is symmetric about the initial line q =

Hence, at C,

from 0 to

and in the region, OECFO,

Fig. 6.52 = 0. In the region OACEO,

varies from

3 Area, A = 2(Area above the initial line)

p p to . 3 2

= 2( Area OACEO + Area OECFO ) ⎡1 p ⎤ 1 p = 2 ⎢ ∫ 3 (1 + cos q ) 2 dq + ∫p2 9 cos 2 q dq ⎥ 0 2 3 ⎣2 ⎦ p

p

⎛ 1 + cos 2q = ∫ 3 (1 + 2 cos q + cos 2 q ) dq + 9 ∫p2 ⎜ 0 ⎝ 2 3

⎞ ⎟⎠ dq

varies

Integral Calculus

p

1 + cos 2q ⎛ = ∫ 3 ⎜1 + 2 cos q + 0 ⎝ 2 p 3

sin 2q 3 = q + 2 sin q + 4 2

0

p

1 + cos 2q ⎞ 2 ⎛ ⎟⎠ dq + 9 ∫p ⎜⎝ 2 3

9 sin 2q + q+ 2 2

p 2 p 3

6.63

⎞ ⎟⎠ dq

⎛p 3 ⎞ 9 ⎛p p 3⎞ + ⎜ − − = ⎜ + 3+ ⎟ 8 ⎠ 2 ⎝ 2 3 4 ⎟⎠ ⎝2

p 9 3 9p 9 3 15p + + − = 2 8 12 8 12 5p = 4 =

Example 8: Find the area common to the circles r = a 2 and r = 2a cos p . Solution: The points of intersection of circles r = a 2 and r = 2a cos q are obtained as, a 2 = 2a cos q cos q =

1

q =±

2 p 4

p 4 The region is symmetric about the iniq =

Hence, at C,

tial line DO, Area,

= 0. In the region OACEO,

Fig. 6.53 varies from 0 to

varies from

to . 2 4 A = 2(Area above the initial line) = 2( Area OACEO + Area OECDO ) ⎡1 p ⎤ 1 p = 2 ⎢ ∫ 4 ( a 2 ) 2 dq + ∫p2 ( 2a cos q ) 2 dq ⎥ 0 2 2 ⎣ ⎦ 4 p

p

⎛ 1 + cos 2q ⎞ = ∫ 4 2a 2 dq + 4 a 2 ∫p2 ⎜ ⎟⎠ dq 0 ⎝ 2 4 = 2a q 2

=

p 4 0

siin 2q + 2a q + 2 2

p 2 ⎛p p 1 ⎞ a + 2a 2 ⎜ − − ⎟ ⎝ 2 4 2⎠ 2

= (p − 1)a 2

p 2 p 4

p and in the region OEC4

Engineering Mathematics

6.64

Example 9: Find the area common to the cardioid r = a(1 – cosp ) and r = a(1 + cosp ). Solution: The points of intersection of the cardioid r r = a(1 + cos q ) are obtained as,

a(1 cos q ) and

a(1 cos q ) a(1 cos q ) 2 cos q = 0 cos q = 0 q =±

p 2

Fig. 6.54 The region is symmetric in all the quadrants. In the region OABO,

varies from 0 to

p . 2

Area, A = 4(Area in the first quadrant) 4

2a 2 = 2a

p 2

1 2

2

0 p 2 0



p 2

0

a 2 (1 cos q ) 2 dq

(1 2 cos q

cos 2 q ) dq

1 + cos 2q ⎞ sin 2q ⎛ 2 3 ⎜⎝1 − 2 cos q + ⎟⎠ dq = 2a q − 2 sin q + 2 2 4

p 2 0

⎛ 3p ⎞ = 2a 2 ⎜ − 2⎟ ⎝ 4 ⎠ Example 10: Find the area inside the cardioid r = 2a(1 + cos q ) and outside 2a the parabola r = . 1 + cos q Solution: The points of intersection of the cardioid r = 2a(1 + cos q ) and parabola 2a r= are obtained as, 1 + cos q

Integral Calculus

6.65

2a 1 + cos q 1 + cos q = 1 cos q = 0 p q =± 2

2a(1 + cos q ) =

q =

Hence, at B,

p 2

The region is symmetric about the initial line q = 0. In the regions OADp BCO and OABCO, varies from 0 to . 2 Area, A = 2(Area above the initial line) = 2( Area OADBCO − Area OABCO )

Fig. 6.55

p ⎡1 p ⎤ 1 4a2 = 2 ⎢ ∫ 2 4 a 2 (1 + cos q ) 2 dq − ∫ 2 dq ⎥ 0 0 (1 + cos q ) 2 2 2 ⎣ ⎦ p

p

= 4 a 2 ∫ 2 (1 + 2 cos q + cos 2 q ) dq − 4 a 2 ∫ 2 0

0

p

(

1 2 cos 2 q 2

)

2

dq

p

1 + cos 2q ⎞ ⎛ 2 2 4 q = 4 a 2 ∫ 2 ⎜1 + 2 cos q + dq ⎟⎠ dq − a ∫0 sec 0 ⎝ 2 2 3 sin 2q = 4a q + 2 sin q + 2 4 2

p 2 0

p

q q q⎞ ⎛ − a 2 ∫ 2 ⎜ tan 2 sec 2 + sec 2 ⎟ dq 0 ⎝ 2 2 2⎠

2 q q ⎡ 3p ⎤ = 4a ⎢ + 2⎥ − a 2 tan 3 + 2 tan 3 2 2 ⎣ 4 ⎦ 2

p 2 0

⎡ [ f (q )]n +1 ⎤ n ⎢∵ ∫ [ f (q )] f ′ (q )dq = ⎥ n +1 ⎦ ⎣ ⎛2 ⎞ = a 2 (3p + 8) − a 2 ⎜ + 2 ⎟ ⎝3 ⎠ 16 ⎞ ⎛ = a 2 ⎜ 3p + ⎟ ⎝ 3⎠ Example 11: Find the area of the loop of the curve x 3 + y 3 = 3axy. Solution: Putting x = r cos q , y = r sin q , equation of the curve becomes, r 3 (cos3 q + sin 3 q ) = 3ar 2 sin q cos q 3a sin q cos q r= cos3 q + sin 3 q

Engineering Mathematics

6.66

p . 2 For the loop of the curve, varies from p 0 to . 2 1 p2 2 Area, A= r dq 2 0 1 p2 9a 2 sin 2 q cos 2 q dq = 2 0 (cos3 q + sin 3 q ) 2 r = 0 at q = 0 and q =

9a 2 2

=

p 2 0

tan 2 q sec 2 q dq (1 + tan 3 q ) 2

1 + tan 3 q = t

Putting

3tan 2 q sec 2 q dq = dt When

q = 0,

t =1

p q = , 2

t→

A=

3a 2 2 3a 2 2

1

Fig. 6.56

1 dt t2 1 t1

3a 2 2

Example 12: Find the area of the loop of the curve x 4 + 3 x 2 y 2 + 2 y 4 = a 2 xy. Solution: Putting x = r cos , y = r sin , the equation of the curve becomes, r 4 cos 4 q + 3r 4 cos 2 q sin 2 q + 2r 4 sin 4 q = a 2 r 2 cos q sin q r2 =

a 2 cos q sin q cos q + 3cos 2 q sin 2 q + 2sin 4 q 4

r = 0 at q = 0 and q =

p . 2

For the loop of the curve,

varies from 0 to

p . 2

Area, 1 p2 2 A= r dq 2 0 cos q sin q a 2 p2 = dq 4 0 2 cos q + 3cos 2 q sin 2 q + 2sin 4 q =

a2 2



p 2

0

tan q sec 2 q dq 1 + 3 tan 2 q + 2 tan 4 q

Fig. 6.57

[Dividing numerator and denomenator by cos4 q ]

Integral Calculus

6.67

Putting

tan2 q = t 2 tanq sec q dq = dt 2

When

t=0 q = 0, p q = , t 2 1 a2 ∞ a2 dt = A= 2 ∫ 4 4 0 1 + 3t + 2t =

a2 4





0



1



0

( 2t + 1) (t + 1)

dt

∞ 2 ⎞ a2 ⎛ 1 d t − + = − log (t + 1) + log ( 2t + 1) 0 ⎜⎝ ⎟⎠ t +1 2 t +1 4

⎡ 1 ∞ 2+ 2t + 1 a2 ⎢ a2 t = = log ⎢ log 1 t +1 0 4 ⎢ 4 1+ ⎢⎣ t 2 a = log 2 4

⎤ 2t + 1 ⎥ − log ⎥ t + 1 t =0 ⎥ ⎥⎦ t →∞

Example 13: Find the area of the loop of the curve (x2 + y2)(3ay – x2 – y2) = 4ay3. Solution: Putting x = r cos , y = r sin , the equation of the curve becomes, r 2 (3ar sin q

r2 )

4ar 3 sin 3 q

a(3sin q 4 sin 3 q ) = a sin 3q p r = 0 at q = 0 and q = . 3 p For the loop of the curve, q varies from 0 to . 3 r

Area,

A=

1 p3 2 1 p3 2 2 r d q = a sin 3q dq 2 ∫0 2 ∫0

a2 = 4



pa = 12

p 3

0

2 (1 − cos 6q ) dq = a q − sin 6q 4 6

p 3 0

2

Exercise 6.8 1. Find the area of the loop of the curve r = a sin 3 . Ans. :

p a2 12

2. Find the area of the limacon r = a + b cosq, a > b, p Ans. : (2a 2 + b 2 ) 2

Engineering Mathematics

6.68

3. Find the area of the limacon r = a cosq + b, a < b. Ans. :

p 2 ( a + 2b 2 ) 2

4. Show that the area of the loop of the curve r = aq cos q lying in the first a 2p 2 quadrant is (p 6). 96 5. Show that the area of a loop of the a2 curve r cosq = a cos 2q is (4 p ). 2 6. Show that area of a loop of the curve 1 r = 3 cos q + sin 3q is p . 3 7. Show that the area of the region enclosed between the two loops of the curve r = a(1 + 2 cos q ) is

(

)

a2 p + 3 3 . 8. Find the area of the ellipse l = 1 + e cos q. r Ans. :

Ans. : a2 e 2 b cot a (e 2g cot a 1) 4 cot a 12. Show that the area contained between the circle r = a and the curve r = a cos5q is equal to three fourths of the area of the circle. 13. Find the area common to two circles r = a cos and r = a(cos q + sin q ). Ans. :

a2 (p 1) 4

14. Find the area of the loop of the curve x 4 + y 4 = 2a 2 xy. 1 Ans. : p a 2 4

p l2 3

(1 e 2 ) 2 9. Show that the area of the loop of the curve r 2 cos = a 2 sin 3 lying in the first quadrant is

11. Find the area of the portion of the curve r = ae cot bounded by the radii vectors q = b and q = b + g where g > 2p .

1 2 e3 a log . 4 4

10. Show that the area bounded by the spiral r = ae m and two radii is proportional to the difference of the squares of these radii.

15. Show that the area of a loop of the 5 curve x 5 + y 5 = 5ax 2 y 2 is a 2 . 2 16. Prove that the area of the loop of the curve x 6 + y 6 = a 2 x 2 y 2 is

p a2 . 12

17. Find the area of the loop of the curve ( x + y )( x 2 + y 2 ) = 2axy. Ans. : a 2 1

p 4

.

6.5 VOLUME OF SOLID OF REVOLUTION A solid generated by revolving a plane area about a line in the plane is called a solid of revolution.

Integral Calculus

6.69

Volume of Solid of Revolution in Cartesian Form Let y = f ( x) be a curve and the area bounded by the curve, the x-axis and the two lines x = a and x = b be revolved about the x-axis. An elementary strip of width dx at point P ( x, y ) of the curve, generates elementary solid of volume y 2 dx, when revolved about the x-axis. Summing up the volumes of revolution of all such strips from x = a to x = b, the volume of solid of revolution is given by, Volume,

V=

b a

Fig. 6.58

p y 2 dx

Similarly, if the area bounded by the curve x = f (y), the y-axis and the two lines, y = c and y = d is revolved about the y-axis, then the volume of solid of revolution is given by, V=

d c

p x 2 dy

The volume of solid of revolution about any axis Fig. 6.59 can be obtained by calculating the length of the perpendicular from point P(x, y) on the axis of revolution. If the area bounded by the curve y = f ( x) is revolved about the line AB, then the volume of the solid of revolution is given by, V = ∫ p ( PM ) 2 d(AM ) with proper limits of integration.

Volume of Solid of Revolution in Parametric Form When the equation of the curve is given in parametric form x = f1 (t), y = f2 (t) with t1 t t2, the volume of the solid of revolution about the x-axis is given by, V=

t2 t1

y2

dx dt dt

Fig. 6.60

Similarly, the volume of the solid of revolution about the y-axis is given by, t2 dy V = ∫ x 2 dt t1 dt

Volume of Solid of Revolution in Polar Form For the curve r = f (q ), bounded

Fig. 6.61 between the radii vectors q = q1 and q = q2 the volume of the solid of revolution about the initial line is given by,

Engineering Mathematics

6.70

2

V

1 2

=

1

2 2 r r sin d 3 2 3 r sin d 3

Similarly, the volume of the solid of revolution about the line through the pole and perpendicular to the initial line is given by, 2 2 V r 2 r cos d 1 3 2 2 r 3 cos d = 1 3 x2 y2 Example 1: Find the volume generated by revolving the ellipse 2 + 2 = 1 a b about the x-axis. Solution: The volume is generated by revolving the upper-half of the ellipse about the x-axis. For the upper half of the ellipse, x varies from a to a. Due to symmetry about y-axis, considering the region in the first quadrant where x varies from 0 to a, a

Volume, V = 2 ∫ p y 2 dx 0

a⎛ x2 ⎞ = 2p b 2 ∫ ⎜1 − 2 ⎟ dx 0⎝ a ⎠

= 2p b 2 x −

x3 3a 2

a

Fig. 6.62

0

⎛ a ⎞ = 2p b 2 ⎜ a − 2 ⎟ 3a ⎠ ⎝ 3

4 p ab 2 3 Example 2: Find the volume generated by revolving the area bounded by the parabola y2 = 8x and its latus rectum about (i) x-axis, (ii) latus rectum, and (iii) y-axis. =

Solution: (i) The volume is generated by revolving the region about the x-axis. For the region above the x-axis, x varies from 0 to 2. Volume, V

2 0

p y 2 dx

p

2 0

8 x dx

8p

x2 2

2

16p 0

Fig. 6.63

Integral Calculus

6.71

(ii) The volume is generated by revolving the region about latus rectum. If P(x, y) is any point on the curve, its distance from latus rectum is 2 x. For the region shown, y varies from 4 to 4. Due to symmetry about x-axis, considering the region in the first quandrant where y varies from 0 to 4, 4

Volume, V = 2 p ( 2 − x ) 2 dy ∫ 0

2

4⎛ y2 ⎞ = 2p ∫ ⎜ 2 − ⎟ dy 0 ⎝ 8 ⎠ 4 4⎛ y2 y4 ⎞ y3 y5 = 2p ∫ ⎜ 4 − + dy = 2p 4 y − + 0 ⎝ 2 64 ⎟⎠ 6 320 0

=

256 p 15

(iii) The volume is generated by revolving the region about the y-axis. For the region shown in Fig 6.64, y varies from 4 to 4. Due to symmetry about x-axis, considering the region in the first quadrant where y varies from 0 to 4, Volume,

4

V = 2 ∫ p x 2 dy 0

= 2p ∫

4

0

=

y4 dy 64

p y5 32 5

4

⎡ y2 ⎤ ⎢∵ x = ⎥ 8⎦ ⎣

0

32 = p 5

Fig. 6.64

Example 3: Find the volume of the solid generated by revolving the region bounded by the curve y = log x and x = 2 about the x-axis. Solution: The volume of the solid is generated by revolving the region about x-axis. For the region shown, x varies from 1 to 2. Volume,

2

V = ∫ p y 2 dx 1

2

= p ∫ (log x ) 2 dx 1

2 2 1 ⎡ ⎤ = p ⎢ (log x ) 2 ⋅ x − ∫ 2 log x ⋅ ⋅ x dx ⎥ 1 1 x ⎣ ⎦ 2 = p ⎡ 2(log 2) 2 − 2 ∫ log x dx ⎤ 1 ⎦⎥ ⎣⎢ 2 1 ⎡ 2 ⎛ ⎞⎤ = 2p ⎢(log 2) 2 − ⎜ log x ⋅ x 1 − ∫ ⋅ x dx ⎟ ⎥ 1 x ⎝ ⎠⎦ ⎣

Fig. 6.65

2 = 2p ⎡(log 2) 2 − 2 log 2 + x 1 ⎤ = 2p [(log 2) 2 − 2 log 2 + 1] ⎣ ⎦ 2 = 2p (1 − log 2)

Engineering Mathematics

6.72

Example 4: Find the volume of the solid formed by the revolution of the curve xy2 = a2 (a - x) through four right angles about the y-axis. Solution: The volume of the solid is formed by revolving the region about the y-axis. For the region shown, y varies from to . Due to symmetry about the x-axis, considering the region in the first quadrant, where y varies from 0 to , Volume,



V = 2 ∫ p x 2 dy 0 ∞ a6 = 2p ∫ dy 0 ( y 2 + a2 )2 ⎡ a3 ⎤ ⎢∵ x = 2 ⎥ y + a2 ⎦ ⎣

Putting When

y = a tanq , dy = a sec2q dq y = 0, q q

y

=

2 p 2

V = 2p a 7

Fig. 6.66

0 p 2

2p a 7 a4

0

sec 2 q dq ( a 2 tan 2 q + a 2 ) 2 cos 2q dq = p a3

sin 2q =p a q + 2 3

p 2

= p a3

0

p 2 0

(1 + cos 2q ) dq

p 2

p2 3 = a 2 Example 5: Find the volume of the solid of revolution of the loop of the curve

y2 =

x 2 (a + x ) about x-axis. a-x

Solution: The volume of the solid of revolution is generated by revolving the upper half of the loop about the x-axis. For the loop, x varies from a to 0. Volume,

V= =p

0 a

p y 2 dx x 2 (a + x) dx a a x

0

Fig. 6.67

Integral Calculus

Putting When

a

6.73

x = t, dx = dt x = a, x = 0,

t = 2a t=a a (a t ) 2 (2a t ) p dt 2a t 2a 1 p [2a3 t ( a 2 4 a 2 ) t 2 (2a 2a) t 3 ]dt a t

V

=p∫

⎛ 2a 3 2 2⎞ ⎜⎝ t − 5a + 4 at − t ⎟⎠ dt

2a

a

= p 2a3 log t − 5a 2 t + 2at 2 −

t3 3

2a

a

8 1⎞ ⎛ = p a3 ⎜ 2 log 2 − 10 + 5 + 8 − 2 − + ⎟ ⎝ 3 3⎠ 2⎞ ⎛ = 2p a3 ⎜ log 2 − ⎟ ⎝ 3⎠ Example 6: Find the volume of the solid generated by revolving the curve

x3 about its asymptote. 2a - x Solution: The volume of the solid is generated by revolving the region about its asymptote. The asymptote is x = 2a. If P(x, y) is any point on the curve, its distance from the asymptote is 2a – x. For the region shown, y varies from – to . Due to symmetry about the x-axis, considering the region in the first quadrant where y varies from 0 to , y2 =

Volume,



V = 2 ∫ p ( 2 a − x ) 2 dy 0

3

y=

But,

x2 2a − x

(3a − x) x 2a − x dx ( 2a − x ) 2 y = 0, x=0 y x = 2a dy =

When

V Putting

2p

2a 0

(3a x ) x 2a x dx

x = 2a sin2q , dx = 4a sinq cosq dq

Fig. 6.68

Engineering Mathematics

6.74

x = 0, q = 0

When

x = 2a, q =

2

p 2 0

V = 2p ∫ (3a − 2a sin 2 q ) 2a cos q sin q ⋅ 4 a sin q cos q dq p

= 16p a3 ∫ 2 (3 − 2 sin 2 q ) sin 2 q cos 2 q dq 0

p ⎛ p ⎞ = 16p a3 ⎜⎝ 3∫ 2 sin 2 q cos 2 q dq − 2 ∫ 2 sin 4 q cos 2 q dq ⎟⎠ 0 0

3 ⋅1 ⋅1 p ⎞ ⎛ 1 ⋅1 p = 16p a3 ⎜ 3 ⋅ ⋅ − 2⋅ ⋅ ⎟ ⎝ 4⋅2 2 6⋅4⋅2 2 ⎠ = 2p 2 a3 Example 7: Find the volume generated by revolving the area cut off from the parabola 9y = 4 (9 - x2) by the line 4x + 3y = 12 about x-axis. Solution: The points of intersection of the parabola 9y = 4(9 x2) and the line 4x + 3y = 12 are obtained as, 3(12

4x) = 36

4x2

4x2 12x = 0 4x (x 3) = 0 x = 0, 3 and y = 4, 0 Hence, A: (3, 0) and B: (0, 4) Fig. 6.69 The volume is generated by revolving the region about the x-axis. For the region shown, x varies from 0 to 3. 3

Volume, V = ∫ p ( y12 − y22 ) dx

where y1 =

0

4 12 − 4 x (9 − x 2 ) and y2 = 9 3

2 2 3 ⎡⎧ 4 ⎫ ⎛ 12 − 4 x ⎞ ⎤ = p ∫ ⎢⎨ ( 9 − x 2 )⎬ − ⎜ ⎟ ⎥ dx 0 ⎭ ⎝ 3 ⎠ ⎥⎦ ⎢⎣⎩ 9 3 ⎡16 16 ⎤ = p ∫ ⎢ (81 − 18 x 2 + x 4 ) − (9 − 6 x + x 2 ) ⎥ dx 0 81 9 ⎣ ⎦

16p = 81 48 = p 5

16p x 5 27 ( x − x + 54 x ) d x = − 9 x 3 + 27 x 2 ∫0 81 5 3

4

3

2

0

Integral Calculus

6.75

Example 8: Find the volume of the solid formed by revolving the area enclosed between the curve 27ay2 = 4(x - 2a)3 and the parabola y2 = 4ax about x – axis. Solution: The points of intersection of parabola y2 = 4ax and the curve 27ay2 = 4(x 2a)3 are obtained as, 3

x

27a (4ax) = 4(x 2a)3 6ax 15a2x 8a3 = 0 (x + a)2 (x 8a) = 0 x = a, 8a 2

But x = a does not lie on the curve. Hence, x = 8a. The volume is generated by revolving the region about the x-axis. For the region shown, x varies from 0 to 8a for y1 and 2a to 8a for y2. 2 2 where y1 = 4 ax and y2 = 8a

4( x − 2a) 27a

Fig. 6.70

3

8a

2 2 Volume, V = ∫ p y1 dx − ∫ p y2 dx 2a

0

8a

= p ∫ 4 ax dx − p ∫ 0

2 8a

x = 4 ap 2

0

8a 2a

4( x − 2a)3 dx 27a

4p ( x − 2a) 4 − 27a 4

8a

2a

= 128p a3 − 48p a3 = 80p a3 Example 9: A quadrant of a circle of radius a, revolves about its chord. Find the volume of the spindle thus generated. Solution: Let the equation of the circle be x2 + y2 = a2 The volume of the spindle is generated by revolving the region about the chord AB. Equation of the chord AB is x + y = a. If P(x, y) is any point on the circle and M is the foot of the perpendicular from P on AB, then x+ y−a x+ y−a PM = = 2 2 2 (1) + (1) ( AM ) 2 = ( AP ) 2 − ( PM ) 2 ⎡1 ⎤ = [( x − a) 2 + ( y − 0) 2 ] − ⎢ ( x + y − a) 2 ⎥ ⎣2 ⎦

Fig. 6.71

Engineering Mathematics

6.76

1 = [2( x − a) 2 + 2 y 2 − ( x − a) 2 − y 2 − 2 y( x − a)] 2 1 2 = (x − a − y) 2 x−a− y AM = 2 1 1 ⎛ dy ⎞ d(AM ) = (dx − dy ) = ⎜1 − ⎟ dx 2 2 ⎝ dx ⎠ Since P lies on circle x2 + y2 = a2, dy 2x 2 y dx

dy dx

0,

x y

1 ⎛ x⎞ ⎜⎝1 + y ⎟⎠ dx 2 For the region shown, x varies from 0 to a. d (AM) =

Volume,

V =∫

When

( PM ) 2 d(AM ) =

0

= Putting

a

2

∫ 2



a

0

⎛ x+ y−a⎞ ⎜⎝ ⎟ 2 ⎠

2

1 ⎛ x⎞ ⎜1 + ⎟ dx 2 ⎝ y⎠

⎛ x⎞ ( x + y − a) 2 ⎜1 + ⎟ dx ⎝ y⎠

a

0

x = a cosq and y = a sinq , dx = a sinq dq p x= ,q= 2 x = a, q = 0

V= = =

=

2

⎛ a cos q ( a cos q + a sin q − a) 2 ⎜1 + ⎝ a sin q 2 0

∫ 2 a3

2

∫ 2

2

0

a3 2



2

0

⎞ ⎟⎠ ( − a sin q ) dq

(cosq + sin q − 1) 2 (cosq + sin q ) dq

( cos q + sin q + cos q sin 2 q + cos 2 q sin q − 1 − 2 cos q sin q ) dq

p a3

1 1 sin q − cos q + sin 3 q − cos3 q − q − sin 2 q 3 3 2

1⎞ p a3 ⎛ 1 p = ⎜⎝1 + − − 1 + 1 + ⎟⎠ 3 2 3 2

p 2 0

Integral Calculus

= =

6.77

p a3 ⎛ 5 p ⎞ ⎜ − ⎟ 2 ⎝3 2 ⎠ p a3 6 2

(10 − 3p )

Example 10: Find the volume of the solid formed by the revolution of a parabolic arc about the line joining the vertex to one extremity of the latus rectum. Solution: Let the equation of the parabola be y2 = 4ax. The volume is generated by revolving the region about the line OA. Equation of the line OA is y = 2x. If P(x, y) is any point on the parabola and M is the foot of the perpendicular from P on OA, then y − 2x y − 2x PM = = 2 2 5 (1) + ( −2)

Fig. 6.72

1 (OM 2 ) = (OP 2 ) − ( PM 2 ) = ( x 2 + y 2 ) − ( y − 2 x ) 2 5 = OM = d(OM ) =

5 x 2 + 5 y 2 − y 2 + 4 xy − 4 x 2 ( x + 2 y ) 2 = 5 5 x + 2y 5 1 5

(dx + 2dy ) =

dy ⎞ 1 ⎛ ⎜1 + 2 ⎟⎠ dx dx 5⎝

Since P lies on the parabola, y2 = 4ax dy 2y = 4a dx dy 2 a = dx y 1 ⎛ 4a ⎞ ⎜⎝1 + y ⎟⎠ dx 5

d(OM) =

For the region, x varies from 0 to a. Volume,

V =∫

a

=



( PM ) 2 d (OM )

0 a

0

1 1 ⎛ 4a ⎞ ( y − 2 x)2 ⎜1 + y ⎟⎠ dx 5 5⎝

Differential Equations

⎛ 2x

I.F. = e =e

10.59



∫ ⎝⎜ x2 −1 + cot x⎠⎟ dx log ⎡⎣( x 2 −1) sinn x ⎤⎦

= elog( x

2

−1) + log sin x

= ( x 2 − 1) sin x

Hence, solution is ( x 2 1) sin x y

( x 2 1) sin x cot x dx c

( x 2 1) cos x dx c

( x 2 1) sin x 2 x( cos x) 2( sin x) c y ( x 2 1) sin x

( x2

3) sin x 2 x cos x c

di

Example 14: Solve L dt + iR = sin t , constants.

t

0,

i (0) = 0 where R, v and L are

Solution: Rewriting the equation, di dt

R i L

sin t L

The equation is linear in i. R , Q L

P

e

I.F.

R dt L

sin t L R

eL

t

Solution is R

R

t

t

eL ⋅i = ∫ eL ⋅

sin t dt + c L

⎡ R t 1 ⎢ eL e ⋅i = ⎢ 2 L⎢R + ⎢⎣ L2 R t L

R

t

eL L = 2 R + 2 L2 i=

2

⎤ ⎛R ⎞⎥ ⎜ sin t − cos t ⎟ ⎥ + c ⎝L ⎠⎥ ⎥⎦

⎛R ⎞ ⎜ sin t − cos t ⎟ + c ⎝L ⎠

R − t 1 R sin t − L cos t ) + ce L 2 2 ( R + L 2

Given i(0) = 0 Putting i = 0, t = 0 in Eq. (1),

1 (0 − L) + ce0 R + 2 L2 L c= 2 R + 2 L2 0=

2

... (1)

Engineering Mathematics

10.60

Hence, solution is

i=

R − t 1 L L ( R sin t − L cos t ) + e R 2 + 2 L2 R 2 + 2 L2

dy + y tan x = sin 2 x , y(0) = 0, show that maximum value of y Example 15: If d x 1 is . 2 Solution: The equation is linear in y. P = tan x, Q = sin2x e

I.F.

tan x dx

elog sec x

sec x

Hence, solution is (sec x) y = ∫ sec x ⋅ sin 2 x dx + c = ∫ sec x ⋅ 2 sin x cos x dx + c = 2∫ sin x dx + c y sec x = −2 cos x + c y = −2 cos 2 x + c cos x

... (1)

Given y(0) = 0 Putting x = 0, y = 0 in Eq. (1),

0 = − 2 cos 0 + c cos 0 = − 2 + c c=2 Hence, solution is 2 cos 2 x 2 cos x

y For maximum or minimum value dy 0 dx 4 cos x( sin x) 2sin x 0 2sin x(2 cos x 1) 0

sin x = 0, x = 0 and 2 cos x 1 0, cos x x = 0 and x

3

0,

3

are the points of extreme values.

Now,

When x

1 ,x 2

d2 y dx 2

2

dy dx

2sin 2 x 2sin x

d2 y dx 2

4 cos 2 x 2 cos x

0, y is minimum at x = 0.

... (2)

Differential Equations

When x x

3

3

,

d2 y dx 2

4 cos

2 3

2 cos

1 2

4

3

10.61

1 2

2

3 0 , y is maximum at

.

Putting x

3

in Eq. (2), we get maximum value of y. 2 cos 2

ymax

3

2 cos

1 1 2

3

1 2

Exercise 10.6 Solve the following differential equations: 1. x 2

dy dx

3x 2

⎡⎣ Ans. : xe y − tan y + c ⎤⎦

2 xy 1 1⎤ c ⎡ ⎢⎣ Ans. : y = x 2 + x + x ⎥⎦

2. (2 y 3 x) dx x dy

dy dx

y cot x

2y

9.

e

2 x

y

dx x dy

10. x cos x

dy dx

[ Ans. :

x2

11. cos 2 x

dy dx

y

⎡⎣ Ans. : ( y + 1)( x − e ) = c ⎤⎦ e y sec 2 y dy

xy = sin x + c cos x ]

tan x

y4 )

dy dx

y ⎡ ⎤ 2x 2 ⎢ Ans. : y 2 = y + c ⎥ ⎣ ⎦

0 y

= 2 x + c ⎤⎦

⎡⎣ Ans. : y = tan x − 1 + ce − tan x ⎤⎦ 12. (2 x

y 6. ( y 1) dx [ x ( y 2)e ]dy

x

y ( x sin x cos x) 1

⎤ ⎡ x2 x2 = + c⎥ Ans. : ye ⎢ 2 ⎦ ⎣

7. dx x dy

1

⎡ Ans. : ye 2 ⎣

cos x

e

e x ( x 1) 2

⎡⎣ Ans. : y = (1 + x)(e x + c) ⎤⎦

( x 1) 4

2y

⎡ sin 2 x ⎤ + c⎥ ⎢ Ans. : y sin x = 2 ⎣ ⎦ 5. 1 dy x dx

y

x

⎡ ⎤ ⎛ x2 ⎞ 2 ⎢ Ans. : y = ⎜ + x + c⎟ ( x + 1) ⎥ ⎝ 2 ⎠ ⎣ ⎦ 4. dy dx

dy dx

0

⎡⎣ Ans. : x 2 y = x 3 + c ⎤⎦ 3. ( x 1)

8. (1 x)

13.

a2

x2

⎡ Ans. : ⎣⎢

dy dx

(x +

y

a2

x2

)

x

x 2 + a 2 y = a 2 x + c ⎤⎥ ⎦

Engineering Mathematics

10.62

14.

dy dx

1

(e x

sin x) x 3

x ey ⎡⎣ Ans. : xe − y = c + y ⎤⎦

15.

2x2

Ans. : y

dy dx

3 y x

x 3 , y (1) Ans. : y

18. If

4

Ans. : y

2 xy

sin x, y

3

0,

show that maximum value of y is

x 3 ( x 3) 19.

2 dy 16. (1 x ) dx y (0) 1

dy 2 y tan x dx

dy dx

y x

2 x(1 x 2 ),

1 . 8

log x, y (1) 1

x log x x 5 ⎤ ⎡ ⎢⎣ Ans. : y = 2 − 4 + 4 x ⎥⎦

(1 x 2 )[1 log(1 x 2 )

20.

dy 2 xy dx

dy 3 y x 4 (e x cos x) 2 x 2 , dx 3 y( ) e 2 2

xe

x2

⎡ ⎤ x2 x2 + c⎥ ⎢ Ans. : ye = 2 ⎣ ⎦

17. x

10.3.7 Non-linear Differential Equations Reducible to Linear Form Type 1: Bernoulli’s Equation The equation of the form dy dx

Py

Qy n

... (1)

where P and Q are functions of x or constants is a non-linear equation known as Bernoulli’s equation. This equation can be made linear using the following method: Dividing Eq. (1) by yn, 1 dy P Q ... (2) n y dx y n 1 Let

1 yn

1

v (1 n) dy y n dx 1 dy y n dx

dv dx dv (1 n) dx 1

Substituting in Eq. (2), dv Pv 1 n dx dv (1 n)Pv dx 1

Q Q

Differential Equations

10.63

The equation is linear in v and can be solved using the method of linear differential 1 , we get the solution of Eq. (1). equations. Finally, substituting v yn 1 Example 1: Solve

dy 2 y + = y2 x2 . dx x

Solution: The equation is in Bernoulli’s form. Dividing the equation by y2,

Let

1 y

v,

1 dy y 2 dx

1 dy y 2 dx

1 2 y x

x2

dv dx

2 v x

x2

dv dx

2 v x

... (1)

dv dx

Substituting in Eq. (1),

x2

... (2)

The equation is linear in v. 2 ,Q x

P I.F.

e

2 dx x

x2 e

2 log x

elog x

2

x

2

1 x2

Solution of Eq. (2) is 1 1 v = ∫ 2 ( − x 2 )dx + c 2 x x = ∫ −dx + c = − x + c v = − x 3 + cx 2 1 = − x 3 + cx 2 y

Hence,

Example 2: Solve

dy + y = y 2 (cos x - sin x ). dx

Solution: The equation is in Bernoulli’s form. Dividing the equation by y2, 1 dy y 2 dx

1 y

cos x sin x

... (1)

Engineering Mathematics

10.64

Let

1 y

v,

1 dy y 2 dx

dv dx

Substituting in Eq. (1), dv dx dv dx

v

cos x sin x

v

cos x sin x

... (2)

The equation is linear in v. P = −1, Q = − cos x + sin x I.F. e

dx

e

x

Solution of Eq. (2) is e − x ⋅ v = ∫ e − x ( − cos x + sin x) dx + c = − ∫ e − x cos x dx + ∫ e − x sin x dx + c ⎤ ⎤ ⎡ e− x ⎡ e− x ( − sin x − cos x) ⎥ + c = −⎢ ( − cos x + sin x) ⎥ + ⎢ 2 2 ⎦ ⎦ ⎣ ⎣ −x −x e v = − e sin x + c v = − sin x + ce x 1 = − sin x + ce x y

Hence,

Example 3: Solve xy(1 + xy 2 )

dy = 1. dx

Solution: Rewriting the equation, dx dy

xy

dx dy

xy x 2 y 3

x2 y3

The equation is in the Bernoulli’s form where x is a dependent variable. Dividing the equation by x2, 1 dx 1 y y3 2 x x dy Let

1 x

v,

1 dx x 2 dy

dv dy

... (1)

Differential Equations

10.65

Substituting in Eq. (1), dv + vy = y 3 dy

... (2)

The equation is linear in v. P

y,

I.F.

e

y3

Q

y2

y dy

e2

Solution of Eq. (2) is y2

y2

e 2 ⋅ v = ∫ e 2 y 3 dy + c Putting

y2 2

t , y dy

dt y2

e 2 ⋅ v = ∫ et ⋅ 2t dt + c = 2(et t − et ) + c y2 2

⎛ y2 ⎞ = 2e (t − 1) + c = 2e ⎜ − 1⎟ + c ⎝ 2 ⎠ t

v = y − 2 + ce 2



Hence,

Example 4: Solve y 4 dx = x



y2 2

−y 1 = y 2 − 2 + ce 2 x

3 4

2

- y 3 x dy .

Solution: Rewriting the equation, 3

dx dy

x 4 y4

x y

x 4 y4

x y

3

dx dy

The equation is in Bernoulli’s form where x is a dependent variable. Dividing the equation by x

3 4

x 7

Let x 4

v,

7 34 dx x 4 dy

dv dy

3 4

, dx dy

7

x4

1 y

1 y4

... (1)

Engineering Mathematics

10.66

Substituting in Eq. (1), 4 dv 7 dy

1 v y

1 y4

dv dy

7 v 4y

7 4 y4

... (2)

The equation is linear in v. P

I.F.

7 ,Q 4y e

7 dy 4y

7 4 y4 7

e4

log y

elog y

7 4

7

y4

Solution of Eq. (2) is 7

7

y4v = ∫ y4 ⋅

7 dy + c 4 y4

⎛ −5 ⎞ 7 −49 7 ⎜ 4y 4 ⎟ = ∫ y dy + c = +c 4 4 ⎜ −5 ⎟ ⎝ ⎠ 7 − 54 y +c 5 5 7 y 3 v = − + cy 4 5 7

y4v = −

Hence,

Example 5: Solve

7 5 7 y 3 x 4 = − + cy 4 5

dr r2 = r tan p . dp cos p

Solution: Rewriting the equation,

dr d

r tan

r2 cos

The equation is in Bernoulli’s form where r is a dependent variable. Dividing the equation by r2, 1 dr tan 1 r cos r2 d 1 = v, r 1 dr dv = d r2 d

Let −

... (1)

Differential Equations

Substituting in Eq. (1), dv v tan d

10.67

1 cos

... (2)

The equation is linear in v. P

1 cos

tan , Q e

I.F.

tan d

elog sec

sec

Solution of Eq. (2) is 1 ⎞ ⎛ sec ⋅ v = ∫ sec ⎜ − ⎟d + c ⎝ cos ⎠ = ∫ − sec 2 d + c = − tan + c ⎛ 1⎞ sec ⎜ − ⎟ = − tan + c ⎝ r⎠ sec = tan − c r

Hence,

Type 2: The equation of the form f ( y )

dy dx

Pf ( y )

Q

... (1)

where P and Q are functions of x or constants can be reduced to the linear form by dy dv putting f ( y ) = v, f ′ ( y ) = in Eq. (1) dx dx dv ... (2) Pv Q dx Equation (2) is linear in v and can be solved using the method of linear differential equation. Finally, substituting v f ( y ), we get the solution of Eq. (1). Example 1: Solve

dy + x sin 2 y = x 3 cos 2 y . dx

Solution: Dividing the equation by cos2y, 1 dy cos 2 y dx

2sin y cos y x cos 2 y dy sec 2 y 2 tan y x dx

Let tan y

v,

sec 2 y

dy dx

dv dx

x3 x3

... (1)

Engineering Mathematics

10.68

Substituting in Eq. (1), dv + 2vx = x 3 dx

... (2)

The equation is linear in v P = 2 x, Q = x 3 I.F. = e

2 x dx

= ex

2

Solution of Eq. (2) is e x v = ∫ e x ⋅ x 3 dx + c 2

Putting x 2 = t ,

2 x dx = dt , x dx =

2

dt 2 e x v = ∫ et t 2

2

(

)

1 dt + c = tet − et + c 2 2

1 x2 2 e ( x − 1) + c 2 2 1 v = ( x 2 − 1) + ce − x 2

ex v =

tan y =

Hence,

Example 2: Solve x

2 1 2 ( x − 1) + ce − x 2

dy + y log y = xye x . dx

Solution: Dividing the equation by xy,

Let log y = v,

1 dy log y + = ex y dx x

... (1)

dv v + = ex dx x

... (2)

1 dy dv = y dx dx

Substituting in Eq. (1),

The equation is linear in v. P=

1 , Q = ex x

I.F. = e

1 dx x

= elog x = x

Differential Equations

10.69

Solution of Eq. (2) is xv = ∫ xe x dx + c = xe x − e x + c xv = e x ( x − 1) + c x log y = e x ( x − 1) + c.

Hence,

dy + tan x tan y = cos x sec y . dx

Example 3: Solve

Solution: Dividing the equation by sec y, 1 dy + tan x sin y = cos x sec y dx cos y Let sin y

v, cos y

dy dx

dy + tan x sin y = cos x dx

... (1)

dv dx

Substituting in Eq. (1), dv dx

tan x v

cos x

... (2)

The equation is linear in v. P I.F.

tan x, Q e

tan x dx

cos x elog sec x

sec x

Solution of Eq. (2) is sec x ⋅ v = ∫ sec x ⋅ cos x dx + c = ∫ dx + c sec x ⋅ v = x + c sec x ⋅ sin y = x + c

Hence, Example 4: Solve

dy = e x - y (e x dx

e y ).

Solution: Dividing the equation by e–y, dy dx

e2 x

dy exe y dx

e2 x

ey ey Let e y

v, e y

dy dx

dv dx

exe y ... (1)

Engineering Mathematics

10.70

Substituting in Eq. (1), dv dx

exv

e2 x

... (2)

The equation is linear in v. P I.F.

ex , Q e

e x dx

e2 x ee

x

Solution of Eq. (2) is e e ⋅ v = ∫ e e ⋅ e 2 x dx + c x

x

Let ex = t, ex dx = dt e e ⋅ v = ∫ e t t dt + c = e t ⋅ t − e t + c x

x

= et (t − 1) + c = ee (e x − 1) + c v = e x − 1 + ce − e

e y = e x − 1 + ce − e

Hence, Example 5: Solve

e

e −2 x

x

dy y3 . = 2x dx e + y 2

Solution: Rewriting the equation,

Let e −2 x = v, − 2e −2 x

x

2x

dx dy

e2 x y3

dx dy e

2x

y

1 y

1 y3

... (1)

dx dv = , dy dy

dx 1 dv =− dy 2 dy

Substituting in Eq. (1), 1 dv v 2 dy y dv 2 v dy y

1 y3 2 y3

... (2)

The equation is linear in v. P

I.F.

2 ,Q y e

2 dy y

2 y3 e 2 log y

elog y

2

y2

Differential Equations

10.71

Solution of Eq. (2) is ⎛ 2⎞ y 2 ⋅ v = ∫ y 2 ⎜ − 3 ⎟ dy + c ⎝ y ⎠ 1 dy + c y = −2 log y + c

y 2 ⋅ v = −2∫

y 2 e −2 x = −2 log y + c

Hence, Example 6: Solve 2 xy

Solution: Let y 2

6

3 dy = ( y 2 + 6) + x 2 ( y 2 + 6)4 . dx

z, 2 y

dy dx

dz dx

Substituting in given equation, dz dx 1 dz z 4 dx 1 dz 1 z 4 dx xz 3 x

1 3 dz dv 1 dz v, 4 , 3 z z dx dx z 4 dx Substituting in Eq. (1), 1 dv 3 dx dv dx The equation is linear in v.

Let

3

x2 z4

z 1 xz 3

1

x2

1

x2

... (1)

1 dv 3 dx v x 3v x P

1

x2 1

3x 2 3 , x

I.F. e

3 dx x

... (2) 1

Q

3x 2 e3log x

elog x

3

x3

Solution Eq. (2) is 1 7 2 9 x 3 v = ∫ x 3 ⋅ 3 x 2 dx + c = 3∫ x 2 dx + c = 3 ⋅ x 2 + c 9

2 92 x + c, 3 9 ⎛ 1⎞ 2 x3 ⎜ − 3 ⎟ = x 2 + c ⎝ z ⎠ 3 x3v =

Engineering Mathematics

10.72

3

Hence,

⎛ x ⎞ 2 9 −⎜ 2 = x 2 + c. ⎟ 3 ⎝ y + 6⎠

Example 7: Solve

dz z z + log z = 2 (log z )2 . dx x x

Solution: Rewriting the equation, 1 dz z (log z ) 2 dx Let

−1 = v, log z

1 1 log z x

1 x2

... (1)

1 1 dz dv ⋅ = (log z ) 2 z dx dx

Substituting in Eq. (1), dv dx

v x

1 x2

... (2)

The equation is linear in v. 1 ,Q x

P I.F.

e

1 dx x

1 x2 e

log x

elog x

1

x

1 x

1

Solution of Eq. (2) is 1 1 1 x −2 ⋅ v = ∫ ⋅ 2 dx + c = ∫ x −3 dx + c = +c x x x −2 Hence,

1⎛ 1 ⎞ 1 − = − 2 +c ⎜ ⎟ x ⎝ log z ⎠ 2x 1 1 = −c x log z 2 x 2

Example 8: Solve x sin p dp + ( x 3 - 2 x 2 cos p + cos p )dx = 0. Solution: Rewriting the equation, x sin

d dx sin

x 3 (2 x 2 1) cos d dx

2x

1 cos x

0 x2

... (1)

Differential Equations

Let

cos

dv dx

d dx

v, sin

10.73

Substituting in Eq. (1), dv dx

2x

1 v x

x2

... (2)

The equation is linear in v. 1 P = 2x − , Q = − x2 x 2x

e

I.F.

1 dx x

ex

2

log x

2

ex e

log x

2

2

e x elog x

1

2

ex x

1

ex x

Solution of Eq. (2) is 2

2

ex ex ⋅v = ∫ ⋅ ( − x 2 ) dx + c x x = − ∫ e x ⋅ x dx + c 2

Let x 2

t , 2 x dx

dt 2

dt , x dx 2

ex dt et ⋅ v = − ∫ et ⋅ + c = − + c x 2 2 2

2

ex ex ⋅v = − +c x 2 2 x v = − + c x e− x 2 Hence,

2 x − cos = − + cx e − x 2

Example 9: Solve (sec x tan x tan y – e x ) dx + sec x sec 2 y dy = 0. Solution: Rewriting the equation, sec x sec 2 y

dy + sec x tan x tan y − e x = 0 dx sec 2 y

dy dx

tan x tan y

ex sec x

... (1)

Engineering Mathematics

10.74

Let tan y = v, sec 2 y

dy dv = dx dx

Substituting in Eq. (1), dv + (tan x)v = e x cos x dx

... (2)

The equation is linear in v. P = tan x, Q = e x cos x tan x dx

I.F. = e

= elog sec x = sec x

Solution of Eq. (2) is (sec x)v = ∫ sec x e x cos x dx + c = ∫ e x dx + c = e x + c sec x tan y = e x + c

Hence, Example 10: Solve

dy + x ( x + y ) = x 3 ( x + y )3 - 1. dx

Solution:

dy dx

x( x

y)

Let x

z, 1

dy dx

dz dy , dx dx

y

x3 ( x

y )3 1

... (1)

dz 1 dx

Substituting in Eq. (1), dz 1 xz x 3 z 3 1 dx dz + xz = x 3 z 3 dx Dividing the Eq. (2) by z3, 1 dz x + = x3 z 3 dx z 2 Let

... (2)

... (3)

1 2 dz dv 1 dz 1 dv = v, − 3 = , =− 2 dx z2 z dx dx z 3 dx

Substituting in Eq. (3), 1 dv xv 2 dx dv 2 xv dx

x3 2 x3

... (4)

Differential Equations

10.75

The equation is linear in v. 2 x3

2 x, Q

P e

I.F.

2 x dx

e

x2

Solution of Eq. (4) is e − x ⋅ v = ∫ e − x ( −2 x 3 )dx + c 2

Let x 2

t , 2 x dx

2

dt e − x ⋅ v = − ∫ te − t dt + c = te − t + e − t + c 2

2

= ( x 2 + 1)e − x + c v = ( x 2 + 1) + ce x

2

Substituting value of v, 2 1 = ( x 2 + 1) + ce x 2 z 2 1 = ( x 2 + 1) + ce x 2 ( x + y)

Hence,

Exercise 10.7 Solve the following differential equations: 1.

dy dx

x3 y 3

⎡ 2 3 2⎛2 ⎞ 2 2⎤ ⎢ Ans. : x = − 3 x y ⎜⎝ 3 + log x⎟⎠ + cy ⎥ ⎣ ⎦

xy

⎡ 1 2 x2 ⎤ ⎢ Ans. : y 2 = x + 1 + ce ⎥ ⎣ ⎦ 2 3 dy 4 y cos x 2. x y x dx ⎡⎣ Ans. : x 3 = y 3 (3 sin x − c) ⎤⎦ 2 2 3. x(3 x 2 y ) dx 2 y (1 x ) dy 0

dy 6. dx

y2ex

y

⎤ ⎡ e− x Ans. : − = x + c⎥ ⎢ y ⎣ ⎦ 7. x dy

y dx

⎡ 2 3 5⎤ ⎢ Ans. : y 5 = 5 x + cx ⎥ ⎣ ⎦

⎡⎣ Ans. : y 2 (1 + x 2 ) = − x 3 + c ⎤⎦ 2 2 4. y dx x(1 3 x y ) dy

0

1 − 2 2 ⎤ ⎡ 6 x y ⎣⎢ Ans. : y = ce ⎦⎥

5. x dy [ y

xy 3 (1 log x)]dx

0

x 3 y 6 dx

8. x

dy dx

y

y3 xn

1

⎡ n −1 2 n +1 ⎤ ⎢ Ans. : y 2 = cx − 2 x ⎥ ⎣ ⎦

Engineering Mathematics

10.76

2 2 9. xy (1 x y )

dy dx

1 ⎤ ⎡ 2 − y2 ⎢⎣ Ans. : x 2 = ce − y + 1⎥⎦ 2 3 3 10. x y dx ( x y 2) dy

19.

0

dy dx

2

yx cos y

dy 3y dx

ey x2

2 21. x

1 x

dy dx

y2 4 x 3 3x

dy dx

0

dy dx

(2 x tan 1 y

x 3 )(1 y 2 )

22.

dr d

⎡ Ans. : 2 tan −1 y = ( x 2 − 1) + ce − x ⎤ ⎣ ⎦ 2

15. tan y

dy dx

tan x

[ Ans. : 16. ( y e y

cos y cos 2 x

sec y sec x = sin x + c ]

e x )dx (1 e y )dy

0

⎡⎣ Ans. : y + e y = ( x + c)e − x ⎤⎦ 2 17. x cos y

dy dx

2 x sin y 1

⎡⎣ Ans. : 3 x sin y = cx 3 + 1⎤⎦

cosec y = 1 + cx ]

1 2

x4e x y3

r sin r2 cos 1 ⎤ ⎡ ⎢ Ans. : r = c cos + sin ⎥ ⎦ ⎣

23. cos x

0

1 tan y sin y x2

1 ⎤ ⎡ ⎢⎣ Ans. : cot y = 2 x + cx ⎥⎦

2 4 − ⎤ ⎡ 2 3 3 ⎢ Ans. : y x + 2 x = c ⎥ ⎦ ⎣

14.

2)3

sin 2 y (sin y cos y ) x

⎡⎣ Ans. : 2 xe − y = 1 + 2cx 2 ⎤⎦ 13. y

2) 2(3 y 2

1 ⎡ ⎞ 6⎤ 1 ⎛ x2 ⎢ Ans. : 2 = ⎜ e + c⎟ x ⎥ y ⎝ ⎠ ⎦⎥ ⎢⎣

y ⎤ ⎡ ⎢⎣ Ans. : x = y sin y + cos y + c ⎥⎦ dy 12. dx

1 tan y x

[ Ans. : 20. x

x

3 x(3 y 2

⎡⎣ Ans. : 4 x 9 = (3 y 2 + 2) 2 ( −3 x8 + c) ⎤⎦

3 ⎡ ⎤ 2 2 3 y ⎢ Ans. : x = + + ce ⎥ y 3 ⎢⎣ ⎥⎦

dx 11. y dy

dy dx

18. 4 x 2 y

1

dy dx

4 y sin x

4 y sec x

⎡ Ans. : y sec 2 x ⎤ ⎢ ⎥ 3 ⎛ tan x ⎞ ⎢ ⎥ = 2 ⎜ tan x + + c⎥ ⎢ ⎟ 3 ⎠ ⎝ ⎣ ⎦ 24. sin y

dy dx

cos x(2 cos y sin 2 x)

⎡ Ans. : 4 cos y = 2 sin 2 x − 2 sin x ⎤ ⎥ ⎢ + 1 − 4ce −2 sin x ⎦ ⎣ y dy 25. e dx 1

ex

⎤ ⎡ e2 x x+ y + c⎥ ⎢ Ans. : e = 2 ⎦ ⎣

Engineering Mathematics

6.78

= =

5

∫ 5

a



a

5 5

0

0

⎡ 2 4a 2 2 2 ⎤ ⎢ y − 4 xy + 4 x + y ( y − 4 xy + 4 x ) ⎥ dx ⎣ ⎦ ⎛ ax 2 ⎞ 2 8 a ax − 16 ax + 8 ax x ax x 4 8 4 − + + ⎜⎝ ⎟ dx ax ⎠ a

=

p 5 5

2

4a ⋅

=

2

x x x 4x + + 8a a − 16 a 3 2 3 2 2

0

p ⎛ 3 4 a 16 a 3⎞ ⎜⎝ 2a + 3 + 3 − 8a ⎟⎠ 5 5 3

=

3 2

3

2pa3 15 5

3

.

Exercise 6.9 1. Find the volume of the solid of revolution generated by revolving the plane area bounded by the curves y = x3, y = 0, and x = 2 about x-axis. 128p ⎤ ⎡ ⎢ Ans. : 7 ⎥ ⎣ ⎦ 2. Find the volume of the solid of revolution generated by revolving the region bounded by the curve ex sin x and x-axis about the x-axis. p 2p ⎡ ⎤ ⎢ Ans. : 8 (e − 1) ⎥ ⎣ ⎦ 3. Show that the volume generated by revolving the loop of the curve y2(a + x) = x2(3a x) about the x-axis is p a3(8 log 2 3). 4. Find the volume generated by revolving the curve x (y2 + a2) = a3 about its asymptote. 1 2 3⎤ ⎡ ⎢ Ans. : 2 p a ⎥ ⎣ ⎦ 5. The area bounded by the curve y = x(x 1) (2 x) and the x-axis between x = 1 and x = 2 is revolved about the x-axis. Prove that the volume 8p . generated is 105

6. The area enclosed by the parabolas x2 = 4ay and x2 = 4a(2a y) revolves about the line y = 2a. Find the volume of the solid so generated. 32 3 ⎤ ⎡ ⎢ Ans. : 3 p a ⎥ ⎣ ⎦ 7. The curve included between the curves y2 = 4ax and x2 = 4ay revolves about the x-axis. Find the volume of the solid of revolution. 96 3 ⎤ ⎡ ⎢ Ans. : 5 p a ⎥ ⎣ ⎦ 8. Find the volume generated by revolving the area between the curve y +8 = x − 2 and the x-axis about the x line x + 5 = 0. [ Ans. : 432p ] 9. Show that the volume of the spindle formed by the revolution of a parabolic arc about the line joining the vertex to one extremity of the latus 2p a3 , 4a being the latus rectum is 15 5 rectum of the parabola.

Integral Calculus

x2 y2 + = 1 is divided a2 b2 a into two parts by the line x = and 2 the smaller part is rotated through four right angles about this line. Find the volume generated. ⎡ ⎛ 3 3 p ⎞⎤ 2 − ⎟⎥ ⎢ Ans. : p a b ⎜ 3 ⎠ ⎥⎦ ⎝ 4 ⎢⎣ 11. The first quadrant of the ellipse x2 y2 + = 1 revolves about the line a2 b2 joining its extremities. Show that the volume of the solid generated is p a2b2 ⎛ 5 p ⎞ ⎜ − ⎟. a2 + b2 ⎝ 3 2 ⎠ 10. The ellipse

12. Find the volume of the solid obtained by revolving the area between the curves y2 = x3 and x2 = y3 about the x-axis. 5 ⎤ ⎡ ⎢ Ans. : 28 p ⎥ ⎣ ⎦

6.79

13. The parabola y2 = 8ax divides the circle x2 + y2 = 9a2 into two arcs the smaller of which is rotated about the x-axis. Show that the volume of solid 28 p a2 . generated is 3 14. Show that the volume obtained by revolving the area enclosed between the curves xy2 = a2(a x) and (a x) y2 = a2x about a p a3 is ( 4 − p ). 2 4 15. The loop of the curve 2ay2 = x(x a)2 revolves about the straight line y = a. Find the volume of the solid generated. ⎡ ⎤ 8 2 p a3 ⎥ ⎢ Ans. : 15 ⎣ ⎦ 16. The area bounded by the hyperbola xy = 4 and the line x + y = 5 is revolved about x-axis. Find the volume of the solid thus formed. [Ans. : 9p ] x=

Parametric Form Example 1: For the cycloid, x = a (p + sinp ), y = a (1 - cosp ), find the volume of the solid generated by the revolution of one arch about (i) the tangent at the vertex (i.e., x-axis), (ii) y-axis, and (iii) the base. Solution: (i)

x = a (q + sinq ) dx = a(1 + cos q ) dq

Fig. 6.73

Engineering Mathematics

6.80

The volume of the solid is generated by revolving one arch about the x-axis. For the region shown, x varies from ap to ap, hence q varies from p to p. Due to symmetry about the y-axis, considering the region in the first quadrant where q varies from 0 to p, π dx dθ V = 2∫ π y 2 Volume, 0 dθ π

= 2π ∫ a 2 (1 − cos θ ) 2 a(1 + cos θ ) dθ 0

θ θ ⋅ 2 cos 2 dθ 2 2 π θ θ = 16π a3 ∫ sin 4 cos 2 dθ 0 2 2 π

= 2π a3 ∫ 4 sin 4 0

q =t 2 d q = 2 dt q = 0, t = 0

Putting

When

q = p, t =

p 2

V = 32 a3 ∫ 2 sin 4 t cos 2 t dt 0

= 32 a3 ⋅

3 ⋅1 ⋅1 ⋅ 6⋅4⋅2 2

= 2 a3 (ii) y = a (1 cosq ) dy = a sin q dq The volume of the solid is generated by revolving one arch about the y-axis. For the region shown, y varies from 0 to 2a, hence q varies from 0 to p. Volume, π dy dθ V = ∫ π x2 0 dθ π

= π ∫ a 2 (θ + sin θ ) 2 a sin θ dθ 0

π

= π a3 ∫ (θ 2 sin θ + 2θ sin 2 θ + siin 3 θ ) dθ 0

π ⎡ 1 ⎤ = π a3 ∫ ⎢θ 2 sin θ + θ (1 − cos 2θ ) + (3 sin θ − sin 3θ ) ⎥ dθ 0 4 ⎣ ⎦ 2 ⎡ ⎛ ⎞⎤ 1 1 ⎞ ⎛θ = π a3 ( −θ 2 cos θ + 2θ sin θ + 2 cos θ ) + ⎢θ ⎜θ − sin 2θ ⎟ − 1 ⎜ + cos 2θ ⎟ ⎥ ⎝ ⎠ 2 2 4 ⎝ ⎠⎦ ⎣

1⎛ 1 ⎞ + ⎜ −3 cos θ + cos 3θ ⎟ ⎠ 4⎝ 3

π

0

Integral Calculus

⎡⎛ = a3 ⎢ ⎜ ⎣⎝ ⎛3 = a3 ⎜ ⎝2

1 3 1⎞ ⎛ 1 3 1 ⎞⎤ + − − ⎜ 2 − − + ⎟⎥ 4 4 12 ⎟⎠ ⎝ 4 4 12 ⎠ ⎦

2 2

−2+ 2

2



2

6.81



8⎞ − ⎟ 3⎠

(iii) The volume of the solid is generated by revolving one arch about the base which is the line joining the cusps. If P(x, y) is any point on the curve, its distance from the base = 2a y = 2a a (1 cosq ) = a(1 + cosq). For the region shown, x varies from ap to ap, hence q varies from p to p. Due to symmetry about the y-axis, considering the region in the first quadrant where q varies from 0 to p, π dx dθ V = 2 ∫ π ( 2a − y ) 2 Volume, 0 dθ π

= 2π ∫ a 2 (1 + cos θ ) 2 a(1 + cos θ ) dθ 0

π

= 2π a3 ∫ (1 + cos θ )3 dθ 0

3

π ⎛ θ⎞ = 2π a3 ∫ ⎜ 2 cos 2 ⎟ dθ 0 ⎝ 2⎠ π

= 16π a3 ∫ cos6 0

θ dθ 2

q = t, 2 d q = 2 dt

Putting

q = 0, t = 0 p q = p, t = 2

When

p

V = 16p a3 ∫ 2 2 cos6 t dt 0

p

= 32p a3 ∫ 2 cos6 t dt 0

5 ⋅ 3 ⋅1 p = 32p a3 ⋅ 6⋅4⋅2 2 = 5p 2 a3 Example 2: Find the volume of the solid generated by the revolution of the loop 1 3 t about the x-axis. of the curve x = t2, y = t 3 Solution:

x = t2 dx = 2t dt

Engineering Mathematics

6.82

The volume of the solid is generated by revolving the upper half of the loop of the curve about the x-axis. For the region shown, x varies from 0 to 3, hence t varies from 0 to 3 . Volume,

V =∫

3

y2

0

=2



=2

0

3

dx dt = dt



3

0

2

⎛ 1 3⎞ ⎜⎝ t − t ⎟⎠ 2t dt 3

⎛ 2 2 4 1 6⎞ ⎜⎝ t − t + t ⎟⎠ t dt = 2 3 9

t 4 2 t6 1 t8 − ⋅ + ⋅ 4 3 6 9 8

3

0



0

3

⎛ 3 2 5 1 7⎞ ⎜⎝ t − t + t ⎟⎠ dt 3 9

Fig. 6.74

9⎞ ⎛9 = 2 ⎜ −3+ ⎟ ⎝4 8⎠

3 = 4 Example 3: Find the volume of the solid of revolution generated by revolving 3 3 the curve x = 2t + 3, y = 4t2 - 9 about the x-axis for t = - to t = . 2 2 Solution:

x = 2t + 3 dx =2 dt

The volume of solid is generated by revolving the curve about the x-axis. For the required region, t varies from 3

Volume,

V = ∫ 23 −

= =4

y2

2



3 2 3 − 2



3 3 to . 2 2

dx dt dt

( 4t 2 − 9) 2 ( 2) dt

3 2

0

(16t 4 − 72t 2 + 81) dt

[∵ (4t 2 – 9)2 is an even function]

3 2 t5 t3 = 4p 16 − 72 + 81t 5 3 0

= 1296p Example 4: Prove that the volume of solid generated by revolving the cissoid x = 2a sin2t, y = 2a

sin 3 t about its asymptote is 2o 2a3. cos t

Integral Calculus

Solution:

6.83

sin 3 t cos t ⎛ 3 sin 2 t cos 2 t + sin 3 t sin t ⎞ dy = 2a ⎜ ⎟⎠ dt cos 2 t ⎝ y = 2a

2 2 ⎛ 3 cos 2 t + sin 2 t ⎞ 2 ⎛ 3 cos t + 1 − cos t ⎞ = 2a sin 2 t ⎜ = 2 a sin t ⎜⎝ ⎟⎠ ⎟⎠ cos 2 t cos 2 t ⎝

⎛ 2 cos 2 t + 1 ⎞ = 2a sin 2 t ⎜ ⎝ cos 2 t ⎟⎠ The volume of solid is generated by revolving the region about its asymptote, i.e., the line x = 2a. If P(x, y) is any point on the curve, its distance from the asymptote is 2a x = 2a 2a sin2 t = 2a cos2 t. For the region shown, y varies from to , hence t varies from to . Due to symmetry about the x-axis, consider2 2 ing the region in the first quadrant where t varies from 0 to 2

Fig. 6.75

,

Volume,

V = 2 ∫ 2 ( 2a − x ) 2 0

=2



2

0

dy dt dt

⎛ 2 cos 2 t + 1 ⎞ 4a 2 cos 4 t ⋅ 2a sin 2 t ⎜ ⎝ cos 2 t ⎟⎠

= 16 a3 ∫ 2 ( 2 cos 4 t sin 2 t + sin 2 t cos 2 t ) dt 0

1 1 ⎤ ⎡ 3 ⋅1 ⋅1 ⋅ + ⋅ ⋅ ⎥ = 16 = 16 a3 ⎢ 2 ⋅ ⎣ 6⋅4⋅2 2 4 2 2 ⎦ 2 3 =2 a

2

⎛6 ⎞ a3 ⎜ ⎟ ⎝ 48 ⎠

Exercise 6.10 1. For the cycloid x = a (q sinq ), y = a (1 cosq ), find the volume of the solid generated by the revolution of an arch about (i) x-axis, (ii) y-axis, and (iii) the tangent at the vertex. [Ans. : 5p 2a 3, 6p 3a 3, p 2a 3] 2. Find the volume formed by revolving one arch of the cycloid x = a(q + sinq ),

y = a(1 + cosq ) about x-axis. [Ans. : 5p 2a3] 3. Find the volume of the solid formed by revolving the tractrix t x = a cos t + a log tan , y = a sin t 2 about its asymptote. ⎡ 2p a3 ⎤ ⎢ Ans. : ⎥ 3 ⎦ ⎣

Engineering Mathematics

6.84

4. If the ellipse x = a cos q, y = b sin q is revolved about the line x = 2a, show that the volume of the solid generated is 4p 2a2b. 5. The area of the curve x = a cos3 q, y = a sin3 q lying between q = −

p and 2

p rotates about the x-axis. Find 2 the volume of solid so generated. q =

16 ⎡ 3⎤ ⎢ Ans. : 105 p a ⎥ ⎣ ⎦

Polar Form Example 1: Find the volume of the solid generated by the revolution about the initial line of the cardioid r = a (1 - cosp ). Solution: The volume of the solid is generated by revolving the upper half of the cardioid about the initial line q = 0. For the region above the initial line, q varies from 0 to p. p 2 Volume, V = ∫ p r 3 sin q dq 0 3 2p p 3 = a (1 − cos q )3 sin q dq 3 ∫0 Putting 1 cosq = t, sinq dq = dt When q = 0, t = 0 q = p, t = 2 V=

2p 3 2 3 2p 3 t 4 a ∫ t dt = a 0 3 3 4

Fig. 6.76

2

0

8 = p a3 . 3 π Example 2: Find the volume of revolution of a loop about the line θ = of the 2 curve r2 = a2 cos2p. Solution: The volume of solid is generated by revolving a loop of the curve about the line q =

p . For the loop of the curve, q varies from 2

p p to . Due to symmetry 4 4

about the line q = 0, considering the loop above the initial line where q varies from 0 to

π p 2 , Volume, V = 2 ∫ 4 π r 3 cos θ dθ 0 3 4

Integral Calculus

6.85

3 4π π4 3 2 2 θ a (cos ) cos θ dθ 3 ∫0 π 3 4 = π a3 ∫ 4 (1 − 2 sin 2 θ ) 2 cos θ dθ 0 3

=

2 sinq = sin t,

Putting

2 cosq dq = cos t dt When

θ =0 t=0 π π θ= , t= 4 2 V= = = =

4 3 2 1 a ∫ cos3 t ⋅ cos t dt 0 3 2 4

Fig. 6.77

a3 ∫ 2 cos 4 t dt 0

3 2 4 3 2 2 3 a 4 2

a3 ⋅

3 ⋅1 ⋅ 4⋅2 2

.

Example 3: Find the volume of the solid generated by revolving the curve r = a + b cosp , (a > b) about the initial line. Solution: The volume of solid is generated by revolving the upper half of the curve about the initial line. For the region above the initial line, q varies from 0 to p. Volume,

V =∫

p

0

2p 3 2p = 3 =

2 3 p r sin q dq 3



p

0

Fig. 6.78

( a + b cos q )3 sin q dq

⎛ 1⎞ p 3 ⎜⎝ − ⎟⎠ ∫0 ( a + b cos q ) ( −b sin q ) dq b p

⎡ −2p ( a + b cos q ) 4 [ f (q )]n +1 ⎤ n ∵ [ f ( q )] f ( q ) d q = ′ ⎢ ∫ ⎥ 3b 4 n +1 ⎦ ⎣ 0 p = − [( a − b) 4 − ( a + b) 4 ] 6b p = − [{( a − b) 2 + ( a + b) 2 }{( a − b) 2 − ( a + b) 2 }] 6b 4 = p a( a 2 + b 2 ) 3

=

Engineering Mathematics

6.86

Example 4: The arc of the cardioid r = a (1 + cosp ) included between θ = − and θ =

π 2

π π is rotated about the line θ = . Find the volume of the solid of 2 2

revolution. Solution: The volume of solid is generated p by revolving the curve about the line q = . 2 p For the region shown, q varies from to 2 p . Due to symmetry about the initial line, 2 considering the region above the initial line p where q varies from 0 to , 2 π

Volume,

V = 2∫ 2 0

Fig. 6.79

2 3 π a (1 + cos θ )3 cos θ dθ 3

4 3 π2 π a ∫ (cos 4 θ + 3 cos3 θ + 3 cos 2 θ + cos θ ) dθ 0 3 2 4 1 π ⎡ 3 ⋅1 π ⎤ ⋅ + 3 ⋅ + 3 ⋅ ⋅ + 1⎥ = π a3 ⎢ 3 3 2 2 ⎣4⋅2 2 ⎦ =

=

π a3 (5 π + 16). 4

Example 5: For the curve r2 = a2 cos 2p, prove that the volume of revolution of 2 3 a a loop about the tangent at the pole is . 8 Solution: The volume is generated by revolving the loop about the tangent at the p pole, i.e., the line q = . If P (r, q ) is 4 any point on the curve, its distance from the line q =

p 4

⎛p ⎞ is r sin ⎜ − q ⎟ , i.e., ⎝4 ⎠

1

r(cosq sin q ). 2 For the region shown, q varies from to

4

.

4

Fig. 6.80

Integral Calculus

6.87

p Volume, V = 4 2 p r 2 ⋅ 1 r (cosq − sin q ) dq ∫− p4 3 2

=

When

3 2



p 4 p − 4

3

(cos 2q ) 2 (cos q − sin q ) dq

=

p 3 3 ⎡ p ⎤ 2 p a3 ⎢ ∫ 4p (cos 2q ) 2 cos q dq − ∫ 4p (cos 2q ) 2 sin q dq ⎥ − 3 ⎣ −4 ⎦ 4

=

p 3 3 ⎡ p ⎤ 2 2 2 p a3 ⎢ 2 ∫ 4 (cos 2q ) 2 cos q dq − 0 ⎥ = p a3 ∫ 4 (cos 2q ) 2 cos q dq 0 0 3 3 ⎣ ⎦

= Putting

2p a3

p 3 2 2 p a3 ∫ 4 (1 − 2 sin 2 q ) 2 cos q dq 0 3

2 sinq = sin t, 2 cosq dq = cos t dt q = 0, t=0 p p q = , t= 4 2 V=

p 2 2 1 p a3 ∫ 2 cos3t cos t dt 0 3 2

p 2 2 3 ⋅1 p = p a3 ∫ 2 cos 4 t dt = p a3 ⋅ ⋅ 0 3 3 4⋅2 2 p2 3 a. = 8

Example 6: A solid is formed by rotating the area between two loops of the curve r = a (1 + 2cosp ) through four right angles. Find the volume generated. Solution: The volume of solid is generated by rotating the area between two loops of the curve through four right angles. For the 2p curve ACOBA, q varies from 0 to . For 3 4p . the curve BEOB, q varies from p to 3 At the pole, r=0 1 + 2 cos q = 0 1 cos q = 2 2 q = 3

Fig. 6.81

Engineering Mathematics

6.88

Volume,

⎛ Volume obtained by revolving ⎞ V =⎜ ⎟⎠ ⎝ the area ACOBA ⎛ Volume obtained by revolving ⎞ −⎜ ⎟⎠ ⎝ the area BEOB 2p 3

2 p a3 (1 + 2 cos q )3 sin q dq 3 4p 2 − ∫ 3 p a3 (1 + 2 cosq )3 sin q dq p 3 3 p a 23p =− (1 + 2 cos q )3 ( −2 sin q )dq 3 ∫0 p a3 43p (1 + 2 cos q )3 ( −2 sin q ) dq + 3 ∫p

=∫

0

p a3 (1 + 2 cos q ) 4 =− 3 4

2p 3

0

p a3 (1 + 2 cos q ) 4 + 3 4

4p 3

p

p a ⎛ 81 ⎞ p a ⎛ 1 ⎞ ⎜− ⎟ + ⎜− ⎟ 3 ⎝ 4⎠ 3 ⎝ 4⎠ 20 = p a3 . 3 3

3

=−

Example 7: Show that if the area lying within the cardioid r = 2a (1 + cosp ) and outside the parabola r (1 + cosp ) = 2a revolves about the initial line, the volume generated is 18o a3. Solution: The points of intersection of the cardioid r = 2a (1 + cosq ) and parabola r (1 + cosq ) = 2a are obtained as, 2a(1 + cos q ) =

2a 1 + cos q

1 + 2 cosq + cos2q = 1 cosq = 0, cosq p q =± 2 p q = Hence, at P, 2

Fig. 6.82 2 (does not exist)

The volume is generated by revolving the region about the initial line. For the regions p OBAPO and OBPO, q varies from 0 to . 2

Integral Calculus

6.89

Volume, V = ⎛ Volume obtained by revolving ⎞ ⎜⎝ ⎟⎠ the area OBAPO ⎛ Volume obtained by revolving ⎞ −⎜ ⎟⎠ ⎝ the area OBPO p

=∫2 0

2 p [2a(1 + cos q )]3 sin q dq 3 3

p 2 ⎛ 2a ⎞ −∫2 p ⎜ sin q dq 0 3 ⎝ 1 + cos q ⎟⎠ p 16 p a3 ∫ 2 (1 + cos q )3 ( − sin q ) dq 0 3 p 16 + p a3 ∫ 2 (1 + cos q ) −3 ( − sin q ) dq 0 3

=−

(1 + cos q ) 4 16 = − p a3 3 4

p 2

0

16 (1 + cos q ) −2 + p a3 3 −2

p 2

0

⎡ [ f (q )]n +1 ⎤ n ∵ d = [ ( q )] ( q ) q f f ′ ⎢ ∫ ⎥ n +1 ⎦ ⎣ 4 8 ⎛3⎞ = − p a3 ( −15) − p a3 ⎜ ⎟ ⎝4⎠ 3 3 = 18p a3 . Exercise 6.11 1. Find the volume of solid formed by revolving the curve r = a(1 + cosq ) about the initial line. 8 3⎤ ⎡ ⎢ Ans. : 3 a ⎥ ⎣ ⎦ 2. Find the volume of solid formed by revolving the curve r2 = a2 cos 2q about (i) the initial line, and (ii) the tangent at the pole. ⎡ ⎤ a3 ⎡ 3 log 2 + 1 − 2 ⎤ ⎥ ⎢ Ans. : (i) ⎣ ⎦ 6 2 ⎢ ⎥ ⎢ a3 ⎡ 3 ⎤⎥ (ii) log 2 + 1 − 1⎥ ⎥ ⎢ 12 ⎢⎣ 2 ⎦⎦ ⎣ 3. Prove that the volume generated by revolving the loop of the curve

(

)

(

)

r = a cos 3q lying between θ = − to

=

6

π 6

about the initial line.

⎡ 19 a 3 ⎤ ⎢ Ans. : ⎥ 960 ⎦ ⎣ 4. Find the volume generated by revolving the curve r = 2a cosq about the initial line. ⎡ 4 a3 ⎤ Ans. : ⎢ ⎥ 3 ⎦ ⎣ 5. Show that the volume of the solid generated by the revolution of the curve r = a + b secq about its 1 ⎞ ⎛2 asymptote is 2 a 2 ⎜ a + b ⎟ . 3 2 ⎠ ⎝

Engineering Mathematics

6.90

6.6 SURFACE OF SOLID OF REVOLUTION Let y = f (x) be a curve included between two lines x = a and x = b. Let P(x, y) be any point on the curve. When the chord PQ is revolved about the x-axis, a solid of revolution is generated. The elementary surface area d S is approximately equal to the circumference of the circle multiplied by the PQ. d S = 2p yPQ = 2p y d s The total surface area of the solid of revolution about x-axis is given by, Fig. 6.83 S = ∫ 2 y ds

Area of Surface of Solid of Revolution in Cartesian Form Area of surface generated by revolving the arc of the curve y = f (x) about the x-axis is given by, b b ds S = ∫ 2 y ds = ∫ 2 y dx a a dx 2

b ⎛ dy ⎞ = ∫ 2 y 1 + ⎜ ⎟ dx a ⎝ dx ⎠

Similarly, the area of the surface generated by revolving the arc of the curve x = f (y) about y-axis is given by, d

d

c

c

S = ∫ 2 x ds = ∫ 2 x

ds dy dy

2

=∫

⎛ dx ⎞ 2 x 1 + ⎜ ⎟ dy ⎝ dy ⎠

d c

Area of Surface of Solid of Revolution in Parametric Form When the equation of the curve is given in parametric form x = f1(t), y = f2(t) with t1 t t2, the area of surface of solid of revolution about the x-axis is given by, 2

2

t2 ds ⎛ dx ⎞ ⎛ dy ⎞ dt = ∫ 2 y ⎜ ⎟ + ⎜ ⎟ dt t1 t 1 dt ⎝ dt ⎠ ⎝ dt ⎠ Similarly, the area of surface of solid of revolution about the y-axis is given by, t2

S=∫ 2 y

t2

S=∫ 2 x t1

2

2

t2 ds ⎛ dx ⎞ ⎛ dy ⎞ dt = ∫ 2 x ⎜ ⎟ + ⎜ ⎟ dt t 1 dt ⎝ dt ⎠ ⎝ dt ⎠

Area of Surface of Solid of Revolution in Polar Form For the curve r = f (q ), bounded between the radii vectors at q = q 1 and q = q 2, the area of surface of the solid of revolution about the initial line q = 0 is given by,

Integral Calculus

6.91

2

θ2 ds ⎛ dr ⎞ dθ = ∫ 2π r sin θ r 2 + ⎜ ⎟ dθ θ 1 dθ ⎝ dθ ⎠

θ2

S = ∫ 2π y θ1

=

Similarly, the area of surface of solid of revolution about the line

2

is given by,

2

θ2 ds ⎛ dr ⎞ dθ = ∫ 2π r cos θ r 2 + ⎜ ⎟ dθ θ 1 dθ ⎝ dθ ⎠

θ2

S = ∫ 2π x θ1

Example 1: Find the area of the surface of revolution generated by revolving the curve x = y3 from y = 0 to y = 2. Solution:

x = y3 dx = 3y2 dy 2

⎛ dx ⎞ ds = 1 + ⎜ ⎟ = 1 + 9 y4 dy ⎝ dy ⎠

Fig. 6.84 The area of the surface is generated by revolving the region about the y-axis. For the region shown, y varies from 0 to 2. Surface area,

2

S=∫ 2 x 0

=

2 36



2

0

2 ds dy = ∫ 2 y 3 1 + 9 y 4 dy 0 dy 1

(1 + 9 y 4 ) 2 (36 y 3 )dy 2

= =

3 4 2

(1 + 9 y ) 3 18 2 27

(145

⎡ [ f ( y )]n +1 ⎤ n ⎢∵ ∫ [ f ( y )] f ′( y ) dy = ⎥ n +1 ⎦ ⎣ 0

)

145 − 1

Example 2: Find the area of the surface of revolution of the solid generated by revolving the ellipse Solution:

x2 y2 + = 1 about the x-axis. 16 4

x2 + 16 2x 2 y + 16 4

y2 =1 4 dy =0 dx x dy =− dx 4y

Engineering Mathematics

6.92

ds ⎛ dy ⎞ = 1+ ⎜ ⎟ dx ⎝ dx ⎠ = 1+

2

x2 16 y 2

x 2 + 16 y 2 4y The area of the surface of solid is generated by revolving the upper half of the ellipse about the x-axis. For the region above the x-axis, x varies Fig. 6.85 from 4 to 4. Due to symmetry about the y-axis, considering the region in the first quadrant where x varies from 0 to 4, =

4

S = 2∫ 2 y

Surface area,

0

4

ds dx dx x 2 + 16 y 2 dx = 4y



=

3∫

=

1 64 x 64 x 3 3 − x 2 + ⋅ sin −1 2 3 2 3 8

0

y



4

=4

0

x 2 + 64 − 4 x 2 dx

2

4

0

⎛ 8 ⎞ 2 ⎜ ⎟ − x dx 3 ⎝ ⎠ 4

0

⎡ 64 32 3⎤ − 16 + sin −1 3 ⎢2 ⎥ 3 3 2 ⎦ ⎣ 4 ⎞ ⎛ = 8 ⎜1 + ⎟ ⎝ 3 3⎠

=

Example 3: The part of the parabola y2 = 4ax cut off by the latus rectum revolves about the tangent at the vertex. Find the surface area of the revolution. Solution: The points of intersection of the parabola y2 = 4ax and its latus rectum x = a are obtained as, y2 = 4a . a = 4a2 y = ± 2a and x = a Hence, A: (a, 2a) and B: (a, 2a) Now,

y2 4a dx 2 y y = = dy 4 a 2 a x=

Fig. 6.86

Integral Calculus

⎛ dx ⎞ ds = 1+ ⎜ ⎟ dy ⎝ dy ⎠ = 1+

6.93

2

y2 4a 2

The surface area is generated by revolving the region about the tangent at the vertex i.e., y-axis. For the region shown, y varies from 2a to 2a. Due to symmetry about x-axis, considering the region in the first quadrant where y varies from 0 to 2a, Surface area,

S = 2∫ = 2∫

Putting When

2a 0 2a 0

2 x

ds dy dy

2 ⋅

y2 y2 1 + 2 dy 4a 4a

y = 2a tan q, dy = 2a sec2q dq y = 0, q = 0 y = 2a, q = S = 4π ∫

4 4a tan 2 θ 1 + tan 2 θ 2a sec 2 θ dθ 4a

π 4 0

2

= 8π a 2 ∫ = 8π a 2

π 4 0

π

tan 2 θ sec3 θ dθ = 8π a 2 ∫ 4 (sec5 θ − sec3 θ ) dθ 0

1 3 3 tan θ sec3 θ + tan θ sec θ + log(sec θ + tan θ ) 4 8 8 π 4 1 1 − tan θ sec θ − log(sec θ + tan θ ) 2 2 0

[Using reduction formula] 3 3 1 1 ⎡1 = 8 a2 ⎢ 2 2 + 2 + log( 2 + 1) − 2 − log 8 8 2 2 ⎣4 = a 2 ⎡3 2 − log ⎣

(

)

(

⎤ 2 +1 ⎥ ⎦

)

2 +1 ⎤ ⎦

Example 4: Find the surface area generated by revolving the loop of the curve 9ay2 = x(3a - x)2 about the x-axis. Solution: The points of intersection of the curve 9ay2 = x(3a obtained as,

Hence,

0 = x(3a x)2 x = 0, 3a, 3a and y = 0, 0, 0 A : (3a, 0)

x)2 and x-axis are

Engineering Mathematics

6.94

Now,

9ay 2 = x(3a − x) 2 dy 18ay = (3a − x) 2 − 2 x(3a − x) dx dy (3a − x) 2 − 2 x(3a − x) = 18ay dx (3a − x)(a − x) = 6ay 2

ds (3a − x) 2 (a − x) 2 ⎛ dy ⎞ = 1+ ⎜ ⎟ = 1+ dx 36a 2 y 2 ⎝ dx ⎠ =

36a 2 y 2 + (3a − x) 2 (a − x) 2 36a 2 y 2

Fig. 6.87

1 4ax(3a − x) 2 + (3a − x) 2 (a − x) 2 6ay 1 (3a − x) 2 (a + x) 2 = 6ay (3a − x)(a + x) = 6ay =

The surface area is generated by revolving the loop about the x-axis. For the loop, x varies from 0 to 3a. 3a 3a ds (3a − x)(a + x) dx Surface area, S = ∫ 2 y dx = 2 ∫ y ⋅ 0 0 dx 6ay =

3a

3a ∫0

=3 a

(3a 2 + 2ax − x 2 ) dx =

3a

3a 2 x + ax 2 −

x3 3

3a

0

2

Example 5: Find the area of the surface of revolution of a quadrant of a circular arc as obtained by revolving it about a tangent at one of its ends. Solution: Let x2 + y2 = a2 be the equation of the circle and let AC be the tangent at A. x2 + y 2 = a2 dy 2x + 2 y =0 dx dy x =− dx y 2

ds x2 ⎛ dy ⎞ = 1+ ⎜ ⎟ = 1+ 2 dx y ⎝ dx ⎠ =

x2 + y 2 a = y y2

Fig. 6.88

Integral Calculus

6.95

The surface area is generated by revolving the quadrant of circular arc APB about the line AC. If P(x, y) is any point on the circle, the distance of P from the tangent at A = a x. For the region shown, x varies from 0 to a. a

ds dx = 2 dx a−x dx a2 − x2

Surface area, S = ∫ 2 (a − x) 0

= 2 a∫

a

0



a

0

a ( a − x ) dx y

a⎛ x a = 2 a ∫ ⎜⎜ − 0 2 2 2 a − x2 ⎝ a −x

⎞ ⎟⎟ dx ⎠

−1 ⎤ a⎡ a 1 = 2 a∫ ⎢ + (a 2 − x) 2 (−2 x) ⎥ dx 0 2 2 2 ⎣ a −x ⎦

= 2 a a sin −1

x + a2 − x2 a

⎡ [ f ( x)]n +1 ⎤ n ⎢∵ ∫ [ f ( x)] f ′( x)dx = ⎥ n +1 ⎦ ⎣

a

0

⎛ ⎞ = 2 a⎜a − a⎟ ⎝ 2 ⎠ 2 = a ( − 2) Exercise 6.12 1. Find the surface area of the solid generated by revolving the arc of the parabola y2 = 4ax bounded by its latus rectum about the x-axis. ⎡ ⎤ 8a 2 Ans. : 2 2 −1 ⎥ ⎢ 3 ⎣ ⎦ 2. Find the area of the curved surface generated when one loop of the curve x2(a2 x2) = 8a2y2 is revolved about the x-axis.

(

)

⎡ a2 ⎤ Ans. : ⎢ ⎥ 4 ⎦ ⎣ 3. Prove that the surface area of the solid obtained by revolving the ellipse b2x2 + a2y2 = a2b2 about the x-axis ⎡ ⎤ ⎛1⎞ is 2 ab ⎢ 1 − e 2 + ⎜ ⎟ sin −1 e ⎥ , e e ⎝ ⎠ ⎣ ⎦ being the eccentricity of the ellipse.

4. Show that the surface area of the solid obtained by revolving the arc of the curve y = sin x from x = 0 to x = p about the x-axis is

(

)

⎡ 2 + log 2 + 1 ⎤ . ⎣ ⎦ 5. Show that the area of the surface formed by rotating the curve y2 = x3 from x = 0 to x = 4 about the y-axis is 128 1 + 125 10 . 1215 6. Find the area of the curved surface of the cup formed by the revolution of the smaller part of the parabola y2 = 4ax cut off by the line x = 3a about its axis. 2

(

)

56 2 ⎤ ⎡ ⎢ Ans. : 3 a ⎥ ⎦ ⎣

Engineering Mathematics

6.96

7. The arc of the parabola y2 = 4ax between its vertex and an extremity of its latus rectum revolves about its axis. Find the surface area traced out. 8 ⎡ 2⎤ ⎢ Ans. : 3 (2 2 − 1) a ⎥ ⎣ ⎦ 8. The arc of the curve a2y = x3 between x = 0 and x = a is revolved about the x-axis. Find the area of the surface so generated. ⎡ ⎤ a2 Ans. : 10 10 − 1 ⎥ ⎢ 27 ⎣ ⎦

(

)

9. Find the surface area of the solid formed by the revolution of the loop of the curve 3ay2 = x (x a)2 about the x-axis. ⎡ a2 ⎤ ⎢ Ans. : ⎥ 3 ⎦ ⎣ 10. Find the surface area of the solid generated by revolving the area bounded by the circle x2 + y2 = a2 about the line y = a. [Ans. : 4p 2a2]

Parametric Form Example 1: Prove that the surface generated by the revolution of the tractrix 1 t x = a cos t + a log tan 2 , y = a sin t about its asymptote is equal to the surface 2 2 of the radius a.

1 t x = a cos t + a log tan 2 2 2 dx 1 2 t 1 = − a sin t + a ⋅ ⋅ sec ⋅ t dt 2 2 tan 2 a = − a sin t + sin t 2 a cos t = sin t y = a sin t dy = a cos t dt

Solution:

2

ds ⎛ dx ⎞ ⎛ dy ⎞ = ⎜ ⎟ +⎜ ⎟ ⎝ dt ⎠ ⎝ dt ⎠ dt

Fig. 6.89

2

a 2 cos 4 t + a 2 cos 2 t sin 2 t a cos t = sin t =

The surface area is generated by revolving the tractrix about its asymptote, i.e., x-axis. For the region shown, x varies from −∞ to , hence t varies from 0 to p . Due to

Integral Calculus

6.97

symmetry about the y-axis, considering the region in the second quadrant where t varp ies from 0 to , 2 p ds S = 2 ∫ 2 2p y dt Surface area, 0 dt p a cos t = 4p ∫ 2 a sin t ⋅ dt 0 sin t p

= 4p a 2 ∫ 2 cos t dt 0

p

= 4p a 2 sin t 02 = 4p a 2 Example 2: Find the surface area of the solid generated by revolving the astroid 2

2

2

x 3 + y 3 = a 3 about the x-axis. Solution: The parametric equations of the astroid are x = a cos3 q , y = a sin 3 q dx = −3a cos 2q sinq , dq dy = 3a sin 2q cosq dq 2

ds ⎛ dx ⎞ ⎛ dy ⎞ = ⎜ + ⎝ dq ⎟⎠ ⎜⎝ dq ⎟⎠ dq

2

= 9a 2 cos 4 q sin 2 q + 9a 2 sin 4 q cos 2 q Fig. 6.90 = 3a siin q cos q The surface area is generated by revolving the upper half of the astroid about the x axis. For the region shown, x varies from a to a, hence q varies from p to 0. Due to symmetry about the y-axis, considering the region in the first quadrant, where p q varies from 0 to , 2 p ds dq Surface area, S = 2 ∫ 2 2p y 0 dq p

= 4p ∫ 2 a sin 3 q ⋅ 3a sin q cos q dq 0

p

= 12p a 2 ∫ 2 sin 4 q cos q dq 0

sin 5 q = 12p a 2 5 12 = p a2 5

p 2

0

⎡ [ f (q ) n +1 ⎤ n ⎥ ⎢∵ ∫ [ f (q ) f ′(q ) dq = n +1 ⎦ ⎣

Engineering Mathematics

6.98

Example 3: Find the surface area of the solid formed by revolving one arch of the cycloid x = a(p − sin p ), y = a(1 − cos p ) about the y-axis. Solution:

x = a(q − sin q )

dx = a(1 − cos q ) dq y = a(1 − cos q ) dy = a sin q dq 2

Fig. 6.91 2

ds ⎛ dx ⎞ ⎛ dy ⎞ = ⎜ + = a 2 (1 − cos q ) 2 + a 2 sin 2 q ⎝ dq ⎟⎠ ⎜⎝ dq ⎟⎠ dq = 2a 2 (1 − cosq q) q 2 The surface area is generated by revolving one arch of the curve about the y-axis. For the region shown, q varies from 0 to 2p . = 2a sin

Surface area, S = ∫

2p

0

2p x

ds dq dq

q dq 2 2p ⎛ q q q⎞ = 4p a 2 ∫ ⎜q sin − 2 sin 2 cos ⎟ dq 0 ⎝ 2 2 2⎠ = 2p ∫

2p

0

a (q − sin q )2a sin

⎡ = 4p a ⎢q ⎣ 2

q⎞ q ⎞⎤ 4 3 q ⎛ ⎛ ⎜⎝ −2 cos ⎟⎠ − 1 ⎜⎝ −4 sin ⎟⎠ ⎥ − sin 2 2 ⎦ 3 2

2p

0

⎡ [ f (q )]n +1 ⎤ n ⎢∵ ∫ [ f (q )] f ′(q )dq = ⎥ n +1 ⎦ ⎣ = 4p a2(4p ) = 16p 2 a2 Example 4: A circular arc of radius a revolves round its chord. Show that the surface of the spindle generated is 4π a 2(sin α α cos α ), where 2` is the angle subtended by the arc at the centre. Find the surface area if the circular arc is a quadrant of circle. Solution: Taking the centre of the circle as origin and radius as a, the equation of the circle is x2 + y2 + a2. The parametric equations of the circle are, x = a cos q , y = a sin q dx dy = − a sin q , = a cos q dq dq

Integral Calculus

2

6.99

2

ds ⎛ dx ⎞ ⎛ dy ⎞ = ⎜ + = a 2 sin 2 q + a 2 cos 2 q = a ⎝ dq ⎟⎠ ⎜⎝ dq ⎟⎠ dq The arc ACB is revolved about the chord AB. If P(x, y) is any point on the circle and M is the foot of perpendicular from P on AB, then PM = ON − OL = x − a cosa For the region shown, q varies from a to a . Due to symmetry about the x-axis, considering the region in the first quadrant where q varies from 0 to a, a

Surface area, S = 2 ∫ 2p ( PM ) 0

ds dq dq

Fig. 6.92

a

= 4p ∫ ( x − a cos a ) adq 0

a

= 4p a∫ ( a cos q − a cos a ) dq 0

= 4p a 2 sin q − q cos a

a 0

= 4p a (sin a − a cos a ) 2

p 4 1 p 1 ⎞ ⎛ S = 4p a 2 ⎜ − ⎝ 2 4 2 ⎟⎠

When circular arc is quadrant of a circle, a =

=

p a2 2

(4 − p )

Example 5: Show that the total surface area of the solid generated by the revo2 ⎛ ⎞ lution of an ellipse about its minor axis is 2 a 2 ⎜ 1 + 1 - e log 1 + e ⎟ , where a is ⎝ 2e 1- e⎠ the semi-major axis and e is the eccentricity.

Solution: The parametric equations of the ellipse are, x = a cos q ,

y = b sin q

dx = − a sin q , dq 2

dy = b cos q dq

ds ⎛ dx ⎞ ⎛ dy ⎞ = ⎜ + ⎝ dq ⎟⎠ ⎜⎝ dq ⎟⎠ dq

2

= a 2 sin 2 q + b 2 cos 2 q

Fig. 6.93

Engineering Mathematics

6.100

The surface area of the solid is generated by the revolution of the ellipse about its minor p p axis. For the region shown, y varies from b to b, hence q varies from to . 2 2 Due to symmetry about x-axis, considering the region in the first quadrant where q varies from 0 to

p , 2 p

S = 2 ∫ 2 2p x

Surface area,

0

p

ds dq dq

= 4p ∫ 2 a cos q a 2 sin 2 q + b 2 cos 2 q dq 0

p

= 4 ap ∫ 2 cos q a 2 sin 2 q + b 2 (1 − sin 2 q ) dq 0

p

= 4 ap ∫ 2 cos q b 2 + a 2 e 2 sin 2 q dq

where e =

0

Putting When

sinq = t cosq dq = d t q = 0, q = S = 4 ap ∫

1

0

1−

b2 a2

t=0

p , t =1 2

b 2 + a 2 e 2 t 2 dt = 4 ap ⋅ ae ∫

1

0

2

⎛b⎞ t 2 + ⎜ ⎟ dt ⎝ ae ⎠ 1

⎛ t 2 b2 b2 b2 ⎞ = 4p a e t + 2 2 + 2 2 log ⎜ t + t 2 + 2 2 ⎟ 2 ae 2a e ae ⎠ ⎝ 0 2

⎡ 1 b2 1 b2 b⎤ ⎛ ⎞ = 4p a 2e ⎢ a 2 e 2 + b 2 + 2 2 log ⎜1 + a 2 e 2 + b 2 ⎟ − 2 2 log ⎥ ⎝ ae ⎠ 2a e ae ⎦ 2a e ⎣ 2ae ⎡ 1 b2 a⎞ b2 b⎤ ⎛ = 4p a 2e ⎢ ⋅ a + 2 2 log ⎜1 + ⎟ − 2 2 log ⎥ ⎝ ⎠ ae ae ae 2 a e a e 2 2 ⎣ ⎦

⎡∵ b = a 1 − e 2 ⎤ ⎣ ⎦

⎡ b2 a(1 + e) ⎤ = 2p ⎢ a 2 + log ⎥ e b ⎦ ⎣ ⎛ 1+ e ⎞ b2 = 2p ⎜ a 2 + log ⎟ ⎝ 2e 1− e ⎠ ⎛ 1 − e2 1+ e ⎞ = 2p a 2 ⎜1 + log ⎟ ⎝ 2e 1− e ⎠

⎡∵ b = a 1 − e 2 ⎤ ⎣ ⎦

Integral Calculus

6.101

Exercise 6.13 1. Find the surface area of the reel formed by the revolution of the cycloid x = a(q + sinq ), y = a(1 – cosq ) about (i) the tangent at the vertex, (ii) y-axis, and (iii) base. 32 2 ⎡ Ans. : ⎤ pa (i) ⎢ ⎥ 3 ⎢ ⎥ 4 ⎞⎥ ⎢ 2⎛ p a p − (ii) 4 2 ⎜⎝ ⎟ ⎢ 3 ⎠⎥ ⎢ ⎥ 64 2 ⎢ ⎥ pa (iii) ⎥⎦ ⎣⎢ 3 2. Find the surface area of the solid generated by the revolution of the t3 loop of the curve x = t 2 , y t 3 about x-axis. [Ans. : 3p ] 3. Show that the area of the surface of the solid generated by revolving the curve x = a(u − tanh u ), y = a sechu, about the x-axis is equal to the area of the surface of a sphere of radius a. 4. Find the area of the surface of

revolution generated by revolving the cardioid x = 2cosq – cos 2q, y = 2sinq – sin 2q, about the x-axis. ⎡ Ans. : 128p ⎤ ⎢ 5 ⎥⎦ ⎣ 5. Find the area of the surface generated by revolving the curve x = 3t(t – 2), 3

y = 8t 2 with 0 y-axis.

t

1 about the [Ans. : 39p ]

6. Find the area of the surface generated by revolving the curve x = a cos 2 t , y = a sin 2 t about x-axis. ⎡ Ans. : 12p 2 ⎤ a ⎥ ⎢ 5 ⎣ ⎦ 7. Show that the ratio of the areas of the surface formed by revolving the arch of the cycloid x = a(q + sin q ), y = a(1 + cos q ) between two consecutive cusps about the x-axis to the area enclosed by the cycloid and 64 . x-axis is 9

Polar Form q

Example 1: The curve r = e 2 is revolved about the initial line. Prove that the area of surface of revolution traced out by the part between the points

θ = π is equal to

2

5 ( e + 1). q

Solution:

r = e2 d r 1 q2 = e dq 2 2

ds 1 ⎛ dr ⎞ = r2 + ⎜ = eq + eq ⎝ dq ⎟⎠ dq 4 =

5 q2 e . 2

= 0 and

Engineering Mathematics

6.102

The surface area is generated by revolving the curve about the initial line. For the region shown,q varies from 0 to p . p ds dq Surface area, S = ∫ 2p y 0 dq q p p ds 5 q2 dq = 2p ∫ e 2 sin q = ∫ 2p r sin q e dq 0 0 dq 2 p

p

= p 5 ∫ eq sin q dq = p 5 0

=

eq (sin q − cos q ) 2 0

p 5 (ep + 1). 2

Example 2: Find the area of the surface of the solid generated by revolving upper half of the cardioid r = a(1 − cos ) about the initial line. Solution: r = a(1 − cos q ) dr = a sin q dq 2

ds ⎛ dr ⎞ = r2 + ⎜ ⎝ dq ⎟⎠ dq

Fig. 6.94

= a 2 (1 − cos q ) 2 + a 2 sin 2 q 2

q⎞ q q⎞ ⎛ ⎛ = a 2 ⎜ 2 sin 2 ⎟ + a 2 ⎜ 2 sin cos ⎟ ⎝ ⎠ ⎝ 2 2 2⎠ = 4 a 2 sin 2 = 2a sin

2

q 2

q 2

The area of the surface of the solid is generated by revolving the upper half of the cardioid about the initial line q = 0. For the region shown, q varies from 0 to p . Surface area,

ds dq dq p ds = ∫ 2p r sin q dq 0 dq p

S = ∫ 2p y 0

q dq 2 p ⎛ q ⎞⎛ q q⎞ q = 4p a 2 ∫ ⎜ 2 sin 2 ⎟ ⎜ 2 sin cos ⎟ sin dq 0 ⎝ 2 2⎠ 2 2⎠⎝ p

= 4p a 2 ∫ (1 − cos q ) sin q sin 0

Integral Calculus

6.103

q q cos dq 2 2 p q 2 4 q 1 = 32p a ∫ sin ⋅ cos dq 0 2 2 2 p

= 16p a 2 ∫ sin 4 0

= 32p a 2

sin 5

q 2

p

⎡ [ f (q ) n +1 ] ⎤ n ]⎥ ⎢∵ ∫ [ f (q ) f ′(q ) dq = n +1 ⎦ ⎣

5 0

32 = p a2 5 Example 3: The arc of cardioid r = a(1 + cos ) included between θ = −

π to is rotated about 2 2

π . Show that the area of the surface 2 48 2 generated is a 2. 5 Solution: r = a(1 + cos q ) the line θ =

Fig. 6.95

dr = − a sin q dq 2

ds ⎛ dr ⎞ = r2 + ⎜ = a 2 (1 + cos q ) 2 + a 2 sin 2 q ⎝ dq ⎟⎠ dq 2

q⎞ q q⎞ ⎛ ⎛ = a 2 ⎜ 2 cos 2 ⎟ + a 2 ⎜ 2 sin cos ⎟ ⎝ ⎝ 2⎠ 2 2⎠ = 4 a 2 cos 2

2

q 2

q 2 p The area of the surface is generated by revolving the cardioid about the line q = . 2 p p . For the region shown, q varies from to Due to symmetry about the initial line 2 2 p considering the region in the first quadrant where q varies from 0 to , 2 p d s Surface area, dq S = 2 ∫ 2 2p y 0 dq p ds = 2 ∫ 2 2p r cos q dq 0 dq p q = 8p a 2 ∫ 2 (1 + cos q ) cos q cos dq 0 2 = 2a cos

Engineering Mathematics

6.104

p q ⎞⎛ q⎞ q ⎛ = 8p a 2 ∫ 2 ⎜ 2 cos 2 ⎟ ⎜1 − 2 sin 2 ⎟ cos dq 0 ⎝ 2⎠⎝ 2⎠ 2 p q ⎞⎛ q⎞ q ⎛ = 8p a 2 ∫ 2 2 ⎜1 − sin 2 ⎟ ⎜1 − 2 sin 2 ⎟ cos dq 0 ⎝ 2⎠⎝ 2⎠ 2

Putting

sin

q = t, 2

1 q cos dq = dt 2 2 q = 0,

When

t=0

p 1 q = , t= 2 2 1

1

S = 32 p a 2 ∫ 2 (1 − t 2 )(1 − 2t 2 )dt = 32 p a 2 ∫ 2 (1 − 3t 2 + 2t 4 ) dt 0

2 = 32 p a 2 t − t 3 + t 5 5 =

0

1 2

0

1 2 1 ⎤ ⎡ 1 = 32 p a 2 ⎢ − + ⋅ ⎥ 5 2 2 2 4 2⎦ ⎣

48 2 2 pa 5

Example 4: Find the surface area of the solid formed by the revolution of the loop about the tangent at the pole of the curve r 2 = a 2 cos 2 . r 2 = a 2 cos 2q dr 2r = −2a 2 sin 2q dq a 2 sin 2q dr sin 2q =− = −a dq a cos 2q cos 2q

Solution:

ds ⎛ dr ⎞ = r2 + ⎜ ⎝ dq ⎟⎠ dq

2

= a 2 cos 2q + a 2 = =

sin 2 2q cos 2q

a 2 cos 2 2q + a 2 sin 2 2q cos 2q a cos 2q

The surface area is formed by the revolution of the loop about the tangent at the pole i.e., q =

p p . If P ( r , q ) is any point on the curve, its distance from the line q = is 4 4

Integral Calculus

6.105

Fig. 6.96 1 p ⎛p ⎞ to r(cosq sin q ). For the region shown, q varies from r sin ⎜ − q ⎟ , i.e., 4 ⎝4 ⎠ 2 p . Due to symmetry about the initial line, considering the region in the first quadrant 4 p where q varies from 0 to , 4 p

Surface area,

S = 2 ∫ 4 2p

1

0

=

4p

∫ 2

p 4

0

2

r (cosq − sin q )

ds dq dq

a cos 2q (cos q − sin q )

a cos 2q

dq

p

= 2 2p a 2 ∫ 4 (cosq − sin q ) dq = 2 2p a 2 sin q + cos q 0

p 4 0

= 2 2 p a ( 2 − 1) 2

Exercise 6.14 1. Find the area of the surface of the solid generated by revolving the curve r 2 = a 2 cos 2q about the initial line. ⎡ 1 ⎞⎤ 2⎛ ⎢ Ans. : 4pa ⎜1 − ⎟⎥ ⎝ 2 ⎠⎦ ⎣ 2. Find the area of the surface of the solid generated by revolving the

curve r = 2a cosq about the initial line. ⎡⎣ Ans. : 4p a 2 ⎤⎦ 3. Find the area of the surface of the solid generated by revolving the curve r = 4 cos q about the initial line. [Ans. : 16p ]

Engineering Mathematics

6.106

FORMULAE m−2 n+2 + m − 1 ∫ sin x cos x dx n +1

Reduction Formulae sin x dx = – 1 cos x sinn –1 x n + n 1 ∫ sin n − 2 x dx n n 1 cos x dx = sin x cosn –1 x n n

(i)

(ii)

(iii)

+ n 1 ∫ cos n tan n 1 x n tan x dx = n 1

n−2

∫ tan

n−2



+ n − 1 ∫ sin m + 2 x cos n − 2 x dx m +1

x dx

– sec n x dx =

(v)

= − x dx

+ m+n+2 m +1

∫ cot

n−2

(b)

+ n −1 m+n (c)

(viii)

∫ sin

m

p 2 0

x cos n x dx

sin n x dx

= n 1 n n n if n is odd, = n 1 n n n if n is even

3 2

n 5 … 2, n 4 3

3 n 5 … 1 p , 2 n 4 2 2

p 2

sin m +1 x cos n −1 x m+n

cos n x dx = n 1 n 3 n n 2 n 5 … 2 , if n is odd n 4 3 n 1 n 3 n 5 … 1 p, = n 2 n 4 2 2 n

x cos n − 2 x dx

if n is even

(ix)

sin m x cos n x dx =

m+2

=

m −1

x sin x m+n m −1 + sin m − 2 x cos n x dx m+n ∫ cos

∫ sin

sin m +1 x cos n +1 x m +1 m + n + 2 + sin m x cos n + 2 x dx m +1 ∫

x dx

tan x sec n 2 x n 1

n +1

cos n +1 x sin m +1 x m +1

sin m x cos n x dx

(f)

+ n 2 ∫ sec n − 2 x dx n 1 cot x cosec n 2 x n (vi) cosec x dx = n 1 + n 2 ∫ cosec n − 2 x dx n 1 (vii) (a) sin m x cos n x dx = −

sin m x cos n x dx

(e)

cot x n 1

cot n x dx =

sin m +1 x cos n −1 x m +1

=

n 1

(iv)

sin m x cos n x dx

(d)

sin m x cos n x dx cos n +1 x sin m −1 x = − n +1

(x)

0

p 2 0

sin m x cos n x dx = m − 1

m+n

m−3 2 1 , … m+n−2 3 + n n +1

Integral Calculus

if m is odd and n may be odd or even m−3 m−5 … = m −1 m+n m+n−2 m+n−4 1 n 1 n 3 … 2, 2+n n n 2 3 if m is even and n is odd m−5 … = m −1 m − 3 m+n m+n−2 m+n−4 1 n 1 n 3 … 1 p , 2+n n n 2 2 2 if m is even and n is even Length of Arc (i) Cartesian form (a) s = ∫ (b) s = ∫

2

⎛ dy ⎞ 1 + ⎜ ⎟ dx ⎝ dx ⎠

b a

2

c

(ii) Parametric form s=∫

2

2 ⎛ dx ⎞ + ⎛ dy ⎞ dt ⎝ dt ⎠ ⎜⎝ dt ⎟⎠

t2

t1

(iii) Polar form (a) s = ∫

q2

q1

(b) s = ∫

r2 r1

2

dr ⎞ dq r2 + ⎛ ⎝ dq ⎠ 2

dq ⎞ dr 1+ r2 ⎛ ⎝ dr ⎠

Areas of Plane Curves (i) Cartesian form b

(a) A = (b) A =

a d c

y dx x dy

(ii) Parametric form t2 dx (a) A = y dt t1 dt t2 dy (b) A = x dt t1 dt (iii) Polar form 1 q2 2 A= r dq 2 q1

Volume of Solid of Revolution (i) Cartesian form b

p y 2 dx (revolution about x-axis)

(a) V =

a

d

(b) V =

c

p x 2 dy

(revolution about y-axis) (ii) Parametric form t2 dx dt (revolution p y2 (a) V = t1 dt about x-axis) t2 dy (b) V = p x2 dt (revolution t1 dt about y-axis) (iii) Polar form

⎛ dx ⎞ 1 + ⎜ ⎟ dy ⎝ dy ⎠

d

6.107

q2

2 3 p r sin q dq 3 (revolution about q = 0) q2 2 (b) V = p r 3 cos q dq q1 3 p revolution about q = 2 (a) V =

q1

(

)

Area of Surface of Solid of Revolution (i) Cartesian form (a) S =

2

⎛ dy ⎞ ∫a 2p y 1 + ⎜⎝ dx ⎟⎠ dx b

(revolution about x-axis) (b) S =



d

c

2

⎛ dx ⎞ 2p x 1 + ⎜ ⎟ dy ⎝ dy ⎠

(revolution about y-axis) (ii) Parametric form (a) S =



t2

t1

2

2 ⎛ dy ⎞ dx 2p y ⎛ ⎞ + ⎜ ⎟ dt ⎝ dt ⎠ ⎝ dt ⎠

(revolution about x-axis) (b) S =



t2

t1

2

2

⎛ dy ⎞ dx 2p x ⎛ ⎞ + ⎜ ⎟ dt ⎝ dt ⎠ ⎝ dt ⎠

(revolution about y-axis)

Engineering Mathematics

6.108

(iii) Polar form 2

(b) S =



q2

q1

q2 (a) S = ∫ 2p r sin q r 2 + ⎛ dr ⎞ dq ⎝ dq ⎠ q1

2

dr ⎞ dq 2p r cos q r 2 + ⎛ ⎝ dq ⎠

(revolution about q = p2 )

(revolution about q = 0) MULTIPLE CHOICE QUESTIONS

Choose the correct alternative in each of the following: p 2

5. The area under the curve y = 7

sin q dq is given by

1. The integral 0

12 16 (a) (b) 35 35 16p 8 (c) (d) 35 35 2. The area enclosed between the parabola y = x2 and the straight line y = x is 1 1 (a) (b) 8 6 1 1 (c) (d) 3 2 3. The sum of areas of all the loops of the curve r = 2 sin 3q is p 3 2 (a) 3 sin 3q dq 0 p 3

(b) 6 sin 2 3q dq 0 p 3

(c) 9 sin 2 3q dq 0 p 3

(d) 12 sin 2 3q dq 0

4. The area bounded by the x-axis, 8 y = 1 + 2 , ordinate at x = 2 and x = 4 is x (a) 2 (b) 4 (c) 8 (d) 1

for 0 1 (a) 4 (c) 1

x

x 1 x2

1 is 1 (b) 2 (d) 16

6. The area between the curves y = 1 x and y = 1 to the right of the line x +1 x = 1 is (a) log 3 (b) log 2 (c) 2 log 3 (d) 2 log 2 7. The area in the first quadrant under is the curve y = 2 1 ( x + 6 x + 10) (a) p (b) p – 2 tan–1 3 2 4 p p –1 (c) – tan 3 (d) + tan–1 3 2 2 8. The area under y = 2 1 2 for (x a ) x a + 1 is 1 (a) 1 log (a + 1) (b) log (a + 1) 2a a 1 (c) log (a + 1) (d) none of these 2 9. The area of the region bounded by y+8 the curve = x – 2 and the x x- axis is (a) 54 (b) 36 (c) 18 (d) 12

Integral Calculus

10. The area common to the curves y2 = x and x2 = y is equal to 2 (a) 1 (b) 3 1 (c) 0 (d) 3 11. The length of the arc of the curve y = log sec x from x = 0 to x = p is 3 (a) log (2 + 3 ) (b) log ( 2 + 3) (c) log ( 2 + 1) (d) log ( 3 + 1) ( x + 2) 3 2

12. The length of the curve y =

2 3

from x = 0 to x = 3 is (a) 10 (b) 12 (c) 3p (d) 6p 13. The whole length of the curve r = 2a sin q is equal to (a) p a (b) 2p a (c) 3p a (d) 4p a 14. Let An =

p 4 0

tan n x dx . Then the value

of A10 + A8 is 1 1 (a) (b) 8 9 1 1 (c) (d) 10 9 15. The figure bounded by graphs of y2 = 4x, y = 0 and x = 1 is rotated round the line x = 1. The volume of the resulting solid is (a) 16p (b) 15p 15 16 16 p 5 (c) (d) p 5 16 16. The area of the region in the first quadrant bounded by the y-axis and the curves y = sin x and y = cos x is (a)

2

(b)

2 +1

(c)

2 –1

(d) 2 2 – 1

6.109

17. The length of the arc of the curve 6xy = x4 + 3 from x = 1 to x = 2 is 13 17 (a) (b) 12 12 19 (c) (d) none of these 12 18. The arc of the sine curve y = sin x from x = 0 to x = p revolved about the x-axis. The area of the surface of the solid generated is (a) 2p [ 2 + log ( 2 + 1)] 2p 2 [ 2 + log ( 2 + 1)] 3 (c) p [ 2 + log ( 2 + 1)] 3 p2 (d) [ 2 + log ( 2 + 1)] 3 19. The volume of the solid generated by revolving the curve x = a cos t, y = b sin t about the x-axis is 4p ab 2 (a) 4p ab (b) 3 4p ab 2 (c) 4p ab (d) 3 20. The volume of solid obtained by revolving the area under y = e–2x about the x-axis is p p (b) (a) 2 4 (c) 2p (d) p (b)

21. The volume of the solid obtained by revolution of the loop of the curve a+x y2 = x2 about the x-axis is a−x (a) 2p a3 2 (b) 2p a3 log 2 − 3

(

(

(c) p a3 log 2 +

2 3

)

)

(d) p a3 22. The volume of solid generated, when the area of the ellipse

Engineering Mathematics

6.110

(a) 40p 5 36 p (c) 5

y2 x2 + = 1 (in the first quadrant) 9 4 is revolved about the y-axis is (a) 16p (b) 12p (c) 8p (d) 6p 23. The value of the integral p 2

1

3

0

8 (b) 4 45 45 8 p (c) (d) 4p 45 45 24. The area of the surface of the solid generated by revolving the curve r = 2a cos q about the initial line is (a) 2p a2 (b) 4p a2 2 (c) p a (d) 8p a2 25. The area of the surface of the solid generated by revolving the curve x = t3 – 3t, y = 3t2, 0 t 1 about the x-axis is (a)

2. 9. 16. 23.

(b) (b) (c) (a)

3. 10. 17. 24.

(b) (d) (b) (b)

x 5 dx is given by 1− x 2 0 4 p (a) (b) 4 15 15 8 p (c) (d) 8 15 15 27. The value of the integral 26. The integral

sin 2 x cos3 x dx is

Answers 1. (b) 8. (d) 15. (a) 22. (b)

(b) 24p 5 48 p (d) 5

4. 11. 18. 25.



x2

∫ (1 + x )

2 4



dx is

0

(a) p (b) p 32 16 2p (c) (d) 1 15 32 28. The area of the surface of the solid generated by the revolution of the line segment y = 2x from x = 0 to x = 2 about the x-axis is equal to

(b) (a) (a) (d)

(a) p 5

(b) 2p 5

(c) 4p 5

(d) 8p 5

5. 12. 19. 26.

(c) (b) (d) (d)

6. 13. 20. 27.

(b) (b) (b) (a)

7. 14. 21. 28.

(c) (b) (b) (d)

Gamma and Beta Functions Chapter

7

7.1 INTRODUCTION There are some special functions which have importance in mathematical analysis, functional analysis, physics or other applications. In this chapter, we will study two special functions, gamma and beta functions. The beta function is also called the Euler integral of the first kind. The gamma function is an extension of the factorial function to real and complex numbers and is also known as Euler integral of the second kind. Gamma function is a component in various probability distribution functions. It also appears in various areas such as asymptotic series, definite integration, number theory, etc.

7.2 GAMMA FUNCTION Gamma function is defined by the improper integral by n .





n = ∫ e − x x n −1dx, n > 0

Hence,

0

Alternate form of gamma function ∞

n = 2 ∫ e − x x 2 n −1dx 2

0

Proof: By definition, ∞

n = ∫ e − x x n −1dx 0

Let x = t2,



0

dx = 2t dt ∞

n = ∫ e − t . t 2 n − 2 . 2t dt 2

0



= 2 ∫ e − t ⋅ t 2 n −1 dt 2

0

Changing the variable t to x, ∞

n = 2 ∫ e − x . x 2 n −1dx 0

2

e − x x n −1dx, n > 0 and is denoted

7.2

Engineering Mathematics

7.3 PROPERTIES OF GAMMA FUNCTION (1) n + 1 = n n ∞

n + 1 = ∫ e − x x n dx

Proof:

0

Integrating by parts, n + 1 = −e − x x n

∞ 0



− ∫ (−e − x ) nx n −1dx 0



= n ∫ e − x x n−1dx 0

=n n

n +1 = n n

Hence,

This is known as recurrence or reduction formula for Gamma function. Note: (i)

n + 1 = n!

if n is a positive integer

(ii)

n +1 = n n

if n is a positive real number

(iii)

n=

(iv)

n 1− n =

n +1 n

if n is a negative fraction

sin n

(2) 1 =

2

Proof: By alternate form of Gamma function, ⎛ 1⎞

2.⎜ ⎟ − 1 ∞ 2 1 = 2∫ e − x x ⎝ 2 ⎠ dx = 2 0 2





0

2

e − x dx

∞ ∞ 2 2 1. 1 = 2 ∫ e − x dx . 2 ∫ e − y dy 0 0 2 2

= 4∫



0





0

e−( x

2

+ y2 )

Changing to polar coordinates, x = r cosq, Limits of x Limits of y

x=0 y=0

dx dy y = r sinq

dx dy = r dr dq to to

x y

This shows that the region of integration is the first quadrant.

Gamma and Beta Functions

7.3

Draw an elementary radius vector in the region which starts from the pole and extends up to . Limits of r

r=0

to

r

Limits of q

q=0

to

θ=

π 2

π 2 1 1 2 ∞ ⋅ = 4 ∫ ∫ e − r . r d r dθ 0 2 2 0

π 2



⎛ 1⎞ 2 = 4 ∫ dθ . ∫ ⎜ − ⎟ e− r (− 2r ) d r ⎝ 2⎠ 0 0 =

4 θ −2

= −2 .

π 2 0

2

e− r

2



⎡∵ e f ( r ) . f ′ (r )dr = e f ( r ) ⎤ ⎣ ∫ ⎦

0

( 0 − 1)

=

1 = 2 Example 1: Find the value of - 5 . 2 n +1 Solution: n= n 5 − = 2



5 +1 2 3 2 =− − 5 5 2 − 2

2 =− . 5

4 . = 15

=−

Fig. 7.1





8 15

3 +1 4 1 2 = − 3 15 2 − 2

1 +1 8 1 2 =− 1 15 2 − 2

7.4

Engineering Mathematics

Example 2: Given

n=

Solution:

12 − = 5

8 12 = 0.8935 , find the value of - . 5 5

n +1 n 2 12 7 − +1 +1 − +1 25 5 5 5 . . 5 =− = 7 2 12 12 84 − − − 5 5 5



3 +1 625 8 125 . 5 625 =− =− =− (0.8935) = 1.108 3 504 5 504 168 5 Ç

Example 3: Evaluate

3

e - x dx .

0

1 1 −2 Solution: Let x3 = t, x = t 3 , dx = t 3 dt 3 When x = 0, t=0

x





0

,

t

1 ∞ −1 3 1 −2 1 1 1 ∞ e − x dx = ∫ e − t . t 3 dt = ∫ e − t t 3 dt = 0 3 3 3 3 0 1

Ç

x

e-

Example 4: Evaluate

x 4 dx .

0

Solution: Let

x = t , x = t2, dx = 2t dt

When

x = 0,

t=0

x





0

e

− x

,

1 4

t



1

x dx = ∫ e ( t ) 4 2t dt −t

2

0

3



5 3 1 1 3 =2. . = p 2 2 2 2 2

= 2 ∫ e − t t 2 dt = 2 0

Example 5: Evaluate





2

( x 2 + 4)e -2 x dx .

0 1



1

t 2 t 2 1 1 − 12 Solution: Let 2x = t, x = ⎛⎜ ⎞⎟ , dx = ⋅ t dt = dt ⎝2⎠ 2 2 2 2 2

Gamma and Beta Functions x = 0,

When

t=0

x





0

( x + 4) e 2

,

=

=



Example 6: Evaluate Solution: Let

Ç



0

=

xne−

ax

1 4

∫ 2



0

1

e − t t 2 dt +



xn e−

ax

0

1

2

+

8 2

,

e−t t



1 2

dt

0

=

2

17 8 2



Example 7: Evaluate



t n

⎛t ⎞ 2t dx = ∫ ⎜ ⎟ e − t ⋅ dt 0 ⎝ a⎠ a 2



=

2 a



n +1

2 a n +1

2

x e − x dx .

0





e − t t 2 n +1dt

0



2n + 2 e− x

2

x

0

dx .

1

Solution: Let

x2 = t, x = t 2 , dx =

When

x = 0,

x e − x dx . ∫



0

e

− x2

1 − 12 t dt 2

t=0 ,

x

0



dx .

=



∫ 2

1 3 2 1 1 .1 1 2 1 + = + 4 2 2 2 2 4 2 2 2 2 2

x

2

2

t2 2t , dx = dt a a x = 0, t=0

When



1

0

ax = t , x =



t

⎛t ⎞ −t t 2 dt ⎜⎝ + 4⎟⎠ e . 2 2 2



dx = ∫

−2 x 2

7.5

t

∞ e−t 1 −1 1 −1 dx = ∫ t e . t 2 dt . ∫ 1 . t 2 dt 0 0 2 2 x t4 1 ∞ 4 −t

=

1 ∞ − t − 14 . ∞ − t − 34 e t dt ∫ e t dt 0 4 ∫0

7.6

Engineering Mathematics

Ç

Example 8: Evaluate

=

1 3. 1 1 1 1 = 1− 4 4 4 4 4 4

=

1 ⋅ 4 sin

e- x

e− x

x

0

Ç

6

x 4 e - x dx .

0

x ∞

2 2

4

dx .

x = 0,

When



2 =

1 1 −2 x3 = t, x = t 3 , dx = t 3 dt 3

Solution: Let

3

1. 4

3

x

0

=



t=0 ,

dx . ∫ x 4 e − x dx = ∫ 6

0



0

t e − t . 1 − 32 . ∞ 34 − t 2 . 1 − 32 t dt ∫ t e t dt 1 0 3 3 6 t

=

1 ∞ − t − 56 . ∞ − t 2 32 e t dt ∫ e t d t 0 9 ∫0

=

∞ 2 2 ⎜ ⎟ −1 1 1 1 ⋅ ⋅ 2 ∫ e − t t ⎝ 6 ⎠ dt 0 9 6 2

=

1 1 1 5 ⋅ 9 6 2 6

⎛ 5⎞

⎡∵ 2 ∞ e − x2 x 2 n −1dx = n ⎤ ∫0 ⎢⎣ ⎥⎦

1 1 1 1 p 1 = = ⋅ p 18 6 6 18 9 sin 6 1

Example 9: Evaluate

(log x )5 dx .

0

Solution: Let log x =

t, x = e t, dx =

x = 0, x = 1,

When



1

0

t t=0

0

(log x)5 dx = ∫ ( −t )5 ( −e − t ) dt ∞



= − ∫ e − t t 5 dt 0

= − 6 = −120

e t dt

Gamma and Beta Functions

Example 10: Evaluate



0

4

⎛ 1⎞ x 3 log ⎜ ⎟ dx . ⎝ x⎠ 4

1 ⎛1⎞ ⎛1⎞ 3 ∫0 x log ⎜⎝ x ⎟⎠ dx = ∫0 x . 4 log ⎜⎝ x ⎟⎠ dx 1 ⎛1⎞ = 4 ∫ x 3 log ⎜ ⎟ dx 0 ⎝x⎠ 1

Solution:

1

7.7

3

1 ⎛1⎞ log ⎜ ⎟ = t , = et , x = e − t , dx = −e − t dt ⎝x⎠ x x = 0, t

Let When

x = 1,

t=0

4

0 ⎛ 1⎞ x 3 log ⎜ ⎟ dx = 4∫ e −3t t ( −e − t ) dt ⎠ ⎝ 0 ∞ x



1



= 4 ∫ e −4t t 2 −1dt 0

= 4⋅ = Example 11: Evaluate



1

0



Solution:

1

0

1 4

dx

.

⎛ 1⎞ x log ⎜ ⎟ ⎝ x⎠ 1

dx ⎛1⎞ x log ⎜ ⎟ ⎝ x⎠

⎡ ∞ − kx n −1 n⎤ ⎢∵ ∫0 e x dx = n ⎥ k ⎦ ⎣

2 (4) 2

=∫ x



1 2

0

⎡ ⎛ 1 ⎞⎤ ⎢log ⎜⎝ x ⎟⎠ ⎥ ⎣ ⎦



1 2

dx

1 ⎛1⎞ Let log ⎜ ⎟ = t , = et , x = e–t, dx = e–t dt ⎝x⎠ x When x = 0, t x = 1,



1

0

1 ⎛1⎞ x log ⎜ ⎟ ⎝x⎠

t=0

0



1

1

−1

dx = ∫ ( e − t ) 2 . t







=∫ e 0



t 2

t 2 dt

1 2

( − e − t ) dt

… (1)

7.8

Engineering Mathematics

1 2

=

⎡ ∞ − kx n −1 n⎤ ⎢∵ ∫0 e x dx = n ⎥ k ⎦ ⎣

1 2

⎛ 1⎞ ⎜⎝ ⎟⎠ 2

= 2 Ç

Example 12: Evaluate

0

a

x dx . ax

Solution: Let ax = et, x log a = t, dx =

1 dt log a

x = 0, x ,

t=0 t

When





0



Example 13: Evaluate

a

∞⎛ xa t ⎞ . 1 . 1 dx = ∫ ⎜ x ⎟ et log a dt 0 ⎝ log a ⎠ a ∞

=

1 (log a ) a +1

=

1 a +1 (log a ) a +1

=

a +1 (log a ) a +1



e − t t a dt

0

2

3−4 x dx .

0 2

3−4 x = e − t , − 4 x 2 log 3 = −t log e, 4 x 2 log 3 = t

Solution: Let

x=

t 2 log 3

x = 0,

When

x





0

,

t

3−4 x dx = ∫ e − t . 0

= =

. 1 dt 2 log 3 2 t

t=0



2

1

, dx =

1 4 log 3

1 4 log 3 1





e−t t



. 1 dt t 1 2

dt

0

1 = 4 log 3 2 4 log 3

Gamma and Beta Functions

Example 14:

Prove that

Ç

xe - ax sin bx dx =

0



Solution:



0

7.9

2ab . (a + b 2 )2 2



xe − ax sin bx dx = ∫ xe − ax [Imaginary part of eibx]dx 0

Im. part





xe − ax . eibx dx

0

Im. part





e − ( a −ib ) x . x dx

0

= Im. part Im. part

⎡ ⎢∵ ⎣

2 (a ib) 2





0

e − kx x n −1 dx =

n⎤ ⎥ kn ⎦

1 (a 2 b 2 ) 2iab

2 2 2ab Im. part ⎡ (a − b ) + 2iab ⎤ = 2 2 2 ⎢ 2 2 2 2 2 ⎥ ⎣ ( a − b ) + 4a b ⎦ ( a + b )

Exercise 7.1 1. Evaluate the following integrals: (i)





x e − x dx 3

0

(ii)





e



x2 4

dx

0

(iii)





e−

0

1

(iv)

x

x

dx

7 4

( x log x) 4 dx

0

1 dx x dx

1

log

(v) 0

(vi)



1



(ix)

3 0





0

1

n

x m e − ax dx = na

3

4

1

(viii)



0

1⎞ ⎛ (vii) ∫ x ⎜ log ⎟ dx 0 x⎠ ⎝ 1

1 x log dx x

( m +1) n

m +1 . n

3. Prove that





0



x 2 e − x dx . ∫ e − x dx = 4

4

0

8 2

.

4. Prove that

2

5−4 x dx.

⎤ ⎥ ⎥ 4! ⎥ (iv) 5 ⎥ 5 ⎥ ⎥ ( vi) p ⎥ ⎥ 4 ⎛ 3 ⎞3 4 ⎥ ⎥ ( viii) ⎜ ⎟ ⎝ 4 ⎠ 3⎥ ⎥ ⎥ ⎥⎦ (ii) p

2. Prove that

− log x

0

315 ⎡ ⎢ Ans. : (i) 16 p ⎢ 8 ⎢ (iii) p ⎢ 3 ⎢ p ⎢ ( v) ⎢ 2 ⎢ ⎢ 6 ⎢ ( vii) 625 ⎢ ⎢ p ⎢ (ix ) ⎢⎣ 4 log 5





0

xe

− x2

dx .





0

e− x

2

x

dx =

2 2

.

7.10

Engineering Mathematics

5. Prove that





0

8. Prove that ∞

xe − x dx . ∫ x 2 e − x dx = 8

4

0

16 2

.



0

6. Prove that 1 0



x m −1 cos ax dx =

9. Prove that

x m (log x) n dx = (−1) n + 1 . (m + 1) n +1





m ⎛m ⎞ cos ⎜ ⎟. m ⎝ 2 ⎠ a

x n −1e − ax sin bx dx

0

n

=

7. Prove that

b⎞ ⎛ sin ⎜ n tan −1 ⎟ . a⎠ ⎝ ( a 2 + b2 ) n

n 2

n

n +1 1⎞ m ⎛ ∫0 x ⎜⎝ log x ⎟⎠ dx = (m + 1)n +1 . 1

7.4 BETA FUNCTION Beta function B(m, n) is defined by 1

B (m, n) = ∫ x m −1 (1 − x) n −1 dx, m > 0, n > 0. 0

B(m, n) is also known as Euler’s integral of first kind.

7.4.1 Trigonometric form of Beta Function B(m, n) = 2 ∫ 2 sin 2 m −1 x cos 2 n −1 x dx 0

B(m, n) =

Proof:



1

0

Let x When

sin2q,

dx

x m −1 (1 − x) n −1 dx

2sinq cosq dq x = 0,

q =0

x = 1,

q =

2

π 2

B(m, n) = ∫ (sin 2 θ ) m −1 (1 − sin 2 θ ) n −1 ⋅ 2 sin θ cos θ dθ 0

π 2

= 2 ∫ sin 2 m −1θ cos 2 n −1θ dθ 0

Changing the variable q to x, 2 2 m −1 x cos 2 n −1 x dx B(m, n) = 2 ∫ sin 0

Gamma and Beta Functions Corollary: Putting 2m 1 = p,

2n 1 = q

p +1 , 2

m=

7.11

q +1 2

n=

⎛ p +1 q +1⎞ 2 p q B⎜ , ⎟ = 2 ∫0 sin x cos x dx ⎝ 2 2 ⎠

7.5 PROPERTIES OF BETA FUNCTION 1. Symmetry

B(m, n) = B(n, m)

Proof:

B(m, n) =

Let 1

x = t,



1

0

dx = dt

When

x m −1 (1 − x) n −1 dx

x = 0,

t=1

x = 1,

t=0

0

B (m, n) = ∫ (1 − t ) m −1 t n −1 (−dt ) = 1

1

∫t

n −1

0

(1 − t ) m −1 dt

B (n, m).

2. Relation between Beta and Gamma Function m n

B(m, n) =

m+n Proof: By alternate form of Gamma function, ∞



m n = 2 ∫ e − x x 2 m −1dx . 2 ∫ e − y y 2 n −1dy 2

0

= 4∫



0

2

0





e−( x

2

+ y2 )

x 2 m −1 y 2 n −1dx dy

0

Changing to polar coordinates x = r cosq, y = r sinq dx dy = r dr dq Limits of x

x=0

to

x

Limits of y

y=0

to

y

Fig. 7.2

This shows that the region of integration is the first quadrant. Draw an elementary radius vector in the region which starts from pole and extends up to . Limits of r

r=0

to

r

Limits of q

q=0

to

q=

m n=4∫

π 2

0





0

2

2

e − r (r cos θ ) 2 m −1 (r sin θ ) 2 n −1 r dr dθ

7.12

Engineering Mathematics

π

π 2

= 4 ∫ (cos θ )

2 m −1

(sin θ )

2 n −1

0

=4.

2 dθ . ∫ e − r r 2 ( m + n ) −1dr 2

0

1 1 B(m, n) . m + n 2 2

m n

B(m, n) =

m+n

3. Duplication Formula 1 o 2m = 2 m -1 2 2

m m+

π 2

B(m, n) = 2 ∫ sin 2 m −1θ cos 2 n −1θ dθ

Proof:

0

Putting n = m, π 2

B(m, m) = 2 ∫ (sin θ cos θ ) 2 m −1 dθ 0

m m 2m Let 2q When

t,

1 d = dt 2 q = 0, q=

2

=

2 22 m −1



π 2

(sin 2θ ) 2 m −1 dθ

0

t=0 t=p

,

m⋅ m 2 1 = 2 m−1 ∫ (sin t ) 2 m−1 ⋅ dt 0 2 2 2m =

1 22 m −1 2

⋅ 2 ∫ (sin t ) 2

=

1 22 m −1

0

(cos t ) dt

0

22 m−1 ∫0 1 ⎛ 1⎞ = 2 m −1 B ⎜ m, ⎟ ⎝ 2⎠ 2 =

2 m −1

2

(sin t ) 2 m−1 (cos t )

1 2 1 m+ 2 m

⎛1⎞ 2 ⎜ ⎟ −1 ⎝2⎠

dt

⎡∵ 2 a f ( x)dx = 2 a f ( x)dx ⎤ ∫0 ⎢ ∫0 ⎥ ⎢ if f (2a − x) = f ( x) ⎥ ⎣ ⎦

Gamma and Beta Functions

m m 2m

=

7.13

1 m 22 m −1 1 m+ 2 m m+

1 2m = . 2 m −1 2 2

Example 1: Find the value of (i)

⎛ 4 5⎞ (ii) B ⎜ , ⎟ . ⎝ 3 3⎠

⎛ 3 1⎞ B⎜ , ⎟ ⎝ 2 2⎠

Solution: (i)

3 1 ⎛3 1⎞ 2 2 B⎜ , ⎟ = ⎝2 2⎠ 2 1 1 1 1 = 2 2 2 = 1 2 =

(ii)

.

2

4 5 ⎛ 4 5⎞ 3 3 B⎜ , ⎟ = 3 ⎝ 3 3⎠

=

1 1 2 1 1 1.2 2 +1 . +1 = . 2 3 3 2 3 3 3 3

=

1 1 1 1 1− = . 9 3 3 9 sin

⎡ ⎤ ⎢⎣∵ n 1− n = sin n ⎥⎦ 3

1 2 2 = . = 9 3 9 3 1 and n is a positive integer, find the value of n. 60 1 B(n, 3) 60

Example 2: If B(n, 3) = Solution:

n 3 n+3 n.2 (n + 2) (n + 1) n n

=

1 60

=

1 60

7.14

Engineering Mathematics n3 + 3n2 + 2n = 120 120 = 0

n3 + 3n2 + 2n

n = 4, 3.5 But n is a positive integer. Hence, n = 4. Example 3: Prove that B(n, n) =

Solution:

B(n, n) =

=

=

π . 22 n −1

n . = 2n 2n

n n

n . 2n

2

2 n −1

1

.

.

1 n+ 2

n n+

1 2

1 2 2n

n+

[By Duplication formula]

22 n −1

1 n+ 2

.

n

n n+

1 2

1 1 o Example 4: Prove that B(n, n) . B ⎛⎜ n + , n + ⎞⎟ = 21- 4 n . ⎝ 2 2⎠ n 1 1⎞ n n . ⎛ Solution: B(n, n) B ⎜ n + , n + ⎟ = 2 2⎠ ⎝ 2n

n+

1 1 n+ 2 2 2n + 1

⎛ 1⎞ ⎜⎝ n n + ⎟⎠ 2

2

⎛ 1⎞ n n+ ⎟ ⎜ 1 2⎟ ⎜ = = ⎜ 2n ⎟ . n 2 2n 2n 2n ⎝ ⎠ 2

=

1- n = Example 5: Prove that n 2

⎞ 1 ⎛ ⎜ 2 n −1 ⎟ = 21− 4 n 2n ⎝ 2 ⎠ n

n 2 . no 1- p 2 cos 2 o

Solution: We know that n 1− n =

sin n

2

Gamma and Beta Functions

Replacing n by

7.15

n +1 , 2 n +1 n +1 1− = 2 2

⎛ n +1⎞ sin ⎜ ⎟ ⎝ 2 ⎠

n +1 1− n = 2 2

n ⎞ ⎛ sin ⎜ + ⎟ ⎝2 2 ⎠

n n n 1 1− n 2 + = n 2 2 2 2 cos 2 n n 1− n 2 = n 2 2n −1 cos 2 n 1− n 2 = n n 2 1− n 2 cos 2 Exercise 7.2 1. Find the value of ⎛5 3⎞ (i) B ⎜ , ⎟ ⎝2 2⎠

⎛1 2⎞ (ii) B ⎜ , ⎟ ⎝2 3⎠

p 2p ⎤ ⎡ ⎢ Ans. : (i) 16 (ii) 3 ⎥ ⎣ ⎦ 1 2. If B(n, 2) = and n is a positive 42 integer, find the value of n. [Ans. : n = 6] 3. Prove that

Example 1: Evaluate



1

1 n+ 1 1⎞ 1 . ⎛ 2 B ⎜ n + , n + ⎟ = 2n 2 2⎠ 2 ⎝ n +1

4. Prove that B(m, n) = B(m, n + 1) + B(m + 1, n). 5. Prove that

3 3 −n +n 2 2

⎛1 ⎞ = ⎜ − n 2 ⎟ p sec np, (–1 < 2n < 1). ⎝4 ⎠

x 3 (1 − x ) dx. 5

0

Solution: Let

.

x = t , x = t2, dx = 2t dt

7.16

Engineering Mathematics

When

∫ x (1 − 1

x = 0,

t=0

x = 1,

t=1

x ) dx = ∫ t 6 (1 − t ) 2t dt 1

5

3

0

5

0

1

= 2 ∫ t 7 (1 − t ) dt 5

2B(8, 6)

0

=2 Example 2: Evaluate

1

x2

0

1− x



4

8 6

=2

14



dx .

1

0

1 7 ! 5! = 13! 5148 dx 1 − x4

1

Solution: Let

x4 = t, x = t 4 , dx =

When

x = 0,

t=0

x = 1,

t=1

.

1 − 34 t dt 4

1



1

0

x2 1 − x4

dx .



1

0

dx 1 − x4

=∫

1

0

3 1 − 1 . 1 − 34 . 1 t 4 dt . ∫0 1 − t 4 t dt 1− t 4

t2

=

1 1 1 −3 − − 1 1 − 14 2 . t 4 (1 − t ) 2 dt t ( − t ) d t 1 ∫0 16 ∫0

=

1 ⎛3 1⎞ . ⎛1 1⎞ B⎜ , ⎟ B⎜ , ⎟ 16 ⎝ 4 2 ⎠ ⎝4 2⎠

3 1 1 1 1 4 2 . 4 2 1 . 1 = = 16 5 16 3 1 1 4 4 4 4 4 Example 3: Evaluate



1

0

1 − y 4 dy . 1

Solution: Let

y4 = t, y = t 4 , dy =

When

y = 0,

t=0

y = 1,

t=1



1

0

1 − 34 t dt 4

1 1 1 −3 1 − y 4 dy = ∫ (1 − t ) 2 . t 4 dt 0 4

3 1 1 ⎛3 1⎞ 1 2 4 = B⎜ , ⎟ = 4 ⎝2 4⎠ 4 7 4

=

p 4

Gamma and Beta Functions

1 1 1 1 2 2 4 1 = = 6 4 3 3 4 4

p

⎛ 1⎞ ⎜⎝ ⎟⎠ 4

7.17

2

3 1 4 4 2

2

⎛ 1⎞ ⎜ ⎟ p ⎝ 4⎠ p = = p 6 6 1 1 1− p 4 4 sin 4 ⎛ 1⎞ ⎜⎝ ⎟⎠ 4

2

⎛ 1⎞ ⎜ ⎟ 1 p ⎝ 4⎠ = = 6 p 2 6 2 Example 4: Evaluate



2

0

⎛ 1⎞ ⎜ ⎟ ⎝ 4⎠

2

1

y 4 ( 8 − y 3 ) 3 dy . −

1 1 −2 Solution: Let y3 = 8t, y = 2t 3 , dy = 2 . t 3 dt 3 When y = 0, t=0

y = 2,



2

0

1

t=1

( ) (8 − 8t ) 1 4

y 4 ( 8 − y 3 ) 3 dy = ∫ 2t 3 −

1

0

=



2a 0

Solution:



2a

0

x 2 2ax - x 2 dx . 2a

5

x 2 2ax − x 2 dx = ∫ x 2 2a − x dx 0

Let x = 2at, dx = 2a dt When

2 − 32 t dt 3

1 16 1 32 16 ⎛ 5 2 ⎞ − 3 dt = ( ) t − t 1 B⎜ , ⎟ ∫ 0 3 3 ⎝3 3⎠

5 2 16 3 3 16 = = 3 3 7 3 Example 5: Evaluate

1 3

x = 0,

t=0

x = 2a,

t=1

2 2 3 3 4. 1 3 3

⎛ 2⎞ 2 ⎜⎝ ⎟⎠ 3 3 =8 1 1 3 3

2

7.18

Engineering Mathematics



2a

0

5

1

x 2 2ax − x 2 dx = ∫ (2at ) 2 2a − 2at . 2a dt 0

1 1 5 ⎛7 3⎞ 4 = 16a 4 ∫ t 2 (1 − t ) 2 dt = 16a B ⎜ , ⎟ ⎝2 2⎠ 0

7 3 16a 4 . 5 . 3 . 1 1 . 1 1 = 16a 2 2 = 5 24 2 2 2 2 2 2 4

=

15 a 4 24

3

Example 6: Evaluate



3

0

x2

dx . ∫

3− x

dx

1

0

1− x

1 4

.

3

Solution: Let x = 3t, dx = 3 dt When

I1 = ∫

x2

3

3− x

0

dx

x = 0,

t=0

x = 3,

t=1 3

I1 = ∫

(3t ) 2

1

3 − 3t

0

⋅ 3 dt

1 3

= 9 ∫ t 2 (1 − t )



1 2

0

⎛5 1⎞ dt = 9 B ⎜ , ⎟ ⎝2 2⎠

5 1 9 3 1 1 1 27 =9 2 2 = . . = 3 2 2 2 2 2 8 1 dx I2 = ∫ 1

0

1− x4

1

Let x 4 = t , x = t4, dx = 4t3 dt When

x = 0,

t=0

x = 1,

t=1 3

1

4t

0

1− t

I2 = ∫

1

dt

= 4 ∫ t 3 (1 − t ) 0



1 2

⎛ 1⎞ dt = 4 B ⎜ 4, ⎟ ⎝ 2⎠

Gamma and Beta Functions

4 =4

9 2

1 2 =4

7.19

1 128 2 = 7 . 5 . 3 . 1 1 35 2 2 2 2 2 3!

3



Hence,

x2

3

dx . ∫

3− x

0

Example 7: Prove that



0

1− x

dx

a

0

dx

1

1 n n

(a n − x ) 1

Solution: Let xn = an t, x = at n , dx = x = 0, x = a,

When



a

0

dx 1 n n

(a n − x )

=∫

1 4

=

=

o ⎛o ⎞ cosec ⎜ ⎟ . ⎝ n⎠ n

a 1n −1 t dt n

t=0 t=1 1

1

0

(a n − a n t )

1 n

1 −1 . a t n dt n

1 1 − −1 1 1 1 ⎛ 1 1⎞ n n 1 ( − ) t t dt = B ⎜ − + 1, ⎟ ∫ 0 n n ⎝ n n⎠

=

1 = . n

1−

1 1 n n =1. n sin 1

⎛ ⎞ = cosec ⎜ ⎟ n ⎝n⎠ Example 8: Prove that



b



9 4

5

Solution: Let (x When



b a

⎡ ⎤ ⎢⎣∵ 1− n n = sin n ⎥⎦ n

( x − a )m (b − x )n dx = (b − a )m + n +1 B( m + 1, n + 1) and

a

hence, deduce that

27 . 128 432 p = 8 35 35

( x − 5)(9 − x ) dx =

⎛ 1⎞ 2⎜ ⎟ ⎝ 4⎠

2

.

3

a) = (b

dx = (b

a) t,

x = a, x = b,

a) dt

t=0 t=1

1

( x − a ) m (b − x) n dx = ∫ [ (b − a )t ] [b − {a + (b − a )t}] (b − a )dt m

n

0

1

= (b − a ) m + n +1 ∫ t m (1 − t ) n dt 0

= (b − a )

m + n +1

B(m + 1, n + 1)

7.20

Engineering Mathematics

1 1 Putting a = 5, b = 9, m = , n = in the above integral, 4 4 1 1 1 1 9 + +1 1 ⎞ ⎛1 ∫5 ( x − 5) 4 (9 − x) 4 dx = (9 − 5) 4 4 B ⎜⎝ 4 + 1, 4 + 1⎟⎠ 2

⎛ 1⎞ ⎛1 1⎞ 5 5 2⎜ ⎟ ⎜ ⎟ ⎝ 4⎠ = 23 4 4 = 8 ⎝ 4 4 ⎠ = 3 5 3.1 1 2 2 2 2



Example 9: Prove that

1

0



1

0

2

x m −1 (1 − x )n −1 B( m , n) dx = and hence, evaluate m+n (a + bx ) ( a + b )m a n

x − 2x + x dx . (1 + x )6 2

3

4

Solution: Let x = When

Also,

a (a + b − bt ) − at (−b) a ( a + b) at dx = dt = , dt 2 (a + b − bt ) (a + b − bt ) 2 a + b − bt x = 0, t=0 x = 1, t=1 1− x = 1−

at (a + b)(1 − t ) = a + b − bt a + b − bt

a + bx = a +

a ( a + b) bat = a + b − bt a + b − bt m −1



1

0

m −1

n −1

x (1 − x) (a + bx) m + n

at ⎛ ⎞ ⎡ (a + b)(1 − t ) ⎤ ⎟ ⎢ 1⎜ ⎥ dx = ∫ ⎝ a + b − bt ⎠ ⎣ a +mb+ n− bt ⎦ 0 ⎡ a ( a + b) ⎤ ⎢⎣ a + b − bt ⎥⎦

=

1 ( a + b) m a n

1

∫t

m −1

0

(1 − t ) n −1 dt =

Putting a = 1, b = 1, m = 3, n = 3 in the above integral



1

0



1

0

1 x 2 (1 − x) 2 B(3, 3) dx = 6 (1 + 1)3 .13 (1 + x)

x 2 − 2 x3 + x 4 1 3 3 1. 4 1 dx = = = 6 (1 + x) 8 6 8 120 240 3

(1 − x 4 ) 4 1 1 ⎛ 1 7⎞ Example 10: Prove that ∫ dx = . 1 B ⎜ , ⎟ . 4 2 0 (1 + x ) 4 4 ⎝ 4 4⎠ 2 1

n −1

. a ( a + b ) dt (a + b − bt ) 2

1 B(m, n) ( a + b) m a n

Gamma and Beta Functions

1

Solution: Let x4 = t, x = t 4 , dx =

1 − 34 t dt 4

x = 0, x = 1,

When

7.21

t=0 t=1

3

3



3

3

1 (1 − t ) 4 (1 − x 4 ) 4 1 −3 1 1 t 4 (1 − t ) 4 ∫0 (1 + x 4 )2 dx =∫0 (1 + t )2 . 4 t 4 dt = 4 ∫0 (1 + t )2 dt 1

u , 2−u

t=

Let

dt =

2 (2 − u ) − u ( −1) du du = 2 ( u )2 2 − (2 − u )

t = 0, t = 1,

When

u=0 u=1 −

3

3

u ⎞4 ⎛ u ⎞ 4⎛ ⎜⎝ ⎟⎠ ⎜⎝1 − ⎟ 1 1 (1 − x ) 2−u 2 − u⎠ 1 2 . x = du d 2 ∫0 (1 + x 4 )2 4 ∫0 ( 2 u )2 − u ⎞ ⎛ ⎜⎝1 + ⎟ 2 − u⎠ 3 4 4

2 1 u = ∫ 4 0



3 4

3

( 2 − 2u ) 4 2

2

1 1 du = . 1 4 4 2



1

0

u



3 4

3

(1 − u ) 4 du

1 1 ⎛1 7⎞ = . 1 B⎜ , ⎟ 4 4 ⎝4 4⎠ 2 Exercise 7.3 1. Evaluate the following integrals: (i) (ii)

1



0 1



0

1 − x dx m

dx 1 − x6

2

1 3 ⎛ ⎞ (iii) ∫ ⎜1 − x 4 ⎟ dx 0 ⎝ ⎠ 1



2

(v)



a

(vi)



(iv)

0

0 1 2

0



1

x 2 ( 2 − x ) 2 dx x 4 a 2 − x 2 dx x 3 1 − 4 x 2 dx.

⎡ Ans. : ⎤ ⎢ ⎥ ⎢ (i) 1 B ⎛⎜ 1 , 3 ⎞⎟ (ii) 1 B ⎛⎜ 1 , 1 ⎞⎟ ⎥ ⎢ m ⎝ m 2⎠ 8 ⎝ 8 2⎠ ⎥ ⎢ ⎥ 64 2 ⎢(iii) 128 ⎥ (iv) ⎢ ⎥ 1155 15 ⎢ ⎥ 6 1 ⎢ ( v) p a ⎥ ( vi) ⎢⎣ ⎥⎦ 32 120

2. Prove that ⎛ 1⎞ 2⎜ ⎟ 7 (i) ∫ 4 ( x − 3)(7 − x) dx = ⎝ 4 ⎠ 3 3

2

7.22

Engineering Mathematics



(ii)

6

5

( x − 5)5 (6 − x)6 dx =



3. Prove that

1

0



1

x 3 (1 − x) (1 + 2 x)



5! 6 ! . 12 !

2 3

and hence, evaluate

0



0

dx = 3

.

7 6

π 2



Solution:

π 2

0

x n −1 ∫0 (1 + cx)(1 − x)n dx 1

5. Prove that =

x m −1 (1 − x) n −1 B(m, n) dx = (1 + x) m + n 2m

Example 1: Evaluate

1 . , 0 < n < 1. (1 + c) n sin n

cot θ dθ .

0

π

1



1

cot θ dθ = ∫ 2 (cos θ ) 2 (sin θ ) 2 dθ 0

1 ⎞ ⎛1 +1 − +1⎟ 1 ⎜2 , 2 ⎟ = B⎜ 2 ⎠ 2 ⎝ 2 3 1 1 ⎛3 1⎞ 1 4 4 = B⎜ , ⎟ = 2 ⎝4 4⎠ 2 1 =

Example 2: Evaluate

π 4

1 1 1 1 1− = ⋅ 2 4 4 2 sin

=

p 2

4

cos 3 2θ sin 4 4θ dθ .

0

Solution:



π 4

0

π

cos3 2θ sin 4 4θ dθ = ∫ 4 cos3 2θ (2 sin 2θ cos 2θ ) 4 dθ 0

π

= 16 ∫ 4 cos 7 2θ sin 4 2θ dθ 0

Let 2q = t, When

x3 − 2 x 2 + x dx. (1 + x)5 1⎤ ⎡ ⎢⎣ Ans. : 48 ⎥⎦

4. Prove that 1



1

1 d = dt 2 q = 0, q=

t=0 ,

4

t=

2

Gamma and Beta Functions



π 4

0

7.23

π

1 cos3 2θ sin 4 4θ dθ = 16 ∫ 2 sin 4 t . cos 7 t ⋅ dt 0 2 5 4 1 5 ⎛ ⎞ = 8. B ⎜ , 4 ⎟ = 4 2 ⎠ 2 ⎝2 13 2 3 .1 1 . 3! 128 2 2 2 = = 4⋅ 11 9 7 5 3 1 1 1155 ⋅ ⋅ ⋅ ⋅ ⋅ 2 2 2 2 2 2 2

Example 3: Evaluate





0

sin 2 θ (1 + cos θ )4 dθ . 2π

I = ∫ sin 2 θ (1 + cos θ ) 4 dθ

Solution:

0

2

4

2π θ θ⎞ ⎛ θ⎞ ⎛ = ∫ ⎜ 2 sin cos ⎟ ⎜ 2 cos 2 ⎟ dθ 0 ⎝ 2 2⎠ ⎝ 2⎠ 2π

= 26 ∫ sin 2 0

Let 2 When

θ θ cos10 dθ 2 2

= t , d = 2dt q = 0, q = 2p,

t=0 t=p

I = 26 ∫ sin 2 t cos10 t . 2dt 0

= 27 . 2 ∫ 2 sin 2 t cos10 t dt 0

⎡∵ f ( x ) dx = 2 ∫ f ( x ) d x ⎤ ⎢ ∫0 ⎥ 0 ⎢ if f (2a − x) = f ( x) ⎥ ⎣ ⎦ 2a

a

3 11 1 3 11 ⎛ ⎞ 7 2 2 = 28 . B ⎜ , ⎟ =2 2 ⎝2 2 ⎠ 7 = Example 4: Evaluate



o 4 o 4

27 . 1 1 9 . 7 . 5 . 3 . 1 1 21p . = 6! 2 2 2 2 2 2 2 2 8 1

(cos p + sin p ) 3 dp . 1

Solution:



π 4 π − 4

1 3

(cos θ + sin θ ) dθ =



π 4 π − 4

⎡ ⎛ 1 1 ⎞⎤ 3 cos θ + sin θ ⎟ ⎥ dθ ⎢ 2 ⎜⎝ ⎠⎦ 2 2 ⎣

7.24

Engineering Mathematics



π 4 π − 4

π ⎛ π ⎞3 2 ⎜ sin cos θ + cos sin θ ⎟ dθ = ⎝ ⎠ 4 4

Let

+

4

= t,

q= π 4 π − 4



⎡ ⎛π ⎞⎤ 3 2 ⎢sin ⎜ + θ ⎟ ⎥ dθ ⎠⎦ ⎣ ⎝4

π 4 π − 4

1 6

dq = dt

q = −

When



1

1

1 6

(cos θ + sin θ ) dθ = 2

1 6

t=0 t=

,

4

1 3

,

4



2

0

2

1 3

(sin t ) dt 1

1 6

1 3

26 ⎛ 4 1 ⎞ 2 ∫ 2 (sin t ) (cos t ) dt = B⎜ , ⎟ 0 2 ⎝6 2⎠

=

2 1 2 1 6 3 2 = 3 = 5 5 7 1 1 26 26 6 6 6

1 5

26 Example 5: Prove that 2

Solution:

0

π 2 0



tan n x dx =

tan n x dx =



2

0

0

2 3 1 6

π ⎛ nπ ⎞ sec ⎜ ⎟. 2 ⎝ 2 ⎠

(sin x) n (cos x) − n dx

1 ⎛ n + 1 −n + 1 ⎞ 1 , = B⎜ ⎟= 2 ⎝ 2 2 ⎠ 2

n + 1 −n + 1 2 2 1

1 n +1 n +1 1− 2 2 2

1 ⋅ 2

2 Example 6: Evaluate Solution:

0

0



⎤ ⎡ ⎥ ⎢∵ n 1− n = sin n ⎦ ⎣

⎛ n +1⎞ sin ⎜ ⎝ 2 ⎟⎠ 1 n ⎞ ⎛ sin ⎜ + ⎝ 2 2 ⎟⎠

=

1 ⎛n ⎞ . = sec ⎜ ⎝ 2 ⎟⎠ 2 cos n 2 2

x sin 7 x cos 4 x dx .

x sin 7 x cos 4 x dx =



0

( − x) sin 7 ( − x) cos 4 ( − x)dx ⎡∵ a f ( x)dx = a f (a − x)dx ⎤ ∫0 ⎣⎢ ∫0 ⎦⎥

Gamma and Beta Functions



sin 7 x cos 4 x dx − ∫ x sin 7 x cos 4 x dx

0

2

x sin 7 x cos 4 x dx =

0

7.25

0

sin 7 x cos 4 x dx

0

⎡ 2 7 ⎤ 4 7 4 2 ⎢ ∫0 sin x cos x dx + ∫0 sin ( − x) cos ( − x)dx ⎥ ⎣ ⎦ ⎡∵ 2 a f ( x)dx = a f ( x)dx + a f (2a − x)dx ⎤ ∫0 ∫0 ⎥⎦ ⎢⎣ ∫0 2

2

0

sin 7 x cos 4 x dx = 2 ⋅

5 2= 13 2

4

0

x sin 7 x cos 4 x dx =

1 ⎛ B ⎜ 4, 2 ⎝

5⎞ ⎟ 2⎠

5 2 11 9 7 5 5 2 2 2 2 2 3!

16 1155

Exercise 7.4 1. Evaluate the following integrals: (i)

π 2 0

(ii)

π 6 0

(iii)

π 3 π − 6

(iv) (v) (vi)

tan θ dθ 2

6

cos 3θ sin 6θ dθ

∫ ( π 2 π − 2 2π

3 sin θ + cos θ

)

1 4





cos3 θ (1 + sin θ )2 dθ



sin 2 θ (1 + cos θ )4 dθ

0

0

⎡ ⎢ Ans. : (i) 2 ⎢ ⎢ 3 ⎢ − (iii) 2 4 ⎢ ⎢ ⎢ ⎢ 21 ⎢ ( v) ⎢⎣ 8

7 ⎤ 384 ⎥ ⎥ ⎥ 8 ⎥ (iv ) ⎥ 5 ⎥ ⎥ ⎥ 8 ⎥ (iv ) 693 ⎥⎦ (ii)

5 8 9 8

2. Prove that



2

0

x sin 5 x cos6 x dx

(sin x) 2 n dx =

1 ⋅ 3 ⋅ 5 (2n − 1) ⋅ . 2n (n !) 2

7.6 BETA FUNCTION AS IMPROPER INTEGRAL B(m, n) =





0

x m −1 dx (1 + x) m + n

Proof: Let x = tan q, dx = 2 tanq sec2q dq 2

7.26

Engineering Mathematics x = 0,

When

x





0

q=0 π θ= 2

Ç,

x m −1 dx = (1 + x) m + n

(tan 2 θ )m −1 ⋅ 2 tan θ sec 2 θ dθ (1 + tan 2 θ )m + n

π 2 0



(tan θ )2 m −1 sec 2 θ dθ (sec θ )2 m + 2 n

π

2∫2 0

π

2 ∫ 2 (sin θ )2 m −1 (cos θ )2 n −1 dθ 0

= B(m, n) Example 1: Prove that value of



∞ 0



x m −1 1 dx = n m B( m , n) and hence, find the m+n (a + bx ) a b



0

5

x . (2 + 3 x )16 a dt b

Solution: Let bx = at, dx =

x = 0,

When

t=0

x

,

t m −1



x m −1 dx = (a + bx) m + n



0





0

⎛a ⎞ ⎜⎝ t ⎟⎠ a b ⋅ dt (a + at ) m + n b

1 a bm



n



0

1 t m −1 dt = n m B(m, n) a b (1 + t ) m + n

Putting a = 2, b = 3, m = 6, n = 10 in the above integral,



x5 1 dx = 10 6 B(6, 10) 16 2 3 (2 + 3x)



0

=

Example 2: Prove that





0

Solution:





0

1

6 10

10

6

2

3

16

=

1 10

2

5! 10 ! 3 15! 6

x 8 (1 − x 6 ) dx = 0 . (1 + x )24

x8 (1 − x 6 ) dx = (1 + x) 24









0

0

∞ x8 x14 x d − ∫0 (1 + x)24 dx (1 + x) 24 ∞ x 9 −1 x15 −1 dx − ∫ dx 9 +15 0 (1 + x )15 + 9 (1 + x)

Gamma and Beta Functions

B(9, 15)

7.27

B(15, 9)

0 Example 3: Prove that

x2 5 2 . dx = (1 + x 4 )3 128





0

1

Solution: Let x4 = t, x = t 4 , dx =

1 43 t dt 4

x = 0,

When

x

t=0 ,

t 1





0

x2 dx = (1 + x 4 )3





0

t2 1 −3 ⋅ t 4 dt 3 (1 + t ) 4 −

1

3

1 ∞ t 4 1 ∞ dt = ∫ 3 ∫ 4 0 (1 + t ) 4 0 1 ⎛3 B⎜ , 4 ⎝4

t4

−1

(1 + t )

3 9 + 4 4

dt

3 9 9⎞ 1 4 4 ⎟= 4⎠ 4 3

3 5 1 1 4 4 4 4 = 5 1 1 1 2! 128 4 4 ⎤ ⎡ 5 ⎥ ⎢∵ 1− n n = n sin 128 sin ⎦ ⎣ 4 5 2 = 128 1 4

Example 4: Prove that





0

Solution:





0

sech 6 x dx =

sech 6 x dx =





0

8 . 15 6

⎛ 2 ⎞ ⎜⎝ x ⎟ dx e + e− x ⎠

26 ⋅

1 ∞ 1 dx x ∫ − ∞ 2 (e + e − x ) 6

25 ∫

e6 x dx − ∞ (e + 1)6 ∞

2x

⎡ e x + e− x ⎤ ⎢∵ cosh x = ⎥ 2 ⎦ ⎣ ⎡∵ a f ( x)dx = 2 a f ( x)dx ⎤ ∫0 ⎢ ∫− a ⎥ ⎢ if f (− x) = f ( x) ⎥ ⎣ ⎦

7.28

Engineering Mathematics

e2x = t, 2e2x dx = dt, x , t=0 x , t

Let When





0

sec h 6 x dx = 25



0 ∞

0

16 Example 5: Prove that Solution:





0

When





0

0

t 3−1 dt (1 + t )3+ 3

33

16

6

24 B(3, 3) 8 2! 2! = . 15 5!

e 2 mx + e − 2 mx 1 dx = B( n + m , n − m ), n > m . x − x 2n 2 (e + e )

e 2 mx + e − 2 mx dx (e x + e − x ) 2 n

e2x = t,

Let





1 dt 2t

t3 1 ⋅ dt 6 2t (t + 1)



24 ∫

dx

1 ∞ (e 2 mx + e −2 mx ) e 2 nx dx 2 ∫− ∞ (e 2 x + 1) 2 n ⎡∵ a f ( x)dx = 2 a f ( x)dx ⎤ ∫0 ⎢ ∫− a ⎥ ⎢ if f (− x) = f ( x) ⎥ ⎣ ⎦

1 ∞ e2( m + n) x + e2( n − m) x dx 2 ∫− ∞ (1 + e 2 x ) 2 n 1 2e2x dx = dt, dx = dt 2t x , t=0 x , t ,

e 2 mx + e −2 mx 1 ∞ t m+n + t n−m 1 dx = ∫ ⋅ dt x − x 2n (e + e ) 2 0 (1 + t ) 2 n 2t ∞ ⎤ 1⎡ ∞ t ( m + n ) −1 t ( n − m ) −1 t d dt ⎥ + ⎢ ∫0 (m+ n)+(n− m) (n− m)+(n+ m) ∫ 0 4 ⎣ (1 + t ) (1 + t ) ⎦

1 [ B(m + n, n − m) + B(n − m, n + m)] 4 1 B (n + m, n − m) [∵ B(m, n) = B(n, m)] 2 Example 6: Prove that

deduce that



o



0

sin x

0

(5 + 3 cos x )

3 2

sin n −1 x dx (a + b cos x )n

dx =

⎛ ⎜ ⎝

3⎞ 4 ⎟⎠

2

2 2o

.

=

2n −1 n 2 2

(a 2 − b )

⎛ n n⎞ B ⎜ , ⎟ and hence, ⎝2 2⎠

Gamma and Beta Functions

Solution: Let tan

7.29

x x 2 = t , = tan −1 t , dx = dt 2 2 1+ t2 1− t2 2t , sin x = 2 1+ t 1+ t2 x = 0, t=0

cos x = When

x = p,



0

n −1

sin x dx = (a + b cos x) n

Let

(a



t ⎛ 2t ⎞ ⎜⎝ ⎟ 1+ t2 ⎠



0

n −1

⎡ ⎛1 − t 2 ⎞⎤ ⎢a + b ⎜ ⎥ ⎝ 1 + t 2 ⎟⎠ ⎦ ⎣

a+b ⋅ u

b) t2 = (a + b)u, t = t = 0, t ,

When

n

2n ∫

2 dt 1+ t2



a−b

0

, dt =

t n −1



⎡⎣(a + b) + (a − b) t 2 ⎤⎦ a+b

2 a−b



1 u

1 2

n

dt

du

u=0 u n −1



0

⎡ (a + b)u ⎤ 2 n −1 ⎢ ( a − b) ⎥ sin x ∞ a+b 1 ⎣ ⎦ dx = 2n n ∫0 [(a + b) + (a + b) u ]n ⋅ 2 a − b 1 du (a + b cos x) u2 n

2n −1 n 2

( a + b) ( a − b) Putting a = 5, b = 3, n =

0

sin x (5 + 3 cos x)



0

−1

(1 + u )

n n + 2 2

3 in the above integral, 2 3



n 2

u2



3 2

dx =

22

−1 3 2 4

(52 − 3 )

⎛3 3⎞ B⎜ , ⎟ ⎝4 4⎠ 2

⎛ 3⎞ 3 3 2⎜ ⎟ ⎝ 4⎠ 2 4 4 = 3 2 3 1 1 23 ⋅ 2 2 2

=

⎛ 3⎞ ⎜ 4⎟ ⎝ ⎠ 2 2

2

du =

2n −1 n 2 2

(a 2 − b )

⎛n B⎜ , ⎝2

n⎞ ⎟ 2⎠

7.30

Engineering Mathematics

Example 7: Prove that



π 2

cos 2 m −1 sin 2 n −1 Β (m , n ) d = 2m 2n . 2a b (a 2 cos 2 + b 2 sin 2 )m + n

0

π 2 0

cos 2 m −1 θ sin 2 n −1 θ dθ (a 2 cos 2 θ + b 2 sin 2 θ )m + n

π 2 0

cos 2 m−1 θ sin 2 n −1 θ (cosθ )−2 m− 2 n dθ (a 2 + b 2 tan 2 θ )m+ n



Solution:

= =

b2 tan 2 a2

Let



= t,

q= π 2 0





a 2( m+ n )

(tan θ )2 n −1 sec 2 θ ⎛ b2 2 ⎞ ⎜⎝1 + a 2 tan θ ⎟⎠

a t, b

tan q =

q = 0,

When

π 2 0

1

m+ n



sec2 q dq =

a 2b

1 t

dt

t=0 ,

2

t

cos 2 m −1 θ sin 2 n −1 θ 1 dθ = 2 ( m + n ) (a cos 2 θ + b 2 sin 2 θ ) m + n a 2



2a b

2n

2 n −1

(1 + t ) m + n

0

1 2m

⎛ a 12 ⎞ ⎜ t ⎟⎠ ∞ ⎝b





0



a 1 dt ⋅ 2b t

t n −1 dt (1 + t ) m + n

1 B(m, n) 2a 2 m b 2 n Example 8: Prove that B(m, n) = Solution: We have B(m, n) =





0



1 0

x m −1 + x n −1 dx . (1 + x )m + n

x m −1 dx (1 + x) m + n

∞ x m −1 x m −1 x d + ∫0 (1 + x)m + n ∫1 (1 + x)m + n dx 1

Consider, Let When

I=





1

1 , y x =1, x=

x

,

x m −1 dx (1 + x) m + n dx = y=1 y=0

1 dy y2

… (1)

Gamma and Beta Functions

I=

⎛1⎞ ⎜⎝ y ⎟⎠

0



m −1

⎛ 1⎞ ⎜⎝1 + y ⎟⎠

1

7.31

⎛ 1 ⎞ ⎜⎝ − y 2 ⎟⎠ dy =

m+n

y n −1 ∫0 ( y + 1)m + n dy 1

Substituting in Eq. (1), B(m, n) =

1 x m −1 y n −1 x d + ∫0 (1 + x)m + n ∫0 (1 + y)m + n dy

B(m, n) =



1

Replacing y by x, 1 x m −1 x n −1 dx + ∫ dx m + n 0 (1 + x ) 0 (1 + x ) m + n 1



1

0

Example 9: Prove that

Solution: Let tan

2

= t,



π 2 0

2

x m −1 + x n −1 dx (1 + x) m + n

dθ 1 1 − sin 2 θ 2

=

π 2 0



2 dt 1+ t2

0, π θ= , 2

t=0

=

1

dθ 1 1 − sin 2 θ 2



t=1

1

0

1−

2∫

1

0

1 −3 Let t4 = u, t = u , dt = u 4 du 4 When

.

2t 1+ t2 q

When

2

4

= tan 1 t , d =

sin q =

⎛ 1⎞ ⎜ 4⎟ ⎝ ⎠

1 ⎛ 2t ⎞ ⎜ ⎟ 2 ⎝1 + t 2 ⎠

1 1 4 2

(1 + t )

1 4

t = 0,

u=0

t = 1,

u=1

dt

2



2 dt 1+ t2

7.32

Engineering Mathematics

π 2 0



dθ 1−

1 sin 2 θ 2

= 2∫

1

0

(1 + u ) 2 1

1 = 2

1 −3 ⋅ u 4 du 4

1

1



1

0

u4

1

−1

1 du = 4

1

(1 + u ) 2

1 ⎛1 B⎜ , 4 ⎝4



1

0

1

−1

u4 + u4 1

−1

+

(1 + u ) 4

1⎞ ⎟ 4⎠

du

[From Ex. 8]

⎛ 1⎞ ⎜⎜ ⎟⎟ ⎝ 4⎠

1 1 1 4 4 = 4 1 2

1 4

2

4

Exercise 7.5 1. Evaluate





0

4. Prove that

dy . 1+ y4 ⎤ ⎡ ⎢ Ans. : 2 2 ⎥ ⎦ ⎣

2. Prove that m −1 ∞ x + x n −1 ∫0 (1 + x)m + n dx = 2B (m, n).



0



0

dx 1 ⎛n B⎜ , = −x n ⎝2 4 (e + e ) x

and hence, evaluate

5. Prove that

3. Prove that ∞



x dx = . (4 + 4 x + x 2 ) 4 2





1

x

p +1





0

n⎞ ⎟ 2⎠

sec h 8 x dx. 16 ⎤ ⎡ ⎢⎣ Ans. : 35 ⎥⎦

dx ( x − 1) q

= B(p + q, 1 – q), if

p < q < 1.

FORMULAE Gamma Function ∞

(i) n = ∫ e − x x n − 1dx, n > 0 0

(ii)



n = 2 ∫ e − x x 2 n − 1dx

n +1 (iii) n = , if n is a negative fractionn

2

0

Properties of Gamma Function (i) n + 1 = n! , if n is a positive integer (ii) n + 1 = n n , if n is a positive real number

(iv) n 1 − n (v)

1 = π 2

π sin nπ

Gamma and Beta Functions

Beta Function



(i) B (m, n) = m > 0, n > 0

1

0

x m − 1 (1 − x ) n − 1 dx,

Properties of Beta Function (i) B (m, n) = B (n, m)

2 m −1 x cos 2 n −1 x dx (ii) B (m, n) = 2 ∫0 sin





0

mn m+n

(ii) B (m, n) =

π 2

(iii) B (m, n) =

7.33

xm − 1 dx (1 + x ) m + n

(iii)

m m+

1 π 2m = 2 m −1 2 2 (Duplication formula)

MULTIPLE CHOICE QUESTIONS Choose the correct alternative in each of the following: 1. The value of the integral I=

1 2p



∫e

⎛ − x2 ⎞ ⎜⎝ ⎟ 8 ⎠

dx is

0

(a) 1 (b) p (c) 2 (d) 2p 2. Match the items in columns I and II for the following special functions I

II

(P) b (p, q)

1 2

(i) ∞

(Q) (R) (S) (a) (b) (c) (d)

y p −1 p q dy (ii) ∫ (1 + y ) ( p + q ) p+q 0 (iii) b (p, q) p p (iv) p 1 p sin pp P-(iv), Q-(iii), R-(i), S-(ii) P-(ii), Q-(iii), R-(i), S-(iv) P-(iii), Q-(ii), R-(i), S-(iv) P-(ii), Q-(iii), R-(iv), S-(i) ∞

3. The value of



3

y e − y dy is

0

p 3 p (c) p (d) 6 4. The value of B (m + 1, n) is n (a) B (m, n) m+n (a)

p 2

(b)

n B (m, n) m +1 m B (m, n) (c) m+n m B (m, n) (d) m +1 (b)

5. The value of

p 2

cot q dq is

0

(a)

p 2 2

(b)

p 2

(c)

p 2 4

(d)

p 4

()

3

⎡ 1 ⎤ 6. The value of ∫ x ⎢log dx is 0 x ⎥⎦ ⎣ (a) 3 (b) 6 325 625 3 (c) (d) 6 625 325 1

4

7. If B (n, 2) = 1 and n is a positive 6 integer, then the value of n is (a) 3 (b) – 2 (c) 2 (d) – 3 8. The value of (a)

p 2

(c) p 2





0

t 2 dt is 1 + t4 (b)

p 2

(d) p 4

7.34

Engineering Mathematics

9. The value of B (m, m) is 1 (a) 21 – 2m B m, 2 (b) 21 – 2m (c) 21 – 2m (d) 21 – 2m

Answers 1. (a) 8. (a)

( ) 1 B ( m + 1, ) 2 1 B ( m + , 1) 2 3 B ( m, ) 2 2. (b) 9. (a)

3. (b) 10. (d)

10. Gamma function is discontinuous for (a) all p < 0 (b) any p > 0 (c) p = 0 only (d) p = 0 and negative integers 11. Beta function B (p, q) is convergent for (a) p > 0, q < 0 (b) p > 0, q > 0 (c) p < 0, q > 0 (d) p < 0, q < 0

4. (c) 11. (b)

5. (a)

6. (b)

7. (c)

Multiple Integral Chapter

8

8.1 INTRODUCTION Integration of functions of two or more variables is normally called multiple integration. The particular case of integration of functions of two variables is called double integration and that of three variables is called triple integration. Sometimes, we have to change the variables to simplify the integrand while evaluating the multiple integrals. Variables can be changed by substitution or by changing the co-ordinate system (polar, spherical or cylindrical coordinates). Integrals can also be solved easily by expressing them in terms of beta and gamma functions. Multiple integrals are useful in evaluating plane area, mass of a lamina, mass and volume of solid regions, etc.

8.2 DOUBLE INTEGRAL Let f (x, y) be a continuous function defined in a closed and bounded region R in the xy-plane. Divide the region R into small elementary rectangles by drawing lines parallel to co-ordinate axes. Let the total number of complete rectangles which lie inside the region R is n. Let Ar be the area of rth rectangle and (xr, yr) be any point in this rectangle. n Consider the sum S= f ( xr , yr ) Ar ... (1) r =1

where Ar = xr · yr. If we increase the number of elementary rectangles, i.e., n, then the area of each rectangle decreases. Hence, as n , Ar 0. The limit of the sum given by the Eq. (1), if it exists, is called the double integral of f (x, y) over the region R and is denoted by f ( x, y ) dA. R

Hence,

∫∫ R

where dA = dx dy

f ( x, y ) dA = lim

n

∑ f ( x , y )δA

n→ ∞ δAr → 0 r =1

r

r

r

Fig. 8.1

8.2

Engineering Mathematics

8.2.1 Evaluation of Double Integral Double integral of a function f (x, y) over region R can be evaluated by two successive integrations. There are two different methods to evaluate a double integral.

Method-I Let the region R, i.e., PQRS is bounded by the curves y = y1(x), y = y2(x) and the lines x = a, x = b. In the region PQRS, draw a vertical strip AB. Along the strip AB, y varies from y1 to y2 and x is fixed. Therefore, the double integral is integrated first w.r.t. y between the limits y1 and y2 treating x as constant. Now, move the strip AB from PS (i.e., x = a) to QR (i.e., x = b) to cover the entire region PQRS. The result of the first integral is integrated w.r.t. x between the limits a and b. Hence, f ( x , y ) dx dy = R

b

y2

a

y1

f ( x , y ) dy dx Fig. 8.2

Method-II Let the region R is bounded by the curves x = x1(y), x = x2(y) and the lines y = c, y = d. In the region PQRS, draw a horizontal strip AB. Along the strip AB, x varies from x1 to x2 and y is fixed. Therefore, the double integral is integrated first w.r.t. x between the limits x1 and x2 treating y as constant. Now, move the strip AB from PQ (i.e., y = c) to RS (i.e., y = d ) to cover the entire region PQRS. The result of the first integral is integrated w.r.t. y between the limits c and d. f ( x, y ) dx dy =

Hence, R

d

x2

c

x1

Fig. 8.3

f ( x, y ) dx dy

Note: (i) If all the four limits are constant, then the function f (x, y) can be integrated w.r.t. any variable first. But if f (x, y) is implicit and is discontinuous within or on the boundry of the region of integration, then the change of the order of integration will affect the result. (ii) If all the four limits are constant and f (x, y) is explicit, then double integral can be written as product of two single integrals.

Multiple Integral

Example 1: Evaluate Solution:

3

1

0

0

3

1

0

0

8.3

( x 2 + 3 y 2 ) dy dx .

( x 2 + 3 y 2 )dy dx =

3 0

1

x 2 y + y 3 0 dx 3

3

=

0

( x 2 + 1) dx =

x3 +x 3 0

= 12 x

1

Example 2: Evaluate

0

0

Solution:

y x

e dy dx . 1

x

0

0

y

1

e x dy dx =

0

y x

xe x dx 0 1

1 0

x(e 1) dx

x2 (e 1) 2 0

1 (e 1) 2

⎡ 1+ x 2 Solution: ∫0 ⎢ ∫0 ⎢ ⎣

1+ x2

1

Example 3: Evaluate

0

0

dx dy . 1 + x2 + y2

1+ x ⎤ 1 y 1 −1 ⎥ dx = 2 ∫0 1 + x 2 tan 1 + x 2 dx ⎥ 0 1+ x2 ) + y2 ⎦ 1 1 (tan 1 1 tan 1 0) dx 0 1+ x2 1 1 . dx = log x + 1 + x 2 = 0 2 4 4 1+ x 2

dy

1

(

(

=

Solution:

0

0

1 y2 2

1 0

1- y2 2

1

Example 4: Evaluate

0

4

dx dy 1 - x2 - y2 dx dy

1 x

2

y

2

=

=

log (1 + 2) .

1 y2 2

1 0

1 0

0

sin

dx (1 y 2 ) x 2

1

1 y2 2

x 1 y2

dy 0

dy

)

1 0

8.4

Engineering Mathematics

1⎛ 1 ⎞ = ∫ ⎜ sin −1 − sin −1 0 ⎟ dy 0⎝ ⎠ 2 1

= Example 5: Evaluate

Solution:

cos 2

4 0

1 2 1 2

0

4 0

4

0

1 2r dr d 2 (1 + r 2 ) 2

cos 2

4 0

y0 =

r d dr . (1 + r 2 )2

cos 2

4 0

4

0

(1 r 2 )

(1 r 2 )

2

cos 2

1 0

2 r dr d

⎡ ⎤ n +1 ⎥ ⎢ n f ( r ) [ ] ⎢∵ [ f ( r ) ] f ′ ( r )dr = ⎥ ⎢ ∫ ⎥ n +1 ⎢ ⎥ n ≠ −1 ⎣ ⎦

d

=−

1 π4 ⎛ 1 ⎞ − 1⎟ dθ ⎜ 2 ∫0 ⎝ 1 + cos 2θ ⎠

=−

1 π4 ⎛ 1 1 π4 ⎛ 1 2 ⎞ ⎞ 1 θ − d = − ⎜⎝ ⎟⎠ ⎜⎝ sec θ − 1⎟⎠ dθ 2 ∫ ∫ 0 0 2 2 2 2 cos θ

1 1 =− tan θ − θ 2 2

π 4 0

1 ⎛1 π π ⎞ = − ⎜ tan − − 0 ⎟ ⎝ ⎠ 2 2 4 4

1 = (π − 2) 8 Example 6: Evaluate

e 0

0

x3

6

y 4 e - y dx dy . x

Solution: Since both the limits are constant and integrand is explicit in x and y, integral can be written as, I Let x 3 = p,

0

e

x3

1

( x ) 2 dx

1 − 32 p dp, 3 When x = 0, p = 0 x → ∞, p → ∞ dx =

e

y6

y 4 dy

y6 = q

1

x = p3 ,

0

1

y = q6 1 − 65 q dq 6 When y = 0, q = 0 dy =

y → ∞, q → ∞ ∞

I = ∫ e− p p 0



=



1 6



4 1 −2 1 −5 ⋅ p 3 d p ⋅ ∫ e − q q 6 ⋅ q 6 dq 3 6 0 ∞

5 1 −1 −1 1 e− p p 6 d p ⋅ ∫ e−q q 6 d q ∫ 18 0 0

Multiple Integral

1 1 18 6

8.5

5 6

⎡∵ ∞ e − x x n −1dx = n⎤ ⎥⎦ ⎣⎢ ∫0

1 5 5 1 18 6 6 1 = . 18 sin 5 6 = Example 7: Show that

⎡ ⎤ ⎢∵ n 1− n = sin n ⎥ ⎣ ⎦

9 1

0

dx

1 0

x- y dy ( x + y )3

1 0

1

dy

0

x- y dx . ( x + y )3

Solution: Consider, L.H.S =

1

dx

0

R.H.S =

2x ( x y )3

1

1

0

0

1

x ( x + 1) 2

0

=

x y dy = 0 ( x + y )3 1

1 0

dx

1 (x

0

2 x ( x y) dy ( x + y )3 1

dy dx

y)2

1 1 x +1 x

1

1 dx x

0

1 2( x y ) 2

2x

1

1 dx x+ y 0 1

1

1

0

( x + 1)2

dx

1 x +1 0

1 2 1 0

dy

1

1

0

0

1 0

1 0

1 0

x y dx = ( x + y )3

dy

1 0

( x y) 2 y dx ( x + y )3

2y dx dy ( x + y )3

1 ( x + y)2 1 x+ y

1 0

1

y dy ( x + y)2 0

1 dy (1 + y ) 2

1 1+ y

1 0

1 1+ y

y (1 + y ) 2

1 y

1 dy y

1

0

1 2 Hence,

1 0

dx

x y dy 0 ( x + y )3 1

1 0

dy

1 0

x y dx ( x + y )3

x y is discontinuous at (0, 0), a point on the boundary of the region (square), ( x + y )3 change of order of integration does not give the same result. Since

8.6

Engineering Mathematics

Exercise 8.1 Evaluate the following: 1.

2

2 y

1

2 y

2 x 2 y 2 dx dy

5. Ans. :

2.

1 0

y 0

xy e

dx dy 1 Ans. : 4e

3.

1

x

0

0

10 0

ye xy dx dy [Ans. : 9(1 e)]

856 945 6.

x 2

1 y

1

log8

log y

1

0

e x + y dx dy [Ans. : 8(log8 1)]

7.

1

y

0

y2

(1 + xy 2 ) dx dy

e x + y dx dy 1 Ans. : (e 1) 2 2

4.

2 0

a (1+ sin ) 0

8.

2

r cos d dr

0

Ans. :

5a3 4

2 ax x 2

2a 0

Ans. :

41 210

Ans. :

2a 4 3

xy dy dx

8.2.2 Working Rule for Evaluation of Double Integral 1. If a region is bounded by more than one curve, then find the points of intersection of all the curves. 2. Draw all the curves and mark their points of intersection. 3. Identify the region bounded by all the curves. 4. Draw a vertical or horizontal strip in the region whichever makes the integration easier. 5. Find the variation of y (or x) along the strip and variation of x (or y) in the region. 6. Write the limits of y and x. Lower limit is always obtained from the curve where the strip starts and upper limit is always obtained from the curve where it terminates. 7. The function is integrated first along the strip (i.e., w.r.t. y first for vertical strip and w.r.t. x for horizontal strip.) 8. Variation along vertical strip is always taken from lower part to upper part and along horizontal strip is always taken from left part to right part of the region. 9. If variation along the strip changes within the region, then the region is divided into parts. Example 1: Evaluate ax + by = 1.

e ax + by dx dy , over the triangle bounded by x = 0, y = 0,

Multiple Integral

8.7

Solution: 1. The region of integration is the OPQ. 2. The integration can be done w.r.t. any variable first. Draw a vertical strip AB parallel to y-axis which starts from x-axis and terminates on the line ax + by = 1. 1 ax 3. Limits of y : y = 0 to y = b 1 Limits of x : x = 0 to x = a 1

I = ∫ ∫ e ax + by dx dy = ∫ a e ax ∫ 0

1

= ∫ a e ax 0

1− ax by b

e b

=

0

1− ax b

0

Fig. 8.4

e by dy dx

1 1a ax (1− ax ) e ⎡⎣e − 1⎤⎦ dx b ∫0

1 1 1 e ax = ∫ a (e − e ax )dx = ex − b 0 b a

1 a

=

0

1 ⎛e e 1⎞ ⎜ − + ⎟ b ⎝a a a⎠

1 = ab xy

Example 2: Evaluate x 2 + y 2 = 1.

1 - y2

dx dy over the first quadrant of the circle

Solution: 1. The region of integration is OPQ. 2. The integration can be done w.r.t. any variable first. Draw a vertical strip AB parallel to y-axis which starts from x-axis and terminates on the circle x 2 + y 2 = 1. 3. Limits of y : y = 0 Limits of x : x = 0 xy

I=

1 y2 1 0

x

1 2

1 x2 0

1 0

to y 1 x2 to x = 1

dx dy 1 1 (1 y 2 ) 2 ( 2 y )dy dx 2 1 x2

1 2 2

x 2(1 y )

dx 0

Fig. 8.5 ⎡ [ f ( x )]n +1 ⎤ n ∵ [ f ( x )] f ( x ) d x = ′ ⎢ ∫ ⎥ n +1 ⎦ ⎣

8.8

Engineering Mathematics

1 2

1 0

2 x ( x 1) dx

1 3 =

x3 3

x2 2

1

0

1 2

1 6 ( a - x )2 dx dy , over the right half of the circle x 2 + y 2 = a 2 .

Example 3: Evaluate Solution:

1. The region of integration is PQR. 2. The integration can be done w.r.t. any variable first. Draw a vertical strip AB parallel to y-axis which starts from the part of the circle x 2 + y 2 = a 2 below x-axis and terminates on the part of the circle x 2 + y 2 = a 2 above x-axis. 3. Limits of a2

y:y

x 2 to y

Limits of x : x = 0

a2

x2

to x = a

I = ∫ ∫ ( a − x ) 2 dx dy a

a2 − x 2

0

− a2 − x 2

= ∫ (a − x)2 ∫

dy dx

a

a2 − x 2

0

− a2 − x 2

= ∫ (a − x)2 y

dx

a

= ∫ ( a 2 + x 2 − 2ax ) 2 a 2 − x 2 dx 0

a

= 2 ∫ ( a 2 + x 2 − 2ax ) a 2 − x 2 dx

Fig. 8.6

0

Putting x = a sinq, dx = a cosq dq When x = 0, q =0 =

x = a,

2

π

I = 2 ∫ 2 ( a 2 + a 2 sin 2 θ − 2a 2 sin θ ) a cosθ ⋅ a cosθ dθ 0

π

= 2a 4 ∫ 2 (cos 2 θ + sin 2 θ cos 2 θ − 2 sin θ cos 2 θ )dθ 0

Multiple Integral

⎡ 3 1 3 3 3⎤ 1 ⎥ ⎢ = a4 ⎢ 2 2 + 2 2 − 2 2 ⎥ 3 5 ⎥ ⎢ 2 ⎢⎣ 2 ⎥⎦

∵2

a4

2 5 8

8

sin p cos q d p +1 q +1 , 2 2

p +1 q +1 2 = 2 p+q+2 2

4 3

4 3 x2 y2 xy 2 + 2 a b

Example 4: Evaluate

2 0

=B

2 ⎡ ⎤ ⎛1 1⎞ 3 ⎥ ⎢1 1 1 ⎜ ⎟ ⎢ ⎥ ⎝2 2⎠ = a4 ⎢ 2 2 2 + −2 2 ⎥ 1 2! 3 3⎥ ⎢ ⎢ 2 2⎥ ⎣ ⎦

a4

8.9

n 2

dx dy, over the first quadrant of the ellipse

x2 y2 + = 1. a 2 b2 Solution: 1. The region of integration is OPQ. 2. The integration can be done w.r.t. any variable first. Draw a vertical strip AB parallel to y-axis which starts from x-axis and 2 2 terminates on the ellipse x + y = 1. 2 2 a b x2 3. Limits of y : y = 0 to y b 1 2 a Limits of x : x = 0 to x = a n

⎛ x2 y2 ⎞ 2 I = ∫ ∫ xy ⎜ 2 + 2 ⎟ dx dy b ⎠ ⎝a a

b 1−

0

0

= ∫ x∫

x2 a2

Fig. 8.7 n 2

b2 ⎛ x 2 y 2 ⎞ 2 y + dy dx 2 ⎜⎝ a 2 b 2 ⎟⎠ b 2 b 1− n

b2 = 2



a

0

⎛ x2 y2 ⎞ 2 + x ⎛ n ⎞ ⎜⎝ a 2 b 2 ⎟⎠ ⎜⎝ + 1⎟⎠ 2 1

x2 a2

+1

dx 0

⎡ [ f ( y)]n+1 ⎤⎥ n ⎢∵ ∫ [ f ( y ) ] f ′( y ) dy = n + 1 ⎥⎦ ⎢⎣

8.10

Engineering Mathematics

b2 ( n + 2)

a 0

b2 a2 n+2 2 =

x a

x 1 1 an+ 2

n+ 2

b2 x 2 ( n + 2) 2

dx

an+ 4 n+4

1 an+ 2

a

x n+ 4 n+4 0

a 2 b 2 ( n + 2) ( n + 2) 2( n + 4)

a2 b2 2( n + 4)

Example 5: Evaluate

( x 2 + y 2 ) dx dy over the ellipse 2 x 2 + y 2 = 1.

Solution: 1. The region of integration is PQRS, the ellipse 2 x 2 + y 2 = 1 or

x2 1 2

1 2

2

+

y2 = 1 with 12

and 1 as axes.

2. The integration can be done w.r.t. any variable first. Draw a vertical strip AB parallel to y-axis which starts from the part of the ellipse 2 x 2 + y 2 = 1 below x-axis and terminates on the part of the ellipse 2 x 2 + y 2 = 1 above x-axis.

Fig. 8.8 3. Limits of y : y Limits of x : x

1 2 x 2 to y 1 2

to x

1 2

1 2x 2

Multiple Integral

1

I=

( x 2 + y 2 ) dx dy =

2 1

1 2 x2 1 2 x2

8.11

( x 2 + y 2 ) dy dx

2 1

y3 3

x2 y

2 1 2

1 2 x2

1 2 x2

1

4

2 0

x2 1 2x2

Putting 2 x 2 = t , x = When

1

dx

2 1

2 x2 1 2x2

2

3 1 (1 2 x 2 ) 2 dx 3

3 1 (1 2 x 2 ) 2 dx 3

t 1 , dx = dt 2 2 2 t

x = 0, t = 0 1 x= , t =1 2 3 1⎡ t ⎤ 1 1 I = 4 ∫ ⎢ 1 − t + (1 − t ) 2 ⎥ dt 0 2 3 ⎣ ⎦2 2 t 3 1 1 1⎡1 ⎤ ⎡ 1 ⎛ 3 3 ⎞ 1 ⎛ 1 5 ⎞⎤ 1 −1 = 2 ∫ ⎢ t 2 (1 − t ) 2 + t 2 (1 − t ) 2 ⎥ dt = 2 ⎢ B ⎜ , ⎟ + B ⎜ , ⎟ ⎥ 0 2 3 ⎣ 2 ⎝ 2 2 ⎠ 3 ⎝ 2 2 ⎠⎦ ⎣ ⎦

⎡ ⎛ 1 1 ⎞2 ⎤ ⎡ 1 3 1 1⎥ 3 3 1 5⎤ ⎢ ⎜ ⎟ ⋅ ⋅ ⎢1 ⎥ ⎢1 ⎝ 2 2 ⎠ 1 2 2 2 2 ⎥ 1 = 2⎢ ⋅ 2 2 + ⋅ 2 2⎥ = 2⎢ ⋅ + ⋅ ⎥ 2 2 3 3 3 ⎥ ⎢2 3 ⎢2 ⎥ ⎢⎣ ⎥⎦ ⎢ ⎥ ⎣ ⎦ ⎤ 3 2 ⎡1 = 2⎢ ⋅ + ⎥= 4 4 8 16 ⎦ ⎣ Example 6: Evaluate (1, 1), (1, 2).

( x 2 - y 2 ) dx dy over the triangle with vertices (0, 1),

Solution: 1. The region of integration is PQR. 2. Equation of the line PQ is y = 1. Equation of the line PR is 2 1 y 1 ( x 0) x 1 0 y = x +1 3. The integration can be done w.r.t. any variable first. Draw a vertical strip AB parallel to y-axis which starts from the line y = 1 and terminates on the line y = x + 1.

Fig. 8.9

8.12

Engineering Mathematics to y = x + 1 to x = 1

4. Limits of y : y = 1 Limits of x : x = 0

I = ∫ ∫ ( x 2 − y 2 ) dx dy = ∫

1



x +1

0 1

=∫

2

0

1

1⎡ ( x + 1)3 1⎤ dx = ∫ ⎢ x 2 ( x + 1) − − x 2 + ⎥ dx 0 3 3⎦ ⎣ 1

x x ( x + 1) 4 x 3 x 1 1 16 1 + − − + = + − + 4 3 12 3 3 0 4 3 12 12 4

=

x +1

y3 x y− 3

1

( x 2 − y 2 ) dy dx

=−

3

2 3 2

e y dx dy over the region bounded by the triangle with

Example 7: Evaluate vertices (0, 0), (2, 1), (0, 1). Solution:

1. The region of integration is OPQ. 2. Equation of the line OQ is x y = or x = 2 y. 2 3. Here, it is easier to integrate w.r.t. x first than with y. Draw a horizontal strip AB parallel to x-axis which starts from y-axis and terminates on the line x = 2y. 4. Limits of x : x = 0 to x = 2y Limits of y : y = 0 to y = 1 1

2

e y dx dy =

0 1 0

ey

2

2y 0

dx dy =

Fig. 8.10 1 0

2

2y

e y x 0 dy

2

e y 2 y dy

= ey

2

1

⎡ ∵ e f ( y ) f ′ ( y ) dy = e f ( y ) ⎤ ⎣ ∫ ⎦

0

e 1 Example 8: Evaluate (0, 0), (1, 1) and (0, 1).

2 xy 5 1 + x2 y2 - y4

dx dy over the triangle having vertices

Multiple Integral

8.13

Solution: 1. The region of integration is the OPQ. 2. Equation of the line OP is y = x. 3. Here, it is easier to integrate w.r.t. x first than with y. Draw a horizontal strip AB parallel to x-axis which starts from y-axis and terminates on the line y = x. 4. Limits of x : x = 0 to x = y Limits of y : y = 0 to y = 1 2 xy 5

I=

1 x2 y2 1 0 1 0

y3

y 0

y4

(1 x 2 y 2

dx dy y4 )

1 2

1 y 4 2

y 3 2(1 x 2 y 2

y )

2 xy 2 dx dy

Fig. 8.11 n

∵ [ f ( x )] f ( x )dx =

dy

0

[ f ( x )]n +1 n +1

1 ⎡ ⎤ = ∫ y 3 ⋅ 2 ⎢1 − (1 − y 4 ) 2 ⎥ dy 0 ⎣ ⎦ 1

1

1

1

= ∫ 2 y 3 dy − 2 ∫ (1 − y 4 ) 2 y 3 dy 0

0

4 1

=2 =

y 4

1

1

− 2 ∫ (1 − y 4 ) 2 y 3 dy 0

0

1 1 1 1 + ∫ (1 − y 4 ) 2 ( − 4 y 3 ) dy 2 2 0

3 1 1 2 + (1 − y 4 ) 2 2 2 3 1 1 1 = − = 2 3 6

1

⎡ [ f ( x )]n +1 ⎤ n ⎥ ⎢∵ ∫ [ f ( x )] f ′( x )d x = n +1 ⎦ ⎣

=

Example 9: Evaluate

0

y dx dy ( a - x ) ax - y 2

over the region bounded by the

parabola y 2 = x and the line y = x. Solution: 1. The region of integration is OPQ. 2. The points of intersection of y 2 = x and y = x are obtained as x2 = x x( x 1) 0 x = 0, 1 and y = 0, 1 Hence O : (0, 0) and P : (1, 1)

8.14

Engineering Mathematics

3. Here, it is easier to integrate w.r.t. y first than with x. Draw a vertical strip AB parallel to y-axis, which starts from the line y = x and terminates on the parabola y 2 = x. 4. Limits of y : y = x to y = x Limits of x : x = 0 to x = 1. I = ∫∫

y dx dy ( a − x ) ax − y 2

1 x ⎛ 1⎞ − 1 − ⎟ ( ax − y 2 ) 2 ( −2 y )dy dx ⎜ ∫ 0 (a − x) x ⎝ 2⎠

=∫

1

=−

1 1 1 1 2 ( ax − y 2 ) 2 ∫ 2 0 (a − x)

Fig. 8.12 ⎡ [ f ( y)]n+1 ⎤⎥ n ⎢∵∫ [ f ( y ) ] f ′( y )dy = n + 1 ⎥⎦ ⎢⎣

x

dx x

1 1 ⎡ 2 2⎤ − − − ( ax x ) ( ax x ) ⎢ ⎥ dx 0 (a − x) ⎣ ⎦

= −∫

1

= −∫

1

0

1 2

x a− x

(

)

a − 1 − a − x dx

Putting x = a sin 2 , dx = 2a sin cos d When

x = 0, q = 0 x = 1, sin −1

= sin

1

1 a

1

(

a sin a cos 2 1 sin −1 sin 2 a = −2 a ∫ 0 cos

I = −∫

a

0

= −2 a ∫

sin −1

1 a

0

= −2 a ∫

sin −1

0

1 a

a − 1 − a cos

(

) 2a sin

a − 1 − a cos

cos d

)d

⎡ ⎛ 1 − cos 2 ⎞ ⎤ a − 1 − a sin 2 ⎥ d ⎢⎜ ⎟ ⎠ ⎣ ⎝ cos ⎦ ⎡ a (1 − cos 2 ⎢ a − 1 (sec − cos ) − 2 ⎣

⎤ )⎥ d ⎦ sin −1

= −2 a

= −2 a

a a sin 2 a − 1 [log (sec + tan ) − sin ] − + 2 4 ⎤ ⎡ ⎛ 1 + sin ⎞ a a sin cos + a − 1 ⎢log ⎜ ⎟⎠ − sin ⎥ − ⎝ cos 2 2 ⎦ ⎣

1 a

0 sin −1

0

1 a

Multiple Integral

8.15

⎤ ⎡ ⎛ ⎞ 1 1 a sin −1 ⎥ ⎢ ⎜ 1+ a ⎟ 1 1 1 a a = −2 a ⎢ a − 1 ⎜ log − + ⋅ ⋅ 1− ⎥ ⎟− ⎢ 2 2 a⎥ 1 a⎟ a ⎜ 1− ⎜⎝ ⎟⎠ ⎢ ⎥ a ⎦ ⎣ = −2 a( a − 1) log

a +1 a −1

Example 10: Evaluate

+ a − 1 + a sin −1

1 a

y dx dy over the region enclosed by the parabola

x2 = y and the line y = x + 2. Solution: 1. The region of integration is POQ. 2. The points of intersection of x 2 = y and y = x + 2 are obtained as x 2 x 2, x 2 x 2 0 ( x 2) ( x 1) x

0 2, 1and y

4, 1

Hence, P : (–1, 1) and Q : (2, 4) Fig. 8.13 3. Here, integration can be done w.r.t. any variable first. Draw a vertical strip AB parallel to y-axis which starts from the parabola x 2 = y and terminates on the line y = x + 2. 4. Limits of y : y = x 2 to y = x + 2 Limits of x : x = 1 to x = 2 I = ∫ ∫ y dx dy = ∫ =∫ =

2 −1

y2 2

2



x+2

−1 x 2

x+2

dx = x2

1 ( x + 2)3 x 5 − 2 3 5

y dy d x

1 2 ⎡( x + 2) 2 − x 4 ⎤⎦ dx 2 ∫−1 ⎣ 2

= −1

1 ⎛ 64 32 1 1 ⎞ ⎜ − − − ⎟⎠ 2⎝ 3 5 3 5

36 = 5 Example 11: Evaluate 2

xy ( x + y ) dx dy , over the region enclosed by the par-

2

abolas x = y , y = - x . Solution: 1. The region of integration is OPQ. 2. The points of intersection of the parabola x 2 = y, and y 2 = − x are obtained as y 4 y, y 0, 1 and x 0, 1. Hence, O : (0, 0) and Q : ( 1, 1)

8.16

Engineering Mathematics

Fig. 8.14 3. Here, it is convenient to integrate w.r.t. x first. Draw a horizontal strip AB parallel to x-axis, which starts from the parabola x 2 = y and terminates on the parabola y2 x. 4. Limits of x : x = − y to x = − y 2 Limits of y : y = 0 to y = 1 I = ∫ ∫ xy ( x + y ) dx dy 1

− y2

1

x3 x 2 y = ∫ y ∫ ( x + xy ) d x d y = ∫ y + 3 2 0 − y 0

− y2

2

1

=∫ 0

dy − y

5 7 ⎛ 7 6 3 ⎞ 8 7 2 2 − y y y y y y 2 y y4 ⎜ + + − ⎟ dy = − + + − ⎝ 3 2 3 2⎠ 24 14 21 8

Example 12: Evaluate

1

=0 0

xy dx dy over the region enclosed by the x-axis, the

line x = 2a and the parabola x 2 = 4ay. Solution: 1. The region of integration is OPQ. 2. The point of intersection of the parabola x 2 = 4 ay and the line x = 2a is obtained as 4 a 2 = 4 ay, y = a. Hence, Q : (2a, a) 3. Here, integration can be done w.r.t. any variable first. Draw a vertical strip AB parallel to y-axis, which starts from x-axis and terminates on the parabola x 2 = 4ay.

Fig. 8.15

Multiple Integral

8.17

x2 4a to x = 2a

4. Limits of y : y = 0 to y = Limits of x : x = 0

2a

I = ∫ ∫ xy dx dy = ∫ x ∫ 0

x2 4a

0

y d y dx

2

=∫

2a

0

y2 x 2

1 x6 = 32a 2 6

x 4a

2a

dx = ∫ x ⋅ 0

0 2a

= 0

x4 dx 32 a 2

1 64 a6 ⋅ 32a 2 6

a4 = 3 Example 13: Evaluate 2

xy dx dy , over the region enclosed by the circle

2

x + y - 2 x = 0, the parabola y 2 = 2 x and the line y = x. Solution: 1. The region of integration is OPQRO. 2. The points of intersection of (i) the circle x 2 y 2 2 x 0 and the line y = x are obtained as x 2 + x 2 − 2 x = 0, x = 0, 1 and y = 0, 1 Hence, O : (0, 0) and P : (1, 1) (ii) the circle x 2 y 2 2 x 0 and the parabola y 2 = 2 x are obtained as x 2 2 x 2 x 0, x 0 and y = 0 Hence, O : (0, 0) (iii) the parabola y 2 = 2 x and the line y = x are obtained as x 2 = 2 x, x = 0, 2 and y = 0, 2 Hence, O : (0, 0) and Q : (2, 2). Fig. 8.16 3. Here, integration can be done w.r.t. to any variable first. To integrate w.r.t. y, first we need to draw vertical strip in the region. But one vertical strip does not cover the entire region, therefore, we divide the region OPQRO into two subregions OPR and RPQ and draw one vertical strip in each subregion. 4. In the subregion OPR strip starts from the circle x 2 y 2 2 x 0 and terminates on the parabola y 2 = 2 x . 2 x x 2 to y 2x Limits of y : y Limits of x : x = 0 to x = 1. 5. In the subregion RPQ, strip starts from the line y = x and terminates on the parabola y 2 = 2 x.

8.18

Engineering Mathematics

Limits of y : y = x to y = 2 x Limits of x : x = 1 to x = 2 I=

xy dx dy =

xy dx dy + OPR

=

1 0

=

1 0

2x

x

2 x x2

y2 2

x

1 2

1 0

2

y dy dx +

1

x

2x

dx + 2 x x2

2 1

x

x(2 x 2 x x 2 )dx

1 x4 2 4

1

0

1 2x3 2 3

xy dx dy

RPQ

x4 4

2

1

2x x

y dy dx

2x

y2 2

dx x

1 2

2 1

x(2 x x 2 )dx

1 8 1 1 2 8 3 3 8

7 = 12 Example 14: Evaluate

x 2 dx dy , over the region in the first quadrant enclosed

by the rectangular hyperbola xy = 16, the lines y = x, y = 0 and x = 8. Solution: 1. The region of integration is OPQR. 2. The point of intersection of (i) the hyperbola xy = 16 and the line y = x is obtained as x 2 = 16, x = ±4, and y = ±4 Hence, R : (4, 4) in the first quadrant. (ii) the hyperbola xy = 16 and line x = 8 is obtained as 8y = 16, y = 2 Hence, Q : (8, 2) 3. Here, integration can be done w.r.t. any variable first. To integrate w.r.t. y, first we need to draw a vertical strip in Fig. 8.17 the region. But here one vertical strip can not cover the entire region therefore we divide the region OPQR into two subregions OMR and RMPQ and draw one vertical strip in each subregion. 4. In subregion OMR, strip starts from x axis and terminates on the line y = x. Limits of y : y = 0 to y = x Limits of x : x = 0 to x = 4 5. In subregion RMPQ, strip starts from x axis and terminates on the rectangular hyperbola xy = 16 16 Limits of y : y = 0 to y = x Limits of x : x = 4 to x = 8

Multiple Integral

I = ∫ ∫ x 2 dx dy =

8.19

∫∫ x dx dy + ∫∫ 2

OMR

x 2 d x dy

RMPQ

4

x

8

16 x

0

0

4

0

= ∫ x 2 ∫ dy dx + ∫ x 2 ∫ dy dx 4

8

16 x

4

0

= ∫ x 2 y 0 dx + ∫ x 2 y x

0

dx 4

=∫

4

0

x4 x2 16 x dx + ∫ x ⋅ d x = + 16 4 x 4 0 2 3

8

8

2

4

= 64 + 8 (64 − 16) = 448 Example 15: Evaluate

dx dy , over the region bounded by the y x4 + y2

Solution: 1. The region of integration is bounded by the line y x 2 i.e., the region above the parabola x 2 = y and x ≥ 1, i.e., the region on the right of line x = 1. 2. The point of intersection of x 2 = y and x = 1 is obtained as 1 = y. Hence, P : (1, 1) 3. Here, it is easier to integrate w.r.t. y first than x. Draw a vertical strip AB parallel to y-axis in the region which starts from the parabola x 2 = y and extends up to infinity. 4. Limits of y : y = x 2 to y Limits of x : x = 1 to x

I = ∫∫ =∫



=∫



1

1

=

4

Fig. 8.18

∞ ∞ dx dy 1 =∫ ∫2 4 dy dx 4 2 1 x x +y x + y2 ∞

∞ 1 1 y tan −1 2 dx = ∫ 2 (tan n −1 ∞ − tan −1 1) dx 2 1 x x x2 x ∞

1 ⎛ 1 ⎞ ⎜ − ⎟ dx = − x2 ⎝ 2 4 ⎠ 4 x1

x2 , x

1.

8.20

Engineering Mathematics

Exercise 8.2 Evaluate the following: 1 dx dy, over the rectangle 1. xy 1

x

2, 1

y

Ans. :

2. [ Ans. : (log 2) 2 ]

2.

1 ab

4.

xy 1 x

10.

gle bounded by x = 0, y = 0 and x + y = 1.

5.

xy

16 945

11.

12.

( x + y + a) dx dy, over the region

7.

a3 ]

xy dx dy, over the region bounded by the x-axis, the line y = 2x and the x2 parabola y = . 4a

( x 2 + y 2 ) dx dy, over the area bounded by the lines y = 4x, x + y = 3, y = 0, y = 2. 463 48

xy( x + y ) dx dy, over the region bounded by the parabolas y2 = x, x2 = y. 3 Ans. : 28 xy ( x + y ) dx dy, over the region bounded by the curve x2 = y and the line x = y. 3 Ans. : 56

bounded by the circle x 2 + y 2 = a 2 . [ Ans. :

1 3 + 3 3 2

Ans. :

y 2 dx dy, over the triangle

having vertices (0, 0), (10, 1), (1, 1). [Ans. : 6] 6.

y 2 ) 2 dx dy, over the trian-

gle bounded by x-axis, the line y = x and x = 1.

y dx dy, over the trian-

Ans. :

(4 x 2

Ans. :

4e 3 1)

217 60

1

9.

bounded by the lines x = 0, y = 0, and x + y = 1. 1 (3e 4 12

y ) dx dy, over the region

Ans. :

e 3 x + 4 y dx dy, over the triangle

Ans. :

(5 2 x

bounded by x-axis, the line x + 2y = 3 and the parabola y 2 = x.

sin ( ax + by ) dx dy, over the triangle bounded by the lines x = 0, y = 0 and ax + by = 1. Ans. :

3.

8.

2048 4 a 3

13.

xy( x 1) dx dy, over the region bounded by the rectangular hyperbola xy = 4, the lines y = 0, x = 1, x = 4 and x-axis. [Ans. : 8 (3 log 4)]

Multiple Integral

8.21

8.3 CHANGE OF ORDER OF INTEGRATION Sometimes, evaluation of double integral becomes easier by changing the order of integration. To change the order of integration, first, we draw the region of integration with the help of the given limits. Then we draw vertical or horizontal strip as per the required order of integration. This change of order also changes the limits of integration.

(I) Change the Order of Integration of the Following Example 1:

1 0

x x

f ( x , y )dy dx .

Solution: 1. The function is integrated first w.r.t. y and then w.r.t. x. 2. Limits of y : y = x to y = x . Limits of x : x = 0 to x = 1 3. The region is bounded by the line y = x and the parabola y 2 = x. 4. The points of intersection of y 2 = x and y = x are obtained as x 2 = x, x = 0, 1 and y = 0, 1 Hence, O : (0,0) and Q : (1, 1) 5. To change the order of integration, i.e., to integrate first w.r.t. x, draw a horizontal Fig. 8.19 strip AB parallel to x-axis which starts from 2 the parabola y = x and terminates on the line y = x. Limits of x : x = y 2 to x = y Limits of y : y = 0 to y = 1 Hence, the given integral after change of order can be written as 1 0

x x

f ( x, y ) dy dx =

1

y

0

y2

f ( x, y ) dx dy

1

Example 2:

1

y3

0

y2

f ( x , y ) dx dy .

Solution: 1. The function is integrated first w.r.t. x and then w.r.t. y. 1

2. Limits of x : x = y 2 to x = y 3 Limits of y : y = 0 to y = 1 3. The region is bounded by the parabola y 2 = x and the cubical parabola y = x 3 . 4. The points of intersection of y2 = x and y = x3 are obtained as x6 = x, x = 0, 1 and y = 0, 1. Hence, O (0, 0) and Q : (1, 1)

Fig. 8.20

8.22

Engineering Mathematics

5. To change the order of integration, i.e., to integrate first w.r.t. y, draw a vertical strip parallel to y-axis which starts from the cubical parabola y = x 3 and terminates on the parabola y 2 = x. Limits of y : y = x 3 to y = x Limits of x : x = 0 to x = 1 Hence, the given integral after change of order can be written as 1

Example 3:

8 0

y 4 y 8 4

1

y3

0

y2

f ( x, y ) dx dy =

1 0

x x3

f ( x, y ) dy dx

f ( x , y ) dx dy .

Solution: 1. The function is integrated first w.r.t. x and then w.r.t. y. y −8 y 2. Limits of x : x = to x = 4 4 Limits of y : y = 0 to y = 8 3. The region is bounded by the line y = 4x + 8, y = 4x, y = 8 and x-axis (y = 0). 4. The point of intersection of y = 4x and y = 8 is obtained as 8 = 4x, x = 2. Hence, P : (2, 8). 5. To change the order of integration, i.e., to integrate first w.r.t. y, divide the region OPQR into two subregions OQR and OPQ. Draw a vertical strip parallel to y-axis in Fig. 8.21 each subregion. (i) In subregion OQR, strip AB starts from x-axis and terminates on the line y = 4x + 8. Limits of y : y = 0 to y = 4x + 8 Limits of x : x = 2 to x = 0 (ii) In subregion OPQ, strip CD starts from the line y = 4x and terminates on the line y = 8. Limits of y : y = 4x to y = 8 Limits of x : x = 0 to x = 2 Hence, the given integral after change of order can be written as 8 0

Example 4:

y 4 y 8 4

f ( x, y ) dx dy =

y2 a -a 0 a

0

4 x +8

2 0

f ( x, y ) dy dx +

2

8

0

4x

f ( x , y ) dx dy .

Solution: 1. The function is integrated first w.r.t. x and then w.r.t. y.

f ( x, y ) dy dx

Multiple Integral

8.23

y2 . a Limits of y : y = a to y = a. 3. The region is bounded by the parabola y 2 = ax, the y-axis and the line y = a and y = a. 4. The point of intersection of 2. Limits of x : x = 0 to x =

(i) y 2

ax and y

a is obtained as a 2

ax, x

a

Hence, R : (a, a) (ii) y 2 = ax and y = a is obtained as a2 = ax, x = a. Hence, Q : (a, a) 5. To change the order of integration, i.e., to integrate first w.r.t. y, divide the region into two subregions ORS and OPQ. Draw vertical strip parallel to y-axis in each subregion. (i) In subregion ORS, strip AB starts from the line y = a and terminates on the parabola y 2 = ax. Limits of y : y = a to y ax Limits of x : x = 0 to x = a (ii) In subregion OPQ, strip CD starts from the parabola y 2 = ax and terminates on the line y = a.

Fig. 8.22

Limits of y : y = ax to y = a Limits of x : x = 0 to x = a Hence, the given integral after change of order can be written as a

y2 a

f ( x, y ) dx dy =

a 0

Example 5:

2

2+ 4-2 y

0

y

a 0

ax a

f ( x, y ) dy dx +

a 0

a ax

f ( x, y ) dy dx

f ( x , y ) dx dy .

Solution: 1. The function is integrated first w.r.t. x and then w.r.t. y. 2. Limits of x : x = y to x = 2 + 4 − 2 y Limits of y : y = 0 to y = 2 3. The region is bounded by the x-axis, the line y = x and the parabola ( x 2) 2 2(2 y ). Fig. 8.23

8.24

Engineering Mathematics

4. The points of intersection of y = x and ( x 2) 2 2(2 y ) are obtained as ( x 2) 2 2(2 x ), x = 0, 2 and y = 0, 2 Hence, O : (0, 0) and Q : (2, 2) 5. To change the order of integration, i.e., to integrate first w.r.t. y, divide the region into two subregions OPQ and PQR. Draw a vertical strip parallel to y-axis in each subregion. (i) In subregion OPQ, strip AB starts from x-axis and terminates on the line y = x. Limits of y : y = 0 to y = x Limits of x : x = 0 to x = 2 (ii) In subregion PQR, strip CD starts from x-axis and terminates on the parabola ( x 2) 2

2(2 y ). Limits of y : y = 0 to y

2x

x2 2

Limits of x : y = 2 to y = 4 Hence, the given integral after change of order can be written as 2

2

0

y

Example 6:

4 2y

a cos 0

f ( x, y ) dx dy = a2 - x 2 x tan

2

x

0

0

f ( x, y ) dy dx +

4

2x

2

0

x2 2

f ( x, y ) dy dx

f ( x , y )d y d x.

Solution: 1. The function is integrated first w.r.t. y and then w.r.t. x. 2. Limits of y : y = x tan to y = a 2 − x 2 Limits of x : x = 0 to x = a cos a 3. The region is bounded by the line y = x tan a, the circle x 2 + y 2 = a 2 and y-axis. Since given limits of x and y are positive, the region lies in the first quadrant.

Fig. 8.24

Multiple Integral

8.25

4. The points of intersection of y = x tan a and x 2 + y 2 = a 2 are obtained as x 2 + x 2 tan 2 = a 2 , x = ± a cos and y = ± a sin Hence, P : (a cos a, a sin a ) and P' : (–a cos a, – a sin a) 5. To change the order of integration, i.e., to integrate first w.r.t. x, divide the region into two subregions OPR and PQR. Draw horizontal strip in each subregion. (i) In subregion OPR, strip AB starts from y-axis and terminates on the line y = x tan a. Limits of x : x = 0 to x = y cot a Limits of y : y = 0 to y = a sin a (ii) In subregion PQR, strip CD starts from y-axis and terminates on the circle x 2 + y 2 = a2 . Limits of x : x = 0 to x a2 y2 Limits of y : y = a sin a to y = a Hence, given integral after change of order can be written as a2 x 2

a cos 0

x tan

Example 7:

f ( x, y ) dy dx = 4

4x

0

4 x - x2

a sin 0

y cot 0

f ( x, y ) dx dy +

a2 y 2

a a sin

0

f ( x, y ) dx dy

f ( x , y )dy dx .

Solution: 1. The function is integrated first w.r.t. y and then w.r.t. x. 2. Limits of y : y = 4 x − x 2 to y = 4 x . Limits of x : x = 0 to x = 4 3. The region is bounded by the circle x 2 y 2 4 x 0, the parabola y 2 = 4 x and the line x = 4. 4. The point of intersection of (i) x 2 y 2 4 x 0 and y 2 = 4 x is obtained as x 2 = 0, x = 0 and y = 0 Hence O : (0, 0)

y2

Fig. 8.25

8.26

Engineering Mathematics

(ii) y 2 = 4 x and x = 4 are obtained as y2 = 16, y = ± 4 Hence, Q : (4, 4) and Q : (4, - 4) 5. To change the order of integration, i.e., to integrate first w.r.t. x, divide the region into three subregions ORT, TPS and RSQ. Draw a horizontal strip parallel to x-axis in each subregion. (i) In subregion ORT, strip AB starts from the parabola y 2 = 4 x and terminates on the circle x 2

y2

4x

0.

2

y to x 2 4 y 2 (Part of the circle where x < 2) 4 Limits of y : y = 0 to y = 2 (ii) In subregion TPS, strip CD starts from the circle x 2 y 2 4 x 0 and terminates on the line x = 4. Limits of x : x

Limits of x : x 2 4 y 2 (Part of circle where x > 2) to x = 4 Limits of y : y = 0 to y = 2 (iii) In subregion RSQ, strip EF starts from the parabola y 2 = 4 x and terminates on the line x = 4. y2 Limits of x : x = to x = 4 4 Limits of y : y = 2 to y = 4 Hence, given integral after change of order can be written as 4 0

4x 4 x x2

f ( x, y ) dy dx = +

Example 8:

2

(4 x )2

0

4 x

2

2

0

y2 4

2

4

0

2

4 y2

4 y2

f ( x, y ) dx dy

f ( x, y ) dx dy +

4

4

2

y2 4

f ( x, y ) dx dy

f ( x , y ) dy dx .

Solution: 1. The function is integrated first w.r.t. y and then w.r.t. x. 2. Limits of y : y = 4 − x to y = ( 4 − x ) 2 . Limits of x : x = 0 to x = 2 3. The region is enclosed by the parabola y 2 = 4 − x and y = ( 4 − x ) 2 , the lines x = 2 and x = 0. 4. The point of intersection of (i) x = 2 and y2 = 4 – x are obtained as y 2 (4 2), y 2 Hence, Q : ( 2, 2 ) and Q ′ ( 2, − 2 ) (ii) x = 2 and y (4 x ) 2 is obtained as y (4 2) 2 4 Hence, S : (2, 4) (iii) x = 0 and y2 = 4 – x are obtained as y 2 = 4, y = ± 2 Hence, P : (0, 2) and P' : (0, 2)

Multiple Integral

8.27

Fig. 8.26 (iv) x = 0 and y (4 x) 2 is obtained as y = 16 Hence, U : (0, 16) 5. To change the order of integration, i.e., to integrate first w. r. t. x, divide the region into three subregions PQR, PRST and STU. Draw a horizontal strip in each subregion. (i) In subregion PQR, strip AB starts from parabola y 2 4 x and terminates on the line x = 2. Limits of x : x 4 y 2 to x 2 Limits of y : y = 2 to y = 2 (ii) In subregion PRST, strip CD starts from y-axis and terminates on the line x = 2. Limits of x : x = 0 to x = 2 Limits of y : y = 2 to y = 4 (iii) In the subregion STU, strip EF starts from y-axis and terminates on the parabola y (4 x) 2 . Limits of x : x = 0 to x 4 y (Part of the parabola where x < 4) Limits of y : y = 4 to y = 16 Hence, given integral after change of order can be written as 2

(4 x )2

0

4 x

2

f ( x, y ) dy dx = +

2 4 y2

2 16

4

4

0

y

f ( x, y ) dx dy + f ( x, y ) dx dy

4

2

2

0

f ( x, y ) dx dy

8.28

Engineering Mathematics

(II) Change the Order of Integration and Evaluate the Following Example 1:

0

x

sin y dy dx . y

Solution: 1. The function is integrated first w.r.t. y, but evaluation becomes easier by changing the order of integration. 2. Limits of y : y = x to y = p . Limits of x : x = 0 to x = p .

Fig. 8.27 3. The region is bounded by the line y = x, y = p and x = 0. 4. The point of intersection of the line y = x and the line y = p is P : (p, p) 5. To change the order of integration, i.e., to integrate first w.r.t. x, draw a horizontal strip AB parallel to x-axis which starts from y-axis and terminates on the line y = x. Limits of x : x = 0 to x = y Limits of y : y = 0 to y = p Hence, the given integral after change of order can be written as sin y sin y y sin y y ∫0 ∫x y dy dx = ∫0 y ∫0 dx dy = ∫0 y x 0 dy

=∫

0

Example 2:

x 0

0

xe

x2 y

sin y ⋅ y dy = ∫ sin y dy 0 y

= − cos y 0 = − cos + cos 0 =2 dy dx .

Solution: 1. The function is integrated first w.r.t. y, but evaluation becomes easier by changing the order of integration.

Multiple Integral

8.29

2. Limits of y : y = 0 to y = x Limits of x : x = 0 to x 3. The region is the part of the first quadrant bounded between the lines y = x and y = 0. 4. To change the order of integration, i.e., to integrate first w.r.t. x, draw a horizontal strip parallel to x-axis which starts from the line y = x and extends up to infinity. Limits of x : x = y to x Limits of y : y = 0 to y Hence, the given integral after change of order can be written as ∞

∫ ∫ 0

x

0



xe

x2 y

dy dx = ∫



0



∞ y



xe

x2 y

Fig. 8.28

dx dy x2

∞⎛ y⎞ ∞ − = ∫ ⎜− ⎟ ∫ e y 0 ⎝ 2⎠ y x2

⎛ 2x ⎞ ⎜⎝ − y ⎟⎠ dx dy



− 1 ∞ =− ∫ ye y 2 0

⎡∵ e f ( x ) f ′( x ) dx = e f ( x ) ⎤ ⎣ ∫ ⎦

dy y

=− =

Example 3:

1- x2

1 0

0

1 ∞ 1 y(0 − e − y ) dy = − ye − y − e − y 2 ∫0 2

∞ 0

1 2 ey

( e y + 1) 1 - x 2 - y 2

dy dx .

Solution: 1. The function is integrated first w.r.t. y, but evaluation becomes easier by changing the order of integration. 2. Limits of y : y = 0 to y 1 x2 Limits of x : x = 0 to x = 1 3. Since given limits of x and y are positive, the region is the part of circle x 2 + y 2 = 1 in the first quadrant. 4. To change the order of integration, i.e., to integrate first w.r.t. x, draw a horizontal strip AB parallel to x-axis which starts from y-axis and terminates on the circle x 2 + y 2 = 1.

Fig. 8.29

8.30

Engineering Mathematics

1 y2 Limits of x : x = 0 to x Limits of y : y = 0 to y = 1 Hence, the given integral after change of order can be written as

0

ey

1 x2

1 0

(e

y

1) 1 x

2

y

2

ey 0 e +1 y

1

1 y2

1

dy dx =

0

(1 y 2 ) x 2

dx dy

1 y2

ey sin = 0 ey +1 1

x

1

1 y2

dy 0

ey (sin 1 1 sin 1 0) dy 0 ey +1 1

ey dy 0 ey +1 2 1

=

Example 4:

2

(

[log(e + 1) − log 2] = 2 log ⎛⎜⎝ 2 a

a − a2 − y2

0

0

)

log e y 1

1



0

f ( y) dy = log f ( y ) f ( y)

e +1⎞ ⎟ 2 ⎠

xy log ( x + a ) dx dy . ( x − a )2

Solution: 1. The function is integrated first w.r.t. x, but evaluation becomes easier by changing the order of integration. 2. Limits of x : x = 0 to x a a2 y2 Limits of y : y = 0 to y = a 3. The region is bounded by the circle (x a)2 + y2 = a2, the lines y = a and x = 0. 4. The point of intersection of (x a)2 + y2 = a2 and y = a is obtained as (x a)2 + a2 = a2, Fig. 8.30 x = a. Hence, P : (a, a). 5. To change the order of integration, i.e., to integrate first w.r.t. y, draw a vertical strip AB parallel to y-axis which starts from the circle (x a)2 + y2 = a2 and terminates on the line y = a. Limits of y : y 2ax x 2 to y Limits of x : x = 0 to x = a

a

Multiple Integral

8.31

Hence, given integral after change of order can be written as a

a − a2 − y 2

0

0

∫ ∫

a x log ( x + a) a xy log ( x + a) dx dy = ∫ y dy d x 2 2 0 ( x − a) ( x − a) 2 ∫ 2 ax − x

=∫

a

0

x log ( x + a) y 2 2 ( x − a) 2

a

2 ax − x 2

dx = ∫

a

0

x log ( x + a) ⎛ a 2 − 2ax + x 2 2 ( x − a) 2 ⎜⎝

=

a 2 ⎤ a x 1 a 1 ⎡ x2 1 ⎢ x x + a x = x + a −∫ log ( ) d log ( ) ⋅ dx ⎥ ∫ 0 2 2 0 2⎢ 2 x+a ⎥ 0 ⎣ ⎦

=

1 ⎡ a2 1 a⎧ a2 ⎫ ⎤ 2 a x a log ( ) − − + ⎨ ⎬ dx ⎥ ⎢ 2⎣ 2 2 ∫0 ⎩ x + a⎭ ⎦

⎞ ⎟⎠ dx

a ⎤ 1 ⎡ a2 1 x2 2 = ⎢ log 2a − − ax + a log ( x + a) ⎥ 2⎢ 2 2 2 0⎥ ⎣ ⎦

Example 5:

=

⎞ a2 1⎛ 2 a log 2a − + a 2 − a 2 log 2a + a 2 log a ⎟ ⎜ 4⎝ 2 ⎠

=

⎞ a2 1 ⎛ a2 + a 2 log a ⎟ = (1 + 2 log a) ⎜ 4⎝ 2 ⎠ 8 1 1- 4 y2 2 0 0

1 + x2 1 - x2 1 - x2 - y2

dx dy .

Solution: 1. The function is integrated first w.r.t. x, but evaluation becomes easier by changing the order of integration. 2. Limits of x : x = 0 to x

1 4 y2

1 2 3. The region is the part of the ellipse in the first quadrant. 4. To change the order of integration, i.e., to integrate first w.r.t. y, draw a vertical strip AB parallel to y-axis which starts from x-axis and terminates on the ellipse x2 + 4y2 = 1. Limits of y : y = 0 to y =

1 1 x2 2 Limits of x : x = 0 to x = 1 Limits of y : y = 0 to y

8.32

Engineering Mathematics

Fig. 8.31 Hence, the given integral after changing the order of integration can be written as 1 2

∫ ∫ 0

1+ x2

1− 4 y 2

1− x

0

=∫

1

0

=∫

1

2

1+ x2 1− x2

1− x2 2sin 1 x



a 0

0

1− x

dx =

x 1 x2 2

2

2

1

0

0

⎛ 0⎜ ⎝

6∫

1



1 1− x 2 2

0

1 (1 − x 2 ) − y 2

dy dx

1 + x 2 ⎛ −1 1 ⎞ − sin −1 0 ⎟ dx ⎜ sin 2 ⎝ ⎠ 2 1− x

⎞ − 1 − x 2 ⎟ dx ⎠ 1− x2 2

1

1 sin 1 x 2 0 2

4 2

2

dx = ∫

8

x dy dx

y 0

1+ x2

1 1− x 2 2

1− x2

6

3 sin 1 1 6 2 Example 6:

2

y

sin −1

2 − (1 − x 2 )

0

6

1− x − y 2

1

dx dy = ∫

x 2 ⎡ 2 2 2⎤ ⎢∵ ∫ a − x dx = 2 a − x ⎥ ⎢ ⎥ a2 ⎢ −1 x ⎥ + sin ⎢⎣ 2 a ⎥⎦

.

( a - x )( a - y )( y - x )

Solution: 1. The function is integrated first w.r.t. x, but evaluation becomes easier by changing the order of integration. 2. Limits of x : x = 0 to x = y Limits of y : y = 0 to y = a 3. The region is bounded by the line y = x, y = a and x = 0. 4. The point of intersection of y = a and y = x is P : (a, a)

Multiple Integral

8.33

5. To change the order of integration, i.e., to integrate first w.r.t. y, draw a vertical strip AB parallel to y-axis which starts from the line y = x and terminates on the line y = a. Limits of y : y = x to y = a Limits of x : x = 0 to x = a Hence, the given integral after changing the order can be written as a 0

x dy dx

y 0

(a

2

2

x )( a x

a

=

y )( y x )

0

a

dy

a

2

x

2

x

(a

dx

y )( y x )

Fig. 8.32

2

Putting y x t , dy 2t dt When y = x, t = 0 y = a, t a x a

∫ ∫ 0

x dy dx

y

0

( a − x )( a − y )( y − x ) 2

= 2∫

2

0

2

x

a

a2 − x 2 x

a 0

2

a2

x2

2( a 2



a− x

a

x

0

a −x

=∫

2

dt (a − x) − t 2

0

(sin 1 1 sin 1 0) dx 1 a 2 2

x )

0

1 0

1 x x



( a − x − t 2 )t 2 x

a

a2 − x 2

0

2

a

2

2t dt

a− x

0

dx = 2 ∫

= a Example 7:

2

0

1 2 (a 2

sin −1

dx a− x

t a− x

dx 0

1 2

x 2 ) ( 2 x ) dx

⎡ [ f ( x)]n+1 ⎤⎥ n ⎢∵ ∫ [ f ( x ) ] f ′( x )dx = n +1 ⎥ ⎢⎣ ⎦

y dx dy . (1 + xy )2 (1 + y 2 )

Solution: 1. The function is integrated first w.r.t. y, but evaluation becomes easier by changing the order of integration. 1 2. Limits of y : y = x to y = x Limits of x : x = 0 to x = 1 3. The region is bounded by the rectangular hyperbola xy = 1, the line y = x and y-axis in the first quadrant. 4. The point of intersection of xy = 1 and y = x is obtained as x2 = 1, x = 1 and y = 1 Hence, P : (1, 1)

8.34

Engineering Mathematics

5. To change the order of integration, i.e., to integrate first w.r.t. x, divide the region into two subregions OPQ and QPR. Draw a horizontal strip parallel to x-axis in each subregion. (i) In subregion OPQ, strip AB starts from y-axis and terminates on the line y = x. Limits of x : x = 0 to x = y Limits of y : y = 0 to y = 1 (ii) In subregion QPR, strip CD starts from y-axis and terminates on the rectangular hyperbola xy = 1. 1 Limits of x : x = 0 to x = y Fig. 8.33 Limits of y : y = 1 to y . Hence, given integral after changing the order can be written as 1

1 x

0

x

y dx dy = (1 + xy ) 2 (1 + y 2 )

1 dy y(1 + xy ) 0

1

1

1 1 1 dy 2 1 y 1 y2

1 0

1 (1 + y 2 ) 2

y 0

1 dx d y + (1 + xy ) 2

1

y 1+ y2

1 y 0

1 dx dy (1 + xy ) 2

1

y

y 0 1 + y2 1

0

y 0 1+ y2 1

1

y 1 + y2

y 1 dy y(1 + xy ) 0

1 1 1 dy 2 1 y 2

1 1 dy 2 2 1+ y

1

1 dy 1 + y2

Putting y = tanq in the first term of first integral, dy = sec 2 d , When y = 0, q = 0 y = 1, 1

∫∫ 0

1 x x

=

4 π

1 ∞ y sec 2 θ dθ 1 dx dy = − ∫ 4 + tan −1 y 0 + tan −1 y 1 2 4 0 (1 + xy ) (1 + y ) sec θ 2 2

π

= −∫ 4 0

(1 + cos 2θ ) dθ + (tan −1 1 − tan −1 0) 2 1 + (tan −1 ∞ − tan −1 1) 2

1 sin 2θ =− θ+ 2 2 =

π −1 4

π 4 0

+

3π π 1 π 3π = − − sin + 8 8 4 2 8

Multiple Integral

cos -1 x

1- y2

1

Example 8:

0

0

8.35

dx dy .

1 - x2 1 - x2 - y2

Solution: 1. The function is integrated first w.r.t. x, but evaluation becomes easier by changing the order of integration. 2. Limits of x : x = 0 to x 1 y2 Limits of y : y = 0 to y = 1 3. Since given limits of x and y are positive, the region is the part of the circle x2 + y2 = 1 in the first quadrant. 4. To change the order of integration, i.e., to integrate first w.r.t. x, draw a vertical strip AB parallel to y-axis in the region. AB starts from x-axis and terminates on the circle x2 + y2 = 1. Limits of y : y = 0 to y 1 x2 Limits of x : x = 0 to x = 1. Hence, the given integral after changing the order can be written as 0

cos -1 x

1- y 2

1 0

1 - x2 1 - x2 - y2 1

cos −1 x

0

1− x2

=∫

(1 − x 2 ) − y 2

0

1

cos −1 x

0

1− x2

=∫



dx dy

1

1− x 2

cos −1 x

−1

⎡(cos 1) − (cos 0) ⎤⎦ 4⎣ ⎡ ⎛ ⎞2 ⎤ = − ⎢0 − ⎜ ⎟ ⎥ 4 ⎢⎣ ⎝ 2 ⎠ ⎥⎦ 3

=

16

cos −1 x

0

1− x2

sin

−1

1− x 2

y 1− x2

dx 0

(sin −1 1 − sin −1 0) dx

−1

=−

1

dy dx = ∫

(cos −1 x ) 2 dx = − =− ∫ − 2 2 2 0 1− x2 1

Fig. 8.34

2

2

1

0

⎡ ⎢∵ ⎣

n ∫ [ f ( x)]

f ′( x ) dx =

[ f ′( x )] n + 1 ⎤ ⎥ n +1 ⎦

8.36

Engineering Mathematics

Exercise 8.3 (I) Change the order of integration of the following: 1.

6

2+ x

0

2 x

f ( x, y ) dy dx

Ans. :

2.

1

2x

0

x

2

6

4

2 y

+

8

6

2

y 2

y

0

y 2

1

y

0

y

1 y 2

1

1

1 x2

f ( x, y ) dx dy

0

2 0

6 x 2 x2 + 4 4

y a

a

a 0

Ans. :

a

+ 5.

a

2a x

0

x2 a

f ( x, y ) dx dy

2

x +6 a 4 x2

a

ay

0

0

+ 3 2

y y

2

6

ax

0

f ( x, y ) dy dx

2

a 0

a

0

f ( x, y )dx dy

Ans. :

x 6

6

+

x 6

x +6

3 2

f ( x, y ) dy dx

x

f ( x, y ) dy dx

f ( x, y )dx dy

f ( x, y )dx dy

6a

2 0

f ( x, y )dx dy

6a+2

2

6a

y 6a

f ( x, y )dx dy

f ( x, y ) dx dy

a2 y 2

2a

2 ax

0

2 ax x 2

Ans. :

f ( x, y ) dx dy 2

y+a

a

Ans. :

11.

6 2y 0

2

a a2 x 2

0

f ( x, y )dx dy 2a y

3

4 y2

+

2a

f ( x, y )dx dy

2

2

0

+ 10.

f ( x, y )dy dx

f ( x, y ) dy dx

Ans. :

f ( x, y ) dy dx

Ans. :

6.

a

2 y 1

+

f ( x, y ) dy dx

a

0

0

0

+

ax

2

1

f ( x, y )dx dy

0

4 x x2 4

4

2

f ( x, y ) dx dy a

f ( x, y )dy dx

0

f ( x, y ) dy dx

2

4.

x 2

2

+

9.

1

)

Ans. :

f ( x, y ) dx dy Ans. :

1 y

f ( x, y )dy dx

f ( x, y )dx dy 2

+

21 2y

Ans. :

8. 1

(

1 0

f ( x, y )dy dx

f ( x, y ) dy dx Ans. :

3.

7.

a

2a a

+

2a

a

a

x a

f ( x, y ) dy dx

f ( x, y ) dx dy

0

+

f ( x, y ) dy dx

a2 y 2

f ( x, y ) dx dy

a2 y 2

a

a

0

y2 2a

2a

2a

a

y2 2a

f ( x, y ) dx dy

f ( x, y ) dx dy

Multiple Integral

12.

a

a2 − x 2

0

a2 − x 2 4

∫ ∫

Ans. :

f ( x, y ) dy dx a 2

a2 y 2 a

0

13.

a2 x

0

x

2

4y

a 2

y

0

0

+ b

mx

a

k x

+

f ( x, y ) dx dy 15.

1

ex

0

a

f ( x, y )dx dy

2 0

ma

b

k a

a

mb

b

ma

y m

e

Ans. : 16.

f ( x, y )dx dy f ( x, y )dx dy f ( x, y )dx dy

f ( x, y ) dy dx

0 1

f ( x, y )dx dy a2 y

b k y

+

f ( x, y ) dx dy

0

a

k a k b

Ans. :

f ( x, y ) dy dx

Ans. :

14.

2

a2 y 2

a

+ a

8.37

x3

1

1

f ( x, y )dx dy

log y

f ( x, y ) dy dx

0

⎡ Ans. : ⎣⎢

f ( x, y ) dy dx

8

2

0

y3

∫∫

f ( x, y )dx dy ⎤ ⎦⎥

1

(II) Change the order of integration and evaluate the following: 1.

1 0

dx

e y y x log y dy

1

Ans. : 2.

1

1

0

y

dy

1

1 e

1

2

0

2y

1

x

0

0

5

2+ x

0

2 x

1

2 x

0

x

0

x 2 0

e 2 x 2 e xy dy dx = 2

cos( x 2 )dy dx =

sin 4 4

a

2 ax

0

0

5

3

2 y

dx dy +

Ans. : 7.

a 0

ay y

2a

a

0

y2 4a

0

x dx dy y 2 2 y x dx dy + 1 0 y

= log

4 e

2

y 2

dx dy

a

x

0

x2 a

= 8.

a

2a x

0

x2 a

⎡ ⎢ Ans. : ⎢ ⎢ ⎣

x 2 dx dy =

4 4 a 7

x dx dy x + y2

y

0

5

2

Ans. :

1

7

x 2 dy dx

x dy dx y Ans. :

2

= 25

cos( x 2 )dx dy 2

dy dx

Ans. :

6.

Ans. : 4.

0

e y y x log y dx =

x 2 e xy dx dy

Ans. : 3.

1

5.

x dy dx x + y2 2

a log 2 2

xy dy dx a

∫ ∫ 0

ay

0

+∫

2a a

⎤ ⎥ 5 ⎥ xy dx dy = a 4 ⎥ 6 ⎦

xy dx dy



2a− y

0

8.38

Engineering Mathematics

sin y

x

9.

0

0

(

x )( x

y)

14.

sin y

Ans. :

0

y

(

x )( x

y)

∫∫ 0

2

1

a

x

0

0

a

a

0

y

y)

1

x 0

dx dy ( y a) ( a x )( x

y)

15.

1 0

1 y3

a

0

y

2

y

( a x )( x

y )(4 5cos y )

2

dy dx

16.

2

2

10 y 2

3 0

y2 9

y )(4 5cos y ) 2

( a x )( x

2

4 y2

0

2

4 y2

0

0

x + y2

=

4 x x2

4

2

+

1 2

1

3 x

0

0

x

y

0

0

x +y 2

2

y

2

2

2 2

1 x2 2

1 0

18.

2

0

2

dy dx

x2 2 0

1

0

1 + 5sin 2

1

dy dx 3 10

y2

dx dy

1+ x2 y2

dy dx

2 3

x + y2 +1 2

dy dx

⎤ dx dy ⎥ x + y +1 ⎥ ⎥ 1 = (5 log 5 − 4) ⎥ ⎦ 4

2

∫ ∫ 0

10 x 2

10

1 x2

x

⎡ ⎢ Ans. : ⎢ ⎢ ⎢ ⎣

dy dx

2

1 x2

Ans. :

0

x +y

0

0

=

dy dx

2 x2

1

1+ x

2

dx dy

1

2

dy dx = 2

1 4y

e 2 2

dx dy

= 17.

y

2 y2

Ans. :

x

2

e x dy dx =

+

0

y

0

3

dx dy

dx dy

Ans. :

0

x

log 5cos a 4

5 2

1

Ans. :

sin y

dx dy

2

log 2 sin y

1

2

e x dx dy

Ans. : a

x + y2

0

Ans. :

a

x

2 y2

2

=

11.

13.

2

+

( y a) ( a x )( x

=

0

dx dy

x + y2

0

dy dx

Ans. :

12.

x

y

0

=2 10.

dy dx

x + y2

x

Ans. :

dx dy

x

2− x2

1

dy dx

x

2

2y

2

2

Multiple Integral

8.39

8.4 DOUBLE INTEGRATION IN POLAR COORDINATES The double integral can be changed from cartesian coordinates (x, y) to polar coordinates (r, q) by putting x = r cosq, y = r sinq . Then f (x, y) dy dx = f (r cos q, r sin q) | J | dr dq where J is the Jacobian (functional determinant) x ( x, y ) r J= = y (r , ) r cos r sin = sin r cos Hence,

f ( x, y )dy dx = =

x y = r (cos 2 + sin 2 ) = r

f ( r cos , r sin ) r dr d f ( r cos , r sin ) r dr d

8.4.1 Limits of Integration The limits of integration, if required can be found with the help of the given curves. Let the region is bounded by the curves r = r1 (q ), r = r2 (q ) and the lines q = q 1, q = q 2. The region of integration is PQRS. Draw an elementary radius vector AB from origin which enters in the region from the curve r = r1 (q ) and leaves at the curve r = r2 (q ). Therefore, limits for r are r1 ( ) to r2 ( ) [i.e., r varies along the AB and q remains constant]

Fig. 8.35 To cover the entire region PQRS, rotate elementary radius vector AB from PQ to RS. Therefore, q varies from q1 to q 2. Hence, double integration in polar form becomes

8.40

Engineering Mathematics

r2 ( )

2

r1 ( )

1

f ( r cos , r sin ) r dr d

Note: The function is integrated first w.r.t. r and then w.r.t. q.

(I) Evaluation of Integral Over a Given Region in Polar Coordinates: Example 1: Evaluate r = a cosp.

r a 2 - r 2 dr d , over the upper half of the circle

Solution: 1. The region of integration is the upper half of the circle r = a cosq . 2. Draw an elementary radius vector OA which starts from the origin and terminates on the circle r = a cosq . Limits of r : r = 0 to r = a cosq Limits of q : q = 0 to

=

2

Fig. 8.36

I=∫

∫r π

=∫2∫ 0

=−

a 2 − r 2 dr dθ

a cosθ

0

1 ⎛ 1⎞ 2 2 2 ( ) ( −2r ) dr dθ − a r − ⎜⎝ ⎟⎠ 2 3 a cosθ 2 2

1 2( a − r ) 3 2∫ π 2 0

2





[ f (r )] f ( r )dr = n

0

=−

1 π2 3 3 a3 3 a sin θ − a d θ = − 3 ∫0 3

(

)

π 2 0



⎛ 3 sin θ − sin 3θ ⎞ − 1⎟ dθ ⎜⎝ ⎠ 4

[ f (r )]n+1 n +1

Multiple Integral

a3 1 3 4

cos 3 3

3cos

8.41

a3 3 1 3 4 12

2

0

2

3

a 2 3 3

2 r 4 cos 3 dr d , over the interior of the circle r = 2a cosp .

Example 2: Evaluate Solution:

1. The region of integration is the interior of the circle r = 2a cos q. 2. Draw an elementary radius vector OA which starts from the origin and terminates on the circle r = 2a cosq. Limits of r : r = 0 to r = 2a cosq Limits of

:

2

to

2

Fig. 8.37 I = ∫ ∫ r 4 cos3 θ dr dθ π 2 π − 2

= ∫ cos θ ∫ 3

2 a cosθ

0

π 2 π − 2

r dr d θ = ∫

π

4

r5 cos θ 5

2 a cosθ

3

dθ 0

π

=

1 2 32a5 3 5 cos ( a cos ) θ θ d θ = ⋅ 2 ∫ 2 cos8 θ dθ 2 π 0 5 ∫− 2 5

=

32a5 ⎛ 9 1 ⎞ ⋅B⎜ , ⎟ ⎝2 2⎠ 5

∵B

p +1 q +1 , =2 2 2

2 0

sin p cos q d

8.42

Engineering Mathematics

9 1 32a5 2 2 5 5 7 5 a = 4

7 5 3 1 1 1 32a5 2 2 2 2 2 2 5 24

Example 3: Evaluate the initial line.

r 2 sin dr d , over the cardioid r = a (1 + cosp ) above

Solution: 1. The region of integration is the part of the cardioid r = a (1 + cosq ) above the initial line (q = 0). 2. Draw an elementary radius vector OA which starts from the origin and terminates on the cardioid r = a (1 + cosq ). 3. Limits of r : r = 0 to r = a (1 + cosq ) Limits of : = 0 to =

Fig. 8.38 I=

r 2 sin dr d = 1 3

0

sin

a3 (1 cos )3 d

a3 (1 + cos ) 4 3 4 =−

0

sin

a (1+ cos ) 0

a3 3

r 2 dr d =

0

4 a3 (0 − 16) = a 2 12 3

r3 3

a (1+ cos )

d 0

(1 cos )3 ( sin ) d ∵

0

0

sin

[ f ( x)]n

f ( x ) dx =

[ f ( x)]n+1 n +1

Multiple Integral

r dr d

Example 4: Evaluate

r 2 + a2

8.43

, over one loop of the lemniscate r2 = a2 cos2p .

Solution: 1. The region of integration is one loop of the lemniscate r 2 = a 2 cos 2 bounded to 4 4 2. Draw an elementary radius vector OA which starts from the origin and terminates on the lemniscate r 2 = a 2 cos 2 . between the lines

3. Limits of r : r = 0 to r = a cos 2 Limits of :

to

4

4

Fig. 8.39 r dr d

I=

r 2 + a2

1 = 2 =

4

=

a cos 2

4 4

1 a cos 2 2 2

2( r + a ) 2

0

4

1 1 2 ( r + a 2 ) 2 (2r ) dr d 2

d

π⎞ ⎛ = a ⎜2 − ⎟ ⎝ 2⎠ a = (4 − π ) 2

[ f (r )]

0

π π 1 ⎡ ⎤ 1 4 4 2 π 2a ⎢(cos 2θ + 1) − 1⎥ dθ = a ∫ π ∫ − 2 −4 ⎣ ⎦ 4

= a 2 sin θ − θ



n

π 4 π − 4

(

)

2 cosθ − 1 dθ

⎡ ⎛ π π⎞ π π⎤ = a ⎢ 2 ⎜ sin + sin ⎟ − − ⎥ ⎝ 4 4⎠ 4 4⎦ ⎣

f

n +1 f (r )] [ ( r ) dr =

n +1

8.44

Engineering Mathematics

r 2 dr d , over the area between the circles r = a sinp

Example 5: Evaluate and r = 2a sinp .

Solution: 1. The region of integration is the area bounded between the circles r = a sinq and r = 2a sinq. 2. Draw an elementary radius vector OAB from the origin which enters in the region from the circle r = a sinq and leaves at the circle r = 2a sinq. 3. Limits of r : r = a sinq to r = 2a sinq Limits of q : q = 0 to =

Fig. 8.40 r 2 dr d

I =

7a 3

0

a sin

3

7a 3 12 =

2 a sin

0

sin 3 d = 3cos

7a 3

cos 3 3

r 2 dr d 3

3sin

sin 3 4

0

0

0

r3 3

7a 3 12

3(cos

2 a sin

d a sin

1 3

0

(8a3 sin 3

a3 sin 3 ) d

d cos 0)

1 (cos 3 3

cos 0)

7a3 16 28 3 = a. 12 3 9

(II) Evaluation of Integral by Changing to Polar Coordinates: Example 1: Evaluate x2 + y2 = 1.

1 - x2 - y2 dx dy over the first quadrant of the circle 1 + x2 + y2

Solution: 1. Putting x = r cosq, y = r sinq , polar form of the circle x2 + y2 = 1 is obtained as r = 1.

Multiple Integral

8.45

2. The region of integration is the part of the circle r = 1 in the first quadrant. 3. Draw an elementary radius vector OA which starts from the origin and terminates on the circle r = 1. 4. Limits of r : r = 0 to r = 1 Limits of q : q = 0 to

=∫2∫ 0

2

1− x2 − y2 dx dy 1+ x2 + y2

I =∫∫ π

=

1

0

1− r2 r dr dθ 1+ r2 Fig. 8.41

Putting r 2 = cos 2t , 2r dr = −2 sin 2t dt When

π 2 0

∫ ∫

0

π 4

r = 0, t =

4 r = 1, t = 0 π 0 1 − cos 2t 2 sin 2 t sin 2t dtt dθ ( − sin 2t dt ) dθ = − ∫ 2 ∫π 0 1 + cos 2t 2 cos 2 t 4 π

π sin t ⋅ 2 sin t cos t dt dθ = ∫ 2 dθ 0 4 cos t 0

= − ∫ 2 ∫π 0

= θ

π 2 0

sin 2t t− 2

π 4 0

dθ = ∫

π 4 0



(1 − cos 2t ) dt

π ⎛π 1 ⎞ π ⎜ − ⎟ = (π − 2) 2 ⎝ 4 2⎠ 8

4 xy - x2 - y2 e dx dy , over the region bounded by the x + y2 circle x2 + y2 - x = 0 in the first quadrant. Example 2: Evaluate

2

Solution: 1. Putting x = r cosq, y = r sinq , polar form of the circle x2 + y2 – x = 0 is r2 r cosq = 0, r = cosq. 2. The region of integration is the part of the circle r = cosq in the first quadrant. 3. Draw an elementary radius vector OA which starts from the origin and terminates on the circle r = cosq . 4. Limits of r : r = 0 to r = cosq Limits of q : q = 0 to

=

2

Fig. 8.42

8.46

Engineering Mathematics

4 xy e 2 x + y2

I=

2

2

0 π 2

x2 y2

dx dy = cos

cos sin

= −2 ∫ cos θ sin θ e − r 0

π

e

0

(

2

r2

cos

2 0

0

4 r 2 cos sin e r2

r2

r dr d

( 2 r ) dr d

cosθ

dθ 0

∵ e f ( r ) f ( r ) dr = e f ( r )

)

= −2 ∫ 2 cos θ sin θ e − cos θ − 1 dθ 0

π 2 0

2

= − ∫ ⎡⎣e − cos θ ( 2 cosθ sin θ ) − sin 2θ ⎤⎦ dθ =−e

− cos 2 θ

2

cos 2θ + 2

π 2 0

cos π − cos 0 ⎞ ⎛ = − ⎜ e 0 − e −1 + ⎟⎠ ⎝ 2

⎛ 1 ⎞ = − ⎜1 − − 1⎟ ⎝ e ⎠ =

1 e

x2 y2 dx dy , over the region bounded by the circles ( x2 + y2 ) x2 + y2 = a2 and x2 + y2 = b2 (a > b). Example 3: Evaluate

Solution: 1. Putting x = r cosq, y = r sinq, polar form of the (i) circle x 2 + y 2 = a 2 is r 2 = a 2 , r = a. (ii) circle x 2 + y 2 = b 2 is r 2 = b 2 , r = b. 2. The region of integration is the part bounded between the circles r = a and r = b. 3. Draw an elementary radius vector OAB from the origin which enters in the region from the circle r = b and leaves at the circle r = a. 4. Limits of r : r = b to r = a Limits of q : q = 0 to = 2

I = ∫∫

4 2 2 2π a r cos θ sin θ x2 y2 = ⋅ r dr d θ x y d d ∫0 ∫b r2 ( x2 + y2 ) a



= ∫ cos 2 θ sin 2 θ 0

Fig. 8.43

2 4 4 2π sin 2θ ( a − b ) r4 dθ = ∫ ⋅ dθ 0 4 b 4 4

Multiple Integral

=

a4 − b4 16





0

⎛ a4 − b4 ⎞ (1 − cos 4θ ) sin 4θ dθ = ⎜ θ− 2 4 ⎝ 32 ⎟⎠

8.47



0

⎛a −b ⎞ ( 2π ) =⎜ ⎝ 32 ⎟⎠ 4

=

4

π 4 (a − b4 ) 16

( x 2 + y 2 )2 dx dy , over the region common to the circles x2 y2 x2 + y2 = ax and x2 + y2 = by (a, b > 0). Example 4: Evaluate

Solution: 1. Putting x = r cosq, y = r sinq, polar form of the (i) circle x 2 + y 2 = ax is r 2 = ar cos , r = a cosq (ii) circle x 2 + y 2 = by is r 2 = br sin , r = b sinq

Fig. 8.44 2. The region of integration is the common part of the circles r = a cosq and r = b sinq. 3. The point of intersection of the circle r = a cosq and r = b sinq, is obtained as a a b sinq = a cosq, tan = , = tan −1 b b a Hence, at P , = tan −1 b 4. Divide the region into two subregions OAP and OBP. Draw an elementary radius vector OA and OB in each subregion. (i) In subregion OAP, elementary radius vector OA starts from the origin and terminates on the circle r = b sinq.

8.48

Engineering Mathematics

Limits of r : r = 0 to r = b sinq a b (ii) In subregion OBP, elementary radius vector OB starts from the origin and terminates on the circle r = a cosq. Limits of r : r = 0 to r = a cosq a Limits of q : = tan 1 to = b 2 Limits of q : q = 0 to q = tan

I=∫∫ =∫

( x 2 + y 2 )2 dx dy x2 y2

tan −1

a b

0

=∫

1



b sin θ

0

tan −1

a b

0

π

a cos θ r4 r4 2 d d ⋅ + r r θ ∫tan−1 a ∫0 r 4 sin 2 θ cos2 θ ⋅ r dr dθ r 4 sin 2 θ cos 2 θ b

1 r2 sin 2 θ cos 2 θ 2

b sin θ

dθ + ∫

0

π 2 tan −1

1 r2 sin 2 θ cos 2 θ 2

a b

a cos θ

dθ 0

π

a

=

1 tan−1 b 1 1 1 ⋅ b 2 sin 2 θ dθ + ∫ 2 −1 a ⋅ a 2 cos 2 θ dθ 2 ∫0 2 tan b sin 2 θ cos 2 θ sin 2 θ cos 2 θ

=

b2 2



tan −1

a b

0

π

sec 2 θ dθ +

a2 2 b2 2 cos ec θ d θ = tan θ a 2 ∫tan−1 b 2

⎤ a2 b2 ⎡ ⎛a⎞ tan tan −1 ⎜ ⎟ − tan 0 ⎥ − ⎢ ⎝b⎠ 2 ⎣ ⎦ 2 ab ab = + = ab 2 b 0

0

e - (x

2

+ y2 )

0

a b

+

a2 − cot θ 2

dx dy .

Solution: 1. Limits of x : x = 0 to x → ∞ Limits of y : y = 0 to y → ∞ 2. The region of integration is the first quadrant. 3. Putting x = r cosq, y = r sinq, the integral changes to polar form. 4. Draw an elementary radius vector which starts from the origin and extends up to infinity. Limits of r : r = 0 to r → ∞ Limits of q : q = 0 to I=

0

0

2 0

0

e e

( x2 y2 )

r2

=

Fig. 8.45

2

dx dy

r dr d

1 2

π 2 tan −1

2 2 ⎡ π ⎤ a ⎛ −1 a ⎞ ⎤ b ⎡ a ⎢cot 2 − cot ⎜⎝ tan b ⎟⎠ ⎥ = 2 ⎢ b − 0 ⎥ − 2 ⎣ ⎦ ⎦ ⎣

=

Example 5:

tan −1

2 0

0

e

r2

( 2r ) dr d

a b

b⎤ ⎡ ⎢0 − a ⎥ ⎣ ⎦

Multiple Integral

1 2 1 2 =

2 0

2 0

e

r2

8.49

∵ e f ( r ) f ( r ) dr = e f ( r )

d 0

(0

)

1 2

e0 d

2 0

4 dx dy

Example 6:

-

3 2 2

2

.

(1 + x + y )

Solution: 1. Limits of x : x → − ∞ to x → ∞ Limits of y : y → − ∞ to y → ∞ 2. The region of integration is the entire coordinate plane. 3. Putting x = r cosq, y = r sinq, integral changes to polar form. 4. Draw an elementary radius vector which starts from origin and extends upto Limits of r : r = 0 to r → ∞ Limits of q : q = 0 to q = 2p

Fig. 8.46 dx dy

I=

=

3 2 2

(1 + x 2 + y ) =

1 2 1 2

2 0

3

(1 + r 2 ) 2

0

(1 + r 2 ) 2 (2r ) dr d 2

2(1 r )

1 2

d 0





0

0

= − ∫ (0 − 1) dθ = θ = 2π

0

3

2 0

r dr d

2 0



[ f ( r ) ] f ( r ) dr = n

[ f (r )]n+1 n +1

8.50

Engineering Mathematics

2 x - x2

1

Example 7:

0

x

( x 2 + y 2 ) dx dy .

Solution: 1. Limits of y : y = x to y 2x x2 Limits of x : x = 0 to x = 1 2. The region of integration is bounded by the line y = x and the circle x2 + y2 3. Putting x = r cosq, y = r sinq, polar form of the (i) line y = x is r sinq = r cosq, tanq = 1, (ii) circle x 2 r2

y2

2x

=

2x = 0.

4

0 is

2r cos

0 r = 2 cosq 4. Draw an elementary radius vector OA which starts from the origin and terminates on the circle r = 2 cosq. Limits of r : r = 0 to r = 2 cosq Limits of q :

=

to

4

=

2

Fig. 8.47 2 x x2

1

I

0

=

2 4

x

r4 4

( x2

y 2 )dx dy

2 4

2 cos 0

r 2 r dr d

2 cos

d =4 0

2 4

cos 4 d = 4

2 4

1 + cos 2 2

2

d

Multiple Integral

8.51

1 + cos 4θ ⎞ ⎛ = ∫π2 1 + 2 cos 2θ + cos 2 2θ dθ = ∫π2 ⎜1 + 2 cos 2θ + ⎟⎠ dθ ⎝ 2 4 4 π

(

)

3 2 sin 2θ sin 4θ = θ+ + 2 2 8 =

π

π 2 π 4

=

π⎞ 1 3 ⎛π π ⎞ ⎛ ⎜ − ⎟ − ⎜ sin π − sin ⎟⎠ + (sinn 2π − sin π ) 2⎝2 4⎠ ⎝ 2 8

2

2

3π +1 8

Example 8:

1

1 – x2

0

x – x2

x y e –( x + y ) dx dy . x2 + y2

Solution: x x 2 to y 1 x2 1. Limits of y : y Limits of x : x = 0 to x = 1. 2. The region of integration is the part of the first quadrant bounded by the circles x2 + y2 x = 0 and x2 + y2 = 1. 3. Putting x = r cosq, y = r sinq, polar form of the (i) circle x 2 y 2 x 0 is r 2 r cos 0, r cos (ii) circle x2 + y2 = 1 is r2 = 1, r = 1 4. Draw an elementary radius vector OAB from the origin which enters in the region from the circle r = cosq and leaves the region at the circle r = 1. Limits of r : r = cosq to r = 1 Limits of q : q = 0 to

=

2

Fig. 8.48

8.52

Engineering Mathematics

I=∫

1

0



1− x 2 x − x2

2

2

2

1 r 2 sin θ cos θ e − r x y e−( x + y ) dx dy = ∫ 2 ∫ ⋅ r dr dθ 2 2 0 cosθ r2 x +y

π

π

1 2 1 2 sin θ cos θ ∫ e − r ( −2r ) dr dθ ∫ 0 θ cos 2 π 2 1 1 ⎡∵ e f ( r ) f ′( r )dr = e f ( r ) ⎤ = − ∫ 2 sin θ cos θ e − r dθ ⎣ ∫ ⎦ cosθ 2 0 π π 2 2 1 1 1 ⎛1 ⎞ = − ∫ 2 sin θ cos θ e −1 − e − cos θ dθ = − ∫ 2 ⎜ sin 2θ − e − cos θ ⋅ 2 sin θ cosθ ⎟ dθ ⎠ 2 0 2 0 2 ⎝e

=−

(

)

1 1 ⎛ cos 2θ ⎞ − cos2 θ =− ⎜− ⎟−e 4 e⎝ 2 ⎠

π 2

⎡∵ e f (θ ) f ′(θ )dθ = e f (θ ) ⎤ ⎣ ∫ ⎦

0

π⎞ ⎛ − ⎜ cos 2 ⎟ 2 ⎤ 1⎡ 1 = − ⎢ − (cos π − cos 0 ) − e ⎝ 2 ⎠ + e − cos 0 ⎥ 4 ⎢⎣ 2e ⎥⎦ 1 ⎡1 1⎤ 1⎡ 1 ⎤ = − ⎢ − ( −2) − e 0 + e −1 ⎥ = − ⎢ − 1 + ⎥ e e⎦ 4 4 ⎣ 2e ⎣ ⎦

=

1 ⎡ 2⎤ 1− 4 ⎢⎣ e ⎥⎦

Example 9:

2

1+ 2 x - x2

0

1- 2 x - x2

dx dy . ( x 2 + y 2 )2

Solution: 1. Limits of y : y 1 2 x x 2 to y 1 2x x2 Limits of x : x = 0 to x = 2 2. The region of integration is the circle x2 + y2 2x 2y + 1 = 0 with centre at (1, 1) and radius 1. 3. Putting x = r cosq, y = r sinq, polar form of the circle x2 + y2 2x 2y + 1 = 0 is r 2 2r (cos sin ) 1 0 . 4. Draw an elementary radius vector OAB from origin which enters in the region from the lower part of the circle where r (cos sin ) sin 2 and leaves the region at the upper part of the circle where

Fig. 8.49

r = cos + sin + sin 2 . Limits of r : r

(cos

Limits of q : q = 0 to

sin ) =

2

sin 2

to r = (cos + sin ) + sin 2

Multiple Integral

I= = =

2

1

2 x x2

0

1

2 x x2

(cos 2

r

2

sin )

1 2 1 2 2 0

2

r dr d r4 where α = cos θ + sin θ

d , 1

2 0

sin 2

+

2

0

=2

dx dy ( x + y 2 )2 2

(cos + sin ) + sin 2

2 0

8.53

1

(

)

2

(

(

)2

(

0

2

(

)2 )2 2 2

)

and

β = sin 2θ

d 1 2

d

(cos + sin ) sin 2 d =2 (1 sin 2 sin 2 ) 2

4

2 0

(

2

2 2

)

3 2 0

d 1

3

1

2 (cos ) 2 (sin ) 2 + (sin ) 2 (cos ) 2 d

⎡ 1 ⎛ 5 3 ⎞ 1 ⎛ 5 3 ⎞⎤ = 2 2 ⎢ B ⎜ , ⎟ + B ⎜ , ⎟⎥ ⎣ 2 ⎝ 4 4 ⎠ 2 ⎝ 4 4 ⎠⎦ ⎡ π2 p 1 ⎛ p + 1 q + 1⎞⎤ q , ⎢∵ ∫0 sin θ cos θ dθ = B ⎜⎝ ⎟⎥ 2 2 ⎠⎦ 2 ⎣ 5 3 1 1 1 1− =2 2 4 4 =2 2 4 4 4 2 1 π = 2 sin π 4 =π Example 10:

4a 0

y y2 4a

∵ n1 n

n sin n

x2 - y2 dx dy . x2 + y2

Solution: y2 to x = y 4a Limits of y : y = 0 to y = 4a. 2. The region of integration is bounded by the line y = x and the parabola y 2 = 4ax. 3. Putting x = r cosq, y = r sinq, polar form of the 1. Limits of x : x =

(i) line y = x is r sinq = r cosq, tanq = 1, 2

2

(ii) parabola y = 4ax is r sin

2

=

4

= 4ar cos , r = 4a cot q cosecq

8.54

Engineering Mathematics

Fig. 8.50 4. Draw an elementary radius vector OA which starts from the origin and terminates on the parabola r = 4a cotq cosecq. Limits of r : r = 0 to r = 4a cotq cosecq : =

Limits of

I=∫

4a

0

2



y y2 4a

x2 − y2 dx dy = ∫ x2 + y2

π 2 π 4

(1 2sin 2 )

4

8a 2

2

to

4

r2 2

=

2

4 a cot θ cosecθ



0

4 a cot cosec

1 2

d 0

(cot 2 cosec 2

r 2 (cos 2 θ − sin 2 θ ) ⋅r dr dθ r2 2

(1 2sin 2 )(4 a) 2 cot 2 cosec 2 d

4

2 cot 2 ) d

4

8a 2

2

{

}

(cot 2 )( cosec 2 )

2cosec 2

2 d

4

8a

2

cot 3 3

⎡ [ f ( )]n+1 ⎤⎥ n ⎢ ∵ ∫ [ f ( ) ] f ′( ) d = n + 1 ⎥⎦ ⎢⎣

2

2 cot

2 4

8a 2

1 cot 3 3 2

cot 3

4

2 cot

2

cot

4

2

2

4

Multiple Integral

1 ( 1) 2( 1) 2 3 4

8a 2 8a 2

8a 2

5 3

8.55

2

5 3

2

a

Example 11:

5 ax - x 2 2 ax

0

x2 + y2 dx dy . y2

Solution: 5ax x 2 1. Limits of y : y 2 ax to y Limits of x : x = 0 to x = a 2. Since the limits of x and y are positive, the region of integration is the part of the first quadrant bounded by the parabola y 2 = 4ax and the circle x 2 y 2 5ax 0 3. Putting x = r cosq, y = r sinq, polar form of the (i) parabola y 2 = 4ax is r 2 sin 2 = 4a r cos , r = 4a cotq cosecq (ii) circle x 2 y 2 5a x 0 is r 2 5a r cos 0, r = 5a cosq 4. The point of intersection of r = 4a cotq cosecq and r = 5a cosq is obtained as 4a cotq cosecq = 5a cosq 4 sin 2 = , 5

2 5

= ± sin −1

Hence, at P,

= sin

Fig. 8.51

1

2

5 5. Draw an elementary radius vector OAB from the origin which enters in the region from the parabola r = 4a cotq cosecq and terminates on the circle r = 5a cosq. Limits of r : r = 4a cotq cosecq to r = 5a cosq 2 Limits of q : = sin 1 to = 2 5 I=∫

a

0

=∫



5 ax − x 2 2 ax

π 2

sin

−1

2 5



x2 + y2 dx dy y2

5 a cos θ 4 a cot θ cosecθ

r r dr dθ r sin 2 θ 2

8.56

Engineering Mathematics

π

= ∫ 2 −1

5 a cos θ

2

sin

=∫

5

π 2 2

sin −1

=∫

cosec 2θ r 4 a cot θ cosec θ dθ cosec 2θ (5a cos θ − 4 a co ot θ cosecθ ) dθ

5

π 2 2

sin −1

⎡⎣5a cot θ cosecθ + 4 a cosec 2θ ( − cosecθ cot θ ) ⎤⎦ dθ

5

cosec3θ = −5a cosecθ + 4 a 3

π 2 sin −1

⎡ [ f (θ )]n+1 ⎤⎥ n ⎢∵ ∫ [ f (θ ) ] f ′(θ )dθ = n + 1 ⎥⎦ ⎢⎣

2 5

⎡ π π 4a 2 ⎞⎤ 2 ⎞ 4a ⎛ ⎛ = ⎢ −5a cosec + 5a cosec ⎜ sin −1 + cosec3 − cosec3 ⎜ sin −1 ⎟⎥ ⎟ ⎝ ⎝ 2 3 2 5⎠ 3 5 ⎠⎦ ⎣ 3

5 5a 5a 2

(

4a 3

4a 3

⎡ ⎛ ⎛ −1 2 ⎞ 5 ⎞⎤ = cosec ⎜ cosec −1 ⎢∵ cosec ⎜ sin ⎥ ⎟ ⎝ 2 ⎟⎠ ⎥ 5⎠ ⎝ ⎢ ⎢ ⎥ 5 ⎢ ⎥ = ⎢⎣ ⎥⎦ 2

5 2

)

a 5 5 11 3 Exercise 8.4

(I) Evaluate the following: − r2

1.

∫ ∫ re a cos sin dr d , over the 2

3.

r sin dA, over the cardioid r = a (1 + cosq ) above the initial line. 4 Ans. : a3 3

4.

∫∫

upper half of the circle r = 2a cosq. Ans. : 2.

a2 1 3+ 4 16 e

3

r dr d , over the region between the circles r = 2 sinq and r = 4 sinq. 45 Ans. : 2

r

dr d , over one loop of r +4 the lemniscate r 2 = 4 cos 2 . 2

[ Ans. : (4

)]

(II) Change to polar coordinates and evaluate the following: 1.

1

2.

dx dy, over the region

xy bounded by the semi-circle x 2 y 2 x 0, y 0.

y 2 dx dy, over the area outside the circle x 2

y2

2

2

the circle x

y

ax 2ax

0 and inside 0.

Ans. :

Ans. : 2

15 a 4 64

Multiple Integral

8.57

sin( x 2 + y 2 ) dx dy, over the circle

3.

3 a4 4

Ans. :

x2 + y 2 = a2 . (1 cos a 2 )

Ans. :

(

xy x 2 + y

4.

10.

4 xy e x + y2

x x2

1 0

( x2 y2 )

2

0

3 2 2

) dx dy, over the first

dx dy

Ans. :

quadrant of the circle x 2 + y 2 = a 2 . Ans. : 3

5.

a2 y 2

2 y

0

a

x

0

0

3 log 3 2

12.

a

a

0

y

a2 y 2

Ans. :

x2 + y2 Ans. :

(

a log 1 + 2 4

)

13.

a

a2 x 2

0

ax x 2

7. a2 x 2

0

0

sin

a

2

(a2

x2

8.

0

0

x2

e

x2 y2

9.

2 ax x 2

2a 0

0

1 2

dx dy (4 a + x 2 + y 2 ) 2 1 8a 2 4

1 2

tan

1

1 2

dx dy a

2

x2

y2

[ Ans. : a] 14.

a2 x 2

a

ax x

0

2

xy e x + y2 2

( x2 y2 )

dx dy

1 ⎡ ⎡ 2 − a2 ⎤ ⎤ ⎢ Ans. : 4a 2 ⎣1 − (1 + a )e ⎦ ⎥ ⎣ ⎦

dx dy Ans. :

a 2 log a

y 2 ) dx dy a2 Ans. : 2

2

4

2

x 3 dx dy

4

a

log e ( x 2 + y 2 ) dx dy Ans. :

x2 + y2

0

Ans. : 6.

a

11.

dy dx

3x

0

a7 14

1 e

(1 4

e

2

)

15.

1

x

dx dy

0

x2

x2 + y2

( x 2 + y 2 ) dx dy

Ans. :

2 1

8.5 CHANGE OF VARIABLES OF INTEGRATION In some cases, evaluation of double integral becomes easier by changing the variables. Let the variables x, y be replaced by new variables u, v by the transformation x = f1 (u, v ), y = f 2 (u, v ), then f ( x, y ) dx dy =

f ( f1 , f 2 ) J du dv

... (1)

8.58

Engineering Mathematics

Jacobian, J =

where

( x, y ) ( u, v )

=

x u y u

x v y v

Using Eq. (1), the double integral can be transformed to new variables. Example 1: Using the transformation x - y = u, x + y = v, evaluate cos

x- y dx dy over the region bounded by the lines x = 0, y = 0, x + y = 1. x+ y

Solution: x

y

u, x y v u v v u x= , y= 2 2 J=

( x, y ) = ( u, v )

x u y u

x v y v

1 1 2 2 1 1 1 = = + = 1 1 4 4 2 2 2 dx dy = J du dv 1 du dv 2 The region bounded by the lines x = 0, y = 0 and x + y = 1 in xy-plane is a triangle OPQ. u+v v−u Under the transformation x = , and y = 2 2 (i) the line x = 0 gets transformed to the line u = –v =

Fig. 8.52

Multiple Integral

8.59

(ii) the line y = 0 gets transformed to the line u = v (iii) the line x + y = 1 gets transformed to the line v = 1 Thus, triangle OPQ in xy-plane gets transformed to triangle OP'Q' in uv-plane bounded by the lines u = v, u = v and v = 1. In the region, draw a horizontal strip AB parallel to u-axis which starts from the line u = v and terminates on the line u = v. Limits of u : u = v to u = v Limits of v : v = 0 to v = 1 I=

cos 1 2

1 0

x x

v sin

y dx dy = y

1 v2 2sin 1 2 2 =

v

u v

1

1 2

dv v

v

0

v

1 0

u 1 du dv v 2

cos

v [sin 1 sin ( 1) ] dv

1

0

1 sin 1 2

Example 2: Using the transformation x 2 - y 2 = u, 2 xy = v , find

( x 2 + y 2 ) dx dy

over the region in the first quadrant bounded by x2 - y2 = 1, x2 - y2 = 2, xy = 4, xy = 2. Solution: x2

y2 = u, 2xy = v

It is difficult to express x and y in terms of u and v, therefore we write Jacobian of u, v in terms of x and y. ∂u ∂ ( u , v ) ∂x J= = ∂ ( x , y ) ∂v ∂x

∂u ∂y

=

∂v ∂y

2x

− 2y

2y

2x

= 4 ( x2 + y2 )

du dv = J dx dy = 4 ( x 2 + y 2 ) dx dy dx dy =

1 du dv 4 ( x + y2 ) 2

The region in xy-plane bounded by the curves x2 y2 = 1, x2 y2 = 2, xy = 4, xy = 2 is transformed to a square in uv-plane bounded by the lines u = 1, u = 2, v = 4, v = 8. I = ∫∫ ( x 2 + y 2 ) dx dy = ∫

2

1

1 2 8 u v 4 1 4 =1 =



8 4

( x2 + y2 )

1 du dv 4 ( x + y2 ) 2

8.60

Engineering Mathematics

Fig. 8.53 Example 3: Using the transformation x + y = u, y = uv, show that 1

1− x

0

0

∫∫

y

e x + y dy dx =

1 ( e − 1). 2

Solution: x + y = u , x

y = uv

u (1 v),

y

uv

( x, y ) = J= ( u, v ) 1 v v

u u

x u y u

x v y v

(1 v ) u u v

u

dx dy = J du dv = u du dv Limits of y : y = 0 to y = 1 x Limits of x : x = 0 to x = 1. The region in xy-plane is the triangle OPQ bounded by the lines x = 0, y = 0 and x + y = 1. Under the transformation x = u (1 v) and y = uv, (i) the line x = 0 gets transformed to the line u = 0 or v = 1 (ii) the line y = 0 gets transformed to the line u = 0 or v = 0 (iii) the line x + y = 1 gets transformed to the line u = 1

Multiple Integral

8.61

Fig. 8.54 Thus, the triangle OPQ in the xy-plane gets transformed to the square OP'Q'R' in uv-plane bounded by the lines u = 0, v = 0, u = 1 and v = 1. In the region, draw a vertical strip AB parallel to the v-axis which starts from the u-axis and terminates on the line v = 1. Limits of v : v = 0 to v = 1 Limits of u : u = 0 to u = 1 I=

1

1 x

0

0

ev

1 0

y

e x + y dx dy =

u2 2

1

1

0

0

1

(e1 e 0 ) 0

e v u du dv

1 2

1 (e 1) 2 Example 4: Using the transformation x = u (1 + v ), y = v (1 + u), u evaluate

2 0

y 0

( x - y ) 2 + 2( x + y ) + 1

1 2

dy dx .

Solution: x = u (1 + v), y = v (1 + u )

( x, y ) J= = ( u, v )

x u y u

x u v 1+ v = = 1+ u + v y v 1 u v

dx dy = J du dv = (1 + u + v ) du dv Limits of x : x = 0 to x = y Limits of y : y = 0 to y = 2.

0, v

0,

8.62

Engineering Mathematics

Fig. 8.55 The region in the xy-plane is the OPQ bounded by the lines x = 0, y = 2 and y = x. Under the transformation x u (1 v ), y v (1 u ), u 0, v 0 (i) the line x = 0 gets transformed to the line u = 0 (ii) the line y = 2 gets transformed to the curve v (1 + u) = 2 (iii) the line y = x gets transformed to the line u = v Thus, the triangle OPQ in the xy-plane gets transformed to the region OP'Q' in uv plane bounded by the lines u = 0, u = v and the curve v (1 + u) = 2. The point of intersection of u = v and v (1 + u) = 2 is obtained as u2 + u 2 = 0, u = 1, 2 and v = 1, 2. Hence, P' : (1, 1) In the region, draw a vertical strip AB parallel to the v-axis which starts from the line u = v and terminates on the curve v (1 + u) = 2. 2 Limits of v : v = u to v = 1+ u Limits of u : u = 0 to u = 1 y

2

I

=

0

0

1

2 1+ u

0

u

1

2 1+ u

0

u

1

2 1+ u

0

u

y)2

(x

2( x

(u v ) 2

dy dx

2(u v 2uv ) 1

1 2

(1 u v ) du dv

(1 + u + v ) 1 (1 + u + v ) dv du dv du

1 0

u2 2 log (1 u ) 2 2 log 2

1 2

y) 1

1 2

2

v 1u+ u du 1

0

1 0

2 u du 1+ u

Multiple Integral

Example 5: Evaluate

8.63

xy dx dy by changing the variables over the region in

the first quadrant bounded by the hyperbolas x2 - y2 = a2, x2 - y2 = b2 and the circles x2 + y2 = c2, x2 + y2 = d2 with 0 < a < b < c < d. Solution: Let x 2

y2

u, x 2

y2

v

x2 =

u v , 2

y2 =

v u 2

1 x x 4 x u v = 1 y y 4y u v 1 dx dy = J du dv = du dv 8 xy ( x, y ) J= = ( u, v )

xy dx dy =

1 1 4x = 1 8 xy 4y

du dv 8

The region bounded by the hyperbolas x2 y2 = a2, x2 y2 = b2 and the circles x2 + y2 = c2, x2 + y2 = d 2 in xy-plane is the curvilinear rectangle PQRS. Under the transformation x2 y2 = u and x2 + y2 = v, (i) the hyperbolas x2 y2 = a2, x2 y2 = b2 get transformed to the lines u = a2, u = b2 respectively. (ii) the circles x2 + y2 = c2, x2 + y2 = d 2 get transformed to the lines v = c2, v = d 2 respectively.

Fig. 8.56

8.64

Engineering Mathematics

Thus, the curvilinear rectangle PQRS in the xy-plane gets transformed to the rectangle P'Q'R'S' in uv-plane bounded by the lines u = a2, u = b2, v = c2 and v = d 2. In the region, draw a vertical strip AB parallel to v-axis which starts from the line v = c2 and terminates on the line v = d 2. Limits of v : v = c2 to v = d 2 Limits of u : u = a2 to u = b2 b2 d2 1 I du dv xy dx dy 2 u=a v = c2 8 1 b2 d 2 1 2 (b a 2 ) ( d 2 c 2 ) u 2 v 2 8 a c 8 ( x + y )2 dx dy , by changing the variables over the paral-

Example 6: Evaluate

lelogram with vertices (1, 0), (3, 1), (2, 2), (0, 1). Solution: The region of integration in xy-plane is the parallelogram PQRS. Equations of the sides of the parallelogram are obtained as 1 0 ( x 1) 3 1 2y x 1 x 2y 1

(i) PQ : y 0

(ii) RS : y 1 2y 2 x 2y

2 1 ( x 0) 2 0 x 2

1 0 ( x 1) 0 1 x+ y =1

(iii) PS : y 0

(iv) QR : y − 1 =

2 −1 ( x − 3) 2−3

Fig. 8.57

y 1 x 3 x+ y = 4

Let x 2 y x=

u, x

y

v

u 2v v u , y= 3 3 ∂x ∂x 1 ∂ ( x , y ) ∂u ∂v 3 J= = = 1 ∂ ( u , v ) ∂y ∂y − ∂u ∂v 3 1 dx dy = J du dv = du dv 3

2 3 1 = 1 3 3

Multiple Integral

Under the transformation x x+y=v

2y = u, and

(i) the lines x 2y = 1, x 2y = 2 get transformed to the lines u = 1, u = 2 respectively. (ii) the lines x + y = 1, x + y = 4 get transformed to the lines v = 1, v = 4 respectively Thus, the parallelogram PQRS in the xy-plane gets transformed to a square P'Q'R'S' in uv-plane bounded by the lines u = 1, u = 2, v = 1 and v = 4. In the region, draw a vertical strip AB parallel to v-axis which starts from the line v = 1 and terminates on the line v = 4. Limits of v : v = 1 to v = 4 Limits of u : u = 2 to u = 1 I=

ellipse

Fig. 8.58

( x + y ) 2 dx dy =

1 1 v3 = u 2 3 3

Example 7: Evaluate

8.65

1 u

4 2 v 1

1 v 2 du dv 3

4

= 21 1

x2 y2 xy 2 + 2 a b

n 2

dx dy , over the first quadrant of the

x2 y2 + = 1. a 2 b2

Solution:

∂x ∂x ∂ ( x , y ) ∂r ∂ J= = ∂ ( r , ) ∂y ∂y ∂r ∂ a cos − a r sin = = ab r b sin b r cos d x d y = | J | d r d = ab r d r d Under the transformation x = ar cos q, x2 y2 y = br sin q, the ellipse 2 + 2 = 1 in a b the xy-plane gets transformed to r2 = 1 or

Fig. 8.59

8.66

Engineering Mathematics

r = 1, circle with centre (0, 0) and radius 1 in the rq -plane. The region of integration is the part of the circle r = 1 in first quadrant in the rq -plane. In the region, draw an elementary radius vector OA from the pole which terminates on the circle r = 1. Limits of r : r = 0 to r = 1 Limits of q : q = 0 to

=

2

n 2

2

x y2 xy 2 + 2 a b

I= =

2 0

1 0

dx dy Fig. 8.60 n 2 2

ab r 2 cos sin ( r ) ab r d r d

= a2b2

2 0

sin 2 2

a 2 b 2 cos 2θ = − 2 2

1 0

π 2 0

(r )n+3 d r 1

r n+ 4 n+4 0

2 2

ab a2b2 1 = ( − cos π + cos 0) ⋅ n + 4 2 ( n + 4) 4

=

Exercise 8.5 1. Using the transformation x + y = u, x

y = v, evaluate

e

x y x+ y

dx dy over

the region bounded by x = 0, y = 0 and x + y = 1. Ans. :

1 1 e 4 e

2. Using the transformation x2 y2 = u, 2xy = v, evaluate ( x 2 y 2 ) dx dy over the region bounded by the hyperbolas x2 y2 = 1, x2 y2 = 9, xy = 2 and xy = 4. [Ans. : 4] 3. Using the transformation x + y = u, y = uv evaluate

0

0

e

( x y)

x p 1 y q 1 dx dy.

Ans. : p q 4. Using the transformation x = u, y = uv, evaluate

1

x

0

0

x 2 + y 2 dx dy.

⎡ ⎤⎤ 1⎡ 2 1 + log 1 + 2 ⎥ ⎥ ⎢ Ans. : ⎢ 3⎣ 2 2 ⎢⎣ ⎦ ⎥⎦

(

5. Evaluate

)

( x + y ) 2 dx dy by changing

the variables over the region bounded by the parallelogram with sides x + y = 0, x + y = 2, 3x 2y = 0 and 3x 2y = 3. Ans. :

8 5

Multiple Integral

6. Evaluate

(x

1

y ) 4 e x + y dx dy, by

changing the variables over the region bounded by the square with vertices at (1, 0), (2, 1), (1, 2), (0, 1). Ans. :

8.67

e3

7. Evaluate [ xy (1 x y )] 2 dx dy, by changing the variables over the region bounded by the triangle with sides x = 0, y = 0, x + y = 1.

e Ans. :

5

2 105

8.6 TRIPLE INTEGRAL Let f (x, y, z) be a continuous function defined in a closed and bounded region V in 3-dimensional space. Divide the region V into small elementary parallelopipeds by drawing planes parallel to the coordinate planes. Let the total number of complete parallelopipeds which lie inside the region V is n. Let dVr be the volume of the r th parallelopiped and (xr, yr, zr) be any point in this parallelopiped. Consider the sum n

S = ∑ f ( xr , yr , zr ) Vr

... (1)

r =1

vr = xr ⋅ yr ⋅ z r

where

If we increase the number of elementary parallelopipeds, i.e., n, then the volume of each parallelopiped decreases. Hence as n → ∞, Vr → 0. The limit of the sum given by Eq. (1), if it exists is called the triple integral of f (x, y, z) over the region V and is denoted by

f ( x , y , z ) dV v

Hence,

∫∫∫

f ( x, y, z ) dV = lim

n

∑ f (x , y , z ) r

n→∞ Vr → 0 r =1

v

r

r

Vr

dV = dx dy dz

where,

8.6.1 Evaluation of Triple Integral Triple integral of a continuous function f (x, y, z) over a region V can be evaluated by three successive integrations. Let the region V is bounded below by a surface z = z1 (x, y) and above by a surface z = z2 (x, y). Let the projection of region V in xy-plane is R which is bounded by the curves y = y1 (x), y = y2 (x) and x = a, x = b. Then the triple integral is defined as I=

b

y2 ( x )

a

y1 ( x )

{

z2 ( x , y ) z1 ( x , y )

}

f ( x , y , z ) dz dy dx

Note: The order of variables in dx dy dz indicates the order of integration. In some cases this order is not maintained. Therefore it is advisable to identify the order of integration with the help of the limits.

8.68

Engineering Mathematics

8.6.2 Triple Integral in Cylindrical Coordinates Cylindrical coordinates r, q, z are used to evaluate the integral in the regions which are bounded by cylinders along z-axis, planes through z-axis, planes perpendicular to the z-axis.

Fig. 8.61 Relations between cartesian (rectangular) coordinates (x, y, z) and cylindrical coordinates (r, q, f ) are given as x = r cosq y = r sinq z=z Then

where,

f ( x, y, z ) dx dy dz =

f ( r cos , r sin , z ) J dr d dz

∂x ∂r ∂ ( x , y , z ) ∂y J= = ∂ ( r , , z ) ∂r ∂z ∂r cos = sin 0

Hence,

f ( x , y , z ) dx dy dz =

r sin r cos 0

∂x ∂ ∂y ∂ ∂z ∂

∂x ∂z ∂y ∂z ∂z ∂z

0 0 =r 1

f ( r cos , r sin , z ) r dr d dz

Multiple Integral

8.69

8.6.3 Triple Integral in Spherical Coordinates Spherical coordinates (r, q, f) are used to evaluate the integral in the regions which are bounded by sphere with centre at the origin, cone with vertices at the origin and axis as z-axis. Relations between cartesian (rectangular) coordinates (x, y, z) and spherical coordinates (r, q, f) are given as x = r sinq cosf y = r sinq sinf z = r cosq Then

where,

f ( x , y , z ) dx dy dz =

f ( r sin cos , r sin sin , r cos ) J dr d d

∂x ∂r ∂ ( x , y , z ) ∂y J= = ∂ ( r , θ , φ ) ∂r ∂z ∂r

sin θ cos φ = sin θ sin φ cosθ = r 2 sin θ

Fig. 8.62

∂x ∂θ ∂y ∂θ ∂z ∂θ

∂x ∂φ ∂y ∂φ ∂z ∂φ

r cos θ cos φ r cosθ sin φ

− r sin θ sin φ r sin θ cos φ

− r sin θ

0

8.70

Engineering Mathematics

Hence,

∫∫∫ f ( x, y, z ) dx dy dz = ∫∫∫ f (r sin θ cos φ , r sinθ sin φ , r cosθ ) r

2

sin θ dr dθ dφ ⋅

Note: If the region of integration is a sphere x2 + y2 + z2 = a2 with centre at (0, 0, 0) and radius a, then limits of r, q, f are (i) For positive octant of a sphere, r : r = 0 to r = a :

= 0 to

=

:

= 0 to

=

2 2

(ii) For hemisphere, r : r = 0 to r = a : = 0 to : = 0 to

=

2 =2

(iii) For complete sphere, r : r = 0 to r = a : = 0 to = : = 0 to = 2

8.6.4 Change of Variable In some cases, evaluation of triple integral becomes easier by changing the variables. Let the variables x, y, z be replaced by new variables u, v, w by the transformation x = f 1(u, v, w), y = f 2(u, v, w), z = f 3(u, v, w). Then

where,

f ( x, y, z ) dx dy dz =

f ( f1 , f 2 , f 3 ) J du dv dw

∂x ∂u ∂ ( x , y , z ) ∂y J= = ∂ ( u, v , w ) ∂u ∂z ∂u

∂x ∂v ∂y ∂v ∂z ∂v

∂x ∂w ∂y ∂w ∂z ∂w

8.6.5 Working Rule for Evaluation of Triple Integral 1. Draw all the planes and surfaces and identify the region of integration. 2. Draw an elementary volume parallel to z (y or x) axis. 3. Find the variation of z (y or x) along the elementary volume.

Multiple Integral

8.71

4. Lower and upper limits of z (y or x) are obtained from the equation of the surface (or plane) where elementary volume starts and terminates respectively. 5. Find the projection of the region on xy (zx or yz) plane. 6. Draw the region of projection in xy (zx or yz) plane. 7. Follow the steps of double integration to find the limits of x and y (z and x or y and z). Note: 1. If the region is bounded by the cylinders along the z-axis, planes through zaxis, the planes perpendicular to the z-axis, then the variables are changed to cylindrical coordinates. 2. If the region is bounded by the sphere, then the variables are changed to spherical coordinates.

(I) Evaluation of Integral when Limits are Given Example 1: Evaluate

2

z

0

1

yz 0

xyz dx dy dz .

Solution: The innermost limits depend on y and z. Hence, integrating first w.r.t. x, I=

2

z

0

1

1 2

x2 2

yz

yz dy dz = 0

4 z

y z 3 dz 4 1

2 0

1 z8 8 8

z4 4

2

1 8

2 0

1 2

2

z

0

1

( y 2 z 2 ) yz dy dz

( z 4 1) z 3 dz

1 (32 4) 8

0

7 = 2 Example 2: Evaluate

1

1- x

1- x - y

0

0

0

1 dx dy dz . ( x + y + z + 1) 3

Solution: The innermost limits depend on x and y. Hence, integrating first w.r.t. z, I=

1

1 x

1 x y

0

0

0

1 2 1 2

(x

1

1 x

0

0

1 0

1 x 2 4

1 y z 1)3

dz dy dx =

y (1 x

2

y ) 1}

1 x

1 dx x y 10 x2 8

1 x

0

0

1 {x

y 4

1

1 2 1

x log ( x 1) 2 0

1 0

(x

1 x 4

2( x

1 y z 1) 2

1 x y

dy dx 0

1 dy dx y 1) 2 1

1

x (1 x ) 1

x 1

1 5 log 2 2 8

dx

8.72

Engineering Mathematics

e

Example 3: Evaluate

1

ex

log y 1

log z dx dy dz .

1

Solution: The inner most limit depends on x and middle limit depends on y. Hence, integrating first w.r.t. z, e

I

1

1

x ex

1

ex

log y

1

e 1

e e

y

1 1

1 y2 y 2

e 1

1 log y 1

z

1 dz dx dy z

[e x x e x 1]dx dy

e log y (log y 2) log y e (1 2) 1 dy

2) log y e 1] dy

y2 2

ex

ex

z log z 1

dx dy

z1

dy

x1

1 ex

log 1 log y

ex

[ y (log y

log y

e

log z dz dx dy

e x log e x

1

e

e

1

log y

e 1

1

ex

log y

e 1

y dy

[( y y2

1) log y 2 y e 1] dy e 1

e

(e 1) y 1

e

2 ⎛e ⎞ ⎛1 ⎞ y = log e ⎜ + e ⎟ − log 1 ⎜ + 1⎟ − + y − (e 2 − 1) + [(e − 1) (e − 1) ] ⎝2 ⎠ 4 ⎝2 ⎠ 1 2

=

⎛ e2 1 ⎞ e2 + e − ⎜ + e − − 1⎟ − e 2 + 1 + e 2 − 2 e + 1 2 4 ⎠ ⎝4

=

13 e2 − 2e + 4 4

Example 4: Evaluate 0

0

0

dx dy dz . (1 + x 2 + y 2 + z 2 )2

Solution: 1. It is difficult to integrate in cartesian form. Putting x = r sinq cosf, y = r sinq sinf, z = r cosq, the integral changes to spherical form. 2. Limits of x : x = 0 to x Limits of y : y = 0 to y Limits of z : z = 0 to z The region of integration is the positive octant of the plane. Limits of r : r = 0 to r = 0 to

Limits of

:

Limits of

: = 0 to

I=∫



0





0

0

∫ ∫

2

=

2 dx dy dz (1 + x 2 + y 2 + z 2 ) 2

π π ∞ 2 2 φ =0 θ =0 r = 0

=∫

=

∫ ∫

r 2 sin θ d r dθ dφ (1 + r 2 ) 2

Fig. 8.63

Multiple Integral

8.73

Putting r = tan t , dr = sec 2 t dt When r = 0, t = 0 r

,t

2

π 2 0

π

π

0

0

I = ∫ dφ ∫ 2 sin θ dθ ∫ 2

π 2 0

π 2 0



sin 2 t dt

3 1 1⎞ π 2 2 π 1 = ⋅ π ⎟= 4 2 2⎠ 4 2

π 1 ⎛3 = ⋅1 ⋅ B ⎜ , 2 2 ⎝2 =

π tan 2 t ⋅ sec 2 t dt = φ 2 − cosθ 4 0 sec t

π2 8

Example 5: Evaluate

a2 - x 2

a 0

a2 - x 2 - y2

0

0

xy z dx dy dz .

Solution: 1. It is difficult to integrate in cartesian form. Putting x = r sinq cosf, y = r sinq sinf, z = r cosq, the integral changes to spherical form. 2. Limits of z : z

0 to z

a2

x2

2

2

y2

a2 x 2

a 0

0

0

2

2

=0

=0

π 2 0

=∫

a2 x 2 y 2

a r =0

q

1 21 a ⋅ 4 3 6 a6 = 48 =

y

O

φ

x

Fig. 8.64

xyz dx dy dz

r 3 sin 2 cos

cos sin

r 2 sin dr d d

π a sin 2φ 1 cos 2φ dφ ∫ 2 sin 3 θ cos θ dθ ∫ r 5 dr = − 0 0 2 2 2 6

P

r

Limits of y : y 0 to y a x Limits of x : x = 0 to x = a The region of integration is the positive octant of the sphere x2 + y2 + z2 = a2. Limits of r : r = 0 to r = a π Limits of θ : θ = 0 to θ = 2 π Limits of φ : φ = 0 to φ = 2 I=

z

π 2 0

1 r6 ⋅ B ( 2,1) 6 2

a

0

8.74

Engineering Mathematics

(II) Evaluation of Integral Over the Given Region Example 1: Evaluate

x 2 yz dx dy dz over the region bounded by the planes

x = 0, y = 0, z = 0 and x + y + z = 1. Solution: 1. Draw an elementary volume AB parallel to z-axis in the region. AB starts from xy-plane and terminates on the plane x + y + z = 1. Limits of z : z = 0 to z = 1 x y.

Fig. 8.65 2. Projection of the plane x + y + z = 1 in xy-plane is OPQ. Putting z = 0 in x + y + z = 1, we get equation of the line PQ as x + y = 1. 3. Draw a vertical strip A'B' in the region OPQ. A'B' starts from the x-axis and terminates on the line x + y = 1. Limits of y : y = 0 to y = 1 x Limits of x : x = 0 to x = 1 I=

1

1 x

1 x y

0

0

0

1 2 1 2 1 2

1

1 x

0

0

1

1 x

0

0

1

1 x

0

0

x 2 yz dz dy dx =

x 2 y (1 x x 2 y (1 x 2

1

1 x

0

0

x2 y

z2 2

1 x y

dy dx 0

y ) 2 dy dx y2

( x 2 y x 4 y x 2 y3

2 x 2 y 2 xy ) dy dx 2 x3 y 2 x 2 y 2

2 x 3 y 2 ) dy dx

Multiple Integral

8.75

1− x

1 1 y2 x2 y4 y3 = ∫ ( x 2 + x 4 − 2 x3 ) + − 2 ( x 2 − x3 ) 2 0 2 4 3 =

dx 0

2 x2 (1 − x )3 ⎤ 1 1⎡ 2 2 (1 − x ) 4 2 + ( 1 − x ) − 2 x ( 1 − x ) ⋅ 1 x ( − x ) ⋅ ⎢ ⎥ dx 2 4 3 ⎦ 2 ∫0 ⎣ 1

1 1 x2 1 (1 − x )5 2 (1 − x )6 (1 − x )7 (1 − x ) 4 dx = ⋅x − ⋅ 2x + ⋅2 = ∫ − 2 0 12 24 −5 30 −210 0 1 ⎛ 1 ⎞ ⎜0 + ⎟ 24 ⎝ 105 ⎠ 1 = 2520

=

Example 2: Evaluate x2 + y2 + z2 = 4.

xyz dx dy dz over the positive octant of the sphere

Solution: Putting x = r sinq cosf, y = r sinq sinf, z = r cosq equation of the sphere x2 + y2 + z2 = 4 reduces to

z

r 2 sin 2 θ cos 2 φ + r 2 sin 2 θ sin 2 φ + r 2 cos 2 θ = 4 r 2 = 4, r = 2. q

The region is the positive octant of the sphere r = 2.

=

xyz dx dy dz = 2 0

sin

3

cos d

1 sin 4 4 2 =

4 3

sin 0

2 0

2 0

y

O φ

Limits of r : r = 0 to r = 2 π Limits of q = 0 to θ = 2 π Limits of f = 0 to φ = 2 I=

P

r

x

Fig. 8.66 2 0

2 0

( r 3 sin 2 cos cos sin ) r 2 sin dr d d

sin 2 d 2 1 (cos 4

2 0

sin 4 r dr = 4 5

6

cos 0)

2 6

2

0

cos 2 4

2 0

r6 6

2

0

8.76

Engineering Mathematics

dx dy dz

Example 3: Evaluate

2

a - x2 - y2 - z2

over the region bounded by the sphere

x2 + y2 + z2 = a2. Solution: 1. Putting x = r sinq cosf, y = r sinq sinf, z = r cosq equation of the sphere x 2 + y 2 + z 2 = a 2 reduces to r = a. 2. For the complete sphere, limits of r : r = 0 to r = a limits of q : q = 0 to q = p limits of f : f = 0 to f = 2p

I=∫



0

π

a

0

0

∫ ∫

r 2 sin θ dr dθ dφ



= φ 0 − cos θ

a2 − r 2 π 0



a

0

⎛ ⎜⎝



π

a

0

0

0

= ∫ dφ ∫ sin θ dθ ∫

r 2 + a2 − a2 a2 − r 2

dr

⎞ − a 2 − r 2 ⎟ dr ⎠ a −r a2

2

2

a

= ( 2π )( − cos π + cos 0) a 2 sin −1

r r 2 2 a 2 −1 r − a − r − sin a 2 a0 2

⎞ ⎛ a2 a2 π = 4π ⎜ sin −1 1⎟ = 4π ⋅ ⋅ = π 2 a 2 2 2 ⎠ ⎝2 Example 4: Evaluate

dx dy dz 1

over the region bounded by the

( x2 + y2 + z2 )2 spheres x 2 + y 2 + z 2 = a 2 and x 2 + y 2 + z 2 = b 2 , a > b > 0. Solution: 1. Putting x = r sinq cosf, y = r sinq sinf, z = r cosq, equation of the spheres x2 + y2 + z2 = a2 and x2 + y2 + z2 = b2 reduces to r = a and r = b respectively.

Fig. 8.67

Multiple Integral

8.77

2. Draw an elementary radius vector OAB from the origin in the region. This radius vector enters in the region from the sphere r = b and leaves the region at the sphere r = a. 3. Limits of r : r = b to r = a. For complete sphere, limits of q : q = 0 to q = p limits of f : f = 0 to f = 2p

I=∫



0

π

a

0

b

∫ ∫

π a 2π r 2 sin θ dr dθ dφ = ∫ dφ ∫ sin θ dθ ∫ r dr b 0 0 r a



= φ 0 − cos θ

r2 (a2 − b2 ) = 2π ( − cos π + cos 0) = 2π ( a 2 − b 2 ) 2 b 2

π 0

z 2 dx dy dz over the region common to the sphere

Example 5: Evaluate

x 2 + y 2 + z 2 = 4 and the cylinder x 2 + y 2 = 2 x . Solution: 1. Putting x = r cos , y = r sin , z = z , equation of the (i) sphere x2 + y2 + z2 = 4 reduces to r2 + z2 = 4 z2 4 r2 (ii) cylinder x 2 + y 2 = 2 x reduces to r2 = 2r cos q, r = 2 cos q 2. Draw an elementary volume parallel to zaxis in the region. This elementary volume starts from the part of the sphere z 2 4 r 2 , below xy-plane and terminates on the part of the sphere z 2 4 r 2 , above xy-plane. Limits of r : z = − 4 − r 2 to z = 4 − r 2 3. Projection of the region in rq plane is the circle r = 2 cosq. 4. Draw an elementary radius vector OA in the region (r = 2 cosq ) which starts from the origin and terminates on the circle r = 2 cosq

Fig. 8.68

Limits of r : r = 0 to r = 2 cosq Limits of

:

z 2 dx dy dz

I

to

2 2 2

2 2

2 cos 0

z3 3

2 cos 0

4 r2

r dr d 4 r2

2 4 r2

z 2 r dz dr d

4 r2

1 3

2 2

2 cos 0

3

2(4 r 2 ) 2 r dr d

8.78

Engineering Mathematics

=

π 3 ⎤ 1 2 2 cosθ ⎡ 2 2 − ( − r ) ( −2r ) dr ⎥ dθ 4 π ∫ ⎢ ∫ 3 −2 0 ⎣ ⎦ 5 2 cosθ

1 2 (4 − r 2 ) 2 = − ∫ 2π 3 −2 5

f ( r )] n +1

⎤ ⎥ ⎥⎦

⎡∵ a f ( ) d = 0, if f ( − ) = − f ( ⎢ ∫− a ⎢ sin 5 ( − ) = − sin 5 ⎣

)⎤ ⎥ ⎥ ⎦

⎡ ⎢∵ ⎢⎣

π

dθ 0

2 15

(2

5

sin 5

2 0 25 15

2

2

=

n +1

)

25 d

2

6

[ n ∫ [ f ( r ) ] f ′ ( r ) dr =

2

2 64 = 15 15

Example 6: Evaluate

x y z dx dy dz , over the region bounded by the planes

x = 0, y = 0, z = 0, z = 1 and the cylinder x 2 + y 2 = 1. Solution: 1. Putting x = r cosq, y = r sinq, z = z, equation of the cylinder x 2 + y 2 = 1 reduces to r2 = 1, r = 1. 2. Draw an elementary volume AB parallel to z-axis in the region. This elementary volume AB starts from xy-plane and terminates on the plane z = 1. Limits of z : z = 0 to z = 1. 3. Projection of the region in rq-plane is the part of the circle r = 1 in the first quadrant. 4. Draw an elementary radius vector OA' in the region in the rq-plane which starts from the origin and terminates on the circle r = 1.

Fig. 8.69

Multiple Integral

8.79

Limits of r : r = 0 to r = 1 Limits of q : q = 0 to I

x y z dx dy dz 1 0

z dz

2 0

sin 2 d 2

= 1

2

z =0 1 0

2

=0

3

r dr

1 r =0

z2 2

r 2 cos sin 1

0

cos 2 4

z r dr d dz 2 0

r4 4

1

0

1 = 16 Example 7: Evaluate

x 2 + y 2 dx dy dz , over the region bounded by the

right circular cone x 2 + y 2 = z 2 , z > 0 and the planes z = 0 and z = 1. Solution: 1. Putting x = r cos , y = r sin , z = z , equation of the cone x 2 + y 2 = z 2 reduces to r2 = z2, r = z. 2. Draw an elementary volume AB parallel to z-axis in the region, which starts from the cone r = z and terminates on the plane z = 1. Limits of z : z = r to z = 1. 3. Projection of the region in rq -plane is the curve of intersection of the cone r = z and the plane z = 1 which is obtained as r = 1, a circle with centre at the origin and radius 1. 4. Draw an elementary radius vector OA' in the region in xy (rq ) plane which starts from the origin and terminates on the circle r = 1. Limits of r : r = 0 to r = 1 Limits of q : q = 0 to q = 2p

Fig. 8.70

8.80

Engineering Mathematics

x2

I 2

1

0

0 2 0

=

=0 2

1

r 2 z r dr d

r3 3

2

y 2 dx dy dz

r4 4

0

1

2 0

d

1

1

r =0

z=r

1 0

r r dz dr d

r 2 (1 r ) dr

1 12

6

Example 8: Evaluate

( x 2 + y 2 ) dx dy dz , over the region bounded by the

paraboloid x 2 + y 2 = 3 z and the plane z = 3. Solution: 1. Putting x = r cosq, y = r sinq, z = z, equation of the paraboloid x2 + y2 = 3z reduces to r2 = 3z.

Fig. 8.71 2. Draw an elementary volume AB parallel to z-axis in the region which starts from the paraboloid r 2 = 3 z and terminates on the plane z = 3. Limits of z : z =

r2 to z = 3. 3

3. Projection of the region in rq -plane is the curve of intersection of the paraboloid r 2 = 3 z and the plane z = 3 which is obtained as r 2 = 9, r = 3, a circle with centre at the origin and radius 1. 4. Draw an elementary radius vector OA' in the region (circle r = 3) which starts from origin and terminates on the circle r = 3. Limits of r : r = 0 to r = 3 Limits of q : q = 0 to q = 2p

Multiple Integral

( x2

I 2

3

0

0

2

y 2 ) dx dy dz

0 2

3

r 3 z r 2 dr d

0

3

3

0

r2 3 3

d

0

3 2 0

=

r6 18 0

35 4

2

r 2 r dz dr d

r3 3

r2 dr 3

36 18

81 2

Example 9: Evaluate the ellipsoid

3

3r 4 4

8.81

x2 y2 z2 - dx dy dz , where V is the volume of a 2 b2 c 2

1-

x2 y2 z2 + + = 1. a 2 b2 c 2

Solution: Evaluation of integral becomes easier by changing the variables. x y z x2 y2 z2 = u, = v, = w , the ellipsoid 2 + 2 + 2 = 1 gets a b c a b c transformed to u 2 + v 2 + w2 = 1, which is a sphere of radius 1 and centre at origin. Under the transformation

∂x ∂u ∂ ( x , y , z ) ∂y Jacobian, J = = ∂ ( u , v , w ) ∂u ∂z ∂u

∂x ∂v ∂y ∂v ∂z ∂v

∂x ∂w a 0 0 ∂y = 0 b 0 = abc ∂w 0 0 c ∂z ∂w

dx dy dz = J du dv dw = abc du dv dw I = ∫∫∫ 1 −

x2 y2 z2 − − dx dy dz a2 b2 c2

= ∫∫∫ 1 − u 2 − v 2 − w 2 abc du dv dw Using u = r sinq cos f, v = r sinq sinf, w = r cosq and du dv dw = r2 sinq dr dq df, the equation of the sphere u2 + v2 + w2 = 1 reduces to r2 = 1, r = 1. For complete sphere, limits of r : r = 0 to r = 1 limits of : = 0 to = limits of : = 0 to = 2 I

2 0

abc

1 0

0 2

0

d

1 r 2 abc r 2 sin dr d d 0

sin d

1 0

r 2 1 r 2 dr

8.82

Engineering Mathematics

Putting r = sin t, dr = cos t dt When r = 0, t = 0

r = 1, t =

2 2π

I = abc φ 0 − cos θ

π 0

π 2 0



sin 2 t ⋅ cos t ⋅ cos t dt

1 ⎛3 3⎞ = abc (2π ) (2) ⋅ B ⎜ , ⎟ 2 ⎝2 2⎠ 1 1 3 3 2 2 = 2 abc 2 2 = 2 abc 2 3 2 abc = 4

2

x 2 y 2 z 2 dx dy dz , over the region bounded by the surfaces

Example 10: Evaluate

xy = 4, xy = 9, yz = 1, yz = 4, zx = 25, zx = 49. Solution: Evaluation of integral becomes easier by changing the variables. Under the transformation xy = u, yz = v, zx = w, surfaces gets transformed to u = 4, u = 9, v = 1, v = 4, w = 25, w = 49. These equations represent the planes parallel to vw, wu and uv planes in the new coordinate system. It is easier to find partial derivatives of u, v, w w.r.t. x, y and z.

( u, v , w ) = ( x, y, z )

Jacobian, J =

y 0

x 0 z y

z

0

u x v x

u y v y

u z v z

w x

w y

w z

y ( zx 0) x (0

x

du dv dw = J dx dy dz = 2 xyz dx dy dz u v w = x2 y 2 z 2 xyz = u v w du dv dw = 2 uvw dx dy dz dx dy dz =

1 2 uvw

d u dv dw

yz ) = 2 xyz

Multiple Integral

8.83

Limits of u : u = 4 to u = 9 Limits of v : v = 1 to v = 4 Limits of w : w = 25 to w = 49 I=

x 2 y 2 z 2 dx dy dz 49

4

9

w = 25 v =1 u = 4

=

=

1 2

49 25

1

w 2 dw

1 2w 2 3

3 49 2

1

uvw 4 1

2v 3

2 uvw

1

v 2 dv 3 4 2

25

9 4

2u 3

1

du dv dw

1

u 2 du

3 9 2

4

4 (343 125) (8 1) (27 8) 27 115976 = 27

Exercise 8.6 (I) Evaluate the following: 1.

1 0

dx

2 0

2

dy

1

x 2 yz dz

5.

4

2 z

0

0

4 z x2 0

dy dx dz

[Ans. : 1] x

log 2

2.

∫ ∫ ∫ 0

0

x+ y

0

Ans. :

3.

a2 r 2

a cos

2 0

0

[Ans. : 8p ]

e x + y + z dz dy dx

0

5 8

6.

a sin

2 0

0

a2 r 2 a 0

r dz dr d Ans. :

r dz dr d a3 Ans. : 3 2

2 3

7.

2

y

0

0

x+ y x y

5a 3 64

( x + y + z ) dz dx dy [Ans. : 16]

4.

π

a (1+ cos θ )

0

0

∫ ∫



h

0

⎡ ⎤ r 2 ⎢1 − ⎥ ⎣ a (1 + cos θ ) ⎦ r dz dr dθ a2 h Ans. : 2

8.

a

∫ ∫ 0

0

a2 − x 2



0

a2 − x 2 − y 2

xyz dz dy dx.

Ans. :

a6 48

8.84

Engineering Mathematics

(II) Evaluate the following over the given region of integration: bounded by the spheres x2 + y2 + z2 = a2 and x2 + y2 + z2 = b2, a > b > 0.

( x + y + z ) dx dy dz , over the tetra-

1.

hedron bounded by the planes x = 0, y = 0, z = 0 and x + y + z = 1. Ans. :

2.

1 8

dx dy dz , over the tetra(1 + x + y + z )3 hedron bounded by the planes x = 0, y = 0, z = 0 and x + y + z = 1. 1 5 Ans. : log 2 2 8

8.

z 2 dx dy dz , over the region common to the spheres x2 + y2 + z2 = a2 and cylinder x2 + y2 = ax. Ans. :

9.

octant of the sphere x 2 + y 2 + z 2 = a 2.

( x 2 + y 2 ) dx dy dz , over the

a6 Ans. : 48

Ans. : 10.

x y z ( x 2 + y 2 + z 2 ) dx dy dz ,

4.

over the sphere of radius a and centre at the origin.

Ans. :

11.

∫∫∫

z2 dx dy dz , over the x2 + y2 + z2

Ans. : dx dy dz

7.

3 2 2

(x + y + z ) 2

2

12. 8

x2 a2

y2 b2

Ans. :

sphere x 2 + y 2 + z 2 = 2. 2 9

, over the region

a3 b 2 c 2 2520

x y z dx dy dz , over the positive octant of the ellipsoid

4 a7 Ans. : 35 6.

x 2 y z dx dy dz , over the tetrahe-

a8 64

( y 2 z 2 + z 2 x 2 + x 2 y 2 ) dx dy dz ,

5.

16 3

dron bounded by the planes x = 0, x y z y = 0, z = 0 and + + = 1. a b c

over the positive octant of the sphere x 2 + y 2 + z 2 = a 2. Ans. :

2 a5 15

region bounded by the paraboloid x 2 + y 2 = 2 z and the plane z = 2.

x y z dx dy dz , over the positive

3.

a b

Ans. : 4 log

z2 c2

1.

a2b2c2 48

x2 y2 z2 + + dx dy dz over 1 4 9 the region bounded by the ellipsoid x2 y2 z2 + + = 1. 1 4 9 [Ans. : 8p ]

Multiple Integral

8.85

8.7 APPLICATIONS OF MULTIPLE INTEGRALS 8.7.1 Area in Cartesian Form (i) The area bounded by the curves y = y1(x) and y = y2(x) intersecting at the points P (a, b) and Q (c, d) is given as Area =

c

y2 ( x )

a

y1 ( x )

dy dx

(ii) If equation of the curves are represented as x = x1 ( y ) and x = x2 ( y ), then Area =

d

x2 ( y )

b

x1 ( y )

dx dy

Note: Consider the symmetricity of the region while finding area. Example 1: Find the area bounded by the ellipse

Fig. 8.72 x2 y2 + = 1, above x-axis. a 2 b2

Solution: 1. The region is symmetric about the y-axis. Total area = 2 (Area bounded by the ellipse in the first quadrant) 2. Draw a vertical strip AB in the region which lies in the first quadrant. AB starts from the x-axis and terminates on the x2 y 2 + 2 = 1. 2 b ellipse a Limits of y : y = 0 to y = b 1 −

x2 a2

Limits of x : x = 0 to x = a

Area, A = 2 a

a

b 1

0

0

b 1−

= 2∫ y 0 0

x2 a2

x2 a2

Fig. 8.73 dy dx a

dx = 2 ∫ b 1 − 0

x2 a2

2b a 2 2b x 2 x a2 a − x 2 dx = a − x 2 + siin −1 ∫ 2 a a 0 a 2 ⎞ 2b ⎛ a 2 ⎞ 2b ⎛ a 2 = ⎜ sin −1 1⎟ = ⎜ ⋅ ⎟ a ⎝ 2 ⎠ a ⎝ 2 2⎠

a

=

=

ab 2

0

Engineering Mathematics

8.86

Example 2: Find the area bounded by the parabola y2 = 4x and the line 2x - 3y + 4 = 0. Solution: 1. The points of intersection of the parabola y2 = 4x and the line 2x – 3y + 4 = 0 are obtained as 2x + 4 3

2

= 4x

( x + 2) 2 = 9 x x2

5x 4 0 x = 1, 4 and y = 2, 4

Hence, P : (1, 2) and Q : (4, 4) 2. Draw a vertical strip AB which starts from the line 2x – 3y + 4 = 0 and terminates on the parabola y2 = 4x. Limits of y : y =

Fig. 8.74

2x + 4 to y = 2 x 3

Limits of x : x = 1 to x = 4 Area, A = ∫

4

1



2 x 2x+4 3

dy d x

4 4⎛ 2x + 4 ⎞ 2 x = ∫ y 2 x + 4 dx = ∫ ⎜ 2 x − ⎟ dx 1 1 3 ⎠ 3 ⎝

4

3

x2 2.2 3

x2 3

4x 3 1

4 1 (8 1) (16 1) 3 3

4 (4 1) 3

1 3

Example 3: Find the area bounded between the parabolas x2 = 4ay and x2 = -4a(y - 2a). Solution: 1. The parabola x 2 = 4ay has vertex (0, 0) and the parabola x 2 (0, 2a). Both the parabolas are symmetric about y-axis.

4a ( y 2a ) has vertex

Multiple Integral

8.87

Fig. 8.75 2

2. The points of intersection of x = 4ay and x 2 4a ( y 2a ) are obtained as 4ay = –4a (y – 2a) 8ay = 8 a2 y = a and x = ± 2a Hence, P : (2a, a) and R : ( 2a, a) 3. The region is symmetric about y-axis. Total area = 2 (Area in the first quadrant) 4. Draw a vertical strip AB in the region which lies in the first quadrant. AB starts from the parabola x2 = 4ay and terminates on the parabola x 2 4a ( y 2a ). x2 x2 to y = 2a − 4a 4a Limits of x : x = 0 to x = 2a Limit of y : y =

Area, A = 2 ∫

2a

= 2∫

2a

0

0



2a −

x2 4a

x2 4a

y

2a −

dy dx

x2 4a

x2 4a

dx

2a ⎛ x2 x2 ⎞ − ⎟ dx = 2 ∫ ⎜ 2a − 0 4a 4a ⎠ ⎝

= 2 2ax −

x3 6a

2a

0

4 ⎞ 16 ⎛ = 2 ⎜ 4a 2 − a 2 ⎟ = a 2 3 ⎠ 3 ⎝

Example 4: Find the larger area bounded by the circle x2 + y2 = 64a2 and the parabola y2 = 12ax.

Engineering Mathematics

8.88

Solution: 1. The points of intersection of the parabola y 2 = 12ax and the circle x 2 + y 2 = 64 a 2 are obtained as x 2 + 12ax − 64a 2 = 0 ( x + 16a ) ( x − 4a ) = 0 x = 4a and y = ±4a 3 [∵x = −16a does not lie on the parabola ]

(

)

(

Hence, P : 4a, 4a 3 and T : 4a, − 4a 3

)

Fig. 8.76 2. The region is symmetric about the x-axis. Total area = 2 (Area above x-axis) 3. Divide the region OPQR above x-axis into two subregions OQR and OPQ. Draw a vertical strip in each subregion. (i) In the subregion OQR, the strip AB starts from the x-axis and terminates on the circle x 2 + y 2 = 64a 2 Limits of y : y = 0 to y = 64a 2 − x 2 Limits of x : x = −8a to x = 0 2 (ii) In the subregion OPQ, the strip CD starts from the parabola y = 12ax and termi2 2 2 nates on the circle x + y = 64a

Limits of y : y = 12ax to 64a 2 − x 2 Limits of x : x = 0 to x = 4a ⎡ ⎤ Area, A = 2 ⎢ ∫∫ dy dx + ∫∫ dy dx ⎥ ⎢⎣ OQR ⎥⎦ OPQ

Multiple Integral

=2 =2 2

2

64 a 2 x 2

0 8a 0 0 8a 0 8a

y0

64 a 2 x 2

64a 2

dx +

64 a 2 x 2

4a

dy dx +

12 ax

0 4a

y

0

4a

x 2 dx

0

(

64 a 2 x 2 12 ax

x 64a 2 2

x2

dy dx

dx

64a 2

2

64a x sin 1 2 8a

8.89

)

x2 0

8a

12ax dx x 64a 2 2

3 4a 2

2

x2

64a x sin 1 2 8a

2 12a

x 3

0

3 ⎤ ⎡ 4a ⎛ 1 ⎞ 2 12a = 2 ⎢ −32a 2 sin −1 (−1) + 48a 2 + 32a 2 sin −1 ⎜ ⎟ − ( 4a ) 2 ⎥ 2 3 ⎝2⎠ ⎣ ⎦

⎡ 64 2 8 3 a 2 ⎤ ⎡ 32a 2 3 ⎤ ⎛ ⎞ a − = 2 ⎢ −32a 2 ⎜ − ⎟ + 8a 2 3 + 32a 2 − = 2 ⎢ ⎥ ⎥ 6 3 ⎦ 3 ⎦ ⎝ 2⎠ ⎣3 ⎣ 16 = a2 8 − 3 3

(

)

Example 5: Find the area common to the circles x 2 + y 2 - 4 y = 0 and x 2 + y 2 - 4 x - 4 y + 4 = 0. Solution: 1. The circles x 2

y2

4y

0 and x 2

y2

4x 4 y 4

0 have equal radii 2.

Fig. 8.77 2. The points of intersection of x 2 y 2 4 y 0 and x 2 obtained as –4x + 4 = 0 x = 1, y = 2 ± 3

(

Hence, P : 1, 2

)

(

3 and Q : 1, 2

3

)

y2

4x 4 y 4

0 are

Engineering Mathematics

8.90

3. The region is symmetric about the line PQ. Total area = 2 (Area P C2 Q) 4. Draw a vertical strip AB in the region P C2 Q which starts from the part of the circle x 2 y 2 4 y 0 below centre line (y < 2) and terminates on the part of the same circle above centre line (y > 2). Limits of y : y = 2 − 4 − x 2 to y = 2 + 4 − x 2 Limits of x : x = 1 to x = 2 Area, A = 2 ∫

2

1 2



(

2 + 4 − x2

2− 4− x

2

2

2 + 4 − x2

dy dx = 2 ∫ y 2 − 1

4 − x2

dx

)

= 2 ∫ 2 + 4 − x 2 − 2 + 4 − x 2 dx 1

2

2 1

2 4 x 2 dx 2

x 4 4 x2 2

1 1 3 2 sin 1 1 sin 1 2 2

4 0

3

4

4 x sin 1 2 21

2

2

2

6

4

2 3

3 2

2 2 2 2 Example 6: Find the area of the loop of the curve x ( x + y ) = a ( x - y ).

Solution: The equation of the curve can be rewritten as y 2 = x 2

a x a+x

1. The point of intersection of the curve with x-axis (y = 0) is obtained as x 2 (a x) 0, x 0, x a The loop of the curve lies between the points O : (0, 0) and P : (a, 0). 2. The region is symmetric about x-axis Total area = 2 (Area above x-axis) 3. Draw a vertical strip AB in the region above x-axis. AB starts from x-axis and a x terminates on the curve y 2 = x 2 . a+x a−x a+x

Limits of y : y = 0 to y = x Limits of x : x = 0 to x = a Area of the loop, A = 2 =2

a 0

a

x

0

0 x

y0

a x a+ x

a x a+ x

Fig. 8.78

dy dx

dx = 2

a 0

x

a x dx a+ x

Multiple Integral

8.91

Putting x = a cosq, dx = –a sinq dq π When x = 0, θ = 2 x = a, θ = 0 0

A= 2

a a cos ( a sin ) d a + a cos

a cos

2

2sin 2 2a

2

2 0

cos sin 2 cos

2 d

2

2 sin

2a 2

2a 2

2 0

cos

2

2sin cos 2 2 1 + cos 2 2

cos

0

2a 2 1

cos

d

2d

2a 2

2 0

cos (1 cos ) d

2 2a 2 sin

2

sin 2 4

2 0

4

Example 7: Find the area of the loop of the curve 2 y 2 = ( x - 2)( x - 10)2 . Solution: 1. The points of intersection of 2 y 2 obtained as ( x 2) ( x 10) 2

0, x

( x 2) ( x 10) 2 with the x-axis (y = 0) are 2,10.

The loop of the curve lies between the points P : (2, 0) and Q : (10, 0). 2. The region is symmetric about x-axis. Total area = 2 (Area of the loop above x-axis) 3. Draw a vertical strip in the region above x-axis. AB starts from x-axis and terminates on the curve 2 y 2 ( x 2) ( x 10) 2 . Limits of y : y

1

0 to y

2

(10 x) x 2

Limits of x : x = 2 to x = 10

Area of the loop, A = 2 2

10 2

10

(10 x )

2

0 (10 x )

y0

x 2 2

x 2 2

Fig. 8.79

dy dx

dx

2

10 2

(10 x)

x 2 2

dx

Engineering Mathematics

8.92

Putting When

x − 2 = t 2 , dx = 2t dt x = 2, t = 0 x = 10, t = 2 2

A =

2

∫ 2

2 2

0

2 2 2 2

(−t 2 + 8) t ⋅ 2t dt

2 2 0

( t4

8t 2 ) dt

128 2 5

2 2

128 2 3

t5 5

8t 3 3

2 2

0

1024 15

2 2 3 Example 8: Find the area bounded between the curve x ( y + a ) = a and its asymptote.

Solution: The equation of the curve can be rewritten as y 2 = x = 0 is the asymptote of the curve.

a 2 (a − x) . The line x

Fig. 8.80 1. The region is symmetric about the x-axis. Total area = 2 (Area above x-axis) 2. Draw a vertical strip AB in the region above x-axis. AB starts from the x-axis and a 2 (a x) , terminates on the curve y 2 = x Limits of y : y = 0 to y = a Limits of x : x = 0 to x = a

a−x x

Multiple Integral

Area,

A = 2∫

a



0

a

0

a

= 2∫ a 0

a−x x

a

a

dy dx = 2 ∫ y 0

a−x x

0

8.93

dx

a−x dx x

Putting x = a sin 2 θ , dx = 2a sin θ cos θ dθ When x = 0, θ = 0 π x = a, θ = 2 π

A = 2∫ 2 a 0

= 2a 2

a − a sin 2 θ ⋅ 2a sin θ cos θ dθ a sin 2 θ 2 0

2a 2

2 cos 2 d = 2a 2 sin 2 2

2

2a 2

0

2 0

2

(1 + cos 2 ) d 1 sin 2

0

= a2 Example 9: Find the area between the rectangular hyperbola 3xy = 2 and the line 12x + y = 6. Solution: 1. The points of intersection of the rectangular hyperbola 3xy = 2 and the line 12x + y = 6 are obtained as 3 x (6 − 12 x) = 2 18 x 2 − 9 x + 1 = 0 1 1 x = , and y = 2, 4 3 6 ⎛1 ⎞ ⎛1 P : ⎜ , 2 ⎟ and Q : ⎜ , Hence, ⎝3 ⎠ ⎝6

⎞ 4⎟ ⎠

2. Draw a vertical strip AB in the region which starts from the rectangular hyperbola 3xy = 2 and terminates on the line 12x + y = 6. 2 Limits of y : y = to y = 6 − 12 x 3x 1 1 Limits of x : x = to x = 6 3

Fig. 8.81

Engineering Mathematics

8.94

1

6 −12 x

6

3x

Area, A = ∫13 ∫ 2 1

= ∫13 y 6

dy dx

6 −12 x 2 3x

1 2 ⎞ ⎛ dx = ∫13 ⎜ 6 − 12 x − ⎟ dx x⎠ 3 ⎝ 6

1

6x 6x

2

3 2 log x 1 3

(2 1) 6

1 9

1 36

2 1 1 log log 3 3 6

6

1 2

2 log 2. 3

Example 10: Find the area of the curvilinear triangle bounded by the parabolas y2 = 12x, x2 = 12y, circle x2 + y2 = 45 and lying outside the circle. Solution: Required curvilinear triangle is PQR.

Fig. 8.82 1. The point of intersection of (i) the parabola x 2 = 12 y and the circle x 2 + y 2 = 45 are obtained as 45 y 2 y

2

12 y 45 y

12 y 0 3, 15

But y = –15 does not lie on the parabola x2 = 12y. Thus, y = 3, x = 6 Hence, P : (6, 3) (ii) the parabolas x 2 = 12 y and y 2 = 12 x are obtained as y4 = 12 y, y = 0, 12 and x = 0, 12 144 Hence, Q : (12, 12)

Multiple Integral

8.95

2 2 2 (iii) the parabola y = 12 x and the circle x + y = 45 are obtained as 45 – x2 = 12x, x2 + 12x – 45 = 0, x = 3, –15 but x = –15 does not lie on the parabola y2 = 12x. Thus x = 3, y = 6 Hence, R : (3, 6) 2. Divide the region PQR into two subregions PRS and PQS. Draw a vertical strip in each subregion. (i) In subregion PRS, strip starts from the circle x 2 + y 2 = 45 and terminates on the parabola y 2 = 12 x.

Limits of y : y = 45 − x 2 to y = 12 x Limits of x : x = 3 to x = 6 (ii) In subregion PQS, strip starts from the parabola x2 = 12y and terminates on the parabola y2 = 12x. x2 to y = 12 x 12 Limits of x : x = 6 to x = 12

Limits of y : y =

Area PQR, A = Area PRS + Area PQS =

6

12 x

3

45 x 2

6 3

(

dy dx +

2x 3

x2 12

)

45 x 2 dx

12 x

12

12 x

12 6

3 2

6

dy dx = 12 6

3

y

12 x 45 x 2

dx +

12 6

y

45 sin 2

1

dx

x2 dx 12

12 x

6

x 45 x 2 2

12 x x2 12

x

12

45

2x 3

12

3 2

3

x 36

3

4 3 6 6 3 3 3

(

)

3 9

3 36 2

4 3 1 12 12 6 6 (123 3 36 45 12 1 42 sin 1 sin 1 2 5 5

(

)

45 sin 2

6 1

2 5

sin

1

1 5

63 )

Example 11: Find the area bounded by the hypocycloid Solution: 1. The hypocycloid is symmetric in the coordinate plane.

x a

2 3

y + b

2 3

=1 .

8.96

Engineering Mathematics

Fig. 8.83 Total Area = 4 (Area in the first quadrant) 2. Draw a vertical strip AB parallel to y-axis in the region which lies in the quadrant. AB starts from x-axis and terminates on the curve 3

2 2 ⎤ ⎡ ⎛ x ⎞3 ⎥ ⎢ Limits of y : y = 0 to y = b 1 − ⎜ ⎟ ⎢ ⎝a⎠ ⎥ ⎦ ⎣ Limits of x : x = 0 to x = a 3

Area, A = 4 ∫

a

0



a

2 ⎤2 ⎡ ⎛ x ⎞3 b ⎢⎢1− ⎜ ⎟ ⎥⎥ a ⎢⎣ ⎝ ⎠ ⎥⎦

0

= 4∫ y 0

dy dx

3 2 ⎤2 x ⎞3 ⎥

⎡ ⎛ b ⎢⎢1− ⎜ ⎟ ⎥ ⎝a⎠ ⎥ ⎣⎢ ⎦ 0

dx 3

2 2 ⎡ ⎤ 3 a x ⎛ ⎞ = 4 ∫ b ⎢1 − ⎜ ⎟ ⎥ dx 0 ⎢ ⎝a⎠ ⎥ ⎣ ⎦

Putting x = a cos3 t , dx = 3a cos 2 t (− sin t ) dt When x = 0, t =

2 x = a, t = 0

x a

2 3

y + b

2 3

=1

Multiple Integral

0

8.97

3 2

A = 4 ∫ b(1 − cos t ) (−3a cos 2 t sin t ) dt 2

2

= 12ab ∫ 2 sin 4 t cos 2 t dt 0

5 3 3 1 1 1 1 ⋅ ⋅ 1 ⎛5 3⎞ 2 2 = 12ab B ⎜ , ⎟ = 6ab = 6ab 2 2 2 2 2 2 ⎝2 2⎠ 3! 4 3 = ab 8 Exercise 8.7 1. Find the area bounded by y-axis, the line y = 2x and the line y = 4. [Ans. : 4] 2. Find the area bounded by the lines y = 2 + x, y = 2 x and x = 5. [Ans. : 25] 3. Find the area bounded by the parabola y2 + x = 0, and the line y = x + 2. 9 Ans. : 2 4. Find the area bounded by the parabola x = y y2 and the line x + y = 0. 4 Ans. : 3 5. Find the area bounded by the curves y2 = 4x and 2x 3y + 4 = 0. 1 Ans. : 3 6. Find the area bounded by the parabola y = x2 3x and the line y = 2x. 125 Ans. : 6 7. Find the area bounded by the parabolas y2 = x, x2 = 8y. Ans. : 8. Find the area bounded by the parabolas y = ax2 and

8 3

y

x2 a

a Ans. :

4 a 3 a2 + 1

9. Find the area bounded by the curve y2 (2a x) = x3 and its asymptote. [Ans. : 3pa2] 10. Find the area of the loop of the a+ x curve y 2 = x 2 a x Ans. : 2a 2

4

1

11. Find the area of one of the loops of x4 + y4 = 2a2 xy. a2 Ans. : 4 12. Find the area enclosed by the curve 9xy = 4 and the line 2x + y = 2. 1 4 log 2 Ans. : 3 9 13. Find the area of the smaller region bounded by the circle x2 + y2 = 9 and a straight line x = 3 y. Ans. : 4

2 3

3 2

Engineering Mathematics

8.98

14. Find the area bounded by the x-axis, circle x2 + y2 = 16 and the line y = x. [Ans. : 2p] 15. Find the area bounded between the curves y = 3x2 x 3 and y = 2x2 + 4x + 7.

Ans. :

45 2

16. Find the area bounded by the asteroid 2

2

2

( x) 3 + ( y ) 3 = (a) 3 3 2⎤ ⎡ ⎢ Ans. : 8 a ⎥ ⎣ ⎦

8.7.2 Area in Polar Form The area bounded by the curves r = r1 (q ), r = r2 (q ) and the lines q = q1 and q = q2 is given as =

2 1

r2 ( ) r1 ( )

r r

Note: Consider the symmetricity of the region while finding area. Example 1: Find the area between the circles r = 2 sinp and r = 4 sinp. Fig. 8.84

Solution: . 2 Total area = 2 (Area in the first quadrant)

1. The region is symmetric about the line

=

Fig. 8.85 2. Draw an elementary radius vector OAB in the region which lies in the first quadrant. OAB enters in the region from the circle r = 2sinq and leaves at the circle r = 4 sinq.

Limits of r : r = 2 sin θ to r = 4 sin θ Limits of θ : θ = 0 to θ =

π 2

Multiple Integral

π 2 0

Area, A = 2 ∫



4 sin θ

2 sin θ

π 2 0

r d r dθ = 2 ∫

8.99

r2 2

4 sin θ

dθ 2 sin θ

π

= ∫ 2 (16 sin 2 θ − 4 sin 2 θ ) dθ 0

2 0

6 (1 cos 2 ) d sin 2 2

6

2

sin

6

2

0

sin 0

3

2

Example 2: Find the area of the crescent bounded by the circles r = 3 and r = 2cosp. Solution: 1. The points of intersection of r = 3 and r = 2 cos

are obtained as

3 = 2 cos θ 3 2 π θ =± 3

cosθ =

Hence, at P,

=

3

.

2. The region is symmetric about the initial line, q = 0. Area of the crescent = 2 (Area above the initial line, q = 0) 3. Draw an elementary radius vecFig. 8.86 tor OAB in the region above the initial line. OAB enters in the region from the circle r = 3 and leaves at the circle r = 2 cosq. Limits of r : r = 3 to r = 2 cos q p Limits of q : q = 0 to q = 3 π

Area, A = 2 ∫ 3 ∫ 0

π

= 2∫ 3 0

3 0

[ 2 (1

sin

2 3

2 cosθ 3

r2 2

r d r dθ

2 cosθ

π

dθ = ∫ 3 (4 cos 2 θ − 3) dθ 0

3

cos 2 ) 3] d

3

3 2

3

,

2

sin 2 2

3 0

Engineering Mathematics

8.100

But area cannot be negative. Hence, numerical value of area =

3

3 . 2

Example 3: Find the area which lies inside the circle r = 3a cosp and outside the cardioid r = a (1 + cosp ). Solution: 1. The points of intersection of the circle r = 3a cosq and the cardioid r = a (1 + cosq ) are obtained as

3a cos θ = a (1 + cos θ ) 1 cosθ = 2 π θ =± 3 Hence, at R, q =

p 3

Fig. 8.87 2. The region is symmetric about the initial line q = 0. Total area = 2 (Area above the initial line) 3. Draw an elementary radius vector OAB from the origin in the area above the initial line. OAB enters in the region from the cardioid r = a (1 + cosq ) and leaves at the circle r = 3a cosq.

Limits of r : r = a (1 + cos θ ) to r = 3a cos θ Limits of θ : θ = 0 to θ =

π 3

Multiple Integral

π 3 0

Area, A = 2 ∫

3 a cosθ



a (1+ cosθ )

8.101

π 3 0

r2 2

r dr dθ = 2 ∫

3 a cosθ

dθ a (1+ cosθ )

π

= ∫ 3 ⎡⎣9a 2 cos 2 θ − a 2 (1 + cos θ ) 2 ⎤⎦ dθ 0

a2

3 0

[4 (1 cos 2 ) 1 2 cos ]d 4sin 2 2

a2 3 a

2

3

3

2 2sin 3

2sin

3 0

2sin

3

= a2 Example 4: Find the area common to the cardioids r = a (1 + cosp ) and r = a (1 - cosp ). Solution: 1. The points of intersection of the cardioids r = a(1 + cosq ) and r = a(1 obtained as a (1 + cosq ) = a(1 cosq = 0

θ =± Hence, at P, θ =

cosq )

π 2

π 2

Fig. 8.88

cosq ) are

8.102

Engineering Mathematics

2. The region is symmetric in all the quadrants Total area = 4 (Area in the first quadrant) 3. Draw an elementary radius vector OA from the origin in the region which lies in the first quadrant. OA starts from the origin and terminates on the cardioid r = a (1 cosq ). Limits of r : r = 0 to r = a (1 − cos θ )

Limits of θ : θ = 0 to θ =

π 2 π

a (1− cosθ )

0

0

π 2 0

r2 2

Area, A = 4 ∫ 2 ∫ = 4∫

2a 2 2a 2 2a

2

2 0

r d r dθ

a (1− cosθ )

π

dθ = 2 ∫ 2 a 2 (1 − cos θ ) 2 dθ 0

0

1 2 cos

3 2

2sin

3 4

2

1 + cos 2 2 sin 2 4

d

2 0

Example 5: Find the area inside the cardioid r = 3 (1 + cosp ) and outside the 3 . parabola r = 1 + cos p Solution: 1. The points of intersection of the cardioid r = 3(1 + cosq ) and the parabola 3 are obtained as r= 1 + cos

Fig. 8.89

Multiple Integral

3 (1 + cosθ ) =

8.103

3 (1 + cosθ )

(1 + cosθ ) 2 = 1 cos θ = 0,θ = ±

π 2

π 2 2. The region is symmetric about the initial line q = 0. Total area = 2 (Area above the initial line) 3. Draw an elementary radius vector OAB from the origin in the region above the 3 initial line q = 0. OAB enters in the region from the parabola r = and 1 + cos leaves at the cardioid r = 3 (1 + cosq ). 3 Limits of r : r = to r = 3 (1 + cosθ ) 1 + cosθ π Limits of θ : θ = 0 to θ = 2 Hence, at P, θ =

π

Area, A = 2 ∫ 2 ∫ 0

3 (1+ cosθ )

3 1+ cosθ

π

r d r dθ = 2 ∫ 2 0

r2 2

3 (1+ cosθ )

dθ 3 1+ cosθ

π ⎡ ⎤ 1 dθ = ∫ 2 9 ⎢(1 + cosθ ) 2 − 2⎥ 0 (1 + cosθ ) ⎦ ⎣ ⎤ ⎡ ⎥ π ⎢ 1 + cos 2θ 1 ⎥ dθ − = 9 ∫ 2 ⎢1 + 2 cosθ + 2 0 ⎢ ⎥ 2 ⎛ 2θ ⎞ ⎢ ⎜ 2 cos ⎟ ⎥ 2⎠ ⎦ ⎝ ⎣

π ⎡3 cos 2θ 1 ⎛ θ⎞ θ⎤ = 9 ∫ 2 ⎢ + 2 cos θ + − ⎜1 + tan 2 ⎟ sec 2 ⎥ dθ 0 2 4⎝ 2⎠ 2⎦ ⎣2 π ⎡3 cos 2θ 1 2 θ 1 θ ⎛1 θ ⎞⎤ − sec − ⋅ tan 2 ⎜ sec 2 ⎟ ⎥ dθ = 9 ∫ 2 ⎢ + 2 cos θ + 0 2 4 2 2 2⎝2 2 ⎠⎦ ⎣2

θ tan sin 2θ 1 θ 1 3θ 2 + 2 sin θ + − ⋅ 2 tan − =9 2 4 4 2 2 3 3

π 2

0 n +1 ⎡ f (θ ) ] ⎤ [ n ⎢∵ ∫ [ f (θ ) ] f ′ (θ )dθ = ⎥ n + 1 ⎥⎦ ⎢⎣

Engineering Mathematics

8.104

9 =9

3 4

2sin

2

sin 4

1 tan 2 4

1 tan 3 6 4

3 4 + 4 3

Example 6: Find the area common to the circles r = cosp and r = 3 sin . Solution: 1. The point of intersection of the circles r = cosq and r = 3 sin 3 sin = cos tan = = Hence, at P, q =

is obtained as

1 3 6

p 6

Fig. 8.90 2. Divide the region OQPR into two subregions OQP and ORP. Draw an elementary radius vector in each subregion. (i) In subregion OQP, radius vector OA starts from the origin and terminates on the circle r = 3 sin . Limits of r : r = 0 to r = 3 sin q Limits of q : q = 0 to q =

p 6

Multiple Integral

8.105

(ii) In the subregion ORP, the radius vector OB starts from the origin and terminates on the circle r = cosq. Limits of r : r = 0 to r = cosq

Limits of

: =

p p to q = 6 2 π

Area, A = ∫ 6 ∫

3 sin θ

0

0

π

r2 2

= ∫6 0

π

r dr dθ + ∫π2 ∫ 6

3 sin θ

π

dθ + ∫π2 6

0

r2 2

cosθ

0

r d r dθ

cosθ

dθ 0

=

1 π2 1 π6 2 2 3 θ θ sin d + π cos θ dθ 2 ∫6 2 ∫0

=

3 π6 ⎛ 1 − cos 2θ ⎜ 2 ∫0 ⎝ 2

1 π2 ⎛ 1 + cos 2θ ⎞ + d θ π ⎜ ⎟ 2 ∫6 ⎝ 2 ⎠

3 4

1 4

3 4 6 5 24

sin 2 2

6 0

1 sin 2 3

sin 2 2

⎞ ⎟ dθ ⎠

2

6

1 4 2

1 sin 2

6

1 sin 2 3

4 3 16

Example 7: Find the area common to the circle r = a and the cardioid r = a (1 + cosp ). Solution: 1. The points of intersection of the circle r = a and the cardioid r = a (1 + cosq ) are obtained as a = a (1 + cosq ) π cos θ = 0, θ = ± 2 π Hence, at Q, θ = 2 2. The region is symmetric about the initial line q = 0 Total area = 2 (Area above the initial line) 3. Divide the region OPQR above the initial line into two subregions OPQ and ORQ. Draw elementary radius vectors in each subregion. (i) In the subregion OPQ, the radius vector OA starts from the origin and terminates on the circle r = a. Limits of r : r = 0 to r = a

Limits of θ : θ = 0 to θ =

π 2

Vector Calculus Chapter

9

9.1 INTRODUCTION Vector algebra deals with the operations of addition, subtraction and multiplication of vectors. Vector calculus deals with the differentiation and integration of vector functions. We will learn about multiplication of vectors in vector algebra and about derivative of a vector function, gradient, divergence and curl in vector differential calculus. In vector integral calculus, we will learn about line integral, surface integral, volume integral and three theorems namely Green’s theorem, divergence theorem and Stoke’s theorem. It plays an important role in the differential geometry and in the study of partial differential equations. It is useful in the study of rigid dynamics, fluid dynamics, heat transfer, electromagnetism, theory of relativity, etc.

9.2 UNIT VECTOR A unit vector is a vector having unit magnitude. If A is a vector, then unit vector in the A direction of A is given as aˆ = . | A| This also shows that A can be represented in terms of unit vector as A = | A | aˆ. Note: (i) The unit vectors in the direction of x, y and z- axes are denoted by iˆ, jˆ and kˆ respectively. (ii) iˆ · iˆ = jˆ · jˆ = kˆ · kˆ = 1, iˆ · jˆ = jˆ · kˆ = kˆ · iˆ = 0, iˆ jˆ = kˆ, jˆ kˆ = iˆ, kˆ iˆ = jˆ

9.3 COMPONENTS OF A VECTOR Let OA represent a vector with initial point at the origin O and terminal point at A. Let (A1, A2, A3) be the rectangular coordinates of the terminal point A. The vectors A1 iˆ, A2 jˆ, A3 kˆ are called the rectangular component vectors or component vectors of A in the x, y and z-directions respectively. A1, A2 and A3 are called the rectangular components or components of A in the x, y and z-directions respectively.

9.2

Engineering Mathematics

z

A (A1, A2, A3)

O

y

x

Fig. 9.1 A

A1iˆ + A2 jˆ + A3kˆ

A = A=

A12 + A22 + A32

In particular, the position vector of the point (x, y, z) w.r.t. origin is denoted by r and is written as r = x iˆ + y jˆ + z kˆ r = r = x2 + y 2 + z 2

9.4 TRIPLE PRODUCT 9.4.1 Scalar Triple Product Scalar triple product of three vectors a, b and c is a dot product of a vector a and

(

vector b

)

c . It is denoted by a b c and is also known as box product of vectors

a, b and c. If

a = a1 iˆ + a2 jˆ + a3 kˆ, b = b iˆ + b jˆ + b kˆ, 1

2

3

c = c1 iˆ + c2 jˆ + c3 kˆ then a1 ⎡⎣ a b c ⎤⎦ = b1 c1

a2 b2 c2

a3 b3 c3

Vector Calculus

9.3

Note:

(

(i) a b

c

) (a

)

b c

1 (ii) Volume of a parallelogram = ⎡⎣ a b c ⎤⎦ and volume of a parallelopiped = ⎡⎣ a b c ⎤⎦ 6 (iii) ⎡⎣ a b c ⎤⎦ = ⎡⎣b c a ⎤⎦ = ⎡⎣c a b ⎤⎦ (iv) If a, b, c are coplanar, then ⎡⎣ a b c ⎤⎦ = 0.

9.4.2 Vector Triple Product Vector triple product of three vectors a, b and c is a cross product of a vector a and

(

)

vector b

(

)

c or vector a

b and vector c .

( b c) ( a c) b ( a b) c b) c ( a c) b ( b c) a

a

(a Note: (i) a (ii) a

(b c) ( a b ) (b c) (b c)

Example 1: If a b

to ( b c ) .

Solution:

(a

d)

(b

c a

c d and a c

c)

(a (c

b d , show that ( a

d ) is parallel

b)

( a c) ( d b) ( d c) d ) (b d ) (b d ) (c d )

=0

Hence, ( a d ) is parallel to ( b c ) . Example 2: If a = ˆi + ˆj – kˆ, b = ˆi – ˆj + kˆ, c = ˆi – ˆj – kˆ, find the vector a

(b c) .

Solution: We know that, a

( b c) ( a c) b ( a b) c [(iˆ + jˆ kˆ) · (iˆ jˆ kˆ)](iˆ jˆ + kˆ) [(iˆ + jˆ kˆ) · (iˆ jˆ + kˆ)] (iˆ jˆ kˆ) = (iˆ · iˆ jˆ · jˆ + kˆ · kˆ)(iˆ jˆ + kˆ) (iˆ · iˆ jˆ · jˆ kˆ · kˆ) (iˆ jˆ kˆ) [∵ iˆ · jˆ = jˆ · kˆ = kˆ · iˆ = 0 ]

9.4

Engineering Mathematics = (1 1 + 1) (iˆ jˆ + kˆ) = iˆ jˆ + kˆ + iˆ jˆ kˆ = 2iˆ 2jˆ.

(1

1

1) (iˆ



kˆ)

Example 3: Find the scalars p and q, if ( a b ) c

(b

a

c ) where,

ˆ , b = iˆ – ˆj , c = 4iˆ + q ˆj + 2k ˆ. a = 2iˆ + ˆj + pk

(a

( a c) b a (b c) ( a c) b ( a b ) c a (b ( a c) b (b c) a ( a c) b ( b c) a ( a b) c

Solution: We know that, and Given,

[(iˆ

(b c) a (a b ) c c) ( a b) c

jˆ) · (4iˆ + q jˆ + 2kˆ)](2iˆ + jˆ + pkˆ) = [(2iˆ + jˆ + p kˆ) · (iˆ jˆ)](4iˆ + q jˆ + 2kˆ) (4 q)(2iˆ + jˆ + pkˆ) = (2 1) (4iˆ + q jˆ + 2kˆ) 8iˆ + 4jˆ + 4p kˆ

(8

b) c

2q) iˆ + (4

2q iˆ

q jˆ

q) jˆ + (4p

pqkˆ = 4iˆ + q jˆ + 2kˆ pq) kˆ = 4iˆ + q jˆ + 2kˆ

Equating coefficients of iˆ, jˆ, and kˆ on both the sides, 8 2q = 4 q=2 and 4p pq = 2 4p 2p = 2 p=1 Example 4: Prove that the four points whose position vectors are 3iˆ - 2jˆ + 4kˆ, 6iˆ + 3jˆ + kˆ, 5iˆ + 7jˆ + 3kˆ and 2iˆ + 2jˆ + 6kˆ are coplanar. Solution: Let A, B, C, D be the four points such that A = 3iˆ

2 jˆ + 4kˆ, B = 6iˆ + 3 jˆ + kˆ, C = 5iˆ + 7 jˆ + 3kˆ, D = 2iˆ + 2 jˆ + 6kˆ AB B A (6iˆ + 3 jˆ + kˆ) (3iˆ 2 jˆ + 4kˆ) = 3iˆ + 5 jˆ

3kˆ

AC

C A (5iˆ + 7 jˆ + 3kˆ) = 2iˆ + 9 jˆ kˆ

(3iˆ

2 jˆ + 4kˆ)

AD

D A (2iˆ + 2 jˆ + 6kˆ) = iˆ + 4 jˆ + 2kˆ

(3iˆ

2 jˆ + 4 kˆ)

Vector Calculus

AB ( AC

3 5 2 9 1 4

AD )

3 1 2

9.5

0

Hence, the four points are coplanar. Example 5: Prove that ( a b ) , ( b c ) , ( c a ) are non-coplanar if a , b and c are non-coplanar. Hence obtain the scalars l, m, n such that a

l ( b c ) m ( c a ) n ( a b) .

Solution: (i) If a, b, c are non-coplanar, then ⎡⎣ a, b, c ⎤⎦ ≠ 0 Consider,

⎡ a × b b × c c × a ⎤ = ( a × b ) ⋅ ⎡⎣(b × c ) × ( c × a )⎤⎦ ⎣ ⎦

Let

b c

(a

b)

(b c) ( c

a)

p

( p a) c ( p c) a ( a b ) { ( b c ) a } c { ( b c ) c} a (a

b)

p

(c

{

a)

(a

}

= ( a × b ) ⋅ ⎡⎣b c a ⎤⎦ c − 0

b)

⎡∵ ⎡b c c ⎤ = 0⎤ ⎦ ⎣ ⎣ ⎦

= ⎡⎣b c a ⎤⎦ ⎡⎣( a × b ) ⋅ c ⎤⎦ = ⎡⎣ a b c ⎤⎦ ⎡⎣ a b c ⎤⎦ 2

= ⎡⎣ a b c ⎤⎦ ≠ 0

⎡∵ ⎡ a b c ⎤ ≠ 0⎤ ⎦ ⎣ ⎣ ⎦

⎡⎣ a × b b × c c × a ⎤⎦ ≠ 0 Hence, (a b), (b c), (c a) are non-coplanar. (ii) a

l ( b c ) m ( c a ) n ( a b ) where l, m, n are scalars to be determined.

Taking scalar product with a on both the sides, a ⋅ a = la ⋅ (b × c ) + ma ⋅ ( c × a ) + na ⋅ ( a × b ) ⎡∵ ⎡ a c a ⎤ = 0 = ⎡ a a b ⎤ ⎤ ⎦ ⎣ ⎦⎦ ⎣ ⎣

= l ⎡⎣ a b c ⎤⎦ + 0 + 0 l=

a⋅a ⎡⎣ a b c ⎤⎦

Similarly, taking, dot product with b and c, m=

a⋅b ⎡⎣ a b c ⎤⎦

and n =

a⋅c ⎡⎣ a b c ⎤⎦

9.6

Engineering Mathematics

Example 6: If ⎡⎣a b c ⎤⎦

0, prove that a vector d can be expressed as

⎡ d b c ⎤⎦ a + ⎡⎣ d c a ⎤⎦ b + ⎡⎣ d a b ⎤⎦ c . d=⎣ ⎡⎣ a b c ⎤⎦ Solution: Since ⎡⎣ a b c ⎤⎦ ≠ 0, a, b, c are non-coplanar vectors, any vector d can be uniquely expressed as a linear combination of a, b, c. Let d = la + m b + nc where l, m, and n are scalars to be determined.

… (1)

Taking dot product with b c on both the sides, d ⋅ (b × c ) = la ⋅ (b × c ) + mb ⋅ (b × c ) + nc ⋅ (b × c ) ⎡∵ ⎡b b c ⎤ = 0 = ⎡c b c ⎤ ⎤ ⎣ ⎦⎦ ⎦ ⎣ ⎣

⎡⎣ d b c ⎤⎦ = l ⎡⎣ a b c ⎤⎦ + 0 + 0 ⎡ d b c ⎤⎦ l= ⎣ ⎡⎣ a b c ⎤⎦

Similarly, taking dot product with c a and a b on both the sides of Eq. (1), ⎡ d c a ⎤⎦ m= ⎣ , n= ⎡⎣ a b c ⎤⎦

⎡⎣ d a b ⎤⎦ ⎡⎣ a b c ⎤⎦

Substituting the values of l, m and n in Eq. (1), ⎡ d b c ⎤⎦ a + ⎡⎣ d c a ⎤⎦ b + ⎡⎣ d a b ⎤⎦ c d=⎣ ⎡⎣ a b c ⎤⎦ Example 7: If a + b + c = O, prove that a b Solution:

b c

c a.

a+b+c = O

… (1)

Taking cross product with b on both the sides,

(a + b + c)× b = O × b ( a × b ) + (b × b ) + ( c × b ) = O ( a × b ) + O = − ( c × b ) = (b × c ) a b

b c

… (2)

Vector Calculus

9.7

Similarly, taking cross-product with c on both the sides of Eq. (1),

b c

c a

b c

c a.

… (3)

From Eqs. (2) and (3), we get a b

1 ˆ Example 8: If aˆ ë (bˆ ë cˆ) = b , find angles which aˆ makes with bˆ and cˆ. 2 1 Solution: aˆ (bˆ cˆ) = bˆ 2 1 (aˆ cˆ) bˆ (aˆ bˆ) cˆ = bˆ 2 Equating the coefficients of bˆ and cˆ on both the sides, 1 and aˆ bˆ = 0 aˆ cˆ = 2 But, aˆ cˆ = |aˆ| |cˆ| cos q, where q is the angle between aˆ and cˆ. 1.1 cos q [aˆ and cˆ are unit vectors] 1 cos = 2 =

3 aˆ bˆ = 0, Thus, aˆ is perpendicular to bˆ . Hence, â makes an angle

3

with cˆ and an angle

2

with bˆ.

Example 9: A vector x satisfies the equation x b that x

c

( a c) b a b

Solution: x b

c b and a x

0, prove

.

c b

Taking cross-product with a on both the sides,

a × ( x × b) = a × (c × b)

(a ⋅ b ) x − (a ⋅ x ) b = (a ⋅ b ) c − (a ⋅ c ) b (a ⋅ b ) x = (a ⋅ b ) c − (a ⋅ c ) b (a ⋅ c ) b x= c− a⋅b

⎡⎣∵a ⋅ x = 0⎤⎦

9.8

Engineering Mathematics

p

q

r

Example 10: Prove that a p a q a r

p q r

(a

b) .

b p b q b r Solution: Let p = p1 iˆ + p2 jˆ + p3 kˆ, q = q1 iˆ + q2 jˆ + q3 kˆ, r = r1 iˆ + r2 jˆ + r3 kˆ, a = a1 iˆ + a2 jˆ + a3 kˆ, b = b iˆ + b jˆ + b kˆ 1

2

p1 ⎡⎣ p q r ⎤⎦ ( a × b ) = q1 r1

3

p2 q2 r2

p3 q3 r3

iˆ a1 b1

ˆj a2 b2

kˆ a3 b3

Interchanging rows by columns in second determinant, p p p iˆ a b 1

⎡⎣ p q r ⎤⎦ ( a × b ) = q1 r1

2

3

q2 r2

1

1

q3 ˆj a2 r3 kˆ a 3

b2

p1iˆ + p2 ˆj + p3 kˆ = q1iˆ + q2 ˆj + q3 kˆ r iˆ + r ˆj + r kˆ 1

2

3

p

p⋅a

p ⋅b

= q

q⋅a

q ⋅b

r

r ⋅a

r ⋅b

b3

p1a1 + p2 a2 + p3 q3

p1b1 + p2 b2 + p3b3

q1a1 + q2 a2 + q3 a3

q1b1 + q2 b2 + q3b3

r1a1 + r2 a2 + r3 a3

r1b1 + r2 b2 + r3b3

Interchanging rows by columns, p

q

r

⎡⎣ p q r ⎤⎦ ( a × b ) = a ⋅ p a ⋅ q a ⋅ r b⋅ p b⋅q b⋅r

9.5 PRODUCT OF FOUR VECTORS 9.5.1 Scalar Product of Four Vectors Scalar product of four vectors a, b, c, d is a dot product of vectors ( a b ) and ( c d ) .

(a

b) (c d )

a c

b c

a d

b d

Vector Calculus

9.9

This result is known as “Lagrange’s identity.” Proof: Let c d

m

( a × b ) ⋅ ( c × d ) = ( a × b ) ⋅ m = a ⋅ (b × m ) = a ⋅ ⎡⎣b × ( c × d ) ⎤⎦

⎡⎣Substituting m ⎤⎦

= a ⋅ ⎡⎣( b ⋅ d ) c − ( b ⋅ c ) d ⎤⎦

( a c )( b d ) ( a d )( b c ) (a

b) (c d )

a c

b c

a d

b d

9.5.2 Vector Product of Four Vectors Vector product of four vectors a, b, c, d is a cross product of vectors ( a b ) and ( c d ) . The vector product of four vectors a, b, c, d can be expressed in terms of vectors a and b as well as in terms of vectors c and d . (i)

( a × b ) × ( c × d ) = ⎡⎣ a c d ⎤⎦ b − ⎡⎣b c d ⎤⎦ a

(ii)

( a × b ) × ( c × d ) = ⎡⎣ a b d ⎤⎦ c − ⎡⎣ a b c ⎤⎦ d

Proof: (i) Let ( c d ) m

(a

b)

(c

d)

(a (a

b) m m)b

a (c

(b m) a d ) b b (c

d) a

( a × b ) × ( c × d ) = ⎡⎣ a c d ⎤⎦ b − ⎡⎣b c d ⎤⎦ a (ii) Let a b

n

(a

b)

(c

d) n

(c d ) (n d )c (n c)d (a b) d c (a

( a × b ) × ( c × d ) = ⎡⎣ a b d ⎤⎦ c − ⎡⎣ a b c ⎤⎦ d

b) c d

9.10

Engineering Mathematics

Example 1: By considering the product ( a b )

(c

d ) in two different ways,

show that ⎡⎣ b c d ⎤⎦ a + ⎡⎣ c a d ⎤⎦ b + ⎡⎣ a b d ⎤⎦ c = ⎡⎣ a b c ⎤⎦ d where a, b, c are non-coplanar vectors. Solution: We know that,

and

( a × b ) × ( c × d ) = ⎡⎣ a b d ⎤⎦ c − ⎡⎣ a b c ⎤⎦ d

... (1)

( a × b ) × ( c × d ) = ⎡⎣ a c d ⎤⎦ b − ⎡⎣b c d ⎤⎦ a

... (2)

Equating Eq. (1) and (2), ⎡⎣ a b d ⎤⎦ c − ⎡⎣ a b c ⎤⎦ d = ⎡⎣ a c d ⎤⎦ b − ⎡⎣b c d ⎤⎦ a ⎡⎣b c d ⎤⎦ a − ⎡⎣ a c d ⎤⎦ b + ⎡⎣ a b d ⎤⎦ c = ⎡⎣ a b c ⎤⎦ d ⎡∵ ⎡ a c d ⎤ = − ⎡c a d ⎤ ⎤ ⎣ ⎦⎦ ⎦ ⎣ ⎣

⎡⎣b c d ⎤⎦ a + ⎡⎣c a d ⎤⎦ b + ⎡⎣ a b d ⎤⎦ c = ⎡⎣ a b c ⎤⎦ d

Example 2: Prove that ( b c )

(a

d)

(c

a)

(b

d)

(a

b)

(c

d)

– 2 ⎡⎣ a b c ⎤⎦ d and hence, show that vector on L.H.S. is parallel to vector d . Solution:

(b

c)

(a

d)

(c

a)

(b

d)

(a

b)

(c

d)

= ⎡⎣b c d ⎤⎦ a − ⎡⎣b c a ⎤⎦ d + ⎡⎣c a d ⎤⎦ b − ⎡⎣c a b ⎤⎦ d + ⎡⎣ a c d ⎤⎦ b − ⎡⎣b c d ⎤⎦ a = − ⎡⎣ a b c ⎤⎦ d − ⎡⎣ a c d ⎤⎦ b − ⎡⎣ a b c ⎤⎦ d + ⎡⎣ a c d ⎤⎦ b = −2 ⎡⎣ a b c ⎤⎦ d

⎡⎣( b × c ) × ( a × d ) + ( c × a ) × ( b × d ) + ( a × b ) × ( c × d )⎤⎦ × d = −2 ⎡ a b c ⎤ d × d = 0. ⎣ ⎦ Hence, the given vector is parallel to vector d . Example 3: Prove that ( b c ) ( a d ) Solution:

(b c ) ( a =

d)

b a

c a

b d

c d

(c +

a ) (b d )

c b

a b

c d

a d

+

(c

a) ( b d )

(a

b) (c d )

a c

b c

a d

b d

(a

b) ( c d )

( b a )( c d ) ( c a )( b d ) ( b c )( a d ) ( a b )( c d ) ( a c )( b d ) ( b c )( a d ) = 0.

0.

Vector Calculus

9.11

Example 4: Prove that a × ⎡⎣ b × ( c × d ) ⎤⎦ = ( b ⋅ c ) ( d × a ) - ( b ⋅ d ) ( c × a ) . Solution: a × ⎡⎣b × ( c × d ) ⎤⎦ = a × ⎡⎣( b ⋅ d ) c − ( b ⋅ c ) d ⎤⎦

= (b ⋅ d ) ( a × c ) − (b ⋅ c ) ( a × d )

= ( b ⋅ d ) ⎡⎣ − ( c × a ) ⎤⎦ − ( b ⋅ c ) ⎡⎣ − ( d × a ) ⎤⎦ a × ⎡⎣b × ( c × d ) ⎤⎦ = ( b ⋅ c ) ( d × a ) − ( b ⋅ d ) ( c × a )

{

}

Example 5: Prove that d ⋅ ⎡⎣ a × b × ( c × d ) ⎤⎦ = ( b ⋅ d ) ⎡⎣ a c d ⎤⎦ . Solution: As proved in Example 4. a × {b × ( c × d )} = ( b ⋅ c ) ( d × a ) − ( b ⋅ d ) ( c × a ) d ⋅ ⎡⎣ a × {b × ( c × d )}⎤⎦ = d ⋅ ⎡⎣( b ⋅ c ) ( d × a ) − ( b ⋅ d ) ( c × a ) ⎤⎦

= ( b ⋅ c ) {d ⋅ ( d × a )} − ( b ⋅ d ) {d ⋅ ( c × a )} = ( b ⋅ c ) (0) − ( b ⋅ d ) ⎡⎣ d c a ⎤⎦

{

}

= − ( b ⋅ d ) − ⎡⎣c d a ⎤⎦

⎡⎣ Interchanging c and d ⎤⎦

= ( b ⋅ d ) ⎡⎣ a c d ⎤⎦

Exercise 9.1 1. If a = iˆ 2 jˆ 3 kˆ, b = 2iˆ + jˆ kˆ, c = iˆ + 8 jˆ 2 kˆ, find a ( b c ) . [Ans. : 21 iˆ 2. Prove that iˆ ( a ( a jˆ) + kˆ ( a

33 jˆ + 15 kˆ]

iˆ) + jˆ kˆ) = 2 a.

3. Prove that ⎡⎣ a + b b + c c + a ⎤⎦ = 2 ⎡⎣ a b c ⎤⎦ and hence, prove that a, b, c are coplanar if and only if

( a + b) , ( b + c) , ( c + a ) 4. Prove that

are coplanar.

a

(b c )

b

(c

a) c

(a

b)

0.

5. Prove that 2

⎡⎣b × c c × a a × b ⎤⎦ = ⎡⎣ a b c ⎤⎦ .

(b c) ( a prove that ( a c )

6. If a

b ) c, then

b

0.

7. Find the scalars p and q such that ( a b) c a ( b c ) , where a = p iˆ + jˆ + 2 kˆ, b = iˆ

jˆ,

c = 4 iˆ + 2 jˆ + q kˆ. [Ans. : p = 2, q = 4]

9.12

Engineering Mathematics

8. If the vector x and a scalar l satisfy x = l a + b and a · x = 2, where a = iˆ + 2 jˆ kˆ and b = 2 iˆ jˆ + q kˆ, find x and l. 1 Ans. : x = iˆ + 7 ˆj + 3kˆ, = 6

the equation a

9. If the vector x and a scalar l satisfy the equation a x =l a + b

11. If a = 2 iˆ + 3 jˆ kˆ, b = iˆ + 2 jˆ 4 kˆ, c = iˆ + jˆ + kˆ, find ( a b ) · ( a c ) . 12. Prove that 2a2 = | a iˆ|2 + | a where a = | a |. 13. Prove that ⎡⎣( a × b ) × ( a × c ) ⎤⎦ · d

and a · x = 1, find the values of l

= ( a ⋅ d ) ⎡⎣ a b c ⎤⎦ .

and x in terms of a and b. Also, determine them if a = iˆ b = 2 iˆ + jˆ 2 kˆ. Ans. :

=

2 jˆ and

14. Prove that (i)

(a b) , x = a (a b) a2

0, x

a2

kˆ|2,

jˆ|2 + | a

(ii)

1 ˆ (3i 4 ˆj 3kˆ) 5

(a (c (a (c

d ) · (b

c ) + (b d ) ·

a) + (c d ) · (a b) = 0 d) a) +

(b (c

c ) + (b d ) d)

(a

b) =

2 ( a b) ( b c) ( c a ) 10. If a , b , c are three vectors defined by q ×r r×p p×q a= ,b= ,c= , 15. If a, b, c, d are four vectors such that [p q r] [p q r] [p q r] x a + y b + z c + t d = 0, then prove that then prove that x y z −t = = = . ( p a ) + ( q b ) + ( r c ) = 0. ⎡⎣b c d ⎤⎦ ⎡⎣c a d ⎤⎦ ⎡⎣ a b d ⎤⎦ ⎡⎣ a b c ⎤⎦

9.6 VECTOR FUNCTION OF A SINGLE SCALAR VARIABLE If, in some interval (a, b) or [a, b], for every value of a scalar variable t, there corresponds a value of r , then r is called a vector function of the scalar variable ‘t’ and is denoted by r = f (t ).

9.6.1 Decomposition of a Vector Function If iˆ, jˆ, kˆ be three unit vectors along the three mutually perpendicular fixed directions (x, y, and z axes), then r = f (t ) can be decomposed as r = f (t ) = f (t) iˆ + f (t) jˆ + f (t) kˆ 1

2

3

where, f1(t), f2(t) and f3(t) are scalar functions of t. This relation can also be denoted by f = ( f1, f2, f3 )

| f (t ) | = [ f1 (t )]2 + [ f 2 (t )]2 + [ f3 (t )]2

Vector Calculus

9.13

9.6.2 Differentiation of a Vector Function Derivative of a vector function f (t ) with respect to a scalar variable t is defined as df f (t = lim t 0 dt

t) t

f (t )

where, d t is the change in t. If f (t)= f1 (t) iˆ + f2 (t) jˆ + f3 (t) kˆ where f1 (t), f2 (t) and f3 (t) are the components of f (t ) in the direction of x, y, z-axes, then derivative in the component form is d f df1 ˆ df 2 ˆ df3 ˆ i+ j+ k. = dt dt dt dt

9.7 VELOCITY AND ACCELERATION Let r (t ) = x (t) iˆ + y (t) jˆ + z (t) kˆ be the position vector of a particle moving along a curve, at time t. Velocity is the rate of change of displacement with respect to time. Velocity, v = d r = dx iˆ + dy ˆj + dz kˆ dt dt dt dt . dr is also denoted by r . dt Acceleration is the rate of change of velocity with respect to time. dv d 2 r = dt dt 2 d2 y d2 z d2 x = 2 iˆ + 2 jˆ + 2 kˆ . dt dt dt

Acceleration, a =

9.8 STANDARD RESULTS Most of the basic rules of differentiation that are true for a scalar function of scalar variable hold good for vector function of a scalar variable, provided the order of factors in vector products is maintained. Let a, b, c are differentiable vector functions of a scalar variable t. 1.

dk = 0, k is a constant vector dt

2.

d ( a ± b ) = d a ± db dt dt dt

3. d dt

( a) =

da d , f is a scalar function of t. +a dt dt

9.14

Engineering Mathematics

4.

d ( a b) dt

5.

d ( a b) dt

6.

⎤ ⎡ db ⎤ ⎡ ⎡ da d ⎡ dc ⎤ b c⎥ + ⎢a c⎥ + ⎢a b ⎥ ⎣ a b c ⎤⎦ = ⎢ dt dt ⎦ ⎣ dt ⎦ ⎣ dt ⎦ ⎣

7.

d a dt

da db b a dt dt da dt

(b c)

db dt

b a

da dt

(b c)

a

db dt

c

a

dc dt

b

9.9 TANGENT VECTOR TO A CURVE AT A POINT Let P(t) and Q (t + d t) be the two points on the curve r = f (t ) . The tangent at P is the limiting position of the chord PQ when Q Let r + d r = f (t + d t) PQ

P.

Q

OQ OP

d r = (r + d r ) − r

Since d t is a scalar, vector parallel to PQ. As d t 0, Q of chord PQ is

t) t

dr

f (t )

P r

r is t

t

0

x

O

P, limiting position lim

Hence,

r+

r f (t = t

f (t)

dr

f (t + d t)

Fig. 9.2

r dr = t dt

dr is a vector parallel to the tangent at P. dt

If s is the arc length measured from a fixed point, and d s is the arc length PQ, then lim

s →0

chord PQ r = lim Q → P arc PQ s dr =1 ds

Hence, d r is a unit vector in the direction of the tangent to the curve at P and is called ds unit tangent vector. It is denoted by ˆt.

Vector Calculus

d Example 1: Write down the formula for dt A = 5t2 iˆ + t jˆ – t3 kˆ, and B = sint iˆ – cost jˆ. Solution: Given,

( A ë B)

9.15

and verify the same for

d ( A B) = d A B + A dB dt dt dt A = 5t2 iˆ + t jˆ t3 kˆ, B = sin t iˆ iˆ

A B

cos t jˆ ˆj kˆ

5t 2 sin t

t3 0

t cos t

iˆ (0 t3 cos t) jˆ (0 + t3 sin t) + kˆ ( 5t2 cos t t sin t) ( t3 cos t) iˆ (t3 sin t) jˆ (5t2 cos t + t sin t) kˆ

(

)

d A B = ( 3t2 cos t + t3 sin t) iˆ (3t2 sin t + t3 cos t) jˆ dt (10t cos t 5t2 sin t + sin t + t cos t) kˆ dA = 10t iˆ + jˆ dt

Now,

… (1)

3t2 kˆ,

dB = cos t iˆ + sin t jˆ dt ˆj iˆ kˆ dA dt

B

1 cos t

iˆ(0

3t2 cos t)

iˆ dB A dt

jˆ(0 + 3t2 sin t) + kˆ ( 10 t cos t

5t 2 t cos t sin t

t3 0 jˆ(0 + t 3 cos t) + kˆ (5t2 sin t

t cos t)

dB = ( 3t2 cos t + t3 sin t) iˆ (3t2 sin t + t3 cos t) jˆ dt (10t cos t + sin t 5t2 sin t + t cos t) kˆ Comparing Eqs. (1) and (2), B

sin t)



ˆj

iˆ(0 + t 3 sin t) dA dt

3t 2 0

10 t sin t

A

(

d A B dt

)

dA dt

B

A

dB dt

… (2)

9.16

Engineering Mathematics

Example 2: If d dt

(u ë v) =

du dv w v = w ë u and = ë , then prove that dt dt

(

)

w ë uëv . d ( u v ) = du v + u dt dt du dv = w u, = w v dt dt

Solution: We know that, But

d (u v) dt

(w

u) v u

dv dt

( w v)

( v w) u ( v u ) w ( u v ) w ( u w) v ( v w) u ( u w) v ( w v ) u ( w u ) v w (u v) 1

Example 3: If r = t3 iˆ + 2t 3

5t

d r ˆ. jˆ, then show that r ë =k dt

2

1

Solution: r = t3 iˆ + 2t 3

jˆ 5t 2 2 ⎞ ⎛ 2 dr = 3t2 iˆ + ⎜ 6t + 3 ⎟ jˆ ⎝ 5 t ⎠ dt

r

dr dt

t 3iˆ

1

2t 3

5t

3t 5 ( iˆ iˆ ) 2t 3 0

6t 5

2

6t 5 1

5t

2

6t 2

2 ˆ k 5

ˆj

3t 2 iˆ

6t 2

2 5

( iˆ ˆj )

6t 5

2 5t 3 6t 5

2 5t 3 3 5

ˆj

( ˆj iˆ)

( ˆj ˆj ) 3 ( kˆ) 0 5

[∵ iˆ × iˆ = 0 =

= kˆ Example 4: If a and b are constant vectors and v is constant and r = a sin v t + b cos v t, prove that r ë Solution: r = a sin w t + b cos w t dr = a w cos w t + b w ( sin w t) dt

dr + v ( a ë b ) = 0. dt

jˆ × jˆ

]

Vector Calculus

9.17

dr = ( a sin t + b cos t ) × ( a cos t − b sin t ) dt



= ( a × a ) sin t cos t − ( a × b ) sin 2 t + ( b × a ) cos 2 t − ( b × b ) cos t sin t = 0 − ( a × b ) sin 2 t − ( a × b ) cos 2 t − 0 ⎡⎣∵ a × a = 0 = b × b ⎤⎦

= − ( a × b ) (sin 2 t + cos 2 t ) = − ( a × b ) dr + ( a b ) w = 0. dt

Hence, r

Example 5: If r = a sinh t + b cosh t, where a and b are constant, then show that d2 r

(i)

dt 2

=r

(ii)

dr dt

d2 r dt 2

constant.

Solution: r = a sinh t + b cosh t, dr = a cosh t + b sinh t dt d 2 r = a sinh t + b cosh t dt 2

(i)

d2 r

Hence,

(ii)

dt 2

[∵ a and b are constant] r

=r

dr d2 r ( × = a cosh t + b sinh t ) × ( a sinh t + b cosh t ) dt dt 2

= ( a × a ) cosh t sinh t + ( a × b ) cosh 2 t + ( b × a ) sinh 2 t + ( b × b ) sinh t cosh t = 0 + ( a × b ) cosh 2 t − ( a × b ) sinh 2 t + 0

= ( a × b ) ( cosh 2 t − sinh 2 t ) = (a × b) Hence,

dr dt

d2 r dt 2

[∵ cosh 2 t − sinh 2 t = 1]

= constant.

ct Example 6: If r = a (sin v t) iˆ + b (sin v t) jˆ + 2 (sin v t) kˆ, prove that d2 r dt

2

+ v2 r =

2c

(cos v t) kˆ.

9.18

Engineering Mathematics

ct Solution: r = a (sin w t) iˆ + b (sin w t) jˆ + 2 (sin w t) kˆ dr c = aw (cos w t) iˆ + bw (cos w t) jˆ + 2 (sin w t + t w cos w t) kˆ dt d2 r dt

c = aw ( w sin w t ) iˆ + bw ( w sin w t) jˆ + 2 [w (cos w t) +

2

w (cos w t) + tw ( w sin w t)] kˆ aw2 (sin w t ) iˆ

bw2 (sin w t) jˆ +

c 2

(2w cos w t

tw2 sin w t) kˆ

ct 2c w2 [a (sin w t) iˆ + b (sin w t) jˆ + 2 (sin w t) kˆ] + (cos w t) kˆ w2 r +

2c

(cos w t) kˆ

Example 7: If r = (a cos t) iˆ + (a sin t) jˆ + (at tan a ) kˆ, prove that (i)

dr dt

d2 r dt 2

⎡ dr d2 r d3 r ⎤ = a 3 tan α . (ii) ⎢ 2 3 ⎥ ⎢⎣ dt dt dt ⎥⎦

a 2 sec

Solution: r = (a cos t) iˆ + (a sin t) jˆ + (at tan a) kˆ,

(i)

dr dt

d2 r

dr dt (ii)

dr = ( a sin t ) iˆ + (a cos t) jˆ + (a tan a) kˆ dt d2 r = ( a cos t) iˆ + ( a sin t) jˆ + 0 kˆ dt 2 d3 r = (a sin t) iˆ + ( a cos t) jˆ + 0 · kˆ dt 3 iˆ jˆ kˆ d2 r a sin t a cos t a tan dt 2 a cos t a sin t 0 = iˆ (0 + a2 sin t tan a) jˆ (0 + a2 cos t tan a) + kˆ (a2 sin2 t + a2 cos2 t) = a2 (sin t tan a) iˆ a2 (cos t tan a) jˆ + a2 kˆ a 4 sin 2 t tan 2

dt 2 dr dt

d2 r

d3 r

2

dt 3

dt

a 4 cos 2 t tan 2

2 2 a 4 = a tan

2 + 1 = a sec a

=[a2(sin t tan a) iˆ – a2 (cos t tan a) jˆ + a2 kˆ]. [(a sin t) iˆ + ( a cos t) jˆ + 0 kˆ] = a3 sin2 t tan a + a3 cos2 t tan a [∵ iˆ · iˆ = jˆ · jˆ = kˆ · kˆ = 1 and iˆ · jˆ = jˆ · kˆ = kˆ · iˆ = 0] 3 a tan a

Vector Calculus

9.19

2 3 ⎤ ⎡ Hence, ⎢ d r d r d r ⎥ = a 3 tan . 2 ⎣ dt dt dt 3 ⎦

Example 8: If A = (sin t) iˆ + (cos t) jˆ + t kˆ, B = (cos t) iˆ – (sin t) jˆ – 3 kˆ, d C = 2 iˆ + 3 jˆ – kˆ, find A dt iˆ

(B

Solution:

(B

A

C)

cos t 2 iˆ sin t

C)

(B

C ) at t = 0.

ˆj sin t 3

kˆ 3 = iˆ (sin t + 9) jˆ( cost + 6) + kˆ (3 cos t + 2 sin t) 1 kˆ t

ˆj cos t

sin t 9 cos t 6 3cos t 2sin t

= iˆ (3 cos2 t + 2 sin t cos t t cos t + 6t) jˆ (3 cos t sin t + 2 sin2 t t sin t 9t) + kˆ (sin t cos t 6 sin t cos t sin t 9 cos t) ⎛3 ⎞ 2 = (3 cos2 t + sin 2t t cos t + 6t)iˆ ⎜⎝ sin 2 t + 2 sin t − t sin t − 9t ⎟⎠ jˆ 2 + ( 6 sin t 9 cos t) kˆ d A dt

(B

C ) = [6 cos t ( sin t) + 2 cos 2t (3 cos 2t + 4 sin t cos t

cos t + t sin t + 6] iˆ sin t t cos t 9) jˆ (6 cos t

9 sin t) kˆ

Putting t = 0, d dt

A

(B

C ) = 7 iˆ + 6 jˆ

6 kˆ.

Example 9: Find the derivative of r

dr dt

d2 r dt 2

with respect to ‘t’.

Solution: ⎛ d r d3 r ⎞ ⎛ d2 r d2 r ⎞ d ⎡ ⎛ dr d2 r ⎞ ⎤ dr ⎛ dr d2 r ⎞ ×⎜ × 2 ⎟ +r×⎜ 2 × 2 ⎟ +r×⎜ × 3 ⎟ ⎢r × ⎜ × 2 ⎟ ⎥ = dt ⎢⎣ ⎝ dt dt ⎠ ⎥⎦ dt ⎝ dt dt ⎠ dt ⎠ ⎝ dt dt ⎠ ⎝ dt ⎤ ⎡ d2 r d2 r ⎛ d r d3 r ⎞ dr ⎛ dr d2 r ⎞ = ×⎜ × 2 ⎟ +r×⎜ × 3 ⎟ ⎢∵ 2 × 2 = 0⎥ dt ⎝ dt dt ⎠ dt ⎝ dt dt ⎠ ⎥⎦ ⎢⎣ dt Example 10: Find

d ⎛ r×a ⎞ ⎜ ⎟ , where r is a vector function of scalar variable dt ⎜⎝ r ⋅ a ⎟⎠

t and r is a constant vector.

d ⎡d( ⎤ r × a )⎥ ( r ⋅ a ) − ( r × a ) ( r ⋅ a ) d ⎛ r × a ⎞ ⎢⎣ dt d t ⎦ Solution: = ⎟ ⎜ 2 dt ⎝ r ⋅ a ⎠ (r ⋅ a)

9.20

Engineering Mathematics

dr dt

=

da dt

a r

( r a) ( r

a)

dr da a r dt dt

( r a )2

da = 0, as a is constant. dt ⎛ dr ⎞ ⎛ dr ⎞ × a⎟ ( r ⋅ a ) − ( r × a ) ⎜ ⋅ a⎟ ⎜ ⎠ ⎝ dt ⎠ d ⎛ r × a ⎞ ⎝ dt = . Hence, ⎟ ⎜ 2 dt ⎝ r ⋅ a ⎠ (r ⋅ a) But,

df if f = r2 r + ( a · r ) b where r is a function of t and dt a , b are constant vectors.

Example 11: Find

Solution: f = r2 r + ( a r ) b df dt

d( 2 ) r r dt dr 2 dt 2r

Hence,

dr dt

d (a r)b dt

(r) (r)

r2

dr dt

r2

da dr r a b dt dt

dr dt

a

( a r ) db dt

dr b dt



da dt

db dt

⎛ dr ⎞ df dr dr = 2rr + r2 + b⎜a⋅ ⎟ . dt dt dt ⎝ dt ⎠

Example 12: If f (t) is a unit vector, prove that f ( t ) Solution: Since f is a unit vector, f · f =1 Differentiating w.r.t. t, df df ⋅f+f⋅ =0 dt dt df 2f ⋅ =0 dt df f⋅ =0 dt This shows that f and d f are perpendicular to each other. dt

d f (t ) dt

d f (t ) . dt

0

Vector Calculus

Now,

f

df dt

f

9.21

df sin n^ dt

df and nˆ is the unit vector perpendicular to the where, q is the angle between f and dt df plane of f and . dt df are perpendicular, Since f and dt f d f = f d f sin nˆ dt 2 dt

Hence,

f

df dt

df nˆ dt

f

df dt

df dt

2

ÈÎ∵ f is a unit vector ˘˚ [∵ | nˆ | = 1]

Example 13: Find the magnitude of the velocity and acceleration of a particle which moves along the curve x = 2 sin 3t, y = 2 cos 3t, z = 8t at any time t > 0. Find unit tangent vector to the curve. Solution: The position vector r of the particle is r = xiˆ + yjˆ + zkˆ = (2 sin 3t)iˆ + (2 cos 3t)jˆ + (8t)kˆ Velocity,

v=

dr = (6 cos 3t) iˆ + ( 6 sin 3t) jˆ + 8kˆ dt

v = 36 cos 2 3t + 36sin 2 3t + 64 = 36 + 64 = 10 Acceleration,

a=

d2 r = ( 18 sin 3t) iˆ + ( 18 cos 3t) jˆ + (0)kˆ dt 2

a = (18) 2 sin 2 3t + (18) 2 cos 2 3t = 18. dr 1 Unit tangent vector = dt = [(6 cos 3t ) iˆ – (6 sin 3t ) jˆ +8kˆ ]. 10 dr dt Example 14: A particle moves along a plane curve such that its linear velocity is perpendicular to the radius vector. Show that the path of the particle is a circle. Solution: Let position vector r of the particle is r = x iˆ + y jˆ

9.22

Engineering Mathematics

dr dt To find path of the particle, we have to develop a relation in x and y. Velocity is perpendicular to the radius vector. v=

Velocity,

dr dt dr 2r dt dr dr r r dt dt d r r dt r r r

( )

0 0 0 0 c 2 , constant

x2 + y2 = c2 which is a circle with center at the origin and radius c. Example 15: Find the magnitude of tangential components of acceleration at any time t of a particle whose position at any time t is given by x = cos t + t sin t, y = sin t - t cos t. Solution: Position vector r of the particle is r = (cos t + t sin t) iˆ + (sin t

Velocity,

t cos t) jˆ

dr dt = ( sin t + sin t + t cos t) iˆ + (cos t = (t cos t) iˆ + (t sin t ) jˆ

v=

d2 r = (cos t dt 2 Unit vector in the direction of the tangent is a=

Acceleration,

cos t + t sin t) jˆ

t sin t) iˆ + (sin t + t cos t) jˆ

dr (t cos t ) iˆ + (t sin t ) ˆj tˆ = dt = = (cos t) iˆ + (sin t) jˆ t 2 cos 2 t + t 2 sin 2 t dr dt Magnitude of tangential component of acceleration

= a ⋅ tˆ [(cos t t sin t) iˆ + (sin t + t cos t) jˆ] · [(cos t) iˆ + (sin t) jˆ] cos2 t t sin t cos t + sin2 t + t cos t sin t 1

Vector Calculus

9.23

Example 16: Show that a particle whose position vector r at any time t is given by r = (a cos nt) iˆ + (b sin nt) jˆ moves in an ellipse whose center is at the origin and that its acceleration varies directly as its distance from the center and is directed towards it. Solution: r = (a cos nt) iˆ + (b sin nt) jˆ x = a cos nt, y = b sin nt x2 y 2 + = cos 2 nt + sin 2 nt = 1 a 2 b2 x2 y 2 + =1 a 2 b2 which is an ellipse with center at origin. dr = ( a n sin nt) iˆ + (b n cos nt) jˆ dt d2 r a = 2 = ( a n2 cos nt) iˆ + ( b n2 sin nt) jˆ dt n2 [(a cos nt) iˆ + (b sin nt) jˆ] = n2 r

Now, Acceleration,

This shows that acceleration of the particle varies directly as its distance r from the origin (center of the ellipse) and negative sign shows that acceleration is directed towards the origin. Exercise 9.2 1. If A = 5t2 iˆ + t jˆ t3 kˆ and B = sin t iˆ cos t jˆ, find the value of (i)

d( A B) dt

(ii)

d( A B) dt

Ans. : (i) (5t 2 1) cos t 11t sin t , (ii) (t 3 sin t 3t 2 cos t ) iˆ ^ (t 3 cos t 3t 2 sin t ) j ^

(5t 2 sin t sin t 11cos t ) k 2. If A = 4t3 iˆ + t2 jˆ 6t2 kˆ and B = (sin t) iˆ (cos t) jˆ, verify the d( formula of A B) . dt

3. If r = A e nt + B e nt, show that d 2 r n2 r = 0. dt 2 1 4. If r = t3 iˆ + 2t 3 jˆ, show 5t 2 dr ˆ that r = k. dt 5. Prove that d ⎡ dr d 2 r ⎤ ⎡ dr d3 r ⎤ ⎥. ⎥ = ⎢r ⎢r dt ⎣ dt dt 2 ⎦ ⎣ dt dt 3 ⎦ 6. Prove that d 2 ⎡ dr d 2 r ⎤ ⎡ d 2 r d3 r ⎤ ⎥ ⎥ = ⎢r ⎢r dt 2 ⎣ dt dt 2 ⎦ ⎣ dt 2 dt 3 ⎦ ⎡ dr d4 r ⎤ . + ⎢r 4 ⎥ ⎣ dt d t ⎦

9.24

Engineering Mathematics the velocity and acceleration at t = 1 in the direction of iˆ 3 jˆ + 2 kˆ [Hint: unit vector in the direction of iˆ 3jˆ + 2kˆ is iˆ 3 ˆj 2kˆ iˆ 3 ˆj 2kˆ = nˆ = , 1+ 9 + 4 14

7. Find the derivatives of the following: (i) r3 r + a

r rb dr (ii) 2 + r a r dt

where, r = r , a and b are constant vectors. Ans. : dr dr r r3 (i) 3r dt dt 2

(ii)

1 dr r 2 dt br

( a r )2

2

Find v and a at t = 1, velocity in the given direction = v nˆ and acceleration in the given direction = a nˆ ]

d2 r a dt 2

r dr r 3 dt

b

dr ( a r ) dt

dr a dt

8. A particle moves along the curve r = e t (cos t) iˆ + e t (sin t) jˆ + e t kˆ. Find the magnitude of velocity and acceleration at time t. Ans. : v = 3e t , a = 5e t 9. A particle moves on the curve x = 2t2, y = t2 4t, z = 3t 5. Find

Ans. : v

8 2 7

,a

2 7

10. A particle is moving along the curve r = a t 2 + b t + c, where a, b, c are constant vectors. Show that acceleration is constant. 11. A particle moves such that its position vector is given by r = (cos w t) iˆ + (sin w t) jˆ. Show that velocity v is perpendicular to r. dr r 0 Hint: Prove that dt

9.10 SCALAR AND VECTOR POINT FUNCTION 9.10.1 Field If a function is defined in any region of space, for every point of the region, then this region is known as field.

9.10.2 Scalar Point Function A function f (x, y, z) is called scalar point function defined in the region R, if it associates a scalar quantity with every point in the region R of space. The temperature distribution in a heated body, density of a body and potential due to gravity are the examples of a scalar point function.

9.10.3 Vector Point Function A function F (x, y, z) is called vector point function defined in the region R, if it associates a vector quantity with every point in the region R of space. The velocity of a moving fluid, gravitational force are the examples of vector point function.

Vector Calculus

9.25

9.10.4 Vector Differential Operator Del ( ) The vector differential operator Del (or nabla) is denoted by iˆ

ˆj

x



y

and is defined as

z

9.11 GRADIENT The gradient of a scalar point function f is written as iˆ

grad

x

ˆj

or grad f and is defined as kˆ

y

z

grad f is a vector quantity. f (x, y, z) is a function of three independent variables and its total differential df is given as ∂ ∂ ∂ d = dx + dy + dz ∂x ∂y ∂z ⎛ ∂ ∂ ∂ ⎞ = ⎜ iˆ + ˆj + kˆ ⎟ ⋅ ( iˆdx + ˆjdy + kˆdz ) ∂ ∂ ∂z ⎠ x y ⎝ dr

...(1) ∵ r

xiˆ

yjˆ zkˆ

dr

iˆdx

ˆjdy

ˆjdz

d r cos where, q is the angle between the vectors direction, then q = 0, df =

and d r. If dr and

are in the same

dr

cos q = 1 is the maximum value of cos q. Hence, df is maximum at q = 0.

9.11.1 Normal Let f (x, y, z) = c represents a family of surfaces for different values of the constant c. Such a surface for which the value of the function is constant is called level surface. Now differentiating f, we get df = 0 But from Eq. (1) of 9.11, d dr dr

0

Hence, and dr are perpendicular to each other. Since vector dr is in the direction of the tangent to the given surface, vector is perpendicular to the tangent to the surface and hence is in the direction of normal to the surface. Thus geometrically represents a vector normal to the surface f (x, y, z) = c.

9.26

Engineering Mathematics

9.11.2 Directional Derivative (i) Let f (x, y, z) be a scalar point function. Then

, , are the directional x y z derivative of f in the direction of the coordinate axes.

f f f are the directional Similarly, if f (x, y, z) be a vector point function, then , , x y z derivative of f in the direction of the coordinate axes. (ii) The directional derivative of a scalar point function f (x, y, z) in the direction of a line whose direction cosines are l, m, n,

=l

x

+m

y

+n

z

(iii) The directional derivative of scalar point function f (x, y, z) in the direction of in the direction of a . If aˆ is the unit vector vector a , is the component of in the direction of a , then directional derivatives of f in the direction of a a

aˆ a

9.11.3 Maximum Directional Derivative Since the component of a vector is maximum in its own direction, [∵ cos q is maximum . Since when q = 0], the directional derivative is maximum in the direction of is normal to the surface, directional derivative is maximum in the direction of normal. Maximum directional derivative cos cos 0 Standard Results: (i) ( ) (ii)

(

(iii)

f (u )

)

(

) (

f (u ) iˆ x

)

ˆj f (u ) kˆ f (u ) y z

f (u ) u.

at (1, - 2, 1), if f = 3x2y - y3 z2.

Example 1: Find

ˆj kˆ x y z ˆ ˆ = i (6xy 0) + j (3x2 3y2z2) + kˆ (0 At x = 1, y = 2, z = 1 = iˆ ( 12) + jˆ (3 12) + kˆ (16) at (1, 2, 1) = 12 iˆ 9 jˆ + 16 kˆ Solution:



2y3z)

Vector Calculus

Example 2: Evaluate

9.27

2

e r , where r2 = x2 + y2 + z2.

Solution: r2 = x2 + y2 + z2 Differentiating partially w.r.t. x, y and z, r r x = 2 x, = 2r x x r r r y = 2 y, = 2r y y r r r z = 2 z, = 2r z z r 2

e

er iˆ x

r2

er iˆ r

2

2

2

r ˆj e y

r x

er kˆ z

r ˆj e r

2 x iˆ(e r 2r ) r

2

ˆj (e r 2

2

er r kˆ r z y ˆ r2 z k (e 2 r ) 2r ) r r r y

(

2 2e r xiˆ

yjˆ zkˆ

)

Example 3: If f (x, y) = log x 2 + y 2 and r = xiˆ + yjˆ + zkˆ, prove that

( kˆ r ) kˆ ( kˆ r ) kˆ r ( kˆ r ) kˆ r

grad f = r

.

Solution: f (x, y) = log x 2 + y 2 = iˆ

f

=

=

1 log( x 2 + y 2 ) 2

1 log( x 2 x 2

1 iˆ 2x 2 x2 + y 2 xiˆ + yjˆ

ˆj

1 log( x 2 y 2

y2 )



1 log( x 2 z 2

y2 )

ˆj 1 2y 0 2 x2 + y 2

x2 + y 2

( xiˆ

xiˆ + yjˆ yjˆ ) ( xiˆ

Now, r = xiˆ + yjˆ + zkˆ kˆ· r = z r = xiˆ + yjˆ + (kˆ · r ) kˆ r

y2 )

(kˆ · r )kˆ = xiˆ + yjˆ

yjˆ ) [∵ iˆ · kˆ = jˆ · kˆ = 0, kˆ · kˆ = 1]

9.28

Engineering Mathematics

Substituting x iˆ + y jˆ in

f,

( kˆ r ) kˆ ( kˆ r ) kˆ r ( kˆ r ) kˆ r

f r Example 4: Prove that

rn

nr n 2 r , r = xiˆ + yjˆ + zkˆ, r = r .

Solution: r = x iˆ + y jˆ + zkˆ, r2 = x2 + y2 + z2

∂r x ∂r y ∂r z = , = , = ∂x r ∂y r ∂z r ∂r n ∂r n ∂r n ∂r n ∂r ˆ ∂r n ∂r ˆ ∂r n ∂r ⋅ +j ⋅ +k +j ⋅ +k =i ∂r ∂y ∂x ∂r ∂z ∂y ∂z ∂r ∂x z x y = iˆ nr n −1 ⋅ + ˆj nr n −1 ⋅ + kˆ nr n−1 ⋅ r r r = nrn 2 (x iˆ + yjˆ + zkˆ ) = nrn 2 r .

∇r n = i

⎛ a ⋅ r ⎞ a n(a ⋅ r ) Example 5: Show that ∇ ⎜ n ⎟ = n − n + 2 ( r ) , where r = xiˆ + yjˆ + zkˆ, ⎜ r ⎟ r r ⎝ ⎠ r = r , a is constant vector. Solution: Let a = a1iˆ + a2 ˆj + a3 kˆ, and

a⋅r = rn

r = xiˆ + yjˆ + zkˆ ⎛ a ⋅ r ⎞ ⎡ ( a iˆ + a2 ˆj + a3 kˆ ) ⋅ ( xiˆ + yjˆ + zkˆ ) ⎤ = ⎜⎜ n ⎟⎟ = ⎢ 1 ⎥ rn ⎦ ⎝ r ⎠ ⎣ ⎛ a x + a2 y + a3 z ⎞ =⎜ 1 ⎟ rn ⎝ ⎠ a1 x + a2 y + a3 z = x x rn =

= But,

x

(a1 x a2 y a3 z ) r n

(a1 x a2 y a3 z )

r 2n a1r n

(a1 x a2 y a3 z )nr n

1

r 2n

r = xiˆ + yjˆ + zkˆ, r2 = x2 + y2 + z2

r x

rn x

Vector Calculus

9.29

r x r y r z = , = , = x r y r z r

x

Similarly, y

and z

=

a1r n

a2 r n

=

(a1 x a2 y a3 z )nr n

y r

1

r 2n

a3 r n

(a1 x a2 y a3 z )nr n

z r

1

r 2n iˆ

=

x r

1

r 2n

=

=

(a1 x a2 y a3 z )nr n

x

( a1iˆ ar

n

ˆj



y

z

+ a2 jˆ + a3 kˆ ) r n − (a1 x + a2 y + a3 z )nr n − 2 ( xiˆ + yjˆ + zkˆ r

− ( a ⋅ r ) nr n − 2 r

)

2n

r 2n ⎡⎣∵ a1 x + a2 y + a3 z = ( a1iˆ + a2 jˆ + a3 kˆ ) ⋅ ( xiˆ + yjˆ + zkˆ ) = a ⋅ r ⎤⎦

( ) ⎛ ⎞ Hence, ∇ ⎜ a ⋅nr ⎟ = an − n an⋅+r2 r . r ⎝ r ⎠ r Example 6: If r = xiˆ + yjˆ + zkˆ and a , b are constant vectors, prove that a

3(a r) (b r) r5

1 r

b

Solution: Let

a b . r3

a = a1iˆ + a2 ˆj + a3 kˆ, b = b1iˆ + b2 ˆj + b3 kˆ ∂ ⎛1⎞ ∂ ⎛1⎞ ∂ ⎛1⎞ ⎛1⎞ ∇ ⎜ ⎟ = iˆ ⎜ ⎟ + ˆj ⎜ ⎟ + kˆ ⎜ ⎟ ∂x ⎝ r ⎠ ∂y ⎝ r ⎠ ∂z ⎝ r ⎠ ⎝r⎠

⎛ 1 ∂r ⎞ ˆ ⎛ 1 ∂r ⎞ ˆ ⎛ 1 ∂r ⎞ = iˆ ⎜ − 2 ⎟ + j ⎜ − 2 ∂y ⎟ + k ⎜ − 2 ∂z ⎟ ⎝ r ⎠ ⎝ r ∂x ⎠ ⎠ ⎝ r ˆ 2 2 2 2 ˆ ˆ r = xi + yj + zk, r = x + y + z r x r y r z = , = , = x r y r z r

But,

1 r



1 x r2 r

ˆj

1 y r2 r



1 z r2 r

(

1 ˆ xi r3

yjˆ zkˆ

)

r . r3

9.30

Engineering Mathematics

1 r

b

xiˆ + yjˆ + zkˆ r3

(b1iˆ b2 ˆj b3 kˆ) b1 x + b2 y + b3 z r3 = , say

1 r

b



x

ˆj

x



y

z

b1 x + b2 y + b3 z r3

x

b1r 3

(b1 x b2 y b3 z )

x

r3

b1r 3

r6

r

r Similarly, y and

z

= = iˆ

3(b r ) y r5

b3 r 2

3(b r ) y r5 ˆj

x

a Hence,

a a

b

y



b 1 r

1 r

3(b r ) x r5

(b1iˆ + b2 ˆj + b3 kˆ) r3

z

3(b r ) r r5

b r3

b1r 2

6

b2 r 2

r x

r6

( b r ) 3r 2 x

b1r 3

(b1 x b2 y b3 z )3r 2

a b r3

3 ( a r )( b r ) r5

3 ( b r ) ( xiˆ yjˆ zkˆ) r5

3 ( a r )( b r ) r5 a b . r3

Example 7: Find the unit vector normal to the surface x2 + y2 + z2 = a2 at a a a , , . 3 3 3 Solution: Given surface is

is the vector which is normal to the surface f (x, y, z) = c x2 + y2 + z2 = a2 f (x, y, z) = x2 + y2 + z2

Vector Calculus



x

( x2

y2

ˆj

z2 )

( x2

y

9.31

z 2 ) kˆ

y2

z

( x2

y2

z2 )

= iˆ(2 x) + ˆj (2 y ) + kˆ(2 z ) At the point

a 3

a

,

,

3

a

,

3

∇ =

2a 3

(iˆ + jˆ + kˆ ) a

Unit vector normal to the surface x2 + y2 + z2 = a2 at

=

=

=

a 3

,

a 3

(iˆ + ˆj + kˆ )

2a

4a 2 4a 2 4a 2 + + 3 3 3

3

(

) = iˆ + ˆj + kˆ .

2a iˆ + ˆj + kˆ 3

3

,

2a 3 3

3

Example 8: Find unit vector normal to the surface x2y + 2xz2 = 8 at the point (1, 0, 2). Solution: Given surface is x2y + 2xz2 = 8 f (x, y, z) = x2y + 2xz2 ( x 2 y 2 xz 2 ) kˆ ( x 2 y 2 xz 2 ) y z 2 2 ˆ = iˆ(2 xy + 2 z ) + ˆj ( x ) + k (4 xz ) At the point (1, 0, 2), = 8iˆ + jˆ + 8kˆ iˆ

x

( x 2 y 2 xz 2 )

ˆj

Unit vector normal to the surface x2y + 2xz2 = 8 at the point (1, 0, 2) =

=

8iˆ

ˆj 8kˆ

64 + 1 + 64

=

ˆj 8kˆ

8iˆ

129

.

Example 9: Find the directional derivatives of f = xy2 + yz2 at the point (2, –1, 1) in the direction of the vector ˆi + 2jˆ + 2kˆ. Solution:



x

( xy 2

yz 2 )

ˆj

y

( xy 2

yz 2 ) kˆ

= iˆy2 + jˆ (2xy + z2) + kˆ (2yz)

z

( xy 2

yz 2 )

9.32

Engineering Mathematics

At the point (2, 1, 1),

= iˆ + jˆ ( 4 + 1) + kˆ ( 2) = iˆ

3 jˆ 2kˆ Directional derivative in the direction of the vector a = iˆ + 2 jˆ + 2kˆ (

(iˆ

a

)

a (1 6 4) 3

3 ˆj 2kˆ

ˆ ˆ ˆ ) ( i + 2 j + 2k ) 1+ 4 + 4

3.

1

Example 10: Find the directional derivative of φ = 2

(x + y

P (1, –1, 1) in the direction of a = ˆi + ˆj + kˆ.

2

1 + z2 )2

at the point

Solution:

∇ = iˆ

∂ ∂x

1 1 2 2

( x2 + y 2 + z )

∂ + ˆj ∂y

2x ⎡ ⎤ˆ = ⎢− i+ 3 ⎥ ⎢⎣ 2( x 2 + y 2 + z 2 ) 2 ⎥⎦ =−

1 1 2 2

( x2 + y 2 + z )

∂ + kˆ ∂z

1 1

( x2 + y 2 + z 2 ) 2

2y 2z ⎤ˆ ⎤ ⎡ ˆj ⎡ − k + ⎢− 3 ⎥ 3 ⎥ ⎢ ⎢⎣ 2( x 2 + y 2 + z 2 ) 2 ⎥⎦ ⎢⎣ 2( x 2 + y 2 + z 2 ) 2 ⎥⎦

( xiˆ + yjˆ + zkˆ ) 3

( x2 + y 2 + z 2 ) 2 At the point (1, 1, 1), −(i − j + k ) ∇ = 3 (3) 2 Directional derivative in the direction of a = iˆ + jˆ + kˆ

=∇ ⋅ =

a a

=

−(i − j + k ) ⋅ (i + j + k ) 3

(3) 2 1 + 1 + 1

1 −1 + 1 − 1 =− . 2 9 3

Example 11: Find the directional derivative of f = xy2 + yz3 at (2, –1, 1) in the direction of the normal to the surface x log z – y2 = –4 at (–1, 2, 1). Solution: Let y = x log z

y2

is normal to the surface x log z iˆ

x

( x log z

iˆ(log z )

y2 )

y2 = 4 ˆj

y

ˆj ( 2 y ) kˆ x z

( x log z

y 2 ) kˆ

z

( x log z

y2 )

Vector Calculus

At the point ( 1, 2, 1), = iˆ (log 1) 4 jˆ kˆ 4 jˆ kˆ ˆ ˆ 4 j k is a vector normal to the surface x log z f = xy + yz 2

Now,



9.33

y2 = 4 at ( 1, 2, 1).

3

x

( xy 2

ˆj

yz 3 )

y

( xy 2

yz 3 ) kˆ

z

( xy 2

yz 3 )

iˆ (y2) + jˆ (2xy + z3) + kˆ (3yz2) At the point (2, 1, 1), = iˆ + jˆ ( 4 + 1) + kˆ ( 3) iˆ 3jˆ

3kˆ Directional derivative of f in the direction of the vector 4 jˆ ( 4 ˆj kˆ) 12 3 15 (iˆ 3 ˆj 3kˆ) 16 + 1 17 17



Example 12: Find directional derivative of the function f = xy2 + yz2 + zx2 along the tangent to the curve x = t, y = t2, z = t3 at the point (1, 1, 1). Solution: Tangent to the curve is dr d ˆ ˆ = ( xi + yj + zkˆ) dt dt d = (tiˆ + t 2 ˆj + t 3 kˆ) = (iˆ + 2tjˆ + 3t 2 kˆ) dt If x = 1, y = 1, z = 1, then t 1 At the point (1, 1, 1), t = 1 T=

T = iˆ + 2jˆ + 3kˆ

f = xy2 + yz2 + zx2 iˆ

x

( xy 2

yz 2

zx 2 )

ˆj

y

( xy 2

yz 2

zx 2 ) kˆ

z

( xy 2

yz 2

zx 2 )

iˆ (y2 + 2xz) + jˆ (2xy + z2) + kˆ (2yz + x2) At the point (1, 1, 1), = 3iˆ + 3jˆ + 3kˆ Directional derivative of f in the direction of the tangent T = iˆ + 2jˆ + 3kˆ at the point (1, 1, 1) T (iˆ + 2 ˆj + 3kˆ) 18 (3iˆ 3 ˆj 3kˆ) 1 4 9 14 + + T Example 13: Find the directional derivative of e = e2x cos yz at the origin in the direction of the tangent to the curve x = a sin t, y = a cos t, z = a t at t =

4

.

9.34

Engineering Mathematics

Solution: Tangent to the curve is dr d T= (a sin t )iˆ + (a cos t ) ˆj + (at )kˆ = dt dt (a cos t) iˆ + ( a sin t) jˆ + (a) kˆ a ˆ a ˆ t i j akˆ At the point ,T 4 2 2 f = e2x cos yz iˆ

x

(e 2 x cos yz )

ˆj

y

(e 2 x cos yz ) kˆ

z

(e 2 x cos yz )

iˆ (2e2x cos yz) + jˆ ( e2x z sin yz) + kˆ ( e2x y sin yz) = 2i At the origin, Directional derivative in the direction of the tangent to the given curve

⎛ a ˆ a ˆ ⎞ i− j + akˆ ⎟ ⎜ T 2 2 ⎠ = 2 a = 1. =∇ = 2iˆ ⋅ ⎝ 2 2 2a T a a + + a2 2 2 Example 14: Find the directional derivative of v2, where v = xy2 ˆi + zy2 ˆj + xz2 kˆ at the point (2, 0, 3) in the direction of the outward normal to the sphere x2 + y2 + z2 = 14 at the point (3, 2, 1). Solution: v2 = v · v ( xy 2 iˆ zy 2 ˆj xz 2 kˆ) ( xy 2 iˆ zy 2 ˆj xz 2 kˆ) x2y4 + z2y4 + x2z4 Let v2 = f ˆj kˆ x y z 4 4 ˆ (2xy + 2xz ) i + (4x2y3 + 4z2y3) jˆ + (2zy4 + 4x2z3) kˆ At the point (2, 0, 3), = (0 + 324) iˆ + (0 + 0) jˆ + (0 + 432) kˆ = 324 iˆ + 432 kˆ iˆ

Given sphere is x2 + y2 + z2 = 14. Let y = x2 + y2 + z2 Normal to the sphere At the point (3, 2, 1),



x ˆ ˆ = 6i + 4j + 2kˆ

ˆj

y



z

= 2xiˆ + 2yjˆ + 2zkˆ

Directional derivative in the direction of normal to the sphere ( 6iˆ + 4 ˆj + 2kˆ ) ∇ψ ( = ∇φ ⋅ = 324iˆ + 432kˆ ) ⋅ ∇ψ 36 + 16 + 4 =

1404 14

.

Vector Calculus

9.35

Example 15: Find the directional derivative of e = x2 - y2 + 2z2 at the point P(1, 2, 3) in the direction of the line PQ where Q is the point (5, 0, 4). In what direction it will be maximum? Find the maximum value of it. Solution: Position vector of the point P OP = iˆ + 2jˆ + 3kˆ Position vector of the point Q OQ = 5iˆ + 0jˆ + 4kˆ PQ = OQ iˆ

2jˆ + kˆ

OP = 4iˆ

x

( x2

y2

2z2 )

ˆj

( x2

y2

2 z 2 ) kˆ

( x2

y2

2z2 )

y z ˆ ˆ ˆ (2x) i + ( 2y) j + (4z) k At the point, (1, 2, 3), = 2iˆ 4jˆ + 12kˆ Directional derivative at the point (1, 2, 3) in the direction of the line PQ 4iˆ 2 ˆj kˆ 2iˆ 4 ˆj 12kˆ 16 + 4 + 1 8 + 8 + 12 28 = = 21 7 3

)(

(

=

)

4 7 3

Directional derivative is maximum in the direction of Maximum value of directional derivative 4 16 144

i.e. 2iˆ

4jˆ + 12kˆ

= 164 = 2 41 Example 16: Find the directional derivative of e = 6x2y + 24y2z - 8z2x at x 1 y 3 z (1, 1, 1) in the direction parallel to the line = = . Hence, find its 2 2 1 maximum value. Solution:



x

(6 x 2 y 24 y 2 z 8 z 2 x)

ˆj

y

(6 x 2 y 24 y 2 z 8 z 2 x)



= (12xy At the point (1, 1, 1),

(6 x 2 y 24 y 2 z 8 z 2 x) z 8z2) iˆ + (6x2 + 48yz) jˆ + (24y2 16zx) kˆ = 4iˆ + 54 jˆ + 8kˆ

9.36

Engineering Mathematics

x 1 y 3 z = = . 2 2 1 Direction ratios of the line are 2, 2, 1. Direction of the line = 2iˆ 2 jˆ + kˆ Directional derivative in the direction of 2iˆ Given line is

= (4iˆ + 54 ˆj + 8kˆ ) =

2jˆ + kˆ at the point (1, 1, 1) ( 2 iˆ − 2 jˆ + kˆ )

4 + 4 +1 8 − 108 + 8 − 92 = = . 3 3 Maximum value of directional derivative = 4iˆ + 54 ˆj + 8kˆ = 16 + 2916 + 64 = 2996. Example 17: Find the values of a, b, c if the directional derivative of e = axy2 + byz + cz2x3 at (1, 2, –1) has maximum magnitude 64 in the direction parallel to the z-axis. Solution: (axy 2 byz cz 2 x 3 ) kˆ (axy 2 byz cz 2 x 3 ) y z (ay2 + 3cz2x2) iˆ + (2axy + bz) jˆ + (by + 2czx3) kˆ iˆ

x

(axy 2

ˆj

byz cz 2 x 3 )

At the point (1, 2, 1), = (4a + 3c) iˆ + (4a

b) jˆ + (2b

2c) kˆ

… (1)

The directional derivative is maximum in the direction of i.e. in the direction of (4a + 3c) iˆ + (4a b) jˆ + (2b 2c) kˆ. But it is given that directional derivative is maxiand mum in the direction of z-axis i.e., in the direction of 0 iˆ + 0 jˆ + kˆ. Therefore, z-axis are parallel. 4a 3c 4a b 2b 2c = = = l , say 0 0 1 4a + 3c = 0 … (2) 4a b = 0 … (3) Substituting in Eq. (1), = (2b 2c) kˆ Maximum value of directional derivative is 64 (2b 2c)kˆ 2b

2c = 64,

b

64

c = 32

. But it is given as 64.

Vector Calculus

9.37

From Eqs. (2) and (3), 4a + 3c = 0, 4a b = 0, Solving, b = 3c Substituting in b c = 32, 4c = 32, c = 8, b = 24, a = 6 Hence, a = 6, b = 24, c = 8. Example 18: For the function e (x, y) =

x 2

x + y2

, find the magnitude of the

directional derivative along a line making an angle 30 with the positive x-axis at (0, 2).

⎞ ˆ ∂ ⎛ x ⎞ ˆ ∂ ⎛ x ⎞ ⎟+ j ⎜ 2 ⎟+k ⎜ 2 ⎟ ∂y ⎝ x + y 2 ⎠ ∂z ⎝ x + y 2 ⎠ ⎠ x( 2 x ) ⎤ ˆ ⎡ x( 2 y ) ⎤ ˆ ⎡ 1 − 2 i + ⎢− 2 j+0 =⎢ 2 2 2 2 ⎥ + + + y 2 ) 2 ⎥⎦ x y x y x ( ) ( ⎣ ⎦ ⎣

Solution: ∇ = iˆ

=

∂ ⎛ x ⎜ ∂x ⎝ x 2 + y 2

2 xy y 2 − x2 ˆ ˆj i− 2 2 2 2 (x + y ) ( x + y 2 )2

At the point (0, 2), y

4 0 ˆ 0 ˆj iˆ i = 2 2 (0 + 4) (0 + 4) 4 Line OA makes an angle 30 with positive x-axis. OA = OB + BA

A

Unit vector in the direction of OA iˆ cos 30 + jˆ sin 30 =

30° O

3ˆ 1ˆ i+ j 2 2

3ˆ 1ˆ i j 2 2

x

Fig. 9.3

Directional derivative in the direction of iˆ 4

B

3ˆ 1ˆ i + j at (0, 2) 2 2

3 8

Example 19: Find the rate of change of e = xyz in the direction normal to the surface x2y + y2x + yz2 = 3 at the point (1, 1, 1). Solution: Rate of change of f in the given direction is the directional derivative of f in that direction.

9.38

Engineering Mathematics



( xyz )

x

ˆj

y

( xyz ) kˆ

At the point (1, 1, 1),

z

( xyz ) = (yz) iˆ + (xz) jˆ + (xy) kˆ

= iˆ + jˆ + kˆ

Given surface is x2y + y2x + yz2 = 3. Let y = x2y + y2x + yz2 Normal to the surface =

ˆj kˆ x y z 2 ˆ 2 = (2xy + y ) i + (x + 2xy + z2) jˆ + (2yz) kˆ iˆ

At the point (1, 1, 1), = 3iˆ + 4 jˆ + 2kˆ Directional derivative in the direction of normal to the given surface

(iˆ

ˆ ˆ ˆ ˆj kˆ) (3i + 4 j + 2k ) 9 + 16 + 4

3+ 4+ 2

9

29

29

Example 20: Find the direction in which temperature changes most rapidly with distance from the point (1, 1, 1) and determine the maximum rate of change if the temperature at any point is given by e (x, y, z) = xy + yz + zx. Solution: Temperature is given by f (x, y, z) = xy + yz + zx. Temperature will change most rapidly i.e., rate of change of temperature, will be maximum in the direction of . iˆ

x

( xy

yz zx)

ˆj

y

yz zx) kˆ

( xy

z

( xy

yz zx)

(y + z) iˆ + (x + z) jˆ + (y + x) kˆ At the point (1, 1, 1), = 2iˆ + 2 jˆ + 2kˆ This shows that temperature will change most rapidly in the direction of 2iˆ + 2jˆ + 2kˆ and maximum rate of change = maximum directional derivative 4 4 4 = 12 = 2 3 Example 21: Find the acute angle between the surfaces x2 + y2 + z2 = 9 and z = x2 + y2 - z at the point (2, -1, 2). Solution: The angle between the surfaces at any point is the angle between the normals to the surfaces at that point. Let f1 = x2 + y2 + z2, f2 = x2 + y2

z

Normal to f1,

1



1

x

ˆj

1

y



1

z

= (2x) iˆ + (2y) jˆ + (2z) kˆ

Vector Calculus

Normal to f2, At (2, 1, 2),



2

1

= 4iˆ

2jˆ + 4kˆ,

2

ˆj

2

x = 4iˆ

1

1



2

y

and

2

2

1

(4iˆ 2 ˆj 4kˆ) (4iˆ 2 ˆj kˆ)

2

z

= (2x) iˆ + (2y) jˆ





2jˆ

Let q be the angle between the normals

9.39

2

. cos

4iˆ 2 ˆj 4kˆ 4iˆ 2 ˆj kˆ cos

(16 4 4)

16 4 16 16 4 1 cos = 36 21 cos

16 = 6 21 cos cos =

16 6 21

= cos

Hence, acute angle

1

=

8 21 63

8 21 = 54°251 63

Example 22: Find the angle between the normals to the surface xy = z2 at P(1, 1, 1) and Q (4, 1, 2). Solution: Given surface is xy = z2. Let f = xy

z2 iˆ

Normal to f,

x

( xy z 2 )

y iˆ + x jˆ

ˆj

y

( xy z 2 ) kˆ

2z kˆ

Normal at point P (1, 1, 1), N1 = iˆ + jˆ

2kˆ

Normal at point Q (4, 1, 2), N 2 = iˆ + 4 jˆ

4kˆ

Let q be the angle between N1 and N 2 . N1 · N 2 = N1 N 2 cos cos q = =

N1 ⋅ N 2 N1 N 2 1+ 4 + 8 6 33

= cos

1

= =

(iˆ + jˆ − 2kˆ ) ⋅ (iˆ + 4 jˆ − 4kˆ ) 1 + 1 + 4 1 + 16 + 16 13 198

13 198

z

( xy z 2 )

9.40

Engineering Mathematics

Example 23: Find the constants a, b such that the surfaces 5x2 - 2yz - 9x = 0 and ax2y + bz3 = 4 cut orthogonally at (1, -1, 2). Solution: If surfaces cut orthogonally, then their normals will also cut orthogonally, i.e., angle between their normals will be 90°. Given surfaces are 5x2 2yz 9x = 0 and ax2y + bz3 = 4. Let f1 = 5x2 2yz 9x and f2 = ax2y + bz3 Normal to f1, f1 = iˆ

x

(5x2 2yz 9x) + jˆ

(5x2 2yz 9x) + kˆ

z

(5x2 2yz 9x)

9) iˆ + ( 2z) jˆ + ( 2y) kˆ

(10x Normal to f2, f2 = iˆ

y

x

(ax2y + bz3) + jˆ

y

(ax2y + bz3) + kˆ

z

(ax2y + bz3)

(2axy) iˆ + (ax2) jˆ + (3bz2) kˆ At the point (1, 1, 2), f1 = iˆ 4 jˆ + 2kˆ f = 2aiˆ + ajˆ + 12bkˆ 2

f1 and f2 are orthogonal. f1 · f2 = | f1| | f2| cos

2

jˆ + 2 kˆ)· ( 2aiˆ + a jˆ + 12bkˆ) = 0 2a 4a + 24b = 0 6a + 24 b = 0 a 4b = 0 The point (1, 1, 2) lies on the surface ax2y + bz3 = 4. a (1)2 ( 1) + b (2)3 = 4 a + 8b = 4 Solving Eqs. (1) and (2), we get a = 4, b = 1 (iˆ

… (1)

… (2)

Example 24. Find the angle between the surfaces ax2 + y2 + z2 – xy = 1 and bx2y + y2z + z = 1 at (1, 1, 0). Solution: Let f1 = ax2 + y2 + z2

xy

f2 = bx2y + y2z + z The point (1, 1, 0) lies on both the surfaces. a (1)2 + (1)2 + 0 (1) (1) = 1 a=1 2 and b (1) + 0 + 0 = 1 b=1

Vector Calculus

9.41

Angle between the given surface is the angle between their normals. Normal to f1, f1 = iˆ

x

(x2 + y2 + z2

xy) + jˆ

y

(x2 + y2 + z2

xy)



(x2 + y2 + z2 xy) z (2x y) iˆ + (2y x) jˆ + (2z) kˆ Normal to f2, f2 = iˆ

x

(x2y + y2z + z) + jˆ

y

(x2y + y2z + z) + kˆ

z

(x2y + y2z + z)

(2xy) iˆ + (x2 + 2yz) jˆ + (y2 + 1) kˆ At the point (1, 1, 0), f1 = iˆ + jˆ + 0kˆ f = 2 iˆ + jˆ + 2kˆ 2

Let the angle between N 1 and N 2 is q. 1

cos

| =

q =

1|

2 +1 2 9

2

| =

2

|

=

(iˆ + ˆj ) ⋅ ( 2iˆ + ˆj + 2kˆ ) 1+1 4 +1+ 4

1 2

p 4

Hence, angle between the surfaces is

4

.

Example 25: Find the constants a, b if the directional derivative of e = ay2 + 2bxy + xz at P (1, 2, –1) is maximum in the direction of the tangent to the curve, r = (t3 – 1) iˆ + (3t – 1) jˆ + (t2 – 1) kˆ at point (0, 2, 0). Solution: f = ay2 + 2bxy + xz f1 = iˆ

x

(ay2 + 2bxy + xz) + jˆ

y

(ay2 + 2bxy + xz) + kˆ

(2by + z) iˆ + (2ay + 2bx) jˆ + (x) kˆ At the point (1, 2, 1), 1) iˆ + (4a + 2b) jˆ + kˆ Tangent to the curve r = (t3 1) iˆ + (3t 1) jˆ + (t2 f = (4b

d r = (3t2) iˆ + 3 jˆ + (2t) kˆ dt At the point (0, 2, 0), i.e., at t = 1 dr = 3 iˆ + 3 jˆ + 2 kˆ dt

1) kˆ is

z

(ay2 + 2bxy + xz)

9.42

Engineering Mathematics

Directional derivative is maximum in the direction of f but it is given that directional derivative is maximum in the direction of the tangent. Hence, f and

dr are parallel. dt 4b 1 4a 2b 1 = = 3 3 2 4b 1 1 4a 2b 1 = and = , 8a + 4b = 3 3 2 3 2 5 5 1 and 8a = 3 − 4b = 3 − = 8 2 2 1 a= 16 1 5 a= , b= . 16 8 b=

Hence,

Example 26: The temperature of the points in space is given by e = x2 + y2 – z. A mosquito located at point (1, 1, 2) desires to fly in such a direction that it will get warm as soon as possible. In what direction should it move? Solution: Temperature is given by f = x2 + y2

z

Rate of change (increase) in temperature = f iˆ

x

(x2 + y2

(2x) iˆ + (2y) jˆ At the point (1, 1, 2), f = 2iˆ + 2 jˆ kˆ

z) + jˆ

y

(x2 + y2

z) + kˆ

z

(x2 + y2

z)



Mosquito will get warm as soon as possible if it moves in the direction in which rate of increase in temperature is maximum, i.e., f is maximum. Now, f is maximum in its own direction, i.e., in the direction of f. Unit vector in the direction of = =

2iˆ 2 ˆj kˆ 4 + 4 +1 2iˆ 2 ˆj kˆ 3

Hence, mosquito should move in the direction of

2iˆ + 2 ˆj − kˆ . 3

Vector Calculus

9.43

Example 27: Find the direction in which the directional derivative of =

( x2

y2 )

at (1, 1) is zero.

xy

Solution:

( x, y )

x y

y , x



x x y

y x

y



1 y

x2

x y y

jˆ x y2

y x



x z y

y x

1 ˆ j, x

At the point (1, 1) f = 2iˆ 2 jˆ. Let the direction in which directional derivative is zero is r = xiˆ + yjˆ.

(2iˆ

xiˆ

yiˆ

x2

y2

0

2 jˆ) · (xi + y jˆ) = 0 2x 2y = 0, x = y r = xiˆ + x jˆ

Unit vector in this direction =

x( iˆ + jˆ ) x 1+1

=

iˆ + jˆ 2

Hence, directional derivative is zero in the direction of

iˆ + ˆj 2

.

Exercise 9.3 1. Find f if (i) f = log (x2 + y2 + z2) (ii) f = (x2 + y2 + z2) e

x2 y 2 z 2

.

⎡ ⎤ 2r −r ⎢ Ans.: (i) 2 (ii) ( 2 − r ) e r ⎥ r ⎢ ⎥ ⎢ where r = xiˆ + yjˆ + zkˆ , ⎥ ⎥ ⎢ ⎥ ⎢ r= r ⎣ ⎦ 2. Find f and | f| if (i) f = 2xz4 x2y at (2, 2, 1) (ii) f = 2xz2 3xy 4x at (1, 1, 2).

⎡Ans. : (i) 10iˆ −4 jˆ − 16kˆ, 2 93 ⎤ ⎢ ⎥ (ii) 7iˆ − 3 jˆ + 8kˆ, 2 29 ⎥⎦ ⎢⎣ 3. If A = 2x2iˆ 3yzjˆ + xz2kˆ and f = 2z x3y find (i) A · f (ii) A f at (1, 1, 1). [Ans. : (i) 5 (ii) 7iˆ jˆ 11kˆ ] 4. If f = 3x2y, = xz2 2y, find ( f· ). ⎡ Ans. : (6yz 2 − 12x ) iˆ ⎤ ⎢ ⎥ + 6xz 2 ˆj + 12xyzkˆ ⎥⎦ ⎢⎣

9.44

Engineering Mathematics

5. If r = xiˆ + yjˆ + zkˆ, r = | r |, prove that (i)

(log r) =

r

(ii)

r2 | r |3 = 3r r

(iii)

f (r) = f (r )

6. Prove that

r . r

a r

a

n

n

r

r

( )

n a r r r n+2

,

where a is a constant vector. 7. Find a unit vector normal to the surface x2y + 2xz = 4 at the point (2, 2, 3). 1 ˆ ⎡ ˆ ⎤ ˆ ⎢⎣ Ans. : 3 (i − 2 j − 2 k ) ⎥⎦ 8. Find unit outward drawn normal to the surface (x 1)2 + y2 + (z + 2)2 = 9 at the point (3, 1, 4). ( 2iˆ + jˆ − 2 kˆ ) ⎤ ⎡ Ans. : ⎥⎦ ⎢⎣ 3 9. Find a unit vector normal to the surface xy3 z2 = 4 at the point ( 1, 1, 2). iˆ 3 ˆj kˆ Ans. : 11 10. Find the directional derivative of f = x2yz + 4xz2 at (1, 2, 1) in the direction of 2iˆ jˆ 2kˆ. Ans.:

37 3

11. Find the directional derivative of f = xy + yz + zx at (1, 2, 0) in the direction of vector iˆ + 2 jˆ + 2kˆ. 10 Ans. : 3 12. Find the maximal directional derivative of x3y2z at (1, 2, 3). Ans. : 4 91 13. In what direction from the point (2, 1, 1) is the directional derivative of f = x2yz3 a maximum? Find its maximum value of magnitude.

⎡ Ans. : maximum in the direction of ⎤ ⎥ ⎢ ∇ = 4iˆ − 4 ˆj + 12kˆ, 4 11 ⎦ ⎣

14. In what direction from the point (3, 1, 2) is the directional derivative of f = x2y2z4 a maximum? Find its maximum value of magnitude. ⎡⎣ Ans. : 96 (iˆ + 3jˆ − 3kˆ), 96 19 ⎦⎤ 15. In what direction from the point (1, 3, 2) is the directional derivative of f = 2xz y2 a maximum? Find its maximum value of magnitude. ⎡⎣ Ans. : 4iˆ − 6 jˆ + 2kˆ , 2 14 ⎤⎦ 16. What is the greatest rate of change of f = xyz2 at the point (1, 0, 3)? [Ans. : f = 9] 17. If the directional derivative of f = ax2 + by + 2z at (1, 1, 1) is maximum in the direction of iˆ + jˆ + kˆ, then find values of a and b. [Ans. : a = 1, b = 2] 18. If the directional derivative of f = ax + by + cz at (1, 1, 1) has maximum magnitude 4 in a direction parallel to x axis, then find values of a, b, c. [Ans. : a = 2, b = 2, c = 2] 19. Find the directional derivative of f = x2y + y2z + z2x2 at (1, 2, 1) in the direction of the normal to the surface x2 + y2 z2x = 1 at (1, 1, 1). 4 Ans. : 3 20. Find the directional derivative of f = x2y + yz2 at (2, 1, 1) in the direction normal to the surface x2y + y2x + yz2 = 3 at (1, 1, 1). 13 Ans. : 29 21. Find the directional derivative of f = x2y + y2z + z2x at (2, 2, 2) in the direction of the normal to the surface 4x2y + 2z2 = 2 at the point (2, 1, 3). 36 Ans. : 41

Vector Calculus 22. Find the rate of change of f = xy + yz + zx at (1, 1, 2) in the direction of the normal to the surface x2 + y2 = z + 4. Ans. :

14

21 23. Find the directional derivative of f = x2yz2 along the curve x = e t, y = 2 sin t + 1, z = t cost at t = 0. 1

Ans. :

6

24. Find the directional derivative of f = x2y2z2 at (1, 1, 1) in the direction of the tangent to the curve x = et, y = 2 sin t + 1, z = t cos t, at t = 0. 2 3 Ans. : 3 25. Find the directional derivative of the scalar function f = x2 + xy + z2 at the point P(1, 1, 1) in the direction of the line PQ where Q has coordinates (3, 2, 1). ⎡ Hint : PQ = OQ − OP ⎤ ⎢ ⎥ ˆ ˆ ˆ ˆ ˆ ˆ ⎢ = (3i + 2 j + k ) − ( −i − j − k ) ⎥ ⎢⎣ = 2iˆ + 3 jˆ + 2kˆ ⎦⎥ Ans. :

1 17

26. Find the directional derivative of f = 2x3y 3y2z at the point P (1, 2, 1) in the direction towards Q (3, 1, 5). In what direction from P is the directional derivative maximum? Find the magnitude of maximum directional derivative. Ans. :

90 ,12iˆ 7

14 jˆ

12kˆ, 22

27. Find the directional derivative of f = 4xz3 3x2y2z at (2, 1, 2) in the direction from this point towards the point (4, 4, 8).

9.45

376 7

Ans. :

28. Find the angle of intersection of the spheres x2 + y2 + z2 = 29 and x2 + y2 + z2 + 4x 6y 8z = 47 at (4, 3, 2). 19 Ans. : cos 1 29 29. Find the angle between the normals to the surface 2x2 + 3y2 = 5z at the point (2, 2, 4) and ( 1, 1, 1). 65 Ans. : cos 1 233 77 30. Find the angle between the normals to the surface xy = z2 at the points (1, 4, 2) and ( 3, 3, 3). Ans. :

= cos

1

1

22

31. Find the acute angle between the surfaces xy2z = 3x + z2 and 3x2 y2 + 2z = 1 at the point (1, 2, 1). Ans. : cos

1

6 14

32. Find the constant a and b so that the surface ax2 byz = (a + 2)x will be orthogonal to the surface 4x2y + z3 = 4 at the point (1, 1, 2). ⎡ Hint : condition for orthogonal - ⎤ ⎥. ⎢ ity is ∇f ⋅ ∇y = 0 ⎦ ⎣ Ans. : a =

5 , b =1 2

33. Find the angle between the two surfaces x2 + y2 + a z2 = 6 and z = 4 y2 + bxy at P (1, 1, 2). ⎡ Hint : (1, 1, 2) lies on both ⎤ ⎢ surfaces, a = 1, b = − 1⎥⎦ ⎣

Ans. : cos

1

6 11

9.46

Engineering Mathematics

34. Find the directional derivative of f = x2 + y2 + z2 in the direction of the x y z line = = at (1, 2, 3). 3 4 5

Let the direction in which directional derivative is zero is r = xiˆ + yjˆ xiˆ + yjˆ 0 x2 + y 2

26 2 5

Ans.

(2iˆ

2 jˆ) · (xiˆ + yjˆ) = 0 2x 2y = 0, x = y r = xiˆ + x jˆ unit vector in this direction x iˆ + ˆj = x 1+1 ˆi + ˆj = 2

35. Find the direction in which the directional derivative of f = (x + y) = ( x2 y 2 ) at (1, 1) is zero. xy

(

x y ⎡ ⎤ ⎢ Hint : ( x, y ) = y − x , ⎥ ⎢ ⎥ ⎢ ⎛ 1 y ⎞ˆ ⎛ x 1 ⎞ ˆ ⎥ ⎢∇ = ⎜ + 2 ⎟ i + ⎜ − 2 − ⎟ j , ⎥ ⎜ x ⎟⎠ ⎥ ⎢ ⎝y x ⎠ ⎝ y ⎢ ⎥ ⎣ At (1,, 1),∇ = 2iˆ − 2 jˆ ⎦

)

Hence, directional derivative is zero in iˆ + ˆj the direction of . 2

9.12 DIVERGENCE The divergence of a vector point function F is denoted by div F or defined as iˆ

F

x

ˆj

y



z

If

F = F1 iˆ + F2 jˆ + F3 kˆ,

then

F

iˆ =

x

ˆj

y



z

F

· (F1 iˆ + F2 jˆ + F3 kˆ )

F F1 F + 2+ 3 x y z

which is a scalar quantity. Note: (i) · F F (ii)

F · , because

F1

x

F1 x

F = iˆ

F2 F2 y

F x

ˆj

· F is a scalar quantity whereas

F3

y

z

is a scalar differential operator.

F3 z F y



F z

(if F = F1 iˆ + F2 jˆ + F3 kˆ )

· F and is

Vector Calculus

9.47

9.12.1 Physical Interpretation of Divergence Consider the case of a homogeneous and incompressible fluid flow. Consider a small rectangular parallelepiped of dimensions d x, d y, d z parallel to x, y and z axes respectively. Let v = v iˆ + v jˆ + v kˆ be the velocity of the fluid at point A (x, y, z). 1

2

3

The velocity component parallel to x of the face PQRS v1 (x + d x, y, z) v1 +

axis (normal to the face PQRS) at any point

v1 d x [expanding by Taylor’s series and ignoring higher x powers of d x]

C S

D

R

dz

V1 d y

O

Q

B

A

P x

y

Fig. 9.4 Mass of the fluid flowing in across the face ABCD per unit time velocity component normal to the face ABCD area of the face ABCD v1 (d y d z) Mass of the fluid flowing out across the face PQRS per unit time velocity component normal to the face PQRS area of the face PQRS v1

v1 x x

y z

Gain of fluid in the parallelepiped per unit time in the direction of x-axis v1 =

v1 x x

v1 x y z x

y z v1 y z

9.48

Engineering Mathematics

Similarly, gain of fluid in the parallelepiped per unit time in the direction of y-axis =

∂v2 x y z ∂y

and gain of fluid in the parallelepiped per unit time in the direction of z-axis =

∂v3 x y z ∂z

Total gain of fluid in the parallelepiped per unit time v v1 v + 2+ 3 x y z

=

x y z

But, d x d y d z is the volume of the parallelepiped. Hence, total gain of fluid per unit volume =

v v1 v + 2+ 3 x y z div v =

· v

Note: A point in a vector field F is said to be a source if div F is positive, i.e., · F > 0 and is said to be a sink if div F is negative, i.e, · F < 0.

9.12.2 Solenoidal Function A vector function F is said to be solenoidal if div F = 0 at all points of the function. For such a vector, there is no loss or gain of fluid.

9.13 CURL The curl of a vector point function F is denoted by curl F or iˆ

F

=

x

+ ˆj

y

+ kˆ



ˆj



x F1

y F2

z F3

iˆ which is a vector quantity.

F3 y

F2 z

( F1iˆ F2 ˆj F3 kˆ)

z

ˆj

F and is defined as

F3 x

F1 z



F2 x

F1 y

Vector Calculus

9.49

9.13.1 Physical Interpretation of Curl Let

be the angular velocity of a rigid body moving about a fixed point. The linear

velocity v of any particle of the body with position vector r w.r.t. to the fixed point is given by, v =w r Let w = w1iˆ + w2 jˆ + w3 kˆ, r = x iˆ + y jˆ + z kˆ v =w

r

iˆ =



ˆj 1

2

x

3

y

iˆ (w2 z Curl v =

z w3 y)

w3 x) + kˆ (w1 y

jˆ (w1 z

w2 x)

v

= 2

z



ˆj



x

y

z

3

y

3

x

iˆ (w1 + w1) jˆ ( w2 2(w1 iˆ + w2 jˆ + w3 kˆ)

z

1

1

y

2

x

w2) + kˆ (w3 + w3)

2w Curl v = 2 w Thus, the curl of the linear velocity of any particle of a rigid body is equal to twice the angular velocity of the body. This shows that curl of a vector field is connected with rotational properties of the vector field and justifies the name rotation used for curl.

9.13.2 Irrotational Field A vector point function F is said to be irrotational, if curl F = 0 at all points of the function, otherwise it is said to be rotational. Note: If F = f, then curl F = Thus, if

F =

f = 0.

F = 0, then we can find a scalar function f so that F = f. A vector

field F which can be derived from a scalar field f so that F = f is called a conservative vector field and f is called the scalar potential.

9.50

Engineering Mathematics

Example 1: If A = x2ziˆ – 2y3z2 jˆ + xy2zkˆ, find Æ · A at the point (1, –1, 1). A A1 A + 2 + 3 , where A = A iˆ + A jˆ + A kˆ 1 2 3 x y z

· A =

Solution:

· A =

x

(x2z) +

( 2 y3z2) +

z

(xy2z)

6 y2z2 + xy2

2xz At the point (1, 1, 1), A

y

2 (1) (1) 6 ( 1) 2 (1) 2 1( 1) 2 6+1

2 3

Example 2: If r = xiˆ + yjˆ + zkˆ, prove that div (grad rn) = n (n + 1) rn–2. Solution: grad r n = iˆ

∂r n ˆ ∂r n ˆ ∂r n + j +k ∂x ∂y ∂z

⎛ ∂r ⎞ ∂r ⎞ ∂r ⎞ ⎛ ⎛ = iˆ ⎜ nr n −1 ⎟ + jˆ ⎜ nr n −1 ⎟ + kˆ ⎜ nr n −1 ⎟ ∂y ⎠ ∂z ⎠ ∂x ⎠ ⎝ ⎝ ⎝

But

r = xiˆ + yjˆ + zkˆ,

r2 = r

2

= x2 + y2 + z2

r x r y r z = , = , = x r y r z r xˆ y ˆ z ˆ i + j+ k r r r

grad r n = nr n–1

= nr

n 1

( xiˆ + yjˆ + zkˆ) r

nr n 2 r div (grad r n) = · (nr n 2 r ) n · rn 2 (xiˆ + yjˆ + zkˆ ) =n =n x

x

(r n 2 x) +

x

rn

2

+ rn

y 2

(r n 2 y ) + +y

y

rn

z 2

(r n 2 z ) + rn

2

+z

z

rn

2

+ rn

2

Vector Calculus

9.51

∂r ∂r ∂r ⎤ ⎡ = n ⎢3r n − 2 + x(n − 2)r n − 3 + y (n − 2)r n − 3 + z (n − 2)r n − 3 ⎥ ∂x ∂y ∂z ⎦ ⎣ n 3r n

2

(n 2)r n

3

n 3r n

2

(n 2)r n

3

n (n + 1) rn

( x2

y2 r

r2 r

nr n

z2 )

2

(3

r x

∵ n

x , r

r y

y , r

r z

z r

2)

2

Example 3: Prove that for vector function A, Æ ë (Æ ë A ) = Æ (Æ · A ) – Æ2 A . Solution: Let A = A1iˆ + A2 jˆ + A3 kˆ

iˆ ∂ ∇× A = ∂x A1

ˆj ∂ ∂y A2

kˆ ∂ ∂z A3

⎛ ∂A ∂A ⎞ = iˆ ⎜ 3 − 2 ⎟ − ∂z ⎠ ⎝ ∂y

(

)

∇× ∇ × A =

ˆj ⎛⎜ ∂A3 − ∂A1 ⎞⎟ + kˆ ⎛⎜ ∂A2 − ∂A1 ⎞⎟ ∂y ⎠ ∂z ⎠ ⎝ ∂x ⎝ ∂x

iˆ ∂ ∂x

ˆj ∂ ∂y

kˆ ∂ ∂z

∂A3 ∂A2 − ∂y ∂z

∂A1 ∂A3 − ∂z ∂x

∂A2 ∂A1 − ∂x ∂y

⎡⎛ ∂ 2 A ∂ 2 A ⎞ ⎛ ∂ 2 A ∂ 2 A 3 2 − 21 ⎟ − ⎜ 21 − = iˆ⎢⎜ ⎜ ⎟ ⎜ y x ∂ ∂ x z ∂ ∂ ∂y ⎠ ⎝ ∂z ⎢⎣⎝

⎞⎤ ⎟⎥ ⎟⎥ ⎠⎦

⎡⎛ ∂ 2 A ∂ 2 A ⎞ ⎛ ∂ 2 A ∂ 2 A ⎞ ⎤ 3 2 1 − ˆj ⎢⎜ 22 − − ⎟⎥ ⎟−⎜ 2 ⎟ ⎜ ⎟ ⎜ ∂ ∂ ∂ ∂ x y y z ∂z ⎠ ⎦⎥ ⎢⎣⎝ ∂x ⎠ ⎝ ⎡⎛ ∂ 2 A ∂ 2 A ⎞ ⎛ ∂ 2 A ∂ 2 A ⎞ ⎤ 3 3 2 1 + kˆ ⎢⎜ − − ⎟⎥ ⎟−⎜ ∂y∂z ⎟⎠ ⎥ ⎢⎣⎜⎝ ∂x∂z ∂x 2 ⎟⎠ ⎜⎝ ∂y 2 ⎦ Consider

⎡⎛ ∂ 2 A2 ∂ 2 A1 ⎞ ⎛ ∂ 2 A1 ∂ 2 A3 ⎞ ⎤ iˆ ⎢⎜ − 2 ⎟−⎜ 2 − ⎟⎥ ∂x ∂z ⎠ ⎥⎦ ⎢⎣⎝ ∂y ∂x ∂y ⎠ ⎝ ∂z

⎡ ∂ ⎛ ∂A ⎞ ∂ 2 A ∂ 2 A ∂ ⎛ ∂A ⎞ ⎤ = iˆ ⎢ ⎜ 2 ⎟ − 21 − 21 + ⎜ 3 ⎟ ⎥ ∂x ⎝ ∂z ⎠ ⎥⎦ ∂z ⎢⎣ ∂x ⎝ ∂y ⎠ ∂y

9.52

Engineering Mathematics

⎡ ∂ ⎛ ∂A ∂A ⎞ ⎛ ∂ 2 A ∂ 2 A ⎞ ⎛ ∂ 2 A ∂ 2 A ⎞ ⎤ = iˆ ⎢ ⎜ 2 + 3 ⎟ − ⎜ 21 + 21 ⎟ + ⎜ 21 − 21 ⎟ ⎥ ∂z ⎠ ⎜⎝ ∂y ∂x ⎟⎠ ⎥⎦ ∂z ⎟⎠ ⎜⎝ ∂x ⎢⎣ ∂x ⎝ ∂y ⎡ ∂ 2 A1 ⎤ ⎥ ⎢ Adding and subtracting ∂x 2 ⎥⎦ ⎢⎣ ⎡ ∂ ⎛ ∂A ∂A ∂A ⎞ ⎛ ∂ 2 A ∂ 2 A ∂ 2 A ⎞ ⎤ 1 = iˆ ⎢ ⎜ 1 + 2 + 3 ⎟ − ⎜ 21 + + 21 ⎟ ⎥ ∂y ∂z ⎠ ⎝ ∂x ∂y 2 ∂z ⎠ ⎥⎦ ⎢⎣ ∂x ⎝ ∂x ∂ = iˆ ( ∇ ⋅ A) − iˆ ∇2 A1 ∂x Similarly,

⎡⎛ ∂ 2 A ∂ 2 A1 ⎞ ⎛ ∂ 2 A3 ∂ 2 A2 − − ˆj ⎢⎜ 22 − ⎟−⎜ ∂x∂y ⎠ ⎝ ∂y∂z ∂z 2 ⎢⎣⎝ ∂x

⎞⎤ ∂ 2 ⎟ ⎥ = ˆj ( ∇ ⋅ A ) − ˆj ∇ A2 y ∂ ⎠ ⎥⎦ 2 2 2 2 ⎡⎛ ∂ A1 ∂ A3 ⎞ ⎛ ∂ A3 ∂ A2 ⎞ ⎤ ∂ 2 and kˆ ⎢⎜ − 2 ⎟−⎜ 2 − ⎟ ⎥ = kˆ ( ∇ ⋅ A ) − kˆ ∇ A3 ∂ x ∂ z ∂ y ∂ z z ∂ ∂ x ∂ y ⎠ ⎥⎦ ⎠ ⎝ ⎣⎢⎝ Hence, ∇ × (∇ × A) = iˆ

x

+ ˆj

y

+ kˆ

z

(∇ ⋅ A)

2

(A1iˆ + A2 jˆ + A3 kˆ )

∇ (∇ ⋅ A) − ∇2 A Example 4: If A = Æ (xy + yz + zx), find Æ · A and Æ ë A. Solution:

A =

(xy + yz + zx) iˆ

x

(xy + yz + zx) + jˆ

y

(xy + yz + zx) + kˆ

(y + z) iˆ + (x + z) jˆ + (y + x) kˆ · A =

· [(y + z) iˆ + (z + x) jˆ + (x + y) kˆ ] x

A

(y + z) +

y

(z + x) +



ˆj



x y+z

y z+x

z x+ y

z

(x + y) = 0

z

(xy + yz + zx)

Vector Calculus

9.53

∂ ∂ ⎤ ⎡∂ ⎤ ⎡∂ = iˆ ⎢ ( x + y ) − ( z + x) ⎥ − jˆ ⎢ ( x + y ) − ( y + z ) ⎥ ∂z ∂z ⎦ ⎣ ∂x ⎦ ⎣ ∂y ∂ ⎤ ⎡∂ + kˆ ⎢ ( z + x) − ( y + z ) ⎥ ∂y ⎦ ⎣ ∂x iˆ (1

1) + kˆ (1

jˆ (1

1)

1)= 0

Example 5: Verify Æ (Æ ë A) = Æ (Æ ⋅ A) – Æ2 A for A = x2yiˆ + x3y2jˆ – 3 x2z2kˆ. Solution:

iˆ ∂ ∇× A = ∂x x2 y

ˆj ∂ ∂y

kˆ ∂ ∂z

x3 y 2

−3 x 2 z 2

∂ ⎤ ⎡∂ = iˆ ⎢ ( −3 x 2 z 2 ) − ( x3 y 2 ) ⎥ − y z ∂ ∂ ⎦ ⎣ ∂ ⎤ ⎡∂ + kˆ ⎢ ( x3 y 2 ) − ( x 2 y ) ⎥ ∂ x ∂ y ⎦ ⎣ 0 · iˆ





ˆj

∇ × (∇ × A) = x

y 6 xz

iˆ (6x2y (6x2y · A = iˆ

x

x

z 2

x

+ kˆ

2 2

jˆ (6xy2

12xz) iˆ + ˆj

(x2y) +

y

(6xy2

+ kˆ

y

z

0) + kˆ (6z2 2x) jˆ + (6z2) kˆ 2x

· (x2yiˆ + x3y2jˆ

(x3y2) +

z

0)

3x2z2kˆ )

( 3x2z2)

6x2z

(2xy + 2x3y

z

x2 )

(3 x y

12xz)

2xy + 2 x3y ∇ (∇ ⋅ A) = iˆ

x2)kˆ

( 6xz2) jˆ + (3x2y2

0

ˆj ⎡ ∂ ( −3 x 2 z 2 ) − ∂ ( x 2 y ) ⎤ ⎢⎣ ∂x ⎥⎦ ∂z

(2xy + 2x3y

6x2z) + jˆ 6x2z)

y

(2xy + 2x3y

6x2z)

9.54

Engineering Mathematics (2y + 6x2y 2 2

A =

x2

0) jˆ + ( 6x2) kˆ

12xz) iˆ + (2x + 2x3

(x2yiˆ + x3y2 jˆ

2

3x2z2kˆ ) +

y2

(x2yiˆ + x3y2 jˆ

3x2z2kˆ )

(x2yiˆ + x3y2jˆ

3x2z2kˆ )

2

z2 x

6z2kˆ ) + 2x3jˆ

(2yiˆ + 6xy2jˆ ∇ (∇ ⋅ A)

2

6xz2kˆ ) +

(2xyiˆ + 3x2y2jˆ

A = (6x2y

2

z

( 6x2z kˆ )

6x2kˆ = 2yiˆ + (6xy2 + 2x3) jˆ – 6 (z2 + x2)kˆ 6xy2) jˆ + (6z2) kˆ

12xz) iˆ + (2x

Hence, ∇ × (∇ × A) = ∇ (∇ ⋅ A)

(x2iˆ + 2x3y jˆ) +

y

A

Example 6: Show that A = 3 y4z2iˆ + 4x3z2 jˆ – 3x2y2kˆ is solenoidal. Solution: A = 3y4z2 iˆ + 4x3z2 jˆ · A =

x

(3y4z2) +

y

3x2y2 kˆ (4x3z2) +

z

( 3x2y2) = 0

· A = 0, A is solenoidal.

Since

Example 7: Determine the constant b such that A = (bx + 4y2z) iˆ + (x3sin z – 3y) jˆ – (ex + 4 cos x2y) kˆ is solenoidal. Solution: If A is solenoidal, then · A =0 x

(bx + 4y2z ) +

y

(x3 sin z

3y) +

z

( ex

4 cos x2y) = 0. b

Example 8: Show that the vector field A =

a ( xiˆ + yjˆ ) x2 + y2

3=0 b=3 is a source field or sink

field according as a > 0 or a < 0. Solution: Vector field A is a source field if field if · A < 0.

· A > 0 and vector field A is a sink

Vector Calculus

ax

A

2

x +y =

2

ay



2

x +y

ax x

2

x +y

+

2

2

ˆj

ay y

2

x + y2

x ⋅ 2x 1 ⎡ = a⎢ − + 3 2 2 ⎢⎣ x + y 2( x 2 + y 2 ) 2 ⎡ 2 ( x2 + y 2 ) = a⎢ − 3 2 2 ⎢⎣ x + y ( x2 + y 2 ) 2 =

Since

9.55

1 x +y 2

2



y⋅2y

⎤ ⎥ 2( x 2 + y ) ⎥⎦ 3 2 2

⎤ ⎥ ⎥⎦

a 2

x + y2

x 2 + y 2 is always positive,

· A > 0 if a > 0, and

· A < 0 if a < 0. Hence,

A is a source field if a > 0 and sink field if a < 0.

Example 9: If A = (ax2y + yz) iˆ + (xy2 – xz2) jˆ + (2xyz – 2x2y2) kˆ is solenoidal, find the constant a. Solution: If A is solenoidal, then · A = 0, · [(ax2y + yz) iˆ + (xy2 xz2) jˆ + (2xyz

(2xyz 2x2y2) = 0 z 2axy + 2xy + 2xy = 0 2a = 4 a= 2 Example 10: Find the curl of A = exyz (iˆ + jˆ + kˆ ) at the point (1, 2, 3). x

(ax2y + yz) +

2x2y2) kˆ ] = 0

Solution: Curl of A = iˆ ∂ = ∂x

ˆj ∂ ∂y

kˆ ∂ ∂z

e xyz

e xyz

e xyz

y

(xy2

xz2) +

A

⎛ ∂ ⎞ ⎛ ∂ ⎞ ∂ ∂ ∂ ⎛ ∂ ⎞ = iˆ ⎜ e xyz − e xyz ⎟ − jˆ ⎜ e xyz − e xyz ⎟ + kˆ ⎜ e xyz − e xyz ⎟ ⎝ ⎠ ∂z ∂x ∂z ∂y ⎝ ∂y ⎠ ⎝ ∂x ⎠ (exyz · xz

exyz · xy) iˆ

(exyz · yz

exyz · xy) jˆ + (exyz · yz

exyz · xz) kˆ

9.56

Engineering Mathematics

At the point (1, 2, 3), Curl A = e6 [iˆ (3 2) e6 (iˆ

2) + kˆ (6 – 3)]

jˆ (6

4 jˆ + 3kˆ )

Example 11: Find curl curl A = x2y iˆ – 2xz jˆ + 2yzkˆ at the point (1, 0, 2). Solution: Curl A =

A

iˆ ∂ = ∂x

kˆ ∂ ∂z

ˆj ∂ ∂y

x 2 y −2 xz 2 yz ∂ ∂ ⎡∂ ⎤ ⎤ ⎡∂ = iˆ ⎢ ( 2 yz ) − ( −2 xz ) ⎥ − ˆj ⎢ ( 2 yz ) − ( x 2 y ) ⎥ x z ∂ ∂ ∂ y z ∂ ⎣ ⎦ ⎦ ⎣ ∂ ⎤ ⎡∂ + kˆ ⎢ ( −2 xz ) − ( x 2 y ) ⎥ x y ∂ ∂ ⎦ ⎣ (2z + 2x) iˆ

(

)

Curl Curl A =

0) jˆ + ( 2z

(0

(

A)

iˆ ∂ ∂x

ˆj ∂ ∂y

kˆ ∂ ∂z

2( z + x)

0

−( x 2 + 2 z )

=

∂ ⎡∂ ⎤ = iˆ ⎢ ( − x 2 − 2 z ) − (0) ⎥ − ∂ ∂z ⎦ ⎣ y ∂ ⎡∂ ⎤ + kˆ ⎢ (0) − 2( z + x) ⎥ ∂ x y ∂ ⎣ ⎦ iˆ (0

0)

jˆ ( 2x

2) + kˆ (0

x2) kˆ

ˆj ⎡ ∂ ( − x 2 − 2 z ) − ∂ 2( z + x) ⎤ ⎢⎣ ∂x ⎥⎦ ∂z

0)

(2x + 2) jˆ At the point (1, 0, 2),

(

)

Curl Curl A = (2 + 2) jˆ 4 jˆ Example 12: Prove that F = 2xyz2iˆ + [x2z2 + z cos (yz) ] jˆ + (2x2yz + y cos yz) kˆ is a conservative vector field.

Vector Calculus

F =0

Solution: Vector field F is conservative if iˆ F



ˆj

x

y

2 xyz

2



(2 x 2 yz

y ˆj kˆ

9.57

z

2 2

2

x z + z cos yz 2 x yz + y cos yz y cos yz )

(2 x 2yz

x

( x2 z 2

x

( x2 z 2

z

y cos yz )

z

z cos yz )

z cos yz )

(2 xyz 2 )

(2 xyz 2 )

y

yz sin yz 2x2z cos yz + zy sin yz) iˆ 4xyz) jˆ + (2xz2 2xz2) kˆ

(2x2z + cos yz (4xyz 0

Hence, F is conservative vector field. Example 13: Determine the constants a and b such that curl of (2xy + 3yz) iˆ + (x2 + axz – 4 z2) jˆ + (3xy + 2byz) kˆ is zero. 4z2) jˆ + (3xy + 2byz) kˆ

Solution: Let F = (2xy + 3yz) iˆ + (x2 + axz Curl F =

F =0



ˆj



x

y

z

2 xy 3 yz iˆ

y

(3 xy 2byz )

z kˆ

(3x + 2bz

ax + 8z) iˆ [(3

( x2

x

x2

axz 4 z 2

axz 4 z 2 )

ˆj

axz 4 z 2 )

(3y

3y) jˆ + (2x + az

a)x + 2z (b + 4)] iˆ

3 xy 2byz

x

( x2

y

=0

(3xy 2byz )

(2 xy 3 yz )

z 0

3z) kˆ = 0 0 jˆ + z (a 3) kˆ = 0 2x

(2 xy 3 yz )

9.58

Engineering Mathematics

Comparing coefficients of iˆ and kˆ, we get (3 a) x + 2 (b + 4) z = 0 (a 3) z = 0 Solving both the equations a = 3, b = 4 Example 14: Show that F = (y2 – z2 + 3yz – 2x) iˆ + (3xz + 2xy) jˆ + (3xy – 2xz + 2z)kˆ is both solenoidal and irrotational . Solution: If F is solenoidal, · F =

x

(y2

z2 + 3yz

· F =0 2x) +

y

(3xz + 2xy) +

z

(3xy

2xz + 2z)

2 + 2x 2x + 2 = 0 Hence, F is solenoidal. F =0

If F is irrotational,

F y2 iˆ

ˆj kˆ (3x



ˆj



x

y

z

z2 y

x x

3 yz 2 x 3 xz 2 xy 3 xy 2 xz 2 z

(3 xy 2 xz 2 z )

z

(3 xy 2 xz 2 z ) (3 xz 2 xy ) 3x) iˆ

(3y

y

z

(3 xz 2 xy )

( y2

( y2

2z + 2z

z2

z2

3 yz 2 x)

3 yz 2 x)

3y) jˆ + (3z + 2y

2y

3z) kˆ = 0

Hence, F is irrotational. Example 15: Find the directional derivative of the divergence of F (x, y, z) = xy iˆ + xy2jˆ + z2kˆ at the point (2, 1, 2) in the direction of the outer normal to the sphere x2 + y2 + z2 = 9. Solution: F (x, y, z) = xy iˆ + xy2 jˆ + z2 kˆ Divergence of F (x, y, z) = =

· F ∂ ∂ ∂ 2 ( xy ) + ( xy 2 ) + (z ) ∂x ∂y ∂z y + 2xy + 2z

Vector Calculus

Gradient of divergence of F =

9.59

( · F)

= iˆ

x

+ ˆj

y

+ kˆ

z

(y + 2xy + 2z)

= 2y iˆ + (1 + 2x) jˆ + 2kˆ At the point (2, 1, 2), ( · F ) = 2 iˆ + 5 jˆ + 2kˆ + ˆj + kˆ (x2 + y2 + z2) x y z = 2 (xiˆ + yjˆ + zkˆ ) Normal at (2, 1, 2) = 2 (2iˆ + jˆ + 2kˆ ) Normal to sphere = iˆ

Directional derivative in the direction of the outer normal to the sphere x2 + y2 + z2 = 9 4iˆ + 2 ˆj + 4kˆ

= (2iˆ + 5 jˆ + 2kˆ ) · =

1 (8 + 10 + 8) 6

=

13 3

16 + 4 + 16

Exercise 9.4 1. Find divergence and curl of x2 cos z iˆ + y log x jˆ yz kˆ. ⎡ Ans. : 2x cos z + log x − y, ⎤ ⎢ ⎥ ˆ − jˆ x 2 sin z + kˆ y ⎥ ⎢ iz ⎣ x⎦ 3 2 4 2. If f = 2x y z , prove that div (grad f) = 12xy2z4 + 4x3z4 + 24x3y2z2.

3. Find curl (curl A ), if A = x2y iˆ 2xz jˆ + 2yz kˆ. [Ans. : (2x + 2) jˆ] 4. If A = 2yziˆ x2yjˆ + x z2kˆ, B = x2iˆ + yzjˆ find

xykˆ and f = 2x2yz3,

(i) ( A · ) f

(ii) A ·

(iii) ( B · ) A

(iv) ( A

(v) A

f

⎡ Ans. : (i) and (ii) 8 xy 2 z 4 − 2 x 4 yz 3 ⎤ ⎢ ⎥ + 6 x 3 yz 4 (iii) ( 2 yz 2 − 2 xy 2) iˆ ⎥ ⎢ ⎢ ⎥ − ( 2 x 3 y + x 2 yz ) jˆ ⎢ ⎥ ⎢ + ( x 2 z 2 − 2 x 2 yz )kˆ (iv ) and ⎥ ⎢ ⎥ (v) − (6 x 4 y 2 z 2 + 2 x 3 z 5 ) iˆ ⎥ ⎢ ⎢ ⎥ + ( 4 x 2 yz 5 − 12 x 2 y 2 z 3 ) jˆ ⎢ ⎥ ⎢⎣ + (4 x 2 yz 4 + 4 x 3 y 2 z 3 )kˆ ⎦⎥ 5. If A = x2 iˆ + xyex jˆ + sin z kˆ, find ·(

[Ans. : 0] 6. If f = tan

f )f

A ).

7. If f = 2x2

1

y , find div (grad f). x [Ans. : 0] 3y2 + 4z2, find curl (grad f). [Ans. : 0]

9.60

Engineering Mathematics

8. Prove that for every field A, div (curl A ) = 0. 9. Prove that gradient field describing a motion is irrotational. [Hint: Prove that f = 0] 10. Prove that A = iˆ yz + jˆ xz + kˆ xy is irrotational and find a scalar function f (x, y, z) such that A = grad f. [Ans. : xyz + c] 11. Prove that A = (6xy + z3) iˆ + (3 x2 z) jˆ + (3x z2 y) kˆ is irrotational. Find the function f such that A = f. [Ans. : f = 3 x2y + x z3 yz] 12. Prove that the velocity given by A = (y + z) iˆ + (z + x) jˆ + (x + y) kˆ is irrotational and find its scalar potential. Is the motion possible for an incompressible fluid? ⎡ Ans. : f = yz + zx + xy, motion is ⎤ ⎥ ⎢ possible because ∇ ⋅ A = 0 ⎦ ⎣ 13. Prove that A = (z2 + 2xy + 3y) iˆ + (3x + 2y + z) jˆ + (y + 2zx) kˆ is irrotational and find scalar potential f such that A = f and f (1, 1, 0) = 4. [Ans. : f = z2x + x2 + 3xy + y2 + yz 1]

14. Prove that A = (z2 + 2x + 3y) iˆ + (3x + 2y + z) jˆ + (y + 2zx) kˆ is conservative and find scalar potential f such that A = f. [Ans. : f = x2 + y2 + z2x + 3xy + zy] 15. Prove that A = (y2cosx + z3) iˆ + (2y sin x 4) jˆ + (3xz2 + 2) kˆ is irrotational and find its scalar potential. [Ans. : f = y2 sin x + z3x 4y + 2z] 16. Prove that a = 1 or b = 0, if (xyz)b (xaiˆ + yajˆ + zakˆ ) is an irrotational vector. 17. Find the constant a if A = (ax + 3y + 4z) iˆ + (x 2y + 3z) jˆ + (3x + 2y z) kˆ is solenoidal. [Ans. : a = 3] 18. Find the constant a if A = (x + 3y2) iˆ + (2y + 2z2) jˆ + (x2 + az) kˆ is solenoidal. [Ans. : a = 3] 19. Find the constants a, b, c if A = (axy + bz2) iˆ + (3x2 cz) jˆ + (3xz2 y) kˆ is irrotational. [Ans. : a = 6, b = 1, c = 1] 20. Find the directional derivative of ( f ) at the point (1, 2, 1) in the direction of the normal to the surface xy2z = 3x + z2, where f = 2x3y2z4.

9.14 PROPERTIES OF GRADIENT, DIVERGENCE AND CURL 9.14.1 Sum and Difference The gradient, divergence and curl are distributive with respect to the sum and difference of the functions. If f, g are scalars and A and B are vectors, then (i) (ii) (iii)

(f

g)

(A (A

B) B)

f

g A)

( (

A)

B)

( (

B)

Vector Calculus

9.61

∂ ∂ ∂ Proof: (i) ∇ ( f ± g ) = iˆ ( f ± g ) + ˆj ( f ± g ) + kˆ ( f ± g ) ∂x ∂y ∂z ⎛ ∂f ∂f ˆ ∂f ⎞ ⎛ ˆ ∂g ^ˆ ∂g ˆ ∂g ⎞ = ⎜ iˆ + jˆ +k ± i +j +k ⎝ ∂x ∂y ∂z ⎟⎠ ⎜⎝ ∂x ∂y ∂z ⎟⎠ = ∇f ± ∇g (ii) Let A = A1 iˆ + A2 jˆ + A3 kˆ, B = B1 iˆ + B2 jˆ + B3 kˆ

(A

B)

( A1 =

x

B1 )iˆ ( A2

( A1 ± B1 ) +

y

A

±

B B1 B + 2+ 3 x y z

B

(iii) ∇ × ( A ± B ) = ∇ × ( A ± B )

= iˆ ×

∂ ( ∂ ∂ A ± B ) + jˆ × ( A ± B ) + kˆ × ( A ± B ) ∂x ∂y ∂z iˆ

x



(

A)

(A

A x

B)

B x

B x



(

A x



B)

9.14.2 Products If f, g are scalars and A and B are vectors, then (i) (ii)

(iii)

(iv)

( f g)

f g

g f or grad ( f g) = f (grad g) + g (grad f )

( A B) ( B ) A ( A ) B B ( or grad ( A B ) ( B ) A ( A ) B

A) B

( f A) f ( A) ( f ) A or div ( f A) = f ( div A) + (grad f ) ⋅ A ( A B ) B ( A) A ( B ) or div ( A B ) B curl A A curl B

( ( curl A)

B)

A

A

B3 )kˆ

( A2 ± B2 ) + z ( A3 ± B3 )

A A1 A + 2+ 3 x y z

=

B2 ) ˆj ( A3

( curl B )

9.62

(v)

Engineering Mathematics

( f A) f ( A) ( f ) A or curl ( f A ) f ( curl A ) (grad f ) ( A B) ( B

(vi)

or curl ( A B )

)A

B(

(B

A)

)A

A

(A )B

B ( div A )

A(

(A )B

B) A ( div B ) .

Proof: (i)

( fg )



x

ˆj

( fg )

g iˆ f x g f iˆ x f g

( A B ) iˆ

(ii)

y g

( fg ) kˆ

z

( fg )

f x

g f iˆ x

g kˆ z

f g iˆ x

ˆj g y

f g iˆ x ˆj f y

f kˆ z

g f

x

( A B ) ˆj



x

y

( A B ) kˆ

( A B)

iˆ A

B x

iˆ A

iˆ B

B x A x

z

( A B) A B x ... (1)

Consider, A



B x

B ˆ i x

A

( A iˆ )

⎡∵ a × ( b × c ) = ( a ⋅ c ) b − ( a ⋅ b ) c ⎤ ⎣ ⎦

B x

( )

⎛ ^ ∂B ⎞ ^⎛ ^ ∂B ∂B ⎞ i ⎜⎜ A ⋅ ⎟⎟ = A × ⎜⎜ i × ⎟⎟ + A ⋅ i ∂x ∂x ⎠ ⎝ ⎝ ∂x ⎠ Similarly, interchanging A and B, iˆ B

A x

B



A x

( B iˆ )

A x

Vector Calculus

9.63

Substituting in Eq. (1),

( A B)

B x



A

B x



A

(

A

B) B

(

A)

( f A)



ˆj

x iˆ

x

(

( A B)

(iv) = ∑i

^



A x

f

f) kˆ

y

( A iˆ )

B x

( A ) B (B B x

( B iˆ )

A x

( B iˆ )

A x

)A

A1iˆ

B x

B A2 ˆj y

B A3 kˆ z

f

(

f x

A

f A iˆ x

(A ˆj

x

A x

( A iˆ )



A x

A)

A x

( f A)

z

( f A)

f iˆ

f



y



B



B



(iii)

B x

( A iˆ )



f

A)

(

A x

A

f iˆ x

f A)

( A B)

z

⎞ ∂ ( ∂B ∂ A ^ ∂ ^ ⎛ ⋅ A × B ) = ∑ i ⋅ ( A × B ) = ∑ i ⋅ ⎜⎜ A × + × B ⎟⎟ ∂x ∂x ∂x ∂x ⎝ ⎠

⎛ ⎞ ⎛ ∂A ∂B ⎞ = ∑ iˆ ⋅ ⎜⎜ A × × B ⎟⎟ ⎟⎟ + ∑ iˆ ⋅ ⎜⎜ ∂x ⎠ ⎝ ⎠ ⎝ ∂x ∂B ∂A ⋅B = ∑ iˆ × A ⋅ + ∑ iˆ × ∂x ∂x = −∑ iˆ ×

∂B ∂A ⋅ A + ∑ iˆ × ⋅B ∂x ∂x

⎡∵ a ⋅ b × c = a × b ⋅ c ⎤ ⎣ ⎦ ⎡ ⎤ ∂B in scalar triple product.⎥ ⎢ Interchanging A annd ∂x ⎣ ⎦

= − ( ∇ × B ) ⋅ A + ( ∇ × A) ⋅ B = B ⋅ ( ∇ × A) − A ⋅ ( ∇ × B )

(v)

( f A)



x iˆ

ˆj

x

y



( f A)

( f A)

z iˆ

f

A x

f A x

9.64

Engineering Mathematics

f iˆ

A x



A x

f f (vi)

( A B)

(

f A x



f iˆ x

A

A) ( f ) A iˆ

ˆj

x iˆ



y

( A B)

z

( A B)

x



( A B)

x

⎞ ⎞ ∂B ∂ A ∂B ⎞ ^ ⎛ ^ ⎛ ^ ⎛ ∂A = ∑ i × ⎜⎜ A × + × B ⎟⎟ = ∑ i × ⎜⎜ A × × B ⎟⎟ ⎟⎟ + ∑ i × ⎜⎜ ∂x ∂x ∂x ⎠ ⎠ ⎝ ⎠ ⎝ ⎝ ∂x B A x



( iˆ A)

B x

( iˆ B )

A x

(b

c)

∵a



A B x

(a c) b (a b) c

⎛ ∂B ⎞ ⎛ ∂A⎞ ∂B ∂A = A∑ ⎜⎜ iˆ ⋅ + ∑ ( B ⋅ iˆ ) ⎟⎟ − B ∑ ⎜⎜ iˆ ⋅ ⎟⎟ − ∑ ( A ⋅ iˆ ) ∂ ∂x ∂ x ∂ x x ⎝ ⎠ ⎝ ⎠ = A ( ∇ ⋅ B ) − B ( ∇ ⋅ A) − ( A ⋅ ∇ ) B + ( B ⋅ ∇ ) A = ( B ⋅ ∇ ) A − B ( ∇ ⋅ A) − ( A ⋅ ∇ ) B + A ( ∇ ⋅ B )

9.15 SECOND ORDER DIFFERENTIAL OPERATOR It is a two fold application of the operator are given below.

(i) Laplacian Operator iˆ = =

ˆj

x

Div (grad f ) =

2

y



f + x y

x 2

f

x

2

2

f

+

2

f

y

2

f

+

. Some second order differential operators

f iˆ x

z

f + y z 2

f

z

2

ˆj f y

x

f)

f kˆ z

f z

2

=

.(

2

2

+

y

2

2

+

z2

f

Vector Calculus

9.65

Thus, the scalar differential operator (read as “nabla squared” or “delta”) 2

2

2

2

x

2

y

2

z2

is known as Laplacian operator. 2

2

f

x

2

f

2

f

y

2

2

f

z

2

is known as Laplacian equation. (ii)

f = curl grad f

⎛ ∂f = ∇ × ⎜ iˆ + ⎝ ∂x ˆj iˆ ∂ ∂x ∂f ∂x

=

∂ ∂y ∂f ∂y

ˆj ∂f + kˆ ∂f ⎞⎟ ∂y ∂z ⎠ kˆ ∂ ∂z ∂f ∂z

⎛ ∂2 f ∂2 f ⎞ − = iˆ ⎜ ⎟− ⎝ ∂y ∂z ∂z ∂y ⎠ Hence, curl grad f = (iii)

(

×

f = 0.

A)

div curl A A = A1iˆ + A2 jˆ + A3 kˆ

Let

iˆ A

(

A)

y A2

z A3

x

A3 y

A2 z

A3 x y

(



ˆj

x A1

2

Hence,

2 2 2 2 ˆj ⎛⎜ ∂ f − ∂ f ⎞⎟ + kˆ ⎛⎜ ∂ f − ∂ f ⎞⎟ = 0 ⎝ ∂x ∂y ∂y ∂x ⎠ ⎝ ∂x ∂z ∂z ∂x ⎠

2

A2 x z

A3 y



y 2

A3 y x

A2 z

A3 x 2

A1 y z

ˆj

A1 z

A3 x

A1 z

A2 x

z 2

A2 z x



A1 y

2

A1 z y

0

A ) = div curl A = 0.

Example 1: If r = xiˆ + yjˆ + z kˆ, show that div ( r n r ) = ( n + 3 ) r n . Solution: r n is a scalar and r is a vector. We know that div

( f A)

f

(

A) ( f ) A

A2 x

A1 y

9.66

Engineering Mathematics

div ( r n r ) = r n ( ∇ ⋅ r ) + (∇r n ) ⋅ r ⎡⎛ ∂ ⎤ ⎛ ∂r n ∂ ∂r n ˆ ∂r n ⎞ ∂ ⎞ = r n ⎢⎜ iˆ + jˆ + kˆ ⎟ ⋅ ( xiˆ + yjˆ + zkˆ ) ⎥ + ⎜ iˆ + ˆj +k ⎟⋅r ∂y ∂y ∂z ⎠ ∂z ⎠ ⎣⎝ ∂x ⎦ ⎝ ∂x ∂r ∂r ∂r ⎤ ⎡ = r n (1 + 1 + 1) + ⎢iˆ( nr n −1 ) + ˆj ( nr n −1 ) + kˆ( nr n −1 ) ⎥ ⋅ r ∂ x ∂ y ∂z ⎦ ⎣

r = xiˆ + yjˆ + zkˆ

r2 = x2 + y2 + z2 r x r y r z = , = , = x r y r z r x Hence, div ( r n r ) = 3r n + nr n −1 ⎛⎜ iˆ + ⎝ r

ˆj y + kˆ z ⎞⎟ ⋅ ( xiˆ + yjˆ + zkˆ ) r r⎠

2 ⎛ x2 + y 2 + z 2 ⎞ n n −1 ⎛ r ⎞ n n = 3r n + nr n −1 ⎜ ⎟ = 3r + nr ⎜ ⎟ = 3r + nr r ⎝ ⎠ ⎝ r ⎠

Hence, div ( r n r ) = (n + 3)r n . Example 2: Find the value of n for which the vector r n r is solenoidal, where r = xiˆ + yjˆ + z kˆ. Solution: If F = r n r is solenoidal, then rn r = 0

… (1)

As proved in Ex. 1., r n r = (n + 3) r n

Substituting in Eq. (1), n (n + 3) r = 0 n= 3

Example 3: Prove that Div (grad r n ) = n (n + 1) rn – 2 , where r = xiˆ + yjˆ + z kˆ. Solution: Div (grad r n )

( rn ) rn iˆ x nr n

1

nr n

1

n ˆj r y

rˆ i nr n x

rn kˆ z 1

r ˆ j nr n y

1

xˆ y z i nr n 1 ˆj nr n 1 kˆ r r r

r ˆ k z

Vector Calculus

nr n n

r

1

2

= n rn

2

( xiˆ + yjˆ + zkˆ ) r

n 2

n rn

9.67

r

(

r ) ( rn 2 ) r iˆ

ˆj

x

rn 2 iˆ x



y

z

n 2 ˆj r y

( xiˆ

yjˆ zkˆ )

rn 2 kˆ z

( xiˆ

yjˆ zkˆ )

⎡ ∂r ∂r ˆ ⎧ = n ⎢ r n − 2 (1 + 1 + 1) + ⎨( n − 2) r n − 3 iˆ + ( n − 2) r n − 3 j ∂x ∂y ⎩ ⎣ + ( n − 2) r n − 3

}

⎤ ∂r ˆ k ⋅ ( xiˆ + yjˆ + zkˆ )⎥ ∂z ⎦

r = xiˆ + yjˆ + zkˆ

r2 = x2 + y2 + z2 r x r y r z = , = , = x r y r z r Hence, ∇ ⋅ (∇r n ) = n ⎡⎢3r n − 2 + ( n − 2)r n −3 ⎛⎜ x iˆ + y jˆ + z kˆ ⎞⎟ ⋅ ( xiˆ + yjˆ + zkˆ ) ⎤⎥ r r ⎠ ⎝r ⎣ ⎦ n 2 n 4 2 2 = n [3r + (n 2) r (x + y + z2)] = n [3r n 2 + (n 2) r n 4 · r2] = n [3rn 2 + (n 2) r n 2] = n (n + 1) rn 2 Example 4: If e and x are two scalar point functions, show that 2

(ex ) = e

2

x+2

x+x

e. 2

Solution:

2

(

)

(

)=

x

2

2

e.

2

(

)

y

2

2

(

)

z2

(

)

2

Consider,

x2

=

x

x

x

(

x

) +

x 2

=

x

x

+

x2

2

+

x

x

+

x2

... (1)

9.68

Engineering Mathematics

2

Similarly,

y2

2

(

)

(

)

y

y2

y

2

and

z2

2

y

y2

y

2

z

2

z2

z

z

z2

z

Substituting in Eq. (1),

⎛ ∂φ ∂ψ ∂φ ∂ψ ∂φ ∂ψ ⎞ ∇ 2 (φψ ) = 2 ⎜ + + ⎟ ⎝ ∂x ∂x ∂y ∂y ∂z ∂z ⎠ ⎛ ∂ 2ψ ∂ 2ψ ∂ 2ψ ⎞ ⎛ ∂ 2φ ∂ 2φ ∂ 2φ ⎞ + φ ⎜ 2 + 2 + 2 ⎟ +ψ ⎜ 2 + 2 + 2 ⎟ ∂z ⎠ ∂y ∂z ⎠ ∂y ⎝ ∂x ⎝ ∂x 2 2 = 2∇φ ⋅ ∇ψ + φ∇ ψ + ψ ∇ φ ∇ 2 (φψ ) = φ∇ 2ψ + 2∇φ ⋅∇ψ + ψ∇ 2φ r

2

Example 5: Prove that

2r 4 , where r = xî + y + z kˆ.

r2

⎛ ⎞ Solution: ∇ ⋅ ⎜ r ⎟ = ∇ ⋅ ( r −2 r ) ⎜ r2 ⎟ ⎝ ⎠

= r −2 ( ∇ ⋅ r ) + (∇r −2 ) ⋅ r ⎡⎛ ∂ ∂ ˆ ∂ ⎞ ˆ ˆ ˆ ⎤ = r −2 ⎢⎜ iˆ + jˆ +k ⎟ ⋅ ( xi + yj + zk ) ⎥ x y ∂ z ∂ ∂ ⎠ ⎦ ⎣⎝ ⎛ ∂r −2 ˆ ∂r −2 ˆ ∂r −2 + ⎜ iˆ +j +k ∂y ∂z ⎝ ∂x

⎞ ⎟⋅r ⎠

⎡ ∂r ∂r ˆ ⎧ = r − 2 ⎢(1 + 1 + 1) + ⎨( − 2 r − 3 ) iˆ + ( − 2 r − 3 ) j ∂ x ∂ y ⎩ ⎣ + (− 2r − 3)

}

⎤ ∂r ˆ k ⋅ ( xiˆ + yjˆ + zkˆ ) ⎥ ∂z ⎦

r = xiˆ + yjˆ + zkˆ

r2 = x2 + y2 + z2 r x r y r z = , = , = x r y r z r Hence,

r r2

r

2

3 2r

3

r

2

3 2r

3

xˆ i r

yˆ j r

z ˆ k r

( x2 + y 2 + z 2 ) r

( xiˆ

y ˆj zkˆ )

Vector Calculus 2

3r 3r r r2

2

2r 4 r2 2r 2 = r

2

2

x

2

(r 2 )

2

=

9.69

2

2

(r 2 ) +

y

2

(r 2 ) +

2

z

2

(r 2 )

Now, ∂ ⎡ ∂r ⎤ ∂ 2 r −2 ( −2r −3 ) ⎥ = 2 ⎢ ∂x ⎣ ∂x ⎦ ∂x x⎤ ∂ ⎡ ∂ = ( −2r −3 ) ⎥ = −2 (r −4 ⋅ x) ⎢ ∂x ⎣ ∂x r⎦ ∂ r x ⎛ ⎞ ⎛ ⎞ = −2 ⎜ −4r −5 x + r −4 ⎟ = −2 ⎜ −4r −5 x + r −4 ⎟ ⎝ ⎠ ⎝ ⎠ ∂x r

2r

4

2r

4

2r

4

4x2 r2

1

4 y2 r2

1

4z2 r2

1

Similarly, 2

r 2 y2

2

r 2 z2

2

r r2

2

r 2 x2

2r

4

2

r 2 y2

2

4( x 2

y2

r 2 z2 z2 )

r2

= 2r –4 Hence,

⎡ ⎛ r ∇ 2 ⎢∇⋅ ⎜⎜ 2 ⎢⎣ ⎝ r

⎞⎤ −4 ⎟⎟ ⎥ = 2r ⎠ ⎥⎦ r r

Example 6: Prove that

Solution:

r r

(r r ) 1

2 r3

r.

3

2r

4

4r 2 r2

3

9.70

Engineering Mathematics

(

)

= r −1 ∇ ⋅ r + (∇r −1 ) ⋅ r ⎤ ⎛ ∂r −1 ⎡⎛ ∂ ∂ ∂r −1 ˆ ∂r −1 ⎞ ∂ ⎞ = r −1 ⎢⎜ iˆ + jˆ + ˆj + kˆ ⎟ ⋅ ( xiˆ + yjˆ + zkˆ ) ⎥ + ⎜ iˆ +k ⎟⋅r ∂y ∂y ∂z ⎠ ∂z ⎠ ⎦ ⎝ ∂x ⎣⎝ ∂x ⎛ ∂r ˆ −2 ∂r ˆ ⎞ ∂r j−r k ⎟⋅r = 3r −1 + ⎜ −r −2 iˆ − r −2 y x ∂ ∂z ⎠ ∂ ⎝

r = xiˆ + yjˆ + zkˆ r2 = x2 + y2 + z2

r x r y r z = , = , = x r y r z r Hence,

r r

r r

3r

1

r

2

3r

1

r

2

3r

1

r

2



x

r r

2

2r

2

= −2r −2

yˆ j r r

2r ˆj

rˆ i 2r x xˆ i r

z ˆ k r

3r

r2 r

(2r 1 )

2r

=−

xˆ i r

y

1

r

r 2

1

(2r 1 ) kˆ r ˆ j 2r y

2

y ˆ j r

(r r ) r

z 2

(2r 1 ) r ˆ k z

z ˆ k r

r r

2 r. r3

Example 7: Show that E = Solution: Curl E

r r2

E r r2

(r 2 r )

is irrotational.

Vector Calculus

( f A)

We know that,

f

(

A) ( f ) A

(r 2 r ) r 2 (

curl E r

2

r

2

9.71



x

r) ( r 2) r r 2 iˆ x

( xiˆ )

( iˆ iˆ )

( 2r 3 )

r

rˆ i x

r

r = xiˆ + yjˆ + zkˆ r2 = x2 + y2 + z2

r x r y r z = , = , = x r y r z r Hence,

(r r ) 2

0 2r 2r

3

2r

4

xˆ i r

3

r

( xiˆ + yjˆ + zkˆ ) (r r )

r

r

=0 Hence, E is irrotational.

(a

Example 8: Prove that

r)

2a , where a is a constant vector.

Solution: Let a = a1 iˆ + a2 jˆ + a3 kˆ r = xiˆ + yjˆ + zkˆ

a r

iˆ a1

ˆj a2

kˆ a3

x

y

z

iˆ (a2 z

a3 y)

jˆ (a1 z



(a

r)

ˆj

a3 x) + kˆ (a1 y kˆ

x y z a2 z a3 y a3 x a1 z a1 y a2 x iˆ (a1 + a1) jˆ ( a2 2(a iˆ + a jˆ + a kˆ) 1

2a

2

3

a2) + kˆ (a3 + a3)

a2 x)

9.72

Engineering Mathematics

r a r rn

Solution:

(2 n)a

a r

Example 9: Prove that

n

r

n(a r )r r n+ 2

n

.

( r n A) , where a r A, say

We know that,

( f A)

f

(

( r n A) r n ( r

n

A) ( f ) A A) ( r n ) A

(a

r iˆ

r)

n

x

ˆj r

n

y

r kˆ

n

A

z

As proved in Ex. 8

(a

r)

2a

( r n A) r n ( 2a ) ( nr

n 1

)

rˆ i x

r ˆ j y

r ˆ k z

A

r x r y r z = , = , = x r y r z r

As proved earlier,

y z ⎞ ⎛x Hence, ∇ × ( r − n A ) = 2ar − n − nr − n −1 ⎜ iˆ + ˆj + kˆ ⎟ × A r r r ⎝ ⎠ 2a n − r n r n +1 n 2a = n − n+2 r r =

=

2a rn

n r

2a rn

na rn

n+2

r ( × a×r) r ⎡⎣( r ⋅ r ) a − ( r ⋅ a ) r ⎤⎦ ⎡∵ a × ( b × c ) = ( a ⋅ c ) b − ( a ⋅ b ) c ⎤ ⎣ ⎦ r2 a

(r a) r

n(a r ) r r n+2

(2 n)a n ( a r ) r . + rn r n+2

Example 10: If a is a constant vector, show that a Solution: Let a = a1 iˆ + a2 jˆ + a3 kˆ r = x1 iˆ + x2 jˆ + x3 kˆ

(

r)

(a r ) (a ) r.

Vector Calculus

r



ˆj



x r1

y r2

z r3

r3 y



a

(

r2 z

r3 x

ˆj

iˆ a1

r)

r1 z

r2 z

r2 x

iˆ a2

r2 x

r1 y

a3

r1 z

r3 x

kˆ a1

r1 z

r3 x

a2

r3 y

r2 z

r iˆ a2 2 x

a3

r3 x

r kˆ a1 1 z

a2

r2 z

ˆj a1 r2 x

a3

r2 z

r1 y

kˆ a3 r3 x

r1 z

r2 x



ˆj a2

r3 y

9.73

r1 x

a1 a3

a1 r3 z

a3

r kˆ a1 3 x

r1 y ˆj a1

r1 x

r2 x

ˆj a1 r1 y r3 z a2

r iˆ a2 1 y

r1 y

a3 a3

a3

r3 y

a2

r3 y

r2 y

r2 z

a2

r1 z

r3 y

⎛ ∂ ∂ ∂ ⎞ = ⎜ iˆ + ˆj + kˆ ⎟ ( a1r1 + a2 r2 + a3 r3 ) ∂ ∂ ∂ x y z⎠ ⎝ ⎛ ∂ ∂ ∂ ⎞ − ⎜ a1 + a2 + a3 ⎟ ( r1iˆ + r2 ˆj + r3 kˆ) ∂ ∂ x y ∂ z⎠ ⎝

= ∇ ( a ⋅ r ) − ( a ⋅∇ ) r

Example 11: If a is a constant vector such that a = a , prove that

(a r ) a

a2 .

Solution: Let a = a1 iˆ + a2 jˆ + a3 kˆ r = xiˆ + yjˆ + zkˆ

r2 y

9.74

Engineering Mathematics

( f A)

We know that,

(a r ) a Since a is constant,

(a r )

iˆ = iˆ

x

A) ( f ) A

( a r )(

a x

(

f

a)

(a r )

a

0

(a r )

ˆj

y

(a r )



(a1 x + a2 y + a3 z ) + ˆj

y

z

(a r )

(a1 x + a2 y + a3 z ) + kˆ

z

(a1 x + a2 y + a3 z )

= a1iˆ + a2 ˆj + a3 kˆ =a Hence,

(a r ) a

0 a a = a2 .

Example 12: If F

( a r ) r where a

is a constant vector, find curl F and

prove that it is perpendicular to a . Solution: Curl F We know that,

(a r ) r

F

( f A)

(

f

A) ( f ) A

(a r ) r

Curl F

( a r )(

Now,

r

r)



ˆj



x x

y y

z z

iˆ (0 0

0)

jˆ (0

As proved in Ex. 11

(a r ) (a r ) r

a 0 a r a r

∇ × ⎡⎣( a ⋅ r ) r ⎤⎦ ⋅ a = ( a × r ) ⋅ a = 0

(a r )

0) + kˆ (0

r

0)

Vector Calculus

(a r ) r

Hence,

is perpendicular to a. a r r

Example 13: Prove that

a r r

Solution:

9.75

r

1

(a

0, where a is a constant vector.

r)

∇ ⋅ ( f A ) = f ( ∇ ⋅ A ) + ( ∇f ) ⋅ A

We know that,

∇ ⋅ ⎡⎣ r −1 ( a × r ) ⎤⎦ = r −1 ⎡⎣∇ ⋅ ( a × r ) ⎤⎦ + (∇r −1 ) ⋅ ( a × r ) ⎛ ∂r −1 ˆ ∂r −1 ˆ ∂r −1 ⎞ = r −1 ⎡⎣∇ ⋅ ( a × r ) ⎤⎦ + ⎜ iˆ +j +k ⎟ ⋅(a × r ) ∂y ∂z ⎠ ⎝ ∂x ∇ ⋅ ( A × B ) = B ⋅ ( ∇ × A) − A ⋅ ( ∇ × B ) ∇ ⋅ ( a × r ) = r ⋅ (∇ × a ) − a ⋅ (∇ × r )

Since a is constant, Also,

r

a

0.

0 as proved in Ex. 12. ∇ ⋅(a × r ) = 0

⎛ ∂r ∂r ˆ −2 ∂r ˆ ⎞ ( Hence, ∇ ⋅ ⎡⎣ r −1 ( a × r ) ⎤⎦ = 0 + ⎜ −r −2 iˆ − r −2 j−r k ⎟⋅ a×r) ∂x ∂y ∂z ⎠ ⎝

⎛ xiˆ + yjˆ + zkˆ ⎞ −3 = 0 − r −2 ⎜ ⎟⎟ ⋅ ( a × r ) = −r ⎡⎣ r ⋅ ( a × r ) ⎤⎦ = 0 ⎜ r ⎝ ⎠

(r

Example 14: Prove that curl

a) b

Solution: We know that, ( r a ) b

b a , where a and b are constants.

(r b) a (a b) r

Let r ⋅ b = f , say and a ⋅ b = g , say

(r curl

(r

a) b

f a gr

a) b

(r

( f a) We know that,

(r

(fa

a) b

gr )

( gr )

( f A)

f

(

A) ( f ) A

a) b

f

(

a) ( f ) a g (

r ) ( g) r

9.76

Engineering Mathematics

Since a is constant,

0. Also

a

r

0

∇ × ⎡⎣( r × a ) × b ⎤⎦ = ⎡⎣∇ ( r ⋅ b )⎤⎦ × a − ⎡⎣∇ ( a ⋅ b )⎤⎦ × r

(r b)

[Substituting f and g] ∵ a and b are constant

a 0

Let b = b1iˆ + b2 jˆ + b3 kˆ, r = xiˆ + yjˆ + zkˆ

(r b)



ˆj

x



y

z

(b1 x b2 y b3 z )

= b1iˆ + b2 ˆj + b3 kˆ = b Substituting in Eq. (1),

(r Hence, curl

(r

a) b

1 rn



=

We know that, n

(r

n 1

r

ˆj

x

r n+1

z

)

rˆ i ( nr x

( nr

n 1

)

xˆ i r

yˆ j r

.

n

r

n 1

n 1

)

r ˆ j ( nr y

n 1

)

r ˆ k z

z ˆ k r

n −n r = − n+ 2 r n +1 r r r

n

r

r

n

(r

( f A)

f

r)

n ( n 2)

n

r kˆ

y

1

( nr

1 rn

r

b a

b a.

Example 15: Prove that

Solution:

a) b

n n n n

n 1

(

1 r

n+2

n +1

3 r n +1 3 r

n +1

r

n +1

3

r

r) A) ( f ) A

(

r) ( r iˆ

x

(n 1)r

ˆj n 2

(n 1) ˆ x i r r n+2

n 1

y

) r kˆ

r iˆ x ˆj y r

z

(r

ˆj r y z kˆ r

n 1

)

r kˆ z r

r

[∵ r

r

3]

Vector Calculus

9.77

⎡ 3 (n + 1) r ⋅ r ⎤ = − n ⎢ n +1 − n + 2 ⎥ r ⎦ ⎣r r ⎡ 3 (n + 1)r 2 ⎤ = − n ⎢ n +1 − n + 2 ⎥ ⎣r r r ⎦ n( 2 − n) =− r n +1 n(n − 2) = . r n +1 Example 16: Prove that

(a

log r

Solution:

)

2

log r

(a r ) r r4 iˆ

r

log r

r2

and hence, show that

where a is a constant vector. ˆj

x



y

z

log r

1 rˆ 1 r ˆ 1 r ˆ i j k r x r y r z 1 xˆ i r r

y ˆ j r

z ˆ k r

r r2

(a

log r )

a

r r2

a r r2 Let a × r = A,

(a We know that,

( f A)

log r ) f

(

a r r2

( r 2 A)

A) ( f ) A

∇ × ( r −2 A ) = r −2 ( ∇ × A ) + ( ∇r −2 ) × A = r −2 ⎡⎣∇ × ( a × r ) ⎤⎦ + ( ∇r −2 ) × ( a × r ) = r −2 ⎡⎣( r ⋅ ∇ ) a − r ( ∇ ⋅ a ) − ( a ⋅ ∇ ) r + a ( ∇ ⋅ r ) ⎤⎦ ⎡⎛ ∂ ∂ ∂ ⎞ ⎤ + ⎢⎜ iˆ + ˆj + kˆ ⎟ r −2 ⎥ × ( a × r ) x y z⎠ ⎦ ∂ ∂ ∂ ⎣⎝

9.78

Engineering Mathematics

Since a is a constant vector,

(

a

0, r

)a

0.

a = a1 iˆ + a2 jˆ + a3 kˆ r = xiˆ + yjˆ + zkˆ

Let

(a

log r )

As proved earlier,

r

2

(a

)r

a(

r)

r ( 2r 3 ) iˆ x

ˆj r y

r kˆ z

(a

r)

r x r y r z = , = , = x r y r z r

⎡ ⎛ ∂r ⎤ ∂r ∂r ⎞ ∇ × ( a × ∇ log r ) = r −2 ⎢ − ⎜⎜ a1 + a2 + a3 ⎟⎟ + a(3) ⎥ ∂y ∂z ⎠ ⎢⎣ ⎝ ∂x ⎥⎦ y z ⎞ ⎛ x + ( −2r −3 ) ⎜ iˆ + jˆ + kˆ ⎟ × ( a × r ) r r ⎠ ⎝r r = r −2 ⎡⎣ − ( a1iˆ + a2 jˆ + a3 kˆ ) + 3a ⎤⎦ + ( −2r −3 ) × ( a × r ) r 2 −2 ( = r −a + 3a ) − 4 ⎡⎣( r ⋅ r ) a − ( r ⋅ a ) r ⎤⎦ r

2a r2

2 2 r a r4

2a r2

2a r2

Example 17: Calculate (1, 1, 0).

2

(r a) r

2(a r ) r r4

2(a r ) r r4

f when f = 3x2z – y2z3 + 4x3y + 2x – 3y – 5 at the point

⎛ ∂2 ∂2 ∂2 ⎞ Solution: ∇ 2 f = ⎜ 2 + 2 + 2 ⎟ (3x 2 z − y 2 z 3 + 4 x 3 y + 2 x − 3 y − 5) ∂y ∂z ⎠ ⎝ ∂x ∂f ∂ = (3 x 2 z − y 2 z 3 + 4 x3 y + 2 x − 3 y − 5) = 6 xz + 12 x 2 y + 2 ∂x ∂x ∂2 f = 6 z + 24 xy ∂x 2 ∂f ∂ = (3 x 2 z − y 2 z 3 + 4 x3 y + 2 x − 3 y − 5) ∂y ∂y = −2 yz 3 + 4 x3 − 3 ∂2 f = −2 z 3 ∂y 2

... (1)

Vector Calculus

9.79

∂f ∂ = (3 x 2 z − y 2 z 3 + 4 x3 y + 2 x − 3 y − 5) = 3 x 2 − 3 y 2 z 2 ∂z ∂z ∂2 f = − 6 y 2 z. ∂z 2 Substituting in Eq. (1), 2

6 z 24 xy 2 z 3 6 y 2 z

f

At the point (1, 1, 0), ∇ 2 f = 24 Example 18: Prove that

2

d2 f

f (r )

dr

2

2 df where r = xiˆ + yjˆ + z kˆ. , r dr

Solution: ∇ 2 f = ∇ ⋅∇f

∇f = iˆ

∂f ( r ) ˆ ∂f ( r ) ˆ ∂f ( r ) +j +k ∂x ∂y ∂z

∂r ⎤ ∂r ⎤ ∂r ⎤ ⎡ ⎡ ⎡ = ⎢ f ′( r ) ⎥ iˆ + ⎢ f ′( r ) ⎥ ˆj + ⎢ f ′( r ) ⎥ kˆ ∂y ⎦ ∂z ⎦ ∂x ⎦ ⎣ ⎣ ⎣ r x r y r z = , = , = x r y r z r

As proved earlier,

f

xˆ i r

f (r )

yˆ j r

z ˆ k r

r f (r ) r = f (r ) = r r 2

We know that, 2

f

f

( f A)

f (r ) r r

Now,

f

(

A) ( f ) A

f (r ) ( r iˆ

r



f (r ) r r

f

x

f (r ) r

r) ˆj

y



z

( xiˆ

r



f (r ) is a scalar function r

yjˆ zkˆ ) = 3

∂ ⎡ f ′( r ) ⎤ ˆ ∂ ⎡ f ′( r ) ⎤ ˆ f ′( r ) ∂ ⎡ f ′( r ) ⎤ ˆ = ⎢ k i + j + ⎢ ⎥ ⎢ ⎥ ∂x ⎣ r ⎦ ∂y ⎣ r ⎦ ∂z ⎣ r ⎥⎦ r d ⎡ f ′( r ) ⎤ ∂r ˆ d ⎡ f ′( r ) ⎤ ∂r ˆ d ⎡ f ′( r ) ⎤ ∂r ˆ k i+ ⎢ j+ ⎢ dr ⎢⎣ r ⎥⎦ ∂x dr ⎣ r ⎥⎦ ∂y dr ⎣ r ⎥⎦ ∂z ⎡ f ′′( r ) f ′( r ) ⎤ ⎛ x ˆ y ˆ z ˆ ⎞ ⎡ f ′′( r ) f ′( r ) ⎤ =⎢ − 2 ⎥⎜ i + j + k ⎟ = ⎢ 2 − 3 ⎥r r r ⎠ ⎣ r r ⎦ ⎣ r r ⎦⎝ r =

9.80

Hence,

Engineering Mathematics

2

f

f (r ) f (r ) f (r ) (3) r r r r2 r3 f (r ) f (r ) 2 3 f (r ) r r r2 r3 d 2 f 2 df 3 f ′(r ) 2 f ′(r ) = + f ′′ (r ) − = f ′′ (r ) + f ′ (r ) = 2 + r r r r dr dr

Exercise 9.5 1. Evaluate div ( A r ) if curl A = 0, r = xiˆ + yjˆ + zkˆ. [Ans. : 0] 2. If r1 and r2 are vectors joining the P1 (x1, y1, z1) and P2 (x2, y2, z2) to a variable point P (x, y, z), show that

(

curl r1 r2

)

2 ( r1 r2 ) .

10. Is A =

a

r rn

a solenoidal vector,

where a is constant vector? [Ans. : Yes]

( )

3. Prove that

(r a) b

9. If f1 and f2 are scalar functions, then prove that, ∇ × ( 1 ∇ 2 ) = ∇ 1 × ∇ 2.

11. Prove that div a r a

b a, where

r = xiˆ + yjˆ + zkˆ and a, b are constant vectors. 4. If a is a constant vector, prove that r ( a r ) 3r a. 5. Prove that f (r ) 1 d 2 r r f (r ) r r 2 dr Hence, or otherwise prove that div ( r n r ) = (n + 3)r n . 6. Prove that log r 1 r (1 2 log r ). r r

(

) = ∇ 1 ×∇ 2 = and deduce that 1)

2

2

(f f)

0.

14. Prove that

( 1 2 2 1 ) 0, where f1 and f2 are scalar functions.

a ) = div f

where a is a constant unit vector. 8. Find f (r), so that the vector f (r) r is both solenoidal and irrotational. Ans. : f (r ) =

12. If r is the positive vector of the point (x, y, z) and r is the modulus of r , then prove that rn r is an irrotational vector for any value of n but is solenoidal only if n = 3. 13. If f1 and f2 are scalar functions, then prove that ∇×( 1 ∇

7. Prove that a grad ( f a ) curl ( f

a2 .

c r3

15. Prove that 2 ( fg ) f

2

g 2 g

f

g

2

f,

where f and g are scalar functions. 16. Calculate

2

f where f = 4x2 + 9y2 + z2. [Ans. : 28]

Vector Calculus

9.81

9.16 LINE INTEGRALS The line integral is a simple generalisation of a definite integral

b a

B A

f ( x) dx which is integrated from x = a to

x = b along x-axis. In a line integral, the integration is done along a curve C in space.

B

C C

Let F (r ) be a vector function defined at every point of A Fig. 9.5 a curve C. If r is the position vector of a point P (x, y, z) on the curve C, then the line integral of F (r ) over a curve C is defined by



C

F ( r ) ⋅ d r = ∫ ( F1dx + F2 dy + F3dz ) C

F = F1iˆ + F2 jˆ + F3 kˆ and r = xiˆ + yjˆ + zkˆ

where If the curve C is represented by a parametric representation r (t ) = x(t )iˆ + y (t ) ˆj + z (t )kˆ , then the line integral along the curve C from t = a to t = b is



C

dr dt dt b⎛ dx dy dz ⎞ = ∫ ⎜ F1 + F2 + F3 ⎟ dt a d t d dt ⎠ t ⎝ b

F (r ) ⋅ d r = ∫ F ⋅ a

If C is a closed curve, then the symbol of line integral is replaced by . C C Note: (1) The curve C is called the path of integration, the points r (a ) and r (b) are called initial and terminal points respectively. (2) The direction from A to B along which t increases is called positive direction on C.

9.16.1 Circulation If F is the velocity of a fluid particle and C is a closed curve, then the line integral



C

F ⋅ d r represents the circulation of F around the curve C.

Note: If circulation of F around every closed curve C in the region R is zero, then F is irrotational, i.e., if



C

F ⋅ d r = 0, F is irrotational.

9.16.2 Work done by a force If F is the force acting on a particle moving along the arc AB of the curve C, then the line integral



B A

F ⋅ d r represents the work done in displacing (moving) the particle

from point A to point B.

Engineering Mathematics

9.82

9.16.3 Path Independence of Line Integral Scalar Potential) If F is conservative, i.e., F where f is scalar potential, then the line integral along the curve C from the points A to B is



C

B

F ⋅d r = ∫ ∇ ⋅d r A

B⎛∂ ⎞ ∂ ∂ = ∫ ⎜ dx + dy + dz A ⎝ ∂x ∂y ∂z ⎟⎠ B

=∫ d A

= ( B) − ( A) Thus, line integral depends only on the start and end values and therefore is independent of the path. Hence, for a conservative force field, line integral is independent of the path. Note 1: If F is conservative and curve C is closed, then



C

F ⋅ d r = ( A) − ( A) = 0

A

Note 2: If F is conservative, then F

0

Fig. 9.6

Hence, conservative field is irrotational. Note 3: Work done in moving a particle from points A to B under a conservative force field is work done = f (B) Example 1: Evaluate



C

f (A)

F ⋅ d r along the parabola y2 = x between the points

(0, 0) and (1, 1) where F = x 2 iˆ + xy ˆj .

y

Solution: (i) Let r = xiˆ + yjˆ A (1, 1)

d r = iˆdx + ˆjdy (ii)

F dr

( x 2 iˆ xy ˆj ) (iˆ dx

ˆj dy )

= x 2 dx + xydy (iii) Path of integration, C is the parabola x = y2 dx = 2ydy Substituting in F d r and integrating between the limits x = 0 to x = 1,

x

O (0, 0) y2 = x

Fig. 9.7

Vector Calculus



9.83

1

C

F ⋅ d r = ∫ y 4⋅ 2 y dy + y 2 ⋅ y dy 0

1

= ∫ ( 2 y 5 + y 3 ) dy 0

y6 y4 =2 + 6 4

1

0

1 1 = + 3 4 7 = 12 Example 2: Prove that

C

F d r = 3o , where F = ziˆ + xjˆ + ykˆ and C is the arc

of the curve r = cos t iˆ + sin t ˆj + t kˆ from t = 0 to t = 2o . r = cos t iˆ + sin t ˆj + t kˆ

Solution : (i)

x = cos t , y = sin t , z = t dx = – sin t dt , dy = cos t dt , dz = dt (ii) F d r

ykˆ) (iˆ dx

( ziˆ x ˆj

ˆj dy kˆ dz )

= zdx + xdy + ydz t ( sin t )dt cos t cos t dt sin t dt ( t sin t cos 2 t sin t )dt (iii) Path of integration, C is the arc of the curve r = cos tiˆ + sin t ˆj + t kˆ from t = 0 to t = 2p.

∫ F ⋅dr = ∫ C

2

0

(−t sin t + cos 2 t + sin t )dt

= − t (− cos t ) − (− sin t ) 0 + ∫ 2

2

0

= −(−2 ) + =2 +

t sin 2t + 2 4

(1 + cos 2t ) 2 dt + − cos t 0 2

2

− (cos 2 − cos 0) 0

2 =3 2

Example 3: If F = (2 x - y + 2 z )iˆ + ( x + y - z ) ˆj + (3 x - 2 y - 5 z )kˆ , calculate the circulation of F along the circle in the xy-plane of radius 2 and centre at the origin. Solution: Circulation =



C

(i) Let r = xiˆ + yjˆ + zkˆ d r = iˆdx + ˆjdy + kˆdz

F ⋅dr

Engineering Mathematics

9.84

(ii) F d r

y z ) ˆj (3 x 2 y 5 z )kˆ (iˆ dx

y 2 z )iˆ ( x

(2 x (2 x

y 2 z )dx ( x

y

ˆj dy kˆ dz )

z )dy (3 x 2 y 5 z )dz

(iii) Path of integration is the circle in xy-plane of radius 2 and centre at the origin, i.e., x2 + y2 = 4 and in xy-plane z = 0 Parametric equation of the circle is x = 2 cosq, y = 2 sinq dx = 2 sinq dq, dy = 2 cosq dq For the complete circle, q varies from 0 to 2p . Substituting in F d r and integrating between the limits q = 0 to q = 2p, 2

Circulation

[ (2

0

4 4

2 0

2 cos

sin 2

( 2 cos sin

2 0

=4 +

sin 2 2

1

cos 2

cos sin ) d

d

2

cos 2 4

Example 4: Evaluate

2sin ) (2 cos d ) ]

2sin ) ( 2sin d ) (2 cos

=8 0



C

F ⋅ d r where F = ( x 2 + y 2 )iˆ - 2 xy ˆj and C is the rect-

angle in the xy-plane bounded by y = 0, x = a, y = b, x = 0. y

Solution : (i) Let r = xiˆ + yjˆ d r = iˆ dx + ˆj dy (ii) F d r

( x2

y 2 )iˆ 2 xy ˆj (iˆ dx

( x2

y 2 )dx 2 xy dy

D (0, b)

ˆj dy )

(iii) Path of integration is the rectangle OABD bounded by the four lines y = 0, x = a, y = b, x = 0.

O (0, 0)

B (a, b)

A (a, 0) x

Fig. 9.8



C

F ⋅d r = ∫ F ⋅d r + ∫ F ⋅d r + ∫ F ⋅d r + ∫ OA

AB

BD

DO

F ⋅d r

(a) Along OA : y = 0, dy = 0 x varies from 0 to a. a



OA

a

F ⋅ d r = ∫ x 2 dx = 0

x3 a3 = 3 0 3

(b) Along AB : x = a, dx = 0 y varies from 0 to b.



AB

b

b

0

0

F ⋅ d r = ∫ (−2ay )dy = − ay 2 = −ab 2

… (1)

Vector Calculus (c) Along BD : y = b, x varies from a to 0.

9.85

dy = 0 0

0

F ⋅ d r = ∫ ( x 2 + b 2 )dx =



BD

a

(d) Along DO : x = 0, y varies from b to 0.

⎛ a3 ⎞ x3 + b2 x = − ⎜ + b2 a ⎟ 3 3 ⎝ ⎠ a

dx = 0 0



DO

F ⋅ d r = ∫ 0 dy = 0 b

Substituting in Eq. (1), C

F dr

Example 5: Evaluate

a3 ab 2 3 2ab 2 C

a3 3

b2 a

F d r where F = (3 x 2 + 6 y )iˆ - 14 yzjˆ + 20 xz 2 kˆ and C

is the straight line joining the points (0, 0, 0) to (1, 1, 1). Solution: (i) Let r = xiˆ + yjˆ + zkˆ d r = iˆdx + ˆjdy + kˆdz (ii) F d r

(3 x 2 (3x 2

6 y )iˆ 14 yzjˆ 20 xz 2 kˆ (iˆdx

ˆjdy kˆdz )

6 y )dx 14 yzdy 20 xz 2 dz

(iii) Path of integration is the straight line joining the points A (0, 0, 0) to B (1, 1, 1). Equation of the line AB is x 0 y 0 z 0 = = 0 1 0 1 0 1 x=y=z dx = dy = dz Substituting in F d r and integrating between the limits x = 0 to x = 1, 1



F ⋅ d r = ∫ ⎡⎣(3 x 2 + 6 x)dx − 14 x 2 dx + 20 x 3 dx ⎤⎦ 0 C 1

= ∫ (20 x 3 − 11x 2 + 6 x) dx 0

= 20

x 4 11x 3 6 x 2 − + 4 3 2

1

0

13 = 3 Example 6: Evaluate

C

F d r along the curve x2 + y2 = 1, z = 1 in the positive

direction from (0, 1, 1) to (1, 0, 1), where F = ( yz + 2 x )iˆ + xzjˆ + ( xy + 2 z )kˆ .

Engineering Mathematics

9.86

r = xiˆ + yjˆ + zkˆ

Solution: (i) Let

d r = iˆdx + ˆjdy + kˆdz F ⋅ d r = [ ( yz + 2 x)iˆ + xzjˆ + ( xy + 2 z )kˆ ] ⋅ ( iˆdx + jˆdy + kˆ dz )

(ii)

= ( yz + 2 x)dx + xzdy + ( xy + 2 z )dz (iii) Path of integration is the part of the curve x2 + y2 = 1, z = 1 from (0, 1, 1) to (1, 0, 1). Parametric equation of the curve is x = cosq, y = sinq, z=1 y dx = – sinq dq, dy = cosq dq, dz = 0 A (0, 1) At point A : x = 0 cosq = 0, = 2 At point B : x = 1 cosq = 1, q = 2p

B (1, 0) x D

Substituting in F d r and integrating between the limits

=

2

to ADB

=2 ,

Fig. 9.9 2

F dr

[ (sin

2 cos )( sin d ) cos (cos d ) ]

(cos 2

sin 2

(cos 2

sin 2 )d

2 2

2 cos sin )d

2 2 2

=

sin 2 cos 2 + 2 2

2

2

1 (sin 4 2 =1 Example 7: Evaluate

C

sin

cos 4

cos )

F d r over the circular path x2 + y2 = a2 where

F = sin y iˆ + x(1 + cos y ) ˆj . Solution: (i) Let r = xiˆ + yjˆ d r = iˆdx + ˆjdy (ii)

F ⋅ d r = [ sin y iˆ + x(1 + cos y ) jˆ ] ⋅ ( iˆdx + jˆdy )

= sin y dx + x(1 + cos y )dy

Vector Calculus

9.87

= sin y dx + x cos y dy + x dy = d ( x sin y ) + xdy (iii) Path of integration is the circle x2 + y2 = a2. Parametric equation of the circle is x = a cosq, y = a sinq dy = a cosq dq dx = a sinq dq, For complete circle, q varies from 0 to 2p. Substituting in F d r and integrating between the limits q = 0 to q = 2p, C

2

F dr

d {a cos sin (a sin )} a cos

0

2

= a cos sin (a sin ) 0 + a2 = 0+ 2 = a

sin 2 + 2

2

a 2

2 0

a cos d

(1 + cos 2 )d

2

0

2

Example 8: Find work done in moving a particle in the force field F = 3 x 2 iˆ + (2 xz - y ) ˆj + zkˆ along the curve x2 = 4y and 3x3 = 8z from x = 0 to x = 2. Solution: Work done

C

F dr

(i) Let r = xiˆ + y ˆj + zkˆ d r = iˆ dx + ˆj dy + kˆ dz (ii)

F dr

3 x 2 iˆ (2 xz 3 x 2 dx (2 xz

y ) ˆj zkˆ) (iˆ dx y )dy

ˆj dy kˆ dz )

zdz

(iii) Path of integration is the curve x2 = 4y and 3x3 = 8z. x2 3 y= , z = x3 4 8 x 9x2 dy = dx , dz = dx 2 8 Substituting in F d r and integrating between the limits x = 0 to x = 2, 2⎡ ⎛ 3x3 x 2 ⎞ x 3x3 9 x 2 ⎤ Work done = ∫ ⎢3 x 2 dx + ⎜ 2 x ⋅ − ⎟ dx + ⋅ dx ⎥ 0 8 4 ⎠2 8 8 ⎝ ⎣ ⎦ 2⎛ 51x 5 x 3 ⎞ 3 x 3 51 x 6 1 x 4 = ∫ ⎜ 3x 2 + − ⎟ dx = + ⋅ − ⋅ 0 64 8 ⎠ 3 64 6 8 4 ⎝ 51 1 = 8+ − 6 2 = 16

2

0

Engineering Mathematics

9.88

Example 9: Find the work done in moving a particle from A(1, 0, 1) to B (2, 1, 2) 2 along the straight line AB in the force field F = x iˆ + ( x - y ) ˆj + ( y + z )kˆ . Solution: Work done

F dr

(i) Let r = xiˆ + yjˆ + zkˆ d r = iˆ dx + ˆj dy + kˆ dz (ii) F ⋅ d r = ⎡⎣ x 2 iˆ + ( x − y ) ˆj + ( y + z )kˆ ⎤⎦ ⋅ (iˆdx + ˆjdy + kˆdz ) x 2 dx ( x y )dy ( y z )dz (iii) Path of integration is the straight line AB joining the points A(1, 0, 1) and B(2, 1, 2). Equation of the line AB is x x1 y y1 z z1 = = x1 x2 y1 y2 z1 z2 x 1 y 0 z 1 = = 1 2 0 1 1 2 x 1 y z 1 x = 1 + y, dx = dy,

z=1+y dz = dy

B (2, 1, 2)

A (1, 0, 1)

Fig. 9.10

Substituting in F d r and integrating between the limits y = 0 to y = 1, 1

Work done

0

=

1 0

(1 y ) 2 dy (1 y

y ) dy ( y 1 y ) dy

(1 + y )3 + 2 y + y2 (1 + y ) + 2 + 2 y dy = 3

1

2

0

8 1 2 1 3 3 16 = 3

Example 10: Find work done in moving a particle along the straight line segments joining the points (0, 0, 0) to (1, 0, 0) then to (1, 1, 0) and finally to (1, 1, 1) under the force field F = (3 x 2 + 6 y ) iˆ - 14 yzjˆ + 20 xz 2 kˆ . Solution: Work done = F d r (i) Let r = xiˆ + yjˆ + zkˆ d r = iˆdx + ˆjdy + kˆdz (ii) F d r

(3 x 2 (3x 2

6 y )iˆ 14 yzjˆ 20 xz 2 kˆ (iˆdx 6 y ) dx 14 yz dy 20 xz 2 dz

ˆjdy kˆdz )

Vector Calculus

9.89

(iii) Path of integration is the line segments joining the points O (0, 0, 0) to A (1, 0, 0), A (1, 0, 0) to B (1, 1, 0) and then B (1, 1, 0) to D (1, 1, 1). Work done = ∫ F ⋅ d r C

= ∫ F ⋅ dr + ∫ F ⋅ dr + ∫ F ⋅ dr OA

AB

... (1)

BD

z

D (1, 1, 1) O (0, 0, 0)

y B (1, 1, 0)

A (1, 0, 0) x

Fig. 9.11 (a) Along OA : y = 0, dy = 0, x varies from 0 to 1. OA

F dr

1 0

z=0 dz = 0

3 x 2 dx

x3

1 0

1

(b) Along AB : x = 1, z=0 dx = 0, dz = 0 y varies from 0 to 1. AB

F dr

1 0

0 dy 0

(c) Along BD : x = 1, y=1 dx = 0, dy = 0 z varies from 0 to 1. z3 F ⋅ d r = 20 z d z = 20 ∫BD ∫0 3 1

1

=

2

0

20 3

Substituting in Eq. (1), Work done = 1 + 0 +

20 23 = 3 3

Example 11: Find the work done by the force F = the particle along the triangle OAB, where OA : 0 x 1, y x , z AB : 0 z 1, x 1, y BO : 0 x 1, y z x

xiˆ - z ˆj + 2 ykˆ in displacing 0 1

Engineering Mathematics

9.90

Solution: Work done

C

z

F dr

(i) Let r = xiˆ + y ˆj + zkˆ d r = iˆ dx + ˆj dy + kˆ dz

B (1, 1, 1)

(ii) F ⋅ d r = ( xiˆ − zjˆ + 2 ykˆ ) ⋅ ( iˆ dx + jˆdy + kˆdz ) O (0, 0, 0)

xdx zdy 2 ydz

y

(iii) Path of integration is the triangle OAB. Work done

C

F dr

OA

F dr

BO

AB

F dr

F dr

(a) Along OA : y = x, dy = dx, x varies from 0 to 1.

A (1, 1, 0)

x

... (1)

Fig. 9.12

z=0 dz = 0

OA

1

F dr

0

x2 2

x dx

1

= 0

1 2

(b) Along AB : x = 1, y=1 dx = 0, dy = 0 z varies from 0 to 1. AB

F dr

1 0

2 dz

1

2z 0

2

(c) Along BO : x = y = z dx = dy = dz x varies from 1 to 0. BO

F dr

0 1

x2

x dx 0 1

0 1

x dx

0 1

2 x dx

1

Substituting in Eq. (1), C

F dr

1 2 1 2

3 2

Example 12: Find the work done by the force F = 16 yiˆ + (3 x 2 + 2) ˆj in moving a particle once round the right half of the ellipse x 2 + a 2 y 2 = a 2 from (0, 1) to (0, - 1). Solution: Work done (i) Let r = xiˆ + yjˆ d r = iˆdx + ˆjdy

C

F dr

Vector Calculus

16 yiˆ (3x 2

(ii) F d r

2) ˆj (iˆdx

9.91

y

ˆjdy )

p 2

q

= 16 y dx + (3 x + 2) dy 2

A (0, 1)

(iii) Path of integration is the right half of the ellipse x2 + a2y2 = a2 from (0, 1) to (0, – 1). Parametric equation of the ellipse is x = a cosq, dx = a sinq dq,

D x

y = sinq dy = cosq dq

B (0, −1) −

q

At point A : y = 1 Fig. 9.13

sin q = 1 q = At point B : y = –1

p 2

2

sin q = –1 q = –

2

Substituting in F d r and integrating between the limits Work done

ADB

to

2

,

F dr 16sin ( a sin d ) (3a 2 cos 2

2

2

2) (cos d )

2

( 16a sin 2

2

3a 2 cos3

2 cos ) d

2

2 2

2 0

( 16a sin 2

3a 2 cos3

1 3 1 B , 2 2 2

16a

3a 2 ∵

2

3 1 8a 2 2 2

2

8a

8a

1 2

2 5 2

3a 2 4 2 2 3

1 2

4a 2

3a 2 2

4

2 cos ) d 1 1 B 2, 2 2 2 0

1

2

1 1 B 1, 2 2

sin p cos q d =

1 2 3 2

1 p +1 q +1 B , 2 2 2

Engineering Mathematics

9.92

Example 13: If F = 2 xyz iˆ + ( x 2 z + 2 y ) ˆj + x 2 ykˆ , then (i) prove that F is irrotational (ii) find its scalar potential e (iii) find the work done in moving a particle under this force field from (0, 1, 1) to (1, 2, 0) Solution : iˆ

ˆj



x 2 xyz

y x z + 2y

z x y

(i) F



y kˆ

( x2 =0

2

( x 2 y)

x

z

2

ˆj

( x 2 z 2 y)

( x 2 z 2 y)

y

x

( x 2 y)

z

(2 xyz )

(2 xyz )

x 2 ) iˆ (2 xy 2 xy ) ˆj (2 xz 2 xz ) kˆ

Hence, F is irrotational. (ii) Since F is irrotational, it is conservative, F (2 xyz ) iˆ + ( x 2 z + 2 y ) ˆj + ( x 2 y ) kˆ = iˆ

x

+ ˆj

y

+ kˆ

z

Comparing coefficient of iˆ, ˆj , kˆ on both the sides, x But,

= 2 xyz ,

y

= x 2 z + 2 y,

d =

x

dx +

z

= x2 y

dy +

y

z

dz

= (2 xyz ) dx + ( x 2 z + 2 y ) dy + ( x 2 y ) dz Integrating both the sides, d =

2 x yz dx +

y, z constant

( x 2 z + 2 y ) dy +

x, z constant

( x 2 y ) dz

x, y constant

Considering only those terms in R.H.S. integral which have not appeared in the previous integral, i.e., omitting x2yz term in second and third integral, = x 2 yz + y 2 + c where c is constant of integration.

Vector Calculus

9.93

(iii) F is conservative and hence the work-done is independent of the path. Work done

C

F dr

(1, 2, 0)

=

(0,1,1)

(1, 2, 0)

d =

(0,1,1)

= x 2 yz + y 2 + c

(1, 2, 0) (0,1,1)

=3 Example 14: If F = ( x 2 - yz ) iˆ + ( y 2 - zx ) ˆj + ( z 2 - xy ) kˆ , then (i) prove that F is conservative (ii) find its scalar potential e (iii) find the work done in moving a particle under this force field from (1, 1, 0) to (2, 0, 1) Solution:

(i) ∇ × F =



ˆj



∂ ∂x x 2 − yz

∂ ∂y y 2 − zx

∂ ∂z z 2 − xy

⎡∂ ⎤ ∂ = iˆ ⎢ ( z 2 − xy ) − ( y 2 − zx ) ⎥ − ∂z ⎣ ∂y ⎦

ˆj ⎡ ∂ ( z 2 − xy ) − ∂ ( x 2 − yz ) ⎤ ⎥ ⎢ ∂x ∂z ⎦ ⎣

⎡∂ ⎤ ∂ + kˆ ⎢ ( y 2 − zx ) − ( x 2 − yz ) ⎥ ∂y ⎣ ∂x ⎦ = ( − x + x ) iˆ − ( − y + y ) ˆj + ( − z + z ) kˆ =0 Thus, F is irrotational and hence is conservative. (ii) Since F is conservative, F yz ) iˆ ( y 2

( x2

xy ) kˆ

zx ) ˆj ( z 2



x

ˆj

y

Comparing coefficients of iˆ, ˆj , kˆ on both the sides, x But,

x2

yz , d =

y x ( x2

y2 dx +

zx,

y

z2

z

dy +

yz ) dx ( y 2

z

xy

dz zx ) dy ( z 2

xy ) dz



z

Engineering Mathematics

9.94

Integrating both the sides, ( x2

d

yz ) dx

y, z constant

( y2

(z2

zx) dy

x, z constant

xy ) dz

x, y constant

Considering only those terms in R.H.S. integral which have not appeared in the previous integral, i.e., omitting xyz term in second and third integral, x3 3

xyz

y3 3

z3 3

c

where c is constant of integration. (iii) F is conservative and hence the work-done is independent of the path. Work done

C

=

F dr

(2, 0,1) (1,1, 0)

x3 3

(2, 0,1)

d =

xyz

(1,1, 0)

y3 3

z3 3

(2, 0,1)

c (1,1, 0)

7 3

Example 15: Prove that the line integral of F = (y sin z - sin x) iˆ + (x sin z + 2yz) ˆj + (xy cos z + y2) kˆ is independent of the path of integration and hence, find its scalar potential e. Solution:

(i)

F



ˆj



x

y

y sin z sin x

x sin z 2 yz

z xy cos z

y2

⎡∂ ⎤ ∂ = iˆ ⎢ ( xy cos z + y 2 ) − ( x sin z + 2 yz ) ⎥ ∂ ∂ y z ⎣ ⎦ ⎡∂ ⎤ ∂ ∂ ⎡∂ ⎤ − ˆj ⎢ ( xy cos z + y 2 ) − ( y sin z − sin x ) ⎥ + kˆ ⎢ ( x sin z + 2 yz ) − ( y sin z − sin x ) ⎥ x y ∂ ∂ ∂z ⎣ ∂x ⎦ ⎣ ⎦ ˆ ˆ ˆ = ( x cos z + 2 y − x cos z − 2 y ) i − ( y cos z − y cos z ) j + (sin z − sin z ) k = 0

Thus F is irrotational and hence its line integral is independent of the path of integration. (ii) Since F is irrotational, it is conservative. F ( y sin z sin x ) iˆ ( x sin z 2 yz ) ˆj ( x y cos z

y 2 ) kˆ



x

ˆj

y



z

Vector Calculus

9.95

Comparing coefficients of iˆ, ˆj , kˆ on both the sides, y sin z sin x,

x But, d

x

dx

y

x sin z 2 yz ,

y

dy

z

y2

x y cos z

z

dz y 2 ) dz

( y sin z sin x ) dx ( x sin z 2 yz ) dy ( x y cos z Integrating both the sides, d

( y sin z sin x) dx

y, z constant

( x sin z 2 yz ) dy

x, z constant

( x y cos z

x, y constant

y 2 ) dz

Considering only those terms in R.H.S. integral which have not appeared in the previous integral, i.e., omitting xy sin z term in second integral and xy sin z, y2z terms in third integral, xy sin z cos x y 2 z c where c is constant of integration. Example 16: Determine constants a, b, c so that vector F = (x + 2y + az) iˆ + (bx - 3y - z) ˆj + (4x + cy + 2z) kˆ is conservative. Also, find (i) scalar potential e (ii) find the work done in moving a particle under this force field from (1, 2, -4) to (3, 3, 2). Solution : (i) If F is conservative, iˆ

F

0 kˆ

ˆj

x

y z x 2 y az bx 3 y z 4 x cy 2 z iˆ

y

(4 x cy 2 z )

z

(bx 3 y z )

ˆj

x

=0

(4 x cy 2 z )

⎡∂ ⎤ ∂ + kˆ ⎢ (bx − 3 y − z ) − ( x + 2 y + a z ) ⎥ = 0 ∂y ⎣ ∂x ⎦ (c + 1) iˆ − ( 4 − a ) ˆj + (b − 2) kˆ = 0 Comparing coefficients of iˆ, ˆj , kˆ on both the sides, c = 1, a = 4, b = 2

z

( x 2 y az )

Engineering Mathematics

9.96

(ii) Since F is conservative, F ( x 2 y 4 z ) iˆ (2 x 3 y z ) ˆj (4 x

y 2 z ) kˆ



x

ˆj

y



z

Comparing coefficients of iˆ, ˆj , kˆ on both the sides, x 2 y 4 z,

x But, d =

y

2 x 3 y z,

4x

z

dx + dy + dz x y z ( x 2 y 4 z ) dx (2 x 3 y z ) dy (4 x

y 2z

y 2 z ) dz

Integrating both the sides, d

( x 2 y 4 z ) dx

(2 x 3 y z ) dy

y, z constant

x, z constant

(4 x

x, y constant

y 2 z ) dz

Considering only those terms in R.H.S. integral which have not appeared in the previous integral, i.e., omitting xy term in second and xz, yz terms in the third integral, x2 2 x2 2

2 xy 4 xz 3y2 2

z2

3y2 2 2 xy

yz z 2

k

yz 4 xz k

where k is constant of integration. (iii) F is conservative and hence the work done is independent of the path. Work done

C

=

F dr

(3, 3, 2) (1, 2, 4)

x2 2 9 2 49 = 2

(3, 3, 2)

d =

3y2 2

(1, 2, 4) (3, 3, 2)

z

2

2 xy

yz 4 xz k

27 4 18 6 24 k 2

(1, 2, 4)

1 12 16 4 8 16 k 2 2

Vector Calculus

9.97

Exercise 9.6 1. Evaluate ∫ F ⋅ d r , where C

F ( x y ) iˆ ( y x) ˆj and C is (i) the parabola y 2 = x between the points (1, 1) and (4, 2) (ii) the straight line joining the points (1, 1) and (4, 2). 34 Ans. : (i) (ii)11 3 2. Evaluate F d r , where F C

(3 x 2 y ) iˆ

( y 2 z ) ˆj x 2 kˆ and C is (i) the curve x = t, y = t2, z = t3 between the points (0, 0, 0) to (1, 1, 1) (ii) the straight line joining the points (0, 0, 0) to (1, 1, 1). (iii) the straight lines from (0, 0, 0) to (0, 1, 0) then to (0, 1, 1) and then to (1, 1, 1). 23 5 Ans. : (i) (ii) (iii) 0 15 3 3. Evaluate C

F d r , where F

(2 x

y 2 ) iˆ

+ (3 y − 4 x ) ˆj and C is the triangle in xy-plane with vertices (0, 0), (2, 0) and (2, 1). 14 Ans. : 3 4. Evaluate F d r , where F C

yz iˆ zx ˆj xy kˆ

and C is the curve y2 = x, z = 0 from (0, 0, 0) to (1, 1, 0) followed by the straight line from (1, 1, 0) to (1, 1, 1). 3 Ans. : 4 5. Evaluate F d r , where F C

and C is the curve

2 x iˆ 4 y ˆj 3 z kˆ

r = cos t iˆ + sin t ˆj + t kˆ from t = 0 to t = p. Ans. :

2

3 2

6. Find the circulation of F ( x 3 y ) iˆ ( y 2 x) ˆj around the ellipse in the xy-plane with the origin as centre and 2 and 3 as semimajor and semi-minor axes respectively. [ Ans. : 6p ] 7. Find the circulation of F = y iˆ + z ˆj + x kˆ around the curve x 2 + y 2 = 1, z = 0.

[ Ans. : − p ] 8. Find the work done in moving a particle in a force field F 3 xy iˆ 5 z ˆj 10 x kˆ along the curve x = 1 + t 2 , y = 2t 2 , z = t 3 from t = 1 to t = 2.

[ Ans. : 303] 9. Find the work done in moving a particle in a force field F 3 x 2 iˆ (2 xz y ) ˆj z kˆ along the (i) straight line joining the points (0, 0, 0) and (2, 1, 3). (ii) curve x 2t 2 , y t , z 4t 2 t from t = 0 to t = 1. Ans. : (i) 16 (ii)

71 5

10. Find the work done in moving a particle in a force field F (2 x y z ) iˆ ( x y z 2 ) ˆj +(3 x − 2 y + 4 z ) kˆ once around the circle in xy-plane with centre at the origin and radius 3.

[ Ans. :18p ]

Engineering Mathematics

9.98

11. Determine whether the force field F 2 xz iˆ ( x 2 y ) ˆj (2 z x 2 ) kˆ

(i) the scalar potential f (ii) the work done in moving a particle under this force field from (0, 0, 0) to (1, 1, 1).

is conservative or not.

[ Ans. : no ]

Ans. : (i) = x 2 y e z + c (ii) e

12. If F = (2 xy + z ) iˆ + x ˆj + 3xz 2 kˆ, then 3

2

15. Find the constant a such that the force field F = (a xy − z 3 ) iˆ (a 2) x 2 ˆj + (1 − a ) a z 3 kˆ is a conservative field. Find its (i) scalar potential f (ii) the work done in moving a particle under this force field from (1, 2, 3) to (1, 4, 2).

(i) prove that F d r is independent of the path. (ii) find its scalar potential f. (iii)find the work done in moving a particle under this force field from (1, 2, 1) to (3, 1, 4). = x 2 y + x z 3 + c (iii) 202

Ans. : (ii) 13. If F

3 x y iˆ ( x

Ans. : a

2 y z ) ˆj +(3 z 2 − 2 y 2 z ) kˆ , then 2

3

2

C

(ii) find its scalar potential f

is independent of the path C. Find the scalar potential f of the force field.

(iii) find the work done in moving a particle under this force field from (2, 1, 1) to (2, 0, 1). x3 y z 3

y2 z2

c

16. Find the constant b such that the (e x z bxy ) iˆ (1 bx 2 ) ˆj (e x bz ) kˆ

(i) prove that F is conservative

Ans. : (ii) (iii)

4 (i) 2 x 2 y 3z 4 (ii)183

Ans. : b = 0, 17. Evaluate F d r where F

c

7

C

= ze x + y

cos y iˆ x sin y ˆj

and C is the curve y 1 x 2 in xy-plane from (1, 0) to (0, 1).

14. If F = 2 xy e z iˆ + x 2 e z ˆj + x 2 y e z kˆ is irrotational, then find

[ Ans. : −1]

9.17 GREEN’S THEOREM IN THE PLANE M N are continuous in , y x some region R of xy-plane bounded by a closed curve C, then Statement: If M (x, y), N (x, y) and their partial derivatives

C

( M dx N dy ) R

N x

M dx dy y

Proof: Let the region R is bounded by the curve C. Let the curve C is divided into two parts, the curves EAB and BDE. Let the equations of the curves

Vector Calculus

9.99

EAB and BDE are x = f1(y), x = f2(y) respectively and are bounded between the lines y = c and y = d Consider, ∂N

d



∫∫ ∂x dx dy = ∫ ⎢⎣ ∫ c

R

f2 ( y ) f1 ( y )

∂N ⎤ dx dy ∂x ⎥⎦

d

f2 ( y )

c

f1 ( y )

= ∫ N ( x, y )

dy

= ∫ [ N ( f 2 , y ) − N ( f1 , y ) ] dy d

c

d

c

c

d

= ∫ N ( f 2 , y ) dy + ∫ N ( f1 , y ) dy =∫

BDE

=



C

N ( x , y ) dy + ∫

EAB

Fig. 9.14

N ( x , y ) dy

N ( x, y ) dy

C

N ( x, y ) dy = R

N dx dy x

... (1)

Similarly, let the curve C is divided into two parts, the curves ABD and DEA. Let the equations of the curves ABD and DEA are y = g1(x), y = g2(x) respectively and are bounded between the lines x = a and x = b. Consider, b ⎡ g 2 ( x ) ∂M ⎤ ∂M ∫∫R ∂y dx dy = ∫a ⎢⎣ ∫g1 ( x ) ∂y dy ⎥⎦ dx b

= ∫ M ( x , y ) g 2 ( x ) dx g ( x)

a

1

= ∫ [ M ( x, g 2 ) − M ( x, g1 ) ] dx b

a

a

b

b

a

= − ∫ M ( x, g 2 ) dx − ∫ M ( x, g1 ) dx = − ⎡ ∫ M ( x, y ) dx + ∫ M ( x, y ) dx ⎤ ABD ⎣ DEA ⎦ = − ∫ M ( x , y ) dx C

C

M ( x, y ) dx R

M dx dy y

Adding Eqs. (1) and (2), C

( M dx N dy ) R

N x

M dx dy y

Note: Vector form of Green’s theorem is given as F dr F ) kˆ dx dy ( C

R

... (2)

Engineering Mathematics

9.100

where F = M iˆ + N ˆj , r = x iˆ + y ˆj , kˆ is the unit vector along z-axis.

Area of the Plane Region

Let A be the area of the plane region R bounded

by a closed curve C. Let M = − y, N = x ∂M ∂N = −1, =1 ∂y ∂x Using Green’s theorem, C

( y dx x dy )

(1 1) dx dy R

2 dx dy

2A

R

1 ( x dy y d x ) 2 C Note : In polar coordinates, Hence, A

x = r cos , dx = cos dr − r sin d , 1 2 1 = 2

A=

∫ [ r cos C



C

y = r sin dy = sin dr + r cos d

(sin dr + r cos d ) − r sin (cos dr − r sin d ) ]

r 2d ( x 2 - 2 xy )dx + ( x 2 y + 3)dy where C

Example 1: Verify Green’s theorem for C

is the boundary of the region bounded by the parabola y = x 2 and the line y = x. Solution: (i) The points of intersection of the parabola y = x 2 and the line y = x are obtained as x = x 2 , x = 0, 1 and y = 0, 1. Hence, B : (1, 1). (ii) M x 2 2 xy, M y (iii)

2 x,

N

x2 y 3

N x

2 xy

( M dx + N dy ) C

=

OAB

( M dx + N dy ) +

BO

( M dx + N d y )

... (1) (a) Along OAB : y = x 2 dy = 2 x dx x varies from 0 to1.

Fig. 9.15

Vector Calculus

OAB

( M dx N dy )

( x2

OAB 1

( x2

0 1

( x2

0

x3 3

2 xy )dx ( x 2 y 3)dy 2 x x 2 )dx ( x 2 x 2

2 x3

2x4 4

1 1 3 2 19 = 6

9.101

2 x5

2 x6 6

3)2 x dx

6 x) dx

6x2 2

1

0

1 3 3

(b) Along BO : y = x dy = dx x varies from : x = 1 to x = 0.



BO

( M dx + N dy ) = ∫ ⎡⎣( x 2 − 2 xy )dx + ( x 2 y + 3)dy ⎤⎦ BO 0

= ∫ ⎡⎣( x 2 − 2 x 2 )dx + ( x3 + 3)dx ⎤⎦ 1 0

= −

x3 x 4 1 1 + + 3x = − − 3 3 4 3 4 1

35 12

=− Substituting in Eq. (1),

19 35 1 ... (2) = 6 12 4 (iv) Let R be the region bounded by the line y = x and the parabola y = x 2 . Along the vertical strip AA , y varies from x 2 to x and in the region R, x varies from 0 to 1. C

R

( M dx N dy )

N x

M dx dy y

1

x

0

x2

1 0

(2 xy 2 x) dy dx

xy 2

x4 4 1 4 =

1 4

2 x3 3 2 3

x

1

x

0

2 xy 2 dx x6 6

( x3

2x2

x5

2 x 3 ) dx

1

0

1 6 ... (3)

Engineering Mathematics

9.102

From Eqs. (2) and (3), C

( M dx + N dy ) = R

N M 1 dx dy = x y 4

Hence, Green’s theorem is verified. Example 2: Verify Green’s theorem for

∫ ⎡⎣( x - y ) dx + 3 xy dy ⎤⎦,

where C is

C

the boundary of the region bounded by the parabolas x 2 = 4 y and y 2 = 4 x. Solution: (i) The points of intersection of the parabolas x 2 = 4 y and y 2 = 4 x are obtained as 2

⎛ y2 ⎞ 3 ⎜ ⎟ = 4 y, y ( y − 64) = 0 ⎝ 4 ⎠ y = 0, 4 x = 0, 4 Hence, C : (4, 4) (ii)

M

x

M y

(iii)

C

y, 1,

( M dx + N dy ) =

N

3 xy

N x

3y

OAC

Fig. 9.16 ( M dx + N d y ) +

CBO

( M dx + N d y )

... (1)

2 (a) Along OAC : x 2 = 4 y, y = x 4 x dy = dx 2

x varies from 0 to 4.



OAC

( M dx + N dy ) = ∫

OAC

[( x − y )dx + (3 xy )dy ]

4⎛ ⎛ x2 ⎞ x2 ⎞ x = ∫ ⎜ x − ⎟ dx + ⎜ 3 x ⋅ ⎟ dx 0 4 ⎠2 4 ⎠ ⎝ ⎝ 4⎛ x2 3 ⎞ x 2 x3 3 x5 − + ⋅ = ∫ ⎜ x − + x 4 ⎟ dx = 0 4 8 ⎠ 2 12 8 5 ⎝

8

16 3

384 5

1192 15

4

0

Vector Calculus

(b) Along CBO : y 2 = 4 x, x = dx =

9.103

y2 4

y dy 2

y varies from 0 to 4. CBO

( M dx N dy )

CBO 0 4

[( x

y )dx 3 xy dy ]

y2 4

y

7 y3 8

0 4

y dy 2

3

y2 dy 2

y2 y dy 4

7 y4 8 4

0

1 y3 2 3

4

7 1 64 64 8 2 3 136 3

Substituting in Eq. (1), C

1192 136 15 3

( M dx N dy ) =

512 15

... (2)

(iv) Let R be the region bounded by the parabolas x 2 = 4 y and y 2 = 4 x. x2 Along the vertical strip AB, y varies from to 2 x and in the region R, 4 x varies from 0 to 4.

R

N x

M dx dy y =

4

2 x

0

x2 4

4 0

(3 y 1) dx dy

3y2 +y 2

4 0

dx x2 4

6x 2 x

3x 2

4 32 x 3

32 48 3 =

2 x

512 15

3 4 x 32 3 x5 32 5

x2 dx 4 1 x3 4 3

4

0

96 16 5 3 ... (3)

Engineering Mathematics

9.104

From Eqs. (2) and (3), C

N x

( M dx N d y ) R

M dx dy y

512 15

Hence, Green’s theorem is verified. Example 3: Verify Green’s theorem for

C

( y - sin x ) dx + cos x dy

where C is

o 2x , y= . 2 o

the plane triangle enclosed by the lines y = 0, x = Solution: (i) The point of intersection of 2x the lines y = and x = is obtained as 2 2 y = ⋅ = 1. 2 Hence B :

2

,1

M = y – sin x, N = cos x

(ii)

M y (iii) C

=

N x

1,

sin x

( M dx + N d y ) OA

Fig. 9.17

( M dx + N d y ) +

AB

( M dx + N d y ) +

BO

( M dx + N d y )

(a) Along OA : y = 0 dy = 0 x varies from 0 to OA

2

.

( M dx N dy )

[( y

OA 2 0

sin x)dx cos x dy ]

( sin x) dx

cos x 02

1 (b) Along AB : x =

, dx = 0 2 y varies from 0 to 1. ( M dx N d y ) AB

AB

=

1 0

[( y

cos

2

sin x)dx cos x dy ] dy = 0

... (1)

Vector Calculus

(c) Along BO : y = x varies from

2x

2 BO

, dy =

2

9.105

dx

to 0. ( M dx N d y )

BO

[( y

sin x)dx cos xdy ]

2x

0

2

sin x dx cos x

dx

2

2 x2 2

cos x

2

0

sin x 2

1

cos 0 1

2

4

cos

2 2

sin

2

2 4

Substituting in Eq. (1), C

( M dx N d y )

1 0 1

2 4

⎛ 2 +8⎞ = −⎜ ⎟ ⎝ 4 ⎠

... (2)

(iv) Let R be the region bounded by the triangle OAB. 2x Along the vertical strip PQ, y varies from 0 to and in the region R, x varies from 0 to

R

2 N x

. M dx dy y

2x 2 0

( sin x 1) dx dy

0

2x 2 0

2

y sin x

x2 2

x( cos x) ( sin x)

sin x

2x

dx

2

0

2

2 2 2

2x

2 0

y 0 dx

cos

2

sin

2

8

0

2

1

8

⎛ 2 +8⎞ = −⎜ ⎟ ⎝ 4 ⎠

... (3)

Engineering Mathematics

9.106

From Eqs. (2) and (3), C

( M dx N d y ) R

N x

2

M dx dy y

8

4

Hence, Green’s theorem is verified. Example 4: Verify Green’s theorem for



C

1 ⎞ ⎛1 ⎜⎝ y dx + x dy ⎟⎠ where C is the

boundary of the region bounded by the parabola y = y = 1.

x and the lines x = 1, x = 4,

Solution: (i) The point of intersection of the (a) parabola y = x and the line x = 1 is obtained as y = 1 =1 Hence, A : (1, 1) (b) parabola y = x and the line x = 4 is obtained as y= 4=2 Hence, D : (4, 2) (ii) M = M y (iii)



1 , y

N= N x

1 , y2 C

1 x

Fig. 9.18 1 x2

( M dx + N dy ) = ∫ ( M dx + N dy ) + ∫ ( M dx + N dy ) AB

BD

+∫

DQA

( M dx + N dy )

(a) Along AB : y = 1, dy = 0 x varies from 1 to 4.



AB

⎛1 1 ⎞ ( M dx + N dy ) = ∫ ⎜ dx + dy ⎟ AB y x ⎠ ⎝ 4

= ∫ dx = x 1 1

=3 (b) Along BD : x = 4, dx = 0 y varies from 1 to 2.

4

... (1)

Vector Calculus

9.107

⎛1 1 ⎞ ( M dx + N dy ) = ∫ ⎜ dx + dy ⎟ BD y x ⎠ ⎝ 21 1 2 = ∫ dy = y 1 1 4 4 1 = 4 1 dx (c) Along DQA : y = x , dy = 2 x



BD

x varies from 4 to 1.



DQA

( M dx + N dy ) = ∫

DQA

⎛1 1 ⎞ ⎜ dx + dy ⎟ x ⎠ ⎝y

1⎛ 1 1 1 1 ⎞ =∫ ⎜ dx + ⋅ dx ⎟ = 2 x − 4 x 2 x ⎠ x ⎝ x 1 = 2 −1− 4 + 2 5 =− 2

1

4

Substituting in Eq. (1), ( M dx N d y )

3

C

=

1 4

5 2

3 4

... (2)

(iv) Let R be the region bounded by the parabola y = x and the lines x = 1, x = 4, y = 1. Along the vertical strip, y varies from 1 to x and in the region R, x varies from 1 to 4. 4 x⎛ ⎛ ∂N ∂M ⎞ 1 1 ⎞ ∫R ∫ ⎜⎝ ∂x − ∂y ⎟⎠ dx dy = ∫1 ∫1 ⎜⎝ − x 2 + y 2 ⎟⎠ dx dy 4

=∫ − 1

= 2x



1 2

1 1 ⋅y− y1 x2 1 2

x

3 1 − − 4⎛ ⎞ 1 = ∫ ⎜ − x 2 − x 2 + 2 + 1⎟ dx 1 ⎝ x ⎠ 4

1 − 2x − + x x 1

1 = 1− 4 − + 4 − 2 + 2 +1−1 4

=

3 4

... (3)

Engineering Mathematics

9.108

From Eqs. (2) and (3), N x

( M dx N dy ) R

M dx dy y

3 4

Hence, Green’s theorem is verified. Example 5: Verify Green’s theorem for

C

(2 xy dx - y 2 dy ) where C is the

boundary of the region bounded by the ellipse 3 x 2 + 4 y 2 = 12. Solution: M

(i)

2 xy,

M = 2 x, y (ii)

C

y2

N N =0 x

( M dx N dy )

C

where C is the ellipse

(2 xy dx

y 2 dy ),

... (1)

x2 y 2 + = 1. 4 3

Parametric equation of the ellipse is x = 2 cos , dx

y = 3 sin

2sin d , dy

3 cos d

For the given ellipse, q varies from 0 to 2p. Substituting in Eq. (1),



Fig. 9.19



C

( M dx + N dy ) = ∫ ⎡⎣(2 ⋅ 2 cos θ ⋅ 3 sin θ )(−2 sin θ dθ ) − 3 sin 2 θ ⋅ 3 cos θ dθ ⎤⎦ 0 =∫



0

( −11

)

3 cos θ sin 2 θ dθ π

= −11 3 ⋅ 2 ∫ cos θ sin 2 θ dθ 0

=0

...(2)

⎡∵ 2 a f ( x)dx = 2 a f ( x)dx, ⎤ ∫0 ⎥ ⎢ ∫0 ⎢if f (2a − x) = f ( x) ⎥ ⎢ ⎥ = 0, ⎢ ⎥ ⎢if f (2a − x) = − f ( x) ⎥ ⎦ ⎣

(iii) Let R be the region bounded by the ellipse,

x2 y 2 + = 1. 4 3

Along the vertical strip PQ, y varies from

3x 2 3x 2 to 3 4 4

R, x varies from 2 to 2.

3

and in the region

Vector Calculus

9.109

3 x2

3− 2 ⎛ ∂N ∂M ⎞ ∫R ∫ ⎜⎝ ∂x − ∂y ⎟⎠ dx dy = ∫−2 ∫− 3 − 34x2 (0 − 2 x) dy dx 4 2

= ∫ −2 x y −2

=0

3 x2 4 3 x2 − 3− 4 3−

2

dx = −4 ∫ x 3 − −2

3x 2 dx 4

⎡∵ f ( x)dx = 0, if f (− x) = − f ( x) ⎤ ⎢⎣ ∫− a ⎥⎦ a

...(3)

From Eqs. (2) and (3),

C

( M dx + N dy ) = R

N x

M dx dy y

0

Hence, Green’s theorem is verified. Example 6: Evaluate

C

( x 2 - cosh y ) dx + ( y + sin x ) dy by Green’s theorem

where C is the rectangle with vertices (0, 0), (o , 0), (o , 1), (0, 1). Solution: By Green’s theorem,



C

⎛ ∂N ∂M ( M dx + N dy ) = ∫ ∫ ⎜ − ∂y R ⎝ ∂x

⎞ ⎟ dx dy ⎠

... (1) where R is the region bounded by the rectangle OABC. M M y

x2

cosh y, N sinh y,

N x

y sin x cos x

Along the vertical strip PQ, y varies from 0 to 1 and in the region R, x varies from 0 to p. Fig. 9.20

Substituting in Eq. (1),



1

C

⎡⎣( x 2 − cosh y )dx + ( y + sin x)dy ⎤⎦ = ∫ ∫ (cos x + sinh y ) dy dx x =0 y =0

= ∫ y cos x + cosh y 0 dx = ∫ (cos x + cosh 1 − 0 − cosh 0) dx 1

0

0

= ∫ (cos x + cosh 1 − 1) dx = sin x + x cosh 1 − x 0 0

sin cosh1 (cosh1 1)

sin 0

Engineering Mathematics

9.110

Example 7: Evaluate by Green’s theorem cardioid r = a (1 + cosp ).

C

( - x 2 y dx + xy 2 dy ) where C is the

Solution: By Green’s theroem,

C

N x

( M dx + N dy ) = R

M dx dy y

... (1) where R is the region bounded by the cardioid r = a (1 + cosq ). M

x 2 y,

N

xy 2

M y

x2 ,

N x

y2

Fig. 9.21

x = r cosq,

Putting

M y

y = r sinq N x

r 2 cos 2 ,

r 2 sin 2

dx dy = r dr dq Along the radius vector OA, r varies from 0 to a (1 + cosq) and in the region R, q varies 0 to 2p. Substituting in Eq. (1),



C

(− x 2 y dx + xy 2 dy ) = ∫



0

=∫



0



a (1+ cosθ )

0

r dr dθ = ∫ 3



0

a 4 2π (1 + cosθ ) 4 dθ 4 ∫0 π a4 = ⋅ 2 ∫ (1 + cosθ ) 4 dθ 0 4



a (1+ cosθ )

0

r4 4

(r 2 sin 2 θ + r 2 cos 2 θ )r dr dθ

a (1+ cosθ )

dθ 0

=

=

Putting When

a4 2



π

0

4

⎛ 2θ ⎞ ⎜ 2 cos ⎟ dθ 2⎠ ⎝

= t , d = 2dt 2 = 0, t=0

⎡∵ 2 a f ( )d = 2 a f ( )d ∫0 ⎢ ∫0 ⎢ if f (2a – ) = f ( ) ⎣

⎤ ⎥ ⎥ ⎦

Vector Calculus

t=

= ,



C

9.111

2

(− x 2 y dx + xy 2 dy ) = 8a 4 ∫ 2 cos8 t ⋅ 2 dt 0

9 1 1 ⎛9 1⎞ 4 = 16a ⋅ B ⎜ , ⎟ = 8a ⋅ 2 2 2 ⎝2 2⎠ 5 4

8a 4 24 35 = 16 =

Example 8: Evaluate

C

7 5 3 1 1 1 ⋅ ⋅ ⋅ ⋅ 2 2 2 2 2 2 a4

( x 2 + 2 y ) dx + (4 x + y 2 ) dy by Green’s theorem where

C is the boundary of the region bounded by y = 0, y = 2x and x + y = 3. Solution: By Green’s theorem, C

N x

( M dx N dy ) R

M dx dy y

... (1)

where R is the region bounded by the triangle OAB. M = x 2 + 2 y, N = 4x + y2 M N = 2, =4 y x Substituting in Eq. (1), ( x 2 + 2 y )dx + (4 x + y 2 )dy C

(4 2)dx dy

2 dx dy

R

R

= 2(Area of ΔOAB ) 1 = 2 ⋅ ⋅3⋅ 2 2 =6

Fig. 9.22

Example 9: Find the area of the region bounded by the parabola y = x 2 and the line y = x + 2. Solution : (1) The points of intersection of the parabola y = x 2 and the line y = x + 2 are obtained as x 2

x2 , x2

x 2

0

(x 2)(x + 1) = 0, x = 2, 1 and y = 4, 1

Engineering Mathematics

9.112

Hence A : ( 1, 1) and B : (2, 4). (ii) By Green’s theorem, area of the region bounded by a closed curve C is A

1 2

C

=

( x dy

y dx )

1⎡ ∫ ( x dy − y dx) 2 ⎢⎣ AOB

+ ∫ ( x dy − y dx) ⎤ ⎥⎦ BA

... (1)

(a) Along AOB : y = x 2 , dy = 2 x dx Fig. 9.23

x varies from 1 to 2. AOB

( x dy

2

y dx )

1

x3 3

=

( x 2 x dx x 2 dx ) 2

= 1

8 1 + 3 3

=3 (b) Along BA : y = x + 2, dy = dx x varies from 2 to 1. BA

( x dy

y dx )

1 2

[ x dx

2 x2

1

( x 2) dx ] 2( 1 2)

=6 Substituting in Eq. (1), 1 9 A = (3 + 6) = 2 2 Example 10: Find the area of the ellipse

x2 y2 + = 1. a 2 b2

Solution: (i) By Green’s theorem, area of the region bounded by a closed curve C is 1 ( x dy y dx ) A 2 C x2 y 2 + = 1 is a 2 b2 x = a cos , y = b sin dx a sin d , dy b cos d

(ii) Parametric equation of the ellipse

... (1)

Vector Calculus

9.113

For the given ellipse, q varies from 0 to 2p . Substituting in Eq. (1), 1 2 [a cos (b cos d ) A = 2 0 b sin ( a sin d )] 1 2 1 2 = ab d = ab 0 0 2 2 = ab Example 11: Find the area of the loop of the folium of descrates x 3 + y 3 = 3a xy.

Fig. 9.24

Solution: (i) Putting x = r cosq, y = r sinq, equation of the curve changes to r 3 (cos3 + sin 3 ) = 3ar 2 sin cos 3a sin cos r= cos3 + sin 3 (ii) By Green’s theorem, area of the region bounded by a closed curve C in polar form is 1 A= r2 d 2 C For the loop of the given curve, q varies from 0 to

A = =

1 2

2 0

9a 2 2

Putting

2

.

9a 2 sin 2 cos 2 d (cos3 + sin 3 ) 2 2 0

tan 2 sec 2 d (1 + tan 3 ) 2

Fig. 9.25

1 + tan 3 = t 3 tan 2 sec 2 d = dt

When q = 0, t = 1 π θ = , t→∞ 2 A =

=

9a 2 2 3a 2

2





1



dt 3a 2 1 = − 3t 2 2 t1

9.114

Engineering Mathematics

Exercise 9.7 (I) Verify Green’s theorem in plane for the following: bounded by the square with vertices 1. ∫ ⎡⎣( x 2 − 2 xy ) dx + ( x 2 y + 3) dy ⎤⎦, C where C is the boundary of the (0, 0), ,0 , , , 0, . 2 2 2 2 region bounded by the parabola y 2 = 8 x and the line x = 2. ⎡ ⎛ − ⎞⎤ ⎢ Ans.: 2 ⎜ e 2 − 1⎟ ⎥ 128 ⎢⎣ ⎝ ⎠ ⎥⎦ Ans. : 5 ( xy 2 2 xy ) dx ( x 2 y 3) dy , 5. C 2. ( xy x 2 ) dx x 2 y dy , where where C is the boundary of the region C bounded by the rectangle with vertiC is the boundary of the triangle ces ( 1, 0), (1, 0), (1, 1) and ( 1, 1). formed by the lines y = 0, x = 1 and [ Ans.: 0] y = x. 1 3 3 Ans.: 6. C ( x dy y dx), where C is the 12 circle x 2 + y 2 = 4. 2 2 3. ∫ ⎡⎣(3 x − 8 y ) dy + (4 y − 6 xy ) dy ⎤⎦, [ Ans.: 48 ] C where C is the boundary of the re7. ∫ ⎡⎣(2 x 2 − y 2 ) dx + ( x 2 + y 2 ) dy, 2 C gion bounded by y = x and y = x . where C is the boundary of the 3 region bounded by the x-axis and the Ans. : 2 circle y 1 x2 . 4 4. ∫ (e − x sin y dx + e − x cos y dy ), where C Ans. : 3 C is the boundary of the region (II) Evaluate the following integrals using Green’s theorem: where C is the boundary of the region 1. e x (cos y dx sin y dy ), where C C in the first quadrant bounded by y-axis is the boundary of the region boundand the parabolas y 1 x 2 , y x 2 . ed by the rectangle with vertices 1 2 Ans.: + . (0, 0), ( , 0), , and 0, . 8 6 2 2 4. ( xy dx + x 3 dy ), where C is the Ans.: 2(1 e ) C boundary of the region bounded by ( x 2 y 2 ) dx (5 x 2 3 y ) dy , 2. x-axis and the circle y 4 x2 . C

where C is the boundary of the region bounded by the parabola x2 = 4y and the line y = 4. 512 Ans. : 5 3 2 ( y xy ) dx ( xy 3xy ) dy , 3. C

[ Ans. : 6 ]

5.

x

e (sin y dx + cos y dy ), C where C is the boundary of the region bounded by the ellipse 4 ( x 1) 2 9 ( y 3) 2 36.

[ Ans. : 0 ]

Vector Calculus

9.115

(III) Find the area of the following regions using Green’s theorem: 2 2 2 3. In the first quadrant, bounded by the 1. Bounded by the astroid x 3 + y 3 = a 3 . lines y = x, x = 4y and rectangular hyperbola xy = 1. 3 2 [ Ans. : log 2] Ans. : a 8 4. Bounded by one loop of the lemnis2. Bounded by one arch of the cycloid cate ( x 2 y 2 ) 2 a 2 ( x 2 y 2 ) x = a (q sinq ), y = a (1 cosq ) and ⎡ a2 ⎤ x-axis. Ans.: ⎥ ⎢ ⎣ 2⎦ [ Ans. : 3 a 2 ]

9.18 SURFACE INTEGRALS The surface integral over a curved surface S is the generalisation of a double integral over a plane region R. Let F = F1 iˆ + F2 ˆj + F3 kˆ be a continuous vector point function defined over a two sided surface S. Dividing S into a finite number of sub-surfaces S1 , S 2 , ......, S m with surface areas S1 , S 2 , ......, S m . Let d Sr be the surface area of Sr and nˆr be the unit vector at some point Pr (in Sr) in the direction of the outward normal to Sr.

Fig. 9.26 If we increase the number of subsurfaces, then the surface area dSr of each subsurface will decrease. Thus, as m , Sr 0 Then, m

lim ∑ F ( Pr ) ⋅ nr S r = ∫∫ F ⋅ n dS

m →∞

r =1

S

This is called surface integral of F over the surface S.

Engineering Mathematics

9.116

The surface integral can also be written as

∫∫ F ⋅ dS , where dS = nˆ dS S

If equation of the surface S is f (x, y, z) = 0, then nˆ =

9.18.1 Flux If F represents velocity of the fluid at any point P on a closed surface S, then surface integral ∫∫ F ⋅ nˆ dS represents the flux of F over S, i.e., volume of the fluid flowing S

out from S per unit time. Note: If

∫∫ F ⋅ nˆ dS = 0 , then

F is called a solenoidal vector point function.

S

9.18.2 Evaluation of Surface Integral A surface integral is evaluated by expressing it as double integral over the region R. The region R is the orthogonal projection of S on one of the coordinate planes (xy, yz or zx) Let R be the orthogonal projection of S on the xy-plane and cosa, cosb, cosg are the direction cosines of nˆ . Then nˆ = cos α iˆ + cos β ˆj + cos γ kˆ dx dy = Projection of dS on xy-plane = dS cosg dx dy dS = cos =

dx dy nˆ kˆ

∫∫ F ⋅ nˆ dS =∫∫ F ⋅ nˆ

Hence,

S

R

dx dy nˆ ⋅ kˆ

Similarly, taking projection on yz and zx-plane,

∫∫ F ⋅ nˆ dS =∫∫ F ⋅ nˆ S

R

dy dz nˆ ⋅ iˆ

and

∫∫ F ⋅ nˆ dS =∫∫ F ⋅ nˆ S

R

dz dx nˆ ⋅ ˆj

Component form of Surface Integral

∫∫ F ⋅ nˆ dS = ∫∫ ( F iˆ + F 1

S

2

ˆj + F3 kˆ ) ⋅ (cos α iˆ + cos β ˆj + cos γ kˆ ) dS

S

= ∫∫ ( F1 dS cos α + F2 dS cos β + F3 dS cos γ ) S

= ∫∫ F1 dy dz + F2 dz dx + F3 dx dy ) S

Vector Calculus

Example 1: Evaluate

∫∫ F ⋅ nˆ dS ,

9.117

where F = 18 z iˆ - 12 ˆj + 3 y kˆ and S is the

S

part of the plane 2x + 3y + 6z = 12 in the first octant. Solution: (i) The given surface is the plane 2x + 3y + 6z = 12 in the first octant. Let = 2 x + 3 y + 6 z z 2iˆ 3 ˆj 6 kˆ nˆ = = 4 + 9 + 36 2iˆ + 3 ˆj + 6 kˆ C(0, 0, 2) = 7 y

(ii) Let R be the projection of the plane 2x + 3y + 6z = 12 (in the first octant) on xy-plane, which is a triangle OAB bounded by the lines y = 0, x = 0 and 2x + 3y = 12. dx dy 7 (iii) dS = = dx dy nˆ ⋅ kˆ 6

O

B(0, 4, 0)

A(6, 0, 0) x

Fig. 9.27

(iv) Along the vertical strip PQ, y varies from 0 to 12 2 x and in the region R, x 3 varies from 0 to 6.

∫∫ F ⋅ nˆ dS = ∫∫ (18 z iˆ − 12 ˆj + 3 y kˆ) ⋅ S

R

⎛ 2iˆ + 3 ˆj + 6kˆ ⎞ 7 ⎜⎜ ⎟⎟ dx dy 7 ⎝ ⎠6 1 = ∫∫ (36 z − 36 + 18 y ) dx dy 6R ⎡ ⎛ 12 − 2 x − 3 y ⎞ ⎤ = 3∫∫ ⎢ 2 ⎜ ⎟ − 2 + y ⎥ dx dy 6 ⎠ ⎦ R ⎣ ⎝ 12 − 2 x 6 3 0 0

=∫



( 6 − 2 x ) dy dx

6

12 − 2 x 3

= 2 ∫ (3 − x) y 0

0

6

dx = 2 ∫ (3 − x) 0

Fig. 9.28

(12 − 2 x) dx 3 6

4 6 4 x3 9 x 2 = ∫ ( x 2 − 9 x + 18) dx = − + 18 x 3 0 3 3 2 0 4 = (72 − 162 + 108) = 24 3

Engineering Mathematics

9.118

Example 2: Evaluate

( yz dy dz + xz dz dx + xy dx dy ) over the surface of the S

sphere x 2 + y 2 + z 2 = 1 in the positive octant. Solution: (i)

∫∫ F ⋅ nˆ dS = yz dy dz + xz dz dx + xy dx dy S

F = yz iˆ + xz ˆj + xy kˆ

(ii) The given surface is the sphere x 2 + y 2 + z 2 = 1 . = x2 + y 2 + z 2

Let

nˆ = =

2 x iˆ + 2 y ˆj + 2 z kˆ 4x2 + 4 y2 + 4z2

= x iˆ + y ˆj + z kˆ

[∵x 2 + y 2 + z 2 = 1]

(iii) Let R be the projection of the sphere x 2 + y 2 + z 2 = 1 (in positive octant) on xy-plane (z = 0), which is the part of the circle x2 + y2 = 1 in the first quadrant. (iv) dS = dx dy = dx dy z nˆ ⋅ kˆ (v)

∫∫ ( yz dy dz + xz dz dx + xy dx dy) = ∫∫ F ⋅ nˆ dS S

S

( yz iˆ xz ˆj xy kˆ ) ( x iˆ R

=

(3xyz ) R

dx dy y ˆj z kˆ ) z

Fig. 9.29

dx dy = 3 xy dx dy z R

Putting x = r cosq, y = r sinq, equation of the circle x2 + y2 = 1 reduces to r = 1 and dx dy = r dr dq . Along the radius vector OP, r varies from 0 to 1 and in the first quadrant of the circle, q varies from 0 to 2

. π

1

∫∫ ( yz dy dz + xz dz dx + xy dx dy) = 3∫ 2 ∫ r cosθ ⋅ r sin θ ⋅ r dr dθ 0

0

S

= 3∫

π 2 0

1 sin 2θ 3 − cos 2θ dθ ⋅ ∫ r 3 dr = 0 2 2 2

3 3 = (− cos π + cos 0) = 16 8

π 2 0



r4 4

1

0

Vector Calculus

9.119

Example 3: Find the flux of F = iˆ - ˆj + xyz kˆ through the circular region S obtained by cutting the sphere x 2 + y 2 + z 2 = a 2 with a plane y = x.

∫∫ F ⋅ nˆ dS

Solution: Flux =

S

2 2 2 2 (i) Surface S is the intersection of the sphere x + y + z = a with a plane y = x, which is an ellipse 2x2 + z2 = a2. (ii) Normal to the ellipse 2x2 + z2 = a2 is also normal to the plane y = x. x y Let iˆ ˆj = nˆ = 2

(iii) Let R be the projection of the surface S on xz-plane, which is an ellipse 2x2 + z2 = a2 dx dz (iv) dS = nˆ ˆj = 2 dx d z (v)

Fig. 9.30 ⎛ iˆ − ˆj ⎞ ⎟ 2 dx dz 2 ⎠

∫∫ F ⋅ nˆ dS = ∫∫ (iˆ − ˆj + xyz kˆ) ⋅ ⎜⎝ S

R

2 dx dz

= R

Putting x =

a 2

r cos , z = a r sin , equation of the ellipse 2x2 + z2 = a2 reduces to

r = 1 and dx dz =

a2 2

r dr d

Along the radius vector OP, r varies from 0 to 1 and for complete ellipse, q varies from 0 to 2p.

∫∫ F ⋅ nˆ dS = 2∫



0

S

or

Hence,



1

0

a2 2

r d r dθ =

2a 2 r 2 2 2

1

θ 0

2π 0

1 = 2 a 2 ⋅ ⋅ 2π = 2 π a 2 2

⎡ ⎤ ⎢ ⎥ 2 2 y ⎢ Area of the ellipse x ⎥ ˆ d 2 d d 2 1 F ⋅ n S = x z = + = 2 2 ∫∫S ∫∫R ⎢ ⎥ a ⎛ a ⎞ ⎢ ⎥ ⎜ ⎟ ⎢⎣ ⎥⎦ ⎝ 2⎠ a = 2⋅ ⋅ a = 2 a2 2 Flux = 2 a 2

Engineering Mathematics

9.120

Example 4: Evaluate

∫∫ F ⋅ nˆ dS where F = 3 y iˆ + 2 z ˆj + x

2

yz kˆ and S is the sur-

S

face y2 = 5x in the positive octant bounded by the planes x = 3 and z = 4. Solution: (i) The given surface is y2 = 5x. y 2 5x Let 5 iˆ 2 y ˆj = nˆ = 25 + 4 y 2 (ii) Let R be the projection of the surface y2 = 5x (in the positive octant) bounded by the planes x = 3 and z = 4 in xz-plane. dx dz (iii) dS = nˆ ⋅ ˆj =

25 + 4 y 2 dx dz 2y

Fig. 9.31

(iv) In the region R, x varies from 0 to 3 and z varies from 0 to 4. ⎛ −5 iˆ + 2 y ˆj ⎞ ⎛ 25 + 4 y 2 ⎞ 2 ∫∫S F ⋅ nˆ dS = ∫∫R (3 y iˆ + 2 z ˆj + x yz kˆ) ⋅ ⎜⎜ 25 + 4 y 2 ⎟⎟ ⎜⎜ 2 y ⎟⎟ dx dz ⎝ ⎠⎝ ⎠ 1 dx dz 1 4 3 = ∫∫ ( −15 y + 4 yz ) = ∫ ∫ ( −15 + 4 z ) dx dz 2R 2 z =0 x =0 y

(

)

1 4 1 4 3 3 −15 x 0 + 4 z x 0 dz = ∫ (−45 + 12 z ) dz 2 ∫0 2 0 4 1 1 = −45 z + 6 z 2 = (−180 + 96) 0 2 2 = −42 =

Exercise 9.8

Evaluate the following integrals: 1. F ⋅ nˆ dS , where F = ( x + y 2 ) iˆ

∫∫ S

−2 x ˆj + 2 yz kˆ and S is the surface of the plane 2x + y + 2z = 6 in the first octant. [ Ans. : 81]

2.

∫∫ F ⋅ nˆ dS , where F = 2 xy iˆ + yz

2

ˆj + xzkˆ

S

and S is the surface

of the parallelepiped 0 x 1, 0 y 2 and 0

z

3.

[ Ans. : 33]

Vector Calculus

3.

∫∫ F ⋅ nˆ dS , where F = x iˆ + ( z

2

9.121

F = x iˆ + y ˆj + z kˆ.

− zx) ˆj − xy kˆ

S

and S is the triangular surface with vertices (2, 0, 0), (0, 2, 0) and (0, 0, 4). Ans.: 4.

22 3

Ans.: 72

20

11

6. Find the flux of the vector field F x iˆ y ˆj x 2 y 2 1 kˆ through the outer side of the hyper-

2 ∫∫ ∇× F ⋅ nˆ dS , where F = y iˆ + y ˆj − xzkˆ

x 2 y 2 1 bounded by boloid z the planes z = 0 and z = 3.

S

and S is the upper half of

the sphere x 2 + y 2 + z 2 = a 2 .

Ans. : 2 3

[ Ans. : 0 ] 5. Find the flux of the vector field F through the portion of the sphere x 2 + y 2 + z 2 = 36 lying between the planes z = 11 and z = 20 where

7. Find the flux of the vector field F 2 y iˆ z ˆj x 2 kˆ across the surface of the parabolic cylinder y2 = 8x in the first octant bounded by the planes y = 4 and z = 6. [ Ans. :132]

9.19 VOLUME INTEGRAL If V be a region in space bounded by a closed surface S, then the volume integral of a vector point function F is F dV . V

Component form of Volume Integral F = F1 iˆ + F2 ˆj + F3 kˆ

If

( F1 iˆ + F2 ˆj + F3 kˆ) dx dy d z

F dV = V

V

= iˆ

F1 dx dy d z + ˆj

F2 dx dy d z + kˆ

F3 dx dy d z

dV , where f is a scalar function.

Another type of volume integral is V

FdV where F = x iˆ + y ˆj + 2 z kˆ and V is the volume

Example 1: Evaluate V

enclosed by the planes x = 0, y = 0, y = a, z = b2 and the surface z = x2. Solution: (i) V is the volume of the cylinder in positive octant with base as OAB and bounded between the planes y = 0 and y = a. y varies from 0 to a.

Engineering Mathematics

9.122

Fig. 9.32 (ii) Along the vertical strip PQ, z varies from x2 to b2 and in the region OAB, x varies from 0 to b. b2

b

∫∫∫ F dV = ∫ ∫ ∫ x=0 z = x

V

=∫

b

=∫

b

0

0

∫ ∫

b2 x2

b2 x

2

2

a y =0

( x iˆ + y jˆ + 2 z kˆ ) dx dy dz

a ⎛ ⎞ y2 a a ˆ ˆ + 2 zkˆ y 0 ⎟ dz dx ⎜ xi y 0 + j 2 0 ⎝ ⎠ 2 ⎛ˆ ˆj a + kˆ 2 za ⎞ dz dx ixa + ⎜⎝ ⎟⎠ 2

2 2 2 b⎛ ˆ z b 2 + ˆj a z b 2 + kˆ a z 2 = ∫ ⎜ ixa x x 0 2 ⎝

⎞ ⎟ dx ⎠

b2 x

2

b⎡ ⎤ a2 = ∫ ⎢iˆ xa (b 2 − x 2 ) + ˆj (b 2 − x 2 ) + kˆ a (b 4 − x 4 ) ⎥ dx 0 2 ⎣ ⎦

b2 x2 iˆ a 2 b4 iˆ a 2 =

x4 4 b4 4

2 ˆj a b 2 x 2 2 ˆj a b3 2

b3 3

x3 3

kˆ a b 4 x

kˆ a b5

x5 5

b

0

b5 5

ab 4 ˆ a 2 b3 ˆ 4ab5 ˆ i+ j+ k. 4 3 5

(∇ × F ) dV , where F = (2 x 2 - 3 z ) iˆ - 2 xy ˆj - 4 x kˆ

Example 2: Evaluate V

and V is the closed region bounded by the planes x = 0, y = 0, z = 0 and 2x + 2y + z = 4.

Vector Calculus

Solution: (i)

F 2x



ˆj



x

y 2 xy

z 4x

2

iˆ (0 0)

3z

9.123

ˆj ( 4 3) kˆ ( 2 y 0)

ˆj 2 y kˆ

Fig. 9.33 (ii) Along the elementary volume PQ, z varies from 0 to 4 2x 2y. Along the vertical strip P'Q', y varies from 0 to 2 x and in the region, x varies from 0 to 2. 2

2− x

4− 2 x − 2 y

0

0

0

∫∫∫ (∇ × F ) dV = ∫ ∫ ∫ V

=∫

2

=∫

2

0

0



2− x



2− x

0

0

( jˆ − 2 y kˆ ) dx dy dz

4−2 x −2 y ( jˆ − 2 y kˆ ) z 0 dy dx = ∫

2

0



2− x

0

( jˆ − 2 y kˆ ) ( 4 − 2 x − 2 y ) dy dx

⎡⎣ ( 4 − 2 x − 2 y ) jˆ − 2 ( 4 − 2 x) y kˆ + 4 y 2 kˆ ⎤⎦ dy dx

2− x ⎡ ⎫⎪ ⎤ ⎧ 2 2− x 2− x y3 2− x ˆj − ⎪⎨2 ( 2 − x) y 2 = ∫ ⎢ ( 4 − 2 x) y 0 − y 2 −4 ⎬ kˆ ⎥ dx 0 0 0 3 ⎢ 0 ⎭⎪ ⎥⎦ ⎩⎪ ⎣ 2⎡ 4 ⎧ ⎫ = ∫ ⎢ 2 ( 2 − x) ( 2 − x) − ( 2 − x) 2 jˆ − ⎨2 ( 2 − x) ( 2 − x) 2 − ( 2 − x)3 ⎬ ˆk 0 3 ⎩ ⎭ ⎣

{

{

}

}

2⎡ 2 ( 2 − x )3 ˆ 2 ( 2 − x ) 4 ˆ ⎤ = ∫ ⎢ ( 2 − x) 2 jˆ − ( 2 − x)3 ˆk ⎥ dx = k j− ⋅ 0 −4 3 −3 3 ⎣ ⎦

8 8 8 = ˆj − kˆ = ( ˆj − kˆ ) 3 3 3

2

0

⎤ ⎥ dx ⎦

Engineering Mathematics

9.124

Exercise 9.9 Evaluate the following integrals:

∫∫∫ (∇ ⋅ F ) dV where

1.

3.

∫∫∫ f dV where f

= 45 x 2 y and V is

V

V

the region bounded by the planes 4x + 2y + z = 8, x = 0, y = 0, z = 0.

2 2 F = 2 x 2 y iˆ − y jˆ +4 xz kˆ and V is region in the first octant bounded by the cylinder y2 + z2 = 9 and the plane z = 2. [ Ans. :180 ]

[ Ans. :128] 4.

∫∫∫ ∇ × F dV where F = ( x + 2 y) iˆ V

∫∫∫ F dV where F = 2 xz iˆ − x ˆj + y

2.

2

−3 z jˆ + x kˆ and V is the closed region in the first octant bounded by the plane 2x + 2y + z = 4. 8 Ans. : (3 iˆ ˆj 2 kˆ) 3



V

and V is the region bounded by the surfaces x = 0, y = 0, y = 6, z = x2, z = 4. Ans. : 128 iˆ 24 ˆj 384 kˆ

9.20 STOKE’S THEOREM Statement: If S be an open surface bounded by a closed curve C and F be a continuous and differentiable vector function, then z



C

F ⋅ d r = ∫∫ ∇ × F ⋅ nˆ dS

S

S

c

where nˆ is the unit outward normal at any point of the surface S. Proof: Let F = F1 iˆ + F2 ˆj + F3 kˆ r = x iˆ + y ˆj + z kˆ

∫∫ ∇ × F ⋅ nˆ dS = ∫∫ ∇ × ( F iˆ + F 1

S

2

y R

x

c1

ˆj + F3 kˆ ) ⋅ nˆ dS

Fig. 9.34

S

= ∫∫ ( ∇ × F1 iˆ ) ⋅ nˆ dS + ∫∫ ( ∇ × F2 jˆ ) ⋅ nˆ dS S

S

... (1)

+ ∫∫ ( ∇ × F3 kˆ ) ⋅ nˆ dS S

Consider, ⎡⎛







∂ ⎞

∫∫ ( ∇ × F iˆ ) ⋅ nˆ dS = ∫∫ ⎢⎣ ⎜⎝ iˆ ∂x + jˆ ∂y + kˆ ∂z ⎟⎠ × F iˆ ⎥⎦ ⋅ nˆ dSS 1

S

1

S

⎛ ∂F ∂F ⎞ = ∫∫ ⎜ −kˆ 1 + ˆj 1 ⎟ ⋅ nˆ dS ∂y ∂z ⎠ S ⎝ ∂F ⎛ ∂F ⎞ = ∫∫ ⎜ 1 ˆj ⋅ nˆ − 1 kˆ ⋅ nˆ ⎟ dS ∂ ∂ z y ⎝ ⎠

... (2)

Vector Calculus

9.125

Let equation of the surface S be z = f (x, y), r = x iˆ + y ˆj + z kˆ = x iˆ + y ˆj + f ( x, y ) kˆ Differentiating partially w.r.t. y, r ˆ f = j + kˆ y y ˆ Taking dot product with n, r nˆ y

f ˆ k nˆ y

ˆj nˆ

... (3)

r is tangential and nˆ is normal to the surface S. y r nˆ 0 y

Substituting in Eq. (3), f ˆ k nˆ y

ˆj nˆ

0

f ˆ k nˆ y

ˆj nˆ

z ˆ k nˆ y

[∵z = f ( x, y )]

Substituting in Eq. (2), ⎡ ∂F1 ⎛ ∂z ⎞ ∂F1 ˆ ⎤ k ⋅ nˆ ⎥ dS ⎜ − kˆ ⋅ nˆ ⎟ − ∂z ⎝ ∂y ⎠ ∂y ⎦

∫∫ ( ∇ × F iˆ ) ⋅ nˆ dS = ∫∫ ⎢⎣ 1

S

S

⎛ ∂F ∂z ∂F ⎞ = − ∫∫ ⎜ 1 ⋅ + 1 ⎟ kˆ ⋅ nˆ dS ∂z ∂y ∂y ⎠ S ⎝

... (4)

Equation of the surface is z = f (x, y). F1 ( x, y, z ) = F1 [ x, y, f ( x, y )] = G ( x, y ) say

F1

Differentiating partially w.r.t. y, G y

F1 y

F1 z

z y

Substituting in Eq. (4),

∫∫ ( ∇ × F1 iˆ ) ⋅ nˆ dS = − ∫∫ S

S

∂G ˆ k ⋅ nˆ dS ∂y

x

z

y x

y

Fig. 9.35

Let R is the projection of S on xy-plane and dx dy is the projection of dS on xy-plane then kˆ ⋅ nˆ dS = dx dy Thus,

∂G

∫∫ ( ∇ × F iˆ ) ⋅ nˆ dS = − ∫∫ ∂y dx dy 1

S

R

=



C1

G dx

[Using Green’s theorem]

Engineering Mathematics

9.126

Since the value of G at each point (x, y) of C1 is same as the value of F1 at each point (x, y, z) of C and dx is same for both the curves C1 and C, we get ∫∫ ( ∇ × F1 iˆ ) ⋅ nˆ dS = ∫ F1 dx ... (5) C

S

Similarly, by projecting surface S on to yz and zx planes, ∫∫ ( ∇ × F2 jˆ ) ⋅ nˆ dS = ∫ F2 dy

... (6)

C

S

∫∫ ( ∇ × F kˆ ) ⋅ nˆ dS = ∫

and

3

C

F3 dz

... (7)

S

Substituting Eqs. (5), (6) and (7) in Eq. (1),

∫∫ ∇ × F ⋅ nˆ dS = ∫

C

( F1 dx + F2 dy + F3 dz ) =

S

∫ ( F ⋅ dr ) C

Note: If surfaces S1 and S2 have same bounding curve C, then

∫∫ ∇ × F ⋅ nˆ dS = ∫∫ ∇ × F ⋅ nˆ dS = ∫ S1

C

F ⋅ dr

S2

Example 1: Verify Stoke’s theorem for the vector field F = ( x 2 - y 2 ) iˆ + 2 xy ˆj in the rectangular region in the xy-plane bounded by the lines x = 0, x = a, y = 0, y = b. Solution: By Stoke’s theorem,

∫∫ ∇ × F ⋅ nˆ dS = ∫

C

F ⋅ dr

S

(i)

∇× F =



ˆj



∂ ∂x

∂ ∂y

∂ ∂z

x2 − y 2

2 xy

0

= iˆ (0) − ˆj (0) + kˆ ( 2 y + 2 y ) = 4 y kˆ

Fig. 9.36

Vector Calculus

9.127

(ii) Surface S is the rectangle ABCD in xy-plane. dx dy nˆ = kˆ and dS = = dx dy nˆ ⋅ kˆ (iii) Let R be the region bounded by the rectangle ABCD in xy-plane. Along the vertical strip PQ, y varies from 0 to b and in the region R, x varies from a to a. ∫∫ ∇ × F ⋅ nˆ dS = ∫∫ 4 y kˆ ⋅ kˆ dx dy S

R

b

= 4∫

a x =− a



b y =0

y dy dx = 4 ∫

a

−a

y2 dx 2 0

a

= 2b 2 x − a = 4ab 2

... (1)

(iv) Let C be the boundary of the rectangle ABC D.



C

F ⋅ dr =



AB

F ⋅ dr +



BC ′

F ⋅ dr +



F ⋅ dr +

C ′D



DA

F ⋅ dr

(a) Along AB : y = 0, dy = 0 x varies from a to a. AB

F dr

( x2

AB a

=

a

y 2 ) dx 2 xy dy

x 2 dx =

x3 3

a

a

2a 3 = 3 (b) Along BC : x = a, dx = 0 y varies from 0 to b.



BC ′

F ⋅ dr = ∫

BC ′

⎡⎣( x 2 − y 2 ) dx + 2 xy dy ⎤⎦

b

= ∫ 2ay dy = 2a 0

y2 2

b

0

= ab (c) Along C D : y = b, dy = 0 x varies from a to a. 2



C ′D

F ⋅ dr = ∫

C ′D

=∫

−a

a

⎡⎣( x 2 − y 2 ) dx + 2 xy dy ⎤⎦ –a

x3 ( x − b ) dx = − b2 x 3 a 2

2

2a 3 + 2ab 2 3 (d) Along DA : x = a, dx = 0 y varies from b to 0. =−

... (2)

Engineering Mathematics

9.128



F ⋅ d r = ∫ ⎡⎣( x 2 − y 2 ) dx + 2 xy dy ⎤⎦ DA

DA

0

= ∫ (−2ay ) dy b

y2 2a 2

0

ab 2 b

Substituting in Eq. (2), C

F dr

2a 3 3 = 4ab2

ab 2

2a 3 3

2ab 2

ab 2 ... (3)

From Eqs. (1) and (3),

∫∫ ∇× F ⋅ nˆ dS = ∫

C

F ⋅ d r = 4ab 2

S

Hence, Stoke’s theorem is verified. Example 2: Verify Stoke’s theorem for F = ( x + y ) iˆ + ( y + z ) ˆj - x kˆ and S is the surface of the plane 2 x + y + z = 2 which is in the first octant. Solution: By Stoke’s theorem,

∫∫ ∇ × F ⋅ nˆ dS = ∫

C

F ⋅ dr

S

(i)

F x



ˆj



x

y

z x

y

iˆ (0 1)

y

z

ˆj ( 1 0) kˆ(0 1)

iˆ ˆj kˆ (ii) Let = 2 x + y + z Fig. 9.37 2iˆ ˆj kˆ = nˆ = 4 +1+1 2iˆ + ˆj + kˆ = 6 (iii) Projection of the plane 2 x + y + z = 2 on xy -plane ( z = 0) is the triangle OAB bounded by the lines x = 0, y = 0, 2x + y = 2. dx dy (iv) dS = = 6 dx dy nˆ ⋅ kˆ (v) Let R be the region bounded by the triangle OAB in xy-plane. Along the vertical strip PQ, y varies from 0 to (2 2x) and in the region R, x varies from 0 to 1.

Vector Calculus

∫∫ ∇ × F ⋅ nˆ dS =∫∫ (−iˆ + ˆj − kˆ) ⋅ S

( 2iˆ + ˆj + kˆ )

=∫

1

0



2−2 x

0 1

6 dx dy

6

R

9.129

( −2 + 1 − 1) dx dy 2−2 x

= −2 ∫ y 0 0

dx

1

2 (2 2 x) dx

x2 2

4x

0

1

0

1 4 1 2

Fig. 9.38

∫∫ ∇ × F ⋅ nˆ dS = − 2

... (1)

S

or

∫∫ ∇ × F ⋅ nˆ dS = − 2∫∫ dx dy S

R

= −2( Area of ΔOAB) 1 = −2 ⋅ ⋅1 ⋅ 2 2 = −2 (vi) Let C be the boundary of the triangle ABC .

F dr ( x

y ) dx ( y z ) dy x dz

∫ F ⋅ dr =∫

AB

F ⋅ dr + ∫

BC ′

F ⋅ dr + ∫

C ′A

F ⋅ dr

(a) Along AB : z = 0, y = 2 2x dz = 0, dy = –2dx x varies from 1 to 0. ∫ F ⋅ d r = ∫ [( x + y)dx + ( y + z )dy − x dz ] AB

AB

= ∫ [( x + 2 − 2 x )dx + ( 2 − 2 x )( −2dx ) ] 0

1

0

= ∫ ( 3 x − 2 ) dx 1

x2 3 2 1 = 2 (b) Along BC : x = 0, dx = 0, y varies from 2 to 0.

0

2x 1

y+z=2 dz = dy

3 2 2

... (2)

Engineering Mathematics

9.130



F ⋅ dr = ∫

BC ′

BC ′

[( x + y)dx + ( y + z )dy − x dz ]

0

= ∫ 2 dy = 2 y 2 0

2

= −4 (c) Along C A : y = 0, 2x + z = 2 dy = 0, dz = dx x varies from 0 to 1.



C ′A

F ⋅ dr = ∫

C ′A

[( x + y)dx + ( y + z )dy − x dz ]

= ∫ [ x dx − x(−2 dx) ] = ∫ 3 x dx 1

1

0

0

x2 =3 2

1

= 0

3 2

Substituting in Eq. (2), C

1 3 4 2 2 2

F dr

... (3)

From Eqs. (1) and (3),

∫∫ ∇ × F ⋅ nˆ dS = ∫

C

F ⋅ d r = −2

S

Hence, Stoke’s theorem is verified. Example 3: Verify Stoke’s theorem for F = x z iˆ + y ˆj + x y 2 kˆ where S is the surface of the region bounded by y = 0, z = 0 and 4 x + y + 2 z = 4 which is not included in the yz-plane. Solution: By Stoke’s theorem, ∫∫ ∇ × F ⋅ nˆ dS = S



C

F ⋅ dr

z

C ′(0, 0, 2)

B (0, 4, 0)

y

O A (1, 0, 0) x

Fig. 9.39

Vector Calculus

(i)

F



ˆj



x xz

y y

z xy 2

iˆ(2 xy 0) ˆj ( y 2 2 xy iˆ ( x y 2 ) ˆj

9.131

x) kˆ(0 0)

(ii) Surface S consists of three surfaces, y = 0, z = 0 and 4x + y + 2z = 4.

∫∫ ∇ × F ⋅ nˆ dS = ∫∫ ∇ × F ⋅ nˆ dS + ∫∫ ∇ × F ⋅ nˆ dS + ∫∫ ∇ × F ⋅ nˆ dS S

S1

S2

... (1)

S3

(a) Surface S1 ( OAC'): y = 0, nˆ = − ˆj and dS = dx dz. Let R1 be the region bounded by the OAC'. Along the vertical strip PQ 1 1, z varies from 0 to 2 2x and in the region R1, x varies from 0 to 1.

∫∫ ∇ × F ⋅ nˆ dS = ∫∫ −( x − y S1

2

z C ′(0, 0, 2) Q1

) dx dz

2x + z = 2

R1

=∫

1

0



2−2 x

0

1

2−2 x

= −∫ x z 0 0

1 0

2 3

P1

O

A(1, 0, 0)

x2

2 x3 3

1

y 0

B (0, 4, 0)

1 3

4x + y = 4 Q2

(b) Surface S2 ( OAB): z = 0, nˆ kˆ and dS = dx dy. Let R2 be the region bounded by the P2 A(1, 0, 0) O OAB. Along the vertical strip P2 Q2, y varies from 0 to 4 4x and in the region Fig. 9.41 R2, x varies from 0 to 1. 2 ∫∫ ∇ × F ⋅ nˆ dS = ∫∫ ⎡⎣2 xy iˆ + ( x − y ) ˆj ⎤⎦ ⋅ (– kˆ) dx dy = 0 S2

R2

(c) Surface S3 (4 x + y + 2 z = 4) : Let

= 4x + y + 2z

x

Fig. 9.40

dx

x(2 2 x) dx

1

[∵ y = 0]

( − x) dx dz

x

Engineering Mathematics

9.132

nˆ =

= =

4iˆ

ˆj 2kˆ

16 + 1 + 4 4iˆ + ˆj + 2kˆ 21

Projection of the plane 4 x + y + 2 z = 4 on xy-plane is the triangle OAB. dS =

dx dy 21 dx dy = ˆ 2 nˆ ⋅ k

Let R3 be the region bounded by the OAB. Along the vertical strip P2Q2, y varies from 0 to 4 4x and x varies from 0 to 1. ˆ⎞ ⎛ ˆ ˆ ˆ + ( x − y 2 ) ˆj ⎤ ⋅ ⎜ 4i + j + 2k ⎟ 21 dx dy ⎡ ˆ ∇ × F ⋅ n S = 2 xy i d ∫∫S ∫∫R ⎣ ⎦ ⎜ 21 ⎟⎠ 2 ⎝ 3 2 1 1 4−4 x = ∫ ∫ (8 xy + x − y 2 ) dy dx 2 0 0 y2 y3 1 1 8 x + xy − ∫ 2 0 2 3

=

4−4 x

dx 0

=

1 1⎡ ( 4 − 4 x )3 ⎤ 2 4 4 4 4 4 x ( − x ) + x ( − x ) − ⎢ ⎥ dx 2 ∫0 ⎣ 3 ⎦

=

64 ⎞ 1 1 ⎛ 256 3 x − 196 x 2 + 132 x − ⎟ dx ⎜ 3 ⎠ 2 ∫0 ⎝ 3 1 256 x 4 x3 x2 196 132 2 3 4 3 2

1

64 x 3 0

1 64 196 64 66 2 3 3 3 1 = 3 Substituting in Eq. (1), 1

1

∫∫ ∇ × F ⋅ nˆ dS = − 3 + 0 + 3 = 0

... (2)

S

(iii) Since surface S does not include yz-plane, it is open on the yz-plane. ΔOBC ′ is the boundary of the surface S. Let C be the boundary of the ΔOBC ′ bounded by the lines y = 0, z = 0, y + 2z = 4. ... (3) F ⋅ dr = F ⋅ dr + F ⋅ dr + F ⋅ dr



C

F dr



C ′O

xz dx





BC ′

OB

2

y dy xy dz

y dy

[∵x = 0, dx = 0]

Vector Calculus z

(a) Along C O : y = 0 dy = 0 z varies from 2 to 0.



9.133

0

C ′O

F ⋅ d r = ∫ y dy = 0 2

C ′(0, 0, 2)

(b) Along OB : z = 0, d z = 0 y varies from 0 to 4. 4

F dr

0

y2 2

y dy

OB

y=0

4

0

BC ′

B(0, 4, 0)

O (0, 0, 0)

(c) Along BC': y = 4 2z, dy = 2 d z z varies from 0 to 2.



y + 2z = 4

8 y

z=0

Fig. 9.42 2

2

0

0

F ⋅ d r = ∫ y dy = ∫ ( 4 − 2 z ) ( −2d z) = −4 2 z −

z2 2

2

= −4( 4 − 2) = −8 0

Substituting in Eq. (3), C

F dr

0 8 8

... (4)

0

From Eqs. (2) and (4),

∫∫ ∇ × F ⋅ nˆ dS = ∫

C

F ⋅ dr = 0

S

Hence, Stoke’s theorem is verified. Example 4: Verify Stoke’s theorem for F = 4 y iˆ - 4 x ˆj + 3kˆ , where S is a disk of radius 1 lying on the plane z = 1 and C is its boundary. Solution: By Stoke’s theorem,

∫∫ ∇ × F ⋅ nˆ dS = ∫

C

F ⋅ dr

S

where S is the surface of the disk of radius 1 lying on the plane z = 1 and C is the circle x 2 + y 2 = 1. ˆj iˆ kˆ (i)

F

x 4y

y 4x

iˆ(0 0) 8kˆ

z 3 ˆj (0 0) kˆ( 4 4)

(ii) Since disc lies on the plane z = 1, parallel to xy-plane, nˆ = kˆ

Fig. 9.43

Engineering Mathematics

9.134

(iii) Projection of the disc in xy-plane is the circle x 2 + y 2 = 1 . (iv) dS = dx dy = dx dy | nˆ ⋅ kˆ | (v) Let R be the region bounded by the circle x 2 + y 2 = 1 in xy-plane. ∇ × F ⋅ nˆ dS = ( −8kˆ ) ⋅ kˆ dx dy

∫∫

∫∫

S

R

= −8∫∫ dx dy R

Putting x = r cosq , y = r sinq, equation of the circle x 2 + y 2 = 1 reduces to r = 1 and dx dy = r dr d . Along, the radius vector OA, r varies from 0 to 1 and for complete circle, q varies from 0 to 2p. 2π 1

∫∫ ∇ × F ⋅ nˆ dS = −8∫ ∫ 0

0

Fig. 9.44

r d r dθ

S

= −8 θ or

2π 0

r2 2

... (1)

1

0

∫∫ ∇ × F ⋅ nˆ dS = −8∫∫ dx dy S

R

= −8( Area of the circle) = −8 (1) 2 = −8 (vi) C is the boundary of the disc, i.e., circle x 2 + y 2 = 1 lying on the plane z = 1. F d r 4 y dx 4 x dy 3dz 4 y dx 4 x dy [∵z = 1, dz = 0] C

F dr

C

(4 y dx 4 x dy )

Parametric equation of the circle is x = cos ,

dx = − sin d ,

y = sin d y = cos d

For complete circle, q varies from 0 to 2p.



C

F ⋅ dr = ∫

2π 0

[ 4 sin θ (− sin θ dθ ) − 4 cosθ (cosθ dθ )]

= −4 ∫

2π 0

dθ = −4 θ

2π 0

... (2)

8 From Eqs. (1) and (2),

∫∫ ∇ × F ⋅ nˆ dS = ∫ S

Hence, Stoke’s theorem is verified.

C

F ⋅ d r = −8

Vector Calculus

9.135

Example 5: Verify Stoke’s theorem for F ( x 2 y 4) iˆ 3 xy ˆj (2 xz z 2 ) kˆ over the surface of the sphere x 2 + y 2 + z 2 = 16 above xy-plane. z

Solution: By Stoke’s theorem,

∫∫ ∇ × F ⋅ nˆ dS = ∫

C

F ⋅ dr

S

(i) iˆ

ˆj

kˆ O

F x2

y

x y z y 4 3 xy 2 xz z 2

ˆj (2 z 0) kˆ(3 y 1) 2 z ˆj (3 y 1)kˆ

iˆ(0 0)

x

Fig. 9.45

= x2 + y 2 + z 2

(ii) Let

nˆ = =

=

2 x iˆ + 2 y ˆj + 2 z kˆ 4x2 + 4 y 2 + z 2 x iˆ + y ˆj + z kˆ 4

⎡⎣∵x 2 + y 2 + z 2 = 16 ⎤⎦

(iii) Let R be the projection of the hemisphere x 2 + y 2 + z 2 = 16 on xy-plane (z = 0) which is a circle x 2 + y 2 = 16. (iv) dS =

dx dy | nˆ ⋅ kˆ |

=

4 dx d y z

(v)

∫∫ ∇ × F ⋅ nˆ dS S

⎛ x iˆ + y ˆj + z kˆ ⎞ 4 dx dy = ∫∫ ⎡⎣ −2 z ˆj + (3 y − 1)kˆ ⎤⎦ ⋅ ⎜ ⎟⎟ ⎜ 4 R ⎝ ⎠ z dx dy [ 2 zy (3 y 1) z ] z R ( y 1) dx dy R

Fig. 9.46

Engineering Mathematics

9.136

Putting x = r cosq, y = r sinq, equation of the circle x 2 + y 2 = 16 reduces to r = 4 and dx dy = r dr dq . Along the radius vector OA, r varies from 0 to 4 and for complete circle, q varies from 0 to 2p. 2π

4

0

0

∫∫ ∇ × F ⋅ nˆ dS = ∫ ∫

( r sin θ − 1) r dr dθ

S

=

r3 3

4

− cos θ

2π 0



0

r2 2

4

θ

2π 0

0

3

=−

4 16 (cos 2π − cos 0) − ⋅ 2π 3 2 16p

... (1)

(vi) The boundary C of the hemisphere S is the circle x + y = 4 in xy-plane (z = 0). 2

F dr

C

F dr

( x2

y 4)dx 3 xy dy (2 xz z 2 )dz

( x2

y 4) dx 3 xy dy

C

(x

2

2

[∵z = 0, dz = 0]

y 4) dx 3 xy dy

Parametric equation of the circle x 2 + y 2 = 4 is x = 4 cos , dx 4sin d ,

y = 4sin dy 4 cos d

For the complete circle, q varies from 0 to 2p.



C

2π F ⋅ d r = ⎡ ∫ (16 cos 2 θ + 4 sin θ − 4)(−4 sin θ dθ ) + (3 ⋅ 4 cosθ ⋅ 4 sin θ )(4 cosθ dθ ) ⎤ ⎢⎣ 0 ⎥⎦ 2π

= ∫ (−64 cos 2 θ sin θ − 16 sin 2 θ + 16 sin θ + 192 cos 2 θ sin θ ) dθ 0

⎡∵ 2 a f (2a − x) = 0, if f (2a − x) = − f ( x) ⎤ ⎣⎢ ∫0 ⎦⎥



= ∫ −16 sin 2 θ dθ 0

2π ⎛ 1 − cos 2θ ⎞ sin 2θ = −16 ∫ ⎜ ⎟ dθ = −8 θ − 0 2 2 ⎝ ⎠ = −16π

From Eqs. (1) and (2),

∫∫ ∇ × F ⋅ nˆ dS = ∫ S

Hence, Stoke’s theorem is verified.

C

F ⋅ d r = −16



0

... (2)

Vector Calculus

9.137

Example 6: Verify Stoke’s theorem for F = yiˆ + zjˆ + xkˆ over the surface x 2 + y 2 = 1 - z , z > 0. Solution: By Stoke’s theorem,

∫∫ ∇ × F ⋅ nˆ dS = ∫ F ⋅ d r C

S

(i)

F



ˆj



x y

y z

z x

ˆj (1 0) kˆ(0 1) ˆj kˆ)

iˆ(0 1) (iˆ

x2

(ii) Let

y2

z

nˆ Fig. 9.47 2 x iˆ 2 y ˆj kˆ 4 x2

4 y2 1

(iii) Let R be the projection of the surface x 2 is a circle x 2 y 2 1. (iv) dS =

1

z on xy-plane (z = 0) which

dx dy nˆ ⋅ kˆ 4x2

(v)

y2

4 y 2 1 dx dy

∫∫ ∇ × F ⋅ nˆ dS S

ˆ ˆ ˆ ˆ + jˆ + kˆ ) ⋅ ( 2 x i + 2 y j + k ) 4 x 2 + 4 y 2 + 1 dx dy − ( i ∫∫R 4x2 + 4 y 2 + 1 = − ∫∫ ( 2 x + 2 y + 1) dx dy

Fig. 9.48

R

Putting x = r cos , y = r sin , circle x 2 + y 2 = 1 reduces to r = 1 and dx dy = r dr d Along the radius vector OA, r varies from 0 to 1 and for complete circle, q varies from 0 to 2p . 2π

1

0

0

∫∫ ∇ × F ⋅ nˆ dS = −∫ ∫ (2r cos θ + 2r sin θ + 1)r dr dθ S

= −∫



0

1 1 ⎡ r2 ⎤ r3 ⎢ 2(cos+ sin θ ) ⎥ dθ + 3 0 2 0⎥ ⎢⎣ ⎦

Engineering Mathematics

9.138

= −∫

2π 0

1⎤ ⎡2 ⎢ 3 (cos θ + sin θ ) + 2 ⎥ dθ ⎣ ⎦ 2π

2 1 (sin θ − cos θ ) + θ 3 2

=−

2 (sin 2 3

cos 2

0

sin 0 cos 0)

= –p

... (1)

(vi) The boundary C of the surface x xy-plane (z = 0). F dr

2

y

2

y dx z dy

1 z is the circle x

y

2

1 in

x dz

[∵ z

y dx F dr

2

0, dz

0]

y dx

C

C

Parametric equation of the circle x 2 + y 2 = 1 is

x dx

y sin dy cos d

cos , sin d ,

For the complete circle, q varies from 0 to 2p.

∫ F ⋅ dr = ∫



0

C

=−

2π ⎛ 1 − cos 2θ sin θ (− sin θ dθ ) = − ∫ ⎜ 0 ⎝ 2

sin 2θ 1 θ− 2 2



0

⎞ ⎟ dθ ⎠

sin 4π 1⎛ ⎞ = − ⎜ 2π − − 0 ⎟ = −π 2⎝ 2 ⎠

... (2)

From Eqs. (1) and (2),

∫∫ ∇ × F ⋅ nˆ dS = ∫ F ⋅ d r = – C

S

Hence, Stoke’s theorem is verified. Example 7: Evaluate by Stoke’s theorem curve x 2 + y 2 = 4, z = 2.

(e x dx + 2 y dy - dz ) where C is the

C

Solution: By Stoke’s theorem,

∫∫ ∇ × F ⋅ nˆ dS = ∫ F ⋅ d r S

∫∫ ∇ × F ⋅ nˆ dS = ∫ (e dx + 2 y dy − dz ) x

S

C

C

... (1)

Vector Calculus

9.139

F = e x iˆ + 2 y ˆj − kˆ

∇× F =



ˆj



∂ ∂x ex

∂ ∂y 2y

∂ ∂z −1

= iˆ(0 − 0) − ˆj (0 − 0) + kˆ(0 − 0) =0 Substituting in Eq. (1), (e x dx 2 y dy dz )

C

Example 8: Evaluate

0

∫∫ ( ∇ × F ) ⋅ nˆ dS

for vector field

S

F = (2 y 2 + 3 z 2 - x 2 )iˆ + (2 z 2 + 3 x 2 - y 2 ) ˆj + (2 x 2 + 3 y 2 - z 2 )kˆ over the part of the sphere x 2 + y 2 + z 2 - 2ax + az = 0 cut off by the plane z = 0. Solution: By Stoke’s theorem,

∫∫ ∇ × F ⋅ nˆ dS = ∫ F ⋅ d r

... (1)

C

S

(i) F d r

(2 y 2

3z 2

x 2 )dx (2 z 2

3x 2

y 2 )dy (2 x 2

3y2

z 2 )dz

(ii) Let C is the boundary of the part of the sphere x 2 + y 2 + z 2 – 2ax + az = 0 cut off by the plane z = 0, which is a circle x 2 + y 2 − 2ax = 0, ( x − a) 2 + y 2 = a 2 . y

A (2a, 0) x

Fig. 9.49

Engineering Mathematics

9.140

Parametric equation of the circle x a dx

a cos , a sin d ,

y a sin dy a cos d

For the complete circle, q varies from 0 to 2p.



C

[∵ z = 0, dz = 0]

∫ ⎡⎣(2 y − x )dx + (3x − y )dy ⎤⎦ = ∫ ⎡⎣{2a sin θ − ( a + a cos θ ) } (−a sin θ dθ )

F ⋅ dr =

2

2

2

2

C



2

2

2

0

+ {3 ( a + a cos θ ) − a 2 sin 2 θ } (a cos θ dθ ).⎤⎦ 2



= a 3 ∫ (−2 sin 3 θ + sin θ + sin θ cos 2 θ + 2 cos θ sin θ 0

+ 3 cos θ + 3 cos3 θ + 6 cos 2 θ − sin 2 θ cos θ )dθ ⎡∵ 2 a f (θ )dθ = 0, if f (2a − θ ) = − f (θ ) ⎤ ⎢ ∫0 ⎥ 0 a ⎢ ⎥ 2 2 + 6 cos θ − sin θ cos θ )dθ ⎢ = 2 ∫ f (θ )dθ , if f (2a − θ ) = f (θ )⎥ 0 ⎣ ⎦ π

= 2a 3 ∫ (3 cos θ + 3 cos3 θ

p

[∵cos (p − q ) = − cosq ]

= 4a 3 ∫ 2 6 cos 2 q dq 0

⎛ 1 + cos 2q ⎞ = 24a 3 ∫ 2 ⎜ ⎟ dq 0 ⎝ 2 ⎠ p

sin 2θ = 12a θ + 2 3

π 2

⎛ π sin π − 0 ⎞ = 12a 3 ⎜ + ⎟ 2 ⎝2 ⎠

0

= 6π a 3 From Eq. (1), F nˆ ds

6 a3

S

Example 9: Evaluate by Stoke’s theorem

(4 y dx + 2 z dy + 6 y dz ) where C is C

2

2

the curve of intersection of the sphere x + y + z 2 = 6 z and the plane z = x + 3. Solution: By Stoke’s theorem,

∫∫ ∇ × F ⋅ nˆ dS = ∫ F ⋅ d r S

(i)

F dr

C

4 y dx 2 z dy 6 y dz

... (1)

Vector Calculus

F

F

9.141

(4 y iˆ 2 z ˆj 6 y kˆ) (iˆ dx 4 y iˆ 2 z ˆj 6 y kˆ iˆ

ˆj



x 4y

y 2z

z 6y

iˆ(6 2) 4iˆ 4kˆ

ˆj dy kˆ dz )

ˆj (0 0) kˆ(0 4)

(ii) Normal to the surface which is bounded by the curve of intersection of the sphere x 2 + y 2 + z 2 = 6 z and the plane z = x + 3 is also normal to the plane z x 3. x z

Let nˆ = =

∇ ∇ iˆ − kˆ

2 dS = dx dz (iii) Let C be the curve of intersection of x2 + y2 + z2 = 6z and z = x + 3 which is a circle x2 + z2 = 6z (since y = 0 on xz-plane). (iv) Let R be the region bounded by the circle x 2 z 2 6 z 0 with radius 3.

Fig. 9.50

∫∫ ∇ × F ⋅ nˆ dS = ∫∫ S

R

= 4 2 ∫∫ dx dz R

4+4 2

dx dz

Engineering Mathematics

9.142

Putting x r cos , z r sin , equation of the circle x 2 z 2 6 z reduces to r = 6 sinq and dx dy r dr d . Along the radius vector OP, r varies from 0 to 6 sinq and for the complete circle, q varies from 0 to p. π

6 sin θ

0

0

∫∫ ∇ × F ⋅ nˆ dS = 4 2 ∫ ∫

r dr dθ

S

= 4 2∫

π

0

36 2

0

r2 2

6 sin θ

dθ = 0

(1 cos 2 )d sin 2 2

36 2

4 2 π 36 sin 2 θ dθ 2 ∫0 sin 2 2

36 2 36

0

2

∫∫ ∇ × F ⋅ nˆ dS = 4 2 ∫ ∫ dx dz

or

S

R

= 4 2 ( Area of the circle C ) = 4 2 ( ⋅ 32 ) = 36

2

F d r 36

2

From Eq. (1), C

Example 10: Using Stoke’s theorem, find the work done in moving a particle once around the perimeter of the triangle with vertices at (2, 0, 0), (0, 3, 0) and (0, 0, 6) under the force field F = ( x + y )iˆ + (2 x - z ) ˆj + ( y + z )kˆ . Solution: Work done By Stoke’s theorem,

C

z

F dr

C (0, 0, 6)

∫ F ⋅d r = ∫∫ ∇ × F ⋅ nˆ dS C

S

Thus, work done = ∫∫ ∇ × F ⋅ nˆ dS S

where S is the surface of the Equation of the

O (0, 0, 0)

ABC .

A (2, 0, 0)

ABC is

x y z 2 3 6 3x 2 y z

1 x

6

Fig. 9.51

B (0, 3, 0) y

Vector Calculus

(i)

F x



ˆj



x

y

z

y 2x z

iˆ(1 1) 2iˆ kˆ (ii) Let nˆ

y

9.143

z

ˆj (0 0) kˆ(2 1)

= 3x + 2 y + z 3iˆ 2 ˆj kˆ 9 4 1 3iˆ 2 ˆj kˆ 14

(iii) Projection of ABC on xy-plane is the 3x + 2y = 6, x = 0. dS =

OAB bounded by the lines y = 0,

dx dy nˆ ⋅ kˆ

= 14 dx dy (iv) Let R be the region bounded by the OAB. Along the vertical strip PQ, y varies 6 3x from 0 to and in the region R, x varies from 0 to 2. 2 ⎛ 3iˆ + 2 ˆj + kˆ ⎞ ∫∫S ∇ × F ⋅ nˆ dS = ∫∫R (2iˆ + kˆ) ⋅ ⎜⎜ 14 ⎟⎟ 14 dx dy ⎝ ⎠ =∫

2

0

6− −3 x 2 0



2

7 dy dx = 7 ∫ y 0 0

6 −3 x 2

y

dx

2 ⎛ 6 − 3x ⎞ = 7∫ ⎜ ⎟ dx 0 ⎝ 2 ⎠

3x 2 = 7 3x − 4 or

B (0, 3, 0) 3x + 2y = 6 Q

2

= 21 0

∫∫ ∇ × F ⋅ nˆ dS = 7 ∫∫ dx dy S

R

= 7 ( Area of ΔOAB ) 1 = 7 ⋅ ⋅ 2 ⋅ 3 = 21 2

O (0, 0, 0)

P

Fig. 9.52

A (2, 0, 0) x

9.144

Engineering Mathematics

Example 11: Evaluate

∫∫ ∇ × F ⋅ nˆ dS

using Stoke’s theorem, where

S

F = y z iˆ + (2 x + y - 1) ˆj + ( x 2 + 2 z ) kˆ and S is the surface of intersection of the cylinders x 2 + y 2 = a 2 and x 2 + z 2 = a 2 in the positive octant. Solution: By Stoke’s theorem,

∫∫ ∇ × F ⋅ nˆ dS = ∫

C

F ⋅ dr

... (1)

S

Fig. 9.53 (i) F ⋅ d r = yz dx + (2 x + y − 1)dy + ( x 2 + 2 z )dz (ii) C is EABDE which is the boundary of the surface of intersection of the cylinders x 2 y 2 a 2 and x 2 z 2 a 2 in the positive octant.

F dr C

EA

F dr

AB

F dr

(a) Along EA: z 0, x 2 dz = 0 Putting x a cos ,

dx

EA

F dr

y2

dy 2

DE

... (2)

F dr y

a2

y

a sin d ,

Along EA, q varies from 0 to



BD

A (0, a)

a sin a cos d

E (a, 0) x

.

F ⋅ d r = ∫ ⎡⎣ yz dx + (2 x + y − 1)dy + ( x 2 + 2 z ) dz ⎤⎦ EA π

= ∫ 2 (2 a cos θ + a sin θ − 1)a cos θ dθ 0

π 2 0

Fig. 9.54

= ∫ (2a cos θ + a sin θ cos θ − a cos θ )dθ = 2a 2 ⋅

2

2

2

1 ⎛3 1⎞ 2 1 B ⎜ , ⎟ + a ⋅ B (1, 1) − a sin θ 2 2 ⎝2 2⎠

π 2 0

Vector Calculus

3 1 a 2 2 2 1 a2 2 2 a a2 2 2 2

(b) Along AB:

a2 1 1 a sin 2 2 2 2 a a 2

9.145

sin 0

a

x 0, y a dx = 0, dy = 0

z varies from 0 to a.



AB

F ⋅ d r = ∫ ⎡⎣ yz dx + (2 x + y − 1)dy + ( x 2 + 2 z )dz ⎤⎦ AB a

a2

0

0

= ∫ 2 z dz = z 2 (c) Along BD:



BD

= a2

x 0, z a dx = 0, dz = 0 y varies from a to 0. F ⋅ d r = ∫ ⎡⎣ yz dx + (2 x + y − 1)dy + ( x 2 + 2 z )dz ⎤⎦ BD 0

y2 a2 = ∫ ( y − 1)dy = − y = − + a. a 2 2 a 0

(d) Along DE: Putting x dx

y dy

0, 0

a cos , a sin d ,

z

x2

z2

a2

a sin

dz

a cos d z

Along DE, q varies from



DE

2

to 2p.

D (0, a)

F ⋅ d r = ∫ ⎡⎣ yz dx + (2 x + y − 1)dy + ( x 2 + 2 z )dz ⎤⎦ DE

E (a, 0)



= ∫π (a 2 cos 2 θ + 2a sin θ ) (a cos θ dθ )

x

2



= a 2 ∫π (a cos3 θ + 2 sin θ cos θ )dθ 2

2π ⎡ a ⎤ = a 2 ∫π ⎢ (cos 3θ + 3 cos θ ) + sin 2θ ⎥ dθ 4 ⎦ 2 ⎣

a 3 sin 3θ = + 3 sin θ 4 3



cos 2θ +a − 2



2

π 2

π 2

Fig. 9.55

Engineering Mathematics

9.146

=

2 a 3 ⎛ sin 6 1 3 ⎞ a + − − − 3 2 sin sin 3 sin (cos 4 − cos ) ⎜ ⎟ 4⎝ 3 3 2 2⎠ 2

=

a3 ⎛ 1 ⎞ a 2 2a 3 − a2 ⎜ − 3 ⎟ − (1 + 1) = − 4 ⎝3 ⎠ 2 3

Substituting in Eq. (2), a2 a2 F dr a a2 2 2 C

a2 2

a

2a 3 3

a2

a2 2

2a 3 3

From Eq. (1),

∫∫ ∇ × F ⋅ nˆ dS = S

a 2 2a 3 . − 2 3

Exercise 9.10 (I) Verify Stoke’s theorem for the following vector point functions: 2 ⎛ ⎞ ⎛ 2 ⎞ 1. F = ⎜ x 3 + yz ⎟ iˆ + ⎜ xz + y 2 ⎟ ˆj 2 ⎠ ⎝ ⎝ 2 ⎠ + ( x y z )kˆ over the surface S of the cube 0 ≤ x ≤ 3, 0 ≤ y ≤ 3, 0 ≤ z ≤ 3. [ Ans. : 0] 2 ˆ ˆ ˆ 2. F x z i y j y x k over the sur-

z4 ˆ i 4 x ˆj ( x z 3 z 2 )kˆ 4 over the upper half surface S of the sphere x 2 y 2 z 2 1.

3. F

x3

[ Ans. : 4p ] 4. F

face S of the tetrahedron bounded by the planes y = 0, z = 0 and 4 x y 2 z 4 above yz-plane.

( x2

y 2)iˆ 2 xy ˆj 4 ze x kˆ

over the surface S of the paraboloid z 9 ( x 2 y 2 ) above xy-plane.

[ Ans. : −9p ]

[ Ans. : 0] (II) Evaluate the following integrals using Stoke’s theorem: 1.

∫∫ ∇ × F ⋅ nˆ dS where F = ( x

2

the plane z = 2 and C is its boundary traversed in the clock-wise direction. [ Ans. : 20p ]

+ y + z ) iˆ

S

+ 2 xy ˆj − (3 xyz + z 3 )kˆ and S is the surface of the hemisphere x 2 y 2 z 2 9 above xy-plane.

[ Ans. : −9p ] 2.

∫∫ ∇ × F ⋅ nˆ dS where F = 3 y iˆ − xz ˆj

3.

( y dx z dy x dz ) where C is the C curve of intersection of the sphere x 2 y 2 z 2 a 2 and the plane x z a.

S

+ yz 2 kˆ and S is the surface of the 2 2 2 z bounded by paraboloid x y

Ans. :

a2 2

.

Vector Calculus

9.147

(III) For the vector field: y

1. F

x

2

y

2



x x

2

y2

ˆj over the surface of the sphere x 2

y2

z2

1 above

xy-plane evaluate (i)

∫∫ ∇ × F ⋅ nˆ dS

(ii) C

S

F d r , where

since

C

F ⋅ dr

S

C is the boundary of S. Are the result compatible with Stoke’s theorem?

[ Ans. : (i) 0 (ii) 2p

∫∫ ∇ × F ⋅ nˆ dS ≠ ∫

Also in this case Stoke’s theorem cannot be applied since at (0, 0) which is inside C, F is neither continuous nor differentiable].

(iii) no

9.21 GAUSS DIVERGENCE THEOREM Statement: If F be a vector point function having continuous partial derivatives in the region bounded by a closed surface S, then ∫∫∫ ∇ ⋅ FdV = ∫∫ F ⋅ nˆ dS V

S

where nˆ is the unit outward normal at any point of the surface S. Proof: Let F F1 iˆ F2 ˆj F3 kˆ ⎛ ∂

∂ ⎞



∫∫∫ ∇ ⋅ F dV = ∫∫∫ ⎜⎝ iˆ ∂x + ˆj ∂y + kˆ ∂z ⎟⎠ ⋅ ( F iˆ + F 1

V

2

ˆj + F3 kˆ )dx dy dz

V

V

F1 x

F2 y

F3 dx dy dz z

... (1)

z ∧ n2 S2

z = f2 (x, y) z = f1 (x, y)

S1

O

∧ n1 y

R x

Fig. 9.56 Assume a closed surface S such that any line parallel to the coordinate axes intersects S at most two points.

Engineering Mathematics

9.148

Divide the surface S into two parts S 1, the lower and S 2, the upper part. Let z = f1 ( x, y ) and z = f 2 ( x, y ) be the equations and nˆ1 and nˆ2 be the normals to the surfaces S1 and S2 respectively. Let R be the projection of the surface S on xy-plane.

∫∫∫ V

∂F3 ⎡ f2 ( x , y ) ∂F3 ⎤ dx dy dz = ∫∫ ⎢ ∫ d z ⎥ dx dy f1 ( x , y ) ∂z ∂z ⎦ R ⎣

= ∫∫ F3 ( x, y, z ) f2 dx dy f

1

R

= ∫∫ [ F3 ( x, y, f 2 ) − F ( x, y, f1 ) ] dx dy R

F3 ( x, y, f 2 ) dx dy

F3 ( x, y, f1 ) dx dy

R

... (2)

R

dx dy = Projection of dS on xy-plane = nˆ kˆ dS For surface S2: z = f2 (x, y) dx dy = nˆ2 kˆ dS2 For surface S1: z = f1 (x, y) dx dy = nˆ1 kˆ dS1 Substituting in Eq. (2),

∫∫∫ V

∂F3 dx dy dz = ∫∫ F3 nˆ2 ⋅ kˆ dS 2 − ∫∫ F3 ( −nˆ1 ⋅ kˆ ) dS1 ∂z S2 S1 =

∫∫ F nˆ 3

S2

=

2

⋅ kˆ dS2 + ∫∫ F3 nˆ1 ⋅ kˆ dS1 S1

∫∫ F nˆ ⋅ kˆ dS

... (3)

3

S

Similarly, projecting the surface S on yz and zx-planes, we get ∂F1

∫∫∫ ∂x dx dy dz = ∫∫ F nˆ ⋅ iˆ dS

... (4)

1

V

S

∂F2 dx dy dz = ∫∫ F2 nˆ ⋅ ˆj dS ∂y V S Substituting Eq. (3), (4) and (5) in Eq. (1),

∫∫∫

and

∫∫∫ ∇ ⋅ FdV = ∫∫ F nˆ ⋅ iˆ dS + ∫∫ F 1

V

S

2

S

... (5)

nˆ ⋅ ˆj dS + ∫∫ F3 nˆ ⋅ kˆ dS S

= ∫∫ ( F1 iˆ ⋅ nˆ + F2 ˆj ⋅ nˆ + F3 kˆ ⋅ nˆ)dS S

= ∫∫ ( F1 iˆ + F2 ˆj + F3 kˆ ) ⋅ nˆ dS S

= ∫∫ F ⋅ nˆ dS S

Vector Calculus

9.149

∫∫∫ ∇ ⋅ FdV = ∫∫ F ⋅ nˆ dS

Hence,

V

S

Note: Cartesian form of Gauss divergence theorem is

V

F1 x

F2 y

F2 z

( F1 dy dz

dx dy dz

F2 dz dx F3 dx dy )

S

Example 1: Verify Gauss divergence theorem for F = 4 x z iˆ - y 2 ˆj + y z kˆ over the cube x = 0, x = 1, y = 0,y = 1, z = 0, z = 1. Solution: By Gauss divergence theorem,

∫∫∫ ∇ ⋅ FdV = ∫∫ F ⋅ nˆ dS V

(i) F

S

4 x z iˆ

y 2 ˆj

y z kˆ F

(4 x z ) ( y2 ) x y 4 z 2y y 4 z y

z

( y z)

z E (0, 0, 1) D (0, 1, 1)

F (1, 0, 1)

G (1, 1, 1) y O (0, 0, 0)

C (0, 1, 0) B (1, 1, 0)

A (1, 0, 0) x

Fig. 9.57 (ii) For the cube: x varies from 0 to 1 y varies from 0 to 1 z varies from 0 to 1 1

1

1

0

0

0

∫∫∫ ∇ ⋅ FdV = ∫ ∫ ∫ (4 z − y) dx dy dz V

=∫

1

0



1

0

1

2 z 2 − yz dx dy 0

1

1

0

0

= ∫ dx ∫ (2 − y )dy

Engineering Mathematics

9.150

y2 2y − 2

1

= x0

1

= 2− 0

1 2

3 2 (iii) Surface S of the cube consists of 6 surfaces, S1, S2, S3, S4, S5 and S6.

... (1)

∫∫ F ⋅ nˆ dS =∫∫ F ⋅ nˆ dS + ∫∫ F ⋅ nˆ dS + ∫∫ F ⋅ nˆ dS S

S1

S2

S3

+ ∫∫ F ⋅ nˆ dS + ∫∫ F ⋅ nˆ dS + ∫∫ F ⋅ nˆ dS S5

S4

... (2)

S6

dx dy = dx dy nˆ ⋅ kˆ

(a) On S1 (OABC ) : z = 0, nˆ = −kˆ , dS = x and y both varies from 0 to 1.

∫∫ F ⋅ nˆ dS =∫∫ (4 x z iˆ − y S1

2

ˆj + y z kˆ ) ⋅ ( − kˆ ) dx dy

S1

0 dx dy (b) On S 2 ( DEFG ) : z = 1, nˆ = kˆ , dS = = dx dy | nˆ ⋅ kˆ |

x and y both varies from 0 to 1.

∫∫ F ⋅ nˆ dS =∫∫ (4 x z iˆ − y S2

2

ˆj + y z kˆ ) ⋅ kˆ dx dy

S2

1

=∫

1

0

=



1

0

y dx dy = ∫

1

0

y2 dx 2 0

1 1 1 x = 2 0 2

dz dx = dz dx (c) On S3 ( OAFE ) : y = 0, nˆ = − jˆ, dS = nˆ⋅ jˆ x and z both varies from 0 to 1.

∫∫ F ⋅ nˆ dS =∫∫ (4 x z iˆ − y S3

2

ˆj + y z kˆ ) ⋅ ( − ˆj ) dz dx

S3

=0 dz dx (d) On S4 ( BCDG ) : y = 1, nˆ = jˆ , dS = = dz dx nˆ ⋅ jˆ

x and z both varies from 0 to 1.

∫∫ F ⋅ nˆ dS =∫∫ (4 x z iˆ − y S4

S4

=∫

1

0



= −1

1

0

− dz dx

2

ˆj + y z kˆ ) ⋅ ( ˆj ) dz dx

Vector Calculus

9.151

dy dz (e) On S5 ( OCDE ) : x = 0, nˆ = − ˆi , dS = = dy dz nˆ ⋅ iˆ

y and z both varies from 0 to 1.

∫∫ F ⋅ nˆ dS =∫∫ (4 x z iˆ − y S5

2

ˆj + y z kˆ ) ⋅ ( −iˆ) dy dz

S5

= 0. (f ) On S6 ( ABGF ) : x = 1, nˆ = iˆ , dS =

dy dz = dy dz nˆ ⋅ ˆi

y and z both varies from 0 to 1. F ⋅ nˆ dS = ( 4 x z iˆ − y 2 ˆj + y z kˆ ) ⋅ iˆ dy dz

∫∫

∫∫

S6

S6

=∫

1

0



1

0

4 z dy dz

1

4 y0

z2 2

1

0

2 Substituting in Eq. (2), 1

∫∫ F ⋅ nˆ dS = 0 + 2 + 0 + (−1) + 0 + 2 S

3 2

... (3)

From Eqs. (1) and (3), 3

∫∫∫ ∇ ⋅ F dV = ∫∫ F ⋅ nˆ dS = 2 V

S

Hence, Gauss divergence theorem is verified. Example 2: Verify Gauss divergence theorem for F = 2 x 2 y iˆ - y 2 ˆj + 4 xz 2 kˆ over the region bounded by the cylinder y 2 + z 2 = 9 and the plane x = 2 in the first octant. Solution: By Gauss divergence theorem,

∫∫∫ ∇ ⋅ F dV = ∫∫ F ⋅ nˆ dS V

S

(i) F = 2 x 2 y iˆ − y 2 ˆj + 4 xz 2 kˆ F

(2 x 2 y )

x y 4 xy 2 y 8 xz

( y2 )

z

(4 xz 2 )

Engineering Mathematics

9.152

(ii)

F dV

(4 xy 2 y 8 xz ) dx dy dz

V

For the given region, x varies from 0 to 2. Putting y = r cosq, z = r sinq, equation of the cylinder y 2 z 2 9 reduces to r = 3 and dy dz = r dr dq. Along the radius vector OP, r varies from 0 to 3 and for the region in the first octant, q varies from 0 to

2

. π 3 2 2 θ =0 r =0 x =0

∫∫∫ ∇ ⋅ F dV = ∫ ∫ ∫ V

π

3

Fig. 9.58 (4 x ⋅ r cos θ − 2 ⋅ r cos θ + 8 x ⋅ r sin θ ) dx ⋅ r dr dθ

(

2

= ∫ 2 ∫ 2r 2 cos θ x 2 − 2r 2 cos θ x 0 + 4 r 2 sin θ x 2 0

0

3

2 0

4

0

r3 3

2

0

(4 r 2 cos

r3 3

16

2 0

) dr dθ

16 r 2 sin ) dr d

3

sin

2 0

0

3

cos

2 0

0

= 36 + 144 = 180 (iii) The surface S consists of 5 surfaces, S1 , S 2 , S3 , S 4 , S5 .

... (1)

∫∫ F ⋅ nˆ dS = ∫∫ F ⋅ nˆ dS + ∫∫ F ⋅ nˆ dS S

S1

S2

+ ∫∫ F ⋅ nˆ dS + ∫∫ F ⋅ nˆ dS S3

x D (2, 0, 0)

S4

+ ∫∫ F ⋅ nˆ dS

E

C

... (2)

S5

(a) On S1(OAED) : z = 0, nˆ

kˆ O (0, 0, 0)

dx dy dS = = dx dy nˆ ⋅ kˆ

z

y

x varies from 0 to 2 and y varies from 0 to 3. 2 2 ∫∫ F ⋅ nˆ dS = ∫∫ (2 x y iˆ − y ˆj + 4 xz kˆ) ⋅ (−kˆ) dx dy 2

S1

S1

=0

B (0, 0, 3)

A (0, 3, 0)

Fig. 9.59

Vector Calculus

9.153

dz dx (b) On S2 (OBCD) : y = 0, nˆ = − ˆj , dS = = dz dx nˆ ⋅ ˆj x varies from 0 to 2 and y varies from 0 to 3.

∫∫ F ⋅ nˆ dS = ∫∫ (2 x S2

2

y iˆ − y 2 ˆj + 4 xz 2 kˆ ) ⋅ ( − ˆj ) dz dx = 0

S2

dy dz (c) On S3 (OAB) : x = 0, nˆ = −iˆ, dS = = dy dz nˆ ⋅ iˆ y and z varius from 0 to 3.

∫∫ F ⋅ nˆ dS = ∫∫ (2 x S3

2

y iˆ − y 2 ˆj + 4 xz 2 kˆ ) ⋅ ( −iˆ) dy dz = 0

S3

dy dz (d) On S 4 ( DEC ) : x = 2, nˆ = iˆ, dS = = dy dz nˆ ⋅ iˆ y and z varius from 0 to 3.

∫∫ F ⋅ nˆ dS = ∫∫ (2 x S4

2

S4

y iˆ − y 2 ˆj + 4 xz 2 kˆ ) ⋅ iˆ dy dz = ∫∫ 8 y dy dz

Putting y = r cosq, z = r sinq, equation of the cylinder y2 + z2 = 9 reduces to r = 3 and dy dz = r dr dq. π

π

3

3

2 ∫∫ F ⋅ nˆ dS = 8∫ 2 ∫ r sin θ ⋅ r dr dθ = 8∫ 2 sin θ dθ ⋅ ∫ r dr 0

0

0

0

S4

= 8 − cos θ

π 2 0

r3 3

3

= 72 0

(e) On S5 (ABCE) : This is curved surface of the cylinder y2 + z2 = 9 bounded between x = 0 and x = 2. Let

y2

z2 nˆ = dS =

∇ ∇

=

4 y2 + 4z2

=

y iˆ + z kˆ 3

[∵ y 2 + z 2 = 9]

dx dy 3 dxx dy = z nˆ ⋅ kˆ

⎛ y ˆj + z kˆ ⎞ 3 dx dy y iˆ − y 2 ˆj + 4 xz 2 kˆ ) ⋅ ⎜ ⎟⎟ ⋅ ⎜ 3 z S5 ⎝ ⎠ 2 3 dx dy = ∫ ∫ ( − y 3 + 4 xz 3 ) x =0 y =0 z

∫∫ F ⋅ nˆ dS = ∫∫ (2 x S5

2 y ˆj + 2 z kˆ

2

Engineering Mathematics

9.154

The parametric equation of the cylinder y2 + z2 = 9, is, y = 3 cosq, z = 3 sinq dy dy 3sin d d zd , z When

y

0,

y

3,

2 0 0

2

π 2

0

∫∫ F ⋅ nˆ dS = ∫ ∫ S5

π 2 0

=∫

( −27 cos3 θ + x 108 sin 3 θ ) ( −dθ )dx

⎛ x2 2 ⎜ −27 cos3 θ x 0 + 108 sin 3 θ ⎜ 2 ⎝

⎞ ⎟ dθ ⎟ 0 ⎠ 2

π

= ∫ 2 −54 cos3 θ + 216 sin 3 θ 0

1 ⎛ 1⎞ 1 ⎛ 1⎞ = −54 ⋅ B ⎜ 2, ⎟ + 216 ⋅ B ⎜ 2, ⎟ 2 ⎝ 2⎠ 2 ⎝ 2⎠ 1 1 1⋅ 2 ⋅ 81 = 2 (−27 + 108) = 5 3 1 1 ⋅ 2 2 2 2 =108 2

⎡∵ sin p cos q d ⎢ ∫ ⎢ ⎛ p +1 q +1⎞ , ⎢= B ⎜ ⎟ 2 ⎠ ⎝ 2 ⎣

⎤ ⎥ ⎥ ⎥ ⎦

Substituting in Eq. (2),

∫∫ F ⋅ nˆ dS = 0 + 0 + 0 + 72 + 108 = 180

... (3)

S

From Eq. (1) and (3),

∫∫∫ ∇ ⋅ F dV = ∫∫ F ⋅ nˆ dS = 180 V

S

Hence, Gauss divergence theorem is verified. Example 3: Verify Gauss divergence theorem for F = 2 xz iˆ + yz ˆj + z 2 kˆ over the upper half of the sphere x 2 + y 2 + z 2 = a 2 . Solution: By Gauss divergence theorem,

∫∫∫ ∇ ⋅ F dV = ∫∫ F ⋅ nˆ dS V

(i) F

2 xz iˆ

S

yz ˆj z 2 kˆ F

x

(2 xz )

y

( yz )

z

(z2 )

2z

z 2z

5z

Vector Calculus

(ii)

F dV V

9.155

5 z dx dy dz V

Putting x = r sinq cosf, y = r sinq sinf, z = r cosq, equation of the sphere x2 + y2 + z2 = a2 reduces to r = a and dx dy dz = r2 sinq dr dq df. For upper half of the sphere (hemisphere), r varies from 0 to a varies from 0 to

2 varies from 0 to 2 π a 2 φ =0 θ =0 r =0 2π

∫∫∫ ∇ ⋅ F dV = 5∫ ∫ ∫ V

π 2 0



= 5 ∫ dφ ∫ 0

r cos θ ⋅ r 2 sin θ dr dθ dφ

r4 cos θ sin θ dθ 4

a 2π

=5φ 0 0

1 cos 2θ ⋅ − 2 2

π 2 0



a4 4

4

=−

5a ⋅ 2π (cos π − cos 0) 16

5 4 a 4

... (1)

(iii) Given surface is not closed. We close this surface from below by the circular surface S2 in xy-plane. Thus, the surface S consists of two surfaces S1 and S2.

∫∫ F ⋅ nˆ dS = ∫∫ F ⋅ nˆ dS + ∫∫ F ⋅ nˆ dS S

S1

Fig. 9.60

S2

... (2)

[∵ x 2

y2

z2

a2 ]

(a) Surface S1(ABCEA) : This is the curved surface of the upper half of the sphere. Let

x2

y2

z2

2 x iˆ 2 y ˆj 2 z kˆ



4x

2

4y

2

4z

2

x iˆ

y ˆj z kˆ a

Let R be the projection of S1 on xy-plane, which is a circle x2 dS =

dx dy a dx dy = z nˆ ⋅ kˆ

[∵ x 2 y2 = a2.

y2

z2

a2 ]

Engineering Mathematics

9.156

∫∫ F ⋅ nˆ dS = ∫∫ (2 xz iˆ + yz ˆj + z S1

2

R

⎛ x iˆ + y ˆj + z kˆ ⎞ a dx dy kˆ ) ⋅ ⎜ ⎟⎟ ⎜ a ⎝ ⎠ z

= ∫∫ ( 2 x 2 + y 2 + z 2 ) dx dy R

= ∫∫ ( 2 x 2 + y 2 + a 2 − x 2 − y 2 ) dx dy

[∵ z 2 = a 2 − x 2 − y 2 ]

R

= ∫∫ ( x 2 + a 2 ) dx dy R

Putting x = r cosq, y = r sinq, equation of the circle x 2 y 2 a 2 reduces to r = a and dx dy = r dr dq. Along the radius vector OP, r varies from 0 to a and for the complete circle, q varies from 0 to 2p. 2π

a

0

0

∫∫ ∇ × F ⋅ nˆ dS = ∫ ∫

( r 2 cos 2 θ + a 2 ) r dr dθ

S1

4 a 2π ⎡ r r2 cos 2 θ + a 2 =∫ ⎢ 0 2 ⎢⎣ 4 0

⎤ ⎥ dθ 0⎥ ⎦ 4 4 2π ⎡ a ⎛ 1 + cos 2θ ⎞ a ⎤ =∫ ⎢ ⎜ + ⎥ dθ ⎟ 0 2 ⎠ 2⎦ ⎣4 ⎝ a4

5 8

1 sin 2 8 2

2

0

a

5 4 a 4

(b) Surface S2 (ABCDA) : This is the circle x 2 nˆ kˆ dS =

Fig. 9.61

y2

dx dy = dx dy nˆ ⋅ kˆ

∫∫ F ⋅ nˆ dS = ∫∫ (2 xz iˆ + yz ˆj + z S2

a 2 in xy-plane z = 0,

2

kˆ ) ⋅ ( −kˆ ) dx dy = 0

[∵ z = 0]

S2

Substituting in Eq. (2), 5

∫∫ F ⋅ nˆ dS = 4

a4

... (3)

S

From Eqs. (1) and (3), 5

∫∫∫ ∇ ⋅ F dV = ∫∫ F ⋅ nˆ dS = 4 V

S

Hence, Gauss divergence theorem is verified.

a4

Vector Calculus

Example 4: Evaluate

9.157

∫∫ ( yz iˆ + zx jˆ + xy kˆ ) ⋅ dS , where S is the surface of the S

sphere in the first octant.

Solution: By Gauss divergence theorem, ∫∫ F ⋅ d S = ∫∫∫ ∇ ⋅ F dV S

… (1)

V

F = yz iˆ + zx ˆj + xy kˆ F From Eq. (1),

x

( yz )

y

( zx)

z

( xy )

0

∫∫ F ⋅ d S = 0 S

( x 3dy dz + x 2 y dz dx + x 2 z dx dy ) where S is the closed

Example 5: Evaluate S

surface consisting of the circular cylinder x 2 + y 2 = a 2 , z = 0 and z = b. Solution: By Gauss divergence theorem,

( F1 dy dz + F2 dz dx + F3 dx dy ) = S

V

F F1 F + 2 + 3 dx dy dz x y z

… (1)

F1 dy dz + F2 dz dx + F3 dx dy = x 3 dy dz + x 2 y dz dx + x 2 z dx dy

(i)

F1 = x 3, F2 = x 2 y, F3 = x 2 z

F F1 F + 2 + 3 = ( x3 ) + ( x 2 y) + ( x 2 z ) x y z x y z

(ii)

= 3x 2 + x 2 + x 2 = 5 x 2 (iii) V

F F1 F + 2 + 3 dx dy dz = x y z

5 x 2 dx dy dz

Putting x = r cosq, y = r sinq, z = z, circular cylinder x 2 + y 2 = a 2 reduces to r = a and dx dy dz = r dr dq dz. Along the radius vector OA, r varies from 0 to a and for complete circle, q varies from 0 to 2p. Along the volume of the cylinder, z varies from 0 to b. z y

z=b

A z=0 y

x

Fig. 9.62

O

r=a x

Engineering Mathematics

9.158

⎛ ∂F1

∫∫∫ ⎜⎝ ∂x V

+

b 2π a ∂F2 ∂F3 ⎞ dx dy dz = 5∫ ∫ ∫ r 2 cos 2 θ ⋅ r dr dθ dz + ⎟ 0 z = θ = =0 0 r ∂y ∂z ⎠ b

=5 z0

5 ba 4 2

a





0

0

⎛ 1 + cos 2θ ⎞ ⎜ ⎟ dθ 2 ⎝ ⎠

5 1 sin 2θ ⋅ b a4 ⋅ θ + 4 2 2

= 4

r4 4



0

5 4 ab 4

2

From Eq. (1), ( x 3 dy dz

x 2 y dz dx x 2 z dx dy )

S

Example 6: Evaluate

5 4 ab 4

( lx + my + nz ) dS , where l, m, n are the direction S

cosines of the outer normal to the surface whose radius is 2. Solution: By Gauss divergence theorem,

∫∫ F ⋅ nˆ dS = ∫∫∫ ∇ ⋅ F dV S

... (1)

V

(i) F nˆ

lx my nz ( x iˆ F

(ii)

F

(iii)

x iˆ x

x

F dV V

y ˆj z kˆ) (l iˆ m ˆj n kˆ) y ˆj z kˆ y

y

z

z

3

3dV V

= 3 (Volume of the region bounded by the sphere of radius 2) 4 3 (2)3 32 3 From Eq. (1),

∫∫ (lx + my + nz ) dS = 32

.

S

dS

Example 7: Prove that 2

S

2

2

2

2 2

a x +b y +c z

=



, where S is the

abc

ellipsoid ax 2 + by 2 + cz 2 = 1. Solution: By Gauss divergence theorem,

∫∫ F ⋅ nˆ dS = ∫∫∫ ∇ ⋅ F dV S

V

... (1)

Vector Calculus

1

F nˆ

(ii)

2

a x

2

b2 y 2

c2 z 2

where nˆ = unit normal to the ellipsoid, ax 2

by 2

2ax iˆ 2by ˆj 2cz kˆ 2

4a x

2

2

a x

2

2

a x

F

a2 x2

4c z

b2 y 2

c2 z 2

2

cz 2

b y

2

2 2

[∵ax 2

by 2

c z

cz 2

1]

ax iˆ by ˆj cz kˆ a2 x2

b2 y 2

c2 z 2

y ˆj z kˆ

x

F dV V

ax iˆ by ˆj cz kˆ

2 2

y ˆj z kˆ)

x iˆ

F

(iv)

2

1

c2 z 2

by 2

2

( x iˆ

(iii)

4b y

b2 y 2

ax 2

Hence,

2

cz 2

1

F nˆ

Now,

9.159

x

y

y

z

z

3

3dV V

= 3 (Volume of the region bounded by the ellipsoid)

4 3

= 3⋅

=

⎤ ⎡ ⎥ ⎢ 2 2 2 x y z 1 1 1 ⎢ ⎥ 1 ∵ ⋅ ⋅ + + = 2 2 2 ⎥ ⎢ a b c ⎛ 1 ⎞ ⎛ 1 ⎞ ⎛ 1 ⎞ ⎢ ⎥ ⎜ ⎟ ⎜ ⎟ ⎜ ⎟ ⎥⎦ ⎝ a⎠ ⎝ b⎠ ⎝ c⎠ ⎣⎢

4 abc

From Eq. (1),

∫∫ S

Example 8: Evaluate

dS a x +b y +c z 2

2

2

∫∫ F ⋅ dS

2

2 2

=

4 abc

using divergence theorem where

S

F

x 3 iˆ

y 3 ˆj

z 3 kˆ and S is the surface of the sphere x 2

y2

z2

a2 .

Solution: By Gauss divergence theorem,

∫∫ F ⋅ d S = ∫∫∫ ∇ ⋅ FdV S

V

... (1)

Engineering Mathematics

9.160

(i)

F (ii)

y 3 ˆj z 3 kˆ

x 3iˆ

F

x3

x

F dV

( x2

3

V

y3

y

y2

z

z3

3x 2

3y2

3z 2

z 2 ) dx dy dz

V

Putting x = r sinq cosf, y = r sinq sinf, z = r cosq equation of the sphere x2 + y2 + z2 = a2 reduces to r = a and dx dy dz = r2 sinq dr dq df For complete sphere, r varies from 0 to a q varies from 0 to p f varies from 0 to 2p 2π

π

∫∫∫ ∇ ⋅ F dV = 3∫ ∫ ∫

a

φ =0 θ =0 r =0

V

r 2 ⋅ r 2 sin θ dr dθ dφ



π

a

0

0

0

= 3∫ dφ ⋅ ∫ sin θ dθ ∫ r dr = 3 φ 0 − cos θ 4

5

3 2 ( cos

cos 0)

a 5



π 0

r5 5

a

0

12 5 a 5

From Eq. (1),

∫∫ F ⋅ d S = S

Example 9: Evaluate

12 a5 . 5

∫∫ F ⋅ d S

using Gauss divergence theorem where

F = 2 xy iˆ + yz ˆj + zx kˆ and S is the surface of the region bounded by x = 0, y = 0, z = 0, y = 3, x + 2z = 6. 2

Solution: By Gauss divergence theorem,

∫∫ F ⋅ d S = ∫∫∫ ∇ ⋅ F dV S

(i)

F

2 xy iˆ

∇⋅F = (ii)

yz 2 ˆj zx kˆ

∂ ∂ ∂ (2 xy ) + ( yz 2 ) + ( zx) = 2 y + z 2 + x ∂x ∂y ∂z

F V

... (1)

V

(2 y

z2

x ) dx dy dz

V

In the given region, y varies from 0 to 3. In xz-plane, region is bounded by the lines x = 0, z = 0, x + 2z = 6. 6 x Along the vertical strip PQ, z varies from 0 to and in the region, x varies from 2 0 to 6.

Vector Calculus

F V

6 x 2

6

x 0 z 0 6 x 2

6 0

0 6 x 2

6 0 6 0 6 0

0

3 y 0

y2

(2 y

z2

z2 y

xy 0 dz dx

z3

9

6 x 2

3 x) dz dx

3 xz

6 x 2 0

3x 2 2

9 x2 27 x 2 2

x3 2

0

27

dx

6 x 2

9x 2

6

x ) dy dz dx 3

(9 3 z 2

9z

9.161

3

6 x 2

3x

6 x 2

3

dx

1 (6 x) 4 8 4

64 162 81 108 32

Fig. 9.63 dx

6

0

351 2

Hence, From Eq. (1),

351 . 2

F ds Example 10: Evaluate

∫∫ F ⋅ nˆ dS using

Gauss divergence theorem where

S

2 2 2 F = 4 xz iˆ + xyz 2 ˆj + 3 z kˆ over the region bounded by the cone z = x + y and

plane z = 4, above xy plane. Solution: By Gauss divergence theorem,

∫∫ F ⋅ nˆ dS = ∫∫∫ ∇ ⋅ F dV S

(i)

F F

4 xz iˆ xyz 2 ˆj 3z kˆ x 4z

(ii)

F dV V

... (1)

V

(4 xz ) x z2 (4 z

y

( xyz 2 )

z

3 xz 2

3) dx dy dz

(3 z )

Engineering Mathematics

9.162

Fig. 9.64 Putting x = r cosq, y = r sinq, z = z equation of the cone z2 = x2 + y2 reduces to z = r, and dx dy dz = r dr dq dz. Along the elementary volume PQ, z varies from r to 4. Projection of the region in rq -plane is the curve of intersection of the cone r = z and plane z = 4 which is a circle r = 4. Along the radius vector OA, r varies from 0 to 4 and for complete circle, q varies from 0 to 2p. 2π

4

4

r =0

z =r

∫∫∫ ∇ ⋅ F dV = ∫ ∫ ∫ θ =0

V

(4 z + r cos θ ⋅ z 2 + 3) r dz dr dθ 4

=∫ =∫





0 2π



0

=∫



=∫



0

4 0 4 0

2 z 2 + r cos θ ⋅

z3 + 3 z r dr dθ 3 r

⎡ ⎤ r 2 cos θ 2 r r − + ( ) (64 − r 3 ) + 3r (4 − r ) ⎥ dr dθ 2 16 ⎢ 3 ⎣ ⎦

⎛ ⎞ 64 r5 44r + r 2 cos θ − 3r 2 − 2r 3 − cos θ ⎟ dr dθ ⎜ 0 3 3 ⎝ ⎠



4

4

0

r3 r4 1 r6 64 22r + cos θ ⋅ − r 3 − − ⋅ cos θ dθ 3 2 3 6 3 0 2

2π ⎛ 2048 ⎞ = ∫ ⎜160 + cos θ ⎟ dθ 0 9 ⎝ ⎠

2

160

0

From Eq. (1),

∫∫ F ⋅ nˆ dS = 320 S

.

2048 sin 9

2 0

160 2

0

320

Vector Calculus

∫∫ ( x

Example 11: Evaluate

2

9.163

i + y 2 j + z 2 k ) ⋅ n dS using Gauss divergence

S

theorem where S is the surface of the ellipsoid

x2 y2 z2 + + = 1. a 2 b2 c 2

Solution: By Gauss divergence theorem,

∫∫ F ⋅ nˆ dS = ∫∫∫ ∇ ⋅ F dV S

(i)

F dV

(ii)

... (1)

V

y 2 ˆj z 2 kˆ

F

x 2 iˆ

F

( x2 ) ( y2 ) x y 2x 2 y 2z

z

(z2 )

(2 x 2 y 2 z ) dx dy dz

V

V

Putting x = ar sinq cosf, y = br sinq sinf, z = cr cosq, equation of the ellipsoid x2 y 2 z 2 1 reduces to r = 1 and dx dy dz = abc r2 sinq dr dq df. a 2 b2 c2 For complete ellipsoid, r varies from 0 to 1 q varies from 0 to p f varies from 0 to 2p 2π

π

∫∫∫ ∇ ⋅ F dV = 2∫ ∫ ∫

1

φ =0 θ =0 r =0

V

(a r sin θ cos φ + br sin θ sin φ + cr cos θ ) abc r 2 sin θ dr dθ dφ 1

=∫



0

=



π

0

(a sin 2 θ cos φ + b sin 2 θ sin φ + c cos θ sin θ )

(

abc ⎡ π a sin 2 θ sin φ 4 ⎢⎣ ∫0 abc 4

0

abc 2 4

2π 0

(0 0 c cos sin cos 2 2

0

∫∫ F ⋅ nˆ dS = 0



+ b sin 2 θ − cos φ 0 + c cos θ sin θ φ 0 2 )d

abc 2 (cos 2 8

From Eq. (1), S



r4 abc dθ dφ 4 0

cos 0)

0

) dθ

Engineering Mathematics

9.164

Exercise 9.11 (I) Verify Gauss divergence theorem for the following: 1. F

3. F 4 x iˆ 2 y 2 ˆj z 2 kˆ over the region bounded by the cylinder x 2 y 2 4 and the planes z = 0, z = 3.

yz ) iˆ ( y 2 zx) ˆj xy ) kˆ over the region R

( x2

(z2 bounded by the parallelepiped 0 x a, 0 y b, 0 z c.

[ Ans. : abc(a + b + c)] 2. F x iˆ y ˆj z kˆ over the region R bounded by the sphere x 2 y 2 z 2 16.

[ Ans. : 84p ] 4. F 2 xy iˆ 6 yz ˆj 3 zx kˆ over the region bounded by the coordinate planes and the plane x + y + z = 2. 22 ⎤ ⎡ ⎢⎣ Ans. : 3 ⎥⎦

[ Ans. : 256p ] (II) Evaluate the following integrals using Gauss divergence theorem: 1.

∫∫ ( y

z iˆ + z 2 x 2 jˆ + z 2 y 2 kˆ ) ⋅ nˆ dS ,

2 2

2.

∫∫ ( x

2

where S is the surface of the sphere ( x 2) 2 ( y 2) 2 ( z 2) 2 4 .

[ Ans. : 32p.]

12

y iˆ + y 3 jˆ + xz 2 kˆ ) ⋅ nˆ dS , where

S

∫∫ (4 x iˆ −2 y

2

(2 xy 2 iˆ x 2 y ˆj x 3 kˆ), where S is

6. S

S is the surface of the parallelopiped 0 x 2, 0 y 3, 0 z 4. [ Ans. : 384] 3.

y dz dx z dx dy ),

S

where S is the part of the sphere x 2 y 2 z 2 1 above xy-plane. Ans. :

( x dy dz

5.

S

the surface of the region bounded by the cone z z = 4.

jˆ + z 2 kˆ ) ⋅ nˆ dS , where

S

S is the surface of the region bounded by y 2 4 x, x 1, z 0, z 3. [ Ans. : 56] ( x dy dz

4.

y dz dx z dx dy ),

x2

y 2 and the plane

Ans. : ( x 3 iˆ

7.

3072 5

y 3 ˆj z 3 kˆ), where S is the

S

surface of the region bounded within

S

where S is the part of the plane x + 2y + 3z = 6 which lies in the first octant. [ Ans. :18]

z

16 x 2

Ans. :

y 2 and x 2

y2

4.

2 (2188 1056 3) 5

Vector Calculus

9.165

FORMULAE Vector Triple Product (i) a

(b

c ) = (a c ) b – (a b ) c

(ii) (a

b)

c = (a c ) b – (b c ) a

Scalar Product of Four Vectors (Lagrange’s identity) (a

b ) (c

∂F1 ∂F2 ∂F3 + + ∂x ∂y ∂z

= Curl

∂ ∂ ∂ + ˆj + kˆ ⎞⎟ F = ⎛⎜ iˆ ∂y ∂z ⎠ ⎝ ∂x

Curl F =

( F1 ˆi + F2 jˆ + F3 kˆ )

d) a c = a d

b c b d

=

Vector Product of Four Vectors (i) (a

b)

(c

d)

b)

(c

ˆj



x F1

y F2

z F3 2

Laplacian Operator

= [a c d ] b − [b c d ] a (ii) (a



Div (grad f ) =

( f) ∂ f ∂2 f ∂2 f + + ∂x 2 ∂y 2 ∂z 2 2

d)

=

= [a b d ] c − [a b c ] d

Green’s Theorem

Gradient ∂f ˆ ∂f ˆ ∂f grad f = f = iˆ + j +k ∂x ∂y ∂z Divergence Div( F ) =

F ∂ ∂ ∂ ⎛ ˆ + ˆj + kˆ ⎞⎟ = ⎜i ∂y ∂z ⎠ ⎝ ∂x ( F1 iˆ + F2 jˆ + F3 kˆ )



C

( M dx + N dy ) =

⎛ ∂N − ∂M ∂y

∫∫ ⎜⎝ ∂x R

⎞ dx dy ⎟⎠

Stoke’s Theorem



C

F ⋅dr =

∫∫ ∇ × F ⋅ nˆ dS S

Gauss Divergence Theorem

∫∫∫ ∇ ⋅ F dV = ∫∫ F ⋅ nˆ dS V

S

MULTIPLE CHOICE QUESTIONS Choose the correct alternative in each of the following: 1. The value of the integral of the function g (x, y) = 4x3 + 10y4 along the straight line segment from the point (0, 0) to the point (1, 2) in the x-y plane is (a) 33 (b) 35 (c) 40 (d) 56 2. Consider points P and Q in the xplane with P = (1, 0) and Q = (0, 1). Q

The line integral 2 ∫ ( x dx + y dy ) P

along the semicircle with the line segment PQ as its diameter (a) is –1 (b) is 0 (c) is 1 (d) depends on the direction (clockwise or anticlockwise) of the semicircle. 3. The following plot shows a function y which varies linearly with x. The

Engineering Mathematics

9.166

2

value of the integral I =

y dx is 1

y

2 1 x –1

1

(a) 1 (c) 4 4.

5.

2

(b) 2.5 (d) 5

( A ), where A is a vector, is equal to (a) A A – 2A (b) 2 A + ( A ) (c) 2 A + A (d) ( A ) – 2 A

∫∫ (∇ × P ) ⋅ ds

where P is a vector,

is equal to (a) ∫ P ⋅ d l (b) (c) (d)

∫ ∇ × ∇ × P ⋅ dl ∫ ∇ × P ⋅ dl ∫∫∫ ∇ ⋅ P dV

6. Stoke’s theorem connects (a) a line integral and a surface integral (b) a surface integral and a volume integral (c) a line integral and a volume integral (d) gradient of a function and its surface integral 7. The line integral ∫ V ⋅ d r of the vector function V (r ) = 2xyz iˆ + x2 z ˆj + x2y kˆ from the origin to the point P(1, 1, 1)

(a) (b) (c) (d)

is 1 is 0 is – 1 cannot be determined without specifying the path 8. The vector field F = x iˆ – y ˆj (where iˆ and ˆj are unit vector) (a) divergence free, but not irrotational (b) irrotational, but not divergence free (c) divergence free and irrotational (d) neither divergence nor irrotational. 9. The divergence of vector r = x iˆ + y ˆj + z kˆ is (a) iˆ + ˆj + kˆ (b) 3 (c) 0 (d) 1 10. The Gauss divergence theorem relates certain (a) surface integrals to volume integrals (b) surface integrals to line integrals (c) vector quantities to other vector quantities (d) line integrals to volume integrals 11. If V is differentiable vector function and f is a sufficient differentiable scalar function, then curl (f V ) is equal to (a) (grad f ) (V ) + (f curl V ) (b) O (c) f curl (V ) (d) (grad f ) (V ) 12. The expression curl (grad f ), where f is a scalar function, is (a) equal to 2f (b) equal to div (grad f ) (c) a scalar of zero magnitude (d) a vector of zero magnitude x2 y2 13. For the scalar field u = + , 2 3

Vector Calculus

the magnitude of the gradient at the point (1, 3) is (a)

13 9

9 2

(b)

9 2 14. For the function f = ax2 y – y3 to represent the velocity potential of an ideal field, 2f should be equal to zero. In that case, the value of ‘a’ has to be: (a) –1 (b) 1 (c) – 3 (d) 3 15. If the velocity vector in a two dimensional flow field is given by V = 2xy iˆ + (2y2 – x2) ˆj, curl V will be (a) 2y2 ˆj (b) 6y kˆ (c) 0 (d) – 4x kˆ (c)

(d)

5

16. The maximum value of the directional derivative of the function f = 2x2 + 3y2 + 5z2 at a point (1, 1, –1) is (a) 10 (b) – 4 (c) 152 (d) 152 17. The directional derivative of the function f (x, y) = x2 + y2 at (1, 2) in the direction of (4 iˆ + 3 ˆj) is 4 (a) (b) 4 5 2 (c) (d) 1 5 18. A velocity vector is given as V = 5xy iˆ + 2y2 ˆj + 3yz2 kˆ . The divergence of this velocity vector at (1, 1, 1) is (a) 9 (b) 10 (c) 14 (d) 15 19. The point of application of a force F = (0, 6, 8) is changed from P (1, –1, 2) to Q (–1, 1, 2). The work done is Answers 1. (a) 8. (b) 15. (d) 22. (c)

2. 9. 16. 23.

(b) (b) (c) (b)

3. 10. 17. 24.

(b) (a) (b) (c)

9.167

(a) 6 (b) 8 (c) 10 (d) 12 20. The total work done in moving a particle in a force field F = 3xy iˆ – 5z ˆj + 10x kˆ along the curve x = t2 + 1, y = 2t2, z = t3 from t = 1 to t = 2 is (a) 18 (b) 120 (c) 360 (d) 303 21. A necessary and sufficient condition that line integral ∫ A ⋅ d r = 0 C for every closed curve C is that (a) A =0 (b) A =0 (c) A 0 (d) A 0 22. The unit normal to the surface 1 x2 + y2 + z2 = 1 at the point ( , 2 1 , 0) is 2 1 ˆ ˆ (a) ( i – j) 2 1 (b) (– iˆ + ˆj) 2 1 ˆ (c) ( i + ˆj) 2 (d) None of the above 23. If ( A B ) C = A ( B C ), then (a) A , B are collinear (b) A , B are perpendicular (c) A , C are collinear (d) A , C are perpendicular 24. If A = 2 iˆ + kˆ , B = iˆ + ˆj + kˆ , C = 4 iˆ – 3 ˆj + 7 kˆ and r B = C B , r A = 0, then r is equal to (a) iˆ – 8 ˆj + 2 kˆ (b) 2 iˆ + 8 ˆj + 4 kˆ (c) – iˆ – 8 ˆj + 2 kˆ (d) –2 iˆ – 6 ˆj + 2 kˆ

4. (d) 11. (a) 18. (d)

5. (a) 12. (d) 19. (d)

6. (a) 13. (c) 20. (d)

7. (a) 14. (d) 21. (b)

Differential Equations

O

Chapter

10

10.1 INTRODUCTION Differential equations are very important in engineering mathematics. A differential equation is a mathematical equation for an unknown function of one or several variables that relates the values of the function itself and of its derivatives of various orders. It provides the medium for the interaction between mathematics and various branches of science and engineering. Most common differential equations are radioactive decay, chemical reactions, Newton’s law of cooling, series RL, RC and RLC circuits, simple harmonic motions, etc.

10.2 DIFFERENTIAL EQUATION A differential equation is an equation which involves variables (dependent and independent) and their derivatives, e.g., d2 y dy 5 6y 2 dx dx d2 y dx 2

2

… (1)

0

… (2)

3

2

dy dx

ex

1

2

z

2

x

2

y2

z

x

y

… (3)

Equations (1) and (2) involve ordinary derivatives and hence called “ordinary differential equations” whereas Eq. (3) involves partial derivatives and hence called “partial differential equation”.

10.2.1 Order Order of a differential equation is the order of the highest derivative present in the equation, e.g., the order of Eqs. (1) and (2) is 2.

10.2

Engineering Mathematics

10.2.2 Degree Degree of a differential equation is the power of the highest order derivative after clearing the radical sign and fraction, e.g., the degree of Eq. (1) is 1 and the degree of Eq. (2) is 2.

10.2.3 Solution or Primitive Solution of a differential equation is a relation between the dependent and independent variables (excluding derivatives), which satisfies the equation. Solution of a differential equation is not always unique. It may have more than one solution or sometimes no solution. General solution of a differential equation of order n contains n arbitrary constants. Particular solution of a differential equation is obtained from the general solution by giving particular values to the arbitrary constants.

10.3 ORDINARY DIFFERENTIAL EQUATIONS OF FIRST ORDER AND FIRST DEGREE A differential equation which contains first order and first degree derivative of y (dependent variable) and known functions of x (independent variable) and y is known as ordinary differential equation of first order and first degree. The general form of this equation can be written as dy =0 F x, y, dx or in explicit form as dy = f ( x, y ) or M( x, y )dx N( x, y )dy 0 dx Solution of the differential equation can be obtained by classifying them as follows: (i) Variable separable (ii) Homogeneous differential equations (iii) Non homogeneous differential equations (iv) Exact differential equations (v) Non-exact differential equations reducible to exact form (vi) Linear differential equations (vii) Non-linear differential equations reducible to linear form

10.3.1 Variable Separable A differential equation of the form M( x)dx N( y )dy 0 … (1) where M (x) is the function of x only and N (y) is the function of y only, is called a differential equation with variables separable as in Eq. (1) function of x and function of y can be separated easily.

Differential Equations

10.3

Integrating Eq. (1) we get the solution as

M( x)dx

N( y )dy

c

or

g ( y )dy

f ( x)dx c

where c is the arbitrary constant. 1

Example 1: Solve y(1 + x 2 ) 2 dy + x 1 + y 2 dx = 0. 1

Solution: y (1 x 2 ) 2 dy



y 1+ y

2

x 1 y 2 dx

dy = − ∫

x 1 + x2

dx + c

1 1 − − 1 1 2 2 2 2 + = − + 1 y 2 y y 1 x ( ) ( ) d ( ) ( 2 x ) dx + c 2∫ 2∫ 1

1

⎡ [ f ( x)]n +1 ⎤ n ⎥ ⎢∵ ∫ [ f ( x)] f ′ ( x)dx = n +1 ⎦ ⎣

1 (1 + x 2 ) 2 1 (1 + y 2 ) 2 ⋅ =− ⋅ +c 1 1 2 2 2 2 1 + x2 + 1 + y 2 = c Example 2: Solve

dy = ex dx

y

+ x2e y . ey

Solution:

dy dx

e y dy ey

ex

x2

(e x ex

x 2 )dx x3 3

c

Example 3: Solve sec2x tan y dx + sec2y tan x dy = 0. Solution: sec2x tan y dx = –sec2y tan x dy sec 2 x

sec 2 y

∫ tan x dx = −∫ tan y dy + c log tan x = − log tan y + c log tan x + log tan y = c log(tan x tan y ) = c tan x tan y = ec = k tan x tan y = k



f ( x) dx f ( x)

log f ( x)

10.4

Engineering Mathematics

Example 4: Solve (4 x + y )2 Solution:

dy dx

(4 x

dx = 1. dy

y)2

… (1)

Let 4x + y = t 4

dy dx

dt dx

dy dx

dt dx

4

t2

4

Substituting in Eq. (1),

dt dx

1 tan 2 Example 5: Solve

dy dx

1

dt dx dt 2 t +4 t 1 tan 1 2 2 4x + y 2

1 tan( y

= t2 + 4 = dx + c = x+c = x+c

x ).

Solution: Let y – x = t dy 1 dx dy dx

dt dx dt 1 dx

Substituting in the given equation, dt 1 1 tan t dx dt dx tan t Integrating both the sides, cos t

∫ sin t dt = ∫ dx + c log sint = x + c



f ( x) dx f ( x)

log f ( x)

Differential Equations

Example 6: Solve

log sin( y

x)

x c

sin( y

x)

ex

10.5

c

dy = (4 x + y + 1)2 , y(0) = 1. dx

Solution: Let 4 x + y + 1 = t dy dt dy dt = , = −4 dx dx dx dx Substituting in the given equation, dt 4 t2 dx dt t2 4 dx dt dx 2 t 4 Integrating both the sides, dt dx c 2 t 4 t 1 x c tan 1 2 2 1 4x y 1 x c tan 1 2 2 4+

Given y (0) 1 Substituting x = 0, y = 1 in Eq. (1),

1 tan 1 (1) 2

0 c, c

8

Hence, solution is 1 tan 2 Example 7: Solve Solution: Let

y x

x

dy dx

1

4x

y

y 1 2

x

y x

x

cos

8 0.

t 1 dy 1 dt − 2 y= x dx x dx dy dt x − y = x2 dx dx

… (1)

10.6

Engineering Mathematics

Substituting in the given equation, dt x2 cos t dx

x

cos t dt

0 dx x

Integrating both the sides,

dx c x log x c

cos t dt sin t y sin x Example 8: Solve 2 x 2 y Solution: Let x 2 y 2

dy dx

log x c

tan( x 2 y 2 ) 2 xy 2 .

t 2 xy 2

x2 2 y

dy dx Substituting in the given equation, 2x2 y

dy dx

2 xy 2

dt dx dt tan t

dt dx dt dx tan t dx

Integrating both the sides,

cot t dt

dx

log sin t

x c

2

2

x c

2

2

sin( x y )

ex

sin( x 2 y 2 )

ke x

log sin( x y )

Example 9: Solve ( x log x )

c

e x ec

dy = 2 y , y(2) = (log 2)2 . dx

dy dx 2 y x log x Integrating both the sides, Solution:

dy

1

1

∫ 2 y = ∫ log x ⋅ x dx

ex k

Differential Equations

10.7

⎡ ⎤ f ′ ( x) ⎢∵ ∫ f ( x) dx = log f ( x) + c ⎥ ⎣ ⎦

1 log y = log(log x) + log c 2 1

log y 2 = log(c log x) 1

y 2 = c log x

… (1)

y (2) (log 2) 2 x = 2, y = (log 2)2 in Eq. (1), (log 2) = c log 2

Given, Putting

c =1 Hence, solution is 1

y2 y

log x (log x) 2

2 Example 10: Solve ( x + y ) x

x Solution:

dy y dx xy

dy dy . + y = xy 1 + dx dx

dy dx ( x y)2 1

d(log xy )

d

1 x

y

Integrating both the sides,

log xy

1 x

y

c

Exercise 10.1 Solve the following differential equations: 1. y 2

dy dx

x2

4. y dy dx

0.

Ans. : x 3 dy 2. (1 x) y (1 y ) x dx

0, x

y3

c

0, y

x

1 y2 .

Ans. : 1 y 2 0.

x c y 3. (e y + 1) cos x dx + e y sin x dy = 0. Ans. : x

xe

5. x(e 4 y 1)

dy dx

( x 1)e

( x 2 1)e 2 y

y log

[ Ans. : (e y 1) sin x

c]

Ans. : cosh(2 y )

log x

x

0, x

x2 2

c 0.

c

10.8

6.

Engineering Mathematics

dy dx

x(2 log x 1) . sin y y cos y

Ans. : e xy

x 2 log x c

Ans. : y sin y

15.

⎡ Ans. : sin y − tan y = − cos x ⎤ ⎢ ⎥ 1 ⎢ ⎥ + (log x) 2 + c ⎣ 2 ⎦ dy 8. y sec x ( y 7) tan x dx 2

Ans. : y tan x 2( y

Ans. : y 10. cos( x

y )dy

ce

x).

x

y y

dy dx

x x 2

dy dx

e

xy

dy dx

x , y (0) 1 y

4(1 x 3 )

2.

2 y3

3x 2

28

x 2 y, y (0) 1.

2y

Ans. : y

y 17. e

dy 1 dx

c( x 1) 2

e

x3 2x 3

1, y (0) 1. Ans. : e y

18.

dy dx

2 y sin 2 x, y Ans. : log y

x) 2

c 4y

1 (1 e)e

2

x

x

1. 1 sin 2 x 2

19. cos y dx (1 e x ) sin y dy y (0)

y . x

y Ans. : sin x 13. x

16.

.

x 2 tan

y

x y

2.

dx.

Ans. : ( y 12. x

dy dx

y

⎡ ⎤ ⎛ x + y⎞ ⎢ Ans. : y − tan ⎜⎝ 2 ⎟⎠ = c ⎥ ⎣ ⎦ 11. dy dx

0, y (1)

Ans. : 3 y 2

0.

7

dy 1 dx

x 2 y dx

Ans. : y 3

log x sin x + dy x . 7. = dx cos y − sec 2 y

9. ( x 1)

14. (1 x 3 )dy

x c

4

0,

. Ans. : (1 e x ) sec y

ce

2

2 2

x

20.

dy dx

y 2 sin x, y (2 ) 1.

y.

[ Ans. : y cos x

1]

10.3.2 Homogeneous Differential Equation A differential equation of the form dy dx

M( x, y ) N( x, y )

… (1)

is called a homogeneous equation if M(x, y) and N(x, y) are homogeneous functions of the same degree, i.e., degree of the R.H.S. of Eq. (1) is zero.

Differential Equations

10.9

Equation (1) can be reduced to variable separable form by putting y = vx. dy dx

v x

dv dx

Equation (1) reduces to dv dx dv x dx dv g (v ) v v x

M( x, vx) N( x, vx)

g (v )

g (v ) v dx x

Above equation is in variable separable form and can be solved by integrating

dv g (v ) v After integrating and replacing v by

dx c x y , we get the solution of Eq. (1). x

Note: Homogeneous functions: A function f (x, y, z) is said to be a homogeneous function of degree n, if for any positive number t, f ( xt , yt , zt ) = t n f ( x, y, z ), where n is a real number. Example 1: Solve x ( x - y )dy + y 2 dx = 0. dy y2 M( x, y ) 2 dx x xy N( x, y ) The equation is homogeneous since M and N are of the same degree 2. Let y = vx Solution:

dy dx

v x

dv dx

Substituting in Eq. (1), v x

dv dx

v2 x2 x (1 v)

v2 1 v

x

dv dx

v2 1 v

v 1 v

2

dx ⎛ v − 1⎞ ⎜⎝ ⎟ dv = v ⎠ x dx ⎛ 1⎞ ⎜⎝1 − ⎟⎠ dv = v x

v

... (1)

10.10

Engineering Mathematics

Integrating both the sides,

1 dv v v log v v

dx x log x log c log v log cx

1

y x y Example 2: Solve

log c xv

log cy x log cy

2 2 dy y + x + y = . dx x

x2 x

y

dy dx

Solution:

y2

M( x, y ) N( x, y )

The equation is homogeneous since M and N are of the same degree 1. Let y = vx dy dv v x dx dx Substituting in Eq. (1), dv vx + x 2 + v 2 x 2 = = v + 1 + v2 dx x dv x = 1 + v2 dx dv dx = 2 x 1+ v

v+ x

Integrating both the sides,



(

dv 1+ v

2

=∫

dx x

)

log v + v 2 + 1 = log x + log c = log cx v + v 2 + 1 = cx y + x

y2 + 1 = cx x2

y + y 2 + x 2 = cx 2

… (1)

Differential Equations

x y

Example 3: Solve 2 ye dx + y - 2 xe

x y

10.11

dy = 0.

x

2 ye y

dy dx

Solution:

M( x, y ) N( x, y )

x y

… (1)

y 2 xe The equation is homogeneous since M and N are of the same degree 1. Let y = vx dy dv v x dx dx Substituting in Eq. (1), x

dv v x dx

2vxe vx x

vx 2 xe vx 1

dv x dx

2ve v 1

v 2e v

v

v2 1

v 2e v

1

v 2e v dv v2

dx x

Integrating both the sides, 1 1 dx ⎛ 1⎞ − ∫ dv − 2∫ e v ⎜ − 2 ⎟ dv = ∫ ⎝ ⎠ v x v 1

− log v − 2e v = log x + c

⎡∵ e f ( x ) f ′ ( x)dx = e f ( x ) ⎤ ⎣ ∫ ⎦

x

− log

y − 2e y = log x + c x x

− log y + log x − 2e y = log x + c x

log y + 2e y + c = 0 Example 4: Solve

Solution:

y y x y y cos dx sin + cos dy = 0. x x y x x dy dx

y y cos x x x y y sin cos y x x

M( x, y ) N( x, y )

The equation is homogeneous since M and N are of the same degree 0.

… (1)

10.12

Engineering Mathematics

Let y = vx dy dv v x dx dx Substituting in Eq. (1), v cos v dv = 1 dx sin v + cos v v v cos v dv − sin v ⋅ v x = −v = sin v + v cos v dx 1 sin v + cos v v dx ⎛ sin v + v cos v ⎞ ⎜⎝ ⎟⎠ dv = x − v sin v v+ x

dx ⎛1 ⎞ ⎜⎝ + cot v⎟⎠ dv = − v x Integrating both the sides,

dx x log x log c

1 cot v dv v log v log sin v c x c x

v sin v y y sin x x y y sin x

c

dy y y - + cosec = 0, y(1) = 0. dx x x dy y y Solution: cosec dx x x The equation is homogeneous since degree of each term is same. Let y = vx dy dv v x dx dx Example 5: Solve

Substituting in Eq. (1), dv dx dv x dx

v x

sin v dv

v cosec v cosec v dx x

… (1)

Differential Equations

10.13

Integrating both the sides,

dx x log x c

sin v dv cos v log x cos v y log x cos x

c … (2)

c

Given y (1) = 0 Putting x = 1, y = 0 in Eq. (2),

log1 cos 0

c

c

1

y x

1

Hence, solution is log x cos

y y 1 1 – – ⎤ ⎤ dy ⎡ ⎡ = x + ⎢ x ( x 2 – y 2 ) 2 + e x ⎥ y , y(1) = 1. Example 6: Solve ⎢ x ( x 2 – y 2 ) 2 + e x ⎥ x ⎦ ⎦ dx ⎣ ⎣

Solution:

dy dx

x

x( x 2 x( x

2

y2 ) 2

y )

1 2

y

1 2

ex y e

y x

x

M( x, y ) N( x, y )

The equation is homogeneous since M and N are of the same degree 1. Let y = vx dy dv v x dx dx Substituting in Eq. (1), vx 1 − ⎤ ⎡ x + ⎢ x( x 2 − v 2 x 2 ) 2 + e x ⎥ vx dv ⎦ ⎣ v+ x = vx 1 dx − ⎤ ⎡ 2 2 2 x 2 ⎢ x( x − v x ) + e ⎥ x ⎣ ⎦

1 − ⎡ 2 2 ⎢(1 − v ) ⎣

1 − ⎡ ⎤ 1 + ⎢(1 − v 2 ) 2 + ev ⎥ v dv 1 ⎣ ⎦ −v = x = 1 1 dx − − ⎡ ⎡ v⎤ v⎤ 2 2 2 2 ⎢(1 − v ) + e ⎥ ⎢(1 − v ) + e ⎥ ⎣ ⎦ ⎣ ⎦ ⎤ dx + e v ⎥ dv = x ⎦

… (1)

10.14

Engineering Mathematics

Integrating both the sides,

1 1 v

dx x

e v dv

2

sin 1 v ev 1

sin

log x c

y

y x

ex

… (2)

log x c

Given y (1) = 1 Putting x = 1, y = 1 in Eq. (2),

sin 1 1 e 2

e

log1 c c

Hence, solution is sin

1

y x

y

ex

log x

2

e

Exercise 10.2 Solve the following differential equations: 1. x ( y x)

dy dx

y (y

x).

y ⎤ ⎡ ⎢⎣ Ans. : x − log xy = c ⎥⎦

dy dx

y sin

y x

0]

y log x cy

7. x

y (log y log x 1).

4. y dx x log y dy 2 x dy x

Ans. : y y x

⎤ y y⎞ ⎛ ⎜⎝ sin + cos ⎟⎠ = c ⎥ x x ⎦

Ans. : cos

Ans. : log

xe

y dy x

3 xy y . 3x 2

[ Ans. : 3x

5.

6. x sin

2

2. dy dx

3. x

y − ⎡ 2 x Ans. : log x − e ⎢ ⎣

y x

cx

0.

c 1 log

y y y sin dx x sin dy x x

x y 0.

dy dx

y

x sec

x dx.

y x

log x c

y . x

Ans. : sin

y x

log(cx)

y ⎛ 2 y⎞ 8. ⎜ x tan − y sec ⎟ dx ⎝ x x⎠ y + x sec 2 dy = 0. x

Ans. : x tan

y x

c

Differential Equations

9.

1 e

x y

dx e

x y

1

x dy y

⎡ ⎤ −1 ⎛ y ⎞ ⎢ Ans. : 3 cos ⎜⎝ x ⎟⎠ − log x = 0⎥ ⎣ ⎦

0. x

ye y

Ans. : x 10. (3 xy y 2 )dx ( x 2 y (1) 1.

11. 2 x ( x

y)

xy )dy

0,

Ans. : x 2 y (2 x

y)

3y2

Ans. : x 4 14. xy log

3

dy 3 y ( x2 dx

x dx y

y2

3 x 2 y )dy

6x2 y2 x 2 log

0,

y4

x dy y

1 0,

e.

4 xy, y (1) 1.

Ans. : y 12. 3 x

13. ( x 3 3 xy 2 )dx ( y 3 y (0) 1.

c

y (1)

dy dx

10.15

2

2 xy

3x

Ans. :

3

1

y2 )2

0, y (1) 1.

x2 x log 2 y 2y 3 1 4e 2

x2 4 y2

log y

10.3.3 Non-Homogeneous Differential Equations A differential equation of the form dy a1 x b1 y c1 … (1) dx a2 x b2 y c2 is called non-homogeneous equation where a1, b1, c1, a2, b2, c2 are all constants. These equations are classified into two parts and can be solved by following methods: a1 b1 m Case I: If a2 b2 a1

a2 m, b1

b2 m ,

then Eq. (1) reduces to dy m(a2 x + b2 y ) + c1 = dx a2 x + b2 y + c2

… (2)

dy dt = , Eq. (2) reduces to variable-separable form and dx dx can be solved using the method of variable-separable equation. a b1 , then substituting Case II: If 1 a2 b2

Putting a2 x + b2 y = t, a2 + b2

x = X + h, y = Y + k in the Eq. (1), a1 ( X dy dY dx dX a2 ( X

h) b1 (Y k ) c1 h) b2 (Y k ) c2

(a1 X b1Y ) (a1h b1k c1 ) (a2 X b2Y ) (a2 h b2 k c2 )

… (3)

10.16

Engineering Mathematics

Choosing h, k such that a1h + b1k + c1 = 0, a2 h + b2 k + c2 = 0 , then Eq. (3) reduces to a X + b1Y dY = 1 dX a2 X + b2Y which is a homogeneous equation and can be solved using the method of homogeneous equation. Finally substituting X = x – h, Y = y – k, we get the solution of Eq. (1).

Problems based on Case I: a1 = b1 b2 b2 Example 1: Solve ( x + y - 1)dx + (2 x + 2 y - 3)dy = 0. dy dx

Solution:

x y 1 2x 2 y 3

The equation is non-homogeneous and

a1 a2

x y 1 2x 2 y 3 b1 b2

… (1)

1 2

Let x + y = t 1+

dy dt dy dt = , = −1 dx dx dx dx

Substituting in Eq. (1), t dt 1 dx 2t t dt = dx 2t 2t 3 dt = dx t 2 2+

1 3 t 1 2t 3 t 2 1 +1 = = 3 2t 3 2t 3

1

dt = dx t 2 Integrating both the sides, 1 2+ dt = dx t 2 2t log (t 2) x c

2( x y ) log ( x x 2 y log ( x

y 2) y 2)

x c c

Example 2: Solve ( x + y )dx + (3 x + 3 y - 4)dy = 0, y (1) = 0. Solution:

dy x y = dx 3 x 3 y 4

… (1)

Differential Equations

The equation is non-homogeneous and

a1 a2

b1 b2

10.17

1 3

Let x + y = t 1

dy dx dy dx

dt dx dt 1 dx

Substituting in Eq. (1), dt 1 dx 3t dt dx 3t 3t 4 dt dx 2t 4 1 2 3 dt 2 t 2

t 4 t 4

1

t 3t 4 3t 4

2t 4 3t 4

dx

Integrating both the sides, 1 ⎛ 2 ⎞ ⎜⎝ 3 + ⎟ dt = ∫ dx ∫ 2 t − 2⎠ 1 [3t + 2 log | (t − 2) |] = x + c 2 3( x + y ) + 2 log | ( x + y − 2) | = 2 x + 2c x + 3 y + 2 log | ( x + y − 2) | = k , where 2c = k Given y (1) = 0 Putting x = 1, y = 0 in the above equation, 1 + 2 log | −1 | = k 1 + 2 log 1 = k k =1 Hence, solution is x 3 y 2 log | x

y 2| 1

Problem Based on Case II: a1 a2

b1 b2

Example 1: Solve ( x + 2 y )dx + ( y - 1)dy = 0. Solution:

dy dx

x 2y y 1

… (1)

10.18

Engineering Mathematics

The equation is non-homogeneous and Let x dx dy dx

X h, dX ,

y dy

1 0

2 1

Y k dY

dY dX

Substituting in Eq. (1), dY ( X h) 2(Y k ) ( X 2Y ) ( h 2k ) dX (Y k ) 1 Y (k 1) Choosing h, k such that h 2k 0, k 1 0

… (2)

… (3)

Solving these equations, k = 1, h = –2 Substituting Eq. (3) in Eq. (2), dY X 2Y dX Y which is a homogeneous equation. Let Y = vX dY dv v X dX dX Substituting in Eq. (4),

dv − X − 2vX −1 − 2v = = dX vX v dv −1 − 2v −1 − 2v − v 2 − (v + 1) 2 X = −v = = dX v v v v dX dv = − X (vv + 1) 2 v+ X

⎡ 1 dX 1 ⎤ ⎢ v + 1 − (v + 1) 2 ⎥ dv = − X ⎣ ⎦ Integrating both the sides, 1

1

∫ v + 1 dv − ∫ (v + 1)

2

dv = − ∫

dX X

1 = − log X + c v +1 1 ⎛Y ⎞ log ⎜ + 1⎟ + = − log X + c ⎝X ⎠ Y +1 X log (v + 1) +

… (4)

Differential Equations

10.19

⎛Y + X log ⎜ ⎝ X

X ⎞ = − log X + c ⎟+ ⎠ Y+X X log (Y + X ) − log X + = − log X + c Y+X X =c log(Y + X ) + Y+X Now,

X=x–h=x+2 Y=y–k=y–1

Hence, solution is

⎛ x+2 ⎞ log ( x + y + 1) + ⎜ =c ⎝ x + y + 1⎟⎠ Example 2: Solve

dy 2 x - 5 y + 3 = . dx 2 x + 4 y - 6

Solution: The equation is non-homogeneous and Let x = X + h, dx = dX, dy dY dx dX

2 2

5 4

y=Y+k dy = dY

Substituting in the given equation, dY 2( X + h) − 5(Y + k ) + 3 (2 X − 5Y ) + (2h − 5k + 3) = = dX 2( X + h) + 4(Y + k ) − 6 (2 X + 4Y ) + (2h + 4k − 6) Choosing h, k such that 2h –5k + 3 = 0, 2h + 4k – 6 = 0 Solving the equations, h=k=1 Substituting Eq. (2) in Eq. (1), dY dX

2X 2X

5Y 4Y

which is a homogeneous equation. Let Y = vX dY dv v X dX dX Substituting in Eq. (3), v+ X

dv 2 X − 5vX 2 − 5v = = dX 2 X + 4vX 2 + 4v

… (1)

… (2)

… (3)

10.20

Engineering Mathematics

dv 2 − 5v 2 − 5v − 2v − 4v 2 −4v 2 − 7v + 2 = −v = = dX 2 + 4v 2 + 4v 2 + 4v 2 + 4v dX dv = − 4v 2 + 7v − 2 X X

2 4v dv (4v 1)(v 2)

dX X

… (4)

Now, 2 + 4v A B dv = + (4v − 1)(v + 2) 4v − 1 v + 2 2 + 4v = A(v + 2) + B (4v − 1) = ( A + 4 B )v + ( 2 A − B ) Comparing coefficients on both the sides, A + 4 B = 4,

2A − B = 2 2 B= 3

4 A= , 3 2 4v (4v 1)(v 2)

4 3(4v 1)

2 3(v 2)

Substituting in Eq. (4), ⎡ 4 2 ⎤ dX ⎢ 3(4v − 1) + 3(v + 2) ⎥ dv = − X ⎣ ⎦ Integrating both the sides, 4 3(4v 1)

2 dv 3(v 2)

dX X

4 log(4v − 1) 2 + log (v + 2) = − log X + log c 3 4 3 c 1 log(4v − 1)(v + 2) 2 = log X 3 1 2 c log(4v − 1) 3 (v + 2) 3 = log X 1 2 c (4v − 1) 3 (v + 2) 3 = X 1

2

1

2

c ⎞3 ⎛ 4Y ⎞ 3 ⎛ Y − 1⎟ ⎜ + 2⎟ = ⎜⎝ ⎠ ⎠ ⎝X X X (4Y − X ) 3 (Y + 2 X ) 3 = c (4Y − X )(Y + 2 X ) 2 = c 3 = k

Differential Equations

X Y

Now,

10.21

x h x 1 y k y 1

Hence, solution is (4 y x 3)( y 2 x 3) 2

k

Exercise 10.3 Solve the following differential equations: 1. ( x 2 y )dx (3x 6 y 3)dy

[ Ans. :

x 3 y 3log | x

0.

10. ( x

0.

[ Ans. :

4 x 8 y log(12 x 8 y 1)

c]

3. ( x

y 3)dy

[ Ans. :

–x

y 3)dx.

y – 3log( x

[ Ans. : 3 x 2

y 1)dx (4 y

0.

5. (2 x 6 y 1)dy ( x 3 y 2)dx

6. ( y

0.

c]

x 2 y log | x 3 y 1| x 2)dy

(y

x) 2

7. (4 x 2 y 5)dy (2 x

4y

c]

y 1)dx

0.

[ Ans. : 10 y − 5 x + 7 log|10 x + 5 y + 9| = c ] 8. (2 x 4 y 5)dy ( x 2 y 3)dx

[ Ans. : x 2 dy 9. dx

13. ( x

4 xy 4 y 2

y 1)dx ( x

14. ( y

x 2)dx ( x

( x 2)

c] 15.

2x y 1 . x y

⎡ ⎡ ⎛ 1⎞ ⎛ 1⎞ ⎤⎤ ⎢ Ans. : log ⎢ 2 ⎜ x + ⎟ + ⎜ y − ⎟ ⎥ ⎥ 3⎠ ⎝ 3 ⎠ ⎥⎦ ⎥ ⎢⎣ ⎝ ⎢ ⎥ ⎢ ⎡ 3y −1 ⎤ ⎥ ⎢ −1 + 2 tan ⎢ ⎥=c⎥ ⎢ ⎣ 2 (3 x + 1) ⎦ ⎦ ⎣

x 1)dy

y 5)dy

y 6)dy

Ans. : ( y 4) 2

0.

6 x 10 y

2

c]

0.

0.

⎡ Ans. : log ⎡⎣( y + 3) 2 + ( x + 2) 2 ⎤⎦ ⎤ ⎢ ⎥ ⎢ ⎥ −1 ⎛ y + 3 ⎞ + 2 tan ⎜ ⎟⎠ = c ⎥ ⎢ ⎝ x+2 ⎣ ⎦

x)dx.

[ Ans. : ( y

8x 2 y

⎡ Ans. : log[4 y 2 + ( x − 1) 2 ⎤ ⎢ ⎥ ⎛ 2y ⎞ ⎢ ⎥ + tan −1 ⎜ = c ⎢⎣ ⎥⎦ ⎝ x − 1⎟⎠

[ Ans. : − 3x + 6 y − 7 log | 3x + 3 y + 2 | = c ]

[ Ans. :

4 xy 7 y 2

0.

c]

y)

y 3)dx (2 x 2 y 1)dy

0.

11. (3 x 2 y 4)dx (2 x 7 y 1)dy

12. ( x 4. ( x

y 1)dy

⎡ Ans. : log[( x − 1) 2 + y 2 ] ⎤ ⎥ ⎢ y ⎞ −1 ⎛ ⎢ = c⎥ − 2 tan ⎜ ⎥⎦ ⎢⎣ ⎝ x − 1⎟⎠

2 y 3 | c]

2. (6 x 4 y 1)dy (3 x 2 y 1)dx

(x

y 1)dx ( x

dy dx

y y

0.

2( x 2)( y 4) 2

c

x 2 . x 4

⎡ Ans. : ( x + 1) 2 − ( y − 3) 2 ⎤ ⎥ ⎢ + 2( x + 1)( y − 3) = c ⎦ ⎣

2

16.

dy dx

2 x 9 y 20 . 6 x 2 y 10

[ Ans. : (2 x

y)2

c( x 2 y 5)]

10.22

Engineering Mathematics

17. (3 x 2 y 3)dx ( x 2 y 1)dy y ( 2) 1.

0,

⎡ Ans. : log ⎡⎣( x − 1) 2 + ( y + 3) 2 ⎤⎦ ⎤ ⎢ ⎥ ⎢ ⎥ −1 ⎛ x − 1 ⎞ + 2 tan ⎜ = 2 log 3⎥ ⎢ ⎟ ⎝ y + 3⎠ ⎦ ⎣

⎡⎣ Ans : (2 x + 2 y + 1) (3 x − 2 y + 9) 4 = −1⎤⎦ 18. ( x y 2)dx ( x y (1) 0.

y 4)dy

0,

10.3.4 Exact Differential Equation Any first order differential equation which is obtained by differentiation of its general solution without any elimination or reduction of terms is known as exact differential equation. If f (x, y) = c is the general solution, then df = 0 f f dx dy 0 x y M( x, y )dx N( x, y )dy

0

… (1)

represents an exact differential equation M( x, y )

where ∂M ∂ 2 f = , ∂y ∂y ∂x

Therefore,

N( x, y )

f y

∂N ∂ 2 f = ∂ x ∂ x∂ y 2

But

f , x

2

f y x

f x y

M y

N x

Thus, necessary condition for a differential equation to be an exact differential equation is M N y x Solution of Eq. (1) can be written as

M( x, y )dx + (terms of N not containing x )dy = c y constant

Sometimes, integration of M w.r.t. x is tedious whereas N can be integrated easily w.r.t. y. In this case solution can be written as

(terms of M not containing y )dx +

N( x, y )dy = c x constant

Differential Equations

10.23

Example 1: Solve ( y 2 - x 2 )dx + 2 xydy = 0. Solution:

Since

M y

M M y

y2

x2 ,

2 y,

N N x

2 xy 2y

N equation is exact. , x

Hence, solution is





M dx +

y constant

N dy = c

terms not containing x

∫(y

− x 2 ) dx + ∫ 0 dy = c

2

xy 2 −

(

x3 =c 3

) (

)

2 2 2 2 Example 2: Solve x 1 - x y - y dy + x + y 1 - x y dx = 0.

Solution: N = x 1 − x 2 y 2 − y,

M = x + y 1 − x2 y 2

⎡ −2 xy 2 ∂N = 1 − x2 y 2 + x ⎢ ∂x ⎢⎣ 2 1 − x 2 y 2 x2 y 2

1 x2 y 2 Since

M y

⎤ ∂M ⎡ −2 x 2 y ⎥, = 1 − x2 y 2 + y ⎢ ⎥⎦ ∂y ⎢⎣ 2 1 − x 2 y 2 1 x2 y 2

1 x2 y 2

x2 y 2 1 x2 y 2

N , equation is exact. x

Hence, solution is



M dx +

y constant

∫ (x + y



N dy = c

terms not containing x

)

1 − x 2 y 2 dx + ∫ ( − y ) dy = c

⎛ 1 ⎞ x2 y2 + y 2 ∫ ⎜ 2 − x 2 ⎟ dx − =c 2 2 ⎝ y ⎠ ⎡x x2 + y2 ⎢ 2 ⎢2 ⎢⎣

⎤ y2 1 1 −1 ⎛ x ⎞ 2 − =c − + x sin ⎜ 1 ⎟ ⎥⎥ 2 y2 2 y2 ⎜ ⎟⎥ ⎝ y⎠⎦

1 x 2 − y 2 xy + 1 − x 2 y 2 + sin −1 ( xy ) = c 2 2 2 x 2 − y 2 + xy 1 − x 2 y 2 + sin −1 ( xy ) = 2c = k

⎤ ⎥ ⎥⎦

Engineering Mathematics

10.24

Example 3: Solve (2 xy cos x 2 - 2 xy + 1)dx + (sin x 2 - x 2 )dy = 0. M = 2 xy cos x 2 − 2 xy + 1,

Solution:

N = sin x 2 − x 2

∂M = 2 x cos x 2 − 2 x, ∂y Since

∂N = (cos x 2 )( 2 x ) − 2 x ∂x

M N = , equation is exact. y x

Hence, solution is

∫ Mdx + ∫

y constant

∫ (2 xy cos x

2

Ndy = c

terms not containing x

− 2 xy + 1) dx + ∫ 0 dy = c

y sin x 2 − x 2 y + x = c Example 4: Solve

⎡∵ {cos f ( x)} f ′ ( x) dx = sin f ( x) ⎤ ⎣ ∫ ⎦

dy y cos x + sin y + y + = 0. dx sin x + x cos y + x

Solution: ( y cos x + sin y + y )dx + (sin x + x cos y + x)dy = 0 M = y cos x + sin y + y,

N = sin x + x cos y + x

∂M = cos x + cos y + 1, ∂y

∂N = cos x + cos y +1 ∂x

Since M = N , equation is exact. y x Hence, solution is

∫ M dx + ∫ y constant

Ndy = c

terms not contaning x

∫ ( y cos x + sin y + y)dx + ∫ 0 dy = c y sin x + x(sin y + y ) = c x x ⎛ ⎞ x⎞ y y ⎛ Example 5: Solve ⎜⎜ 1 + e ⎟⎟ dx + e ⎜ 1 – ⎟ dy = 0, y(0) = 4. y⎠ ⎝ ⎝ ⎠

Solution:

x y

M = 1+ e ,

x y

⎛ x⎞ N = e ⎜1 − ⎟ ⎝ y⎠

Differential Equations

x

⎛ x ⎞ ∂M = e y ⎜− 2 ⎟, ∂y ⎝ y ⎠

x

x

⎛1⎞⎛ x ⎞ ∂N = e y ⎜ ⎟ ⎜1 − ⎟ + e y ∂x ⎝ y⎠⎝ y⎠

x

⎛ 1⎞ ⎜⎝ − y ⎟⎠

x

−x = 2 ey, y Since

10.25

x = − 2 ey y

∂M ∂N = , equation is exact. ∂y ∂x

Hence, solution is

∫ Mdx + ∫ Ndy = c y constant

terms not containing x

x ⎛ ⎞ y 1 e + ⎟ dx + ∫ 0 dy = c ∫ ⎜⎝ ⎠ x y

1+

e =c 1 y x y

1 + ye = c

... (1)

Given y (0) = 4 Substituting in Eq. (1),

1 + 4e0 = c 5=c Hence, solution is x

1 + ye y = 5

Example 6: Solve log( x 2 + y 2 ) +

2 x2 2 xy dx + 2 dy = 0. 2 2 x +y x + y2

⎡ 2x2 ⎤ Solution: M = ⎢log( x 2 + y 2 ) + 2 ⎥, x + y2 ⎦ ⎣ ∂M 1 2x2 = 2 ⋅ − ⋅ 2 y, 2 y ∂y ( x 2 + y 2 )2 x + y2 =

2y 4x2 y − x 2 + y 2 ( x 2 + y 2 )2

N=

2 xy x + y2 2

∂N 2y 2 xy = 2 − 2 ⋅ 2x 2 ∂x x + y ( x + y 2 )2 =

2y 4x2 y − x 2 + y 2 ( x 2 + y 2 )2

Engineering Mathematics

10.26

Since

M N = , equation is exact. y x

Hence, solution is

∫ M dx

∫ N dy = c

+

terms not containing y x constant

∫ 0 dx + ∫ x

2 xy dy = c + y2

2

x log ( x 2 + y 2 ) = c Example 7: For what values of a and b, the differential equation (y + x3)dx + (ax + by3)dy = 0 is exact. Also find the solution of the equation. Solution:

M = y + x3,

N = ax + by3

∂M = 1, ∂y

∂N =a ∂x

Equation will be exact if M N = y x 1= a Hence, equation is exact for a = 1 and for all values of b. Substituting a = 1 in the equation, (y + x3) dx + (x + by3) dy = 0, which is exact. Hence, solution is ∫ M dx + ∫ N dy = c y constant

terms not containing x

∫ ( y + x ) dx + ∫ by dy = c 3

xy +

3

x 4 by 4 + =c 4 4

Example 8: Solve (cos x + y sin x )dx = (cos x )dy , y( ) = 0. Solution: (cos x

y sin x) dx (cos x) dy M cos x y sin x, M = sin x, y

Since

M N = , equation is exact. y x

0 N cos x N = sin x x

Differential Equations

10.27

Hence, solution is

∫ Mdx + ∫ Ndy = c y constant

terms not containing x

∫ (cos x + y sin x) dx + ∫ 0 dy = c sin x − y cos x = c

... (1)

Given y ( ) = 0 Substituting x = , y = 0 in Eq. (1),

sin − 0 = c 0=c Hence, solution is sin x

y cos x

0

y = tan x Example 9: Solve ( ye xy + 4 y 3 )dx + ( xe xy + 12 xy 2 - 2 y )dy = 0, y (0) = 2. Solution:

Since

M = ye xy + 4 y 3 , ∂M = e xy + ye xy ⋅ x + 12 y 2 , ∂y

N = xe xy + 12 xy 2 − 2 y ∂N = e xy + xe xy ⋅ y +12 y 2 ∂x

M N , equation is exact. = y x

Hence, solution is

∫ M dx + ∫ y constant

∫ ( ye

xy

N dy = c

terms not containing x

+ 4 y 3 ) dx + ∫ −2 y dy = c y

e xy + 4 y3 x − y 2 = c y

e xy + 4 xy 3 − y 2 = c

... (1)

Given y (0) = 2 Substituting x = 0, y = 2 in Eq. (1)

e 0 + 0 − 4 = c, Hence, solution is e xy

4 xy 3

y2

3

−3 = c

Engineering Mathematics

10.28

Exercise 10.4 Solve the following differential equations: 1. (2 x 3 + 3 y ) dx + (3 x + y − 1) dy = 0.

8.

(1+ x

)

⎡⎣ Ans. : x 4 + 6 xy + y 2 − 2 y = c ⎤⎦

3. sinh x cos y dx − cosh x sin y dy = 0.

[ Ans. : cosh x cos y = c ] x2 + y2

dx + y (1 + e y (0) = 0.

4. xe

x2 + y2

5.

4 x3 y 3

1 dx x

2

+ y2

10. ( x 2 1)

x =1 y

(2 xy dx x 2 dy )

0.

⎡⎣ Ans. : x 4 y 3 − x 2 y = c ⎤⎦ 2

1)dx e x dy

0.

⎡ Ans. : ye x − x 2 = c ⎤ ⎣ ⎦

x3

2 xy x.

dy x 2 2 xy = . dx x 2 sin y

⎡⎣ Ans. : x 3 − 3( x 2 y + cos y ) = c ⎤⎦ 12.

dy y +1 = dx ( y 2)e y

x

.

⎡⎣ Ans. : ( y + 1) ( x − e y ) = c ⎤⎦ 13. ( x y

6. (4 x 3 y 3 dx + 3 x 4 y 2 dy )

dy dx

⎡⎣ Ans. : x 4 − 4 x 2 y + 2 x 2 − 4 y = c ⎤⎦

=1

1 dy 0, y

3x 4 y 2

Ans. : x 4 y 3 + log

2

⎡⎣ Ans. : sin x(e y + 1) = c ⎤⎦

11.

y (1) = 1.

7. 2 x( ye x

9. (e y + 1) cos x dx + e y sin x dy = 0.

)dy = 0,

Ans. : y 2 + e x

)

⎡ ⎤ 2 3 32 ⎢ Ans. : 2 xy + x y = c ⎥ 3 ⎣ ⎦

2. (1 + e x ) dx + y dy = 0.

⎤ ⎡ y2 x Ans. : x + e + = c⎥ ⎢ 2 ⎦ ⎣

(

y y dx + x 2 y + 2 x dy = 0.

2

y cos x)dx sin x dy 2

0,

= 1. 2 ⎡ ⎤ 2 − 2⎥ ⎢ Ans. : x − 2 y sin x = 4 ⎣ ⎦

14. (2 xy + e y ) dx + ( x 2 + xe y ) dy = 0, y (1) = 1.

2

Ans. : x 2 y + xe y = e + 1

10.3.5 Non-Exact Differential Equations Reducible to Exact Form Sometimes a differential equation is not exact but can be made exact by multiplying with a suitable function. This function is known as Integrating factor (I.F.). There may exists more than one integrating factor to a differential equation.

Differential Equations

10.29

Here, we will discuss different methods to find an I.F. to a non exact differential equation, M dx + N dy = 0

Case I: If

M y

N x N

= f ( x ), (function of x alone), then I.F. = e

f ( x )dx

After multiplication with the I.F. the equation becomes exact and can be solved using the method of exact differential equations. Example 1: Solve ( x 2 + y 2 + 1)dx - 2 xy dy = 0. Solution:

M = x2 + y2 + 1,

N = – 2 xy

∂M = 2 y, ∂y Since

M y

∂N = −2 y ∂x

N , equation is not exact. x M y

N x

2 y ( 2 y) 2 xy

N I.F. = e

2 dx x

=e

2 log x

2

= elog x = x

2

2 x =

1 x2

Multiplying D.E. by I.F., 1 2 (x x2

y 2 1)dx 1

M1 = 1 +

1 2 xy dy x2

y2 + 1 dx x2

2y dy x

y2 +1 , x2

∂M1 2 y = 2, ∂y x Since

M1 N1 = , equation is exact. y x

Hence, solution is

∫M

dx +

1 y constant



N dy = c

1 terms not containing x

0 0

2y x ∂ N1 2 y = 2 ∂x x N1 = −

Engineering Mathematics

10.30

⎛ y 2 + 1⎞ 1 + ∫ ⎜⎝ x 2 ⎟⎠ dx + ∫ 0 dy = c

y2 + 1 =c x x2 − y 2 − 1 = c x x−

1

Example 2: Solve

xy 2 - e x

3

dx - x 2 y dy = 0. 1

Solution:

Since

M y

M = xy 2 − e x , ∂M = 2 xy, ∂y

N = − x2 y ∂N = −2 xy ∂x

3

N , equation is not exact. x M y

N x N

2 xy ( 2 xy ) x2 y

I.F. = e

4 dx x

=e

4 log x

4 x 4

= e log x = x

Multiplying D.E. by I.F., 1

1 1 3 ( xy 2 − e x ) dx − 4 ( x 2 y ) dy = 0 x4 x 1 ⎛ 2 x3 y e ⎜ − ⎝ x3 x 4

⎞ ⎟ dx − y dy = 0 ⎠ x2

1 3

y2 e x M1 = 3 − 4 , x x ∂M1 2 y = 3, ∂y x Since

N1 = −

∂ N1 2 y = 3 ∂x x

M1 N1 = , equation is exact. y x

Hence, solution is

∫ M d x + ∫ N dy = c

1 y constant

y x2

1 terms not containing x

4

=

1 x4

Differential Equations

10.31

1 ⎛ 2 ⎞ x3 y e ⎜ − ∫ ⎜ x3 x 4 ⎟⎟ dx + ∫ 0 dy = c ⎝ ⎠ 1

y2 1 3⎞ 3 ⎛ − 2 + ∫ e x ⎜ − 4 ⎟ dx = c ⎝ x ⎠ 3 2x 1



y 2 1 x3 + e =c 2x2 3

⎡∵ e f ( x ) f ′ ( x)dx = e f ( x ) + c ⎤ ⎣ ∫ ⎦

Example 3: Solve (2 x log x - xy )dy + 2 y dx = 0. Solution: 2 y dx (2 x log x xy )dy

0

M = 2 y, ∂M = 2, ∂y Since

M y

N = 2 x log x − xy ∂N = 2 log x + 2 − y ∂x

N , equation is not exact. x M y

N x N

=

2 (2 log x 2 y ) 2 x log x xy (2 log x y ) 1 x(2 log x y ) x

I.F. = e

1 dx x

=x1=

=e

log x

= elog x

1

1 x

Multiplying D.E. by I.F., 1 1 (2 y ) dx (2 x log x xy ) dy 0 x x 2y dx (2 log x y )dy 0 x 2y , M1 = N1 = 2 log x − y x ∂M1 2 ∂ N1 2 = , = ∂y x ∂x x Since

M1 N1 = , equation is exact. y x

Engineering Mathematics

10.32

Hence, solution is

∫ M d x + ∫ N dy = c

1 y constant



Example 4: Solve x sin x Solution:

1 terms not containing x

2y dx + ∫ ( − y ) dy = c x y2 2 y log x − =c 2

dy + y( x cos x - sin x ) = 2. dx

x sin x dy ( xy cos x ( xy cos x

y sin x 2) dx

0

y sin x 2) dx x sin x dy

0

M = xy cos x − y sin x − 2 ∂M = x cos x − sin x ∂y Since

M y

N = x sin x ∂N = sin x + x cos x ∂x

N , equation is not exact. x M y

N x N

=

( x cos x sin x ) (sin x x cos x ) x sin x 2sin x 2 x sin x x

I.F. = e

2 dx x

=e

2 log x

2

= elog x = x

2

=

1 x2

Multiplying D.E. by I.F., 1 ( xy cos x x2 y cos x x

y sin x 2) dx y sin x x2

2 dx x2

y y 2 cos x − 2 sin x − 2 , x x x ∂M1 cos x sin x = − 2 , ∂y x x M1 =

Since

M1 N1 = , equation is exact. y x

1 ( x sin x) dy x2 1 sin x dy x

0 0

sin x x ∂N1 cos x sin x = − 2 ∂x x x N1 =

Differential Equations

10.33

Hence, solution is

∫M

dx +

1 terms not containing y

2

∫− x

2

∫N

dy = c

1 x constant

sin x dy = c x 2 ⎛ sin x ⎞ +⎜ ⎟ y=c x ⎝ x ⎠

dx + ∫

2 y sin x + =c x x

Case II: If then I.F. = e

N x

M y M

= f ( y ), (function of y alone),

f ( y )dy

After multiplying with the I.F., the equation becomes exact and can be solved using the method of exact differential equation. Example 1: Solve ( y 4 + 2 y )dx + ( xy 3 + 2 y 4 - 4 x )dy = 0. Solution:

M = y 4 + 2 y, ∂M = 4 y 3 + 2, ∂y

Since

N = xy 3 + 2 y 4 − 4 x ∂N = y3 − 4 ∂x

∂M ∂N ≠ , equation is not exact. ∂y ∂x ∂N ∂M − y 3 − 4 − ( 4 y 3 + 2) −3( y 3 + 2) 3 ∂x ∂y = = =− 4 3 y M y + 2y y ( y + 2) 3

I.F. = e

∫ − y dy

−3

= e −3 log y = e log y = y −3 =

1 y3

Multiplying D.E. by I.F., 1 4 1 ( y + 2 y )dx + 3 ( xy 3 + 2 y 4 − 4 x) dy = 0 3 y y ⎛ ⎛ 2⎞ 4x ⎞ ⎜⎝ y + y 2 ⎟⎠ dx + ⎜⎝ x + 2 y − y 3 ⎟⎠ dy = 0

Engineering Mathematics

10.34

M1 = y +

2 , y2

N1 = x + 2 y −

∂M1 4 = 1− 3 , ∂y y Since,

4x y3

∂ N1 4 = 1− 3 ∂x y

M1 N1 = , equation is exact. y x

Hence, solution is

∫ M d x + ∫ N dy = c

1 y constant



1 terms not containing x

2⎞ dx + ∫ 2 y dy = c 2⎟ ⎠

∫ ⎜⎝ y + y

⎛ 2⎞ 2 ⎜⎝ y + y 2 ⎟⎠ x + y = c Example 2: Solve (2 xy 4 e y + 2 xy 3 + y )dx + ( x 2 y 4 e y - x 2 y 2 - 3 x )dy = 0. Solution:

Since

M y

M = 2 xy 4 e y + 2 xy 3 + y, ∂M = 2 x( y 4 e y + 4 y 3 e y + 3 y 2 ) + 1, ∂y

N = x 2 y 4 e y − x 2 y 2 − 3x ∂N = 2 xy 4 e y − 2 xy 2 − 3 ∂x

N , equation is not exact. x ∂N ∂M − ( 2 xy 4 e y − 2 xy 2 − 3) − ( 2 xy 4 e y + 8 xy 3 e y + 6 xy 2 + 1) ∂x ∂y = M 2 xy 4 e y + 2 xy 3 + y =

−4 ( 2 xy 3 e y + 2 xy 2 + 1) 4 =− 3 y 2 y y ( 2 xy e + 2 xy + 1) 4

I.F. = e Multiplying D.E. by I.F., 1 (2 xy 4 e y y4

∫ − y dy

2 xy 3 2 xe y

−4

= e −4 log y = e log y = y −4 =

y )dx 2x y

1 2 4 y (x y e y4

1 dx y3

x2e y

1 y4

x2 y2

3 x)dy

0

x2 y2

3x dy y4

0

Differential Equations

M1 = 2 xe y +

2x 1 + , y y3

10.35

N1 = x 2 e y −

∂M1 2x 3 = 2 xe y − 2 − 4 , ∂y y y

x 2 3x − y2 y4

∂ N1 2x 3 = 2 xe y − 2 − 4 ∂x y y

Since M1 = N1 , equation is exact. y x Hence, solution is

∫ M d x + ∫ N dy = c

1 y constant



∫ ⎜⎝ 2 xe

y

+

1 terms not containing x

2x 1 ⎞ + dx + ∫ 0 dy = c y y 3 ⎟⎠ x2e y +

x2 x + =c y y3

Example 3: Solve xe x (dx - dy ) + e x dx + ye y dy = 0. Solution:

( xe x

e x )dx ( ye y

xe x )dy

M = xe + e , ∂M = 0, ∂y x

Since

M y

0 N = ye y − xe x ∂N = −e x − xe x ∂x

x

N , equation is not exact. x N x

M y

e x ( x 1) 0 e x ( x + 1)

M I.F. = e

dy

=e

1

y

Multiplying D.E. by I.F., e y ( xe x

e x )dx e y ( ye y

M1 = e − y ( xe x + e x ), ∂M1 = −e − y ( xe x + e x ), ∂y M1 N1 , equation is exact. = y x Hence, solution is ∫ M1dx + ∫ N1dy = c

Since

y constant

terms not containing x

xe x )

0

N1 = y − xe x − y ∂N1 = − e − y ( xe x + e x ) ∂x

Engineering Mathematics

10.36

∫e

−y

( xe x + e x )dx + ∫ y dy = c y2 =c 2 y2 + =c 2

e − y ( xe x − e x + e x ) + xe x − y

y sec y - tan y dx + (sec y log x - x )dy = 0. x

Example 4: Solve Solution:

Since

M y

y sec y − tan y, x ∂M 1 y = sec y + sec y tan y − sec 2 y, ∂y x x M=

N = sec y log x − x ∂N sec y = −1 x ∂x

N , equation is not exact. x N x

M y M

sec y sec y y 1 sec y tan y sec 2 y x x x = y sec y tan y x y sec y tan y tan 2 y x tan y y sec y tan y x

I.F. = e

tan y dy

=e

log sec y

1

= elog(sec y ) = (sec y ) 1 = cos y

Multiplying D.E. by I.F., cos y

y sec y tan y dx cos y (sec y log x x) dy x

0

y sin y dx (log x x cos y ) dy x

0

y − sin y, x ∂M1 1 = − cos y, ∂y x M1 =

Since

M1 N1 = , equation is exact. y x

N1 = log x − x cos y ∂ N1 1 = − cos y ∂x x

Differential Equations

10.37

Hence, solution is

∫ M d x + ∫ N dy = c

1 y constant

⎛y

1 terms not containing x



∫ ⎜⎝ x − sin y⎟⎠ dx + ∫ 0 dy = c y log x − x sin y = c Case III: If differential equation is of the form f1 ( xy ) y dx + f 2 ( xy ) x dy = 0, then I.F. = provided Mx Ny

1 , where M = f1 (xy)y, N = f2 (xy) x Mx Ny

0

After multiplying with the I.F., the equation becomes exact and can be solved using the method of exact differential equation. Example 1: Solve y(1 + xy + x 2 y 2 )dx + x (1 - xy + x 2 y 2 )dy = 0. Solution: Equation is of the form f1 ( xy ) y dx + f 2 ( xy ) x dy = 0 I.F. =

1

Mx Ny 1 = 2 2 2x y

=

1 ( xy

2

x y

2

3

3

x y ) ( xy

Multiplying D.E. by I.F., y 2

2x y

2

(1 + xy + x 2 y 2 ) dx +

x 2x2 y 2

(1 − xy + x 2 y 2 ) dy = 0

⎛ 1 ⎛ 1 1 y⎞ 1 x⎞ ⎜⎝ 2 x 2 y + 2 x + 2 ⎟⎠ dx + ⎜⎝ 2 xy 2 − 2 y + 2 ⎟⎠ dy = 0

M1 =

1 1 y + + , 2 2x y 2x 2

∂M1 1 1 =− 2 2 + , ∂y 2 2x y Since

M1 N1 = , equation is exact. y x

N1 =

1 1 x − + 2 2y 2 2 xy

∂ N1 1 1 =− 2 2 + ∂x 2 2x y

x2 y2

x3 y 3 )

Engineering Mathematics

10.38

Hence, solution is

∫ M d x + ∫ N dy = c

1 y constant

⎛ 1

∫ ⎜⎝ 2 x −

2

y

+

1 terms not containing x

1 y⎞ 1 + ⎟ dx + ∫ − dy = c 2x 2 ⎠ 2y

1 1 xy 1 + log x + − log y = c 2 xy 2 2 2 1 xy 1 x − + + log = c 2 xy 2 2 y

Example 2: Solve ( xy sin xy + cos xy ) y dx + ( xy sin xy - cos xy ) x dy = 0. Solution: M = x y2 sin xy + y cos xy,

N = x2 y sin xy – x cos xy

The equation is in the form f1 ( xy ) y dx + f 2 ( xy ) x dy = 0 I.F. =

1 Mx Ny =

1 x 2 y 2 sin xy

xy cos xy

Multiplying D.E. by I.F., 1 ( xy sin xy cos xy ) y dx 2 xy cos xy

x 2 y 2 sin xy

Since

1 dx 2x

y tan xy 1 + , 2 2x

N1 =

∂M1 tan xy + xy sec 2 xy = , ∂y 2

1 2 xy cos xy

x tan xy 2

1 dy 2y

x tan xy 1 − 2 2y

∂N1 taan xy + xy sec 2 xy = ∂x 2

M1 N1 = , equation is exact. y x

Hence, solution is

∫ M d x + ∫ N dy = c

1 y constant

=

1 ( xy sin xy cos xy ) x dy 2 xy cos xy

y tan xy 2 M1 =

xy cos xy

1 terms not containing x

1 ⎛ 1⎞ 1 ⎜ y tan xy + ⎟ dx + ∫ − dy = c ∫ 2 ⎝ x⎠ 2y

0 0

Differential Equations

10.39

1⎛ y ⎞ 1 log sec xy + log x ⎟ − log y = c ⎜ 2⎝ y ⎠ 2 log( x sec xy ) − log y = 2c ⎛x ⎞ log ⎜ sec xy ⎟ = 2c ⎝y ⎠

x sec xy = e 2 c = k , y

x sec xy = k y

Case IV: If differential equation Mdx + Ndy = 0 is homogeneous equation in x and y (degree of each term is same), then I.F. =

1 provided Mx Ny Mx + Ny

0.

After multiplying with the I.F., the equation becomes exact and can be solved using the method of exact differential equations. Example 1: Solve ( x 2 y - 2 xy 2 )dx - ( x 3 - 3 x 2 y )dy = 0. x 2 y 2 xy 2 ,

Solution: M

x3

N

3x 2 y

Differential equation is homogeneous as each term is of degree 3. 1 1 1 = 3 = 2 2 I.F. = 2 2 3 2 2 Mx + Ny x y 2 x y x y 3x y x y Multiplying D.E. by I.F., 1 1 ( x 2 y − 2 xy 2 )dx − 2 2 ( x 3 − 3x 2 y )dy = 0 2 x y x y 2

⎛ x 3⎞ ⎛1 2⎞ ⎜⎝ y − x ⎟⎠ dx − ⎝⎜ y 2 − y ⎟⎠ dy = 0 M1 =

1 2 − , y x

∂M1 1 =− 2, ∂y y Since

M1 N1 = , equation is exact. y x

Hence, solution is

∫ M d x + ∫ N dy = c

1 y constant

⎛1

2⎞

1 terms not containing x

3

∫ ⎜⎝ y − x ⎟⎠ dx + ∫ y dy = c

N1 = −

x 3 + y2 y

∂ N1 1 =− 2 ∂x y

Engineering Mathematics

10.40

x − 2 log x + 3 log y = c y x y3 + log 2 = c y x Example 2: Solve x

dy y 2 + = y. dx x

Solution: x 2 dy + y 2 dx = xy dx ( y2

xy )dx x 2 dy M

0 y2

xy, N

x2

Differential equation is homogeneous as each term is of degree 2. 1 I.F. = Mx + Ny 1 1 = 2 = 2 2 2 xy x y x y xy Multiplying D.E. by I.F., 1 x2 2 ( y − xy ) d x + dy = 0 xy 2 xy 2 1 x

1 dx y

x dy y2

M1 =

0

1 1 − , x y

∂M1 1 = 2, ∂y y Since

M1 N1 = , equation is exact. y x

Hence, solution is

∫ M d x + ∫ N dy = c

1 y constant

⎛1

1 terms not containing x

1⎞

∫ ⎜⎝ x − y ⎟⎠ dx + ∫ 0 dy = c log x −

x =c y

N1 =

x y2

∂ N1 1 = 2 ∂x y

Differential Equations

10.41

Example 3: Solve 3 x 2 y 4dx + 4 x 3 y 3dy = 0, y(1) = 1. Solution: M = 3 x 2 y 4 , N = 4 x 3 y 3 Differential equation is homogeneous as each term is of degree 6. 1 I.F. = Mx + Ny 1 1 = 3 4 = 3x y + 4 x3 y 4 7 x3 y 4 Multiplying D.E. by I.F., 1 1 (3 x 2 y 4 )dx + 3 4 (4 x 3 y 3 )dy = 0 3 4 7x y 7x y 3 4 dx + dy = 0 7x 7y M1 =

3 , 7x

N1 =

∂M1 = 0, ∂y Since

4 7y

∂ N1 =0 ∂x

M1 N1 , equation is exact. = y x

Hence, solution is

∫ M d x + ∫ N dy = c

1 y constant

1 terms not containing x

3

4

∫ 7 x dx + ∫ 7 y dy = log c 3 4 log x + log y = log c 7 7 3

4

log x 7 + log y 7 = log c ⎛ 3 4⎞ log ⎜ x 7 y 7 ⎟ = log c ⎝ ⎠ 3

4

x7 y7 = c

... (1)

Given y (1) = 1 Substituting x = 1, y = 1 in Eq. (1), 3

4

(1) 7 ⋅ (1) 7 = c,

1= c

Engineering Mathematics

10.42

Hence, solution is 3

4

x7 y7 = 1 Case V: If the differential equation is of the type x m1 y n1 (a1 y dx + b1 x dy ) + x m2 y n2 (a2 y dx + b2 x dy ) = 0 , then I.F. = x h y k where

m1 + h + 1 n1 + k + 1 = a1 b1

and

m2 + h + 1 n2 + k + 1 = a2 b2

Solving these two equations, we get the values of h and k. Example 1: Solve x (4 y dx + 2 x dy ) + y 3 (3 y dx + 5 x dy ) = 0. Solution: xy 0 (4 y dx + 2 x dy ) + x 0 y 3 (3 y dx + 5 x dy ) = 0 m1 = 1, n1 = 0, a1 = 4, b1 = 2, m2 = 0, n2 = 3, a2 = 3, b2 = 5 m1 + h + 1 n1 + k + 1 = a1 b1 1+ h +1 0 + k +1 = 4 2 2h + 4 = 4k + 4 h = 2k

... (1)

m2 + h + 1 n2 + k + 1 = a2 b2

and

0 + h +1 3 + k +1 = 3 5 5h + 5 = 3k + 12 5h 3k 7 Solving Eqs. (1) and (2), h = 2, k = 1 I.F. = x 2 y Multiplying D.E. by I.F., x 3 y (4 y dx + 2 x dy ) + x 2 y 4 (3 y dx + 5 x dy ) = 0 (4 x 3 y 2 + 3 x 2 y 5 ) dx + (2 x 4 y + 5 x 3 y 4 ) dy = 0

... (2)

Differential Equations

M = 4 x 3 y 2 + 3x 2 y 5 ,

N = 2 x 4 y + 5x3 y 4

∂M = 8 x 3 y + 15 x 2 y 4 , ∂y Since

10.43

∂N = 8 x 3 y + 15 x 2 y 4 ∂x

M N = , equation is exact. y x

Hence, solution is

∫ Mdx + ∫ Ndy = c y constant

∫ (4 x

3

terms not containing x

y 2 + 3 x 2 y 5 ) dx + ∫ 0 dy = c

x 4 y 2 + x3 y 5 = c Example 2: Solve ( x 7 y 2 + 3 y )dx + (3 x 8 y - x )dy = 0. Solution:

Since

M y

M = x 7 y 2 + 3 y, ∂M = 2 x 7 y + 3, ∂y

N = 3x8 y − x ∂N = 24 x 7 y − 1 ∂x

N , equation is not exact. x

Rewriting the equation, x 7 y 2 dx 3 x 8 y dy 3 y dx x dy

0

7

0

x y ( y dx 3 x dy ) (3 y dx x dy )

m1 = 7, n1 = 1, a1 = 1, b1 = 3, m2 = 0, n2 = 0, a2 = 3, b2 = –1 m1 + h + 1 n1 + k + 1 = a1 b1 7 + h +1 1+ k +1 = 1 3 3h + 24 = k + 2 3h k 22 and

... (1)

m2 + h + 1 n2 + k + 1 = a2 b2 0 + h +1 0 + k +1 = 3 −1 − h − 1 = 3k + 3 h + 3k = −4

... (2)

Engineering Mathematics

10.44

Solving Eqs. (1) and (2), h = –7, k = 1 I.F. = x 7 y Multiplying D.E. by I.F., x 7 y( x7 y 2 (y

Since

3

3 y )dx x 7 y (3 x8 y x)dy 7

2

3 x y )dx (3 xy

2

6

x y )dy

0 0

M1 = y 3 + 3 x −7 y 2 ,

N1 = 3 xy 2 − x −6 y

∂M1 = 3 y 2 + 6 x −7 y, ∂y

∂ N1 = 3 y 2 + 6 x −7 y ∂x

M1 N1 = , equation is exact. y x

Hence, solution is

∫ M d x + ∫ N dy = c

1 y constant

∫ (y

3

1 terms not containing x

+ 3 x −7 y 2 ) dx + ∫ 0 dy = c 3 x −6 y 2 =c −6 x −6 y 2 =c xy 3 − 2

xy 3 +

Case VI: Integrating Factors by Inspection Sometimes integrating factor can be identified by regrouping the terms of the differential equation. The following table helps in identifying the I.F. after regrouping the terms. Sr. No. Group of Terms

Integrating Factor

Exact Differential Equation

1.

dx ± dy

1 x± y

dx ± dy = d[log( x ± y )] x± y

2.

y dx + x dy

1 2xy

2 x y dy + 2 xy 2 dx = d( x 2 y 2 )

1 xy

y dx + x dy = d[log( xy )] xy

1 ( xy ) n

y dx + x dy = d( xy ) 2

y dx + x dy ( xy ) n

d

( xy )1 n ,n 1 1 n Contd.

Differential Equations

3.

1 y2

y dx x dy

y dx x dy x =d 2 y y

1 x2 + y 2 1 x2 1 xy 4.

x dx ± y dy

10.45

y dx x dy = d tan x2 + y 2 y dx x dy x2

d

1

x y

y x

y dx x dy x = d log xy y

2

2 x dx ± 2 y dy = d( x 2 ± y 2 )

1 (x ± y2 ) 1 ( x 2 ± y 2 )n

2 x dx ± 2 y dy = d[log( x 2 ± y 2 )] x2 ± y 2

2

2 x dx ± 2 y dy ( x 2 ± y 2 )1 =d 2 2 n 2(1 n) (x ± y )

5.

2y dx + x dy

x

2xy dx + x2 dy = d(x2y)

6.

y dx + 2x dy

y

y2 dx + 2xy dy = d(xy2)

7.

2y dx – x dy

x y2

2 xy dx x 2 dy x2 =d 2 y y

8.

2x dy – y dx

y x2

2 xy dy y 2 dx y2 =d 2 x x

Example 1: Solve x dy - y dx + 2 x 3 dx = 0. Solution: Dividing the equation by x2, x dy

y dx x

2

d

+ 2 x dx = 0

y + d( x 2 ) = 0 x

Integrating both the sides,

y + x2 = c x Example 2: Solve x dx + y dy + 2( x 2 + y 2 )dx = 0. Solution: Dividing the equation by x2 + y2,

n

Engineering Mathematics

10.46

x dx + y dy + 2 dx = 0 x2 + y 2 1 d[log( x 2 + y 2 )] + 2 dx = 0 2 Integrating both the sides,

1 log( x 2 + y 2 ) + 2 x = c 2 Example 3: Solve (1 xy ) y dx (1 xy ) x dy

0.

Solution: y dx + xy2 dx + x dy – x2y dy = 0 Regrouping the terms, ( y dx x dy ) ( xy 2 dx x 2 y dy ) 2 2

Dividing the equation by x y , y dx + x dy x2 y2 d

1 xy

dx x

dy y

0

dx x

dy y

0

Integrating both the sides,



1 + log x − log y = c xy x 1 − + log = c xy y

Example 4: Solve x dy - y dx = 3 x 2 ( x 2 + y 2 )dx . Solution: Dividing the equation by (x2 + y2), x dy y dx = 3 x 2 dx x2 + y 2 d tan

1

tan

1

y x

= d( x 3 )

Integrating both the sides, y = x3 + c x

Example 5: Solve ( xy - 2 y 2 )dx - ( x 2 - 3 xy )dy = 0. Solution: xy dx 2 y 2 dx x 2 dy 3 xy dy

0

0

Differential Equations

Regrouping the terms, x ( y dx x dy ) 2 y 2 dx 3 xy dy 0 Dividing the equation by xy2, y dx x dy y2 x y

d

2 dx x

3 dy y

0

2 dx x

3 dy y

0

Integrating both the sides, x − 2 log x + 3 log y = c y

x − log x 2 + log y 3 = c y x y3 + log 2 = c y x Example 6: Solve y(2 xy + e x )dx = e x dy . Solution: 2 xy 2 dx e x y dx e x dy

0

Dividing the equation by y2, 2 x dx + 2 x dx + d

ye x dx − e x dy =0 y2 ex =0 y

Integrating both the sides,

x2 +

ex =c y

Example 7: Solve ydx + x(x2y – 1)dy = 0. Solution: y dx x 3 y dy

x dy

0

Regrouping the terms, y dx x dy Dividing the equation by

x3 , y

x 3 y dy

0

10.47

Engineering Mathematics

10.48

y 2 dx xy dy + y 2 dy = 0 x3 1 ⎛ y 2 ⋅ 2 x dx − x 2 ⋅ 2 y dy ⎞ 2 ⎟⎠ + y dy = 0 2 ⎜⎝ x4 1 ⎛ y2 ⎞ d − + y 2 dy = 0 2 ⎜⎝ x 2 ⎟⎠ Integrating both the sides,

1 y 2 y3 + =c 2 x2 3 y2 y3 − 2+ =c 3 2x



3 xy 3 xy Example 8: Solve y( x e - y )dx + x ( y + x e )dy = 0. 3 xy Solution: x ye dx

y 2 dx xy dy

x 4 e xy dy

0

Regrouping the terms, x 3 ye xy dx + x 4 e xy dy − y 2 dx + xy dy = 0 Dividing the equation by x3, 1 ⎛ y 2 ⋅ 2 x dx − x 2 ⋅ 2 y d y ⎞ ye xy dx + xe xy dy − ⎜ ⎟⎠ = 0 2⎝ x4 1 y2 d(e xy ) + d 2 = 0 2 x Integrating both the sides, e xy +

1 y2 =c 2 x2

Example 9: If xn is an integrating factor of ( y - 2 x 3 )dx - x (1 - xy )dy = 0, then find n and solve the equation. Solution: If xn is an I.F., then after multiplication with xn, the equation becomes exact. ( x n y 2 x n + 3 )dx x n +1 (1 xy )dy where and

M

xn y 2xn+3 , N

0 is an exact D.E. x n +1

M N = y x

xn+ 2 y

Differential Equations

10.49

x n = − (n + 1) x n + (n + 2) x n +1 y (n + 2) x n (1 + x y ) = 0 n+2 = 0 n = −2 Putting n = –2 in the equation, ( x −2 y − 2 x )dx − x −1 (1 − xy )dy = 0 ⎛ y ⎞ ⎛1 ⎞ ⎜⎝ 2 − 2 x ⎟⎠ dx − ⎜⎝ − y ⎟⎠ dy = 0 x x y − 2 x, x2 ∂M 1 = 2, ∂y x

N=−

M=

1 +y x

∂N 1 = ∂x x 2

Since M = N , equation is exact. y x Hence, solution is

∫ Mdx + ∫ Ndy = c y constant

⎛ y

∫ ⎜⎝ x

2

terms not containing x

⎞ − 2 x ⎟ dx + ∫ y dy = c ⎠ −

y y2 − x2 + =c x 2

Example 10: If (x + y)n is an integrating factor of (4x2 + 2xy + 6y) dx + (2x2 + 9y + 3x) = 0, then find n and solve the equation. Solution: Since (x + y)n is an I.F., after multiplication with (x + y)n, the equation becomes exact. i.e.,

( x + y ) n (4 x 2 + 2 xy + 6 y )dx + ( x + y ) n (2 x 2 + 9 y + 3 x)dy = 0

... (1)

is an exact D.E. where M = ( x + y ) n (4 x 2 + 2 xy + 6 y ), and

N = ( x + y ) n (2 x 2 + 9 y + 3 x)

M N = y x

( x + y ) n−1 (4nx 2 + 2nxy + 6ny + 2 x 2 + 2 xy + 6 x + 6 y ) = ( x + y ) n−1 (2nx 2 + 9ny + 3nx + 4 x 2 + 4 xy + 3 x + 3 y )

Engineering Mathematics

10.50

2nx 2 + 2nxy − 3ny − 3nx = 2 x 2 + 2 xy − 3 y − 3 x n(2 x 2 + 2 xy − 3 y − 3 x) = 2 x 2 + 2 xy − 3 y − 3 x n =1 Putting n = 1 in Eq. (1), ( x + y )( 4 x 2 + 2 xy + 6 y )dx + ( x + y )( 2 x 2 + 9 y + 3x )dy = 0 ( 4 x 3 + 6 x 2 y + 2 xy 2 + 6 xy + 6 y 2 )dx + ( 2 x 3 + 12 xy + 3 x 2 + 2 x 2 y + 9 y 2 )dy = 0 M = 4 x 3 + 6 x 2 y + 2 xy 2 + 6 xy + 6 y 2 , ∂M = 6 x 2 + 4 xy + 6 x + 12 y ∂y

N = 2 x 3 + 12 xy + 3 x 2 + 2 x 2 y + 9 y 2 ∂N = 6 x 2 + 12 y + 6 x + 4 xy ∂x

∂M ∂N = , equation is exact. ∂y ∂x Hence, solution is Since

∫ Mdx + ∫ Ndy = c y constant

∫ (4 x

3

terms not containing x

+ 6 x 2 y + 2 xy 2 + 6 xy + 6 y 2 )dx + ∫ 9 y 2 dy = c x 4 + 2 x3 y + x 2 y 2 + 3x 2 y + 2 y 3 + 3 y 3 = c

Exercise 10.5 Solve the following differential equations: 1. ( x 2 + y 2 + x) dx + xy dy = 0.

Ans. :

⎡⎣ Ans. : 3 x 4 + 4 x 3 + 6 x 2 y 2 = c ⎤⎦ 2. ( y 2 x 3 ) dx ( x x 2 y ) dy Ans. : xy 2

5. (e x x 4

0.

2 y 2 x3

x 2 y 2 3 x)dy

0.

⎤ ⎡ x2 x 2 y ⎢ Ans. : x e + + 3 = c ⎥ y y ⎦ ⎣ 4.

2 x sinh

y y + 3 y cosh dx x x

3 x cosh

y dy x

0.

y x

2mxy 2 ) dx 2mx 2 y dy

2

cx 3 0.

Ans. : x 2 e x + my 2 = cx 2

cx

3. (2 xy 4 e y + 2 xy 3 + y ) dx + ( x 2 y 4 e y

3sinh

6.

y+

y3 x2 1 + dx + ( x + xy 2 ) dy = 0. 3 2 4

⎡⎣ Ans. : x 6 + 3 x 4 y + x 4 y 3 = c ⎤⎦ 7. (x sec2 y – x2 cos y) dy = (tan y – 3x4)dx.

tan y ⎤ ⎡ 3 ⎢⎣ Ans. : x + x − sin y = c ⎥⎦

Differential Equations

8. (4 xy 3 y 2

x) dx x( x 2 y ) dy

0.

10.51

19. (1 xy ) y dx (1 xy ) x dy

⎤ ⎡ x4 4 3 2 + − = c⎥ Ans. : x y x y ⎢ 4 ⎦ ⎣ 9. ( x 2 + y 2 + 2 x) dx + 2 y dy = 0.

Ans. : log

(1 xy x 2 y 2

11. y ( xy e x ) dx e x dy

3

2

0.

21.

⎤ ⎡ x2 ex + = c⎥ ⎢ Ans. : 2 y ⎦ ⎣ 12. (3x2 y3 e y + y3 + y2)dx + (x3 y3 e y – xy)dy = 0. ⎡ ⎤ x 3 y ⎢ Ans. : x e + x + y = c ⎥ ⎣ ⎦ 13. y ( x 2 y e x ) dx e x dy

14. (xy3 + y)dx + 2(x2 y 2 + x + y 4) dy = 0.

⎡⎣ Ans. : 3 x y + 6 xy + 2 y = c ⎤⎦ 2

2

4

x

15. (2 x y e ) y dx (e

2

x

6

3

y ) dy

Ans. : 4 x 3 y 3 y 3

6e x

16. y log y dx ( x log y ) dy

17. ( x

y ) dx 2 xy dy

cy

0.

⎤ ⎡ y + log x = c ⎥ ⎢ Ans. : x ⎦ ⎣ 18. 2 xy dx ( y

2

x ) dy

22. y (sin xy + xy cos xy ) dx x( xy cos xy sin xy ) dy 0.

⎡ ⎤ x sin( xy ) = c⎥ ⎢ Ans. : y ⎣ ⎦

0.

Ans. : x + y = cy 2

23. y ( x y ) dx x ( y x) dy 0.

y ⎤ ⎡ ⎢⎣ Ans. : log xy − 2 x = c ⎥⎦ 24. x 2 y dx ( x 3

2

y 3 ) dy 0. x3

Ans. : y = ce

3 y3

25. 3 y dx + 2 x dy = 0, y (1) =1. 3

0.

2

2

⎡ ⎤ 1 x 1 ⎢ Ans. : 3 log y 2 − 3 x 2 y 2 = c ⎥ ⎣ ⎦

0.

Ans. : 2 x log y = c + (log y ) 2 2

x2 y3 + 2 y . 2 x 2 x3 y 2

dy dx

0.

⎤ ⎡ x3 e x + = c⎥ ⎢ Ans. : 3 y ⎦ ⎣

x3 y 3 ) x dy 0.

⎡ ⎤ 1 2 ⎢ Ans. : xy − xy − log y = c ⎥ ⎣ ⎦

x 2 ) dy 0.

Ans. : x y + x = cy 3

x 1 =c+ y xy

20. (1 + xy + x 2 y 2 + x 3 y 3 ) y dx

⎡⎣ Ans. : e x ( x 2 + y 2 ) = c ⎤⎦ 10. (3 x 2 y 4 2 xy ) dx (2 x 3 y 3

0.

Ans. : yx 2 = 1 26. ( y 2 2 x 2 y ) dx (2 x 3 xy ) dy 0. 1 1 ⎡ ⎤ 2 − 32 32 2 2 ⎢ Ans. : − x y + 4 x y = c ⎥ 3 ⎣ ⎦

27. (2 x 2 y 2

y ) dx ( x 3 y 3 x ) dy

0.

⎤ ⎡ 7 107 − 75 7 − 74 − 127 Ans. : x y − x y = c⎥ ⎢ 5 4 ⎦ ⎣

Engineering Mathematics

10.52

28. If yn is an integrating factor of

and solve the equation.

(2 xy e + 2 xy + y ) dx 4

y

2

3

4

(x y e

y

2

x y

2

Ans. :

3 x) dy 0 , find n

n

4, x 2 y 3e y

x2 y 2

x

cy 3

10.3.6 Linear Differential Equations If each term in a differential equation including derivative is linear in terms of dependent variable, then the equation is called linear. A differential equation of the form dy + Py = Q dx

... (1)

where P and Q are functions of x, is called linear differential equation and is linear in y. To solve Eq. (1), obtain the integrating factor (I.F.) as I.F. = e

Pdx

Multiplying Eq. (1) by I.F., e

Pdx

dy + Pe dx d e dx

Pdx

Pdx

y = Qe

Pdx

y = Qe

Pdx

Integrating w.r.t x, e∫

Pdx

y = ∫ Qe∫

Pdx

dx + c

(I.F.) y = ∫ (I.F.) Q + c

or

Eq. (2) is the solution of differential Eq. (1). Example 1: Solve

dy 3 y sin x + = 3 . dx x x

Solution: The equation is linear in y. P= I.F. = e

3 dx x

3 , x

Q=

sin x x3 3

= e3log x = elog x = x 3

... (2)

Differential Equations

10.53

Hence, solution is sin x dx + c x3

x3 y = x3

sin x dx c cos x x3

y

Example 2: Solve

cos x c

c x3

dy 4x 1 . y= 2 + dx 1 + x 2 ( x + 1)3

Solution: The equation is linear in y. P=

4x 1 , Q= 2 2 1+ x ( x + 1)3 4x

I.F. = e

1+ x 2

dx

2

= e 2 log(1+ x ) = elog(1+ x

2 2

)

= (1 + x 2 ) 2

Hence, solution is (1 + x 2 ) 2 y = ∫ (1 + x 2 ) 2 ⋅ =∫ Example 3: Solve (1 x 2 )

dy dx

1 dx + c ( x 2 + 1)3

1 dx + c = tan −1 x + c x2 + 1 2 xy

x 1 x2 .

Solution: Rewriting the equation, dy 2x x + y= 2 dx 1 x 1 x2 The equation is linear in y. P= 2x

I.F.

e

1 x2

2x x , Q= 2 1 x 1 x2

dx

e

log(1 x 2 )

elog(1

x2 )

1

(1 x 2 )

Hence, solution is ⎛ 1 ⎜ 2 ⎝1− x

⎞ ⎟y=∫ ⎠ =∫

⎛ 1 ⎜ 2 ⎝1− x

⎞⎛ x ⎟ ⎜⎜ ⎠ ⎝ 1 − x2

x 3 2 2

(1 − x )

dx + c

⎞ ⎟⎟ dx + c ⎠

1

1 1 x2

Engineering Mathematics

10.54

=−

3 − 1 2 2 1 ( − x ) ( −2 x) dx + c 2∫

1 (1 − x 2 ) =− ⋅ 1 2 − 2



1 2

⎡ [ f ( x)]n + 1 ⎤ n ⎥ ⎢∵ ∫ [ f ( x)] f ′ ( x) dx = n +1 ⎦ ⎣

+c

1 − y 2 2 = − +c ( x ) 1 1 − x2

y = 1 − x 2 + c (1 − x 2 ) Example 4: Solve x log x

dy + y = 2 log x . dx

Solution: Rewriting the equation, dy 1 2 + y= dx x log x x The equation is linear in y. P=

1 2 , Q= x log x x

I.F. = e

1 dx x log x

= elog (log x ) = log x

Hence, solution is 2 (log x) 2 (log x) y = ∫ (log x) ⋅ dx + c = 2 +c 2 x

⎡ ⎢∵ ⎣



f ( x ) ⋅ f ′ ( x ) dx =

[ f ( x)]2 ⎤ ⎥ 2 ⎦

= (log x) 2 + c y log x = (log x) 2 + c Example 5: Solve (1 + x + xy 2 )dy + ( y + y 3 )dx = 0. Solution: Rewriting the equation, (1 + x + xy 2 ) + ( y + y 3 )

dx =0 dy

dx (1 + y 2 ) x 1 + + =0 3 dy y+ y y + y3 dx dy

1 x y

1 y (1 + y 2 )

... (1)

Differential Equations

10.55

The equation is linear in x. 1 , Q y

P

1 dy y

I.F. = e

1 y (1 + y 2 )

= elog y = y

Hence, solution is ⎡ ⎤ 1 1 dy + c = − ∫ dy + c yx = ∫ y ⎢ − 2 ⎥ 1+ y2 ⎣ y (1 + y ) ⎦ = − tan −1 y + c xy = c − tan −1 y

Example 6: Solve y log y dx + ( x - log y )dy = 0. Solution: Rewriting the equation, y log y

dx dy

x log y

0

dx 1 1 x= + dy y log y y The equation is linear in x. P=

1 1 , Q= y log y y

1

I.F. = e

∫ y log y dy

⎡ ⎢∵ ⎣

= elog(log y )



⎤ f ′( y) dy = log f ( y ) + c ⎥ f ( y) ⎦

= log y Hence, solution is 1 (log y ) x = (log y ) dy + c y x log y =

(log y ) 2 +c 2

Example 7: Solve (1 + sin y )dx = (2 y cos y - x sec y - x tan y )dy . Solution: Rewriting the equation, (1 sin y ) (1 + sin y )

dx dy

2 y cos y (sec y tan y ) x

dx 1 + sin y + x = 2 y cos y dy cos y dx 1 2 y cos y + x= dy cos y 1 sin y

Engineering Mathematics

10.56

The equation is linear in x. P= I.F. = e

1 dy cos y

=e

sec y dy

1 2 y cos y , Q= cos y 1 + sin y = elog(sec y + tan y ) = sec y + tan y

Hence, solution is (sec y + tan y ) x = (sec y + tan y ) =2

1 + sin y cos y

2 y cos y dy + c 1 + sin y

y cos y dy + c = 2 y d y + c 1 + sin y

(sec y + tan y ) x = y 2 + c Example 8: Solve (1 + y 2 )dx = (tan

1

y - x )dy .

Solution: Rewriting the equation, (1 y 2 )

dx dy

tan 1 y

x

dx 1 tan 1 y x + = dy 1 y2 1 y2 The equation is linear in x. P=

1 tan 1 y , Q = 1+ y2 1+ y2 1 1+ y 2

I.F. = e

dy

= e tan

1

y

Hence, solution is (e tan

1

y

1

) x = e tan

y

tan 1 y dy + c 1+ y2

Let tan 1 y = t 1 dy = dt 1+ y2 (e tan

1

y

et t dt c

)x e x

tan

1

y

1

tet

et

1

(tan y 1) c

tan y 1 ce

tan

1

y

c

Differential Equations

10.57

Example 9: Solve dr + (2r cot p + sin 2p )dp = 0. Solution: Rewriting the equation, dr d

(2 cot )r

sin 2

The equation is linear in r. P

2 cot ,

2 cot d

I.F. = e

Q

sin 2

= e 2 log sin = elog sin

2

= sin 2

Hence, solution is sin 2 ⋅ r = ∫ sin 2 (− sin 2 ) d + c = −2 ∫ sin 3 cos d + c = −2 r sin 2

=−

sin 4 2

sin 4 4

⎡ [ f ( )]n +1 ⎤ n ⎢∵ ∫ [ f ( )] f ′( ) d = ⎥ n +1 ⎦ ⎣

+c

+c

Example 10: Solve (4r 2 s - 6)dr + r 3 ds = 0. Solution: 4r 2 s 6 r 3

ds dr

0 ds 4 s 6 + = 3 dr r r

The equation is linear in s. P=

4 , r

I.F. = e

4 dr r

Q=

6 r3 4

= e 4 log r = elog r = r 4

Hence, solution is r4 s

r4

6 dr c r3

6 r dr c

r2 + c = 3r 2 + c 2 3 c s= 2 + 4. r r =6

Example 11: Solve cosh x

Solution:

dy = 2cosh 2 x sinh x - y sinh x . dx

dy + (tanh x) y = 2 cosh x sinh x dx

Engineering Mathematics

10.58

The equation is linear in y. P = tanh x, Q = 2 cosh x sinh x tanh x dx

I.F. = e

=e

sinh x dx cosh x

= elog cosh x = cosh x

Hence, solution is (cosh x) y = ∫ cosh x (2 cosh x sinh x) dx + c = 2∫ cosh 2 x ⋅ sinh x dx + c = 2 y cosh x =

2 cosh 3 x + c 3

Example 12: Solve x ( x - 1)

Solution:

dy dx

cosh 3 x +c 3

dy - ( x - 2) y = x 3 (2 x - 1). dx

x 2 (2 x 1) ( x 1)

( x 2) y x( x 1)

The equation is linear in y. P=−

x−2 , x( x − 1)

Q=

x 2 (2 x − 1) x −1

1 ⎞ ⎛2 = −⎜ − ⎝ x x − 1⎟⎠ ⎛ 2

I.F. = e

1 ⎞

∫ ⎜⎝ − x + x −1⎟⎠ dx

= e −2 log x + log( x −1) = e

⎛ x −1⎞ log ⎜ 2 ⎟ ⎝ x ⎠

=

x −1 x2

Hence, solution is x 1 x2

x 1 x2

y

x2

2x 1 dx c x 1

x2

x c

x 3 ( x 1) cx 2 + x 1 x 1 2 cx y = x3 + x 1 y=

Example 13: Solve ( x 2 - 1)sin x

Solution:

dy + [2 x sin x + ( x 2 1)cos x ] y = ( x 2 - 1)cos x . dx

dy 2x + 2 + cot x y = cot x dx x 1

The equation is linear in y. P=

2x x

2

1

+ cot x, Q = cot x

Differential Equations

10.77

10.4 HOMOGENEOUS LINEAR DIFFERENTIAL EQUATIONS OF HIGHER ORDER WITH CONSTANT COEFFICIENTS An ordinary differential equation of the form a0

dn y dn 1 y dn 2 y a a + + + … + an y = 0 1 2 dx n dx n 1 dx n 2

... (1)

where a0, a1, a2, …, an are constants, is known as homogeneous linear differential equation of order n with constant coefficients. This equation is known as linear since degree of dependent variable y and all its differential coefficients is one. Equation (1) can also be written as (a0 D n + a1D n 1 + a2 D n 2 + … + an ) y = 0 f (D) y = 0 n

where f (D) = a0 D + a1D

n 1

+ a2 D

n 2

+ … + an .

d is known as differential operator. dx The operator D obeys the laws of algebra.

Here D

10.4.1 General Solution of Homogeneous Linear Differential Equation The homogeneous equation f (D) y = 0 ... (2) can be solved by replacing D by m in f (D) and solving the auxiliary equation (A.E.) f ( m) = 0 ... (3) The general solution of Eq. (2) depends upon the nature of the roots of auxiliary Eq. (3). If m1, m2, m3, …, mn are n roots of the A.E., following cases arise: Case I: Real and distinct roots: If roots m1, m2, m3, …, mn are real and distinct, then the solution of Eq. (1) is given as y = c1e m1 x + c2 e m2 x + c3 e m3 x + … + cn e mn x Case II: Real and repeated roots: If two roots m1, m2 are real and equal and remaining (n – 2) roots m3, m4, …, mn are all real and distinct, then the solution of Eq. (1) is given as y = (c1 + c2 x)e m1 x + c3 e m3 x + c4 e m4 x + … + cn e mn x Note: If, however, r roots m1, m2, m3, …, mr are equal and remaining (n – r) roots mr+1, mr+2, …, mn are all real and distinct, then the solution of Eq. (1) is given as y = (c1 + c2 x + c3 x 2 + … + cr x r 1 )e m1 x + cr +1e mr +1 x + … + cn e mn x

Engineering Mathematics

10.78

Case III: Imaginary roots: If two roots m1, m2 are imaginary say, m1 i , m2 i (conjugate pair) and remaining (n – 2) roots m3, m4, …, mn are real and distinct, then the solution of Eq. (1) is given as y = e x (c1 cos x + c2 sin x) + c3 e m3 x + c4 e m4 x + … + cn e mn x Here, roots.

is the real part and

is the imaginary part of the conjugate pair of complex

Note: If, however, two pair of imaginary roots m1, m2 and m3, m4 are equal, say, m1 = m2 = + i , m3 m4 i and remaining (n 4) roots m5, m6, …, mn are real and distinct, then the solution of Eq. (1) is given as y = e x [(c1 + c2 x) cos x + (c3 + c4 x) sin x] + c5 e m5 x + c6 e m6 x + ... + cn e mn x Remark: (i) In all the above cases, c1, c2, …, cn are arbitrary constants. (ii) In the general solution of a homogeneous equation, the number of arbitrary constants is always equal to the order of that homogeneous equation. Example 1: Solve 2D2y + Dy – 6y = 0. Solution: The equation can be written as (2D 2

D 6) y

0

2m 2 m 6 (2m 3)(m 2)

0 0

Auxiliary equation is

2,

m

3 2

The roots are real and distinct. Hence, solution is 3

y = c1e −2 x + c2 e 2

x

Example 2: Solve (D 3 + D2 - 2D) y = 0. Solution: Auxiliary equation is m3

m2

2m

0

2

m(m m 2) 0 m(m 1)(m 2) 0 m 0, 1, 2 The roots are real and distinct. Hence, solution is y = c1e0 x + c2 e x + c3 e −2 x y = c1 + c2 e x + c3 e −2 x

Differential Equations

10.79

Example 3: Solve 2D2 y - 2Dy - y = 0. Solution: The equation can be written as (2D 2 2D 1) y 0 Auxiliary equation is 2m 2

2m 1 0 m= m=

2 ± 4 + 8 2 ± 2 3 1± 3 = = 4 4 2 1

3 2

,

1

3 2

The roots are real and distinct. Hence, solution is ⎛ 1+ 3 ⎞ ⎜ ⎟x 2 ⎠

y = c1e⎝

⎛ 1− 3 ⎞ ⎜ ⎟x 2 ⎠

+ c2 e⎝

Example 4: Solve D2 y + 6Dy + 9 y = 0. Solution: The equation can be written as

(D 2 + 6D + 9) y = 0 Auxiliary equation is m 2 + 6m + 9 = 0 (m + 3) 2 = 0 m 3,

3

The roots are repeated twice. Hence, solution is y = (c1 + c2 x)e

3x

Example 5: Solve (D4 - 6D 3 + 12D2 - 8D) y = 0. Solution: Auxiliary equation is m4 m( m

6m3 12m 2 8m 3

6m

2

m(m 2)(m

2

0

12m 8)

0

4m 4)

0

2

m(m 2)(m 2) 0 m = 0, 2, 2, 2 The root m = 2 is repeated three times. Hence, solution is y = c1e0 x + (c2 + c3 x + c4 x 2 )e 2 x y = c1 + (c2 + c3 x + c4 x 2 )e 2 x

Engineering Mathematics

10.80

Example 6: Solve (D4 - 6D 3 + 13D2 - 12D + 4) y = 0. Solution: Auxiliary equation is m4

6m3 13m 2 12m 4 2

(m 1) (m 2) m = 1, 1, 2, 2

2

0

0

The roots m = 1 and m = 2 are repeated twice. Hence, solution is y = (c1 + c2 x)e x + (c3 + c4 x)e 2 x Example 7: Solve (D4 + 4D2 ) y = 0. Solution: Auxiliary equation is m 4 + 4m 2 = 0 m 2 (m 2 + 4) = 0 m

0, 0 and m 2

4, m

2i

The root m = 0 is real and repeated twice and two roots are imaginary with Hence, solution is y = (c1 + c2 x)e0 x + c1 cos 2 x + c2 sin 2 x = c1 + c2 x + c1 cos 2 x + c2 sin 2 x Example 8: Solve (D4 + 4) y = 0. Solution: Auxiliary equation is m4 + 4 = 0 m 4 + 4 + 4m 2 − 4m 2 = 0 (m 2 + 2) 2 − (2m) 2 = 0 (m 2 + 2 + 2m)(m 2 + 2 − 2m) = 0 (m 2 + 2m + 2)(m 2 − 2m + 2) = 0 m = −1 ± i and m = 1 ± i The roots are imaginary with α1 = −1, β1 = 1 and α 2 = 1, β 2 = 1. Hence, solution is y = e x (c1 cos x + c2 sin x) + e x (c3 cos x + c4 sin x)

= 0,

= 2.

Differential Equations

10.81

Example 9: Solve (D 3 - 5D2 + 8D - 4) y = 0. Solution: Auxiliary equation is m3 5m 2 8m 4 (m 1)(m

2

4m 4)

0 0

2

(m 1)(m 2) 0 m = 1, 2, 2 The roots are real and distinct, but second root m = 2 is repeated twice. Hence, solution is y = c1e x + (c2 + c3 x)e 2 x Example 10: Solve (D4 + 8D2 + 16) y = 0. Solution: Auxiliary equation is m 4 + 8m 2 + 16 = 0 (m 2 + 4) 2 = 0 m = ±2i, ± 2i The pair of roots is imaginary and repeated twice with = 0, = 2. Hence, solution is y = e0 x [(c1 + c2 x) cos 2 x + (c3 + c4 x) sin 2 x] = (c1 + c2 x) cos 2 x + (c3 + c4 x) sin 2 x Example 11: Solve (D2 + 1) 3 (D2 + D + 1)2 y = 0. Solution: Auxiliary equation is (m 2 + 1)3 (m 2 + m + 1) 2 = 0 m 2 + 1 = 0, m 2 + m + 1 = 0 m = ±i , m =

1 i 3 2

Both pair of roots are imaginary and first pair is repeated thrice with 1 3 , . second pair is repeated twice with 2 2 Hence, solution is

= 0,

y = e0 x [(c1 + c2 x + c3 x 2 ) cos x + (c4 + c5 x + c6 x 2 ) sin x] +e

x 2

(c7 + c8 x) cos

3 3 x + (c9 + c10 x) sin x 2 2

= 1 and

10.82

Engineering Mathematics

Example 12: Solve (D 3 - 2D2 - 5D + 6) y = 0, y(0) = 0, y (0) = 0, y (0) = 1. Solution: Auxiliary equation is m3

2m 2 5m 6

0

2

(m 1)(m m 6) 0 (m 1)(m 2)(m 3) 0 m 1,

2, 3

The roots are real and distinct. Hence, solution is y = c1e x + c2 e

2x

+ c3 e3 x

... (1)

Differentiating Eq. (1), c1e x

3c3 e3 x

... (2)

y ′′ = c1e x + 4c2 e −2 x + 9 c3 e3 x

... (3)

y

2c2 e

2x

Differentiating Eq. (2), Putting x = 0 in Eqs. (1), (2) and (3), y (0) = c1 + c2 + c3 0 = c1 + c2 + c3 c1 + c2 + c3 = 0 y (0) c1 2c2 0 c1 2c2

3c3

... (4) 3c3

c1 2c2 3c3 0

... (5)

y (0) = c1 + 4c2 + 9c3 1 = c1 + 4c2 + 9c3 c1 + 4c2 + 9c3 = 1

... (6)

Solving Eqs. (4), (5) and (6), 1 1 1 , c2 , c3 6 15 10 1 x 1 2 x 1 3x e e e 6 15 10

c1 y

Example 13: Solve (D 3 + o 2 D) y = 0, y(0) = 0, y(1) = 0, y (0) + y (1) = 0. Solution: Auxiliary equation is m3 +

2

m = 0, m = 0,

m( m 2 +

2

)=0 m=± i

First root is real and second pair of roots is imaginary with Hence, solution is y = c1 + c2 cos x + c3 sin x

= 0,

= . ... (1)

Differential Equations

10.83

Differentiating Eq. (1), y 0 c2 sin x c3 cos x Putting x = 0 in Eqs. (1) and (2) and using given initial conditions,

... (2)

y (0) = 0, c1 + c2 = 0 y (1) 0, c1 c2 0 y ′(0) + y ′(1) = 0

... (3) ... (4)

c3 − c3 = 0 Solving Eqs. (3) and (4), c1 = 0, c2 = 0 and c3 cannot be determined. Hence, solution is y = c3 sin x, where c3 is arbitrary constant. Exercise 10.8 Solve the following differential equations: 1. (D 2 D 2) y 0. 10. (9D 2 30D 25) y [Ans. : y = c1e 2. (4D 2 8D 5 y )

2x

4D 12) y

5x 2

11. (D 2

2x

Ans. : y = e

+ c2 e(

2

3)x

3 14. (D 9D) y

Ans. : y = (c1 + c2 x)e 7. (D 2 + 2p D + p 2 ) y = 0. [Ans. : y = (c1 + c2 x)e 2 8. (9D 12D 4) y 0. Ans. : y = (c1 + c2 x)e 20D 4) y

0.

Ans. : y = c1 + c2 e3 x + c3 e

6. (4D 2 + 4D + 1) y = 0.

9. (25D 2

(c1 cos 2 x + c2 sin 2 x)

13. [D 2 2aD (a 2 b 2 ) y ] 0. [ Ans. : y = e ax (c1 cos bx + c2 sin bx)]

5. (D 2 + 4D + 1) y = 0. 3)x

3x

4x

Ans. : y = c1e + c2 e 2

0.

12. (D 2 + 6D + 11) y = 0.

0. 2x

Ans. : y = c1e(

6D 25) y

[Ans. : y = e3 x (c1 cos 4 x + c2 sin 4 x)]

0.

Ans. : y = c1e6 x + c2 e 2D 8) y

Ans. : y = (c1 + c2 x)e 3

0. x

4. (D 2

5x

+ c2 e ]

Ans. : y = c1e 2 + c2 e 3. (D 2

0.

x

x 2

D 3) y

0. x

Ans. : y = c1e + c2 e x + c3 e3 x x

2x 3

]

16. (D3 6D 2 11D 6) y

0.

Ans. : y = c1e x + c2 e 2 x + c3 e3 x 17. (D3 6D 2 12D 8) y

0.

Ans. : y = (c1 + c2 x + c3 x 2 )e 2 x

0.

Ans. : y = (c1 + c2 x)e

15. (D3 3D 2

3x

2x 5

18. (D3 + D) y = 0. [ Ans. : y = c1 + c2 cos x + c3 sin x]

Engineering Mathematics

10.84

29. (4D 2 + 12D + 9) y = 0, y (0) 1, y (0) 2.

19. (D3 + 5D 2 + 8D + 6) y = 0. Ans. : y = c1e 4 20. (8D

3x

+ e x (c2 cos x + c3 sin x)

6D3 7D 2

6D 1) y

21. (D − 2D + D ) y = 0. 3

2 30. (D 4D 5) y y (0) 2, y (0)

Ans. : y

22. (D 4 3D3 3D 2

x

31. (9D 2

23. (D 4 8D 2 9) y

Ans. : y

0.

D3 14D 2 16D 32) y

0.

Ans. :

25. (D

2D

3

+ c3 cos 4 x + c4 sin 4 x

9D

2

0,

0.

y = c1e x + c2 e x + c3 cos3 x + c4 sin 3 x

2x

6D 1) y

y (1) = e , y (2) = 1. D) y

Ans. :

4

e 2 x (2 cos x 5sin x)

1 3

Ans. : y = c1 + (c2 + c3 x + c4 x 2 )e x

y = c1e x + c2 e

0, 1.

2

Ans. : y = c1 + c2 x + (c3 + c4 x)e

24. (D 4

3x 2

0.

x x ⎡ ⎤ 4 2 Ans.: y = c e + c e + c3 e x + c4 e − x ⎥ ⎢ 1 2 ⎣ ⎦ 4

x 1 e 2

Ans. : y

10D 50) y

0.

⎡ Ans. : y = e 2 x (c1 cos x + c2 sin x) ⎤ ⎥ ⎢ + e −3 x (c3 cos x + c4 sin x) ⎥⎦ ⎢⎣ 26. (D 4 + 18D3 + 81) y = 0. Ans. : y = (c1 + c2 x) cos 3 x

25) y

0.

Ans. : y = e x [(c1 + c2 x) cos 2 x + (c3 + c4 x) sin 2 x] 28. (D2 + D – 2)y = 0, y(0) = 4, y (0) = –5. Ans. : y = e x + 3e

2x

2 3

1 x

2 e

2 3

x

e3

32. (4D3 4D 2 9D 9) y 0, y (0) = 1, y (0) = 0, y (0) = 0. 3x −3 x ⎡ ⎞⎤ 1⎛ x 2 2 9 5 Ans. : y = e − e + e ⎢ ⎜ ⎟⎠ ⎥ 5⎝ ⎦ ⎣

33. (D3 D 2 2) y 0, y (0) y (0) 2, y (0) 3.

2,

Ans. : y = e x + e x (cos x + 2sin x) 34. (D 4 3D3 ) 0, y (0) 2, y (0) = 5, y (0) = 15, y (0) = 27. ⎡⎣ Ans. : y = 1 + 2 x + 3 x 2 + e3 x ⎤⎦

+ (c3 + c4 x) sin 3 x 4 3 2 27. (D 4D 14D 20D

e

35. (D 4

3D3 2D 2 ) y 0, y (0) 2, y (0) = 0, y (0) = 2, y (0) = 2. Ans. : y

2(e x

x)

Differential Equations

10.85

10.5 NON-HOMOGENEOUS LINEAR DIFFERENTIAL EQUATIONS OF HIGHER ORDER WITH CONSTANT COEFFICIENTS An ordinary differential equation of the form dn y dn 1 y dn 2 y + a1 n 2 + a2 n 2 + … + an y = Q( x) ... (1) n dx dx dx where a0, a1, a2, ..., an are constants and Q is a function of x, is known as Nonhomogeneous linear differential equation with constant coefficients. Equation (1) can also be written as a0

(a0 D n + a1D n 1 + a2 D n 2 + … + an ) y = Q( x)

... (2)

f (D) y = Q( x) n

where f (D) = a0 D + a1D

n 1

+ a2 D

n 2

+ … + an

10.5.1 General Solution of Non-Homogeneous Linear Differential Equation A general solution of Eq. (1) is obtained in two parts as General solution = complimentary function + particular integral y = C.F + P.I. Complimentary function (C.F.) is the general solution of the homogeneous equation obtained by putting Q(x) = 0 in Eq. (1). Particular integral (P.I.) is any particular solution of the non-homogeneous Eq. (1) and contains no arbitrary constants.

Inverse Operator and Particular Integral 1 f (D) is known as differential operator and is known as inverse differential f (D) operator. ⎡ 1 ⎤ f ( D) ⎢ Q( x) ⎥ = Q(x) f ( D ) ⎣ ⎦ This shows that

1 1 Q( x) satisfies the equation f (D) y = Q( x) and since Q( x) f (D) f (D)

does not contain any arbitrary constants, gives the P.I. of the equation f (D) y = Q( x) . P.I. =

Hence,

1 Q( x) f (D)

(i) If f (D) = D, then P.I. =

1 Q( x) = Q( x) dx D

Engineering Mathematics

10.86

(ii) If f (D)

D a, then equation f (D) y = Q( x) becomes (D a ) y

dy ay dx is a first order linear differential equation.

Q( x) Q( x)

I.F. = e

adx

=e

ax

Solution is ye

ax

= e

ax

Q( x) dx + c

y = e ax Q( x)e

ax

dx + ce ax

Here, ceax is the complimentary function since it contains arbitrary constant c and e ax Q( x)e ax dx is the particular integral. P.I. =

Hence,

1 D a

Q( x) = e ax Q( x)e

ax

dx

10.5.2 Direct (Short-cut) Method of Obtaining Particular Integral (P.I) This method depends on the nature of Q(x) in Eq. (1). Particular Integral by this method can be obtained when Q(x) has the following forms: (i) Q( x) = e ax + b (ii) Q( x) = sin(ax + b) or cos(ax + b) (iii) Q( x) = x m or polynomial in x (iv) Q( x) = e ax v( x) (v) Q( x) = xv( x) Case I: Q( x) = e ax + b : f (D) y = e ax + b Now, Consider

D(e ax + b ) = ae ax + b , D 2 (e ax + b ) = a 2 e ax + b , … , D n e ax + b = a n e ax + b f (D)(e ax b ) = (a0 D n + a1D n 1 + … + an )e ax = (a0 a n + a1a n 1 + … + an )e ax

Operating both the sides with

1 , f (D)

(

)

1 1 ⎡ f (D) e ax + b ⎤ = ⎡ f (a )e ax + b ⎤⎦ ⎣ ⎦ f ( D) f ( D) ⎣ 1 e ax + b = f (a ) e ax + b f ( D)

b b

= f (a )e ax

Differential Equations

1 ax + b e f (a) 1 e ax + b f (D)

1 e ax + b , f (D) 1 ax + b e , f (a)

10.87

f (a)

0

f (a)

0

1 ax + b e if f (a) 0 f (a) Note: If f (a ) = 0, then (D – a) is a factor of f (D) and hence, above rule fails. Let f (D) (D a )f (D), where f (a ) 0 Hence, P.I. =

1 ax +b 1 = e e ax +b ( D − a ) ( D) f ( D) 1 1 = ⋅ e ax +b (a) (D − a) 1 1 = ⋅ e ax ∫ e − ax e ax +b dx = ⋅ e ax ⋅ xeb (a) (a) 1 ax +b =x e (a) f (D) (D a ) (D) f (D) (D a ) (D) (D) f (a ) =f (a )

P.I. =

Since

Substituting in Eq. (3), 1 e ax + b f (D)

x

1 e ax + b where f (a ) f (a )

... (3)

0

If f (a ) = 0, then repeating the above process, 1 1 e ax + b x x e ax + b f (D) f (a ) 1 e ax + b f (a) In general if (D – a)r is a factor of f (D) , then 1 1 e ax = x r ( r ) e ax + b f (D) f (a) = x2

Hence,

P.I. = x r

Example 1: Solve (D2 - 3D + 2) y = e 3 x . Solution: Auxiliary equation is m 2 3m 2 0 m = 1, 2

1 e ax + b . f ( r ) (a)

where f (a) 0

Engineering Mathematics

10.88

C.F. = c1e x + c2 e 2 x P.I. =

1 1 1 e3 x = 2 e3 x = e3 x 2 3 − 3(3) + 2 D − 3D + 2 2

Hence, the general solution is 1 y = c 1e x + c2 e 2 x + e3 x 2 Example 2: Solve (D2

5x

6D 9) y

log 2.

Solution: Auxiliary equation is m2

6m 9

0, (m 3) 2

C.F. = (c1 + c2 x)e P.I.

0,

m

3, 3

3x

1 (5 x log 2) D + 6D + 9 1 1 (e x log 5 ) (log 2)e0 x (D + 3) 2 (D + 3) 2 1 1 e x log 5 log 2 e0 x 2 (log 5 + 3) (0 + 3) 2 2

5x (log 5 + 3) 2

log 2 9

Hence, the general solution is y

(c1 c2 x)e

3x

5x (log 5 + 3) 2

log 2 9

Example 3: Solve (D 3 - D2 + 4D - 4) y = e x . Solution: Auxiliary equation is m3

m2

4m 4 0, (m 1)(m 2 4) 0 m 1 0, m2 4 0 m = 1, m = ± 2i

C.F. = c1e x + c2 cos 2 x + c3 sin 2 x P.I.

1 D

3

D

2

4D 4

ex

x

3D

2

1 ex 2D 4

x

1 ex 3 2 4

Hence, the general solution is y = c1e x + c2 cos 2 x + c3 sin 2 x +

xe x 5

x x e 5

Differential Equations

10.89

Example 4: Solve ( D6 – 64) y = e x cosh 2 x . Solution: Auxiliary equation is m6 3 2

(m )

64 (8)

2

0 (m3 8)(m3 8)

0,

(m 2)(m 2

2m 4)(m 2)(m 2 m + 2 = 0, m m

m

2

2m 4)

0 0

2m + 4 = 0,

2 = 0, m + 2m + 4 = 0 2

2, m 1 i 3,

m

2,

m

1 i 3

Two roots are real and two pair of roots are complex. C.F. = c1e Now,

2x

(

)

+ c2 e 2 x + e x c3 cos 3 x + c4 sin 3 x + e e2 x + e 2

e x cosh 2 x = e x

1

P.I.

2x

D 64 1 1 e3 x 6 2 3 64

( c cos 5

3 x + c6 sin 3 x

1 3x (e + e x ) 2

=

1

1 3x (e e x) D 64 2 1 1 e3 x e x x e 2 665 63 ( 1)6 64

e x cosh 2 x

6

x

6

Hence, the general solution is y = c1e

2x

+ c2 e 2 x + e x (c3 cos 3 x + c4 sin 3 x) 1 e3 x 2 665

e x (c5 cos 3 x c6 sin 3 x)

Example 5: Solve (D 3 - 4D) y = 2cosh 2 (2 x ). Solution: Auxiliary equation is m3

4m

m( m 2

0,

m = 0,

4)

0

m = ±2 0x

C.F. = c1e + c2 e 2 x + c3 e

2x

= c1 + c2 e 2 x + c3 e

2x

Now, X = 2 cosh 2 2 x = 2

e2 x + e 2

2x

2

=

1 4x (e + e 2

4x

+ 2)

e x 63

)

Engineering Mathematics

10.90

P.I.

1 D

3

2 cosh 2 2 x

4D 1

1 4x 1 1 e 4 x 2) (e (e 4 x e 2 D3 4D D3 4D 2 1 1 1 1 2e 0 x e4 x + e 4x + x 2 = 2 43 16 ( 4)3 16 3D 4 1 1 4x e 2 48 1 e4 x e 2 48

1 e ( 48) 4x

4x

x

2e0 x )

1 2e 0 x 0 4

sinh 4 x 48

x 2

4x

x 4

Hence, the general solution is y

c1 c2 e 2 x

c3 e

sinh 4 x 48

2x

x 4

x

Example 6: Solve (4D2 - 4D + 1) y = e 2 . Solution: Auxiliary equation is 4m 2 − 4m + 1 = 0, (2m − 1) 2 = 0,

m=

1 1 , 2 2

x

C.F. = (c1 + c2 x)e 2 P.I.

4D 2 x 2 2x = e 8 Hence, the general solution is

x 1 e2 4D 1

x

y = (c1 + c2 x)e 2 +

x

x 1 e2 8D 4

x2

1 2x e 8

x 2 2x e 8

Example 7: Solve (D 3 - 5D2 + 8D - 4) y = e 2 x + 2e x + 3e - x + 2. Solution: Auxiliary equation is m3 5m 2 8m 4 0 2 (m − 1)(m − 4m + 4) = 0, (m − 1)(m − 2) 2 = 0 m = 1, m = 2, 2 C.F. = c1e x + (c2 + c3 x)e 2 x

1 (e 2 x + 2e x + 3e − x + 2e0⋅ x ) D3 − 5D 2 + 8D − 4 1 1 1 1 0⋅ x = x⋅ 2 ⋅ e2 x + x ⋅ 2 2e x + 3e − x + 2e −1 − 5 − 8 − 4 −4 3D − 10D + 8 3D − 10D + 8

P.I. =

Differential Equations

1 1 1 1 e2 x + x 2e x − ⋅ 3e − x − 6D − 10 3 − 10 + 8 18 2 1 1 −x 1 2 2x x =x e + 2 xe − e − 2 12 − 10 6 2 x 1 1 = e 2 x + 2 xe x − e − x − 2 6 2 Hence, the general solution is x2 2x y c1e x (c2 c3 x)e 2 x e 2

10.91

= x2 ⋅

2 xe x

1 e 6

Case II: Q( x) = sin(ax + b) or cos( ax + b) (i) If Q(x) = sin(ax + b), then Eq. (2) reduces to f (D) y = sin(ax + b) Now

D[sin(ax + b)] = a cos(ax + b) D 2 [sin( ax b)]

( a 2 ) sin( ax b)

D3 [sin(ax b)]

a 3 cos(ax b)

D 4 [sin( ax + b)] = a 4 sin( ax + b) In general,

(D 2 ) 2 [sin( ax b)]

( a 2 ) 2 sin( ax b)

(D 2 ) r [sin( ax b)]

( a 2 ) r sin( ax b)

(D 2 ) sin(ax b)

( a 2 ) sin(ax b)

This shows that

Operating both the sides with 1 (D 2 )

1 , f (D 2 )

(D 2 ) sin(ax b)

1 (D 2 )

sin(ax + b) = f (−a 2 )

( a 2 ) sin(ax b) 1 sin( ax + b) f (D 2 )

1 1 sin( ax + b) = sin( ax + b) f (−a 2 ) f (D2 ) 1 1 sin(ax + b) = sin(ax + b) 2 f (D ) f (−a 2 ) If f (D) = (D 2 ), then 1 P.I. = sin(ax + b) f ( D)

=

1 1 sin(ax + b) = sin(ax + b), if f (−a 2 ) ≠ 0 2 2 f (D ) f (− a )

x

1 2

Engineering Mathematics

10.92

2 If ( a )

0, then (D 2 + a 2 ) is a factor of f (D 2 ) and hence, above rule fails. 1 1 1 ⎡ I.P. of ei ( ax + b ) ⎤⎦ = I.P. of sin(ax + b) = ei ( ax + b ) (D 2 ) (D 2 ) ⎣ (D 2 ) 1 ⎡⎣∵ (i 2 a 2 ) = (−a 2 ) = 0 ⎤⎦ = I.P. of x ⋅ ei ( ax + b ) ′(D 2 ) 1 1 = I.P. of x ⋅ ei ( ax + b ) ei ( ax + b ) = I.P. of x ⋅ 2 2 ′(i a ) ′(−a 2 ) 1 = x⋅ sin(ax + b) ′(−a 2 )

P.I. =

If

( a2 )

0, then

1 sin(ax b) f (D 2 ) In general, if P.I. =

(r )

( a2 )

x2

1 sin(ax b), f ( a2 )

(ii) Similarly, if Q(x) = cos(ax + b) 1 1 P.I. cos(ax b) cos(ax b), f ( a 2 ) 2 f (D ) f ( a2 )

where

( r +1)

( a2 )

0

0

0, then P.I.

If

0

0, then

1 1 sin(ax + b) = x ( r +1) ( r +1) sin(ax + b), 2 f (D ) f ( a2 )

2 If ( a )

( a2 )

where

1 cos(ax b) f (D 2 )

x

1 cos(ax b) f ( a2 )

( a2 )

0, then 1 P.I. cos(ax b) f (D 2 )

In general, if P.I.

(r )

( a2 )

x2

1 cos(ax b), f ( a2 )

where f ( a 2 )

0

0, then

1 cos(ax b) f (D 2 )

x r +1

1 cos(ax b), f ( r +1) ( a 2 )

where

( r +1)

( a2 )

0

Note: If after replacing D2 by –a2, f (D) contains terms of D, then denominator is rationalised to obtained the even powers of D. Example 1: Solve (D2 + 9) y = sin 4 x . Solution: Auxiliary equation is m 2 + 9 = 0, m = ± 3i (imaginary) C.F. = c1 cos 3 x + c2 sin 3 x P.I.

1 sin 4 x D2 9

1 sin 4 x 42 9

1 sin 4 x 7

Differential Equations

10.93

Hence, the general solution is y

c1 cos 3 x c2 sin 3 x

1 sin 4 x 7

Example 2: Solve (D4 + 2a 2 D2 + a 4 ) y = 8 cos ax . Solution: Auxiliary equation is m 4 + 2a 2 m 2 + a 4 = 0 (m 2 + a 2 ) 2 = 0, m = ± ia, ± ia (imaginary and repeatedd twice) C.F. = (c1 + c2 x) cos ax + (c3 + c4 x) sin ax 1 1 ⋅ 8 cos ax = x ⋅ 3 ⋅ 8 cos ax 2 2 4 4D + 4a 2 D D + 2a D + a 1 1 ⋅ 8 cos ax = x2 ⋅ ⋅ 8 cos ax = x 2 ⋅ 12D 2 + 4a 2 12(− a 2 ) + 4a 2

P.I. =

4

=−

x2 cos ax a2

Hence, the general solution is y

(c1 c2 x) cos ax (c3

c4 x) sin ax

x2 cos ax a2

Example 3: Solve (D2 + 3D + 2) y = sin 2 x . Solution: Auxiliary equation is m 2 + 3m + 2 = 0, (m + 1)(m + 2) = 0 m 1, 2 (Real and distinct) C.F. = c1e x + c2 e

2x

1 1 ⋅ sin 2 x = ⋅ sin 2 x −4 + 3D + 2 D 2 + 3D + 2 1 1 (3D + 2) = ⋅ sin 2 x = ⋅ sin 2 x 3D − 2 (3D − 2) (3D + 2) (3D + 2) 3D + 2 sin 2 x = sin 2 x = 9( −4) − 4 9D 2 − 4 3 1 3D + 2 sin 2 x = − (D sin 2 x) − sin 2 x = 40 20 −40 3 1 1 = − ⋅ 2 cos 2 x − sin 2 x = − (3 cos 2 x + sin 2 x) 40 20 20

P.I. =

Hence, the general solution is y

c1e

x

c2 e

2x

1 (3cos 2 x sin 2 x) 20

Engineering Mathematics

10.94

Example 4: Solve (D 3 - 3D2 + 4D - 2) y = e x + cos x . Solution: Auxiliary equation is m3 3m 2 (m

1)(m

2

4m 2 2m

m 1 0, m = 1,

2) m

x

0

2

0 2m 2 0 m = 1± i

(imaginary)

x

C.F. = c1e + e (c2 cos x + c3 sin x) P.I. = = = = = =

1 (e x + cos x) D3 − 3D 2 + 4D − 2 1 1 x⋅ 2 ex + cos x 2 3D − 6D + 4 D( −1 ) − 3( −12 ) + 4D − 2 1 1 1 (3D − 1) ⋅ ex + ⋅ x cos x = xe x + cos x 3−6+ 4 3D + 1 (3D + 1) (3D − 1) 3D − 1 3D − 1 cos x = xe x + cos x xe x + 2 9D − 1 9( −12 ) − 1 1 1 xe x − (3D cos x − cos x) = xe x − ( −3 sin x − cos x) 10 10 1 xe x + (3 sin x + cos x) 10

Hence, the general solution is y = (c1 + c2 cos x + c3 sin x)e x + xe x + Example 5: Solve (D - 1)2 (D2 + 1) y = e x + sin 2

x . 2

Solution: Auxiliary equation is (m 1) 2 (m 2 1) (m − 1) 2 = 0,

0

m2 + 1 = 0

m = 1, 1 (repeated twice), m = ± i (imaginary) C.F. = (c1 + c2 x)e x + c3 cos x + c4 sin x Now,

Q( x) = e x + sin 2 P.I. =

1 − cos x x = ex + 2 2

⎛ x e0 x cos x ⎞ 1 − ⎜e + ⎟ 2 2 ⎠ (D − 1) 2 (D 2 + 1) ⎝

1 (3sin x + cos x) 10

Differential Equations

=

e0 x cos x 1 1 1 1 x e ⋅ + ⋅ − 2 ⋅ 2 2 2 2 (D − 1) (1 + 1) (0 − 1) (0 + 1) 2 (D + 1)(D − 2D + 1) 2

= x⋅ =

10.95

1 1 cos x ex 1 ⋅ + − 2 ⋅ 2(D − 1) 2 2 (D + 1)(−12 − 2D + 1) 2

1 1 1 x2ex 1 1 x2 ex 1 1 cos x = + + cos x dx ⋅ + + ⋅ 2 2 2 2 4 (D + 1) D 4 2 4 (D 2 + 1) ∫ x2ex 4 2 x xe 4

1 2 1 2

1 1 x2ex 1 1 1 sin x x sin x 2 4 D +1 4 2 4 2D x x2ex 1 x sin x dx ( cos x) 8 4 2 8

Hence, the general solution is (c1 c2 x)e x

y

c3 cos x c4 sin x

x2ex 4

1 2

x cos x 8

Case III: Q(x) = x m In this case, Eq. (2) reduces to f (D) y = x m . Hence, P.I. =

1 xm f (D)

= [ f (D)] 1 x m = [1 + f (D)] 1 x m Expanding in ascending powers of D up to Dm using Binomial Expansion, since Dnxm = 0 when n > m, P.I. = (a0 + a1D + a2 D 2 + … + am D m ) x m Example 1: Solve (D2 + 2D + 1) y = x . Solution: Auxiliary equation is m 2 + 2m + 1 = 0 (m 1) 2

0,

m

C.F. = (c1 + c2 x)e P.I.

1, 1 (real and repeated twice) x

1 x D + 2D + 1 2

1 x (1 + D) 2

(1 D) 2 x (1 2D 3D 2 …) x x 2 0 x 2 Hence, the general solution is y (c1 c2 x)e

x

x 2

x 2Dx 3D 2 x …

Engineering Mathematics

10.96

Example 2: Solve (D4 - 2D 3 + D2 ) y = x 3 . Solution: Auxiliary equation is m4

2m3

m2

0,

2

0,

2

m (m 1)

m 2 (m 2

2m 1) m

0

0, 0, m

1, 1

Both the roots are real and repeated twice. C.F. = (c1 + c2 x)e0 x + (c3 + c4 x)e x = c1 + c2 x + (c3 + c4 x)e x 1 1 ⋅ x3 = 2 2 x3 D 4 − 2D3 + D 2 D (D − 2D + 1) 1 1 1 = 2 ⋅ x 3 = 2 (1 − D) −2 x 3 = 2 (1 + 2D + 3D 2 + 4D3 + 5D 4 + …) x 3 D D (1 − D) 2 D 1 = 2 ( x 3 + 2Dx 3 + 3D 2 x 3 + 4D3 x 3 + 5D 4 x 3 + …) D 1 1 = 2 ( x 3 + 2 ⋅ 3x 2 + 3 ⋅ 6 x + 4 ⋅ 6 + 0) = 2 ( x 3 + 6 x 2 + 18 x + 24) D D ⎛ x4 ⎞ x3 x2 = ∫ ⎡⎣ ∫ ( x3 + 6 x 2 + 18 x + 24)dx ⎤⎦ dx = ∫ ⎜ + 6 + 18 + 24 x⎟ dx 3 2 ⎝ 4 ⎠

P.I. =

=

x5 x4 x3 x 2 x5 x 4 + + 3x 3 + 12 x 2 + 2 + 9 + 24 = 20 4 3 2 20 2

Hence, the general solution is x5 x 4 + + 3 x 3 + 12 x 2 20 2

y = c1 + c2 x + (c3 + c4 x)e x +

Example 3: Solve (D 3 - D2 - 6D) y = 1 + x 2 . Solution: Auxiliary equation is m3

m2

6m

m(m 3)(m 2)

0, m(m 2 0,

m 6) m

0x

3x

0

0, 3,

2

2x

(real and distinct) 3x

C.F. = c1e + c2 e + c3 e = c1 + c2 e + c3 e 2 x 1 1 P.I. = 3 (1 + x 2 ) = (1 + x 2 ) 2 D − D − 6D ⎡ D2 − D ⎤ − 6D ⎢1 − 6 ⎥⎦ ⎣ −1

=−

1 ⎡ ⎛ D2 − D ⎞ ⎤ 2 ⎢1 − ⎥ (1 + x ) 6D ⎣ ⎝⎜ 6 ⎟⎠ ⎦

=−

2 ⎤ 1 ⎡ ⎛ D2 − D ⎞ ⎛ D2 − D ⎞ ⎢1 + ⎜ + + …⎥⎥ (1 + x 2 ) ⎟ ⎜ ⎟ 6D ⎢ ⎝ 6 ⎠ ⎝ 6 ⎠ ⎥⎦ ⎣

Differential Equations

10.97

=−

⎤ 1 ⎡ D 2 − D D 4 − 2D3 + D 2 + + …⎥ (1 + x 2 ) ⎢1 + 6D ⎣ 6 36 ⎦

=−

⎤ 1 ⎡ D 7D2 + Higher powers of D ⎥ (1 + x 2 ) ⎢1 − + 6D ⎣ 6 36 ⎦

=−

1 ⎡ 1 7 ⎤ (1 + x 2 ) − D(1 + x 2 ) + D 2 (1 + x 2 ) ⎥ ⎢ 6D ⎣ 6 36 ⎦

=−

1 ⎡ 1 7 ⎤ 1 + x 2 − ( 2 x ) + ( 2) ⎥ 6D ⎢⎣ 6 36 ⎦

=−

1 ⎡ 2 x 25 ⎤ 1 ⎛ x 25 ⎞ x − + ⎥ = − ∫ ⎜ x 2 − + ⎟ dx ⎢ ⎝ 6D ⎣ 3 18 ⎦ 6 3 18 ⎠

1 ⎛ x 3 x 2 25 ⎞ x =− ⎜ − + 6⎝ 3 6 18 ⎟⎠ Hence, the general solution is c1 c2 e3 x

y

c3 e

2x

x2 2

1 3 x 18

25 x 6

Example 4: Solve ( D 3 − 2D + 4) y = x 4 + 3 x 2 – 5 x + 2. Solution: Auxiliary equation is m3 2m 4 (m 2)(m

2

0 2m 2)

0

2

m 2 0, m 2 (real),

m 2m 2 0 m 1 i (complex)

C.F. = c1e −2 x + e x (c2 cos x + c3 sin x) P.I.

(D3

1 ( x4 2D 4)

1 D3 2D 1 4 4 D3

1 1 4

3x 2

1

( x4

3x 2

D3

2D 4 2D 4

D3

2D 4

+

5 x 2)

2D

2

D3

4

D3 1 1 4

5 x 2)

2D

3

4

4

… ( x4 4D 2 16

4D 4 16

3 x 2 – 5 x 2) 8D3 64

16D 4 + higher powers of D ( x 4 256

3x 2

5 x 2)

Engineering Mathematics

10.98

D2 4

1 D 1 4 2

D3 8

3D 4 16

higher powers of D ( x 4

3x 2

5 x 2)

1 1 1 2 4 ( x 4 3 x 2 5 x 2) D( x 4 3 x 2 5 x 2) D ( x 3 x 2 5 x 2) 4 2 4 1 3 4 3 4 4 D ( x 3 x 2 5 x 2) D ( x 3 x 2 5 x 2) 8 16 1 1 1 1 3 ( x 4 3x 2 5 x 2) (4 x 3 6 x 5) (12 x 2 6) (24 x) (24) 4 2 4 8 16 1 4 x 4

2 x3

6x2

7 2

5x

Hence, the general solution is y

c1e

2x

e x (c2 cos x c3 sin x)

1 4 x 4

2 x3 6 x 2 5 x

7 2

Example 5: Solve (D4 + 2D 3 - 3D2 ) y = x 2 + 3e 2 x . Solution: Auxiliary equation is m4 2

2m3 3m 2

0

2

m (m 2m 3) 0, m 2 (m 1)(m 3) 0 m 0, 0 (repeated twice), m 1, 3 (real and distinct) C.F. = (c1 + c2 x)e0 x + c3 e x + c4 e

3x

= c1 + c2 x + c3 e x + c4 e

3x

1 1 x2 + 4 3e 2 x D 4 + 2D3 − 3D 2 D + 2D3 − 3D 2 1 1 = x2 + 3e 2 x 2 1 6 + 16 − 12 ⎛ ⎞ + D 2 D −3D 2 ⎜1 − ⎟ ⎝ 3 ⎠

P.I. =

1 ⎛ D 2 + 2D ⎞ = − 2 ⎜1 − ⎟ 3 ⎠ 3D ⎝

−1

x2 +

3e 2 x 20

2 ⎤ 3e 2 x 1 ⎡ D 2 + 2D ⎛ D 2 + 2D ⎞ 2 … x = − 2 ⎢1 + +⎜ + ⎥ + 3 ⎟⎠ 20 3 3D ⎢ ⎝ ⎥⎦ ⎣

=−

⎞ 1 ⎛ D 2 + 2D 4D 2 3e 2 x 1+ + + higher powers of D⎟ x 2 + 2 ⎜ 3 9 20 3D ⎝ ⎠

=−

1 3D 2

1 ⎛ 2 2 2 7 2 2 ⎞ 3 2x ⎜⎝ x + Dx + D x ⎟⎠ + e = − 2 20 3 9 3D

7 ⎤ 3e 2 x ⎡ 2 2 ⎢⎣ x + 3 (2 x) + 9 (2) ⎥⎦ + 20

Differential Equations

1 3

4 14 3e 2 x x dx dx 3 9 20

x2

1 x4 3 12

2 x3 9

7 x2 9

10.99

1 3

x3 3

2 2 x 3

x2 x2 9 4

2x 3

7 3

14 3e 2 x x dx 9 20

3e 2 x 20

Hence, the general solution is c1 c2 x c3 e x

y

c4 e

3x

3e 2 x 20

Example 6: Solve (D2 + 2) y = x 3 + x 2 + e -2 x + cos 3 x. Solution: Auxiliary equation is m 2 + 2 = 0,

m = ± i 2 (imaginary)

C.F. = c1 cos 2 x + c2 sin 2 x 1 ( x 3 + x 2 + e 2 x + cos 3 x) D2 + 2 1 1 1 ( x3 + x 2 ) + 2 e −2 x + 2 cos 3x = 2 D +2 D +2 ⎛ D ⎞ 2 ⎜1 + ⎟ 2 ⎠ ⎝

P.I. =

−1

=

1 ⎛ D2 ⎞ 1 −2 x 1 3 2 e + 2 cos 3 x ⎜1 + ⎟ (x + x ) + 2⎝ 2 ⎠ 4+2 −3 + 2

=

⎞ 3 1 ⎛ D2 D4 e −2 x cos 3 x 2 … 1 − + − + + − ( x x ) ⎜ ⎟ 2⎝ 2 4 6 7 ⎠

1 3 1 e −2 x cos 3 x ( x + x 2 ) − D2 ( x3 + x 2 ) + − 2 4 6 7 −2 x 1 1 cos 3 x e = ( x 3 + x 2 ) − ( 6 x + 2) + − 2 4 6 7 Hence, the general solution is 1 3 e 2x y c1 cos 2 x c2 sin 2 x ( x x 2 3 x 1) 2 6 =

Case IV: Q = eaxV, where V is a function of x. In this case, Eq. (2) reduces to f (D)y = eaxV. Let u be a function of x. Then

D(e ax u ) = e ax Du + ae ax u = e ax (D + a )u D 2 (e ax u ) = D e ax (D + a )u = ae ax (D + a )u + e ax (D 2 + aD)u = e ax (D 2 + 2aD + a 2 )u = e ax (D + a ) 2 u

cos 3 x 7

Engineering Mathematics

10.100

In general,

D r (e ax u ) = e ax (D + a ) r u

Let D r = f (D),

(D + a ) r = f (D + a )

f (D)(e ax u ) = e ax f (D + a )u 1 , Operating both the sides with f (D) 1 1 f (D)(e ax u ) = e ax f (D + a )u f (D) f (D) 1 e ax u = e ax f (D + a )u f (D) 1 Putting f (D + a )u = V, u = V f (D + a ) 1 1 V (e ax V) e ax f (D + a ) f (D) 1 1 Hence, V P.I. e ax V e ax f (D + a ) f (D) Example 1: Solve (D2 - 2D - 1) y = e x cos x . Solution: Auxiliary equation is m 2 2m 1 0 m=

2± 4+4 2±2 2 = = 1 ± 2 (real and distinct) 2 2

C.F. = c1e(1 P.I. =

2)x

+ c2 e(1

x

1 1 cos x e x cos x = e x 2D 1 (D 1) 2 2(D 1) 1

D2

1

ex

(D 2 2) Hence, the general solution is y

2)

c1e(1

2)x

c2 e(1

1

ex

cos x

2)x

12

2

cos x

e x cos x 3

e x cos x 3

Example 2: Solve (D3 + 3D2 - 4D - 12) y = 12 xe -2 x . Solution: Auxiliary equation is m3 3m 2

4m 12

0

m (m 3) 4(m 3)

0

2

(m 3)(m

2

4) 0 m 3, 2, 2 (real and distinct) C.F. = c1e 3 x + c2 e 2 x + c3 e 2 x

Differential Equations

P.I. =

1 (D 3)(D 2)(D 2)

= 12e

2x

= 12e −2 x = 12e −2 x

2x

12 xe

1 x (D 2 3)(D 2 2)(D 2 2) 1 1 x = 12e −2 x x (D + 1)D(D − 4) D(D 2 − 3D − 4) 1 x ⎛ 3D − D 2 ⎞ −4D ⎜1 + ⎟ 4 ⎝ ⎠ −1

= −3e

−2 x

= −3e −2 x = −3e −2 x

1 ⎛ 3D − D 2 ⎞ ⎜1 + ⎟ x D⎝ 4 ⎠ ⎞ 1 ⎛ 3D − D 2 + Higher powers of D ⎟ x ⎜1 − D⎝ 4 ⎠ 1⎡ 3 1⎛ 3⎞ ⎤ x − D( x) ⎥ = −3e −2 x ⎜ x − ⎟ ⎢ D⎣ D⎝ 4 4⎠ ⎦

⎛ x2 3 ⎞ 3⎞ ⎛ = −3e −2 x ∫ ⎜ x − ⎟ dx = −3e −2 x ⎜ − x ⎟ 4⎠ ⎝ ⎝ 2 4 ⎠ Hence, the general solution is y

c1e

3x

c2 e

2x

c3 e 2 x

3e

2x

x2 2

3 x 4

Case V: Q = xV, where V is a function of x. In this case Eq. (2) reduces to f (D)y = xV. Let u be a function of x. Then

D( xu ) = xDu + u D 2 ( xu ) = D( xDu + u ) = xD 2 u + Du + Du = xD 2 u + 2Du D3 ( xu ) = D( xD 2 u + 2Du ) = xD3u + D 2 u + 2D 2 u = xD3u + 3D 2 u

In general, D r ( xu ) = xD r u + rD r 1u = xD r u + Let

D r = f (D) d f (D) u dD = xf (D)u + f (D)u

f (D)( xu ) = x f (D)u +

d (D r ) u dD

10.101

Engineering Mathematics

10.102

Putting f (D)u = V, u =

1 V in above equation, f (D)

⎡ ⎤ ⎡ 1 ⎤ 1 f ( D) ⎢ x V ⎥ = xV + f ′ (D) ⎢ V⎥ ⎣ f ( D) ⎦ ⎣ f ( D) ⎦ ⎡ 1 ⎤ ⎡ ⎤ 1 xV = f (D) ⎢ x V ⎥ − f ′ ( D) ⎢ V⎥ f ( D ) f ( D ) ⎣ ⎦ ⎣ ⎦ 1 , Operating both the sides with f (D) ⎛ ⎞⎤ ⎛ 1 ⎞ ⎤ 1 1 ⎡ 1 1 ⎡ xV = V ⎥− V⎥ ⎢ f ( D) ⎜ x ⎢ f ′ ( D) ⎜ f ( D) f ( D) ⎣ ⎝ f (D) ⎟⎠ ⎦ f (D) ⎣ ⎝ f (D) ⎟⎠ ⎦ =x

Hence,

P.I. =

1 f ′ ( D) V− V f ( D) [ f (D)]2

1 xV f ( D)

x

1 V f (D)

f (D) 2

[ f (D)]

V

Example 1: Solve (D2 - 5D + 6) y = x cos 2 x . Solution: Auxiliary equation is m 2 − 5m + 6 = 0, (m − 2)(m − 3) = 0 m = 2, 3 (real and distinct ) C.F. = c1e 2 x + c2 e3 x 1 x cos 2 x D 2 − 5D + 6 1 2D − 5 =x 2 cos 2 x − 2 cos 2 x D − 5D + 6 (D − 5D + 6) 2 1 2D − 5 cos 2 x − cos 2 x =x −4 − 5D + 6 ( −4 − 5D + 6) 2 1 ( 2 + 5D ) 2D − 5 cos 2 x − cos 2 x =x ⋅ ( 2 − 5D ) ( 2 + 5D ) (4 − 20D + 25D 2 ) (2 + 5D) 2D − 5 cos 2 x − cos 2 x =x 2 (4 − 20D − 100) 4 − 25D 2D − 5 (2 + 5D) =x cos 2 x + cos 2 x 4 + 100 4(5D + 24) x 2D − 5 (5D − 24) = (2 cos 2 x − 10 sin 2 x) + ⋅ cos 2 x 104 4(5D + 24) (5D − 24)

P.I. =

Differential Equations

=

10.103

(10D 2 − 73D + 120) x (2 cos 2 x − 10 sin 2 x) + cos 2 x 104 4(25D 2 − 576)

x (10D 2 − 73D + 120) ⋅ cos 2 x (cos 2 x − 5 sin 2 x) + 52 4( −100 − 576) x 1 (cos 2 x − 5 sin 2 x) + = ( − 40 cos 2 x + 146 sin 2 x + 120 cos 2 x) 52 2704 Hence, the general solution is x 1 y = c1e 2 x + c2 e3 x + (cos 2 x − 5 sin 2 x) + (40 cos 2 x + 73 sin 2 x) 52 1352 =

Example 2: Solve (D2 + 3D + 2) y = xe x sin x . Solution: Auxiliary equation is m 2 + 3m + 2 = 0, (m + 1)(m + 2) = 0 m 1, 2 (real and distinct) C.F. = c1e x + c2 e

2x

1 1 xe x sin x = e x x sin x (D + 1)(D + 2) (D + 1 + 1)(D + 1 + 2) 1 1 = ex x sin x = e x 2 x sin x (D + 2)(D + 3) D + 5D + 6

P.I. =

⎡ ⎤ 1 2D + 5 = ex ⎢ x 2 sin x − 2 sin x ⎥ 2 5 6 5 6 D + D + D + D + ( ) ⎣ ⎦ ex x ex x

1 2D + 5 sin x sin x 1 5D 6 ( 1 5D 6) 2 1 (D 1) 2D 5 sin x sin x 5(D 1) (D 1) 25(D 2 + 2D + 1)

ex

2D 5 x (D 1) sin x sin x 2 5 (D 1) 25( 1 2D 1)

ex

x 1 5 (cos x sin x) 1 sin x 10 25 2D

ex

1 ⎛ 5 ⎡ x ⎞⎤ = e x ⎢ − (cos x − sin x) − ⎜ sin x + ∫ sin x dx⎟ ⎥ ⎠⎦ 25 ⎝ 2 ⎣ 10 ⎡ x 1 ⎛ 5 ⎞⎤ = e x ⎢ − (cos x − sin x) − ⎜ sin x − cos x⎟ ⎥ ⎠⎦ 25 ⎝ 2 ⎣ 10

2D 5 x (D 1) sin x sin x 5 ( 1 1) 25(2D)

Engineering Mathematics

10.104

Hence, the general solution is y

c1e

x

c2 e

2x

ex x 1 5 (cos x sin x) sin x cos x 5 2 5 2

Example 3: Solve (4D2 + 8D + 3) y = xe

-

x 2

cos x.

Solution: Auxiliary equation is 4m 2 + 8m + 3 = 0, (2m + 1)(2m + 3) = 0 1 3 , (real and distinct) m 2 2 x 2

C.F. = c1e P.I. =

3x 2

x x − − 1 1 2 2 xe cos x e x coss x = 2 2 4D + 8D + 3 1⎞ 1⎞ ⎛ ⎛ 4⎜ D − ⎟ + 8⎜ D − ⎟ + 3 2⎠ 2⎠ ⎝ ⎝

=e



e = 4 e = 4 e = 4 =

+ c2 e

e 4

e = 4 e = 8 e = 8 e = 8

x 2











x 2

x 2

x 2

x 2

x 2



x 2



x 2



x 2

x − 1 1 x cos x = e 2 x cos x ( 4 D 2 + 4 D) ⎛ 2 1 ⎞ 4 ⎜ D + − D ⎟ + 8D − 4 + 3 4 ⎝ ⎠

1⎛ 1 ⎞ e ⋅ ⎜ x cos x ⎟ = D ⎝ D +1 ⎠ 4



x 2



⎤ 1⎡ 1 1 cos x − cos x ⎥ ⎢x ⋅ D ⎣ D +1 (D + 1) 2 ⎦



1 ⎛ D −1 1 ⎞ cos x − 2 cos x⎟ ⎜⎝ x ⋅ 2 ⎠ D D −1 D + 2D + 1



⎤ 1 1 ⎡ (D − 1) cos x − cos x ⎥ x −1 + 2D + 1 D ⎢⎣ ( −1 − 1) ⎦



1⎡ x 1 ⎤ − (D cos x − cos x) − ∫ cos x dx ⎥ D ⎢⎣ 2 2 ⎦



1 1⎡ x ⎤ − (− sin x − cos x) − sin x ⎥ 2 D ⎢⎣ 2 ⎦

⎡ x(sin x + cos x)dx + sin x dx ⎤ ∫ ⎣∫ ⎦

[ x(− cos x + sin x) − (− sin x − cos x) − cos x ] [ x(sin x − cos x) + sin x]

Differential Equations

10.105

Hence, the general solution is x

y

x 2

c1e

3x 2

c2 e

e 2 [ x(sin x cos x) sin x] 8

Exercise 10.9 Solve the following differential equations: 6. (3D 2

x 2 1. (D + D + 2) y = e 2 .

Ans. : y = e

x 2

2. (D 2 4) y

3. (D

1 (3cos x 4sin x) 25

+

4 2x e 11

D 1) y

e

3x

6e

x

2e x

x 2

c1 cos e

2x

4. (D 2

1 2x xe 4 3e

2x

5.

e3 x 3x 3x + c2 sin + 2 2 13

5

x

e 10

x

e 2

e2 x

sin 2 x.

3 2 2 8. (D + 2D + D) y = sin x.

x⎤ ⎡ −x ⎢ Ans. : y = c1 + (c2 + c3 x)e + 2 ⎥ ⎢ ⎥ 1 ⎢ + (3 sin 2 x + 4 cos 2 x) ⎥ ⎢⎣ ⎥⎦ 100 9. (D 2

D 6) y

e 2 x.

Ans. : y = c1e 2 x + c2 e

4D 5) y

Ans. : y = e

4D) y

Ans. : y = c1 + e x ( c2 cos 3 x

2x

2 x e 3

2D 2

1 + c3 sin 3x ) + (e 2 x + sin 2 x) 8

Ans. : y=e

7. (D3

(1 e x ) 2 .

Ans. : y = c1e 2 x + c2 e

2

Ans. : y = c1e 2 x + c2 e 3

7x 2

7x 2

1 4

sin x cos x. x

c1 cos

+ c2 sin

7D 2) y

2x

2 cosh x 2 x. (c1 cos x + c2 sin x)

2x (log 2) 2 + 4(log 2) + 5

3x

+

xe 2 x 5

x

10. (9D 2 + 6D + 1) y = e 3. Ans. : y = (c1 + c2 x)e

x 3

+

x2 e 18

x 3

11. (D 2 + 4) y = e x + sin 2 x. 5. (D3 + D 2 + D + 1) y = sin 2 x. Ans. : y = c1e x + c2 cos x + c3 sin x 1 (2 cos 2 x sin 2 x) 15

Ans. : y = c1 cos 2 x + c2 sin 2 x +

ex 5

x cos 2 x 4

Engineering Mathematics

10.106

2 12. (D

x 2.

4) y

Ans. : y 13. (D 2

c1e

2x

c2 e

x2

D) y Ans. : y

14. (D

2

1) y

x

e

x

c1 c2 e 2x

2x

e 5 2

x3 3

20. (D 2

1 cosh 2 x x 3 6 x 5 x ex sin 2

(c1 c2 x)e

3D 2) y

Ans. : y

2 17. (D 2D 10) y 24e x sin 3 x.

x

2

x.

(c3

x

c4 x)e 2

x x e 8

x

x 2e x cos . 2

8 x c1e c2 e e 5 x x 2sin cos 2 2 x

21. (4D 2

9D 2) y

16e x cos 3 x

9D 10) y

24e x sin 2 x.

e x (c2 cos 2 x

c3 sin 2 x)

6 xe x (2sin 2 x cos 2 x) 5

2x

sin x sin 3 x.

xe

2x

.

Ans. : c1e

22. (D 2

2x

c2 e

4) y

Ans. : y

x 4

1 (7 x 2 8 x)e 98

2x

x sin x. c1 cos 2 x c2 sin 2 x 1 (3 x sin x 2 cos x) 9

2x

Ans. : c1e 2 x

4D 8) y 12e

⎡ Ans. : y = e −2 x (c1 cos 2 x + c2 sin 2 x) ⎤ ⎢ ⎥ 1 ⎢ + e −2 x (3 x sin 2 x + cos 4 x) ⎥ ⎢⎣ 2 ⎦⎥

y

⎡ Ans. : y = e x (c1 cos 3 x + c2 sin 3 x) ⎤ ⎢ ⎥ xe x ⎢ + (8 sin 3x − 12 cos 3x) ⎥ ⎢⎣ ⎥⎦ 3

y

e 2 (c2 cos x c3 sin x) 4 xe 2 (2 cos x 3sin x) 13

4x

c1 cos x c2 sin x

2

4D 2

c1e 2 x

x

cosh 2 x x .

1 1 cos x 2 8

18. (D3

12D

2 13D 10) y 16e cos x.

x

Ans. : y

3

15. (D 1) (D 1) y

2 16. (D

19. (4D

1 2

2

2 x 4.

Ans. : y

Ans. : y

1 2 x 4

2x

3

23. (D 2

9) y

xe 2 x cos x.

⎤ ⎡ Ans. : y = c1 cos 3 x + c2 sin 3 x ⎥ ⎢ 2x ⎢ + e [(30 x − 11) cos x + (10 x − 2) sin x ]⎥ ⎥⎦ ⎢⎣ 400 24. (D 2

4D 4) y

8 x 2 e 2 x sin 2 x.

⎡ Ans. : y = (c1 + c2 x)e 2 x − e 2 x [4 x cos 2 x ⎤ ⎥ ⎢ + (2 x 2 − 3) sin 2 x]⎦ ⎣ 25. (D 2 1) y

x sin x (1 x 2 )e x .

1 ⎤ ⎡ x −x ⎢ Ans. : y = c1e + c2 e − 2 ( x sin x ⎥ ⎢ ⎥ 1 ⎢ + cos x) + xe x (2 x 2 − 3x + 9) ⎥ ⎢⎣ ⎦⎥ 12

Differential Equations

10.107

10.5.3 General Method of Obtaining Particular Integral (P.I.) In a linear differential equation f (D) y

Q( x)

if Q(x) is not in any of the standard forms discussed in the previous section, then particular integral is obtained using the general method described below.

P.I. = =

1 Q( x ) f ( D) 1 Q( x ) (D − m1 )(D − m2 )… (D − mn )

An ⎞ A2 ⎛ A1 [ Using Partial Fraction ] Q( x ) =⎜ + +… + D − mn ⎟⎠ ⎝ D − m1 D − m2 1 1 1 = A1 ⋅ Q( x ) + … + A n ⋅ Q( x ) Q( x ) + A 2 ⋅ D − mn D − m1 D − m2 = A1e m1 x ∫ Q( x) ⋅ e − m1 x dx + A 2 e m2 x ∫ Q( x) ⋅ e − m2 x dx + … + A n e mn x ∫ Q( x)e − mn x dx This method can be applied for any form of Q(x). But sometimes integration of the terms become complicated and lengthy therefore direct (short-cut) methods are preferred to find the P.I. and general method is used only if direct method can not be applied. x

2 e Example 1: Solve (D + 3D + 2) y = e .

Solution: Auxiliary equation is m 2 + 3m + 2 = 0, (m + 1)(m + 2) = 0, m = –1, –2 (real and distinict) C.F. = c1 e –x + c2e–2x P.I.

D

x 1 ee 3D 2

2

1 (D

(e 2)

x

x 1 ee (D 2)(D 1) x 1 ( x ex ) e e e x dx e e D 2

)

x

e

2x

e x e e e 2 x dx

e

2x

e e e x dx

x

Hence, the general solution is y c1e

e

x

2 x ex

e

c2 e

2x

e

2 x ex

e

∵ e f ( x ) f ( x ) dx

e f ( x)

Engineering Mathematics

10.108

Example 2: Solve (D2 - 1) y = (1 + e - x ) -2. Solution: Auxiliary equation is m 2 1 0, m C.F. c1e x

1 (real and distinct) c2 e

1

P.I.

D

2

1

x

(1 e x )

1 1 1 (D 1) D 1 (1 e x ) 2

2

1 1 D 1 (1 e x ) 2 Let 1 e x

t , e x dx

e

x

1 e x dx (1 e x ) 2

e

1 1 1 1 2 D 1 D 1 (1 e x ) 2 x

e2 x e x dx (e x 1) 2

dt

2 1 1 ⎛ 2 1⎞ − x (t − 1) e ⋅ = dt = e − x ∫ ⎜ 1 − + 2 ⎟ d t −x 2 2 ∫ (D + 1) (1 + e ) t ⎝ t t ⎠ 1⎞ ⎛ = e − x ⎜ t − 2 log t − ⎟ t⎠ ⎝

1 ⎤ ⎡ = e − x ⎢1 + e x − 2 log(1 + e x ) − 1 + e x ⎥⎦ ⎣ e− x = e − x + 1 − 2e − x log(1 + e x ) − 1 + ex 1 1 (D 1) (1 e x ) 2 Let 1 e

x

t , e x dx

ex

1 e x dx (1 e x ) 2

ex

1 ( dt ) t2

dt

1 1 (D 1) (1 e x ) 2

ex 1 e P.I.

ex

1 t

x

1 ex 2 1 e

x

e

x

1 2e x log(1 e x )

e

x

1 ex

Hence, the general solution is 1 ⎡ ex e− x ⎤ y = c1e x + c2 e − x + ⎢ − e − x − 1 + 2e − x log(1 + e x ) + ⎥ −x 2 ⎣1 + e 1 + ex ⎦ Example 3: Solve (D2 + a 2 ) y = sec ax . Solution: Auxiliary equation is m2

a2

0,

m

ia (complex)

C.F. c1 cos ax c2 sin ax

Differential Equations

10.109

1 sec ax D + a2 1 = sec ax (D − ia )(D + ia )

P.I. =

=

2

1 ⎛ 1 1 ⎞ − ⎜ ⎟ seec ax 2ia ⎝ D − ia D + ia ⎠

1 sec ax = eiax ∫ sec ax ⋅ e − iax dx D − ia 2 = eiax ∫ iax e − iax dx e + e − iax 2 = eiax ∫ e −2iax dx 1 + e −2iax Let 1 e

2 iax

t , 2iae

2 iax

dx

dt

1 2 ⎛ dt ⎞ eiax sec ax = eiax ∫ ⎜ − log t ⎟=− D − ia t ⎝ 2ia ⎠ ia eiax log(1 + e −2iax ) ia eiax =− log(1 + cos 2ax − i sin 2ax) ia eiax =− log(2 cos 2 ax − 2i sin ax cos ax) ia eiax =− log (2 cos ax )(cos ax − i sin ax) ia eiax =− [log (2 cos ax) + log e − iax ] ia 1 eiax sec ax = − [log (2 cos ax) − iax] D − ia ia =−

Replacing i by –i in Eq. (1), 1 e − iax sec ax = [log (2 cos ax) + iax] D + ia ia P.I. =

⎤ 1 ⎡ eiax e − iax {log (2 cos ax) − iax} − {log (2 cos ax) + iax}⎥ ⎢− 2ia ⎣ ia ia ⎦

=

1 ⎡ log (2 cos ax) iax ⎤ (e + e − iax ) + x(eiax − e − iax ) ⎥ − 2ia ⎢⎣ ia ⎦

=

log (2 cos ax) x (2 cos ax) + (2i sin ax) 2ia 2a 2

=

1 x sin ax [log(2 cos ax)] cos ax + a a2

... (1)

Engineering Mathematics

10.110

Hence, the general solution is y = c1 cos ax + c2 sin ax +

1 x sin ax [log (2 cos ax)]cos ax + a a2

Example 4: Solve (D2 + 5D + 6) y = e -2 x sec 2 x (1 + 2 tan x ). Solution: Auxiliary equation is m2 m

5m 6 0, (m 2)(m 3) 2, 3 (real and distinct) C.F. c1e P.I.

0 2x

c2 e

3x

1 e 5D 6

D2

2x

sec 2 x(1 2 tan x)

1 e (D 2)(D 3) 1 D 2 1 e D 2

2x

sec 2 x(1 2 tan x)

2x

1 e D 3 2x

sec 2 x(1 2 tan x) 2x

sec 2 x(1 2 tan x)

e

2x

e

sec 2 x(1 2 tan x) e 2 x dx

e

2x

sec 2 x(1 2 tan x)dx

e

2x

2 1 e D 3

2x

sec 2 x(1 2 tan x)

e

3x

e

2x

e

3x

e x sec 2 x(1 2 tan x)dx

e

3x

e

3x

(1 2 tan x) 2 2

sec 2 x(1 2 tan x) e3 x dx

( e sec x dx (e sec x e x

2

x

2

e x sec 2 x 2 tan x dx x

2sec x sec x tan x dx

e x sec 2 x 2 tan x dx e

3x x

2

e sec x

e

2x

)

)

2

sec x

e −2 x (1 + 2 tanx ) 2 − e −2 x sec 2 x 4 e −2 x e −2 x = (1 + 4 tan 2 x + 4 tan x ) − e −2 x (1 1 + tan 2 x ) = ( 4 tan x − 3) 4 4

P.I.=

Hence, the general solution is y

c1e

2x

c2 e

3x

e

2x

4

(4 tan x 3)

Differential Equations

10.111

Exercise 10.10 Solve the following differential equations: 1. (D 2

3D 2) y

Ans. : y

c1e

x

sin e x. c2 e

2x

5. (D 2 e

2x

D) y

sin e x

1 . 1 ex Ans. : y

2. (D

2

1) y

Ans. : y

cosec x.

e x [e x log(e

c1 cos x c2 sin x x cos x

6. (D 2

2D 2) y

sin x log(sin x) Ans. : y 3. (D

2

4) y

tan 2 x.

x

1) log(e x 1)] e x tan x.

e x (c1 cos x c2 sin x)

e x cos x log(sec x tan x)

⎡ Ans. : y = c1 cos 2 x + c2 sin 2 x ⎤ ⎢ ⎥ 1 ⎢ − cos 2 x log(sec 2 x + tan 2 x) ⎥ ⎣ 4 ⎦ 4. (D 2 1) y

x

c1 c2 e

x cot x.

7. (D 2

4D 4) y

8 x 2 e 2 x sin 2 x.

⎡ Ans. : y = (c1 + c2 x)e 2 x − e 2 x (2 x 2 sin 2 x ⎤ ⎥ ⎢ + 4 x cos 2 x − 3 sin 2 x) ⎦ ⎣ 2 8. (D

⎡ Ans. : y = c1 cos x + c2 sin x − x cos 2 x ⎤ ⎢ ⎥ 2 ⎣⎢ + x sin x − sin x log(cosec x − cot x) ⎥⎦

2D 1) y

e x log x.

⎡ Ans. : y = (c1 + c2 x)e − x ⎤ ⎢ ⎥ x2 3⎞ ⎛ ⎢ + e − x ⎜ log x − ⎟ ⎥ ⎢⎣ ⎝ 2 2 ⎠ ⎥⎦

10.6 HIGHER ORDER LINEAR DIFFERENTIAL EQUATIONS WITH VARIABLE COEFFICIENTS In this section we will discuss two types of differential equations with variable coefficients. These differential equations have variable coefficients and can be solved by reducing to linear differential equation with constant coefficients form.

10.6.1 Cauchy’s Linear Equation An equation of the form a0 x n

dn y dx n

a1 x n

1

dn 1 y dx n 1

a2 x n

2

dn 2 y dy … an 1 x n 2 dx dx

an y

Q( x)

where a0, a1, a2, …, an 1, an are constants, is called Cauchy’s linear equation. To solve Eq. (1), Let x = ez, 1 e z

dz dz , dx dx

1 ez

1 x

... (1)

Engineering Mathematics

10.112

dy dx

Now, x

dy dz dz dx

1 dy x dz

dy dy = , xDy = Dy, dx dz

where D ≡

d d and D = dz dx

d 2 y d ⎛ dy ⎞ d ⎛ 1 dy ⎞ 1 dy 1 d ⎛ dy ⎞ = ⎜ ⎟= ⎜ + ⎟=− 2 ⎜ ⎟ 2 dx ⎝ dx ⎠ dx ⎝ x dz ⎠ dx x dz x dx ⎝ dz ⎠ 1 dy 1 d ⎛ dy ⎞ dz 1 dy 1 d 2 y 1 =− 2 + ⋅ ⎜ ⎟⋅ = − 2 + ⋅ x dz x dz ⎝ dz ⎠ dx x dz x dz 2 x x2 Similarly,

d 2 y ⎛ d 2 y dy ⎞ =⎜ − ⎟ or x 2 D 2 y = D (D − 1) y dx 2 ⎝ dz 2 dz ⎠

x 3 D3 y = D (D − 1)(D − 2) y ........................................... ........................................... x n D n y = D (D − 1)(D − 2)...[ D − (n − 1) ] y

Substituting these derivatives in Eq. (1),

[ a D(D − 1)… (D − n + 1) + a D(D − 1)… (D − n + 2) + … + a D + a ] y = Q(e ) 0

1 z

n −1

n

which is a linear differential equation with constant coefficients and can be solved by usual methods described in previous section. Example 1: Solve (4 x 2 D2 + 16 xD + 9) y = 0. Solution: Putting x = ez,

[ 4D (D − 1) + 16D + 9] y = 0,

where D

d dz

(4D 2 + 12D + 9) y = 0

Auxiliary equation is 4m 2 12m 9

0

(2m 3) 2

0,

m

3 , 2

C.F. (c1 c2 z )e

3 z 2

Since Q(ez) = 0, P.I. = 0 Hence, the general solution is y

(c1 c2 log x) x

3 2

3 (real and repeated twice) 2 (c1 c2 log x) x

3 2

Differential Equations

10.113

2 2 Example 2: Solve (4 x D + 1) y = 19cos(log x ) + 22sin(log x ).

Solution: Putting x = ez, [4D (D

1) + 1]y = 19 cos z + 22 sin z,

where D

(4D 2 − 4D + 1) y = 19 cos z + 22 sin z

d dz

which is a linear equation with constant coefficients. Auxiliary equation is 4m 2 4m 1 0 (2m 1) 2 1

(c1 c2 z )e 2

C.F.

z

0,

m

1 (real and repeated twice) 2

1

(c1 c2 log x) x 2

1 (19 cos z + 22 sin z ) 4D − 4D + 1 1 1 (4D − 3) = ⋅ (19 cos z + 22 sin z ) (19 cos z + 22 sin z ) = − 4 − 4D + 1 −(4D + 3) (4D − 3)

P.I. =

2

4D − 3 4D − 3 (19 cos z + 22 sin z ) (19 cos z + 22 sin z ) = 2 −(−16 − 9) −(16D − 9) 1 = [ 4(−19 sin z + 22 cos z ) − 3(19 cos z + 22 sin z ) ] 25 1 1 (31 cos z − 142 sin z ) = [31 cos(log x) − 142 sin(log x) ] = 25 25 Hence, the general solution is 1 1 y (c1 c2 log x) x 2 [ 31cos(log x) 142sin(log x)] 25 1 Example 3: Solve ( x 3 D 3 + x 2 D2 - 2) y = x + 3 . x Solution: Putting x = ez, d where D [ D (D − 1)(D − 2) + D (D − 1) − 2] y = e z + e−3 z dz (D 3 − 2D 2 + D − 2) y = e z + e −3 z =

which is a linear equation with constant coefficients. Auxiliary equation is m3

2m 2

m 2

(m 2)(m C.F. c1e 2 z c1 x

2

2

1)

0 0, m

2, m

c2 cos z c3 sin z c2 cos(log x) c3 sin(log x)

i (imaginary)

Engineering Mathematics

10.114

1 (e z + e −3 z ) 2 D − 2D + D − 2 1 1 ez + e −3 z = 1− 2 +1− 2 (−3)3 − 2(−3) 2 − 3 − 2 1 1 1 1 1 1 = − e z − e −3 z = − x − ( x) −3 = − x − 2 50 2 50 2 50 x 3 Hence, the general solution is 1 1 y c1 x 2 c2 cos(log x) c3 sin(log x) x 2 50 x 3 P.I. =

3

xD2 + D -

Example 4: Solve

1 y = - ax 2. x

Solution: Multiplying the given equation by x, ( x 2 D2

ax 3

xD 1) y

which is Cauchy’s linear equation. Putting x = ez,

[ D (D − 1) + D − 1] y = −ae3 z

where D ≡

d dz

(D 2 − 1) y = −ae3 z which is a linear equation with constant coefficients. Auxiliary equation is m 2 1 0,

m

C.F. c1e z

P.I. =

c2 e

z

1 (real and distinct) c1 x c2 x

1

c1 x

c2 x

a 1 1 (−ae3 z ) = −a e3 Z = − x 3 8 8 D −1 2

Hence, the general solution is

1 Example 5: Solve D + x Solution: D

1 x

2

y

y

c1 x

y=

1 . x4

d dx

1 x

d dx

1 x

2

d2 y dx 2

c2 x

2

1 dy x dx

a 3 x 8

y

d dx

1 x

dy dx

y x

d2 y dx 2

y x2

1 dy x dx

d dx

1 y x d y dx x

y x2

d2 y dx 2

1 dy x dx 2 dy x dx

y x2

Differential Equations

10.115

Substituting in the given equation, d2 y dx 2

2 dy x dx

D2

2 D y x

( x 2 D2

2 xD) y

1 x4 1 x4

Multiplying the equation by x2, 1 x2

which is Cauchy’s equation. Putting x = ez, 1 e2 z (D 2 + D ) y = e −2 z

[ D (D − 1) + 2 D ] y =

which is a linear equation with constant coefficients. Auxiliary equation is m2

m

0,

c1 c2 e

C.F.

P.I. =

m z

0, 1 (real and distinct)

c1 c2 x

1

c2 x

c1

1 1 −2 z 1 −2 z 1 −2 1 e −2 z = e = e = ( x) = 2 4−2 2 2 2x D +D 2

Hence, the general solution is y

c1

c2 x

1 2x2

Example 6: Solve ( x 2 D2 + xD - 1) y =

x3 . 1 + x2

Solution: Putting x = ez,

[ D (D − 1) + D − 1] y = (D 2 − 1) y =

e3 z , 1 + e2 z

where D ≡

d dz

e3 z 1 + e2 z

which is a linear equation with constant coefficients. Auxiliary equation is m 2 1 0,

m

1 (real and distinct)

C.F. = c1e z + c2 e − z = c1 x +

c2 x

Engineering Mathematics

10.116

⎛ e3 z ⎞ ⎞ 1 ⎜ ⎟= 2z ⎟ ⎠ (D + 1)(D − 1) ⎝ 1 + e ⎠ 1⎛ 1 1 ⎞ ⎛ e3 z ⎞ = ⎜ − ⎟ ⎟⎜ 2 ⎝ D − 1 D + 1 ⎠ ⎝ 1 + e2 z ⎠

P.I. =

Putting 1 e 2 z

t , 2e 2 z dz

⎛ e3 z 1 ⎜ (D − 1) ⎝ 1 + e 2 z 2

=

1 ⎡ 1 ⎛ e3 z ⎞ 1 ⎛ e3 z ⎞ ⎤ − ⎢ ⎜ ⎟⎥ ⎜ ⎟ 2 ⎣ D − 1 ⎝ 1 + e2 z ⎠ D + 1 ⎝ 1 + e2 z ⎠ ⎦

=

⎤ 1 ⎡ z e3 z − z e3 z −z e e z − e ⋅ e z dz ⎥ d ⎢ ∫ 2z 2z ∫ 2 ⎣ 1+ e 1+ e ⎦

=

⎤ 1 ⎡ z e2 z e4 z −z dz ⎥ z e e − d ⎢ ∫ 2z 2z ∫ 2 ⎣ 1+ e 1+ e ⎦

dt P.I.

1 z 1 dt e 2 t 2

e

z

t 1 dt t 2

1 ez e z log t (t log t ) 2 2 2 1 z e log (1 e 2 z ) e z {1 e 2 z log (1 e 2 z )} 4 1 x log (1 x 2 ) ( x) 1{1 x 2 log (1 x 2 )} 4 x 1 x 1 log (1 x 2 ) log (1 x 2 ) 4 4x 4 4x Hence, the general solution is c y c1 x 2 x c2 c1 x x where

c1

c1

1 , 4

x log (1 x 2 ) 4 x log (1 x 2 ) 4 c2

c2

1 x 1 log (1 x 2 ) 4x 4 4x 1 log (1 x 2 ) 4x 1 4

Example 7: Solve ( x 2 D2 - 4 xD + 6) y = - x 4 sin x . Solution: Putting x = ez,

[ D (D − 1) − 4D + 6] y = −e4 z sin e z (D 2 − 5D + 6) y = −e 4 z sin e z which is a linear equation with constant coefficients.

where D ≡

d dz

Differential Equations

Auxiliary equation is m 2 5m 6 0 (m 2)(m 3) 0, C.F. c1e 2 z P.I. =

m c2 e3 z

10.117

2, 3 (real and distinct)

c1 x 2

c2 x 3

1 1 (−e 4 z sin e z ) = (−e 4 z sin e z ) (D − 2)(D − 3) D − 5D + 6 2

1 ⎞ ⎛ 1 4z z =⎜ − ⎟ (−e sin e ) 3 2 − − D D ⎝ ⎠ 1 1 4z z = (e sin e ) − (e 4 z sin e z ) D−2 D −3 = e 2 z ∫ e 4 z sin e z ⋅ e −2 z dz − e3 z ∫ e 4 z sin e z ⋅ e −3 z dz = e 2 z ∫ sin e z ⋅ e 2 z dz − e3 z ∫ sin e z ⋅ e z dz z Putting e

t , e z dz

dt

P.I. = e 2 z ∫ sin t ⋅ tdt − e3 z ∫ sin t dt = e 2 z (−t cos t + sin t ) − e3 z (− cos t ) = e 2 z (−e z cos e z + sin e z ) + e3 z cos e z = e 2 z sin e z = x 2 sin x Hence, the general solution is y

c1 x 2

c2 x 3

x 2 sin x

Example 8: Solve ( x 2 D2 - xD + 2) y = 6, y(1) = 1, y (1) = 2. Solution: Putting x = ez,

[ D (D − 1) − D + 2] y = 6, (D 2 − 2D + 2) y = 6 which is a linear equation with constant coefficients. Auxiliary equation is m 2 2m 2 0, m 1 i (imaginary) C.F. = e z (c1 cos z + c2 sin z ) = x[c1 cos(log x) + c2 sin(log x)] P.I. =

1 1 6e0 z = ⋅ 6 = 3 2 D 2 − 2D + 2

where D

d dz

Engineering Mathematics

10.118

Hence, the general solution is y x [ c1 cos(log x) c2 sin(log x) ] 3

... (1)

1 1⎤ ⎡ y ′ = [ c1 cos(log x ) + c2 sin(log x) ] + x ⎢ −c1 sin(log x) ⋅ + c2 cos(log x) ⋅ ⎥ x x⎦ ⎣ y (c1 c2 ) cos(log x) (c2 c1 ) sin(log x) ... (2) Given y (1) 1, y (1) 2 Putting x = 1, y = 1 and y 2 in Eqs. (1) and (2), 1 c1 cos(0) c2 sin(0) 3 c1 3 c1 and

2 2

(c1 c2 ) cos(0) (c2

2

c1 c2

c1 ) sin 0

c2 4 Hence, the general solution is y 2 x cos(log x) 4 x sin(log x) 3 Example 9: Solve (4 x 2 D2 + 1) y = log x , x > 0, y(1) = 0, y( e ) = 5. Solution: Putting x = ez, [4D (D − 1) + 1] y = z ,

where D

d dz

(4D 2 − 4D + 1) y = z which is a linear equation with constant coefficients. Auxiliary equation is 4m 2

(2m − 1) 2 = 0,

m= 1

1 1 , 2 2

4m 1 0

(real and repeated )

z

1

C.F. = (c1 + c2 z )e 2 = (c1 + c2 log x) x 2 P.I. =

1 1 1 z = (1 − 2D ) −2 z z= z= 2 4D − 4D + 1 (2D − 1) (1 − 2D ) 2 2

= (1 + 4D + 6D 2 + …) z = z + 4Dz + 6D 2 z + … = z + 4 + 0 = log x + 4 Hence, the general solution is y (c1 c2 log x) x log x 4 Given y (1) = 0, y (e) = 5 Putting x = 1, y = 0 and then x = e, y = 5 in Eq. (1), 0 c1 c2 log1 log1 4 c1 4 c1

4

... (1)

Differential Equations

and

5

(c1 c2 log e) e log e 4

c2

10.119

e ( 4 c2 ) 1 4

4

Hence, the general solution is y ( 4 4 log x) x log x 4 log x . x2

Example 10: Solve ( x 2 D2 + 5 xD + 3) y = Solution: Putting x = ez,

[ D (D − 1) + 5D + 3] y =

z , e2 z

where D

d dz

(D 2 + 4D + 3) y = e −2 z z

which is a linear equation with constant coefficients. Auxiliary equation is m 2 4m 3 0 (m 1)(m 3) 0, m 1, 3 (real and distinct) c c2 1 C.F. c1e z c2 e 3 z c1 ( x) 1 c2 ( x) 3 x x3 1 1 P.I. = 2 e −2 z z = e −2 z z 2 D + 4D + 3 (D − 2) + 4(D − 2) + 3 1 1 − D 2 ) −1 z = −e −2 z (1 + D 2 + D 4 + …) z = e −2 z 2 z = −e −2 z (1 D −1 = −e −2 z ( z + D 2 z + D 4 z + …) = −e −2 z ( z + 0) = −( x) −2 (log x) = − Hence, the general solution is c1 x

y

c2 x3

log x x2

Example 11: Solve ( x 2 D2 + xD + 1) y = log x sin(log x ). Solution: Putting x = ez, [D (D − 1) + D + 1] y = z sin z ,

where D

(D 2 + 1) y = z sin z which is a linear equation with constant coefficients. Auxiliary equation is m 2 1 0,

m

i

(imaginary)

C.F. = c1 cos z + c2 sin z = c1 cos(log x ) + c2 sin(log x) P.I. =

1 1 z sin z = 2 z (Imaginary part of eiz ) D2 +1 D +1

d dz

log x x2

Engineering Mathematics

10.120

⎡ ⎤ 1 ⎡ 1 ⎤ = I.P. ⎢ 2 zeiz ⎥ = I.P. ⎢eiz ⋅ z⎥ 2 (D + i ) + 1 ⎦ ⎣ D +1 ⎦ ⎣ 1 ⎡ iz ⎤ 1 ⋅ z⎥ e ⋅ ⎡ ⎤ = I.P. ⎢eiz ⋅ 2 z ⎥ = I.P. ⎢ D ⎞ ⎛ ⎢ 2i D ⎜1 + ⎟ ⎥ D + 2iD ⎦ ⎣ ⎢⎣ ⎝ 2i ⎠ ⎥⎦ ⎡ eiz 1 ⎛ D ⎞ −1 ⎤ 1+ z ⋅ = I.P. ⎢⎣ 2i D ⎜⎝ 2i ⎟⎠ ⎥⎦

⎡ eiz 1 ⎛ D D 2 ⎡ eiz 1 ⎛ ⎞ ⎤ 1 ⎞⎤ = I.P. ⎢ ⋅ ⎜1 − + 2 − … ⎟ z ⎥ = I.P. ⎢ ⋅ ⎜ z − + 0 ⎟ ⎥ 2 2 2 2 D D i i i i 4 i ⎝ ⎠⎦ ⎣ ⎝ ⎠ ⎦ ⎣ iz iz 2 ⎡ −ie ⎛ z ⎡e ⎛ 1⎞ ⎤ z ⎞⎤ = I.P. ⎢ ∫ ⎜ z − ⎟ dz ⎥ = I.P. ⎢ ⎜ − ⎟⎥ 2i ⎠ ⎦ ⎣ 2i ⎝ ⎣ 2 ⎝ 2 2i ⎠ ⎦ 2 ⎡ −i (cos z + i sin z )(i z − z ) ⎤ = I.P. ⎢ ⎥ 4i ⎣ ⎦ − z 2 cos z + z sin z 4 (log x) 2 log x =− cos(log x) + sin(log x) 4 4 Hence, the general solution is =

y

c1 cos(log x) c2 sin(log x)

(log x) 2 log x cos(log x) sin(log x) 4 4

Exercise 10.11 Solve the following differential equations: 1. ( x 2 D 2

xD 1) y

4. ( x 3 D3

0.

Ans. : y 2

2. (9 x D

2

3xD 10) y

c1 x

c2 x

0.

1

y

x 3 c1 cos(log x)

c2 sin(log x) ]

Ans. : y

c1 x2

x[c2 cos(4 log x) c3 sin(4 log x)]

c1 x

(c2

c3 log x) x 2

3 xD 4) y

Ans. : y 6. ( x 3D 3

3. ( x 3 D3 − 2 xD + 4) y = 0.

0.

Ans. :

2 2 5. ( x D

Ans. :

3 x 2 D 2 14 xD 34) y

Ans. : y

x 3.

(c1 c2 log x) x 2

6 x 2 D 2 12) y c1 x 2

c2 x2

x3

12 . x2 c3 3 log x x3 x 2

Differential Equations

7. (4 x 3 D3 12 x 2 D 2 50sin(log x).

xD 1) y

10.121

Ans. : y

1

Ans. : y

8. ( x 2 D 2

c3 x sin(log x) 7 cos(log x)

(c1 c2 log x) x 2

3 xD 3) y

d2 y dx 2

3x

dy dx

y

log x 2

(

)

(

3 xD 5) y

x 2 sin(log x).

14. ( x 2 D 2

3 xD 1) y Ans. : y

2 2 15. ( x D

2

3

11. ( x D

3xD

2

D) y

Ans. : y

1 [2 cos(3log x) x

x log x.

2) y 10 x

5 , 2

9 x 2 , y (1)

x2 2

3sin(3log x)]

x3 (log x 1) 27 2 x2 D2

1 . (1 x) 2 1 (c1 c2 log x) x 1 x log x x 1

8.

2

Ans. : c1 c2 log x c3 (log x) 2

12. ( x 3 D3

1 4

3 xD 10) y

y (1)

1 [(log x) 2 2

c1 x c2 x 2

c2 sin(log x)] x2 log x cos(log x) 2

2 log x x (log x) 2 log x 2.

log x]

)

x 2 [c1 cos(log x)

Ans. : y

Ans. : y

sin(log x) 1 . x

⎡ Ans. : y = x 2 ⎡c cosh 3 log x ⎤ ⎣ 1 ⎢ ⎥ ⎢ ⎥ + c2 sinh 3 log x ⎤ ⎥ ⎢ ⎦ ⎢ ⎥ 1 1 ⎢ [5 sin(log g x) ⎥ + + ⎢ ⎥ 6 x 61x ⎢ ⎥ + 6 cos(log x)] ⎣ ⎦ 10. ( x 2 D 2

5x 2 2 13. ( x D 2 xD 2) y

2 3log x.

c1 x c2 x 3

Ans. : y

9. x 2

c1 x[c2 cos(log x) x c3 sin(log x)]

16. (2 x 2 D 2 y (4)

1 . x

3 xD 1) y 41 . 16 Ans. : y

x, y (1) 1,

1 4

x

1 x

x 2

10.6.2 Legendre’s Linear Equation An equation of the form dn y d n −1 y dn−2 y + a1 (a + bx) n −1 n −1 + a2 (a + bx) n − 2 n − 2 + … n dx dx dx dy …+ an −1 (a + bx) + an y = Q( x) dx where a0, a1, a2, …, an–1, an are constants, is called Legendre’s linear equation. a0 (a + bx) n

... (1)

Engineering Mathematics

10.122

Let (a bx)

ez

b

ez

Now,

dy dx

dz , dx

dz dx

dy dz dz dx

dy b dz (a bx)

(a bx)

dy dx

b

b ez

b a bx

dy dz

(a + bx)Dy = bDy

where D

d and D dx

d dz

dy ⎞ d 2 y d ⎛ dy ⎞ d ⎛ b = ⋅ ⎟ = ⎜ 2 ⎝ ⎠ dx dx dx ⎝ a + bx dz ⎠ dx =–

dy ⎛ dy ⎞ b b ⋅b⋅ + ⋅ d dz (a a + bx) dx ⎝ dz ⎠ (a + bx) 2

=–

dy ⎛ dy ⎞ b2 b ⋅ + ⋅ d ⎝ ⎠ ⋅ dz 2 (a + bx) dz (a + bx) dz dz dx

=–

dy d2 y b2 b ⋅ + ⋅ 2 ⎛⎜ b ⎞⎟ 2 (a + bx) dz (a + bx) dz ⎝ a + bx ⎠

⎛ d 2 y dy ⎞ d2 y = b2 ⎜ 2 − ⎟ 2 ⎝ dz dz ⎠ dx 2 2 2 2 (a + bx) D y = b (D − D ) y = b 2D (D − 1) y

(a + bx) 2

Similarly,

(a + bx)3 D3 y = b3D (D − 1)(D − 2) y ......................................................... ......................................................... (a + bx) n D n y = b nD (D − 1)(D − 2) …[D − (n − 1)] y

Substituting these derivatives in Eq. (1), we get [a0 b nD (D − 1) … (D − n + 1) + a1b n −1D (D − 1) … (D − n + 2) +…+ an +1D + an ] y ⎛ ez − a ⎞ = Q⎜ ⎟ ⎝ b ⎠

which is a linear differential equation with constant coefficients and can be solved by usual methods described in previous section. 2 2 Example 1: Solve [( x + 1) D + ( x + 1)D]y = ( 2 x + 3)( 2 x + 4).

Solution: Here a = 1, b = 1 Putting x + 1 = ez, [D (D − 1) + D ] y = [2 (e z − 1) + 3][2 (e z − 1) + 4], D 2 y = 4 e 2 z + 6e z + 2 which is a linear equation with constant coefficients.

where D

d dz

Differential Equations

10.123

Auxiliary equation is m 2 = 0, m = 0, 0, (real and repeated twice)

C.F. = (c1 + c2 z ) e0 z = c1 + c2 z = c1 + c2 log ( x + 1) 1 1 1 1 (4e 2 z + 6e z + 2) = 4 ⋅ 2 ⋅ e 2 z + 6 ⋅ 2 e z + 2 ⋅ 2 e0⋅ z D2 D D D 1 1 0⋅ z 1 e = 4 ⋅ 2 e2 z + 6 ⋅ 2 e z + 2 z ⋅ 2D 1 2 1 = e 2 z + 6e z + 2 z 2 ⋅ e 0 ⋅ z = e 2 z + 6e z + z 2 2 = ( x + 1) 2 + 6 ( x + 1) + [log ( x + 1)]2 = x 2 + 8 x + 7 + [log ( x + 1)]2

P.I. =

Hence, the general solution is y = c1 + c2 log ( x + 1) + x 2 + 8 x + 7 + [log ( x + 1)]2 Aliter: After putting x + 1 = ez,

D 2 y = 4e 2 z + 6e z + 2 1 y = 2 (4e 2 z + 6e z + 2) = ∫ ⎡⎣ ∫ (4e 2 z + 6e z + 2) dz ⎤⎦ dz D = ∫ (2e 2 z + 6e z + 2 z + A) dz = e 2 z + 6e z + z 2 + Az + B = ( x + 1) 2 + 6 ( x + 1) + [log ( x + 1)]2 + A log ( x + 1) + B = x 2 + 8 x + 7 + [log ( x + 1)]2 + A log ( x + 1) + B Example 2: Solve ( 2 + 3 x )2

d2 y dy − 36 y = 3 x 2 + 4 x + 1. + 3 (2 + 3 x ) dx dx 2

Solution: Here a = 2, b = 3 Putting 2 + 3x = ez, 2

⎛ ez − 2 ⎞ ⎛ ez − 2 ⎞ [9D (D − 1) + 3 ⋅ 3D − 36] y = 3 ⎜⎝ 3 ⎟⎠ + 4 ⎜⎝ 3 ⎟⎠ + 1,

where D

d dz

e 2 z − 4e z + 4 + 4e z − 8 + 3 3 1 2z ( D 2 − 4) y = (e − 1) 27

(9D 2 − 36) y =

which is a linear equation with constant coefficients. Auxiliary equation is

m 2 − 4 = 0,

m = ± 2 (real and distinct)

C.F. = c1e 2 z + c2 e − 2 z = c1 (2 + 3 x) 2 + c2 (2 + 3 x) − 2 P.I. =

1 1 1 ⎛ 1 1 ⋅ (e 2 z − 1) = e2 z − 2 e0⋅ z ⎞⎟ 27 ⎜⎝ D 2 − 4 D 2 − 4 27 D −4 ⎠

Engineering Mathematics

10.124

) (

(

)

1 1 2z 1 0⋅ z 1 1 1 z⋅ e − e = z ⋅ e2 z + 27 2D 0−4 27 4 4 1 1 ( ze 2 z + 1) = [llog (2 + 3 x) ⋅ (2 + 3 x) 2 + 1]. = 108 108 Hence, the general solution is =

y = c1 (2 + 3 x) 2 +

c2 1 [(2 + 3x) 2 log (2 + 3x) + 1] + (2 + 3x) 2 108

d3 y d2 y dy + 2( x − 1)2 2 − 4( x − 1) + 4 y = 4 log( x − 1). 3 x d dx dx Solution: Here a = 1, b = 1 Example 3: Solve ( x − 1) 3

Putting (x – 1) = ez, [D (D − 1)(D − 2) + 2D (D − 1) − 4D + 4] y = 4 z (D 3 − D 2 − 4D + 4) y = 4 z which is a linear equation with constant coefficients.

Auxiliary equation is m3 − m 2 − 4m + 4 = 0 (m 2 − 4)(m − 1) = 0,

m = ± 2, 1 (real and distinct)

C.F. = c1e + c2 e + c3 e − 2 z = c1 ( x − 1) + c2 ( x − 1) 2 + c3 ( x − 1) − 2 z

P.I. =

2z

1 1 ⋅ 4z ⋅ 4z = D − D − 4D + 4 ⎛ 4D + D 2 − D 3 ⎞ 4 ⎜1 − ⎟ 4 ⎝ ⎠ 3

2

−1

⎛ 4D + D 2 − D 3 ⎞ = ⎜1 − ⎟ z 4 ⎝ ⎠ ⎡ 4D + D 2 − D 3 ⎛ 4D + D 2 − D 3 ⎞ 2 ⎤ = ⎢1 + +⎜ ⎟ + …⎥ z 4 4 ⎢⎣ ⎥⎦ ⎝ ⎠ her powers of D ) z = z + D ( z ) + (High = z + 1 + 0 = log ( x − 1) + 1.

Hence, the general solution is y = c1 ( x − 1) + c2 ( x − 1) 2 + c3 ( x − 1) − 2 + log ( x − 1) + 1 Exercise 10.12 Solve the following differential equations: 1. [(1 + x)2 D2 + (1 + x) D + 1] y = 2 sin log (x + 1). Ans. : y = c1 cos log (1 + x) ⎡ ⎤ ⎢ ⎥ + c sin log ( 1 + x ) 2 ⎢ ⎥ ⎢⎣ − log (1 + x) cos log (1 + x) ⎥⎦

2. [(x + 2)2 D2 – (x + 2)D + 1] y = 3x + 4. ⎡ Ans. : y = [c1 + c2 log ( x + 2)] ( x + 2) ⎤ ⎢ ⎥ 3 ⎢ + [log ( x + 2)]2 ( x + 2) − 2⎥ ⎣ 2 ⎦

Differential Equations 3. [(x – 1)3 D3 + 2(x – 1)2 D2 – 4 (x – 1) D + 4] y = 4 log (x – 1). ⎤ ⎡ Ans. : y = c1 ( x − 1) + c2 ( x − 1) ⎥ ⎢ −2 + c3 ( x − 1) + log ( x − 1) + 1⎦ ⎣ 2

4. [(x + 2)2 D2 – (x + 2) D + 1] y = 3x + 4 ⎡ Ans. : y = ( x + 2) ⎤ ⎢ ⎥ log ( 2 ) c + c x + [ 1 2 ⎢ ⎥ ⎢ ⎥ 3 ⎤ + log ( x + 2) 2 ⎥ − 2⎥ ⎢ 2 ⎦ ⎦ ⎣

{

}

5. [(2x + 1)2 D2 – 2(2x + 1) D – 12] y = 6x. ⎤ ⎡ Ans. : y = c1 (2 x + 1) −1 ⎥ ⎢ 3 1 ⎢ + c2 (2 x + 1)3 − x + ⎥ ⎢⎣ 8 16 ⎥⎦ 6. [(x + a)2 D2 – 4D + 6] y = x. ⎡ Ans. : y = c1 ( x + a )3 + c2 ( x + a ) 2 ⎤ ⎥ ⎢ 1 ⎥ ⎢ + (3 x + 2a ) ⎥⎦ ⎢⎣ 6 7. [3x + 1)2 D2 – 3(3x + 1)D – 12] y = 9x.

10.125

8. [(2x + 5)2 D2 – 6D + 8] y = 6x. ⎡ Ans. : y = (2 x + 5) 2 ⎡c (2 x + 5) ⎣ 1 ⎢ ⎢ + c2 (2 x + 5) − 2 ⎤⎦ ⎢ 3 45 ⎢ − x− ⎢⎣ 2 8

2

⎤ ⎥ ⎥ ⎥ ⎥ ⎦⎥

9. [(2 + 3x)2 D2 + 5(2 + 3x) D – 3] y = x2 + x + 1. 1 ⎡ −1 ⎤ 3 ⎢ Ans. : c1 (2 + 3 x) + c2 (2 + 3x) ⎥ ⎥ ⎢ 1 ⎡1 ⎢ + (2 + 3x) 2 ⎥ ⎢ 27 ⎣15 ⎥ ⎢ ⎥ ⎢ 1 ⎤ ⎥ ⎢ + (2 + 3x) − 7 ⎥ ⎥⎦ ⎢⎣ 4 ⎦

10. [(2x – 1)3 D3 + (2x –1) D – 2] y = 0. ⎡ Ans. : y = c1 (2 x − 1) ⎢ ⎡ ⎢ + (2 x − 1) ⎣⎢c2 (2 x − 1) ⎢ ⎢ 3⎤ − 2 ⎢ + − c ( 2 x 1 ) ⎥⎦ 3 ⎣

⎤ ⎥ 3 2 ⎥ ⎥ ⎥ ⎥ ⎦

⎤ ⎡ Ans. : y = (3 x + 1) ⎥ ⎢ 7 ⎡ ⎥ ⎢ 12 c ( 3 x + 1 ) ⎢ ⎣ 1 ⎥ ⎢ ⎢ 7 ⎤⎥ − ⎢ + c2 (3 x + 1) 12 ⎦⎥ ⎥ ⎥ ⎢ ⎢ ⎡ 3x + 1 1 ⎤ ⎥ − 3⎢ + ⎥⎥ ⎢ 4⎦ ⎦ ⎣ 7 ⎣

10.7 METHOD OF VARIATION OF PARAMETERS This method is used to find the particular integral if complimentary function is known. In this method, the particular integral is obtained by varying the arbitrary constants of the complimentary function and hence known as variation of parameters method. Consider a linear non-homogeneous differential equation of second order with constant coefficients d2 y dy ... (1) + a1 + a2 y = Q ( x) 2 dx dx Let the complimentary function is C.F. = c1 y1 + c2 y2 ... (2)

Engineering Mathematics

10.126

where y1, y2 are the solution of

d2 y dy + a1 + a2 y = 0 dx dx 2

... (3)

Let the particular integral is y = v1(x) y1 + v2(x) y2 where v1 and v2 are unknown functions of x. Differentiating Eq. (4) w.r.t. x, y ′ = v1 y1′ + v2 y2′ + v1′y1 + v2′ y2

... (4)

Let v1, v2 satisfy the equation v1′y1 + v2′ y2 = 0 y ′ = v1 y1′ + v2 y2′

Then

... (5)

Differentiating Eq. (5) w.r.t. x, y ′′ = v1 y1′′+ v2 y2′′+ v1′y1′ + v2′ y2′ Substituting y , y and y in Eq. (1), v1 y1′′ + v2 y2′′ + v1′y1′ + v2′ y2′ + a1 (v1 y1′ + v2 y2′ ) + a2 (v1 y1 + v2 y2 ) = Q ( x) v1 ( y1′′ + a1 y1′ + a2 y1 ) + v2 ( y2′′ + a1 y2′ + a2 y2 ) + v1′y1′ + v2′ y2′ = Q( x) Since y1 and y2 satisfy Eq. (3), we get v1′y1′ + v2′ y2′ = Q Solving Eqs. (5) and (6) by using Cramer’s rule,

... (6)

0 y2 Q y2′ − y2 Q v1′ = = y1 y2 y1 y2′ − y1′y2 y1′ y2′ 0 y′ Q − y1Q = v2′ = 1 y1 y2 y1 y2′ − y1′y2 y1′ y2′ y2 Q yQ v1 = ∫ − dx = ∫ − 2 d x y1 y2′ − y1′y2 W y1

v2 = ∫

y1Q yQ dx = ∫ 1 dx y1 y2′ − y1′y2 W

y1 y2 is known as Wronskian of y1, y2. y1′ y2′ Hence, the required general solution of Eq. (1) is, general solution = C.F. + P.I. where W =

= c1 y1 + c2 y2 + v1 y1 + v2 y2 where v1, v2 are obtained using formulas (7) and (8).

... (7) ... (8)

Differential Equations

10.127

Note: The above method can also be extended for third order differential equation. Let complementary function of a third order differential equation is C.F. = c1 y1 + c2 y2 + c3 y3 Let P.I. = v1(x)y1 + v2(x)y2 + v3(x)y3 where

( y2 y3′ − y3 y2′ ) Q dx W ( y y′ − y y′) Q dx v2 ( x) = ∫ 3 1 1 3 W ( y y ′ − y2 y1′) Q v3 ( x) = ∫ 1 2 dx W v1 ( x) = ∫

y1 y2 y3 W = y1′ y2′ y3′ y1′′ y2′′ y3′′

Wronskian,

Example 1: Solve (D2 + 1) y = cosec x. Solution: Auxiliary equation is m2 = 1, m = i (imaginary) C.F. = c1 cos x + c2 sin x y1 = cos x, y2 = sin x Wronskian,

W= =

y1

y2

y1′

y2′

cos x sin x − sin x cos x

=1 Let particular integral is P.I. = x1(x) cos x + v2(x) sin x where

y2 Q dx W sin x cosec x =∫ dx = − x 1 yQ v2 = ∫ 1 dx W cos x cosec x =∫ dx 1 v1 = ∫ −

= ∫ cot x dx = log sin x P.I. = – x cos x + (log sin x) sin x.

Engineering Mathematics

10.128

Hence, the general solution is y = c1 cos x + c2 sin x – x cos x + sin x log sin x Example 2: Solve (D2 – 1) y = e–x sin (e–x) + cos (e–x). Solution: Auxiliary equation is m2 – 1, = 0, m = 1 C.F. = c1e x + c2e –x y1 = e x, y2 = e –x Wronskian,

(real and distinct)

W=

y1 y1′

y2 y2′

=

ex ex

e− x −e − x

= − 2. Let particular integral is P.I. = v1(x) e x + v2(x)e–x yQ v1 = ∫ − 2 dx W e − x [e − x sin (e − x ) + cos (e − x ) dx = −∫ −2

where,

Putting e–x = t, – e–x dx = dt 1 v1 = ∫ − (t sin t + cos t ) dt 2 1 = − [t ( − cos t ) − ( − sin t ) + sin t )] 2 1 1 = t cos t − sin t = e − x cos(e − x ) − sin (e − x ) 2 2 y1Q e x [e − x sin (e − x ) + cos (e − x )] =∫ dx −2 W 1 = − ∫ [sin (e − x ) + e x cos (ee − x )] dx 2 1 1 = − ∫ d [e x cos (e − x )] = e x cos (e − x ) 2 2 1 1 x −x −x P.I. = cos (e ) − e sin (e ) − cos (e − x ) 2 2 x −x = − e sin (e ). v2 = ∫

Hence, the general solution is y = c1e x + c2e–x – e x sin (e–x).

Differential Equations

Example 3: Solve (D3 – 6D2 + 12D – 8) y =

e2 x x

Solution: Auxiliary equation is m2 – 6m2 + 12m – 8 = 0 (m – 2)3 = 0 m = 2, 2, 2 C.F. = (c1 + c2 x + c3 x2) e2x

(repeated thrice)

y1 = e2x, y2 = xe2x, y3 = x2e2x Wronskian,

y1 y2 W = y1′ y2′ y1′′ y2′′ e2 x = 2e 2 x 4e 2 x

y3 y3′ y3′′

xe 2 x (2 x + 1) e 2 x 4 ( x + 1) e 2 x

x 2 e2 x ( 2 x + 2 x ) e 2 x = 2e 6 x (4 x 2 + 8 x + 2) e 2 x 2

Let particular integral is P.I. = v1 ( x)e 2 x + v2 ( x) ⋅ xe 2 x + v3 ( x) ⋅ x 2 e 2 x where, ( y y ′ − y3 y2′ ) Q dx v1 = ∫ 2 3 W [ xe 2 x ⋅ (2 x 2 + 2 x) e 2 x − x 2 e 2 x ⋅ (2 x + 1) e 2 x ] e 2 x =∫ dx ⋅ x 2e6 x x x2 = ∫ dx = 2 4 ( y3 y1′ − y1 y3′ ) Q dx v2 = ∫ W [ x 2 e 2 x ⋅ 2e 2 x − e 2 x ⋅ (2 x 2 + 2 x) e 2 x ] e 2 x ⋅ dx =∫ x 2e6 x = ∫ − dx = − x v3 = ∫ =∫ =∫ =

( y1 y2′ − y2 y1′ ) Q dx W e 2 x ⋅ (2 x + 1) e 2 x − xe 2 x ⋅ 2e 2 x e 2 x dx ⋅ x 2e6 x 1 dx 2x

1 log x 2

x2 2x 2 2x 1 e − x e + log x ⋅ x 2 e 2 x 4 2 3x 2 2 x x 2 2 x =− e + e log x 4 2

P.I. =

10.129

Engineering Mathematics

10.130

Hence, the general solution is y = (c1 + c2 x + c3 x 2 ) e 2 x −

3x 2 2 x x 2 2 x e + e log x. 4 2

Exercise 10.13 Solve the following differential equations using variation of parameter method: 1. (D 2 + 1) y = tan x.

8. (D 2 − 2D) y = e x sin x.

⎤ ⎡ Ans. : y = c1 cos x + c2 sin x ⎢ − cos x log(sec x + tan x) ⎥⎦ ⎣ 2. (D 2 + 4) y = sec 2 2 x. 1⎤ ⎡ ⎢ Ans. : y = c1 cos 2 x + c2 sin 2 x − 4 ⎥ ⎢ ⎥ sin 2 x ⎢ + log(sec 2 x + tan 2 x) ⎥ ⎢⎣ ⎦⎥ 4 3. (D 2 + 1) y = co sec cot x.

2 x 10. (D − 2D + 1) y = x 2 e .

⎡ 4 72 x ⎤ x Ans. : y = ( c + c x ) e + x e ⎥ ⎢ 1 2 35 ⎣ ⎦ 11. (D 2 − 3D + 2) y = xe x + 2 x.

1 4. (D + 1) y = . 1 + sin x 2

⎡ Ans. : y = c1 cos x + c2 sin x ⎤ ⎢ − (1 − sin x + x cos x) ⎥⎥ ⎢ ⎢⎣ + sin x log(1 + sin x) ⎥⎦

⎡ x2 x ⎤ 2x x ⎢ Ans. : y = c1e + c2 e − e ⎥ 2 ⎥ ⎢ 3 ⎥ ⎢ − xe − x + x + ⎥⎦ ⎢⎣ 2 12. (D 2 + 1) y = x cos 2 x.

2 . 1 − ex −x

⎤ ⎡ Ans. : y = c1e + c2 e ⎥ ⎢ x −x x + e log(1 + e ) − e − 1⎥ ⎢ ⎢ ⎥ − e − x log(1 + e x ) ⎦ ⎣ 3x e 2 6. ( D − 6D + 9) y = 2 . x ⎡⎣ Ans. : y = (c1 + c2 x)e3 x − (1 + log x)e3 x ⎤⎦ x



−x

−1

−x

⎡ Ans. : y = c1e + c2 e − e sin (e ) ⎤ ⎥ ⎢ 1 ⎥ ⎢ 2x 2 −x − (e − 1) e ⎦ ⎣ x

⎡ Ans. : y = c1 cos x + c2 sin x ⎤ ⎢ ⎥ x 4 ⎢ − cos 2 x + sin 2 x ⎥ ⎢⎣ ⎥⎦ 2 9 13. (D 2 + 1) y = log cos x. ⎤ ⎡ Ans. : y = c1 cos x + c2 sin x ⎢ ⎥ + (log cos x − 1) ⎢ ⎥ ⎢⎣ + sin x log (sec x + tan x) ⎥⎦ 14. (D 2 + 4D + 8) y = 16e −2 x co sec 2 2 x.

1

2 −2 x 7. (D − 1) y = 2(1 − e ) 2 . x

⎡ ex ⎤ −x −2 x ⎢ Ans. : y = c1e + c2 e + ⎥ 6⎥ ⎢ 2 ⎢ ⎛x 3x 7 ⎞ ⎤ ⎥ +⎜ − + ⎟⎥ ⎥ ⎢ 2 4 ⎠ ⎦ ⎥⎦ ⎝ 2 ⎢⎣ 3

⎤ ⎡ Ans. : y = c1 cos x + c2 sin x ⎢ − cos x log |sin x | − x sin x ⎥⎥ ⎢ ⎢⎣ − sin x cot x ⎦⎥

5. (D 2 − 1) y =

⎤ ⎡ ex 2x sin x ⎥ Ans. : y = c + c e − 1 2 ⎢ 2 ⎣ ⎦ 2 x 2 ( D + 3 D + 2 ) y = e + x . 9.

⎡ Ans. : y = e −2 x (c1 cos 2 x + c2 sin 2 x) ⎤ ⎢ ⎥ + 4e −2 x cos 2 x log |cosec x ⎢ ⎥ ⎢ ⎥ −2 x + cot 2 x | − 4 e ⎣ ⎦

Differential Equations

10.131

10.8 METHOD OF UNDETERMINED COEFFICIENTS This method can be used to find the particular integral only if linearly independent derivatives of Q(x) are finite in number. This restriction implies that Q(x) can only have the terms such as k, xn, eax, sin ax, cos ax and combinations of such terms where k, a are 1 constants and n is a positive integer. However, when Q( x) = or tan x or secx, etc., x this method fails, since each function has an infinite number of linearly independent derivatives. In this method, particular integral is assumed as a linear combination of the terms in Q(x) and all its linearly independent derivatives. Some of the choices of particular integral are given below. Sr. No.

1. 2. 3. 4. 5. 6.

7.

Particular Integral

Q(x) ke

ax

ax

Ae

k sin(ax + b) or k cos (ax + b) A sin (ax + b)+ B cos (ax + b) keax sin (bx + c) or keax cos (bx + c) kxn n = 0, 1, 2, ... kxn eax n = 0, 1, 2, .... kxn sin (ax + b) or kxn cos (ax + b)

kxn eax sin (bx + c) or kxn eax cos (bx + c)

A eax sin (bx + c) + B eax cos (bx + c) Anxn + An–1 x n–1 + ...... + A2x2 +A1x+A0 eax (Anxn +An–1 xn–1 + ...... +A2 x2 +A1 x+A0) xn [An sin (ax + b) + Bncos (ax + b)] + xn–1 [An–1 sin(ax + b) + Bn–1 cos (ax + b)]+..... + x [A1sin(ax + b) + B1 cos(ax + b)] + [A0 sin(ax + b) + B0 cos(ax + b)] eax [xn {An sin (ax + b) + Bncos (ax + b)} + xn–1{An–1 sin(ax + b) + Bn–1 cos (ax + b)} + ..... + x {A1sin(ax + b) + B1 cos (ax + b) + {A0 sin (ax + b) + B0 cos (ax + b)}]

In the table, A0, A1, A2, …, An are coefficients to be determined. To obtain the values of these coefficients, we use the fact that the particular integral satisfies the given differential equation. However, before assuming the particular integral it is necessary to compare the terms of Q(x) with the complimentary function. While comparing the terms following different cases arise. Case I: If no terms of Q(x) occurs in the complimentary function, then particular integral is assumed from the table depending on the nature of Q(x). Case II: If a term u of Q(x) is also a term of the complimentary function corresponding to an r-fold root, then assumed particular integral corresponding to u should be multiplied by xr. Case III: If xsu is a term of Q(x) and only u is a term of complimentary function corresponding to an r-fold root, then assumed particular integral corresponding to xsu should be multiplied by xr.

Engineering Mathematics

10.132

Note: In case (ii) and (iii) initially similar type of terms appear in complimentary function and in assumed particular integral. After multiplication by xr the terms of particular integral changes. Hence this method avoids the repetition of similar terms in complimentary function and particular integral. Example 1: Solve ( D2 − 2D + 5) y = 25 x 2 + 12 Solution: Auxiliary equation is m 2 − 2m + 5 = 0 m=

2 ± 4 − 20 = 1 ± 2i 2

(imaginary)

C.F. = e x (c1 cos 2 x + c2 sin 2 x) Q = 25x2 + 12 Let particular integral is y = A1 x 2 + A 2 x + A 3 Dy = 2A1 x + A 2 D 2 y = 2A1 Substituting these derivatives in the given equation, 2A1 − 2(2A1 x + A2 ) + 5(A1 x 2 + A 2 x + A 3 ) = 25 x 2 + 12 5A1 x 2 + ( −4 A1 + 5A 2 ) x + (2A1 − 2A 2 + 5A 3 ) = 25 x 2 + 12 Comparing coefficients on both the sides, 5A1 = 25,

A1 = 5

4 A1 = 4 5 1 2A1 − 2A 2 + 5A 3 = 12, A 3 = (12 − 10 + 8) = 2 5 P.I. = 5 x 2 + 4 x + 2 Hence, the general solution is y = e x (c1 cos 2 x + c2 sin 2 x) + 5 x 2 + 4 x + 2 − 4A1 + 5A 2 = 0,

A2 =

Example 2: Solve ( D2 − 2D + 3) y = x 3 + sin x . Solution: Auxiliary equation is m 2 − 2m + 3 = 0 m = 1± i 2

(

(imaginary)

C.F. = e c1 cos 2 x + c2 sin 2 x Q = x3 + sin x x

)

Differential Equations

10.133

Let particular integral is y = A1 x 3 + A 2 x 2 + A 3 x + A 4 + A 5 sin x + A 6 cos x Dy = 3A1 x 2 + 2A 2 x + A 3 + A 5 cos x − A 6 sin x D 2 y = 6A1 x + 2A 2 − A 5 sin x − A 6 cos x Substituting these derivatives in the given equation, (6A1 x + 2 A2 − A 5 sin x − A 6 cos x) − 2(3A1 x 2 + 2A 2 x + A 3 + A 5 cos x − A 6 sin x) + 3(A1 x 3 + A 2 x 2 + A 3 x + A 4 + A 5 sin x + A 6 cos x) = x 3 + sin x

3A1 x 3 + ( − 6 A1 + 3A 2 ) x 2 + (6A1 − 4 A2 + 3A 3 ) x + (2 A2 − 2A 3 + 3A 4 ) − 2(A 5 − A6 ) coss x + 2(A 5 + A 6 ) sin x = x 3 + sin x Comparing coefficients on both the sides, 3A1 = 1, −6A1 + 3A 2 = 0, 6A1 − 4A 2 + 3A 3 = 0, 2A 2 − 2A 3 + 3A 4 = 0, 2(A 5 − A 6 ) = 0, 2(A 5 + A 6 ) = 1, P.I. =

A1 =

1 3

A 2 = 2A1 =

2 3

2 1 (4A 2 − 6A1 ) = 9 3 2 8 A 4 = (A3 − A 2 ) = − 3 27 A5 = A6 A3 =

2(A 5 + A 5 ) = 1,

A5 =

1 1 , A6 = 4 4

8 1 1 3 2 2 2 x + x + x− + (sin x + cos x) 3 3 9 27 4

Hence, the general solution is 2 2 8 1 1 y = e x (c1 cos 2 x + c2 sin 2 x) + x 3 + x 2 + x − + (sin x + cos x) 3 3 9 27 4 Example 3: Solve ( D2 − 9) y = x + e 2 x − sin 2 x . Solution: Auxiliary equation is m2 − 9 = 0 m = ±3 (real and distinct ) C.F. = c1e3 x + c2 e −3 x Q = x + e 2x – sin 2x

Engineering Mathematics

10.134

Let particular integral is y = A1 x + A 2 + A 3 e 2 x + A 4 sin 2 x + A 5 cos 2 x Dy = A1 + 2A 3 e 2 x + 2A 4 cos 2 x − 2A 5 sin 2 x D 2 y = 4A 3 e 2 x − 4A 4 sin 2 x − 4A 5 cos 2 x Substituting these derivatives in the given equation, 4A 3 e 2 x − 4A 4 sin 2 x − 4A 5 cos 2 x − 9(A1 x + A 2 + A 3 e 2 x + A 4 sin 2 x + A 5 cos 2 x) = x + e 2 x − sin 2 x ( −5A 3 )e 2 x − 9A1 x − 9A 2 + sin 2 x( −13A 4 ) + cos 2 x( −13A 5 ) = x + e 2 x − sin 2 x Comparing coefficients on both the sides, 1 5 1 A1 = − 9 A2 = 0

−5A 3 = 1,

A3 = −

−9A1 = 1, −9A 2 = 0,

1 13 A5 = 0

−13A 4 = −1,

A4 =

−13A 5 = 0,

1 1 1 P.I. = − x − e 2 x + sin 2 x 9 5 13 Hence, the general solution is y = c1e3 x + c2 e −3 x −

x e 2 x sin 2 x − + 9 5 13

Example 4: Solve ( D2 – 2D) y = e x sin x . Solution: Auxiliary equation is m 2 2m m C.F.

0 0,

2

c1 c2 e

(real and distinct) 2x

Q = e x sin x Let particular integral is y = A1e x sin x + A 2 e x cos x Dy = A1 (e x sin x + e x cos x) + A 2 (e x cos x − e x sin x) = (A1 − A 2 )e x sin x + (A1 + A 2 )e x cos x D 2 y = (A1 − A 2 )(e x sin x + e x cos x) + (A1 + A 2 )(e x cos x − e x sin x) = −2A 2 e x sin x + 2A1e x cos x

Differential Equations

10.135

Substituting these derivatives in the given equation, −2A 2 e x sin x + 2A1e x cos x − 2(A1 − A 2 )e x sin x − 2(A1 + A 2 )e x cos x = e x sin x − 2A1e x sin x − 2A 2 e x cos x = e x sin x Comparing coefficients on both the sides, −2A1 = 1,

A1 = −

2A 2 = 0,

A2 = 0

1 2

1 P.I. = − e x sin x 2 Hence, the general solution is 1 y = c1 + c2 e 2 x − e x sin x 2 Example 5: Solve ( D 3 + 3D2 + 2D) y = x 2 + 4 x + 8 . Solution: Auxiliary equation is m3 + 3m 2 + 2m = 0 m(m + 1)(m + 2) = 0 m

0, 1, 2

C.F. = c1 + c2 e

−x

(real and distinct) + c3 e −2 x

Q = x2 + 4x + 8 Let particular integral is y = A1 x 2 + A 2 x + A 3 Since constant occurs in Q(x) and is also a part of C.F. corresponding to 1-fold root m = 0, multiplying assumed particular integral by x. y = A1 x 3 + A 2 x 2 + A 3 x Dy = 3A1 x 2 + 2A 2 x + A 3 D 2 y = 6A1 x + 2A 2 D3 y = 6A1 Substituting these derivatives in the given equation, 6A1 + 3(6A1 x + 2A 2 ) + 2(3A1 x 2 + 2A 2 x + A 3 ) = x 2 + 4 x + 8 6A1 x 2 + (18A1 + 4 A2 ) x + (6A1 + 6A 2 + 2A 3 ) = x 2 + 4 x + 8 Comparing coefficients on both the sides, 6A1 = 1, 18A1 + 4A 2 = 4,

1 6 1 1 A 2 = (4 − 3) = 4 4 A1 =

Engineering Mathematics

10.136

6A1 + 6A 2 + 2A 3 = 8,

A3 =

1 1⎛ 3 ⎞ 11 (8 − 6A1 − 6A 2 ) = ⎜ 8 − 1 − ⎟ = ⎝ 2 2 2⎠ 4

P.I. =

1 3 1 2 11 x + x + x 6 4 4

Hence, the general solution is y = c1 + c2 e − x + c3 e −2 x +

x3 x 2 11x + + 6 4 4

Example 6: Solve ( D2 + 1) y = 4 x cos x − 2 sin x . Solution: Auxiliary equation is m2 + 1 = 0 m = ±i C.F. = c1 cos x + c2 sin x Q = 4x cos x – 2 sin x Let particular integral is y = A1 x sin x + A 2 x cos x + A 3 sin x + A 4 cos x Since x cos x and its derivatives occur in Q(x) and cos x is a part of C.F. corresponding to 1 fold pair of complex root m = ±i, multiplying assumed particular integral by x, y = A1 x 2 sin x + A 2 x 2 cos x + A 3 x sin x + A4 x cos x Dy = A1 x 2 cos x + 2 A1 x sin x − A 2 x 2 sin x + 2A 2 x cos x + A 3 x cos x + A 3 sin x − A 4 x sin x + A 4 cos x = (A1 cos x − A 2 sin x) x 2 + (2A1 − A 4 ) x sin x + (2A 2 + A 3 ) x cos x + A 3 sin x + A 4 cos x D y = ( − A1 sin x − A 2 cos x) x 2 + (A1 cos x − A 2 sin x)(2 x) + (2A1 − A 4 ) sin x 2

+ (2A1 − A 4 ) x cos x + (2A 2 + A3 ) cos x − (2A 2 + A 3 ) x sin x + A 3 cos x − A 4 sin x = − A1 x 2 sin x − A 2 x 2 cos x + (4A1 − A 4 ) x cos x − (4A 2 + A 3 ) x sin x + 2(A1 − A 4 ) sin x + 2(A 2 + A 3 ) cos x Substituting these derivatives in given equation, − A1 x 2 sin x − A 2 x 2 cos x + (4A1 − A 4 ) x cos x − (4A 2 + A 3 ) x sin x + 2(A1 − A 4 ) sin x + 2(A 2 + A 3 ) cos x + A1 x 2 sin x + A 2 x 2 cos x + A 3 x sin x + A 4 x cos x = 4 x cos x − 2 sin x 4 A1 x cos x − 4A 2 x sin x + 2(A1 − A 4 ) sin x + 2(A 2 + A 3 ) cos x = + 4 x cos x − 2 sin x

Differential Equations

10.137

Comparing coefficients on both the sides, 4A1 = 4,

A1 = 1

−4A 2 = 0,

A2 = 0

2(A1 − A 4 ) = −2,

A 4 = A1 + 1 = 2

2(A 2 + A 3 ) = 0,

A3 = 0

P.I. = x sin x + 2 x cos x 2

Hence, the general solution is y = c1 cos x + c2 sin x + x 2 sin x + 2 x cos x Example 7: Solve ( D3 − D2 − 4D + 4) y = 2 x 2 − 4 x − 1 + 2 x 2 e 2 x + 5 xe 2 x + e 2 x . Solution: Auxiliary equation is m3 − m 2 − 4m + 4 = 0 (m − 1)(m 2 − 4) = 0 m = 1, ± 2 (real and distinct ) C.F. = c1e x + c2 e 2 x + c3 e −2 x Q = 2x2 – 4x – 1 + 2x2 e2x + 5x e2x + e2x Let particular integral is y = A1 x 2 + A 2 x + A 3 + A 4 x 2 e 2 x + A5 xe 2 x + A 6 e 2 x Since x2e2x and its derivatives occur in Q(x) and e2x is a part of C.F. corresponding to 1-fold root m = 2, multiplying assumed particular integral corresponding to x2e2x by x, y = A1 x 2 + A 2 x + A 3 + A4 x 3 e 2 x + A 5 x 2 e 2 x + A 6 xe 2 x Dy = 2A1 x + A 2 + 2e 2 x (A 4 x 3 + A 5 x 2 + A6 x) + e 2 x (3A 4 x 2 + 2 A5 x + A 6 ) = 2A1 x + A2 + e 2 x [2A 4 x 3 + (3A 4 + 2A 5 ) x 2 + (2A 5 + 2A 6 ) x + A 6 ] D 2 y = 2A1 + 2e 2 x [2A 4 x 3 + (3A 4 + 2A 5 ) x 2 + (2A 5 + 2A 6 ) x + A 6 ] + e 2 x [6A 4 x 2 + (3A 4 + 2A 5 )2 x + (2A 5 + 2A 6 )] = 2A1 + e 2 x [4A 4 x 3 + (12A 4 + 4A 5 ) x 2 + (6A 4 + 8A 5 + 4 A6 ) x + (2 A5 + 4A 6 )] D3 y = 2e 2 x [4A 4 x 3 + (12A 4 + 4A 5 ) x 2 + (6A 4 + 8A 5 + 4 A6 ) x + (2A 5 + 4A 6 )] + e 2 x [12A 4 x 2 + (12A 4 + 4A 5 )2 x + (6A 4 + 8A 5 + 4A 6 )] = e 2 x [8A 4 x 3 + (36A 4 + 8A 5 ) x 2 + (36A 4 + 24A 5 + 8A 6 ) x + (6A 4 + 12A 5 + 12A 6 )] Substituting these derivatives in the given equation, e 2 x [(8A 4 − 4A 4 − 8A 4 + 4A 4 ) x 3 + (36A 4 + 8A 5 − 12A 4 − 4A 5 − 12A 4 − 8A 5 + 4A 5 ) x 2 + (36A 4 + 24A 5 + 8A 6 − 6A 4 − 8 A5 − 4A 6 − 8A 5 − 8A 6 + 4A 6 ) x + (6A 4 + 12A 5 + 12A 6 − 2A 5 − 2A 6 − 4A 6 )]

Engineering Mathematics

10.138

− 2A1 − 8A1 x − 4A 2 + 4A1 x 2 + 4A 2 x + 4A 3 = 2 x 2 − 4 x − 1 + 2 x 2 e 2 x + 5 xe 2 x + e 2 x 12A 4 x 2 e 2 x + (30A 4 + 8 A5 ) xe 2 x + (6A 4 + 10 A5 + 6 A6 )e 2 x + 4A1 x 2 + (4 A2 − 8A1 ) x + ( −2A1 − 4A 2 + 4 A3 ) = 2 x 2 − 4 x − 1 + 2 x 2 e 2 x + 5 xe 2 x + e 2 x Comparing coefficients on both the sides, 1 6 1 A 5 = (5 − 5) = 0 8 1 A 6 = (1 − 1 − 0) = 0 6 1 A1 = 2 A 2 = −1 + 2A1 = −1 + 1 = 0

12A 4 = 2,

A4 =

30A 4 + 8A 5 = 5, 6A 4 + 10 A5 + 6A 6 = −1, 4A1 = 2, 4 A2 − 8A1 = −4,

1 1 1 A A3 = − + 1 + A 2 = − + + 0 = 0 4 2 4 4

−2A1 − 4A 2 + 4A 3 = −1, Particular integral is y=

1 2 1 3 2x x + xe 2 6

Hence, the general solution is y = c1e x + c2 e 2 x + c3 e −2 x +

1 2 1 3 2x x + xe 2 6

Example 8: Solve ( D − 3)2 ( D + 2) y = x 2 e 3 x + x 2 . Solution: Auxiliary equation is (m − 3) 2 (m + 2) = 0 m=3

(repeated twice), m = −2

C.F. = (c1 + c2 x)e3 x + c3 e −2 x Q = x2 e3x + x2 Let particular integral is y = A1 x 2 e3 x + A 2 xe3 x + A 3 e3 x + A 4 x 2 + A 5 x + A 6 Since x2e3x occurs in Q(x) and e3x is a part of C.F. corresponding to a 2 fold root m = 3, multiplying assumed particular integral corresponding to x2e3x by x2, y = A1 x 4 e3 x + A 2 x 3 e3 x + A 3 x 2 e3 x + A 4 x 2 + A 5 x + A 6 Dy = 3e3 x (A1 x 4 + A 2 x 3 + A 3 x 2 ) + e3 x (4A1 x 3 + 3A 2 x 2 + 2A 3 x) + 2A 4 x + A 5 = e3 x [3 3A1 x 4 + (3A 2 + 4A1 ) x 3 + (3A 3 + 3A 2 ) x 2 + 2A 3 x] + 2A 4 x + A5

Differential Equations

10.139

D 2 y = 3e3 x [3A1 x 4 + (3A 2 + 4A1 ) x 3 + (3A 3 + 3A 2 ) x 2 + 2A 3 x] + e3 x [12 A1 x 3 + 3(33A 2 + 4A1 ) x 2 + 2(3A 3 + 3A 2 ) x + 2 A3 ] + 2A 4 = e3 x [9A1 x 4 + (24A1 + 9A 2 ) x3 + (12A1 + 18 A2 + 9A 3 ) x 2 + (6A 2 + 12A 3 ) x + 2A 3 ] + 2A 4 D3 y = 3e3 x [9A1 x 4 + (24A1 + 9A 2 ) x 3 + (12A1 + 18A 2 + 9A 3 ) x 2 + (6A 2 + 12A 3 ) x + 2 A3 ] + e3 x [36A1 x3 + 3(24A1 + 9A 2 ) x 2 + 2(12A1 + 18A 2 + 9A 3 ) x + 6A 2 + 12 A3 ] = e3 x [27A1 x 4 + (108A1 + 27A 2 ) x 3 + (108A1 + 81A 2 + 27 A3 ) x 2 + (24A1 + 54A 2 + 54A 3 ) x + 6A 2 + 12A 3 ] Substituting these derivatives in the given equation, (D − 3) 2 (D + 2) y = x 2 e3 x + x 2 (D3 − 4D 2 − 3D + 18) y = x 2 e3 x + x 2 (27 A1 − 39A1 − 9A1 + 18A1 ) x 4 e3 x + (108A1 + 27 A 2 − 96A1 − 36A 2 −12A1 − 9A 2 + 18A 2 ) x 3 e3 x + (108A1 + 81A 2 + 27 A 3 − 48A1 − 72A 2 −36A 3 − 9A 3 − 9A 2 + 18A 3 ) x 2 e3 x + (24A1 + 54A 2 + 54A 3 − 24A 2 −48A 3 − 6A 3 ) xe3 x + (6A 2 + 12A 3 − 8A 3 )e3 x − 8A 4 − 3A 5 +18A 6 + (−6A 4 + 18A 5 )x + 18A 4 x 2 = x 2 e3 x + x 2 (60A1 ) x 2 e3 x + (24A1 + 30A 2 ) xe3 x + (6A 2 + 4A 3 )e3 x + 18A 4 x 2 + (−6A 4 + 18A 5 ) x + (−8A 4 − 3A 5 + 18A 6 ) = x 2 e3 x + x 2 Comparing coefficients on both the sides, 60A1 = 1, 24A1 + 30A 2 = 0, 6A 2 + 4A 3 = 0, 18A 4 = 1, −6A 4 + 18A 5 = 0, −8A 4 − 3A 5 + 18A 6 = 0,

1 60 1 A2 = − 75 3A 1 A3 = − 2 = 2 50 1 A4 = 18 A4 1 A5 = = 3 54 A1 =

A6 =

1 1 (8A 4 + 3A 5 ) = 18 36

Engineering Mathematics

10.140

Particular integral is 1 1 4 3x 1 3 3x 1 2 3x 1 2 1 xe − xe + xe + x + x+ 60 75 50 18 54 36 Hence, the general solution is P.I. =

7 3 7 2 ⎞ 3x 1 2 1 1 ⎛ 1 x+ y = (c1 + c2 x)e3 x + c3 e −2 x + ⎜ x 4 − x + x ⎟e + x + ⎝ 60 150 100 ⎠ 18 544 36 Exercise 10.14 Solve the following differential equations using method of undetermined coefficients: 1. (D 2 + 6D + 8) y = e −3 x + e x .

⎡ ex ⎤ −2 x −4 x −3 x ⎢ Ans. : y = c1e + c2 e − e + ⎥ 15 ⎦ ⎣ 2. (4D 2 − 1) y = e x + e3 x. x x − ⎡ ⎤ 2 2 ⎢ Ans. : y = c1e + c2 e ⎥ ⎢ ⎥ 1 ⎢ + (35e x + 3e3 x ) ⎥ 105 ⎣ ⎦ 3. (D 2 + D − 6) y = 39 cos 3 x.

⎡ Ans. : y = c1e 2 x + c2 e −3 x ⎤ ⎢ ⎥ 1 ⎢ + (sin 3 x − 5 cos 3 x) ⎥ ⎢⎣ ⎥⎦ 2 4. (D 2 + 2D + 5) y = 6 sin 2 x + 7 cos 2 x. ⎡ Ans. : y = e − x (c1 cos 2 x + c2 sin 2 x) ⎤ ⎥ ⎢ + 2 sin 2 x − cos 2 x ⎦ ⎣ 2 5. (D + 4D − 5) y = 34 cos 2 x − 2 sin 2 x.

⎤ ⎡ Ans. : y = c1e x + c2 e −5 x ⎥ ⎢ + 2(sin 2 x − cos 2 x) ⎦ ⎣ 3 2 6. (D − D + D − 1) y = 6 cos 2 x.

⎡ Ans. : y = c1e x + c2 cos x + c3 sin x ⎤ ⎢ ⎥ 2 ⎢ + (cos 2 x − 2 sin 2 x) ⎥ ⎢⎣ ⎥⎦ 5 2 3 7. (2D − D − 3) y = x + x + 1.

3x ⎡ ⎤ −x 2 ⎢ Ans. : y = c1e + c2 e ⎥ ⎢ ⎥ 1 ⎢ − (9 x 3 − 9 x 2 + 51x − 20) ⎥ 27 ⎣ ⎦ 8. (D 2 + 4) y = 8 x 2 .

⎡ Ans. : y = c1 cos 2 x + c2 sin 2 x ⎤ ⎥ ⎢ + 2x2 − 1 ⎦ ⎣ 2 −2 x 9. (3D + 2D − 1) y = e + x. x ⎡ ⎤ −x 3 ⎢ Ans. : y = c1e + c2 e ⎥ ⎢ ⎥ 1 ⎢ + (e −2 x − 7 x − 14) ⎥ 7 ⎣ ⎦ 2 2 10. (D − 2D + 3) y = x + sin x.

⎡ Ans. : y = e x ( c1 cos 2 x + c2 sin 2 x ) ⎤ ⎢ ⎥ ⎢ + 1 (9 x 2 + 6 x − 8) + 1 (sin x + cos x) ⎥ ⎢⎣ 27 ⎥⎦ 4 4 4 11. (D − 1) y = x + 1.

⎡ Ans. : y = c1e x + c2 e − x + c3 cos x ⎤ ⎥ ⎢ + c4 sin x − x 4 − 25 ⎥⎦ ⎢⎣ 12. (D 2 − 1) y = e3 x cos 2 x − e 2 x sin 3 x. ⎡ Ans. : y = c1e x + c2 e − x ⎤ ⎢ ⎥ 1 ⎢ + e 2 x (2 cos 3 x + sin 3 x) ⎥ ⎢ ⎥ 30 ⎢ ⎥ 1 ⎢ + e3 x (cos 2 x + 3 sin 2 x) ⎥ 40 ⎣ ⎦

Differential Equations

2 −x 3 13. (D + 3D + 2) y = 12e sin x.

⎤ ⎡ e− x −x −2 x [(cos 3x ⎥ ⎢ Ans. : y = c1e + c2 e + 10 ⎥ ⎢ + 3 sin 3 x) − 45(cos x + sin x)]⎥⎦ ⎢⎣ 14. (D 2 + 4D + 3) y = 6e − x .

15. (D − D − 6) y = 5e

−2 x

2 3 2x 2x 22. ( D − 4D + 4) y = x e + xe .

⎡ Ans. : y = (c1 + c2 x)e 2 x ⎤ ⎢ ⎥ 5 3 ⎛x x ⎞ 2x ⎥ ⎢ +⎜ + ⎟e ⎥ ⎢ ⎝ 20 6 ⎠ ⎦ ⎣ 2 2x 23. (D − 3D + 2) y = xe + sin x.

⎡⎣ Ans. : y = c1e − x + c2 e −3 x + 3 xe − x ⎤⎦ 2

10.141

+ 10e . 3x

⎡⎣ Ans. : y = c1e3 x + c2 e −2 x + 2 xe3 x − xe −2 x ⎤⎦ 2 16. (D + 16) y = 16 sin 4 x.

⎡ Ans. : y = c1 cos 4 x + c2 sin 4 x ⎤ ⎥ ⎢ − 2 x cos 4 x ⎦ ⎣ 17. (D 2 + 25) y = 50 cos 5 x + 30 sin 5 x. ⎡ Ans. : y = c1 cos 5 x + c2 sin 5 x ⎤ ⎢ − x(3 cos 5 x − 5 sin 5 x) ⎥⎦ ⎣ 18. (D3 – 2D2 + 4D – 8)y = 8(x2 + cos2x) ⎡ Ans. : y = c1e 2 x + c2 cos 2 x ⎤ ⎢ ⎥ + c3 sin 2 x − ( x 2 + x) ⎥ ⎢ ⎢ ⎥ x ⎢ − (cos 2 x + sin 2 x) ⎥ ⎣ 2 ⎦ 2 2x 19. (D − 4D + 5) y = 16e cos x.

⎡ Ans. : y = e 2 x (c1 cos x + c2 sin x) ⎤ ⎥ ⎢ + 8 xe 2 x sin x ⎦ ⎣ 2 3x 20. (D − 6D + 13) y = 6e sin x cos x.

⎡ Ans. : y = e3 x (c1 cos 2 x ⎤ ⎢ ⎥ 3x ⎢ + c2 sin 2 x) − e3 x cos 2 x ⎥ ⎢⎣ ⎥⎦ 4 3 2 x 2 21. (D + 2D − D − 2) y = e + x .

⎡ Ans. : y = c1e x + c2 e − x + c3 e −2 x ⎤ ⎢ ⎥ x2 x 5 1 ⎢ + xe x − + − ⎥ ⎢⎣ 6 2 2 4 ⎥⎦

⎡ ⎛ x2 ⎞ 2x ⎤ 2x x Ans. : y = c e + c e + ⎢ 1 2 ⎜⎝ 2 − x⎟⎠ e ⎥ ⎥ ⎢ ⎥ ⎢ 1 3 + sin x + cos x ⎥ ⎢ 10 10 ⎣ ⎦ 2 3 24. (D + 1) y = sin x.

⎡ Ans. : y = c1 cos x + c2 sin x ⎤ ⎢ ⎥ 1 3 ⎢ + sin 3 x − x cos x ⎥ ⎢⎣ ⎥⎦ 32 8 2 2 −x 25. (D + 2D + 1) y = x e .

⎡ x4 − x ⎤ −x ⎢ Ans. : y = (c1 + c2 x)e + e ⎥ 12 ⎣ ⎦ 26. (D3 − D 2 − 4D + 4) y = 2 x 2 − 4 x −1 + 2 x 2 e 2 x + 5 xe 2 x + e 2 x . ⎡ Ans. : y = c1e x + c2 e 2 x + c3 e −2 x ⎤ ⎢ ⎥ x 2 x3 2 x ⎢ ⎥ + + e ⎢⎣ ⎥⎦ 2 6 2 3x 27. ( D − 5D − 6) y = e , y (0) = 2, y ′ (0) = 1

10 6 x 45 − x 1 3 x ⎤ ⎡ ⎢ Ans. : y = 21 e + 28 e − 12 e ⎥ ⎣ ⎦ 2 x 28. (D − 5D + 6) y = e (2 x − 3), y (0) = 1, y ′ (0) = 3.

⎡⎣ Ans. : y = e 2 x + xe x ⎤⎦ 29. (D3 − D) y = 4e − x + 3e 2 x, y (0) = 0, y ′ (0) = −1, y ′′ (0) = 2.

Engineering Mathematics

10.142

⎡ Ans. : y = c1 + c2 e x + c3 e − x ⎤ ⎥ ⎢ 1 ⎢ + 2 xe − x + e 2 x ⎥ ⎥⎦ ⎢⎣ 2

3 2 x 30. (D − 2D + D) y = 2e + 2 x, y (0) = 0, y ′ (0) = 0, y ′′ (0) = 0.

⎡⎣ Ans. : y = x 2 + 4 x + 4 + e x ( x 2 − 4) ⎤⎦

10.9 SIMULTANEOUS LINEAR DIFFERENTIAL EQUATIONS WITH CONSTANT COEFFICIENTS Sometimes we come across the linear differential equation with more than one dependent variable and a single independent variable. Such equations are called simultaneous linear differential equations with constant coefficients and can be solved by eliminating one of the dependent variable. This method is known as elimination method. These equations can also be solved by using Laplace transform method, matrices method, short cut operator method. Here, we will discuss the elimination method only. Note: The total number of arbitrary constants in the general solution is equal to the order of the differential equation of that dependent variable which is obtained first. If total number of arbitrary constants are more than the order of the differential equation (degree of auxiliary equation), then arbitrary constants are obtained by putting dependent variables and their derivatives (as required) in the given simultaneous equation. dx dy = 5 x + y, = y − 4 x. Example 1: Solve dt dt Solution: Putting

d dt

D, equations reduce to (D − 5) x − y = 0

... (1)

4 x + (D − 1) y = 0

... (2)

Eliminating y from Eqs. (1) and (2) by operating Eq. (1) by (D - 1) and then adding, (D − 5)(D − 1) x + 4 x = 0 ( D 2 − 6 D + 9) x = 0 (D − 3) 2 x = 0 Auxiliary equation,

(m − 3) 2 = 0 m = 3, 3 (repeated twice) C.F. = (c1 + c2t)e3t, P.I. = 0

Hence, x = (c1 + c2 t )e3t D( x) = c2 e3t + (c1 + c2 t )3e3t = (c2 + 3c1 + 3c2 t )e3t

Differential Equations

10.143

Putting the value of x and Dx in Eq. (1), y = (c2 + 3c1 + 3c2 t )e3t − 5(c1 + c2 t )e3t = ( −2c1 + c2 − 2c2 t )e3t Hence, the general solution is x = (c1 + c2 t )e3t y = ( −2c1 + c2 − 2c2 t )e3t Example 2: Solve Solution: Putting

dx dy dx − 3x − 6 y = t 2 , + − 3 y = et . dt dt dt

d dt

D, equations reduce to (D − 3) x − 6 y = t 2 Dx + (D − 3) y = e

... (1) ... (2)

t

Eliminating y from Eqs. (1) and (2) by operating Eq. (1) by (D Eq. (2) by 6 and then adding,

3) and multiplying

(D − 3) 2 x + 6 Dx = (D − 3)t 2 + 6et ( D2 + 9) x = 2t − 3t 2 + 6et Auxiliary equation, m 2 + 9 = 0, m = ± 3i C.F. = c1 cos 3t + c2 sin 3t −1

P.I. = =

1 1 1 ⎛ D2 ⎞ 6et (2t − 3t 2 ) + 2 .6et = ⎜1 + (2t − 3t 2 ) + 2 ⎟ 9⎝ 9 ⎠ 10 D +9 D +9 ⎞ 1 ⎛ D2 D4 3 1⎡ 1 ⎤ 3 1− + − …⎟ (2t − 3t 2 ) + et = ⎢(2t − 3t 2 ) − (0 − 6) + 0⎥ + et 9 ⎜⎝ 9 81 5 9⎣ 9 ⎦ 5 ⎠

t 2 2t 2 3 t + + + e 3 9 27 5 t 2 2t 2 3 t Hence, x = c1 cos 3t + c2 sin 3t − + + + e 3 9 27 5 2t 2 3 Dx = −3c1 sin 3t + 3c2 cos 3t − + + et 3 9 5 Putting the value of x and Dx in Eqs. (1), =−

2t 2 3 t + + e 3 9 5 ⎛ t 2 2t 2 3 t ⎞ 2 −3 ⎜ c1 cos 3t + c2 sin 3t − + + + e − t = 6y 3 9 27 5 ⎟⎠ ⎝ −3c1 sin 3t + 3c2 cos 3t −

1 1 2t 1 y = − (c1 + c2 ) sin 3t + (c2 − c1 ) cos 3t − − et 2 2 9 5

Engineering Mathematics

10.144

Hence, the general solution is t 2 2t 2 3 t + + + e 3 9 27 5 1 2t 1 1 y = − (c1 + c2 ) sin 3t + (c2 − c1 ) cos 3t − − et 9 5 2 2

x = c1 cos 3t + c2 sin 3t −

Example 3: Solve D2 y = x − 2, D2 x = y + 2. Solution:

D 2 y − x = −2

... (1)

− y + D2 x = 2

... (2)

Eliminating y from Eqs. (1) and (2) by operating Eq. (2) by D2 and then adding, − x + D 4 x = −2 + D 2 (2) (D 4 − 1) x = −2 Auxiliary equation m4 − 1 = 0 m = 1, − 1, i, − i C.F. = c1et + c2 e − t + c3 cos t + c4 sin t 1 1 ( −2) = 4 ( −2e0t ) D −1 D −1 =2

P.I. =

4

Hence, x = c1et + c2 e − t + c3 cos t + c4 sin t + 2 Dx = c1et − c2 et − c3 sin t + c4 cos t D 2 x = c1et + c2 e − t − c3 cos t − c4 sin t Putting D2x in Eq. (2), y = D2 x − 2 = c1et + c2 e − t − c3 cos t − c4 sin t − 2 Hence, the general solution is x = c1e t + c2e–t + c3cos t + c4 sin t + 2 y = c1e t + c2e–t – c3 cos t – c4 sin t – 2 Example 4: Solve ( D2 + 4) x − 3Dy = 0, 3 Dx + ( D2 + 4) y = 0. Solution:

(D 2 + 4) x − 3Dy = 0

... (1)

3Dx + (D + 4) y = 0

... (2)

2

Differential Equations

10.145

Eliminating y from Eqs. (1) and (2) by operating Eq. (1) by (D2 + 4) and Eqs. (2) by 3D and then adding, (D 2 + 4) 2 x + 9D 2 x = 0 (D 4 + 17 D 2 + 16) x = 0 Auxiliary equation, m 4 17 m 2 16 (m

2

1)(m

2

16) m

0 0 i,

4i

c1 cos t c2 sin t c3 cos 4t c4 sin 4t

C.F. P.I. =

1 ⋅0 = 0 D 4 + 17 D 2 + 16

x = c1 cos t + c2 sin t + c3 cos 4t + c4 sin 4t

Hence,

Dx = − c1 sin t + c2 cos t − 4c3 sin 4t + 4c4 cos 4t D 2 x = −c1 cos t − c2 sin t − 16c3 cos 4t − 16c4 sin 4t Putting x and D2x in Eq. (1), 3Dy = −c1 cos t − c2 sin t − 16c3 cos 4t − 16c4 sin 4t + 4(c1 cos t + c2 sin t + c3 coos 4t + c4 sin 4t ) 1 Dy = (3c1 cos t + 3c2 sin t − 12c3 cos 4t − 12c4 sin 4t ) 3 Integrating w.r.t. t,

y = c1 sin t − c2 cos t − c3 sin 4t + c4 cos 4t + k ,

Putting Dx, D2y, and y in Eq. (2), we get k1 = 0 Hence, the general solution is x = c1 cos t + c2 sin t + c3 cos 4t + c4 sin 4t y = c1 sin t − c2 cos t − c3 sin 4t + c4 coos 4t Example 5: Solve (D2 + D + 1)x + (D2 + 1) y = et, (D2 + D)x + D2 y = e–t. Solution:

(D 2 + D + 1) x + (D 2 + 1) y = et ( D 2 + D) x + D 2 y = e − t

... (1) ... (2)

Eliminating y from Eqs. (1) and (2) by operating Eq. (1) by D2 and Eq. (2) by (D2 + 1) and then subtracting, D 2 (D 2 + D + 1) x − (D 2 + 1)(D 2 + D) x = D 2 et − (D 2 + 1)e − t − Dx = et − e − t − e − t Dx = −et + 2e − t

... (3)

10.146

Engineering Mathematics

Integrating w.r.t. t, x = −et − 2e − t + c1 D 2 x = − e t − 2e − t Putting Dx, D2x in Eq. (2), D 2 y = e − t − D2 x − Dx = e − t + et + 2e − t + et − 2e − t D 2 y = e − t + 2et Integrating w.r.t. t, Dy = −e − t + 2et + k1 Integrating again w.r.t. t, y = e − t + 2et + k1t + k2 Since the order of the Eq. (3) is one, there should be only one arbitrary constant in the general solution. Putting x, Dx, D2x, y, D2y in Eq. (1), (D 2 + D + 1) x + (D 2 + 1) y = et ( −et − 2e − t − et + 2e − t − et − 2e − t + c1 ) + (e − t + 2et + e − t + 2et + k1t + k2 ) = et et + c1 + k1t + k2 = et k1t + k2 = −c1 Therefore, y = e − t + 2et − c1 Hence, the general solution is x = −et − 2e − t + c1 y = 2et + e − t − c1 Example 6: Solve 2 D2 x + 3Dy − 4 = 0, 2D2 y − 3Dx = 0 where x = y + Dx = Dy = 0 at t = 0. Solution:

2D 2 x + 3Dy − 4 = 0

... (1)

−3Dx + 2D 2 y = 0

... (2)

Eliminating y from Eqs. (1) and (2) by operating Eq. (1) by 2D and multiplying Eq. (2) by 3 and then subtracting, 4 D3 x + 9Dx − 2D4 = 0 D(4D 2 + 9) x = 0

Differential Equations

Auxiliary equation, D ( 4 D 2 + 9) = 0 D = 0, ±

3i 2

3 3 C.F. = c1e0t + c2 cos t + c3 sin t 2 2 1 P.I. = ⋅0 = 0 4D3 + 9D 3 3 x = c1 + c2 cos t + c3 sin t 2 2

Hence,

3 3 3 3 Dx = − c2 sin t + c3 cos t 2 2 2 2 At t = 0, x = 0 and Dx = 0 c1 + c2 = 0 and c1 = 0 3 x = c1 − c1 cos t 2 3 3 Dx = c1 sin t 2 2 Putting the value of Dx in Eq. (2), 3 3 2D 2 y = 3 ⋅ c1 sin t 2 2 9⎛ 3 ⎞ D 2 y = ⎜ c1 sin t ⎟ 4⎝ 2 ⎠ Integrating w.r.t. t, 3 ⎞ ⎛ cos t 9⎜ 2 ⎟ + k = −3c1 cos 3 t + k Dy = ⎜ −c1 1 3 ⎟ 1 2 2 4 ⎜⎝ ⎟⎠ 2

Integrating again w.r.t. t, 3 ⎞ ⎛ −3c1 ⎜ sin 2 t ⎟ y= + k1t + k2 2 ⎜ 3 ⎟ ⎜⎝ ⎟⎠ 2 3 y = −c1 sin t + k1t + k2 2

10.147

Engineering Mathematics

10.148 At t = 0, y = 0 and Dy = 0

k2 = 0

k1 =

and

3c1 2

3 3c y = −c1 sin t + 1 t 2 2

Hence,

Dy = − D2 x =

Also,

3c1 3 3c cos t + 1 2 2 2

9 3 c1 cos t 4 2

Putting the value of D2x and Dy in Eq. (1), 9 3 9 3 9 c1 cos t − c1 cos t + c1 − 4 = 0 2 2 2 2 2 8 c1 = 9 Hence, the general solution is x=

8⎛ 3 ⎞ ⎜⎝1 − cos t ⎟⎠ 9 2

y=

4⎛ 2 3 ⎞ ⎜⎝ t − sin t ⎟⎠ 3 3 2

Example 7: Solve D2 x + 2Dy + 8 x = 32t , D2 y + 3Dx − 2 y = 60e t where at t = 0, x = 6, Dx = 8, y = -24 and Dy = 0. (D 2 + 8) x + 2Dy = 32t

Solution:

3Dx + (D − 2) y = 60e 2

... (1) −t

Eliminating y from Eqs. (1) and (2) by operating Eq. (1) by (D and then subtracting,

... (2) 2

2) and Eq. (2) by 2D

(D 2 − 2)(D 2 + 8) x − 6D 2 x = (D 2 − 2)32t − 2D(60e − t ) (D 4 − 16) x = −64t + 120e − t Auxiliary equation m 4 − 16 = 0 m = −2, 2, − 2i, 2i C.F. = c1e −2t + c2 e 2t + c3 cos 2t + c4 sin 2t

... (3)

Differential Equations

P.I. =

10.149

1 ( −64t + 120e − t ) D − 16 4

−1

⎛ D4 ⎞ 120e − t −64 ⎛ D 4 ⎞ 1 = − t + = 4 1+ + …⎟ t − 8e − t ⎜ ⎟ ⎜ 1 − 16 −16 ⎝ 16 ⎠ ⎝ 16 ⎠ = 4t − 8e − t Hence,

x = c1e −2t + c2 e 2t + c3 cos 2t + c4 sin 2t + 4t − 8e − t Dx = −2c1e −2t + 2c2 e 2t − 2c3 sin 2t + 2c4 cos 2t + 4 + 8e − t D 2 x = 4c1e −2t + 4c2 e 2t − 4c3 cos 2t − 4c4 sin 2t − 8e − t

Putting x and D2x in Eq. (1), 2Dy = 32t − D 2 x − 8 x = 32t − 4(c1e −2t + c2 e 2t − c3 cos 2t − c4 sin 2t − 2e − t ) − 8(cc1e −2t + c2 e 2t + c3 cos 2t + c4 sin 2t + 4t − 8e − t ) Dy = 36e − t − 6c1e −2t − 6c2 e 2t − 2c3 cos 2t − 2c4 sin 2t Integrating w.r.t. t, y = −36e − t + 3c1e −2t − 3c2 e 2t − c3 sin 2t + c4 cos 2t + k1 Since the order of the Eq. (3) is 4, there should be only 4 arbitrary constants in the general solution. Putting Dx, y, D2y in Eq. (2), D 2 y + 3Dx − 2 y = 60e − t −36e − t + 12 + 24e − t + 72e − t − 2k1 = 60e − t Comparing costant term on both the sides, k1 = 6 Hence, y = −36e − t + 3c1e −2t − 3c2 e 2t − c3 sin 2t + c4 cos 2t + 6 At t = 0, x = 6, Dx = 8 and y = – 24, Dy = 0 6 = c1 + c2 + c3 − 8

... (4)

c1 + c2 + c3 = 14 and or

8 = −2c1 + 2c2 + 2c4 + 4 + 8 −c1 + c2 + c4 = −2

... (5) −24 = −36 + 3c1 − 3c2 + c4 + 6

or and or

3c1 − 3c2 + c4 = 6

... (6)

0 = 36 − 6c1 − 6c2 − 2c3 3c1 + 3c2 + c3 = 18

... (7)

Engineering Mathematics

10.150

Solving Eqs. (4), (5), (6) and (7), we get c1 = 2, c2 = 0, c3 = 12, c4 = 0 Hence, the general solution is x = 2e −2t + 12 cos 2t + 4t − 8e − t y = −36e − t + 6e −2t − 12 sin 2t + 6 Exercise 10.15 Solve the following differential equations: dy 1. dx = 3 x + 8 y, = −x − 3y dt dt ⎡ Ans. : x = −4c1et − 2c2 e − t ,⎤ ⎥ ⎢ y = c1et − c2 e − t ⎥⎦ ⎢⎣ dx dy = 2 y − 1, = 1 + 2x 2. dt dt 1 ⎤ ⎡ 2t −2 t ⎢ Ans. : x = c1e + c2 e − 2 ,⎥ ⎥ ⎢ 1 ⎥ 2t −2 t ⎢ y = c1e − c2 e + ⎢⎣ 2 ⎥⎦ 3. (D + 6)y Dx = 0, (3 D) x 2 Dy = 0 with x = 2, y = 3 at t = 0 ⎡ Ans. : x = 4e 2t − 2e −3t , ⎤ ⎢ ⎥ y = e 2t + 2e −3t ⎣ ⎦ 4.

dx dy + y − 1 = sin t , + x = cos t dt dt −t

⎤ ⎡ Ans. : x = c1e + c2 e , ⎢ t −t ⎥ y = 1 + sin t − c1e + c2 e ⎥⎦ ⎢⎣ t

t 5. (D + 5) x + (D + 7) y = 2e , (2D + 1) x + (3D + 1) y = et

1 ⎧ 5 −2t ⎫ ⎤ ⎡ t ⎢Ans. : x = 1 + 5t ⎨ (2 − 8c2 )e + 2 c1e ⎬ , ⎥ ⎩ ⎭ ⎥ ⎢ −2 t t ⎢⎣ y = c1e + c2 e ⎦⎥

6.

d2 x + y = sin t , dt 2

⎡ Ans. : ⎢ ⎢ ⎢ ⎢ ⎢ ⎢ ⎣

d2 y + x = cos t dt 2

⎤ x = c1et + c2 e − t + c3 cos t ⎥ t t + c4 sin t − cos t + sin t ⎥ 4 4 ⎥ t −t y = − c1e − c2 e + c3 cos t ⎥ ⎥ 1 + c4 sin t + ( 2 + t )(sin t − cos t ) ⎥ ⎦ 4

7. D 2 x + 3 x − 2 y = 0, D 2 x + D 2 y 3x + 5y = 0 with x = 0, y = 0, Dx = 3, Dy = 2 when t = 0 1 1 ⎡ ⎢ Ans. : x = 4 (11sin t + 3 sin 3t ), ⎢ 1 ⎢ y = (11sin t − sin 3t ) ⎢⎣ 4

⎤ ⎥ ⎥ ⎥ ⎥⎦

10.10 APPLICATIONS OF ORDINARY DIFFERENTIAL EQUATIONS OF FIRST ORDER AND FIRST DEGREE 10.10.1 Orthogonal Trajectories Two families of curves are called orthogonal trajectories of each other if every curve of one family cuts each curve of another family at right angles.

Working rule: (a) Cartesian curve f (x, y, c) = 0

Differential Equations

10.151

dy ⎞ ⎛ (i) Obtain the differential equation F ⎜ x, y, ⎟ = 0 by differentiating and elim⎝ dx ⎠ inating c from the equation of the family of curves. dy dx by in the above differential equation to obtain the differendx dy ⎛ dx ⎞ tial equation of the family of orthogonal trajectories as F ⎜ x, y, − ⎟ = 0. ⎝ dy ⎠ ⎛ dx ⎞ (iii) Solve the differential equation F ⎜ x, y, − ⎟ = 0 to obtain the equation of ⎝ dy ⎠ (ii) Replace

the family of orthogonal trajectories. (b) Polar curve f (r , , c) = 0 dr (i) Obtain the differential equation F ⎛⎜ r , , ⎞⎟ = 0 by differentiating and elimid ⎠ ⎝ nating c from the equation of the family of curves.

(ii) Replace

d dr by r 2 in the above differential equation to obtain the differend dr

d ⎞ tial equation of the family of orthogonal trajectories as F ⎛⎜ r , , − r 2 ⎟ = 0. dr ⎠ ⎝ d ⎞ ⎛ (iii) Solve the differential equation F ⎜ r , , − r 2 ⎟ = 0 to obtain the equation dr ⎠ ⎝ of the family of orthogonal trajectories. Example 1: Find the orthogonal trajectories of the family of semicubical parabolas ay2 = x3. Solution: The equation of the family of curves is ay2 = x3

... (1)

Differentiating Eq. (1) w.r.t. x, a⋅2y Substituting a =

dy = 3x 2 dx

x3 from Eq. (1), y2

dy x3 ⋅2y = 3x 2 2 dx y 2 x dy =3 y dx This is the differential equation of the given family of curves. dy dx Replacing by in Eq. (2), dx dy

... (2)

Engineering Mathematics

10.152

−2 x dx =3 y dy

... (3)

This is the differential equation of the family of orthogonal trajectories. Separating the variables and integrating Eq. (3),

∫ −2 x dx = ∫ 3 y dy 3y2 +c 2 −2 x 2 = 3 y 2 + 2c − x2 =

2 x 2 + 3 y 2 + 2c = 0 which is the equation of the required orthogonal trajectories. Example 2: Find the orthogonal trajectories of the family of curves x2 y2 + = 1, where is a parameter. a 2 b2 + Solution: The equation of the family of curves is x2 y2 + a 2 b2 +

Differentiating Eq. (1) w.r.t. x,

=1

2x 2y + 2 2 a b + x y + 2 2 a b +

dy =0 dx dy =0 dx y =− 2 b +

y2 b2 +

=−

... (1)

... (2)

x ⎛ dy ⎞ a2 ⎜ ⎟ ⎝ dx ⎠ xy ⎛ dy ⎞ a2 ⎜ ⎟ ⎝ dx ⎠

... (3)

Substituting Eq. (3) in Eq. (1), x2 − a2

xy =1 ⎛ dy ⎞ a2 ⎜ ⎟ ⎝ dx ⎠

( x2 − a2 )

dy = xy dx

This is the differential equation of given family of curves. Replacing

dy by dx

dx in Eq. (4), dy

... (4)

Differential Equations

dx = xy dy This is the differential equation of the orthogonal trajectories. Separating the variables and integrating Eq. (5), a2 − x2 ∫ y dy = ∫ x d x + c 1 2 1 y = a 2 log x − x 2 + c 2 2 2 2 2 x + y = 2a log x + 2c (a 2 − x 2 )

10.153

... (5)

which is the equation of the required orthogonal trajectories. Example 3: Find the equation of the family of all orthogonal trajectories of the family of circles, which pass through the origin (0, 0) and have centres on the y-axis. Solution: The equation of the family of circles passing through (0, 0) and having centres on y-axis is x2 + y 2 + 2 f y = 0 Differentiating Eq. (1) w.r.t. x, 2x + 2 y

dy dy +2f =0 dx dx dy −x = dx y + f

... (1)

... (2)

From Eq. (1), f =−

x2 + y 2 2y

y+ f = y−

x2 + y 2 y 2 − x2 = 2y 2y

Substituting in Eq. (2), dy −2 xy = 2 dx y − x 2 This is the differential equation of the given family of circles. Replacing

dy by dx

dx in Eq. (3), dy dx 2 xy = 2 dy y − x 2

... (3)

Engineering Mathematics

10.154

This is the differential equation of the family of orthogonal trajectories. ( y 2 − x 2 )dx − 2 xy dy = 0

... (4)

M = y 2 − x2 , ∂M = 2 y, ∂y ∂M ∂N ≠ ∂y ∂x

N = −2 xy ∂N = −2 y ∂x

The equation is not exact. ∂M ∂N − 4y 2 ∂y ∂x = =− N −2 xy x 2

I.F. = e Multiplying Eq. (4) by

∫ − x dx

−2

= e −2 log x = elog x = x −2 =

1 x2

1 , x2

⎛ y2 ⎞ 2y ⎜⎝ x 2 − 1⎟⎠ dx − x dy = 0 M1 =

y2 − 1, x2

N1 = −

2y x

∂M1 ∂N1 2 y = = 2, ∂y ∂x x The equation is exact. Hence, solution is ⎛ y2 ⎞ ∫ ⎜ x 2 − 1⎟⎠ dx − ∫ 0 dy = c y constant ⎝ − y2 −x=c x x 2 + y 2 + cx = 0 which is the equation of the required orthogonal trajectories representing the equation of the family of the circles with centre on x-axis and passing through origin. x2 y2 Example 4: Show that the family of confocal conics + = 1 is selforthogonal. Here a is the parameter and b is the constant. a a − b Solution: The equation of the family of curves is x2 y2 + =1 a a −b

... (1)

Differential Equations

10.155

Differentiating Eq. (1) w.r.t. x, 2x 2 y dy + =0 a a − b dx dy yy ′ x where y ′ = =− , dx a−b a ayy ′ = − ax + bx a ( x + yy ′ ) = bx bx a= x + yy ′ Putting value of a in Eq. (1), x 2 ( x + yy ′ ) + bx

y2 =1 bx −b x + yy ′

x( x + yy ′ ) y 2 ( x + yy ′ ) + =1 b −byy ′ xy ′ − y b = y′ x + yy ′

... (2)

This is the differential equation of the given family of curves. Replacing y by −

1 in Eq. (2), y′ −

x −y b y′ = 1 ⎛ y⎞ − x+⎜− ⎟ y′ ⎝ y′ ⎠

by ′ xy ′ − y xy ′ − y b = y′ x + yy ′ x + yy ′ =

which is same as Eq. (2). Therefore, differential equation of the family of orthogonal trajectories is the same as differential equation of the family of curves. Hence, the given family of curves is self orthogonal. Example 5: Find the orthogonal trajectories of the family of the curves r n sin n = a n . Solution: The family of the curves is given by the equation r n sin n = a n

... (1)

Engineering Mathematics

10.156

Differentiating Eq. (1) w.r.t.q , nr n −1

dr ⋅ sin n + r n n cos n = 0 d dr = −r cot n d

... (2)

This is the differential equation of the given family of curves. dr 2 d by r in Eq. (2), Replacing dr d d = −r cot n dr d r = cot n dr

−r 2

... (3)

This is the differential equation of the family of orthogonal trajectories. Separating the variables and integrating Eq. (3),

∫ tan n

d =∫

dr r

log sec n = log r + log c n log sec n = n log rc = log(rc) n sec n = c n r n

1 cn which is the equation of the required orthogonal trajectories. r n cos n = k where k =

Example 6: Find the orthogonal trajectories of the family of the curves r = 4a secp tanp. Solution: The equation of the family of curves is r = 4a sec tan

... (1)

Differentiating Eq. (1) w.r.t.q, dr = 4a (sec tan tan + sec sec 2 ) d

Substituting 4a =

r sec tan

from Eq. (1),

dr = r (tan + cot sec 2 ) d

... (2)

Differential Equations

This is the differential equation of the given family of curves. d dr Replacing by r 2 in Eq. (2), dr d d −r 2 = r (tan + cot sec 2 ) dr d 1 sin −r = + dr cos cos sin d siin 2 +1 −r = dr cos sin

10.157

... (3)

This is the differential equation of the family of orthogonal trajectories. Separating the variables and integrating Eq. (3), 1 2 cos sin dr − ∫ d =∫ 2 2 sin + 1 r 1 − log(1 + sin 2 ) = log r − log c 2 − log(1 + sin 2 ) = 2 log r − 2 log c = log r 2 − log c 2 log r 2 (1 + sin 2 ) = log c 2 r 2 (1 + sin 2 ) = c 2 which is the equation of the family of orthogonal trajectories. Example 7: Find the orthogonal trajectories of the family of the curves r = a (1 + sin2p ) . Solutin: The equation of the family of the curves is r = a (1 + sin2q )

... (1)

Differentiating Eq. (1) w.r.t. q, dr = a ⋅ 2sin cos d

Substituting a =

r 1 + sin 2

from Eq. (1), dr r = d 1 + sin 2

⋅ 2 sin cos

... (2)

This is the differential equation of the given family of curves. dr 2 d Replacing by r in Eq. (2), dr d d r = ⋅ 2 sin cos dr 1 + sin 2 d 2 sin cos −r = dr 1 + sin 2 −r 2

This is the differential equation of the family of orthogonal trajectories.

... (3)

Engineering Mathematics

10.158

Separating the variables and integrating Eq. (3), ⎛ 1 + sin 2 ⎞ dr ⎟ d = −∫ cos ⎠ r tan ⎞ dr ⎛ ∫ ⎜⎝ cosec 2 + 2 ⎟⎠ d = −∫ r

∫ ⎜⎝ 2 sin

log ( cosec 2 − cot 2

)

+

log sec 2

= − log r + log c 2 ⎡ ⎛ 1 – cos 2 ⎞ ⎤ 2 2 log ⎢sec ⎜ ⎟ ⎥ = –2 log r + 2 log c = – log r + log c ⎝ sin 2 ⎠ ⎦ ⎣

⎡ ⎤ c2 2 sin 2 log ⎢sec ⋅ ⎥ = log 2 2 sin cos ⎦ r ⎣ c2 r2 2 r = c 2 cos cot

sec tan =

which is the equation of the family of orthogonal trajectories. Exercise 10.16 1. Find the orthogonal trajectories of the family of following curves: (i) y2 = 4 ax (ii) x 2 − y 2 = ax x3 2 (iii) y = a−x (iv) x2 + y2 + 2ay + b = 2 (v) (a + x) y 2 = x 2 (3a − x) ⎡ Ans. : (i) ⎢ ( ii) ⎢ ⎢ (iii) ⎢ (iv) ⎢ ⎢ ( v) ⎣

⎤ 2x 2 + y 2 = c ⎥ 2 2 y ( y + 3x ) = c ⎥ ( x 2 + y 2 ) 2 = c(2 x 2 + y 2 ) ⎥ ⎥ x 2 + y 2 + 2cx − b = 0 ⎥ ( x 2 + y 2 )5 = cy 3 (5 x 2 + y 2 ) ⎥⎦

2. Show that the family of confocal x2 y2 conics 2 + 2 = 1 is self ora +c b +c thogonal. Here a and b are constants and c is the parameter. 3. Find the value of the constant d such that the parabolas y = c1x2 + d are the

orthogonal trajectories of the family of the ellipses x 2 + 2 y 2 − y = c2 1⎤ ⎡ ⎢⎣ Ans.: d = 4 ⎥⎦ 4. Find the orthogonal trajectories of the family of following curves: (i) r = a(1 + cos q ) 2a (ii) r = 1 + cos (iii) r2 = a sin2 q (iv) rn = an cos nq (v) r = a (secq + tanq ) (vi) r = aeq

⎡ Ans. : (i) ⎢ ( ii) ⎢ ⎢ ⎢ (iii) ⎢ (iv) ⎢ ⎢ ( v) ⎢ ⎢ ( vi) ⎣

r = c(1 – cos ) ⎤ ⎥ c r= ⎥ 1 – cos ⎥ ⎥ r 2 = c 2 cos 2 ⎥ n n r = c si n n ⎥ ⎥ log r = – sin + c ⎥ ⎥ r = ce − ⎦

Differential Equations

10.159

10.10.2 Electrical Circuit A simple electric circuit consists of a voltage source, resistor, inductor and capacitor. To find current, voltage or change in an electric circuit, a differential equation is formed using Kirchhoff ’s voltage Law (KVL) which states that the algebraic sum of all the voltages in a closed loop or circuit is zero. The voltage across resistor, inductor and capacitor are given by, vR = R i vL = L vC =

di dt

1 i dt C∫

R-L circuit The figure shows a simple R-L circuit. Applying Kirchhoff ’s voltage law to the circuit, di Ri + L = e (t ) dt The differential equation is di R e (t ) + i= dt L L

R

e (t)

The figure shows a simple R-C circuit. Applying Kirchhoff ’s voltage Law to the circuit, 1 idt = e(t ) C∫

L

Fig. 10.1

R-C circuit

Ri +

i

R

e (t)

i

C

Differentiating the equation, R

d di i + = e(t ) dt C dt

Fig. 10.2

The differential equation is di de(t ) 1 + i= dt RC dt Example 1: A circuit consisting of a resistance R and inductance L is connected in series with a voltage E. (a) Find the value of the current at any time t. Given that i = 0 at t = 0. (b) Show that the current builds up to half its maximum value L in log 2 seconds. R Solution: Applying Kirchhoff ’s law to series R-L circuit, di =E dt di R E + i= dt L L

Ri + L

10.160

Engineering Mathematics

The equation is linear in i. P=

R E , Q= L L R

I.F. = e

∫ L dt

R

= eL

t

Solution is R

E RL t E L Rt e dt + c = ⋅ e L + c L L R R t − E i = +c e L R

t

i.e L = ∫

At t = 0, i = 0 E +c R −E c= R 0=

Hence, i=

R − t⎞ E E − RL t E ⎛ − e = ⎜1 − e L ⎟ R R R⎝ ⎠

(b) The current reaches its maximum value as t → ∞ E i (∞) = = I max R When I E i = max = 2 2R R − t⎞ E E⎛ = ⎜1 − e L ⎟ 2R R ⎝ ⎠ R − t 1 = 1− e L 2

e

R − t L

R

t

=

1 2

eL = 2 R t = log 2 L L t = log 2 R

Differential Equations

10.161

Example 2: The current in a circuit containing an inductance L, resistance di + Ri = E sin t . If initially there is no dt R t⎤ ⎡ E L sin( – ) sin ω φ φ ⋅ t + e ⎥ where 2 2 2 ⎢ R +ω L ⎣ ⎦

R and voltage E sin v t is given by L current in the circuit show that i = ωL tan φ = . R Solution:

dt + Ri = E sin t dt di R E + i = sin t dt L L

L

The equation is linear in i.

E R , Q = sin t L L

P=

R

I.F. = e

∫ L dt

R

= eL

t

Solution is R

i.e

R t L

R E E = ∫ sin t ⋅ e t + c = L L

i=

t

⎞ ⎛R sin t − cos t ⎟ + c 2 ⎜ L R ⎠ ⎝ 2 + 2 L eL

R − t E R sin t − L cos t ) + ce L 2 2 ( R + L 2

At t = 0, i = 0 L

0 = −E c = E⋅

R + 2

2 2

L

+c

L R2 +

2 2

L

Hence, R − t E E L ( R sin t − L cos t ) + e L 2 2 2 R + L R + 2 L2 ⎛ ⎞ 1 R L sin t − cos t ⎟⎟ = E⋅ ⎜ 2 2 2 2 2 2 2 2 ⎜ 2 R + L ⎝ R + L R + L ⎠

i=

+e

2

R − t L

E R + 2

2 2

L



L R + 2

2 2

L

Engineering Mathematics

10.162

Putting

R R +ω L 2

2 2

ωL

= cos φ and

R + ω 2 L2 2

= sin φ

R − t ⎤ ⎡ sin(ωt − φ ) + e L sin φ ⎥ ⎢ 2 2 2 R +ω L ⎣ ⎦

E

i=

Example 3: A circuit consisting of resistance R and a condenser of capacity C is connected in series with a voltage E. Assuming that there is no charge on condenser at t = 0, find the value of current i, voltage and charge q at any time t. Solution: Applying Kirchoff ’s law to series R-C circuit, Ri +

1 i dt = E C∫

But

i=

dq dt

dq q + =E dt C dq 1 E + q= dt RC R R

The equation is linear in q. P=

1 E , Q= RC R t

1

I.F. = e

∫ RC dt

= e RC

Solution is t

t

q e RC = ∫ e RC

E dt + k R

t

qe

t RC

t E e RC = + k = CE e RC + k R 1 RC

q = CE + ke At t = 0, q = 0

0 = CE + k k = CE



t RC

Differential Equations

Hence,

q = CE − CE e



t RC

t t − − ⎛ ⎞ ⎛ ⎞ = CE ⎜1 − e RC ⎟ = CE ⎜1 − e RC ⎟ ⎝ ⎠ ⎝ ⎠

i=

t − ⎞ dq d ⎛ = ⎜ CE − CE e RC ⎟ dt dt ⎝ ⎠

= CE = e= At t = 0, e = 0

10.163

t − ⎞ CE − RCt d⎛ 1 − e RC ⎟ = e ⎜ dt ⎝ ⎠ RC

E − RCt e R t − 1 1 E − RCt RC d i t = e = − E e +k C∫ C∫R

0 E+k k=E e = − Ee



t RC

+E

t − ⎛ ⎞ = E ⎜1 − e RC ⎟ ⎝ ⎠

Example 4: An emf e = 200 e–5t is applied to a series circuit consisting of 20 ohm resistor and 0.01 F capacitor. Find the charge and current at any time assuming that there is no initial charge on capacitor. Solution: Applying Kirchoff’s law to series R-C circuit, Ri + But

1 i dt = e(t ) C∫ dq i= dt

dq q + = e(t ) dt C dq 1 e + q= dt RC R Putting the values of R, C and e (t), R

dq 1 200e −5t q= + dt 20 × 0.01 20 dq + 5q = 10e −5t dt

Engineering Mathematics

10.164

Equation is linear in q. P = 5, Q = 10e I.F. = e

5 dt

5t

= e5t

Solution is qe5t = ∫ e5t 10e −5t dt + k = ∫ 10 dt + k = 10 t + k q = 10 te −5t + ke −5t At t = 0, q=0 0=0+k k=0 Hence,

q = 10 te–5t dq d i (10 te dt dt

5t

)

10 e

5t

50 te 5t .

Exercise 10.17 1. A coil having a resistance of 15 ohms and an inductance of 10 henry is connected to 90 volt supply. Determine the value of current after 2 seconds. [Ans. : 5.985 amp] 2. If a voltage of 20 cos 5t is applied to a series circuit consisting of 10 ohm resistor and 2 henry inductor, determine the current at any time t > 0. [Ans. : i cos 5t sin 5t e 5t ] 3. A capacitor of C farad with voltage v0 is discharged through a resistance of R ohms. Show that if q coulomb is the charge on capacitor, i ampere is

the current and v volt is the voltage t

at time t, show that v = v0 e RC . 4. Find the current in series R – C circuit with R = 10 , c = 0.1 F, e (t) = 110 sin 314 t, i (0) = 0. ⎤ ⎡ Ans. : i (t ) = 0.035 (cos 314 t ⎢ −t ⎥ + 314 sin 314 t − e ) ⎦ ⎣ 5. Determine the charge and current at any time t in a series R – C circuit with R = 10 , C = 2 10–4 F and E = 100 V. Given that q(0) = 0. 1 e 500t , i (t ) = 10e –500t Ans. : q (t ) = 50

10.10.3 Mechanical System If a body moves in a straight line starting from a fixed point O and covers a distance x at any instant t, then velocity of the body is given by dx v= dt

Differential Equations

10.165

and acceleration of the body is given by a= or

a

dv d 2 x = dt dt 2 dv dx dv v dx dt dx

v

dv dx

If mass of the body is m and force acting on it is F, then by Newton’s second law of motion dv F = ma = m dt dv F = mv or dx This is the equation of the motion of the particle. Example 1: A chain coiled up near the edge of a smooth table just starts to fall dv over the edge. The velocity v when a length x has fallen is given by xv + v 2 = gx . dx 2 gx . Show that V = 3 Solution: The equation of the motion is given by dv xv + v 2 = gx dx dv 2v 2 = 2g 2v + x dx

... (1)

dv dz = dx dx Substituting in Eq. (1), dz 2 + z = 2g dx x The equation is linear in z. Putting v2 = z, 2v

P= I.F. = e

2 dx x

... (2)

2 , Q = 2g x

Solution of Eq. (2) is z x 2 = ∫ 2 g ⋅ x 2 dx + c x3 +c 3 x3 x2v2 = 2g + c 3 x2 z = 2g

2

= e 2 log x = elog x = x 2

Engineering Mathematics

10.166

Initially, x = 0 when 0 = 0 + c,

v=0 c=0 2 gx 3 3 2 gx 2 v = 3 2 gx v= 3

x2v2 =

Hence,

Example 2: A particle of mass m moves in a horizontal straight line with acceleration

mk directed towards the origin at a distance x from the origin. If initially x3

the particle was at rest at a distance a from the origin, show that it will be at a a2 3 a . distance from the origin at t = 2 k 2 Solution: Since acceleration is directed towards the origin, equation of motion is given by mv

dv dx

mk x3

v

dv dx

k x3

Separating the variables and integrating, k

∫ vdv = ∫ − x

3

dx

v2 k = 2 +c 2 2x Initially, when v = 0, x = a k +c 2a 2 k c=− 2 2a

0=

Hence, v2 k k = 2− 2 2 2x 2a 2 k ( a − x2 ) v2 = a2 x2

Differential Equations

v=± k

a2 − x2 ax

dx =− k dt

a2 − x2 ax

10.167

(Negative sign is taken since x is decreasing with time)

Separating the variables and integrating,

∫ −

ax a − x2 2

dx = − ∫ k dt

1 − a (a 2 − x 2 ) 2 ( −2 x) dx = − k ∫ dt ∫ 2 1 a − ⋅ 2(a 2 − x 2 ) 2 = − kt + c ′ 2

n



[ f ( x)] f '( x)dx =

[ f ( x)]n +1 n +1

1 2 2

− a (a 2 − x ) = − kt + c ′ At t = 0, x = a c' = 0 1

Hence,

a(a 2

x2 ) 2

When

kt

x=

a 2

1

⎛ a2 ⎞ 2 a ⎜ a 2 − ⎟ = kt ⎝ 4⎠ t=

a2 2

3 k

Example 3: A particle is projected with velocity v0 along a smooth horizontal plane in the medium whose resistance per unit mass is l times the cube of the velocity. Show 1 that the distance covered by the particle in time t is 1 l v02 t 1 . l v0 Solution: Resistance per unit mass = m v3 where v is the velocity at any instant t. By Newton’s second law, v

dv dx dv v2

v3 dx

Engineering Mathematics

10.168

Separating the variables and integrating, dv ∫ v 2 = ∫ − dx 1 − = − x+c v Initially, v = v0, c=−

x=0 1 v0

1 v

Hence,

1 v0

x

1 = v

v0 x + 1 v0

v=

v0 v0 x + 1

dx = dt

v0 v0 x + 1

where x is the distance travelled at any instant t. Separating the variables and integrating,

∫(

v0 x + 1)dx = ∫ v0 dt

x2 + x = v0 t + k 2 At t = 0, x = 0 k = 0 v0

Hence,

v0 x 2 + 2 x − 2v0 t = 0

x=

4 4 m v02 t

2

2 m v0

=

1

1 m v02 t m v0

But distance is always positive. Hence,

x=

1

1 m v02 t m v0

Example 4: A body of mass m, falling from rest, is subjected to the force of gravity and an air resistance proportional to the square of the velocity (i.e. kv2). If it falls through a distance x and possesses a velocity v at that instant, prove that 2kx a2 where mg = ka2. = log 2 m a v2 Solution: The forces acting on the body are (i) its weight mg acting downwards (ii) air resistance kv2 acting upwards

Differential Equations

mg kv 2

Net force acting upon the body By Newton’s second law,

ka 2

kv 2

10.169

k (a 2

v2 )

dv = k (a 2 − v 2 ) dx v k dv = dx m a2 − v2 mv

Integrating both the sides, 1 k − log (a 2 − v 2 ) = x + c 2 m At x = 0, v = 0 1 − log a 2 = c 2

1 log(a 2 2

Hence,

v2 ) 2 kx m

k 1 x log a 2 m 2 log a 2 = log

log(a 2

v2 )

a2 a

2

v2

Example 5: A moving body is opposed by a force per unit mass of value k1x and resistance per unit mass of valve k2v2, where x and v are the displacement and velocity of the body at some instant. If equation of motion of the moving body is dv given as v = - k1 x - k2 v 2 , find the velocity of the body in terms of x, if it starts dx from the rest. Solution: The equation of the motion of the moving body is given by, v v Putting v2 = z,

2v

dv dx

k1 x k2 v 2

dv dx

k2 v 2

k1 x

dz dx

2k 2 z

2k1 x

dv dz = dx dx

Substituting in Eq. (1),

... (1)

Engineering Mathematics

10.170

Equation is linear in z. P = 2k2, Q = 2k1x I.F. = e

2 k2 dx

= e 2 k2 x

Solution is ⎡ e 2 k2 x e 2 k2 x ⎤ ze 2 k2 x = − ∫ 2k1 xe 2 k2 x dx + c = −2k1 ⎢ x ⋅ − (1) 2 ⎥ + c 4k 2 ⎦ ⎣ 2k 2 kx k = − 1 e 2 k2 x + 1 2 e 2 k2 x + c k2 2k 2 v 2 e 2 k2 x = − v2 = −

k1 2 k2 x k1 2 k2 x xe + 2 e + c k2 2k 2 k1 k x + 12 + c e −2 k2 x k2 2k 2

At x = 0, v = 0 k1 +c = 0 2k22 c=− v2

Hence,

k1 2k22 k1 x k2

k1 2k22

k1 (1 e 2k22

2 k2 x

k1 e 2k22 )

2 k2 x

k1 x k2

Exercise 10.18 1. A moving body is opposed by a force per unit mass of value Cx and resistance per unit mass of value bv2 where x and v are the displacement and velocity of the particle at that instant. Find the velocity of the particle in terms of x, if it starts from rest. Ans. : v

1 C (1 2bx e b 2

2 bx

)

2. When a bullet is fired into a sand tank, its retardation is proportional to

the square root of its velocity. How long will it take to come to rest if it enters the sand tank with velocity v0? 2 Ans. : t = v0 k 3. A particle of mass m is projected vertically with velocity v. If the air resistance is directly proportional to the velocity, then show that the particle will reach maximum height in time ⎛ kv 2 ⎞ m log ⎜1 + . k ⎝ mg ⎟⎠

Differential Equations

4. A body of mass m falls from rest under gravity in a fluid whose resistance to motion at any instant is mk times the velocity where k is a constant. Find the terminal velocity of the body and also the time required to attain one half of its terminal velocity. Hint: Terminal velocity is velocity at t→∞. g 1 Ans. : v = , t = log 2 k k 5. A particle is moving in a straight line a4 with acceleration k x + 3 directed x towards origin. If it starts from rest at a distance a from the origin, show that it will reach at the origin at the end of time

4 k

.

10.171

6. A vehicle starts from rest and its t , acceleration is given by k 1 T where k and T are constants. Find the maximum speed and the distance travelled when the maximum speed is attained. ⎡ kT kT 2 ⎤ Ans.: v = , x = max ⎥ ⎢ 2 3 ⎦ ⎣ 7. The distance x descended by a parachuter satisfies the differential 2 −2 gx ⎡ ⎤ ⎛ dx ⎞ 2 k2 ⎦, ⎜⎝ ⎟⎠ = k ⎣1 − e dt

equation

where k and g are constants. Show that x =

k2 gt log cosh g k

if x = 0

at t = 0.

10.10.4 Rate of Growth or Decay If the rate of change of a quantity y at any instant t is directly proportional to the quantity present at that time, then the differential equation of (i) growth is dy = ky dt (ii) decay is dy dt

ky

Example 1: In a culture of yeast, at each instant, the time rate of change of active ferment is proportional to the amount present. If the active ferment doubles in two hours, how much can be expected at the end of 8 hours at the same rate of growth. Find also, how much time will elapse, before the active ferment grows to eight times its initial value. Solution: Let y quantity of active ferment be present at any time t. The equation of fermentation of yeast is dy = ky, where k is a constant dt

Engineering Mathematics

10.172

Separating the variables and integrating, dy = k dt y ∫ log y = kt + c



Let at t = 0,

y = y0

log y0 = c log y = kt + log y0

Hence,

log ⎛ y ⎞ ⎜y ⎟ ⎝ 0⎠

kt

… (1)

The active ferment doubles in two hours. Therefore, at t = 2, y = 2y0

log

2 y0 = k (2) y0 k=

1 log 2 2

Substituting in Eq. (1), log

y t = log 2 2 y0 t

log 2

y = y0 e 2 (i) When t = 8 y

y0 e 4 log 2

y0 elog 2

4

y0 2 4

y = 16 y0 Hence, active ferment grows 16 times of its initial value at the end of 8 hours. (ii) When y = 8y0 t

8 y0 = y0 e 2

log 2

t log 2 2 t log 23 = log 2 2 t 3log 2 = log 2 2 t = 6 hours Hence, active ferment grows 8 times its initial value at the end of 6 hours. log 8 =

Differential Equations

10.173

Example 2: Find the half-life of uranium, which disintegrates at a rate proportional to the amount present at any instant. Given that m1 and m2 grams of uranium are present at time t1 and t2 respectively. Solution: Let m grams of uranium be present at any time t. The equation of disintegration of uranium is dm where k is a constant km dt dm k dt m Integrating both the sides, log m = − k t + c At t = 0, m = m0 log m0 = c Hence,

log m

k t log m0 log m

… (1)

kt1 = log m0 − log m1

… (2)

kt2 = log m0 − log m2

… (3)

kt

log m0

At t = t1, m = m1 and at t = t2, m = m2

Subtracting Eqs. (2) from (3), k (t2 t1 )

log m1 log m2 log

k=

m1 m2

t2 t1

Let T be the half-life of uranium, i.e., at t = T, m =

1 m0 2

From Eq. (1), m0 log 2 2 log 2 (t2 t1 ) log 2 T= = m k log 1 m2

kT

log m0

log

Exercise 10.19 1. If the population of a country doubles in 50 years, in how many years will it triple under the assumption that the

rate of increase is proportional to the number of inhabitants? [Ans. : 79 years]

10.174

Engineering Mathematics

2. The number N of bacteria in a culture grew at a rate proportional to N. The value of N was initially 100 and increased to 332 in one hour, what would 1 be the value of N after 1 hours? 2 [Ans. : 605] 3. A radioactive substance disintegrates at a rate proportional to its mass. When mass is 10 mgm, the rate of disintegration is 0.051 mgm per day. How long will it take for the mass to be reduced from 10 mgm to 5 mgm? [Ans. : 136 days]

4. Radium decomposes at a rate proportional to the amount present. If a fraction p of the original amount disappears in 1 year, how much will remain at the end of 21 years? Ans. :

1

1 p

21

times the initial

amount 5. Find the time required for the sum of money to double itself at 5% per annum compounded continuously. [Ans. : 13.9 years]

10.10.5 Newton’s Law of Cooling It states that rate of change of temperature of a body is directly proportional to the difference between the temperature of the body and that of the surrounding medium. If T is the temperature of the body and T0 is the temperature of the surrounding medium at any time t then its differential equation is dT dt

k (T T0 )

where k is a constant.

Example 1: According to Newton’s law of cooling, the rate at which a substance cools in moving air is proportional to the difference between the temperature of the substance and that of the air. If the temperature of the air is 40°C and the substance cools from 80°C to 60°C in 20 minutes, what will be the temperature of the substance after 40 minutes? Solution: Let T be the temperature of the substance at the time t. dT = − k (T − 40) dt Separating the variables and integrating, dT

∫ T − 40 = ∫ − kdt log(T − 40) = − kt + c At

t = 0, T = 80 log 40 = c

Hence,

kt

log 40 log(T

40)

Differential Equations At t = 20,

10.175

T = 60 20k

log 40 log 20 1 log 2 k= 20

Hence,

t

1 log 2 20

log 40 log (T

40

1 log 2 20

log 40 log(T

log 2

40)

At t = 40, 40)

40 40 40 log 4 = log T 40

2 log 2 = log

4= T

T

40 T 40 50 C

Exercise 10.20 1. Water at temperature 100°C cools in 10 minutes to 88°C in a room of temperature 25°C. Find the temperature of water after 20 minutes. [Ans. : 77.9°C]

2. If the temperature of the air is 30 C and the substance cools from 100 C to 70 C in 15 minutes, find when the temperature will be 40 C. [Ans. : 52.5 minutes]

10.11 APPLICATIONS OF HIGHER ORDER LINEAR DIFFERENTIAL EQUATIONS 10.11.1 Simple Harmonic Motion If a particle moves in a straight line with an acceleration directly proportional to its displacement from a fixed point O and is always directed towards O, then the motion is said to be simple harmonic motion. A B P Let the displacement of the particle O from a fixed point O at some instant t x is x, then 2 a d x w 2x 2 dt Fig. 10.3 (D 2 + w 2 ) x = 0 ... (1)

Engineering Mathematics

10.176

where

D

d dt

Solution of differential Eq. (1) is x = c1 cos w t + c2 sin w t

... (2)

Velocity of the particle at point P is

dx = − c1 sin t + c2 cos t dt Let the particle starts from rest at distance a from the fixed point O. Then at t = 0, x = a, v = 0 From Eq. (2) a = c1 From Eq. (3) 0 = w c2 c2 = 0 Hence, displacement x = a cos w t v=

and velocity, v

aw 1 cos 2 w t

aw sin w t

aw 1

... (3)

x2 a2

v w a2 x2 Nature of motion: The particle starts from rest and moves towards O and attains its maximum velocity at O. | vmax | = aw Hence, At O acceleration is zero but velocity is maximum. Hence, particle moves further and comes to rest at B such that OA = OB. Then it retraces its path and oscillates between A and B. (i) The amplitude (maximum displacement from O) = a 2 (ii) The time period (time for a complete oscillation) = w (iii) The frequency (number of oscillations per second) =

w 1 = . time period 2

Example 1: A Particle is executing simple harmonic motion with amplitude 5 metres and time 4 seconds. Find the time required by the particle in passing between points which are at distances 4 and 2 metres from the centre of force and arc on the same side of it. Solution: Amplitude, a = 5 meter 2 Time period, = 4sec w w=

2

Differential Equations

10.177

Let particle is at distances 4 and 2 meter from the centre at time t1 and t2 seconds respectively. x = a cos ω t Since ⎛π ⎞ 4 = 5 cos ⎜ t1 ⎟ ⎝2 ⎠ 2 4 t1 = cos −1 = 23.47 sec π 5 and

⎛ ⎞ 2 = 5 cos ⎜ t2 ⎟ 2 ⎝ ⎠ 2 −1 2 t2 = cos = 42.29 sec 5

Time required in passing between the points at distances 4 and 2 meter = t2 t1 = 18.82 seconds. Example 2: A particle of mass 4 g executing S.H.M. has velocities 8 cm/sec and 6 cm/sec when it is at distances 3 cm and 4 cm from the centre of its path. Find its period and amplitude. Find also the force acting on the particle when it is at a distance 1 cm from the centre. Solution: Velocity of the particle when it is at a distance x from the centre is

v2 = At x = 3, v = 8 and at x = 4, v = 6 (8) 2

2

(a 2 − x 2 )

w 2 a2

(3) 2

64 w 2 (a 2 9) and

(6)

2

w

2

a

2

(4)

36 w 2 (a 2 16) Dividing Eqs. (1) and (2), 64 a 2 9 = 36 a 2 16 a 2 = 25 a=5 Hence, amplitude = 5 cm. Putting a = 5 in Eq. (1) 64 = w 2 (25 − 9) w2 = 4 w =2

... (1) 2

... (2)

10.178

Engineering Mathematics

2π 2π = = π sec ω 2 Acceleration = w 2 x At x = 1 acceleration = w 2 = 4 Force = mass acceleration 4 ( 4) 16 dynes Negative sign indicates that the force is acting towards the centre.

Hence,

time period =

Exercise 10.21 1. A particle is executing simple harmonic motion with amplitude 20 cm. and time 4 sec. Find the time required by the particle in passing between points which are at distance 15 cm and 5 cm from the centre of force and are the same side of it. [Ans. : 0.38 sec]

per minute. Assuming its motion to be S.H.M, show that the maximum force upon the particle is 484 p 2 dynes. 3. Find the time of a complete oscillation in a simple harmonic motion if x = x1, x = x2 and x = x3 when t = 1 sec, t = 2 sec, t = 3 sec respectively. Ans. :

2. A particle of mass 4 gm vibrates through one cm on each side of the centre making 330 complete vibrations

2

wherecos =

x1 + x3 2 x2

10.11.2 Simple Pendulum A simple pendulum consists of a heavy mass m called bob attached to one end of a light inextensible string with other end fixed. The mass of the string is negligible as compared to the mass m (bob). Let the pendulum is suspended from a fixed point O. Let l be the length of the light string and m be the mass of the bob. Let P be the position of the bob at any instant t. Let arc AP = s and q is the angle which OP makes with vertical line OA, then s = lq. The equation of motion of the bob along the tangent is d2s m 2 mg sin dt d2 g sin (l ) dt 2 3 d2 l 2 g dt 3!

O q T B P q A mg sin q

mg cos q mg

Fig. 10.4

5

5!

...

Differential Equations

10.179

For sufficiently small q, higher powers of q can be neglected. d2 g 2 l dt d2 g + =0 dt 2 l d2 +w 2 = 0 dt 2

where

2

=

g l

This shows that the motion of the bob is simple harmonic motion. Time period =

2 =2 w

l . g

The motion of the bob from one extreme position to another extreme position completes half an oscillation and is called a beat or a swing. Hence, time of one beat =

l . g

Change in the number of beats: If a simple pendulum of length l makes n beats in time t, then t=n n=

log n

t

log

l g g l

t

1 ( log g log l ) 2

Differentiating both sides, dn n

1 dg 2 g

dl l

This gives the change in number of beats as g and (or) l changes. (i) If l is constant and g changes, dn 1 dg = n 2 g (ii) If l changes and g is constant, dn n

1 dl . 2 l

10.180

Engineering Mathematics

Example 1: A clock with a seconds pendulum is gaining 2 minutes a day. Prove that the length of the pendulum must be decreased by 0.0028 of its original length to make it go correctly. Solution: Total number of beats per day, n

24 60 60 86400 sec

dn = 2 minutes = 120 sec Let l be the original length and dl be the change in length. Assuming g to be constant gain per day,

dn n

1 dl 2 l

dl l dl

2 120 86400 0.0028 l

0.0028

Hence, length must be decreased by 0.0028 of its original length. Example 2: The differential equation of a simple pendulum is d2 x dx + w 2 x = F sin nt , where v and F are constants. If at t = 0, x = 0, = 0, dt 2 dt determine the motion when n = v . Solution: The differential equation is given as d2 x + w 2 x = F sin nt dt 2 (D2 + w 2) x = F sin nt Auxiliary equation is m2 + w 2 = 0 m = ± iw C.F. = c1 cos w t + c2 sin w t P.I. =

1 F sin nt D +w 2 2

If n = w P.I.

1 1 F sin w t Ft sin w t 2D D2 + w 2 Ft Ft sin w t dt cos w t 2 2w

... (1)

10.181

Differential Equations Hence, the general solution of Eq. (1) is x = C.F + P.I.

= c1 cos t + c2 sin t − dx dt At t = 0, x = 0 and

c1w sin w t c2w cos w t

Ft cos t 2

F (cos w t tw sin w t) 2w

dx =0 dt

0 = c1 and

0

c2w

c2 =

F 2w 2

F 2w

Hence, the equation of motion is x

F Ft sin w t cos w t. 2 2w 2w

Exercise 10.22 1. A clock loses five seconds a day, find alteration required in the length of its pendulum in order to keep correct time. 1 of Ans. : Shortened by 8640 its original length 2. A seconds pendulum which gains 10 seconds per day at one place loses 10 seconds per day at another. Compare

the acceleration due to gravity at the two places. Ans. :

4321 4319

3. If a pendulum clock loses 9 minutes per week, what change is required in the length of the pendulum in order to keep correct time? [Ans. : 1.7 mm]

10.11.3 Oscillation of Spring Consider a spring suspended vertically from a fixed point support. Let a mass m attached to the lower end A of the spring stretches the spring by a length e called elongation and comes to rest at B. This position is called as static equilibrium. Now mass is set in motion from equilibrium position. Let at any time t the mass is at P such that BP = x. The mass m experience the following forces: (i) gravitational force mg acting downwards (ii) restoring force k (e + x) due to displacement of spring acting upwards

Engineering Mathematics

10.182

dx (iii) damping (frictional or resistance) force c of the medium opposing the dt motion (acting upward) (iv) external force F (t) considering downward direction as positive. By Newton's second law, the differential equation of motion of the mass m is

m

dx d2 x = mg − k (e + x) − c + F (t ) 2 dt dt

At equilibrium position B, mg = ke dx d2 x Hence, m 2 = − kx − c + F (t ) dt dt d 2 x c dx k + + x = F (t ) dt 2 m dt m Let

c =2 m

and

k =w2 m

d2 x dx +2 + w 2 x = F (t ) 2 dt dt

Fig. 10.5

which represent the equation of motion and its solution gives the displacement x of the mass m at any instant t. Let us consider the different cases of motion.

(a) Free Oscillation If the external force F(t) is absent and damping force is negligible, then Eq. (1) reduces to d2 x + w 2x = 0 dt 2 which represents the equation of simple harmonic motion. Hence, motion of mass m is S.H.M. m 2π Time period = = 2π ω k Frequency =

w 1 = 2 2

k m

(b) Free Damped Oscillations If the external force F(t) is absent and damping is present, then Eq. (1) reduces to

d2 x dx + 2λ + ω2x = 0 2 dt dt

(c) Forced Undamped Oscillation If an external periodic force F (t) = Q cos nt is applied to the support of the spring and damping force is negligible then Eq. (1) reduces to

10.183

Differential Equations

d2 x + w 2 x = Q cos nt dt 2 (D 2 + w 2 ) x = Q cos nt

... (2)

C.F. = c1 cos w t + c2 sin w t 1 P.I. = 2 Q cos nt D +w 2 Hence general solution of Eq. (2) is x = C.F. + P.I. ⎛ n ⎞ If frequency of the external force ⎜ ⎟ and the natural frequency ⎝2 ⎠ w = n, then resonance occurs.

⎛ω ⎞ ⎜ ⎟ are same i.e. ⎝ 2π ⎠

(d) Forced Damped Oscillation If an external periodic force F(t) = Q cos nt is applied to the support of the spring and damping force is present then Eq. (1) reduces to d2 x dx +2 + w 2 x = Q cos nt 2 dt dt Auxiliary Equation is p2 + 2l p + w2 = 0 The general solution is x = C.F. + P.I.= xc + xp

... (3)

The xc is known as transient term and tends to zero as t . Thus term represent damped oscillations. The xp is known as steady-state term. This term represent simple harmonic motion of period

p=

2 . n

− 2λ ± 4λ 2 − 4ω 2 = −λ ± λ 2 − ω 2 2

The motion of the mass depends on the nature of the roots of the Eq. (3), i.e., on discriminant 2 w 2. Case I: If

2

2

0 then the roots of Eq. (3) are real and distinct.

(

xc = e − λt c1e xc

λ 2 −ω 2 t

λ 2 −ω 2 t

+ c2 e −

0 as t

).

.

This shows that in this case damping is so large that no oscillation can occur. Hence, the motion is called over damped or dead-beat motion. Case II: If 2 w 2 0. then roots of Eq. (2) are equal and real. xc = (c1 + c2 t )e − xc

0 as t

t

.

10.184

Engineering Mathematics

In this case damping is just enough to prevent oscillation. Hence the motion is called critically damped. Case III: If

2

w2

0 then the roots of Eq. (2) are imaginary p

Hence,

i w2

2

xc = e − λt ⎡⎣c1 cos ( ω 2 − λ 2 ) t + c2 sin ( ω 2 − λ 2 ) t ⎤⎦

In this case motion is oscillatory due to the presence of the trigonometric factor. Such a motion is called damped oscillatory motion.

Free oscillation Example 1: A body weighing 20 kg is hung from a spring. A pull of 40 kg weight will stretch the spring to 10 cm. The body is pulled down to 20 cm below the static equilibrium position and then released. Find the displacement of the body from its equilibrium position at time t seconds, the maximum velocity and the period of oscillation. Solution: Since a pull of 40 kg weight stretches the spring to 10 cm,. i.e., 0·1 m 40 k 0 1 k = 400 kg/m Weight of the body, W = 20 kg m=

W 20 = g 9.8

The equation of motion is k d2 x + w 2 x = 0, where w 2 = = 196 m dt 2 d2 x + 196 x = 0 dt 2 (D 2 + 196) x = 0 Auxiliary equation is m2 + 196 = 0 m = ± 14i Hence, the general solution of Eq. (1) is x = c1 cos14t + c2 sin14t dx = −14c1 sin 14t + 14c2 cos 14t dt dx At t = 0, x = 20 cm = 0.2 m, v = = 0, 0.2 = c1 and 0 = 14c2, c2 = 0 dt

... (1)

Differential Equations

10.185

(i) Hence, displacement of the body from its equilibrium position at time t is given by x = 0·2 cos14t (ii) Amplitude = 20 cm = 0.2 m maximum velocity amplitude 14 0 2 2 8 m/sec (iii) Period of oscillation =

2 2 = = 0.45 sec 14 w

Free Damped Oscillation Example 2: A 3 lb weight on a spring stretches it to 6 inches. Suppose a damping force kv is present (k > 0). Show that the motion is (a) critically damped if k = 1.5 (b) overdamped if k > 1.5 (c) oscillatory if k < 1.5. Solution: A 3 lb weight stretches the spring to 6 inches, i.e.,

1 ft 2

1 2 k = 6 lb/ft 3

k

weight = 3 lb 3 3 mass = = g 32 damping force = lv =

dx dt

where l > 0

The equation of motion is m

d2 x dx + kx + =0 2 dt dt

3 2 D x + 6 x + Dx = 0 32 ⎛ 2 32 ⎞ D + 64 ⎟ x = 0 ⎜D + 3 ⎝ ⎠

where D =

d dt

Auxiliary equation is

m2 +

32 m + 64 = 0 3

... (1) 2

− m= =

32 ⎛ 32 ⎞ − 256 ± ⎜ ⎝ 3 ⎟⎠ 3 2

−32 ± 1024 6

2

− 2304

10.186

Engineering Mathematics

(a) The motion is critically damped when roots of Eq. (1) are equal, i.e., 1024 2 2304 0. l = 1·5. (b) The motion is overdamped when roots of Eq. (1) are real and distinct, i.e., 1024 2 2304 0 . > 1.5. (c) The motion is oscillatory when roots of Eq. (1) are imaginary, i.e., 1024 2 2304 0. l < 1.5.

Forced Undamped Oscillation Example 3: Determine whether resonance occurs in a system consisting of a weight 32 lb attached to a spring with constant k = 4 lb/ft and external force 16 sin 2t 1 dx and no damping force present. Initially x = and 4. 2 dt Solution: The equation of motion is d2 x + kx = 16sin 2t dt 2 32 d 2 x + 4 x = 16sin 2t g dt 2 m

⎡⎣∵ g = 32 ft / sec 2 ⎤⎦

(D 2 + 4) x = 16sin 2t Auxiliary equation is m2 + 4 = 0 m 2i C.F. = c1cos 2t + c2 sin 2 t 1 16sin 2t D +4 1 = 16t sin 2t 2D

P.I. =

2

8t sin 2t dt

8t

cos 2t 2

4t cos 2t Hence, general solution of Eq. (1) is x dx dt Initially at

t = 0, 1 = c1 2

c1 cos 2t c2 sin 2t 4t cos 2t 2c1 sin 2t 2c2 cos 2t 4 cos 2t 8t sin 2t x=

1 dx and dt 2

4

... (1)

Differential Equations

and Hence, x

4 2c2 c2 = 0

10.187

4

1 cos 2t 4t cos 2t 2 2

k 4 = m 1 w=2 =

Also, n = 2 Frequency of the external force = Natural frequency =

2

=

n 2 1 = = cycles /sec 2 2

2 1 = cycles/sec 2

Since both the frequencies are same, resonance occurs in the system.

Forced Damped Oscillations Example 4: Determine the transient and steady-state solutions of mechanical system with weight 6 lb, stiffness constant 12 lb/ft, damping force 1.5 times instan1 dx taneous velocity, external force 24 cos 8t and initial conditions x = f t , = 0. 3 dt Solution: Weight = 6 lb, k = 12 lb/ft m=

6 6 = g 32

dx dt The equation of motion is Damping force = 1.5

d2 x dx + kx + 1.5 = 24 cos 8t 2 dt dt 2 6 d x dx + 12 x + 1.5 = 24 cos 8t 32 dt 2 dt 2 d x dx + 8 + 64 x = 128cos 8t dt dt 2 2 (D + 8D + 64) x = 128cos 8t m

Auxiliary equation is m 2 + 8m + 64 = 0 m=

− 8 ± 64 − 256 = −4 ± i 4 3 2

C.F. = e 4t (c1 cos 4 3t + c2 sin 4 3t )

Engineering Mathematics

10.188

which gives the transient solution P.I.

1 1 128cos 8t 128 cos 8t 64 8D 64 D 8D 64 sin 8t = 16 cos 8t dt = 16 = 2sin 8t 8 2

P.I. = 2 sin 8t which gives the steady-state solution. Exercise 10.23 1. A body weighing 4·9 kg is hung from a spring. A pull of 10 kg will stretch the spring to 5 cm. The body is pulled down 6 cm below the static equilibrium position and then released. Find the displacement of the body from its equilibrium position at time t seconds, the maximum velocity and the period of oscillation. ⎡ Ans. : ⎤ ⎢ 0.06 cos 20 t , 1.2 m / sec , 0.314 sec ⎥ ⎣ ⎦ 2. A mass of 200 gm is tied at the end of a spring which extends to 4 cm under a force of 196, 000 dynes. The spring is pulled 5 cm and released. Find the displacement t seconds after released, if there be a damping force of 2000 dynes per cm per second. What should be the damping force for the dead beat motion? Ans. : e

5t

5cos 220t +

25 220

sin 220t ,

6261 3. A spring of negligible weight which stretches 1 inch under tension of 2 lb is fixed at one end and is attached to a weight of W lb at the other. It is found that resonance occurs when an axial

periodic force 2 cos 2t lb acts on the weight. Show that when the free vibrations have died out, the forced vibrations are given by x = ct sin 2t, and find the values of W and c. 1⎤ ⎡ ⎢⎣ Ans. :W = 6g , c = 12 ⎥⎦ 4. Find the steady-state and transient oscillations of the mechanical system corresponding to the differential equa.. . tion x 2x 2x = sin 2t 2 cos 2t, . x (0) x (0) 0. [Ans. : 0.5 sin 2t, e t sin t] 5. If weight W = 16 lb, spring constant dx k = 10 lb/ft, damping force 2 , dt external force F(t) is 5 cos 2t, find the motion of the weight given x(0) = x(0) = 0. Write the transient and steady-state solutions. 3 1 sin 4t cos 4t 8 2 1 1 cos 2t sin 2t 2 4 5e 2t Transient solution : cos (4t 0.64) 8 Ans.: x(t )

e

2t

Transient solution :

5 cos(2t 0.46) 4

Differential Equations

10.189

When a beam made up of fibres is bent the fibres of the upper half are compressed and of lower half are stretched. In between there is a region where the fibres are neither compressed nor stretched. This region is called the neutral surface of the beam and the curve of any particular fibre on neutral surface is called elastic curve or deflection curve of the beam. The line at which any plane section of the beam cuts the neutral surface is called neutral axis of that section. The bending moment M created by the forces acting above and below the neutral surface in opposite direction is M = where

EI R

E = modulus of elasticity of the beam I = moment of inertia of the section about neutral axis R = radius of curvature of elastic curve at any point P(x, y) 1+ =

dy dx

2

d2 y dx 2

3 2

, where y represent deflection at a distance x from one end.

Assuming deflection to be very small,

dy dx

2

can be neglected

1 d2 y dx 2 d2 y M = EI 2 dx R=

... (1)

which is the differential equation of the elastic curve.

Boundary Conditions The arbitrary constants in the solution of Eq. (1) can be found using following boundary conditions: d2 y =0 dx 2 dy =0 (ii) End fixed horizontally: y = 0, dx (i) End freely supported: y = 0,

(iii) End perfectly free:

d2 y d3 y = 0, =0 dx 2 dx 3

Example 1: A light horizontal strut AB is freely pinned at A and B. It is under the action of equal and opposite compressive forces P at its ends and it carries a

10.190

Engineering Mathematics

l 2

load W at its centre. Then for 0 < x < , EI

d2 y 1 + Py + Wx = 0. Also y = 0 at x = 0 2 2 dx

dy l W = 0 at x = . Prove that y = dx 2 2P

P sin nx - x where n2 = . nl EI n cos 2 Solution: The equation of action of forces is

and

1 d2 y + Py + Wx = 0 2 2 dx W d2 y P y+ + x=0 2 EI dx 2 EI EI

P d W = n 2 and D = x, where EI dx 2 EI n 2W (D 2 + n 2 ) y = − x 2P Auxiliary equation is m2 + n2 = 0 m = ±in C.F. = c1 cos nx + c2 sin nx (D 2 + n 2 ) y = −

P.I.

1 2 D + n2

n 2W x 2P

W D2 1 2P n2

n 2W 1 D2 1 2P n2 n2

D4 n4

... x

Wx 2P Hence, the general solution of Eq. (1) is y dy dx

Wx 2P

c1 cos nx c2 sin nx

c1n sin nx c2 n cos nx

At x = 0, y = 0 0 = c1 l dy =0 At x = , 2 dx

0 c2

c2 n cos

nl 2 1

W 2P

W 2 P n cos nl 2

W 2P

... (1)

1

x

10.191

Differential Equations

Hence,

y=

W ⎛ sin nx Wx ⎞ − 2 P ⎜ n cos nl 2 P ⎟ ⎜ ⎟ ⎝ ⎠ 2

where

n2 =

P . EI

Example 2: Find the equation of the elastic curve and its maximum deflection for the simply supported beam of length 2l, having uniformly distributed load W per unit length. Solution: Consider the segment AP = x. The forces acting on the segment AP are (i) The upward thrust W l at A (ii) The load Wx at the midpoint of AP. Moment M

Wlx W x

x 2

Wlx

W x2 2

... (1) B

x

A x 2

P Wx

Fig. 10.6 The equation of elastic curve is M = EI

d2 y dx 2

... (2)

Equating equations (1) and (2), EI

d2 y W x2 Wlx = − 2 dx 2

Integrating w.r.t. x, EI

dy x 2 W x3 = Wl − + c1 2 6 dx

Integrating again w.r.t. x, EI y =

Wlx 3 W x 4 − + c1 x + c2 6 24

Since ends A and B are freely supported, at A, x = 0, y = 0 and at B, x = 2l, y = 0 Putting in Eq. (3), 0 = c2 Wl W 0 (2l )3 (2l ) 4 c1 (2l ) and 6 24 Wl 3 c1 = 3

... (3)

10.192

Engineering Mathematics

Hence,

EI y =

Wlx 3 Wx 4 Wl 3 x − − 6 24 3 2 3 ⎛ Wx lx x l3 ⎞ y= − − EI ⎜⎝ 6 24 3 ⎟⎠

Deflection of the beam is max. at x = l (mid point) Wl l 3 l 3 l 3 ymax EI 6 24 3

5Wl 4 24 EI

Exercise 10.24 1. A horizontal tie-rod of length 2l with concentrated load W at the centre and ends freely hinged, satisfies the differential equation W d2 y EI 2 Py x. With conditions 2 dx dy = 0, prove x = 0, y = 0 and x = l, dx that the deflection and the bending moment M at the centre (x = l) are W given by (nl tanh nl ) and 2 Pn W M = − tanh nl where n2EI = P. 2n 2. The shape of a strut of length l subjected to an end thrust P and lateral load W per unit length, when the ends are built in, is given by d2 y W x 2 Wlx EI 2 + Py = − + M, 2 2 dx where M is the moment at a fixed end. Find y in terms of x, given that dy dy = 0 at y = 0, = 0 at x = 0 and dx dx l x= . 2

⎡ Wl nl nl ⎞ ⎤ ⎛ ⎢ Ans. : y = 2 Pn cosec 2 cos ⎜ nx − 2 ⎟ ⎥ ⎝ ⎠⎥ ⎢ ⎢ ⎥ Wl nl W 2 cot + ( x − lx) ⎥ − ⎢ 2 Pn 2 2P ⎣ ⎦ 3. A uniform horizontal strut of length l freely supported at both ends, carries a uniformly distributed load W per unit length. If the thrust at each end is P, prove that the maximum deflecW al Wl2 tion is where sec 1 2 8P Pa 2 P = a 2 . Prove also that the maxiEI mum bending moment is of magniW al tude 2 sec 1 . 2 a 4. A horizontal tie-rod is freely pined at each end. It carries a uniform load W per unit length and has a horizontal pull P. Find the central deflection and the maximum bending moment taking the origin at one of its ends. ⎡ W ⎛ al ⎞ Wl 2 ⎤ Ans. : sech − 1⎟ + ,⎥ ⎢ ⎜ 2 Pa 2 ⎝ ⎠ 8P ⎥ ⎢ ⎢ ⎥ W⎛ al ⎞ sech − 1⎟ ⎢ ⎥ 2 ⎜ 2 a ⎝ ⎠ ⎣ ⎦

10.193

Differential Equations

10.11.5 Electrical Circuit A second order electrical circuit consists of a resistor, an inductor and a capacitor in series with an emf e(t) as shown in the figure. Applying Kirchhoff ’s voltage law to the circuit Ri + L

di 1 + idt = e(t ) dt C ∫

i=

But

L

... (1)

dq dt

R

e(t)

dq 1 d2q + R + q = e(t ) dt C dt 2

L

i

... (2)

C

Fig. 10.7

Differentiating Eq. (1) w.r.t. t L

d 2i di i de(t ) +R + = dt C dt dt 2

... (3)

The Eqs. (2) and (3) are both second order linear non homogeneous ordinary differential equations. Example 1: A circuit consists of an inductance of 2 henrys, a resistance of 4 ohms and capacitance of 0.05 farads. If q = i = 0 at t = 0, (a) find q(t) and i(t) when there is a constant emf of 100 volts (b) Find steady state solutions. Solution: (a) The differential equation of the RLC circuit d2q dq q + R + = e(t ) 2 dt C dt d2q dq q = 100 2 2 +4 + dt 0.05 dt d2q dq + 2 + 10q = 50 2 dt dt (D 2 + 2D + 10)q = 50 L

Auxiliary equation is m 2 + 2m + 10 = 0 m 1 3i C.F. = e t (c1 cos 3t + c2 sin 3t ) 1 (50e0t ) D + 2D + 10 1 = ⋅ 50 = 5 10

P.I. =

2

Engineering Mathematics

10.194

General solution is q = e t (c1 cos 3t + c2 sin 3t ) + 5 t = 0,

At

... (1)

q=0 0 = c1 + 5 c1

5

Differentiating Eq. (1) w.r.t. t, i=

dq = e − t ( −3c1 sin 3t + 3c2 cos 3t ) − e − t (c1 cos 3t + c2 sin 3t ) dt

t = 0, i = 0

At

0

3c2

c1

3c2 = c1 c2 Hence, q (t ) and

i (t )

5 3

5 t e (3cos 3t sin 3t ) 3 5 sin 3t ) e t (15sin 3t 5cos 3t ) e t (5cos 3t 3 5 t 5 t e (3cos 3t 9sin 3t ) e (3cos 3t sin 3t ) 3 3

5 e t ( 5cos 3t

5 sin 3t ) 3

5

(b) The steady state solution is obtained by putting t q(t) = 5 i(t) = 0

.

Example 2: (a) Determine q and i in an RLC circuit with L = 0.5 H, R = 6 W, C = 0.02 F, e = 24 sin 10t and initial conditions q = i = 0 at t = 0. (b) Find steady state and transient solutions. Solution: The differential equation of the RLC circuit is d2q dq q +R + =e dt C dt 2 2 d q dq q = 24 sin 10t 0.5 2 + 6 + dt 0.02 dt d2q dq + 12 + 100q = 48 sin 10t dt dt 2 2 (D + 12D + 100)q = 48 sin 10t L

Auxiliary solution is m 2 + 12m + 100 = 0 m 6 8i C.F. = e 6t (c1 cos8t + c2 sin 8t )

Differential Equations

P.I. =

10.195

1 48sin10t D + 12D + 100 1 48 sin10t 102 12D 100 cos10t 2 4 cos10t 10 5 2

48 sin10t dt 12

General solution is e 6t (c1 cos8t c2 sin 8t )

q

2 cos10t 5

... (1)

Differentiating Eq. (1) w.r.t. t i

dq 2 6e 6t (c1 cos8t c2 sin 8t ) e 6t ( 8c1 sin 8t 8c2 cos8t ) 10sin10t dt 5 e 6t [( 6c1 8c2 ) cos8t (6c2 8c1 ) sin 8t ] 4sin10t

At t = 0, q = 0, i = 0 0

c1

c1 =

2 5

2 5

and

0 c2 =

6c1 8c2 6c1 3 = 8 10 6t

2 3 cos8t sin 8t 5 10

Hence,

q (t )

e

and

i (t )

e 6t ( 5sin 8t ) 4sin10t

The steady state solution is obtained by putting t

.

2 cos10t 5 i (t ) = 4sin10t

q (t )

The transient solution q (t ) = e i (t )

6t

2 3 cos8t + sin 8t 5 10

e 6t ( 5sin 8t )

2 cos10t 5

10.196

Engineering Mathematics

Exercise 10.25 1. A circuit consists of resistance of 5 ohms, inductance of 0.05 henrys and capacitance of 4 10-4 farads. If q(0) = 0, i(0) = 0, find q(t) and i(t) when an emf of 110 volts is applied. ⎤ ⎡ 11 −50 t ⎛ ⎢ Ans. : q (t ) = e ⎜⎝ − 250 cos 50 19t ⎥ ⎥ ⎢ ⎥ ⎢ ⎞ 11 11 19 sin 50 19t ⎟ + , ⎥ − ⎢ ⎠ 250 ⎥ 4750 ⎢ ⎥ ⎢ 44 −50t i (t ) = e sin 50 19t ⎥ ⎢ 19 ⎦ ⎣ 2. Determine the charge on the capacitor at any time t in series circuit having a resistor of 2 , inductor of

1 F and 260 e(t) = 100 sin 60t. If the initial current 0.1 H capacitor of

and initial charge on capacitor are both zero, find steady state solution. ⎡ ⎤ 6e −10t Ans. : q ( t ) = (6 sin 50t ⎢ ⎥ 61 ⎢ ⎥ 5 ⎢ ⎥ ⎢ + 5 cos 50t ) − 61 (5 sin 60t ⎥ ⎢ ⎥ ⎢ + 6 cos 60t ), Stteady state solution:⎥ ⎢ ⎥ ⎢ q (t ) = − 5 (5 sin 60t + 6 cos 60t ). ⎥ 61 ⎣ ⎦

FORMULAE First Order Differential Equation Sr. No. 1.

3.

4.

5.

M (x, y) dx + N (x, y) dy = 0

M (x, y) dx + N (x, y) dy = 0

M (x, y) dx + N (x, y) dy = 0

f1 (xy) y dx + f2 (xy) x dy = 0,

M (x, y) dx + N (x, y) dy = 0

Type

Integrating Factor

Exact i.e. N M = x y

(i)

M (x, y) dx + (terms of N not containing x) dy = c

(ii)

(terms of M not containing y) dx + N (x, y) dy = c

(i)

M1 (x, y) dx + (terms of N1 not containing x) dy = c

(ii)

(terms of M1 not containing y) dx + N1 (x, y) dy = c

Non-Exact I.F. = e (i) and (ii) ∂N − ∂M ∂x ∂y = f (y) M 1 Non-Exact I.F. = , (i) Mx Ny (ii)

M1 (x, y) dx + (terms of N1 not containing x) dy = c

Non-Exact and ∂M − ∂N ∂y ∂x = f (x), N

-

Solution

I.F. = e

f ( x ) dx

f ( y ) dy

Non-Exact

1 Mx + Ny

M1 (x, y) dx + (terms of N1 not containing x) dy = c (terms of M1 not containing y) dx + N1 (x, y) dy = c

(i)

M1 (x, y) dx + (terms of N1 not containing x) dy = c

(ii)

(terms of M1 not containing y) dx + N1 (x, y) dy = c

10.197

and Homogeneous

I.F. =

(terms of M1 not containing y) dx + N1 (x, y) dy = c

Differential Equations

2.

Differential Equation

6.

Differential Equation x m1 y n1 (a1y dx + b1x dy)

Type Non-Exact

9.

dy + Py = Qy n dx

f (y)

dy + Pf (y) = Q dx

(i)

where

P dx

Linear in y

I.F. = e

Non linear

I.F. = e where P1 = (1 – n) v and v = y 1–n

Non linear

P1 dx

I.F. = e

P dx

M1 (x, y) dx + (terms of N1 not containing x) dy = c (terms of M1 not containing y) dx + N1 (x, y) dy = c

ye

ve

P dx

P1 dx

∫ P dx dx + c

=

∫ Qe

=

Q1e

P1 dx

dx + c where

Q1 = (1 – n)Q ve

P dx

=

∫ Qe

∫ P dx dx + c where f (y) = v

Note: In the cases 1 to 6 after multiplication by I.F., differential equation reduces to M1(x, y) dx + N1 (x, y) dy = 0

Engineering Mathematics

8.

dy + Py = Q, where P and Q dx are functions of x

I.F. = xh yk

Solution

m1 + h + 1 (ii) a1 n + k +1 = 1 b1 m2 + h + 1 and a2 n2 + k + 1 = b2

+ x m2 y n2 (a2y dx + b2x dy) = 0

7.

Integrating Factor

10.198

Sr. No.

Higher Order Differential Equations Homogeneous Linear Differential Equations with constant coefficients Sr. No.

Roots

Complimentary Function(C.F.)

1.

Real and distinct roots (m1, m2, … mn)

y = c1e m1 x + c2 e m2 x + c2 e m3 x + … + cn e mn x

2.

Real and repeated roots (m1 = m2)

y = (c1 + c2 x) e m1 x + c3 e m3 x + c4 e m4 x + … + cn e mn x

3.

Imaginary roots (m1 = a + ib, m2 = a – ib) y = ea x (c1 cos b x + c2 sin b x) + c3 e m3 x + c4 e m4 x + … + cn e mn x

4.

Imaginary and repeated roots

y = ea x [(c1 + c2x) cos b x + (c3 + c4x) sin b x] + c5 e m5 x + c6 e m6 x + … + cn e mn x

Non-Homogeneous Linear Differential Equations with constant coefficients Sr. No.

Q(x) e ax + b

2.

sin (ax + b) or cos (ax + b)

3.

xm

4.

eax V

5.

xV

1 ax + b e if f (a) 0 f (a) (ii) xr ( r1) eax + b if f (r – 1) (a) = 0 and f (r) (a) 0 f (a) 1 sin (ax + b) or 1 cos (ax + b) if f (– a2) 0 (i) 2 f( a ) f ( a2) (ii) x r ( r ) 1 2 cos (ax + b), if f (r – 1) (– a2) = 0 and f (r) (– a2) 0 f ( a) (i)

[f (D)]–1 xm = [1 + f (D)]–1 xm = (a0 + a1D + a2 D2 + … + am Dm) xm 1 eax V f ( D + a) f ′( D) x 1 V− V f ( D) [ f ( D)]2

10.199

1.

Particular Integral (P.I.)

Differential Equations

(m1 = m2 = a + ib, m3 = m4 = a – ib)

Engineering Mathematics

10.200

If Q(x) is not in any of the above 5 forms, then the solution of the differential equation can be obtained by the following methods: (i) f (D) is factorized as linear factors of D and P.I. is obtained using the formula 1 Q(x) = eax Q(x)e–ax dx D a (ii) Variation of parameters: If C.F. = c1 y1 + c2 y2, assume P.I. = y = v1(x) y1 + v2 (x) y2 y1 y2 −y Q y1Q where v1 = ∫ 2 dx , v2 = dx and W = y1 y2 W W MULTIPLE CHOICE QUESTIONS Choose the correct alternative in each of the following: 1. The order of the differential equation whose general solution is given by y = (c1 + c2)cos (x + c3) – c4 e x + c5 , where c1, c2, c3, c4, c5 are arbitrary constants is (a) 5 (b) 4 (c) 3 (d) 2 2 2. The value of n so that e n y is an integrating factor of the differential y2 2

equation (e xy ) dy – dx = 0 is (a) –1 (b) 1 1 1 (c) (d) 2 2 3. The solution of the differential equady tion y = x – 1 satisfying y (1) = 1 dx is (a) y2 = x2 – 2x + 2 (b) y2 = 2x2 – x – 1 (c) y = x2 – 2x + 2 (d) none of these 4. The order of the differential equation 3 ⎛ dy ⎞ d2 y 4 –t + ⎝ dt ⎠ + y = e is dt 2 (a) 1 (b) 2 (c) 3 (d) none of above 5. Match each differential equation in Group I to its family of solution curves from Group II.

Group I Group II dy y (P) = 1. Circles dx x dy y (Q) = 2. Straight lines dx x dy x (R) = 3. Hyperbolas dx y dy x (S) = dx y (a) P-2, Q-3, R-3, S-1 (b) P-1, Q-3, R-2, S-1 (c) P-2, Q-1, R-3, S-3 (d) P-3, Q-2, R-1, S-2 6. Which of the following is a solution dx to the differential equation + 3x = 0? dt –t (a) x = 3e (b) x = 2e–3t 3 2 (c) x = t (d) x = 3t2 2 7. The solution of the differential equad2 y tion k 2 2 = y – y2 under the dx boundary conditions (i) y = y1 at x = 0 and (ii) y = y2 at x = , where k, y1 and y2 are constants, is ⎛−x⎞ ⎜ 2 ⎠⎟

(a) y = (y1 – y2) e ⎝ k

+ y2

(b) y = (y2 – y1) e

+ y1

⎛−x⎞ ⎜⎝ ⎟ k ⎠

Differential Equations

(c) y = (y1 – y2) sinh

( kx ) + y

⎛−x⎞ ⎜ ⎟ k ⎠

(d) y = (y1 – y2) e ⎝

1

+ y2

8. A solution of the following differential equation is given by dy d2 y –5 + 6y = 0 2 dx dx (a) y = e2x + e – 3x (b) y = e2x + e 3x (c) y = e–2x + e3x (d) y = e–2x + e–3x 9. The solution of the differential equa2 dy tion + 2xy = e x with y (0) = 1 is dx 2 2 (a) (1 + x)e x (b) (1 + x) e − x x) e x

(c) (1

2

(d) (1

x) e

x2

d2 y dy +4 + 3y = 3e2x, the 2 dx dx particular integral is 1 2x 1 2x (a) e (b) e 15 5 (c) 3e 2x (d) c1e –x + c2e–3x

10. For

11. The solution of the differential equady tion + y2 = 0 is dx x3 (a) y = 1 (b) y = +c 3 x+c x

(c) c e (d) unsolvable as equation is nonlinear 12. The differential equation d2 y dy + + sin y = 0 is 2 dx dx (a) linear (b) non-linear (c) homogeneous(d) of degree 2 13. The particular solution for the differential equation d2 y dy +3 + 2y = 5 cos x is 2 dx dx (a) 0.5 cos x + 1.5 sin x

10.201

(b) 1.5 cox x + 0.5 sin x (c) 1.5 sin x (d) 0.5 cos x 14. The general solution of the differend2 y dy tial equation x2 2 – x + y = 0 is dx dx (a) Ax + Bx 2 (b) Ax + B log x (c) Ax + Bx2 log x (d) Ax + Bx log x where A and B are constants dy 2 log x 15. If x2 + 2xy = and y (1) = 0, dx x then what is y (e)? (a) e (b) 1 1 (c) (d) 12 e e 16. The family of conics represented by the solution of differential equation (4x + 3y + 1) dx + (3x + 2y + 1)dy = 0 is (a) circles (b) parabolas (c) hyperbolas (d) ellipses 17. Which one of the following does not satisfy the differential equation d3 y – y = 0? dx 3 (a) ex (b) e–x (c) e

x 2

⎛ 3 ⎞ x⎟ sin ⎜⎝ 2 ⎠

⎛ 3 ⎞ x⎟ cos ⎜⎝ 2 ⎠ 18. The orthogonal trajectory of family of curve y = ax2 is (a) x2 + 2y2 = c (b) x2 + y2 = c (c) x2 – y2 = c (d) 2x2 + y2 = c 19. If e –x and x e –x are the fundamental d2 y dy solution of +a + y = 0, the 2 dx dx value of a is (d) e

x 2

Engineering Mathematics

10.202

(a) 1 (c) 2

(b) 3 (d) 4

dy xy 2 y = into dx x exact equation the differential equation becomes xy 1 (a) dx – 1 2 dy = 0 x2 y xy

20. On conversion

(b)

x 1 dx – 1 dy = 0 xy xy

(c) 1 dx – 1 dy = 0 x y (d) None of above 21. The orthogonal trajectories of the cardioid r = k (1 – cos q), where k is a parameter, is (a) r = c (1 + cos q) (b) r = c (1 – sin q) (c) r (1 + cos q) = c (d) r (1 – sin q) = c d and z = log x, then the 22. If D dz differential equation d2 y dy x 2 +2 = 6x becomes dx dx (a) D (D – 1) y = 6ez (b) D (D – 1) y = 6e2z (c) D (D + 1) y = 6e2z (d) D (D + 1) y = 6ez 23. The solution of the equation d2 y – y = k (k = a non zero constant) dx 2 which vanishes when x = 0 and which tends to a finite limit as x tends to infinity, is Answers 1. (c) 8. (b) 15. (d) 22. (c)

2. 9. 16. 23.

(c) (a) (c) (b)

3. 10. 17. 24.

(a) (b) (b) (b)

4. 11. 18. 25.

(a) y = k (1 + e–x) (b) y = k (e–x – 1) (c) y = k (e x + e–x – 2) (d) y = k (e x – 1) 24. If the rate of growth is proportional to the amount x of the substance present and dx = kx, then x is equal dt to (with c1 constant) (a) c1e –kt (b) c1e kt –2kt (c) c1e (d) c1e 2kt 25. The solution of differential equation ⎛ y⎞ f⎝ ⎠ dy y x = + is dx x ⎛ y⎞ f′⎝ ⎠ x ⎛ y⎞ (a) f ⎝ ⎠ = k x x ⎛ y⎞ (b) f ⎝ ⎠ = k y x ⎛ y⎞ (c) x f ⎝ ⎠ = k x ⎛ y⎞ (d) y f ⎝ ⎠ = k x for some constant k. 26. m = 2 is a double root and m = –1 is another root of the auxiliary equation of a homogeneous differential with constant coefficient. The differential equation is (a) (D3 + 3D2 + 4)y = 0 (b) (D3 + 3D2 – 4)y = 0 (c) (D3 – 3D2 + 4)y = 0 (d) (D3 – 3D2 – 4)y = 0

(b) (a) (a) (a)

5. 12. 19. 26.

(a) (b) (c) (c)

6. (b) 13. (a) 20. (a)

7. (d) 14. (d) 21. (a)

Matrices

11.1

Matrices

O

Chapter

11

11.1 INTRODUCTION A matrix is a rectangular table of elements which may be numbers or any abstract quantities that can be added and multiplied. Matrices are used to describe linear equations, keep track of the coefficients of linear transformation and to record data that depend on multiple parameters. There are many applications of matrices, viz., in mathematics, graph theory, probability theory, statistics, computer graphics, geometrical optics, etc.

11.2 MATRIX A set of mn elements (real or complex) arranged in a rectangular array of m rows and n columns enclosed by a pair of square brackets is called a matrix of order m by n, written as m n. A m n matrix is usually written as a11 a12 ... a1n a21 a22 ... a2 n A = ... ...

... ...

am1

am 2

... ...

... ...

... amn

m n

The matrix can also be expressed in the form A = [aij]m n , where aij is the element in the ith row and jth column, written as (i, j)th element of the matrix.

11.3 SOME DEFINITIONS ASSOCIATED WITH MATRICES (1) Row Matrix A matrix having only one row and any number of columns, is called a row matrix or row vector, e.g., A = [2 5 3 4]

11.2

Engineering Mathematics

(2) Column Matrix A matrix, having only one column and any number of rows, is called a column matrix or column vector, e.g., 1 A 3 4

(3) Zero or Null Matrix A matrix, whose all the elements are zero, is called zero or null matrix and is denoted by O, e.g., ⎡ 0 0 0⎤ ⎡0 0⎤ ⎢ ⎥ , ⎢ 0 0⎥ ⎢ 0 0 0⎥ ⎦ ⎢ ⎣ ⎣0 0 0⎥⎦

(4) Square Matrix A matrix, in which the number of rows is equal to the number of columns, is called a square matrix, e.g., ⎡ 1 3 2⎤ ⎡ 2 3⎤ ⎢ ⎥ ⎢ 1 4⎥ , ⎢ −1 4 −5⎥ ⎦ ⎢ ⎣ ⎣ 2 6 8⎦⎥

(5) Diagonal Matrix A square matrix, whose all non-diagonal elements are zero and at least one diagonal element is non-zero, is called a diagonal matrix. e.g., ⎡ 1 0 0⎤ ⎡ 2 0⎤ ⎢ ⎥ , ⎢ 0 4⎥ ⎢0 4 0⎥ ⎦ ⎢ ⎣ ⎣0 0 8⎥⎦

(6) Unit or Identity Matrix A diagonal matrix, whose all diagonal elements matrix and is denoted by I, e.g., ⎡1 0 ⎡ 1 0⎤ ⎢ ⎢0 1⎥ , ⎢0 1 ⎦ ⎢ ⎣ ⎣0 0

are unity, is called a unit or identity 0⎤ 0⎥⎥ 1⎥⎦

(7) Scalar Matrix A square matrix, whose all diagonal elements are equal, is called a scalar matrix, e.g., ⎡ 2 0 0⎤ ⎡ 3 0⎤ ⎢ ⎥ ⎢0 3⎥ , ⎢ 0 2 0⎥ ⎦ ⎢ ⎣ ⎣ 0 0 2⎦⎥

Matrices

11.3

(8) Upper Triangular Matrix A square matrix, in which all the elements below the diagonal are zero, is called an upper triangular matrix, e.g., 1 3 2 0 4 5 0 0 8

(9) Lower Triangular Matrix A square matrix, in which all the elements lower triangular matrix, e.g., 1 0 1 4 2 6

above the diagonal are zero, is called a 0 0 8

(10) Trace of a Matrix The sum of all the diagonal elements of a square matrix is called the trace of a matrix, e.g.,

⎡ 2 −1 0⎤ A = ⎢⎢ 4 6 −2⎥⎥ ⎢⎣ −1 0 3⎥⎦ Trace of A = 2 + 6 + 3 = 11

(11) Transpose of a Matrix A matrix obtained by interchanging rows and columns of a matrix is called transpose of a matrix and is denoted by A or AT, e.g., A

1 0

1 3 2 6

4

1 5

i.e., if A = [aij]m n , then AT = [aji]n

T

A

1 0 1 2

4 1

3 6

5

m

(12) Symmetric Matrix A square matrix A = [aij]m m is called symmetric if aij = aji for all i and j, i.e., A = AT, e.g., 2 4 , 4 3

1 i 3i

i 2 4

3i 4 3

(13) Skew Symmetric Matrix A square matrix A = [aij]m m is called skew symmetric if aij = aji for all i and j, i.e., A = AT. Thus, the diagonal elements of a skew symmetric matrix are all zero, e.g.,

11.4

Engineering Mathematics

0 3i 4

3i 0 8

4 8 0

(14) Conjugate of a Matrix A matrix obtained from any given matrix A, on replacing its elements by the corresponding conjugate complex numbers is called the conjugate of A and is denoted by A , e.g., A=

1 3i 2 5i 8 , i 6 9 i

A=

1 3i 2 5i 8 i 6 9 i

(15) Transposed Conjugate of a Matrix The conjugate of the transpose of a matrix A is called the conjugate transpose or transposed conjugate of A and is denoted by A , e.g., Aq = ( A)T = ( AT )

e.g., If

Then,

⎡1 − 2i 2 + 3i 3 + 4i ⎤ A = ⎢⎢ 4 − 5i 5 + 6i 6 − 7i ⎥⎥ , ⎢⎣ 8 7 + 8i 7 ⎥⎦

8 ⎤ ⎡1 − 2i 4 − 5i ⎢ A = ⎢ 2 + 3i 5 + 6i 7 + 8i ⎥⎥ ⎢⎣3 + 4i 6 − 7i 7 ⎥⎦ T

8 ⎤ ⎡1 + 2i 4 + 5i A = ⎢⎢ 2 − 3i 5 − 6i 7 − 8i ⎥⎥ ⎢⎣3 − 4i 6 + 7i 7 ⎥⎦

(16) Hermitian Matrix ___

A square matrix A = [aij] is called Hermitian if aij = a ji for all i and j, i.e., A = A , e.g., 2 + 3i 3 − 4i ⎤ ⎡ 1 ⎢ 2 − 3i 0 2 − 7i ⎥⎥ ⎢ ⎢⎣3 + 4i 2 + 7i 2 ⎥⎦

(17) Skew Hermitian Matrix ___

A square matrix A = [aij] is called skew-Hermitian if aij A

a ji for all i and j, i.e.,

Aq . Hence, diagonal elements of a skew Hermitian matrix must be either purely

imaginary or zero, e.g., 2 + 3i 2 3i 0 i

Matrices

11.5

(18) Orthogonal Matrix A square matrix A is called orthogonal if AAT = I. Two matrices A1 and A2 are orthogonal if A1T A2 = A1 A2T = O (For real matrix) A1 A2 = A1 A2 = O

and

(For complex mattrix)

(19) Unitary Matrix A square matrix A is called unitary if AAq = I .

(20) Determinant of a Matrix If A is a square matrix, then determinant of A is represented as | A |. 2 3 1 A= 1 2 3 ,

if

2 3 1 then | A |= 1 2 3

1 1 0

1 1 0

(21) Singular and Non-Singular Matrices A square matrix A is called singular if | A | = 0 and non-singular if | A | 0.

(22) Minor of an Element of a Determinant a11 If | A |= a21 a31

a12 a22 a32

a13 a23 , then minor of an element of a determinant is a determinant a33

obtained by leaving the row and column passing through the element, e.g., minor of element a11 =

a22 a32

a21 a23 , minor of element a12 = a31 a33

minor of element a13 =

a21 a31

a22 a32

a23 , a33

(23) Cofactor of an Element of a Determinant Cofactor of an element aij of a determinant is the minor multiplied by ( 1)i+j, e.g., Cofactor of the element a11

( 1)1+1

a22 a32

a23 a33

Cofactor of the element a12

( 1)1+ 2

a21 a31

a23 a33

Cofactor of the element a13

( 1)1+ 3

a21 a31

a22 a32

11.6

Engineering Mathematics

(24) Inverse of a Matrix If A is a square matrix and A

0 , then AA 1 = I = A 1 A where, A 1 is called inverse of the matrix A.

Example 1: Show that every square matrix can be uniquely expressed as the sum of a symmetric matrix and a skew symmetric matrix. Solution: Let A be a square matrix. 1( 1( A AT ) A AT ) P Q 2 2 1 where, P = ( A + AT ) 2 1( and Q A AT ) 2 T 1 1 T ( T )T Now, PT = ( A + AT ) = A + A 2 2 1 = ( AT + A) = P 2 Hence, P is a symmetric matrix. T 1( 1 T ( T )T Also, QT A AT ) A A 2 2 1( T 1( A A) A AT ) = Q 2 2 Hence, Q is a skew symmetric matrix. Thus, every matrix can be expressed as the sum of a symmetric matrix and a skew symmetric matrix. Uniqueness Let A = R + S, where R is a symmetric and S is a skew symmetric matrix. A

AT = ( R + S )T = RT + ST = R

S

Now,

1( A AT ) 2

1 [( R S ) ( R S )] 2

R

P

and

1( A AT ) 2

1 [( R S ) ( R S )] 2

S

Q

Hence, representation A = P + Q is unique.

Example 2: Express the matrix A and a skew symmetric matrix.

1 1 8

5 2 2

7 4 as the sum of a symmetric 13

Matrices

Solution:

Let P =

5 2 2

7 4 , 13

1 5 7

T

A

1 8 2 2 4 13

1( A + AT ) 2 5 2 2

7 4 13

1 5 7

1 8 2 2 4 13

1 2

1( A AT ) 2 1 5 1 1 2 2 8 2

7 4 13

1 5 7

1 8 2 2 4 13

1 2

1 2 Q

A

1 1 8

11.7

1 1 8

2 4 15

4 15 4 2 2 26

0 6 6 0 1 6

1 6 0

We know that P is a symmetric and Q is a skew symmetric matrix. A=P+Q

1 2

2 4 15

4 4

15 2

2 26

1 2

0 6 6 0

1 6

1 6

0

Example 3: Show that every square matrix can be uniquely expressed as the sum of a Hermitian matrix and a skew Hermitian matrix. Solution: Let A be a square matrix. 1( A A ) 2 1 where, P = (A+ A ) 2 1( and Q A A ) 2 1 Now, P = (A+ A ) 2 1( = A + A) 2 Hence, P is a Hermitian matrix. 1( Also, Q A A ) 2 1( A A) 2 Hence, Q is a skew Hermitian matrix. A

1( A A 2

=

) P Q

1 A +(A 2

)

=P 1 A 2 = Q

(A )

11.8

Engineering Mathematics

Thus, every square matrix can be expressed as the sum of a Hermitian matrix and a skew Hermitian matrix. Uniqueness Let A = R + S where R is a Hermitian and S is a skew Hermitian matrix. Aq = (R + S)q = Rq + Sq = R S 1( 1 Now, A A ) [ ( R S ) ( R S )] R P 2 2 1( 1 and A A ) [ ( R S ) ( R S )] S Q 2 2 Hence, representation A = P + Q is unique. 2 + 3i Example 4: Express the matrix A = 5 1 i Hermitian and a skew Hermitian matrix.

Solution:

A=

1 Let P = ( A + A 2 1 2

=

1 2

)

4 5

5 0

6i 5 1 3i

)

0 i

4i 8

1 i

3 i

6

2 3i 0

5 i

1 i 3 i

4i

8

6

as the sum of a

2 3i 5

0 i

4i 8

2 3i 0

5 i

1 i 3 i

1 i

3 i

6

4i

8

6

2 3i 5

0 i

4i 8

2 3i 0

5 i

1 i 3 i

1 i

3 i

6

4i

8

6

1 + 5i 5 i

1 5i 5 i

1( A A 2

Q

1 2

2 + 3i 5

( A )T

A

0 4i i 8 3 i 6

1 2 5 2i 11 i

12

1 3i 11 + i 0

We know that P is a Hermitian and Q is a skew Hermitian matrix. 4 5 1 5i 6i 5 1 3i 1 1 A=P+Q 5 0 5 i 5 2i 11 i 2 2 1 5i 5 i 12 1 3i 11 i 0

Matrices

11.9

Example 5: Show that every square matrix can be uniquely expressed as P + iQ where P and Q are Hermitian matrices. Solution: Let A be a square matrix. A

1( A A 2

) i 1 ( A A ) P iQ 2i

1( 1( A + A ) and Q A A 2 2i 1 1 P = (A+ A ) = A +(A ) Now, 2 2 1 = ( A + A) = P 2 Hence, P is a Hermitian matrix. where,

P=

)

1( 1 A A ) A (A ) 2i 2i 1( 1( A A) A A ) 2i 2i =Q Hence, Q is a Hermitian matrix. Thus, every square matrix can be expressed as P + iQ where P and Q are Hermitian matrices. Uniqueness Let A = R + iS where R and S are Hermitian matrices. Aq = (R + iS)q = Rq + (iS)q = R iS 1( 1 A A ) Now, [ ( R iS ) ( R iS )] R P 2 2 1( 1 and A A ) [ ( R iS ) ( R iS )] iS iQ 2 2 Hence, representation A = P + iQ is unique. Also,

Q

Example 6: Express the matrix A =

2i 0

3 2 + 3i

3i

3 2i 2 5i

Q are both Hermitian.

Solution:

2i 3 1 i A = 0 2 + 3i 1 + i 3i 3 2i 2 5i A

2i 0 3i 3 2 3i 3 2i 1 i 1 i 2 5i

1 i 1+ i

as P + iQ where P and

11.10

Engineering Mathematics

1 Let P = ( A + A 2 1 2

Q

)

2i 3 1 i 0 2 3i 1 i 3i 3 2i 2 5i

1 2

2i 0 3i 3 2 3i 3 2i 1 i 1 i 2 5i

0 3 1 2i 3 4 4 i 1 2i 4 i 4

1( A A 2i

)

2i 3 1 i 0 2 3i 1 i 3i 3 2i 2 5i

1 2i

2i 0 3i 3 2 3i 3 2i 1 i 1 i 2 5i

1 − 4i ⎤ −3 ⎡ 4i 1⎢ 6i = ⎢ 3 −2 + 3i ⎥⎥ 2i ⎢⎣ −1 − 4i 2 + 3i −10i ⎥⎦ We know that P and Q are Hermitian matrices. −3 1 − 4i ⎤ −3 1 + 2i ⎤ ⎡ 4i ⎡ 0 1⎢ 1⎢ ⎥ 4 4−i ⎥+ ⎢ 3 6i −2 + 3i ⎥⎥ A = P + iQ = ⎢ −3 2 2 ⎢⎣ −1 − 4i 2 + 3i −10i ⎥⎦ ⎢⎣1 − 2i 4 + i 4 ⎥⎦ Example 7: Prove that every Hermitian matrix can be written as P + iQ where P is a real symmetric and Q is a real skew symmetric matrix. Solution: Let A be a Hermitian matrix. A where, P =

Aq = A 1( 1 A A) i ( A A) 2 2i

1 ( A + A) and Q 2

P iQ

1 ( A A) are real matrices. 2i T

Now,

T 1( 1 A + A) = [ A + A] 2 2 T T T T T⎤ 1 1⎡ = ( A ) + A = ⎣{( A ) } + ( A ) ⎦ 2 2

PT =

1 1( A + A ) = ( A + A) 2 2 =P Hence, P is a real symmetric matrix. =

Also,

QT

1 ( A A) 2i

T

1 [A 2i

T

A]

Matrices

1 2i

( A )T A

T

=

11.11

{

T 1⎡ ⎣ ( A) 2i

}

T

T⎤ − ( A) ⎦

1( A A 2i

)

1( 1( A A) A A) Q 2i 2i Hence, Q is a real skew symmetric matrix. Thus, every Hermitian matrix can be written as P + iQ, where P is a real symmetric and Q is a real skew symmetric matrix. −i 1+ i ⎤ ⎡ 1 Example 8: Express the Hermitian matrix A = ⎢⎢ i 0 2 − 3i ⎥⎥ as P + iQ ⎢⎣1 − i 2 + 3i 2 ⎥⎦ where P is a real symmetric and Q is a real skew symmetric matrix. 1+ i ⎤ −i ⎡ 1 ⎢ 0 2 A= ⎢ i − 3i ⎥⎥ ⎢⎣1 − i 2 + 3i 2 ⎥⎦

Solution:

1 i

A

i 0

1 i 2 3i 1 Let P = ( A + A ) 2

1 2

i

1 i 0 2 3i 1 i 2 3i 2 1 i

1 i 2 3i 2 i

1 i 0 2 3i 1 i 2 3i 2 1 i

2 0 2 1 0 1 1 = 0 0 4 = 0 0 2 2 2 4 4 1 2 2

Q

1( A A) 2i

1 2i

0 2i 2i

1 2i 2i 0 6i

i

1 i 0 2 3i 1 i 2 3i 2 1 i

2i 6i 0

0 1 1

1 0 3

i

1 i 0 2 3i 1 i 2 3i 2 1 i

1 3 0

We know that P is a real symmetric and Q is a real skew symmetric matrix. A = P +iQ

1 0 1 0 0 2 1 2 2

0 i i

i 0 3i

i 3i 0

11.12

Engineering Mathematics

Example 9: Prove that every skew Hermitian matrix can be written as P + iQ where P is a real skew symmetric and Q is a real symmetric matrix. Solution: Let A be a skew Hermitian matrix. Aq = A 1 1 A = ( A + A ) + i ( A − A ) = P + iQ 2 2i 1 1 ( A A) are real matrices. where, P = ( A + A ) and Q 2 2i T

Now,

PT

1( 1 [ A A A) 2 2 T 1 1 ( A )T A 2 2

T

A]

{( A) } T

T

( A )T

1( 1( A A ) A A) 2 2 1( A A) = P 2 Hence, P is a real skew symmetric matrix. QT

1( A A) 2i

T

1 [ A 2i

T

A]

{

}

T T 1 ( A)T A = 1 ⎡⎣ − ( A)T − ( A)T ⎤⎦ 2i 2i 1( 1 ( A A) A A ) 2i 2i 1( A A) = Q 2i Hence, Q is a real symmetric matrix. Thus, every skew Hermitian matrix can be written as P +iQ where P is a real skew symmetric and Q is a real symmetric matrix.

Example 10: Express the skew Hermitian matrix A

2i 2 i 1 i 2 i i 3i 1 i 3i 0

P + iQ, where P is a real skew symmetric and Q is a real symmetric matrix.

Solution:

2 + i 1 − i⎤ ⎡ 2i ⎢ 3i ⎥⎥ A = ⎢ −2 + i −i ⎢⎣ −1 − i 3i 0 ⎥⎦

as

Matrices

2i 2 i 1 i 2 i i 3i 1 i 3i 0

A

1 Let P = ( A + A ) 2 1 2

Q

1 2

1 ( A A) 2i 1 2i

2i 2 i 1 i i 2 i 3i 1 i 3i 0

0 4 2 4 0 0 2 0 0

11.13

2i 2 i 1 i i 2 i 3i 1 i 3i 0

0 2 1 2 0 0 1 0 0 2i 2 i 1 i 2 i i 3i 1 i 3i 0

1 2i

4i 2i

2i 2i

2i 6i

2 1

1 1

1 3

2i

6i

0

1

3

0

2i 2 i 1 i 2 i i 3i 1 i 3i 0

We know that P is a real skew symmetric and Q is a real symmetric matrix. A = P + iQ

0 2 1 2 0 0

2i i

i i

i 3i

1 0 0

i

3i

0

Example 11: Prove that matrix A is unitary and hence find A-1. 1 i 2 (i) A = 1 i 2

Solution: (i)

1 i 2 1 i 2 ⎡1 + i ⎢ 2 A= ⎢ ⎢1 + i ⎢⎣ 2 ⎡ 1+ i ⎢ 2 AT = ⎢ ⎢ −1 + i ⎢⎣ 2 ⎡ 1− i ⎢ A =⎢ 2 ⎢ −1 − i ⎢⎣ 2

2 (ii) A

−1 + i ⎤ 2 ⎥ ⎥ 1− i ⎥ 2 ⎥⎦ 1+ i ⎤ 2 ⎥ ⎥ 1− i ⎥ 2 ⎥⎦ 1− i ⎤ 2 ⎥ ⎥ 1+ i ⎥ 2 ⎥⎦

i 2 0

i 2 0 2 0

0 . 2

11.14

Engineering Mathematics

⎡1 + i ⎢ AA = ⎢ 2 ⎢1 + i ⎢⎣ 2 =

−1 + i ⎤ ⎡ 1 − i 2 ⎥⎢ 2 ⎥⎢ 1 − i ⎥ ⎢ −1 − i 2 ⎥⎦ ⎢⎣ 2

1− i ⎤ 2 ⎥ ⎥ 1+ i ⎥ 2 ⎥⎦

1 1 i2 i2 1 1 i2 i2 1 1 4 0 = 2 2 2 2 4 1 i 1 i 1 i 1 i 4 0 4

=

1 0 0 1

=I Hence, A is a unitary matrix. Since AAq = Aq A = I and AA–1 = A–1A = I, A–1 = Aq for unitary matrix. ⎡ 1− i 1− i ⎤ ⎢ 2 ⎥ A−1 = A = ⎢ 2 ⎥ ⎢ −1 − i 1 + i ⎥ ⎢⎣ 2 2 ⎥⎦

(ii)

A

T

A

A

AA

2 1 i 2 2 0

i 2

0

2

0 2

2

i 2

0

2 0

0 2

2 1 i 2 2 0

i 2

0

2 0

0 2

2 1 i 2 4 0

i 2

0

2

2

0 2

i 2 0

i 2

0

2

0 2

1 2

0

i 2 0

0

1 0 0 = 0 1 0 =I 0 0 1 Hence, A is a unitary matrix. For unitary matrix, A

1

A

2 1 i 2 2 0

0

i 2

0

2

0 2

0

4 0 0 1 = 0 4 0 4 0 0 4

Matrices

11.15

Example 12: Verify if the following matrices are orthogonal and hence find their inverse: (i) A =

⎡ 1 2 2⎤ 1⎢ 2 1 −2⎥⎥ 3⎢ ⎢⎣ −2 2 −1⎥⎦

Solution: (i)

cos f (ii) A = sin f 0

sin f cos f 0

⎡ 1 2 2⎤ 1⎢ A = ⎢ 2 1 −2⎥⎥ 3 ⎢⎣ −2 2 −1⎥⎦ AT =

1 1 2 3 2

2 1 2

2 2 1

⎡ 1 2 2⎤ ⎡ 1 2 −2⎤ 1⎢ 1 2⎥⎥ AA = ⎢ 2 1 −2⎥⎥ ⎢⎢ 2 9 ⎢⎣ −2 2 −1⎥⎦ ⎣⎢ 2 −2 −1⎥⎦ T

9 0 0 1 = 0 9 0 9 0 0 9 1 0 0 = 0 1 0 =I 0 0 1 Hence, A is an orthogonal matrix. Since AAT = AT A = I and AA–1 = A–1A = I, A–1 = AT for orthogonal matrix. 1 1 A =A = 2 3 2 1

(ii)

T

cos A = sin 0 T

A

cos sin 0

2 1

2 2

2

1

sin cos 0

0 0 1

sin cos 0

0 0 1

0 0 . 1

11.16

Engineering Mathematics

cos sin 0

T

AA

sin cos 0

0 0 1

cos sin 0

sin cos 0

0 0 1

sin cos 0

0 1 0 0 0 = 0 1 0 1 0 0 1

=I Hence, A is an orthogonal matrix. For orthogonal matrix, A

1

cos sin 0

T

A

⎡ 0 2m n ⎤ Example 13: Find l, m, n and A-1 if A = ⎢ l m − n⎥ is orthogonal. ⎢ ⎥ ⎣ l −m n ⎦ Solution: Since the matrix A is orthogonal, AAT = I 0 2m n 0 l l m n 2m m l 2

4m 2m 2

2

m

n 2

n n2

n 2

1 0 0 0 1 0

n

0 0 1

2

2

m2

n2

2m n l m2 n2

2m n l m2 n2

1 0 0 0 1 0

l2

l2

0 0 1

2

2m 2 n 2 Equating the two matrices,

n

l m

m2

2

n2

4m 2 + n 2 = 1 2m 2 n 2 0 2 l + m2 + n2 = 1

… (1) … (2) … (3)

Solving Eqs. (1), (2) and (3) 1 , 2 1 m2 = , 6 1 n2 = , 3 l2 =

l=± m=± n=± 0

A 1 = AT = ± ±

1 2 1 6 1 3 ±

2 6 1 3

± ∓

1 2 1 6 1 3

± ∓ ±

1 2 1 6 1 3

Matrices

l1 Example 14: If A = l2 l3

m1 m2 m3

n1 n2 n3

11.17

is orthogonal, find the relationship among

l1, m1, n1, …n3. Solution: Since the matrix A is orthogonal, AAT = I l1 m1 n1 l1 l2 l3 1 0 0 l2 m2 n2 m1 m2 m3 = 0 1 0 l3 m3 n3 n1 n2 n3 0 0 1 l12 + m12 + n12 l1l2 + m1m2 + n1n2 l1l3 + m1m3 + n1n3

l1l2 + m1m2 + n1n2 l1l3 + m1m3 + n1n3 1 0 0 l22 + m22 + n22 l2 l3 + m2 m3 + n2 n3 = 0 1 0 l2 l3 + m2 m3 + n2 n3 l32 + m32 + n32 0 0 1

Equating the two matrices, l12 + m12 + n12 = l22 + m22 + n22 = l32 + m32 + n32 = 1 l1l2 + m1m2 + n1n2 = l1l3 + m1m3 + n1n3 = l2 l3 + m2 m3 + n2 n3 = 0

and

Exercise 11.1 1. Express the following matrices as the sum of a symmetric and a skew– symmetric matrix: (i)

(iii)

(iii)

9

4

1 0 –2 1 3 2 4 –4

0 5 6 7 2

1 + i 2 + 3i 2 –i i

0

2 4 + i 6i 6 5–i 4

0 1 – i 8i 3. Express the following matrices as P + iQ, where P and Q are both Hermitian.

3 –2 6 2 7 –1 5

1 1– i 2 – 3i

0 5 –3 1 1 1 4 5

(ii)

(ii)

3 1 1 0

2. Express the following matrices as the sum of a Hermitian and a skew Hermitian matrix. 2 2+i 3 (i) –2 + i 0 4 –i 3 – i 1– i

(i)

2 3 – i 1 + 2i i 0 1 1 + 2i 1 3i

⎡1 + 2i 2 3 − i ⎤ (ii) ⎢ 2 + 3i 2i 1 – 2i ⎥ ⎢ ⎥ ⎢⎣ 1 + i 0 3 + 2i ⎥⎦ 4. Express the following Hermitian matrices as P + iQ, where P is a real symmetric and Q is a real skew symmetric.

Engineering Mathematics

11.18

(i)

2 2+i 2–i 3 2i –i

(ii)

2 1– i i

⎡ cos φ cos θ (ii) ⎢ − sin φ cos θ ⎢ ⎢⎣ − sin θ

–2i i 1

1+ i 0 –3 + i

–i –3 – i –1

5. Express the following skew Hermitian matrices as P + iQ, where P is real and skew symmetric and Q is real and symmetric. (i)

(ii)

0 –2 – 3i

2 – 3i 1 + i 2i 2–i

–1 + i

–2 – i

i 2i

2i 2i

–1 + 3i 2–i

1 + 3i

–2 – i

3i

–i

6. Show that the following matrices are unitary.

(i)

2+i 3 2i 3

(ii)

i 2 3 2

1 1+ i (iii) 2 1+ i

2i 3 2–i 3 3 2 i 2 –1 + i 1– i

7. Show that following matrices are orthogonal and hence find their inverse: 1⎤ ⎡ 8 –4 1⎢ (i) ⎢ 1 4 –8⎥⎥ 9 ⎢⎣ 4 7 4 ⎥⎦

⎡ ⎢ ⎢ ⎢ (iii) ⎢ ⎢ ⎢ ⎢ ⎣ ⎡ ⎢ ⎢ ⎢ (iv) ⎢ ⎢ ⎢ ⎢ ⎣

sin φ cos φ 0

1

1

3 1

6 –2

3 1

6 1

3

6

0 2 3 1 3

1 2 1 6 1 3

cos φ sin θ ⎤ − sin φ sin θ ⎥⎥ cos θ ⎦⎥

1 ⎤ ⎥ 2⎥ ⎥ 0 ⎥ ⎥ 1 ⎥ – ⎥ 2⎦ –

1 ⎤ ⎥ 2⎥ 1 ⎥ ⎥ 6⎥ 1 ⎥ ⎥ 6⎦

8. Find l, m, n and A 1 m n 0⎤ ⎡ l ⎢ 0 0 0 –1⎥ ⎥ is if A = ⎢ ⎢ n l –m 0⎥ ⎢ ⎥ 0⎦ ⎣ – m n –l orthogonal. –8 4 a 1 1 4 b is 9. Find a, b, c if A = 9 4 7 c orthogonal. [Ans. : a = 1, b = 8, c = 4] 10. If (ar, br, cr) where r = 1, 2, 3 be the direction cosines of the three mutually perpendicular lines referred to an orthogonal co-ordinate system, ⎡ a1 b1 c1 ⎤ then prove that A = ⎢ a2 b2 c2 ⎥ is ⎢ ⎥ ⎢⎣ a3 b3 c3 ⎥⎦ orthogonal.

Matrices

11.19

11.4 ADJOINT OF A SQUARE MATRIX The transpose of the matrix of the cofactors is called the adjoint of the matrix. Let A be a non-singular n-rowed square matrix and |A| be its determinant. a11 Let A = a21 .... an1

a12 a22 .... an 2

.... a1n .... a2 n .... .... .... ann

The matrix formed by the cofactors of the elements of A is A11 A21 .... An1

A12 A22 .... An 2

.... A1n .... A2 n .... .... .... Ann

The transpose of this matrix of cofactors is called the adjoint of A and is denoted by adj A.

adj A =

A11 A12

A21 A22

.... An1 .... An 2

.... A1n

.... A2 n

.... ....

.... Ann

Example 1: If A is a non-singular square matrix of order n, then show that (i) A (adj A) = (adj A)A = |A| In (ii) |adj A| = |A|n

1

(iii) adj (adj A) = |A|n 2 A. Solution: (i) Let A be a non-singular square matrix of order n. a11 a21

a12 a22

.... a1n .... a2 n

A = .... .... an1

.... .... an 2

.... .... .... .... .... ann

A11 A21 adj A = Transpose of .... .... An1 where Aij is the cofactor of aij in |A|.

A12 A22 .... .... An 2

.... A1n .... A2 n .... .... .... .... .... Ann

Engineering Mathematics

11.20

A11 A12 adj A = .... .... A1n

A21 A22 .... .... A2 n

.... An1 .... An 2 .... .... .... .... .... Ann

(i, j)th element in A (adj A) = ai1Aj1 + ai2 Aj2 + .… + ain Ajn = |A| if i = j =0 if i j Hence, in product A (adj A), each diagonal element is |A| and each non-diagonal element is 0. 1 0 A 0 .... 0 0 1 0 A .... 0 A (adj A) = .... .... .... .... = |A| .... .... .... .... .... .... .... .... 0 .... 0 .... .... A

.... 0 .... 0 .... .... .... .... .... 1

= |A| In Similarly, (adj A) A = |A| In A (adj A) = (adj A) A = |A| In (ii) We know that A (adj A) = |A| In |A (adj A)| = |(|A| In)| |A||adj A| = |A|n |adj A| = |A|n

(... |In| = 1)

1

(iii) We know that A (adj A) = |A| In Replacing A by adj A, adj A (adj adj A) = |adj A|In = |A|n 1 In Premultiplying both sides by A, A (adj A) (adj adj A) = A|A|n 1 In |A| In (adj adj A) = |A|n 1 A In adj (adj A) = |A|n 2 A Example 2: Verify A (adj A) = (adj A) A = |A| I for the following matrix.

2 3 1 A= 1 2 3 3 1 2

Matrices

11.21

2 3 1 A= 1 2 3 3 1 2

Solution:

⎡ 1 7 −5⎤ ⎢ 1 7⎥⎥ The matrix of cofactors of elements of A = ⎢ –5 ⎢⎣ 7 –5 1⎥⎦ ⎡ 1 –5 7⎤ 1 –5⎥⎥ adj A = ⎢⎢ 7 ⎢⎣ −5 7 1⎥⎦ 2 3 1 A (adj A) = 1 2 3 3 1 2

18 0 0 ⎡ 1 –5 7 ⎤ ⎢ 7 ⎥ 1 –5⎥ = 0 18 0 ⎢ ⎢⎣ −5 7 1⎥⎦ 0 0 18

= 18 I 2 3 1 1 2 3 = 2 (1) |A| = 3 1 2

3 ( 7) + 1 ( 5) = 18

A (adj A) = |A| I ⎡ 1 −5 7⎤ (adj A) A = ⎢⎢ 7 1 −5⎥⎥ ⎢⎣ −5 7 1⎥⎦ = 18 I = |A| I

2 3 1 18 0 0 1 2 3 = 0 18 0 3 1 2 0 0 18

⎡ –1 –8 4⎤ 1⎢ Example 3: Find adj (adj A), where A = ⎢ –4 4 7 ⎥⎥ 9 ⎢⎣ –8 –1 –4⎥⎦ Solution:

⎡ –1 –8 4 ⎤ 1⎢ 7⎥⎥ A = ⎢ –4 4 9 ⎢⎣ –8 –1 –4 ⎥⎦

We know that, adj (adj A) = |A|n 2 A and |kA| = kn |A| Here, n=3

Engineering Mathematics

11.22

–8 4 3 –1 ⎛1⎞ 7 |A| = ⎜ ⎟ –4 4 ⎝9⎠ –8 –1 –4 1 [(–1)(–16 + 7) – (–8)(16 + 56) + 4(4 + 36)] 729 1 729 = 1 = 729 adj (adj A) = |A|3 2A =A =

Example 4: If A =

–1 –2 –2 2 1 –2 , show that adj A = 3 AT. 2 –2 1

A=

Solution:

–1 –2 –2 2 1 –2 2 –2

1

–3 –6 –6 The matrix of cofactors of elements of A = 6 3 –6 6 –6 –3 adj A = –6

3

6 6 –1 3 –6 = 3 –2

–6 –6

3

2 2 1 –2

–2 –2

= 3 AT 1 2 1 Example 5: If A = a 0 4 and adj (adj A) = A, find a. 1 1 1

Solution:

1 2 1 A= a 0 4 1 1 1

We know that adj (adj A) = |A|n 2 A Here, n=3 1 2 1 |A| = a 0 4 = 1(0 1 1 1

4)

2(a

4) + 1(a

0) = a + 4

1

Matrices

11.23

adj (adj A) = ( a + 4)1 A = ( a + 4) A Given adj (adj A) = A ( a + 4) A = A a+4=1 a=3

( |A|

0)

Exercise 11.2 1. Verify that A (adj A) = (adj A)A = |A| In for the following matrices: (i)

(ii)

1 –2 3 2 3 –1 3 1 2 –1 –2 3 –2 1 1 4

(i)

–5 2

2. Show that the adjoint of the matrix –4 –3 –3 A = 1 0 1 is A itself. 4

4

⎡ –4 –3 –2⎤ 1⎥⎥ , then show 3. If A = ⎢⎢ −1 0 ⎢⎣ 2 3 4 ⎥⎦ that adj A is symmetric. 4. Verify that (adj A)T = (adj AT) for following matrices: 2 3 1 1 2 3 3 1 2 (ii)

3

1 2 1 0 1 1 1 1 2

11.5 INVERSE OR RECIPROCAL OF A MATRIX If A be any n-rowed square matrix, then a matrix B, if it exists such that AB = BA = In 1 is called the inverse of A, In being a unit matrix and A 1 = adj A A Example 1: Prove that every invertible matrix possesses a unique inverse. Solution: Let A be any n-rowed invertible matrix. Let B and C be two inverses of A. Then

AB = BA = In AC = CA = In

Premultiplying Eq. (1) by C, C (AB) = C (BA) = C In = C C (AB) = C CA (B) = C

… (1) … (2)

11.24

Engineering Mathematics In B = C B=C

[From Eq. (2)]

Hence, inverse of a matrix is unique. Example 2: Prove that the necessary and sufficient condition for a square matrix A to possess an inverse is that |A| ñ 0, i.e., A is non-singular. Solution: Necessary condition: Let A be any n-rowed square matrix and let B be the inverse of A. Then AB = In |AB| = |In| = 1 |A||B| = 1 |A| 0 Sufficient condition: Let |A| 0 1 adj A B= A AB = A =

1 1 adjA = A (A adj A) A

1 |A| In A

= In Also,

1 ⎛ 1 ⎞ BA = ⎜ adj A⎟ A = (adj A) A ⎝ A ⎠ A 1 = A In A = In AB = BA = In

Hence, B is the inverse of A. Example 3: Prove that if A, B are two n-rowed non-singular matrices, then AB is also non-singular and (AB)-1 = B-1A-1 i.e., the inverse of a product is the product of the inverses taken in the reverse order. Solution: Let A, B are n-rowed non-singular matrices. |A| 0 |B| 0 |AB| 0 Hence, matrix AB is non-singular and possesses its inverse.

[∵ |AB| = |A| |B|]

Matrices

11.25

(AB) (B 1 A 1) = A(BB 1) A 1 = A In A 1 = AA 1 = In and (B 1A 1) (AB) = B 1 (A 1A)B = B 1 In B = B 1 B = In Hence, B 1A 1 is the inverse of AB. i.e., (AB) 1 = B 1 A 1 Now,

Example 4: Prove that if A is an n ë n non-singular matrix, then (A-1)T = (AT)-1. Solution: Let A is a non-singular matrix. |A| 0 |A| = |AT| |AT| 0 and (AT)

Also, Hence, We have

1

exists.

AA 1 = A 1 A = In Taking transpose of both sides, (AA 1)T = (A 1 A)T = (In)T (A 1)TAT = AT(A 1)T = (In) (A 1)T = (AT) 1

[Using reversal law]

Example 5: Prove that if A be an n ë n non-singular matrix, then (A-1)q = (Aq )-1. Solution: Let A is a non-singular matrix. |A| 0 – – |Aq | = |(A )T| = |A| 0 Hence, Aq is non-singular and (Aq ) 1 exists. Now, AA 1 = A 1 A = In Taking conjugate transpose of both the sides, (AA 1)q = (A 1A)q = (In)q (A 1)q Aq = Aq(A 1)q = In (A 1)q = (Aq ) 1

[Using reversal law]

Example 6: Prove that if A is a skew symmetric matrix of odd order, then |A| = 0 and hence its inverse does not exists. Solution: Let A is a skew symmetric matrix. AT = A |AT| = | A| = ( 1)n |A| |A| = ( 1)n |A| If n is odd, then ( 1)n = 1 |A| = |A| 2|A| = 0 |A| = 0 Hence, A is singular and its inverse does not exist.

[∵ |AT| = |A|]

Engineering Mathematics

11.26

Example 7: Find the inverse of the following matrices: (i)

1 2 1 0 2 2 2 1 1

Solution: (i)

(ii)

1 1 1 1 2 –3 2 –1 3

(iii)

cos` sin` 0

– sin` cos` 0

1 2 1 A= 0 2 2 2 1 1

0 4 –4 The matrix of cofactors of elements of A = –1 –1 3 2 –2 2 0 –1 2 4 –1 –2

adj A =

–4 1 2 1 |A| = 0 2 2 = 1(2 2 1 1 1 1 A1= adj A = 4 A

(ii)

3

2) 2 (0

2

4) + 1(0

4) = 4

0 –1 2 4 –1 –2 –4 3 2

1 1 1 A = 1 2 –3 2 –1 3

3 –9 –5 The matrix of cofactors of elements of A = –4 1 3 –5 4 1 3 –4 –5 1 4 adj A = –9 –5 3 1 1 1 1 |A| = 1 2 –3 = 1(6 2 –1 3

3)

1(3 + 6) + 1( 1

4) = –11

0 0 1

Matrices

(iii)

cos A = sin 0

– sin cos 0

11.27

0 0 1

cos The matrix of cofactors of elements of A = sin 0 cos adj A = – sin 0 cos |A| = sin 0

– sin cos 0

0 0 1

sin cos 0

0 0 1

sin cos 0

0 0 =1 1

cos 1 A = adj A = – sin A 0 1

sin cos

0 0

0

1

–2 1 3 Example 8: Find matrix A, if adj A = –2 –3 11 . 2

Solution:

1 –5

–2 1 3 adj A = –2 –3 11 2

1 –5

We know that Here, Now,

|adj A| = |A|n 1 n=3 |adj A| = |A|2 |adj A| = 2(15 |A| = 4

11)

1(10

The matrix of cofactors of elements of adj A =

22) + 3( 2 + 6) = 16 4 8

12 4 4 4

20 16 8

Engineering Mathematics

11.28

4 8 20 adj (adj A) = 12 4 16 4 4 8

(adj A) 1 =

4 8 20 1 1 12 4 16 adj (adj A) = 16 adjA 4 4 8

1 2 5 = 1 3 1 4 4 1 1 2 Now,

(adj A) 1 =

A A

A = |A| (adj A)

1

1 2 5 1 2 5 1 = 4. 3 1 4 = 3 1 4 4 1 1 2 1 1 2 Example 9: Find the matrix A if

4 2 2 –1 2 2 A = 2 3 –3 2 3 7 Solution:

Then

Let B =

4 2 2 3

C=

2 –1 –3 2

D=

2 2 3 7

BAC = D AC = B 1D A=B1DC

1

B 1=

1 1 3 –2 adj B = B 8 –2 4

C 1=

1 2 1 1 2 1 = adj C = C 3 2 1 3 2

Matrices

A=

Hence,

=

11.29

1 3 –2 2 2 2 1 8 –2 4 3 7 3 2 1 –24 –16 8 88 56 –3 –2 11 7

=

0 1 1 Example 10: If A = 1 0 1 1 1 0

q+r 2 r–q and B = 2 q–r 2

r– p 2 r+ p 2 p–r 2

q– p 2 p–q , 2 p+q 2

prove that ABA-1 is a diagonal matrix.

Solution:

0 1 1 A= 1 0 1 1 1 0 |A| = 0(0

1)

1(0

1) + 1(1

0) = 2

Hence, A 1 exists.

–1 1 1 The matrix of cofactors of elements of A = 1 –1 1 1 1 –1 adj A =

–1 1 1 1 –1 1 1 1 –1

1 1 A = adj A = A 2 1

Now,

0 1 1 AB = 1 0 1 1 1 0

–1 1 1 1 –1 1 1 1 –1

q+r 2 r–q 2 q–r 2

r–p 2 r+ p 2 p–r 2

q– p 2 ⎡0 p–q ⎢ = ⎢q 2 ⎢⎣ r p+q 2

p 0 r

p⎤ q ⎥⎥ 0 ⎥⎦

Engineering Mathematics

11.30

⎡0 ⎢ ABA = ⎢ q ⎢⎣ r 1

p 0 r

p ⎤ ⎡ –1 1 1 ⎤ 1 q ⎥⎥ ⎢⎢ 1 –1 1 ⎥⎥ 2 0 ⎦⎥ ⎣⎢ 1 1 –1⎥⎦

⎡ 2 p 0 0 ⎤ ⎡ p 0 0⎤ 1⎢ ⎥ ⎥ ⎢ = ⎢ 0 2q 0 ⎥ = ⎢ 0 q 0⎥ 2 ⎢⎣ 0 0 2r ⎥⎦ ⎢⎣ 0 0 r ⎥⎦ Hence, ABA 1 is a diagonal matrix. Example 11: If A =

0 1 + 2i , show that (I - A) (I + A)-1 is a unitary matrix, –1 + 2i 0

where I is a unit matrix. Solution:

A=

0 1 + 2i –1 + 2i 0

I

A=

1 1 – 2i

1 1 + 2i –1 + 2i 1

I+A= |I + A| = 1 adj (I + A) = (I + A) 1 =

(I

A) (I + A) 1 =

–1 – 2i 1

(1 + 2i) ( 1 + 2i) = 6 1 1 – 2i

–1 – 2i 1

1 1 1 adj (I + A) = I+A 6 1 – 2i

1 1 6 1 – 2i

–1 – 2i 1 1 1 – 2i

–1 – 2i 1

–1 – 2i 1 –4 = 1 6 2 – 4i

–2 – 4i = B (Say) –4

For unitary matrix, BBq = I BT =

–4 2 – 4i 1 –4 6 –2 – 4i

Bq =

–4 2 + 4i 1 –4 6 –2 + 4i

BBq =

1 ⎡ –4 36 ⎢⎣ 2 – 4i

–2 – 4i ⎤ ⎡ –4 2 + 4i ⎤ 1 ⎡36 0 ⎤ = =I ⎥ ⎢ –4 ⎦ ⎣ –2 + 4i –4 ⎥⎦ 36 ⎢⎣ 0 36 ⎥⎦

Hence, (I – A) (I + A) 1 is a unitary matrix.

Matrices

11.31

Example 12: Show that [diag (`, a, f)]-1 = diag 0 Solution: Let A = 0 0

1 1 1 , , if `af ñ 0. ` a f

0 0

0 0

The matrix of cofactors of elements of A =

0 adj A =

0 0

0 0

0 0

0

0 0

0

a A= 0 0

0 0 b 0 = abg 0 g

1 adj A A

A–1 =

1 0

1

=

0 0

0 0

=

0

1

tan

Solution: Let A = – tan

2

1

2

0

0 0 1

1 1 1 , ,

⎡ 1 − sin θ ⎤ ⎢ = ⎢ cos θ ⎥⎦ ⎢ θ tan ⎢⎣ 2

⎡ cos θ Example 13: Show that ⎢ ⎣ sin θ

1

0 0

= diag

0

θ⎤ − tan ⎥ 2 ⎥ 1 ⎥ ⎥⎦

⎡ ⎢ 1 ⎢ ⎢ − tan θ ⎢⎣ 2

θ⎤ tan ⎥ 2 ⎥ 1 ⎥ ⎥⎦

−1

11.32

Engineering Mathematics

1

tan

2

The matrix of cofactors of elements of A = – tan

⎤ − tan ⎥ 2 ⎥ 1 ⎥ ⎥⎦

⎡ ⎢ 1 adj A = ⎢ ⎢ tan ⎢⎣ 2

1

tan

2

|A| = tan

A1=

⎡ ⎢ 1 Now, ⎢ ⎢ tan ⎢⎣ 2

⎤⎡ – tan ⎥ ⎢ 1 2 ⎥⎢ 1 ⎥ ⎢ − tan ⎥⎦ ⎢⎣ 2

1

2

= 1 + tan 2

1

2

1 adj A = A

1

1 sec 2

2

–1

⎡ ⎤ tan ⎥ ⎢ 1 2 ⎥ =⎢ ⎢ tan 1 ⎥ ⎢⎣ ⎥⎦ 2

tan

sec 2 cos 2 =

– tan

2

2

1

2

1 – tan 2 2 2

2sin cos sin

=

= sec 2

⎤ ⎡ – tan ⎥ 1 1 ⎢ 2 ⎥ ⎢ 2 ⎢ tan 1 ⎥ sec ⎥⎦ 2 ⎢⎣ 2

1

=

2

tan

2

+ tan

– sin 2

2

cos

2

2

– tan

2

–2sin 2

2

2

cos

– sin

– sin cos

1 a 0 1 0 0 Example 14: Find the inverses of A = 0 1 a and B = b 1 0 0 0 1 0 b 1

1 + ab a 0 and hence, find inverse of C = b 1 + ab a 0

b

1

– tan

2

– tan 2

2

cos

⎤ – tan ⎥ 2 ⎥ 1 ⎥ ⎥⎦

2

2

2

2

2

+1

Matrices

Solution:

11.33

1 a 0 A= 0 1 a 0 0 1

1 The matrix of cofactors of elements of A = – a a2

0 0 1 0 –a 1

⎡1 – a a 2 ⎤ ⎢ ⎥ adj A = ⎢0 1 – a ⎥ ⎢0 0 1 ⎥⎦ ⎣ 1 a 0 |A| = 0 1 a = 1 0 0 1 A1=

1 adj A A

⎡1 – a a 2 ⎤ ⎢ ⎥ = ⎢0 1 – a ⎥ ⎢0 0 1 ⎥⎦ ⎣ 1 0 0 Replacing a by b, A T becomes b 1 0 which is equal to matrix B. 0 b 1 Hence, replacing a by b in transpose of A 1, we get 0 0⎤ ⎡1 ⎢ –b 1 0⎥ B = ⎢ ⎥ ⎢⎣ b 2 – b 1 ⎥⎦ a 0⎤ ⎡1 + ab ⎥ ⎢ b ab a 1 + C= ⎢ ⎥ = AB ⎢⎣ 0 b 1 ⎥⎦ 1

Now,

C 1 = (AB) 1 = B 1 A 1 0 0 = –b 1 0 b 2 –b 1

1

1 –a a2 0 1 –a 0 0 1

1 –a a2 = – b 1 + ab – a 2b – a b 2 – ab 2 – b a 2 b 2 + ab + 1

Engineering Mathematics

11.34

0 1 1 Example 15: Find the inverse of the matrix S = 1 0 1 and if 1 1 0 4 –1 1 1 A = –2 3 –1 , show that SAS-1 is a diagonal matrix diag. (2, 3, 1). 2 2 1 5

Solution: S =

0 1 1 1 0 1 1 1 0

The matrix of cofactors of elements of S =

–1 1 1 1 –1 1 1 1 –1

–1 1 1 adj S = 1 –1 1 1 1 –1 0 1 1 |S| = 1 0 1 = 1 ( 1) + 1(1) = 2 1 1 0 1 1 S = adj S = S 2 1

0 1 1 1 SA = 1 0 1 2 1 1 0

–1 1 1 1 –1 1 1 1 –1 4 –1 1 0 4 4 0 2 2 1 6 0 6 = 3 0 3 –2 3 –1 = 2 2 2 0 1 1 0 2 1 5

⎡0 2 2⎤ ⎡ –1 1 1⎤ 1⎢ ⎥⎢ ⎥ SAS = ⎢ 3 0 3⎥ ⎢ 1 –1 1⎥ 2 ⎢⎣1 1 0 ⎥⎦ ⎣⎢ 1 1 –1⎥⎦ 1

=

4 0 0 2 0 0 1 0 6 0 = 0 3 0 2 0 0 2 0 0 1

= diag. (2, 3, 1)

Matrices

11.35

Exercise 11.3 1. Find the linverse of the following matrices: (i)

1 2 1 2 1 1 4 5 1

(iii)

1 2 1 0 2 2 2 1 1

(ii)

3 2 6 1 1 2 2 2 5

⎡ ⎡ –4 3 1⎤ ⎤ 1⎢ ⎢ ⎥ ⎥ ⎢ Ans. : (i) 6 ⎢ 2 –3 1⎥ ⎥ ⎢ ⎢⎣ 6 3 –3⎥⎦ ⎥ ⎢ ⎥ ⎡ 1 2 –2⎤ ⎥ ⎢ ⎢ (ii) ⎢⎢ –1 3 0 ⎥⎥ ⎥⎥ ⎢ ⎢ ⎢⎣ 0 –2 1⎥⎦ ⎥ ⎢ ⎥ ⎡ 0 –1 2⎤ ⎥ ⎢ 1 ⎢ (iii) ⎢⎢ 4 –1 –2⎥⎥ ⎥ ⎢ ⎥ 4 ⎢⎣ –4 3 2⎥⎦ ⎥⎦ ⎢⎣ 2. Find the inverse of the matrix 0 1 1 S = 1 0 0 and 1 1 0 A=

3 –2 –1 1 –1 4 1 and show that 2 1 2 5

SAS 1 is the diagonal matrix diag. (3, 2, 1). 3 –3 4 3. If A = 2 –3 4 , show that A3 = A 1. 0 –1 1 8 4 3 4. If A = 2 1 1 , prove that 1 2 1

2 –4 –5 1 A2= –10 47 –11 9 9 –54 27

1 2 –1 5. If A = 0 1 –1 , find A 1 if it 2 2 3 exists. Hence, find the inverse of

3 6 –3 B = 0 3 –3 6 6

9

5 –8 –1 1 Ans. : A = –2 5 1 , 3 –2 2 1 1

5 –8 –1 1 B = –2 5 1 9 –2 2 1 1

1 1 1 6. If A = 1 2 3 1 4 9

and

2 5 3 B = 3 1 2 , show that 1 2 1 1 (AB) = B 1 A 1. 7. Find the matrix A if 2 1 –3 2 –2 4 A = 3 2 5 –3 3 –1 Ans. :

24 13 –34 –18

8. Find the inverse of A if 1 0 0 1 –2 9 1 0 0 2 –1 0 A 0 1 –6 = 0 1 0 –2 1 1 0 0 1 0 0 1

–21 11 9 Ans. : A = 14 –7 –6 –2 1 1 1

11.36

Engineering Mathematics

11.5.1 Solution of Non-Homogenous Linear Equations by Inverse Method Suppose

a11x1 + a12x2 + … + a1nxn = b1 a21x1 + a22x2 + … + a2nxn = b2 ..…………………………… ..…………………………… an1x1 + an2x2 + … + a1nxn = bn

is a system of non-homogeneous linear equations in n unknowns x1, x2, … xn. Writing these equations in matrix form, AX = B … (1) Multiplying Eq. (1) by A 1, A 1AX = A 1B IX = A 1B X = A 1B This gives the solution of non-homogeneous linear equations by inverse method. Note: This method is applicable only for non-singular matrix. Example 1: Solve the simultaneous equations: (i)

x+y+z=3 x + 2y + 3z = 4 x + 4y + 9z = 6

(ii) 3x + 2y + 4z = 7 2x + y + z = 4 x + 3y + 5z = 2.

Solution: (i) In matrix form, AX =B 1 1 1 1 2 3

x 3 y = 4

1 4 9

z

6 1 1 1 A= 1 2 3 1 4 9

6 –6 2 The matrix of cofactors of elements of A = –5 8 –3 1 –2 1 6 –5 1 adj A = –6 8 –2 2 –3 1 |A| = 1(18

12)

1(9

3) + 1(4

2) = 2

Matrices

11.37

6 –5 1 1 1 –6 8 –2 A = adj A = 2 A 2 –3 1 1

x 6 –5 1 3 4 2 1 1 –1 X= y =A B= –6 8 –2 4 = 2 = 1 2 2 z 2 –3 1 6 0 0 x = 2, y = 1, z = 0 (ii) In matrix form, AX = B 3 2 4 2 1 1 1 3 5

x 7 y = 4 z 2 3 2 4 A= 2 1 1 1 3 5

The matrix of cofactors of elements of A =

2 –9 5 2 11 –7 –2

5

–1

⎡ 2 2 −2⎤ adj A = ⎢⎢ −9 11 5⎥⎥ ⎢⎣ 5 −7 −1⎥⎦ |A| = 3(5 A1=

3) 2(10

1) + 4(6

1) = 8

2 2 –2 1 1 –9 11 5 adj A = A 8 5 –7 –1

9 4 2 2 –2 7 18 1 1 9 X=A1B= –9 11 5 4 = –9 = – 8 8 8 5 –7 –1 2 5 5 8 x=

9 ,y= 4

9 5 ,z= 8 8

11.38

Engineering Mathematics

Exercise 11.4 1. Solve the following equations: (i) x + y + z = 8, x y + 2z = 6, 9x + 5y 7z = 44 (ii) 3x + y + 2z = 3, 2x 3y z = 3, x + 2y + z = 4 (iii) x + y + z = 6, x y + 2z =5, 3x + y + z = 8 (iv) 4x + 2y z = 9, x y + 3z = 4, 2x + z = 1 (v) x + 2y + 3z = 1, 2x + 3y + 8z = 2, x+y+z=3

5 4 Ans. : (i) x = 5, y = , z = 3 3 (ii) x 1, y 2, z 1 (iii) x = 1, y = 2, z = 3 (iv) x 1, y 2, z 1 9 1 (v) x ,y 1, z 2 2

11.6 ELEMENTARY TRANSFORMATIONS Elementary transformation is any one of the following operations on a matrix. (i) The interchange of any two rows (or columns) (ii) The multiplication of the elements of any row (or column) by any non-zero number (iii) The addition or subtraction of k times the elements of a row (or column) to the corresponding elements of another row (or column), where k 0 Symbols to be used for elementary transformation: (i) Rij : Interchange of i th and j th row : Multiplication of i th row by a non zero number k (ii) kRi (iii) Ri + kRj : Addition of k times the j th row to the i th row The corresponding column transformations are denoted by Cij, kCi and Ci + kCj respectively.

11.6.1 Elementary Matrices A matrix obtained from a unit matrix by subjecting it to any row or column transformation is called an elementary matrix.

11.6.2 Inverse of a Matrix by Elementary Transformation Let A be any non-singular square matrix. Then A = IA Applying suitable elementary row transformation to A on the R.H.S and I on the L.H.S so that A reduces to I. I = BA Hence, B=A1 Similarly, inverse can also be obtained by applying suitable column transformation to A on the R.H.S and to I on the L.H.S of the equation A = AI so that A reduces to I.

Matrices

11.39

I = AB B=A1

Hence,

Example 1: Using elementary row transformations, find the inverse of the following matrices:

(i)

2 3 4 4 3 1 1 2 4

1 –1 0 2 0 1 1 –1 2 1 2 1 3 –2 1 6

(ii)

A = I3 A

Solution: (i)

⎡ 2 3 4⎤ ⎡1 0 0⎤ ⎢ 4 3 1 ⎥ = ⎢ 0 1 0⎥ A ⎥ ⎥ ⎢ ⎢ ⎢⎣1 2 4⎥⎦ ⎢⎣0 0 1 ⎥⎦ R13 ⎡1 2 4⎤ ⎡ 0 0 1 ⎤ ⎢ 4 3 1 ⎥ = ⎢ 0 1 0⎥ A ⎥ ⎥ ⎢ ⎢ ⎢⎣ 2 3 4⎥⎦ ⎢⎣1 0 0⎥⎦ R2

4R1, R3

2R1

1 2 4 0 0 1 0 –5 –15 = 0 1 –4 A 0

–1

–4

1 0 –2

1 R1 + 2R3, – R2 5 0 –3 1 4 A 0 1 3 = 0 – 5 5 0 –1 –4 1 0 –2 1

0 –4

2

R3 + R2 0 –3⎤ ⎡2 ⎡ 1 0 –4 ⎤ ⎢ 4 ⎥⎥ ⎢0 1 3⎥ = ⎢0 – 1 A ⎢ ⎥ ⎢ 5 5⎥ ⎢⎣0 0 −1⎥⎦ ⎢ 1 6⎥ ⎢1 – – ⎥ 5 5⎦ ⎣ R1

4 R3, R2 + 3R3

Engineering Mathematics

11.40

4 ⎡ ⎢ –2 5 0 0⎤ ⎢ 4 1 0 ⎥⎥ = ⎢ 3 – ⎢ 5 0 –1⎥⎦ ⎢ ⎢ 1 −1 ⎢⎣ 5 1) R3

9⎤ 5⎥ ⎥ 14 ⎥ – A 5⎥ ⎥ 6 − ⎥ 5 ⎥⎦

4 ⎡ ⎢ –2 5 ⎡ 1 0 0⎤ ⎢ ⎢0 1 0 ⎥ = ⎢ 3 – 4 ⎢ ⎥ ⎢ 5 ⎢⎣0 0 1⎥⎦ ⎢ 1 ⎢ –1 ⎢⎣ 5

9⎤ 5⎥ ⎥ 14 – ⎥ A 5⎥ ⎥ 6⎥ 5 ⎥⎦

⎡1 ⎢0 ⎢ ⎢⎣0 (

4 5 4 A1= 3 – 5 1 –1 5 –2

9 5 –10 4 9 14 1 – = 15 –4 –14 5 5 –5 1 6 6 5

A = I4 A

(ii)

1 –1 0 2 0 1 1 –1 2 1 2 3 –2 1 R3

1 6

3 2 –3 1 1 0 R3

=

3R1 1 0 0 0 0 1 0 0

A

–2 0 1 0 –3 0 0 1

3R2, R4 R2

1 –1 0 2 1 0 0 0 1 1 –1 0 1 0 = 0 0 –1 0 –2 –3 1 0 0 0 1 –3 –1 0 R1

A

0 0 1 0 0 0 0 1

2R1, R4

1 –1 0 2 0 1 1 –1 0 0

=

1 0 0 0 0 1 0 0

2R4, R2 + R3

0 0 A 0 1

Matrices

11.41

⎡ 1 –1 0 0 ⎤ ⎡ 7 2 0 −2⎤ ⎢ 0 1 0 –1⎥ ⎢ –2 –2 1 0 ⎥ ⎥ ⎥=⎢ ⎢ ⎢ 0 0 –1 0 ⎥ ⎢ –2 –3 1 0 ⎥ A ⎥ ⎥ ⎢ ⎢ 1⎦ ⎣ 0 0 0 1⎦ ⎣ –3 –1 0 1 0 0 0

R2 + R4 –1 0 1 0 0 –1 0 0

0 7 2 0 –2 0 –5 –3 1 1 A = 0 –2 –3 1 0 1 –3 –1 0 1

R1 + R2, ( 1)R3 1 0 0 0

0 1 0 0

0 0 1 0

0 2 –1 1 –1 0 –5 –3 1 1 A = 0 2 3 –1 0 1 –3 –1 0 1 ⎡ 2 –1 1 –1⎤ ⎢ –5 –3 1 1⎥ ⎥ A1= ⎢ ⎢ 2 3 –1 0 ⎥ ⎥ ⎢ ⎣ –3 –1 0 1⎦

Example 2: Using elementary column transformations, find the inverse of the following matrices:

(i)

1 –1

2 2 1 1

3 –2 1

Solution: (i)

(ii)

1 1 1 1 –2 1 –1 –2 –4 –2 –3 1 1 1 1 0 A = AI3

1 2 2 1 0 0 –1 1 1 = A 0 1 0 3 –2 1 0 0 1 C2 2C1, C3 2C1 1 0 0 1 –2 –2 –1 3 3 = A 0 1 0 3 –8 –5 0 0 1

11.42

Engineering Mathematics

1 , 1 C3 C2 3 3 2 2⎤ ⎡ ⎢1 – 3 – 3⎥ 0 0⎤ ⎡ 1 ⎥ ⎢ 1 ⎥ ⎢ –1 ⎢ 1 1⎥ = A ⎢0 0 ⎥⎥ ⎢ 3 ⎢ ⎥ ⎢ 8 5⎥ 1⎥ – ⎥ ⎢ 3 – ⎢0 0 3 3⎦ ⎣ ⎢⎣ 3 ⎥⎦ C3 0 ⎡1 ⎢0 1 ⎢ ⎢1 8 – ⎢ 3 ⎣3

C2, C1 + C2 2 ⎤ ⎡1 0⎥ ⎢3 – 3 0⎤ ⎥ ⎢ 1 1 1 0 ⎥⎥ = A ⎢ – ⎥⎥ ⎢3 3 3 ⎥ ⎥ ⎢ 1 1⎥ ⎥ ⎢0 0 ⎦ ⎢⎣ 3 ⎥⎦

1 8 C1 – C3 , C2 + C3 3 3 1 3 1 0 0 4 0 1 0 =A 9 0 0 1 1 – 9 1 3 4 A1= 9 1 – 9 (ii)

2 3 5 – 9 8 9



2 3 5 – 9 8 9

0 –



0 ⎡ 3 –6 0 ⎤ 1⎢ 1 ⎥ – = ⎢ 4 –5 –3⎥ 9 3 ⎢⎣ –1 8 3⎥⎦ 1 3

A = AI4 1 1 1 1 1 0 –2 1 –1 –2 0 1 =A –4 –2 –3 1 0 0 1 1 1 0 0 0

0 0 1 0

1 3 1 3

0 0 0 1

Matrices

11.43

C2 C1, C3 C1, C4 1 –2 –4 1

0 3 2 0

0 0 1 –1 –1 –1 1 0 0 1 0 0 =A 1 5 0 0 1 0 0 –1 0 0 0 1 C1 + 2C3, C2

⎡ 1 ⎢ 0 ⎢ ⎢ –2 ⎢ ⎣ 1

0 1 0 0

2C3

0 0⎤ ⎡ –1 1 –1 –1⎤ ⎢ 0 ⎥ 1 0⎥ 1 0 0 ⎥⎥ = A⎢ ⎢ 2 –2 1 0 ⎥ 1 5⎥ ⎥ ⎢ ⎥ 0 –1⎦ ⎣ 0 0 0 1⎦

C3 C2 1 0 0 0 1 0

0 0

–2 0 1 5 1 0 0 –1 C4 5C3 1 0 0 0 0 1 0 0 –2 0 1 0 1 0 0 –1 ⎡1 ⎢0 ⎢ ⎢0 ⎢ ⎣0

C1

=A

=A

–1 0

1 –2 –1 1 –1 0

2 –2 0 0 –1 0

3 0

0 1

1 –2 1 –1

2 –2 0 0

9 5

3 –15 0 1

C1 + 2C3 0 0 0⎤ 1 –2 –9 ⎤ ⎡ −5 ⎥ ⎢ −2 1 0 0⎥ 1 –1 –5⎥⎥ = A⎢ ⎢ 8 –2 3 15⎥ 0 1 0⎥ ⎥ ⎥ ⎢ 0 0 1⎦ ⎣ 0 0 0 –1⎦ 1 –2 –9⎤ ⎡ −5 ⎢ −2 1 –1 –5⎥⎥ A1= ⎢ ⎢ 8 –2 3 15⎥ ⎥ ⎢ ⎣ 0 0 0 –1⎦

Exercise 11.5 1. Using elementary row transformations, find the inverse of the following matrices:

(i)

8 4 3 2 1 1 1 2 1

Engineering Mathematics

11.44

⎡ 5 ⎢ (ii) ⎢ 0 ⎢⎣ –5 2 3 (iii) 2 4

(iv)

(v)

0 1 2 2

–1 5⎤ 2 0 ⎥⎥ 3 −15⎥⎦ 4 3 2 6 5 2 5 2 –3 5 14 14 1 1 2 3

2 2 2 3

2 3 3 3

–1 –3 3 –1 1 1 –1 0 2 –5 2 –3 –1 1 0 1

⎡ ⎤ ⎡ 1 –2 –1⎤ ⎢ Ans. : (i) 1 ⎢ 1 –5 2 ⎥ ⎥ ⎢ ⎥ ⎥ 3⎢ ⎢ ⎥ ⎣ –3 12 0 ⎦ ⎢ ⎥ – 10 4 9 ⎤ ⎡ ⎢ ⎥ 1⎢ 15 –4 –14 ⎥ (ii) ⎢ ⎥ ⎥ 5⎢ ⎢ ⎥ –5 1 6⎦ ⎣ ⎢ ⎥ ⎢ 64 18 ⎤ ⎥ ⎡ ⎢ ⎢ –23 29 – 5 – 5 ⎥ ⎥ ⎢ ⎢ ⎥⎥ 26 7 ⎥⎥ ⎢ ⎢ 10 –12 ⎢ 5 5 ⎥⎥ (iii) ⎢⎢ ⎢ 6 2 ⎥⎥ ⎢ ⎢ 1 –2 ⎥⎥ 5 5 ⎥⎥ ⎢ ⎢ 3 1 ⎥⎥ ⎢ ⎢ 2 –2 ⎢ ⎢⎣ 5 5 ⎥⎦ ⎥ ⎢ ⎥ ⎢ ⎥ ⎡ –3 3 –3 2 ⎤ ⎢ ⎥ ⎢ 3 –4 4 –2 ⎥ (iv) ⎢ ⎢ ⎥ ⎥ ⎢ ⎥ ⎢ –3 4 –5 3⎥ ⎢ ⎥ ⎢⎣ 2 –2 3 –2 ⎥⎦ ⎢ ⎥ ⎡ 0 2 1 3⎤ ⎢ ⎥ ⎢ 1 1 –1 –2 ⎥ ⎢ ⎥ (v) ⎢ ⎥ ⎢ ⎥ 1 2 0 1 ⎢ ⎥ ⎢ ⎥ ⎢⎣ –1 1 2 6 ⎥⎦ ⎢⎣ ⎥⎦

2. Using elementary column transformations, find the inverse of the following matrices: (i)

3 –3 4 2 –3 4 (ii) 0 –1 1

1 –1 1 2 2 –3 1 –4 9

(iii)

1 2 3 2 3 0 0 1 2

(iv)

2 –6 –2 –3 5 –13 –4 –7 –1 4 1 2 0 1 0 1

(v)

2 0 0 –4 2 6 0 –16 1 0 3 –2 0 0

–5 10

1 –1 0 Ans. : (i) –2 3 –4 –2

3

3

6 5 1 1 (ii) –21 8 5 17 –10 3 4 6 –1 –9 1 (iii) –4 2 6 4 2 –1 –1 –2 1 (iv) –4 –1 5 1 (v) 0 1

1 0 1 0 2 –1 1 –3 1 0 –2 2 0 1 0 0

0 0 2 0

2 2 1 1

Matrices

3. Find the matrix A if

11.45

⎡ ⎡ 0 ⎢ ⎢ ⎢ Ans. : ⎢ 1 ⎢ ⎢ 1 ⎢ ⎢ ⎢⎣ ⎣ –1

–1 –3 –3 –1 1 1 –1 0 A1= 2 –5 2 –3 –1 1 0 1

2 1 3⎤ ⎤ ⎥ 1 –1 –2⎥⎥ ⎥ 2 0 1⎥ ⎥ ⎥⎥ 1 2 6 ⎦ ⎥⎦

11.7 RANK OF A MATRIX The rank of a matrix A is said to be r if it possesses the following properties: (i) There is at least one minor of order r which is non-zero. (ii) Every minor of order greater than r is zero. Rank of matrix A is denoted by r (A). Note: (1) The rank of a matrix is less than or equal to r, if all (r + 1) rowed minors of the matrix are zero. (2) The rank of a matrix is greater than or equal to r, if there is at least one r-rowed minor of the matrix which is not equal to zero. (3) The rank of a null matrix is zero. (4) The rank of a non-singular square matrix is always equal to its order. e.g., consider the matrix

1 2 3 A= 3 4 5 4 5 6

⎡1 2 |A| = ⎢ 3 4 ⎢ ⎢⎣ 4 5 Therefore, rank of A is less than 3. There is 4 5 0, Hence, rank of A, i.e., r (A) = 2 5 6

3⎤ 5 ⎥⎥ = 0 6 ⎥⎦ at least one minor of A of order 2, i.e.,

Result: (1) The rank of a matrix remains unchanged by elementary transformations. (2) The rank of the transpose of a matrix is same as that of the original matrix. (3) The rank of the product of two matrices cannot exceed the rank of either matrix. i.e. r (AB) r (A) or r (AB) r (B)

11.7.1 Echelon Form of a Matrix A matrix A is said to be in echelon form if (i) Every zero row of matrix A occurs below a non-zero row (ii) The number of zeros before the first non-zero element in a row is less than the number of such zeros in the next row

Engineering Mathematics

11.46

The rank of a matrix in echelon form is equal to the number of non-zero rows of the matrix. 0 1 3 –1 e.g., A= 0 0 1 4 0 0 0 0 r (A) = 2

11.7.2 Reduction to Normal Form ⎡ I 0⎤ n can be reduced to the form ⎢ r ⎥ by elementary trans⎣ 0 0⎦ formation where r is the rank of the matrix. This form is known as normal form or first cannonical form of a matrix. Any matrix of order m

Results: (1) The rank of a matrix A of order m n is r if and only if it can be reduced to the ⎡ I 0⎤ normal form ⎢ r ⎥ by elementary transformations. ⎣ 0 0⎦ (2) If A be an m such that

n matrix of rank r, then there exists non-singular matrices P and Q ⎡Ir PAQ = ⎢ ⎣0

0⎤ 0⎥⎦

11.7.3 Equivalence of Matrices If B be an m n matrix obtained from an m n matrix A by elementary transformations of A, then A is called equivalent to B. Symbolically, we can write A B. Example 1: Find the rank of the following matrices: (i)

2 3 4 4 3 1

(ii)

3 5 7

1 2 4

(iii)

4 8

2 4

3 6 3 –2 –1 – 2

Solution: (i)

1 2 3 2 3 4

(iv)

1 2

2 –1 –4 4 3 5

–1 –2

6 –7

2 3 4 A= 4 3 1 1 2 4

Matrices

11.47

2 3 4 |A| = 4 3 1 1 2 4 = 2(12 = 5 0

2) 3(16

1) + 4(8

3)

A is a non-singular matrix of order 3. Hence, r (A) = 3 1 2 3 (ii) A= 2 3 4 3 5 7 1 2 3 |A| = 2 3 4 3 5 7 = 1(21 =0

20)

2(14

Therefore, rank of A is less than 3. The minor of order 2 is

12) + 3(10 1 2 2 3

9)

0.

r (A) = 2

Hence,

3⎤ ⎡ 4 2 ⎢ 8 4 6 ⎥⎥ (iii) A= ⎢ ⎢ 3⎥ ⎢ –2 –1 – ⎥ ⎣ 2⎦ 4 2 3 8 4 6 |A| = 3 –2 –1 – 2 = 4( 6 + 6) 2( 12 + 12) + 3( 8 + 8) =0 Therefore, rank of A is less than 3. Consider all the minors of A of order 2, i.e., 4 2 2 3 4 3 = 0, = 0, = 0, 8 4 4 6 8 6 2

3 3 = 0, –1 – 2

4 2 = 0, –2 1

4

3 3 =0 –2 – 2

All the minors of A of order 2 are zero. Therefore, rank of A is less than 2.

11.48

Hence,

Engineering Mathematics r (A) = 1 1 2 –1 –4 A= 2 4 3 5 –1 –2 6 –7

(iv)

Consider all the minors of A of order 3, i.e., 1 2 –1 2 4 3 = 0, –1 –2 6

2 –1 –4 4 3 5 = 0, –2 6 –7

1 2 –1

2 –4 4 5 = 0, 2 –7

1 2 –1

1 –4 3 5 = 120 6 –7

One minor of rank 3 is not equal to zero. Hence, r (A) = 3 3– x 2 2 Example 2: For what value of x, will the matrix A = 1 4– x 0 –2 –4 1 – x of rank (i) equal to 3 (ii) less than 3. 3– x 2 2 1 4 – x 0 Solution: |A| = –2 –4 1 – x = (3 x)[(4 x) (1 x) 0] 2(1 = (3 x)(4 x)(1 x) + 2(3 x) = (3 x) (x2 5x + 6) = (3 x) (x 3) (x 2) = (x 3)2 (x 2) (i) r (A) = 3 if |A| 0 (x 3)2 (x 2) 0 x 2, 3 (ii) r (A) < 3 if |A| = 0 (x 3)2 (x 2) = 0 x = 2, 3

x) +2( 4 + 8

2x)

Example 3: Find the value of P for which the following matrix A will be of (i) rank one (ii) rank two (iii) rank three.

3 A= P P Solution:

3 P P |A| = P 3 P P P 3

P 3 P

P P 3

be

Matrices = 3(9 = (3 = (P = (P = (P

(i) If P = 3,

11.49

P2) P(3P P2) + P(P2 3P) P) (9 + 3P P2 P2) 3) (2P2 3P 9) 3) (P 3)(2P + 3) 3)2 (2P + 3)

3 3 3 A= 3 3 3 3 3 3 R2 R1,

R3

R1

3 3 3 0 0 0 0 0 0 r (A) = 1 Rank of A will be 1 if P = 3 (ii) Rank of A will be 2 if |A| = 0 but P 3 (P 3)2 (2P + 3) = 0 but P 3 2P + 3 = 0 3 P= 2 (iii) Rank of A will be 3 if |A| 0 (P 3)2 (2P + 3) 0 3 P 3, P – 2 Example 4: Determine the value of b such that the rank of A is 3 where

A=

1 1 –1 0 4 4 –3 1 b 2 9 9

Solution:

2 2 b 3 1 4 A= b 9

1 –1 0 4 –3 1 2 2 2 9 b 3

Rank of A will be 3 if |A| = 0 and at least one minor of A of order 3 must be non-zero. By elementary transformation, C2 C1 and C3 + C1. 1 4

0 0

0 1

0 1

b 2–b b+2 2 9 0 b+9 3

11.50

Engineering Mathematics

1 0 0 4 0 1 |A| = b 2–b b+2 9 0 b+9

0 0 1 1 1 = 2–b b+2 2 2 0 b+9 3 3

= (2 b) (–3 + b + 9) = (2 b) (b + 6) When |A| = 0, (2 b) (b + 6) = 0 b = 2, 6 For b = 2, 6, one of the minor of order 3, 4 –3 1 2 2 2 0 9 2 3 Example 5: Find the rank of the following matrices by reducing to echelon form: 3 –1 ⎤ ⎡ 1 2 3 –2 0 –1 5 3 14 4 ⎢ − 2 − 1 − 3 –1 ⎥ 0 2 2 1 ⎥ (i) 0 1 2 1 (ii) ⎢ (iii) ⎢ 1 0 1 1⎥ 1 –2 –3 2 1 –1 2 0 ⎢ ⎥ – 0 1 1 1 0 1 2 1 ⎣ ⎦

Solution: (i)

5 A= 0

3 14 4 1 2 1

1 –1

2 0

R13 1 –1 0 1 5 R3

2 0 2 1

3 14 4 5R1

1 –1 2 0 0 1 2 1 0 R3

8 4 4 8R2

1 –1 2 0 0 1 2 1 0 0 –12 –4 The equivalent matrix is in echelon form. Number of non-zero rows = 3 r (A) = 3

Matrices

(ii)

1 2 3 –1 –2 –1 –3 –1 A= 1 0 1 1 0 1 1 –1 R2 + 2R1, R3

R1

1 2 3 –1 0 3 3 –3 0 –2 –2 2 0 1 1 –1 1 R 3 2

R3 +

2 R2, R4 3

⎡1 ⎢0 ⎢ ⎢0 ⎢ ⎣0

2 3 –1⎤ 3 3 –3⎥⎥ 0 0 0⎥ ⎥ 0 0 0⎦

The equivalent matrix is in echelon form. Number of non-zero rows = 2 r (A) = 2

(iii)

A=

3 –2 0 2

0 –1 2 1

1 –2 –3 0 1 2

2 1

R13 1 –2 –3 2 0 2 2 1 3 –2 0 –1 0 1 2 1

R3

3R1, R24

1 –2 –3

2

0

1

2

1

0

4

9 –7

0

2

2

R3

1

4R2, R4 2R2

11.51

Engineering Mathematics

11.52

1 –2 –3 2 0 1 2 1 0 0 1 –11 0 0 –2 –1

R4 + 2R3 1 –2 –3 2 0 1 2 1 0 0 1 –11 0 0 0 –23 The equivalent matrix is in echelon form. Number of non-zero rows = 4 r (A) = 4 Example 6: Find the rank of the following matrices by reducing to normal form: ⎡ − 1 2 –1 –2 ⎤ (i) ⎢⎢ –2 5 3 0⎥⎥ ⎢⎣ 1 0 1 10⎥⎦

(iii)

1 –1 2 –3 4 1 0 2 0 0

3 0 1 0

4 2

1 2 3 4

(ii)

–1 3 0 –1

–1 0 –2

(iv)

7

1 2 3 –1 –1 –1 –3 –1 1 0

0 1

1 1 1 –1

Solution: R2 − 2 R1 , R3 + R1 (i)

⎡ −1 2 –1 –2⎤ A = ⎢⎢ –2 5 3 0 ⎥⎥ ⎢⎣ 1 0 1 10 ⎥⎦ C4

⎡ 1 2 −1 −2⎤ ⎢0 1 5 4⎥ ⎢ ⎥ ⎢⎣0 2 0 8⎥⎦ C4 + 10C1

4C2

⎡ 1 2 –1 0 ⎤ ⎢0 1 5 0 ⎥ ⎢ ⎥ ⎢⎣0 2 0 0 ⎥⎦

1 2 –1 –10 0 1 5 0 0 2 0 0 C3 + C1 1 2 0 0 0 1 5 0 0 2 0 0

R1

R3

1 0 0 0 0 1 5 0 0 2 0 0

Matrices

11.53

1 1 R2 , R3 5 2 1 0 0 0 0 0 1 0 0 1 0 0

1 R3 2 1 0 0 0 0 0 5 0 0 2 0 0

R2

R23 ⎡1 0 0 0⎤ ⎢0 1 0 0⎥ ⎥ ⎢ ⎢⎣0 0 1 0⎥⎦

[ I3

0]

r (A) = 3 R2 3R1 , R3 (ii)

1 2 3 4

A=

1 2 –1 3 0 –2 3 –10 0 2 –3 10

–1 3 0 –1

–1 0 –2 R3 + R2

7

C2

2C1 , C3 C1 , C4 3C1

1 2 –1 3 0 –2 3 –10 0 0 0 0

1 0 0 0 0 –2 3 –10 0 0 0 0

1 C2 , 2 1 0

C3 C2 , C4 C2

1 1 C3 , C4 3 10 0 0

1 0 0 0 0 1 0 0 0 0 0 0

0 1 1 1 0 0 0 0 I2 0

0 0

r (A) = 2

(iii)

R1

R2 − 4 R1

1 –1 2 –3 4 1 0 2 A= 0 3 0 4 0 1 0 2 C2 1 0 0 0

C1 , C3

⎡ 1 –1 2 –3⎤ ⎢0 5 –8 14 ⎥ ⎥ ⎢ ⎢0 3 0 4 ⎥ ⎥ ⎢ ⎣0 1 0 2⎦

2C1 , C4

0 0 0 5 –8 14 3 0 4 1 0 2

3C1

R24 ⎡1 ⎢0 ⎢ ⎢0 ⎢ ⎣0

0 0 0⎤ 1 0 2⎥⎥ 3 0 4⎥ ⎥ 5 –8 14 ⎦

Engineering Mathematics

11.54

C4

R3 3R2 , R4 5 R2

2C2

1 0

0

0

1 0

0 1 0 0 0 3 0 –2 0 5 –8 4

0

0 1 0 0 0 0 0 –2 0 0 –8 4 1 1 C3 , C4 2 8 1 0 0 0 0 1 0 0 0 0 1 0 0 0 –2 1

C34 1 0 0 0

0

0 0 0 1 0 0 0 –2 0 0 4 –8

R4 + 2 R3 1 0 0 0 0 1 0 0 0 0 1 0

[ I4 ]

0 0 0 1 r (A) = 4 R2

(iv)

A=

1 2 3 –1 –1 –1 –3 –1 1 0

0 1

1 0

2 1 1 1 –1 –1

0 –2 0 1 R1

C2

0 0

R3 , R2

0 0 R4

1 0 1 0 0 0 –1 0 0 –2 0 0 0 1 0 0

R1

1 2 3 –1 0 1 0 –2 0 –2 –2 2

1 1 1 –1

C3 C2 , C4

R1 , R3

0

1

1 –1

C 4 − C3 ⎡1 2 1 ⎢0 1 –1 ⎢ ⎢0 –2 0 ⎢ 1 0 ⎣0

0⎤ 0 ⎥⎥ 0⎥ ⎥ 0⎦

R1 + R2 , R3 + 2 R4 ⎡1 ⎢0 ⎢ ⎢0 ⎢ ⎣0

0 0 0 −1 0 0 1 0

0⎤ 0 ⎥⎥ 0⎥ ⎥ 0⎦

Matrices

R34

R24 1 0 0 0

11.55

0 0 1 0 0 0 0 –1

0 0 0 0

1 0 0 0

0 0 0 1 0 0 0 –1 0 0 0 0

− R3 ⎡1 ⎢0 ⎢ ⎢0 ⎢ ⎣0

0 1 0 0

0 0 1 0

0⎤ 0⎥⎥ 0⎥ ⎥ 0⎦

[ I3

0]

r (A) = 3 Example 7: Find non-singular matrices P and Q such that PAQ is in the normal form and hence, find q (A) for the following matrices: (i)

1 1 1 1 –1 –1 3

(iii)

1

1 2 3 2 1 4

(ii)

1 0 –2 2 3 –4 3 3 –6

1 4 3

(iv)

3 0 5 –10

2 1 1 3 1 0 1 2 3 1 2 5

1 1 1 A = 1 –1 –1

Solution: (i)

3

1

1

A = I3 AI3 1 1 1 1 0 0 1 0 0 1 –1 –1 = 0 1 0 A 0 1 0 3 R2

1

1

R1, R3

0 0 1

0 0 1

3R1

1 1 1 1 0 0 1 0 0 0 –2 –2 = –1 1 0 A 0 1 0 0 –2 –2 –3 0 1 0 0 1 1 1 – R2 , – R3 2 2

Engineering Mathematics

11.56

1 0 0 1 1 1 1 0 0 1 1 0 1 1 = – 0 A 0 1 0 2 2 0 1 1 0 0 1 3 1 0 – 2 2 C3

C2

1 0 0 1 1 0 1 0 0 1 1 0 1 0 = – 0 A 0 1 –1 2 2 0 1 0 0 0 1 3 1 0 – 2 2 R1

R2, R3

R2

1 1 2 2 1 0 0 1 1 0 1 0 = – 2 2 0 0 0 1 1 2 I2 0

0 1 0

0 A 0 1 –1 0 0 1 1 – 2

0 = PA Q 0 1 2 1 P= 2

1 2 1 – 2 1 1 2

0 1 0 0 0 , Q = 0 1 –1 0 0 1 1 – 2

r (A) = 2 1 0 –2 A = 2 3 –4 3 3 –6

(ii) A = I3 A I3

1 0 –2 1 0 0 1 0 0 2 3 –4 = 0 1 0 A 0 1 0 3 3 –6 0 0 1 0 0 1 1 – C3 2

0

Matrices

11.57

1 0 1 1 0 0 1 0 0 2 3 2 = 0 1 0 A 0 1 0 3 3 3 0 0 1 1 0 0 – 2 C3 C1 1 0 0 1 0 0 1 0 –1 2 3 0 = 0 1 0 A 0 1 0 3 3 0 0 0 1 1 0 0 – 2 R2 2R1, R3 R2 1 0 0 1 0 0 1 0 –1 0 3 0 = –2 1 0 A 0 1 0 1 0 0 0 –1 1 1 0 0 – 2 1 R , R R1 3 2 3 1 0 0 1 0 0 1 0 –1 2 1 0 1 0 = – 0 A 0 1 0 3 3 0 0 0 1 –1 –1 1 0 0 – 2 I2 0 = PA Q 0 0 1 0 0 1 0 –1 2 1 0 ,Q= 0 1 P= – 0 3 3 1 –1 –1 1 0 0 – 2 r (A) = 2 1 2 3 4 A= 2 1 4 3 3 0 5 –10

(iii) A = I3 A I4 1 2 3

4

1 0 0

1 0 0 0 0 1 0 0

2 1 4 3 = 0 1 0 A 0 0 1 0 3 0 5 –10 0 0 1 0 0 0 1

Engineering Mathematics

11.58

R2

2R1, R3

3R1

⎡1 3 4⎤ ⎡ 1 0 0⎤ ⎢ ⎡1 2 ⎢0 –3 –2 –5⎥ = ⎢ –2 1 0 ⎥ A ⎢0 ⎢0 –6 –4 –22 ⎥ ⎢ –3 0 1⎥ ⎢0 ⎣ ⎦ ⎣ ⎦ 0 ⎣ C2 2C1, C3 3C1, C4 4C1 ⎡1 0⎤ ⎡ 1 0 0⎤ ⎢ ⎡1 0 0 ⎢ 0 –3 –2 –5⎥ = ⎢ –2 1 0 ⎥ A ⎢0 ⎢ 0 –6 –4 –22 ⎥ ⎢ –3 0 1⎥ ⎢0 ⎣ ⎦ ⎣ ⎦ 0 ⎣

R3

0 1 0 0

0 0 1 0

0⎤ 0⎥ 0⎥ ⎥ 1⎦

– 2 –3 – 4 ⎤ 1 0 0⎥ 0 1 0⎥ ⎥ 0 0 1⎦

2R2

⎡1 0⎤ ⎡ 1 0 0⎤ ⎢ ⎡1 0 0 0 ⎢0 –3 –2 –5⎥ = ⎢ –2 1 0⎥ A ⎢ ⎢0 0 0 –12 ⎥ ⎢ 1 –2 1⎥ ⎢0 ⎣ ⎦ ⎣ ⎦ 0 ⎣ 1 1 1 – C2, – C3, – C4 2 3 5 ⎡ ⎢1 ⎢ ⎡1 0 0 0 ⎤ ⎡ 1 0 0 ⎤ ⎢0 ⎢0 1 1 1 ⎥ = ⎢ –2 1 0⎥ A ⎢ ⎢ ⎢ ⎥ ⎢ 12 1 –2 1⎥⎦ ⎢0 ⎢0 0 0 ⎥ ⎣ ⎢ 5⎦ ⎣ ⎢0 ⎢⎣ C3 C2, C4 C3 ⎡ ⎢1 ⎢ ⎡1 0 0 0 ⎤ ⎡ 1 0 0 ⎤ ⎢0 ⎢0 1 0 0 ⎥ = ⎢ –2 1 0⎥ A ⎢ ⎢ ⎢ 12 ⎥ ⎢⎣ 1 –2 1⎥⎦ ⎢0 ⎢0 0 0 ⎥ ⎢ 5⎦ ⎣ ⎢0 ⎢⎣ 5 C 12 4 ⎡ ⎢1 ⎢ ⎡1 0 0 0 ⎤ ⎡ 1 0 0 ⎤ ⎢ 0 ⎢0 1 0 0 ⎥ = ⎢ –2 1 0⎥ A ⎢ ⎢0 0 0 1 ⎥ ⎢ 1 –2 1⎥ ⎢0 ⎣ ⎦ ⎣ ⎦ ⎢ ⎢ ⎢0 ⎣

–2 –3 –4 ⎤ 1 0 0⎥ 0 1 0⎥ ⎥ 0 0 1⎦

4⎤ 5⎥ ⎥ 0 0⎥ ⎥ ⎥ 1 0 − 0⎥ 2 ⎥ 1 0 0 – ⎥ 5 ⎥⎦

2 3 1 – 3

2 3 1 – 3 0 0

2 3 1 – 3 0 0

3 2

5 7⎤ − ⎥ 6 10 ⎥ 1 0⎥ 3 ⎥ 1 1⎥ – ⎥ 2 2⎥ 1 0 – ⎥ 5 ⎥⎦

7⎤ 24 ⎥ ⎥ 0⎥ ⎥ 5⎥ – ⎥ 24 ⎥ 1 0 – ⎥ 12 ⎦

5 6 1 3 1 2



Matrices

11.59

C34 2 7 – 3 24 1 1 0 0 0 1 0 0 0 – 0 3 0 1 0 0 = –2 1 0 A 5 0 0 – 0 0 1 0 1 –2 1 24 1 0 0 – 12

5 6 1 3 1 2

1

[ I3

0] = P A Q 2 3 1 0 – 3 1

1 P = –2

0 0 1 0 , Q=

1 –2 1

0

1 2 1 1 3 1 0 0 0 1 0 1 2 = 0 1 0 A 0 3 1 2 5 0 0 1 0 R12

0 1 0 0

0 0 1 0

0 0 0 1

1 1 0 1 2 0 1 0 0 2 1 1 3 = 1 0 0 A 0 3 1 2 5 0 0 1 0

0 1 0 0

0 0 1 0

0 0 0 1

2R1, R3

3R1

⎡1 1 0⎤ ⎢ ⎡ 1 0 1 2⎤ ⎡0 ⎢0 1 –1 –1⎥ = ⎢ 1 –2 0 ⎥ A ⎢0 ⎢ ⎥ ⎢ ⎥ ⎢0 ⎢⎣0 1 –1 –1⎥⎦ ⎢⎣0 −3 1⎥⎦ ⎢ ⎣0

5 24 1 – 12

0

A = I3 A I4

0 0 0⎤ 1 0 0 ⎥⎥ 0 1 0⎥ ⎥ 0 0 1⎦

7 24 0

0 –

2 1 1 3 A= 1 0 1 2 3 1 2 5

(iv)



0

r (A) = 3

R2

0

5 6 1 3 1 2 0

Engineering Mathematics

11.60

C3 + C2, C4 + C2 ⎡1 1 0⎤ ⎢ ⎡1 0 1 2 ⎤ ⎡ 0 ⎢ 0 1 0 0 ⎥ = ⎢ 1 –2 0 ⎥ A ⎢ 0 ⎢ ⎥ ⎢ ⎥ ⎢0 ⎣ 0 1 0 0 ⎦ ⎣ 0 0 –3 ⎦ ⎢ 0 ⎣

C3

C1, C4

0 1 1 0

0⎤ 1⎥ ⎥ 0⎥ 1 ⎥⎦

2C1

⎡1 1 0⎤ ⎢ ⎡1 0 0 0 ⎤ ⎡ 0 ⎢0 1 0 0 ⎥ = ⎢ 1 –2 0 ⎥ A ⎢0 ⎢ ⎥ ⎢ ⎥ ⎢0 ⎣0 1 0 0 ⎦ ⎣0 0 –3⎦ ⎢0 ⎣

R3

0 1 0 0

0 –1 –2 ⎤ 1 1 1⎥ ⎥ 0 1 0⎥ 0 0 1⎥⎦

R2

⎡1 1 0⎤ ⎢ ⎡1 0 0 0 ⎤ ⎡ 0 ⎢ 0 1 0 0 ⎥ = ⎢ 1 –2 0 ⎥ A ⎢ 0 ⎢ ⎥ ⎢ ⎥ ⎢0 ⎣ 0 0 0 0 ⎦ ⎣ –1 2 –3 ⎦ ⎢ 0 ⎣ ⎡I2 ⎢0 ⎣

0 –1 –2 ⎤ 1 1 1⎥ ⎥ 0 1 0⎥ 0 0 1⎥⎦

0⎤ = PA Q 0 ⎥⎦ 1 0⎤ ⎡ 0 P = ⎢ 1 –2 0 ⎥ , Q = ⎢ ⎥ ⎣ –1 2 –3 ⎦

⎡1 ⎢0 ⎢ ⎢0 ⎢⎣0

0 –1 –2 ⎤ 1 1 1⎥ ⎥ 0 1 0⎥ 0 0 1⎥⎦

r (A) = 2 Exercise 11.6 1. Find the ranks of A, B, AB and verify that rank of the product of two matrices cannot exceed the rank of either matrix. 2 4 1 1 2 3 A= 3 6 2 , B= 3 1 2 4 8 3 4 3 5 2. Find the possible values of P, for which the following matrix A will have (i) rank 1 (ii) rank 2 (iii) rank 3. ⎡P A = ⎢⎢ 2 ⎢⎣ P

P P 2

2⎤ P ⎥⎥ P ⎥⎦

⎡ Ans. : (i) P = 2 (ii) P = −2⎤ ⎢ (iii) P ≠ −1, P ≠ 2 ⎥⎦ ⎣ 3. Find the rank of x –1 x +1 x A = –1 x 0 , where x is real. 0 1 1 [Ans. : 3] 1 0 1 4. If A = 0 x 1 , prove that rank –1 –1 x of A is 3, where x is a real number.

Matrices 5. Find the value of l for which rank of 3 1 2 the matrix A = –1 4 5 7 2 (i) is less than 3 (ii) equal to 3 6. Find the rank of the following matrices by reducing to echelon form:

(i)

(ii)

1 1 –1 1 1 –1 2 –1 3 1 0 1 4 8

2 4

3 6

–2 –1 –1.5

(iii)

(iv)

1 2 –2 3 2 5 –4 6 –1 –3 2 –2 2 4 –1 6 0 0 3 1

1 –3 –1 0 1 1 1 0 2 1 2 0

(v)

3 –2 0 –1 –7 0 2 2 1 –5 1 –2 –3 –2 1 0 1 2 1 6

(vi)

1 2 –1 4 2 4 3 5 –1 –2 6 7 ⎡ Ans. : (i) 2 (ii) 1 (iii) 4 ⎤ ⎢ (iv) 2 (v) 4 (vi) 2⎥⎦ ⎣

11.61

7. Find the rank of the following matrices by reducing to normal form:

(i)

1 2 1 0

2 1 0 1

3 –1 3 1 (ii) 1 1 1 1

1 2 3 6

(iii)

0 1 3 1

(iv)

1 0 2 1 0 1 –2 1 1 –1 4 0 –2 2 8 0

(v)

3 4 1 1 2 4 3 6 –1 –2 6 4 1 –1 2 –2

(vi)

2 4 2 8

3 3 1 7

0 2 3 5

1 –3 –1 0 1 1 1 0 2 1 –2 0

2 3 1 4 5 2 3 0 9 8 0 8 ⎡ Ans. : (i) 2 (ii) 4 (iii) 3 ⎤ ⎢ (iv) 3 (v) 4 (vi) 3⎥⎦ ⎣

8. Find non-singular matrices P and Q such that PAQ is in normal form. Also find their rank. (i)

2 1 –3 –6 3 –3 1 2 1 1 1 2

(ii)

1 2 3 –2 2 –2 1 3 3 0 4 1

Engineering Mathematics

11.62

(iii)

1 –1 –1 1 1 1 (iv) 3 1 1

(v)

1 –1 2 –1 4 2 –1 2 2 2 –2 0

(vi)

1 2 1 0 –2 4 3 0 1 0 2 –8

1 2 3 2 –1 0 3 1 2

⎤ ⎡ Ans. : ⎥ ⎢ 0 0 1 ⎡ ⎤ ⎥ ⎢ ⎥, ⎥ ⎢ (i) P = ⎢ −1 0 2 ⎢ ⎥ ⎥ ⎢ ⎢ 3 1 9⎥ ⎥ ⎢ ⎢⎣ − ⎥ ⎢ 14 28 28 ⎥⎦ ⎥ ⎢ ⎡ 1 −1 4 0 ⎤ ⎥ ⎢ ⎢0 1 −5 0 ⎥ ⎥ ⎢ ⎥ , rank = 3 ⎥ Q=⎢ ⎢ ⎢0 0 1 −2⎥ ⎥ ⎢ ⎢ ⎥ ⎥ ⎢ 0 0 1 0 ⎣ ⎦ ⎥ ⎢ ⎥ ⎢ ⎡ 1 0 0⎤ ⎥ ⎢ ⎢ ⎥ ⎥ ⎢ (ii) P = ⎢ −2 1 0 ⎥ , ⎥ ⎢ ⎢⎣ −1 −1 −1⎥⎦ ⎥ ⎢ 1 4 1⎤ ⎡ ⎥ ⎢ − − ⎥ ⎢1 ⎥ ⎢ 3 3 3 ⎢ ⎥ ⎥ ⎢ 7⎥ ⎢0 − 1 – 5 ⎢ Q=⎢ , rank = 2⎥ ⎥ ⎢ 6 6 6⎥ ⎢ ⎥ ⎥ ⎢ 0 1 0⎥ ⎢0 ⎥ ⎢ ⎢⎣0 ⎥ ⎢ 0 0 1⎥⎦ ⎥ ⎢ 0 0⎤ ⎡ 1 ⎥ ⎢ ⎢ 1 ⎥ ⎥ ⎢ 1 ⎥ ⎢(iii) P = ⎢ − 0⎥ , 2 ⎢ 2 ⎥ ⎥ ⎢ ⎢ 1 ⎥ ⎢ 1 1⎥ − ⎢− ⎥ ⎥ ⎢ ⎣ 4 2 4⎦ ⎥ ⎢ ⎥ ⎢ ⎡ 1 1 0⎤ ⎥ ⎢ ⎢ ⎥ Q = ⎢0 1 −1⎥ , rank = 2 ⎥ ⎢ ⎥⎥ ⎢⎢⎣ ⎣⎢0 0 1⎦⎥ ⎦⎥ ⎢

⎤ 0 0⎤ ⎡ ⎡1 ⎥ ⎢ ⎢2 ⎥ 1 ⎥ ⎢(iv) P = ⎢ − 0⎥ , ⎥ ⎢ 5 ⎢5 ⎥ ⎥ ⎢ ⎢1 ⎥ − 1 1 ⎣ ⎦ ⎥ ⎢ ⎥ ⎢ 3 ⎡ ⎤ ⎥ ⎢ ⎢ 1 −2 − 5 ⎥ ⎥ ⎢ ⎢ ⎥ ⎥ ⎢ 6 Q = ⎢0 1 − ⎥ , rank = 3 ⎥ ⎢ ⎢ ⎥ 5 ⎥ ⎢ ⎢ ⎥ 1⎥⎦ ⎥ ⎢ ⎢⎣ 0 0 ⎥ ⎢ 0⎤ ⎡ 1 0 ⎥ ⎢ ⎢ 2 1 ⎥ ⎥ ⎢ 1 − ⎥, ⎥ ⎢ ( v) P = ⎢ − 2⎥ ⎢ 3 6 ⎥ ⎢ ⎢ 1 1 ⎥ ⎢ 1⎥ ⎢− − ⎥ ⎥ ⎢ ⎢⎣ 3 3 2 ⎥⎦ ⎥ ⎢ ⎥ ⎢ 1⎤ ⎡ ⎥ ⎢ ⎢1 1 0 − 2⎥ ⎥ ⎢ ⎥ ⎢ 3⎥ ⎥ ⎢ ⎢0 1 −1 , rank = 3 ⎥ ⎢ ⎥ ⎢ 2 ⎥ ⎢ ⎥ ⎢ 1⎥ ⎥ ⎢ ⎢0 0 0 ⎥ ⎢ ⎢⎣0 0 1 0 ⎥⎦ ⎥ ⎢ ⎥ ⎢ ⎡ 1 0 0⎤ ⎥ ⎢ ⎢ ⎥ 1⎥ , ⎥ ⎢( vi) P = ⎢ −1 0 ⎥ ⎢ ⎢⎣ 7 1 −5⎥⎦ ⎥ ⎢ ⎥ ⎢ 4 1 ⎤ ⎡ ⎥ ⎢ ⎢1 −1 − 18 45 ⎥ ⎥ ⎢ ⎢ ⎥ 1 1⎥ ⎥ ⎢ ⎢0 0 − ⎥ ⎢ ⎢ 18 18 ⎥ Q=⎢ ⎢ ⎥ , rank = 3⎥ 2 4 ⎥ ⎥ ⎢ ⎢0 1 ⎥ ⎢ ⎢ 18 45 ⎥ ⎥ ⎢ ⎢ ⎥ 1 ⎥ ⎢ ⎢0 0 ⎥ 0 ⎢⎣ ⎢⎣ ⎥⎦ 40 ⎥⎦ ⎢ ⎥

Matrices

11.63

11.8 NON-HOMOGENEOUS LINEAR EQUATIONS a11x1 + a12x2 + … + a1n xn = b1 a21x1 + a22x2 + … + a2n xn = b2 .…………………………...… ……………………………… am1x1 + am2x2 + … + amnxn = bm is a system of m non-homogeneous equations in n unknowns x1, x2, …, xn. Writing these equations in matrix form, AX = B where A is any matrix of order m n, X is any vector of order n 1 and B is any vector of order m 1. When the system of equations has one or more solutions, the system is said to be consistent, otherwise it is inconsistent. Suppose

⎡ a11 a12 ⎢a a22 The matrix [A : B] = ⎢ 21 ⎢ ..... ..... ⎢ ⎣ am1 am 2

b1 ⎤ b2 ⎥⎥ is called the augmented matrix of ..... ..... .....⎥ ⎥ ..... amn bm ⎦ ..... a1n ..... a2 n

the given system of equations.

11.8.1 Condition for Consistency The system of equations AX = B is consistent if and only if the coefficient matrix A and the augmented matrix [A : B] are of the same rank. i.e., r (A) = r [A : B] There are two cases: Case I: If r (A) = r [A : B] = n, number of unknowns, the system has a unique solution. Case II: If r (A) = r [A : B] < n, number of unknowns, the system has infinite solutions. In this case, n-r unknowns called parameters can be assigned arbitrary values. The remaining unknowns can be expressed in terms of these parameters. When r (A) r [A : B], the system is said to be inconsistent and has no solution. Example 1: Discuss the consistency of the system and if consistent, solve the equations: (i) x+y+z=6 (ii) 4x - 2y + 6z = 8 (iii) 2x - 3y + 7z = 5 x + 2y + 3z = 14 x + y - 3z = -1 3x + y - 3z = 13 2x + 4y + 7z = 30 15x - 3y + 9z = 21 2x + 19y - 47z = 32 (iv) 2x - y + z = 9 (v) x1 - x2 + x3 - x4 + x5 = 1 3x - y + z = 6 2x1 - x2 + 3x3 + 4x5 = 2 4x - y + 2z = 7 3x1 - 2x2 + 2x3 + x4 + x5 = 1 -x + y - z = 4 x1 + x3 + 2x4 + x5 = 0

Engineering Mathematics

11.64

Solution: (i) In matrix form, A X =B 1 1 1 x 6 1 2 3 y = 14 2 4 7 z 30 Augmented matrix 1 [A : B] = 1 2

1 2 4

1 3 7

6 14 30

R2

R1, R3

2R1

1 0 0 R3

1 1 1 2 2 5 2R2

6 8 18

1 0

1 1

1 2

6 8

0

0

1

2

r (A) = r [A : B] = 3 (Number of unknowns) Hence, the system is consistent and has a unique solution. x+y+z=6 y + 2z = 8 z=2 Solving these equations, y = 4, x=0 Hence, x = 0, y = 4, z = 2 is the solution of the system. (ii) In matrix form, A X =B 4 –2 6 x 8 1 1 –3 y = –1 15 –3

9

z

21

Augmented matrix [A : B] =

4 1 15 R12

–2 1 –3

6 –3 9

8 –1 21

1 4 15

1 –2 –3

–3 6 9

1 8 21

Matrices

11.65

1 1 R2, R3 2 3 1 1 –3 2 –1 3 5 –1 3 R2

2R1, R3

1 0 0

1 –3 –6

R3

2R2

1 0 0

1 4 7 5R1

–3 9 18

1 6 12

1 –3

–3 9

1 6

0

0

0

r (A) = r [A : B] = 2 < 3 (Number of unknowns) Hence, the system is consistent and has infinite solutions. x + y 3z = 1 3y + 9z = 6 Number of parameters = 3 2 = 1 Let z=t Then y = 3t 2 and x = 1 (3t 2) + 3t = 1 Hence, x = 1, y = 3t 2, z = t is the solution of the system where t is a parameter. (iii) In matrix form, A X =B 2 –3 3 1

7 –3

2 19 –47

x 5 y = 13 z

32

Augmented matrix 2 [A : B] = 3 2 R1 –1 3 2

–3 1 19

7 –3 –47

5 13 32

R2 –4 1 19

10 –3 –47

8 13 32

11.66

Engineering Mathematics

R2 + 3R1, R3 + 2R1 ⎡ –1 ⎢ 0 ⎢ ⎣ 0

–4 –11 11

10 27 –27

–8 ⎤ –11 ⎥ ⎥ 16 ⎦

R3 + R2

–4 ⎡ –1 ⎢ 0 –11 ⎢ 0 ⎣ 0 r (A) = 2 r [A : B] = 3 r [A : B] r (A)

10 27 0

–8 ⎤ –11 ⎥ ⎥ 5⎦

Hence, the system is inconsistent and has no solution. (iv) In matrix form, A X =B ⎡ 2 –1 1⎤ ⎡9 ⎤ ⎢ 3 –1 1⎥ ⎡ x ⎤ ⎢ 6 ⎥ ⎢ ⎥ ⎢ y⎥ = ⎢ ⎥ ⎢ 4 –1 2 ⎥ ⎢ z ⎥ ⎢7 ⎥ ⎢⎣ –1 1 –1⎥⎦ ⎣ ⎦ ⎢⎣ 4 ⎥⎦ Augmented matrix ⎡ 2 ⎢ [A : B] = ⎢ 3 ⎢ 4 ⎢⎣ –1 R14

–1 –1 –1 1

1 1 2 –1

9⎤ 6⎥ ⎥ 7⎥ 4 ⎥⎦

⎡ –1 ⎢ 3 ⎢ ⎢ 4 ⎢⎣ 2

1 –1 –1 –1

–1 1 2 1

4⎤ 6⎥ ⎥ 7⎥ 9 ⎥⎦

R2 + 3R1, R3 + 4R1, R4 + 2R1 ⎡ –1 ⎢ 0 ⎢ ⎢ 0 ⎢⎣ 0

1 2 3 1

R2

2R4

⎡ –1 ⎢ 0 ⎢ ⎢ 0 ⎢⎣ 0

1 0 3 1

–1 –2 –2 –1

4⎤ 18 ⎥ ⎥ 23 ⎥ 17 ⎥⎦

–1 0 –2 –1

4⎤ –16 ⎥ ⎥ 23 ⎥ 17 ⎥⎦

Matrices

11.67

R24 –1 0 0 0

1 1 3 0

–1 –1 –2 0

4 17 23 16

r (A) = 3 r [A : B] = 4 r [A : B] r (A) Hence, the system is inconsistent and has no solution. (v) In matrix form, A X =B x1 1 –1 1 –1 1 1 x2 2 –1 3 0 4 2 x3 = 3 –2 2 1 1 1 x4 1 0 1 2 1 0 x5 Augmented matrix [A : B] =

1 2

–1 –1

1 3

–1 0

1 4

1 2

3 1

–2 0

2 1

1 2

1 1

1 0

R2

2R1, R3

3R1, R4

R1

1 0

–1 1

1 1

–1 2

1 2

1 0

0 0

1 1

–1 0

4 3

–2 0

2 1

R3

R2, R4

R2

1 0

–1 1

1 1

–1 2

1 2

1 0

0 0

0 0

–2 –1

2 1

–4 –2

2 1

R4

1 R3 2 1 1 –2 0

–1 2 2 0

1 2 –4 0

1 0 2 0

1 0 0 0

–1 1 0 0

Engineering Mathematics

11.68

1 R 2 3 1 –1 1 –1 1 1 0 1 1 2 2 0 0 0 –1 1 –2 1 0 0 0 0 0 0 r (A) = r [A : B] = 3 < 5 (Number of unknowns) Hence, the system is consistent and has infinite solutions. x1 x2 + x3 x4 + x5 = 1 x2 + x3 + 2x4 + 2x5 = 0 x3 + x4 2x5 = 1 Number of parameters = 5 3 = 2 Let x4 = t1 x5 = t2 Then x3 = 1 + t1 2t2 x2 = (1 + t1 2t2) 2t1 2t2 = 1 3t1 x1 = 1 + ( 1 3t1) (1 + t1 2t2) + t1 t2 = 1 3t1 + t2 Hence, x1 = 1 3t1 + t2, x2 = 1 3t1, x3 = 1 + t1 2t2, x4 = t1 is the solution of the system where t1 and t2 are parameters. Example 2: Investigate for what values of k and l the equations x + 2y + z = 8 2x + 2y + 2z = 13 3x + 4y + kz = l have (i) no solution (ii) unique solution (iii) many solutions. Solution: In matrix form, A X =B 1 2 1 2 2 2 3 4

x 8 y = 13 z

Augmented matrix 1 2 [A : B] = 2 2 3 4 R2

1 8 2 13

2R1, R3

1 2 0 –2 0 –2

1 0 –3

3R1 8 3 – 24

Matrices

R3

11.69

R2

1 8 ⎤ ⎡1 2 ⎢ 0 –2 0 –3 ⎥ ⎢ ⎥ – 21⎦ ⎣0 0 l – 3 (i) If l = 3, m 21, then r (A) r [A : B] Hence, system is inconsistent and has no solution. (ii) If l 3 and m have any value, then r (A) = r [A : B] = 3 (Number of unknowns) Hence, system is consistent and has unique solution. (iii) If l = 3, m = 21, then r (A) = r [A : B] = 2 < Number of unknowns Hence, the system is consistent and has infinite (many) solutions. Example 3: Determine the values of k for which the following equations are consistent. Also, solve the system for these values of k . x + 2y + z = 3 x+y+z=k 3x + y + 3z = k2. Solution: In matrix form, A X =B ⎡1 2 1⎤ ⎡ x ⎤ ⎡ 3 ⎤ ⎢1 1 1⎥ ⎢ y ⎥ = ⎢ ⎥ ⎢ ⎥⎢ ⎥ ⎢ 2⎥ ⎣3 1 3⎦ ⎣ z ⎦ ⎢⎣ ⎥⎦ Augmented matrix ⎡1 2 1 [A : B] = ⎢ 1 1 1 ⎢ ⎢⎣3 1 3 R2

R1, R3

⎡1 2 1 ⎢0 –1 0 ⎢ ⎢⎣0 –5 0 R3 5R2 ⎡1 2 1 ⎢ 0 –1 0 ⎢ ⎢⎣0 0 0 r (A) = 2 The equations will be consistent if r (A) = r [A : B] i.e. l2 5l + 6 = 0 l = 3 or l = 2 Case I: When l = 3, x + 2y + z = 3 –y = 0

3⎤ ⎥ ⎥ 2 ⎥⎦ 3R1 3 ⎤ –3 ⎥ ⎥ 2 – 9 ⎥⎦

2

⎤ 3 –3 ⎥ ⎥ – 5 + 6 ⎥⎦

11.70

Engineering Mathematics

Let z = t Then x = 3 2(0) t = 3 t Hence, x = 3 t, y = 0, z = t is the solution of the system where t is a parameter. Case II: When l=2 x + 2y + z = 3 y=1 Let z = t Then x = 3 2(1) t = 1 t Hence, x = 1 t, y = 1, z = t is the solution of the system where t is a parameter. Example 4: Show that the system of equations 3x + 4y + 5z = ` 4x + 5y + 6z = a 5x + 6y + 7z = f are consistent only if `, a and f are in arithmetic progression (A.P.). Solution: In matrix form, A X =B ⎡ 3 4 5 ⎤ ⎡ x ⎤ ⎡α ⎤ ⎢4 5 6⎥ ⎢ y ⎥ = ⎢ β ⎥ ⎢ ⎥⎢ ⎥ ⎢ ⎥ ⎢⎣ 5 6 7 ⎥⎦ ⎢⎣ z ⎥⎦ ⎢⎣ γ ⎥⎦ Augmented matrix 3 [A : B] = 4

4 5

5 6

5

6

7

R2

R1, R3

R1

3 4 5 1 1 1



2 2 2



R3

2R2

a ⎡3 4 5 ⎤ ⎢1 1 1 ⎥ b – a ⎢ ⎥ ⎢⎣0 0 0 g – 2 b + a ⎥⎦

r (A) = 2 The system of equations are consistent if r (A) = r [A : B] g 2b+a=0 + b= 2 Hence, a, b and g are in arithmetic progression (A.P.).

Matrices

11.71

Example 5: Show that if k ñ 0, the system of equations 2x + y = a x + ky - z = b y + 2z = c has a unique solution for every value of a, b, c. If k = 0, determine the relation satisfied by a, b, c such that the system is consistent. Find the solution by taking k = 0, a = 1, b = 1, c = -1. Solution: In matrix form, A X =B 2 1 1 0 1

0 –1 2

x a y = b z c

The system has a unique solution if |A| 0 |A| = 2(2l + 1) 1 (2 + 0) 0 4l 0 l 0 Hence, the system has a unique solution if l 0 for any value of a, b, c. If l = 0, the system is either inconsistent or has infinite number of solutions. When l = 0, 2 1 0 x a 1 0 –1 y = b 0

1

2

z

c

R12 1 0 –1 2 1 0 0 1 2

R2

x b y = a z c

2R1

1 0 –1 x b 0 1 2 y = a – 2b 0 1 R3

2

z

c

R2

1 0 –1 0 1 2 0 0 0

x b y = a – 2b z c – a + 2b

x z=b y + 2z = a 2b

11.72

Engineering Mathematics

Let z = t Then

y = a 2b 2t x=b+t Hence, x = b + t, y = a 2b 2t, z = t is the solution of the system where t is a parameter. When a = 1, b = 1, c = 1, z=t y = 1 2t x=1+t Exercise 11.7 1. Discuss the consistency of the system and if consistent, solve the equation: (i) 2x 3y z = 3 x + 2y z = 4 5x 4y 3z = 2 (ii) x + 2y z = 1 x + y + 2z = 9 2x + y z = 2 (iii) 6x + y + z = 4 2x 3y z = 0 x 7y 2z = 7 (iv) 2x y z = 2 x + 2y + z = 2 4x 7y 5z = 2 (v) 2x1 + x2 + 2x3 + x4 = 6 6x1 6x2 + 6x3 + 12x4 = 36 4x1 + 3x2 + 3x3 3x4 = 1 2x1 + 2x2 x3 + x4 = 10 (vi) x + 2y + z = 1 6x + y + z = 4 2x 3y z = 0 x 7y 2z = 7 x y=1 (vii) x+y+z=6 x 2y + 2z = 5 3x + y + z = 8 2x 2y + 3z = 7 (viii) 2x1 + x2 + 5x4 = 4 3x1 2x2 + 2x3 = 2 5x1 8x2 4x3 = 1

⎡ Ans. : ⎢(i) Inconsistent ⎢ ⎢(ii) Consistent ⎢ x = 2, y = 1, z = 3 ⎢ ⎢(iii) Consistent ⎢ x = − 1, y = − 2, z = − 4 ⎢ ⎢(iv) Consistent ⎢ 6+t 2 − 3t ⎢ x= ,y= ,z =t ⎢ 5 5 ⎢ ⎢(v) Consistent ⎢ x1 = 2, x2 = 1, x3 = − 1, x4 = ⎢ ⎢(vi) Consistent ⎢ x = − 1, y = − 2, z = 4 ⎢ ⎢(vii) Consistent ⎢ x = − 1, y = − 2, z = 3 ⎢ ⎢⎣(viii) Inconsistent

⎤ ⎥ ⎥ ⎥ ⎥ ⎥ ⎥ ⎥ ⎥ ⎥ ⎥ ⎥ ⎥ ⎥ ⎥ 3⎥ ⎥ ⎥ ⎥ ⎥ ⎥ ⎥ ⎥ ⎥⎦

2. Investigate for what values of l and m, the system of simultaneous equations x+y+z=6 x + 2y + 3z = 10 x + 2y + l z = m have (i) no solution (ii) unique solution (3) infinite number of solutions. ⎡ Ans. : (i) λ = 3, μ ≠ 10 ⎤ ⎢ ⎥ (ii) λ ≠ 3, any value of μ ⎢ ⎥ ⎢⎣ ⎥⎦ (iii) λ = 3, μ = 10

Matrices

3. Investigate for what values of k the equations x+y+z=1 2x + y + 4z = k 4x + y + 10z = k2 have infinite number of solutions. [Ans. : k = 1, 2] 4. Determine the values of l for which the following equation. 3x y + l z = 0 2x + y + z = 2 x 2y l z = 1 will fail to have unique solution. For this value of l, are the equations consistent?

11.73

7 ⎤ ⎡ ⎢⎣ Ans. : l = − 2 , no solution ⎥⎦ 5. Find for what values of l, the set of equations 2x 3y + 6z 5t = 3 y 4z + t = 1 4x 5y + 8z 9t = l has (i) no solution (ii) infinite number of solution and find the solution of the equations when they are consistent. ⎡ Ans. : (i) l ≠ 7, ⎤ ⎥ ⎢ (ii) l = , x = k + k + , 7 3 3 1 2 ⎥ ⎢ ⎢ y = 4 k1 − k2 + 1, z = k1 ,⎥ ⎥ ⎢ t = k2 ⎦ ⎣

11.9 HOMOGENEOUS LINEAR EQUATIONS a11x1 + a12x2 + … + a1n xn = 0 a21x1 + a22x2 + … + a2n xn = 0 ………………………...…… .…………………………….. am1x1 + am2 x2 + … + amn xn = 0 is a system of m homogeneous equations in n unknowns x1, x2, …, xn. Writing these equations in matrix form. AX = O … (1) where, A is any matrix of order m × n, X is a vector of order n × 1 and O is a null vector of order m × 1. The matrix A is called coefficient matrix of the system of equations. This system is always consistent. Suppose

11.9.1 Solution of Homogeneous Linear Equations The number of linearly independent solutions of the equation AX = O is n r, where n is the number of unknowns and r is the rank of the coefficient matrix A. There are two cases: Case I: If rank of matrix A is equal to the number of unknowns, i.e., n = r, then the equation AX = O will have no linearly independent solutions. In this case, X = O, i.e., x1 = x2 = … = xn = 0 is the only solution and is known as trivial solution. Case II: If rank of matrix A is less than the number of unknowns, i.e., n < r, then the equation AX = O will have n r linearly independent solutions and are known as nontrivial solutions. The equation AX = O will have an infinite number of solutions. In this case, n r unknowns, called parameters, can be assigned arbitrary values. The remaining unknowns can be expressed in terms of these parameters.

Engineering Mathematics

11.74

Example 1: Solve the following system of equations: (i) x + 2y + 3z = 0 (ii) 3x - y - z = 0 2x + 3y + z = 0 x + y + 2z = 0 4x + 5y + 4z = 0 5x + y + 3z = 0 x + 2y - 2z = 0 (iii) x + y - z + w = 0 x - y + 2z - w = 0 3x + y + w = 0. Solution: (i) In matrix form, A X =O 1 2 4 1

2 3 3 1 5 4 2 –2

0 x 0 y = 0 z 0

2R1, R3

4R1, R4

1 2 3 0 –1 –5

0 x 0 y = 0 z 0

R2

0 –3 –8 0 –1 –5 R3

3R2, R4

1 2 3 0 –1 –5 0 0

0 0

7 0

R1

R2 0 x 0 y = 0 z 0

r (A) = 3 (Number of unknowns) Hence, the system has a trivial solution, x = 0, y = 0, z = 0 (ii) In matrix form, A X =O 3 –1 –1 1 1 2 5 1 3

x 0 y = 0 z 0

R12 1 1 2 3 –1 –1 5 1 3

x 0 y = 0 z 0

Matrices

R2

3R1, R3

5R1

1 1 2 0 –4 –7 0 –4 –7 R3

11.75

x 0 y = 0 z 0

R2

1 1 2 0 –4 7 0 0 0

x 0 y = 0 z 0

r (A) = 2 < 3 (Number of unknowns) Hence, the system has non-trivial solutions. x + y + 2z = 0 4y 7z = 0 Number of parameters = 3 2 = 1 Let z = t 7 Then y = – t, 4 t 4 t 7 Hence, x = – , y = – t, z = t is the solution of the system where t is a parameter. 4 4 (iii) In matrix form, A X =O x 1 1 –1 1 0 y 1 –1 2 –1 = 0 z 3 1 0 1 0 w x=

7 t 4

2t = –

R2

R1, R3

3R1

1 1 –1 1 0 –2 3 –2 0 –2 3 –2 R3

x

0 y = 0 z 0 w

R2

1 1 –1 1 0 –2 3 –2 0 0 0 0

x 0 y = 0 z 0 w

r (A) = 2 < 4 (Number of unknowns)

Engineering Mathematics

11.76

Hence, the system has non-trivial solutions. x+y z+w=0 2y + 3z 2w = 0 Number of parameters = 4 2 = 2 Let w = t1, z = t2 3 t t, 2 2 1 1 ⎛ 3 ⎞ x = ⎜ − t2 + t1 ⎟ + t2 t1 = – t2 ⎝ 2 ⎠ 2 1 3 Hence, x = – t2, y = t2 t1, z = t2, w = t1 is the solution of the system where t1 and 2 2 t2 are parameters. Then y =

Example 2: Discuss for all values of k, the system of equations 2x + 3ky + (3k + 4)z = 0 x + (k + 4)y + (4k + 2)z = 0 x + 2(k + 1)y + (3k + 4)z = 0. Solution: In matrix form, A X =O 3k 3k + 4 ⎤ ⎡ x ⎤ ⎡0 ⎤ ⎡2 ⎢ 1 k + 4 4k + 2 ⎥ ⎢ y ⎥ = ⎢ 0 ⎥ ⎢ ⎥⎢ ⎥ ⎢ ⎥ ⎣1 2k + 2 3k + 4 ⎦ ⎣ z ⎦ ⎣0 ⎦ R12 ⎡ 1 k + 4 4k + 2 ⎤ ⎡ x ⎤ ⎡ 0 ⎤ ⎢2 3k 3k + 4 ⎥ ⎢ y ⎥ = ⎢0 ⎥ ⎢ ⎥⎢ ⎥ ⎢ ⎥ ⎣1 2k + 2 3k + 4 ⎦ ⎣ z ⎦ ⎣0 ⎦ R2

2R1, R3

R1

⎡1 k + 4 4k + 2 ⎤ ⎡ x ⎤ ⎡ 0 ⎤ ⎢0 k – 8 –5k ⎥ ⎢ y ⎥ = ⎢0 ⎥ ⎢ ⎥⎢ ⎥ ⎢ ⎥ ⎣0 k – 2 – k + 2 ⎦ ⎣ z ⎦ ⎣0 ⎦ 1 k + 4 4k + 2 A = 0 k – 8 –5k = (k 0 k – 2 –k + 2

8) ( k + 2) + 5k (k

= (k 2) ( k + 8 + 5k) = 4 (k 2) (k + 2) (a) When k ± 2, |A| 0, r (A) = 3 (Number of unknowns) Hence, the system has a trivial solution, x = 0, y = 0, z = 0

2)

Matrices

11.77

(b) When k = ± 2, |A| = 0, r (A) < 3 Hence, the system has non-trivial solutions. Case I: When k = 2, ⎡ 1 6 10 ⎤ ⎡ x ⎤ ⎡0 ⎤ ⎢0 –6 –10 ⎥ ⎢ y ⎥ = ⎢0 ⎥ ⎢ ⎥⎢ ⎥ ⎢ ⎥ 0⎦ ⎣ z ⎦ ⎣0⎦ ⎣0 0 r (A) = 2 < 3 (Number of unknowns) x + 6y + 10z = 0 6y 10z = 0 Number of parameters = 3 2 = 1 Let z = t 5 Then y = – t, 3 x=0 5 Hence, x = 0, y = – t, z = t is the solution of the system where t is a parameter. 3 Case II: When k = 2 2 ⎡1 ⎢0 –10 ⎢ ⎣ 0 –4 2 R3 R2 5 2 ⎡1 ⎢0 –10 ⎢ 0 ⎣0

–6 ⎤ ⎡ x ⎤ ⎡ 0 ⎤ 10 ⎥ ⎢ y ⎥ = ⎢0 ⎥ ⎥⎢ ⎥ ⎢ ⎥ 4⎦ ⎣ z ⎦ ⎣0⎦

–6 ⎤ ⎡ x ⎤ ⎡0 ⎤ 10 ⎥ ⎢ y ⎥ = ⎢0 ⎥ ⎥⎢ ⎥ ⎢ ⎥ 0⎦ ⎣ z ⎦ ⎣0⎦

r (A) = 2 < 3 (Number of unknowns) x + 2y 6z = 0 10y + 10z = 0 Number of parameters = 3 2 = 1 Let z = t Then y = t, x = 4t Hence, x = 4t, y = t, z = t is the solution of the system where t is a parameter. Example 3: Find the only real value of k for which following system of equations has non-zero solutions. Also, solve the following equations: x + 2y + 3z = k x 3x + y + 2z = k y 2x + 3y + z = k z . Solution: The system of equations is (1 l)x + 2y + 3z = 0 3x + (1 l)y + 2z = 0 2x + 3y + (1 l)z = 0

Engineering Mathematics

11.78

In matrix form, A X =O 2 ⎡1 – ⎢ 3 1– ⎢ 3 ⎣ 2

⎤ ⎡ x ⎤ ⎡0⎤ ⎥ ⎢ y ⎥ = ⎢0⎥ ⎥⎢ ⎥ ⎢ ⎥ ⎦ ⎣ z ⎦ ⎣0⎦ The system will have a non-zero solution if |A| = 0 A=

1– 3 2

2 1– 3

3 2 1–

3 2 = (1 1–

l)[(1

l)2

6]

2 (3

3l

4) + 3(9

2 + 2l) = 0

(1 l) (l2 2l 5) + 2 + 6l + 21 + 6l = 0 l2 2l 5 l3 + 2l2 + 5l + 12l + 23 = 0 l3 + 3l2 + 15l + 18 = 0 2 l (l 6) 3l (l 6) 3(l 6) = 0 (l 6) (l2 + 3l + 3) = 0 l 6 = 0, l2 + 3l + 3 = 0 l = 6, l = 1.5 ± 0.866 i For the real value, l = 6, the system has a non-zero solution. –5 2 3 x 0 3 –5 2 y = 0 2 R2

3 –5

z

0

R3

–5 2 1 –8 2 R12

3 –5

1 –8 –5 2 2

3 7

7 3

3 –5

R2 + 5R1, R3

x 0 y = 0 z

0

x 0 y = 0 z 0

2R1

1 –8 7 0 –38 38 0 19 –19

x 0 y = 0 z 0

1 R 2 2 1 –8 7 0 –38 38 0 0 0

x 0 y = 0 z 0

R3 +

r (A) = 2 < 3 (Number of unknowns)

Matrices

11.79

Hence, the system has a non-trivial solution. x 8y + 7z = 0 38y + 38z = 0 Number of parameters = 3 2 = 1 Let z = t Then y = t, x = 8t 7t = t Hence, x = t, y = t, z = t is the solution of the system where t is a parameter. Example 4: If the following system has a non-trivial solution, then prove that a + b + c = 0 or a = b = c. ax + by + cz = 0 bx + cy + az = 0 cx + ay + bz = 0. Solution: In matrix form, A X =O ⎡ a b c ⎤ ⎡ x ⎤ ⎡0⎤ ⎢ b c a ⎥ ⎢ y ⎥ = ⎢0⎥ ⎢ ⎥⎢ ⎥ ⎢ ⎥ ⎣ c a b ⎦ ⎣ z ⎦ ⎣0⎦ The system has a non-trivial solution if |A| = 0 a b c 2 2 2 A = b c a = a (bc a ) b(b ac) + c(ab c ) = 0 c a b a3 + b3 + c3 3abc = 0 2 2 (a + b + c) (a + b + c2 ab bc ca) = 0 a + b + c = 0 or a2 + b2 + c2 ab bc ca = 0 1 [(a b)2 + (b c)2 + (c a)2] = 0 2 a b=0 a=b b c=0 b=c c=a c a=0 Hence, the system has a non-trivial solution if a + b + c = 0 or a = b = c a b c x 0 Now, b c a y = 0 c a b

z

0

R3 + R1 + R2 a b c b c a a+b+c a+b+c a+b+c ax + by + cz = 0 bx + cy + az = 0 (a + b + c) (x + y + z) = 0

x 0 y = 0 z 0

11.80

Engineering Mathematics

(i) When a + b + c = 0, we have only two equations ax + by + cz = 0 bx + cy + az = 0 y x z = – 2 = =t a – bc ab – c 2 ac – b 2 x = (ab (ii) When a = b = c, we have only one equation. x+y+z=0 Let z = t1 y = t2 Then x = (t1 + t2)

c2)t, y = (bc

a2)t, z = (ac

b2)t

Exercise 11.8 1. Solve the following equations: (i) x y + z = 0, x + 2y + z = 0, 2x + y + 3z = 0 (ii) x 2y + 3z = 0, 2x + 5y + 6z = 0 (iii) 2x 2y + 5z + 3w = 0, 4x y + z + w = 0, 3x 2y + 3z + 4w = 0, x 3y + 7z + 6w = 0 (iv) 2x y + 3z = 0, 3x + 2y + z = 0, x 4y + 5z = 0 (v) 7x + y 2z = 0, x + 5y 4z = 0, 3x 2y + z = 0, 2x 7y + 5z = 0 (vi) 3x + 4y z 9w = 0, 2x + 3y + 2z 3w = 0, 2x + y 14z 12w = 0, x + 3y + 13z + 3w = 0 (vii) x1 + 2x2 + 3x3 + x4 = 0, x1 + x2 x3 x4 = 0, 3x1 x2 + 2x3 + 3x4 = 0 (viii) 2x1 x2 + 3x3 = 0, 3x1 + 2x2 + x3 = 0, x1 4x2 + 5x3 = 0

⎡ Ans. : (i) x = 0, y = 0, z = 0 ⎤ ⎢ ⎥ (ii) x = −3t , y = 0, z = t ⎢ ⎥ ⎢ 211 7 ⎥ (iii) x = t , y = 4t , z = t ,⎥ ⎢ 9 9 ⎥ ⎢ w = t ⎢ ⎥ ⎢ ⎥ (iv) x = −t , y = t , z = t ⎢ ⎥ 3 13 ⎢ ⎥ (v) x = t , y = t , z = t ⎥ ⎢ 17 17 ⎢ ⎥ (vi) x = 11t , y = −8t , z = t , ⎥ ⎢ ⎢ ⎥ w=0 ⎢ ⎥ 1 2 ⎢ ⎥ (vii) x1 = − t , x2 = t , ⎢ ⎥ 3 3 ⎢ ⎥ 2 ⎢ ⎥ x3 = − t , x4 = t 3 ⎢ ⎥ ⎢⎢ ⎥⎥ x = − x = − x = (viii) t ⎣ ⎦ 1 2 3 2. For what value of l does the following system of equations possess a nontrivial solution? Obtain the solution for real values of l. (i) 3x + y lz = 0 4x 2y 3z = 0 2lx + 4y lz = 0 (ii) (1 l)x1 + 2x2 + 3x3 = 0 3x1 + (1 l)x2 + 2x3 = 0 2x1 + 3x2 + (1 l)x3 = 0

Matrices

⎡ Ans.: ⎤ ⎢ (i) Non-trivial solution l = 1, −9 ⎥ ⎢ ⎥ For l = 1, x = −t , y = t , z = −2t ⎢ ⎥ ⎢ ⎥ For l = −9, x = −3t , y = −9t , z = 2t ⎥ ⎢ ⎢⎣ (ii) l = 6, x = y = z = t ⎥⎦

3. Show that the system of equations 2x 2y + z = lx, 2x 3y + 2z = ly, x + 2y = lz can posses a non trivial

11.81 solution only if l = 1, l = 3. Obtain the general solution in each case. ⎡ Ans. : For ⎢ ⎢ ⎢ For ⎢ ⎢⎣

l = 1, x = 2t 2 − t1 ,⎤ ⎥ y = t 2 , z = t1 ⎥ l = −3, x = −t , ⎥ ⎥ y = −2t , z = t ⎥⎦

11.10 LINEAR DEPENDENCE AND INDEPENDENCE OF VECTORS 11.10.1 Linear Dependence A set of r vectors X1, X2, ……. Xr is said to be linearly dependent if there exist r scalars (numbers) k1, k2, …… kr not all zero, such that k1X1 + k2X2 + ……+ kr Xr = O

11.10.2 Linear Independence A set of r vectors X1, X2, ……. Xr is said to be linearly independent if there exists r scalars (numbers) k1, k2, …… kr such that if k1X1 + k2X2 + ……+ kr Xr = O then k1 = k2 = ….. = kr = 0

11.10.3 Linear Combination of Vectors A vector X which can be expressed in the form X = k1X1 + k2X2 + ……+ kr Xr is said to be a linear combination of the vectors X1, X2 …. Xr, where k1, k2 …. kr are any numbers. Results: (1) If a set of vectors is linearly independent, then at least one vector of the set can be expressed as a linear combination of the remaining vectors. (2) If a set of vectors is linearly dependent, then no vector of the set can be expressed as a linear combination of the remaining vectors. Example 1: Examine whether the following vectors are linearly independent or dependent: (i) [1, 1, -1], [2, 3, -5], [2, -1, 4] (ii) [1, -1, 1], [2, 1, 1], [3, 0, 2] (iii) [1, 2, -1, 0], [1, 3, 1, 2], [4, 2, 1, 0], [6, 1, 0, 1]

11.82

Engineering Mathematics

(iv) [2, -1, 3, 2], [1, 3, 4, 2], [3, -5, 2, 2] (v) [1, 0, 2, 1], [3, 1, 2, 1], [4, 6, 2, 4], [-6, 0, -3, 0] Solution: (i) X1 = [1, 1, 1], X2 = [2, 3, 5], X3 = [2, 1, 4] Let k1X1 + k2X2 + k3X3 = O k1[1, 1, 1] + k2[2, 3, 5] + k3[2, 1, 4] = [0, 0, 0] k1 + 2k2 + 2k3 = 0 k1 + 3k2 k3 = 0 k1 5k2 + 4k3 = 0 In matrix form, 1 2 2 k1 0 1 3 1 k2 0 1 5 4 k3 0 R2

R1, R3 + R1

1 ∼ 0

2 1

2 k1 3 k2

0 0

0

3

6

0

k3

R3 + 3R2 1 2 ∼ 0 1 0 0

2 k1 3 k2

0 0

3

0

k3

k1 + 2k2 + 2k3 = 0 k2 3k3 = 0 3k3 = 0 Solving these equations, k1 = 0, k2 = 0, k3 = 0 Since all k1, k2, k3 are zero, the vectors are linearly independent. (ii) X1 = [1, 1, 1], X2 = [2, 1, 1], X3 = [3, 0, 2] Let k1X1 + k2X2 + k3X3 = O k1[1, 1, 1] + k2[2, 1, 1] + k3[3, 0, 2] = [0, 0, 0] k1 + 2k2 + 3k3 = 0 k1 + k2 + 0k3 = 0 k1 + k2 + 2k3 = 0 In matrix form, 1 2 3 k1 0 1 1 0 k2 0 1 1 2 k3 0

Matrices R2 + R1, R3

11.83

R1

⎡ 1 2 3⎤ ⎡ k1 ⎤ ⎡ 0 ⎤ ∼ ⎢0 3 3⎥ ⎢ k2 ⎥ = ⎢ 0 ⎥ ⎢ ⎥⎢ ⎥ ⎢ ⎥ ⎣0 −1 −1⎦ ⎣ k3 ⎦ ⎣ 0 ⎦ 1 R3 + R2 3 ⎡ 1 2 3⎤ ⎡ k1 ⎤ ⎡0 ⎤ ∼ ⎢0 3 3⎥ ⎢ k2 ⎥ = ⎢0 ⎥ ⎢ ⎥⎢ ⎥ ⎢ ⎥ ⎣ 0 0 0 ⎦ ⎣ k3 ⎦ ⎣ 0 ⎦ k1 + 2k2 + 3k3 = 0 3k2 + 3k3 = 0 Number of parameters = Number of unknowns Rank of coefficient matrix = 3 2 =1 Let k3 = t Then k2 = t, k1 = 2t 3t = t Since k1, k2, k3 are not all zero, the vectors are linearly dependent. t X1 t X2 + tX3 = O X1 + X2 X3 = O (iii) X1 = [1, 2, 1, 0], X2 = [1, 3, 1, 2], X3 = [4, 2, 1, 0], X4 = [6, 1, 0, 1] Let k1X1 + k2X2 + k3X3 + k4X4 = O k1[1, 2, 1, 0] + k2[1, 3, 1, 2] + k3[4, 2, 1, 0] + k4[6, 1, 0, 1] = [0, 0, 0, 0] k1 + k2 + 4k3 + 6k4 = 0 2k1 + 3k2 + 2k3 + k4 = 0 k1 + k2 + k3 = 0 2 k2 + k4 = 0 In matrix form, ⎡ 1 1 4 6 ⎤ ⎡ k1 ⎤ ⎡ 0 ⎤ ⎢ 2 3 2 1 ⎥ ⎢ k ⎥ ⎢0⎥ ⎢ ⎥⎢ 2⎥ = ⎢ ⎥ 1 1 1 0 − ⎢ ⎥ ⎢ k3 ⎥ ⎢ 0 ⎥ ⎢⎣ 0 2 0 1 ⎥⎦ ⎢⎣ k4 ⎥⎦ ⎢⎣ 0 ⎥⎦ R2 2R1, R3 + R1 ⎡1 ⎢0 ∼⎢ ⎢0 ⎢⎣0

R3 ⎡1 ⎢0 ∼⎢ ⎢0 ⎢⎣0

1 4 6 ⎤ ⎡ k1 ⎤ ⎡ 0 ⎤ 1 −6 −11⎥ ⎢ k2 ⎥ ⎢ 0 ⎥ ⎥⎢ ⎥ = ⎢ ⎥ 2 5 6 ⎥ ⎢ k3 ⎥ ⎢ 0 ⎥ 2 0 1⎥⎦ ⎢⎣ k4 ⎥⎦ ⎢⎣ 0 ⎥⎦ 2R2, R4 – 2R2 1 1 0 0

4 6 ⎤ ⎡ k1 ⎤ ⎡0 ⎤ −6 −11⎥ ⎢ k2 ⎥ ⎢0 ⎥ ⎥⎢ ⎥ = ⎢ ⎥ 17 28 ⎥ ⎢ k3 ⎥ ⎢0 ⎥ 12 23⎥⎦ ⎢⎣ k4 ⎥⎦ ⎢⎣0 ⎥⎦

Engineering Mathematics

11.84

R4 ⎡1 ⎢0 ⎢ ∼ ⎢0 ⎢ ⎢⎣0

6⎤ ⎡ k ⎤ ⎡0 ⎤ −11⎥ ⎢ 1 ⎥ ⎢ ⎥ 0 ⎥ k 28 ⎥ ⎢ 2 ⎥ = ⎢ ⎥ k 0 55 ⎥ ⎢ 3 ⎥ ⎢ ⎥ k ⎢ ⎥ ⎢ ⎣ 4 ⎦ ⎣0 ⎥⎦ 17 ⎥⎦

1 4 1 −6 0 17 0

12 R3 17

0

k1 + k2 + 4k3 + 6k4 = 0 k2 6k3 11k4 = 0 17k3 + 28k4 = 0 55 k4 = 0 17 Solving these equations, k1 = 0, k2 = 0, k3 = 0, k4 = 0 Since all k1, k2, k3, k4 are zero, the vectors are linearly independent. (iv) X1 = [2, 1, 3, 2 ], X2 = [1, 3, 4, 2], X3 = [3, 5, 2, 2] Let k1X1 + k2X2 + k3X3 = O k1 [2, 1, 3, 2 ] + k2[1, 3, 4, 2] + k3[3, 5, 2, 2] = [0, 0, 0, 0] 2k1 + k2 + 3k3 = 0 k1 + 3k2 5k3 = 0 3k1 + 4k2 + 2k3 = 0 2k1 + 2k2 + 2k3 = 0 In matrix form, ⎡ 2 ⎢ −1 ⎢ ⎢ 3 ⎢⎣ 2

1 3⎤ ⎡0⎤ ⎡k ⎤ 3 −5⎥ ⎢ 1 ⎥ ⎢0 ⎥ ⎥ k =⎢ ⎥ 4 2⎥ ⎢ 2 ⎥ ⎢ 0 ⎥ k 2 2⎥⎦ ⎣ 3 ⎦ ⎢⎣0 ⎥⎦

R1 + R2 ⎡ 1 ⎢ −1 ∼⎢ ⎢ 3 ⎢⎣ 2

4 −2⎤ ⎡0⎤ ⎡k ⎤ 3 −5⎥ ⎢ 1 ⎥ ⎢0 ⎥ ⎥ k =⎢ ⎥ 4 2⎥ ⎢ 2 ⎥ ⎢0 ⎥ k 2 2⎥⎦ ⎣ 3 ⎦ ⎢⎣0 ⎥⎦

R2 + R1, R3 ⎡ 1 ⎢ 0 ∼⎢ ⎢ 0 ⎢⎣ 0 1 R2 , 7

3R1, R4

2R1

4 −2⎤ ⎡0⎤ ⎡k ⎤ 7 −7⎥ ⎢ 1 ⎥ ⎢ 0 ⎥ ⎥ k =⎢ ⎥ −8 8⎥ ⎢ 2 ⎥ ⎢ 0 ⎥ k −6 6 ⎥⎦ ⎣ 3 ⎦ ⎢⎣ 0 ⎥⎦ 1 1 R3 , R4 8 6

Matrices

1 0 ∼ 0 0

4 1 1 1

11.85

2 0 k1 1 0 k2 = 1 0 k 1 3 0

R3 + R2, R4 + R2 1 0 ∼ 0 0

4 1 0 0

2 0 k1 1 0 k2 = 0 0 k3 0 0

k1 + 4k2 2k3 = 0 k2 k3 = 0 Number of parameters = Number of unknowns Rank of coefficient matrix = 3 2 = 1 Let k3 = t Then k2 = t, k1 = 4t + 2t = 2t ( 2t)X1 + t X2 + tX3 = O 2X1 X2 X3 = O Since k1, k2, k3 are not all zero, the vectors are linearly dependent. (v) X1 = [1, 0, 2, 1], X2 = [3, 1, 2, 1], X3 = [4, 6, 2, 4], X4 = [ 6, 0, 3, 0] Let k1X1 + k2X2 + k3X3 + k4X4 = O k1[1, 0, 2, 1] + k2[3, 1, 2, 1] + k3[4, 6, 2, 4] + k4[ 6, 0, 3, 0] = [0, 0, 0, 0] k1 + 3k2 + 4k3 6k4 = 0 k2 + 6k3 = 0 3k4 = 0 k1 + k2 + 4k3 = 0

2k1 + 2k2 + 2k3 In matrix form, 1 3 4 0 1 6 2 2 2 1 1 4 R3 1 0 ∼ 0 0

6 0

4 6 6 0

0 0

k2 = 3 k3 0 0 k4 0

2R1, R4 3 1 4 2

k1

R1 6 0 9 6

k1 0 k2 0 = k3 0 k4 0

Engineering Mathematics

11.86

R3 + 4R2, R4 + 2R2 ⎡ 1 3 4 −6 ⎤ ⎡ k1 ⎤ ⎡0 ⎤ ⎢0 1 6 0 ⎥ ⎢ k ⎥ ⎢0 ⎥ ∼⎢ ⎥⎢ 2⎥ = ⎢ ⎥ ⎢0 0 18 9⎥ ⎢ k3 ⎥ ⎢0 ⎥ ⎢⎣0 0 12 6 ⎥⎦ ⎢⎣ k4 ⎥⎦ ⎢⎣0 ⎥⎦ 1 1 R3 , R4 9 6 ⎡1 ⎢0 ∼⎢ ⎢0 ⎢⎣0 R4

3 1 0 0 R3

4 −6 ⎤ ⎡ k1 ⎤ ⎡0 ⎤ 6 0 ⎥ ⎢ k2 ⎥ ⎢0 ⎥ ⎥⎢ ⎥ = ⎢ ⎥ 2 1⎥ ⎢ k3 ⎥ ⎢0 ⎥ 2 1⎥⎦ ⎢⎣ k4 ⎥⎦ ⎢⎣0 ⎥⎦

⎡ 1 3 4 −6 ⎤ ⎡ k1 ⎤ ⎡0 ⎤ ⎢0 1 6 0 ⎥ ⎢ k2 ⎥ ⎢0 ⎥ ∼⎢ ⎥⎢ ⎥ = ⎢ ⎥ 1⎥ ⎢ k3 ⎥ ⎢0 ⎥ ⎢0 0 2 0 ⎥⎦ ⎢⎣ k4 ⎥⎦ ⎢⎣0 ⎥⎦ ⎢⎣0 0 0 k1 + 3k2 + 4k3 6k4 = 0 k2 + 6k3 = 0 2k3 + k4 = 0 Number of parameters = Number of unknowns Rank of coefficient matrix = 4 3 = 1 Let k4 = t t Then k3 = , 2 k2 = 3t, ⎛ t⎞ k1 = 3(3t) 4 ⎜ − ⎟ + 6t = t ⎝ 2⎠ Since k1, k2, k3, k4 are not all zero, the vectors are linearly dependent. ( t)X1 + (3t)X2 2X1

⎛ t ⎞ X + tX = O ⎜⎝ ⎟⎠ 3 4 2 6X2 + X3 2X4 = O

Example 2: Examine whether the following vectors are linearly independent or dependent. (i) X1 = [1, 2, 4]T, X2 = [3, 7, 10]T (ii) X1 = [1, 2, 3]T, X2 = [3, -2, 1]T, X3 = [1, -6, -5]T. Solution: (i)

⎡1 ⎤ X1 = [1, 2, 4]T = ⎢ 2 ⎥ ⎢ ⎥ ⎣4⎦ ⎡3⎤ X2 = [3, 7, 10]T = ⎢ 7 ⎥ ⎢ ⎥ ⎣10 ⎦

Matrices

11.87

Let k1X1 + k2X2 = O 1 3 0 k1 2 + k2 7 = 0 4 10 0 k1 + 3k2 = 0 2k1 + 7k2 = 0 4k1 + 10k2 = 0 In matrix from, 1 3 0 k1 2 7 = 0 k2 4 10 0 R2 1 ∼ 0 0

2R1, R3

4R1

3 0 k1 1 = 0 k2 2 0

R3 + 2R2 1 3 0 k1 = 0 ∼ 0 1 k2 0 0 0 k1 + 3k2 = 0 k2 = 0 k1 = 0 Since all k1, k2, are zero, the vectors are linearly independent. 1 X1 = [1, 2, 3] = 2 T

(ii)

3 X2 = [3, 2, 1] = T

3 2 1 1 6 5

X3 = [1, 6, 5] = T

Let k1X1 + k2X2 + k3X3 = O 1 k1 2 3

k2

3 2 1

k3

1 6 5

0 0 0

Engineering Mathematics

11.88

k1 + 3k2 + k3 = 0 2k1 2k2 6k3 = 0 3k1 + k2 5k3 = 0 In matrix from, 1 2 3 R2

3 1 2 6 1 5 2R1, R3

1 ∼ 0 0

3 8 8

1 R2 , 8 1 ∼ 0 0 R3

k1 k2 k3 3R1

0 0 0

1 k1 8 k2 8 k3

0 0 0

1 R3 8 3 1 k1 0 1 1 k2 = 0 1 1 k3 R2

0

1 3 1 k1 0 ∼ 0 1 1 k2 = 0 0 0 0 k3 0 k1 + 3k2 + k3 = 0 k 2 + k3 = 0 Number of parameters = Number of unknowns Rank of coefficient matrix = 3 2 = 1 Let k3 = t Then k2 = t, k1 = 3( t) t = 2t Since k1, k2, k3, are not all zero, the vectors are linearly dependent. (2t)X1 + ( t)X2 + tX3 = O 2X1 X2 + X3 = O Example 3: Show that the rows of the following matrix are linearly dependent and find the relationship between them. 1 3 4 6

0 1 6 0

2 2 2 3

1 1 4 4

Solution: Let the row vectors of the matrix be given by X1, X2, X3, X4. Let k1X1 + k2X2 + k3X3 + k4X4 = O

Matrices

11.89

k1[1, 0, 2, 1] + k2[3, 1, 2, 1] + k3[4, 6, 2, 4] + k4[ 6, 0, 3, 4] = [0, 0, 0, 0] k1 + 3k2 + 4k3 6k4 = 0 k2 + 6k3 = 0 2k1 + 2k2 + 2k3 3k4 = 0 k1 + k2 4k3 4k4 = 0 In matrix form, 1 3 4 6 k1 0 0 1 6 0 k2 0 = 2 2 2 3 k3 0 1 1 4 4 k4 0 R24 1 1 ∼ 2 0 R3

3 1 2 1 2R2

1 3 1 1 ∼

4 4 2 6

6 k1 0 4 k2 0 = 3 k3 0 0 k4 0

4 4

6 4

0 0 10 0 1 6 R2 R1

k1

0 0

k2 = 5 k3 0 0 k4 0

⎡ 1 3 4 −6 ⎤ ⎡ k1 ⎤ ⎡0 ⎤ ⎢0 −2 −8 2⎥ ⎢ k ⎥ ⎢0 ⎥ ⎥⎢ 2⎥ = ⎢ ⎥ ∼⎢ ⎢0 0 10 5⎥ ⎢ k3 ⎥ ⎢ 0 ⎥ ⎢ ⎥⎢ ⎥ ⎢ ⎥ 1 6 0 ⎦ ⎢⎣ k4 ⎥⎦ ⎣0 ⎦ ⎣0 1 1 R2 , R3 2 5 3 1

4 4

0 0 0 1 R4 + R2

2 6

6 k1 0 1 k2 0 = 1 k3 0 0 k4 0

4 4 2 2

6 1 1 1

1 0 ∼

1 0 ∼ 0 0

3 1 0 0

k1 0 k2 0 = k3 0 k4 0

Engineering Mathematics

11.90

R4

R3

⎡ 1 3 4 −6 ⎤ ⎡ k1 ⎤ ⎡0 ⎤ ⎢0 −1 −4 1⎥ ⎢ k2 ⎥ ⎢0 ⎥ ∼⎢ ⎥⎢ ⎥ = ⎢ ⎥ 2 1⎥ ⎢ k3 ⎥ ⎢0 ⎥ ⎢0 0 0 0 ⎥⎦ ⎢⎣ k4 ⎥⎦ ⎢⎣0 ⎥⎦ ⎢⎣0 0 k1 + 3k2 + 4k3 6k4 = 0 k2 4k3 + k4 = 0 2k3 + k4 = 0 Number of parameters = Number of unknowns Rank of coefficient matrix = 4 3 = 1 Let k4 = t t Then k3 = , 2 t k2 = 4 + t = 3t, 2 t k1 = 3(3t) 4 + 6t = t 2 Since k1, k2, k3, k4 are not all zero, the vectors are linearly dependent. t ( t)X1 + (3t)X2 X + tX4 = O 2 3 2X1 6X2 + X3 2X4 = O Exercise 11.9 Examine whether the following vectors are linearly independent or dependent: 1. [3, 1, 1], [2, 0, 1], [4, 2, 1] 4. [1, 1, 1, 3], [1, 2, 3, 4], [2, 3, 4, 7] [Ans. : Independent] [Ans. : Dependent, X1 + X2 X3 = O] 2. [3, 1, 4], [2, 2, 3], [0, 4, 1] 5. [1, 0, 2, 1], [3, 1, 2, 1], [4, 6, 2, 4], [ 6, 0, 3, 0] [Ans. : Dependent, 2X1 3X2 X3 = O] [Ans. : Dependent, 2X1 6X2 + X3 2X4 = O] 3. [1, 2, 1, 0], [1, 3, 1, 2], [4, 2, 1, 0], 6. [2, 2, 1]T, [1, 3, 1]T, [1, 2, 2]T [6, 1, 0, 1] [Ans. : Independent] [Ans. : Independent]

11.11 EIGEN VALUES AND EIGEN VECTORS Any non-zero vector X is said to be a characteristic vector or eigen vector of a matrix A, if there exists a number l such that AX = l X x1 x where A = [aij]n n is a n-rowed square matrix and X = 2 is a column vector. : xn Also, l is said to be characteristic root or latent root or characteristic value or eigen value or proper value of the matrix A.

Matrices

11.91

Now AX = lX = lIX

lI)X = O (∵ X O ) A lI = O |A lI| = 0 The matrix (A lI) is called the characteristic matrix. The determinant |A lI| is called the characteristic polynomial of A. The equation |A lI| = 0 is called the characteristic equation of A. Note: The determinant of the characteristic equation of a matrix of order 3 can be solved easily using the following formula. l3 – (sum of diagonal elements) l2 + (sum of cofactors of diagonal elements) l – |A| = 0 (A

11.11.1 Properties of Characteristic Roots or Eigen Values (1) The sum of the eigen values of a matrix is the sum of the elements of the principal diagonal. (2) The product of the eigen values of a matrix is equal to the determinant of the matrix. (3) If l is the eigen value of matrix A, then eigen values of following matrices are given as Matrix Eigen value (i) AT 1 (ii) A1 (iii) (iv) (v)

An kA A ± kI

(vi)

adj A

ln kl l ±k | A|

l (vii) (viii) (ix) (x) (xi) (xii) (xiii) (xiv) (xv)

Aq Singular Hermitian Skew Hermitian Real symmetric Skew real symmetric Unitary Orthogonal Triangular

l at least one zero all real either zero or purely imaginary all real either zero or purely imaginary ±1 ±1 diagonal elements

Note: Eigen vectors are same as that of matrix A for all the above matrices.

11.11.2 Properties of Characteristic Vectors or Eigen Vectors (1) If X is an eigen vector of the matrix A corresponding to the eigen value l, then kX is also an eigen vector of A corresponding to the same eigen value l where k is any non- zero scalar.

Engineering Mathematics

11.92

(2) If X is an eigen vector of the matrix A, then X cannot correspond to more than one eigen value of A. (3) The eigen vectors corresponding to distinct eigen values of a matrix are linearly independent. (4) If two or more eigen values are equal, then the corresponding eigen vectors may or may not be linearly independent. (5) The eigen vectors corresponding to distinct eigen values of a real symmetric matrix are orthogonal. (6) Any two eigen vectors corresponding to two distinct eigen values of a unitary matrix are orthogonal. Note: Two eigen vectors X1 and X2 are orthogonal if and

X 1T X 2 = X 1 X 2T = O

( For real vector)

X1 X 2 = X1 X 2 = O

( For complex vecctor)

11.11.3 Algebraic and Geometric Multiplicity of an Eigen Value If l is an eigen value of the characteristic equation |A lI| = 0 repeated n times, then n is called the algebraic multiplicity of l. The number of linearly independent solutions of [A lI] X = O is called the geometric multiplicity of l. Example 1: Find the eigen values and eigen vectors for the following matrices: (i)

(v)

4 1

6 3

6 2

1

4

3

0 1 1 1 0 1

(ii)

(vi)

1 1 0

1 0 1 2

1 1

2 2

3

1 2 2 0 2 1

(iii)

(vii)

1 2 2

A=

Solution: (i)

8 6

6 7

2 4

2

4

3

0 0

1 0 0 1 .

1

3 3

4 1

6 3

6 2

1

4

3

The characteristic equation is |A 4 l 1 1

6 3 l 4 l3 4l2

(a) For l

[A

l1I]X = O

lI| = 0

6 2 3 l

0

l+4=0 l 1, 1, 4

(iv)

2 2

2 1

3 6

1

2

0

Matrices

11.93

6 6 x 0 4 2 y 0 4 2 z 0 5x + 6y + 6z = 0 x + 4y + 2z = 0 x y z 6 6 5 6 5 6 4 2 1 2 1 4 5 1 1

x 12 x 6

y 4 y 2

z 14 z 7

X1

6 2 7

(b) For l

[A

l2I]X = O 3 1

6 2

6 2

x y

0 0

1

4

4

z

0

2 4

x + 2y + 2z = 0 x 4y 4z = 0 y z 1 2 1 2 1 4 1 4

x 2 4

x 0 x 0

y 2 y 1

z 2 z 1 0 1 1

X2 (c) For l = 4, [A

l3I]X = O 0 1 1

6 1 4

6 2 7

x y z

0 0 0

6y + 6z = 0

Engineering Mathematics

11.94

x x 6 6 1 2

x y + 2z = 0 4y 7z = 0 y z 0 6 0 6 1 2 1 1 x 18 x 3

y 6 y 1

z 6 z 1

X3

3 1 1

1 0 A= 1 2 2 2

(ii)

1 1 3

The characteristic equation is lI | = 0

|A 1 l 1

2 l

1 1

2

2

3 l

0

l

3

2

6l + 11l

0 l

(a) For l

[A

l1I]X = O 0 0 1 1

1 1

x y

2 2

2

z

Taking

(b) For l

[A

0 0

0 z=0 x+y+z=0 y = 1, x = 1 1 X1 1 0

l2I]X = O x 0 y 0 z 0 x+z=0 2x+ 2y + z = 0

1 0 1 0 2 2

1 1 1

0

Matrices

x

y

0 1 2 1

z

1 1 2 1 x y 2 1 X2

(c) For l

11.95

1 0 2 2 z 2 2 1 2

l3I]X = O

[A

2 1 2

0 1 2

x 0 1

1 1

1 x 0 1 y 0 0 z 0 2x z = 0 x y+z=0 2x + 2y = 0 y z 2 1 2 0 1 1 1 1 x y z 1 1 2 1 X3 1 2 A=

(iii) The characteristic equation is |A

8− −6

−6 7− −4

2 l

3

lI| = 0 2 −4 = 0

3−

18l + 45l = 0 2

l (a) For l = 0, [A

l1I] = O 8 6 2

6 7 4

2 4 3

x y z

0 0 0

8 6

6 7

2 4

2

4

3

Engineering Mathematics

11.96

8x 6y + 2z = 0 6x + 7y 4z = 0 2x 4y + 3z = 0 x y z =− = −6 2 8 2 8 −6 7 −4 −6 −4 −6 7 x y z 10 20 20 x y z = = 1 2 2

(b) For l

[A

X1

1 2 2

l2I]X = O 5 6

6 4

2 4

x y

0 0

2

4

0

z

0

5x 6y + 2z = 0 6x + 4y 4z = 0 2x 4y = 0 x y z 6 2 5 2 5 6 4 4 6 4 6 4 x y z 16 8 16 x y z 2 1 2 2 1

X2

2 (c) For l

[A

l3I]X = O 7 6 2

6 8 4

2 4 12

x y z

0 0 0

7x 6y + 2z = 0 6x 8y 4z = 0 2x 4y 12z = 0

Matrices

x −6 2 −8 −4

y

=−

−7 2 −6 −4 x y 40 40 x y = = 2 2 X3

=

11.97

z −7 −6 −6 −8

z 20 z 1 2 2 1

Note: The eigen vectors corresponding to distinct eigen values of a real symmetric matrix are orthogonal which can be verfied with this example. ⎡ 2⎤ X X 2 = [1 2 2] ⎢⎢ 1⎥⎥ = 0 ⎢⎣ −2⎥⎦ T 1

⎡ 2⎤ X X 3 = [2 1 − 2] ⎢⎢ −2⎥⎥ = 0 ⎢⎣ 1⎥⎦ T 2

⎡1⎤ X X 1 = [2 − 2 1] ⎢⎢ 2⎥⎥ = 0 ⎢⎣ 2⎥⎦ Thus X1, X2 and X3 are orthogonal to each other. T 3

A=

(iv) The characteristic equation is |A

−2 − λ 2 −1

[A

−3

2

1 − λ −6 = 0 −2 −λ

l3 + l2 (a) For l

lI| = 0

21l

45 = 0 l

l1I]X = O 7 2 1

2 4 2

3 6 5

x y z

0 0 0

2 2

2 1

3 6

1

2

0

Engineering Mathematics

11.98

7x + 2y 2x 4y x 2y y 7 2 x = 24 x 1

x 2 4

(b) For l

[A

3 6

3z = 0 6z = 0 5z = 0 z 3 7 6 2 y z = 48 24 y z 2 1 1 X1 2 1

2 4

l2I]X = O x y z

0 0 0

1 R2 , R1 R3 2 1 2 3 x 0 0 0 y 0 0 0 z

0 0 0

1 2 1 R1

2 4 2

3 6 3

Rank of matrix = 1 Number of unknowns = 3 Number of linearly independent solutions = 3 x + 2y 3z = 0 Taking x = 3 and y = 0, z = 1 Taking x = 2 and y = 1, z = 0 2 3 , X 1 X2 0 3 0 1 0 1 1 A= 1 0 1 1 1 0

(v) The characteristic equation is |A l 1 1 l 1 1 l3 3l

lI| = 0 1 1 0 l 2=0 l

1=2

Matrices (a) For l

[A

l1I]X = O 2 1 1

1 x 1 y 2 z

1 2 1

x 1 1 2 1

[A

0 0 0

2x + y + z = 0 x 2y + z = 0 x + y 2z = 0 y z 2 1 2 1 1 1 1 2 x 3 x 1

(b) For l

11.99

y 3 y 1

z 3 z 1 1 X1 = 1 1

l2I]X = O 1 1 1 1 1 1

x y

0 0

1 1 1 z 0 R2 R1, R3 R1 1 1 1 0 0 0

x y

0 0

0 0 0

z

0

Rank of matrix = 1 Number of unknowns = 3 Number of linearly independent solutions = 3 x+y+z=0 Taking z = 0 and y = 1, x = 1 Taking z = 2 and y = 1, x = 1 1 1 X2 = 1 , X3 = 1 0 2 (vi)

A=

1=2

1 2 2 0 2 1 1 2 2

Engineering Mathematics

11.100

The characteristic equation is |A 1 l 0 1

2 2 l 2

l3 (a) For l

[A

lI| = 0 2 1

0

2 l 4=0 l 2, 2

5l2 + 8l

l1I]X = O 0 2 2 0 1 1 1 2 1

x y z

0 0 0

y+z=0 x + 2y + z = 0 y z 0 1 0 1 1 1 1 2

x 1 1 2 1

x 1 x 1

y 1 y 1

z 1 z 1 1 1

X1 =

1 (b) For l

[A

l2I]X = O 1 2 2 0 0 1

x y

0 0

1 2 0

z

0

2 1 2

x y z

0 0 0

1 2 2 0 0 1 0 0 0

x y z

0 0 0

R3

R1

1 2 0 0 0 0 R3 + 2R2

Matrices

11.101

Rank of matrix = 2 Number of unknowns = 3 Number of linearly independent solution = 3 2 = 1 x + 2y + 2z = 0 z=0 Taking y = 1, x = 2, 2 X2 = 1 0 Hence, there is one eigen vector corresponding to the repeated root l = 2. 0 1 0 (vii) A= 0 0 1 1 3 3 The characteristic equation is |A lI| = 0 l 1 0 0 l 1 0 1 3 3 l l3 For l

[A

3l2 + 3l

1=0 l 1, 1

l1I]X = O 1 1 0 x 0 1 1 y 1 3 2 z R3 + R1

0 0 0

1 0 x 1 1 y 2 2 z

0 0 0

1 0 0 R3

2R2 1 0 0

1 0 x 1 1 y 0 0 z

0 0 0

Rank of matrix = 2 Number of unknowns = 3 Number of linearly independent solutions = 3 2 = 1 x+y=0 y+z=0 x y z 1 0 1 0 1 1 1 1 0 1 0 1

Engineering Mathematics

11.102

x y z = = 1 1 1 1 X= 1 1 Hence, there is only one eigen vector corresponding to the repeated root l = 1. 3 –1 1 –1 5 –1 , find eigen values and eigen vectors for the 1 –1 3

Example 2: If A =

following matrices: (i) AT (ii) A-1 (iii) Ap (iv) 4A-1 (v) A2 (vi) A2 - 2A + I (vii) A3 + 2I

(viii) adj A .

3 –1 1 A = –1 5 –1

Solution:

1 –1

3

The characteristic equation is A – lI = 0 3 l 1

1 5 l

=0 l

1

l3

1

11l2 + 36l

36 = 0 l = 2, 3, 6

(i) Eigen values of AT = lT (ii) Eigen values of A 1 = l

: 1

l1I] X = O

1 –1 1 –1 3 –1 1 –1 1

x 0 y = 0 z 0

:

18, 12, 6

: 1

(v) Eigen values of A2 = l2 (vi) Eigen values of A2 2A + I = l2 (vii) Eigen values of A3 + 2I = l3 + 2 A (viii) Eigen values of adj A = (a) For l = 2, [A

: : :

:

(iii) Eigen values of Aq = (iv) Eigen values of 4A 1 = 4l

2, 3, 6 1 1 1 , , 2 3 6 2, 3, 6 4 2 2, , 3 3 4, 9, 36 1, 4, 25 10, 29, 218

: 2l + 1

Matrices x y+z=0 x + 3y z = 0 x y z =– = –1 1 1 1 1 –1 3 –1 –1 –1 –1 3

(b) For l = 3,

[A

y z = 0 2 y z = 0 –1

X1 =

1 0 –1

l2I] X = O

0 –1 1 –1 2 –1 1 –1

x = –2 x = 1

0

x 0 y = 0 z 0

y+z=0 x + 2y z = 0 x y=0 x y z =– = –1 1 0 1 0 –1 2 –1 –1 –1 –1 2 x = –1 x = 1

y z = –1 –1 y z = 1 1

1 X2 = 1 1 (c) For l = 6,

[A

l3I ] X = O

–3 –1 1 –1 –1 –1 1 –1 –3

x 0 y = 0 z 0

3x y + z = 0 –x y z = 0 x y 3z = 0

11.103

Engineering Mathematics

11.104

x y z =– = –1 1 –3 1 –3 –1 –1 –1 –1 –1 –1 –1 x y z = = 2 –4 2 x y z = = 1 –2 1

1 X3 = –2 1 Eigen vectors remain the same as eigen vectors of A for all the above matrices. Example 3: Find the values of l which satisfy the equation A100 X = lX 2 1 –1 where A = 0 –2 –2 . 1

1

0 2 1 –1 0 –2 –2 A= 1 1 0

Solution: The characteristic equation is

A– I = 0 2–l 0

1 –2 – l

1

1

–1 –2 = 0 –l

l3 l = 0 l = 0, 1, 1 If l is an eigen value of A, it satisfies the equation AX = lX . Thus, for equation A100 X = mX, m represents eigen values of A100. Eigen values of A100 = l100, i.e., 0, 1, 1. Hence, values of m are 0, 1, 1. Example matrices: 1 (i) 0 -1

Solution:

4: Determine algebraic and geometric multiplicity of the following 2 2 2 1 2 2

(ii)

2 1 0 0 2 1 0 0 2 1 2 2 A= 0 2 1 –1 2 2

Matrices

11.105

The characteristic equation is A– I = 0

1– 0 –1

2 2– 2

l3

2 1 2–

=0

5l2 + 8l

4=0 l = 1, 2, 2 [ A − 1I ] X = O

(a) For l = 1

0 2 2 0 1 1 –1 2 1 R13

x 0 y = 0 z 0

–1 2 1 0 1 1

x 0 y = 0

0 R3 –1 0

2 2 2R2 2 1 1 1

z

0

x 0 y = 0

0 0 0 z 0 Rank of matrix = 2 Number of unknowns = 3 Number of linearly independent solutions = 3 2 = 1 Hence, geometric multiplicity is 1. Since eigen value 1 is non-repeated, its algebraic multiplicity is 1. Algebraic multiplicity = geometric multiplicity = 1 (b) For l = 2 [A − 2I ] X = O –1 2 2 0 0 1 –1 2 0 R3 R1

x 0 y = 0 z

0

–1 2 2 0 0 1 0 0 –2 R3 + 2R2

x 0 y = 0 z 0

–1 2 2 0 0 1 0 0 0

x 0 y = 0 z 0

Engineering Mathematics

11.106

Rank of matrix = 2 Number of unknowns = 3 Number of linearly independent solutions = 3 2 = 1 Hence, geometric multiplicity is 1. Since eigen value 2 is repeated twice, its algebraic multiplicity is 2. 2 1 0 (ii) A= 0 2 1 0 0 2 The characteristic equation is A– I = 0 2– 0 0

1 2– 0

0 1 2–

=0

l3 6l2 + 12l For l = 2, [A

8=0 l = 2, 2, 2

l I]X = O 0 1 0 0 0 1

x 0 y = 0

0 0 0

z

0

Rank of matrix = 2 Number of unknowns = 3 Number of linearly independent solutions = 3 2 = 1 Hence, geometric multiplicity is 1. Since eigen value 2 is repeated thrice, its algebraic multiplicity is 3. Example 5: Find the characteristic root and characteristic vectors of A=

cos q sin q

- sin q cos q

and verify that characteristic roots are of unit modulus and

characteristic vectors are orthogonal. A=

Solution:

cos sin

The characteristic equation is A– I = 0 cos – sin (cosq

– sin cos –

=0

l)2 + sin2q = 0 (cosq l)2 = sin2q

– sin cos

Matrices

cosq

11.107

l = ± i sinq l = cosq ± i sinq = e±iq = cos 2 + sin 2

= cos ± i sin

=1

(a) For l = cosq + i sinq, [A l I ] X = O – i sin – sin x 0 = sin – i sin y 0 sinq y = 0

i sinq x Taking x = 1, y =

i 1 X1 = –i

(b) For l = cosq

i sinq, [ A –

2

I]X = O

i sin sin

– sin i sin

i sinq x Taking

x 0 = y 0 sinq y = 0 x = 1, y = i 1 X2 = i

For orthogonality of complex matrix,

1

X 1 X 2 = [1 i ]

i = [1 + i ] = [0] =O Similarly, X1 X2q = O Hence, characteristic vectors are orthogonal. 2

Example 6: If A =

2 1 - 2i , verify whether eigen vectors are mutually 1 + 2i -2

orthogonal. Solution:

A=

2 1 – 2i 1 + 2i –2

The characteristic equation is A– I = 0

2– 1 + 2i

1 – 2i =0 –2 – l2 9 = 0 l = 3, 3

Engineering Mathematics

11.108

(a) For l = 3, [A

l1I ]X = O 1 – 2i ⎤ ⎡ x ⎤ ⎡0 ⎤ ⎡ 5 = ⎢1 + 2i 1 ⎥⎦ ⎢⎣ y ⎥⎦ ⎢⎣0 ⎥⎦ ⎣ 5x + (1

Taking

2i)y = 0 x=1

2i, y = 5

1 – 2i ⎤ X1 = ⎡⎢ ⎥ ⎣ –5 ⎦ (b) For l = 3, [A

l2I]X = O ⎡ –1 1 – 2i ⎤ ⎡ x ⎤ ⎡0 ⎤ = ⎢1 + 2i –5 ⎥⎦ ⎢⎣ y ⎥⎦ ⎢⎣0 ⎥⎦ ⎣ x + (1 2i)y = 0 x = 1 – 2i, y = 1 ⎡1 − 2i ⎤ X2 = ⎢ ⎥ ⎣ 1 ⎦

Taking

For complex matrix, ⎡1 − 2i ⎤ 5] ⎢ ⎥ = [(1 + 2i) (1 – 2i) – 5] = [0] ⎣ 1 ⎦

X1q X2 = [1 + 2i

Similarly, X1 X2q = O Hence, eigen vectors are mutually orthogonal. Example 7: Find orthogonal eigen vectors for the following matrix: ⎡ 1 2 3⎤ A = ⎢ 2 4 6⎥ ⎢ ⎥ ⎣ 3 6 9⎦ ⎡1 2 3⎤ A = ⎢2 4 6⎥ ⎢ ⎥ ⎣3 6 9⎦

Solution: The characteristic equation is

A– I = 0 1– 2 3

2 4– 6

3 6 9–

=0

l3 14 l2 = 0 l = 0, 0, 14 (a) For l = 14, [A

l1I ]X = O 2 3⎤ ⎡ x ⎤ ⎡ 0 ⎤ ⎡ –13 ⎢ 2 –10 6 ⎥ ⎢ y ⎥ = ⎢0 ⎥ ⎢ ⎥⎢ ⎥ ⎢ ⎥ 6 –5⎦ ⎣ z ⎦ ⎣0 ⎦ ⎣ 3

Matrices 13x + 2y + 3z = 0 2x 10y + 6z = 0 3x + 6y 5z = 0 x y 2 3 –13 3 –10 6 2 6 x y z = = 42 84 126 x y z = = 1 2 3

11.109

z –13 2 2 –10

1 X1 = 2 3 (b) For l = 0, [A

l2I ] X = O 1 2 3 2 4 6

x 0 y = 0

3 6 9 z 0 R2 2R1, R3 3R1 1 2 3 0 0 0 0 0 0

x 0 y = 0 z 0

Rank of matrix = 1 Number of unknowns = 3 Number of linearly independent solutions = 3 1 = 2 x + 2y + 3z = 0 Taking z = 0 and y = 1, x = 2 –2 X2 = 1 0 We must choose X3 such that X1, X2, X3 are orthogonal. l Let X 3 = m n Since X1 and X3 are orthogonal, X1T X3 = O ⎡l ⎤ [1 2 3] ⎢⎢ m⎥⎥ = [0] ⎢⎣ n ⎥⎦ l + 2m + 3n = 0

… (1)

Engineering Mathematics

11.110

Also, X2 and X3 are orthogonal, X2T X3 = O

[

⎡l ⎤ 2 1 0] ⎢⎢ m ⎥⎥ = [0] ⎢⎣ n ⎥⎦ 2l + m = 0

… (2)

Solving Eqs. (1) and (2), l m n =– = 2 3 1 3 1 2 1 0 –2 0 –2 1 l = –3 l = 3

m n = –6 5 m n = 6 −5

⎡3⎤ X 3 = ⎢⎢ 6 ⎥⎥ ⎢⎣ −5⎥⎦ Exercise 11.10 1. Find the eigen values and eigen vectors for the following matrices: (i)

9 –1 3 –1 –7

(ii)

(iii)

(v)

(vii)

(viii)

2 1 1 2 3 2 3 3 4

2 –3 1 –6 0

1 0 0 2 0 1 3 1 0

(ix)

2 4 –6 4 2 –6 –6 –6 –15

(x)

7 0 –2 0 5 –2 –2 –2 6

0 –3

4 0 1 –2 1 0 –2 0 1

–2 2

–1 –2

1 –1 2 1

2 2 1 1 3 1 1 2 2

1 –7

–1 1 0 –2 0

(iv)

9 3

1 1 –2 –1 2 1 0

(vi)

Matrices

(xi)

7 –2 –2 –2 1 4 –2 4 1

(xii)

3 –1 1 –1 5 –1 1 –1 3

(xiii)

6 –2 2 –2 3 –1 2 –1 3

(xiv)

7 –2 1 –2 10 –2 1 –2 7

(xv)

2 –1 1 –1 2 –1 1 –1

2

⎡ ⎡1⎤ ⎡ 1⎤ ⎡ 4 ⎤ ⎤ ⎢ ⎢1⎥ , ⎢ 0 ⎥ , ⎢ 1⎥ ⎥ Ans. : (i) − 1 , 0 , 2 ; ⎢ ⎢⎥ ⎢ ⎥ ⎢ ⎥ ⎥ ⎢ ⎢⎣1⎥⎦ ⎢⎣ −1⎥⎦ ⎢⎣ −3⎥⎦ ⎥ ⎢ ⎥ ⎡1 ⎤ ⎡1⎤ ⎡ 3⎤ ⎢ ⎥ ⎢ ⎥ (ii) −1, 2, 1; ⎢⎢0 ⎥⎥ , ⎢⎢3⎥⎥ , ⎢⎢ 2⎥⎥ ⎢ ⎥ ⎢⎣1 ⎥⎦ ⎢⎣1⎥⎦ ⎢⎣1 ⎥⎦ ⎢ ⎥ ⎢ ⎥ ⎡1 ⎤ ⎡ 1⎤ ⎡ 1⎤ ⎥ ⎢ ⎢ (iii) − 1, − 2, − 3; ⎢⎢0 ⎥⎥ , ⎢⎢ −1⎥⎥ , ⎢⎢ 2⎥⎥ ⎥ ⎢ ⎥ ⎢⎣0 ⎥⎦ ⎢⎣ 0 ⎥⎦ ⎢⎣ −2⎥⎦ ⎥ ⎢ ⎢ ⎥ ⎡0 ⎤ ⎡ −1⎤ ⎡ −1⎤ ⎢ ⎥ ⎢ ⎥ ⎢ ⎥ ⎢ ⎥ ⎢ ⎥ (iv) 1, 2, 3; 1 , 2 , 1 ⎢ ⎥ ⎢ ⎥ ⎢ ⎥ ⎢ ⎥ ⎢⎣0 ⎥⎦ ⎢⎣ 2⎥⎦ ⎢⎣ 1⎥⎦ ⎢ ⎥ ⎢ ⎥ ⎡1 ⎤ ⎡ 0 ⎤ ⎡ 1⎤ ⎢ ⎥ ⎢ ⎥ ⎢ ⎥ ⎢ ⎥ (v) 1, 1, 7; 2 , 1 , 0 ⎢ ⎥ ⎢ ⎥ ⎢ ⎥ ⎢ ⎥ ⎢ ⎥ ⎢⎣ 3⎥⎦ ⎢⎣ −1⎥⎦ ⎣⎢ −1⎥⎦ ⎢ ⎥ ⎢ ⎥ 1 2 1 ⎡⎤ ⎡ ⎤ ⎡ ⎤ ⎢ ⎥ ⎢ ⎥ ⎢ ⎥ ⎢ ⎥ (vi) 5, 1, 1; 1 , −1 , 0 ⎢ ⎥ ⎢⎥ ⎢ ⎥ ⎢ ⎥ ⎢ ⎥ ⎢ ⎥ ⎢ ⎥ ⎢ ⎥ 1 0 1 − ⎣ ⎦ ⎣ ⎦ ⎣ ⎦ ⎣ ⎦

11.111

(xvi)

2 0 1 0 3 0 1 0 2

(xvii)

1 2 3 0 2 3 0 0 2

(xviii)

3 10 5 –2 –3 –4 3 5 7

(xix)

–3 –7 –5 2 4 3 1 2 2

(xx)

2 1 0 0 2 1 0 0 2

⎡ ⎡ 1⎤ ⎡ 2⎤ ⎡ 3⎤ ⎤ ⎢ ⎢ 2⎥ , ⎢ −1⎥ , ⎢0 ⎥ ⎥ (vii) 5 , − 3 , − 3 ; ⎢ ⎢ ⎥ ⎢ ⎥ ⎢ ⎥⎥ ⎢ ⎢⎣ −1⎥⎦ ⎢⎣ 0 ⎥⎦ ⎢⎣1 ⎥⎦ ⎥ ⎢ ⎥ ⎡ 0⎤ ⎡0⎤ ⎡1⎤ ⎥ ⎢ ⎢(viii) − 1, 1, 1; ⎢ −1⎥ , ⎢1 ⎥ , ⎢0 ⎥ ⎥ ⎢ ⎥ ⎢ ⎥ ⎢ ⎥ ⎥ ⎢ ⎢⎣ 1⎥⎦ ⎢⎣1 ⎥⎦ ⎢⎣0 ⎥⎦ ⎥ ⎢ ⎢ ⎥ ⎡ 0 ⎤ ⎡ 2⎤ ⎡ 1 ⎤ ⎥ ⎢ ⎢(ix) − 2, 9, − 18; ⎢ −1⎥ , ⎢ 2⎥ , ⎢ 1 ⎥ ⎥ ⎢ ⎥ ⎢ ⎥ ⎢ ⎥⎥ ⎢ ⎢ ⎣⎢ 0 ⎦⎥ ⎣⎢1 ⎦⎥ ⎣⎢ 4 ⎦⎥ ⎥ ⎢ ⎥ ⎡ 1 ⎤ ⎡ 2⎤ ⎡ 2 ⎤ ⎢ ⎥ ⎢ ⎥ ⎢ ⎥ ⎢ ⎥ ⎢(x) 3, 6, 9; 2 , −2 , 1 ⎥ ⎢ ⎥ ⎢ ⎥ ⎢ ⎥ ⎢ ⎥ ⎢⎣ 2⎥⎦ ⎢⎣ 1⎥⎦ ⎢⎣ −2⎥⎦ ⎢ ⎥ ⎢ ⎡ 0 ⎤ ⎡1⎤ ⎡ −2⎤ ⎥ ⎢ ⎥ ⎢ ⎥ ⎢⎥ ⎢(xi) − 3, 3, 9; ⎢ 1 ⎥ , ⎢1⎥ , ⎢⎢ 1 ⎥⎥ ⎥ ⎢ ⎢⎣ −1⎥⎦ ⎢⎣1⎥⎦ ⎢⎣ 1 ⎥⎦ ⎥ ⎢ ⎥ ⎢ − 1 1 1 ⎡ ⎤ ⎡⎤ ⎡ ⎤ ⎥ ⎢ ⎢ ⎥ ⎢⎥ ⎢ ⎥ ⎥ ⎢(xii) 2, 3, 6; ⎢ 0 ⎥ , ⎢1⎥ , ⎢ −2⎥ ⎥ ⎢ ⎥ ⎣⎢ 1⎦⎥ ⎣⎢1⎦⎥ ⎣⎢ 1⎦⎥ ⎦ ⎣

11.112

Engineering Mathematics

⎡ ⎡ 2⎤ ⎡ 0 ⎤ ⎡ 1⎤ ⎤ ⎢ ⎢ ⎥ ⎢ ⎥ ⎢ ⎥⎥ ⎢(xiii) 8, 2, 2; ⎢ −1⎥ , ⎢ 1 ⎥ , ⎢ 1⎥ ⎥ ⎢ ⎢⎣ 1⎥⎦ ⎢⎣ 1 ⎥⎦ ⎢⎣ −1⎥⎦ ⎥ ⎢ ⎥ ⎡ 1⎤ ⎡ 1⎤ ⎡1⎤ ⎥ ⎢ ⎢(xiv) 12, 6, 6; ⎢ −2⎥ , ⎢ 0 ⎥ , ⎢1⎥ ⎥ ⎢ ⎥ ⎢ ⎥ ⎢ ⎥⎥ ⎢ ⎢⎣ 1⎥⎦ ⎢⎣ −1⎥⎦ ⎢⎣1⎥⎦ ⎥ ⎢ ⎢ ⎥ ⎡ 1⎤ ⎡1 ⎤ ⎡ −1⎤ ⎥ ⎢ ⎢( xv ) 4, 1, 1; ⎢ −1⎥ , ⎢1 ⎥ , ⎢ 1⎥ ⎥ ⎢ ⎥ ⎢ ⎥ ⎢ ⎥ ⎥ ⎢ ⎢ ⎣⎢ 1⎥⎦ ⎢⎣0 ⎥⎦ ⎢⎣ 2⎥⎦ ⎥ ⎢ ⎥ ⎡ 1⎤ ⎡1⎤ ⎡ 1⎤ ⎥ ⎢ ⎢(xvi) 1, 3, 3; ⎢ 0 ⎥ , ⎢1⎥ , ⎢ −2⎥ ⎥ ⎢ ⎥ ⎢⎥ ⎢ ⎥ ⎥ ⎢ ⎢⎣ −1⎥⎦ ⎢⎣1⎥⎦ ⎢⎣ 1⎥⎦ ⎥ ⎢ ⎢ ⎥ ⎡ 1 ⎤ ⎡ 2⎤ ⎢ ⎥ ⎢ ⎥ ⎢ ⎥ ⎢(xvii) 1, 2, 2; ⎢0 ⎥ , ⎢1 ⎥ ⎥ ⎢ ⎥ ⎢⎣0 ⎥⎦ ⎢⎣0 ⎥⎦ ⎢ ⎥ ⎢ ⎥ ⎡ 1⎤ ⎡ 5⎤ ⎢ ⎥ ⎢ ⎥ ⎢ ⎥ ⎢(xviii) 3, 2, 2; ⎢ 1⎥ , ⎢ 2⎥ ⎥ ⎢ ⎥ ⎢⎣ −2⎥⎦ ⎢⎣ −5⎥⎦ ⎢ ⎥ ⎢ ⎥ ⎡ −3⎤ ⎢ ⎥ ⎢ ⎥ ⎢(xix) 1, 1, 1; ⎢ 1⎥ ⎥ ⎢ ⎥ ⎢⎣ 1⎥⎦ ⎢ ⎥ ⎢ ⎥ ⎡1 ⎤ ⎢ ⎥ ⎢0 ⎥ (xx) , 2 , 2 ; 2 ⎢ ⎥ ⎢ ⎥ ⎢⎣ ⎥⎦ ⎢⎣0 ⎥⎦ 2. Determine algebraic and geometric multiplicity of the following matrices: 2 1 1 0 1 0 (i) 0 0 1 (ii) 1 2 1 0 0 1 1 –3 3

⎡ Ans. : (i) l = 1, 1, 1; GM = 1, AM = 3 ⎤ ⎢ (ii) For l = 1, AM = 2, GM = 2⎥⎥ ⎢ ⎢⎣ For l = 3, AM = 1, GM M = 1 ⎥⎦

2 2 1 3. If A = 1 3 1 , find eigen values 1 2 2 of the following matrices: (i) A3 + I (ii) A 1 (iii) A2 2A + I (iv) adj A (v) A3 3A2 + A 1 ⎤ ⎡ ⎢ Ans. : (i) 2, 2, 126 (ii) 1, 1, 5 ⎥ ⎢ ⎥ (iii) 0, 0, 16 (iv ) 5, 5, 1⎥ ⎢ ⎢⎣ ⎥⎦ ( v ) − 1, −1, 55 4. Verify that X = [2, 3, 2, 3]T is an eigen vector corresponding to the eigen value l = 2 of the matrix 1 –4 –1 –4 2 0 5 –4 A= –1 1 –2 3 –1 4 –1 6

3 –1 1 5. If A = –1 3 –1 , then check 1 –1

3

whether eigen vectors of A are orthogonal. 3 10 5 6. If A = –2 –3 –4 , then verify 3

5

7

whether eigen vectors are linearly independent or not.

11.12 CAYLEY–HAMILTON THEOREM Statement: Every square matrix satisfies its own characteristic equation. Proof: Let A be n-rowed square matrix. Its characteristic equation is |A l I| = ( 1)n (ln + a1ln 1 + a2 ln 2 + ………+ an) (A l I ) adj (A l I ) = |A l I|I

Matrices

11.113

Since adj [A l I] has element as cofactors of elements of |A l I|, the elements of adj [A l I] are polynomials in l of degree n 1 or less, as the elements of [A l I] are at most of the first degree in l . Hence, adj [A l I] can be written as a matrix polynomial in l . adj (A l I) = B0l n 1 + B1l n 2 + …………+ Bn 2l + Bn 1 B0, B1, ………………………Bn 1 are matrices of order n. (A l I) adj [A l I] = (A l I) [B0l n 1 + B1l n 2 + …………+ Bn 2l + Bn 1] |A l I| I = (A l I) [B0l n 1 + B1l n 2 + …………+ Bn 2l + Bn 1] n n ( 1) [Il + a1Il n 1 + a2Il n 2 + ……..+ an 1Il + an I ] = ( IB0) l n + (AB0 IB1) l n 1 + (AB1 IB2) l n 2 + …………+ (ABn 2 IBn 1)l + ABn 1 Comparing both the sides, IB0 = ( 1)nI AB0 IB1 = ( 1)n a1I AB1 IB2 = ( 1)n a2I .. .. .. .. ABn 2 IBn 1 = ( 1)n an 1I ABn 1 = ( 1)n anI Premultiplying the above equations successively by An, An 1, An 2, ……..I and adding, ( 1)n [An + a1An 1 + a2An 2 + ………+ anI] = O Hence, An + a1An 1 + a2An 2 + ……….+ anI = O ... (1) Corollary: If A is a non singular matrix. i.e., |A| 0, then premultiplying Eq. (1) by A 1, we get An 1 + a1An 2 + a2An 3 +………+ anA 1 = O 1 A 1 = – n [An 1 + a1An 2 + ………+ an 1I] a Example 1: Verify Cayley–Hamilton theorem for the following matrix and hence, find A-1 and A4. 2 -1 1 A = -1 2 -1 . 1 -1

2

2 –1 1 A = –1 2 –1 1 –1 2

Solution: The characteristic equation is |A

2– –1 1

–1 2– –1 l3

lI| = 0

1 –1 = 0 2–

6l2 + 9l 4 = 0

Engineering Mathematics

11.114

2 –1 1 2 –1 1 6 –5 5 A = –1 2 –1 –1 2 –1 = –5 6 –5 1 –1 2 1 –1 2 5 –5 6 2

⎡ 6 –5 5⎤ ⎡ 2 –1 1⎤ ⎡ 22 –21 21⎤ A = ⎢⎢ –5 6 –5⎥⎥ ⎢⎢ –1 2 –1⎥⎥ = ⎢⎢ –221 22 –21⎥⎥ ⎢⎣ 5 –5 6 ⎥⎦ ⎣⎢ 1 –1 2⎥⎦ ⎢⎣ 21 –21 22⎥⎦ 3

3

A

9⎤ ⎡ 4 0 0 ⎤ ⎡ 22 –21 21⎤ ⎡ 36 –30 30 ⎤ ⎡ 18 –9 ⎢ –21 22 –21⎥ ⎢ –30 36 –30 ⎥ + ⎢ – 9 18 – 9⎥ – ⎢ 0 4 0 ⎥ 6A + 9A 4I = ⎢ ⎥⎢ ⎥ ⎢ ⎥ ⎢ ⎥ ⎢⎣ 21 –21 22⎥⎦ ⎢⎣ 30 –30 36 ⎥⎦ ⎢⎣ 9 –9 18⎥⎦ ⎢⎣ 0 0 4 ⎥⎦ 2

⎡0 0 0⎤ = ⎢⎢0 0 0⎥⎥ = O ⎢⎣0 0 0⎥⎦ The matrix A satisfies its own characteristic equation. Hence, Cayley–Hamilton theorem is verified. Premultiplying by A 1, A 1(A3 6A2 + 9A 4I) = O A2 6A + 9I 4A 1 = O 1 2 4A = A 6A + 9I 6 –5 5 12 –6 6 9 0 0 = –5 6 –5 – –6 12 –6 + 0 9 0 = 5 –5 1 A = 4

6

6 –6 12

0 0 9

3 1 –1 1 3 1 –1 1

3

3 1 –1 1 3 1

–1

–1 1

3

Multiplying by A,

A4 = 6A3

A(A3 6A2 + 9A 4I) = O A4 6A3 + 9A2 4A = O 9A2 + 4A

⎡ 132 –126 126 ⎤ ⎡ 54 –45 45⎤ ⎡ 8 –4 4 ⎤ = ⎢⎢ –126 132 –126 ⎥⎥ – ⎢⎢ –45 54 –45⎥⎥ + ⎢⎢ –4 8 –4 ⎥⎥ ⎢⎣ 126 –126 132⎥⎦ ⎢⎣ 45 –45 54 ⎥⎦ ⎢⎣ 4 –4 8⎥⎦

⎡ 86 – 85 85⎤ = ⎢⎢ – 85 86 – 85⎥⎥ ⎢⎣ 85 – 85 86 ⎥⎦

Matrices

11.115

0 c -b Example 2: Show that the matrix A = - c 0 a satisfies Cayley–Hamilb -a 0 ton theorem and hence find A-1, if it exists. 0 c –b A = –c 0 a b –a 0

Solution: The characteristic equation is |A

– –c b

c – –a

l I| = 0

–b a =0 –

l 3 + (a2 + b2 + c2)l = 0 0 0 –c2 – b2 c –b c –b 0 a –c 0 a = A2 = – c ab 0 0 b –a b –a ac –c2 – b2 A = ab 3

ab –c – a2 2

bc

ab –c2 – a2

ac bc

0 –c

c 0

–b a

bc

–b 2 – a 2

b

–a

0

ac

0 3 = c + ca 2 + cb 2

– c 3 – cb 2 – ca 2 0

b3 + bc 2 + ba 2 – ab 2 – ac 2 – a 3

ac 2 + ab 2 + a 3

0

– bc 2 – b3 – a 2 b

ac bc –b 2 – a 2

0

c –b = –(a + b + c ) – c 0 a = – (a2 + b2 + c2) A 0 b –a 2

2

2

A3 + (a2 + b2 + c2) A = O The matrix A satisfies its own characteristic equation. Hence, Cayley–Hamilton theorem is verified. 0 A = –c

c –b 0 a = – c(0 – ab) – b(ac – 0)

b –a

0 = abc

Hence, A 1 does not exist.

abc = 0

Engineering Mathematics

11.116

1 4 and 2 3 and also express

Example 3: Find the characteristic roots of the matrix A = verify Cayley–Hamilton theorem for this matrix. Find A-1 A5 - 4A4 - 7A3 + 11A2 - A - 10I as a linear polynomial in A. Solution:

A=

1 4 2 3

The characteristic equation is |A 1– 2

l I| = 0 4 3–

=0

4l 5 = 0 l = 1, 5 By Cayley–Hamilton theorem, matrix A must satisfy its characteristic equation, i.e., A2 4A 5I = O l2

A2 =

1 4 2 3

1 4 9 16 = 2 3 8 17

⎡9 16 ⎤ ⎡ 4 16 ⎤ ⎡ 5 0 ⎤ ⎡0 0 ⎤ 5I = ⎢ ⎥=O ⎥=⎢ ⎥–⎢ ⎥–⎢ ⎣8 17⎦ ⎣ 8 12⎦ ⎣0 0 ⎦ ⎣0 0 ⎦ Hence, the matrix A satisfies Cayley–Hamilton theorem. Premultiplying by A 1, A 1(A2 4A 5I) = O A 4I 5A 1 = O 1 A 1 = ( A – 4I ) 5

A2

4A

=

… (1)

1 –3 4 5 2 –1

Eigen value of A5 – 4A4 – 7A3 + 11A2 – A – 10I is l5 – 4 l4 – 7 l3 + 11 l2 – l – 10 Let l5 – 4 l4 – 7 l3 + 11 l2 – l – 10 = (l2 – 4 l – 5) f (l) + a l + b = 0 + al + b … (2) Putting l = – 1, 4 = a (– 1) + b a – b = –4 … (3) Putting l = 5, 10 = a (5) + b 5a + b = 10 … (4) Solving equations (2) and (3), a = 1, b = 5

Matrices

11.117

Replacing l by A is Eq. (2) and substituting values of a and b, A5 – 4A4 – 7A3 + 11A2 – A – 10I = A + 5I which is a linear polynomial in A. ⎡ 2 1 1⎤ Example 4: Find the characteristic equation of the matrix A = ⎢ 0 1 0 ⎥ and ⎢ ⎥ ⎣ 1 1 2⎦ hence, find the matrix represented by A8 - 5A7 + 7A6 - 3A5 + A4 - 5A3 + 8A2 - 2A + I. ⎡2 1 1⎤ A = ⎢0 1 0⎥ ⎢ ⎥ ⎣1 1 2⎦

Solution: The characteristic equation is

|A – lI| = 0 2–l 1 1 0 1– l 0 =0 1 1 2–l l3 5l2 + 7l 3 = 0 By Cayley–Hamilton theorem, A3 5A2 + 7A 3I = O Now, A8 5A7 + 7A6 3A5 + A4 5A3 + 8A2 2A + I = A5(A3 5A2 + 7A 3I) + A(A3 5A2 + 7A 3I) + (A2 + A + I) = (A3 5A2 + 7A 3I) (A5 + A) + (A2 + A + I) = 0 + (A2 + A + I) = A2 + A + I

⎡2 1 1⎤ ⎡2 1 A = ⎢0 1 0⎥ ⎢0 1 ⎢ ⎥⎢ ⎣1 1 2 ⎦ ⎣1 1 ⎡5 4 4⎤ ⎡2 A2 + A + I = ⎢ 0 1 0⎥ + ⎢0 ⎢ ⎥ ⎢ ⎣ 4 4 5 ⎦ ⎣1 2

A8

5A7 + 7A6

3A5 + A4

1 ⎤ ⎡5 4 4⎤ 0⎥ = ⎢0 1 0⎥ ⎥ ⎢ ⎥ 2⎦ ⎣4 4 5⎦ 1 1 ⎤ ⎡1 0 0 ⎤ ⎡8 5 5 ⎤ 1 0 ⎥ + ⎢0 1 0⎥ = ⎢0 3 0⎥ ⎥ ⎢ ⎥ ⎢ ⎥ 1 2⎦ ⎣0 0 1 ⎦ ⎣5 5 8 ⎦ ⎡8 5 5 ⎤ 5A3 + 8A2 2A + I = ⎢0 3 0 ⎥ ⎢ ⎥ ⎣5 5 8 ⎦

⎡ 1 2⎤ Example 5: Apply Cayley–Hamilton theorem to A = ⎢ and deduce that ⎣ 2 –1⎥⎦ A8 = 625I. Solution:

⎡1 2 ⎤ A=⎢ ⎥ ⎣ 2 –1⎦

11.118

Engineering Mathematics

The characteristic equation is |A 1– l 2

lI| = 0

2 =0 –1 – l l2 5 = 0

By Cayley–Hamilton theorem, A2 = 5I A4 = 25I A8 = 625I

⎡ 1 0 0⎤ Example 6: If A = ⎢ 1 0 1⎥ , prove by induction that for every integer n ê 3, ⎢ ⎥ ⎣ 0 1 0⎦ An = An-2 + A2 - I. Hence, find A50. Solution:

⎡1 0 0 ⎤ A = ⎢1 0 1 ⎥ ⎢ ⎥ ⎣0 1 0⎦

The characteristic equation is |A lI| = 0 1– l 0 0 1 –l 1 = 0 0 1 –l l3 l2 l + 1 = 0 By Cayley–Hamilton theorem, A3 A2 A + I = O A3 = A2 + A I = A3 2 + A2 I … (1) n n 2 2 Hence, A = A + A I is true for n = 3. Assuming that Eq. (1) is true for n = k, Ak = Ak 2 + A2 I Multiplying both the sides by A, Ak+1 = Ak 1 + A3 A 3 Substituting the value of A , Ak+1 = Ak 1 + (A2 + A I) A = A(k+1) 2 + A2 I Hence, An = An 2 + A2 I is true for n = k + 1. Thus, by mathematical induction, it is true for every integer n 3. We have, An = An 2 + A2 I = (An 4 + A2 I) + A2 I = An 4 + 2(A2 I) = (An 6 + A2 I) + 2 (A2 I) = An 6 + 3(A2 I) .. .. n A = An 2r + r(A2 I)

Matrices

11.119

n = 50 and r = 24, we get A = A50 2(24) + 24 (A2 I) = A2 + 24A2 24I = 25A2

Putting

50

24I

⎡1 0 0 ⎤ ⎡1 0 0 ⎤ ⎡1 0 0 ⎤ A = ⎢1 0 1 ⎥ ⎢1 0 1 ⎥ = ⎢1 1 0 ⎥ ⎢ ⎥⎢ ⎥ ⎢ ⎥ ⎣0 1 0 ⎦ ⎣0 1 0 ⎦ ⎣1 0 1 ⎦ 2

⎡ 25 0 0 ⎤ ⎡ 24 0 0 ⎤ ⎡ 1 0 0⎤ ⎢ ⎥ ⎢ ⎥ A = 25 25 0 – 0 24 0 = ⎢ 25 1 0 ⎥ ⎢ ⎢ ⎥ ⎢ ⎥ ⎥ ⎣ 25 0 25⎦ ⎣ 0 0 24 ⎦ ⎣ 25 0 1 ⎦ 50

Exercise 11.11 1. Verify Cayley–Hamilton theorem for the matrix A and hence, find A 1 and A4. 1 2 –2 1 2 3 (i) –1 3 0 (ii) 2 –1 4 0 –2

1

2 0 –1 0 2 0

(iii)

–1 0

2

3 (iv)

1 –1

1 –1

2 –2 3 0

0 –2

1

Ans. :

⎡ 1 2 0⎤ A = ⎢ 2 –1 0 ⎥ satisfies the charac⎢ ⎥ ⎣ 0 0 –1⎦ teristic equation and hence, find A 2. ⎡ Ans. : A3 + A2 − 5A − 5I = O,⎤ ⎢ ⎥ ⎡1 0 0⎤ ⎢ ⎥ 1⎢ –2 ⎢ ⎥ A = 0 1 0⎥ ⎥ 5⎢ ⎢ ⎥ ⎣0 0 1⎦ ⎣ ⎦

248 101 218

3. Use Cayley–Hamilton theorem to find 2A5 3A4 + A2 4I, where ⎡ 3 1⎤ . A=⎢ ⎥ ⎣ –1 2 ⎦ ⎡ ⎡ 11 138⎤ ⎤ ⎢ Ans. : 138A − 403I = ⎢ –138 127 ⎥ ⎥ ⎣ ⎦⎦ ⎣ 1 4 4. If A = , find A7 9A2 + I. 1 1

272 109 50 104 98 204

5. Verify Cayley–Hamilton theorem for

3 2 6 –55 104 24 (i) 1 1 2 , –20 –15 32 2 2 5 (ii)

2. Verify that the matrix

32 –42 13

–3 5 11 1 14 –10 2 , 40 5 5 –5

4 0 2 41 0 –40 1 (iii) 0 3 0 , 0 16 0 6 2 0 4 –40 0 41 3 2 6 –55 104 24 (iv) 1 1 2 , –20 –15 32 2 2 5 32 –40 –23

[Ans. : 609A + 640I] 1 2⎤ 2 4⎤ (i) A = ⎡⎢ (ii) A = ⎡⎢ ⎥ ⎥ ⎣3 4 ⎦ ⎣1 2 ⎦ and hence, find A 1 and A3 5A2. ⎤ 1 ⎤ ⎡ ⎡ –2 ⎥ ⎢ Ans. : (i) ⎢1.5 –0.5⎥ , 2A ⎣ ⎦ ⎥ ⎢ ⎢⎣ (ii) A–1 does not exist, A2 ⎥⎦

11.120

Engineering Mathematics

6A8 + 10A7 3A6 + ⎡ 1 2 3⎤ A + I, where A = ⎢ –1 3 1⎥ . ⎢ ⎥ ⎣ 1 0 2⎦

6. Compute A9

⎡ ⎡ 2 2 3⎤ ⎤ ⎢ Ans. : ⎢ –1 4 1⎥ ⎥ ⎢ ⎢ ⎥⎥ ⎢⎣ ⎣ 1 0 3⎦ ⎥⎦

7. Verify Cayley–Hamilton theorem for A=

1 2 and evaluate 2 2

2A4

5A3 7A + 6I Ans. :

36 32 32 52

11.13 MINIMAL POLYNOMIAL AND MINIMAL EQUATION OF A MATRIX Let f (x) be a polynomial in x and A be a square matrix of order n. If f (A) = O, then f (x) is said to annihilate the matrix A. Note: The characteristic polynomial of A is a non-zero polynomial that annihilates the matrix A.

11.13.1 Monic Polynomial A polynomial in x in which the coefficient of the highest power of x is unity is called a monic polynomial. The coefficient of the highest power of x is called the leading coefficient of the polynomial. Thus, x3 2x2 + 5x 5 is a monic polynomial but 2x3 2x2 + 5x 5 is not a monic polynomial.

11.13.2 Minimal Polynomial The monic polynomial of lowest degree that annihilates a matrix A is called the minimal polynomial of A. If f (x) is the minimal polynomial of A, then f (x) = O is called the minimal equation of the matrix A. If A is of order n, then its characteristic polynomial is of degree n. Since the characteristic polynomial of A always annihilates A, the minimal polynomial of A cannot be of degree greater than n. Hence degree of minimal polynomial must be less than or equal to order of A. Results: (1) The minimal polynomial of a matrix is unique. (2) The minimal polynomial of a matrix is a divisor of every polynomial that annihilates this matrix. (3) Every root of the minimal equation of a matrix is also a characteristic root or eigen value of the matrix. (4) Every root of the characteristic equation of a matrix is also the root of the minimal equation of the matrix.

11.13.3 Derogatory and Non-Derogatory Matrices An n-rowed matrix is said to be derogatory if the degree of its minimal equation is less than n, the order of the matrix and is said to be non-derogatory if the degree of its minimal equation is equal to n, the order of the matrix.

Matrices

11.121

If the roots of the characteristic equation of a matrix are all distinct, then the matrix is non-derogatory. Example 1: Show the following matrices are non-derogatory. ⎡ 1 0 -1⎤ (i) ⎢ 1 2 1⎥ ⎢ ⎥ ⎣ 2 2 3⎦

⎡ -2 5 4⎤ (ii) ⎢ 5 7 5⎥ ⎢ ⎥ ⎣ 4 5 –2 ⎦

⎡ 2 -1 1⎤ (iii) ⎢ 2 2 -1⎥ . ⎢ ⎥ ⎣ 1 2 -1⎦

⎡ 1 0 –1⎤ A = ⎢ 1 2 1⎥ ⎢ ⎥ ⎣ 2 2 3⎦

Solution: (i) The characteristic equation is |A

lI| = 0

1– l 0 –1 1 2–l 1 =0 2 2 3–l l3 6l2 + 11l 6 = 0 l = 1, 2, 3 Since all eigen values are distinct, matrix A is non-derogatory.

⎡ –2 5 4 ⎤ A=⎢ 5 7 5⎥ ⎢ ⎥ ⎣ 4 5 –2 ⎦

(ii) The characteristic equation is |A

lI| = 0

–2 – l 5 4 5 7–l 5 =0 4 5 –2 – l l3 3l2 90l 216 = 0 l = 3, 6, 12 Since all the eigen values are distinct, matrix A is non-derogatory.

⎡ 2 –1 1⎤ A = ⎢ 2 2 –1⎥ ⎢ ⎥ ⎣ 1 2 –1⎦

(iii) The characteristic equation is |A 2–l 2 1

l3

–1 2–l 2

lI| = 0

1 –1 = 0 –1 – l

3l2 + 3l 1 = 0 (l 1)3 = 0 l = 1, 1, 1

Engineering Mathematics

11.122

Let f1(x) = x

1 f1(A) = A

Let f2(x) = (x

⎡ 1 –1 1⎤ I = ⎢ 2 1 –1⎥ ≠ O ⎢ ⎥ ⎣ 1 2 –2 ⎦

1)2

f2(A) = (A

⎡ 1 –1 1⎤ ⎡ 1 –1 1⎤ ⎡0 0 0 ⎤ I)2 = ⎢ 2 1 –1⎥ ⎢ 2 1 –1⎥ = ⎢ 3 –3 3⎥ ≠ O ⎢ ⎥⎢ ⎥ ⎢ ⎥ ⎣ 1 2 –2 ⎦ ⎣ 1 2 –2 ⎦ ⎣ 3 –3 3⎦

Now, f3(x) = (x 1)3 By Cayley–Hamilton theorem, every square matrix satisfies its characteristic equation. Hence, f3(x) = (x 1)3 = 0. Since f3(x) is the minimal polynomial of A with degree 3, which is equal to the order of A, matrix A is non derogatory. Example 2: Show that the following matrices are derogatory and find its minimal polynomial. ⎡ 7 4 –1⎤ (ii) ⎢ 4 7 –1⎥ . ⎢ ⎥ ⎣ –4 –4 4 ⎦

⎡1 0 0 ⎤ (i) ⎢1 –1 0⎥ ⎢ ⎥ ⎣1 0 –1⎦

⎡1 0 0 ⎤ A = ⎢1 –1 0 ⎥ ⎢ ⎥ ⎣1 0 –1⎦

Solution: (i) The characteristic equation is |A 1– l 1 1

0 –1 – l 0

l3 + l2 Let

lI| = 0

0 0 =0 –1 – l

l 1=0 l = 1, 1, 1

f (x) = (x 1) (x + 1) = x2 1 f (A) = A2 I ⎡1 0 0 ⎤ ⎡1 A = ⎢1 –1 0 ⎥ ⎢1 ⎢ ⎥⎢ ⎣1 0 –1⎦ ⎣1 ⎡1 0 f (A) = A2 I = ⎢0 1 ⎢ ⎣0 0 2

f (x) = x2

0 0 ⎤ ⎡1 0 0 ⎤ –1 0 ⎥ = ⎢ 0 1 0 ⎥ ⎥ ⎢ ⎥ 0 –1⎦ ⎣0 0 1 ⎦ 0 ⎤ ⎡1 0 0 ⎤ ⎡0 0 0⎤ 0 ⎥ − ⎢0 1 0 ⎥ = ⎢0 0 0⎥ = O ⎥ ⎢ ⎥ ⎢ ⎥ 1 ⎦ ⎣0 0 1 ⎦ ⎣0 0 0⎦ 1 annihilates A.

Matrices

11.123

Hence, f (x) is the minimal polynomial of A with degree 2. Since its degree is less than the order of A, matrix A is derogatory. (ii)

7 4 –1 4 7 –1 –4 –4 4

A=

The characteristic equation is |A 7– 4 –4

4 7– –4

lI| = 0

–1 –1 = 0 4–

18l2 + 81l 108 = 0 l = 3, 3, 12 f (x) = (x 3) (x 12) = x2 15x + 36 f (x) = A2 15A + 36I 7 4 –1 7 4 –1 69 60 –15 A2 = 4 7 –1 4 7 –1 = 60 69 –15 l3

Let

–4 –4

4

–4 –4

4

–60 –60

24

f (A) = A

15A + 36I ⎡ 69 60 –15⎤ ⎡ 105 60 –15⎤ ⎡36 0 0 ⎤ ⎡0 0 0 ⎤ = ⎢⎢ 60 69 –15⎥⎥ – ⎢⎢ 60 105 –15⎥⎥ + ⎢⎢ 0 36 0 ⎥⎥ = ⎢⎢0 0 0 ⎥⎥ = O ⎢⎣ –60 –60 24 ⎥⎦ ⎢⎣ –60 –60 60 ⎥⎦ ⎢⎣ 0 0 36 ⎥⎦ ⎢⎣0 0 0 ⎥⎦ f (x) = x2 15x + 36 annihilates A. Hence, f (x) is the minimal polynomial of A with degree 2. Since its degree is less than the order of A, matrix A is derogatory. 2

Exercise 11.12 1. Show that the following matrices are non-derogatory. 1 2 3 2 –2 3 (i) 2 3 4 (ii) 1 1 1 3 4 5 1 3 –1 (iii)

1 1 –2 –1 2 1 0 1 –1

(iv)

1 2 2 0 2 1 –1 2 2

2. Show that the following matrices are derogatory and find the minimal polynomial in each case.

(i)

6 –2 2 –2 3 –1 2

(iii)

–1

2 –3 3 0 3 –1 0 –1 3

(ii)

1 0 2 0 1 0 0 0 1

3 (iv)

1 –6 –4 0 4 2 0 –6 –3

⎡ Ans. : (i) f ( x ) = x 2 − 10 x + 16 ⎤ ⎢ ⎥ (ii) f ( x ) = x − 1 ⎢ ⎥ ⎢ (iii) f ( x ) = x 2 − 6 x + 8 ⎥ ⎢ ⎥ (iv ) f ( x ) = x 2 − x ⎢⎣ ⎥⎦

Engineering Mathematics

11.124

11.14 FUNCTION OF SQUARE MATRIX Let A be n rowed square matrix. By Cayley–Hamilton theorem, every function of A can be expressed as a polynomial of degree n 1 in A. f (A) = an 1 An 1 + an 2 An 2 + ……………………..+ a1A + a0I where an 1, an 2, ………. a1, a0 are constants to be determined. Example 1: If A =

2 1 , find A50. 1 2

Solution:

A=

2 1 1 2

The characteristic equation is A– I = 0 2– 1 l2

1 2–

=0

4l + 3 = 0 l = 1, 3

Let f (A) = A50 = a1A + a0I Assuming that Eq. (1) is satisfied by the characteristic roots of A, l50 = a1l + a0 Putting l = 1, 1 = a1 + a0 Putting l = 3, 350 = 3a1 + a0 Solving Eqs. (2) and (3), 1 a1 = ( –1 + 350 ) 2 1( a0 = 3 – 350 ) 2 1 1 A50 = ( –1 + 350 ) A + ( 3 – 350 ) I 2 2 50 –1 + 350 ⎤ 1 ⎡3 − 350 0 ⎤ 1 ⎡ 2 ( –1 + 3 ) = ⎢ ⎥+ ⎢ ⎥ 50 50 2 ⎢⎣ –1 + 3 3 – 350 ⎦ 2 ( –1 + 3 )⎥⎦ 2 ⎣ 0 =

1 1 + 350 2 –1 + 350

Example 2: If A = Solution:

–1 + 350 1 + 350

0 1 , find eAt. –1 0 A=

0 1 –1 0

… (1)

… (2) … (3)

Matrices

11.125

The characteristic equation is A– I = 0 – –1

1 –

=0

l2 + 1 = 0 l=±i

Let f (A) = eAt = a1A + a0I Assuming that Eq. (1) is satisfied by the characteristic roots of A. elt = a1l + a0 Putting l = i, eit = a1i + a0 Putting l = i, e it = a1i + a0 Solving Eqs. (2) and (3), a1 = sin t a0 = cos t ⎡1 0⎤ ⎡ 0 1⎤ + cos t ⎢ eAt = sin t ⎢ ⎥ ⎥ ⎣0 1⎦ ⎣ –1 0 ⎦ = ⎡o Example 3: If A = ⎢ 2 ⎢ ⎣o

3o 2 o

… (1)

… (2) … (3)

cos t sin t – sin t cos t

⎤ ⎥ , find cos A. ⎥ ⎦ ⎡p A=⎢2 ⎢ ⎣p

Solution:

3p 2 p

⎤ ⎥ ⎥ ⎦

The characteristic equation is A– I = 0 p 3p –l 2 2 =0 p p –l

l2

3p 2

p2=0 l = 2p , –p 2

Let f (A) = cos A = a1A + a0I Assuming that Eq. (1) is satisfied by the characteristic roots of A, cos l = a1l + a0

… (1)

Engineering Mathematics

11.126

l = 2p , cos 2p = a1. 2p + a0 1 = 2p a1 + a0

Putting

λ=–

Putting

… (2)

π , 2

⎛ ⎞ ⎛ ⎞ cos ⎜ – ⎟ = a1 ⎜ – ⎟ + a0 ⎝ 2⎠ ⎝ 2⎠ 0= – Solving Eqs. (2) and (3),

2

a1 + a0

… (3)

2 5p 1 a0 = 5 a1 =

cos A =

= Example 4: If A =

⎡ p 3p ⎤ 1 ⎡1 0 ⎤ 2 ⎢ 2 2 ⎥+ ⎢ ⎥ 5 ⎣0 1 ⎥⎦ 5p ⎢ ⎣p p ⎦ 1 2 3

5 2 3

3 –2 , find 3A57 + 2A18.. 4 –3

Solution:

A=

3 –2 4 –3

The characteristic equation is A– I = 0 3– 4

–2 –3 –

=0

l2

1=0 l = ±1

Let f (A) = 3A57 + 2A18 Assuming that Eq. (1) is satisfied by the characteristic roots of A, 3l57 + 2l18 = a1l + a0 Putting l = 1, 3 + 2 = a1 + a0 5 = a1 + a0 Putting l = 1, 3 + 2 = a1 + a0 1 = a1 + a0

… (1)

… (2)

… (3)

Matrices

11.127

Solving Eqs. (2) and (3), a0 = 2 a1 = 3 ⎡3 3 A57 + 2 A18 = 3 ⎢ ⎣4 ⎡11 =⎢ ⎣12

⎡3 ⎢2 Example 5: If A = ⎢ ⎢1 ⎢⎣ 2

1⎤ 2⎥ ⎥ , find (i) eA 3⎥ 2 ⎥⎦

(ii) 4A.

⎡3 ⎢2 A=⎢ ⎢1 ⎢⎣ 2

Solution:

–2 ⎤ ⎡1 0 ⎤ + 2⎢ ⎥ ⎥ –3 ⎦ ⎣0 1 ⎦ –6 ⎤ –7 ⎥⎦

1⎤ 2⎥ ⎥ 3⎥ 2 ⎥⎦

The characteristic equation is A– I = 0 3 1 −l 2 2 = 0 1 3 −l 2 2 2 l 3l + 2 = 0 l = 1, 2

(i) Let f (A) = eA = a1A + a0I Assuming that Eq. (1) is satisfied by characteristic roots of A, el = a1l + a0 Putting l = 1, e = a1 + a0 Putting l = 2, e2 = 2a1 + a0 Solving Eqs. (2) and (3), a1 = e2 e a0 = e2 + 2e ⎡3 1⎤ ⎥ ⎢ ⎡1 0 ⎤ e A = (e 2 – e) ⎢ 2 2 ⎥ + ( – e 2 + 2e) ⎢ ⎥ ⎣0 1 ⎦ ⎢1 3⎥ ⎢⎣ 2 2 ⎥⎦ 1 2 ⎤ ⎡3 2 ⎢ 2 ( e – e ) 2 ( e – e ) ⎥ ⎡ – e 2 + 2e 0 ⎤ =⎢ ⎥+⎢ ⎥ 2 1 3 + 2e ⎦ 0 – e ⎢ (e 2 – e) (e 2 – e) ⎥ ⎣ ⎥⎦ ⎢⎣ 2 2

… (1)

… (2) … (3)

Engineering Mathematics

11.128

e2 e + 2 2 = 2 e e – 2 2

e2 e – 2 2 e2 e + 2 2

=

1 e2 + e e2 – e 2 e2 – e e2 + e

(ii) Let f (A) = 4A Replacing e by 4 in eA, 10 6 1 20 12 4A = = 6 10 2 12 20 Example 6: If A =

–1 4 , prove that 3 tan A = A tan3. 2 1 A=

Solution:

–1 4 2 1

The characteristic equation is A– I = 0 −1 − l 4 =0 2 1− l

l2

9=0 l = 3, 3

Let f (A) = tan A = a1A + a0 Assuming that Eq. (1) is satisfied by the characteristic roots of A, tanl = a1l + a0 Putting l = 3, tan 3 = 3a1 + a0 Putting l = 3, tan( 3) = 3a1 + a0 tan 3 = 3a1 + a0 Solving Eqs. (2) and (3), a0 = 0 1 a1 = tan 3 3 1 tan A = tan 3. A 3 3 tan A = A tan 3 Example 7: If A = Solution:

7 3 , find An. 2 6 A=

7 3 2 6

… (1)

… (2)

… (3)

Matrices

11.129

The characteristic equation is A– I = 0 7−l 3 =0 2 6−l l2 13l + 36 = 0 l = 4, 9

… (1) Let f (A) = An = a1A + a0 I Assuming that Eq. (1) is satisfied by the characteristic roots of A, ln = a1l + a0 Putting l = 4, 4n = 4a1 + a0 ... (2) Putting l = 9, ... (3) 9n = 9a1 + a0 Solving Eqs. (2) and (3), 9n – 4n a1 = 5 9.4n – 4.9n a0 = 5 n 9 – 4n 7 3 9.4n – 4.9n 1 0 An = + 2 6 0 1 5 5 1 0 0 Example 8: If A = 1 0 1 , show that An = An-2 + A2 - I for every integer 0 1 0 50

n > 3. Hence, find A . Solution:

1 0 0 A= 1 0 1 0 1 0

The characteristic equation is A– I = 0 1– 0 0 1 – 1 =0 0 1 – (1 l) (l2 1) = 0 l = 1, 1, 1 Let f (A) = An An 2 = a2A2 + a1A + a0I Assuming that the Eq. (1) is satisfied by the characteristic roots of A, ln ln 2 = a2l2 + a1l + a0 Putting l = 1, ( 1)n (–1)n 2 = a2 a1 + a0

… (1) … (2) … (3)

Engineering Mathematics

11.130

Putting (1)n

l = 1, (1)n 2 = a2 + a1 + a0

… (4)

Differentiating Eq. (2), nln 1 (n 2)ln 3 = 2a2l + a1 Putting l = 1, n(1)n 1 (n 2)(1)n 3 = 2a2 + a1 2 = 2a2 + a1 From Eqs. (3), (4) and (5) a2 = 1 a1 = 0 a0 = 1 n n 2 A A = A2 I An = An–2 + A2 – I Putting n = 50, 48, 46 ... 4, A50 = A48 + A2 I A48 = A46 + A2 I A46 = A44 + A2 I A4 = A2 + A2

… (5)

I

Adding all the equations, A50 = A2 + 24(A2 I) = 25A2 24I 1 0 0 1 0 0 1 0 0 = 25 1 0 1 1 0 1 – 24 0 1 0 0 1 0

0 1 0

0 0 1

1 0 0 = 25 1 0 25 0 1

Exercise 11.13 1. If A =

2 3 , find A100. –3 –4 Ans. :

2. If A = A50

–299 –300 300 301

1 4 , prove that 2 3

5A49 =

4 –4 . –2 2

3. If A =

5 3 , find An in terms of A 1 3

and A4. Ans. : An = + A4 =

6n – 2n 5 3 1 3 4 3.24 – 64 1 0 , 0 1 2 976 960 320 336

Matrices

⎡ ⎢p 4. If A = ⎢ ⎢0 ⎢⎣

p⎤ 4 ⎥ , find cos A. ⎥ p⎥ 2 ⎥⎦

1 0 0 7. If A = 2 2 0 , find eA. 4 5 –1

⎡ 1 ⎤⎤ ⎡ ⎢ Ans. : ⎢ –1 − 2 ⎥ ⎥ ⎢ ⎥⎥ ⎢ 0 ⎦ ⎥⎦ ⎢⎣ ⎣ 0 y y

5. If A = A

e =e

y

y , prove that y

3p 2 p

⎡ Ans. : ⎤ ⎢ ⎥ ⎤⎥ ⎢⎡ e 0 0 ⎥⎥ ⎢⎢ ⎥ 2 2 ⎢⎢ e 2(e – e) 0 ⎥⎥ ⎢ ⎢ ⎥ ⎥ 5 2 ⎢⎢1 2 –1 –1 –1 ⎥ e e e e e e ( – – ) ( – ) 10 9 ⎥⎥ ⎢ ⎢⎣ 3 3 ⎦⎦ ⎣

8. Find tan 1A, if A =

cosh y sinh y sinh y cosh y

⎡p 6. If A = ⎢ 2 ⎢ ⎣p



3 5 2 5

2 4 . 1 2

⎡ 4⎤ ⎤ 1 –1 ⎡ 2 ⎢ Ans. : tan 4 ⎢ ⎥⎥ 4 ⎣1 2 ⎦ ⎦ ⎣

⎤ ⎥ , find sin A. ⎥ ⎦

Ans. :

11.131

3 5 2 – 5

11.15 SIMILARITY OF MATRICES If A and B are two square matrices of order n, then B is said to be similar to A, if there exists a non-singular matrix P such that B = P 1 AP Note: (1) Similarity of matrices is an equivalence relation. (2) Similar matrices have the same determinant. (3) Similar matrices have the same characteristic polynomial and hence, the same eigen values. If X is an eigen vector of A corresponding to the eigen value l, then P 1X is an eigen vector of B corresponding to the eigen value l where B = P 1 AP. (4) If A is similar to a diagonal matrix D, the diagonal elements of D are the eigen values of A. (5) Two n n matrices with the same set of n distinct eigen values are similar.

11.15.1 Diagonalisation A matrix A is said to be diagonalisable if it is similar to a diagonal matrix. A matrix A is diagonalisable if there exists an invertible matrix P such that P 1AP = D where D is a diagonal matrix, also known as spectral matrix. The matrix P is then said to diagonalise A or transform A to diagonal form and is known as modal matrix.

Engineering Mathematics

11.132

Note: (1) The necessary and sufficient condition for a square matrix to be similar to a diagonal matrix is that the algebraic multiplicity of each of its eigen values is equal to the geometric multiplicity. (2) For a distinct eigen value, algebraic multiplicity is always equal to its geometric multiplicity. (3) If the eigen values of an n n matrix are all distinct, then it is always similar to a diagonal matrix. (4) An n n matrix is diagonalisable if and only if it possesses n linearly independent solutions.

11.15.2 Orthogonally Similar Matrices If A and B are two square matrices of order n, then B is said to be orthogonally similar to A, if there exists an orthogonal matrix P such that B = P 1 AP 1 T Since P is orthogonal, P = P B = P 1AP = PTAP Results: (1) Every real symmetric matrix is orthogonally similar to a diagonal matrix with real elements. (2) A real symmetric matrix of order n has n mutually orthogonal real eigen vectors. (3) Any two eigen vectors corresponding to two distinct eigen values of a real symmetric matrix are orthogonal. Note: To find orthogonal matrix P, each element of the eigen vector is divided by its norm (length).

11.15.3 Unitarily Similar Matrices If A and B are two square matrices of order n, then B is said to be unitarily similar to A, if there exists a unitary matrix P such that B = P 1AP –1 q Since P is orthogonal, P = P B = P –1 AP = Pq AP Results: (1) Every Hermitian matrix is unitarily similar to a diagonal matrix. (2) A Hermitian matrix of order n has n mutually orthogonal eigen vectors in the complex vector space. (3) Any two eigen vectors corresponding to two distinct eigen values of a Hermitian matrix are orthogonal. Example 1: Show that the following matrices are not diagonalisable. (i)

1 2 2 0 2 1 -1 2 2

(ii)

1 -2 0 1 2 2 . 1 2 3

Matrices

11.133

1 2 2 A= 0 2 1 –1 2 2

Solution: (i) The characteristic equation is

A– I = 0 1–

2 2– 2

0 –1

2 1 2–

=0

5l2 + 8l

4=0 l = 1, 2, 2 (a) For l = 1, algebraic multiplicity = geometric multiplicity = 1 (b) For l = 2, [A l2I] X = O l3

–1 2 2 x 0 0 0 1 y = 0 –1 2 0 R3

z

0

R1

–1 2 2 x 0 0 0 1 y = 0 0 0 2 R3

z

0

2R2

–1 2 2 x 0 0 0 1 y = 0 0 0 0 z 0 Rank of matrix = 2. Number of unknowns = 3. Number of linearly independent solutions = 3 2 = 1. Hence, geometric multiplicity is 1. Since the eigen value 2 is repeated twice, its algebraic multiplicity is 2. Thus, algebraic multiplicity geometric multiplicity, Hence, matrix A is not diagonalisable. (ii)

1 –2 0 A= 1 2 2 1 2 3

The characteristic equation is A– I = 0

Engineering Mathematics

11.134

1– 1 1

–2 2– 2

0 2 3–

=0

6l2 + 9l

4=0 l = 1, 1, 4 (a) For l = 4, algebraic multiplicity = geometric multiplicity = 1. (b) For l = 1, [A l I] X = O l3

0 –2 0 1 1 2 1 2 2

x 0 y = 0 z 0

R13 1 1

2 2 1 2

0 –2 0 R2

z

0

R1

1 2 2 0 –1 0 0 –2 0 R3

x 0 y = 0

x 0 y = 0 z 0

2R2

1 2 2 0 –1 0

x 0 y = 0

0

z

0 0

0

Rank of matrix = 2. Number of unknowns = 3. Number of linearly independent solutions = 3 2 = 1. Hence, geometric multiplicity is 1. Since the eigen value 1 is repeated twice, its algebraic multiplicity is 2. Thus, algebraic multiplicity geometric multiplicity. Hence, matrix A is not diagonalisable. Example 2: Show that the following matrices are similar to diagonal matrices. Find the diagonal and modal matrix in each case.

(i)

–9 4 4 –8 3 4 –16 8 7

⎡ 4 2 −2 ⎤ ⎢ ⎥ (ii) ⎢ –5 3 2⎥ ⎢⎣ –2 4 1⎥⎦

(iii)

1 –6 –4 0 4 2 . 0 –6 –3

Matrices

11.135

–9 4 4 A = –8 3 4 –16 8 7

Solution: (i) The characteristic equation is

A– I = 0

–9 – –8

4 3–

4 4

–16

8

7–

=0

3=0 l = 1, 1, 3 (a) For l = 1, algebraic multiplicity = 2 [A l1I ] X = O l3

l2

5l

–8 4 4 –8 4 4 –16 8 8 R2

2R1, R3 –8 4 4 0 0 0 0 0 0

x 0 y = 0 z

0

2R1 x 0 y = 0 z 0

Rank of matrix = 1. Number of unknowns = 3. Number of linearly independent solutions = 3 Geometric multiplicity = 2. 8x + 4y + 4z = 0 Taking y = 1 and z = 1, x=1 Taking y = 1 and z = 1, x = 0 1 X1 = 1 ,

X2 =

1

1 = 2.

0 1 –1

(b) For l = 3, algebraic multiplicity = geometric multiplicity = 1 [A l2I] X = O –12 4 4 –8 0 4 –16 8 4

x 0 y = 0 z 0

12x + 4y + 4z = 0 8x + 4z = 0 16x + 8y + 4z = 0

Engineering Mathematics

11.136

x 4 4 0 4

y z = –12 4 –12 4 –8 4 –8 0

=–

x y z = = 16 16 32 x y z = = 1 1 2 1 X3 = 1 2 Since algebraic multiplicity for each eigen value is equal to the geometric multiplicity, the matrix A is diagonalisable. –1 0 0 Diagonal matrix D = 0 –1 0 0 1 Modal matrix P = 1

0 3 0 1 1 1

1 –1 2 ⎡ 4 2 −2⎤ A = ⎢⎢ –5 3 2⎥⎥ ⎢⎣ –2 4 1⎥⎦

(ii) The characteristic equation is

A– I = 0 4– –5

2 3–

–2

4

–2 2 =0 1–

10 = 0 l = 1, 2, 5 Since all the eigen values are distinct, A is diagonalisable. (a) For l = 1, [A l1I] X = O l

3

8l + 17l 2

3 2 –2 –5 2 2 –2 4 0

x 0 y = 0 z 0

3x + 2y 2z = 0 5x + 2y + 2z = 0 2x + 4y = 0

Matrices

x 2 –2 2 2

=–

11.137

y 3 –2 –5 2

=

z 3 2 –5 2

x y z = = 8 4 16 x y z = = 2 1 4 2 X1 = 1 4 (b) For l = 2, [A

l2I] X = O 2 2 –2 –5 1 2 –2 4

x 0 y = 0 –1 z 0

2x + 2y 2z = 0 5x + y + 2z = 0 2x + 4y z = 0 x y =– = 2 –2 2 –2 1

2

–5

2

z 2 2 –5

1

x y z = = 6 6 12 x y z = = 1 1 2 1 X2 = 1 2 (c) For l = 5, [A

l3I] X = O –1 2 –2 –5 –2 2 –2 4 –4

x 0 y = 0 z 0

x + 2y 2z = 0 5x 2y + 2z = 0 x y z =– = 2 –2 –1 –2 –1 2 –2 2 –5 2 –5 –2

Engineering Mathematics

11.138

x y z = = 0 12 12 x y z = = 0 1 1 0 X3 = 1 1 1 0 0 Diagonal matrix D = 0 2 0 0 0 5 2 1 0 Modal matrix P = 1 1 1 4 2 1 1 –6 –4 A= 0 4 2

(iii)

0 –6

–3

The characteristic equation is |A

lI | = 0

1– l 0

–6 4–l

–4 2

0

–6

–3 – l

=0

2l2 + l = 0 l = 0, 1, 1 (a) For l = 0, algebraic multiplicity = geometric multiplicity = 1. [A l1I] X = O l3

1 –6 –4 0 4 2

x 0 y = 0

0 –6 –3

z

x

0

6y 4z = 0 4y + 2z = 0 6y 3z = 0 x y z =– = –6 –4 1 –4 1 –6 4 2 0 2 0 4

Engineering Mathematics

11.140

1 2⎤ ⎡ 2 ⎢ −1 –2 2⎥ Modal matrix P = ⎢ ⎥ ⎢⎣ 2 3 −3⎥⎦ Example 3: Determine diagonal matrices orthogonally similar to the following real symmetric matrices. Also, find modal matrices. (i)

3 –1 1 –1 5 –1 1 –1 3

8 –6 2 –6 7 –4 2 –4 3

(ii)

6 –2 2 –2 3 –1 2 –1 3

3 –1 1 A = –1 5 –1 1 –1 3

Solution: (i) The characteristic equation is

lI| = 0

|A 3– –1

–1 5–

1

–1

l3

(iii)

1 –1 = 0 3–

11l 2 + 36l 36 = 0 l = 2, 3, 6

(a) For l = 2, [A

l 1I] X = O 1 –1 1 x 0 –1 3 –1 y = 0 1 –1 R3

1

z

0

R1

1 –1 1 x 0 –1 3 –1 y = 0 0

0

0

z

0

x y+z=0 x + 3y z = 0 x y z =– = –1 1 1 1 1 –1 3 –1 –1 –1 –1 3 x y z = = –2 0 2

Matrices

11.139

x y z = = 4 –2 4 x y z = = 2 –1 2 2 X 1 = –1 2 (b) For l = 1, algebraic multiplicity = 1 [A l2I] X = O 0 –6 –4 0 3 2 0 –6 –4

x 0 y = 0 z 0

1 1 R1 , R3 2 2 0 3 2 0 3 2

x 0 y = 0

0 3 2

z

R2

R1, R3

0

R1

0 3 2 0 0 0

x 0 y = 0

0 0 0

z

0

Rank of coefficient matrix = 1. Number of unknowns = 3. Number of linearly independent solutions = 3 Geometric multiplicity = 2. 0x + 3y + 2z = 0 Taking x = 1 and y = 2, z = 3 Taking x = 2 and y = 2, z = –3 ⎡ 2⎤ 1 X 3 = ⎢⎢ 2⎥⎥ X 2 = –2 , ⎢⎣ −3⎥⎦ 3

1 = 2.

Since algebraic multiplicity for each eigen value is equal to the geometric multiplicity, the matrix A is diagonalisable. 0 0 0 Diagonal matrix D = 0 1 0 0 0 1

Matrices

11.141

x y z = = –1 0 1 –1 X1 = 0 1 (b) For l = 3, [A

l 2I] X = O 0 –1 1 x 0 –1 2 –1 y = 0 1 –1 0 z 0 y+z=0 x + 2y z = 0 x y=0 x y z =– = –1 1 0 1 0 –1 2 –1

–1 –1

–1

2

x y z = = –1 –1 –1 x y z = = 1 1 1 1 X2 = 1 1 (c) For l = 6, [A

l 3I] X = O –3 –1 1 x 0 –1 –1 –1 y = 0 1 –1 –3

z

0

3x y + z = 0 –x y z = 0 x y 3z = 0 x y z =– = –1 1 –3 1 –3 –1 –1 –1 –1 –1 –1 –1 x y z = = 2 –4 2

Engineering Mathematics

11.142

x y z = = 1 –2 1 1 X 3 = –2 1 2 0 0 Diagonal matrix D = 0 3 0 0 0 6 Length of vector X1 =

(–1) 2 + (1) 2 = 2

Length of vector X2 =

12 + 12 + 12 = 3

Length of vector X3 =

12 + (–2) 2 + 12 = 6

The normalised eigen vectors are – X1 =

1

1 2 0 , 1

X2 =

2

3 1

,

3 1 3 –

Modal matrix P =

1 2 0 1 2

1 6 2 X3 = – 6 1 6 1

1

3 1

6 2

3 1 3

8 –6 2 A = –6 7 –4 2 –4 3

(ii) The characteristic equation is |A

8– –6 2

–6 7– –4

lI| = 0

2 –4 = 0 3–



6 1 6

Matrices l3 (a) For l = 0, [A

11.143

18l 2 + 45l = 0 l = 0, 3, 15

l 1I] = O 8 –6 2 –6 7 –4 2 –4 3

x 0 y = 0 z 0

8x 6y + 2z = 0 6x + 7y 4z = 0 2x 4y + 3z = 0 x y z =– = –6 2 8 2 8 –6 7 –4 –6 –4 –6 7 x y z = = 10 20 20 x y z = = 1 2 2 1 X1 = 2 2 (b) For l = 3, [A

l 2I] = O 5 –6 2 –6 4 –4

x 0 y = 0

2 –4 0 z 0 5x 6y + 2z = 0 6x + 4y 4z = 0 2x 4y = 0 x y z =– = –6 2 5 2 5 –6 4 –4

–6 –4

–6

4

x y z = = 16 8 –16 x y z = = 2 1 –2 2 X2 = 1 –2

11.144

(c) For l = 15, [A

Engineering Mathematics l 3I] X = O –7 –6 2 x 0 –6 –8 –4 y = 0 2 –4 –12 z 0 7x 6y + 2z = 0 6x 8y 4z = 0 2x 4y 12z = 0 x y z =– = –6 2 –7 2 –7 –6 –8 –4 –6 –4 –6 –8 x y z = = 40 –40 20 x y z = = 2 –2 1 2 X 3 = –2 1 0 0 0 Diagonal matrix D = 0 3 0 0 0 15 Length of vector X1 =

12 + 22 + 22 = 3

Length of vector X2 =

22 + 12 + (–2) 2 = 3

Length of vector X3 = 22 + (–2) 2 + 12 = 3 The normalised eigen vectors are 1 2 2 3 3 3 1 2 2 X1 = , X2 = , X3 = – 3 3 3 2 1 2 – 3 3 3 1 3 2 Modal matrix P = 3 2 3

2 3 1 3 2 3

2 3 2 – 3 1 3

Matrices

11.145

6 –2 2 A = –2 3 –1 2 –1 3

(iii) The characteristic equation is

lI| = 0

|A

6– –2 2 l3 (a) For l = 2, [A

–2 3– –1

2 –1 = 0 3–

12l 2 + 36l

32 = 0 l = 2, 2, 8

l 1I] X = O 4 –2 2 x 0 –2 1 –1 y = 0 2 –1 1 z 0 R3 –

1 1 R2 , R2 + R1 2 2

4 –2 2 0 0 0 0

0 0

x 0 y = 0 z 0

Rank of coefficient matrix = 1 Number of unknowns = 3 Number of linearly independent solutions = 3 4x 2y + 2z = 0 Taking y = 1 and z = 1, x = 0 Taking y = 1 and z = 1, x = 1 0 1 X1 = 1 , X 2 = 1 1 (b) For l = 8, [A

–1

l 2I] X = O –2 –2 2 x 0 –2 –5 –1 y = 0 2 –1 –5 z 0 2x 2y + 2z = 0 2x 5y z = 0

1=2

Engineering Mathematics

11.146

2x

5z = 0

y

x y z =– = –2 2 –2 2 –2 –2 –5 –1 –2 –1 –2 –5 x y z = = 12 –6 6 x y z = = 2 –1 1 2 X 3 = –1 1 2 0 0 Diagonal matrix D = 0 2 0 0 0 8 Length of vector X1 =

02 + 12 + 12 = 2

Length of vector X2 =

12 + 12 + (–1) 2 = 3

Length of vector X3 = Normalised eigen vectors are

22 + (–1) 2 + 12 = 6

X1 =

0 1 2 1 2

,

1 3 1 X2 = , 3 1 – 3

1

2

1

3 1

6 1

2 1

3 1

0 Modal matrix P =

2 6 1 X3 = – 6 1 6

2



3

6 1 6

Example 4: For a symmetric matrix A, the eigen vectors are [1, 1, 1]T, [1, -2, 1]T corresponding to k 1 = 6 and k 2 = 12. If k 3 = 6, find the matrix A. Solution: Let X3 = (x3, y3, z3)T be the eigen vector corresponding to l3 = 6 X1 = (1, 1, 1)T,

X2 = (1, 2, 1)T

Matrices

11.147

Since A is real symmetric matrix, X1, X2 and X3 are orthogonal. i.e., X1TX3 = O and X2TX3 = O ⎡ x3 ⎤ ⎢ ⎥ [1 1 1] ⎢ y3 ⎥ = [0] and [1 2 1] ⎢⎣ z3 ⎥⎦ x3 + y3 + z3 = 0 x3 2y3 + z3 = 0 Solving these equations, x3 y =− 3 = 1 1 1 1 –2 1 1 1

⎡ x3 ⎤ ⎢ y ⎥ = [0 ] ⎢ 3⎥ ⎢⎣ z3 ⎥⎦

z3 1 1 1 –2

x3 y3 z3 3 0 –3 x3 y3 z3 = = 1 0 –1 1 0

X3 =

–1 Length of vector X1 =

12 + 12 + 12 = 3

Length of vector X2 =

12 + (–2) 2 + 12 = 6

Length of vector X3 =

12 + 02 + (–1) 2 = 2

Normalised eigen vectors are

X1 =

1

1

3 1

6 2

3 1

,

X2

3

1 ,

6 1

2 0 1 2

6 1

Modal matrix P =

X3 =

1

1

3 1

6 2

3 1

6

3

1 6

2 0 1 2

Engineering Mathematics

11.148

6 0 0 Diagonal matrix D = 0 12 0 = PT AP 0 0 6 D = P –1AP [∵ P is orthogonal, PT = P –1] PDP –1 = PP –1A PP –1 PDPT = IAI = A 1

A

1

1

3 1

6 2

3 1

6

3

1 6

1

2 0 1

6 0 0 0 12 0 0 0 6

1

1

3 1

3 2

3 1

6 1

6

6 1

2

2

0

2

7 –2 1 = –2 10 –2 1 –2 7 Example 5: Determine the diagonal matrices unitarily similar to the Hermitian matrices. Also, find modal matrices. (i)

⎡ 1 i⎤ ⎥ ⎣ – i 1⎦

2 1 – 2i 1 + 2i –2

(ii) ⎢

Solution:

A=

2 1 – 2i 1 + 2i –2

The characteristic equation is lI| = 0

|A 2– 1 + 2i

1 – 2i =0 –2 – l2

(a) For l = 3, [A

9=0 l = 3, 3

l 1I] X= O 5 1 – 2i 1 + 2i 1 5x + (1

x 0 = y 0 2i) y = 0 x = 1 2i y= 5

Matrices

X1 = (b) For l = 3, [A

11.149

1 – 2i –5

l 2I] X = O –1 1 – 2i 1 + 2i –5 x + (1

x 0 = y 0 2i) y = 0 x = 1 – 2i y = 1

X2 =

1 − 2i 1 –3 0 0 3

Diagonal matrix D = Length of X1 =

[12 + ( −2) 2 ] + ( −5) 2 = 30

Length of X2 =

[(1) 2 + ( −2) 2 ] + 1 = 6

Normalised eigen vectors are 1 – 2i X1 =

30 5

,

30

⎡1 − 2i ⎤ ⎢ 6 ⎥ ⎥ X2 = ⎢ ⎢ 1 ⎥ ⎢ ⎥ ⎣ 6 ⎦

⎡ 1 – 2i ⎢ 30 Modal matrix P = ⎢ ⎢ 5 ⎢− ⎣ 30 (ii)

A=

1 i –i 1

The characteristic equation is |A 1– –i

i 1– l2

lI| = 0 =0 2l = 0 l = 0, 2

1 − 2i ⎤ ⎥ 6 ⎥ 1 ⎥ ⎥ 6 ⎦

11.150

(a) For l = 0, [A

Engineering Mathematics l 1I] X= O 1 i –i 1

x 0 = y 0

x+iy=0 x=1 y= i X1 = (b) For l = 2, [A

1 i

l 2I] X = O –1 i – i –1

x 0 = y 0

x + iy = 0 x= 1 y= i ⎡ −1⎤ X2 = ⎢ ⎥ ⎣i ⎦ Diagonal matrix D =

0 0 0 2

Length of vector X1 =

(1) 2 + (1) 2 =

Length of vector X2 =

( −1) 2 + (1) 2 =

2 2

Normalised eigen vectors are 1 X1 =



2 , i

X2 =

Modal matrix P =

2 i 2

2 i 2

2 1

1



1 2 i 2

,

Matrices

11.151

Exercise 11.14 1. Show that the following matrices are not similar to diagonal matrices. 2 3 4 0 2 –1 0 0 1

(i)

2 –1 1 2 2 –1 1 2 –1

(ii)

1 2 3 3 10 5 (iv) 0 2 0 –2 –3 –4 0 0 2 3 5 7 2. Show that the following matrices are similar to diagonal matrices. Find the diagonal and modal matrix in each case.

3. Determine diagonal matrices orthogonally similar to the following real symmetric matrices. Also, find modal matrix in each case. 7 4 –4 4 –8 –1 (i) –4 –1 8

(iii)

(i)

–2 2

2 –3 1 –6

–1 –2

(iii)

(ii)

0

–9

1 –6 –4 0 4 2 0 –6

–17 18 –6 –18 19 –6

(iv)

–3

9

(ii)

7 0 –2 0 5 –2 –2 –2 6

9 0 Ans. : (i) D = 0 –9 0

2

8 –8 –2 4 –3 –2 3 –4

1 –

Ans. : 5 0 (i) D = 0 –3 0

0 0 ,P =

0 –3

–2 0 0 (ii) D =

1 –2 3 2 1 0 –1

0 1

1 0 0 (iii) D = 0 1 0 , 0 0 0 1 2 2 P = –2 –2 1 3 3 –2 1 0 0 4 3 2 (iv) D = 0 2 0 , P = 3 2 1 0 0 3 2 1 1

1 3 2 3 2 3

0

18 1

1

18 1

2 1

18

2

3 0 0 (ii) D = 0 6 0 , 0 0 9

2 1 –1

0 1 0 ,P = 2 1 0 0 1 1 0

0 –9 4

P

0 0 ,

0 3 P

1 3 2 3 2 3

2 3 2 3 1 3

2 3 1 3 2 3

4. Find the symmetric matrix A having eigen values l1 = 0, l2 = 3 and l3 = 15 with the corresponding eigen vectors X1 = [1, 2, 2]T, X2 = [ 2, 1, 2]T and X3. 8 –6 2 Ans. : –6 7 –4 2 –4 3

Engineering Mathematics

11.152

11.16 QUADRATIC FORM A homogeneous polynomial of second degree in n variables is called a quadratic form. n

An expression of the form

n

∑∑a

ij

i =1 j =1

xi xj where aij = aji are all real, is called a quadratic

form in n variables x1, x2 …… xn.

Matrix of a Quadratic Form The quadratic form corresponding to a symmetric matrix A can be written as n

Q = X TAX =

n

∑∑a

ij

i =1 j =1

a11 a21 A= ... an1

where

a12 a22 ... an 2

xi xj

… (1)

... a1n ... a2 n ... ... ... ann

Coefficient of xi xj in Eq. (1)

= aij + aji = 2aij = 2aji

Coefficient of xi2 in Eq. (1)

= aii

11.16.1 Linear Transformation Let Q = X TAX be a quadratic form and X = PY be a non-singular linear transformation. X = PY X T = (PY)T = Y TP T Q = X TAX = Y TP TAPY = Y TBY where B = P TAP The form Y BY is called linear transformation of the quadratic form X TAX under a non-singular transformation X = PY and P is called the matrix of the transformation. [... A is symmetric] Further, B T = (P TAP) T = P TA TP = P TAP T

=B Hence, matrix B is also symmetric.

11.16.2 Rank of Quadratic Form The rank of the coefficient matrix A is called the rank of the quadratic form X TAX. The number of non-zero eigen values of A also gives the rank of the quadratic form of A. If ( A) < n (order of A), i.e., |A| = 0, then the quadratic form is singular, otherwise it is non-singular.

Matrices

11.153

11.16.3 Canonical or Normal Form Let Q = XTAX be a quadratic form of rank r. An orthogonal transformation X = PY r

which diagonalise A, i.e., P TAP = D, transforms the quadratic form Q to

i

yi 2 (i.e.,

i =1

sum of r squares) or in matrix form Y TDY in new variables. This new quadratic form containing only the squares of yi is called the canonical form or sum of squares form of the given quadratic form. Index The number of positive terms in the canonical form is called the index of the quadratic form and is depoted by P. Signature The difference between the number of positive and negative terms in the canonical form is called the signature of the quadratic form. If index is P and total terms are r, then signature = P (r P) = 2p r. The signature of a quadratic form is invariant for all normal reductions.

11.16.4 Value Class or Nature of Quadratic Form Let Q = XTAX be the quadratic form in n variables x1, x2 …… xn. Let r be the rank and P be the number of positive terms in the canonical form of Q. Then we have the following criteria for the definiteness of value class of Q. Value Class

Criteria

Cannonical Form

Positive definite

r=P=n

2.

Positive semidefinite

r = P, P< n

3.

Negative definite

r = n, P = 0

4.

Negative semidefinite r < n, P = 0

5.

Indefinite

Otherwise

⎫ ⎪ ⎪ i =1 ⎬ only positive terms r 2 ⎪ yi ∑ ⎪⎭ i =1 n ⎫ – ∑ yi 2 ⎪ ⎪ i =1 ⎬ only negative terms r 2⎪ – ∑ yi ⎪⎭ i =1 both positive and negative terms n

∑y

1.

2

i

Criteria for the Value Class of a Quadratic form in Terms of the Nature of Eigen Values Value Class 1. 2. 3. 4. 5.

Positive definite Positive semidefinite Negative definite Negative semidefinite Indefinite

Nature of Eigen Values positive eigen values positive eigen values and at least one is zero negative eigen values negative eigen values and at least one is zero positive as well as negative eigen values

Engineering Mathematics

11.154

Criteria for the Value Class of a Quadratic form in Terms of Leading Principal Minors

For the matrix

a11 a21 A= ... an1

a12 a22 ... an 2

... a1n ... a2 n ... ... ... ann

The leading principal minors of matrix A are those determinants starting with a11 of orders 1, 2, … n. a11 a12 ... a1n a11 a12 a13 a a22 ... a2 n a11 a12 i.e., |a11|, , a21 a22 a23 , … 21 ... ... ... ... a21 a22 a31 a32 a33 an1 an 2 ... ann Value Class 1. 2. 3. 4. 5.

Positive definite Positive semidefinite Negative definite Negative semidefinite Indefinite

Nature of Leading Principal Minors positive leading principal minors positive leading principal minors and at least one is zero negative leading principal minors negative leading principal minors and at least one is zero positive as well as negative leading principal minors

11.16.5 Methods to Reduce Quadratic Form to Canonical Form (1) Orthogonal Transformation If Q = X TAX is a quadratic form, then there exists a real orthogonal transformation X = PY (where P is an orthogonal matrix) which transforms the given quadratic form X TAX to l1 y12 + l2 y22 + ………………+ lr yr2 where l1, l2 … lr are the r non-zero eigen values of matrix A.

(2) Congruent Transformation Congruent transformation consists of a pair of elementary transformations, one row and one similar column such that pre and post matrices are transpose of each other. If Q = XTAX is a quadratic form, then there exists a non-singular linear transformation X = PY which transforms the given quadratic form XTAX to a sum of square terms. b1 y12 + b2 y22 + … + br yr2 Example 1: Express the following quadratic forms in matrix notation: (i) x2 - 6xy + y2 (ii) 2x2 + 3y2 - 5z2 - 2xy + 6xz - 10yz (iii) x12 + 2x22 + 3x32 + x42 - 2x1x2 + 4x1x3 - 2x1x4 + 4x2x3 - 6x2x4 + 8x3x4 .

Matrices

Solution: (i)

X TAX = [ x

(ii)

X AX = [ x T

X TAX = [ x1

(iii)

11.155

x y

y]

1 –3 –3 1

y

2 –1 3 z ] –1 3 –5 3 –5 –5

x2

x4 ]

x3

x y z

1 –1 2 –1 x1 –1 2 2 –3 x2 2 2 3 4 x3 –1 –3 4 1 x4

Example 2: Write down the quadratic forms corresponding to the following matrices:

(i)

2 1

1 5 3 –2

5 –2

(ii)

4

0 1 2 3 1 2 3 4 2 3 4 5 3 4 5 6

Solution: (i) Q = 2x12 + 3x22 + 4x32 + 2x1x2 + 10x1x3

.

4x2x3

(ii) Q = 2x + 4x + 6x + 2x1x2 + 4x1x3 + 6x1x4 + 6x23 + 8x2x4 + 10x3x4 2 2

2 3

2 4

Example 3: Determine the nature (value class), index and signature of the following quadratic forms: (i) x12 + 5x22 + x32 + 2x2x3 + 6x3x1 + 2x1x2 (ii) 6x12 + 3x22 + 3x32 - 4x1x2 - 2x2x3 + 4x3x1 (iii) x12 + 4x22 + x32 - 4x1x2 + 2x3x1 - 4x2x3 (iv) -3x12 - 3x22 - 3x32 - 2x1x2 - 2x1x3 + 2x2x3. Solution: (i)

Q = x12 + 5x22 + x32 + 2x2x3 + 6x3x1 + 2x1x2 Q = XTAX = x1

x2

x3

1 1 3 A= 1 5 1 3 1 1 The characteristic equation is |A

lI | = 0

1 1 3 1 5 1 3 1 1

x1 x2 x3

Engineering Mathematics

11.156

1– l 1 3

1 3 5–l 1 =0 1 1– l l3 7l2 + 36 = 0 l = 2, 3, 6 Since there are positive as well as negative eigen values, value class of quadratic form is indefinite. Index = Number of positive eigen values = 2 Signature = Difference between the number of positive and negative eigen values =2 1=1 (ii) Q = 6x12 + 3x22 + 3x32 4x1x2 2x2x3 + 4x3x1

Q = X TAX = [ x1

x2

⎡ 6 –2 2 ⎤ ⎡ x1 ⎤ x 3 ] ⎢ –2 3 –1⎥ ⎢ x2 ⎥ ⎢ ⎥⎢ ⎥ ⎣ 2 –1 3⎦ ⎣ x3 ⎦

⎡ 6 –2 2 ⎤ A = ⎢ –2 3 –1⎥ ⎢ ⎥ ⎣ 2 –1 3⎦ The characteristic equation is |A lI | = 0 6–l –2 2 –2 3 – l –1 = 0 2 –1 3 – l l3 12l2 + 36l 32 = 0 l = 8, 2, 2 Since all the eigen values of A are positive, value class of quadratic form is positive definite. Index = Number of positive eigen values = 3 Signature = Difference between the number of positive and negative eigen values =3 0=3 (iii) Q = x12 + 4x22 + x32 4x1x2 + 2x3x1 4x2x3 Q = XTAX = [ x1

x2

1⎤ ⎡ x1 ⎤ ⎡ 1 –2 x 3 ] ⎢ –2 4 –2 ⎥ ⎢ x2 ⎥ ⎢ ⎥⎢ ⎥ 1⎦ ⎣ x3 ⎦ ⎣ 1 –2

The characteristic equation is |A 1– l –2 1

lI | = 0

–2 1 4 – l –2 = 0 –2 1 – l

l3

6l2 = 0 l = 0, 0, 6

Matrices

11.157

Since the eigen values of A are positive and two eigen values are zero, value class of quadratic form is positive semidefinite. Index = Number of positive eigen values = 1. Signature = Difference between the number of positive and negative eigen values =1 0=1 (iv) Q = 3x12 3x22 3x32 2x1x2 2x1x3 + 2x2x3 Q = X AX = [ x1 T

x2

x3 ]

–3 –1 –1 x1 –1 –3 1 x2 –1 1 –3 x3

–3 –1 –1 A = –1 –3 1 –1 1 –3 The characteristic equation is |A

lI | = 0

–3 – l –1

–1 –3 – l

–1 1

–1

1

–3 – l

=0

l3 + 9l2 + 24l + 16 = 0 l = 1, 4, 4 Since all the eigen values of A are negative, the quadratic form is negative definite. Index = Number of positive eigen values = 0 Signature = Difference between the number of positive and negative eigen values =0 3= 3 Example 4: Find the value of k so that the value class of quadratic form k (x12 + x22 + x32) + 2x1x2 - 2x2x3 + 2x3x1 is positive definite. Solution:

Q = k(x12 + x22 + x32) + 2x1x2 Q = X TAX = ⎡⎣ x1

x2

⎡k x 3 ⎤⎦ ⎢⎢1 ⎢⎣1

1 1⎤ ⎡k ⎢ A = ⎢ 1 k –1⎥⎥ ⎢⎣ 1 –1 k ⎥⎦ The characteristic equation is |A k –l 1 1

1 k –l –1

2x2x3 + 2x3x1

lI | = 0 1 –1 = 0 k –l

1 1 ⎤ ⎡ x1 ⎤ k –1⎥⎥ ⎢⎢ x2 ⎥⎥ –1 k ⎥⎦ ⎢⎣ x3 ⎥⎦

Engineering Mathematics

11.158

(k l) [(k l)2 1] 1 (k l + 1) + 1 [ 1 (k l)] = 0 (k l) (k l + 1) (k l 1) (k l + 1) (k l + 1) = 0 (k l + 1) [(k l) (k l 1) 2] = 0 (k l + 1) [(k l)2 (k l) 2] = 0 (k l + 1) (k l + 1) (k l 2) = 0 l = (k + 1), (k + 1), (k 2) For value class of quadratic form to be positive definite, all the eigen values should be greater than zero, i.e., k > 1 and k > 2 Hence, value class of quadratic form is positive definite if k > 2. Example 5: Reduce the following quadratic forms to canonical forms by orthogonal transformation. Also find the rank, index, signature and value class (nature) of the quadratic forms. (i) Q = 2x12 + 2x22 + 2x32 + 2x1x3 (ii) Q = 3x12 + 5x22 + 3x32 - 2x1x2 - 2x2x3 + 2x1x3 (iii) Q = 2x2 + 2y2 - z2 - 4yz + 4xz - 8xy. Solution: (i)

Q = 2x12 + 2x22 + 2x32 + 2x1x3 Q = XTAX = [ x1

x2

⎡2 0 1⎤ A = ⎢0 2 0⎥ ⎢ ⎥ ⎣1 0 2 ⎦

⎡ 2 0 1 ⎤ ⎡ x1 ⎤ x 3 ] ⎢ 0 2 0 ⎥ ⎢ x2 ⎥ ⎢ ⎥⎢ ⎥ ⎣1 0 2 ⎦ ⎣ x3 ⎦

The characteristic equation is |A 2–l 0 1

l3 (a) For l = 1, [A

0 2–l 0

lI | = 0

1 0 =0 2–l

6l2 + 11l

6=0 l = 1, 2, 3

l1I]X = O

Taking x1 = 1, x3 = 1

⎡1 0 1 ⎤ ⎡ x1 ⎤ ⎡ 0 ⎤ ⎢0 1 0⎥ ⎢ x ⎥ = ⎢0⎥ ⎢ ⎥⎢ 2⎥ ⎢ ⎥ 1 0 1 ⎣ ⎦ ⎣ x3 ⎦ ⎣ 0 ⎦ x1 + x3 = 0 x2 = 0 ⎡1⎤ X 1 = ⎢⎢ 0 ⎥⎥ ⎢⎣ –1⎥⎦

Matrices (b) For l = 2, [A

11.159

l2I] X = O ⎡0 0 1 ⎤ ⎡ x1 ⎤ ⎡0 ⎤ ⎢0 0 0 ⎥ ⎢ x ⎥ = ⎢0 ⎥ ⎢ ⎥⎢ 2⎥ ⎢ ⎥ ⎣1 0 0 ⎦ ⎣ x3 ⎦ ⎣0 ⎦ x3 = 0 x1 = 0

Taking x2 = 1

⎡0 ⎤ X 2 = ⎢1 ⎥ ⎢ ⎥ ⎣0 ⎦ (c) For l = 3, [A

l3I] X = O

⎡ –1 0 1⎤ ⎡ x1 ⎤ ⎡0 ⎤ ⎢ 0 –1 0 ⎥ ⎢ x ⎥ = ⎢ 0 ⎥ ⎢ ⎥⎢ 2⎥ ⎢ ⎥ ⎣ 0 0 –1⎦ ⎣ x3 ⎦ ⎣0 ⎦ x1+ x3 = 0 x2 = 0 Taking x1 = 1, x3 = 1 ⎡1 ⎤ X 3 = ⎢0 ⎥ ⎢ ⎥ ⎣1 ⎦ Length of vector X1 =

12 + 02 + (–1) 2 = 2

Length of vector X2 =

0 2 + 12 + 0 2 = 1

Length of vector X3 = Normalised eigen vectors are,

12 + 02 + 12 = 2

1 X1 =

2 0 , –1

1 0 X2 = 1 , 0

X3 =

1

2 ⎡ 1 ⎢ ⎢ 2 P= ⎢ 0 ⎢ ⎢− 1 ⎢⎣ 2 1 T P is orthogonal matrix, i.e., P = P

2 0 , 2

0 1 0

1 ⎤ ⎥ 2⎥ 0 ⎥ ⎥ 1 ⎥ 2 ⎥⎦

Engineering Mathematics

11.160

Let X = PY be the orthogonal transformation which transforms Q to canonical form. Q = Y T (PTAP)Y = Y TDY ⎡1 0 0 ⎤ ⎡ y1 ⎤ = [y1 y2 y3] ⎢⎢0 2 0 ⎥⎥ ⎢⎢ y2 ⎥⎥ ⎢⎣0 0 3⎥⎦ ⎢⎣ y3 ⎥⎦ = y12 + 2y22 + 3y32 Rank r = Number of non-zero terms in canonical form = 3 Index P = Number of positive terms in canonical form = 3 Signature = Difference between the number of positive and negative terms in canonical form = 3 0 = 3 Since only positive terms occur in the canonical form, value class of quadratic form is positive definite. (ii) Q = 3x12 + 5x22 + 3x32 2x1x2 2x2x3 + 2x1x3 Q = X T AX = [ x1

x2

x3 ]

3 –1 1 x1 –1 5 –1 x2 1 –1

3 –1 1 A = –1 5 –1 1 –1

3

The characteristic equation is lI | = 0

|A 3–l –1

–1 5–l

1

l

3

(a) For l = 2, [A

–1

1 –1 = 0 3–l

11l + 36l 2

36 = 0 l = 2, 3, 6

l1I] X = O 1 –1 1 x1 0 –1 3 –1 x2 = 0 1 –1 1 x3 0 x1 x2 + x3 = 0 x1 + 3x2 x3 = 0 x3 x1 x2 =– = –1 1 1 1 1 –1 3 –1 –1 –1 –1 3 x1 x2 x3 = = –2 0 2

3

x3

Matrices

x1 x2 x3 = = –1 0 1 –1 X1 = 0 1 (b) For l = 3, [A

l2I] X = O 0 –1 1 x1 0 = –1 2 –1 x2 0 0 0 0 x3 0 x2 + x3 = 0 x1 + 2x2 x3 = 0 x3 x1 x2 =– = –1 1 0 1 0 –1 2 –1 –1 –1 –1 2 x1 x2 x3 = = –1 −1 –1 1 X2 = 1 1

(c) For l = 6, [A

l3I] X = O ⎡ −3 –1 1⎤ ⎡ x1 ⎤ ⎡0 ⎤ ⎢ –1 –1 –1⎥ ⎢ x ⎥ = ⎢0 ⎥ ⎢ ⎥⎢ 2⎥ ⎢ ⎥ ⎢⎣ 1 –1 –3⎥⎦ ⎢⎣ x3 ⎥⎦ ⎢⎣0 ⎥⎦ 3x1 x2 + x3 = 0 x1 x2 x3 = 0 x1 x2 3x3 = 0 x1 –1

1

–1 –1

=–

x2 –3

1

=

–1 –1 x1 x =– 2 = 2 4 x1 x2 = = 1 –2

x3 –3 –1 –1 –1 x3 2 x3 1

1 X 3 = –2 1

11.161

Engineering Mathematics

11.162

Length of vector X1 =

(–1) 2 + 02 + 12 = 2

Length of vector X2 =

12 + 12 + 12 = 3

Length of vector X3 = The normalised eigen vectors are ⎡ 1 ⎤ ⎥ ⎢− 2⎥ ⎢ X1 = ⎢ 0 ⎥ , ⎥ ⎢ ⎢ 1 ⎥ ⎢⎣ 2 ⎥⎦

X2 =

12 + (–2) 2 + 12 = 6 1

1

3 1

6 –2

3 1

,

X3 =

6 1 6

3

⎡ 1 ⎢− 2 ⎢ ⎢ P= ⎢ 0 ⎢ ⎢ 1 ⎢ ⎣ 2

1 3 1 3 1 3

,

1 ⎤ ⎥ 6 ⎥ 2 ⎥ – ⎥ 6⎥ 1 ⎥ ⎥ 6 ⎦

P is orthogonal matrix i.e., P = P Let X = PY be the orthogonal transformation which transforms Q to canonical form. Q = YT (PTAP)Y = Y T DY ⎡ 2 0 0⎤ ⎡ y1 ⎤ ⎥⎢ ⎥ ⎢ = [y1 y2 y3] ⎢ 0 3 0⎥ ⎢ y2 ⎥ ⎢⎣ 0 0 6⎥⎦ ⎢⎣ y3 ⎥⎦ 2 2 = 2y1 + 3y2 + 6y32 Rank r = Number of non-zero terms in canonical form = 3 Index P = Number of positive terms in canonical form = 3 Signature = Difference between the number of positive and negative terms in canonical form = 3 0 = 3 Since only positive terms occur in the canonical form, value class of quadratic form is positive definite. (iii) Q = 2x2 + 2y2 z2 8xy + 4xz 4yz 1

= X AX = [ x T

T

y

2 –4 2 A = –4 2 –2 2 –2 –1

2 –4 2 z ] –4 2 –2 2 –2 –1

x y z

Matrices

11.163

The characteristic equation is lI | = 0

|A 2–l –4 2

l3 (a) For l =

2, [A

–4 2–l –2

3l2

2 –2 = 0 –1 – l

28 = 0 l = 2, 2, 7

24l

l1I] X = O x 0 y = 0 z 0

4 –4 2 –4 4 –2 2 –2 1

1 R1 2 4 –4 2 x 0 0 0 0 y = 0

R2 + R1,

0

R3

0

z

0

0

4x 4y 2z = 0 Rank of coefficient matrix = 1 Number of unknowns = 3 Number of linearly independent solutions = 3 1 = 2 Taking z = 2 and y = 1, x = 2 z = 2 and y = 2, x = 1 2 1 X1 = 1 , X 2 = 2 2 (b) For l = 7, [A

–2

l2I ] X = O –5 –4 2 –4 –5 –2

x 0 y = 0

2 –2 –8 z 0 5x 4y + 2z = 0 4x 5y 2z = 0 2x 2y 8z = 0 x y z =– = –4 2 –5 2 –5 –4 –5 –2 –4 –2 –4 –5 x y z =– = 18 18 9

Engineering Mathematics

11.164

x y z = = 2 –2 1 2 X 3 = –2 1 Length of vector X1 =

22 + 12 + 22 = 3

Length of vector X2 =

12 + 22 + (–2) 2 = 3

Length of vector X3 = 22 + (–2) 2 + 12 = 3 The normalised eigen vectors are 2 2 1 3 3 3 –2 1 2 X1 = , X2 = , X3 = 3 3 3 2 –2 1 3 3 3 2 3 1 P= 3

1 3 2 3

2 3 –2 3

2 3

–2 3

1 3

P is orthogonal matrix, i.e., P 1 = PT Let X = PY be the orthogonal transformation which transforms Q to cannonical form. Q = YT (PTAP)Y = Y T DY ⎡ –2 0 0 ⎤ ⎡ y1 ⎤ = [y1 y2 y3] ⎢⎢ 0 –2 0 ⎥⎥ ⎢⎢ y2 ⎥⎥ ⎢⎣ 0 0 7⎥⎦ ⎢⎣ y3 ⎥⎦ =

2y12

2y22 + 7y32

Rank r = Number of non-zero terms in canonical form = 3 Index P = Number of positive terms in canonical form = 1 Signature = Difference between the number of positive and negative terms in canonical form = 1 2 = 1 Since both positive and negative terms occur in canonical form, value class of quadratic form is indefinite. Example 6: Reduce the following quadratic forms to canonical form by congruent transformation. Also find the rank, index, signature and value class (nature) of the quadratic forms.

Matrices

11.165

(i) x12 + 2x22 + 3x32 + 2x1x2 - 2x1x3 + 2x2x3 (ii) 2x12 + x22 - 3x32 - 8x2x3 - 4x1x3 + 12x1x2 (iii) x2 + 2y2 + 2z2 - 2xy - 2yz + zx. Solution: (i)

Q = x12 + 2x22 + 3x32 + 2x1x2 Q = X AX= x1 T

x2

x3

2x1x3 + 2x2 x3 1 1 –1 x1 1 2 1 x2 –1 1 3 x3

1 1 –1 A= 1 2 1 –1 1 3 Let A = I3 AI3 1 1 –1 1 0 0 1 0 0 1 2 1 = 0 1 0 A 0 1 0 –1 1 3 0 0 1 R2 R1, R3 + R1

0 0 1

1 1 –1 1 0 0 1 0 0 0 1 2 = –1 1 0 A 0 1 0 0 2 C2

2

1 0

1

0 0 1

C1, C3 + C1

1 0 0 1 0 0 1 –1 1 0 1 2 = –1 1 0 A 0 1 0 0 2 2 1 0 1 0 0 1 R3

2R2

1 0 0 1

0 1 2 = –1

0 0 –2 C3

0 0 1 –1 1 1 0 A 0 1 0

3 –2

1

0

0

1

2C2

1 0 0 1 0 0 1 –1 3 0 1 0 = –1 1 0 A 0 1 –2 0 0 –2 3 –2 1 0 0 1 1 2

R3 ,

1 2

C3

11.166

Engineering Mathematics

3 ⎤ ⎡ 0 ⎤ ⎢ 1 –1 2 ⎥⎥ 0⎥ ⎢ ⎥ A ⎢0 1 – 2 ⎥ 1 ⎥ ⎢ 1 ⎥ ⎥ 2 ⎥⎦ ⎢0 0 2 ⎥⎦ ⎢⎣

0 ⎡ 1 ⎡ 1 0 0⎤ ⎢ 1 1 – ⎢0 1 0 ⎥ = ⎢ ⎢ ⎥ ⎢ 3 – 2 ⎣0 0 –1⎦ ⎢ ⎣ 2

⎡ 1 0 0⎤ D = ⎢0 1 0 ⎥ ⎢ ⎥ ⎣0 0 –1⎦ 3 ⎤ ⎡ ⎢ 1 –1 2 ⎥⎥ ⎢ P = ⎢0 1 – 2 ⎥ ⎢ 1 ⎥ ⎢0 0 ⎥ 2 ⎥⎦ ⎢⎣

Let X = PY be the linear transformation. 3 ⎤ ⎡ ⎢ 1 –1 2 ⎥⎥ ⎡ y1 ⎤ ⎡ x1 ⎤ ⎢ ⎢ x ⎥ = ⎢0 1 – 2 ⎥ ⎢ y ⎥ ⎢ 2⎥ ⎢ ⎢ 2⎥ 1 ⎥ ⎣ y3 ⎦ ⎣ x3 ⎦ ⎢0 0 ⎥ 2 ⎥⎦ ⎢⎣ x1 = y1 x2 = y2 x3 =

3

y2 +

2

y3

2 y3

1 2

y3

Thus, transformation X = PY transforms the given quadratic form to the canonical form y12 + y22 y32 Rank r = Number of non-zero terms in cannonical form = 3 Index P = Number of positive terms in cannonical form = 2 Signature = Difference between the number of positive and negative terms in canonical form = 2 1 = 1 Since both positive and negative terms occur in the canonical form, value class of quadratic form is indefinite. (ii) Q = 2x12 + x22 3x32 8x2x3 4x1x3 + 12x1x2 Q = XTAX = ⎡⎣ x1

x2

⎡ 2 6 –2 ⎤ ⎡ x1 ⎤ x 3 ⎤⎦ ⎢ 6 1 –4 ⎥ ⎢ x2 ⎥ ⎢ ⎥⎢ ⎥ ⎣ –2 –4 –3⎦ ⎣ x3 ⎦

Matrices

11.167

⎡ 2 6 –2 ⎤ A=⎢ 6 1 –4 ⎥ ⎢ ⎥ ⎣ –2 –4 –3⎦ Let A = I3 AI3 ⎡ 2 6 –2 ⎤ ⎡ 1 0 0 ⎤ ⎡ 1 0 0 ⎤ ⎢ 6 1 –4 ⎥ = ⎢ 0 1 0 ⎥ A ⎢ 0 1 0 ⎥ ⎢ ⎥ ⎢ ⎥ ⎢ ⎥ ⎣ –2 –4 –3⎦ ⎣ 0 0 1 ⎦ ⎣ 0 0 1 ⎦ R2 3R1, R3 + R1 6 –2 ⎤ ⎡ 1 0 0 ⎤ ⎡ 1 0 0 ⎤ ⎡2 ⎢ 0 –17 2 ⎥ = ⎢ –3 1 0 ⎥ A ⎢0 1 0 ⎥ ⎢ ⎥ ⎢ ⎥ ⎢ ⎥ 2 –5⎦ ⎣ 1 0 1⎦ ⎣0 0 1 ⎦ ⎣0 C2 3C1, C3 + C1 0 0 ⎤ ⎡ 1 0 0 ⎤ ⎡ 1 –3 1⎤ ⎡2 ⎢ 0 –17 2 ⎥ = ⎢ –3 1 0 ⎥ A ⎢0 1 0⎥ ⎢ ⎥ ⎢ ⎥ ⎢ ⎥ 2 –5⎦ ⎣ 1 0 1⎦ ⎣0 0 1⎦ ⎣0 2 R3 + R2 17 0 0⎤ ⎡ 1 0 0⎤ ⎡2 ⎡ 1 –3 1⎤ ⎢ 0 –17 2 ⎥ ⎢ –3 1 0⎥ ⎢ ⎢ ⎥=⎢ ⎥A 0 1 0⎥ 81 ⎥ ⎢ 11 2 ⎢ ⎥ ⎢0 ⎥ 0 – 1 ⎣0 0 1⎦ ⎢⎣ ⎥ ⎢ ⎥ 17 ⎦ ⎣17 17 ⎦ 2 C2 C3 + 17 11 ⎤ ⎡ 0 0 ⎤ ⎡ 1 0 0 ⎤ ⎢ 1 –3 ⎡2 17 ⎥ ⎢ 0 –17 ⎥ 0 ⎥ ⎢ –3 1 0⎥ ⎢ 2⎥ ⎢ ⎥=⎢ ⎥ A ⎢0 1 81 11 2 ⎢0 17 ⎥ 0 – ⎥ ⎢ 1⎥ ⎢ ⎢⎣ ⎥⎦ ⎢0 0 17 ⎥⎦ ⎢⎣17 17 1⎥⎦ ⎣ 1 1 1 1 R1 , C1 , R2 , C2 2 2 17 17 17 R3 , 81

17 C3 81

⎡ 1 ⎢ 2 1 0 0 ⎡ ⎤ ⎢ ⎢ ⎢0 –1 0 ⎥ = – 3 ⎢ ⎥ ⎢ 17 0 0 1 – ⎣ ⎦ ⎢ ⎢ 11 ⎢ ⎣ 9 17

0 1 17 2 9 17

⎤ ⎡ 0 ⎥ ⎢ ⎥ ⎢ ⎥ ⎢ 0 ⎥ A⎢ ⎥ ⎢ 17 ⎥ ⎢ ⎥ 9 ⎦ ⎢⎣

1 2 0 0



3 17 1 17 0

11 ⎤ ⎥ 9 17 ⎥ 2 ⎥ 9 17 ⎥ ⎥ 17 ⎥ 9 ⎥⎦

Engineering Mathematics

11.168

1 0 0 D = 0 –1 0 0 0 –1 1



2 P=

0 0

3

11

17 1

9 17 2

17

9 17

0

17 9

Let X = PY be the linear transformation. 1 2

x1 x2 = x3

0 0

x1 =

1

x2 =

1

2

–3

11

17 1

9 17 2

17

9 17

0

17 9 3

y1

17

y2 +

17

y2 +

2 9 17

y1 y2 y3

11 9 17

y3

y3,

17 y3 9 Thus, transformation X = PY transforms the given quadratic form to canonical form y12 y22 y32 Rank r = Number of non-zero terms in canonical form = 3 Index P = Number of positive terms in canonical form = 1 Signature = Difference between the number of positive and negative terms in canonical form = 1 2 = 1 Since both positive and negative terms occur in the canonical form, value class of quadratic form is indefinite. (iii) Q = x2 + 2y2 + 2z2 2xy 2yz + zx x3 =

1 2 2 –1

1 –1 = X AX = [ x T

y

z ] –1 1 –1 2

2

x y z

Matrices

11.169

1 2 2 –1

1 –1 A = –1 1 –1 2

2

Let A = I3 AI3 1 1 0 0 1 0 0 2 –1 2 –1 = 0 1 0 A 0 1 0 1 0 0 1 0 0 1 –1 2 2 1 –1

R2 + R1, C2 + C1, R3 – 1

1 1 R1 , C3 – C1 2 2

0

0 1 1 0 0 1 1 – 1 2 0 1 1 1 0 2 = A 0 1 0 1 1 7 – 0 1 0 – 0 0 1 2 2 4 1 1 R3 + R2 , C3 + C2 2 2 1 0 1 1

0 0

1 1

0

1 0

0

0 1

1 0 A 0 1 = 2 1 3 1 1 0 0 1 2 2

0 0 2 R3, 3

2 C 3 3

1 1 0 0 1 0 1 0 = 0 0 1 0

0

0

1

0

1 6

1 0 0 D= 0 1 0 0 0 1

2 3

1 1 A

0 1 0 0

0 1 6 2 3

Engineering Mathematics

11.170

1 1

0 1

P= 0 1

6 2 3

0 0 Let X = PY be the linear transformation. 1 1 x y = 0 1 z

0 1 6

u v w

2 3

0 0 x=u+v y=v+

1 6

w

2 w 3 Thus transformation X = PY transforms the given quadratic form to the cannonical form u2 + v2 + w2 Rank r = Number of non-zero terms in canonical form = 3 Index P = Number of positive terms in canonical form = 3 Signature = Difference between the number of positive and negative terms in canonical form = 3 0 = 3 Since only positive terms occur in the canonical form, value class of quadratic form is positive definite. z=

Example 7: Show that 5x12 + 26x22 + 10x32 + 4x2x3 + 14x1x3 + 6x1x2 is positive semi-definite and find a non-zero set of values of x1, x2, x3 which makes the form zero. Solution:

Q = 5x12 + 26x22 + 10x32 + 4x2x3 + 14x1x3 + 6x1x2 = X AX = x1 T

5 3 7 A = 3 26 2 7 2 10

x2

x3

5 3 7 3 26 2 7 2 10

x1 x2 x3

Matrices

11.171

Let A = I3 AI3 5 3 7 1 0 0 1 0 0 3 26 2 = 0 1 0 A 0 1 0 7 2 10 0 0 1 0 0 1 3 3 R2 – R1 , C2 – C1 5 5 R3 –

7 7 R1 , C3 – C1 5 5

5

0 0 1 0 0 3 1 – 121 11 3 5 0 – – 1 0 A 5 5 = 5 0 1 11 1 7 0 0 0 – – 0 1 5 5 5 1 , 1 R2 C3 + C2 11 11 1 0 0 3 5 0 0 1 – 5 3 121 – 1 0 0 0 = A 5 0 1 5 16 1 0 0 0 – 1 0 0 11 11

7 5 0 1



R3 +

5

0 0 121 D= 0 0 5 0 0 0 1 –

3 5

P= 0

1

0

0

16 11 1 11 1



Let X = PY be the linear transformation 3 1 – x1 5 x2 = 0 1 x3 0 0

16 11 1 11 1



y1 y2 y3

16 11 1 11 1



Engineering Mathematics

11.172

x1 = y1

3 y2 5

x2 = y2 +

1 y3, 11

16 y3 11

x3 = y3 Thus, transformation X = PY transforms the given quadratic form to the canonical form 121 2 y 5 2 Rank r = Number of non-zero terms in canonical form = 2 Index P = Number of positive terms in canonical form = 2 Signature = Difference between the number of positive and negative terms in canonical form = 2 0 = 2 Since all the terms in canonical form is greater than or equal to zero and at least one term is zero, value class of quadratic form is positive semi-definite. The set of values y1 = 0, y2 = 0, y3 = 1 will reduce quadratic form to zero. For this set of value, 1 16 x3 = 1, x2 = , x1 = – 11 11 5y12 +

This is a non-zero set of values of x1, x2, x3 which will make the quadratic form zero. Exercise 11.15 1. Express the following quadratic forms in matrix notation: (i) 2x2 + 3y2 + 6xy (ii) 2x2 + 5y2 6z2 2xy yz + 8zx (iii) x12 + 2x22 7x32 + x42 4x1x2 + 8x1x3 6x3x4 (iv) x12 + 2x22 + 2x32 2x1x2 2x2x3 (v) x12 + 2x22 + 3x32 + 4x42 + 2x1x2 + 4x1x3 6x1x4 4x2x3 8x2x4 + 12x3x4. ⎡ ⎢ ⎢ Ans. : (i) ⎢ ⎢ ⎢ ⎢ ⎢

⎡ 2 −1 4 ⎤ ⎢ 2 3 ⎡ ⎤ 1 ⎥⎥ ⎢ 1 5 ( ii ) − − ⎢ 3 3⎥ 2⎥ ⎢ ⎣ ⎦ ⎢ ⎥ 1 −6 ⎥ ⎢ 4 − ⎣ ⎦ 2

⎤ ⎥ ⎥ ⎥ ⎥ ⎥ ⎥ ⎥

⎢ ⎢ ⎢ ⎢ ⎢ ⎢ ⎢ ⎢ ⎢ ⎢ ⎢(iv ) ⎢ ⎢ ⎣⎢

⎡ 1 −2 4 0 ⎤ ⎢ −2 2 0 0 ⎥ ⎥ (iii) ⎢ ⎢ 4 0 −7 −3⎥ ⎢ ⎥ ⎣ 0 0 −3 1⎦ ⎡ 1 1 2 ⎡ 1 −1 0 ⎤ ⎢ ⎢ −1 2 −1⎥ ( v ) ⎢ 1 2 −2 ⎢ ⎥ ⎢ 2 −2 3 ⎢⎣ 0 −1 2⎦⎥ ⎢ ⎣ −3 −4 6

⎥ ⎥ ⎥ ⎥ ⎥ ⎥ ⎥ ⎥ −3⎤ ⎥ −4 ⎥⎥ ⎥ ⎥ 6⎥ ⎥ ⎥ 4 ⎦ ⎦⎥⎥

2. Write down the quadratic forms corresponding to following matrices: ⎡ 1 1 −2 0 ⎤ ⎡ 1 2 −1⎤ ⎢ 1 −4 0 0 ⎥ ⎥ (i) ⎢ 2 0 3⎥ (ii) ⎢ ⎢ ⎥ ⎢ −2 0 6 −3⎥ ⎢⎣ −1 3 1⎥⎦ ⎢ ⎥ ⎣ 0 0 −3 2⎦

Matrices

3 ⎡ ⎤ −2⎥ ⎢ 2 −1 2 ⎢ ⎥ 5 ⎢ −1 −3 − 3⎥ ⎢ ⎥ 2 (iii) ⎢ ⎥ 1⎥ ⎢ 3 −5 4 ⎢ 2 2 2⎥ ⎢ ⎥ 1 ⎢ −2 3 1⎥ ⎢⎣ ⎥⎦ 2 ⎡ Ans. : ⎤ ⎢ ⎥ 2 2 ⎢(i) x1 + x3 + 4 x1 x2 − 2 x1 x3 + 6 x2 x3 ⎥ ⎢(ii) x 2 − 4 x 2 + 6 x 2 + 2 x 2 + 2 x x ⎥ 1 2 3 4 1 2 ⎢ ⎥ − 4 x1 x3 − 6 x3 x4 ⎥ ⎢ ⎢ ⎥ 2 2 2 2 ⎢(iii) 2 x1 − 3 x2 + 4 x3 + x4 − 2 x1 x2 ⎥ ⎢ + 3 x1 x3 − 4 x1 x4 − 5 x2 x3 ⎥ ⎢ ⎥ + 6 x2 x4 + x3 x4 ⎢⎣ ⎥⎦ 3. Reduce the following quadratic forms to canonical forms by orthogonal transformation. Also find rank, index and signature. (i) 3x2 + 5y2 + 3z2 2xy 2yz + 2zx (ii) 2x12 + 2y12 + 2z12 2x1x2 + 2x1x3 2x2x3

11.173

(iii) 3x2

2y2

z2

4xy + 8xz + 12yz.

⎡ Ans. : (i) 2 y12 + 2 y22 + 6 y32 ; r = 3, ⎤ ⎢ ⎥ P = 3, signature = 3 ⎢ ⎥ 2 2 2 ⎢ (ii) 4 y1 + y2 + y3 ; r = 3, ⎥ ⎢ ⎥ P = 3, signature = 3 ⎥ ⎢ ⎢ ⎥ (iii) 3y12 + 6 y22 − 9 y32 ; r = 3,⎥ ⎢ ⎢⎣ P = 2, signature = 1 ⎥⎦ 4. Reduce the following quadratic forms to canonical forms by congruent transformation. Also find rank, index and signature. (i) x2 2y2 + 3z2 4yz + 6zx (ii) 2x2 2y2 + 2z2 2xy 8yz + 6zx (iii) x2 + 3y2 + 8z2 + 4w2 + 4xy + 6xz 4xw + 12yz 8yw 12zw. ⎡ Ans. : (i) y12 − y22 − y32 ; r = 3, ⎤ ⎢ ⎥ P = 1, signature = −1 ⎥ ⎢ ⎢ (ii) y12 − y22 − y32 ; r = 3, ⎥ ⎢ ⎥ P = 1, signature = −1⎥ ⎢ ⎢ ⎥ (iii) y12 − y22 − y32 ; r = 3, ⎥ ⎢ ⎢⎣ P = 1, signature = −1 ⎥⎦

MULTIPLE CHOICE QUESTIONS Choose the correct alternative in each of the following: ⎡ cos 2 q cos q sin q ⎤ 1. If A(q ) = ⎢ ⎥ sin 2 q ⎦ ⎣cos q sin q and q and f differ- by an odd multiple of p /2, then A(q ) A(f ) is a (a) null matrix (b) unit matrix (c) diagonal matrix (d) none of these 2. The matrix A satisfying the equation ⎡1 3⎤ ⎡ 1 1⎤ ⎢0 1⎥ A = ⎢0 −1⎥ is ⎦ ⎦ ⎣ ⎣ ⎡ 1 4⎤ (a) ⎢ ⎥ ⎣ −1 0⎦

⎡1 − 4⎤ (b) ⎢ ⎥ ⎣1 0⎦

⎡ 1 4⎤ (c) ⎢ ⎥ ⎣0 −1⎦

(d) none of these

3. If A is an orthogonal matrix, then A–1 equals (a) A (b) AT 2 (c) A (d) none of these 4. If D = diag (d1, d2, d3, … dn), where di 0 for all i = 1, 2, … n, then D–1 is equal to (a) D (b) diag (d1–1, d2–1, … dn–1) (c) In (d) none of these

11.174

Engineering Mathematics

5. If A = diag (d1, d2, d3, … dn) then An is equal to (a) diag (d1n – 1, d2n – 1, d3n – 1, … dnn –1) (b) diag (d1n, d2n, d3n … dnn) (c) A (d) none of these 6. If A is a symmetric matrix and n N, then An is (a) symmetric (b) skew symmetric (c) diagonal matrix (d) none of these ⎡ 1 −5 7⎤ 7. If A = ⎢⎢ 0 7 9⎥⎥ , then trace of ⎣⎢11 8 9⎦⎥ the matrix A is (a) 17 (b) 25 (c) 3 (d) 12 8. All the four entries of the 2 2 ⎡ P11 P12 ⎤ matrix P = ⎢ ⎥ are non-zero, ⎣ P21 P22 ⎦ and one of its eigen values is zero. Which of the following statements are true? (a) P11 P22 – P12 P21 = 1 (b) P11 P22 – P12 P21 = –1 (c) P11 P22 – P12 P21 = 0 (d) P11 P22 + P12 P21 = 0 9. The system of linear equations 4x + 2y = 7, 2x + y = 6 has (a) a unique solution (b) no solution (c) an infinite number of solution (d) exactly two distinct solutions 1⎤ ⎡ 0 10. Consider the matrix P = ⎢ ⎥. − 2 − 3 ⎦ ⎣ The value of e P is ⎡ 2e − 2 − 3e −1 e −1 − e − 2 ⎤ (a) ⎢ − 2 ⎥ −1 5e − 2 − e −1 ⎦ ⎣ 2e − 2e ⎡ e −1 + e − 2 2e −2 − e −1 ⎤ (b) ⎢ −1 ⎥ −2 3e −1 + 2e − 2 ⎦ ⎣ 2e − 4e

⎡ 5e − 2 − e −1 3e −1 − e − 2 ⎤ (c) ⎢ − 2 ⎥ −1 4e − 2 + e −1 ⎦ ⎣ 2e − 6e ⎡ 2e −1 − e − 2 e −1 − e − 2 ⎤ (d) ⎢ ⎥ −1 −2 − e −1 + 2e −2 ⎦ ⎣ − 2e + 2e ⎡1 1 1⎤ 11. The rank of the matrix ⎢⎢1 −1 0⎥⎥ is ⎢⎣1 1 1⎥⎦ (a) 0 (b) 1 (c) 2 (d) 3 12. The eigen values and the corresponding eigenvectors of a 2 2 matrix are given by eigen value eigen vector ⎡1⎤ l1 = 8 x1 = ⎢ ⎥ ⎣1⎦ l2 = 4

⎡ 1⎤ x2 = ⎢ ⎥ ⎣ −1⎦

The matrix is ⎡ 6 2⎤ (a) ⎢ ⎥ ⎣ 2 6⎦ ⎡ 2 4⎤ (c) ⎢ ⎥ ⎣ 4 2⎦

⎡4 (b) ⎢ ⎣6 ⎡4 (d) ⎢ ⎣8

6⎤ 4⎥⎦ 8⎤ 4⎥⎦

⎡ 2 − 0.1⎤ 13. Let A = ⎢ and 3⎥⎦ ⎣0 ⎡ 1 a⎤ ⎥ . Then (a + b) is –1 A = ⎢2 ⎢ 0 b⎥ ⎣ ⎦ 7 3 (a) (b) 20 20 19 11 (c) (d) 60 20 14. Given an orthogonal matrix ⎡ 1 1 1 1⎤ ⎢ 1 1 −1 −1⎥ ⎥ , [AA–1] is A= ⎢ ⎢ 1 −1 0 0⎥ ⎢ ⎥ ⎣0 0 1 1⎦

Matrices

⎡1 ⎢4 ⎢ ⎢0 (a) ⎢ ⎢0 ⎢ ⎢ ⎣0 ⎡1 ⎢2 ⎢ ⎢0 (b) ⎢ ⎢0 ⎢ ⎢ ⎢0 ⎣ ⎡1 ⎢0 (c) ⎢ ⎢0 ⎢ ⎣0 ⎡1 ⎢4 ⎢ ⎢0 (d) ⎢ ⎢0 ⎢ ⎢ ⎢0 ⎣

0

0 0⎤ ⎥ ⎥ 0 0⎥ ⎥ 1 0⎥⎥ 2 ⎥ 0 1⎦

0

0

1 2

0

0

1 2

0

0

0 1 4 0

0⎤ ⎥ ⎥ 0⎥ ⎥ 0⎥⎥ ⎥ 1⎥ 2⎦

0 0 0⎤ 1 0 0⎥⎥ 0 1 0⎥ ⎥ 0 0 1⎦ 0

0

1 4

0

0

1 4

0

0

0⎤ ⎥ ⎥ 0⎥ ⎥ 0 ⎥⎥ ⎥ 1⎥ 4⎦

15. The characteristic equation of a 3 3 matrix P is defined as a (l) = | l I – P | = l3 + l2 + 2l + 1 = 0 If I denotes identity matrix, then the inverse of matrix P will be (a) P2 + P + 2I (b) P2 + P + I (c) – (P2 + P + I) (d) – (P2 + P + 2I) 16. If the rank of a (5 6) matrix Q is 4, then which one of the following statements is correct? (a) Q will have four linearly independent rows and four linearly independent columns.

11.175

(b) Q will have four linearly independent rows and five linearly independent columns. (c) QQT will be invertible (d) Q T Q will be invertible. 17. A is a m n full rank matrix with m > n and I is an identity matrix. Let matrix A+ = (AT A) –1 AT. Then, which one of the following statements is FALSE? (a) AA+ A = A (b) (AA+)2 = AA+ (c) A + A = I (d) AA+ A = A + ⎡ 3 − 2 2⎤ ⎢ ⎥ 18. For the matrix P = ⎢0 − 2 1 ⎥ , one ⎢⎣0 0 1 ⎥⎦ of the eigen value is equal to –2. Which of the following is an eigen vector? ⎡ 3⎤ ⎡ − 3⎤ (a) ⎢⎢ − 2⎥⎥ (b) ⎢⎢ 2⎥⎥ ⎢⎣ 1⎥⎦ ⎢⎣ −1⎥⎦ ⎡ 1⎤ (c) ⎢⎢ − 2⎥⎥ ⎢⎣ 3⎥⎦

⎡ 2⎤ (d) ⎢⎢ 5⎥⎥ ⎢⎣ 0⎥⎦

⎡1 0 −1⎤ ⎢ ⎥ 19. If R = ⎢ 2 1 −1⎥ , the top row of ⎢⎣ 2 3 2 ⎥⎦ R –1 is (a) [5 6 4] (b) [5 – 3 1] 1⎤ –1] (d) ⎡ 2 −1 ⎢⎣ 2 ⎥⎦ 20. Consider the following system of equations in three real variables x1, x2 and x3. 2x1 – x2 + 3x3 = 1 3x1 + 2x2 + 5x3 = 2 – x1 + 4x2 + x3 = 3 The system equation has (a) no solution (c) [2

0

Engineering Mathematics

11.176

(b) a unique solution (c) more than one but a finite number of solutions (d) an infinite number of solutions ⎡ 3 2⎤ 21. Eigen values of a matrix S = ⎢ ⎥ ⎣ 2 3⎦ are 5 and 1. What are the eigen values of the matrix S2? (a) 1 and 25 (b) 6 and 4 (c) 5 and 1 (d) 2 and 10 22. Match the items in columns I and II I II (P) Singular (1) Determinant matrix is not defined (Q) Non-square (2) Determinant matrix is always ±1 (R) Real (3) Determinant symmetric is zero matrix (S) Orthogonal (4) Eigen values matrix are always real (5) Eigen values are not defined (a) P-3, Q-1, R-4, S-2 (b) P-2, Q-3, R-4, S-1 (c) P-3, Q-2, R-5, S-4 (d) P-3, Q-4, R-2, S-1 23. Multiplication of matrices E and F is G. Matrices E and G are, ⎡cos q - sin q 0⎤ ⎢ ⎥ E = ⎢ sin q cos q 0⎥ and 0 1 ⎦⎥ ⎣⎢ 0 ⎡1 0 0⎤ G = ⎢0 1 0⎥ , what is the matrix F? ⎢ ⎥ ⎢⎣0 0 1 ⎥⎦ ⎡cos q - sin q 0⎤ (a) ⎢⎢ sin q cos q 0⎥⎥ 0 1 ⎦⎥ ⎣⎢ 0 ⎡ sin q (b) ⎢⎢ − cos q ⎢⎣ 0

cos q sin q 0

0⎤ 0⎥⎥ 1 ⎥⎦

⎡ cos q (c) ⎢⎢ − sin q ⎢⎣ 0 ⎡ sin q (d) ⎢⎢cos q ⎢⎣ 0

sin q cos q 0

0⎤ 0⎥⎥ 1 ⎥⎦

− cos q sin q 0

0⎤ 0⎥⎥ 1 ⎥⎦

24. A is a 3 4 real matrix and Ax = b is an inconsistent system of equations. The highest possible rank of A is (a) 1 (b) 2 (c) 3 (d) 4 25. The sum of eigen values of the matrix ⎡1 1 3⎤ ⎢1 5 1⎥ is ⎢ ⎥ ⎢⎣3 1 1⎥⎦ (a) 5 (b) 7 (c) 9 (d) 18 26. For which value of X will be the ⎡ 8 x 0⎤ matrix ⎢⎢ 4 0 2⎥⎥ become singular? ⎢⎣12 6 0⎥⎦ (a) 4 (b) 6 (c) 8 (d) 12 27. Let A and B be real symmetric matrices of size n n. Then which one of the following is true? (a) AA = I (b) A = A–1 (c) AB = BA (d) (AB) = BA 28. Among the following, the pair of vectors orthogonal to each other is, (a) [3, 4, 7], [3, 4, 7] (b) [1, 0, 0], [1, 1, 0] (c) [1, 0, 2], [0, 5, 0] (d) [1, 1, 1], [–1, –1, –1] 29. A 5 7 matrix has all its entries equal to –1. The rank of the matrix is (a) 7 (b) 5 (c) 1 (d) zero

Matrices

30. The eigen values of the matrix ⎡ 2 −1 0 0⎤ ⎢ 0 3 0 0⎥ ⎢ ⎥ are ⎢ 0 0 −2 0⎥ ⎢ ⎥ ⎣ 0 0 1 4⎦ (a) 2, –2, 1, –1 (b) 2, 3, –2, 4 (c) 2, 3, 1, 4 (d) none of these ⎡1 1 1⎤ 31. The eigen values of ⎢⎢1 1 1⎥⎥ are ⎢⎣1 1 1⎥⎦ (a) 0, 0, 0 (b) 0, 0, 1 (c) 0, 0, 3 (d) 1, 1, 1 32. The minimum and maximum ⎡1 1 3⎤ eigen values of the matrix ⎢⎢1 5 1⎥⎥ ⎢⎣3 1 1⎥⎦ are –2 and 6 respectively. What is the other eigenvalue? (a) 5 (b) 3 (c) 1 (d) –1 ⎡1 0 0⎤ 33. The inverse of the matrix ⎢⎢0 2 0⎥⎥ is ⎢⎣0 0 3⎥⎦ 1 0 0 ⎡ ⎤ ⎢ ⎥ 1 ⎢0 0⎥ (a) ⎢ 2 ⎥ ⎢ 1⎥ 0 0 3 ⎦⎥ ⎣⎢ ⎡1 0 ⎢ ⎢0 1 (b) ⎢ 3 ⎢ ⎢⎣0 0

0⎤ ⎥ 0⎥ ⎥ 1⎥ 2 ⎥⎦

⎡1 0 0⎤ ⎢2 ⎥ ⎢ (c) 0 1 0 ⎥ ⎢ ⎥ ⎢ 1⎥ ⎢⎣ 0 0 3 ⎥⎦ (d) none of these

11.177

34. Are the following vectors linearly dependent? X1 = [3, 2, 7], X2 = [2, 4, 1] and X3 = [1, –2, 6] (a) Dependent (b) Independent (c) Can’t say (d) None of these ⎡1 0⎤ 35. If A = ⎢ ⎥ , then the value ⎣1 7 ⎦ of k for which (a) 5 (c) 7 36. The matrix form tions 6x12 + 3x22 18x3 x1 + 4x1 x2 is

A2 = 8A + kI is (b) –5 (d) –7 of quadratic equa+ 14x32 + 4x2 x3 +

⎡ 6 2 9⎤ (a) ⎢⎢ 2 3 2⎥⎥ ⎢⎣ 9 2 14⎥⎦ ⎡ 6 4 18⎤ (b) ⎢⎢ 4 3 4⎥⎥ ⎢⎣18 4 14⎥⎦ ⎡ − 6 − 2 − 9⎤ (c) ⎢⎢ − 2 − 3 − 2⎥⎥ ⎢⎣ 9 2 −14⎥⎦ ⎡ − 6 4 18⎤ (d) ⎢⎢ 4 − 3 4⎥⎥ ⎢⎣ 18 4 −14⎥⎦ ⎡ a + i c −b + id ⎤ 37. A = ⎢ ⎥ is unitary ⎣b + id a − i c ⎦ matrix if and only if (a) a2 + b2 + c2 = 0 (b) b2 + c2 + d2 = 0 (c) a2 + b2 + c2 + d2 = 1 (d) a2 + b2 + c2 + d2 = 0 38. The quadratic form of the symmetric matrix diag [l 1, l 2, … ln] is (a) l1 + l2 + … + ln (b) l 12 x1 + l2 x2 + … + l n2 xn (c) l1 x12 + l2 x22 + … + ln xn2 (d) l1 x1 + l 2 x2 + … + ln xn

Engineering Mathematics

11.178

39. The minimal polynomial associated ⎡0 0 3⎤ with the matrix ⎢1 0 2⎥ is ⎢ ⎥ ⎢⎣0 1 1 ⎥⎦ (a) x3 – x2 – 2x – 3 (b) x3 – x2 + 2x – 3 (c) x3 – x2 – 3x – 3 (d) x3 – x2 + 3x – 3 40. Which of the following matrices is not diagonalizable? ⎡1 1 ⎤ (a) ⎢ ⎥ ⎣1 2⎦

⎡1 0⎤ (b) ⎢ ⎥ ⎣3 2 ⎦

⎡0 −1⎤ (c) ⎢ ⎥ ⎣1 0 ⎦

⎡1 1⎤ (d) ⎢ ⎥ ⎣0 1⎦

Answers 1. (a) 8. (c) 15. (d) 22. (a) 29. (c) 36. (a)

2. 9. 16. 23. 30. 37.

(c) (b) (a) (c) (b) (c)

3. 10. 17. 24. 31. 38.

(b) (d) (d) (b) (b) (c)

4. 11. 18. 25. 32. 39.

⎡ cos 2q cos q sin q ⎤ 41. If E(q ) = ⎢ ⎥, sin 2q ⎦ ⎣cos q sin q q and f differ by an odd multiple of p , then E(q ) E(f) is a 2 (a) null matrix (b) unit matrix (c) diagonal matrix (d) none of these 0 0 ⎡1 ⎤ ⎢ ⎥ ⎢ i −1 + i 3 ⎥ 0 42. If A = ⎢ ⎥, 2 ⎢ ⎥ −1 − i 3 ⎥ ⎢0 1 + 2i ⎢⎣ ⎥⎦ 2 102 then the trace of A is (a) 0 (b) 1 (c) 2 (d) 3

(b) (c) (d) (b) (b) (a)

5. 12. 19. 26. 33. 40.

(b) (a) (b) (a) (a) (d)

6. 13. 20. 27. 34. 41.

(a) (a) (b) (d) (a) (d)

7. 14. 21. 28. 35. 42.

(a) (c) (a) (c) (d) (d)

Chapter

12

12.1 INTRODUCTION Laplace transform is the most widely used integral transform. It is a powerful mathematical technique which enables us to solve linear differential equations by using algebraic methods. It can also be used to solve systems of simultaneous differential equations, partial differential equations and integral equations. It is applicable to continuous functions, piecewise continuous functions, periodic functions, step functions and impulse functions. It has many important applications in mathematics, physics, optics, electrical engineering, control engineering, signal processing and probability theory.

12.2 LAPLACE TRANSFORM If f (t) is a function of t defined for all t

0, then

e

st

f (t) dt is defined as Laplace

0

transform of f (t), provided the integral exists and is denoted by L { f (t)}. L{ f (t)} =

e

st

f (t) dt

0

The integral is a function of the parameter s and is denoted by F (s), f (s) or f (s).

The Laplace transform of function f (t) exists when the following sufficient conditions are satisfied: (i) f (t) is piecewise continuous, i.e, f (t) is continuous in every subinterval and f (t) has finite limits at the end points of each subinterval. (ii) f (t) is of exponential order of a, i.e, there exists M, a such that | f (t)| Mea t, for all t 0. In other words,

{

}

lim e -a t f (t ) = finite quantity t

Engineering Mathematics

12.2

12.3 LAPLACE TRANSFORM OF SOME STANDARD FUNCTIONS (i)

f (t) = k Proof: L{k} =

(ii)

k k dt = k

st

e

e

st

s

0

= 0

k s

n

f (t) = t Proof: L{t n} =

st

e

Putting st = x, dt = L{t n } =

0

t ndt

0

e

dx s x s

x

n

dx 1 = n +1 s s

e

0

x

x n dx =

n +1 s n +1

s > 0, n + 1 > 0

If n is a positive integer, n + 1 = n ! L tn =

n! s n +1

e st e at dt =

e

{ }

(iii)

f (t) = eat

{ }

Proof: L e at =

(iv)

0

f (t) =

0

( s a )t

dt =

1 e ( s a )t = ( s a) 0 s a

at

Proof: L {sin at} = L

e

iat

e 2i

iat

1 L e iat 2i

{ } L {e } iat

1 s ia s ia 1 1 1 = 2i s2 + a2 2i s ia s ia a 1 2ia = 2 = 2 2 s + a2 2i s + a

(v)

f (t) =

at

Proof: L {cos at} = L

e iat + e 2

iat

=

=

1 1 1 + 2 s ia s ia

=

s s + a2 2

{ } { }

1 L e iat + L e 2 =

iat

1 s ia s ia 2 s2 + a2

12.3

Laplace Transform

f (t) =

(vi)

at

Proof: L {sinh at } = L

e

at

e 2

at

{ } L {e }

1 1 2 s a a = 2 s a2

f (t) =

(vii)

1 L e at 2 1

=

s a

at

1 s a s a 2 s2 a2

at at

Proof: L {cosh at} = L

at

e +e 2

=

1 L e at + L e 2

{ } { } at

1 1 1 1 s a s a + = 2 s a s a 2 s2 a2 s = 2 s a2

=

(i) (iii)

(v) (vii)

(i)

Find the Laplace transforms by definition: f (t) = 3 02 (iv) f (t) = 1 =0 0 0) . x →∞ x n

7. Prove that lim

log x = 0. x → 0 cot x

8. Prove that lim

1. Prove that lim

log x 2 = 0. x → 0 cot x 2

2. Prove that lim 3. Prove that lim x →1

log (1 − x) = 0. cot x

[Hint : Put x 2 = y ]

log sin x cos x 4. Prove that lim = 4. x →0 x log x cos sin 2 2

5. Prove that lim log tan x tan 2 x = 1. x →0

6. Prove that lim log sin x sin 2 x = 1.

xm = 0 (m > 0) . x →∞ e x

9. Prove that lim 10. Prove that 2

3

n

⎛1⎞ ⎛1⎞ ⎛1⎞ ⎛1⎞ ⎜ ⎟ + ⎜ ⎟ + ⎜ ⎟ + ...... + ⎜ ⎟ e⎠ ⎝e⎠ ⎝e⎠ ⎝ ⎝ e ⎠ = 0. lim n →∞ n

x →0

2.10.5 Type 3 : (0 ë Ç) lim [ f ( x) ⋅ g ( x)], when lim f ( x) = 0, lim g ( x) = ∞ (i.e. 0 × ∞ form) x →a

x →a

f ( x) ⋅ g ( x) = lim We write lim x →a x →a

x →a

f ( x) g ( x) or lim ⋅ x →a 1 1 g ( x) f ( x) 0 or 0

L’Hospital’s rule. Example 1: Prove that lim sin x log x = 0 . xã 0

Solution: Let l = lim sin x log x x→0

[0 × ∞ ]

Differential Calculus I

= lim x →0

2.137

⎡∞⎤ ⎢∞⎥ ⎣ ⎦

log x cosec x

1 x = lim x → 0 −cosec x cot x

[Applying L’Hospital’s rule]

tan x x tan x = − lim sin x ⋅ lim x →0 x →0 x = − lim sin x ⋅ x →0

tan x ⎤ ⎡ = 1⎥ ⎢⎣∵lim x →0 x ⎦

=0 ⎛ a ⎞ Example 2: lim 2 x ⋅ sin ⎜ x ⎟ = a . x→∞ ⎝2 ⎠

a Solution: Let l = lim 2 x ⋅ sin ⎛⎜ x ⎞⎟ x →∞ ⎝2 ⎠ Taking 2 x = 1 , t = 1x , t 2 when x → ∞, 2 x → ∞, l = lim t →0

t →0

sin at a sin at = lim = a ⋅1 = a t → 0 t at



sin x ⎤ ⎡ = 1⎥ ⎢∵ lim x →0 x ⎣ ⎦



1

Example 3: Prove that lim ⎜ a x − 1⎟ x = log a . x→∞ ⎝ ⎠ ⎛ 1 ⎞ Solution: Let l = lim ⎜ a x − 1⎟ ⋅ x x →∞ ⎝ ⎠

[0 × ∞ ]

⎛ 1x ⎞ ⎜ a − 1⎟ ⎠ ⎡0⎤ = lim ⎝ ⎢⎣ 0 ⎥⎦ x →∞ 1 x Taking

1 = t , when x → ∞, t → 0 x l = lim t →0

at − 1 t

⎡0⎤ ⎢0⎥ ⎣ ⎦

a t log a t →0 1 0 = a log a = log a = lim

[Applying L’Hospital’s rule]

Engineering Mathematics

2.138

⎛ o x⎞ Example 4: lim tan 2 ⎜ ⎟ (1 + sec o x ) = −2.. x →1 ⎝ 2 ⎠ ⎛ x⎞ Solution: Let l = lim tan 2 ⎜ ⎟ (1 + sec x) x →1 ⎝ 2 ⎠

[ ∞ × 0]

1 + sec x ⎡0⎤ ⎢0⎥ ⎛ x⎞ ⎣ ⎦ cot 2 ⎜ ⎟ ⎝ 2 ⎠ sec x tan x = lim x →1 x⎞ ⎛ x ⎞⎛ 2 2 cot ⎜ ⎟ ⎜ −cosec ⎟ 2 ⎠2 ⎝ 2 ⎠⎝ = lim x →1

[Applying L’Hospital’s rule]

⎛ ⎞⎛ ⎞ sec p x ⎟ ⎜ tan p x ⎟ ⎜ = − ⎜ lim ⎟ ⎜ lim x →1 x →1 px⎟ 2 px cot ⎟ cosec ⎜ ⎟⎜ ⎝ 2 ⎠⎝ 2 ⎠ ⎛ ⎜ sec p = −⎜ p ⎜ cosec 2 ⎝ 2

⎞ tan p x ⎟ ⎟ lim x →1 px cot ⎟ ⎠ 2

⎡0⎤ ⎢0⎥ ⎣ ⎦

sec 2 x x →1 ⎛ x⎞ 2 ⎜ −cosec ⎟ 2 ⎠2 ⎝

= −(−1) lim

= −2

sec 2 cosec 2

Example 5: lim sin −1 x →a

= −2 2

a− x 1 . cosec a 2 − x 2 = a+ x 2a

Solution: Let l = lim sin −1 x→a

sin −1 = lim x→a

[Applying L’Hospital’s rule]

a−x cosec a 2 − x 2 a+x

[0 × ∞ ]

a−x a+ x

sin a 2 − x 2

Here applying L’Hospital’s rule will make the expression complicated, so we rearrange the terms to apply the limits directly.

Differential Calculus I

Let

2.139

a−x = a , a2 − x2 = b a+x

when x

a, α

0 and β

0

l = limsin −1 α lim

Hence,

α →0

β →0

1 sin β

⎡ ⎛ sin −1 α ⎞ ⎤ ⎡ ⎛ β = ⎢lim ⎜ ⎟ ⋅ α ⎥ ⎢lim ⎜ α →0 β →0 ⎣ ⎝ α ⎠ ⎦ ⎣ ⎝ sin β

= lim α ⋅ lim α →0

= lim x→a

β →0

⎞ 1⎤ ⎟⋅ ⎠ β⎦ ⎡ ⎤ ⎛ sin −1 x ⎞ ⎢∵lim ⎜ ⎟ =1 ⎥ x →0 x ⎝ ⎠ ⎢ ⎥ ⎢ ⎛ sin x ⎞ ⎥ ⎢and lim ⎜ ⎟ = 1⎥ x →0 ⎢⎣ ⎝ x ⎠ ⎦⎥

1 β

a−x 1 ⋅ 2 a + x a − x2

[Resubstituting a and b ]

a−x 1 ⋅ x→a a+x a+x a−x 1 1 = lim = x→a a + x 2a

= lim

m n Example 6: Evaluate lim x (log x ) , where m and n are positive integers. x

0

Solution: Let l = lim x m (log x) n

[0

x →0

⎡∞⎤ ⎢∞⎥ ⎣ ⎦

(log x) n x →0 1 xm

= lim

n (log x) n −1 = lim x →0

1 x

−m ( x) − m −1

(−1)1 n (log x) n −1 x →0 m ( x) − m

= lim

= lim x →0

⎡∞⎤ ⎢∞⎥ ⎣ ⎦

(−1)1 n (n − 1) (log x) n − 2 ⋅

1 x

[Applying L’Hospital’s rule]

m (−m)1 ( x) − m −1

(−1) 2 n (n − 1) (log x) n − 2 x →0 m 2 ( x) − m

= lim

]

⎡∞⎤ ⎢∞⎥ ⎣ ⎦

Engineering Mathematics

2.140

Applying L’Hospital’s rule (n

( −1) n n !(log x)0 x→0 m n ( x) − m

l = lim

( −1) n n ! m ⋅x = 0 x→0 mn

= lim Exercise 2.12 1. Prove that lim x log x = 0.

7. Prove that x⎞ 1 ⎛ log ⎜ 2 − ⎟ cot ( x − a ) = − . a⎠ a ⎝ 8. Prove that ⎛ x⎞ lim log (1 − x) cot ⎜ ⎟ = 0. x →1 ⎝ 2 ⎠ ⎛1+ x ⎞ 9. Prove that lim log ⎜ ⎟ cot x = 2. x →0 ⎝ 1− x ⎠

x →0

2 −x 2. Prove that lim x e = 0. x →∞

2 gy − 2 ⎞ ⎛ x 2 ⎜1 − e x ⎟ = 2 gy. 3. Prove that lim ⎜ ⎟ x →∞ ⎝ ⎠

4. Prove that lim tan x log x = 0. x →0

5. Prove that

10. Prove that

4 ⎛ x⎞ lim ( x 2 − 1) tan ⎜ ⎟=− . x →1 ⎝ 2 ⎠

lim x →a

6. Prove that

11. Prove that

x lim (1 + sec x) tan = 0. x →1 2

lim x →2

2.10.6 Type 4 : (Ç

a+x tan −1 a 2 − x 2 = 2a. a−x

2+ x tan −1 4 − x 2 = 4. 2− x

Ç)

To evaluate the limits of the type lim[ f ( x) g ( x)] , when lim f ( x) = ∞ and, x a x →a 0 lim g ( x) = ∞ [i.e., (∞ − ∞) form], we reduce the expression in the form of or by x →a 0 taking LCM or by rearranging the terms and then appling L’Hospital’s rule. Example 1: Prove that lim (cosh -1 x - log x ) = log 2 . x →∞

Solution: Let l = lim (cosh −1 x − log x) x →∞

)

(

[∞ − ∞ ]

= lim ⎡log x + x 2 − 1 − log x ⎤ ⎢ x →∞ ⎣ ⎦⎥ ⎛ x + x2 − 1 ⎞ = lim log ⎜ ⎟ x →∞ ⎜ ⎟ x ⎝ ⎠ ⎛ 1 ⎞ = lim log ⎜⎜1 + 1 − 2 ⎟⎟ x →∞ x ⎠ ⎝

(

)

= log 1 + 1 − 0 = log 2

Differential Calculus I

2.141

⎛ 1 1 x ⎞ Example 2: Prove that lim ⎜ =- . x →1 ⎝ log x 2 x − 1 ⎟⎠ ⎛ 1 x ⎞ Solution: Let l = lim ⎜ − x →1 ⎝ log x x − 1⎟⎠ ⎡ x − 1 − x log x ⎤ = lim ⎢ ⎥ x →1 ⎣ ( x − 1) log x ⎦ 1 1 − x ⋅ − log x x = lim x →1 1 ( x − 1) ⋅ + log x x = lim x →1

= lim x →1

− log x 1 1 − + log x x −

1 x

1 1 + x2 x

=−

[∞ − ∞ ] ⎡0⎤ ⎢⎣ 0 ⎥⎦

[Applying L’Hospital’s rule] ⎡0⎤ ⎢0⎥ ⎣ ⎦

1 2

[Applying L’Hospital’s rule]

x⎞ ⎛a Example 3: Prove that lim ⎜ - cot ⎟ = 0. x→0 ⎝ x a⎠ x⎞ ⎛a Solution: Let l = lim ⎜ − cot ⎟ x →0 x a⎠ ⎝ Taking

[∞ − ∞]

x = y, when x → 0, y → 0 a

⎛1 ⎞ l = lim ⎜ − cot y ⎟ y→0 ⎝ y ⎠ ⎛1 1 ⎞ = lim ⎜ − y→0 ⎝ y tan y ⎟⎠

[∞ − ∞ ]

⎛ tan y − y ⎞ = lim ⎜ y → 0 ⎝ y tan y ⎟ ⎠

⎡0⎤ ⎢⎣ 0 ⎥⎦

⎛ tan y − y ⎞ 1 = lim ⎜ ⋅ lim 2 ⎟ y→0 ⎝ y → 0 ⎠ y ⎛ tan y ⎞ ⎝⎜ y ⎟⎠

= lim y →0

tan y − y ⋅1 y2

⎡0⎤ ⎢⎣ 0 ⎥⎦

⎡ tan y ⎤ = 1⎥ ⎢∵ lim y →0 y ⎣ ⎦

Engineering Mathematics

2.142

sec 2 y − 1 y→0 2y 2 sec y ⋅ sec y tan y = lim y →0 2 =0

⎡0⎤ ⎢⎣ 0 ⎥⎦

= lim

[Applying L’Hospital’s rule] [Applying L’Hospital’s rule]

⎡ 1 ⎤ o 1 Example 4: Prove that lim ⎢ ⎥= 4. ox x→0 2 x 1 + ( ) x e ⎣ ⎦ ⎡1 ⎤ 1 Solution: Let l = lim ⎢ − ⎥ x x →0 2 x x e 1 ( + ) ⎣ ⎦

[∞ − ∞ ]

ep x + 1 − 2 x → 0 2 x ( ep x + 1)

⎡0⎤ ⎢⎣ 0 ⎥⎦

= lim

p ep x x → 0 2 ⎡( ep x + 1) + x (p ep x )⎤ ⎣ ⎦

= lim =

p e0 p = 0 2 (e + 1) 4

⎤ 1 1⎞ ⎡ 1⎞ ⎛ ⎛ Example 5: Prove that lim ⎜ x + ⎟ ⎢ log ⎜ x + ⎟ - log x ⎥ = . x→∞ ⎝ ⎝ 2⎠ ⎣ 2⎠ ⎦ 2 1⎞ ⎡ ⎛ 1⎞ ⎤ ⎛ Solution: Let l = lim ⎜ x + ⎟ ⎢log ⎜ x + ⎟ − log x ⎥ x →∞ ⎝ ⎠ ⎝ ⎠ 2 ⎣ 2 ⎦

[∞ − ∞ ]

1⎞ ⎛ ⎜ x+ 2 ⎟ 1⎞ ⎛ = lim ⎜ x + ⎟ log ⎜ ⎟ x →∞ 2⎠ ⎝ ⎜⎜ x ⎟⎟ ⎝ ⎠ 1 ⎞ 1 ⎡ 1 ⎞⎤ ⎛ ⎛ = lim ⎢ x log ⎜1 + ⎟ + log ⎜1 + ⎟ ⎥ x →∞ 2 x x ⎠⎦ 2 2 ⎝ ⎝ ⎠ ⎣ 2x

1 1 ⎞ 1 1 ⎞ ⎛ ⎛ = lim log ⎜ 1 + ⎟ + lim log ⎜1 + ⎟ x →∞ 2 2 x →∞ ⎝ 2x ⎠ ⎝ 2x ⎠ =

1 1 log e + log 1 2 2

1 = . 2

ax ⎡ ⎤ 1 ⎞ ⎛ + 1 ⎢∵ lim ⎜ ⎟ = e⎥ x →∞ ⎝ ax ⎠ ⎥⎦ ⎢⎣

Differential Calculus I

2.143

⎛ a cot x b ⎞ 1 Example 6: If lim ⎜ + 2 ⎟ = , find a and b. x→0 ⎝ x x ⎠ 3 Solution:

1 ⎛ a cot x b ⎞ = lim ⎜ + 2⎟ x → 0 3 x ⎠ ⎝ x

b⎞ ⎛ a = lim ⎜ + 2⎟ x → 0 ⎝ x tan x x ⎠ ⎛ ax + b tan x ⎞ = lim ⎜ 2 ⎟ x→0 ⎝ x tan x ⎠ = lim x→0

⎡0⎤ ⎢⎣ 0 ⎥⎦

(ax + b tan x) ⎛ tan x ⎞ ( x 2 ⋅ x) ⎜ ⎝ x ⎟⎠

= lim

ax + b tan x ⎛ x ⎞ ⋅ lim ⎜ ⎟ 3 x → 0 x ⎝ tan x ⎠

= lim

ax + b tan x ⋅1 x3

x →0

x →0

⎡0⎤ ⎢0⎥ ⎣ ⎦

x ⎡ ⎤ = 1⎥ ⎢∵ lim x → 0 tan x ⎣ ⎦

⎛ a + b sec 2 x ⎞ = lim ⎜ ⎟ x →0 3x 2 ⎝ ⎠ =

[Applying L’Hospital’s rule]

a + b sec 0 a + b = 0 0

But limit is finite, therefore, numerator must be zero. a + b = 0, a = – b 1 −b + b sec 2 x = lim 3 x →0 3x 2

= lim x →0

... (1)

⎡0⎤ ⎢⎣ 0 ⎥⎦

b ⋅ 2 sec x sec x tan x 6x

[Applying L’Hospital’s rule]

b tan x ⎞ ⎛ ⎞ ⎛ = ⎜ lim sec 2 x ⎟ ⋅ ⎜ lim ⎟ x →0 3 x →0 x ⎠ ⎝ ⎠ ⎝ b tan x ⎤ ⎡ = 1⎥ = sec 0 ⋅1 ⎢⎣∵ xlim →0 x 3 ⎦ 1 b = , b =1 3 3 From Eq. (1),

a

b

1

Hence,

a = −1, b = 1.

Engineering Mathematics

2.144

Exercise 2.12 5. Prove that

1⎞ ⎛ 1. Prove that lim ⎜ cot x − ⎟ = 0. x →0 x⎠ ⎝

⎡ 1 ⎤ 1 1 lim ⎢ − =− . ⎥ x →0 x − a 1 2 log ( x + − a ) ⎦ ⎣ 6. Prove that ⎡ 1 ⎤ 1 1 lim ⎢ − ⎥ = − 2. x →3 x − 3 2 log ( ) x − ⎣ ⎦

2. Prove that ⎡ 1 ⎤ lim ⎢ − cot( x − a ) ⎥ = 0. x →a x − a ⎣ ⎦ 3. Prove that

⎡ 1 log (1 + x) ⎤ 1 7. Prove that lim ⎢ − ⎥⎦ = 2 . x →0 x x2 ⎣

lim (sec x − tan x) = 0. x→

2

4. Prove that

1 ⎞ 1 ⎛1 8. Prove that lim ⎜ − x ⎟= . x →0 x e −1 ⎠ 2 ⎝

2 x sec x ⎞ 2 ⎛ lim ⎜ tan x − ⎟= . x→ ⎝ ⎠ 2

2.10.7 Type 5 : 1Ç, Ç 0, 0 0 To evaluate the limits of the type lim[ f ( x)]g ( x ) which takes any one of the above x a form, we proceed as follows: l = lim [ f ( x)]g ( x )

Let

x →a

log l = lim [ g ( x) ⋅ log f ( x)] x→a

[if f ( x) > 0]

∞ × 0, i.e., type 3 form. 1

Example 1: Prove that lim ( a x + x ) x = ae . x

0

1

Solution: Let l = lim (a x + x) x x →0

[1∞ ]

1 log l = lim ⋅ log (a x + x) x →0 x log (a x + x) ⎡0⎤ = lim ⎢⎣ 0 ⎥⎦ x →0 x 1 (a x log a + 1) x + x a = lim x →0 1 a 0 log a + 1 log e a + log e e = = a0 + 0 1

[Applying L’Hospital’s rule]

Differential Calculus I

2.145

log l = log ae

Hence,

l = ae 1

⎛ ax + bx ⎞ x Example 2: Prove that lim ⎜ = ab . x→0 ⎝ 2 ⎟⎠ 1

⎛ ax + bx ⎞ x Solution: Let l = lim ⎜ x→0 ⎝ 2 ⎟⎠

[1∞ ]

⎛ ax + bx ⎞ 1 log l = lim log ⎜ x→0 x ⎝ 2 ⎟⎠ ⎛ ax + bx ⎞ log ⎜ ⎝ 2 ⎟⎠ = lim x→0 x

⎡0⎤ ⎢⎣ 0 ⎦⎥

x x ⎛ 2 ⎞ (a log a + b log b) = lim ⎜ x ⋅ ⎟ x x→0 ⎝ a + b ⎠ 2

[Applying L’Hospital’s rule]

0 0 ⎛ 2 ⎞ (a log a + b log b) =⎜ 0 ⎟ ⎝ a + b0 ⎠ 2

=

1 ⋅ log ab 2 1

log l = log(ab) 2

Hence,

l = ab 1 1 1 ⎛ 1x x x x a b c d + + + Example 3: Prove that lim ⎜ x→∞ ⎜ 4 ⎝ 1 1 1 ⎛ 1x ⎞ a + bx + cx + d x ⎟ ⎜ Solution: Let l = lim x →∞ ⎜ ⎟ 4 ⎝ ⎠

Taking

1 = y, x

x

x

,y 1

⎛ ay + by + cy + d y ⎞ y l = lim ⎜ ⎟ y →0 4 ⎝ ⎠

x

⎞ 1 ⎟ = ( abcd ) 4 . ⎟ ⎠

[1 ]

Engineering Mathematics

2.146

⎛ ay + by + cy + d y ⎞ 1 log l = lim log ⎜ ⎟ y →0 y 4 ⎝ ⎠ y y y y ⎛ a +b +c +d ⎞ log ⎜ ⎟ 4 ⎠ ⎝ = lim y →0 y 4 ⎛ = lim ⎜ y y →0 a + b y + c y + d y ⎝

⎡0⎤ ⎢⎣ 0 ⎥⎦

y y y y ⎞ ⎛ a log a + b log b + c log c + d log d ⎞ ⎟ ⎟⎜ 4 ⎠⎝ ⎠

[Applying L’Hospital’s rule] log a + log b + log c + log d 4 1 = log (abcd ) 4 =

1

log l = log(abcd ) 4

Hence,

1

l = (abcd ) 4 x

⎛ ax + 1 ⎞ a Example 4: Prove that lim ⎜ ⎟ =e . x → ∞ ⎝ ax - 1 ⎠ ⎛ ax + 1⎞ l = lim ⎜ ⎟ x →∞ ⎝ ax − 1⎠

Solution: Let

2

x

1⎞ ⎛ 1+ ⎜ ⎟ ax = lim ⎜ x →∞ 1⎟ ⎜1− ⎟ ⎝ ax ⎠

x

[1∞ ]

1 ⎞ ⎛ ⎜ 1 + ax ⎟ log l = lim x log ⎜ ⎟ x →∞ ⎜ 1− 1 ⎟ ⎝ ax ⎠ 1 ⎞ 1 ⎞⎤ ax ⎡ ⎛ ⎛ = lim ⎢log ⎜1 + ⎟ − log ⎜1 − ⎟ ⎥ x →∞ a ax ax ⎝ ⎠ ⎝ ⎠⎦ ⎣ 1⎡ ⎛ 1 ⎞ 1 ⎞ ⎛ ⎢log ⎜1 + ⎟ + log ⎜1 − ⎟ x →∞ a ax ax ⎠ ⎝ ⎠ ⎢⎣ ⎝ ax

= lim =

1 1 2 (log e + log e) = (1 + 1) = a a a

− ax

⎤ ⎥ ⎥⎦

ax ⎡ ⎤ 1 ⎞ ⎛ ∵ lim + 1 ⎢ x →∞ ⎜ ⎟ = e⎥ ⎝ ax ⎠ ⎢⎣ ⎥⎦

Differential Calculus I

log l =

Hence,

2.147

2 a 2

l = ea x⎞ ⎛ Example 5: Prove that lim ⎜ 2 - ⎟ x→a ⎝ a⎠ x Solution: Let l = lim ⎛⎜ 2 − ⎞⎟ x→a a⎠ ⎝

tan

tan

ox 2a

2

= eo .

x 2a

[1∞ ]

x⎞ ⎛ p x⎞ ⎛ log l = lim tan ⎜ ⎟ log ⎜ 2 − ⎟ x→a ⎝ 2a ⎠ ⎝ a⎠ x⎞ ⎛ log ⎜ 2 − ⎟ ⎝ a⎠ = lim x→a ⎛ p x⎞ cot ⎜ ⎟ ⎝ 2a ⎠ = lim x→a

=

1 x⎞ ⎛ ⎜⎝ 2 − ⎟⎠ a

⎡0⎤ ⎢⎣ 0 ⎥⎦

1 ⎛ 1⎞ ⎜⎝ − ⎟⎠ a ⎛ 2 p x⎞ ⎛ p ⎞ ⎜⎝ − cosec ⎟⎜ ⎟ 2a ⎠ ⎝ 2a ⎠

[Applying L’Hospital’s rule]

2 p log l =

Hence,

2 2

l=e 1

⎛ tan x ⎞ x2 3 Example 6: Prove that lim ⎜ ⎟ =e . x→0 ⎝ x ⎠ 1

1

⎛ tan x ⎞ x 2 Solution: Let l = lim ⎜ ⎟ x→0 ⎝ x ⎠ 1 ⎛ tan x ⎞ log ⎜ ⎟ 2 x ⎝ x ⎠ ⎛ tan x ⎞ log ⎜ ⎟ ⎝ x ⎠ = lim 2 x →0 x

[1∞ ]

tan x ⎡ ⎤ = 1⎥ ⎢∵ lim ⎣ x→0 x ⎦

log l = lim x →0

⎡0⎤ ⎢0⎥ ⎣ ⎦

x ⎛ x sec 2 x − tan x ⎞ 1 ⎜ ⎟⋅ x → 0 tan x x2 ⎝ ⎠ 2x

= lim

[Applying L’Hospital’s rule]

Engineering Mathematics

2.148

x sec 2 x − tan x 2 x3

x ⎡0⎤ ⎡ ⎤ = 1⎥ ⎢⎣ 0 ⎥⎦ ⎢⎣∵ lim x →0 tan x ⎦ 2 2 2 sec x + x ⋅ 2 sec x tan x − sec x = lim [Applying L’Hospital’s rule] x →0 6 x2 sec 2 x tan x 1 = lim ⋅ = x →0 x 3 3 = lim x →0

1 3

log l =

Hence,

1

l = e3 . 1

1 ⎛ sinh x ⎞ x2 6 . Example 7: Prove that lim ⎜ = e ⎟ x→0 ⎝ x ⎠ 1

sinh x ⎞ x 2 Solution: Let l = lim ⎛⎜ ⎟ x→0 ⎝ x ⎠ log l = lim x→0

[1 ]

1 ⎛ sinh x ⎞ log ⎜ ⎝ x ⎟⎠ x2

⎛ sinh x ⎞ log ⎜ ⎝ x ⎟⎠ = lim x→0 x2 = lim x→0

⎡0⎤ ⎢0⎥ ⎣ ⎦

x ⎛ x cosh x − sinh x ⎞ 1 ⎜ ⎟⎠ ⋅ 2x sinh x ⎝ x2

x cosh x − sinh x ⎡ 0 ⎤ ⎢⎣ 0 ⎥⎦ 2 x3 x sinh x + cosh x − cosh x = lim x→0 6x2

[Applying L’Hospital’s rule] x ⎤ ⎡ = 1⎥ ⎢⎣∵ lim x → 0 sinh x ⎦

= lim x→0

[Applying L’Hospital’s rule] sinh x ⎤ ⎡ = 1⎥ ⎢⎣∵ lim x →0 x ⎦

1 sinh x 1 = lim ⋅ = x→0 6 x 6 Hence,

log l =

sinh x ⎤ ⎡ = 1⎥ ⎢⎣∵lim x→0 x ⎦

1 6 1

l = e6 ⎛1⎞ Example 8: Prove that lim ⎜ ⎟ x→0 ⎝ x ⎠ ⎛ 1⎞ Solution: Let l = lim ⎜ ⎟ x→0 ⎝ x ⎠

1 - cos x

=1.

1− cos x

[∞ 0 ]

Differential Calculus I

2.149

⎛1⎞ log l = lim(1 − cos x ) log ⎜ ⎟ x→0 ⎝x⎠ x⎞ ⎛ = lim ⎜ 2 sin 2 ⎟ ( − log x ) x→0 ⎝ 2⎠ 2

= lim

x⎞ ⎛x⎞ ⎛ 2 ⎜ sin ⎟ ⎜ ⎟ ⎝ 2⎠ ⎝2⎠

x→0

⎛x⎞ ⎜⎝ ⎟⎠ 2

2

2

( − log x )

⎡ x⎞ ⎤ ⎛ sin ⎢ ⎜ ⎟ ⎥ 2 = 1⎥ ⎢∵ lim ⎜ x→0 x ⎟ ⎥ ⎢ ⎜ ⎟ ⎝ 2 ⎠ ⎥⎦ ⎢⎣ ⎡∞⎤ ⎢∞⎥ ⎣ ⎦

x 2 ( − log x ) = lim x→0 2 = lim x→0

1 ( − log x ) 2 ⎛1⎞ ⎜⎝ 2 ⎟⎠ x

⎛ −1 ⎞ ⎜ ⎟ 1 = lim ⎜ x ⎟ 2 x →0 ⎜ −2 ⎟ ⎜ 3 ⎟ ⎝x ⎠ = Hence,

[Applying L’Hospital’s rule]

⎛ x2 ⎞ 1 lim ⎜ ⎟ = 0 2 x →0 ⎝ 2 ⎠

log l = 0 l = e0 = 1 sinh -1 x

Example 9: Prove that lim e

cosh -1 x

x→∞

= e.

sinh −1 x

Solution: Let l = lim e

cosh −1 x

x →∞

(

= lim e sinh x →∞

−1

1 x cosh −1 x

)

[∞ 0 ]

sinh −1 x ⋅ log e x →∞ cosh −1 x

log l = lim = lim

x →∞

log ( x + x 2 + 1) log ( x + x − 1)) 2

⎡∞⎤ ⎢∞⎥ ⎣ ⎦

Engineering Mathematics

2.150

⎛ ⎞ 1 ⋅ 2x ⎟ ⎜1 + 2 ⎠ x + x +1 ⎝ 2 x +1 = lim x →∞ ⎛ ⎞ 1 1 ⋅ 2x ⎟ 1+ ⎜ ⎠ x + x2 −1 ⎝ 2 x2 −1 1

2

[Applying L’Hospital’s rule]

1 2 x = lim =1 = lim x →∞ x2 + 1 x→∞ 1 + 1 x2 1−

x2 −1

log l = 1

Hence,

l = e1 = e . 1

⎛ 1⎞x Example 10: Prove that lim ⎜ ⎟ = 1 . x→Ç ⎝ x ⎠ 1

x Solution: Let l = lim ⎛⎜ 1 ⎞⎟ x →∞ ⎝ x ⎠

[0 0 ]

1 − log x ⎛ 1⎞ log l = lim log ⎜ ⎟ = lim x →∞ x ⎝ x ⎠ x →∞ x

Hence,

1 = − lim x x →∞ 1 0 log l = 0

⎡∞⎤ ⎢⎣ ∞ ⎥⎦

[Applying L’Hospital’s rule]

l = e0 = 1 1

Example 11: Prove that lim(1 - x 2 ) log (1- x ) = e . x

1

1

Solution: Let l = lim(1 − x 2 ) log (1− x ) x →1

log l = lim x →1

[0 0 ]

1 log (1 − x 2 ) log(1 − x)

−2 x (1 − x 2 ) = lim x →1 1 ( −1) (1 − x) = lim x →1

2 x(1 − x) 2x = lim =1 x → 1 1+ x (1 − x)(1 + x)

⎡∞⎤ ⎢∞⎥ ⎣ ⎦

[Applying L’Hospital’s rule]

Differential Calculus I Hence,

2.151

log l = 1 l=e

Example 12: Prove that lim x→0

ex x ⎡⎛ 1⎞ ⎤ ⎢ ⎜1 + ⎟ ⎥ x ⎠ ⎥⎦ ⎢⎣ ⎝

x ⎡ 1 ⎤ Solution: Let l = lim ⎢⎛⎜1 + ⎞⎟ ⎥ x →0 ⎢⎣⎝ x ⎠ ⎥⎦

⎛ 1⎞ = lim ⎜1 + ⎟ x→0 ⎝ x⎠

x

=1 .

x

x2

(∞ 0 )

⎛ 1⎞ log l = lim x 2 log ⎜1 + ⎟ x→0 ⎝ x⎠ ⎛ 1⎞ log ⎜1 + ⎟ ⎝ x⎠ = lim x→0 1 x2 1 ⎛ 1⎞ − 1 ⎜⎝ x 2 ⎟⎠ 1+ x = lim x→0 2 − 3 x = lim x→0

x ⎛ 1⎞ 2 ⎜1 + ⎟ ⎝ x⎠

⎡∞⎤ ⎢⎣ ∞ ⎥⎦

[Applying L’Hospital’s rule]

x2 =0 x → 0 2 ( x + 1)

= lim

log l = 0

Hence,

l = e0 = 1 x

⎡⎛ 1 ⎞ x ⎤ lim ⎢⎜1 + ⎟ ⎥ = 1 x→0 ⎝ x ⎠ ⎥⎦ ⎢⎣

Hence,

lim x→0

ex ⎡⎛ 1 ⎞ x ⎤ ⎢⎜1 + ⎟ ⎥ ⎢⎣⎝ x ⎠ ⎥⎦

x

=

e0 1 = = 1. 1 1

Engineering Mathematics

2.152

1 x sin x 0 x log x

Example 13: Prove that lim x

Solution: Let l1 = lim x sin x x→0

1.

[0 0 ]

log l1 = lim sin x ⋅ log x = lim x→0

x→0

⎡∞⎤ ⎢∞⎥ ⎣ ⎦

log x cosec x

1 x = lim x → 0 − cosec x cot x

[Applying L’Hospital’s rule] 2

= lim − x→0

x sin 2 x ⎛ sin x ⎞ = − lim ⎜ =0 ⎟ ⋅ x→0 ⎝ x ⎠ x cos x cos x

log l1 = 0, l1 = e 0 = 1, ∴ lim x sin x = 1 x→0

Let l2 = lim x log x

[0 × ∞ ]

x→0

log x 1 x 1 = lim x x →0 1 − 2 x = lim(− x) = 0 = lim x →0

sin x ⎡ ⎤ = 1⎥ ⎢∵ lim ⎣ x →0 x ⎦ … (1)

⎡∞⎤ ⎢∞⎥ ⎣ ⎦

[Applying L’Hospital’s rule] ... (2)

x →0

1 − x sin x x → 0 x log x

Let l = lim

0 0

[Using Eqs (1) and (2)]

1 − esin x log x x →0 x log x

= lim 1 − x sin x lim = lim x → 0 x log x x →0

⎛ sin x ⎞ + cos x ⋅ log x ⎟ − esin x log x ⎜ x ⎝ ⎠ 1 + log x

⎤ ⎡⎛ sin x ⎞ 1 − x sin x ⎢⎜ + cos x ⎥ ⎟⎠ ⋅ ⎝ log x ⎦ ⎣ x = lim x→0 1 +1 log x

[Applying L’Hospital’s rule]

... (3)

[Dividing numerator and denominator by log x] ⎛ 1 ⎞ 1⎜1 ⋅ + cos 0 ⎟ ∞ ⎠ = −1 =− ⎝ 1 +1 ∞

sin x ⎤ ⎡ = 1⎥ ⎢ Using Eq. (1) and lim x x →0 ⎣ ⎦

Differential Calculus I

2.153

Exercise 2.13 1. Prove that

17. Prove that lim(cos x)cot

1 x

x 2

x n

⎛ 1⎞ 18. Prove that lim ⎜ ⎟ x→0 ⎝ x ⎠

1 n

= (a1a2 ...an ) . 2. Prove that

.

= 1.

= 24 . 1 2x

cot ( x − a )

x⎞ ⎛ 20. Prove that lim ⎜ 2 − ⎟ x→ a ⎝ a⎠



1

= e a.

21. Prove that lim (tan x)cos 2 x = 1.

1 2

x

=e .

4

22. Prove that lim (sec x)cot x = 1.

1 = . e

x

2

⎛ 1⎞ 23. Prove that lim ⎜ ⎟ x→0 ⎝ x ⎠

5. Prove that lim (sin x) tan x = 1. x

e

tan x

4x

x→0

x →1

1

=

x

3. Prove that lim (1 + sin x) 4. Prove that lim ( x)

x

⎛ 2⎞ 19. Prove that lim ⎜1 + ⎟ = e 2 . x →∞ ⎝ x⎠

1 1 1 ⎛ 1x ⎞ 1 + 2 x + 3x + 4 x ⎟ ⎜ lim x →∞ ⎜ ⎟ 4 ⎝ ⎠

1 1− x

2

x→0

⎛ a + a + ... + a ⎞ lim ⎜ ⎟ . x →0 n ⎝ ⎠ x 1

2 sin x

= 1.

2 1

24. Prove that lim (sin x) tan x = 1.

1

− ⎛ sin x ⎞ x2 6 6. Prove that lim ⎜ ⎟ =e . x→0 ⎝ x ⎠

x→0

1

25. Prove that lim (1 − x n ) log(1− x ) = e . x →1

x

⎛ a⎞ 7. Prove that lim ⎜1 + ⎟ = e a . x →∞ ⎝ x⎠

26. Prove that 1

⎡1 ⎛ a x ⎞ ⎤ x−a = 1. lim ⎢ ⎜ + ⎥ x→a 2 a ⎟⎠ ⎦⎥ ⎢⎣ ⎝ x

x

⎛ x + 1⎞ 2 8. Prove that lim ⎜ ⎟ =e . x →∞ ⎝ x − 1⎠ x

⎛ 2 x + 1⎞ 9. Prove that lim ⎜ ⎟ = e. x →∞ ⎝ 2 x − 1⎠

1⎞ ⎛ 27. Prove that lim ⎜ x sin ⎟ x →∞ ⎝ x⎠

10. Prove that lim (1 + sin x)cosec x = e. x→0

11. Prove that lim (1 + sin x)

cot x

x→0

28. Prove that lim x

= e.

⎛ x⎞ tan ⎜ ⎟ ⎝ 2 ⎠

x →0

x→0

⎡ ⎛ ⎞⎤ lim ⎢sin 2 ⎜ ⎟⎥ x →0 ⎝ 2 − ax ⎠ ⎦ ⎣

1 x

1 13. Prove that lim (1 − tan x) = . x→0 e 1

1

2

x→0

e

x→0

bx

= e.

⎛ ⎞ sec2 ⎜ ⎟ ⎝ 2 − bx ⎠

a2 b2

x →0

⎛ 3⎞ ⎜ 2 ⎟⎠

31. Prove that lim (cos 2 x)⎝ x x→0

=e

=e



30. Prove that lim (e3 x − 5 x) x = e −2 .

⎛ sin x ⎞ 15. Prove that lim ⎜ ⎟ = 0. x→0 ⎝ x ⎠ 2

1

1

.

1 x

16. Prove that lim (cos ax)cosec



= e 6.

29. Prove that

12. Prove that lim (1 + tan x)cot x = e .

14. Prove that lim (cos x) x =

x2



= e −6 .

a2 2 b2

.

sin x 32. Prove that lim (cot x) = 1 . x →0

.

Engineering Mathematics

2.154

2.10.8 Type 6 : Using Expansion In some cases, it is difficult to differentiate the numerator or denominator, or in some cases, power of x in the denominator is very large. In such cases, we use expansion of the function to find the limit.

sin -1 x - x 1 = . 0 6 x3

Example 1: Prove that lim x

Solution: Let l = lim x→0

sin −1 x − x x3

⎛ ⎞ x3 3 5 x x + ...⎟ − x + + ⎜⎝ 6 40 ⎠ = lim 3 x→0 x ⎛1 3 2 ⎞ 1 = lim ⎜ + x + ...⎟ = . x→0 ⎝ 6 ⎠ 6 40

sin x - tan -1 x . 0 x 2 log (1 + x )

Example 2: Evaluate lim x

sin x − tan −1 x x → 0 x 2 log (1 + x )

Solution: Let l = lim

⎛ ⎞ ⎛ ⎞ x3 x5 x3 x5 ... x x − + − − − + − …⎟ ⎜ ⎟ ⎜ 3! 5! 3 5 ⎠ ⎝ ⎠ = lim ⎝ 4 2 3 x →0 ⎛ x ⎞ x x x 2 ⎜1 − + − + ... ⎟ 4 2 3 ⎝ ⎠ ⎛ 1 23 2 ⎞ x3 ⎜ − x + ... ⎟ 6 120 ⎝ ⎠ =1 = lim . 2 3 x →0 ⎞ 6 x x x 3⎛ x ⎜1 − + − + ... ⎟ ⎝ 2 3 4 ⎠ sin x sin -1 x - x 2 . 0 x6

Example 3: Evaluate lim x

Solution: Let

⎛ ⎞⎛ ⎞ x3 x5 x3 3 5 x − + − ... x + + x + ...⎟ − x 2 ⎜⎝ ⎟ ⎜ 3! 5 ! 6 40 ⎠⎝ ⎠ l = lim 6 x →0 x ⎛ 2 x 4 3 6 x 4 x 6 x8 x 6 ⎞ x8 x x + + − − − + + + ... ⎟ − x 2 ⎜ 6 40 6 36 80 120 720 ⎠ = lim ⎝ x →0 x6

Differential Calculus I

2.155

x6 + Higher powers of x = lim 18 x →0 x6 1 = . 18 tan x tan −1 x − x 2

Example 4: Evaluate lim

x6

x →0

.

⎛ ⎞⎛ ⎞ x3 2 5 x3 x5 x 7 x + + x + ... x − + − + ... ⎟ − x 2 ⎜ ⎟⎜ 3 15 3 5 7 ⎠⎝ ⎠ Solution: Let l = lim ⎝ 6 x →0 x ⎛ 2 x 4 x 6 x8 x 4 x 6 x8 2 6 2 8 ⎞ 2 ⎜ x − + − + − + + x − x + ... ⎟ − x 3 5 7 3 9 15 15 45 ⎠ = lim ⎝ 6 x →0 x ⎛1 1 2 ⎞ x 6 ⎜ − + ⎟ + Higher powers of x more than 6 5 9 15 ⎠ = lim ⎝ x →0 x6 2 = . 9 Example 5: Prove that lim x

Solution: Let

0

l = lim x →0

= lim x→0

e x sin x - x - x 2 2

x + x log (1 - x )

=-

2 . 3

e x sin x − x − x 2 x 2 + x log (1 − x) ⎛ ⎞⎛ ⎞ x 2 x3 x3 + + + + ... − + ... ⎟ − x − x 2 1 x x ⎜⎝ ⎟ ⎜ 2 ! 3! 3! ⎠⎝ ⎠ ⎛ ⎞ x 2 x3 x 2 + x ⎜ − x − − − ... ⎟ 2 3 ⎝ ⎠

⎛ ⎞ x3 x 4 x3 x5 x4 x6 2 2 ⎜⎝ x − 3! + x − 3! + 2 ! − 2 !3! + 3 − 3!3! + ... ⎟⎠ − x − x = lim x→0 x3 x 4 x 2 − x 2 − − − ... 2 3

x3 x5 − + ... = lim 3 3 124 x →0 x x − − − ... 2 3

Engineering Mathematics

2.156

1 x2 − + ... = lim 3 12 x →0 1 x − − − ... 2 3 1 2 = 3 =− . 1 3 − 2 1

(1 + x ) x - e e Example 6: Prove that lim =- . x 0 x 2 1

(1 + x) x − e l = lim x→0 x

Solution: Let

1

= lim

ex

log (1+ x )

x→0

= lim

x e

⎞ 1 ⎛ x 2 x3 ⎜ x − + − ...⎟ 2 3 x⎝ ⎠

x→0

= lim

−e

x e

⎛ x x2 ⎞ ⎜⎜1− + −... ⎟⎟ ⎝ 2 3 ⎠

x →0

= lim

−e

−e

x ee

⎛ x x2 ⎞ ⎜⎜ − + −... ⎟⎟ ⎝ 2 3 ⎠

x →0

−e

x

2 ⎤ ⎡ ⎛ x x2 ⎞ 1⎛ x ⎞ e ⎢1 + ⎜ − + − ... ⎟ + ⎜ − + ... ⎟ + ...⎥ − e ⎠ ⎢ ⎝ 2 3 ⎠ 2⎝ 2 ⎦⎥ = lim ⎣ x →0 x ⎛ 1 x ⎞ = lim e ⎜ − + − ... ⎟ x →0 2 3 ⎝ ⎠ e =− . 2 1

(1 + x ) x - e + Example 7: Prove that lim x

x2

0

1

Solution: Let l = lim x→0

(1 + x) x − e + x2

ex 2

ex 2 = 11e . 24

Differential Calculus I

1

ex = lim

log (1+ x )

−e+

2.157

ex 2

x2

x→0

e

⎞ 1 ⎛ x 2 x3 x 4 ⎜ x − + − + ...⎟ 2 3 4 x⎝ ⎠

= lim

−e+

ex 2

x2

x→0

e

⎛ x x 2 x3 ⎞ ⎜1− + − + ...⎟ ⎝ 2 3 4 ⎠

= lim

−e+

ex 2

−e+

ex 2

x2

x→0

ee = lim x→0

⎛ x x 2 x3 ⎞ ⎜ − + − + ...⎟ ⎝ 2 3 4 ⎠

x2

⎡ ⎛ x x 2 x3 ⎞ 1 ⎛ x x2 − ⎢1 + ⎜ − + − + ... ⎟ + ⎜ − + ⎢⎣ ⎝ 2 3 4 ⎠ 2! ⎝ 2 3 = lim x →0 x2

⎛ ⎞ ex ex 2 ex 2 ex 3 ex 3 ex e − + + − − + ... ⎟ − e + ⎜ 2 3 8 4 6 2 ⎠ = lim ⎝ 2 x →0 x 1e 5e ⎛ 11 ⎞ 11e − x + ... ⎟ = = lim ⎜ x →0 ⎝ 24 12 ⎠ 24 1

1 ⎡ ⎛ cosh x - 1 ⎞ ⎤ x2 12 e = Example 8: Prove that lim ⎢ 2 ⎜ . ⎟⎠ ⎥ x→0 x2 ⎦ ⎣ ⎝ 1

⎡ ⎛ cosh x − 1 ⎞ ⎤ x 2 Solution: Let l = lim ⎢ 2 ⎜ ⎟⎠ ⎥ x→0 x2 ⎦ ⎣ ⎝ log l = lim x→0

1 ⎡ ⎛ cosh x − 1⎞ ⎤ log ⎢ 2 ⎜ ⎟⎠ ⎥ 2 x x2 ⎣ ⎝ ⎦

⎡ ⎧ ⎛ x2 x4 x6 ⎞ ⎫⎤ ⎢ ⎪ ⎜1 + + + + ...⎟ − 1⎪ ⎥ 2!! 4 ! 6 ! ⎠ ⎪⎥ 1 ⎪⎝ = lim 2 log ⎢ 2 ⎨ ⎬⎥ 2 ⎢ x→0 x x ⎪⎥ ⎢ ⎪ ⎪ ⎪⎭ ⎥ ⎣⎢ ⎩ ⎦ ⎡ ⎧ x2 x4 ⎫⎤ x6 + + + ... ⎪⎥ ⎢ ⎪⎪ 1 ⎪ = lim 2 ⋅ log ⎢ 2 ⎨ 2 24 2 720 ⎬⎥ x →0 x x ⎢ ⎪ ⎪⎥ ⎢⎣ ⎪⎩ ⎪⎭⎥⎦

2

⎞ ⎟ + ⎠

⎤ ex ⎥−e+ 2 ⎦⎥

Engineering Mathematics

2.158

= lim x →0

⎡ ⎛ x2 x4 ⎞⎤ 1 ⋅ log ⎢1 + ⎜ + + ... ⎟ ⎥ 2 x ⎠⎦ ⎣ ⎝ 12 360

2 ⎤ ⎞ 1 ⎛ x2 x4 ⎞ 1 ⎡⎛ x 2 x 4 = lim 2 ⋅ ⎢⎜ + + ... ⎟ − ⎜ + + ... ⎟ + ...⎥ x →0 x ⎢⎣⎝ 12 360 ⎥⎦ ⎠ 2 ⎝ 12 360 ⎠

⎛1 ⎞ x2 x2 = lim ⎜ + − + ... ⎟ x → 0 12 360 288 ⎝ ⎠ 1 = 12

Hence,

log l =

1 12

1

l = e12 . 2x - 1

Example 9: Prove that lim x

= 2 log 2.

1

0

(1 + x ) 2 - 1 Solution: Let l = lim x →0

2x − 1 1

(1 + x) 2 − 1

= lim x →0

e x log 2 − 1 1

(1 + x) 2 − 1

⎡ ⎤ x2 2 ⎢1 + x log 2 + 2 ! (log 2) + ...⎥ − 1 ⎦ = lim ⎣ x →0 ⎡ 1⎛1 ⎞ ⎤ ⎜ − 1⎟ ⎢ 1 ⎥ 2 2 ⎠ 2 ⎢1 + x + ⎝ x + ...⎥ − 1 2! ⎢ 2 ⎥ ⎢⎣ ⎥⎦ x (log 2) 2 + ... 2 ! = lim x →0 1 1⎛ 1⎞ x + ⎜ − ⎟ + ... 2 2 ⎝ 2 ⎠ 2! log 2 = = 2 log 2. 1 2 log 2 +

e x e sin x = 1. 0 x sin x

Example 10: Prove that lim x

e x − esin x x → 0 x − sin x

Solution: Let l = lim

Differential Calculus I

e −e x

sin x

= e −e x

2.159

⎛ x3 x5 ⎞ ⎜ x − + − ...⎟ 3! 5 ! ⎠ ⎝ ⎞ ⎛ x3 x5 ⎜ − + − ...⎟ x ⎝ 3! 5 ! ⎠

= e −e e x

= ex − exez

where z = −

x3 x5 + − ... 3! 5!

⎡ ⎛ ⎞⎤ z2 = e x ⎢1 − ⎜1 + z + + ... ⎟ ⎥ 2! ⎠⎦ ⎣ ⎝ 2 ⎛ ⎞ z = e x ⎜ − z − − ... ⎟ 2 ! ⎝ ⎠ ⎛ ⎞ ⎛ x3 x5 ⎞ x3 x5 x − sin x = x − ⎜ x − + − ...⎟ = ⎜ − + ...⎟ = − z 3! 5! ⎝ ⎠ ⎝ 3! 5! ⎠

⎛ ⎞ z2 −e x ⎜ z + + ... ⎟ 2! ⎝ ⎠ l = lim x→0 −z z ⎛ ⎞ = lim e x ⎜1 + + ... ⎟ x →0 ⎝ 2! ⎠ ⎡∵ lim z = 0 ⎤ ⎢⎣ x → 0 ⎥⎦

= e0 = 1 Example 11: Prove that lim x

Solution: Let

l = lim x→0

0

x sin(sin x ) - sin 2 x 1 = . 18 x6

x sin (sin x) − sin 2 x x6

3

sin (sin x) = sin x −

sin x sin 5 x + − ... 3! 5! 3

5

⎞ ⎞ ⎛ ⎞ 1⎛ x3 x5 1⎛ x3 x5 x3 x5 = ⎜ x − + − ...⎟ − ⎜ x − + − ...⎟ + ⎜ x + + − ...⎟ − ... 3! 5! 5! ⎝ 3! 5 ! 3! 5! ⎠ ⎠ ⎝ ⎠ 3! ⎝ ⎛ ⎞ 1⎛ ⎞ x3 x5 x3 1 = ⎜x− + − ...⎟ − ⎜ x 3 − 3 x 2 ⋅ + ...⎟ + ( x 5 − ...) 6 120 6 ⎝ ⎠ 6⎝ ⎠ 120 = x−

x3 1 5 + x − ... 3 10

x sin (sin x) = x 2 −

x4 x6 + − ... 3 10

⎛ ⎞ x3 x5 sin 2 x = ⎜ x − + − ... ⎟ 3 5 ! ! ⎝ ⎠

2

Engineering Mathematics

2.160

x6 x3 x5 − 2x + 2x ⋅ + ... 36 6 120 x4 2 6 = x2 − + x − ... 3 45 = x2 +

x6 2 6 − x − ... 10 45 1 = x 6 − ... 18

x sin (sin x) − sin 2 x =

1 6 x − Higher powers of x 1 18 l = lim = ⋅ x→0 18 x6

Hence,

Example 12: Find a, b, c if lim x

Solution: 1 = lim x→0

0

x ( a + b cos x ) - c sin x = 1. x5

x (a + b cos x) − c sin x x5

⎡ ⎛ x2 x4 x6 ⎞⎤ ⎛ ⎞ x3 x5 x 7 x ⎢ a + b ⎜1 − + − + ... ⎟ ⎥ − c ⎜ x − + − + ... ⎟ ! ! ! ! ! 7 ! 2 4 6 3 5 ⎝ ⎠⎦ ⎝ ⎠ = lim ⎣ x →0 x5 ⎛ b c⎞ ⎛ b c⎞ ⎛b c⎞ (a + b − c) x + x 3 ⎜ − + ⎟ + x 5 ⎜ − ⎟ + x 7 ⎜ − + ⎟ + ... 2 6 4 5 ! ! ⎠ ⎝ 6! 7 ! ⎠ ⎝ ⎠ ⎝ = lim 5 x →0 x b c⎞ 4⎛ b c ⎞ 6⎛ b c ⎞ 2⎛ (a + b − c) + x ⎜ − + ⎟ + x ⎜ − ⎟ + x ⎜ − + ⎟ + ... ⎝ 4 ! 5! ⎠ ⎝ 6! 7 ! ⎠ ⎝ 2 6⎠ = lim x →0 x4 But limit is given as 1. b c b c a + b − c = 0, − + = 0 , − = 1, 2 6 24 120 a + b − c = 0, − 3b + c = 0, 5b − c = 120 ⋅ Solving all the equations, we get a = 120, b = 60, c = 180. Exercise 2.13 1. Prove that lim

x − sin x 1 = . 6 tan 3 x

2. Prove that lim

tan x − x 1 = . x 2 tan x 3

x→0

x→0

tan 2 x − x 2 2 = . x → 0 x 2 tan 2 x 3

3. Prove that lim 4. Prove that

sin x − x + lim x→0

x5

x3 6 = 1 . 120

Differential Calculus I

2 sinh x − 2 x 1 = . x→0 3 x 2 sin x

10. Prove that

5. Prove that lim

lim log (1 + x 3 ) = 1. x→0 sin 3 x

6. Prove that lim 7. Prove that lim x→0

tanh x − 2 sin x + x 7 = . 60 x5

e x − e x cos x 8. Prove that lim = 3. x → 0 x − sin x

2.161

(

e x sin x − cosh x 2

x→0

x

4

)=1. 6

11. Prove that ⎛ 3⎞ 2 2 x 2 − 2e x + 2 cos ⎜ x 2 ⎟ + sin 3 x ⎝ ⎠ lim = −1 . 4 x→0 x

12. Prove that lim x→0

1 sinh x − x = . sin x − x cos x 2

sin x − tan −1 x 1 = . x → 0 x 2 log (1 + x ) 3

9. Prove that lim

FORMULAE nth Order Derivative of Some Standard Functions (i)

(ii)

dn (ax + b)m dx n a n m !(a x + b) m − n = , if n < m (m − n)! = n! an, if n = m = 0, if n > m dn (ax + b)–m dx n (m + n − 1)! an = (–1)n (m − 1)! (ax + b) m + n dn (ax + b)–1 dx n = (–1)n n!

an (ax + b)1+ n

dn log (ax + b) dx n (−1) n −1 (n − 1)! a n = (ax + b) n n d ax (iv) e = an eax dx n

(iii)

d n mx a = mn amx (log a)n dx n dn (vi) [sin (ax + b)] dx n = an sin n + ax + b 2 (v)

(vii)

dn [cos (ax + b)] dx n

(

)

(

)

= an cos n + ax + b 2

d n ax [e sin (bx + c)] dx n = r n eax sin (bx + c + nq ), where r = a 2 + b 2 , q = tan–1 b . a n d (ix) [eax cos (bx + c)] dx n = r n eax cos (bx + c + nq ),

(viii)

where r =

a 2 + b 2 , q = tan–1 b . a

Leibnitz’s Theorem yn = (uv)n = unv + nC1 un – 1 v1 + nC2 un – 2 v2 + nC3 un – 3 v3 + … + nC n uvn.

Engineering Mathematics

2.162

Taylor’s Series (i) f (x + h) = f (x) + h f (x) +

h2 f (x) 2!

h3 hn n f (x) + … + f (x) + … 3! n! (ii) f (x) = f (a) + (x – a) f (a) ( x a)2 ( x a )3 + f (a) + 2! 3! n ( x a) n f (a) + … + f (a) + … n! +

Maclaurin’s Series x2 f (0) 2! x3 xn n + f (0) + … + f (0) + … 3! n!

f (x) = f (0) + xf (0) +

List of Expansion of Some Standard Functions x2 x3 (i) ex = 1 + x + + +… 2! 3! 1!

(ii) sin x = x –

x3 x5 x7 + – +… 3! 5! 7!

(iii) cos x = 1 –

x2 x4 x6 + – +… 2! 4! 6!

(iv) tan x = x +

x3 2 x5 + +… 3 15

(v) sinh x = x +

x3 x5 x7 + + +… 3! 5! 7!

(vi) cosh x = 1 +

x4 x6 x2 + + +… 4! 6! 2!

(vii) tanh x = x –

x3 2 x5 + –… 3 15

x2 x3 x 4 + – … 2 3 4 m (m 1) 2 (ix) (1 + x)m = 1 + mx + x +… 2! L’Hospital’s Rule (viii) log (1 + x) = x –

If lim f (x) = 0, lim g (x) = 0, x

then

a

x

lim

x

a

a

f ( x) f ( x) = lim g ( x) x a g ( x)

MULTIPLE CHOICE QUESTIONS Choose the correct alternative in each of the following: 1. If f (x) = 4x2, then the value of c in the interval ] – 1, 3[ for which f (c) f (3) f ( 1) is 4 (a) 0 (b) 1 (c) 2 (d) 3 x

2. If f (x) =

∑2

n

x n , then f 33 (0) is

n=0

(a) (33!)333 (b) (32!)333 (c) (3!)232 (d) (33!)233 –1 3. If f (x) = tan x, then f 99(0) is (a) 97 ! (b) –98 ! (c) 99 ! (d) none of these 1 4. If f (x) = 2 then f 36(0) is x + x +1

(a) –36 ! (c) 236

(b) 36 ! (d) none of these 2

5. If f (x) = e x then f 100(0) is (a) 100 ! (b) 2100 100 ! (c) (d) 100 ! 50 ! 50 ! 6. If f (x) = x` log x and f (0) = 0, then the value of ` for which Rolle’s theorem can be applied in [0, 1] is (a) – 2 (b) – 1 1 (c) 0 (d) 2 7. The abscissae of the points of the curve y = x3 in the interval [–2, 2], where the slope of the tangents can

Differential Calculus I

be obtained by mean value theorem for the interval [– 2, 2] are 2 (b) ± 3 (a) ± 3 (c) ±

3 2

(d) 0

8. For which interval does the function x 2 3x satisfy all the conditions x 1 of Rolle’s theorem? (a) [0, 3] (b) [–3, 0] (c) [1.5, 3] (d) for no interval 9. The function f defined by f (x) = (x + 2) e–x is (a) decreasing for all x (b) decreasing in (– , –1) and increasing in (–1, ) (c) increasing for all x (d) decreasing in (–1, ) and increasing in (– , –1) 10. y = [x(x – 3)]2 increases for all values of x lying in the interval 3 (a) 0 < x < (b) 0 < x < 2 (c) – < x < 0 (d) 1 < x < 3 11. The value of a in order that f (x) = 3 sin x – cos x – 2ax + b decreases for all real values of x, is given by, (a) a < 1 (b) a 1 2 (d) a < 2 (c) a 12. As x is increased from – to , the function f (x) =

ex

f (x) = a1x2 + a2x + a3 in (a, b) is (a) b + a (b) b – a b+a ( b a) (c) (d) 2 2 sin x 14. The Taylor series expansion of x at x = p is given by (a) 1 +

( x − )2 + ... 3!

(b) −1 −

( x − )2 + ... 3!

( x − )2 + ... 3! ( x − )2 + ... (d) −1 + 3! 15. Which of the following functions would have only odd powers of x in its Taylor series expansion about the point x = 0? (b) sin (x2) (a) sin (x3) 3 (c) cos (x ) (d) cos (x2) 16. In the Taylor series expansion of ex + sin x about the point x = p, the coefficient of (x – p )2 is (b) 0.5 ep (a) ep p (c) e + 1 (d) ep – 1 17. The limit of the following series (c) 1 −

as x approaches

is 2 x3 x5 x 7 + − + ... f (x) = x − 3! 5! 7! (a)

x

1+ e (a) monotonically increases (b) monotonically decreases (c) increases to a maximum value and then decreases (d) decreases to a minimum value and then increases 13. The value of c in the mean value theorem of f (b) – f (a) = (b – a) f (c) for

2.163

(c)

2 3

(b)

2

(d) 1 3 18. If f and F be both continuous in [a, b], and derivable in (a, b) and if f (x) = F (x) for all x in [a, b] then f (x) and F(x) differ (a) by 1 in [a, b] (b) by x in [a, b] (c) by a constant in [a, b]

Engineering Mathematics

2.164

(d) none of these 19. Consider the following statements: 1. Rolle’s theorem ensures that there is a point on the curve, the tangent at which is parallel to the x-axis 2. Lagrange’s mean value therom ensures that there is a point on the curve, the tangent at which is parallel to the y-axis 3. Cauchy’s mean value thorem can be deduced from Lagrange’s mean value theorem. 4. Rolle’s mean value therom can be deduced from Lagrange’s mean value theorem. Which of the above statement (s) is/ are correct? (a) 1 and 4 (b) 2 and 4 (c) 1 alone (d) 1, 2 and 3 20. lim

sin ( / 2)

5 18 2 (c) 0 (d) 1 5 25. lim n n is equal to

0

(a) 0.5 (b) 1 (c) 2 (d) not defined sin 2 x 21. lim is equal to x 0 x (a) 0 (b) (c) 1 (d) –1 ( x 2 − 1) is equal to x →1 ( x − 1)

(a) 0 (c)

23. lim

x→∞

(b) 0 (d) 1 x 3 − cos x

x 2 + (sin x ) 2

is equal to

1

24. lim

x →3

2x2 − 7x + 3 5 x 2 − 12 x − 9

is equal to

1 ⎞n ⎟ n⎠

1

⎛ ⎜⎝1 +

2⎞n ⎛ ⎟ ... ⎜⎝1 + n⎠

1

n⎞n ⎟ n⎠

is equal to 2 e 4 (d) e

(a) 1

(b)

3 e

(c)

ax + b , x +1 lim f (x) = 2 and lim f (x) = 1,

27. It is given that f (x) = x→ ∞

0

then value of f (–2) is (a) 0 (b) 1 (c) e (d) f ( x) − 2 28. lim = 0, then lim f (x) is x 1 x →1 f ( x ) + 2 equal to (a) 1 (b) –1 (c) –2 (d) 2 29. lim e n

n log n

is equal to

0

(a) 1 (c) – 1 30. If lim

(b) 0 (d) does not exist x (1 − cos x ) − ax 2 sin x

x5 exists and is finite, then the value of a must be 1 (a) 1 (b) 2 1 1 (d) (c) 3 4 x→0

(a) (b) 0 (c) 2 (d) does not exist

(b) 1 (d) –

⎛ 26. lim ⎜1 + n→ ∞ ⎝

22. lim

(a) (c) 2

(b)

n→ ∞

x

is equal to

1 3

(a)

Differential Calculus I 31. The function f (x) = – 2x3 – 9x2 – 12x + 1 is an increasing function in the interval (a) – 2 < x < –1 (b) –2 < x < 1 (c) –1 < x < 2 (d) 1 < x < 2 32. Let f (x) and g(x) be differentiable for 0 x 2, such that f (0) = 4, f (2) = 8, g(0) = 0 and f (x) = g (x) for all x in [0, 2], then the value of g(2) must be (a) 2 (b) –2 (c) 4 (d) –4 33. If the function f and g be defined and continuous on [l, m], and be differentiable on (l, m) then which one of the following is not correct ? (a) When f (l) = f (m), there is p (l, m), such that f (p) = 0 (l, m) such that (b) There is p f (m) – f (l) = f (p) (m – l) (c) There is p (l, m) such that f (m) – f (l) = f (p) g(m) – g(l)] (d) There is p (l, m) such f ( m) − f (l ) f ′( p) = g ( m) − g (l ) g ′( p) where g(m) = g(l) and f (p), g (p) are not simultaneously zero.

that

Answers 1. (b) 8. (b) 15. (a) 22. (c) 29. (d) 36. (d)

2. 9. 16. 23. 30.

(d) (d) (b) (a) (b)

3. 10. 17. 24. 31.

(b) (a) (d) (b) (a)

4. 11. 18. 25. 32.

2.165

34. Let f (x) = x2 – 4x + 3. The following statements are associated with f : 1. f is increasing in (2, ) 2. f is decreasing in (– , –2) 3. f has a stationary point at x = 2 Which of these statements are correct? (a) 1 and 2 (b) 1 and 3 (c) 2 and 3 (d) 1, 2 and 3 35. The expansion of tan x in powers of x by Maclaurin’s theorem is valid in the interval. ⎛ −3 3 ⎞ , (b) ⎜ (a) (– , ) ⎟ ⎝ 2 2 ⎠

⎛− ⎞ , (d) ⎜ ⎝ 2 2 ⎟⎠ 36. The first three terms in the power series for log (1 + sin x) are 1 1 (a) x − x 3 + x 5 2 4 1 3 1 5 (b) x + x + x 2 4 1 3 1 5 (c) − x − x + x 2 4 1 2 1 3 (d) x − x + x 2 6 (c) (–p , p )

(b) (b) (c) (b) (c)

5. 12. 19. 26. 33.

(c) (a) (a) (a) (d)

6. 13. 20. 27. 34.

(d) (c) (c) (a) (b)

7. 14. 21. 28. 35.

(a) (d) (a) (d) (a)

Fourier Series

Fourier Series Chapter

13.1

13

13.1 INTRODUCTION Fourier series is used in the analysis of periodic functions. Many of the phenomena studied in engineering and sciences are periodic in nature e.g., current and voltage in an ac circuit. These periodic functions can be analysed into their constituent components by a Fourier analysis. Fourier series makes use of orthogonality relationships of the sine and cosine functions and exponential functions. It decomposes a periodic function into a sum of sine-cosine functions or exponential functions. The computation and study of Fourier series is known as harmonic analysis. It has many applications in electrical engineering, vibration analysis, acoustics, optics, signal processing, image processing, etc.

13.2 ORTHOGONALITY OF FUNCTIONS Consider two functions f1(x) and f2(x). Suppose we want to approximate f1(x) in terms of f2(x) in a certain interval (a, b) f1 ( x)

c12 f 2 ( x)

The error in this approximation will be f e ( x)

f1 ( x) c12 f 2 ( x)

c12 is selected such that error between f1(x) and c12 f2(x) is minimum. For minimising the error fe(x) in the interval (a, b), we have to minimise the average (or mean) of the square of the error function fe(x). The mean square error is given by, 1 b 2 f e ( x ) dx b a a 1 b [ f1 ( x) c12 f 2 ( x)]2 dx b a a 1 b [{ f1 ( x)}2 2c12 f1 ( x) f 2 ( x) c122 { f 2 ( x)}2 ]dx b a a b b b 1 { f1 ( x)}2 dx 2c12 f1 ( x) f 2 ( x) dx c122 { f 2 ( x)}2 dx a a a b a

13.2

Engineering Mathematics

To find the value of c12 which will minimise , we must have 1 c12

b

c12 b a 1

b

b a

a

a

c12

1 0 2 b a When

c12

b

{ f1 ( x)}2 dx 2c12 b

{ f1 ( x)}2 dx 2

b

a

b

f1 ( x) f 2 ( x)dx 2c12

a

b

f1 ( x) f 2 ( x)dx c122

f1 ( x) f 2 ( x)dx 2c12

a

c12

a

a b

a

=0

{ f 2 ( x)}2 dx

{ f 2 ( x)}2 dx

{ f 2 ( x)}2 dx

= 0, we get b a

b

f1 ( x) f 2 ( x)dx = c12 b a

c12 =

a

[ f 2 ( x)]2 dx

f1 ( x) f 2 ( x)dx b a

[ f 2 ( x)]2 dx

When c12 is zero, the function f1(x) contains no component of function f2(x) and two functions are said to be orthogonal in the interval (a, b). Thus two functions are orthogonal in the interval (a, b) if b a

If, in addition,

b a

f1 ( x) f 2 ( x)dx = 0

[ f1 ( x)]2 dx = 1 and

b a

[ f 2 ( x)]2 dx = 1, the functions are said to be

normalised and hence are called, orthonormal. Note: 1. A set of functions f1(x), f2(x)……..fn(x),… is said to be orthogonal in the interval (a, b) if these functions are mutually orthogonal, i.e., b a

f m ( x) f n ( x)dx = 0, m

n

2. The orthonormal set of functions is constructed by dividing the orthogonal set of b

functions by its norm, || f (x)||, i.e.,

[ f ( x)]2 dx . Hence, the orthonormal set of a

functions is, f1 ( x) , f1 ( x)

f 2 ( x) , f 2 ( x)

f n ( x) , f n ( x)

3. Any function f (x) can be represented by a complete set of orthogonal functions, in a certain interval which is the basis of a Fourier series representation.

Fourier Series

13.3

Example 1: Show that the following functions are orthogonal in the given interval. (i) f1(x) = x,

f2(x) = cos2x in -

o o , 2 2

(ii) f1(x) = ex,

f2(x) = sin x

(iii) f1(x) = 1,

f2(x) = x, f3 ( x ) =

o 5o , 4 4

in 3x2 - 1 2

[ -1, 1]

in

Solution: (i)



2



2

f1 ( x) f 2 ( x) dx = ∫ 2 x cos 2 x dx −

x

2

sin 2 x 2



(ii)

4

f1 ( x) f 2 ( x)dx = ∫

5 4 4

2

=0 2

, 2 2

Hence, x and cos 2x are orthogonal in the interval

5 4

cos 2 x 4

1

ex e sin x dx = (sin x − cos x) 2 x

5 4

4

1 e 2

5 4

1 e 2

5 4

5 4

cos

1

1

sin

2

2

5 4



1 −1

1

f1 ( x) f 2 ( x)dx = ∫ 1⋅ x dx = −1

x2 2

1

e4

Hence, ex and sin x are orthogonal in the interval (iii)

e 4 sin

1

2 4

4

,

2 5 4

cos

4

0

.

1

=0 −1

1 1 ⎛ 3 x 2 − 1⎞ 1 3 f ( x ) f ( x ) d x = 1 ∫−1 1 3 ∫−1 ⎜⎝ 2 ⎟⎠ dx = 2 x − x −1 = 0 1

⎛ 3 x 2 − 1⎞ 1 1 1 3x 4 x 2 3 3 f ( x ) f ( x ) d x = x d x = ( x − x ) d x = − 2 3 ∫−1 ∫−1 ⎜⎝ 2 ⎟⎠ 2 ∫−1 2 4 2 1

1

1

=0 −1

2 Hence, 1, x and 3 x 1 are orthogonal in the interval [ 1, 1] . 2

Example 2: Show that the functions f1(x) = 1 and f2(x) = x are orthogonal in the interval ( -1, 1) and determine the constant a and b so that the function f3(x) = 1 + ax + bx2 is orthogonal to both f1(x) and f2(x) in that interval. Solution:

x2 ∫−1 f1 ( x) f 2 ( x) dx = ∫−11⋅ x dx = 2 1

1

1

=0 −1

13.4

Engineering Mathematics

Hence, the functions f1(x) and f2(x) are orthogonal in the interval ( 1, 1) . The function f3(x) is orthogonal to both f1(x) and f2(x).

∫ ∫

1

1

f ( x) f 3 ( x)dx = 0

−1 1

1 (1 + ax + bx 2 )dx = 0

−1

x

ax 2 2

bx 3 3 2



and



1 −1

1 −1

1

0 1

2b 3 b

0 3

f 2 ( x) f 3 ( x)dx = 0

x (1 + ax + bx 2 )dx = 0 x2 2

ax 3 3

bx 4 4

1

0 1

2a =0 3 a=0 Example 3: Show that the set of functions {sin (2n + 1) x}, n = 0, 1, 2, … is orthogonal in the interval 0,

2

. Hence, construct corresponding orthonormal

set of functions. Solution: Let f m ( x) f n ( x) Case I: If m



2 0

sin(2m 1) x, sin(2n 1) x,

m = 0, 1, 2, … n = 0, 1, 2, …

n f m ( x) f n ( x)dx = ∫ 2 sin(2m + 1) x ⋅ sin(2n + 1) x dx 0

1 2 =

2 0

[cos(2m 2n) x cos(2m 2n 2) x]dx

1 sin(2m − 2n) x sin(2m + 2n + 2) x 2 − 2 2m − 2n 2m + 2n + 2 0

=0

[∵ sin p = 0 where p is an integer]

Fourier Series

13.5

Case II: If m = n 2 0

[ f n ( x)]2 dx

2 0

1 2

sin 2 (2n 1) x dx 2 0

[1 cos 2(2n 1) x]dx

1 sin 2(2n 1) x 2 x 2 2(2n 1) 0

0

4

Hence, the set of functions {sin (2n + 1) x}, n = 0, 1, 2, … is orthogonal in the interval . The orthonormal set of functions is constructed by dividing the orthogonal 2 set of functions by its norm. 0,

f ( x) =

2 0

[ f ( x)]2 dx =

2 Hence, the required orthonormal set of functions is, ⎧ 2 ⎫ sin(2n + 1) x ⎬ , n = 0, 1, 2, … ⎨ ⎩ ⎭ Example 4: Show that {cos x , cos 2x , cos 3x , …} is a set of orthogonal function in the interval ( , ). Hence, construct an orthonormal set. Solution: Let f m ( x) = cos mx, f n ( x) = cos nx, Case I: If m

m = 1, 2, 3, … n = 1 , 2 ,3,…

n





f m ( x) f n ( x) dx = ∫ cos mx ⋅ cos nx dx −

1 2

[cos(m n) x cos(m n) x]dx

1 sin(m n) x 2 m n

=0

sin(m n) x m n

[∵ sin p = 0 where p is an integer]

Case II: If m = n [ f n ( x)]2 dx

cos 2 nx dx 1 sin 2nx x 2 2n

1 2

(1 cos 2nx)dx 0

Hence, the set of functions is orthogonal in the interval ( , ). The orthonormal set of functions is constructed by dividing the orthogonal set of functions by its norm. f ( x) =

cos 2 nx dx =

13.6

Engineering Mathematics

Hence, the required set of orthonormal functions is 1 1 ⎧ 1 ⎫ cos x, cos 2 x, cos 3x, …⎬ ⎨ ⎩ ⎭

3π x 5π x ⎧ πx ⎫ , sin , sin , …⎬ is Example 5: Prove that the set of functions ⎨sin l l l ⎩ ⎭ orthogonal in the interval [0, l ] and construct the corresponding orthonormal set. (2m 1) x , l (2n 1) x f n ( x) = sin , l

Solution: Let f m ( x) = sin

Case I: If m

m = 0, 1, 2, … n = 0, 1, 2, …

n



l

0

l

f m ( x) f n ( x)dx = ∫ sin 0

1 2 1 2

l 0

(2m + 1) x (2n + 1) x sin dx l l (2m 2n) x (2m 2n 2) x cos cos dx l l (2m 2n) x l (2m 2n) l

(2m 2n 2) x l (2m 2n 2) l

sin

l

sin

0

[∵ sin p = 0 where p is an integer]

=0 Case II: If m = n l

∫ [f 0

l

n

( x)]2 dx = ∫ sin 2 0

1 l⎡ (2n + 1) x 2 (2n + 1) ⎤ dx = ∫ ⎢1 − cos x ⎥ dx 2 0⎣ l l ⎦ l

=

2 (2n + 1) x l l = ≠0 2(2n + 1) 2 l 0

sin

1 x− 2

Hence, the set of functions is orthogonal in the interval [0, l]. The othonormal set of functions is constructed by dividing the orthogonal set of functions by its norm. f ( x) =

l 0

[ f ( x)]2 dx =

l 2

Hence, the required orthonormal set of functions is, ⎧⎪ 2 x sin , ⎨ l ⎪⎩ l

2 3 x sin , l l

⎫⎪ 2 5 x sin , …⎬ l l ⎪⎭

Fourier Series

13.7

Example 6: Prove that the set of functions {1, sin x , cos x , sin 2x , cos 2x , …} is orthogonal in the interval (0, 2 ) and construct the corresponding orthonormal set. Solution: Let f n ( x) = sin nx,

n = 1, 2, 3, …

g n ( x) = cos nx, h( x ) = 1

n = 1, 2, 3, …

(a) Case I: If m

n



2

0

2

f m ( x) f n ( x) dx = ∫ sin mx sin nx dx 0

1 2

2

[cos(m n) x cos(m n) x]dx

0

1 sin(m n) x 2 m n

2

sin(m n) x m n 0

[∵ sin p = 0 where p is an integer]

=0 Case II: If m = n 2 0

[ f n ( x)]2 dx =

2

sin 2 nx dx

0

1 2

2

2

(1 cos 2nx)dx

0

1 sin 2nx x 2 2 0

0 (b) Case I: If m

n



2

0

2

g m ( x) g n ( x) dx = ∫ cos mx cos nx dx 0

1 2

2

[cos(m n) x cos(m n) x]dx

0

1 sin(m n) x 2 m n

2

sin(m n) x m n 0

[∵ sin p = 0 where p is an integer]

=0 Case II: If m = n 2 0

[ g n ( x)]2 dx =

2

cos 2 nx dx

0

1 2

2 0

2

(1 cos 2nx)dx 0

1 sin 2nx x 2 2n 0

13.8

(c)



2

0

Engineering Mathematics

2

f m ( x) g n ( x) dx = ∫ sin mx cos nx dx 0

2

1 2 1 cos( m + n) x cos( m − n) x = ∫ [sin(m + n) x + sin(m − n) x]dx = − − 0 m−n 2 2 ( m + n) 0

1 ⎡ − cos(m + n)2 + cos 0 cos(m − n)2 − cos 0 ⎤ − ⎥ 2 ⎢⎣ m+n m−n ⎦ [∵ cos 2 p = 1 where p is an integer ] =0 =

(d)

(e) (f )



2



2

0

0 2

0

h( x) f n ( x)dx = ∫

2

0

2

cos nx 1 1 ⋅ sin nx dx = − = (−1 + 1) = 0 n 0 n 2

2

h( x) g n ( x) dx = ∫ 1 ⋅ cos nx dx = 0

[h( x)]2 dx

2 0

1 dx

2

x0

sin nx =0 n 0

2

0

Hence, the set of functions {1, sin x, cos x, sin 2x, cos 2x, …} is orthogonal in the interval (0, 2 ). The orthonormal set of functions is constructed by dividing the each term of orthogonal set of functions by its norm. Hence, the required orthonormal set of functions is, sin x cos x sin 2 x cos 2 x ⎧ 1 ⎫ , , , , , …⎬ ⎨ ⎩ 2 ⎭

⎧ −x −x ⎫ 1 −x 2 Example 7: Prove that the set of functions ⎨ e 2 , e 2 ( − x + 1), e 2 ( x − 4 x + 2)⎬ 2 ⎩ ⎭ is orthonormal in the interval (0, ). Solution:





0



f1 ( x) f 2 ( x)dx = ∫ e



x 2

0

0



x

⋅ e 2 ( − x + 1)dx

e x ( x 1)dx

= xe

e x ( x 1) e x ( 1)

x 0

=0





0

x ∞ − 1 −x f 2 ( x) f 3 ( x)dx = ∫ e 2 ( − x + 1) ⋅ e 2 ( x 2 − 4 x + 2)dx 0 2 1 e x ( x 1)( x 2 4 x 2)dx 2 0 1 e x ( x 3 5 x 2 6 x 2)dx 2 0

0

Fourier Series

13.9

1 −x −e ( − x 3 + 5 x 2 − 6 x + 2) − e − x ( −3 x 2 + 10 x − 6) + ( −e − x )( −6 x + 10) − e − x ( −6) 2 =0 x ∞ ∞ − 1 − 2x 2 1 ∞ 2 f x f x x e e ( x − 4 x + 2)dx = ∫ e − x ( x 2 − 4 x + 2)dx ( ) ( ) d = ⋅ ∫0 1 3 ∫0 2 2 0 ∞ 1 = −e − x ( x 2 − 4 x + 2) − e − x (2 x − 4) + ( −e − x )(2) 0 = 0 2 =









[ f1 ( x)] dx = ∫ 2

0

0



0

∞ 0

2

∞ ⎛ − 2x ⎞ −x −x ⎜⎝ e ⎟⎠ dx = ∫0 e dx = −e

∞ 0

=1≠ 0

2

x ∞⎡ − ∞ ⎤ [ f 2 ( x)]2 dx = ∫ ⎢e 2 ( − x + 1) ⎥ dx = ∫ e − x ( − x + 1) 2 dx 0 0 ⎣ ⎦



= −e − x ( − x + 1) 2 − e − x ⋅ 2( − x + 1)( −1) + ( −e − x ) 2( −1)( −1) 0 = 1 − e − x ( − x + 1) 2 + 2e − x ( − x + 1) − 2e − x

∞ 0

=1≠ 0





0

[ f 3 ( x)] dx = ∫ 2



0

2

⎡ 1 − 2x 2 ⎤ 1 ∞ −x 2 2 ⎢ e ( x − 4 x + 2) ⎥ dx = ∫0 e ( x − 4 x + 2) dx 2 4 ⎣ ⎦

1 ∞ −x 4 e ( x + 16 x 2 + 4 − 8 x 3 − 16 x + 4 x 2 )dx 4 ∫0 1 ∞ = ∫ e − x ( x 4 − 8 x 3 + 20 x 2 − 16 x + 4)dx 4 0 =

−x −x 4 3 2 3 2 1 −e ( x − 8 x + 20 x − 16 x + 4) − e (4 x − 24 x + 40 x − 16) = 4 + ( −e − x )(12 x 2 − 48 x + 40) − e − x (24 x − 48) + e − x (24)



0

1 = (4) = 1 ≠ 0 4 Hence, the set of functions is orthonormal in the interval (0, ). Example 8: If f ( x ) = c1e1 ( x ) + c2e2 ( x ) + c3e 3 ( x ) where c1, c2 and c3 are constants and e1 ( x ), e2 ( x ), e 3 ( x ) are orthonormal functions in the internal (a, b), show that b a

[ f ( x )]2 dx = c12 + c22 + c32 .

Solution: Since



b

a

and b a



b

a

1

1

( x),

2

( x) and

3

( x) are orthonormal in the interval (a, b),

b

b

a

a

[ 1 ( x)]2 dx = ∫ [ 2 ( x)]2 dx = ∫ [ 3 ( x)]2 dx = 1

( x) 2 ( x)dx = ∫

[ f ( x)]2 dx

b

a

b a

2

( x) 3 ( x)dx = ∫

b

a

3

( x) 1 ( x)dx = 0

[c1 2 ( x) c2 2 ( x) c3 3 ( x)]2 dx

13.10

Engineering Mathematics

b a

[c12 { 1 ( x)}2

c22 { 2 ( x)}2

c32 { 3 ( x)}2

2c1c2 1 ( x) 2 ( x)

2c1c3 1 ( x) 3 ( x) 2c2 c3 2 ( x) 3 ( x)]dx b 2 1 a

c

[

b

2

1

( x) ] dx c22

2c1c2

b a

1

a

[

2

2

( x) ] dx c3

( x) 2 ( x)dx 2c1c3

b a

1

b a

[

2

3

( x) ] dx

( x) 3 ( x)dx 2c2 c3

b a

2

( x) 3 ( x)dx

= c12 (1) c22 (1) c32 (1) 2c1c2 (0) 2c1c3 (0) 2c2 c3 (0) c12

c22

c32

Exercise 13.1 1. Show that the following functions are orthogonal in the given interval. (i) f1(x) = x3, f2(x) = x2 + 1 in [ 1, 1] (ii) f1(x) = sin2x, f2(x) = cos x in [0, 1 ] 2. Show that the set of functions {sin(2n – 1)x}, n = 0, 1, 2, … is orthogonal over the interval 0,

2 Hence, construct the corresponding orthonormal set of functions.

.

3. Show that {sin nx}, n = 1, 2, … is orthogonal in the interval (0, 2 ) 4. Show that the set of functions ⎧ sin x cos x sin 2 x cos 2 x ⎫ , , , , …⎬ ⎨1, l l l l ⎭ ⎩ form an orthogonal set in the interval ( l, l) and construct an orthonormal set.

5. Show that if 1 ( x), 2 ( x) form an orthogonal set in the interval [a, b], then the functions 1 ( x ), 2 ( x )

form an orthogonal set for b > 0 in a b the interval , . 6. Prove that the functions f1(x) = b and f2(x) = x3 are orthogonal in the interval (- a, a) where a and b are real constants. Determine constants A and B such that the function f3(x) = 1 + Ax + Bx2 is orthogonal to both f1(x) and f2(x) in the interval ( a, a). Ans. : A

0, B

3 a2

7. Determine the constants a, b, c, l, m, n b cx, so that 1 ( x) = a, 2 ( x) 2 ( x ) l mx nx form an ortho3 normal set in the interval [- 1, 1]. Ans. : a = l=

1 2 5

2 2

, b = 0, c = , m = 0, n =

3 , 2 3 5 2 2

13.3 FOURIER SERIES Representation of a function over a certain interval by a linear combination of mutually orthogonal functions is called Fourier series representation.

Fourier Series

13.11

13.3.1 Convergence of the Fourier Series (Dirichlet’s Conditions) A function f (x) can be represented by a complete set of orthogonal functions within the interval (c, c + 2l). The Fourier series of the function f (x) exists only if the following conditions are satisfied: (i) f (x) is periodic, i.e., f (x) = f (x + 2l), where 2l is the period of function f (x). (ii) f (x (iii) f (x f (x) is piecewise continuous in c, c + 2l). f (x These conditions are known as Dirichlet’s conditions.

13.3.2 Trignometric Fourier Series n x n x and cos are orthogonal in the interval l l (c, c + 2l) for any value of c where n = 1, 2, 3, …. c 2l m x n x i.e., sin sin dx = 0, m n c l l = l , m = n, We know that the set of function sin

c 2l c

cos

m x n x cos dx = 0, l l =l,

c 2l c

sin

m

n

m=n

m x n x cos dx = 0 for all m, n l l

Hence, any function f (x) can be represented in terms of these orthogonal functions in the interval (c, c + 2l) for any value of c. f ( x)

a0

an cos n =1

n x l

bn sin n =1

n x l

This series is known as a trignometric Fourier series or simply a Fourier series. For example, a square function can be constructed by adding orthogonal sine components as shown in Fig. 13.1.

13.3.3 Euler’s Formula Let f (x) be a periodic function with period 2l in the interval (c, c + 2l). Then the Fourier series of f (x) is given by, n x n x f ( x) a0 an cos bn sin … (1) l l n =1 n =1

13.12

Engineering Mathematics

f (x) One sine component x f (x) x

Addition of two sine components

f (x) Addition of three sine x components

.. .. .. .

f (x)

x

Addition of many sine components

..

f (x)

Square function x

Fig. 13.1

(i) Determination of a0: Integrating both the sides of Eq. (1) w.r.t. x in the interval (c, c + 2l), c 2l c

f ( x)dx

a0

c 2l c

dx

c 2l c

an cos n =1

n x dx l

c 2l c

bn sin n =1

n x dx l

a0 (c 2l c) 0 0 = a0 (2l ) Hence, a0 =

1 2l

c 2l c

f ( x)dx

… (2)

(ii) Determination of an: Multiplying both the sides of Eq. (1) by cos and integrating w.r.t. x in the interval (c, c + 2l), c 2l c

f ( x) cos

n x dx l

a0

c 2l c

cos

n x dx l

c 2l c

0 lan

0

c 2l c

bn sin n =1

an cos n =1

n x l

n x n x cos dx l l

n x n x cos dx l l

Fourier Series

Hence, an =

1 l

c + 2l c

13.13

n x dx l

f ( x) cos

… (3)

(iii) Determination of bn: Multiplying both the sides of Eq. (1) by sin and integrating w.r.t. x in the interval (c, c + 2l ), c + 2l c

f ( x) sin

n x dx = a0 l

c + 2l c

sin

n x dx + l +

c + 2l c

an cos n =1

c + 2l c

bn sin n =1

n x l

n x n x sin dx l l

n x n x sin dx l l

= 0 + 0 + lbn Hence, bn =

1 l

c + 2l c

f ( x) sin

n x dx l

… (4)

The formulae (2), (3) and (4) are known as Euler’s Formulae which give the values of coefficients a0, an and bn. These coefficients are known as Fourier coefficients. Cor. 1: When c = 0 and 2l = 2 f ( x) = a0 +

an cos nx + n =1

where,

a0 = an =

1 2 1

f ( x) dx

0 2

f ( x) cos nx dx

0

1

bn = Cor. 2: When c

2

2

f ( x) sin nx dx

0

and 2l = 2 f ( x) = a0 +

an cos nx + n =1

where,

bn sin nx n =1

a0 = an = bn =

1 2 1

bn sin nx n =1

f ( x) dx f ( x) cos nx dx

1

f ( x) sin nx dx

Cor. 3: When c = 0 f ( x) = a0 +

an cos n =1

where

a0 =

1 2l

2l 0

f ( x ) dx

n x n x + bn sin l l n =1

13.14

Engineering Mathematics

1 l 1 bn = l

2l

an =

Cor. 4: When c

0 2l 0

n x dx l n x f ( x) sin dx l f ( x) cos

l,

f ( x)

a0

an cos n =1

1 2l 1 an = l 1 bn = l

l

a0 =

where

l

n x l

bn sin n =1

n x l

f ( x)dx

n x dx l n x f ( x) sin dx l

l

f ( x) cos

l l l

13.4 PARSEVAL’S IDENTITY Let f (x) be a periodic function with period 2l and is piecewise continuous in the interval (c, c + 2l ). Then 1 2l

c 2l c

[ f ( x)]2 dx

a02

1 ( an 2 2 n =1

bn 2 )

This is known as Parseval’s Identity for the function f (x) in the interval (c, c + 2l). Proof: We know that

f ( x)

a0

an cos n =1

n x l

bn sin n =1

n x l

… (1)

Multiplying both the sides of Eq. (1) by f (x) and integrating term by term w.r.t. x in the interval (c, c + 2l ), c 2l c

[ f ( x)]2 dx

a0

c 2l c

f ( x)dx

c 2l c

an f ( x) cos n =1

c 2l c

a0 (2la0 ) a02

n =1

an (lan ) n =1

1 c + 2l [ f ( x)]2 dx 2l ∫c

bn f ( x) sin

1 (an2 2 n =1

bn (lbn ) n =1

bn2 )

n x dx l

n x dx l

2la02

an2

l n =1

bn2 n =1

Fourier Series

13.15

Cor. 1: When c = 0 and 2l = 2 , 1 2 Cor. 2: When c

2

a02

1 (an2 2 n =1

bn2 )

[ f ( x)]2 dx

a02

1 (an2 2 n =1

bn2 )

[ f ( x)]2 dx

a02

1 (an2 2 n =1

bn2 )

[ f ( x)]2 dx

a02

1 (an2 2 n =1

bn2 )

[ f ( x)]2 dx

0

and 2l = 2 , 1 2

Cor. 3: When c = 0, 1 2l Cor. 4: When c

2l 0

l,

1 2l

l l

Fourier Series Expansion with Period 2o. Example 1: Find the Fourier series of f (x) = x in the interval (0, 2 ). Solution: The Fourier series of f (x) with period 2 f ( x)

a0

an cos nx n =1

a0 =

Hence,

1 2



2

0

is given by,

bn sin nx n =1

f ( x)dx =

1 2



2

x dx =

0

2

= 0

an =

1

=

1

1 ⎛ cos 2n 1 ⎞ ⎛ cos nx ⎞ ⎛ sin nx ⎞ − 2 ⎟=0 x⎜ ⎟ − (1) ⎜ − 2 ⎟ = ⎜ 2 n ⎠0 n ⎠ ⎝ n ⎝ ⎝ n ⎠

bn =

1



=

1

2



f ( x) cos nx dx =

0



2

0

x cos nx dx 2

2

f ( x) sin nx dx =

0

f ( x)

1

1 x2 2 2

1



2

0

x sin nx dx 2

1⎡ 2 ⎛ sin nx ⎞ ⎛ cos nx ⎞ ⎛ cos 2n ⎞ ⎤ x⎜− ⎟ − (1) ⎜ − 2 ⎟ = ⎢ −2 ⎜ ⎟⎥ = − n n n n ⎝ ⎝ ⎠0 ⎠ ⎝ ⎠⎦ ⎣ 2

1 sin nx n =1 n

Example 2: Find the Fourier series of f (x) = x2 in the interval (0, 2 ) and 2

hence, deduce that

12

1 12

1 22

1 32

1 42

….

13.16

Engineering Mathematics

Solution: The Fourier series of f (x) with period 2 f ( x)

a0

an cos nx

bn sin nx

n =1

a0 =

1 2

1

an =

n =1

2

f ( x)dx =

0

2



1 2

2

x 2 dx =

0

1

f ( x) cos nx dx =

0

is given by,



2

1 x3 2 3

2

=

2

4

0

3

x 2 cos nx dx

0

2

1⎛4 ⎞ 4 ⎛ sin nx ⎞ ⎛ cos nx ⎞ ⎛ sin nx ⎞ = x ⎜ ⎟ − 2x ⎜ − 2 ⎟ + 2 ⎜ − 3 ⎟ = ⎜ 2 ⎟ = 2 n ⎠ n ⎠0 ⎝n ⎠ n ⎝ n ⎠ ⎝ ⎝ 1 2 1 2 2 bn = ∫ f ( x) sin nx dx = ∫ x sin nx dx 1

2

0

Hence, f ( x) Putting x =

2

1⎛ 4 2⎞ 4 ⎛ cos nx ⎞ ⎛ sin nx ⎞ ⎛ cos nx ⎞ x ⎜− ⎟=− ⎟ − 2x ⎜ − 2 ⎟ + 2 ⎜ 3 ⎟ = ⎜ − n n ⎠ n ⎠ ⎝ n ⎠ ⎝ ⎝ ⎝ n ⎠0

1

=

0

2

2

4

1 cos nx 4 2 n n =1

4

3

1 sin nx n =1 n

(1)

... (1)

in Eq. (1),

f( )=

2

=

2

4 3

(−1) n 2 n =1 n ∞

+ 4∑

2

⎛ 1 1 1 1 ⎞ + 4 ⎜ − 2 + 2 − 2 + 2 …⎟ 3 ⎝ 1 2 3 4 ⎠ 2 1 1 1 1 = − + − +… 12 12 22 32 42 2

=

4

Example 3: Find the Fourier series of f ( x ) Hence, deduce that

1

4

1 3

1 5

1 ( 2

1 …. 7

Solution: The Fourier series of f (x) with period 2 f ( x)

a0

an cos nx n =1

1 a0 = 2 an =

1



2

0



2

0

x ) in the interval (0, 2 ).

is given by,

bn sin nx n =1

1 f ( x ) dx = 2



2

0

f ( x) cos nx dx =

1

1 1 ( − x ) dx = 2 4



2

0

1 ( − x) cos nx dx 2 2

=

x2 x− 2

1 ⎛ sin nx ⎞ ⎛ cos nx ⎞ ( − x) ⎜ ⎟ − (−1) ⎜ − 2 ⎟ = 0 2 n n ⎠0 ⎝ ⎠ ⎝

2

=0 0

Fourier Series

bn =

1



2

0

13.17

1 ( − x) sin nx dx 2 2

=

1 1 ⎛ ⎛ sin nx ⎞ ⎛ cos nx ⎞ ⎞ 1 ( − x) ⎜ − ⎜ + ⎟= ⎟ − (−1) ⎜ − 2 ⎟ = 2 2 n n n n ⎝ ⎠ n ⎝ ⎠0 ⎝ ⎠ f ( x) =

Hence, Putting x =

1 sin nx n =1 n

… (1)

in Eq. (1),

2

⎛ ⎞ 1⎛ ⎞ ∞ 1 ⎛n ⎞ f ⎜ ⎟ = ⎜ ⎟ = ∑ sin ⎜ ⎟ n 2 2 2 ⎝ ⎠ ⎝ ⎠ n =1 ⎝ 2 ⎠ 1 1 1 = 1 − + − +… 4 3 5 7 Example 4: Find the Fourier series of f ( x ) = 2

(0, 2 ). Hence, deduce that

6

= 1+

3x2

a0

an cos nx n =1

a0 =

1 2



2

0

1



2

0

f ( x)dx =

1 2



2

0

f ( x) cos nx dx =

1 (3 x 2 − 6 x + 2 12

=

1 ⎛6 6 ⎞ 1 ⎜ 2 + 2 ⎟= 2 12 ⎝ n n ⎠ n

=

1



2

0

in the interval

is given by,

bn sin nx

=

bn =

2

n =1

3x 2 − 6 x + 2 12



dx

2

=0

x 0

2

0

⎛ 3x 2 − 6 x + 2 ⎜ 12 ⎝

2

⎞ ⎟ cos nx dx ⎠ 2

2

f ( x) sin nx dx =

1 (3 x 2 − 6 x + 2 12

1

2

2

⎛ x2 ⎞ ⎛ x3 ⎞ 1 3⎜ ⎟ − 6 ⎜ ⎟ + 2 = 24 ⎝ 2 ⎠ ⎝ 3⎠

an =

2

1 1 + +… . 2 2 32

Solution: The Fourier series of f (x) with period 2 f ( x)

6x 12

⎛ cos nx ⎞ ⎛ sin nx ⎞ ⎛ sin nx ⎞ )⎜ ⎟ − (6 x − 6 ) ⎜ − 2 ⎟ + 6 ⎜ − 3 ⎟ n ⎠ ⎝ n ⎠0 ⎝ ⎝ n ⎠

1



2

0

⎛ 3x 2 − 6 x + 2 ⎜ 12 ⎝

2

⎞ ⎟ sin nx dx ⎠ 2

2

⎛ cos nx ⎞ ⎛ sin nx ⎞ ⎛ cos nx ⎞ )⎜ − ⎟ − (6 x − 6 ) ⎜ − 2 ⎟ + 6 ⎜ 3 ⎟ = 0 n n ⎠ ⎝ n ⎠0 ⎝ ⎠ ⎝

13.18

Engineering Mathematics

1 cos nx 2 n =1 n

Hence, f ( x) =

… (1)

Putting x = 0 in Eq. (1), ∞

2

f (a) =

1 2 n n =1

=∑

6

2

6

1 1 1 + + +… 22 32 42

= 1+

-x Example 5: Find the Fourier series of f ( x ) = e in the interval (0, 2 ). ∞ 1 ( −1) n . =∑ 2 Hence, deduce that 2 sinh n= 2 n + 1

Solution: The Fourier series of f (x) with period 2 f ( x)

a0

an cos nx

bn sin nx

n =1

1 2

a0 an =

2

1

2 0

n =1

1 2

f ( x)dx

0

2

e x dx

0

f ( x) cos nx dx =

1

2

1 (1 e (n 2 1)

1



2

0

1



2

0

1 e 2

Hence, f ( x) Putting x =

1− e

2

(1 − e

1 e

0

2

2

0

2

= 0

⎤ 1 ⎡ e −2 1 ( − n) − 2 ( − n) ⎥ ⎢ 2 n +1 ⎣ n +1 ⎦

)

−2

2

1 n =1

n

2

1

cos nx

1 e

2

n n =1

n

2

1

sin nx

… (1)

in Eq. (1),

f ( ) = e− = =

(n 2 + 1)

1 e 2

x 2

e − x sin nx dx

1 e− x = (− sin nx − n cos nx) n2 + 1 =

e

)

2

f ( x) sin nx dx =

n

1 2

e x cos nx dx

0

1 e x ( cos nx n sin nx) n2 1

bn =

is given by,

−2

1 − e −2 1 − e −2 + 2 ∞

(−1) 2 +1

∑n n=2

n



(−1) n 1 − e −2 1 − e −2 ⎡ 1 ∞ (−1) n ⎤ = + ⎢− + ∑ 2 ⎥ 2 2 +1 ⎣ 2 n = 2 n + 1⎦

∑n n =1

Fourier Series

13.19

(−1) n 2 n=2 n + 1 ∞

e (1 − e −2 )

=∑

(−1) n 2 n=2 n + 1 ∞

=∑

e − e−

1 2 sinh

Hence,

=

( 1) n 2 1 n=2 n

Example 6: Find the Fourier series of f (x) = x sin x in the interval (0, 2 ) and 1 3 hence, deduce that = . 2 1 4 n= 2 n Solution: The Fourier series of f (x) with period 2 f ( x)

a0

an cos nx n =1

a0 =

1 2

an =

1

=

1



2

0



2



2

0

0

f ( x ) dx =

1 2



2

0

is given by,

bn sin nx n =1

x sin x dx =

1 2 x(− cos x) − (− sin x) 0 = −1 2

f ( x) cos nx dx x sin x cos nx dx =

1 2



2

0

x[sin(n + 1) x − sin(n − 1) x] dx 2

=

⎧ sin(n + 1) x sin(n − 1) x ⎫ 1 ⎧ cos(n + 1) x cos(n − 1) x ⎫ + x ⎨− + ⎬ , ⎬ − (1) ⎨− 2 2 n +1 n −1 ⎭ (n − 1) 2 ⎭ 0 ⎩ ⎩ (n + 1)

cos(n − 1)2 ⎫⎤ 1 ⎡ ⎧ cos(n + 1)2 + 2 ⎨− ⎬⎥ , n ≠ 1 ⎢ n +1 n −1 2 ⎣ ⎩ ⎭⎦ 1 1 , n ≠1 =− + n +1 n −1 2 , n ≠1 = 2 n −1

=

For n = 1,

a1 =

1



2

0

x sin x cos x dx =

1 2



2

0

x sin 2 x dx 2

=

1 ⎛ cos 2 x ⎞ 1 ⎛ sin 2 x ⎞ x⎜− ⎟ − (1) ⎜ − ⎟ =− 2 2 ⎠ 4 ⎠0 2 ⎝ ⎝

n ≠1

13.20

bn =

Engineering Mathematics

1

2 0

1

2 0

f ( x) sin nx dx x sin x sin nx dx

1 sin(n 1) x x n 1 2 1 2

cos(n 1)2 (n 1) 2

0,

n 1

1 2

2

x[cos(n 1) x cos(n 1) x]dx

0

sin(n 1) x n 1

(1)

cos(n 1)2 n 1

cos(n 1) x (n 1) 2

1 (n 1) 2

cos(n 1) x (n 1) 2

1 , (n 1) 2

2

,

n 1

0

n 1

For n = 1,

b1 =

1

=

1



2



2

0

0

x sin x sin x dx x sin 2 x dx =

1 2



2

0

x(1 − cos 2 x)dx 2

⎛ x 2 cos 2 x ⎞ 1 ⎛ sin 2 x ⎞ 1 = x⎜ x − − ( ) (2 1 ⎜ + ⎟ = ⎟ 2 2 ⎠ 4 ⎠0 2 ⎝ ⎝ 2 Hence, f ( x)

1

1 cos x 2

n=2

2 cos nx n2 1

2

)=

sin x

… (1)

Putting x = 0 in Eq. (1), f (0) = 0 = −1 − 1 n=2

n

2

1

=

1 ∞ 2 +∑ 2 n= 2 n2 − 1

3 4

Example 7: Find the Fourier series of f ( x ) 1 1 Hence, deduce that = . 2 2 n = 1 4n - 1

1 cos x in the interval (0, 2 ).

Solution: The Fourier series of f (x) with period 2 f ( x)

a0

an cos nx

bn sin nx

n =1

f ( x) a0

n =1

1 cos x 1 2

2 0

is given by,

2 sin

f ( x)dx

1 2

x 2 2 0

x 2 sin dx 2

2 2

2

2 cos

x 20

2 2

Fourier Series

an =

1

=

1

=



2



2

13.21

f ( x) cos nx dx

0

⎡ ⎛ 2n + 1 ⎞ ⎛ 2n − 1 ⎞ ⎤ ⎢sin ⎜ 2 ⎟ x − sin ⎜ 2 ⎟ x ⎥ dx ⎝ ⎠ ⎦ ⎠ ⎣ ⎝

2 2 x 2 sin cos nx dx = 2 2 ∫0

0

2

2



2

2 2 ⎛ 2n − 1 ⎞ ⎛ 2n + 1 ⎞ cos ⎜ cos ⎜ ⎟x ⎟x+ 2n + 1 2n − 1 ⎝ 2 ⎠ 0 ⎝ 2 ⎠

=

2⎡ 2 2 2 2 ⎤ − + cos(2n + ) + cos(22n − ) − 2 ⎢⎣ 2n + 1 2n + 1 2n − 1 2n − 1 ⎥⎦

=

2⎡ 4 4 ⎤ − 2 ⎢⎣ 2n + 1 2n − 1 ⎥⎦ 4 2

=− bn =

1

4n − 1 2

2

2 0

2 2

x 2 sin sin nx dx 2 2n 1 2n 1 cos x cos x dx 2 2

f ( x) sin nx dx =

0

2

1 2

1

2

0

2

2 2n 1 2 2n 1 sin sin x x 2n 1 2 2n 1 2 0 f ( x)

Hence,

2 2

4 2

0

1 n =1

4n

2

1

cos nx

Putting x = 0 in Eq. (1),

f (0) = 0 =

2 2



4 2



∑ 4n n =1

1 = 2

n =1

4n

1

x x

2 .

Solution: The Fourier series of f (x) with period 2 a0

an cos nx n =1

1 2 f ( x ) dx = 2 ∫0 1 ⎡ 2 = −x 0 + 2x 2 ⎣

a0 =

−1

1 2

Example 8: Find the Fourier series of f ( x) 1 0 2

f ( x)

1 2

bn sin nx n =1 2 1 ⎡ (−1)dx + ∫ 2dx ⎤ ∫ ⎢ 0 ⎣ ⎦⎥ 2 ⎤=1 ⎦ 2

is given by,

… (1)

13.22

an

Engineering Mathematics

1

2

sin nx n 0

1

bn

1

1

f ( x) cos nx dx

0

2

2

0

sin nx n

1 cos nx n 0

2 cos nx dx

2

0

1

f ( x) sin nx dx

0

2

( 1) cos nx dx

0

( 1) sin nx dx

2

2 cos nx n

1 cos n n

2

1 n

2sin nx dx 2 cos 2n n

2 cos n n

3 [( 1) n 1] n 1 3 ∞ ⎡ (−1) n − 1 ⎤ Hence, f ( x) = + ∑ ⎢ ⎥ sin nx n 2 n =1 ⎣ ⎦ Example 9: Find the Fourier series of f (x) = x + x2 in the interval ( hence, deduce that 2

(i)

1 12

12 2

(ii)

6

=

1 22

1 32

1 42



1 1 1 1 + + + +… . 12 22 32 42

Solution: The Fourier series of f (x) with period 2 f ( x)

a0

an cos nx n =1

1 2

a0

an =

1

=

1





bn sin nx n =1

f ( x)dx

1 x2 2 2

is given by,

x3 3

1 2

( x x 2 )dx 2

3

f ( x) cos nx dx =

1





( x + x 2 ) cos nx dx

⎛ sin nx ⎞ ⎛ cos nx ⎞ ⎛ sin nx ⎞ ( x + x2 ) ⎜ ⎟ − (1 + 2 x) ⎜ − 2 ⎟ + 2 ⎜ − 3 ⎟ n ⎠ n ⎠− ⎝ n ⎠ ⎝ ⎝

1⎡ cos n cos(−n ) ⎤ 1 ⎡ cos n ⎤ ⎢(1 + 2 ) n 2 − (1 − 2 ) ⎥ = ⎢4 n2 n 2 ⎥⎦ ⎣ ⎦ ⎣ 4(−1) n = n2

=

,

) and

Fourier Series

bn =

1

=

1



f ( x) sin nx dx =



1



13.23

( x + x 2 ) sin nx dx



⎛ cos nx ⎞ ⎛ sin nx ⎞ ⎛ cos nx ⎞ ( x + x2 ) ⎜ − ⎟ − (1 + 2 x) ⎜ − 2 ⎟ + 2 ⎜ 3 ⎟ n n ⎝ ⎠ ⎝ ⎠ ⎝ n ⎠



1⎡ 2 ⎤ cos n ⎥ ⎢− ⎦ ⎣ n n −2(−1) = n

=

2

Hence, f ( x)

3

4

( 1) n ( 1) n cos nx 2 sin nx 2 n n =1 n n =1

… (1)

(i) Putting x = 0 in Eq. (1), 2

f (0) = 0 = 2

12 (ii) Putting x =

3

(−1) n 2 n =1 n ∞

+ 4∑

1 1 1 1 − + − +… 12 22 32 42

=

in Eq. (1), 2

+

f (p) = Putting x

⎛1 1 1 1 ⎞ + 4 ⎜ 2 + 2 + 2 + 2 + …⎟ 3 ⎝1 2 3 4 ⎠

=

2

… (2)

in Eq. (1), 2

f (– p) = − +

2

=

3

⎛1 1 1 1 ⎞ + 4 ⎜ 2 + 2 + 2 + 2 + …⎟ 1 2 3 4 ⎝ ⎠

… (3)

Adding Eqs. (2) and (3), 2

6

1 12

1 22

1 32

1 42



Example 10: Find the Fourier series of f ( x ) = e ax in the interval ( Solution: The Fourier series of f (x) with period 2 f ( x)

a0

an cos nx n =1

a0 =

1 2 1 e ax 2 a

=

sinh a a

bn sin nx n =1

f ( x ) dx =

1 2

1 ( a e 2 a

e ax dx e

a

)

is given by,

,

).

13.24

Engineering Mathematics

an =

1

=

1



f ( x) cos nx dx =



1



e ax cos nx dx



e ax (a cos nx + n sin nx) a + n2

=−

2



a cos n ( a e − e− a ) (a 2 + n 2 )

(−1) 2a sinh a (a 2 + n 2 ) n

= bn =

1

=

1

= Hence,



f ( x) sin nx dx =



1



e ax sin nx dx



e ax n cos n ( a (a sin nx − n cos nx) = − e − e− a ) 2 2 2 2 a +n ( a + n ) −

−2n(−1) n sinh a (a 2 + n 2 )

f ( x)

sinh a a

2a sinh a

sinh a a

2sinh a

( 1) n 2sinh a cos nx 2 2 n n =1 a ( 1) n (a cos nx n sin nx) 2 n2 n =1 a

Example 11: Find the Fourier series of f (x) = -o =x o2 1 1 1 Hence, deduce that = 2 + 2 + 2 +…. 8 1 3 5 Solution: The Fourier series of f (x) with period 2 f ( x)

a0

an cos nx

1 2 1 2

an

1 1

-o 0

x 0 x o

is given by,

bn sin nx

n =1

a0

n( 1) n sin nx 2 n2 n =1 a

n =1

f ( x)dx x

1 2 x2 2

0

0

1 ( 1) n 1 n2

x

)dx

(

0

x dx

4

0

f ( x) cos nx dx sin nx n

0

1

0

sin nx n

(

) cos nx dx cos nx n2 0

0

x cos nx dx

1 cos n n2

1 n2

Fourier Series

1

bn

f ( x) sin nx dx cos nx n

1

0

x

1

0

13.25

) sin nx dx

(

cos nx n

sin nx n2 0

0

x sin nx dx

1 [1 2 cos n ] n

1 [1 2( 1) n ] n ( 1) n 1 cos nx n2

1

f ( x)

Hence,

4

n =1

n =1

1 2( 1) n sin nx n

... (1)

1 − +0 =− ⎡ lim f ( x) + lim+ f ( x) ⎤ = x→0 ⎦ 2 ⎣ x → 0− 2 2

At x = 0, f (0) =

Putting x = 0 in Eq. (1),

f ( 0) = − 2

8

2

1 12

=− 1 32

4

+

1 52

⎡ (−1) n − 1 ⎤ ⎥ n2 ⎦ n =1 ⎣ ∞

1

∑⎢



Example 12: Find the Fourier series of f ( x )

x = x+

Solution: The Fourier series of f (x) with period 2 f ( x)

a0

an cos nx n =1

x

0

0

x

.

is given by,

bn sin nx n =1

1 f ( x) dx 2 ∫− 1 ⎡ 0 = (− x − )dx + ∫ ( x + ) dx ⎤ 0 ⎦⎥ 2 ⎣⎢ ∫− 0 ⎤ 1 ⎡ x2 x2 ⎢− − x + = + x ⎥ 2 ⎢ 2 2 − 0 ⎥ ⎣ ⎦

a0 =

= an =

1





f ( x) cos nx dx =

1⎡ ⎢ (− x − ⎢⎣ 2 = [(−1) n n2

=

2

1⎡ 0 (− x − ) cos nx dx + ∫ ( x + ) cos nx dx ⎤ 0 ⎣⎢ ∫− ⎦⎥

⎛ sin nx ⎞ ⎛ cos nx ⎞ )⎜ ⎟ − (−1) ⎜ − 2 ⎟ n ⎠ ⎝ n ⎠ ⎝ − 1]

0



⎛ sin nxx ⎞ ⎛ cos nx ⎞ ⎤ + (x + ) ⎜ ⎟ − (1) ⎜ − 2 ⎟ ⎥ n ⎠ 0 ⎥⎦ ⎝ n ⎠ ⎝

13.26

bn = =

1

Engineering Mathematics





f ( x) sin nx dx

1⎡ 0 (− x − ) sin nx dx + ∫ ( x + ) sin nx dx ⎤ ⎢⎣ ∫− 0 ⎦⎥

0 1⎡ ⎛ cos nx ⎞ ⎛ cos nx ⎞ ⎛ sin nx ⎞ ⎛ sin nx ⎞ 1 − − − ( ) ⎢ (− x − ) ⎜ − ⎟ ⎟ − (1) ⎜ − 2 ⎟ ⎜ ⎟ + (x + ) ⎜ − 2 n n n ⎠ n ⎝ ⎠ ⎝ ⎠ ⎝ ⎠− ⎝ ⎢⎣ 2 = [1 − (−1) n ] n

=

f ( x) =

Hence,

2

+

∞ ⎡1 − (−1) n ⎤ ⎡ (−1) n − 1 ⎤ ⎥ cos nx + 2∑ ⎢ ⎥ sin nx 2 n n ⎦ n =1 ⎣ n =1 ⎣ ⎦ ∞

2

∑⎢

Example 13: Find the Fourier series of f (x) = 0

x

= sin x Hence, deduce that

a0

an cos nx

an

1 2 1 2

n =1

f ( x) dx 0

1

0 dx

0

0

1

1 2

0 cos nx dx

( 1) n 1 n 1 n 1 1

1 [1 ( 1) n ], (n 2 1)

0

0

cos x 0

[sin(n 1) x sin(n 1) x]dx ( 1) n 1 n 1

1 2

1 2

sin x dx

f ( x) cos nxdx

1 2

For n = 1, 1 a1 =

is given by,

bn sin nx

n =1

a0 =

x

1 1 1 1 = + + +… . 2 1 3 3 5 5 7

Solution: The Fourier series of f (x) with period 2 f ( x)

0

0

sin x cos x dx = cos 2 x 2 0

0

1 2

1 , n 1

n 1

0

1 2

sin 2 x dx

1

0

sin x cos nx dx

cos(n 1) x n 1 n 1

cos(n 1) x , n 1 n 1 0

0

⎤ ⎥ ⎥⎦

Fourier Series

bn =

1

=

1 2

=

1 sin(n − 1) x sin(n + 1) x , − n −1 n +1 0 2



f ( x) sin nx dx =





13.27

1⎡ 0 0 ⋅ sin nx dx + ∫ sin x sin nx dx ⎤ 0 ⎣⎢ ∫− ⎦⎥

[cos(n − 1) x − cos( n + 1) x] dx

0

= 0,

n ≠1

n ≠1

For n = 1, 1 b1 = sin x sin x dx 0

1 2 =

0

1 sin 2 x x 2 2 0

(1 cos 2 x) dx

1 2 1

f ( x)

Hence,

1 n=2

1 ( 1) n 1 cos nx sin x 2 n2 1

... (1)

1 ⎡ lim f ( x) + lim+ f ( x) ⎤ = 0 x→0 ⎦ 2 ⎣ x → 0−

At x = 0,

Putting x = 0 in Eq. (1), 1 + (−1) n 1 2 ⎛ 1 1 1 ⎞ = − ⎜ + + + …⎟ 2 3 15 35 n − 1 ⎝ ⎠ n=2 1 1 1 1 = + + +… 2 1⋅ 3 3 ⋅ 5 5 ⋅ 7

f (0) = 0 =

1



1





Example 14: Find the Fourier series of f ( x ) = x =o -x Solution: The Fourier series of f (x) with period 2 f ( x)

a0

an cos nx n =1

a0 =

1 2



3 2



f ( x ) dx =

2

⎡ 1 ⎢ x2 = 2 ⎢ 2 ⎢⎣

2 −

+ 2

bn sin nx n =1

3 ⎤ 1 ⎡ 2 2 ⎢ ∫− x dx + ∫ ( − x)dx ⎥ 2 ⎣ 2 ⎦ 2

x2 x− 2

3 2

2

⎤ ⎥=0 ⎥ ⎥⎦

-o 2 o 2

is given by,

x x

o 2 3o 2

.

13.28

an = =

Engineering Mathematics

1



3 2



f ( x) cos nx dx

2

3 ⎤ 1⎡ 2 2 ⎢ ∫− x cos nx dx + ∫ ( − x) cos nx dx ⎥ ⎣ 2 ⎦ 2

⎡ 1 ⎛ sin nx ⎞ ⎛ sin nx ⎞ ⎛ cos nx ⎞ 2 ⎛ cos nx ⎞ = ⎢ x⎜ − (1) ⎜ − 2 ⎟ + ( − x) ⎜ ⎟ ⎟ − (−1) ⎜ − 2 ⎟ ⎢ ⎝ n ⎠ n ⎠ n ⎠− ⎝ ⎝ n ⎠ ⎝ ⎢⎣ 2 3n n ⎞ 1 ⎛ n ⎞⎤ 1⎡ ⎛ 3n − cos = ⎢ − ⎜ sin + sin ⎟ − 2 ⎜ cos ⎟ n 2 2 2 ⎠ ⎥⎦ n 2 2 ⎝ ⎠ ⎝ ⎣

3 2

2

⎤ ⎥ ⎥ ⎥⎦

2 1⎡ n n ⎤ ⎢ − n sin n cos 2 + n 2 sin 2 sin n ⎥ ⎣ ⎦ =0

=

bn = =

1



3 2



f ( x) sin nx dx

2

3 ⎤ 1⎡ 2 2 x sin nx d x + ( − x) sin nx dx ⎥ ⎢ ∫− ∫ ⎣ 2 ⎦ 2

⎡ 1 ⎛ cos nx ⎞ ⎛ sin nx ⎞ = ⎢ x⎜− ⎟ − (1) ⎜ − 2 ⎟ ⎢ ⎝ n ⎠ n ⎠ ⎝ ⎢⎣

2 −

2

⎛ cos nx ⎞ ⎛ sin nx ⎞ + ( − x) ⎜ − ⎟ − (−1) ⎜ − 2 ⎟ n ⎠ n ⎠ ⎝ ⎝

3 2

2

⎤ ⎥ ⎥ ⎥⎦

n 3 3n 1 3n ⎤ n 1⎡ = ⎢ − cos + 2 sin + cos − 2 sin 2 2n 2 2 ⎥⎦ 2 n n ⎣ 2n 1 3n ⎤ n 1⎡ ⎛ 3n n ⎞ 3 ⎢ 2n ⎜ cos 2 − cos 2 ⎟ + n 2 sin 2 − n 2 sin 2 ⎥ ⎠ ⎦ ⎣ ⎝ 1 3n ⎤ n 1⎡ 3 n = ⎢ − sin − 2 sin sin n + 2 sin 2 2 ⎥⎦ n 2 n n ⎣ =

=

1 n2

3n ⎤ n ⎡ ⎢3 sin 2 − sin 2 ⎥ ⎣ ⎦

Hence, f ( x) =

1



3n ⎤ 1 ⎡ n 3 sin − sin sin nx 2 ⎢ 2 2 ⎥⎦ ⎣

∑n n =1

Fourier Series Expansion with Period 2l Example 15: Find the Fourier series of f (x) = x2 in the interval (0, 4). Hence, 2 1 1 1 deduce that = 2 + 2 + 2 +…. 6 1 2 3

Fourier Series

13.29

Solution: The Fourier series of f (x) with period 2l = 4 is given by, n x n x + bn sin l l n =1 n =1 n x n x = a0 + an cos + bn sin 2 2 n =1 n =1

f ( x) = a0 +

a0 =

an =

an cos

1 2l 1 4 1 x3 f ( x)dx = ∫ x 2 dx = ∫ 2l 0 4 0 4 3

4

= 0

16 3

1 2l 1 4 n x n x f ( x) cos dx = ∫ x 2 cos dx ∫ 2 0 2 l 0 l 4

=

1 2⎛ 2 4 n x ⎞ ⎛ −8 n x⎞ n x⎞ ⎛ x ⎜ sin ⎟ − (2 x) ⎜ − 2 2 cos ⎟ + 2 ⎜ 3 3 sin ⎟ 2 ⎝n 2 ⎠ 2 ⎠ ⎝n 2 ⎠0 ⎝ n

1 ⎡ ⎛ 4 ⎞⎤ 16 8 ⎜ 2 2 ⎟⎥ = 2 2 ⎢ 2⎣ ⎝n ⎠⎦ n 1 2l 1 4 n x n x bn = ∫ f ( x)sin dx = ∫ x 2 sin dx 0 0 2 2 l l =

4

1 ⎛ −2 n x⎞ ⎛ 8 n x⎞ 4 n x⎞ ⎛ = x2 ⎜ cos ⎟ − 2 x ⎜ − 2 2 sin ⎟ + 2 ⎜ 3 3 cos ⎟ 2 ⎝n 2 ⎠ n n 2 2 ⎠0 ⎝ ⎠ ⎝ 16 1 ⎛ 32 ⎞ = ⎜− ⎟=− n 2⎝ n ⎠ Hence,

16 3

f ( x)

16 2

1 n x 16 1 n x cos sin 2 2 2 n =1 n n =1 n

… (1)

Putting x = 0 in Eq. (1),

f (0) = 0 =

16 16 ∞ 1 16 16 ⎛ 1 1 1 ⎞ + 2 ∑ 2 = + 2 ⎜ 2 + 2 + 2 + …⎟ 3 3 n 1 2 3 ⎝ ⎠ n =1

1 1 ⎛1 1 1 ⎞ − = 2 ⎜ 2 + 2 + 2 + …⎟ 3 ⎝1 2 3 ⎠ Putting x = 4 in Eq. (1),

16 16 ⎛ 1 1 1 ⎞ + 2 ⎜ 2 + 2 + 2 + …⎟ 3 ⎝1 2 3 ⎠ 2 1 ⎛1 1 1 ⎞ = 2 ⎜ 2 + 2 + 2 + …⎟ 3 ⎠ ⎝1 2 3

… (2)

f (4) = 16 =

Adding Eqs. (2) and (3), 1 2 1 1 1 = 2 2 + 2 + 2 +… 3 1 2 3 2 1 1 1 = + + +… 6 12 22 32

… (3)

13.30

Engineering Mathematics

Example 16: Find the Fourier series of f ( x )

4 x 2 in the interval (0, 2).

2

1 1 1 + + +…. 6 12 22 32 Solution: The Fourier series of f (x) with period 2l = 2 is given by, Hence, deduce that

f ( x) = a0 +

=

an cos n =1

= a0 +

n x n x + bn sin l l n =1

an cos n x + n =1

bn sin n x n =1

1 2l 1 2 1 x3 a0 = ∫ f ( x)dx = ∫ (4 − x 2 )dx = 4 x − 2l 0 2 0 2 3

an =

2

= 0

8 3

2 1 2l n x f ( x) cos dx = ∫ (4 − x 2 ) cos n x dx ∫ 0 0 l l 2

4 ⎛ cos n x ⎞ ⎛ sin n x ⎞ ⎛ sin n x ⎞ = (4 − x 2 ) ⎜ ⎟ − (−2 x) ⎜ − 2 2 ⎟ + (−2) ⎜ − 3 3 ⎟ = − 2 n n n n ⎠ ⎝ ⎠ ⎝ ⎠0 ⎝ bn =

2

2 1 2l n x f ( x)sin dx = ∫ (4 − x 2 )sin n x dx ∫ 0 0 l l 2

4 ⎛ sin n x ⎞ ⎛ cos n x ⎞ ⎛ cos n x ⎞ = (4 − x 2 ) ⎜ − ⎟ − (−2 x) ⎜ − 2 2 ⎟ + (−2) ⎜ 3 3 ⎟ = n n n n ⎠ ⎝ ⎠ ⎝ ⎠0 ⎝ Hence,

f ( x)

8 3

4 2

1 cos n x 2 n n =1

4

1 sin n x n =1 n

… (1)

Putting x = 0 in Eq. (1),

f (0) = 4 =

8 4 − 2 3



1

∑n n =1

2

=

8 4 ⎛1 1 1 ⎞ − 2 ⎜ 2 + 2 + 2 + …⎟ 3 ⎝1 2 3 ⎠

1 1 ⎛1 1 1 ⎞ = − 2 ⎜ 2 + 2 + 2 + …⎟ 3 ⎝1 2 3 ⎠ Putting x = 2 in Eq. (1), 8 4 ⎛1 1 1 ⎞ f (2) = 0 = − 2 ⎜ 2 + 2 + 2 + … ⎟ 3 ⎝1 2 3 ⎠ 2 1 ⎛1 1 1 ⎞ − = − 2 ⎜ 2 + 2 + 2 + …⎟ 3 ⎝1 2 3 ⎠ Adding Eqs. (2) and (3), 1 2 1 1 1 2 3 12 22 32 2 1 1 1 = 2 + 2 + 2 +… 6 1 2 3



… (2)

… (3)

Fourier Series

13.31

Example 17: Find the Fourier series of f ( x ) = 2 x - x 2 in the interval (0, 3). 2 1 1 1 Hence, deduce that = 2 + 2 + 2 +… . 6 1 2 3 Solution: The Fourier series of f (x) with period 2l = 3 is given by, n x n x + bn sin l l n =1 n =1 2n x 2n x = a0 + an cos + bn sin 3 3 n =1 n =1

f ( x) = a0 +

a0 =

an cos

1 2l 1 3 1 2 x3 2 = 2 − = f ( x ) d x ( x x ) d x x − 2l ∫0 3 ∫0 3 3

3

=0 0

1 2l 2 3 2n x n x f ( x) cos dx = ∫ (2 x − x 2 ) cos dx ∫ 0 0 3 l l l 2n x ⎞ 9 2n x ⎞ 2 ⎛ ⎛ 3 sin = (2 x − x 2 ) ⎜ ⎟ ⎟ − (2 − 2 x) ⎜ − 2 2 cos 3 ⎠ 3 ⎠ 3 ⎝ 4n ⎝ 2n

an =

3

2n x ⎞ 27 ⎛ + (−2) ⎜ − 3 3 sin ⎟ 3 ⎠0 ⎝ 8n 2⎡ 9 3 ⎢⎣ 4n 2 9 =− 2 2 n =

2

⎤ (−4 − 2) ⎥ ⎦

1 2l 2 3 n x 2n x f ( x) sin dx = ∫ (2 x − x 2 ) sin dx ∫ l 0 3 0 l 3 2 2n x ⎞ 9 2n x ⎞ ⎛ 3 ⎛ = (2 x − x 2 ) ⎜ − cos ⎟ − (2 − 2 x) ⎜ − 2 2 sin ⎟ n 3 3 2 3 ⎠ n 4 ⎝ ⎠ ⎝

bn =

3

2n x ⎞ ⎛ 27 +(−2) ⎜ 3 3 cos ⎟ 3 ⎠0 ⎝ 8n = Hence, f ( x)

2⎛ 9 ⎞ 3 ⎜ ⎟= 3 ⎝ 2n ⎠ n 9 2

1 2n x cos 2 3 n n =1

3

1 2n x sin n 3 n =1

… (1)

Putting x = 0 in Eq. (1),

9 ⎛1 1 1 ⎞ + + + …⎟ 2 ⎜ 2 ⎝ 1 22 32 ⎠ 1 1 1 0 = 2 + 2 + 2 +… 1 2 3

f (0) = 0 = −

… (2)

13.32

Engineering Mathematics

Putting x = 3 in Eq. (1),

9 ⎛1 1 1 ⎞ + 2 + 2 + …⎟ 2 ⎜ 2 1 2 3 ⎝ ⎠ 2 1 1 1 = + + +… 3 12 22 32

f (3) = −3 = −

… (3)

Adding Eqs. (2) and (3), 2

1 1 1 + + +… 3 12 22 32 2 1 1 1 = 2 + 2 + 2 +… 6 1 2 3 =2

Example 18: Find the Fourier series of f ( x ) = x =0

0< x